You are on page 1of 482

G.R. No.

L-63915 April 24, 1985


LORENZO M. TAADA, ABRAHAM F. SARMIENTO, and MOVEMENT OF ATTORNEYS FOR BROTHERHOOD, INTEGRITY
AND NATIONALISM, INC. [MABINI], petitioners,
vs.
HON. JUAN C. TUVERA, in his capacity as Executive Assistant to the President, HON. JOAQUIN VENUS, in his capacity
as Deputy Executive Assistant to the President , MELQUIADES P. DE LA CRUZ, in his capacity as Director, Malacaang
Records Office, and FLORENDO S. PABLO, in his capacity as Director, Bureau of Printing, respondents.

ESCOLIN, J.:
Invoking the people's right to be informed on matters of public concern, a right recognized in Section 6, Article IV of the 1973
Philippine Constitution, 1 as well as the principle that laws to be valid and enforceable must be published in the Official Gazette
or otherwise effectively promulgated, petitioners seek a writ of mandamus to compel respondent public officials to publish,
and/or cause the publication in the Official Gazette of various presidential decrees, letters of instructions, general orders,
proclamations, executive orders, letter of implementation and administrative orders.
Specifically, the publication of the following presidential issuances is sought:
a] Presidential Decrees Nos. 12, 22, 37, 38, 59, 64, 103, 171, 179, 184, 197, 200, 234, 265, 286, 298, 303, 312,
324, 325, 326, 337, 355, 358, 359, 360, 361, 368, 404, 406, 415, 427, 429, 445, 447, 473, 486, 491, 503, 504, 521,
528, 551, 566, 573, 574, 594, 599, 644, 658, 661, 718, 731, 733, 793, 800, 802, 835, 836, 923, 935, 961, 10171030, 1050, 1060-1061, 1085, 1143, 1165, 1166, 1242, 1246, 1250, 1278, 1279, 1300, 1644, 1772, 1808, 1810,
1813-1817, 1819-1826, 1829-1840, 1842-1847.
b] Letter of Instructions Nos.: 10, 39, 49, 72, 107, 108, 116, 130, 136, 141, 150, 153, 155, 161, 173, 180, 187, 188,
192, 193, 199, 202, 204, 205, 209, 211-213, 215-224, 226-228, 231-239, 241-245, 248, 251, 253-261, 263-269,
271-273, 275-283, 285-289, 291, 293, 297-299, 301-303, 309, 312-315, 325, 327, 343, 346, 349, 357, 358, 362,
367, 370, 382, 385, 386, 396-397, 405, 438-440, 444- 445, 473, 486, 488, 498, 501, 399, 527, 561, 576, 587, 594,
599, 600, 602, 609, 610, 611, 612, 615, 641, 642, 665, 702, 712-713, 726, 837-839, 878-879, 881, 882, 939-940,
964,997,1149-1178,1180-1278.
c] General Orders Nos.: 14, 52, 58, 59, 60, 62, 63, 64 & 65.
d] Proclamation Nos.: 1126, 1144, 1147, 1151, 1196, 1270, 1281, 1319-1526, 1529, 1532, 1535, 1538, 1540-1547,
1550-1558, 1561-1588, 1590-1595, 1594-1600, 1606-1609, 1612-1628, 1630-1649, 1694-1695, 1697-1701, 17051723, 1731-1734, 1737-1742, 1744, 1746-1751, 1752, 1754, 1762, 1764-1787, 1789-1795, 1797, 1800, 1802-1804,
1806-1807, 1812-1814, 1816, 1825-1826, 1829, 1831-1832, 1835-1836, 1839-1840, 1843-1844, 1846-1847, 1849,
1853-1858, 1860, 1866, 1868, 1870, 1876-1889, 1892, 1900, 1918, 1923, 1933, 1952, 1963, 1965-1966, 19681984, 1986-2028, 2030-2044, 2046-2145, 2147-2161, 2163-2244.
e] Executive Orders Nos.: 411, 413, 414, 427, 429-454, 457- 471, 474-492, 494-507, 509-510, 522, 524-528, 531532, 536, 538, 543-544, 549, 551-553, 560, 563, 567-568, 570, 574, 593, 594, 598-604, 609, 611- 647, 649-677,
679-703, 705-707, 712-786, 788-852, 854-857.
f] Letters of Implementation Nos.: 7, 8, 9, 10, 11-22, 25-27, 39, 50, 51, 59, 76, 80-81, 92, 94, 95, 107, 120, 122, 123.
g] Administrative Orders Nos.: 347, 348, 352-354, 360- 378, 380-433, 436-439.
The respondents, through the Solicitor General, would have this case dismissed outright on the ground that petitioners have no
legal personality or standing to bring the instant petition. The view is submitted that in the absence of any showing that
petitioners are personally and directly affected or prejudiced by the alleged non-publication of the presidential issuances in
question 2 said petitioners are without the requisite legal personality to institute this mandamus proceeding, they are not being
"aggrieved parties" within the meaning of Section 3, Rule 65 of the Rules of Court, which we quote:
SEC. 3. Petition for Mandamus.When any tribunal, corporation, board or person unlawfully neglects the
performance of an act which the law specifically enjoins as a duty resulting from an office, trust, or station, or
unlawfully excludes another from the use a rd enjoyment of a right or office to which such other is entitled, and there
is no other plain, speedy and adequate remedy in the ordinary course of law, the person aggrieved thereby may file
a verified petition in the proper court alleging the facts with certainty and praying that judgment be rendered
commanding the defendant, immediately or at some other specified time, to do the act required to be done to
Protect the rights of the petitioner, and to pay the damages sustained by the petitioner by reason of the wrongful
acts of the defendant.
Upon the other hand, petitioners maintain that since the subject of the petition concerns a public right and its object is to compel
the performance of a public duty, they need not show any specific interest for their petition to be given due course.
The issue posed is not one of first impression. As early as the 1910 case of Severino vs. Governor General, 3 this Court held
that while the general rule is that "a writ of mandamus would be granted to a private individual only in those cases where he has
some private or particular interest to be subserved, or some particular right to be protected, independent of that which he holds
with the public at large," and "it is for the public officers exclusively to apply for the writ when public rights are to be subserved
[Mithchell vs. Boardmen, 79 M.e., 469]," nevertheless, "when the question is one of public right and the object of the mandamus
is to procure the enforcement of a public duty, the people are regarded as the real party in interest and the relator at whose
instigation the proceedings are instituted need not show that he has any legal or special interest in the result, it being sufficient
to show that he is a citizen and as such interested in the execution of the laws [High, Extraordinary Legal Remedies, 3rd ed.,
sec. 431].
Thus, in said case, this Court recognized the relator Lope Severino, a private individual, as a proper party to the mandamus
proceedings brought to compel the Governor General to call a special election for the position of municipal president in the town
of Silay, Negros Occidental. Speaking for this Court, Mr. Justice Grant T. Trent said:
We are therefore of the opinion that the weight of authority supports the proposition that the relator is a proper party
to proceedings of this character when a public right is sought to be enforced. If the general rule in America were
otherwise, we think that it would not be applicable to the case at bar for the reason 'that it is always dangerous to
apply a general rule to a particular case without keeping in mind the reason for the rule, because, if under the
particular circumstances the reason for the rule does not exist, the rule itself is not applicable and reliance upon the
rule may well lead to error'
No reason exists in the case at bar for applying the general rule insisted upon by counsel for the respondent. The
circumstances which surround this case are different from those in the United States, inasmuch as if the relator is
not a proper party to these proceedings no other person could be, as we have seen that it is not the duty of the law
officer of the Government to appear and represent the people in cases of this character.

The reasons given by the Court in recognizing a private citizen's legal personality in the aforementioned case apply squarely to
the present petition. Clearly, the right sought to be enforced by petitioners herein is a public right recognized by no less than the
fundamental law of the land. If petitioners were not allowed to institute this proceeding, it would indeed be difficult to conceive of
any other person to initiate the same, considering that the Solicitor General, the government officer generally empowered to
represent the people, has entered his appearance for respondents in this case.
Respondents further contend that publication in the Official Gazette is not a sine qua non requirement for the effectivity of laws
where the laws themselves provide for their own effectivity dates. It is thus submitted that since the presidential issuances in
question contain special provisions as to the date they are to take effect, publication in the Official Gazette is not indispensable
for their effectivity. The point stressed is anchored on Article 2 of the Civil Code:
Art. 2. Laws shall take effect after fifteen days following the completion of their publication in the Official Gazette,
unless it is otherwise provided, ...
The interpretation given by respondent is in accord with this Court's construction of said article. In a long line of decisions, 4 this
Court has ruled that publication in the Official Gazette is necessary in those cases where the legislation itself does not provide
for its effectivity date-for then the date of publication is material for determining its date of effectivity, which is the fifteenth day
following its publication-but not when the law itself provides for the date when it goes into effect.
Respondents' argument, however, is logically correct only insofar as it equates the effectivity of laws with the fact of publication.
Considered in the light of other statutes applicable to the issue at hand, the conclusion is easily reached that said Article 2 does
not preclude the requirement of publication in the Official Gazette, even if the law itself provides for the date of its effectivity.
Thus, Section 1 of Commonwealth Act 638 provides as follows:
Section 1. There shall be published in the Official Gazette [1] all important legisiative acts and resolutions of a public
nature of the, Congress of the Philippines; [2] all executive and administrative orders and proclamations, except
such as have no general applicability; [3] decisions or abstracts of decisions of the Supreme Court and the Court of
Appeals as may be deemed by said courts of sufficient importance to be so published; [4] such documents or
classes of documents as may be required so to be published by law; and [5] such documents or classes of
documents as the President of the Philippines shall determine from time to time to have general applicability and
legal effect, or which he may authorize so to be published. ...
The clear object of the above-quoted provision is to give the general public adequate notice of the various laws which are to
regulate their actions and conduct as citizens. Without such notice and publication, there would be no basis for the application
of the maxim "ignorantia legis non excusat." It would be the height of injustice to punish or otherwise burden a citizen for the
transgression of a law of which he had no notice whatsoever, not even a constructive one.
Perhaps at no time since the establishment of the Philippine Republic has the publication of laws taken so vital significance that
at this time when the people have bestowed upon the President a power heretofore enjoyed solely by the legislature. While the
people are kept abreast by the mass media of the debates and deliberations in the Batasan Pambansaand for the diligent
ones, ready access to the legislative recordsno such publicity accompanies the law-making process of the President. Thus,
without publication, the people have no means of knowing what presidential decrees have actually been promulgated, much
less a definite way of informing themselves of the specific contents and texts of such decrees. As the Supreme Court of Spain
ruled: "Bajo la denominacion generica de leyes, se comprenden tambien los reglamentos, Reales decretos, Instrucciones,
Circulares y Reales ordines dictadas de conformidad con las mismas por el Gobierno en uso de su potestad. 5
The very first clause of Section I of Commonwealth Act 638 reads: "There shall be published in the Official Gazette ... ." The
word "shall" used therein imposes upon respondent officials an imperative duty. That duty must be enforced if the Constitutional
right of the people to be informed on matters of public concern is to be given substance and reality. The law itself makes a list of
what should be published in the Official Gazette. Such listing, to our mind, leaves respondents with no discretion whatsoever as
to what must be included or excluded from such publication.
The publication of all presidential issuances "of a public nature" or "of general applicability" is mandated by law. Obviously,
presidential decrees that provide for fines, forfeitures or penalties for their violation or otherwise impose a burden or. the people,
such as tax and revenue measures, fall within this category. Other presidential issuances which apply only to particular persons
or class of persons such as administrative and executive orders need not be published on the assumption that they have been
circularized to all concerned. 6
It is needless to add that the publication of presidential issuances "of a public nature" or "of general applicability" is a
requirement of due process. It is a rule of law that before a person may be bound by law, he must first be officially and
specifically informed of its contents. As Justice Claudio Teehankee said in Peralta vs. COMELEC 7:
In a time of proliferating decrees, orders and letters of instructions which all form part of the law of the land, the
requirement of due process and the Rule of Law demand that the Official Gazette as the official government
repository promulgate and publish the texts of all such decrees, orders and instructions so that the people may know
where to obtain their official and specific contents.
The Court therefore declares that presidential issuances of general application, which have not been published, shall have no
force and effect. Some members of the Court, quite apprehensive about the possible unsettling effect this decision might have
on acts done in reliance of the validity of those presidential decrees which were published only during the pendency of this
petition, have put the question as to whether the Court's declaration of invalidity apply to P.D.s which had been enforced or
implemented prior to their publication. The answer is all too familiar. In similar situations in the past this Court had taken the
pragmatic and realistic course set forth in Chicot County Drainage District vs. Baxter Bank 8 to wit:
The courts below have proceeded on the theory that the Act of Congress, having been found to be unconstitutional,
was not a law; that it was inoperative, conferring no rights and imposing no duties, and hence affording no basis for
the challenged decree. Norton v. Shelby County, 118 U.S. 425, 442; Chicago, 1. & L. Ry. Co. v. Hackett, 228 U.S.
559, 566. It is quite clear, however, that such broad statements as to the effect of a determination of
unconstitutionality must be taken with qualifications. The actual existence of a statute, prior to such a determination,
is an operative fact and may have consequences which cannot justly be ignored. The past cannot always be erased
by a new judicial declaration. The effect of the subsequent ruling as to invalidity may have to be considered in
various aspects-with respect to particular conduct, private and official. Questions of rights claimed to have become
vested, of status, of prior determinations deemed to have finality and acted upon accordingly, of public policy in the
light of the nature both of the statute and of its previous application, demand examination. These questions are
among the most difficult of those which have engaged the attention of courts, state and federal and it is manifest
from numerous decisions that an all-inclusive statement of a principle of absolute retroactive invalidity cannot be
justified.
Consistently with the above principle, this Court in Rutter vs. Esteban 9 sustained the right of a party under the Moratorium Law,
albeit said right had accrued in his favor before said law was declared unconstitutional by this Court.
Similarly, the implementation/enforcement of presidential decrees prior to their publication in the Official Gazette is "an operative
fact which may have consequences which cannot be justly ignored. The past cannot always be erased by a new judicial
declaration ... that an all-inclusive statement of a principle of absolute retroactive invalidity cannot be justified."

From the report submitted to the Court by the Clerk of Court, it appears that of the presidential decrees sought by petitioners to
be published in the Official Gazette, only Presidential Decrees Nos. 1019 to 1030, inclusive, 1278, and 1937 to 1939, inclusive,
have not been so published. 10 Neither the subject matters nor the texts of these PDs can be ascertained since no copies
thereof are available. But whatever their subject matter may be, it is undisputed that none of these unpublished PDs has ever
been implemented or enforced by the government. In Pesigan vs. Angeles, 11 the Court, through Justice Ramon Aquino, ruled
that "publication is necessary to apprise the public of the contents of [penal] regulations and make the said penalties binding on
the persons affected thereby. " The cogency of this holding is apparently recognized by respondent officials considering the
manifestation in their comment that "the government, as a matter of policy, refrains from prosecuting violations of criminal laws
until the same shall have been published in the Official Gazette or in some other publication, even though some criminal laws
provide that they shall take effect immediately.
WHEREFORE, the Court hereby orders respondents to publish in the Official Gazette all unpublished presidential issuances
which are of general application, and unless so published, they shall have no binding force and effect.
SO ORDERED.

G.R. No. L-14858

December 29, 1960

MARIANO S. GONZAGA, petitioner-appellee,


vs.
AUGUSTO CE DAVID, as Registrar of the Motor Vehicles Office of Cagayan, respondent-appellant.
Office of the Asst. Solicitor General Guillermo E. Torres and Solicitor E. D. Ignacio for appellant.
Ventura V. Perez for appellee.

REYES, J.B.L., J.:


The essential antecedents of this case are not disputed. On February, 1957, Mariano Gonzaga, as owner, registered with the
Motor Vehicles Office a cargo truck and a passenger bus, paying the first installment for registration fees due on said vehicles
for 1957. To cover the second installment for registration fees, Gonzaga remitted to the Provincial Treasurer of Cagayan, by
registered mail, P500.00, under postal money orders Nos. 18553, 18554 and 18555, purchased from and issued by the Post
Office of Camalaniugan, Cagayan. The postal cancellation mark on the envelope containing the remittance of Gonzaga bears
the date August 31, 1957; so does the postal cancellation mark on the face of the money orders.
The Registrar of the Motor Vehicles Office of Cagayan ruled that pursuant to Section 8 (1), Act 3992, otherwise known as the
Revised Motor Vehicle Law, the second installment for registration fees was payable on or before the last working day of
August; that the last working day of August, 1957 was Friday, August 30, 1957; that consequently, the remittance of Gonzaga
bearing postal cancellation mark dated August 31, 1957 was made beyond the time fixed by law. Accordingly, said official
sought to impose a 50% delinquency penalty, or otherwise, threatened to confiscate the certificate of registration for the two
trucks (Annexes "B" & "C").lawphil.net
Gonzaga brought this action in the Court of First Instance, which, upon a stipulation of facts, rendered judgment, the dispositive
part reading
POR TANTO, el Juzgado dicta decision declarando, como por la presente declara, que el pago hecho con los giros
postales Nos. 18553, 18554 y 18555, por el recurrente, se ha hecho dentro del plazo fijado por ley; y, por tanto, el
recurrente no ha incurrido con morosidad en cuanto a dicho pago.
Se ordena al recurrido, sus agentes y representantes, que se abstengan de confiscar el certificado de registro de los dos
trucks del recurrente, por la alegada morosidad del citado pago.
Sin costas.
ASI SE ORDENA.
The only issue in this appeal is whether the remittance of petitioner-appellee covering the second installment of registration fees
for 1957, made by registered mail with postal cancellation dated August 31, 1957, was within the time fixed by law.
The following are the pertinent provisions of Act 3992 as amended
Sec. 8 (I) ". . . The registration fees provided in this Act for trucks may be payable in two equal installments, the first to be
paid on or before the last working day of February, and the second to be paid on or before the last working day of August.
(Emphasis supplied)
Sec. 6 (b) "The date of cancellation of the postage stamps of envelopes containing money orders, checks, or cash shall
be considered as the date of
application. . . .
In support of its contention that August 30, and not August 31, was the last working day of August, 1957, respondent-appellant
invokes Republic Act No. 1880, otherwise known as the "40-Hour Week Law", pursuant to which government offices are to hold
office from Monday to Friday only, unless one of those expressly exempted therefrom.
As correctly held by the court below, the fact that pursuant to Republic Act 1880, the Motor Vehicles Office in Tuguegarao,
Cagayan, had no office on Saturday, Aug. 31, 1957, is immaterial in the case. The last working day contemplated in Sec. 8(I) of
Act 3992 as amended should not necessarily mean the last working day for Motor Vehicle Office. Under Sec. 6(b) of said Act,
providing for payment of registration fees by mail, the date of cancellation of the postage stamps of the envelope containing the
remittance is considered the date of application. Consequently, where the manner of payment falls under said Section 6(b), the
law, in recognizing the date of cancellation as the date of application, impliedly permits of a remittance or payment within that
last day of August that the Post Office may still effect cancellation; and the remittance, in fact, bears a postal cancellation, dated
August 31, 1957. Moreover, it is not pretended by respondent-appellant that the Post Office ceased or has ceased to transact
business and discharge its functions on Saturdays by reason alone of Republic Act No. 1880. Clearly, therefore,the remittance
by petitioner-appellee was within the by law, as provided in Section 8 (I), in connection with Section 6 (b) of Act 3992, as
amended.lawphil.net
The fact that August 31, 1957 was declared a special public holiday by Proclamation No. 437 (dated August 21, 1957) of the
President of the Philippines did not have the effect of making the preceding day, August 30, the last day for paying registration
fees without penalty. On the contrary, Section 31 of the Revised Administrative Code provides
Sec. 31. Pretermission of holiday. Where the day, or the last day, for doing any act required or permitted by law falls on
a holiday, the act may be done the next succeeding business day.
In Calano vs. Cruz, 91 Phil., 247, we ruled as follows:
The complaint filed by the petitioner herein was presented in the court a quo on November 23, 1951, exactly on the eight
day after the proclamation of the respondent as duly elected councilor for the Municipality of Orion, Bataan. It happens,
however, that November 22, 1951, the last day of the seven-day period prescribed by Section 173 of the Revised Election
Code, was declared a "Special Public Holiday For National Thanksgiving" by Proclamation No. 290, series of 1951, of the
President of the Philippines. The trial court held that the provisions of Section 1 of Rule 28 of the Rules of Court could not
be applied to the case at bar because it is an election case (Rule 132, Rules of Court), and declared that the complaint
was filed outside of the period provided for by law. Assuming that Section 1 of Rule 28 of the Rules of Court is not
applicable, the law applicable is Section 31 of the Revised Administrative Code, which provides that "Where the day, or
the last day, for doing any act required or permitted by law falls on a holiday, the act may be done on the next succeeding
business day." The court a quo, therefore, committed an error in declaring that the complaint was filed out of time.
The ruling is on all fours on the issue before us, and against respondent-appellant.

G.R. No. L-32116 April 2l, 1981


RURAL BANK OF CALOOCAN, INC. and JOSE O. DESIDERIO, JR., petitioners,
vs.
THE COURT OF APPEALS and MAXIMA CASTRO, respondents.

DE CASTRO, * J.:
This is a petition for review by way of certiorari of the decision 1 of the Court of Appeals in CA-G.R. No. 39760-R entitled
"Maxima Castro, plaintiff-appellee, versus Severino Valencia, et al., defendants; Rural Bank of Caloocan, Inc., Jose Desiderio,
Jr. and Arsenio Reyes, defendants-appellants," which affirmed in toto the decision of the Court of First Instance of Manila in
favor of plaintiff- appellee, the herein private respondent Maxima Castro.
On December 7, 1959, respondent Maxima Castro, accompanied by Severino Valencia, went to the Rural Bank of Caloocan to
apply for an industrial loan. It was Severino Valencia who arranged everything about the loan with the bank and who supplied to
the latter the personal data required for Castro's loan application. On December 11, 1959, after the bank approved the loan for
the amount of P3,000.00, Castro, accompanied by the Valencia spouses, signed a promissory note corresponding to her loan in
favor of the bank.
On the same day, December 11, 1959, the Valencia spouses obtained from the bank an equal amount of loan for P3,000.00.
They signed a promissory note (Exhibit "2") corresponding to their loan in favor of the bank and had Castro affixed thereon her
signature as co-maker.
The two loans were secured by a real-estate mortgage (Exhibit "6") on Castro's house and lot of 150 square meters, covered by
Transfer Certificate of Title No. 7419 of the Office of the Register of Deeds of Manila.
On February 13, 1961, the sheriff of Manila, thru Acting Chief Deputy Sheriff Basilio Magsambol, sent a notice of sheriff's sale
addressed to Castro, announcing that her property covered by T.C.T. No. 7419 would be sold at public auction on March 10,
1961 to satisfy the obligation covering the two promissory notes plus interest and attorney's fees.
Upon request by Castro and the Valencias and with conformity of the bank, the auction sale that was scheduled for March 10,
1961 was postponed for April 10, 1961. But when April 10, 1961 was subsequently declared a special holiday, the sheriff of
Manila sold the property covered by T.C.T. No. 7419 at a public auction sale that was held on April 11, 1961, which was the next
succeeding business day following the special holiday.
Castro alleged that it was only when she received the letter from the Acting Deputy Sheriff on February 13, 1961, when she
learned for the first time that the mortgage contract (Exhibit "6") which was an encumbrance on her property was for P6.000.00
and not for P3,000.00 and that she was made to sign as co-maker of the promissory note (Exhibit "2") without her being
informed of this.
On April 4, 1961, Castro filed a suit denominated "Re: Sum of Money," against petitioners Bank and Desiderio, the Spouses
Valencia, Basilio Magsambol and Arsenio Reyes as defendants in Civil Case No. 46698 before the Court of First Instance of
Manila upon the charge, amongst others, that thru mistake on her part or fraud on the part of Valencias she was induced to sign
as co-maker of a promissory note (Exhibit "2") and to constitute a mortgage on her house and lot to secure the questioned note.
At the time of filing her complaint, respondent Castro deposited the amount of P3,383.00 with the court a quo in full payment of
her personal loan plus interest.
In her amended complaint, Castro prayed, amongst other, for the annulment as far as she is concerned of the promissory note
(Exhibit "2") and mortgage (Exhibit "6") insofar as it exceeds P3,000.00; for the discharge of her personal obligation with the
bank by reason of a deposit of P3,383.00 with the court a quo upon the filing of her complaint; for the annulment of the
foreclosure sale of her property covered by T.C.T. No. 7419 in favor of Arsenio Reyes; and for the award in her favor of
attorney's fees, damages and cost.
In their answers, petitioners interposed counterclaims and prayed for the dismissal of said complaint, with damages, attorney's
fees and costs. 2
The pertinent facts arrived from the stipulation of facts entered into by the parties as stated by respondent Court of Appeals are
as follows:
Spawning the present litigation are the facts contained in the following stipulation of facts submitted by the parties
themselves:
1. That the capacity and addresses of all the parties in this case are admitted .
2. That the plaintiff was the registered owner of a residential house and lot located at Nos. 1268-1270 Carola Street,
Sampaloc, Manila, containing an area of one hundred fifty (150) square meters, more or less, covered by T.C.T. No.
7419 of the Office of the Register of Deeds of Manila;
3. That the signatures of the plaintiff appearing on the following documents are genuine:
a) Application for Industrial Loan with the Rural Bank of Caloocan, dated December 7, 1959 in the amount of
P3,000.00 attached as Annex A of this partial stipulation of facts;
b) Promissory Note dated December 11, 1959 signed by the plaintiff in favor of the Rural Bank of Caloocan for the
amount of P3,000.00 as per Annex B of this partial stipulation of facts;
c) Application for Industrial Loan with the Rural Bank of Caloocan, dated December 11, 1959, signed only by the
defendants, Severino Valencia and Catalina Valencia, attached as Annex C, of this partial stipulation of facts;
d) Promissory note in favor of the Rural Bank of Caloocan, dated December 11, 1959 for the amount of P3000.00,
signed by the spouses Severino Valencia and Catalina Valencia as borrowers, and plaintiff Maxima Castro, as a comaker, attached as Annex D of this partial stipulation of facts;
e) Real estate mortgage dated December 11, 1959 executed by plaintiff Maxima Castro, in favor of the Rural Bank
of Caloocan, to secure the obligation of P6,000.00 attached herein as Annex E of this partial stipulation of facts;
All the parties herein expressly reserved their right to present any evidence they may desire on the circumstances
regarding the execution of the above-mentioned documents.
4. That the sheriff of Manila, thru Acting Chief Deputy Sheriff, Basilio Magsambol, sent a notice of sheriff's sale,
address to the plaintiff, dated February 13, 1961, announcing that plaintiff's property covered by TCT No. 7419 of
the Register of Deeds of the City of Manila, would be sold at public auction on March 10, 1961 to satisfy the total
obligation of P5,728.50, plus interest, attorney's fees, etc., as evidenced by the Notice of Sheriff's Sale and Notice of
Extrajudicial Auction Sale of the Mortgaged property, attached herewith as Annexes F and F-1, respectively, of this
stipulation of facts;

5. That upon the request of the plaintiff and defendants-spouses Severino Valencia and Catalina Valencia, and with
the conformity of the Rural Bank of Caloocan, the Sheriff of Manila postponed the auction sale scheduled for March
10, 1961 for thirty (30) days and the sheriff re-set the auction sale for April 10, 1961;
6. That April 10, 1961 was declared a special public holiday; (Note: No. 7 is omitted upon agreement of the parties.)
8. That on April 11, 1961, the Sheriff of Manila, sold at public auction plaintiff's property covered by T.C.T. No. 7419
and defendant, Arsenio Reyes, was the highest bidder and the corresponding certificate of sale was issued to him
as per Annex G of this partial stipulation of facts;
9. That on April 16, 1962, the defendant Arsenio Reyes, executed an Affidavit of Consolidation of Ownership, a copy
of which is hereto attached as Annex H of this partial stipulation of facts;
10. That on May 9, 1962, the Rural Bank of Caloocan Incorporated executed the final deed of sale in favor of the
defendant, Arsenio Reyes, in the amount of P7,000.00, a copy of which is attached as Annex I of this partial
stipulation of facts;
11. That the Register of Deeds of the City of Manila issued the Transfer Certificate of Title No. 67297 in favor of the
defendant, Arsenio Reyes, in lieu of Transfer Certificate of Title No. 7419 which was in the name of plaintiff, Maxima
Castro, which was cancelled;
12. That after defendant, Arsenio Reyes, had consolidated his title to the property as per T.C.T. No. 67299, plaintiff
filed a notice of lis pendens with the Register of Deeds of Manila and the same was annotated in the back of T.C.T.
No. 67299 as per Annex J of this partial stipulation of facts; and
13. That the parties hereby reserved their rights to present additional evidence on matters not covered by this partial
stipulation of facts.
WHEREFORE, it is respectfully prayed that the foregoing partial stipulation of facts be approved and admitted by
this Honorable Court.
As for the evidence presented during the trial, We quote from the decision of the Court of Appeals the statement thereof, as
follows:
In addition to the foregoing stipulation of facts, plaintiff claims she is a 70-year old widow who cannot read and write
the English language; that she can speak the Pampango dialect only; that she has only finished second grade
(t.s.n., p. 4, December 11, 1964); that in December 1959, she needed money in the amount of P3,000.00 to invest in
the business of the defendant spouses Valencia, who accompanied her to the defendant bank for the purpose of
securing a loan of P3,000.00; that while at the defendant bank, an employee handed to her several forms already
prepared which she was asked to sign on the places indicated, with no one explaining to her the nature and
contents of the documents; that she did not even receive a copy thereof; that she was given a check in the amount
of P2,882.85 which she delivered to defendant spouses; that sometime in February 1961, she received a letter from
the Acting Deputy Sheriff of Manila, regarding the extrajudicial foreclosure sale of her property; that it was then when
she learned for the first time that the mortgage indebtedness secured by the mortgage on her property was
P6,000.00 and not P3,000.00; that upon investigation of her lawyer, it was found that the papers she was made to
sign were:
(a) Application for a loan of P3,000.00 dated December 7, 1959 (Exh. B-1 and Exh. 1);
(b) Promissory note dated December 11, 1959 for the said loan of P3,000.00 (Exh- B-2);
(c) Promissory note dated December 11, 1959 for P3,000.00 with the defendants Valencia spouses as borrowers
and appellee as co-maker (Exh. B-4 or Exh. 2).
The auction sale set for March 10, 1961 was postponed co April 10, 1961 upon the request of defendant spouses
Valencia who needed more time within which to pay their loan of P3,000.00 with the defendant bank; plaintiff claims
that when she filed the complaint she deposited with the Clerk of Court the sum of P3,383.00 in full payment of her
loan of P3,000.00 with the defendant bank, plus interest at the rate of 12% per annum up to April 3, 1961 (Exh. D).
As additional evidence for the defendant bank, its manager declared that sometime in December, 1959, plaintiff was
brought to the Office of the Bank by an employee- (t.s.n., p 4, January 27, 1966). She wept, there to inquire if she
could get a loan from the bank. The claims he asked the amount and the purpose of the loan and the security to he
given and plaintiff said she would need P3.000.00 to be invested in a drugstore in which she was a partner (t.s.n., p.
811. She offered as security for the loan her lot and house at Carola St., Sampaloc, Manila, which was promptly
investigated by the defendant bank's inspector. Then a few days later, plaintiff came back to the bank with the wife of
defendant Valencia A date was allegedly set for plaintiff and the defendant spouses for the processing of their
application, but on the day fixed, plaintiff came without the defendant spouses. She signed the application and the
other papers pertinent to the loan after she was interviewed by the manager of the defendant. After the application of
plaintiff was made, defendant spouses had their application for a loan also prepared and signed (see Exh. 13). In his
interview of plaintiff and defendant spouses, the manager of the bank was able to gather that plaintiff was in joint
venture with the defendant spouses wherein she agreed to invest P3,000.00 as additional capital in the laboratory
owned by said spouses (t.s.n., pp. 16-17) 3
The Court of Appeals, upon evaluation of the evidence, affirmed in toto the decision of the Court of First Instance of Manila, the
dispositive portion of which reads:
FOR ALL THE FOREGOING CONSIDERATIONS, the Court renders judgment and:
(1) Declares that the promissory note, Exhibit '2', is invalid as against plaintiff herein;
(2) Declares that the contract of mortgage, Exhibit '6', is null and void, in so far as the amount thereof exceeds the
sum of P3,000.00 representing the principal obligation of plaintiff, plus the interest thereon at 12% per annum;
(3) Annuls the extrajudicial foreclosure sale at public auction of the mortgaged property held on April 11, 1961, as
well as all the process and actuations made in pursuance of or in implementation thereto;
(4) Holds that the total unpaid obligation of plaintiff to defendant Rural Bank of Caloocan, Inc., is only the amount of
P3,000.00, plus the interest thereon at 12% per annum, as of April 3, 1961, and orders that plaintiff's deposit of
P3,383.00 in the Office of the Clerk of Court be applied to the payment thereof;
(5) Orders defendant Rural Bank of Caloocan, Inc. to return to defendant Arsenio Reyes the purchase price the
latter paid for the mortgaged property at the public auction, as well as reimburse him of all the expenses he has
incurred relative to the sale thereof;

(6) Orders defendants spouses Severino D. Valencia and Catalina Valencia to pay defendant Rural Bank of
Caloocan, Inc. the amount of P3,000.00 plus the corresponding 12% interest thereon per annum from December 11,
1960 until fully paid; and
Orders defendants Rural Bank of Caloocan, Inc., Jose Desiderio, Jr. and spouses Severino D. Valencia and Catalina
Valencia to pay plaintiff, jointly and severally, the sum of P600.00 by way of attorney's fees, as well as costs.
In view of the conclusion that the court has thus reached, the counterclaims of defendant Rural Bank of Caloocan,
Inc., Jose Desiderio, Jr. and Arsenio Reyes are hereby dismissed, as a corollary
The Court further denies the motion of defendant Arsenio Reyes for an Order requiring Maxima Castro to deposit
rentals filed on November 16, 1963, resolution of which was held in abeyance pending final determination of the
case on the merits, also as a consequence of the conclusion aforesaid. 4
Petitioners Bank and Jose Desiderio moved for the reconsideration 5 of respondent court's decision. The motion having been
denied, 6 they now come before this Court in the instant petition, with the following Assignment of Errors, to wit:
I
THE COURT OF APPEALS ERRED IN UPHOLDING THE PARTIAL ANNULMENT OF THE PROMISSORY NOTE,
EXHIBIT 2, AND THE MORTGAGE, EXHIBIT 6, INSOFAR AS THEY AFFECT RESPONDENT MAXIMA CASTRO
VIS-A-VIS PETITIONER BANK DESPITE THE TOTAL ABSENCE OF EITHER ALLEGATION IN THE COMPLAINT
OR COMPETENT PROOF IN THE EVIDENCE OF ANY FRAUD OR OTHER UNLAWFUL CONDUCT COMMITTED
OR PARTICIPATED IN BY PETITIONERS IN PROCURING THE EXECUTION OF SAID CONTRACTS FROM
RESPONDENT CASTRO.
II
THE COURT OF APPEALS ERRED IN IMPUTING UPON AND CONSIDERING PREJUDICIALLY AGAINST
PETITIONERS, AS BASIS FOR THE PARTIAL ANNULMENT OF THE CONTRACTS AFORESAID ITS FINDING OF
FRAUD PERPETRATED BY THE VALENCIA SPOUSES UPON RESPONDENT CASTRO IN UTTER VIOLATION
OF THE RES INTER ALIOS ACTA RULE.
III
THE COURT OF APPEAL ERRED IN NOT HOLDING THAT, UNDER THE FACTS FOUND BY IT, RESPONDENT
CASTRO IS UNDER ESTOPPEL TO IMPUGN THE REGULARITY AND VALIDITY OF HER QUESTIONED
TRANSACTION WITH PETITIONER BANK.
IV
THE COURT OF APPEALS ERRED IN NOT FINDING THAT, BETWEEN PETITIONERS AND RESPONDENT
CASTRO, THE LATTER SHOULD SUFFER THE CONSEQUENCES OF THE FRAUD PERPETRATED BY THE
VALENCIA SPOUSES, IN AS MUCH AS IT WAS THRU RESPONDENT CASTRO'S NEGLIGENCE OR
ACQUIESCENSE IF NOT ACTUAL CONNIVANCE THAT THE PERPETRATION OF SAID FRAUD WAS MADE
POSSIBLE.
V
THE COURT OF APPEALS ERRED IN UPHOLDING THE VALIDITY OF THE DEPOSIT BY RESPONDENT
CASTRO OF P3,383.00 WITH THE COURT BELOW AS A TENDER AND CONSIGNATION OF PAYMENT
SUFFICIENT TO DISCHARGE SAID RESPONDENT FROM HER OBLIGATION WITH PETITIONER BANK.
VI
THE COURT OF APPEALS ERRED IN NOT DECLARING AS VALID AND BINDING UPON RESPONDENT
CASTRO THE HOLDING OF THE SALE ON FORECLOSURE ON THE BUSINESS DAY NEXT FOLLOWING THE
ORIGINALLY SCHEDULED DATE THEREFOR WHICH WAS DECLARED A HOLIDAY WITHOUT NECESSITY OF
FURTHER NOTICE THEREOF.
The issue raised in the first three (3) assignment of errors is whether or not respondent court correctly affirmed the lower court
in declaring the promissory note (Exhibit 2) invalid insofar as they affect respondent Castro vis-a-vis petitioner bank, and the
mortgage contract (Exhibit 6) valid up to the amount of P3,000.00 only.
Respondent court declared that the consent of Castro to the promissory note (Exhibit 2) where she signed as co-maker with the
Valencias as principal borrowers and her acquiescence to the mortgage contract (Exhibit 6) where she encumbered her
property to secure the amount of P6,000.00 was obtained by fraud perpetrated on her by the Valencias who had abused her
confidence, taking advantage of her old age and ignorance of her financial need. Respondent court added that "the mandate of
fair play decrees that she should be relieved of her obligation under the contract" pursuant to Articles 24 7 and 1332 8 of the Civil
Code.
The decision in effect relieved Castro of any liability to the promissory note (Exhibit 2) and the mortgage contract (Exhibit 6) was
deemed valid up to the amount of P3,000.00 only which was equivalent to her personal loan to the bank.
Petitioners argued that since the Valencias were solely declared in the decision to be responsible for the fraud against Castro, in
the light of the res inter alios acta rule, a finding of fraud perpetrated by the spouses against Castro cannot be taken to operate
prejudicially against the bank. Petitioners concluded that respondent court erred in not giving effect to the promissory note
(Exhibit 2) insofar as they affect Castro and the bank and in declaring that the mortgage contract (Exhibit 6) was valid only to
the extent of Castro's personal loan of P3,000.00.
The records of the case reveal that respondent court's findings of fraud against the Valencias is well supported by evidence.
Moreover, the findings of fact by respondent court in the matter is deemed final. 9 The decision declared the Valencias solely
responsible for the defraudation of Castro. Petitioners' contention that the decision was silent regarding the participation of the
bank in the fraud is, therefore, correct.
We cannot agree with the contention of petitioners that the bank was defrauded by the Valencias. For one, no claim was made
on this in the lower court. For another, petitioners did not submit proof to support its contention.
At any rate, We observe that while the Valencias defrauded Castro by making her sign the promissory note (Exhibit 2) and the
mortgage contract (Exhibit 6), they also misrepresented to the bank Castro's personal qualifications in order to secure its
consent to the loan. This must be the reason which prompted the bank to contend that it was defrauded by the Valencias. But to
reiterate, We cannot agree with the contention for reasons above-mentioned. However, if the contention deserves any
consideration at all, it is in indicating the admission of petitioners that the bank committed mistake in giving its consent to the
contracts.
Thus, as a result of the fraud upon Castro and the misrepresentation to the bank inflicted by the Valencias both Castro and the
bank committed mistake in giving their consents to the contracts. In other words, substantial mistake vitiated their consents

given. For if Castro had been aware of what she signed and the bank of the true qualifications of the loan applicants, it is
evident that they would not have given their consents to the contracts.
Pursuant to Article 1342 of the Civil Code which provides:
Art. 1342. Misrepresentation by a third person does not vitiate consent, unless such misrepresentation has created
substantial mistake and the same is mutual.
We cannot declare the promissory note (Exhibit 2) valid between the bank and Castro and the mortgage contract (Exhibit 6)
binding on Castro beyond the amount of P3,000.00, for while the contracts may not be invalidated insofar as they affect the
bank and Castro on the ground of fraud because the bank was not a participant thereto, such may however be invalidated on
the ground of substantial mistake mutually committed by them as a consequence of the fraud and misrepresentation inflicted by
the Valencias. Thus, in the case of Hill vs. Veloso, 10this Court declared that a contract may be annulled on the ground of vitiated
consent if deceit by a third person, even without connivance or complicity with one of the contracting parties, resulted in mutual
error on the part of the parties to the contract.
Petitioners argued that the amended complaint fails to contain even a general averment of fraud or mistake, and its mention in
the prayer is definitely not a substantial compliance with the requirement of Section 5, Rule 8 of the Rules of Court. The records
of the case, however, will show that the amended complaint contained a particular averment of fraud against the Valencias in
full compliance with the provision of the Rules of Court. Although, the amended complaint made no mention of mistake being
incurred in by the bank and Castro, such mention is not essential in order that the promissory note (Exhibit 2) may be declared
of no binding effect between them and the mortgage (Exhibit 6) valid up to the amount of P3,000.00 only. The reason is that the
mistake they mutually suffered was a mere consequence of the fraud perpetrated by the Valencias against them. Thus, the
fraud particularly averred in the complaint, having been proven, is deemed sufficient basis for the declaration of the promissory
note (Exhibit 2) invalid insofar as it affects Castro vis-a-vis the bank, and the mortgage contract (Exhibit 6) valid only up to the
amount of P3,000.00.
The second issue raised in the fourth assignment of errors is who between Castro and the bank should suffer the
consequences of the fraud perpetrated by the Valencias.
In attributing to Castro an consequences of the loss, petitioners argue that it was her negligence or acquiescence if not her
actual connivance that made the fraud possible.
Petitioners' argument utterly disregards the findings of respondent Court of Appeals wherein petitioners' negligence in the
contracts has been aptly demonstrated, to wit:
A witness for the defendant bank, Rodolfo Desiderio claims he had subjected the plaintiff-appellee to several
interviews. If this were true why is it that her age was placed at 61 instead of 70; why was she described in the
application (Exh. B-1-9) as drug manufacturer when in fact she was not; why was it placed in the application that she
has income of P20,000.00 when according to plaintiff-appellee, she his not even given such kind of information -the
true fact being that she was being paid P1.20 per picul of the sugarcane production in her hacienda and 500 cavans
on the palay production. 11
From the foregoing, it is evident that the bank was as much , guilty as Castro was, of negligence in giving its consent to the
contracts. It apparently relied on representations made by the Valencia spouses when it should have directly obtained the
needed data from Castro who was the acknowledged owner of the property offered as collateral. Moreover, considering Castro's
personal circumstances her lack of education, ignorance and old age she cannot be considered utterly neglectful for having
been defrauded. On the contrary, it is demanded of petitioners to exercise the highest order of care and prudence in its
business dealings with the Valencias considering that it is engaged in a banking business a business affected with public
interest. It should have ascertained Castro's awareness of what she was signing or made her understand what obligations she
was assuming, considering that she was giving accommodation to, without any consideration from the Valencia spouses.
Petitioners further argue that Castro's act of holding the Valencias as her agent led the bank to believe that they were
authorized to speak and bind her. She cannot now be permitted to deny the authority of the Valencias to act as her agent for
one who clothes another with apparent authority as her agent is not permitted to deny such authority.
The authority of the Valencias was only to follow-up Castro's loan application with the bank. They were not authorized to borrow
for her. This is apparent from the fact that Castro went to the Bank to sign the promissory note for her loan of P3,000.00. If her
act had been understood by the Bank to be a grant of an authority to the Valencia to borrow in her behalf, it should have
required a special power of attorney executed by Castro in their favor. Since the bank did not, We can rightly assume that it did
not entertain the notion, that the Valencia spouses were in any manner acting as an agent of Castro.
When the Valencias borrowed from the Bank a personal loan of P3,000.00 evidenced by a promissory note (Exhibit 2) and
mortgaged (Exhibit 6) Castro's property to secure said loan, the Valencias acted for their own behalf. Considering however that
for the loan in which the Valencias appeared as principal borrowers, it was the property of Castro that was being mortgaged to
secure said loan, the Bank should have exercised due care and prudence by making proper inquiry if Castro's consent to the
mortgage was without any taint or defect. The possibility of her not knowing that she signed the promissory note (Exhibit 2) as
co-maker with the Valencias and that her property was mortgaged to secure the two loans instead of her own personal loan
only, in view of her personal circumstances ignorance, lack of education and old age should have placed the Bank on
prudent inquiry to protect its interest and that of the public it serves. With the recent occurrence of events that have supposedly
affected adversely our banking system, attributable to laxity in the conduct of bank business by its officials, the need of extreme
caution and prudence by said officials and employees in the discharge of their functions cannot be over-emphasized.
Question is, likewise, raised as to the propriety of respondent court's decision which declared that Castro's consignation in court
of the amount of P3,383.00 was validly made. It is contended that the consignation was made without prior offer or tender of
payment to the Bank, and it therefore, not valid. In holding that there is a substantial compliance with the provision of Article
1256 of the Civil Code, respondent court considered the fact that the Bank was holding Castro liable for the sum of P6,000.00
plus 12% interest per annum, while the amount consigned was only P3,000.00 plus 12% interest; that at the time of
consignation, the Bank had long foreclosed the mortgage extrajudicially and the sale of the mortgage property had already been
scheduled for April 10, 1961 for non-payment of the obligation, and that despite the fact that the Bank already knew of the
deposit made by Castro because the receipt of the deposit was attached to the record of the case, said Bank had not made any
claim of such deposit, and that therefore, Castro was right in thinking that it was futile and useless for her to make previous offer
and tender of payment directly to the Bank only in the aforesaid amount of P3,000.00 plus 12% interest. Under the foregoing
circumstances, the consignation made by Castro was valid. if not under the strict provision of the law, under the more liberal
considerations of equity.
The final issue raised is the validity or invalidity of the extrajudicial foreclosure sale at public auction of the mortgaged property
that was held on April 11, 1961.
Petitioners contended that the public auction sale that was held on April 11, 1961 which was the next business day after the
scheduled date of the sale on April 10, 1961, a special public holiday, was permissible and valid pursuant to the provisions of
Section 31 of the Revised Administrative Code which ordains:
Pretermission of holiday. Where the day, or the last day, for doing any act required or permitted by law falls on a
holiday, the act may be done on the next succeeding business day.

Respondent court ruled that the aforesaid sale is null and void, it not having been carried out in accordance with Section 9 of
Act No. 3135, which provides:
Section 9. Notice shall be given by posting notices of the sale for not less than twenty days in at least three public
places of the municipality or city where the property is situated, and if such property is worth more than four hundred
pesos, such notice shall also be published once a week for at least three consecutive weeks in a newspaper of
general circulation in the municipality or city.
We agree with respondent court. The pretermission of a holiday applies only "where the day, or the last day for doing any
act required or permitted by law falls on a holiday," or when the last day of a given period for doing an act falls on a holiday. It
does not apply to a day fixed by an office or officer of the government for an act to be done, as distinguished from a period of
time within which an act should be done, which may be on any day within that specified period. For example, if a party is
required by law to file his answer to a complaint within fifteen (15) days from receipt of the summons and the last day falls on a
holiday, the last day is deemed moved to the next succeeding business day. But, if the court fixes the trial of a case on a certain
day but the said date is subsequently declared a public holiday, the trial thereof is not automatically transferred to the next
succeeding business day. Since April 10, 1961 was not the day or the last day set by law for the extrajudicial foreclosure sale,
nor the last day of a given period but a date fixed by the deputy sheriff, the aforesaid sale cannot legally be made on the next
succeeding business day without the notices of the sale on that day being posted as prescribed in Section 9, Act No. 3135.
WHEREFORE, finding no reversible error in the judgment under review, We affirm the same in toto. No pronouncement as to
cost.

G.R. No. L-6791

March 29, 1954

THE PEOPLE OF THE PHILIPPINES, plaintiff-appellee,


vs.
QUE PO LAY, defendant-appellant.
Prudencio de Guzman for appellant.
First Assistant Solicitor General Ruperto Kapunan, Jr., and Solicitor Lauro G. Marquez for appellee.
MONTEMAYOR, J.:
Que Po Lay is appealing from the decision of the Court of First Instance of Manila, finding him guilty of violating Central Bank
Circular No. 20 in connection with section 34 of Republic Act No. 265, and sentencing him to suffer six months imprisonment, to
pay a fine of P1,000 with subsidiary imprisonment in case of insolvency, and to pay the costs.
The charge was that the appellant who was in possession of foreign exchange consisting of U.S. dollars, U.S. checks and U.S.
money orders amounting to about $7,000 failed to sell the same to the Central Bank through its agents within one day following
the receipt of such foreign exchange as required by Circular No. 20. the appeal is based on the claim that said circular No. 20
was not published in the Official Gazette prior to the act or omission imputed to the appellant, and that consequently, said
circular had no force and effect. It is contended that Commonwealth Act. No., 638 and Act 2930 both require said circular to be
published in the Official Gazette, it being an order or notice of general applicability. The Solicitor General answering this
contention says that Commonwealth Act. No. 638 and 2930 do not require the publication in the Official Gazette of said circular
issued for the implementation of a law in order to have force and effect.
We agree with the Solicitor General that the laws in question do not require the publication of the circulars, regulations and
notices therein mentioned in order to become binding and effective. All that said two laws provide is that laws, resolutions,
decisions of the Supreme Court and Court of Appeals, notices and documents required by law to be of no force and effect. In
other words, said two Acts merely enumerate and make a list of what should be published in the Official Gazette, presumably,
for the guidance of the different branches of the Government issuing same, and of the Bureau of Printing.
However, section 11 of the Revised Administrative Code provides that statutes passed by Congress shall, in the absence of
special provision, take effect at the beginning of the fifteenth day after the completion of the publication of the statute in the
Official Gazette. Article 2 of the new Civil Code (Republic Act No. 386) equally provides that laws shall take effect after fifteen
days following the completion of their publication in the Official Gazette, unless it is otherwise provided. It is true that Circular
No. 20 of the Central Bank is not a statute or law but being issued for the implementation of the law authorizing its issuance, it
has the force and effect of law according to settled jurisprudence. (See U.S. vs. Tupasi Molina, 29 Phil., 119 and authorities
cited therein.) Moreover, as a rule, circulars and regulations especially like the Circular No. 20 of the Central Bank in question
which prescribes a penalty for its violation should be published before becoming effective, this, on the general principle and
theory that before the public is bound by its contents, especially its penal provisions, a law, regulation or circular must first be
published and the people officially and specifically informed of said contents and its penalties.
Our Old Civil code, ( Spanish Civil Code of 1889) has a similar provision about the effectivity of laws, (Article 1 thereof), namely,
that laws shall be binding twenty days after their promulgation, and that their promulgation shall be understood as made on the
day of the termination of the publication of the laws in the Gazette. Manresa, commenting on this article is of the opinion that the
word "laws" include regulations and circulars issued in accordance with the same. He says:
El Tribunal Supremo, ha interpretado el articulo 1. del codigo Civil en Sentencia de 22 de Junio de 1910, en el sentido de
que bajo la denominacion generica de leyes, se comprenden tambien los Reglamentos, Reales decretos,
Instrucciones, Circulares y Reales ordenes dictadas de conformidad con las mismas por el Gobierno en uso de su
potestad. Tambien el poder ejecutivo lo ha venido entendiendo asi, como lo prueba el hecho de que muchas de sus
disposiciones contienen la advertencia de que empiezan a regir el mismo dia de su publicacion en la Gaceta, advertencia
que seria perfectamente inutil si no fuera de aplicacion al caso el articulo 1.o del Codigo Civil. (Manresa, Codigo Civil
Espaol, Vol. I. p. 52).
In the present case, although circular No. 20 of the Central Bank was issued in the year 1949, it was not published until
November 1951, that is, about 3 months after appellant's conviction of its violation. It is clear that said circular, particularly its
penal provision, did not have any legal effect and bound no one until its publication in the Official Gazzette or after November
1951. In other words, appellant could not be held liable for its violation, for it was not binding at the time he was found to have
failed to sell the foreign exchange in his possession thereof.
But the Solicitor General also contends that this question of non-publication of the Circular is being raised for the first time on
appeal in this Court, which cannot be done by appellant. Ordinarily, one may raise on appeal any question of law or fact that has
been raised in the court below and which is within the issues made by the parties in their pleadings. (Section 19, Rule 48 of the
Rules of Court). But the question of non-publication is fundamental and decisive. If as a matter of fact Circular No. 20 had not
been published as required by law before its violation, then in the eyes of the law there was no such circular to be violated and
consequently appellant committed no violation of the circular or committed any offense, and the trial court may be said to have
had no jurisdiction. This question may be raised at any stage of the proceeding whether or not raised in the court below.
In view of the foregoing, we reverse the decision appealed from and acquit the appellant, with costs de oficio.

G.R. No. 137873

April 20, 2001

D. M. CONSUNJI, INC., petitioner,


vs.
COURT OF APPEALS and MARIA J. JUEGO, respondents.
KAPUNAN, J.:
At around 1:30 p.m., November 2, 1990, Jose Juego, a construction worker of D. M. Consunji, Inc., fell 14 floors from the
Renaissance Tower, Pasig City to his death.
PO3 Rogelio Villanueva of the Eastern Police District investigated the tragedy and filed a report dated November 25, 1990,
stating that:
x x x. [The] [v]ictim was rushed to [the] Rizal Medical Center in Pasig, Metro Manila where he was pronounced dead on
arrival (DOA) by the attending physician, Dr. Errol de Yzo[,] at around 2:15 p.m. of the same date.
Investigation disclosed that at the given time, date and place, while victim Jose A. Juego together with Jessie Jaluag and
Delso Destajo [were] performing their work as carpenter[s] at the elevator core of the 14 th floor of the Tower D,
Renaissance Tower Building on board a [p]latform made of channel beam (steel) measuring 4.8 meters by 2 meters wide
with pinulid plywood flooring and cable wires attached to its four corners and hooked at the 5 ton chain block, when
suddenly, the bolt or pin which was merely inserted to connect the chain block with the [p]latform, got loose xxx causing
the whole [p]latform assembly and the victim to fall down to the basement of the elevator core, Tower D of the building
under construction thereby crushing the victim of death, save his two (2) companions who luckily jumped out for safety.
It is thus manifest that Jose A. Juego was crushed to death when the [p]latform he was then on board and performing
work, fell. And the falling of the [p]latform was due to the removal or getting loose of the pin which was merely inserted to
the connecting points of the chain block and [p]latform but without a safety lock. 1
On May 9, 1991, Jose Juegos widow, Maria, filed in the Regional Trial Court (RTC) of Pasig a complaint for damages against
the deceaseds employer, D.M. Consunji, Inc. The employer raised, among other defenses, the widows prior availment of the
benefits from the State Insurance Fund.
After trial, the RTC rendered a decision in favor of the widow Maria Juego. The dispositive portion of the RTC decision reads:
WHEREFORE, judgment is hereby rendered ordering defendant to pay plaintiff, as follows:
1. P50,000.00 for the death of Jose A. Juego.
2. P10,000.00 as actual and compensatory damages.
3. P464,000.00 for the loss of Jose A. Juegos earning capacity.
4. P100,000.00 as moral damages.
5. P20,000.00 as attorneys fees, plus the costs of suit.
SO ORDERED.2
On appeal by D. M. Consunji, the Court of Appeals (CA) affirmed the decision of the RTC in toto.
D. M. Consunji now seeks the reversal of the CA decision on the following grounds:

THE APPELLATE COURT ERRED IN HOLDING THAT THE POLICE REPORT WAS ADMISSIBLE EVIDENCE OF
THE ALLEGED NEGLIGENCE OF PETITIONER.

THE APPELLATE COURT ERRED IN HOLDING THAT THE DOCTRINE OF RES IPSA LOQUITOR [sic] IS
APPLICABLE TO PROVE NEGLIGENCE ON THE PART OF PETITIONER.

THE APPELLATE COURT ERRED IN HOLDING THAT PETITIONER IS PRESUMED NEGLIGENT UNDER
ARTICLE 2180 OF THE CIVIL CODE, AND

THE APPELLATE COURT ERRED IN HOLDING THAT RESPONDENT IS NOT PRECLUDED FROM
RECOVERING DAMAGES UNDER THE CIVIL CODE.3

Petitioner maintains that the police report reproduced above is hearsay and, therefore, inadmissible. The CA ruled otherwise. It
held that said report, being an entry in official records, is an exception to the hearsay rule.
The Rules of Court provide that a witness can testify only to those facts which he knows of his personal knowledge, that is,
which are derived from his perception.4 A witness, therefore, may not testify as what he merely learned from others either
because he was told or read or heard the same. Such testimony is considered hearsay and may not be received as proof of the
truth of what he has learned.5 This is known as the hearsay rule.
Hearsay is not limited to oral testimony or statements; the general rule that excludes hearsay as evidence applies to written, as
well as oral statements.6
The theory of the hearsay rule is that the many possible deficiencies, suppressions, sources of error and untrustworthiness,
which lie underneath the bare untested assertion of a witness, may be best brought to light and exposed by the test of crossexamiantion.7 The hearsay rule, therefore, excludes evidence that cannot be tested by cross-examination. 8
The Rules of Court allow several exceptions to the rule, 9 among which are entries in official records. Section 44, Rule 130
provides:
Entries in official records made in the performance of his duty made in the performance of his duty by a public officer of
the Philippines, or by a person in the performance of a duty specially enjoined by law areprima facie evidence of the facts
therein stated.
In Africa, et al. vs. Caltex (Phil.), Inc., et al.,10 this Court, citing the work of Chief Justice Moran, enumerated the requisites for
admissibility under the above rule:
(a) that the entry was made by a public officer or by another person specially enjoined by law to do so;
(b) that it was made by the public officer in the performance of his duties, or by such other person in the performance of a
duty specially enjoined by law; and
(c) that the public officer or other person had sufficient knowledge of the facts by him stated, which must have been
acquired by him personally or through official information.
The CA held that the police report meets all these requisites. Petitioner contends that the last requisite is not present.

The Court notes that PO3 Villanueva, who signed the report in question, also testified before the trial court. InRodriguez vs.
Court of Appeals,11 which involved a Fire Investigation Report, the officer who signed the fire report also testified before the trial
court. This Court held that the report was inadmissible for the purpose of proving the truth of the statements contained in the
report but admissible insofar as it constitutes part of the testimony of the officer who executed the report.
x x x. Since Major Enriquez himself took the witness stand and was available for cross-examination, the portions of the
report which were of his personal knowledge or which consisted of his perceptions and conclusions were not hearsay. The
rest of the report, such as the summary of the statements of the parties based on their sworn statements (which were
annexed to the Report) as well as the latter, having been included in the first purpose of the offer [as part of the testimony
of Major Enriquez], may then be considered as independently relevant statements which were gathered in the course of
the investigation and may thus be admitted as such, but not necessarily to prove the truth thereof. It has been said that:
"Where regardless of the truth or falsity of a statement, the fact that it has been made is relevant, the hearsay rule
does not apply, but the statement may be shown. Evidence as to the making of such statement is not secondary but
primary, for the statement itself may constitute a fact in issue, or be circumstantially relevant as to the existence of
such a fact."
When Major Enriquez took the witness stand, testified for petitioners on his Report and made himself available for crossexamination by the adverse party, the Report, insofar as it proved that certain utterances were made (but not their truth),
was effectively removed from the ambit of the aforementioned Section 44 of Rule 130. Properly understood, this section
does away with the testimony in open court of the officer who made the official record, considers the matter as an
exception to the hearsay rule and makes the entries in said official record admissible in evidence as prima facie evidence
of the facts therein stated. The underlying reasons for this exceptionary rule are necessity and trustworthiness, as
explained in Antillon v. Barcelon.
The litigation is unlimited in which testimony by officials is daily needed; the occasions in which the officials would be
summoned from his ordinary duties to declare as a witness are numberless. The public officers are few in whose
daily work something is not done in which testimony is not needed from official sources. Were there no exception for
official statements, hosts of officials would be found devoting the greater part of their time to attending as witnesses
in court or delivering deposition before an officer. The work of administration of government and the interest of the
public having business with officials would alike suffer in consequence. For these reasons, and for many others, a
certain verity is accorded such documents, which is not extended to private documents. (3 Wigmore on Evidence,
Sec. 1631).
The law reposes a particular confidence in public officers that it presumes they will discharge their several trusts with
accuracy and fidelity; and, therefore, whatever acts they do in discharge of their duty may be given in evidence and
shall be taken to be true under such a degree of caution as to the nature and circumstances of each case may
appear to require.
It would have been an entirely different matter if Major Enriquez was not presented to testify on his report. In that case the
applicability of Section 44 of Rule 143 would have been ripe for determination, and this Court would have agreed with the
Court of Appeals that said report was inadmissible since the aforementioned third requisite was not satisfied. The
statements given by the sources of information of Major Enriquez failed to qualify as "official information," there being no
showing that, at the very least, they were under a duty to give the statements for record.
Similarly, the police report in this case is inadmissible for the purpose of proving the truth of the statements contained therein
but is admissible insofar as it constitutes part of the testimony of PO3 Villanueva.
In any case, the Court holds that portions of PO3 Villanuevas testimony which were of his personal knowledge suffice to prove
that Jose Juego indeed died as a result of the elevator crash. PO3 Villanueva had seen Juegos remains at the
morgue,12 making the latters death beyond dispute. PO3 Villanueva also conducted an ocular inspection of the premises of the
building the day after the incident13 and saw the platform for himself.14 He observed that the platform was crushed15 and that it
was totally damaged.16 PO3 Villanueva also required Garcia and Fabro to bring the chain block to the police headquarters.
Upon inspection, he noticed that the chain was detached from the lifting machine, without any pin or bolt. 17
What petitioner takes particular exception to is PO3 Villanuevas testimony that the cause of the fall of the platform was the
loosening of the bolt from the chain block. It is claimed that such portion of the testimony is mere opinion. Subject to certain
exceptions,18 the opinion of a witness is generally not admissible. 19
Petitioners contention, however, loses relevance in the face of the application of res ipsa loquitur by the CA. The effect of the
doctrine is to warrant a presumption or inference that the mere fall of the elevator was a result of the person having charge of
the instrumentality was negligent. As a rule of evidence, the doctrine of res ipsa loquituris peculiar to the law of negligence
which recognizes that prima facie negligence may be established without direct proof and furnishes a substitute for specific
proof of negligence.20
The concept of res ipsa loquitur has been explained in this wise:
While negligence is not ordinarily inferred or presumed, and while the mere happening of an accident or injury will not
generally give rise to an inference or presumption that it was due to negligence on defendants part, under the doctrine of
res ipsa loquitur, which means, literally, the thing or transaction speaks for itself, or in one jurisdiction, that the thing or
instrumentality speaks for itself, the facts or circumstances accompanying an injury may be such as to raise a
presumption, or at least permit an inference of negligence on the part of the defendant, or some other person who is
charged with negligence.
x x x where it is shown that the thing or instrumentality which caused the injury complained of was under the control or
management of the defendant, and that the occurrence resulting in the injury was such as in the ordinary course of things
would not happen if those who had its control or management used proper care, there is sufficient evidence, or, as
sometimes stated, reasonable evidence, in the absence of explanation by the defendant, that the injury arose from or was
caused by the defendants want of care.21
One of the theoretical based for the doctrine is its necessity, i.e., that necessary evidence is absent or not available. 22
The res ipsa loquitur doctrine is based in part upon the theory that the defendant in charge of the instrumentality which
causes the injury either knows the cause of the accident or has the best opportunity of ascertaining it and that the plaintiff
has no such knowledge, and therefore is compelled to allege negligence in general terms and to rely upon the proof of the
happening of the accident in order to establish negligence. The inference which the doctrine permits is grounded upon the
fact that the chief evidence of the true cause, whether culpable or innocent, is practically accessible to the defendant but
inaccessible to the injured person.
It has been said that the doctrine of res ipsa loquitur furnishes a bridge by which a plaintiff, without knowledge of the
cause, reaches over to defendant who knows or should know the cause, for any explanation of care exercised by the
defendant in respect of the matter of which the plaintiff complains. The res ipsa loquitur doctrine, another court has said, is
a rule of necessity, in that it proceeds on the theory that under the peculiar circumstances in which the doctrine is
applicable, it is within the power of the defendant to show that there was no negligence on his part, and direct proof of
defendants negligence is beyond plaintiffs power. Accordingly, some court add to the three prerequisites for the
application of the res ipsa loquitur doctrine the further requirement that for the res ipsa loquitur doctrine to apply, it must

appear that the injured party had no knowledge or means of knowledge as to the cause of the accident, or that the party to
be charged with negligence has superior knowledge or opportunity for explanation of the accident. 23
The CA held that all the requisites of res ipsa loquitur are present in the case at bar:
There is no dispute that appellees husband fell down from the 14 th floor of a building to the basement while he was
working with appellants construction project, resulting to his death. The construction site is within the exclusive control
and management of appellant. It has a safety engineer, a project superintendent, a carpenter leadman and others who are
in complete control of the situation therein. The circumstances of any accident that would occur therein are peculiarly
within the knowledge of the appellant or its employees. On the other hand, the appellee is not in a position to know what
caused the accident. Res ipsa loquitur is a rule of necessity and it applies where evidence is absent or not readily
available, provided the following requisites are present: (1) the accident was of a kind which does not ordinarily occur
unless someone is negligent; (2) the instrumentality or agency which caused the injury was under the exclusive control of
the person charged with negligence; and (3) the injury suffered must not have been due to any voluntary action or
contribution on the part of the person injured. x x x.
No worker is going to fall from the 14th floor of a building to the basement while performing work in a construction site
unless someone is negligent[;] thus, the first requisite for the application of the rule of res ipsa loquitur is present. As
explained earlier, the construction site with all its paraphernalia and human resources that likely caused the injury is under
the exclusive control and management of appellant[;] thus[,] the second requisite is also present. No contributory
negligence was attributed to the appellees deceased husband[;] thus[,] the last requisite is also present. All the requisites
for the application of the rule of res ipsa loquitur are present, thus a reasonable presumption or inference of appellants
negligence arises. x x x.24
Petitioner does not dispute the existence of the requisites for the application of res ipsa loquitur, but argues that the
presumption or inference that it was negligent did not arise since it "proved that it exercised due care to avoid the accident
which befell respondents husband."
Petitioner apparently misapprehends the procedural effect of the doctrine. As stated earlier, the defendants negligence is
presumed or inferred25 when the plaintiff establishes the requisites for the application of res ipsa loquitur. Once the plaintiff
makes out a prima facie case of all the elements, the burden then shifts to defendant to explain. 26 The presumption or inference
may be rebutted or overcome by other evidence and, under appropriate circumstances disputable presumption, such as that of
due care or innocence, may outweigh the inference. 27 It is not for the defendant to explain or prove its defense to prevent the
presumption or inference from arising. Evidence by the defendant of say, due care, comes into play only after the circumstances
for the application of the doctrine has been established.1wphi1.nt
In any case, petitioner cites the sworn statement of its leadman Ferdinand Fabro executed before the police investigator as
evidence of its due care. According to Fabros sworn statement, the company enacted rules and regulations for the safety and
security of its workers. Moreover, the leadman and the bodegero inspect the chain block before allowing its use.
It is ironic that petitioner relies on Fabros sworn statement as proof of its due care but, in arguing that private respondent failed
to prove negligence on the part of petitioners employees, also assails the same statement for being hearsay.
Petitioner is correct. Fabros sworn statement is hearsay and inadmissible. Affidavits are inadmissible as evidence under the
hearsay rule, unless the affiant is placed on the witness stand to testify thereon. 28 The inadmissibility of this sort of evidence is
based not only on the lack of opportunity on the part of the adverse party to cross-examine the affiant, but also on the
commonly known fact that, generally, an affidavit is not prepared by the affiant himself but by another who uses his own
language in writing the affiants statements which may either be omitted or misunderstood by the one writing them. 29 Petitioner,
therefore, cannot use said statement as proof of its due care any more than private respondent can use it to prove the cause of
her husbands death. Regrettably, petitioner does not cite any other evidence to rebut the inference or presumption of
negligence arising from the application of res ipsa loquitur, or to establish any defense relating to the incident.
Next, petitioner argues that private respondent had previously availed of the death benefits provided under the Labor Code and
is, therefore, precluded from claiming from the deceaseds employer damages under the Civil Code.
Article 173 of the Labor Code states:
Article 173. Extent of liability. Unless otherwise provided, the liability of the State Insurance Fund under this Title shall be
exclusive and in place of all other liabilities of the employer to the employee, his dependents or anyone otherwise entitled
to receive damages on behalf of the employee or his dependents. The payment of compensation under this Title shall not
bar the recovery of benefits as provided for in Section 699 of the Revised Administrative Code, Republic Act Numbered
Eleven hundred sixty-one, as amended, Republic Act Numbered Six hundred ten, as amended, Republic Act Numbered
Forty-eight hundred sixty-four as amended, and other laws whose benefits are administered by the System or by other
agencies of the government.
The precursor of Article 173 of the Labor Code, Section 5 of the Workmens Compensation Act, provided that:
Section 5. Exclusive right to compensation. The rights and remedies granted by this Act to an employee by reason of a
personal injury entitling him to compensation shall exclude all other rights and remedies accruing to the employee, his
personal representatives, dependents or nearest of kin against the employer under the Civil Code and other laws because
of said injury x x x.
Whether Section 5 of the Workmens Compensation Act allowed recovery under said Act as well as under the Civil Code used to
be the subject of conflicting decisions. The Court finally settled the matter in Floresca vs.Philex Mining Corporation,30 which
involved a cave-in resulting in the death of the employees of the Philex Mining Corporation. Alleging that the mining corporation,
in violation of government rules and regulations, failed to take the required precautions for the protection of the employees, the
heirs of the deceased employees filed a complaint against Philex Mining in the Court of First Instance (CFI). Upon motion of
Philex Mining, the CFI dismissed the complaint for lack of jurisdiction. The heirs sought relief from this Court.
Addressing the issue of whether the heirs had a choice of remedies, majority of the Court En Banc,31 following the rule
in Pacaa vs. Cebu Autobus Company, held in the affirmative.
WE now come to the query as to whether or not the injured employee or his heirs in case of death have a right of selection
or choice of action between availing themselves of the workers right under the Workmens Compensation Act and suing in
the regular courts under the Civil Code for higher damages (actual, moral and exemplary) from the employers by virtue of
the negligence or fault of the employers or whether they may avail themselves cumulatively of both actions, i.e., collect the
limited compensation under the Workmens Compensation Act and sue in addition for damages in the regular courts.
In disposing of a similar issue, this Court in Pacaa vs. Cebu Autobus Company, 32 SCRA 442, ruled thatan injured
worker has a choice of either to recover from the employer the fixed amounts set by the Workmens Compensation Act or
to prosecute an ordinary civil action against the tortfeasor for higher damages but he cannot pursue both courses of action
simultaneously. [Underscoring supplied.]
Nevertheless, the Court allowed some of the petitioners in said case to proceed with their suit under the Civil Code despite
having availed of the benefits provided under the Workmens Compensation Act. The Court reasoned:

With regard to the other petitioners, it was alleged by Philex in its motion to dismiss dated May 14, 1968 before the court a
quo, that the heirs of the deceased employees, namely Emerito Obra, Larry Villar, Jr., Aurelio Lanuza, Lorenzo Isla and
Saturnino submitted notices and claims for compensation to the Regional Office No. 1 of the then Department of Labor
and all of them have been paid in full as of August 25, 1967, except Saturnino Martinez whose heirs decided that they be
paid in installments x x x. Such allegation was admitted by herein petitioners in their opposition to the motion to dismiss
dated may 27, 1968 x x x in the lower court, but they set up the defense that the claims were filed under the Workmens
Compensation Act before they learned of the official report of the committee created to investigate the accident which
established the criminal negligence and violation of law by Philex, and which report was forwarded by the Director of
Mines to then Executive Secretary Rafael Salas in a letter dated October 19, 1967 only x x x.
WE hold that although the other petitioners had received the benefits under the Workmens Compensation Act, such my
not preclude them from bringing an action before the regular court because they became cognizant of the fact that Philex
has been remiss in its contractual obligations with the deceased miners only after receiving compensation under the Act.
Had petitioners been aware of said violation of government rules and regulations by Philex, and of its negligence, they
would not have sought redress under the Workmens Compensation Commission which awarded a lesser amount for
compensation. The choice of the first remedy was based on ignorance or a mistake of fact, which nullifies the choice as it
was not an intelligent choice. The case should therefore be remanded to the lower court for further proceedings. However,
should the petitioners be successful in their bid before the lower court, the payments made under the Workmens
Compensation Act should be deducted from the damages that may be decreed in their favor. [Underscoring supplied.]
The ruling in Floresca providing the claimant a choice of remedies was reiterated in Ysmael Maritime Corporation vs.
Avelino,32 Vda. De Severo vs. Feliciano-Go,33 and Marcopper Mining Corp. vs. Abeleda.34 In the last case, the Court again
recognized that a claimant who had been paid under the Act could still sue under the Civil Code. The Court said:
In the Robles case, it was held that claims for damages sustained by workers in the course of their employment could be
filed only under the Workmens Compensation Law, to the exclusion of all further claims under other laws. In Floresca, this
doctrine was abrogated in favor of the new rule that the claimants may invoke either the Workmens Compensation Act or
the provisions of the Civil Code, subject to the consequence that the choice of one remedy will exclude the other and that
the acceptance of compensation under the remedy chosen will preclude a claim for additional benefits under the other
remedy. The exception is where a claimant who has already been paid under the Workmens Compensation Act may still
sue for damages under the Civil Code on the basis of supervening facts or developments occurring after he opted for the
first remedy. (Underscoring supplied.)
Here, the CA held that private respondents case came under the exception because private respondent was unaware of
petitioners negligence when she filed her claim for death benefits from the State Insurance Fund. Private respondent filed the
civil complaint for damages after she received a copy of the police investigation report and the Prosecutors Memorandum
dismissing the criminal complaint against petitioners personnel. While stating that there was no negligence attributable to the
respondents in the complaint, the prosecutor nevertheless noted in the Memorandum that, "if at all," the "case is civil in nature."
The CA thus applied the exception in Floresca:
x x x We do not agree that appellee has knowledge of the alleged negligence of appellant as early as November 25, 1990,
the date of the police investigators report. The appellee merely executed her sworn statement before the police
investigator concerning her personal circumstances, her relation to the victim, and her knowledge of the accident. She did
not file the complaint for "Simple Negligence Resulting to Homicide" against appellants employees. It was the investigator
who recommended the filing of said case and his supervisor referred the same to the prosecutors office. This is a
standard operating procedure for police investigators which appellee may not have even known. This may explain why no
complainant is mentioned in the preliminary statement of the public prosecutor in her memorandum dated February 6,
1991, to wit: "Respondent Ferdinand Fabro x x x are being charged by complainant of "Simple Negligence Resulting to
Homicide." It is also possible that the appellee did not have a chance to appear before the public prosecutor as can be
inferred from the following statement in said memorandum: "Respondents who were notified pursuant to Law waived their
rights to present controverting evidence," thus there was no reason for the public prosecutor to summon the appellee.
Hence, notice of appellants negligence cannot be imputed on appellee before she applied for death benefits under ECC
or before she received the first payment therefrom. Her using the police investigation report to support her complaint filed
on May 9, 1991 may just be an afterthought after receiving a copy of the February 6, 1991 Memorandum of the
Prosecutors Office dismissing the criminal complaint for insufficiency of evidence, stating therein that: "The death of the
victim is not attributable to any negligence on the part of the respondents. If at all and as shown by the records this case is
civil in nature." (Underscoring supplied.) Considering the foregoing, We are more inclined to believe appellees allegation
that she learned about appellants negligence only after she applied for and received the benefits under ECC. This is a
mistake of fact that will make this case fall under the exception held in the Floresca ruling.35
The CA further held that not only was private respondent ignorant of the facts, but of her rights as well:
x x x. Appellee [Maria Juego] testified that she has reached only elementary school for her educational attainment; that
she did not know what damages could be recovered from the death of her husband; and that she did not know that she
may also recover more from the Civil Code than from the ECC. x x x.36
Petitioner impugns the foregoing rulings. It contends that private respondent "failed to allege in her complaint that her
application and receipt of benefits from the ECC were attended by ignorance or mistake of fact. Not being an issue submitted
during the trial, the trial court had no authority to hear or adjudicate that issue."
Petitioner also claims that private respondent could not have been ignorant of the facts because as early as November 28,
1990, private respondent was the complainant in a criminal complaint for "Simple Negligence Resulting to Homicide" against
petitioners employees. On February 6, 1991, two months before the filing of the action in the lower court, Prosecutor Lorna Lee
issued a resolution finding that, although there was insufficient evidence against petitioners employees, the case was "civil in
nature." These purportedly show that prior to her receipt of death benefits from the ECC on January 2, 1991 and every month
thereafter, private respondent also knew of the two choices of remedies available to her and yet she chose to claim and receive
the benefits from the ECC.
When a party having knowledge of the facts makes an election between inconsistent remedies, the election is final and bars any
action, suit, or proceeding inconsistent with the elected remedy, in the absence of fraud by the other party. The first act of
election acts as a bar.37 Equitable in nature, the doctrine of election of remedies is designed to mitigate possible unfairness to
both parties. It rests on the moral premise that it is fair to hold people responsible for their choices. The purpose of the doctrine
is not to prevent any recourse to any remedy, but to prevent a double redress for a single wrong. 38
The choice of a party between inconsistent remedies results in a waiver by election. Hence, the rule in Florescathat a claimant
cannot simultaneously pursue recovery under the Labor Code and prosecute an ordinary course of action under the Civil Code.
The claimant, by his choice of one remedy, is deemed to have waived the other.
Waiver is the intentional relinquishment of a known right.39
[It] is an act of understanding that presupposes that a party has knowledge of its rights, but chooses not to assert them. It
must be generally shown by the party claiming a waiver that the person against whom the waiver is asserted had at the
time knowledge, actual or constructive, of the existence of the partys rights or of all material facts upon which they
depended. Where one lacks knowledge of a right, there is no basis upon which waiver of it can rest. Ignorance of a

material fact negates waiver, and waiver cannot be established by a consent given under a mistake or misapprehension of
fact.
A person makes a knowing and intelligent waiver when that person knows that a right exists and has adequate knowledge
upon which to make an intelligent decision.
Waiver requires a knowledge of the facts basic to the exercise of the right waived, with an awareness of its consequences.
That a waiver is made knowingly and intelligently must be illustrated on the record or by the evidence. 40
That lack of knowledge of a fact that nullifies the election of a remedy is the basis for the exception in Floresca.
It is in light of the foregoing principles that we address petitioners contentions.
Waiver is a defense, and it was not incumbent upon private respondent, as plaintiff, to allege in her complaint that she had
availed of benefits from the ECC. It is, thus, erroneous for petitioner to burden private respondent with raising waiver as an
issue. On the contrary, it is the defendant who ought to plead waiver, as petitioner did in pages 2-3 of its Answer; 41 otherwise,
the defense is waived. It is, therefore, perplexing for petitioner to now contend that the trial court had no jurisdiction over the
issue when petitioner itself pleaded waiver in the proceedings before the trial court.
Does the evidence show that private respondent knew of the facts that led to her husbands death and the rights pertaining to a
choice of remedies?
It bears stressing that what negates waiver is lack of knowledge or a mistake of fact. In this case, the "fact" that served as a
basis for nullifying the waiver is the negligence of petitioners employees, of which private respondent purportedly learned only
after the prosecutor issued a resolution stating that there may be civil liability. InFloresca, it was the negligence of the mining
corporation and its violation of government rules and regulations. Negligence, or violation of government rules and regulations,
for that matter, however, is not a fact, but aconclusion of law, over which only the courts have the final say. Such a conclusion
binds no one until the courts have decreed so. It appears, therefore, that the principle that ignorance or mistake of fact nullifies a
waiver has been misapplied in Floresca and in the case at bar.
In any event, there is no proof that private respondent knew that her husband died in the elevator crash when on November 15,
1990 she accomplished her application for benefits from the ECC. The police investigation report is dated November 25, 1990,
10 days after the accomplishment of the form. Petitioner filed the application in her behalf on November 27, 1990.
There is also no showing that private respondent knew of the remedies available to her when the claim before the ECC was
filed. On the contrary, private respondent testified that she was not aware of her rights.
Petitioner, though, argues that under Article 3 of the Civil Code, ignorance of the law excuses no one from compliance
therewith. As judicial decisions applying or interpreting the laws or the Constitution form part of the Philippine legal system
(Article 8, Civil Code), private respondent cannot claim ignorance of this Courts ruling inFloresca allowing a choice of remedies.
The argument has no merit. The application of Article 3 is limited to mandatory and prohibitory laws. 42 This may be deduced
from the language of the provision, which, notwithstanding a persons ignorance, does not excuse his or her compliance with
the laws. The rule in Floresca allowing private respondent a choice of remedies is neither mandatory nor prohibitory.
Accordingly, her ignorance thereof cannot be held against her.
Finally, the Court modifies the affirmance of the award of damages. The records do not indicate the total amount private
respondent ought to receive from the ECC, although it appears from Exhibit "K" 43 that she received P3,581.85 as initial payment
representing the accrued pension from November 1990 to March 1991. Her initial monthly pension, according to the same
Exhibit "K," was P596.97 and present total monthly pension was P716.40. Whether the total amount she will eventually receive
from the ECC is less than the sum of P644,000.00 in total damages awarded by the trial court is subject to speculation, and the
case is remanded to the trial court for such determination. Should the trial court find that its award is greater than that of the
ECC, payments already received by private respondent under the Labor Code shall be deducted from the trial court' award of
damages. Consistent with our ruling in Floresca, this adjudication aims to prevent double compensation.
WHEREFORE, the case is REMANDED to the Regional Trial Court of Pasig City to determine whether the award decreed in its
decision is more than that of the ECC. Should the award decreed by the trial court be greater than that awarded by the ECC,
payments already made to private respondent pursuant to the Labor Code shall be deducted therefrom. In all other respects,
the Decision of the Court of Appeals is AFFIRMED.
SO ORDERED.

G.R. No. L-15127

May 30, 1961

EMETERIO CUI, plaintiff-appellant,


vs.
ARELLANO UNIVERSITY, defendant-appellee.
G.A.S. Sipin, Jr., for plaintiff-appellant.
E. Voltaire Garcia for defendant-appellee.
CONCEPCION, J.:
Appeal by plaintiff Emeterio Cui from a decision of the Court of First Instance of Manila, absolving defendant Arellano University
from plaintiff's complaint, with costs against the plaintiff, and dismissing defendant's counter claim, for insufficiency of proof
thereon.
In the language of the decision appealed from:
The essential facts of this case are short and undisputed. As established by the agreement of facts Exhibits X and by the
respective oral and documentary evidence introduced by the parties, it appears conclusive that plaintiff, before the school
year 1948-1949 took up preparatory law course in the defendant University. After finishing his preparatory law course
plaintiff enrolled in the College of Law of the defendant from the school year 1948-1949. Plaintiff finished his law studies in
the defendant university up to and including the first semester of the fourth year. During all the school years in which
plaintiff was studying law in defendant law college, Francisco R. Capistrano, brother of the mother of plaintiff, was the
dean of the College of Law and legal counsel of the defendant university. Plaintiff enrolled for the last semester of his law
studies in the defendant university but failed to pay his tuition fees because his uncle Dean Francisco R. Capistrano
having severed his connection with defendant and having accepted the deanship and chancellorship of the College of Law
of Abad Santos University, plaintiff left the defendant's law college and enrolled for the last semester of his fourth year law
in the college of law of the Abad Santos University graduating from the college of law of the latter university. Plaintiff,
during all the time he was studying law in defendant university was awarded scholarship grants, for scholastic merit, so
that his semestral tuition fees were returned to him after the ends of semester and when his scholarship grants were
awarded to him. The whole amount of tuition fees paid by plaintiff to defendant and refunded to him by the latter from the
first semester up to and including the first semester of his last year in the college of law or the fourth year, is in total
P1,033.87. After graduating in law from Abad Santos University he applied to take the bar examination. To secure
permission to take the bar he needed the transcripts of his records in defendant Arellano University. Plaintiff petitioned the
latter to issue to him the needed transcripts. The defendant refused until after he had paid back the P1,033 87 which
defendant refunded to him as above stated. As he could not take the bar examination without those transcripts, plaintiff
paid to defendant the said sum under protest. This is the sum which plaintiff seeks to recover from defendant in this case.
Before defendant awarded to plaintiff the scholarship grants as above stated, he was made to sign the following contract
covenant and agreement:
"In consideration of the scholarship granted to me by the University, I hereby waive my right to transfer to another school
without having refunded to the University (defendant) the equivalent of my scholarship cash.
(Sgd.) Emeterio Cui".
It is admitted that, on August 16, 1949, the Director of Private Schools issued Memorandum No. 38, series of 1949, on the
subject of "Scholarship," addressed to "All heads of private schools, colleges and universities," reading:
1. School catalogs and prospectuses submitted to this, Bureau show that some schools offer full or partial scholarships to
deserving students for excellence in scholarship or for leadership in extra-curricular activities. Such inducements to
poor but gifted students should be encouraged. But to stipulate the condition that such scholarships are good only if the
students concerned continue in the same school nullifies the principle of merit in the award of these scholarships.
2. When students are given full or partial scholarships, it is understood that such scholarships are merited and earned.
The amount in tuition and other fees corresponding to these scholarships should not be subsequently charged to the
recipient students when they decide to quit school or to transfer to another institution. Scholarships should not be offered
merely to attract and keep students in a school.
3. Several complaints have actually been received from students who have enjoyed scholarships, full or partial, to the
effect that they could not transfer to other schools since their credentials would not be released unless they would pay the
fees corresponding to the period of the scholarships. Where the Bureau believes that the right of the student to transfer is
being denied on this ground, it reserves the right to authorize such transfer.
that defendant herein received a copy of this memorandum; that plaintiff asked the Bureau of Private Schools to pass upon the
issue on his right to secure the transcript of his record in defendant University, without being required to refund the sum of
P1,033.87; that the Bureau of Private Schools upheld the position taken by the plaintiff and so advised the defendant; and that,
this notwithstanding, the latter refused to issue said transcript of records, unless said refund were made, and even
recommended to said Bureau that it issue a written order directing the defendant to release said transcript of record, "so that the
case may be presented to the court for judicial action." As above stated, plaintiff was, accordingly, constrained to pay, and did
pay under protest, said sum of P1,033.87, in order that he could take the bar examination in 1953. Subsequently, he brought
this action for the recovery of said amount, aside from P2,000 as moral damages, P500 as exemplary damages, P2,000 as
attorney's fees, and P500 as expenses of litigation.
In its answer, defendant reiterated the stand it took, vis-a-vis the Bureau of Private Schools, namely, that the provisions of its
contract with plaintiff are valid and binding and that the memorandum above-referred to is null and void. It, likewise, set up a
counterclaim for P10,000.00 as damages, and P3,000 as attorney's fees.
The issue in this case is whether the above quoted provision of the contract between plaintiff and the defendant, whereby the
former waived his right to transfer to another school without refunding to the latter the equivalent of his scholarships in cash, is
valid or not. The lower court resolved this question in the affirmative, upon the ground that the aforementioned memorandum of
the Director of Private Schools is not a law; that the provisions thereof are advisory, not mandatory in nature; and that, although
the contractual provision "may be unethical, yet it was more unethical for plaintiff to quit studying with the defendant without
good reasons and simply because he wanted to follow the example of his uncle." Moreover, defendant maintains in its brief that
the aforementioned memorandum of the Director of Private Schools is null and void because said officer had no authority to
issue it, and because it had been neither approved by the corresponding department head nor published in the official gazette.
We do not deem it necessary or advisable to consider as the lower court did, the question whether plaintiff had sufficient
reasons or not to transfer from defendant University to the Abad Santos University. The nature of the issue before us, and its far
reaching effects, transcend personal equations and demand a determination of the case from a high impersonal plane. Neither
do we deem it essential to pass upon the validity of said Memorandum No. 38, for, regardless of the same, we are of the opinion
that the stipulation in question is contrary to public policy and, hence, null and void. The aforesaid memorandum merely
incorporates a sound principle of public policy. As the Director of Private Schools correctly pointed, out in his letter, Exhibit B, to
the defendant,

There is one more point that merits refutation and that is whether or not the contract entered into between Cui and
Arellano University on September 10, 1951 was void as against public policy. In the case of Zeigel vs. Illinois Trust and
Savings Bank, 245 Ill. 180, 19 Ann. Case 127, the court said: 'In determining a public policy of the state, courts are limited
to a consideration of the Constitution, the judicial decisions, the statutes, and the practice of government officers.' It might
take more than a government bureau or office to lay down or establish a public policy, as alleged in your communication,
but courts consider the practices of government officials as one of the four factors in determining a public policy of the
state. It has been consistently held in America that under the principles relating to the doctrine of public policy, as applied
to the law of contracts, courts of justice will not recognize or uphold a transaction which its object, operation, or tendency
is calculated to be prejudicial to the public welfare, to sound morality or to civic honesty (Ritter vs. Mutual Life Ins. Co.,
169 U.S. 139; Heding vs. Gallaghere 64 L.R.A. 811; Veazy vs. Allen, 173 N.Y. 359). If Arellano University understood
clearly the real essence of scholarships and the motives which prompted this office to issue Memorandum No. 38, s.
1949, it should have not entered into a contract of waiver with Cui on September 10, 1951, which is a direct violation of
our Memorandum and an open challenge to the authority of the Director of Private Schools because the contract was
repugnant to sound morality and civic honesty. And finally, in Gabriel vs. Monte de Piedad, Off. Gazette Supp. Dec. 6,
1941, p. 67 we read: 'In order to declare a contract void as against public policy, a court must find that the contract as to
consideration or the thing to be done, contravenes some established interest of society, or is inconsistent with sound
policy and good morals or tends clearly to undermine the security of individual rights. The policy enunciated in
Memorandum No. 38, s. 1949 is sound policy. Scholarship are awarded in recognition of merit not to keep outstanding
students in school to bolster its prestige. In the understanding of that university scholarships award is a business
scheme designed to increase the business potential of an education institution. Thus conceived it is not only inconsistent
with sound policy but also good morals. But what is morals? Manresa has this definition. It is good customs; those
generally accepted principles of morality which have received some kind of social and practical confirmation. The practice
of awarding scholarships to attract students and keep them in school is not good customs nor has it received some kind of
social and practical confirmation except in some private institutions as in Arellano University. The University of the
Philippines which implements Section 5 of Article XIV of the Constitution with reference to the giving of free scholarships
to gifted children, does not require scholars to reimburse the corresponding value of the scholarships if they transfer to
other schools. So also with the leading colleges and universities of the United States after which our educational practices
or policies are patterned. In these institutions scholarships are granted not to attract and to keep brilliant students in
school for their propaganda mine but to reward merit or help gifted students in whom society has an established
interest or a first lien. (Emphasis supplied.)
WHEREFORE, the decision appealed from is hereby reversed and another one shall be entered sentencing the defendant to
pay to the plaintiff the sum of P1,033.87, with interest thereon at the legal rate from September 1, 1954, date of the institution of
this case, as well as the costs, and dismissing defendant's counterclaim. It is so ordered.

G.R. No. L-22595

November 1, 1927

Testate Estate of Joseph G. Brimo, JUAN MICIANO, administrator, petitioner-appellee,


vs.
ANDRE BRIMO, opponent-appellant.
Ross, Lawrence and Selph for appellant.
Camus and Delgado for appellee.

ROMUALDEZ, J.:
The partition of the estate left by the deceased Joseph G. Brimo is in question in this case.
The judicial administrator of this estate filed a scheme of partition. Andre Brimo, one of the brothers of the deceased, opposed it.
The court, however, approved it.
The errors which the oppositor-appellant assigns are:
(1) The approval of said scheme of partition; (2) denial of his participation in the inheritance; (3) the denial of the motion for
reconsideration of the order approving the partition; (4) the approval of the purchase made by the Pietro Lana of the deceased's
business and the deed of transfer of said business; and (5) the declaration that the Turkish laws are impertinent to this cause,
and the failure not to postpone the approval of the scheme of partition and the delivery of the deceased's business to Pietro
Lanza until the receipt of the depositions requested in reference to the Turkish laws.
The appellant's opposition is based on the fact that the partition in question puts into effect the provisions of Joseph G. Brimo's
will which are not in accordance with the laws of his Turkish nationality, for which reason they are void as being in violation or
article 10 of the Civil Code which, among other things, provides the following:
Nevertheless, legal and testamentary successions, in respect to the order of succession as well as to the amount of the
successional rights and the intrinsic validity of their provisions, shall be regulated by the national law of the person whose
succession is in question, whatever may be the nature of the property or the country in which it may be situated.
But the fact is that the oppositor did not prove that said testimentary dispositions are not in accordance with the Turkish laws,
inasmuch as he did not present any evidence showing what the Turkish laws are on the matter, and in the absence of evidence
on such laws, they are presumed to be the same as those of the Philippines. (Lim and Lim vs. Collector of Customs, 36 Phil.,
472.)
It has not been proved in these proceedings what the Turkish laws are. He, himself, acknowledges it when he desires to be
given an opportunity to present evidence on this point; so much so that he assigns as an error of the court in not having
deferred the approval of the scheme of partition until the receipt of certain testimony requested regarding the Turkish laws on
the matter.
The refusal to give the oppositor another opportunity to prove such laws does not constitute an error. It is discretionary with the
trial court, and, taking into consideration that the oppositor was granted ample opportunity to introduce competent evidence, we
find no abuse of discretion on the part of the court in this particular. There is, therefore, no evidence in the record that the
national law of the testator Joseph G. Brimo was violated in the testamentary dispositions in question which, not being contrary
to our laws in force, must be complied with and executed. lawphil.net
Therefore, the approval of the scheme of partition in this respect was not erroneous.
In regard to the first assignment of error which deals with the exclusion of the herein appellant as a legatee, inasmuch as he is
one of the persons designated as such in will, it must be taken into consideration that such exclusion is based on the last part of
the second clause of the will, which says:
Second. I like desire to state that although by law, I am a Turkish citizen, this citizenship having been conferred upon me
by conquest and not by free choice, nor by nationality and, on the other hand, having resided for a considerable length of
time in the Philippine Islands where I succeeded in acquiring all of the property that I now possess, it is my wish that the
distribution of my property and everything in connection with this, my will, be made and disposed of in accordance with the
laws in force in the Philippine islands, requesting all of my relatives to respect this wish, otherwise, I annul and cancel
beforehand whatever disposition found in this will favorable to the person or persons who fail to comply with this request.
The institution of legatees in this will is conditional, and the condition is that the instituted legatees must respect the testator's
will to distribute his property, not in accordance with the laws of his nationality, but in accordance with the laws of the
Philippines.
If this condition as it is expressed were legal and valid, any legatee who fails to comply with it, as the herein oppositor who, by
his attitude in these proceedings has not respected the will of the testator, as expressed, is prevented from receiving his legacy.
The fact is, however, that the said condition is void, being contrary to law, for article 792 of the civil Code provides the following:
Impossible conditions and those contrary to law or good morals shall be considered as not imposed and shall not
prejudice the heir or legatee in any manner whatsoever, even should the testator otherwise provide.
And said condition is contrary to law because it expressly ignores the testator's national law when, according to article 10 of the
civil Code above quoted, such national law of the testator is the one to govern his testamentary dispositions.
Said condition then, in the light of the legal provisions above cited, is considered unwritten, and the institution of legatees in said
will is unconditional and consequently valid and effective even as to the herein oppositor.
It results from all this that the second clause of the will regarding the law which shall govern it, and to the condition imposed
upon the legatees, is null and void, being contrary to law.
All of the remaining clauses of said will with all their dispositions and requests are perfectly valid and effective it not appearing
that said clauses are contrary to the testator's national law.
Therefore, the orders appealed from are modified and it is directed that the distribution of this estate be made in such a manner
as to include the herein appellant Andre Brimo as one of the legatees, and the scheme of partition submitted by the judicial
administrator is approved in all other respects, without any pronouncement as to costs.
So ordered.

G.R. No. L-23678

June 6, 1967

TESTATE ESTATE OF AMOS G. BELLIS, deceased.


PEOPLE'S BANK and TRUST COMPANY, executor.
MARIA CRISTINA BELLIS and MIRIAM PALMA BELLIS, oppositors-appellants,
vs.
EDWARD A. BELLIS, ET AL., heirs-appellees.
Vicente R. Macasaet and Jose D. Villena for oppositors appellants.
Paredes, Poblador, Cruz and Nazareno for heirs-appellees E. A. Bellis, et al.
Quijano and Arroyo for heirs-appellees W. S. Bellis, et al.
J. R. Balonkita for appellee People's Bank & Trust Company.
Ozaeta, Gibbs and Ozaeta for appellee A. B. Allsman.
BENGZON, J.P., J.:
This is a direct appeal to Us, upon a question purely of law, from an order of the Court of First Instance of Manila dated April 30,
1964, approving the project of partition filed by the executor in Civil Case No. 37089 therein.1wph1.t
The facts of the case are as follows:
Amos G. Bellis, born in Texas, was "a citizen of the State of Texas and of the United States." By his first wife, Mary E. Mallen,
whom he divorced, he had five legitimate children: Edward A. Bellis, George Bellis (who pre-deceased him in infancy), Henry A.
Bellis, Alexander Bellis and Anna Bellis Allsman; by his second wife, Violet Kennedy, who survived him, he had three legitimate
children: Edwin G. Bellis, Walter S. Bellis and Dorothy Bellis; and finally, he had three illegitimate children: Amos Bellis, Jr.,
Maria Cristina Bellis and Miriam Palma Bellis.
On August 5, 1952, Amos G. Bellis executed a will in the Philippines, in which he directed that after all taxes, obligations, and
expenses of administration are paid for, his distributable estate should be divided, in trust, in the following order and manner: (a)
$240,000.00 to his first wife, Mary E. Mallen; (b) P120,000.00 to his three illegitimate children, Amos Bellis, Jr., Maria Cristina
Bellis, Miriam Palma Bellis, or P40,000.00 each and (c) after the foregoing two items have been satisfied, the remainder shall
go to his seven surviving children by his first and second wives, namely: Edward A. Bellis, Henry A. Bellis, Alexander Bellis and
Anna Bellis Allsman, Edwin G. Bellis, Walter S. Bellis, and Dorothy E. Bellis, in equal shares.1wph1.t
Subsequently, or on July 8, 1958, Amos G. Bellis died a resident of San Antonio, Texas, U.S.A. His will was admitted to probate
in the Court of First Instance of Manila on September 15, 1958.
The People's Bank and Trust Company, as executor of the will, paid all the bequests therein including the amount of
$240,000.00 in the form of shares of stock to Mary E. Mallen and to the three (3) illegitimate children, Amos Bellis, Jr., Maria
Cristina Bellis and Miriam Palma Bellis, various amounts totalling P40,000.00 each in satisfaction of their respective legacies, or
a total of P120,000.00, which it released from time to time according as the lower court approved and allowed the various
motions or petitions filed by the latter three requesting partial advances on account of their respective legacies.
On January 8, 1964, preparatory to closing its administration, the executor submitted and filed its "Executor's Final Account,
Report of Administration and Project of Partition" wherein it reported, inter alia, the satisfaction of the legacy of Mary E. Mallen
by the delivery to her of shares of stock amounting to $240,000.00, and the legacies of Amos Bellis, Jr., Maria Cristina Bellis
and Miriam Palma Bellis in the amount of P40,000.00 each or a total of P120,000.00. In the project of partition, the executor
pursuant to the "Twelfth" clause of the testator's Last Will and Testament divided the residuary estate into seven equal
portions for the benefit of the testator's seven legitimate children by his first and second marriages.
On January 17, 1964, Maria Cristina Bellis and Miriam Palma Bellis filed their respective oppositions to the project of partition
on the ground that they were deprived of their legitimes as illegitimate children and, therefore, compulsory heirs of the
deceased.
Amos Bellis, Jr. interposed no opposition despite notice to him, proof of service of which is evidenced by the registry receipt
submitted on April 27, 1964 by the executor.1
After the parties filed their respective memoranda and other pertinent pleadings, the lower court, on April 30, 1964, issued an
order overruling the oppositions and approving the executor's final account, report and administration and project of partition.
Relying upon Art. 16 of the Civil Code, it applied the national law of the decedent, which in this case is Texas law, which did not
provide for legitimes.
Their respective motions for reconsideration having been denied by the lower court on June 11, 1964, oppositors-appellants
appealed to this Court to raise the issue of which law must apply Texas law or Philippine law.
In this regard, the parties do not submit the case on, nor even discuss, the doctrine of renvoi, applied by this Court in Aznar v.
Christensen Garcia, L-16749, January 31, 1963. Said doctrine is usually pertinent where the decedent is a national of one
country, and a domicile of another. In the present case, it is not disputed that the decedent was both a national of Texas and a
domicile thereof at the time of his death.2 So that even assuming Texas has a conflict of law rule providing that the domiciliary
system (law of the domicile) should govern, the same would not result in a reference back (renvoi) to Philippine law, but would
still refer to Texas law. Nonetheless, if Texas has a conflicts rule adopting the situs theory (lex rei sitae) calling for the application
of the law of the place where the properties are situated, renvoi would arise, since the properties here involved are found in the
Philippines. In the absence, however, of proof as to the conflict of law rule of Texas, it should not be presumed different from
ours.3Appellants' position is therefore not rested on the doctrine of renvoi. As stated, they never invoked nor even mentioned it
in their arguments. Rather, they argue that their case falls under the circumstances mentioned in the third paragraph of Article
17 in relation to Article 16 of the Civil Code.
Article 16, par. 2, and Art. 1039 of the Civil Code, render applicable the national law of the decedent, in intestate or testamentary
successions, with regard to four items: (a) the order of succession; (b) the amount of successional rights; (e) the intrinsic validity
of the provisions of the will; and (d) the capacity to succeed. They provide that
ART. 16. Real property as well as personal property is subject to the law of the country where it is situated.
However, intestate and testamentary successions, both with respect to the order of succession and to the amount of
successional rights and to the intrinsic validity of testamentary provisions, shall be regulated by the national law of the
person whose succession is under consideration, whatever may he the nature of the property and regardless of the
country wherein said property may be found.
ART. 1039. Capacity to succeed is governed by the law of the nation of the decedent.
Appellants would however counter that Art. 17, paragraph three, of the Civil Code, stating that
Prohibitive laws concerning persons, their acts or property, and those which have for their object public order, public policy
and good customs shall not be rendered ineffective by laws or judgments promulgated, or by determinations or
conventions agreed upon in a foreign country.

prevails as the exception to Art. 16, par. 2 of the Civil Code afore-quoted. This is not correct. Precisely, Congressdeleted the
phrase, "notwithstanding the provisions of this and the next preceding article" when they incorporated Art. 11 of the old Civil
Code as Art. 17 of the new Civil Code, while reproducing without substantial change the second paragraph of Art. 10 of the old
Civil Code as Art. 16 in the new. It must have been their purpose to make the second paragraph of Art. 16 a specific provision in
itself which must be applied in testate and intestate succession. As further indication of this legislative intent, Congress added a
new provision, under Art. 1039, which decrees that capacity to succeed is to be governed by the national law of the decedent.
It is therefore evident that whatever public policy or good customs may be involved in our System of legitimes, Congress has
not intended to extend the same to the succession of foreign nationals. For it has specifically chosen to leave, inter alia,
the amount of successional rights, to the decedent's national law. Specific provisions must prevail over general ones.
Appellants would also point out that the decedent executed two wills one to govern his Texas estate and the other his
Philippine estate arguing from this that he intended Philippine law to govern his Philippine estate. Assuming that such was
the decedent's intention in executing a separate Philippine will, it would not alter the law, for as this Court ruled in Miciano v.
Brimo, 50 Phil. 867, 870, a provision in a foreigner's will to the effect that his properties shall be distributed in accordance with
Philippine law and not with his national law, is illegal and void, for his national law cannot be ignored in regard to those matters
that Article 10 now Article 16 of the Civil Code states said national law should govern.
The parties admit that the decedent, Amos G. Bellis, was a citizen of the State of Texas, U.S.A., and that under the laws of
Texas, there are no forced heirs or legitimes. Accordingly, since the intrinsic validity of the provision of the will and the amount of
successional rights are to be determined under Texas law, the Philippine law on legitimes cannot be applied to the testacy of
Amos G. Bellis.
Wherefore, the order of the probate court is hereby affirmed in toto, with costs against appellants. So ordered.

Human Relations
G.R. No. 88694 January 11, 1993
ALBENSON ENTERPRISES CORP., JESSE YAP, AND BENJAMIN MENDIONA, petitioners,
vs.
THE COURT OF APPEALS AND EUGENIO S. BALTAO, respondents.
Puruganan, Chato, Chato & Tan for petitioners.
Lino M. Patajo, Francisco Ma. Chanco, Ananiano Desierto and Segundo Mangohig for private respondent.

BIDIN, J.:
This petition assails the decision of respondent Court of Appeals in
CA-GR CV No. 14948 entitled "Eugenio S. Baltao, plaintiff-appellee vs. Albenson Enterprises Corporation, et al, defendantsappellants", which modified the judgment of the Regional Trial Court of Quezon City, Branch XCVIII in Civil Case No. Q-40920
and ordered petitioner to pay private respondent, among others, the sum of P500,000.00 as moral damages and attorney's fees
in the amount of P50,000.00.
The facts are not disputed.
In September, October, and November 1980, petitioner Albenson Enterprises Corporation (Albenson for short) delivered to
Guaranteed Industries, Inc. (Guaranteed for short) located at 3267 V. Mapa Street, Sta. Mesa, Manila, the mild steel plates
which the latter ordered. As part payment thereof, Albenson was given Pacific Banking Corporation Check No. 136361 in the
amount of P2,575.00 and drawn against the account of E.L. Woodworks (Rollo, p. 148).
When presented for payment, the check was dishonored for the reason "Account Closed." Thereafter, petitioner Albenson,
through counsel, traced the origin of the dishonored check. From the records of the Securities and Exchange Commission
(SEC), Albenson discovered that the president of Guaranteed, the recipient of the unpaid mild steel plates, was one "Eugenio S.
Baltao." Upon further inquiry, Albenson was informed by the Ministry of Trade and Industry that E.L. Woodworks, a single
proprietorship business, was registered in the name of one "Eugenio Baltao". In addition, upon verification with the drawee
bank, Pacific Banking Corporation, Albenson was advised that the signature appearing on the subject check belonged to one
"Eugenio Baltao."
After obtaining the foregoing information, Albenson, through counsel, made an extrajudicial demand upon private respondent
Eugenio S. Baltao, president of Guaranteed, to replace and/or make good the dishonored check.
Respondent Baltao, through counsel, denied that he issued the check, or that the signature appearing thereon is his. He further
alleged that Guaranteed was a defunct entity and hence, could not have transacted business with Albenson.
On February 14, 1983, Albenson filed with the Office of the Provincial Fiscal of Rizal a complaint against Eugenio S. Baltao for
violation of Batas Pambansa Bilang 22. Submitted to support said charges was an affidavit of petitioner Benjamin Mendiona, an
employee of Albenson. In said affidavit, the above-mentioned circumstances were stated.
It appears, however, that private respondent has a namesake, his son Eugenio Baltao III, who manages a business
establishment, E.L. Woodworks, on the ground floor of the Baltao Building, 3267 V. Mapa Street, Sta. Mesa, Manila, the very
same business address of Guaranteed.
On September 5, 1983, Assistant Fiscal Ricardo Sumaway filed an information against Eugenio S. Baltao for Violation of Batas
Pambansa Bilang 22. In filing said information, Fiscal Sumaway claimed that he had given Eugenio S. Baltao opportunity to
submit controverting evidence, but the latter failed to do so and therefore, was deemed to have waived his right.
Respondent Baltao, claiming ignorance of the complaint against him, immediately filed with the Provincial Fiscal of Rizal a
motion for reinvestigation, alleging that it was not true that he had been given an opportunity to be heard in the preliminary
investigation conducted by Fiscal Sumaway, and that he never had any dealings with Albenson or Benjamin Mendiona,
consequently, the check for which he has been accused of having issued without funds was not issued by him and the signature
in said check was not his.
On January 30, 1984, Provincial Fiscal Mauro M. Castro of Rizal reversed the finding of Fiscal Sumaway and exonerated
respondent Baltao. He also instructed the Trial Fiscal to move for dismissal of the information filed against Eugenio S. Baltao.
Fiscal Castro found that the signature in PBC Check No. 136361 is not the signature of Eugenio S. Baltao. He also found that
there is no showing in the records of the preliminary investigation that Eugenio S. Baltao actually received notice of the said
investigation. Fiscal Castro then castigated Fiscal Sumaway for failing to exercise care and prudence in the performance of his
duties, thereby causing injustice to respondent who was not properly notified of the complaint against him and of the
requirement to submit his counter evidence.
Because of the alleged unjust filing of a criminal case against him for allegedly issuing a check which bounced in violation of
Batas Pambansa Bilang 22 for a measly amount of P2,575.00, respondent Baltao filed before the Regional Trial Court of
Quezon City a complaint for damages against herein petitioners Albenson Enterprises, Jesse Yap, its owner, and Benjamin
Mendiona, its employee.
In its decision, the lower court observed that "the check is drawn against the account of "E.L. Woodworks," not of Guaranteed
Industries of which plaintiff used to be President. Guaranteed Industries had been inactive and had ceased to exist as a
corporation since 1975. . . . . The possibility is that it was with Gene Baltao or Eugenio Baltao III, a son of plaintiff who had a
business on the ground floor of Baltao Building located on V. Mapa Street, that the defendants may have been dealing
with . . . ." (Rollo, pp. 41-42).
The dispositive portion of the trial court 's decision reads:
WHEREFORE, judgment is hereby rendered in favor of plaintiff and against defendants ordering the latter to pay
plaintiff jointly and severally:
1. actual or compensatory damages of P133,350.00;
2. moral damages of P1,000,000.00 (1 million pesos);
3. exemplary damages of P200,000.00;
4. attorney's fees of P100,000.00;
5 costs.

Defendants' counterclaim against plaintiff and claim for damages against Mercantile Insurance Co. on the bond for
the issuance of the writ of attachment at the instance of plaintiff are hereby dismissed for lack of merit. (Rollo, pp.
38-39).
On appeal, respondent court modified the trial court's decision as follows:
WHEREFORE, the decision appealed from is MODIFIED by reducing the moral damages awarded therein from
P1,000,000.00 to P500,000.00 and the attorney's fees from P100,000.00 to P50,000.00, said decision being hereby
affirmed in all its other aspects. With costs against appellants. (Rollo, pp. 50-51)
Dissatisfied with the above ruling, petitioners Albenson Enterprises Corp., Jesse Yap, and Benjamin Mendiona filed the instant
Petition, alleging that the appellate court erred in:
1. Concluding that private respondent's cause of action is not one based on malicious prosecution but one for abuse
of rights under Article 21 of the Civil Code notwithstanding the fact that the basis of a civil action for malicious
prosecution is Article 2219 in relation to Article 21 or Article 2176 of the Civil Code . . . .
2. Concluding that "hitting at and in effect maligning (private respondent) with an unjust criminal case was, without
more, a plain case of abuse of rights by misdirection" and "was therefore, actionable by itself," and which "became
inordinately blatant and grossly aggravated when . . . (private respondent) was deprived of his basic right to notice
and a fair hearing in the so-called preliminary investigation . . . . "
3. Concluding that petitioner's "actuations in this case were coldly deliberate and calculated", no evidence having
been adduced to support such a sweeping statement.
4. Holding the petitioner corporation, petitioner Yap and petitioner Mendiona jointly and severally liable without
sufficient basis in law and in fact.
5. Awarding respondents
5.1. P133,350.00 as actual or compensatory damages, even in the absence of sufficient evidence to
show that such was actually suffered.
5.2. P500,000.00 as moral damages considering that the evidence in this connection merely involved
private respondent's alleged celebrated status as a businessman, there being no showing that the act
complained of adversely affected private respondent's reputation or that it resulted to material loss.
5.3. P200,000.00 as exemplary damages despite the fact that petitioners were duly advised by counsel
of their legal recourse.
5.4. P50,000.00 as attorney's fees, no evidence having been adduced to justify such an award (Rollo,
pp. 4-6).
Petitioners contend that the civil case filed in the lower court was one for malicious prosecution. Citing the case ofMadera vs.
Lopez (102 SCRA 700 [1981]), they assert that the absence of malice on their part absolves them from any liability for malicious
prosecution. Private respondent, on the other hand, anchored his complaint for Damages on Articles 19, 20, and 21 ** of the
Civil Code.
Article 19, known to contain what is commonly referred to as the principle of abuse of rights, sets certain standards which may
be observed not only in the exercise of one's rights but also in the performance of one's duties. These standards are the
following: to act with justice; to give everyone his due; and to observe honesty and good faith. The law, therefore, recognizes
the primordial limitation on all rights: that in their exercise, the norms of human conduct set forth in Article 19 must be observed.
A right, though by itself legal because recognized or granted by law as such, may nevertheless become the source of some
illegality. When a right is exercised in a manner which does not conform with the norms enshrined in Article 19 and results in
damage to another, a legal wrong is thereby committed for which the wrongdoer must be held responsible. Although the
requirements of each provision is different, these three (3) articles are all related to each other. As the eminent Civilist Senator
Arturo Tolentino puts it: "With this article (Article 21), combined with articles 19 and 20, the scope of our law on civil wrongs has
been very greatly broadened; it has become much more supple and adaptable than the Anglo-American law on torts. It is now
difficult to conceive of any malevolent exercise of a right which could not be checked by the application of these articles"
(Tolentino, 1 Civil Code of the Philippines 72).
There is however, no hard and fast rule which can be applied to determine whether or not the principle of abuse of rights may
be invoked. The question of whether or not the principle of abuse of rights has been violated, resulting in damages under
Articles 20 and 21 or other applicable provision of law, depends on the circumstances of each case. (Globe Mackay Cable and
Radio Corporation vs. Court of Appeals, 176 SCRA 778 [1989]).
The elements of an abuse of right under Article 19 are the following: (1) There is a legal right or duty; (2) which is exercised
in bad faith; (3) for the sole intent of prejudicing or injuring another. Article 20 speaks of the general sanction for all other
provisions of law which do not especially provide for their own sanction (Tolentino, supra, p. 71). Thus, anyone who,
whether willfully or negligently, in the exercise of his legal right or duty, causes damage to another, shall indemnify his victim for
injuries suffered thereby. Article 21 deals with acts contra bonus mores, and has the following elements: 1) There is an act
which is legal; 2) but which is contrary to morals, good custom, public order, or public policy; 3) and it is done with intent to
injure.
Thus, under any of these three (3) provisions of law, an act which causes injury to another may be made the basis for an award
of damages.
There is a common element under Articles 19 and 21, and that is, the act must be intentional. However, Article 20 does not
distinguish: the act may be done either "willfully", or "negligently". The trial court as well as the respondent appellate court
mistakenly lumped these three (3) articles together, and cited the same as the bases for the award of damages in the civil
complaint filed against petitioners, thus:
With the foregoing legal provisions (Articles 19, 20, and 21) in focus, there is not much difficulty in ascertaining the
means by which appellants' first assigned error should be resolved, given the admitted fact that when there was an
attempt to collect the amount of P2,575.00, the defendants were explicitly warned that plaintiff Eugenio S. Baltao is
not the Eugenio Baltao defendants had been dealing with (supra, p. 5). When the defendants nevertheless insisted
and persisted in filing a case a criminal case no less against plaintiff, said defendants ran afoul of the legal
provisions (Articles 19, 20, and 21 of the Civil Code) cited by the lower court and heretofore quoted (supra).
Defendants, not having been paid the amount of P2,575.00, certainly had the right to complain. But that right is
limited by certain constraints. Beyond that limit is the area of excess, of abuse of rights. (Rollo, pp.
44-45).
Assuming, arguendo, that all the three (3) articles, together and not independently of each one, could be validly made the bases
for an award of damages based on the principle of "abuse of right", under the circumstances, We see no cogent reason for such
an award of damages to be made in favor of private respondent.

Certainly, petitioners could not be said to have violated the aforestated principle of abuse of right. What prompted petitioners to
file the case for violation of Batas Pambansa Bilang 22 against private respondent was their failure to collect the amount of
P2,575.00 due on a bounced check which they honestly believed was issued to them by private respondent. Petitioners had
conducted inquiries regarding the origin of the check, and yielded the following results: from the records of the Securities and
Exchange Commission, it was discovered that the President of Guaranteed (the recipient of the unpaid mild steel plates), was
one "Eugenio S. Baltao"; an inquiry with the Ministry of Trade and Industry revealed that E.L. Woodworks, against whose
account the check was drawn, was registered in the name of one "Eugenio Baltao"; verification with the drawee bank, the
Pacific Banking Corporation, revealed that the signature appearing on the check belonged to one "Eugenio Baltao".
In a letter dated December 16, 1983, counsel for petitioners wrote private respondent demanding that he make good the
amount of the check. Counsel for private respondent wrote back and denied, among others, that private respondent ever
transacted business with Albenson Enterprises Corporation; that he ever issued the check in question. Private respondent's
counsel even went further: he made a warning to defendants to check the veracity of their claim. It is pivotal to note at this
juncture that in this same letter, if indeed private respondent wanted to clear himself from the baseless accusation made against
his person, he should have made mention of the fact that there are three (3) persons with the same name, i.e.: Eugenio Baltao,
Sr., Eugenio S. Baltao, Jr. (private respondent), and Eugenio Baltao III (private respondent's son, who as it turned out later, was
the issuer of the check). He, however, failed to do this. The last two Baltaos were doing business in the same building Baltao
Building located at 3267 V. Mapa Street, Sta. Mesa, Manila. The mild steel plates were ordered in the name of Guaranteed of
which respondent Eugenio S. Baltao is the president and delivered to Guaranteed at Baltao building. Thus, petitioners had
every reason to believe that the Eugenio Baltao who issued the bouncing check is respondent Eugenio S. Baltao when their
counsel wrote respondent to make good the amount of the check and upon refusal, filed the complaint for violation of BP Blg.
22.
Private respondent, however, did nothing to clarify the case of mistaken identity at first hand. Instead, private respondent waited
in ambush and thereafter pounced on the hapless petitioners at a time he thought was propitious by filing an action for
damages. The Court will not countenance this devious scheme.
The criminal complaint filed against private respondent after the latter refused to make good the amount of the bouncing check
despite demand was a sincere attempt on the part of petitioners to find the best possible means by which they could collect the
sum of money due them. A person who has not been paid an obligation owed to him will naturally seek ways to compel the
debtor to pay him. It was normal for petitioners to find means to make the issuer of the check pay the amount thereof. In the
absence of a wrongful act or omission or of fraud or bad faith, moral damages cannot be awarded and that the adverse result of
an action does not per se make the action wrongful and subject the actor to the payment of damages, for the law could not have
meant to impose a penalty on the right to litigate (Rubio vs. Court of Appeals, 141 SCRA 488 [1986]).
In the case at bar, private respondent does not deny that the mild steel plates were ordered by and delivered to Guaranteed at
Baltao building and as part payment thereof, the bouncing check was issued by one Eugenio Baltao. Neither had private
respondent conveyed to petitioner that there are two Eugenio Baltaos conducting business in the same building he and his
son Eugenio Baltao III. Considering that Guaranteed, which received the goods in payment of which the bouncing check was
issued is owned by respondent, petitioner acted in good faith and probable cause in filing the complaint before the provincial
fiscal.
To constitute malicious prosecution, there must be proof that the prosecution was prompted by a sinister design to vex and
humiliate a person, and that it was initiated deliberately by the defendant knowing that his charges were false and groundless.
Concededly, the mere act of submitting a case to the authorities for prosecution does not make one liable for malicious
prosecution. (Manila Gas Corporation vs. Court of Appeals, 100 SCRA 602 [1980]). Still, private respondent argues that liability
under Articles 19, 20, and 21 of the Civil Code is so encompassing that it likewise includes liability for damages for malicious
prosecution under Article 2219 (8). True, a civil action for damages for malicious prosecution is allowed under the New Civil
Code, more specifically Articles 19, 20, 26, 29, 32, 33, 35, and 2219 (8) thereof. In order that such a case can prosper, however,
the following three (3) elements must be present, to wit: (1) The fact of the prosecution and the further fact that the defendant
was himself the prosecutor, and that the action was finally terminated with an acquittal; (2) That in bringing the action, the
prosecutor acted without probable cause; (3) The prosecutor was actuated or impelled by legal malice (Lao vs. Court of
Appeals, 199 SCRA 58, [1991]).
Thus, a party injured by the filing of a court case against him, even if he is later on absolved, may file a case for damages
grounded either on the principle of abuse of rights, or on malicious prosecution. As earlier stated, a complaint for damages
based on malicious prosecution will prosper only if the three (3) elements aforecited are shown to exist. In the case at bar, the
second and third elements were not shown to exist. It is well-settled that one cannot be held liable for maliciously instituting a
prosecution where one has acted with probable cause. "Probable cause is the existence of such facts and circumstances as
would excite the belief, in a reasonable mind, acting on the facts within the knowledge of the prosecutor, that the person
charged was guilty of the crime for which he was prosecuted. In other words, a suit will lie only in cases where a legal
prosecution has been carried on without probable cause. The reason for this rule is that it would be a very great
discouragement to public justice, if prosecutors, who had tolerable ground of suspicion, were liable to be sued at law when their
indictment miscarried" (Que vs. Intermediate Appellate Court, 169 SCRA 137 [1989]).
The presence of probable cause signifies, as a legal consequence, the absence of malice. In the instant case, it is evident that
petitioners were not motivated by malicious intent or by sinister design to unduly harass private respondent, but only by a wellfounded anxiety to protect their rights when they filed the criminal complaint against private respondent.
To constitute malicious prosecution, there must be proof that the prosecution was prompted by a sinister design to
vex and humiliate a person, that it was initiated deliberately by the defendant knowing that his charges were false
and groundless. Concededly, the mere act of submitting a case to the authorities for prosecution does not make one
liable for malicious prosecution. Proof and motive that the institution of the action was prompted by a sinister design
to vex and humiliate a person must be clearly and preponderantly established to entitle the victims to damages
(Ibid.).
In the case at bar, there is no proof of a sinister design on the part of petitioners to vex or humiliate private respondent by
instituting the criminal case against him. While petitioners may have been negligent to some extent in determining the liability of
private respondent for the dishonored check, the same is not so gross or reckless as to amount to bad faith warranting an
award of damages.
The root of the controversy in this case is founded on a case of mistaken identity. It is possible that with a more assiduous
investigation, petitioners would have eventually discovered that private respondent Eugenio S. Baltao is not the "Eugenio
Baltao" responsible for the dishonored check. However, the record shows that petitioners did exert considerable effort in order
to determine the liability of private respondent. Their investigation pointed to private respondent as the "Eugenio Baltao" who
issued and signed the dishonored check as the president of the debtor-corporation Guaranteed Enterprises. Their error in
proceeding against the wrong individual was obviously in the nature of an innocent mistake, and cannot be characterized as
having been committed in bad faith. This error could have been discovered if respondent had submitted his counter-affidavit
before investigating fiscal Sumaway and was immediately rectified by Provincial Fiscal Mauro Castro upon discovery thereof,
i.e., during the reinvestigation resulting in the dismissal of the complaint.
Furthermore, the adverse result of an action does not per se make the act wrongful and subject the actor to the payment of
moral damages. The law could not have meant to impose a penalty on the right to litigate, such right is so precious that moral
damages may not be charged on those who may even exercise it erroneously. And an adverse decision does not ipso
facto justify the award of attorney's fees to the winning party (Garcia vs. Gonzales, 183 SCRA 72 [1990]).

Thus, an award of damages and attorney's fees is unwarranted where the action was filed in good faith. If damage results from
a person's exercising his legal rights, it is damnum absque injuria (Ilocos Norte Electric Company vs. Court of Appeals, 179
SCRA 5 [1989]).
Coming now to the claim of private respondent for actual or compensatory damages, the records show that the same was
based solely on his allegations without proof to substantiate the same. He did not present proof of the cost of the medical
treatment which he claimed to have undergone as a result of the nervous breakdown he suffered, nor did he present proof of
the actual loss to his business caused by the unjust litigation against him. In determining actual damages, the court cannot rely
on speculation, conjectures or guesswork as to the amount. Without the actual proof of loss, the award of actual damages
becomes erroneous (Guilatco vs. City of Dagupan, 171 SCRA 382 [1989]).
Actual and compensatory damages are those recoverable because of pecuniary loss in business, trade, property, profession,
job or occupation and the same must be proved, otherwise, if the proof is flimsy and unsubstantiated, no damages will be
given (Rubio vs. Court of Appeals, 141 SCRA 488 [1986]). For these reasons, it was gravely erroneous for respondent court to
have affirmed the award of actual damages in favor of private respondent in the absence of proof thereof.
Where there is no evidence of the other party having acted in wanton, fraudulent or reckless, or oppressive manner, neither may
exemplary damages be awarded (Dee Hua Liong Electrical Equipment Corporation vs. Reyes, 145 SCRA 488 [1986]).
As to the award of attorney's fees, it is well-settled that the same is the exception rather than the general rule. Needless to say,
the award of attorney's fees must be disallowed where the award of exemplary damages is eliminated (Article 2208, Civil Code;
Agustin vs. Court of Appeals, 186 SCRA 375 [1990]). Moreover, in view of the fact that there was no malicious prosecution
against private respondent, attorney's fees cannot be awarded him on that ground.
In the final analysis, there is no proof or showing that petitioners acted maliciously or in bad faith in the filing of the case against
private respondent. Consequently, in the absence of proof of fraud and bad faith committed by petitioners, they cannot be held
liable for damages (Escritor, Jr. vs. Intermediate Appellate Court, 155 SCRA 577 [1987]). No damages can be awarded in the
instant case, whether based on the principle of abuse of rights, or for malicious prosecution. The questioned judgment in the
instant case attests to the propensity of trial judges to award damages without basis. Lower courts are hereby cautioned anew
against awarding unconscionable sums as damages without bases therefor.
WHEREFORE, the petition is GRANTED and the decision of the Court of Appeals in C.A. G.R. C.V. No. 14948 dated May 13,
1989, is hereby REVERSED and SET ASIDE. Costs against respondent Baltao.
SO ORDERED.

G.R. No. 154259

February 28, 2005

NIKKO HOTEL MANILA GARDEN and RUBY LIM, petitioners,


vs.
ROBERTO REYES, a.k.a. "AMAY BISAYA," respondent.
DECISION
CHICO-NAZARIO, J.:
In this petition for review on certiorari, petitioners Nikko Hotel Manila Garden (Hotel Nikko) 1 and Ruby Lim assail the Decision2 of
the Court of Appeals dated 26 November 2001 reversing the Decision 3 of the Regional Trial Court (RTC) of Quezon City, Branch
104, as well as the Resolution4 of the Court of Appeals dated 09 July 2002 which denied petitioners motion for reconsideration.
The cause of action before the trial court was one for damages brought under the human relations provisions of the New Civil
Code. Plaintiff thereat (respondent herein) Roberto Reyes, more popularly known by the screen name "Amay Bisaya," alleged
that at around 6:00 oclock in the evening of 13 October 1994, while he was having coffee at the lobby of Hotel Nikko, 5 he was
spotted by his friend of several years, Dr. Violeta Filart, who then approached him. 6 Mrs. Filart invited him to join her in a party at
the hotels penthouse in celebration of the natal day of the hotels manager, Mr. Masakazu Tsuruoka. 7 Mr. Reyes asked if she
could vouch for him for which she replied: "of course." 8 Mr. Reyes then went up with the party of Dr. Filart carrying the basket of
fruits which was the latters present for the celebrant. 9 At the penthouse, they first had their picture taken with the celebrant after
which Mr. Reyes sat with the party of Dr. Filart.10 After a couple of hours, when the buffet dinner was ready, Mr. Reyes lined-up
at the buffet table but, to his great shock, shame and embarrassment, he was stopped by petitioner herein, Ruby Lim, who
claimed to speak for Hotel Nikko as Executive Secretary thereof. 11 In a loud voice and within the presence and hearing of the
other guests who were making a queue at the buffet table, Ruby Lim told him to leave the party ("huwag ka nang kumain, hindi
ka imbitado, bumaba ka na lang").12 Mr. Reyes tried to explain that he was invited by Dr. Filart. 13 Dr. Filart, who was within
hearing distance, however, completely ignored him thus adding to his shame and humiliation. 14 Not long after, while he was still
recovering from the traumatic experience, a Makati policeman approached and asked him to step out of the hotel. 15 Like a
common criminal, he was escorted out of the party by the policeman. 16 Claiming damages, Mr. Reyes asked for One Million
Pesos actual damages, One Million Pesos moral and/or exemplary damages and Two Hundred Thousand Pesos attorneys
fees.17
Ruby Lim, for her part, admitted having asked Mr. Reyes to leave the party but not under the ignominious circumstance painted
by the latter. Ms. Lim narrated that she was the Hotels Executive Secretary for the past twenty (20) years. 18 One of her
functions included organizing the birthday party of the hotels former General Manager, Mr. Tsuruoka. 19 The year 1994 was no
different. For Mr. Tsuruokas party, Ms. Lim generated an exclusive guest list and extended invitations accordingly. 20 The guest
list was limited to approximately sixty (60) of Mr. Tsuruokas closest friends and some hotel employees and that Mr. Reyes was
not one of those invited.21 At the party, Ms. Lim first noticed Mr. Reyes at the bar counter ordering a drink. 22 Mindful of Mr.
Tsuruokas wishes to keep the party intimate, Ms. Lim approached Mr. Boy Miller, the "captain waiter," to inquire as to the
presence of Mr. Reyes who was not invited.23 Mr. Miller replied that he saw Mr. Reyes with the group of Dr. Filart. 24 As Dr. Filart
was engaged in conversation with another guest and as Ms. Lim did not want to interrupt, she inquired instead from the sister of
Dr. Filart, Ms. Zenaida Fruto, who told her that Dr. Filart did not invite Mr. Reyes. 25 Ms. Lim then requested Ms. Fruto to tell Mr.
Reyes to leave the party as he was not invited. 26 Mr. Reyes, however, lingered prompting Ms. Lim to inquire from Ms. Fruto who
said that Mr. Reyes did not want to leave.27 When Ms. Lim turned around, she saw Mr. Reyes conversing with a Captain Batung
whom she later approached.28 Believing that Captain Batung and Mr. Reyes knew each other, Ms. Lim requested from him the
same favor from Ms. Fruto,i.e., for Captain Batung to tell Mr. Reyes to leave the party as he was not invited. 29 Still, Mr. Reyes
lingered. When Ms. Lim spotted Mr. Reyes by the buffet table, she decided to speak to him herself as there were no other
guests in the immediate vicinity.30 However, as Mr. Reyes was already helping himself to the food, she decided to wait. 31When
Mr. Reyes went to a corner and started to eat, Ms. Lim approached him and said: "alam ninyo, hindo ho kayo dapat nandito.
Pero total nakakuha na ho kayo ng pagkain, ubusin na lang ninyo at pagkatapos kung pwede lang po umalis na kayo."32 She
then turned around trusting that Mr. Reyes would show enough decency to leave, but to her surprise, he began screaming and
making a big scene, and even threatened to dump food on her.331awphi1.nt
Dr. Violeta Filart, the third defendant in the complaint before the lower court, also gave her version of the story to the effect that
she never invited Mr. Reyes to the party.34 According to her, it was Mr. Reyes who volunteered to carry the basket of fruits
intended for the celebrant as he was likewise going to take the elevator, not to the penthouse but to Altitude 49. 35 When they
reached the penthouse, she reminded Mr. Reyes to go down as he was not properly dressed and was not invited. 36 All the while,
she thought that Mr. Reyes already left the place, but she later saw him at the bar talking to Col. Batung. 37 Then there was a
commotion and she saw Mr. Reyes shouting.38 She ignored Mr. Reyes.39 She was embarrassed and did not want the celebrant
to think that she invited him.40
After trial on the merits, the court a quo dismissed the complaint,41 giving more credence to the testimony of Ms. Lim that she
was discreet in asking Mr. Reyes to leave the party. The trial court likewise ratiocinated that Mr. Reyes assumed the risk of
being thrown out of the party as he was uninvited:
Plaintiff had no business being at the party because he was not a guest of Mr. Tsuruoka, the birthday celebrant. He assumed
the risk of being asked to leave for attending a party to which he was not invited by the host. Damages are pecuniary
consequences which the law imposes for the breach of some duty or the violation of some right. Thus, no recovery can be had
against defendants Nikko Hotel and Ruby Lim because he himself was at fault (Garciano v. Court of Appeals, 212 SCRA 436).
He knew that it was not the party of defendant Violeta Filart even if she allowed him to join her and took responsibility for his
attendance at the party. His action against defendants Nikko Hotel and Ruby Lim must therefore fail. 42
On appeal, the Court of Appeals reversed the ruling of the trial court as it found more commanding of belief the testimony of Mr.
Reyes that Ms. Lim ordered him to leave in a loud voice within hearing distance of several guests:
In putting appellant in a very embarrassing situation, telling him that he should not finish his food and to leave the place within
the hearing distance of other guests is an act which is contrary to morals, good customs . . ., for which appellees should
compensate the appellant for the damage suffered by the latter as a consequence therefore (Art. 21, New Civil Code). The
liability arises from the acts which are in themselves legal or not prohibited, but contrary to morals or good customs. Conversely,
even in the exercise of a formal right, [one] cannot with impunity intentionally cause damage to another in a manner contrary to
morals or good customs.43
The Court of Appeals likewise ruled that the actuation of Ms. Lim in approaching several people to inquire into the presence of
Mr. Reyes exposed the latter to ridicule and was uncalled for as she should have approached Dr. Filart first and both of them
should have talked to Mr. Reyes in private:
Said acts of appellee Lim are uncalled for. What should have been done by appellee Lim was to approach appellee Mrs. Filart
and together they should have told appellant Reyes in private that the latter should leave the party as the celebrant only wanted
close friends around. It is necessary that Mrs. Filart be the one to approach appellant because it was she who invited appellant
in that occasion. Were it not for Mrs. Filarts invitation, appellant could not have suffered such humiliation. For that, appellee
Filart is equally liable.
...

The acts of [appellee] Lim are causes of action which are predicated upon mere rudeness or lack of consideration of one
person, which calls not only protection of human dignity but respect of such dignity. Under Article 20 of the Civil Code, every
person who violates this duty becomes liable for damages, especially if said acts were attended by malice or bad faith. Bad faith
does not simply connote bad judgment or simple negligence. It imports a dishonest purpose or some moral obliquity and
conscious doing of a wrong, a breach of a known duty to some motive or interest or ill-will that partakes of the nature of fraud
(Cojuangco, Jr. v. CA, et al., 309 SCRA 603).44
Consequently, the Court of Appeals imposed upon Hotel Nikko, Ruby Lim and Dr. Violeta Filart the solidary obligation to pay Mr.
Reyes (1) exemplary damages in the amount of Two Hundred Thousand Pesos (P200,000); (2) moral damages in the amount
of Two Hundred Thousand Pesos (P200,000); and (3) attorneys fees in the amount of Ten Thousand Pesos (P10,000). 45 On
motion for reconsideration, the Court of Appeals affirmed its earlier decision as the argument raised in the motion had "been
amply discussed and passed upon in the decision sought to be reconsidered." 46
Thus, the instant petition for review. Hotel Nikko and Ruby Lim contend that the Court of Appeals seriously erred in
I.
NOT APPLYING THE DOCTRINE OF VOLENTI NON FIT INJURIA CONSIDERING THAT BY ITS OWN FINDINGS, AMAY
BISAYA WAS A GATE-CRASHER
II.
HOLDING HOTEL NIKKO AND RUBY LIM JOINTLY AND SEVERALLY LIABLE WITH DR. FILART FOR DAMAGES SINCE
BY ITS OWN RULING, AMAY BISAYA "COULD NOT HAVE SUFFERED SUCH HUMILIATION," "WERE IT NOT FOR DR.
FILARTS INVITATION"
III.
DEPARTING FROM THE FINDINGS OF FACT OF THE TRIAL COURT AS REGARDS THE CIRCUMSTANCES THAT
ALLEGEDLY CAUSED THE HUMILIATION OF AMAY BISAYA
IV.
IN CONCLUDING THAT AMAY BISAYA WAS TREATED UNJUSTLY BECAUSE OF HIS POVERTY, CONSIDERING THAT
THIS WAS NEVER AN ISSUE AND NO EVIDENCE WAS PRESENTED IN THIS REGARD
V.
IN FAILING TO PASS UPON THE ISSUE ON THE DEFECTS OF THE APPELLANTS BRIEF, THEREBY DEPARTING
FROM THE ACCEPTED AND USUAL COURSE OF JUDICIAL PROCEEDINGS
Petitioners Lim and Hotel Nikko contend that pursuant to the doctrine of volenti non fit injuria, they cannot be made liable for
damages as respondent Reyes assumed the risk of being asked to leave (and being embarrassed and humiliated in the
process) as he was a "gate-crasher."
The doctrine of volenti non fit injuria ("to which a person assents is not esteemed in law as injury" 47 ) refers to self-inflicted
injury48 or to the consent to injury49 which precludes the recovery of damages by one who has knowingly and voluntarily
exposed himself to danger, even if he is not negligent in doing so. 50 As formulated by petitioners, however, this doctrine does not
find application to the case at bar because even if respondent Reyes assumed the risk of being asked to leave the party,
petitioners, under Articles 19 and 21 of the New Civil Code, were still under obligation to treat him fairly in order not to expose
him to unnecessary ridicule and shame.
Thus, the threshold issue is whether or not Ruby Lim acted abusively in asking Roberto Reyes, a.k.a. "Amay Bisaya," to leave
the party where he was not invited by the celebrant thereof thereby becoming liable under Articles 19 and 21 of the Civil Code.
Parenthetically, and if Ruby Lim were so liable, whether or not Hotel Nikko, as her employer, is solidarily liable with her.
As the trial court and the appellate court reached divergent and irreconcilable conclusions concerning the same facts and
evidence of the case, this Court is left without choice but to use its latent power to review such findings of facts. Indeed, the
general rule is that we are not a trier of facts as our jurisdiction is limited to reviewing and revising errors of law. 51 One of the
exceptions to this general rule, however, obtains herein as the findings of the Court of Appeals are contrary to those of the trial
court.52 The lower court ruled that Ms. Lim did not abuse her right to ask Mr. Reyes to leave the party as she talked to him
politely and discreetly. The appellate court, on the other hand, held that Ms. Lim is liable for damages as she needlessly
embarrassed Mr. Reyes by telling him not to finish his food and to leave the place within hearing distance of the other guests.
Both courts, however, were in agreement that it was Dr. Filarts invitation that brought Mr. Reyes to the party.
The consequential question then is: Which version is credible?
From an in depth review of the evidence, we find more credible the lower courts findings of fact.
First, let us put things in the proper perspective.
We are dealing with a formal party in a posh, five-star hotel, 53 for-invitation-only, thrown for the hotels former Manager, a
Japanese national. Then came a person who was clearly uninvited (by the celebrant) 54 and who could not just disappear into the
crowd as his face is known by many, being an actor. While he was already spotted by the organizer of the party, Ms. Lim, the
very person who generated the guest list, it did not yet appear that the celebrant was aware of his presence. Ms. Lim, mindful of
the celebrants instruction to keep the party intimate, would naturally want to get rid of the "gate-crasher" in the most hush-hush
manner in order not to call attention to a glitch in an otherwise seamless affair and, in the process, risk the displeasure of the
celebrant, her former boss. To unnecessarily call attention to the presence of Mr. Reyes would certainly reflect badly on Ms.
Lims ability to follow the instructions of the celebrant to invite only his close friends and some of the hotels personnel. Mr.
Reyes, upon whom the burden rests to prove that indeed Ms. Lim loudly and rudely ordered him to leave, could not offer any
satisfactory explanation why Ms. Lim would do that and risk ruining a formal and intimate affair. On the contrary, Mr. Reyes, on
cross-examination, had unwittingly sealed his fate by admitting that when Ms. Lim talked to him, she was very close. Close
enough for him to kiss:
Q: And, Mr. Reyes, you testified that Miss Lim approached you while you were at the buffet table? How close was she
when she approached you?
A: Very close because we nearly kissed each other.
Q: And yet, she shouted for you to go down? She was that close and she shouted?
A: Yes. She said, "wag kang kumain, hindi ka imbitado dito, bumaba ka na lang."
Q: So, you are testifying that she did this in a loud voice?
...

A: Yes. If it is not loud, it will not be heard by many.55


In the absence of any proof of motive on the part of Ms. Lim to humiliate Mr. Reyes and expose him to ridicule and shame, it is
highly unlikely that she would shout at him from a very close distance. Ms. Lim having been in the hotel business for twenty
years wherein being polite and discreet are virtues to be emulated, the testimony of Mr. Reyes that she acted to the contrary
does not inspire belief and is indeed incredible. Thus, the lower court was correct in observing that
Considering the closeness of defendant Lim to plaintiff when the request for the latter to leave the party was made such that
they nearly kissed each other, the request was meant to be heard by him only and there could have been no intention on her
part to cause embarrassment to him. It was plaintiffs reaction to the request that must have made the other guests aware of
what transpired between them. . .
Had plaintiff simply left the party as requested, there was no need for the police to take him out. 56
Moreover, another problem with Mr. Reyess version of the story is that it is unsupported. It is a basic rule in civil cases that he
who alleges proves. Mr. Reyes, however, had not presented any witness to back his story up. All his witnesses Danny
Rodinas, Pepito Guerrero and Alexander Silva - proved only that it was Dr. Filart who invited him to the party. 57
Ms. Lim, not having abused her right to ask Mr. Reyes to leave the party to which he was not invited, cannot be made liable to
pay for damages under Articles 19 and 21 of the Civil Code. Necessarily, neither can her employer, Hotel Nikko, be held liable
as its liability springs from that of its employee. 58
Article 19, known to contain what is commonly referred to as the principle of abuse of rights, 59 is not a panacea for all human
hurts and social grievances. Article 19 states:
Art. 19. Every person must, in the exercise of his rights and in the performance of his duties, act with justice, give everyone his
due, and observe honesty and good faith.1awphi1.nt
Elsewhere, we explained that when "a right is exercised in a manner which does not conform with the norms enshrined in Article
19 and results in damage to another, a legal wrong is thereby committed for which the wrongdoer must be responsible." 60 The
object of this article, therefore, is to set certain standards which must be observed not only in the exercise of ones rights but
also in the performance of ones duties.61 These standards are the following: act with justice, give everyone his due and observe
honesty and good faith.62 Its antithesis, necessarily, is any act evincing bad faith or intent to injure. Its elements are the
following: (1) There is a legal right or duty; (2) which is exercised in bad faith; (3) for the sole intent of prejudicing or injuring
another.63 When Article 19 is violated, an action for damages is proper under Articles 20 or 21 of the Civil Code. Article 20
pertains to damages arising from a violation of law 64 which does not obtain herein as Ms. Lim was perfectly within her right to
ask Mr. Reyes to leave. Article 21, on the other hand, states:
Art. 21. Any person who willfully causes loss or injury to another in a manner that is contrary to morals, good customs or public
policy shall compensate the latter for the damage.
Article 2165 refers to acts contra bonus mores and has the following elements: (1) There is an act which is legal; (2) but which is
contrary to morals, good custom, public order, or public policy; and (3) it is done with intent to injure.66
A common theme runs through Articles 19 and 21,67 and that is, the act complained of must be intentional. 68
As applied to herein case and as earlier discussed, Mr. Reyes has not shown that Ms. Lim was driven by animosity against him.
These two people did not know each other personally before the evening of 13 October 1994, thus, Mr. Reyes had nothing to
offer for an explanation for Ms. Lims alleged abusive conduct except the statement that Ms. Lim, being "single at 44 years old,"
had a "very strong bias and prejudice against (Mr. Reyes) possibly influenced by her associates in her work at the hotel with
foreign businessmen."69 The lameness of this argument need not be belabored. Suffice it to say that a complaint based on
Articles 19 and 21 of the Civil Code must necessarily fail if it has nothing to recommend it but innuendos and conjectures.
Parenthetically, the manner by which Ms. Lim asked Mr. Reyes to leave was likewise acceptable and humane under the
circumstances. In this regard, we cannot put our imprimatur on the appellate courts declaration that Ms. Lims act of personally
approaching Mr. Reyes (without first verifying from Mrs. Filart if indeed she invited Mr. Reyes) gave rise to a cause of action
"predicated upon mere rudeness or lack of consideration of one person, which calls not only protection of human dignity but
respect of such dignity."70 Without proof of any ill-motive on her part, Ms. Lims act of by-passing Mrs. Filart cannot amount to
abusive conduct especially because she did inquire from Mrs. Filarts companion who told her that Mrs. Filart did not invite Mr.
Reyes.71 If at all, Ms. Lim is guilty only of bad judgment which, if done with good intentions, cannot amount to bad faith.
Not being liable for both actual and moral damages, neither can petitioners Lim and Hotel Nikko be made answerable for
exemplary damages72 especially for the reason stated by the Court of Appeals. The Court of Appeals held
Not a few of the rich people treat the poor with contempt because of the latters lowly station in life.l^vvphi1.net This has to be
limited somewhere. In a democracy, such a limit must be established. Social equality is not sought by the legal provisions under
consideration, but due regard for decency and propriety (Code Commission, pp. 33-34). And by way of example or correction for
public good and to avert further commission of such acts, exemplary damages should be imposed upon appellees. 73
The fundamental fallacy in the above-quoted findings is that it runs counter with the very facts of the case and the evidence on
hand.l^vvphi1.net It is not disputed that at the time of the incident in question, Mr. Reyes was "an actor of long standing; a cohost of a radio program over DZRH; a Board Member of the Music Singer Composer (MUSICO) chaired by popular singer
Imelda Papin; a showbiz Coordinator of Citizen Crime Watch; and 1992 official candidate of the KBL Party for Governor of
Bohol; and an awardee of a number of humanitarian organizations of the Philippines." 74 During his direct examination on
rebuttal, Mr. Reyes stressed that he had income 75 and nowhere did he say otherwise. On the other hand, the records are bereft
of any information as to the social and economic standing of petitioner Ruby Lim. Consequently, the conclusion reached by the
appellate court cannot withstand scrutiny as it is without basis.
All told, and as far as Ms. Lim and Hotel Nikko are concerned, any damage which Mr. Reyes might have suffered through Ms.
Lims exercise of a legitimate right done within the bounds of propriety and good faith, must be his to bear alone.
WHEREFORE, premises considered, the petition filed by Ruby Lim and Nikko Hotel Manila Garden is GRANTED. The Decision
of the Court of Appeals dated 26 November 2001 and its Resolution dated 09 July 2002 are hereby REVERSED and SET
ASIDE. The Decision of the Regional Trial Court of Quezon City, Branch 104, dated 26 April 1999 is hereby AFFIRMED. No
costs.
SO ORDERED.

G.R. No. 142943

April 3, 2002

Spouses ANTONIO and LORNA QUISUMBING, petitioners,


vs.
MANILA ELECTRIC COMPANY (MERALCO), respondent.
PANGANIBAN, J.:
Under the law, the Manila Electric Company (Meralco) may immediately disconnect electric service on the ground of alleged
meter tampering, but only if the discovery of the cause is personally witnessed and attested to by an officer of the law or by a
duly authorized representative of the Energy Regulatory Board.
The Case
Before us is a Petition for Review under Rule 45 of the Rules of Court, assailing the February 1, 2000 Decision 1and the April 10,
2000 Resolution2 of the Court of Appeals (CA) in CA-GR SP No. 49022. The decretal portion of the said Decision reads as
follows:
"WHEREFORE, the challenged decision in Civil Case No. Q-95-23219 is hereby SET ASIDE and the complaint against
defendant-appellant MERALCO is hereby DISMISSED. Plaintiffs-appellees are herebyORDERED to pay defendantappellant MERALCO the differential billing of P193,332.00 representing the value of used but unregistered electrical
consumption."3
The assailed Resolution denied petitioner's Motion for Reconsideration.
The Facts
The facts of the case are summarized by the Court of Appeals in this wise:
"Defendant-appellant Manila Electric Company (MERALCO) is a private corporation, authorized by law to charge all
persons, including the government, for the consumption of electric power at rates duly authorized and approved by the
Board of Energy (now the Energy Regulatory Board).
"Plaintiffs-appellees Spouses Antonio and Lorna Quisumbing are owners of a house and lot located at No. 94
Greenmeadows Avenue, Quezon City, which they bought on April 7, 1994 from Ms. Carmina Serapio Santos. They
alleged to be business entrepreneurs engaged in the export of furnitures under the business name 'Loran Industries' and
recipient of the 1993 Agora Award and 1994 Golden Shell Award. Mrs. Quisumbing is a member of the Innerwheel Club
while Mr. Quisumbing is a member of the Rotary Club, Chairman of Cebu Chamber of Commerce, and Director of
Chamber of Furniture.
"On March 3, 1995 at around 9:00 a.m., defendant-appellant's inspectors headed by Emmanuel C. Orlino were assigned
to conduct a routine-on-the-spot inspection of all single phase meters at Greenmeadows Avenue. House no. 94 of Block
8, Lot 19 Greenmeadows Avenue owned by plaintiffs-appellees was inspected after observing a standard operating
procedure of asking permission from plaintiffs-appellees, through their secretary which was granted. The secretary
witnessed the inspection. After the inspection, defendant-appellant's inspectors discovered that the terminal seal of the
meter was missing; the meter cover seal was deformed; the meter dials of the meter was mis-aligned and there were
scratches on the meter base plate. Defendant-appellant's inspectors relayed the matter to plaintiffs-appellees' secretary,
who in turn relayed the same to plaintiff-appellee, Lorna Quisumbing, who was outraged of the result of the inspection and
denied liability as to the tampering of the meter. Plaintiffs-appellees were advised by defendant-appellant's inspectors that
they had to detach the meter and bring it to their laboratory for verification/confirmation of their findings. In the event the
meter turned out to be tampered, defendant-appellant had to temporarily disconnect the electric services of plaintiffsappellees. The laboratory testing conducted on the meter has the following findings to wit:
'1. Terminal seal was missing.
'2. Lead cover seals ('90 ERB 1-Meralco 21) were tampered by forcibly pulling out from the sealing wire.
'3. The 1000th, 100th and 10th dial pointers of the register were found out of alignment and with circular scratches at
the face of the register which indicates that the meter had been opened to manipulate the said dial pointers and set
manually to the desired reading. In addition to this, the meter terminal blades were found full of scratches.'
"After an hour, defendant-appellant's head inspector, E. Orlina returned to the residence of plaintiffs-appellees and
informed them that the meter had been tampered and unless they pay the amount ofP178,875.01 representing the
differential billing, their electric supply would be disconnected. Orlina informed plaintiffs-appellees that they were just
following their standard operating procedure. Plaintiffs-appellees were further advised that questions relative to the results
of the inspection as well as the disconnection of her electrical services for Violation of Contract (VOC) may be settled with
Mr. M. Manuson of the Special Accounts, Legal Service Department. However, on the same day at around 2:00 o'clock in
the afternoon defendant-appellant's officer through a two-way radio instructed its service inspector headed by Mr. Orlino to
reconnect plaintiffs-appellees' electric service which the latter faithfully complied.
"On March 6, 1995, plaintiffs-appellees filed a complaint for damages with prayer for the issuance of a writ of preliminary
mandatory injunction, despite the immediate reconnection, to order defendant-appellant to furnish electricity to the
plaintiffs-appellees alleging that defendant-appellant acted with wanton, capricious, malicious and malevolent manner in
disconnecting their power supply which was done without due process, and without due regard for their rights, feelings,
peace of mind, social and business reputation.
"In its Answer, defendant-appellant admitted disconnecting the electric service at the plaintiffs-appellees' house but denied
liability citing the 'Terms and Conditions of Service,' and Republic Act No. 7832 otherwise known a 'Anti-Electricity and
Electric Transmission Lines/Materials Pilferage Act of 1994.'
"After trial on the merits, the lower court rendered judgment, ruling in favor of plaintiffs-appellees." 4(Citations omitted)
Ruling of the Trial Court
The trial court held that Meralco (herein respondent) should have given the Quisumbing spouses (herein petitioners) ample
opportunity to dispute the alleged meter tampering.
It held that respondent had acted summarily and without procedural due process in immediately disconnecting the electric
service of petitioners. Respondent's action, ruled the RTC, constituted a quasi delict.
Ruling of the Court of Appeals
The Court of Appeals overturned the trial court's ruling and dismissed the Complaint. It held that respondent's representatives
had acted in good faith when they disconnected petitioners' electric service. Citing testimonial and documentary evidence, it
ruled that the disconnection was made only after observing due process. Further, it noted that petitioners had not been able to
prove their claim for damages. The appellate court likewise upheld respondent's counterclaim for the billing differential in the

amount of P193,3325 representing the value of petitioners' used but unregistered electrical consumption, which had been
established without being controverted.
Hence, this Petition.6
The Issues
In their Memorandum,7 petitioners submit the following issues for our consideration:
"4.1 Whether a prima facie presumption of tampering of electrical meter enumerated under Sec. 4 (a) iv of RA 7832 (AntiElectricity and Electric Transmission Lines/Materials Pilferage Act of 1994) may be had despite the absence of an ERB
representative or an officer of the law?
"4.2 Whether the enumeration of instances to establish a prima facie presumption of tampering of electrical meter
enumerated under Sec. 4 (a) iv of RA 7832 (Anti-Electricity and Electric Transmission Lines/Materials Pilferage Act of
1994) is exclusive?
"4.3 What constitutes notice prior to disconnection of electricity service? Corollarily, whether the definition of notice under
Meralco v. Court of Appeals (157 SCRA 243) applies to the case at bar?
"4.4 Whether a prima facie presumption may contradict logic?
"4.5 Whether documentary proof is pre-requisite for award of damages?" 8
In sum, this Petition raises three (3) main issues which this Court will address: (1) whether respondent observed the requisites
of law when it disconnected the electrical supply of petitioners, (2) whether such disconnection entitled petitioners to damages,
and (3) whether petitioners are liable for the billing differential computed by respondent.
The Court's Ruling
The Petition is partly meritorious.
First Issue:
Compliance with Requisites of Law
Petitioners contend that the immediate disconnection of electrical service was not validly effected because of respondent's
noncompliance with the relevant provisions of RA 7832, the "Anti-Electricity and Electric Transmission Lines/Materials Pilferage
Act of 1994." They insist that the immediate disconnection of electrical supply may only be validly effected only when there is
prima facie evidence of its illegal use. To constitute prima facie evidence, the discovery of the illegal use must be "personally
witnessed and attested to by an officer of the law or a duly authorized representative of the Energy Regulatory Board (ERB)."
Respondent, on the other hand, points out that the issue raised by petitioners is a question of fact which this Court cannot pass
upon. It argues further that this issue, which was not raised in the court below, can no longer be taken up for the first time on
appeal. Assuming arguendo that the issue was raised below, it also contends that petitioners were not able to specifically prove
the absence of an officer of the law or a duly authorized representative of the ERB when the discovery was made.1wphi1.nt
Prima facie Evidence of Illegal Use of Electricity
We agree with petitioners. Section 4 of RA 7832 states:
(a) The presence of any of the following circumstances shall constitute prima facie evidence of illegal use of electricity, as
defined in this Act, by the person benefitted thereby, and shall be the basis for: (1) the immediate disconnection by the
electric utility to such person after due notice, x x x
xxx

xxx

xxx

(viii) x x x Provided, however, That the discovery of any of the foregoing circumstances, in order to constituteprima
facie evidence, must be personally witnessed and attested to by an officer of the law or a duly authorized representative
of the Energy Regulatory Board (ERB)."9 (Italics supplied)
Under the above provision, the prima facie presumption that will authorize immediate disconnection will arise only upon the
satisfaction of certain requisites. One of these requisites is the personal witnessing and attestation by an officer of the law or by
an authorized ERB representative when the discovery was made.
As a rule, this Court reviews only questions of law, not of facts. However, it may pass upon the evidence when the factual
findings of the trial court are different from those of the Court of Appeals, as in this case. 10
A careful review of the evidence on record negates the appellate court's holding that "the actions of defendant-appellant's
service inspectors were all in accord with the requirement of the law." 11
Respondent's own witnesses provided the evidence on who were actually present when the inspection was made. Emmanuel
C. Orlino, the head of the Meralco team, testified:
"Q
When you were conducting this inspection, and you discovered these findings you testified earlier, who was
present?
A

The secretary, sir."12

"ATTY. REYES - Who else were the members of your team that conducted this inspection at Greenmeadows Avenue on
that day, March 3, 1995?
A

The composition of the team, sir?

Yes.

Including me, we are about four (4) inspectors, sir.

You were four (4)?

Yes, sir.

Who is the head of this team?

I was the head of the team, sir."13

Further, Catalino A. Macaraig, the area head of the Orlino team, stated that only Meralco personnel had been present during the
inspection:

"Q

By the way you were not there at Green Meadows on that day, right?

Yes, sir.

Only Mr. Orlino and who else were there?

Two or three of his men.

All members of the inspection team?

Yes, sir."14

These testimonies clearly show that at the time the alleged meter tampering was discovered, only the Meralco inspection team
and petitioners' secretary were present. Plainly, there was no officer of the law or ERB representative at that time. Because of
the absence of government representatives, the prima facie authority to disconnect, granted to Meralco by RA 7832, cannot
apply.
Neither can respondent find solace in the fact that petitioners' secretary was present at the time the inspection was made. The
law clearly states that for the prima facie evidence to apply, the discovery "must be personally witnessed and attested to by an
officer of the law or a duly authorized representative of the Energy Regulatory Board (ERB)." 15 Had the law intended the
presence of the owner or his/her representative to suffice, then it should have said so. Embedded in our jurisprudence is the
rule that courts may not construe a statute that is free from doubt. 16 Where the law is clear and unambiguous, it must be taken
to mean exactly what it says, and courts have no choice but to see to it that the mandate is obeyed. 17
In fact, during the Senate deliberations on RA 7832, Senator John H. Osmea, its author, stressed the need for the presence of
government officers during inspections of electric meters. He said:
"Mr. President, if a utility like MERALCO finds certain circumstances or situations which are listed in Section 2 of this bill to
be prima facie evidence, I think they should be prudent enough to bring in competent authority, either the police or the
NBI, to verify or substantiate their finding. If they were to summarily proceed to disconnect on the basis of their findings
and later on there would be a court case and the customer or the user would deny the existence of what is listed in
Section 2, then they could be in a lot of trouble." 18 (Italics supplied)
Neither can we accept respondent's argument that when the alleged tampered meter was brought to Meralco's laboratory for
testing, there was already an ERB representative present.
The law says that before immediate disconnection may be allowed, the discovery of the illegal use of electricity must have been
personally witnessed and attested to by an officer of the law or by an authorized ERB representative. In this case, the
disconnection was effected immediately after the discovery of the alleged meter tampering, which was witnessed only by
Meralco's employees. That the ERB representative was allegedly present when the meter was examined in the Meralco
laboratory will not cure the defect.
It is undisputed that after members of the Meralco team conducted their inspection and found alleged meter tampering, they
immediately disconnected petitioners' electrical supply. Again, this verity is culled from the testimony of Meralco's Orlina:
"A
When she went inside then she came out together with Mrs. Lourdes Quis[u]mbing at that time. We did tell our
findings regarding the meter and the consequence with it. And she was very angry with me.
Q

When you say consequence of your findings, what exactly did you tell Mrs. Quisumbing?

A
We told her that the service will be temporarily disconnected and that we are referring to our Legal Department so
could know the violation, sir."19
"A

Yes, sir. At that time, I referred her to Mr. Macaraig, sir.

What is the fist name of this supervisor?

Mr. Catalino Macara[i]g, sir.

Then after talking to Mr. Catalino Macara[i]g, this is over the telephone, what happened?

A
The supervisor advised her that the service will be temporarily disconnected and she has to go to our Legal
Department where she could settle the VOC, sir.
Q

You are talking of 'VOC,' what is this all about Mr. Orlino?

'VOC' is violation of contract, sir."20

As to respondent's argument that the presence of an authorized ERB representative had not been raised below, it is clear,
however, that the issue of due process was brought up by petitioners as a valid issue in the CA. The presence of government
agents who may authorize immediate disconnections go into the essence of due process. Indeed, we cannot allow respondent
to act virtually as prosecutor and judge in imposing the penalty of disconnection due to alleged meter tampering. That would not
sit well in a democratic country. After all, Meralco is a monopoly that derives its power from the government. Clothing it with
unilateral authority to disconnect would be equivalent to giving it a license to tyrannize its hapless customers.
Besides, even if not specifically raised, this Court has already ruled that "[w]here the issues already raised also rest on other
issues not specifically presented, as long as the latter issues bear relevance and close relation to the former and as long as
they arise from matters on record, the Court has the authority to include them in its discussion of the controversy as well as to
pass upon them."21
Contractual Right to Disconnect
Electrical Service
Neither may respondent rely on its alleged contractual right to disconnect electrical service based on Exhibits "10" 22 and
"11,"23 or on Decisions of the Board of Energy (now the Energy Regulatory Board). The relevant portion of these documents
concerns discontinuance of service. It provides:
"The Company reserves the right to discontinue service in case the Customer is in arrears in the payment of bills or for
failure to pay the adjusted bills in those cases where the meter stopped or failed to register the correct amount of energy
consumed, or for failure to comply with any of these terms and conditions, or in case of or to prevent fraud upon the
Company. Before disconnection is made in case of or to prevent fraud, the Company may adjust the bill of said Customer
accordingly and if the adjusted bill is not paid, the Company may disconnect the same. In case of disconnection, the
provisions of Revised Order No. 1 of the former Public Service Commission (now the Board of Energy) shall be observed.
Any such suspension of service shall not terminate the contract between the Company and the Customer." 24
Petitioners' situation can fall under disconnection only "in case of or to prevent fraud upon the Company." However, this too has
requisites before a disconnection may be made. An adjusted bill shall be prepared, and only upon failure to pay it may the

company discontinue service. This is also true in regard to the provisions of Revised Order No. 1 of the former Public Service
Commission, which requires a 48-hour written notice before a disconnection may be justified. In the instant case, these
requisites were obviously not complied with.
Second Issue
Damages
Having ruled that the immediate disconnection effected by Meralco lacks legal, factual or contractual basis, we will now pass
upon on the right of petitioners to recover damages for the improper disconnection.
Petitioners are asking for the reinstatement of the RTC Decision, which awarded them actual, moral and exemplary damages as
well as attorney's fees. All these were overturned by the CA.
As to actual damages, we agree with the CA that competent proof is necessary before our award may be made. The appellate
court ruled as follows:
"Considering further, it is a settled rule that in order for damages to be recovered, the best evidence obtainable by the
injured party must be presented. Actual and compensatory damages cannot be presumed but must be duly proved and
proved with reasonable degree and certainty. A court cannot rely on speculation, conjecture or guess work as to the fact
and amount of damages, but must depend upon competent proof that they have been suffered and on evidence of actual
amount thereof. If the proof is flimsy and unsubstantial, no damages will be awarded." 25
Actual damages are compensation for an injury that will put the injured party in the position where it was before it was
injured.26 They pertain to such injuries or losses that are actually sustained and susceptible of measurement. 27 Except as
provided by law or by stipulation, a party is entitled to an adequate compensation only for such pecuniary loss as it has duly
proven.28
Basic is the rule that to recover actual damages, not only must the amount of loss be capable of proof; it must also be actually
proven with a reasonable degree of certainty, premised upon competent proof or the best evidence obtainable. 29
Petitioners' claim for actual damages was premised only upon Lorna Quisumbing's bare testimony as follows:
"A

Actually that da[y] I was really scheduled to go to that furniture exhibit. That furniture exhibit is only once a year.

What is this furniture exhibit?

A
The SITEM, that is a government agency that takes care of exporters and exclusive marketing of our products
around the world. We always have that once a year and that's the time when all our buyers are here for us to show what
we had that was exhibited to go around. So, my husband had to [fly] from Cebu to Manila just for this occasion. So we
have an appointment with our people and our buyers with SITEM and also that evening we will have to treat them [to]
dinner.
Q

Whereat?

At our residence, we were supposed to have a dinner at our residence.

What happened to this occasion?

So when they disconnected our electric power we had to get in touch with them and change the venue.

Which venue did you transfer your dinner for your buyers?

A
We brought them in a restaurant in Makati at Season's Restaurant. But it was very embar[r]assing for us because
we faxed them ahead of time before they came to Manila.
Q
Now as a result of this change of your schedule because of the disconnection of the electric power on that day,
Friday, what damage did you suffer?
A

I cancelled the catering service and that is so much of a h[a]ssle it was so embarras[s]ing for us.

Can you tell us how much amount?

Approximately P50,000.00."30

No other evidence has been proffered to substantiate her bare statements. She has not shown how she arrived at the amount
of P50,000; it is, at best, speculative. Her self-serving testimonial evidence, if it may be called such, is insufficient to support
alleged actual damages.
While respondent does not rebut this testimony on the expenses incurred by the spouses in moving the dinner out of their
residence due to the disconnection, no receipts covering such expenditures have been adduced in evidence. Neither is the
testimony corroborated. To reiterate, actual or compensatory damages cannot be presumed, but must be duly proved with a
reasonable degree of certainty. It is dependent upon competent proof of damages that petitioners have suffered and of the
actual amount thereof.31 The award must be based on the evidence presented, not on the personal knowledge of the court; and
certainly not on flimsy, remote, speculative and unsubstantial proof. 32 Consequently, we uphold the CA ruling denying the grant
of actual damages.
Having said that, we agree with the trial court, however, that petitioners are entitled to moral damages, albeit in a reduced
amount.
The RTC opined as follows:
"This Court agrees with the defendant regarding [its] right by law and equity to protect itself from any fraud. However, such
right should not be exercised arbitrarily but with great caution and with due regard to the rights of the consumers. Meralco
having a virtual monopoly of the supply of electric power should refrain from taking drastic actions against the consumers
without observing due process. Even assuming that the subject meter has had history of meter tampering, defendant
cannot simply assume that the present occupants are the ones responsible for such tampering. Neither does it serve as a
license to deprive the plaintiffs of their right to due process. Defendant should have given the plaintiffs simple opportunity
to dispute the electric charges brought about by the alleged meter-tampering, which were not included in the bill rendered
them. Procedural due process requires reasonable notice to pay the bill and reasonable notice to discontinue supply.
Absent due process the defendant may be held liable for damages. While this Court is aware of the practice of
unscrupulous individuals of stealing electric curre[n]t which causes thousands if not millions of pesos in lost revenue to
electric companies, this does not give the defendant the right to trample upon the rights of the consumers by denying
them due process."33
Article 2219 of the Civil Code lists the instances when moral damages may be recovered. One such case 34 is when the rights of
individuals, including the right against deprivation of property without due process of law, are violated. 35

Moral damages include physical suffering, mental anguish, fright, serious anxiety, besmirched reputation, wounded feelings,
moral shock, social humiliation, and similar injury.36 Although incapable of pecuniary computation, such damages may be
recovered if they are the proximate results of the defendant's wrongful act or omission. 37
Case law establishes the following requisites for the award of moral damages: (1) there is an injury -- whether physical, mental
or psychological -- clearly sustained by the claimant; (2) there is a culpable act or omission factually established; (3) the
wrongful act or omission of the defendant is the proximate cause of the injury sustained by the claimant; and (4) the award of
damages is predicated on any of the cases stated in Article 2219 of the Civil Code. 38
To reiterate, respondent had no legal right to immediately disconnect petitioners' electrical supply without observing the
requisites of law which, in turn, are akin to due process. Had respondent been more circumspect and prudent, petitioners could
have been given the opportunity to controvert the initial finding of alleged meter tampering. Said the RTC:
"More seriously, the action of the defendant in maliciously disconnecting the electric service constitutes a breach of public
policy. For public utilities, broad as their powers are, have a clear duty to see to it that they do not violate nor transgress
the rights of the consumers. Any act on their part that militates against the ordinary norms of justice and fair play is
considered an infraction that gives rise to an action for damages. Such is the case at bar." 39
Indeed, the Supreme Court has ruled in Meralco v. CA40 that respondent is required to give notice of disconnection to an alleged
delinquent customer. The Court said:
"x x x One can not deny the vital role which a public utility such as MERALCO, having a monopoly of the supply of
electrical power in Metro Manila and some nearby municipalities, plays in the life of people living in such areas. Electricity
has become a necessity to most people in these areas, justifying the exercise by the State of its regulatory power over the
business of supplying electrical service to the public, in which petitioner MERALCO is engaged. Thus, the state may
regulate, as it has done through Section 97 of the Revised Order No. 1 of the Public Service Commission, the conditions
under which and the manner by which a public utility such as MERALCO may effect a disconnection of service to a
delinquent customer. Among others, a prior written notice to the customer is required before disconnection of the service.
Failure to give such prior notice amounts to a tort." 41
Observance of the rights of our people is sacred in our society. We cannot allow such rights to be trifled with or trivialized.
Although the Court sympathizes with respondent's efforts to stamp out the illegal use of electricity, such action must be done
only with strict observance of the rights of our people. As has been we succinctly said: "there is a right way to do the right thing
at the right time for the right reason."42
However, the amount of moral damages, which is left largely to the sound discretion of the courts, should be granted in
reasonable amounts, considering the attendant facts and circumstances. 43 Moral damages, though incapable of pecuniary
estimation, are designed to compensate the claimant for actual injury suffered and not to impose a penalty. 44 Moral damages
are not intended to enrich a plaintiff at the expense of the defendant. 45 They are awarded only to obtain a means, a diversion or
an amusement that will serve to alleviate the moral suffering the injured party has undergone by reason of the defendant's
culpable action.46 They must be proportionate to the suffering inflicted. 47
It is clear from the records that respondent was able to restore the electrical supply of petitioners on the same day. Verily, the
inconvenience and anxiety they suffered as a result of the disconnection was thereafter corrected. Thus, we reduce the RTC's
grant of moral damages to the more equitable amount of P100,000.
Exemplary damages, on the other hand, are imposed by way of example or correction for the public good in addition to moral,
temperate, liquidated or compensatory damages. 48 It is not given to enrich one party and impoverish another, but to serve as a
deterrent against or as a negative incentive to socially deleterious actions. 49In this case, to serve an example -- that before a
disconnection of electrical supply can be effected by a public utility like Meralco, the requisites of law must be faithfully complied
with -- we award the amount of P50,000 to petitioners.
Finally, with the award of exemplary damages, the award of attorney's fees is likewise granted. 50 It is readily apparent that
petitioners needed the services of a lawyer to argue their cause, even to the extent of elevating the matter to this Court; 51 thus,
an award of P50,000 is considered sufficient.
Final Issue:
Billing Differential
Finally, this Court holds that despite the basis for the award of damages -- the lack of due process in immediately disconnecting
petitioners' electrical supply -- respondent's counterclaim for the billing differential is still proper. We agree with the CA that
respondent should be given what it rightfully deserves. The evidence it presented, both documentary and testimonial, sufficiently
proved the amount of the differential.
Not only did respondent show how the meter examination had been conducted by its experts, but it also established the amount
of P193,332.96 that petitioners owed respondent. The procedure through which this amount was arrived at was testified to by
Meralco's Senior Billing Computer Enrique Katipunan. His testimony was corroborated by documentary evidence showing the
account's billing history and the corresponding computations. Neither do we doubt the documents of inspections and
examinations presented by respondent to prove that, indeed there had been meter tampering that resulted in unrecorded and
unpaid electrical consumption.
The mere presentation by petitioners of a Contract to Sell with Assumption of Mortgage 52 does not necessarily mean that they
are no longer liable for the billing differential. There was no sufficient evidence to show that they had not been actually residing
in the house before the date of the said document. Lorna Quisumbing herself admitted 53 that they did not have any contract for
electrical service in their own name. Hence, petitioners effectively assumed the bills of the former occupants of the premises.
Finally, the CA was correct in ruling that the convincing documentary and testimonial evidence presented by respondent, was
not controverted by petitioners.1wphi1.nt
WHEREFORE, the Petition is hereby PARTLY GRANTED. The assailed CA Decision is MODIFIED as follows: petitioners
are ORDERED to pay respondent the billing differential of P193,332.96; while respondent is ordered to pay
petitioners P100,000 as moral damages, P50,000 as exemplary damages, and P50,000 as attorney's fees. No pronouncement
as to costs.
SO ORDERED.

G.R. No. 132344

February 17, 2000

UNIVERSITY OF THE EAST, petitioner,


vs.
ROMEO A. JADER, respondent.
YNARES-SANTIAGO, J.:
May an educational institution be held liable for damages for misleading a student into believing that the latter had satisfied all
the requirements for graduation when such is not the case? This is the issue in the instant petition for review premised on the
following undisputed facts as summarized by the trial court and adopted by the Court of Appeals (CA), 1 to wit:
Plaintiff was enrolled in the defendants' College of Law from 1984 up to 1988. In the first semester of his last year (School
year 1987-1988), he failed to take the regular final examination in Practice Court I for which he was given an incomplete
grade (Exhibits "2", also Exhibit "H"). He enrolled for the second semester as fourth year law student (Exhibit "A") and on
February 1, 1988 he filed an application for the removal of the incomplete grade given him by Professor Carlos Ortega
(Exhibits "H-2", also Exhibit "2") which was approved by Dean Celedonio Tiongson after payment of the required fee. He
took the examination on March 28, 1988. On May 30, 1988, Professor Carlos Ortega submitted his grade. It was a grade
of five (5). (Exhibits "H-4", also Exhibits "2-L", "2-N").1wphi1.nt
In the meantime, the Dean and the Faculty Members of the College of Law met to deliberate on who among the fourth
year students should be allowed to graduate. The plaintiff's name appeared in the Tentative List of Candidates for
graduation for the Degree of Bachelor of Laws (LL.B) as of Second Semester (1987-1988) with the following annotation:
JADER ROMEO A.
Def. Conflict of Laws x-1-87-88, Practice Court I Inc., 1-87-88 C-1 to submit transcript with S.O. (Exhibits "3", "3-C-1",
"3-C-2").
The 35th Investitures & Commencement Ceremonies for the candidates of Bachelor of Laws was scheduled on the 16th
of April 1988 at 3:00 o'clock in the afternoon, and in the invitation for that occasion the name of the plaintiff appeared as
one of the candidates. (Exhibits "B", "B-6", "B-6-A"). At the foot of the list of the names of the candidates there appeared
however the following annotation:
This is a tentative list Degrees will be conferred upon these candidates who satisfactorily complete requirements as
stated in the University Bulletin and as approved of the Department of Education, Culture and Sports (Exhibit "B-7A").
The plaintiff attended the investiture ceremonies at F. dela Cruz Quadrangle, U.E., Recto Campus, during the program of
which he went up the stage when his name was called, escorted by her (sic) mother and his eldest brother who assisted
in placing the Hood, and his Tassel was turned from left to right, and he was thereafter handed by Dean Celedonio a rolled
white sheet of paper symbolical of the Law Diploma. His relatives took pictures of the occasion (Exhibits "C" to "C-6", "D3" to "D-11").
He tendered a blow-out that evening which was attended by neighbors, friends and relatives who wished him good luck in
the forthcoming bar examination. There were pictures taken too during the blow-out (Exhibits "D" to "D-1").
He thereafter prepared himself for the bar examination. He took a leave of absence without pay from his job from April 20,
1988 to September 30, 1988 (Exhibit "G") and enrolled at the pre-bar review class in Far Eastern University. (Exhibits "F"
to "F-2"). Having learned of the deficiency he dropped his review class and was not able to take the bar examination. 2
Consequently, respondent sued petitioner for damages alleging that he suffered moral shock, mental anguish, serious anxiety,
besmirched reputation, wounded feelings and sleepless nights when he was not able to take the 1988 bar examinations arising
from the latter's negligence. He prayed for an award of moral and exemplary damages, unrealized income, attorney's fees, and
costs of suit.
In its answer with counterclaim, petitioner denied liability arguing mainly that it never led respondent to believe that he
completed the requirements for a Bachelor of Laws degree when his name was included in the tentative list of graduating
students. After trial, the lower court rendered judgment as follows:
WHEREFORE, in view of the foregoing judgment is hereby rendered in favor of the plaintiff and against the defendant
ordering the latter to pay plaintiff the sum of THIRTY FIVE THOUSAND FOUR HUNDRED SEVENTY PESOS
(P35,470.00) with legal rate of interest from the filing of the complaint until fully paid, the amount of FIVE THOUSAND
PESOS (P5,000.00) as attorney's fees and the cost of suit.
Defendant's counterclaim is, for lack of merit, hereby dismissed.
SO ORDERED.3
which on appeal by both parties was affirmed by the Court of Appeals (CA) with modification. The dispositive portion of the CA
decision reads:
WHEREFORE, in the light of the foregoing, the lower Court's Decision is hereby AFFIRMED with the MODIFICATION that
defendant-appellee, in addition to the sum adjudged by the lower court in favor of plaintiff-appellant, is also ORDERED to
pay plaintiff-appellant the amount of FIFTY THOUSAND (P50,000.00) PESOS for moral damages. Costs against
defendant-appellee.
SO ORDERED.4
Upon the denial of its motion for reconsideration, petitioner UE elevated the case to this Court on a petition for review under
Rule 45 of the Rules of Court, arguing that it has no liability to respondent Romeo A. Jader, considering that the proximate and
immediate cause of the alleged damages incurred by the latter arose out of his own negligence in not verifying from the
professor concerned the result of his removal exam.
The petition lacks merit.
When a student is enrolled in any educational or learning institution, a contract of education is entered into between said
institution and the student. The professors, teachers or instructors hired by the school are considered merely as agents and
administrators tasked to perform the school's commitment under the contract. Since the contracting parties are the school and
the student, the latter is not duty-bound to deal with the former's agents, such as the professors with respect to the status or
result of his grades, although nothing prevents either professors or students from sharing with each other such information. The
Court takes judicial notice of the traditional practice in educational institutions wherein the professor directly furnishes his/her
students their grades. It is the contractual obligation of the school to timely inform and furnish sufficient notice and information to
each and every student as to whether he or she had already complied with all the requirements for the conferment of a degree
or whether they would be included among those who will graduate. Although commencement exercises are but a formal
ceremony, it nonetheless is not an ordinary occasion, since such ceremony is the educational institution's way of announcing to

the whole world that the students included in the list of those who will be conferred a degree during the baccalaureate ceremony
have satisfied all the requirements for such degree. Prior or subsequent to the ceremony, the school has the obligation to
promptly inform the student of any problem involving the latter's grades and performance and also most importantly, of the
procedures for remedying the same.
Petitioner, in belatedly informing respondent of the result of the removal examination, particularly at a time when he had already
commenced preparing for the bar exams, cannot be said to have acted in good faith. Absence of good faith must be sufficiently
established for a successful prosecution by the aggrieved party in a suit for abuse of right under Article 19 of the Civil Code.
Good faith connotes an honest intention to abstain from taking undue advantage of another, even though the forms and
technicalities of the law, together with the absence of all information or belief of facts, would render the transaction
unconscientious.5 It is the school that has access to those information and it is only the school that can compel its professors to
act and comply with its rules, regulations and policies with respect to the computation and the prompt submission of grades.
Students do not exercise control, much less influence, over the way an educational institution should run its affairs, particularly
in disciplining its professors and teachers and ensuring their compliance with the school's rules and orders. Being the party that
hired them, it is the school that exercises general supervision and exclusive control over the professors with respect to the
submission of reports involving the students' standing. Exclusive control means that no other person or entity had any control
over the instrumentality which caused the damage or injury.6
The college dean is the senior officer responsible for the operation of an academic program, enforcement of rules and
regulations, and the supervision of faculty and student services. 7 He must see to it that his own professors and teachers,
regardless of their status or position outside of the university, must comply with the rules set by the latter. The negligent act of a
professor who fails to observe the rules of the school, for instance by not promptly submitting a student's grade, is not only
imputable to the professor but is an act of the school, being his employer.
Considering further, that the institution of learning involved herein is a university which is engaged in legal education, it should
have practiced what it inculcates in its students, more specifically the principle of good dealings enshrined in Articles 19 and 20
of the Civil Code which states:
Art. 19. Every person must, in the exercise of his rights and in the performance of his duties, act with justice, give
everyone his due, and observe honesty and good faith.
Art. 20. Every person who, contrary to law, wilfully or negligently causes damage to another, shall indemnify the latter for
the same.
Art. 19 was intended to expand the concept of torts by granting adequate legal remedy for the untold number of moral wrongs
which is impossible for human foresight to provide specifically in statutory law.8 In civilized society, men must be able to assume
that others will do them no intended injury that others will commit no internal aggressions upon them; that their fellowmen,
when they act affirmatively will do so with due care which the ordinary understanding and moral sense of the community exacts
and that those with whom they deal in the general course of society will act in good faith. The ultimate thing in the theory of
liability is justifiable reliance under conditions of civilized society.9 Schools and professors cannot just take students for granted
and be indifferent to them, for without the latter, the former are useless.
Educational institutions are duty-bound to inform the students of their academic status and not wait for the latter to inquire from
the former. The conscious indifference of a person to the rights or welfare of the person/persons who may be affected by his act
or omission can support a claim for damages.10 Want of care to the conscious disregard of civil obligations coupled with a
conscious knowledge of the cause naturally calculated to produce them would make the erring party liable. 11 Petitioner ought to
have known that time was of the essence in the performance of its obligation to inform respondent of his grade. It cannot feign
ignorance that respondent will not prepare himself for the bar exams since that is precisely the immediate concern after
graduation of an LL.B. graduate. It failed to act seasonably. Petitioner cannot just give out its student's grades at any time
because a student has to comply with certain deadlines set by the Supreme Court on the submission of requirements for taking
the bar. Petitioner's liability arose from its failure to promptly inform respondent of the result of an examination and in misleading
the latter into believing that he had satisfied all requirements for the course. Worth quoting is the following disquisition of the
respondent court:
It is apparent from the testimony of Dean Tiongson that defendant-appellee University had been informed during the
deliberation that the professor in Practice Court I gave plaintiff-appellant a failing grade. Yet, defendant-appellee still did
not inform plaintiff-appellant of his failure to complete the requirements for the degree nor did they remove his name from
the tentative list of candidates for graduation. Worse, defendant-appellee university, despite the knowledge that plaintiffappellant failed in Practice Court I, again included plaintiff-appellant's name in the "tentative list of candidates for
graduation which was prepared after the deliberation and which became the basis for the commencement rites program.
Dean Tiongson reasons out that plaintiff-appellant's name was allowed to remain in the tentative list of candidates for
graduation in the hope that the latter would still be able to remedy the situation in the remaining few days before
graduation day. Dean Tiongson, however, did not explain how plaintiff appellant Jader could have done something to
complete his deficiency if defendant-appellee university did not exert any effort to inform plaintiff-appellant of his failing
grade in Practice Court I.12
Petitioner cannot pass on its blame to the professors to justify its own negligence that led to the delayed relay of information to
respondent. When one of two innocent parties must suffer, he through whose agency the loss occurred must bear it. 13 The
modern tendency is to grant indemnity for damages in cases where there is abuse of right, even when the act is not illicit. 14 If
mere fault or negligence in one's acts can make him liable for damages for injury caused thereby, with more reason should
abuse or bad faith make him liable. A person should be protected only when he acts in the legitimate exercise of his right, that
is, when he acts with prudence and in good faith, but not when he acts with negligence or abuse. 15
However, while petitioner was guilty of negligence and thus liable to respondent for the latter's actual damages, we hold that
respondent should not have been awarded moral damages. We do not agree with the Court of Appeals' findings that respondent
suffered shock, trauma and pain when he was informed that he could not graduate and will not be allowed to take the bar
examinations. At the very least, it behooved on respondent to verify for himself whether he has completed all necessary
requirements to be eligible for the bar examinations. As a senior law student, respondent should have been responsible enough
to ensure that all his affairs, specifically those pertaining to his academic achievement, are in order. Given these considerations,
we fail to see how respondent could have suffered untold embarrassment in attending the graduation rites, enrolling in the bar
review classes and not being able to take the bar exams. If respondent was indeed humiliated by his failure to take the bar, he
brought this upon himself by not verifying if he has satisfied all the requirements including his school records, before preparing
himself for the bar examination. Certainly, taking the bar examinations does not only entail a mental preparation on the subjects
thereof; there are also prerequisites of documentation and submission of requirements which the prospective examinee must
meet.
WHEREFORE, the assailed decision of the Court of Appeals is AFFIRMED with MODIFICATION. Petitioner is ORDERED to
PAY respondent the sum of Thirty-five Thousand Four Hundred Seventy Pesos (P35,470.00), with legal interest of 6% per
annum computed from the date of filing of the complaint until fully paid; the amount of Five Thousand Pesos (P5,000.00) as
attorney's fees; and the costs of the suit. The award of moral damages is DELEIED.1wphi1.nt
SO ORDERED.

G.R. No. 81262 August 25, 1989


GLOBE MACKAY CABLE AND RADIO CORP., and HERBERT C. HENDRY, petitioners,
vs.
THE HONORABLE COURT OF APPEALS and RESTITUTO M. TOBIAS, respondents.
Atencia & Arias Law Offices for petitioners.
Romulo C. Felizmena for private respondent.

CORTES, J.:
Private respondent Restituto M. Tobias was employed by petitioner Globe Mackay Cable and Radio Corporation (GLOBE
MACKAY) in a dual capacity as a purchasing agent and administrative assistant to the engineering operations manager. In
1972, GLOBE MACKAY discovered fictitious purchases and other fraudulent transactions for which it lost several thousands of
pesos.
According to private respondent it was he who actually discovered the anomalies and reported them on November 10, 1972 to
his immediate superior Eduardo T. Ferraren and to petitioner Herbert C. Hendry who was then the Executive Vice-President and
General Manager of GLOBE MACKAY.
On November 11, 1972, one day after private respondent Tobias made the report, petitioner Hendry confronted him by stating
that he was the number one suspect, and ordered him to take a one week forced leave, not to communicate with the office, to
leave his table drawers open, and to leave the office keys.
On November 20, 1972, when private respondent Tobias returned to work after the forced leave, petitioner Hendry went up to
him and called him a "crook" and a "swindler." Tobias was then ordered to take a lie detector test. He was also instructed to
submit specimen of his handwriting, signature, and initials for examination by the police investigators to determine his complicity
in the anomalies.
On December 6,1972, the Manila police investigators submitted a laboratory crime report (Exh. "A") clearing private respondent
of participation in the anomalies.
Not satisfied with the police report, petitioners hired a private investigator, retired Col. Jose G. Fernandez, who on December
10, 1972, submitted a report (Exh. "2") finding Tobias guilty. This report however expressly stated that further investigation was
still to be conducted.
Nevertheless, on December 12, 1972, petitioner Hendry issued a memorandum suspending Tobias from work preparatory to the
filing of criminal charges against him.
On December 19,1972, Lt. Dioscoro V. Tagle, Metro Manila Police Chief Document Examiner, after investigating other
documents pertaining to the alleged anomalous transactions, submitted a second laboratory crime report (Exh. "B") reiterating
his previous finding that the handwritings, signatures, and initials appearing in the checks and other documents involved in the
fraudulent transactions were not those of Tobias. The lie detector tests conducted on Tobias also yielded negative results.
Notwithstanding the two police reports exculpating Tobias from the anomalies and the fact that the report of the private
investigator, was, by its own terms, not yet complete, petitioners filed with the City Fiscal of Manila a complaint for estafa
through falsification of commercial documents, later amended to just estafa. Subsequently five other criminal complaints were
filed against Tobias, four of which were for estafa through Falsification of commercial document while the fifth was for of Article
290 of' the Revised Penal Code (Discovering Secrets Through Seizure of Correspondence).lwph1.t Two of these
complaints were refiled with the Judge Advocate General's Office, which however, remanded them to the fiscal's office. All of the
six criminal complaints were dismissed by the fiscal. Petitioners appealed four of the fiscal's resolutions dismissing the criminal
complaints with the Secretary of Justice, who, however, affirmed their dismissal.
In the meantime, on January 17, 1973, Tobias received a notice (Exh. "F") from petitioners that his employment has been
terminated effective December 13, 1972. Whereupon, Tobias filed a complaint for illegal dismissal. The labor arbiter dismissed
the complaint. On appeal, the National Labor Relations Commission (NLRC) reversed the labor arbiter's decision. However, the
Secretary of Labor, acting on petitioners' appeal from the NLRC ruling, reinstated the labor arbiter's decision. Tobias appealed
the Secretary of Labor's order with the Office of the President. During the pendency of the appeal with said office, petitioners
and private respondent Tobias entered into a compromise agreement regarding the latter's complaint for illegal dismissal.
Unemployed, Tobias sought employment with the Republic Telephone Company (RETELCO). However, petitioner Hendry,
without being asked by RETELCO, wrote a letter to the latter stating that Tobias was dismissed by GLOBE MACKAY due to
dishonesty.
Private respondent Tobias filed a civil case for damages anchored on alleged unlawful, malicious, oppressive, and abusive acts
of petitioners. Petitioner Hendry, claiming illness, did not testify during the hearings. The Regional Trial Court (RTC) of Manila,
Branch IX, through Judge Manuel T. Reyes rendered judgment in favor of private respondent by ordering petitioners to pay him
eighty thousand pesos (P80,000.00) as actual damages, two hundred thousand pesos (P200,000.00) as moral damages,
twenty thousand pesos (P20,000.00) as exemplary damages, thirty thousand pesos (P30,000.00) as attorney's fees, and costs.
Petitioners appealed the RTC decision to the Court of Appeals. On the other hand, Tobias appealed as to the amount of
damages. However, the Court of Appeals, an a decision dated August 31, 1987 affirmed the RTC decision in toto. Petitioners'
motion for reconsideration having been denied, the instant petition for review on certiorari was filed.
The main issue in this case is whether or not petitioners are liable for damages to private respondent.
Petitioners contend that they could not be made liable for damages in the lawful exercise of their right to dismiss private
respondent.
On the other hand, private respondent contends that because of petitioners' abusive manner in dismissing him as well as for the
inhuman treatment he got from them, the Petitioners must indemnify him for the damage that he had suffered.
One of the more notable innovations of the New Civil Code is the codification of "some basic principles that are to be observed
for the rightful relationship between human beings and for the stability of the social order." [REPORT ON THE CODE
COMMISSION ON THE PROPOSED CIVIL CODE OF THE PHILIPPINES, p. 39]. The framers of the Code, seeking to remedy
the defect of the old Code which merely stated the effects of the law, but failed to draw out its spirit, incorporated certain
fundamental precepts which were "designed to indicate certain norms that spring from the fountain of good conscience" and
which were also meant to serve as "guides for human conduct [that] should run as golden threads through society, to the end
that law may approach its supreme ideal, which is the sway and dominance of justice" (Id.) Foremost among these principles is
that pronounced in Article 19 which provides:
Art. 19. Every person must, in the exercise of his rights and in the performance of his duties, act with justice, give
everyone his due, and observe honesty and good faith.

This article, known to contain what is commonly referred to as the principle of abuse of rights, sets certain standards which must
be observed not only in the exercise of one's rights but also in the performance of one's duties. These standards are the
following: to act with justice; to give everyone his due; and to observe honesty and good faith. The law, therefore, recognizes a
primordial limitation on all rights; that in their exercise, the norms of human conduct set forth in Article 19 must be observed. A
right, though by itself legal because recognized or granted by law as such, may nevertheless become the source of some
illegality. When a right is exercised in a manner which does not conform with the norms enshrined in Article 19 and results in
damage to another, a legal wrong is thereby committed for which the wrongdoer must be held responsible. But while Article 19
lays down a rule of conduct for the government of human relations and for the maintenance of social order, it does not provide a
remedy for its violation. Generally, an action for damages under either Article 20 or Article 21 would be proper.
Article 20, which pertains to damage arising from a violation of law, provides that:
Art. 20. Every person who contrary to law, wilfully or negligently causes damage to another, shall indemnify the latter
for the same.
However, in the case at bar, petitioners claim that they did not violate any provision of law since they were merely exercising
their legal right to dismiss private respondent. This does not, however, leave private respondent with no relief because Article 21
of the Civil Code provides that:
Art. 21. Any person who wilfully causes loss or injury to another in a manner that is contrary to morals, good
customs or public policy shall compensate the latter for the damage.
This article, adopted to remedy the "countless gaps in the statutes, which leave so many victims of moral wrongs helpless, even
though they have actually suffered material and moral injury" [Id.] should "vouchsafe adequate legal remedy for that untold
number of moral wrongs which it is impossible for human foresight to provide for specifically in the statutes" [Id. it p. 40; See
also PNB v. CA, G.R. No. L-27155, May 18,1978, 83 SCRA 237, 247].
In determining whether or not the principle of abuse of rights may be invoked, there is no rigid test which can be applied. While
the Court has not hesitated to apply Article 19 whether the legal and factual circumstances called for its application [See for e.g.,
Velayo v. Shell Co. of the Phil., Ltd., 100 Phil. 186 (1956); PNB v. CA, supra;Grand Union Supermarket, Inc. v. Espino, Jr., G.R.
No. L-48250, December 28, 1979, 94 SCRA 953; PAL v. CA, G.R. No. L-46558, July 31,1981,106 SCRA 391; United General
Industries, Inc, v. Paler G.R. No. L-30205, March 15,1982,112 SCRA 404; Rubio v. CA, G.R. No. 50911, August 21, 1987, 153
SCRA 183] the question of whether or not the principle of abuse of rights has been violated resulting in damages under Article
20 or Article 21 or other applicable provision of law, depends on the circumstances of each case. And in the instant case, the
Court, after examining the record and considering certain significant circumstances, finds that all petitioners have indeed
abused the right that they invoke, causing damage to private respondent and for which the latter must now be indemnified.
The trial court made a finding that notwithstanding the fact that it was private respondent Tobias who reported the possible
existence of anomalous transactions, petitioner Hendry "showed belligerence and told plaintiff (private respondent herein) that
he was the number one suspect and to take a one week vacation leave, not to communicate with the office, to leave his table
drawers open, and to leave his keys to said defendant (petitioner Hendry)" [RTC Decision, p. 2; Rollo, p. 232]. This, petitioners
do not dispute. But regardless of whether or not it was private respondent Tobias who reported the anomalies to petitioners, the
latter's reaction towards the former upon uncovering the anomalies was less than civil. An employer who harbors suspicions that
an employee has committed dishonesty might be justified in taking the appropriate action such as ordering an investigation and
directing the employee to go on a leave. Firmness and the resolve to uncover the truth would also be expected from such
employer. But the high-handed treatment accorded Tobias by petitioners was certainly uncalled for. And this reprehensible
attitude of petitioners was to continue when private respondent returned to work on November 20, 1972 after his one week
forced leave. Upon reporting for work, Tobias was confronted by Hendry who said. "Tobby, you are the crook and swindler in this
company." Considering that the first report made by the police investigators was submitted only on December 10, 1972 [See
Exh. A] the statement made by petitioner Hendry was baseless. The imputation of guilt without basis and the pattern of
harassment during the investigations of Tobias transgress the standards of human conduct set forth in Article 19 of the Civil
Code. The Court has already ruled that the right of the employer to dismiss an employee should not be confused with the
manner in which the right is exercised and the effects flowing therefrom. If the dismissal is done abusively, then the employer is
liable for damages to the employee [Quisaba v. Sta. Ines-Melale Veneer and Plywood Inc., G.R. No. L-38088, August 30, 1974,
58 SCRA 771; See also Philippine Refining Co., Inc. v. Garcia, G.R. No. L-21871, September 27,1966, 18 SCRA 107] Under the
circumstances of the instant case, the petitioners clearly failed to exercise in a legitimate manner their right to dismiss Tobias,
giving the latter the right to recover damages under Article 19 in relation to Article 21 of the Civil Code.
But petitioners were not content with just dismissing Tobias. Several other tortious acts were committed by petitioners against
Tobias after the latter's termination from work. Towards the latter part of January, 1973, after the filing of the first of six criminal
complaints against Tobias, the latter talked to Hendry to protest the actions taken against him. In response, Hendry cut short
Tobias' protestations by telling him to just confess or else the company would file a hundred more cases against him until he
landed in jail. Hendry added that, "You Filipinos cannot be trusted." The threat unmasked petitioner's bad faith in the various
actions taken against Tobias. On the other hand, the scornful remark about Filipinos as well as Hendry's earlier statements
about Tobias being a "crook" and "swindler" are clear violations of 'Tobias' personal dignity [See Article 26, Civil Code].
The next tortious act committed by petitioners was the writing of a letter to RETELCO sometime in October 1974, stating that
Tobias had been dismissed by GLOBE MACKAY due to dishonesty. Because of the letter, Tobias failed to gain employment with
RETELCO and as a result of which, Tobias remained unemployed for a longer period of time. For this further damage suffered
by Tobias, petitioners must likewise be held liable for damages consistent with Article 2176 of the Civil Code. Petitioners,
however, contend that they have a "moral, if not legal, duty to forewarn other employers of the kind of employee the plaintiff
(private respondent herein) was." [Petition, p. 14; Rollo, p. 15]. Petitioners further claim that "it is the accepted moral and
societal obligation of every man to advise or warn his fellowmen of any threat or danger to the latter's life, honor or property. And
this includes warning one's brethren of the possible dangers involved in dealing with, or accepting into confidence, a man whose
honesty and integrity is suspect" [Id.]. These arguments, rather than justify petitioners' act, reveal a seeming obsession to
prevent Tobias from getting a job, even after almost two years from the time Tobias was dismissed.
Finally, there is the matter of the filing by petitioners of six criminal complaints against Tobias. Petitioners contend that there is
no case against them for malicious prosecution and that they cannot be "penalized for exercising their right and prerogative of
seeking justice by filing criminal complaints against an employee who was their principal suspect in the commission of forgeries
and in the perpetration of anomalous transactions which defrauded them of substantial sums of money" [Petition, p. 10, Rollo, p.
11].
While sound principles of justice and public policy dictate that persons shall have free resort to the courts for redress of wrongs
and vindication of their rights [Buenaventura v. Sto. Domingo, 103 Phil. 239 (1958)], the right to institute criminal prosecutions
can not be exercised maliciously and in bad faith [Ventura v. Bernabe, G.R. No. L-26760, April 30, 1971, 38 SCRA 5871.]
Hence, in Yutuk V. Manila Electric Co., G.R. No. L-13016, May 31, 1961, 2 SCRA 337, the Court held that the right to file
criminal complaints should not be used as a weapon to force an alleged debtor to pay an indebtedness. To do so would be a
clear perversion of the function of the criminal processes and of the courts of justice. And in Hawpia CA, G.R. No. L-20047,
June 30, 1967. 20 SCRA 536 the Court upheld the judgment against the petitioner for actual and moral damages and attorney's
fees after making a finding that petitioner, with persistence, filed at least six criminal complaints against respondent, all of which
were dismissed.
To constitute malicious prosecution, there must be proof that the prosecution was prompted by a design to vex and humiliate a
person and that it was initiated deliberately by the defendant knowing that the charges were false and groundless [Manila Gas

Corporation v. CA, G.R. No. L-44190, October 30,1980, 100 SCRA 602]. Concededly, the filing of a suit by itself, does not
render a person liable for malicious prosecution [Inhelder Corporation v. CA, G.R. No. 52358, May 301983122 SCRA 576]. The
mere dismissal by the fiscal of the criminal complaint is not a ground for an award of damages for malicious prosecution if there
is no competent evidence to show that the complainant had acted in bad faith [Sison v. David, G.R. No. L-11268, January
28,1961, 1 SCRA 60].
In the instant case, however, the trial court made a finding that petitioners acted in bad faith in filing the criminal complaints
against Tobias, observing that:
xxx
Defendants (petitioners herein) filed with the Fiscal's Office of Manila a total of six (6) criminal cases, five (5) of
which were for estafa thru falsification of commercial document and one for violation of Art. 290 of the Revised Penal
Code "discovering secrets thru seizure of correspondence," and all were dismissed for insufficiency or lack of
evidence." The dismissal of four (4) of the cases was appealed to the Ministry of Justice, but said Ministry invariably
sustained the dismissal of the cases. As above adverted to, two of these cases were refiled with the Judge Advocate
General's Office of the Armed Forces of the Philippines to railroad plaintiffs arrest and detention in the military
stockade, but this was frustrated by a presidential decree transferring criminal cases involving civilians to the civil
courts.
xxx
To be sure, when despite the two (2) police reports embodying the findings of Lt. Dioscoro Tagle, Chief Document
Examiner of the Manila Police Department, clearing plaintiff of participation or involvement in the fraudulent
transactions complained of, despite the negative results of the lie detector tests which defendants compelled plaintiff
to undergo, and although the police investigation was "still under follow-up and a supplementary report will be
submitted after all the evidence has been gathered," defendants hastily filed six (6) criminal cases with the city
Fiscal's Office of Manila, five (5) for estafa thru falsification of commercial document and one (1) for violation of Art.
290 of the Revised Penal Code, so much so that as was to be expected, all six (6) cases were dismissed, with one
of the investigating fiscals, Asst. Fiscal de Guia, commenting in one case that, "Indeed, the haphazard way this case
was investigated is evident. Evident likewise is the flurry and haste in the filing of this case against respondent
Tobias," there can be no mistaking that defendants would not but be motivated by malicious and unlawful intent to
harass, oppress, and cause damage to plaintiff.
xxx
[RTC Decision, pp. 5-6; Rollo, pp. 235-236].
In addition to the observations made by the trial court, the Court finds it significant that the criminal complaints were filed during
the pendency of the illegal dismissal case filed by Tobias against petitioners. This explains the haste in which the complaints
were filed, which the trial court earlier noted. But petitioners, to prove their good faith, point to the fact that only six complaints
were filed against Tobias when they could have allegedly filed one hundred cases, considering the number of anomalous
transactions committed against GLOBE MACKAY. However, petitioners' good faith is belied by the threat made by Hendry after
the filing of the first complaint that one hundred more cases would be filed against Tobias. In effect, the possible filing of one
hundred more cases was made to hang like the sword of Damocles over the head of Tobias. In fine, considering the haste in
which the criminal complaints were filed, the fact that they were filed during the pendency of the illegal dismissal case against
petitioners, the threat made by Hendry, the fact that the cases were filed notwithstanding the two police reports exculpating
Tobias from involvement in the anomalies committed against GLOBE MACKAY, coupled by the eventual dismissal of all the
cases, the Court is led into no other conclusion than that petitioners were motivated by malicious intent in filing the six criminal
complaints against Tobias.
Petitioners next contend that the award of damages was excessive. In the complaint filed against petitioners, Tobias prayed for
the following: one hundred thousand pesos (P100,000.00) as actual damages; fifty thousand pesos (P50,000.00) as exemplary
damages; eight hundred thousand pesos (P800,000.00) as moral damages; fifty thousand pesos (P50,000.00) as attorney's
fees; and costs. The trial court, after making a computation of the damages incurred by Tobias [See RTC Decision, pp. 7-8;
Rollo, pp. 154-1551, awarded him the following: eighty thousand pesos (P80,000.00) as actual damages; two hundred thousand
pesos (P200,000.00) as moral damages; twenty thousand pesos (P20,000.00) as exemplary damages; thirty thousand pesos
(P30,000.00) as attorney's fees; and, costs. It must be underscored that petitioners have been guilty of committing several
actionable tortious acts, i.e., the abusive manner in which they dismissed Tobias from work including the baseless imputation of
guilt and the harassment during the investigations; the defamatory language heaped on Tobias as well as the scornful remark
on Filipinos; the poison letter sent to RETELCO which resulted in Tobias' loss of possible employment; and, the malicious filing
of the criminal complaints. Considering the extent of the damage wrought on Tobias, the Court finds that, contrary to petitioners'
contention, the amount of damages awarded to Tobias was reasonable under the circumstances.
Yet, petitioners still insist that the award of damages was improper, invoking the principle of damnum absqueinjuria. It is argued
that "[t]he only probable actual damage that plaintiff (private respondent herein) could have suffered was a direct result of his
having been dismissed from his employment, which was a valid and legal act of the defendants-appellants (petitioners
herein).lwph1.t " [Petition, p. 17; Rollo, p. 18].
According to the principle of damnum absque injuria, damage or loss which does not constitute a violation of a legal right or
amount to a legal wrong is not actionable [Escano v. CA, G.R. No. L-47207, September 25, 1980, 100 SCRA 197; See also
Gilchrist v. Cuddy 29 Phil, 542 (1915); The Board of Liquidators v. Kalaw, G.R. No. L-18805, August 14, 1967, 20 SCRA 987].
This principle finds no application in this case. It bears repeating that even granting that petitioners might have had the right to
dismiss Tobias from work, the abusive manner in which that right was exercised amounted to a legal wrong for which petitioners
must now be held liable. Moreover, the damage incurred by Tobias was not only in connection with the abusive manner in which
he was dismissed but was also the result of several other quasi-delictual acts committed by petitioners.
Petitioners next question the award of moral damages. However, the Court has already ruled in Wassmer v. Velez, G.R. No. L20089, December 26, 1964, 12 SCRA 648, 653, that [p]er express provision of Article 2219 (10) of the New Civil Code, moral
damages are recoverable in the cases mentioned in Article 21 of said Code." Hence, the Court of Appeals committed no error in
awarding moral damages to Tobias.
Lastly, the award of exemplary damages is impugned by petitioners. Although Article 2231 of the Civil Code provides that "[i]n
quasi-delicts, exemplary damages may be granted if the defendant acted with gross negligence," the Court, in Zulueta v. Pan
American World Airways, Inc., G.R. No. L- 28589, January 8, 1973, 49 SCRA 1, ruled that if gross negligence warrants the
award of exemplary damages, with more reason is its imposition justified when the act performed is deliberate, malicious and
tainted with bad faith. As in the Zuluetacase, the nature of the wrongful acts shown to have been committed by petitioners
against Tobias is sufficient basis for the award of exemplary damages to the latter.
WHEREFORE, the petition is hereby DENIED and the decision of the Court of Appeals in CA-G.R. CV No. 09055 is
AFFIRMED.
SO ORDERED.
G.R. No. L-46061 November 14, 1984

ST. LOUIS REALTY CORPORATION, petitioner,


vs.
COURT OF APPEALS and CONRADO J. ARAMIL, respondents.
Romeo Z. Comia for petitioner.
Roman R. Bersamin for private respondent.

AQUINO, J.:
This case is about the recovery of damages for a wrongful advertisement in the Sunday Times where Saint Louis Realty
Corporation misrepresented that the house of Doctor Conrado J. Aramil belonged to Arcadio S. Arcadio.
St. Louis Realty caused to be published with the permission of Arcadio S. Arcadio (but without permission of Doctor Aramil) in
the issue of the Sunday Times of December 15, 1968 an advertisement with the heading "WHERE THE HEART IS". Below that
heading was the photograph of the residence of Doctor Aramil and theArcadio family and then below the photograph was the
following write-up:
Home is where the heart is. And the hearts of MR. AND MRS. ARCADIO S. ARCADIO and their family have been
captured by BROOKSIDE HILLS. They used to rent a small 2-bedroom house in a cramped neighborhood, sadly
inadequate and unwholesome for the needs of a large family. They dream(ed) of a more pleasant place free from
the din and dust of city life yet near all facilities. Plans took shape when they heard of BROOKSIDE HILLS. With
thrift and determination, they bought a lot and built their dream house ... for P31,000. The Arcadios are now part of
the friendly, thriving community of BROOKSIDE HILLS... a beautiful first-class subdivision planned for wholesome
family living.
The same advertisement appeared in the Sunday Times dated January 5, 1969. Doctor Aramil a neuropsychiatrist and a
member of the faculty of the U. E. Ramon Magsaysay Memorial Hospital, noticed the mistake. On that same date, he wrote St.
Louis Realty the following letter of protest:
Dear Sirs:
This is anent to your advertisements appearing in the December 15, 1968 and January 5, 1969 issues of
the Sunday Times which boldly depicted my house at the above-mentioned address and implying that it belonged to
another person. I am not aware of any permission or authority on my partfor the use of my house for such publicity.
This unauthorized use of my house for your promotional gain and much more the apparent distortions therein are I
believe not only transgression to my private property but also damaging to my prestige in the medical profession I
have had invited in several occasions numerous medical colleagues, medical students and friends to my house and
after reading your December 15 advertisement some of them have uttered some remarks purporting doubts as to
my professional and personal integrity. Such sly remarks although in light vein as "it looks like your house," "how
much are you renting from the Arcadios?", " like your wife portrayed in the papers as belonging to another
husband," etc., have resulted in no little mental anguish on my part.
I have referred this matter to the Legal Panel of the Philippine Medical Association and their final advice is pending
upon my submission of supporting ownership papers.
I will therefore be constrained to pursue court action against your corporation unless you could satisfactorily explain
this matter within a week upon receipt of this letter.
The letter was received by Ernesto Magtoto, an officer of St. Louis Realty in charge of advertising. He stopped publication of the
advertisement. He contacted Doctor Aramil and offered his apologies. However, no rectification or apology was published.
On February 20, 1969, Aramil's counsel demanded from St. Louis Realty actual, moral and exemplary damages of P110,000
(Exh. D). In its answer dated March 10, St. Louis Realty claimed that there was an honest mistake and that if Aramil so desired,
rectification would be published in the Manila Times (Exh. 3).
It published in the issue of the Manila Times of March 18, 1969 a new advertisement with the Arcadio family and their real
house. But it did not publish any apology to Doctor Aramil and an explanation of the error.
On March 29, Aramil filed his complaint for damages. St. Louis Realty published in the issue of the Manila Times of April 15,
1969 the following "NOTICE OF RECTIFICATION" in a space 4 by 3 inches:
This will serve as a notice that our print ad 'Where the Heart is' which appeared in the Manila Timesissue of March
18, 1969 is a rectification of the same ad that appeared in the Manila Times issues rectification of the same ad that
appeal of December 15, 1968 and January 5, 1969 wherein a photo of the house of another Brookside Homeowner
(Dr. Aramil-private respondent) was mistakenly used as a background for the featured homeowner's the Arcadio
family.
The ad of March 18, 1969 shows the Arcadio family with their real house in the background, as was intended all
along.
Judge Jose M. Leuterio observed that St. Louis Realty should have immediately published a rectification and apology. He found
that as a result of St. Louis Realty's mistake, magnified by its utter lack of sincerity, Doctor Aramil suffered mental anguish and
his income was reduced by about P1,000 to P1,500 a month. Moreover, there was violation of Aramil's right to privacy (Art. 26,
Civil Code).
The trial court awarded Aramil P8,000 as actual damages, P20,000 as moral damages and P2,000 as attorney's fees. St. Louis
Realty appealed to the Court of Appeals.
The Appellate Court affirmed that judgment, with Acting Presiding Justice Magno S. Gatmaitan as ponente, and Justices Sixto
A. Domondon and Samuel F. Reyes concurring.
The Appellate Court reasoned out that St. Louis Realty committed an actionable quasi-delict under articles 21 and 26 of the Civil
Code because the questioned advertisements pictured a beautiful house which did not belong to Arcadio but to Doctor Aramil
who, naturally, was annoyed by that contretemps.
In this appeal, St. Louis Realty contends that the Appellate Court ignored certain facts and resorted to surmises and
conjectures. This contention is unwarranted. The Appellate Court adopted the facts found by the trial court. Those factual
findings are binding on this Court.
St. Louis Realty also contends that the decision is contrary to law and that the case was decided in a way not in conformity with
the rulings of this Court. It argues that the case is not covered by article 26 which provides that "every person shall respect the
dignity, personality, privacy and peace of mind of his neighbors and other persons". "Prying into the privacy of another's

residence" and "meddling with or disturbing the private life or family relations of another" and "similar acts", "though they may
not constitute a criminal offense, shall produce a cause of action for damages, prevention and other relief".
The damages fixed by Judge Leuterio are sanctioned by Articles 2200, 2208 and 2219 of the Civil Code. Article 2219 allows
moral damages for acts and actions mentioned in Article 26. As lengthily explained by Justice Gatmaitan, the acts and
omissions of the firm fan under Article 26.
St. Louis Realty's employee was grossly negligent in mixing up the Aramil and Arcadio residences in a widely circulated
publication like the Sunday Times. To suit its purpose, it never made any written apology and explanation of the mix-up. It just
contented itself with a cavalier "rectification ".
Persons, who know the residence of Doctor Aramil, were confused by the distorted, lingering impression that he was renting his
residence from Arcadio or that Arcadio had leased it from him. Either way, his private life was mistakenly and unnecessarily
exposed. He suffered diminution of income and mental anguish.
WHEREFORE, the judg

Breach of Promise to Marry

G.R. No. 97336 February 19, 1993


GASHEM SHOOKAT BAKSH, petitioner,
vs.
HON. COURT OF APPEALS and MARILOU T. GONZALES, respondents.
Public Attorney's Office for petitioner.
Corleto R. Castro for private respondent.

DAVIDE, JR., J.:


This is an appeal by certiorari under Rule 45 of the Rules of Court seeking to review and set aside the Decision 1of the
respondent Court of Appeals in CA-G.R. CV No. 24256 which affirmed in toto the 16 October 1939 Decision of Branch 38
(Lingayen) of the Regional Trial Court (RTC) of Pangasinan in Civil Case No. 16503. Presented is the issue of whether or not
damages may be recovered for a breach of promise to marry on the basis of Article 21 of the Civil Code of the Philippines.
The antecedents of this case are not complicated:
On 27 October 1987, private respondent, without the assistance of counsel, filed with the aforesaid trial court a complaint 2 for
damages against the petitioner for the alleged violation of their agreement to get married. She alleges in said complaint that:
she is twenty-two (22) years old, single, Filipino and a pretty lass of good moral character and reputation duly respected in her
community; petitioner, on the other hand, is an Iranian citizen residing at the Lozano Apartments, Guilig, Dagupan City, and is
an exchange student taking a medical course at the Lyceum Northwestern Colleges in Dagupan City; before 20 August 1987,
the latter courted and proposed to marry her; she accepted his love on the condition that they would get married; they therefore
agreed to get married after the end of the school semester, which was in October of that year; petitioner then visited the private
respondent's parents in Baaga, Bugallon, Pangasinan to secure their approval to the marriage; sometime in 20 August 1987,
the petitioner forced her to live with him in the Lozano Apartments; she was a virgin before she began living with him; a week
before the filing of the complaint, petitioner's attitude towards her started to change; he maltreated and threatened to kill her; as
a result of such maltreatment, she sustained injuries; during a confrontation with a representative of the barangay captain of
Guilig a day before the filing of the complaint, petitioner repudiated their marriage agreement and asked her not to live with him
anymore and; the petitioner is already married to someone living in Bacolod City. Private respondent then prayed for judgment
ordering the petitioner to pay her damages in the amount of not less than P45,000.00, reimbursement for actual expenses
amounting to P600.00, attorney's fees and costs, and granting her such other relief and remedies as may be just and equitable.
The complaint was docketed as Civil Case No. 16503.
In his Answer with Counterclaim, 3 petitioner admitted only the personal circumstances of the parties as averred in the complaint
and denied the rest of the allegations either for lack of knowledge or information sufficient to form a belief as to the truth thereof
or because the true facts are those alleged as his Special and Affirmative Defenses. He thus claimed that he never proposed
marriage to or agreed to be married with the private respondent; he neither sought the consent and approval of her parents nor
forced her to live in his apartment; he did not maltreat her, but only told her to stop coming to his place because he discovered
that she had deceived him by stealing his money and passport; and finally, no confrontation took place with a representative of
the barangay captain. Insisting, in his Counterclaim, that the complaint is baseless and unfounded and that as a result thereof,
he was unnecessarily dragged into court and compelled to incur expenses, and has suffered mental anxiety and a besmirched
reputation, he prayed for an award of P5,000.00 for miscellaneous expenses and P25,000.00 as moral damages.
After conducting a pre-trial on 25 January 1988, the trial court issued a Pre-Trial Order 4 embodying the stipulated facts which
the parties had agreed upon, to wit:
1. That the plaintiff is single and resident (sic) of Baaga, Bugallon, Pangasinan, while the defendant is single,
Iranian citizen and resident (sic) of Lozano Apartment, Guilig, Dagupan City since September 1, 1987 up to the
present;
2. That the defendant is presently studying at Lyceum Northwestern, Dagupan City, College of Medicine, second
year medicine proper;
3. That the plaintiff is (sic) an employee at Mabuhay Luncheonette , Fernandez Avenue, Dagupan City since July,
1986 up to the present and a (sic) high school graduate;
4. That the parties happened to know each other when the manager of the Mabuhay Luncheonette, Johhny Rabino
introduced the defendant to the plaintiff on August 3, 1986.
After trial on the merits, the lower court, applying Article 21 of the Civil Code, rendered on 16 October 1989 a decision 5 favoring
the private respondent. The petitioner was thus ordered to pay the latter damages and attorney's fees; the dispositive portion of
the decision reads:
IN THE LIGHT of the foregoing consideration, judgment is hereby rendered in favor of the plaintiff and against the
defendant.
1. Condemning (sic) the defendant to pay the plaintiff the sum of twenty thousand (P20,000.00) pesos as moral
damages.
2. Condemning further the defendant to play the plaintiff the sum of three thousand (P3,000.00) pesos as atty's fees
and two thousand (P2,000.00) pesos at (sic) litigation expenses and to pay the costs.
3. All other claims are denied. 6
The decision is anchored on the trial court's findings and conclusions that (a) petitioner and private respondent were lovers, (b)
private respondent is not a woman of loose morals or questionable virtue who readily submits to sexual advances, (c) petitioner,
through machinations, deceit and false pretenses, promised to marry private respondent, d) because of his persuasive promise
to marry her, she allowed herself to be deflowered by him, (e) by reason of that deceitful promise, private respondent and her
parents in accordance with Filipino customs and traditions made some preparations for the wedding that was to be held at
the end of October 1987 by looking for pigs and chickens, inviting friends and relatives and contracting sponsors, (f) petitioner
did not fulfill his promise to marry her and (g) such acts of the petitioner, who is a foreigner and who has abused Philippine
hospitality, have offended our sense of morality, good customs, culture and traditions. The trial court gave full credit to the
private respondent's testimony because, inter alia, she would not have had the temerity and courage to come to court and
expose her honor and reputation to public scrutiny and ridicule if her claim was false. 7
The above findings and conclusions were culled from the detailed summary of the evidence for the private respondent in the
foregoing decision, digested by the respondent Court as follows:
According to plaintiff, who claimed that she was a virgin at the time and that she never had a boyfriend before,
defendant started courting her just a few days after they first met. He later proposed marriage to her several times
and she accepted his love as well as his proposal of marriage on August 20, 1987, on which same day he went with

her to her hometown of Baaga, Bugallon, Pangasinan, as he wanted to meet her parents and inform them of their
relationship and their intention to get married. The photographs Exhs. "A" to "E" (and their submarkings) of
defendant with members of plaintiff's family or with plaintiff, were taken that day. Also on that occasion, defendant
told plaintiffs parents and brothers and sisters that he intended to marry her during the semestral break in October,
1987, and because plaintiff's parents thought he was good and trusted him, they agreed to his proposal for him to
marry their daughter, and they likewise allowed him to stay in their house and sleep with plaintiff during the few days
that they were in Bugallon. When plaintiff and defendant later returned to Dagupan City, they continued to live
together in defendant's apartment. However, in the early days of October, 1987, defendant would tie plaintiff's hands
and feet while he went to school, and he even gave her medicine at 4 o'clock in the morning that made her sleep the
whole day and night until the following day. As a result of this live-in relationship, plaintiff became pregnant, but
defendant gave her some medicine to abort the fetus. Still plaintiff continued to live with defendant and kept
reminding him of his promise to marry her until he told her that he could not do so because he was already married
to a girl in Bacolod City. That was the time plaintiff left defendant, went home to her parents, and thereafter
consulted a lawyer who accompanied her to the barangay captain in Dagupan City. Plaintiff, her lawyer, her
godmother, and a barangay tanod sent by the barangay captain went to talk to defendant to still convince him to
marry plaintiff, but defendant insisted that he could not do so because he was already married to a girl in Bacolod
City, although the truth, as stipulated by the parties at the pre-trial, is that defendant is still single.
Plaintiff's father, a tricycle driver, also claimed that after defendant had informed them of his desire to marry Marilou,
he already looked for sponsors for the wedding, started preparing for the reception by looking for pigs and chickens,
and even already invited many relatives and friends to the forthcoming wedding. 8
Petitioner appealed the trial court's decision to the respondent Court of Appeals which docketed the case as CA-G.R. CV No.
24256. In his Brief, 9 he contended that the trial court erred (a) in not dismissing the case for lack of factual and legal basis and
(b) in ordering him to pay moral damages, attorney's fees, litigation expenses and costs.
On 18 February 1991, respondent Court promulgated the challenged decision 10 affirming in toto the trial court's ruling of 16
October 1989. In sustaining the trial court's findings of fact, respondent Court made the following analysis:
First of all, plaintiff, then only 21 years old when she met defendant who was already 29 years old at the time, does
not appear to be a girl of loose morals. It is uncontradicted that she was a virgin prior to her unfortunate experience
with defendant and never had boyfriend. She is, as described by the lower court, a barrio lass "not used and
accustomed to trend of modern urban life", and certainly would (sic) not have allowed
"herself to be deflowered by the defendant if there was no persuasive promise made by the defendant to marry her."
In fact, we agree with the lower court that plaintiff and defendant must have been sweethearts or so the plaintiff must
have thought because of the deception of defendant, for otherwise, she would not have allowed herself to be
photographed with defendant in public in so (sic) loving and tender poses as those depicted in the pictures Exhs. "D"
and "E". We cannot believe, therefore, defendant's pretense that plaintiff was a nobody to him except a waitress at
the restaurant where he usually ate. Defendant in fact admitted that he went to plaintiff's hometown of Baaga,
Bugallon, Pangasinan, at least thrice; at (sic) the town fiesta on February 27, 1987 (p. 54, tsn May 18, 1988), at (sic)
a beach party together with the manager and employees of the Mabuhay Luncheonette on March 3, 1987 (p. 50,
tsn id.), and on April 1, 1987 when he allegedly talked to plaintiff's mother who told him to marry her daughter (pp.
55-56, tsn id.). Would defendant have left Dagupan City where he was involved in the serious study of medicine to
go to plaintiff's hometown in Baaga, Bugallon, unless there was (sic) some kind of special relationship between
them? And this special relationship must indeed have led to defendant's insincere proposal of marriage to plaintiff,
communicated not only to her but also to her parents, and (sic) Marites Rabino, the owner of the restaurant where
plaintiff was working and where defendant first proposed marriage to her, also knew of this love affair and
defendant's proposal of marriage to plaintiff, which she declared was the reason why plaintiff resigned from her job
at the restaurant after she had accepted defendant's proposal (pp. 6-7, tsn March 7, 1988).
Upon the other hand, appellant does not appear to be a man of good moral character and must think so low and
have so little respect and regard for Filipino women that he openly admitted that when he studied in Bacolod City for
several years where he finished his B.S. Biology before he came to Dagupan City to study medicine, he had a
common-law wife in Bacolod City. In other words, he also lived with another woman in Bacolod City but did not
marry that woman, just like what he did to plaintiff. It is not surprising, then, that he felt so little compunction or
remorse in pretending to love and promising to marry plaintiff, a young, innocent, trustful country girl, in order to
satisfy his lust on her. 11
and then concluded:
In sum, we are strongly convinced and so hold that it was defendant-appellant's fraudulent and deceptive
protestations of love for and promise to marry plaintiff that made her surrender her virtue and womanhood to him
and to live with him on the honest and sincere belief that he would keep said promise, and it was likewise these (sic)
fraud and deception on appellant's part that made plaintiff's parents agree to their daughter's living-in with him
preparatory to their supposed marriage. And as these acts of appellant are palpably and undoubtedly against
morals, good customs, and public policy, and are even gravely and deeply derogatory and insulting to our women,
coming as they do from a foreigner who has been enjoying the hospitality of our people and taking advantage of the
opportunity to study in one of our institutions of learning, defendant-appellant should indeed be made, under Art. 21
of the Civil Code of the Philippines, to compensate for the moral damages and injury that he had caused plaintiff, as
the lower court ordered him to do in its decision in this case. 12
Unfazed by his second defeat, petitioner filed the instant petition on 26 March 1991; he raises therein the single issue of
whether or not Article 21 of the Civil Code applies to the case at bar. 13
It is petitioner's thesis that said Article 21 is not applicable because he had not committed any moral wrong or injury or violated
any good custom or public policy; he has not professed love or proposed marriage to the private respondent; and he has never
maltreated her. He criticizes the trial court for liberally invoking Filipino customs, traditions and culture, and ignoring the fact that
since he is a foreigner, he is not conversant with such Filipino customs, traditions and culture. As an Iranian Moslem, he is not
familiar with Catholic and Christian ways. He stresses that even if he had made a promise to marry, the subsequent failure to
fulfill the same is excusable or tolerable because of his Moslem upbringing; he then alludes to the Muslim Code which
purportedly allows a Muslim to take four (4) wives and concludes that on the basis thereof, the trial court erred in ruling that he
does not posses good moral character. Moreover, his controversial "common law life" is now his legal wife as their marriage had
been solemnized in civil ceremonies in the Iranian Embassy. As to his unlawful cohabitation with the private respondent,
petitioner claims that even if responsibility could be pinned on him for the live-in relationship, the private respondent should also
be faulted for consenting to an illicit arrangement. Finally, petitioner asseverates that even if it was to be assumed arguendo that
he had professed his love to the private respondent and had also promised to marry her, such acts would not be actionable in
view of the special circumstances of the case. The mere breach of promise is not actionable. 14
On 26 August 1991, after the private respondent had filed her Comment to the petition and the petitioner had filed his Reply
thereto, this Court gave due course to the petition and required the parties to submit their respective Memoranda, which they
subsequently complied with.
As may be gleaned from the foregoing summation of the petitioner's arguments in support of his thesis, it is clear that questions
of fact, which boil down to the issue of the credibility of witnesses, are also raised. It is the rule in this jurisdiction that appellate
courts will not disturb the trial court's findings as to the credibility of witnesses, the latter court having heard the witnesses and

having had the opportunity to observe closely their deportment and manner of testifying, unless the trial court had plainly
overlooked facts of substance or value which, if considered, might affect the result of the case. 15
Petitioner has miserably failed to convince Us that both the appellate and trial courts had overlooked any fact of substance or
values which could alter the result of the case.
Equally settled is the rule that only questions of law may be raised in a petition for review on certiorari under Rule 45 of the
Rules of Court. It is not the function of this Court to analyze or weigh all over again the evidence introduced by the parties
before the lower court. There are, however, recognized exceptions to this rule. Thus, inMedina vs. Asistio, Jr., 16 this Court took
the time, again, to enumerate these exceptions:
xxx xxx xxx
(1) When the conclusion is a finding grounded entirely on speculation, surmises or conjectures (Joaquin v. Navarro,
93 Phil. 257 [1953]); (2) When the inference made is manifestly mistaken, absurb or impossible (Luna v. Linatok, 74
Phil. 15 [1942]); (3) Where there is a grave abuse of discretion (Buyco v. People, 95 Phil. 453 [1955]); (4) When the
judgment is based on a misapprehension of facts (Cruz v. Sosing,
L-4875, Nov. 27, 1953); (5) When the findings of fact are conflicting (Casica v. Villaseca, L-9590 Ap. 30, 1957;
unrep.) (6) When the Court of Appeals, in making its findings, went beyond the issues of the case and the same is
contrary to the admissions of both appellate and appellee (Evangelista v. Alto Surety and Insurance Co., 103 Phil.
401 [1958]);
(7) The findings of the Court of Appeals are contrary to those of the trial court (Garcia v. Court of Appeals, 33 SCRA
622 [1970]; Sacay v. Sandiganbayan, 142 SCRA 593 [1986]); (8) When the findings of fact are conclusions without
citation of specific evidence on which they are based (Ibid.,); (9) When the facts set forth in the petition as well as in
the petitioners main and reply briefs are not disputed by the respondents (Ibid.,); and (10) The finding of fact of the
Court of Appeals is premised on the supposed absence of evidence and is contradicted by the evidence on record
(Salazar v. Gutierrez, 33 SCRA 242 [1970]).
Petitioner has not endeavored to joint out to Us the existence of any of the above quoted exceptions in this case. Consequently,
the factual findings of the trial and appellate courts must be respected.
And now to the legal issue.
The existing rule is that a breach of promise to marry per se is not an actionable wrong. 17 Congress deliberately eliminated from
the draft of the New Civil Code the provisions that would have made it so. The reason therefor is set forth in the report of the
Senate Committees on the Proposed Civil Code, from which We quote:
The elimination of this chapter is proposed. That breach of promise to marry is not actionable has been definitely
decided in the case of De Jesus vs. Syquia. 18 The history of breach of promise suits in the United States and in
England has shown that no other action lends itself more readily to abuse by designing women and unscrupulous
men. It is this experience which has led to the abolition of rights of action in the so-called Heart Balm suits in many
of the American states. . . . 19
This notwithstanding, the said Code contains a provision, Article 21, which is designed to expand the concept of torts or quasidelict in this jurisdiction by granting adequate legal remedy for the untold number of moral wrongs which is impossible for
human foresight to specifically enumerate and punish in the statute books. 20
As the Code Commission itself stated in its Report:
But the Code Commission had gone farther than the sphere of wrongs defined or determined by positive law. Fully
sensible that there are countless gaps in the statutes, which leave so many victims of moral wrongs helpless, even
though they have actually suffered material and moral injury, the Commission has deemed it necessary, in the
interest of justice, to incorporate in the proposed Civil Code the following rule:
Art. 23. Any person who wilfully causes loss or injury to another in a manner that is contrary to morals,
good customs or public policy shall compensate the latter for the damage.
An example will illustrate the purview of the foregoing norm: "A" seduces the nineteen-year old daughter of "X". A
promise of marriage either has not been made, or can not be proved. The girl becomes pregnant. Under the present
laws, there is no crime, as the girl is above nineteen years of age. Neither can any civil action for breach of promise
of marriage be filed. Therefore, though the grievous moral wrong has been committed, and though the girl and
family have suffered incalculable moral damage, she and her parents cannot bring action for damages. But under
the proposed article, she and her parents would have such a right of action.
Thus at one stroke, the legislator, if the forgoing rule is approved, would vouchsafe adequate legal remedy for that
untold number of moral wrongs which it is impossible for human foresight to provide for specifically in the statutes.

21

Article 2176 of the Civil Code, which defines a quasi-delict thus:


Whoever by act or omission causes damage to another, there being fault or negligence, is obliged to pay for the
damage done. Such fault or negligence, if there is no pre-existing contractual relation between the parties, is called
a quasi-delict and is governed by the provisions of this Chapter.
is limited to negligent acts or omissions and excludes the notion of willfulness or intent. Quasi-delict, known in Spanish
legal treatises as culpa aquiliana, is a civil law concept while torts is an Anglo-American or common law concept. Torts is
much broader than culpa aquiliana because it includes not only negligence, but international criminal acts as well such as
assault and battery, false imprisonment and deceit. In the general scheme of the Philippine legal system envisioned by the
Commission responsible for drafting the New Civil Code, intentional and malicious acts, with certain exceptions, are to be
governed by the Revised Penal Code while negligent acts or omissions are to be covered by Article 2176 of the Civil
Code. 22 In between these opposite spectrums are injurious acts which, in the absence of Article 21, would have been
beyond redress. Thus, Article 21 fills that vacuum. It is even postulated that together with Articles 19 and 20 of the Civil
Code, Article 21 has greatly broadened the scope of the law on civil wrongs; it has become much more supple and
adaptable than the Anglo-American law on torts. 23
In the light of the above laudable purpose of Article 21, We are of the opinion, and so hold, that where a man's promise to marry
is in fact the proximate cause of the acceptance of his love by a woman and his representation to fulfill that promise thereafter
becomes the proximate cause of the giving of herself unto him in a sexual congress, proof that he had, in reality, no intention of
marrying her and that the promise was only a subtle scheme or deceptive device to entice or inveigle her to accept him and to
obtain her consent to the sexual act, could justify the award of damages pursuant to Article 21 not because of such promise to
marry but because of the fraud and deceit behind it and the willful injury to her honor and reputation which followed thereafter. It
is essential, however, that such injury should have been committed in a manner contrary to morals, good customs or public
policy.
In the instant case, respondent Court found that it was the petitioner's "fraudulent and deceptive protestations of love for and
promise to marry plaintiff that made her surrender her virtue and womanhood to him and to live with him on the honest and
sincere belief that he would keep said promise, and it was likewise these fraud and deception on appellant's part that made

plaintiff's parents agree to their daughter's living-in with him preparatory to their supposed marriage." 24 In short, the private
respondent surrendered her virginity, the cherished possession of every single Filipina, not because of lust but because of moral
seduction the kind illustrated by the Code Commission in its example earlier adverted to. The petitioner could not be held
liable for criminal seduction punished under either Article 337 or Article 338 of the Revised Penal Code because the private
respondent was above eighteen (18) years of age at the time of the seduction.
Prior decisions of this Court clearly suggest that Article 21 may be applied in a breach of promise to marry where the woman is
a victim of moral seduction. Thus, in Hermosisima vs. Court of Appeals, 25 this Court denied recovery of damages to the woman
because:
. . . we find ourselves unable to say that petitioner is morally guilty of seduction, not only because he is
approximately ten (10) years younger than the complainant who was around thirty-six (36) years of age, and as
highly enlightened as a former high school teacher and a life insurance agent are supposed to be when she
became intimate with petitioner, then a mere apprentice pilot, but, also, because the court of first instance found
that, complainant "surrendered herself" to petitioner because, "overwhelmed by her love" for him, she "wanted to
bind" him by having a fruit of their engagement even before they had the benefit of clergy.
In Tanjanco vs. Court of Appeals, 26 while this Court likewise hinted at possible recovery if there had been moral seduction,
recovery was eventually denied because We were not convinced that such seduction existed. The following enlightening
disquisition and conclusion were made in the said case:
The Court of Appeals seem to have overlooked that the example set forth in the Code Commission's memorandum
refers to a tort upon a minor who had been seduced. The essential feature is seduction, that in law is more than
mere sexual intercourse, or a breach of a promise of marriage; it connotes essentially the idea of deceit, enticement,
superior power or abuse of confidence on the part of the seducer to which the woman has yielded (U.S. vs.
Buenaventura, 27 Phil. 121; U.S. vs. Arlante, 9 Phil. 595).
It has been ruled in the Buenaventura case (supra) that
To constitute seduction there must in all cases be some sufficient promise or inducementand the woman
must yield because of the promise or other inducement. If she consents merely from carnal lust and the
intercourse is from mutual desire, there is no seduction (43 Cent. Dig. tit. Seduction, par. 56) She must
be induced to depart from the path of virtue by the use of some species of arts, persuasions and wiles,
which are calculated to have and do have that effect, and which result in her person to ultimately
submitting her person to the sexual embraces of her seducer (27 Phil. 123).
And in American Jurisprudence we find:
On the other hand, in an action by the woman, the enticement, persuasion or deception is the essence
of the injury; and a mere proof of intercourse is insufficient to warrant a recovery.
Accordingly it is not seduction where the willingness arises out of sexual desire of curiosity of the female,
and the defendant merely affords her the needed opportunity for the commission of the act. It has been
emphasized that to allow a recovery in all such cases would tend to the demoralization of the female
sex, and would be a reward for unchastity by which a class of adventuresses would be swift to profit. (47
Am. Jur. 662)
xxx xxx xxx
Over and above the partisan allegations, the fact stand out that for one whole year, from 1958 to 1959, the plaintiffappellee, a woman of adult age, maintain intimate sexual relations with appellant, with repeated acts of intercourse.
Such conduct is incompatible with the idea of seduction. Plainly there is here voluntariness and mutual passion; for
had the appellant been deceived, had she surrendered exclusively because of the deceit, artful persuasions and
wiles of the defendant, she would not have again yielded to his embraces, much less for one year, without exacting
early fulfillment of the alleged promises of marriage, and would have cut short all sexual relations upon finding that
defendant did not intend to fulfill his defendant did not intend to fulfill his promise. Hence, we conclude that no case
is made under article 21 of the Civil Code, and no other cause of action being alleged, no error was committed by
the Court of First Instance in dismissing the complaint. 27
In his annotations on the Civil Code, 28 Associate Justice Edgardo L. Paras, who recently retired from this Court, opined that in a
breach of promise to marry where there had been carnal knowledge, moral damages may be recovered:
. . . if there be criminal or moral seduction, but not if the intercourse was due to mutual lust. (Hermosisima vs. Court
of Appeals,
L-14628, Sept. 30, 1960; Estopa vs. Piansay, Jr., L-14733, Sept. 30, 1960; Batarra vs. Marcos, 7 Phil. 56 (sic);
Beatriz Galang vs. Court of Appeals, et al., L-17248, Jan. 29, 1962). (In other words, if the CAUSE be the promise to
marry, and the EFFECT be the carnal knowledge, there is a chance that there was criminal or moral seduction,
hence recovery of moral damages will prosper. If it be the other way around, there can be no recovery of moral
damages, because here mutual lust has intervened). . . .
together with "ACTUAL damages, should there be any, such as the expenses for the wedding presentations (See
Domalagon v. Bolifer, 33 Phil. 471).
Senator Arturo M. Tolentino

29

is also of the same persuasion:

It is submitted that the rule in Batarra vs. Marcos, 30 still subsists, notwithstanding the incorporation of the present
article 31 in the Code. The example given by the Code Commission is correct, if there was seduction, not necessarily
in the legal sense, but in the vulgar sense of deception. But when the sexual act is accomplished without any deceit
or qualifying circumstance of abuse of authority or influence, but the woman, already of age, has knowingly given
herself to a man, it cannot be said that there is an injury which can be the basis for indemnity.
But so long as there is fraud, which is characterized by willfulness (sic), the action lies. The court, however, must
weigh the degree of fraud, if it is sufficient to deceive the woman under the circumstances, because an act which
would deceive a girl sixteen years of age may not constitute deceit as to an experienced woman thirty years of age.
But so long as there is a wrongful act and a resulting injury, there should be civil liability, even if the act is not
punishable under the criminal law and there should have been an acquittal or dismissal of the criminal case for that
reason.
We are unable to agree with the petitioner's alternative proposition to the effect that granting, for argument's sake, that he did
promise to marry the private respondent, the latter is nevertheless also at fault. According to him, both parties are in pari delicto;
hence, pursuant to Article 1412(1) of the Civil Code and the doctrine laid down inBatarra vs. Marcos, 32 the private respondent
cannot recover damages from the petitioner. The latter even goes as far as stating that if the private respondent had "sustained
any injury or damage in their relationship, it is primarily because of her own doing, 33 for:
. . . She is also interested in the petitioner as the latter will become a doctor sooner or later. Take notice that she is a
plain high school graduate and a mere employee . . . (Annex "C") or a waitress (TSN, p. 51, January 25, 1988) in a

luncheonette and without doubt, is in need of a man who can give her economic security. Her family is in dire need
of financial assistance. (TSN, pp. 51-53, May 18, 1988). And this predicament prompted her to accept a proposition
that may have been offered by the petitioner. 34
These statements reveal the true character and motive of the petitioner. It is clear that he harbors a condescending, if not
sarcastic, regard for the private respondent on account of the latter's ignoble birth, inferior educational background, poverty and,
as perceived by him, dishonorable employment. Obviously then, from the very beginning, he was not at all moved by good faith
and an honest motive. Marrying with a woman so circumstances could not have even remotely occurred to him. Thus, his
profession of love and promise to marry were empty words directly intended to fool, dupe, entice, beguile and deceive the poor
woman into believing that indeed, he loved her and would want her to be his life's partner. His was nothing but pure lust which
he wanted satisfied by a Filipina who honestly believed that by accepting his proffer of love and proposal of marriage, she would
be able to enjoy a life of ease and security. Petitioner clearly violated the Filipino's concept of morality and brazenly defied the
traditional respect Filipinos have for their women. It can even be said that the petitioner committed such deplorable acts in
blatant disregard of Article 19 of the Civil Code which directs every person to act with justice, give everyone his due and
observe honesty and good faith in the exercise of his rights and in the performance of his obligations.
No foreigner must be allowed to make a mockery of our laws, customs and traditions.
The pari delicto rule does not apply in this case for while indeed, the private respondent may not have been impelled by the
purest of intentions, she eventually submitted to the petitioner in sexual congress not out of lust, but because of moral
seduction. In fact, it is apparent that she had qualms of conscience about the entire episode for as soon as she found out that
the petitioner was not going to marry her after all, she left him. She is not, therefore, in pari delicto with the petitioner. Pari
delicto means "in equal fault; in a similar offense or crime; equal in guilt or in legal fault." 35 At most, it could be conceded that
she is merely in delicto.
Equity often interferes for the relief of the less guilty of the parties, where his transgression has been brought about
by the imposition of undue influence of the party on whom the burden of the original wrong principally rests, or
where his consent to the transaction was itself procured by
fraud. 36
In Mangayao vs. Lasud, 37 We declared:
Appellants likewise stress that both parties being at fault, there should be no action by one against the other (Art.
1412, New Civil Code). This rule, however, has been interpreted as applicable only where the fault on both sides is,
more or less, equivalent. It does not apply where one party is literate or intelligent and the other one is not. (c.f.
Bough vs. Cantiveros, 40 Phil. 209).
We should stress, however, that while We find for the private respondent, let it not be said that this Court condones the
deplorable behavior of her parents in letting her and the petitioner stay together in the same room in their house after giving
approval to their marriage. It is the solemn duty of parents to protect the honor of their daughters and infuse upon them the
higher values of morality and dignity.
WHEREFORE, finding no reversible error in the challenged decision, the instant petition is hereby DENIED, with costs against
the petitioner.
SO ORDERED.

G.R. No. L-20089

December 26, 1964

BEATRIZ P. WASSMER, plaintiff-appellee,


vs.
FRANCISCO X. VELEZ, defendant-appellant.
Jalandoni & Jamir for defendant-appellant.
Samson S. Alcantara for plaintiff-appellee.
BENGZON, J.P., J.:
The facts that culminated in this case started with dreams and hopes, followed by appropriate planning and serious endeavors,
but terminated in frustration and, what is worse, complete public humiliation.
Francisco X. Velez and Beatriz P. Wassmer, following their mutual promise of love, decided to get married and set September 4,
1954 as the big day. On September 2, 1954 Velez left this note for his bride-to-be:
Dear Bet
Will have to postpone wedding My mother opposes it. Am leaving on the Convair today.
Please do not ask too many people about the reason why That would only create a scandal.
Paquing
But the next day, September 3, he sent her the following telegram:
NOTHING CHANGED REST ASSURED RETURNING VERY SOON APOLOGIZE MAMA PAPA LOVE .
PAKING
Thereafter Velez did not appear nor was he heard from again.
Sued by Beatriz for damages, Velez filed no answer and was declared in default. Plaintiff adduced evidence before the clerk of
court as commissioner, and on April 29, 1955, judgment was rendered ordering defendant to pay plaintiff P2,000.00 as actual
damages; P25,000.00 as moral and exemplary damages; P2,500.00 as attorney's fees; and the costs.
On June 21, 1955 defendant filed a "petition for relief from orders, judgment and proceedings and motion for new trial and
reconsideration." Plaintiff moved to strike it cut. But the court, on August 2, 1955, ordered the parties and their attorneys to
appear before it on August 23, 1955 "to explore at this stage of the proceedings the possibility of arriving at an amicable
settlement." It added that should any of them fail to appear "the petition for relief and the opposition thereto will be deemed
submitted for resolution."
On August 23, 1955 defendant failed to appear before court. Instead, on the following day his counsel filed a motion to defer for
two weeks the resolution on defendants petition for relief. The counsel stated that he would confer with defendant in Cagayan
de Oro City the latter's residence on the possibility of an amicable element. The court granted two weeks counted from
August 25, 1955.
Plaintiff manifested on June 15, 1956 that the two weeks given by the court had expired on September 8, 1955 but that
defendant and his counsel had failed to appear.
Another chance for amicable settlement was given by the court in its order of July 6, 1956 calling the parties and their attorneys
to appear on July 13, 1956. This time. however, defendant's counsel informed the court that chances of settling the case
amicably were nil.
On July 20, 1956 the court issued an order denying defendant's aforesaid petition. Defendant has appealed to this Court. In his
petition of June 21, 1955 in the court a quo defendant alleged excusable negligence as ground to set aside the judgment by
default. Specifically, it was stated that defendant filed no answer in the belief that an amicable settlement was being negotiated.
A petition for relief from judgment on grounds of fraud, accident, mistake or excusable negligence, must be duly supported by
an affidavit of merits stating facts constituting a valid defense. (Sec. 3, Rule 38, Rules of Court.) Defendant's affidavit of merits
attached to his petition of June 21, 1955 stated: "That he has a good and valid defense against plaintiff's cause of action, his
failure to marry the plaintiff as scheduled having been due to fortuitous event and/or circumstances beyond his control." An
affidavit of merits like this stating mere conclusions or opinions instead of facts is not valid. (Cortes vs. Co Bun Kim, L-3926,
Oct. 10, 1951; Vaswani vs. P. Tarrachand Bros., L-15800, December 29, 1960.)
Defendant, however, would contend that the affidavit of merits was in fact unnecessary, or a mere surplusage, because the
judgment sought to be set aside was null and void, it having been based on evidence adduced before the clerk of court. In
Province of Pangasinan vs. Palisoc, L-16519, October 30, 1962, this Court pointed out that the procedure of designating the
clerk of court as commissioner to receive evidence is sanctioned by Rule 34 (now Rule 33) of the Rules of Court. Now as to
defendant's consent to said procedure, the same did not have to be obtained for he was declared in default and thus had no
standing in court (Velez vs. Ramas, 40 Phil. 787; Alano vs. Court of First Instance, L-14557, October 30, 1959).
In support of his "motion for new trial and reconsideration," defendant asserts that the judgment is contrary to law. The reason
given is that "there is no provision of the Civil Code authorizing" an action for breach of promise to marry. Indeed, our ruling
in Hermosisima vs. Court of Appeals (L-14628, Sept. 30, 1960), as reiterated in Estopa vs. Biansay (L-14733, Sept. 30, 1960),
is that "mere breach of a promise to marry" is not an actionable wrong. We pointed out that Congress deliberately eliminated
from the draft of the new Civil Code the provisions that would have it so.
It must not be overlooked, however, that the extent to which acts not contrary to law may be perpetrated with impunity, is not
limitless for Article 21 of said Code provides that "any person who wilfully causes loss or injury to another in a manner that is
contrary to morals, good customs or public policy shall compensate the latter for the damage."
The record reveals that on August 23, 1954 plaintiff and defendant applied for a license to contract marriage, which was
subsequently issued (Exhs. A, A-1). Their wedding was set for September 4, 1954. Invitations were printed and distributed to
relatives, friends and acquaintances (Tsn., 5; Exh. C). The bride-to-be's trousseau, party drsrses and other apparel for the
important occasion were purchased (Tsn., 7-8). Dresses for the maid of honor and the flower girl were prepared. A matrimonial
bed, with accessories, was bought. Bridal showers were given and gifts received (Tsn., 6; Exh. E). And then, with but two days
before the wedding, defendant, who was then 28 years old,: simply left a note for plaintiff stating: "Will have to postpone
wedding My mother opposes it ... " He enplaned to his home city in Mindanao, and the next day, the day before the wedding,
he wired plaintiff: "Nothing changed rest assured returning soon." But he never returned and was never heard from again.
Surely this is not a case of mere breach of promise to marry. As stated, mere breach of promise to marry is not an actionable
wrong. But to formally set a wedding and go through all the above-described preparation and publicity, only to walk out of it
when the matrimony is about to be solemnized, is quite different. This is palpably and unjustifiably contrary to good customs for
which defendant must be held answerable in damages in accordance with Article 21 aforesaid.

Defendant urges in his afore-stated petition that the damages awarded were excessive. No question is raised as to the award of
actual damages. What defendant would really assert hereunder is that the award of moral and exemplary damages, in the
amount of P25,000.00, should be totally eliminated.
Per express provision of Article 2219 (10) of the New Civil Code, moral damages are recoverable in the cases mentioned in
Article 21 of said Code. As to exemplary damages, defendant contends that the same could not be adjudged against him
because under Article 2232 of the New Civil Code the condition precedent is that "the defendant acted in a wanton, fraudulent,
reckless, oppressive, or malevolent manner." The argument is devoid of merit as under the above-narrated circumstances of
this case defendant clearly acted in a "wanton ... , reckless [and] oppressive manner." This Court's opinion, however, is that
considering the particular circumstances of this case, P15,000.00 as moral and exemplary damages is deemed to be a
reasonable award.
PREMISES CONSIDERED, with the above-indicated modification, the lower court's judgment is hereby affirmed, with costs.

G.R. No. L-14628

September 30, 1960

FRANCISCO HERMOSISIMA, petitioner,


vs.
THE HON. COURT OF APPEALS, ET AL., respondents.
Regino Hermosisima for petitioner.
F.P. Gabriel, Jr. for respondents.
CONCEPCION, J.:
An appeal by certiorari, taken by petitioner Francisco Hermosisima, from a decision of Court of Appeals modifying that of the
Court of First Instance of Cebu.
On October 4, 1954, Soledad Cagigas, hereinafter referred to as complaint, filed with said of her child, Chris Hermosisima, as
natural child and moral damages for alleged breach of promise. Petitioner admitted the paternity of child and expressed
willingness to support the latter, but denied having ever promised to marry the complainant. Upon her motion, said court ordered
petitioner, on October 27, 1954, to pay, by way of alimony pendente lite, P50.00 a month, which was, on February 16, 1955,
reduced to P30.00 a month. In due course, later on, said court rendered a decision the dispositive part of which reads:
WHEREFORE, judgment is hereby rendered, declaring the child, Chris Hermosisima, as the natural daughter of
defendant, and confirming the order pendente lite, ordering defendant to pay to the said child, through plaintiff, the sum of
thirty pesos (P30.00), payable on or before the fifth day of every month sentencing defendant to pay to plaintiff the sum of
FOUR THOUSAND FIVE HUNDRED PESOS (P4,500.00) for actual and compensatory damages; the sum of FIVE
THOUSAND PESOS (P5,000.00) as moral damages; and the further sum of FIVE HUNDRED PESOS (P500.00) as
attorney's fees for plaintiff, with costs against defendant.
On appeal taken by petitioner, the Court of Appeals affirmed this decision, except as to the actual and compensatory damages
and the moral damages, which were increased to P5,614.25 and P7,000.00, respectively.
The main issue before us is whether moral damages are recoverable, under our laws, for breach of promise to marry. The
pertinent facts are:
Complainant Soledad Cagigas, was born in July 1917. Since 1950, Soledad then a teacher in the Sibonga Provincial High
School in Cebu, and petitioner, who was almost ten (10) years younger than she, used to go around together and were
regarded as engaged, although he had made no promise of marriage prior thereto. In 1951, she gave up teaching and became
a life insurance underwriter in the City of Cebu, where intimacy developed among her and the petitioner, since one evening in
1953, when after coming from the movies, they had sexual intercourse in his cabin on board M/V "Escao," to which he was
then attached as apprentice pilot. In February 1954, Soledad advised petitioner that she was in the family way, whereupon he
promised to marry her. Their child, Chris Hermosisima, was born on June 17, 1954, in a private maternity and clinic. However,
subsequently, or on July 24, 1954, defendant married one Romanita Perez. Hence, the present action, which was commenced
on or about October 4, 1954.
Referring now to the issue above referred to, it will be noted that the Civil Code of Spain permitted the recovery of damages for
breach to marry. Article 43 and 44 of said Code provides:
ART. 43. A mutual promise of marriage shall not give rise to an obligation to contract marriage. No court shall entertain
any complaint by which the enforcement of such promise is sought.
ART. 44. If the promise has been in a public or private instrument by an adult, or by a minor with the concurrence of the
person whose consent is necessary for the celebration of the marriage, or if the banns have been published, the one who
without just cause refuses to marry shall be obliged to reimburse the other for the expenses which he or she may have
incurred by reason of the promised marriage.
The action for reimbursement of expenses to which the foregoing article refers must be brought within one year, computed
from the day of the refusal to celebrate the marriage.
Inasmuch as these articles were never in force in the Philippines, this Court ruled in De Jesus vs. Syquia (58 Phil., 866), that
"the action for breach of promises to marry has no standing in the civil law, apart from the right to recover money or property
advanced . . . upon the faith of such promise". The Code Commission charged with the drafting of the Proposed Civil Code of
the Philippines deem it best, however, to change the law thereon. We quote from the report of the Code Commission on said
Proposed Civil Code:
Articles 43 and 44 the Civil Code of 1889 refer to the promise of marriage. But these articles are not enforced in the
Philippines. The subject is regulated in the Proposed Civil Code not only as to the aspect treated of in said articles but
also in other particulars. It is advisable to furnish legislative solutions to some questions that might arise relative to
betrothal. Among the provisions proposed are: That authorizing the adjudication of moral damages, in case of breach of
promise of marriage, and that creating liability for causing a marriage engagement to be broken.1awphl.nt
Accordingly, the following provisions were inserted in said Proposed Civil Code, under Chapter I, Title III, Book I thereof:
Art. 56. A mutual promise to marry may be made expressly or impliedly.
Art. 57. An engagement to be married must be agreed directly by the future spouses.
Art. 58. A contract for a future marriage cannot, without the consent of the parent or guardian, be entered into by a male
between the ages of sixteen and twenty years or by a female between the ages of sixteen and eighteen years. Without
such consent of the parents or guardian, the engagement to marry cannot be the basis of a civil action for damages in
case of breach of the promise.
Art. 59. A promise to marry when made by a female under the age of fourteen years is not civilly actionable, even though
approved by the parent or guardian.
Art. 60. In cases referred to in the proceeding articles, the criminal and civil responsibility of a male for seduction shall not
be affected.
Art. 61. No action for specific performance of a mutual promise to marry may be brought.
Art. 62. An action for breach of promise to marry may be brought by the aggrieved party even though a minor without the
assistance of his parent or guardian. Should the minor refuse to bring suit, the parent or guardian may institute the action.
Art. 63. Damages for breach of promise to marry shall include not only material and pecuniary losses but also
compensation for mental and moral suffering.
Art. 64. Any person, other than a rival, the parents, guardians and grandparents, of the affianced parties, who cause a
marriage engagement to be broken shall be liable for damages, both material and moral, to the engaged person who is
rejected.

Art. 65. In case of breach of promise to marry, the party breaking the engagement shall be obliged to return what he or
she has received from the other as gift on account of the promise of the marriage.
These article were, however, eliminated in Congress. The reason therefor are set forth in the report of the corresponding Senate
Committee, from which we quote:
The elimination of this Chapter is proposed. That breach of promise to marry is not actionable has been definitely decide in the
case of De Jesus vs. Syquia, 58 Phil., 866. The history of breach of promise suit in the United States and in England has shown
that no other action lends itself more readily to abuse by designing women and unscrupulous men. It is this experience which
has led to the abolition of the rights of action in the so-called Balm suit in many of the American States.
See statutes of:
Florida 1945 pp. 1342 1344
Maryland 1945 pp. 1759 1762
Nevada 1943 p. 75
Maine 1941 pp. 140 141
New Hampshire 1941 p. 223
California 1939 p. 1245
Massachusetts 1938 p. 326
Indiana 1936 p. 1009
Michigan 1935 p. 201
New York 1935
Pennsylvania p. 450
The Commission perhaps though that it has followed the more progression trend in legislation when it provided for breach
of promise to marry suits. But it is clear that the creation of such causes of action at a time when so many States, in
consequence of years of experience are doing away with them, may well prove to be a step in the wrong direction.
(Congressional Record, Vol. IV, No. 79, Thursday, May 19, 1949, p. 2352.)
The views thus expressed were accepted by both houses of Congress. In the light of the clear and manifest intent of our law
making body not to sanction actions for breach of promise to marry, the award of moral damages made by the lower courts is,
accordingly, untenable. The Court of Appeals said award:
Moreover, it appearing that because of defendant-appellant's seduction power, plaintiff-appellee, overwhelmed by her love
for him finally yielded to his sexual desires in spite of her age and self-control, she being a woman after all, we hold that
said defendant-appellant is liable for seduction and, therefore, moral damages may be recovered from him under the
provision of Article 2219, paragraph 3, of the new Civil Code.
Apart from the fact that the general tenor of said Article 2219, particularly the paragraphs preceding and those following the one
cited by the Court of Appeals, and the language used in said paragraph strongly indicates that the "seduction" therein
contemplated is the crime punished as such in Article as such in Article 337 and 338 of the Revised Penal Code, which
admittedly does not exist in the present case, we find ourselves unable to say that petitioner is morally guilty of seduction, not
only because he is approximately ten (10) years younger than the complainant who around thirty-six (36) years of age, and
as highly enlightened as a former high school teacher and a life insurance agent are supposed to be when she became
intimate with petitioner, then a mere apprentice pilot, but, also, because, the court of first instance found that, complainant
"surrendered herself" to petitioner because, "overwhelmed by her love" for him, she "wanted to bind" "by having a fruit of their
engagement even before they had the benefit of clergy."
The court of first instance sentenced petitioner to pay the following: (1) a monthly pension of P30.00 for the support of the child:
(2) P4,500, representing the income that complainant had allegedly failed to earn during her pregnancy and shortly after the
birth of the child, as actual and compensation damages; (3) P5,000, as moral damages; and (4) P500.00, as attorney's fees.
The Court of Appeals added to the second item the sum of P1,114.25 consisting of P144.20, for hospitalization and medical
attendance, in connection with the parturiation, and the balance representing expenses incurred to support the child and
increased the moral damages to P7,000.00.
With the elimination of this award for damages, the decision of the Court of Appeals is hereby affirmed, therefore, in all other
respects, without special pronouncement as to cost in this instance. It is so ordered.

G.R. No. 57227 May 14, 1992


AMELITA CONSTANTINO and MICHAEL CONSTANTINO, the latter represented herein by the former, his mother and
natural guardian, petitioners,
vs.
IVAN MENDEZ and the HONORABLE COURT OF APPEALS, respondents.
Roberto M. Sarenas for petitioners.
Bienvinido D. Cariaga for private respondent.

BIDIN, J.:
This is a petition for review on certiorari questioning the decision 1 dated April 30, 1981 of the Court of Appeals in CA-G.R. No.
61552-R which dismissed petitioner's complaint and set aside the resolution 2 dated October 21, 1976 of the then Court of First
Instance of Davao, 16th Judicial District, amending the dispositive portion of its decision dated June 21, 1976 and ordering
private respondent Ivan Mendez: (1) to acknowledge the minor Michael Constantino as his illegitimate child; (2) to give a
monthly support of P300.00 to the minor child; (3) to pay complainant Amelita Constantino the sum of P8,200.00 as actual and
moral damages; and (4) to pay attorney's fees in the sum of P5,000 plus costs.
It appears on record that on June 5, 1975, petitioner Amelita Constantino filed an action for acknowledgment, support and
damages against private respondent Ivan Mendez. The case was filed with the then CFI of Davao, 10th Judicial District and
docketed as Civil Case No. 8881. In her complaint, Amelita Constantino alleges, among others, that sometime in the month of
August, 1974, she met Ivan Mendez at Tony's Restaurant located at Sta. Cruz, Manila, where she worked as a waitress; that the
day following their first meeting, Ivan invited Amelita to dine with him at Hotel Enrico where he was billeted; that while dining,
Ivan professed his love and courted Amelita; that Amelita asked for time to think about Ivan's proposal; that at about 11:00
o'clock in the evening, Amelita asked Ivan to bring her home to which the latter agreed, that on the pretext of getting something,
Ivan brought Amelita inside his hotel room and through a promise of marriage succeeded in having sexual intercourse with the
latter; that after the sexual contact, Ivan confessed to Amelita that he is a married man; that they repeated their sexual contact
in the months of September and November, 1974, whenever Ivan is in Manila, as a result of which Amelita got pregnant; that
her pleas for help and support fell on deaf ears; that Amelita had no sexual relations with any other man except Ivan who is the
father of the child yet to be born at the time of the filing of the complaint; that because of her pregnancy, Amelita was forced to
leave her work as a waitress; that Ivan is a prosperous businessman of Davao City with a monthly income of P5,000 to P8,000.
As relief, Amelita prayed for the recognition of the unborn child, the payment of actual, moral and exemplary damages,
attorney's fees plus costs.
In his answer dated August 5, 1975, Ivan admitted that he met Amelita at Tony's Cocktail Lounge but denied having sexual
knowledge or illicit relations with her. He prayed for the dismissal of the complaint for lack of cause of action. By way of
counterclaim, he further prayed for the payment of exemplary damages and litigation expense including attorney's fees for the
filing of the malicious complaint.
On September 1, 1975, Amelita Constantino filed a motion for leave to amend the complaint impleading as co-plaintiff her son
Michael Constantino who was born on August 3, 1975. In its order dated September 4, 1975, the trial court admitted the
amended complaint.
On September 11, 1975, Ivan Mendez filed his answer to the amended complaint reiterating his previous answer denying that
Michael Constantino is his illegitimate son.
After hearing, the trial court rendered a decision dated June 21, 1976, the dispositive portion of which reads, viz:
WHEREFORE, in view of the foregoing, judgment is hereby rendered in favor of plaintiff Amelita Constantino and
against defendant Ivan Mendez, ordering the latter to pay Amelita Constantino the sum of P8,000.00 by way of
actual and moral damages; and, the sum of P3,000.00, as and by way of attorney's fees. The defendant shall pay
the costs of this suit.
SO ORDERED.
From the above decision, both parties filed their separate motion for reconsideration. Ivan Mendez anchored his motion on the
ground that the award of damages was not supported by evidence. Amelita Constantino, on the other hand, sought the
recognition and support of her son Michael Constantino as the illegitimate son of Ivan Mendez.
In its resolution dated October 21, 1976, the trial court granted Amelita Constantino's motion for reconsideration, and amended
the dispositive portion of its decision dated June 21, 1976 to read as follows, viz:
WHEREFORE, in view of the foregoing, judgment is hereby rendered in favor of plaintiff Amelita Constantino and
plaintiff-minor Michael Constantino, and against defendant Ivan Mendez ordering the latter to pay Amelita
Constantino the sum of P8,000.00 by way of actual and moral damages and the sum of P200.00 as and by way of
payment of the hospital and medical bills incurred during the delivery of plaintiff-minor Michael Constantino; to
recognize as his own illegitimate child the plaintiff-minor Michael Constantino who shall be entitled to all the rights,
privileges and benefits appertaining to a child of such status; to give a permanent monthly support in favor of plaintiff
Michael Constantino the amount of P300.00; and the sum of P5,000.00 as and by way of attorney's fees. The
defendant shall pay the costs of this suit.
Let this Order form part of the decision dated June 21, 1976.
SO ORDERED.
On appeal to the Court of Appeals, the above amended decision was set aside and the complaint was dismissed. Hence, this
petition for review.
Basically, the issue to be resolved in the case at bar is whether or not the Court of Appeals committed a reversible error in
setting aside the decision of the trial court and in dismissing the complaint.
Petitioners contend that the Court of Appeals erred in reversing the factual findings of the trial and in not affirming the decision
of the trial court. They also pointed out that the appellate court committed a misapprehension of facts when it concluded that
Ivan did not have sexual access with Amelita during the first or second week of November, 1976 (should be 1974), the time of
the conception of the child.
It must be stressed at the outset that factual findings of the trial court have only a persuasive and not a conclusive effect on the
Court of Appeals. In the exercise of its appellate jurisdiction, it is the duty of the Court of Appeals to review the factual findings of
the trial court and rectify the errors it committed as may have been properly assigned and as could be established by a reexamination of the evidence on record. It is the factual findings of the Court of Appeals, not those of the trial court, that as a rule
are considered final and conclusive even on this Court (Hermo v. Hon. Court of Appeals, et al., 155 SCRA 24 [1987]). This being
a petition for certiorari under Rule 45 of the Rules of Court, this Court will review only errors of law committed by the Court of

Appeals. It is not the function of this Court to re-examine all over again the oral and documentary evidence submitted by the
parties unless the findings of facts of the Court of Appeals is not supported by the evidence on record or the judgment is based
on misapprehension of facts (Remalante v. Tibe, et al., 158 SCRA 138 [1988]; Hernandez v. Court of Appeals, et al., 149 SCRA
97 [1987]).
It is the conclusion of the Court of Appeals, based on the evidence on record, that Amelita Constantino has not proved by clear
and convincing evidence her claim that Ivan Mendez is the father of her son Michael Constantino. Such conclusion based on
the evaluation of the evidence on record is controlling on this Court as the same is supported by the evidence on record. Even
the trial court initially entertained such posture. It ordered the recognition of Michael as the illegitimate son of Ivan only when
acting on the motions for reconsideration, it reconsidered, on October 21, 1976, its earlier decision dated June 21, 1976.
Amelita's testimony on cross-examination that she had sexual contact with Ivan in Manila in the first or second week of
November, 1974 (TSN, December 8, 1975, p. 108) is inconsistent with her response that she could not remember the date of
their last sexual intercourse in November, 1974 (Ibid, p. 106). Sexual contact of Ivan and Amelita in the first or second week of
November, 1974 is the crucial point that was not even established on direct examination as she merely testified that she had
sexual intercourse with Ivan in the months of September, October and November, 1974.
Michael Constantino is a full-term baby born on August 3, 1975 (Exhibit 6) so that as correctly pointed out by private
respondent's counsel, citing medical science (Williams Obstetrics, Tenth Ed., p. 198) to the effect that "the mean duration of
actual pregnancy, counting from the day of conception must be close to 267 days", the conception of the child (Michael) must
have taken place about 267 days before August 3, 1975 or sometime in the second week of November, 1974. While Amelita
testified that she had sexual contact with Ivan in November, 1974, nevertheless said testimony is contradicted by her own
evidence (Exh. F), the letter dated February 11, 1975, addressed to Ivan Mendez requesting for a conference, prepared by her
own counsel Atty. Roberto Sarenas to whom she must have confided the attendant circumstances of her pregnancy while still
fresh in her memory, informing Ivan that Amelita is four (4) months pregnant so that applying the period of the duration of actual
pregnancy, the child was conceived on or about October 11, 1974.
Petitioner's assertion that Ivan is her first and only boyfriend (TSN, December 8, 1975, p. 65) is belied by Exhibit 2, her own
letter addressed to Mrs. Mendez where she revealed the reason for her attachment to Ivan who possessed certain traits not
possessed by her boyfriend. She also confided that she had a quarrel with her boyfriend because of gossips so she left her
work. An order for recognition and support may create an unwholesome atmosphere or may be an irritant in the family or lives of
the parties so that it must be issued only if paternity or filiation is established by clear and convincing evidence. The burden of
proof is on Amelita to establish her affirmative allegations that Ivan is the father of her son. Consequently, in the absence of
clear and convincing evidence establishing paternity or filiation, the complaint must be dismissed.
As regards Amelita's claim for damages which is based on Articles 19 3 & 21 4 of the Civil Code on the theory that through Ivan's
promise of marriage, she surrendered her virginity, we cannot but agree with the Court of Appeals that more sexual intercourse
is not by itself a basis for recovery. Damages could only be awarded if sexual intercourse is not a product of voluntariness and
mutual desire. At the time she met Ivan at Tony's Restaurant, Amelita was already 28 years old and she admitted that she was
attracted to Ivan (TSN, December 3, 1975, p. 83). Her attraction to Ivan is the reason why she surrendered her womanhood.
Had she been induced or deceived because of a promise of marriage, she could have immediately severed her relation with
Ivan when she was informed after their first sexual contact sometime in August, 1974, that he was a married man. Her
declaration that in the months of September, October and November, 1974, they repeated their sexual intercourse only
indicates that passion and not the alleged promise of marriage was the moving force that made her submit herself to Ivan.
WHEREFORE, the instant petition is Dismissed for lack of merit.
SO ORDERED.

G.R. No. L-19671

November 29, 1965

PASTOR B. TENCHAVEZ, plaintiff-appellant,


vs.
VICENTA F. ESCAO, ET AL., defendants-appellees.
I. V. Binamira & F. B. Barria for plaintiff-appellant.
Jalandoni & Jarnir for defendants-appellees.
REYES, J.B.L., J.:
Direct appeal, on factual and legal questions, from the judgment of the Court of First Instance of Cebu, in its Civil Case No. R4177, denying the claim of the plaintiff-appellant, Pastor B. Tenchavez, for legal separation and one million pesos in damages
against his wife and parents-in-law, the defendants-appellees, Vicente, Mamerto and Mena, 1 all surnamed "Escao,"
respectively.2
The facts, supported by the evidence of record, are the following:
Missing her late afternoon classes on 24 February 1948 in the University of San Carlos, Cebu City, where she was then enrolled
as a second year student of commerce, Vicenta Escao, 27 years of age (scion of a well-to-do and socially prominent Filipino
family of Spanish ancestry and a "sheltered colegiala"), exchanged marriage vows with Pastor Tenchavez, 32 years of age, an
engineer, ex-army officer and of undistinguished stock, without the knowledge of her parents, before a Catholic chaplain, Lt.
Moises Lavares, in the house of one Juan Alburo in the said city. The marriage was the culmination of a previous love affair and
was duly registered with the local civil register.
Vicenta's letters to Pastor, and his to her, before the marriage, indicate that the couple were deeply in love. Together with a
friend, Pacita Noel, their matchmaker and go-between, they had planned out their marital future whereby Pacita would be the
governess of their first-born; they started saving money in a piggy bank. A few weeks before their secret marriage, their
engagement was broken; Vicenta returned the engagement ring and accepted another suitor, Joseling Lao. Her love for Pastor
beckoned; she pleaded for his return, and they reconciled. This time they planned to get married and then elope. To facilitate the
elopement, Vicenta had brought some of her clothes to the room of Pacita Noel in St. Mary's Hall, which was their usual trysting
place.
Although planned for the midnight following their marriage, the elopement did not, however, materialize because when Vicente
went back to her classes after the marriage, her mother, who got wind of the intended nuptials, was already waiting for her at
the college. Vicenta was taken home where she admitted that she had already married Pastor. Mamerto and Mena Escao
were surprised, because Pastor never asked for the hand of Vicente, and were disgusted because of the great scandal that the
clandestine marriage would provoke (t.s.n., vol. III, pp. 1105-06). The following morning, the Escao spouses sought priestly
advice. Father Reynes suggested a recelebration to validate what he believed to be an invalid marriage, from the standpoint of
the Church, due to the lack of authority from the Archbishop or the parish priest for the officiating chaplain to celebrate the
marriage. The recelebration did not take place, because on 26 February 1948 Mamerto Escao was handed by a maid, whose
name he claims he does not remember, a letter purportedly coming from San Carlos college students and disclosing an
amorous relationship between Pastor Tenchavez and Pacita Noel; Vicenta translated the letter to her father, and thereafter
would not agree to a new marriage. Vicenta and Pastor met that day in the house of Mrs. Pilar Mendezona. Thereafter, Vicenta
continued living with her parents while Pastor returned to his job in Manila. Her letter of 22 March 1948 (Exh. "M"), while still
solicitous of her husband's welfare, was not as endearing as her previous letters when their love was aflame.
Vicenta was bred in Catholic ways but is of a changeable disposition, and Pastor knew it. She fondly accepted her being called
a "jellyfish." She was not prevented by her parents from communicating with Pastor (Exh. "1-Escao"), but her letters became
less frequent as the days passed. As of June, 1948 the newlyweds were already estranged (Exh. "2-Escao"). Vicenta had
gone to Jimenez, Misamis Occidental, to escape from the scandal that her marriage stirred in Cebu society. There, a lawyer
filed for her a petition, drafted by then Senator Emmanuel Pelaez, to annul her marriage. She did not sign the petition (Exh. "B5"). The case was dismissed without prejudice because of her non-appearance at the hearing (Exh. "B-4").
On 24 June 1950, without informing her husband, she applied for a passport, indicating in her application that she was single,
that her purpose was to study, and she was domiciled in Cebu City, and that she intended to return after two years. The
application was approved, and she left for the United States. On 22 August 1950, she filed a verified complaint for divorce
against the herein plaintiff in the Second Judicial District Court of the State of Nevada in and for the County of Washoe, on the
ground of "extreme cruelty, entirely mental in character." On 21 October 1950, a decree of divorce, "final and absolute", was
issued in open court by the said tribunal.
In 1951 Mamerto and Mena Escao filed a petition with the Archbishop of Cebu to annul their daughter's marriage to Pastor
(Exh. "D"). On 10 September 1954, Vicenta sought papal dispensation of her marriage (Exh. "D"-2).
On 13 September 1954, Vicenta married an American, Russell Leo Moran, in Nevada. She now lives with him in California, and,
by him, has begotten children. She acquired American citizenship on 8 August 1958.
But on 30 July 1955, Tenchavez had initiated the proceedings at bar by a complaint in the Court of First Instance of Cebu, and
amended on 31 May 1956, against Vicenta F. Escao, her parents, Mamerto and Mena Escao, whom he charged with having
dissuaded and discouraged Vicenta from joining her husband, and alienating her affections, and against the Roman Catholic
Church, for having, through its Diocesan Tribunal, decreed the annulment of the marriage, and asked for legal separation and
one million pesos in damages. Vicenta claimed a valid divorce from plaintiff and an equally valid marriage to her present
husband, Russell Leo Moran; while her parents denied that they had in any way influenced their daughter's acts, and
counterclaimed for moral damages.
The appealed judgment did not decree a legal separation, but freed the plaintiff from supporting his wife and to acquire property
to the exclusion of his wife. It allowed the counterclaim of Mamerto Escao and Mena Escao for moral and exemplary
damages and attorney's fees against the plaintiff-appellant, to the extent of P45,000.00, and plaintiff resorted directly to this
Court.
The appellant ascribes, as errors of the trial court, the following:
1. In not declaring legal separation; in not holding defendant Vicenta F. Escao liable for damages and in dismissing the
complaint;.
2. In not holding the defendant parents Mamerto Escano and the heirs of Doa Mena Escao liable for damages;.
3 In holding the plaintiff liable for and requiring him to pay the damages to the defendant parents on their counterclaims;
and.
4. In dismissing the complaint and in denying the relief sought by the plaintiff.
That on 24 February 1948 the plaintiff-appellant, Pastor Tenchavez, and the defendant-appellee, Vicenta Escao, were validly
married to each other, from the standpoint of our civil law, is clearly established by the record before us. Both parties were then
above the age of majority, and otherwise qualified; and both consented to the marriage, which was performed by a Catholic

priest (army chaplain Lavares) in the presence of competent witnesses. It is nowhere shown that said priest was not duly
authorized under civil law to solemnize marriages.
The chaplain's alleged lack of ecclesiastical authorization from the parish priest and the Ordinary, as required by Canon law, is
irrelevant in our civil law, not only because of the separation of Church and State but also because Act 3613 of the Philippine
Legislature (which was the marriage law in force at the time) expressly provided that
SEC. 1. Essential requisites. Essential requisites for marriage are the legal capacity of the contracting parties and
consent. (Emphasis supplied)
The actual authority of the solemnizing officer was thus only a formal requirement, and, therefore, not essential to give the
marriage civil effects,3 and this is emphasized by section 27 of said marriage act, which provided the following:
SEC. 27. Failure to comply with formal requirements. No marriage shall be declared invalid because of the absence of
one or several of the formal requirements of this Act if, when it was performed, the spouses or one of them believed in
good faith that the person who solemnized the marriage was actually empowered to do so, and that the marriage was
perfectly legal.
The good faith of all the parties to the marriage (and hence the validity of their marriage) will be presumed until the contrary is
positively proved (Lao vs. Dee Tim, 45 Phil. 739, 745; Francisco vs. Jason, 60 Phil. 442, 448). It is well to note here that in the
case at bar, doubts as to the authority of the solemnizing priest arose only after the marriage, when Vicenta's parents consulted
Father Reynes and the archbishop of Cebu. Moreover, the very act of Vicenta in abandoning her original action for annulment
and subsequently suing for divorce implies an admission that her marriage to plaintiff was valid and binding.
Defendant Vicenta Escao argues that when she contracted the marriage she was under the undue influence of Pacita Noel,
whom she charges to have been in conspiracy with appellant Tenchavez. Even granting, for argument's sake, the truth of that
contention, and assuming that Vicenta's consent was vitiated by fraud and undue influence, such vices did not render her
marriage ab initio void, but merely voidable, and the marriage remained valid until annulled by a competent civil court. This was
never done, and admittedly, Vicenta's suit for annulment in the Court of First Instance of Misamis was dismissed for nonprosecution.
It is equally clear from the record that the valid marriage between Pastor Tenchavez and Vicenta Escao remained subsisting
and undissolved under Philippine law, notwithstanding the decree of absolute divorce that the wife sought and obtained on 21
October 1950 from the Second Judicial District Court of Washoe County, State of Nevada, on grounds of "extreme cruelty,
entirely mental in character." At the time the divorce decree was issued, Vicenta Escao, like her husband, was still a Filipino
citizen.4 She was then subject to Philippine law, and Article 15 of the Civil Code of the Philippines (Rep. Act No. 386), already in
force at the time, expressly provided:
Laws relating to family rights and duties or to the status, condition and legal capacity of persons are binding upon the
citizens of the Philippines, even though living abroad.
The Civil Code of the Philippines, now in force, does not admit absolute divorce, quo ad vinculo matrimonii; and in fact does not
even use that term, to further emphasize its restrictive policy on the matter, in contrast to the preceding legislation that admitted
absolute divorce on grounds of adultery of the wife or concubinage of the husband (Act 2710). Instead of divorce, the present
Civil Code only provides for legal separation (Title IV, Book 1, Arts. 97 to 108), and, even in that case, it expressly prescribes
that "the marriage bonds shall not be severed" (Art. 106, subpar. 1).
For the Philippine courts to recognize and give recognition or effect to a foreign decree of absolute divorce betiveen Filipino
citizens could be a patent violation of the declared public policy of the state, specially in view of the third paragraph of Article 17
of the Civil Code that prescribes the following:
Prohibitive laws concerning persons, their acts or property, and those which have for their object public order, policy and
good customs, shall not be rendered ineffective by laws or judgments promulgated, or by determinations or conventions
agreed upon in a foreign country.
Even more, the grant of effectivity in this jurisdiction to such foreign divorce decrees would, in effect, give rise to an irritating and
scandalous discrimination in favor of wealthy citizens, to the detriment of those members of our polity whose means do not
permit them to sojourn abroad and obtain absolute divorces outside the Philippines.
From this point of view, it is irrelevant that appellant Pastor Tenchavez should have appeared in the Nevada divorce court.
Primarily because the policy of our law cannot be nullified by acts of private parties (Civil Code,Art. 17, jam quot.); and
additionally, because the mere appearance of a non-resident consort cannot confer jurisdiction where the court originally had
none (Area vs. Javier, 95 Phil. 579).
From the preceding facts and considerations, there flows as a necessary consequence that in this jurisdiction Vicenta Escao's
divorce and second marriage are not entitled to recognition as valid; for her previous union to plaintiff Tenchavez must be
declared to be existent and undissolved. It follows, likewise, that her refusal to perform her wifely duties, and her denial
of consortium and her desertion of her husband constitute in law a wrong caused through her fault, for which the husband is
entitled to the corresponding indemnity (Civil Code, Art. 2176). Neither an unsubstantiated charge of deceit nor an anonymous
letter charging immorality against the husband constitute, contrary to her claim, adequate excuse. Wherefore, her marriage and
cohabitation with Russell Leo Moran is technically "intercourse with a person not her husband" from the standpoint of Philippine
Law, and entitles plaintiff-appellant Tenchavez to a decree of "legal separation under our law, on the basis of adultery" (Revised
Penal Code, Art. 333).
The foregoing conclusions as to the untoward effect of a marriage after an invalid divorce are in accord with the previous
doctrines and rulings of this court on the subject, particularly those that were rendered under our laws prior to the approval of
the absolute divorce act (Act 2710 of the Philippine Legislature). As a matter of legal history, our statutes did not recognize
divorces a vinculo before 1917, when Act 2710 became effective; and the present Civil Code of the Philippines, in disregarding
absolute divorces, in effect merely reverted to the policies on the subject prevailing before Act 2710. The rulings, therefore,
under the Civil Code of 1889, prior to the Act above-mentioned, are now, fully applicable. Of these, the decision in Ramirez vs.
Gmur, 42 Phil. 855, is of particular interest. Said this Court in that case:
As the divorce granted by the French Court must be ignored, it results that the marriage of Dr. Mory and Leona Castro,
celebrated in London in 1905, could not legalize their relations; and the circumstance that they afterwards passed for
husband and wife in Switzerland until her death is wholly without legal significance. The claims of the very children to
participate in the estate of Samuel Bishop must therefore be rejected. The right to inherit is limited to legitimate,
legitimated and acknowledged natural children. The children of adulterous relations are wholly excluded. The word
"descendants" as used in Article 941 of the Civil Code cannot be interpreted to include illegitimates born
of adulterous relations. (Emphasis supplied)
Except for the fact that the successional rights of the children, begotten from Vicenta's marriage to Leo Moran after the invalid
divorce, are not involved in the case at bar, the Gmur case is authority for the proposition that such union is adulterous in this
jurisdiction, and, therefore, justifies an action for legal separation on the part of the innocent consort of the first marriage, that
stands undissolved in Philippine law. In not so declaring, the trial court committed error.

True it is that our ruling gives rise to anomalous situations where the status of a person (whether divorced or not) would depend
on the territory where the question arises. Anomalies of this kind are not new in the Philippines, and the answer to them was
given in Barretto vs. Gonzales, 58 Phil. 667:
The hardship of the existing divorce laws in the Philippine Islands are well known to the members of the Legislature. It is
the duty of the Courts to enforce the laws of divorce as written by Legislature if they are constitutional. Courts have no
right to say that such laws are too strict or too liberal. (p. 72)
The appellant's first assignment of error is, therefore, sustained.
However, the plaintiff-appellant's charge that his wife's parents, Dr. Mamerto Escao and his wife, the late Doa Mena Escao,
alienated the affections of their daughter and influenced her conduct toward her husband are not supported by credible
evidence. The testimony of Pastor Tenchavez about the Escao's animosity toward him strikes us to be merely conjecture and
exaggeration, and are belied by Pastor's own letters written before this suit was begun (Exh. "2-Escao" and "Vicenta," Rec. on
App., pp. 270-274). In these letters he expressly apologized to the defendants for "misjudging them" and for the "great
unhappiness" caused by his "impulsive blunders" and "sinful pride," "effrontery and audacity" [sic]. Plaintiff was admitted to the
Escao house to visit and court Vicenta, and the record shows nothing to prove that he would not have been accepted to marry
Vicente had he openly asked for her hand, as good manners and breeding demanded. Even after learning of the clandestine
marriage, and despite their shock at such unexpected event, the parents of Vicenta proposed and arranged that the marriage
be recelebrated in strict conformity with the canons of their religion upon advice that the previous one was canonically defective.
If no recelebration of the marriage ceremony was had it was not due to defendants Mamerto Escao and his wife, but to the
refusal of Vicenta to proceed with it. That the spouses Escao did not seek to compel or induce their daughter to assent to the
recelebration but respected her decision, or that they abided by her resolve, does not constitute in law an alienation of
affections. Neither does the fact that Vicenta's parents sent her money while she was in the United States; for it was natural that
they should not wish their daughter to live in penury even if they did not concur in her decision to divorce Tenchavez (27 Am.
Jur. 130-132).
There is no evidence that the parents of Vicenta, out of improper motives, aided and abetted her original suit for annulment, or
her subsequent divorce; she appears to have acted independently, and being of age, she was entitled to judge what was best
for her and ask that her decisions be respected. Her parents, in so doing, certainly cannot be charged with alienation of
affections in the absence of malice or unworthy motives, which have not been shown, good faith being always presumed until
the contrary is proved.
SEC. 529. Liability of Parents, Guardians or Kin. The law distinguishes between the right of a parent to interest himself
in the marital affairs of his child and the absence of rights in a stranger to intermeddle in such affairs. However, such
distinction between the liability of parents and that of strangers is only in regard to what will justify interference. A parent
isliable for alienation of affections resulting from his own malicious conduct, as where he wrongfully entices his son or
daughter to leave his or her spouse, but he is not liable unless he acts maliciously, without justification and from unworthy
motives. He is not liable where he acts and advises his child in good faith with respect to his child's marital relations in the
interest of his child as he sees it, the marriage of his child not terminating his right and liberty to interest himself in, and be
extremely solicitous for, his child's welfare and happiness, even where his conduct and advice suggest or result in the
separation of the spouses or the obtaining of a divorce or annulment, or where he acts under mistake or misinformation,
or where his advice or interference are indiscreet or unfortunate, although it has been held that the parent is liable for
consequences resulting from recklessness. He may in good faith take his child into his home and afford him or her
protection and support, so long as he has not maliciously enticed his child away, or does not maliciously entice or cause
him or her to stay away, from his or her spouse. This rule has more frequently been applied in the case of advice given to
a married daughter, but it is equally applicable in the case of advice given to a son.
Plaintiff Tenchavez, in falsely charging Vicenta's aged parents with racial or social discrimination and with having exerted efforts
and pressured her to seek annulment and divorce, unquestionably caused them unrest and anxiety, entitling them to recover
damages. While this suit may not have been impelled by actual malice, the charges were certainly reckless in the face of the
proven facts and circumstances. Court actions are not established for parties to give vent to their prejudices or spleen.
In the assessment of the moral damages recoverable by appellant Pastor Tenchavez from defendant Vicente Escao, it is
proper to take into account, against his patently unreasonable claim for a million pesos in damages, that (a) the marriage was
celebrated in secret, and its failure was not characterized by publicity or undue humiliation on appellant's part; (b) that the
parties never lived together; and (c) that there is evidence that appellant had originally agreed to the annulment of the marriage,
although such a promise was legally invalid, being against public policy (cf. Art. 88, Civ. Code). While appellant is unable to
remarry under our law, this fact is a consequence of the indissoluble character of the union that appellant entered into
voluntarily and with open eyes rather than of her divorce and her second marriage. All told, we are of the opinion that appellant
should recover P25,000 only by way of moral damages and attorney's fees.
With regard to the P45,000 damages awarded to the defendants, Dr. Mamerto Escao and Mena Escao, by the court below,
we opine that the same are excessive. While the filing of this unfounded suit must have wounded said defendants' feelings and
caused them anxiety, the same could in no way have seriously injured their reputation, or otherwise prejudiced them, lawsuits
having become a common occurrence in present society. What is important, and has been correctly established in the decision
of the court below, is that said defendants were not guilty of any improper conduct in the whole deplorable affair. This Court,
therefore, reduces the damages awarded to P5,000 only.
Summing up, the Court rules:
(1) That a foreign divorce between Filipino citizens, sought and decreed after the effectivity of the present Civil Code (Rep. Act
386), is not entitled to recognition as valid in this jurisdiction; and neither is the marriage contracted with another party by the
divorced consort, subsequently to the foreign decree of divorce, entitled to validity in the country;
(2) That the remarriage of divorced wife and her co-habitation with a person other than the lawful husband entitle the latter to a
decree of legal separation conformably to Philippine law;
(3) That the desertion and securing of an invalid divorce decree by one consort entitles the other to recover damages;
(4) That an action for alienation of affections against the parents of one consort does not lie in the absence of proof of malice or
unworthy motives on their part.
WHEREFORE, the decision under appeal is hereby modified as follows;
(1) Adjudging plaintiff-appellant Pastor Tenchavez entitled to a decree of legal separation from defendant Vicenta F. Escao;
(2) Sentencing defendant-appellee Vicenta Escao to pay plaintiff-appellant Tenchavez the amount of P25,000 for damages and
attorneys' fees;
(3) Sentencing appellant Pastor Tenchavez to pay the appellee, Mamerto Escao and the estate of his wife, the deceased Mena
Escao, P5,000 by way of damages and attorneys' fees.
Neither party to recover costs.

G.R. No. 147902

March 17, 2006

SPOUSES VICENTE YU AND DEMETRIA LEE-YU, Petitioners,


vs.
PHILIPPINE COMMERCIAL INTERNATIONAL BANK, Respondent.
DECISION
AUSTRIA-MARTINEZ, J.:
Before the Court is a Petition for Review on Certiorari of the Decision 1 dated November 14, 2000 of the Court of Appeals (CA) in
CA-G.R. SP No. 58982 and the CA Resolution dated April 26, 2001, which denied petitioners Motion for Reconsideration.
The factual background of the case is as follows:
Under a Real Estate Mortgage dated August 15, 1994 2 and Amendments of Real Estate Mortgage dated April 4, 1995 3 and
December 4, 1995,4 spouses Vicente Yu and Demetria Lee-Yu (petitioners) and spouses Ramon T. Yu and Virginia A. Tiu, or Yu
Tian Hock aka Victorino/Vicente Yu, mortgaged their title, interest, and participation over several parcels of land located in
Dagupan City and Quezon City, in favor of the Philippine Commercial International Bank (respondent) as security for the
payment of a loan in the amount of P9,000,000.00.5
As the petitioners failed to pay the loan, the interest, and the penalties due thereon, respondent filed on July 21, 1998 with the
Office of the Clerk of Court and Ex-Officio Sheriff of the Regional Trial Court of Dagupan City a Petition for Extra-Judicial
Foreclosure of Real Estate Mortgage on the Dagupan City properties. 6 On August 3, 1998, the City Sheriff issued a Notice of
Extra-Judicial Sale scheduling the auction sale on September 10, 1998 at 10:00 oclock in the morning or soon thereafter in
front of the Justice Hall, Bonuan, Tondaligan, Dagupan City.7
At the auction sale on September 10, 1998, respondent emerged as the highest bidder. 8 On September 14, 1998, a Certificate
of Sale was issued in favor of respondent.9 On October 1, 1998, the sale was registered with the Registry of Deeds of Dagupan
City.
About two months before the expiration of the redemption period, or on August 20, 1999, respondent filed an Ex-Parte Petition
for Writ of Possession before the Regional Trial Court of Dagupan City, docketed as Special Proceeding No. 99-00988-D and
raffled to Branch 43 (RTC Branch 43).10 Hearing was conducted on September 14, 1999 and respondent presented its evidence
ex-parte.11 The testimony of Rodante Manuel was admitted ex-parte and thereafter the petition was deemed submitted for
resolution.
On September 30, 1999, petitioners filed a Motion to Dismiss and to Strike Out Testimony of Rodante Manuel stating that the
Certificate of Sale dated September 14, 1998 is void because respondent violated Article 2089 of the Civil Code on the
indivisibility of the mortgaged by conducting two separate foreclosure proceedings on the mortgage properties in Dagupan City
and Quezon City and indicating in the two notices of extra-judicial sale that petitioners obligation is P10,437,015.2012 as of
March 31, 1998, when petitioners are not indebted for the total amount of P20,874,031.56.13
In the meantime, petitioners filed a complaint for Annulment of Certificate of Sale before the Regional Trial Court of Dagupan
City, docketed as Civil Case No. 99-03169-D and raffled to Branch 44 (RTC Branch 44).
On February 14, 2000, RTC Branch 43 denied petitioners Motion to Dismiss and to Strike Out Testimony of Rodante Manuel,
ruling that the filing of a motion to dismiss is not allowed in petitions for issuance of writ of possession under Section 7 of Act
No. 3135.14
On February 24, 2000, petitioners filed a Motion for Reconsideration, further arguing that the pendency of Civil Case No. 9903169-D in RTC Branch 44 is a prejudicial issue to Spec. Proc. No. 99-00988-D in RTC Branch 43, the resolution of which is
determinative on the propriety of the issuance of a writ of possession. 15
On May 8, 2000, RTC Branch 43 denied petitioners Motion for Reconsideration, holding that the principle of prejudicial question
is not applicable because the case pending before RTC Branch 44 is also a civil case and not a criminal case. 16
On June 1, 2000, petitioners filed a Petition for Certiorari with the CA. 17 On November 14, 2000, the CA dismissed petitioners
Petition for Certiorari on the grounds that petitioners violated Section 8 of Act No. 3135 and disregarded the rule against
multiplicity of suits in filing Civil Case No. 99-03169-D in RTC Branch 44 despite full knowledge of the pendency of Spec. Proc.
No. 99-00988-D in RTC Branch 43; that since the one-year period of redemption has already lapsed, the issuance of a writ of
possession in favor of respondent becomes a ministerial duty of the trial court; that the issues in Civil Case No. 99-03169-D are
not prejudicial questions to Spec. Proc. No. 99-00988-D because: (a) the special proceeding is already fait accompli, (b) Civil
Case No. 99-03169-D is deemed not filed for being contrary to Section 8 of Act No. 3135, (c) the filing of Civil Case No. 9903169-D is an afterthought and dilatory in nature, and (d) legally speaking what seems to exist is litis pendentia and not
prejudicial question.18
Petitioners filed a Motion for Reconsideration 19 but it was denied by the CA on April 26, 2001.20
Hence, the present Petition for Review on Certiorari.
Petitioners pose two issues for resolution, to wit:
A. Whether or not a real estate mortgage over several properties located in different locality [sic] can be separately
foreclosed in different places.
B. Whether or not the pendency of a prejudicial issue renders the issues in Special Proceedings No. 99-00988-D as [sic]
moot and academic.21
Anent the first issue, petitioners contend that since a real estate mortgage is indivisible, the mortgaged properties in Dagupan
City and Quezon City cannot be separately foreclosed. Petitioners further point out that two notices of extra-judicial sale
indicated that petitioners obligation is P10,437,015.2022 each as of March 31, 1998 or a total ofP20,874,030.40,23 yet their own
computation yields only P9,957,508.90 as of February 27, 1998.
As to the second issue, petitioners posit that the pendency of Civil Case No. 99-03169-D is a prejudicial issue, the resolution of
which will render the issues in Spec. Proc. No. 99-00988-D moot and academic. Petitioners further aver that they did not violate
Section 8 of Act No. 3135 in filing a separate case to annul the certificate of sale since the use of the word "may" in said
provision indicates that they have the option to seek relief of filing a petition to annul the certificate of sale in the proceeding
involving the application for a writ of possession or in a separate proceeding.
Respondent contends24 that, with respect to the first issue, the filing of two separate foreclosure proceedings did not violate
Article 2089 of the Civil Code on the indivisibility of a real estate mortgage since Section 2 of Act No. 3135 expressly provides
that extra-judicial foreclosure may only be made in the province or municipality where the property is situated. Respondent
further submits that the filing of separate applications for extra-judicial foreclosure of mortgage involving several properties in
different locations is allowed by A.M. No. 99-10-05-0, the Procedure on Extra-Judicial Foreclosure of Mortgage, as further
amended on August 7, 2001.

As to the second issue, respondent maintains that there is no prejudicial question between Civil Case No. 99-03169-D and
Spec. Proc. No. 99-00988-D since the pendency of a civil action questioning the validity of the mortgage and the extra-judicial
foreclosure thereof does not bar the issuance of a writ of possession. Respondent also insists that petitioners should have filed
their Petition to Annul the Certificate of Sale in the same case where possession is being sought, that is, in Spec. Proc. No. 9900988-D, and not in a separate proceeding (Civil Case No. 99-01369-D) because the venue of the action to question the validity
of the foreclosure is not discretionary since the use of the word "may" in Section 8 of Act No. 3135 refers to the filing of the
petition or action itself and not to the venue. Respondent further argues that even if petitioners filed the Petition to Annul the
Certificate of Sale in Spec. Proc. No. 99-00988-D, the writ of possession must still be issued because issuance of the writ in
favor of the purchaser is a ministerial act of the trial court and the one-year period of redemption has already lapsed.
Anent the first issue, the Court finds that petitioners have a mistaken notion that the indivisibility of a real estate mortgage
relates to the venue of extra-judicial foreclosure proceedings. The rule on indivisibility of a real estate mortgage is provided for
in Article 2089 of the Civil Code, which provides:
Art. 2089. A pledge or mortgage is indivisible, even though the debt may be divided among the successors in interest of the
debtor or of the creditor.
Therefore, the debtors heir who has paid a part of the debt cannot ask for the proportionate extinguishment of the pledge or
mortgage as the debt is not completely satisfied.
Neither can the creditors heir who received his share of the debt return the pledge or cancel the mortgage, to the prejudice of
the other heirs who have not been paid.
From these provisions is excepted the case in which, there being several things given in mortgage or pledge, each one of them
guarantees only a determinate portion of the credit.
The debtor, in this case, shall have a right to the extinguishment of the pledge or mortgage as the portion of the debt for which
each thing is specially answerable is satisfied.
This rule presupposes several heirs of the debtor or creditor 25 and therefore not applicable to the present case. Furthermore,
what the law proscribes is the foreclosure of only a portion of the property or a number of the several properties mortgaged
corresponding to the unpaid portion of the debt where, before foreclosure proceedings, partial payment was made by the debtor
on his total outstanding loan or obligation. This also means that the debtor cannot ask for the release of any portion of the
mortgaged property or of one or some of the several lots mortgaged unless and until the loan thus secured has been fully paid,
notwithstanding the fact that there has been partial fulfillment of the obligation. Hence, it is provided that the debtor who has
paid a part of the debt cannot ask for the proportionate extinguishment of the mortgage as long as the debt is not completely
satisfied.26 In essence, indivisibility means that the mortgage obligation cannot be divided among the different lots, 27 that is,
each and every parcel under mortgage answers for the totality of the debt. 28
On the other hand, the venue of the extra-judicial foreclosure proceedings is the place where each of the mortgaged property is
located, as prescribed by Section 2 of Act No. 3135, 29 to wit:
SECTION 2. Said sale cannot be made legally outside of the province in which the property sold is situated; and in case the
place within said province in which the sale is to be made is subject to stipulation, such sale shall be made in said place or in
the municipal building of the municipality in which the property or part thereof is situated.
A.M. No. 99-10-05-0,30 the Procedure on Extra-Judicial Foreclosure of Mortgage, lays down the guidelines for extra-judicial
foreclosure proceedings on mortgaged properties located in different provinces. It provides that the venue of the extra-judicial
foreclosure proceedings is the place where each of the mortgaged property is located. Relevant portion thereof provides:
Where the application concerns the extrajudicial foreclosure of mortgages of real estates and/or chattels in different locations
covering one indebtedness, only one filing fee corresponding to such indebtedness shall be collected. The collecting Clerk of
Court shall, apart from the official receipt of the fees, issue a certificate of payment indicating the amount of indebtedness, the
filing fees collected, the mortgages sought to be foreclosed, the real estates and/or chattels mortgaged and their respective
locations, which certificate shall serve the purpose of having the application docketed with the Clerks of Court of the
places where the other properties are located and of allowing the extrajudicial foreclosures to proceed thereat.
(Emphasis supplied)
The indivisibility of the real estate mortgage is not violated by conducting two separate foreclosure proceedings on mortgaged
properties located in different provinces as long as each parcel of land is answerable for the entire debt. Petitioners assumption
that their total obligation is P20,874,030.40 because the two notices of extra-judicial sale indicated that petitioners obligation
is P10,437,015.2031 each, is therefore flawed. Considering the indivisibility of a real estate mortgage, the mortgaged properties
in Dagupan City and Quezon City are made to answer for the entire debt of P10,437,015.29.32
As to the second issue, that is, whether a civil case for annulment of a certificate of sale is a prejudicial question to a petition for
issuance of a writ of possession, this issue is far from novel and, in fact, not without precedence. In Pahang v. Vestil, 33 the Court
said:
A prejudicial question is one that arises in a case the resolution of which is a logical antecedent of the issue involved therein,
and the cognizance of which pertains to another tribunal. It generally comes into play in a situation where a civil action and a
criminal action are both pending and there exists in the former an issue that must be preemptively resolved before the criminal
action may proceed, because howsoever the issue raised in the civil action is resolved would be determinative juris et de jure of
the guilt or innocence of the accused in the criminal case. The rationale behind the principle of prejudicial question is to avoid
two conflicting decisions. 1avvph!l.net
In the present case, the complaint of the petitioners for Annulment of Extrajudicial Sale is a civil action and the respondents
petition for the issuance of a writ of possession of Lot No. 3-A, Block 1, Psd-07-021410, TCT No. 44668 is but an incident in the
land registration case and, therefore, no prejudicial question can arise from the existence of the two actions. A similar issue was
raised in Manalo v. Court of Appeals, where we held that:
At any rate, it taxes our imagination why the questions raised in Case No. 98-0868 must be considered determinative of Case
No. 9011. The basic issue in the former is whether the respondent, as the purchaser in the extrajudicial foreclosure
proceedings, may be compelled to have the property repurchased or resold to a mortgagors successor-in-interest (petitioner);
while that in the latter is merely whether the respondent, as the purchaser in the extrajudicial foreclosure proceedings, is entitled
to a writ of possession after the statutory period for redemption has expired. The two cases, assuming both are pending, can
proceed separately and take their own direction independent of each other.34
In the present case, Civil Case No. 99-01369-D and Spec. Proc. No. 99-00988-D are both civil in nature. The issue in Civil Case
No. 99-01369-D is whether the extra-judicial foreclosure of the real estate mortgage executed by the petitioners in favor of the
respondent and the sale of their properties at public auction are null and void, whereas, the issue in Spec. Proc. No. 99-00988D is whether the respondent is entitled to a writ of possession of the foreclosed properties. Clearly, no prejudicial question can
arise from the existence of the two actions. The two cases can proceed separately and take their own direction independently of
each other.

Nevertheless, there is a need to correct the CAs view that petitioners violated Section 8 of Act No. 3135 and disregarded the
proscription on multiplicity of suits by instituting a separate civil suit for annulment of the certificate of sale while there is a
pending petition for issuance of the writ of possession in a special proceeding.
Section 8 of Act No. 3135 provides:
Sec. 8. Setting aside of sale and writ of possession. The debtor may, in the proceedings in which possession was requested,
but not later than thirty days after the purchaser was given possession, petition that the sale be set aside and the writ of
possession cancelled, specifying the damages suffered by him, because the mortgage was not violated or the sale was not
made in accordance with the provisions hereof, and the court shall take cognizance of this petition in accordance with the
summary procedure provided for in section one hundred and twelve of Act Numbered Four hundred and ninety-six; and if it finds
the complaint of the debtor justified, it shall dispose in his favor of all or part of the bond furnished by the person who obtained
possession. Either of the parties may appeal from the order of the judge in accordance with section fourteen of Act Numbered
Four hundred and ninety-six; but the order of possession shall continue in effect during the pendency of the appeal. (Emphasis
supplied)
Under the provision above cited, the mortgagor may file a petition to set aside the sale and for the cancellation of a writ of
possession with the trial court which issued the writ of possession within 30 days after the purchaser mortgagee was given
possession. It provides the plain, speedy, and adequate remedy in opposing the issuance of a writ of possession. 35 Thus, this
provision presupposes that the trial court already issued a writ of possession. In Sps. Ong v. Court of Appeals, 36 the Court
elucidated:
The law is clear that the purchaser must first be placed in possession of the mortgaged property pending proceedings assailing
the issuance of the writ of possession. If the trial court later finds merit in the petition to set aside the writ of possession, it shall
dispose in favor of the mortgagor the bond furnished by the purchaser. Thereafter, either party may appeal from the order of the
judge in accordance with Section 14 of Act 496, which provides that "every order, decision, and decree of the Court of Land
Registration may be reviewedin the same manner as an order, decision, decree or judgment of a Court of First Instance
(RTC) might be reviewed." The rationale for the mandate is to allow the purchaser to have possession of the foreclosed
property without delay, such possession being founded on his right of ownership. 37
Accordingly, Section 8 of Act No. 3135 is not applicable to the present case since at the time of the filing of the separate civil suit
for annulment of the certificate of sale in RTC Branch 44, no writ of possession was yet issued by RTC Branch 43.
Similarly, the Court rejects the CAs application of the principle of litis pendentia to Civil Case No. 99-03169-D in relation to
Spec. Proc. No. 99-00988-D. Litis pendentia refers to that situation wherein another action is pending between the same parties
for the same cause of actions and that the second action becomes unnecessary and vexatious. For litis pendentia to be
invoked, the concurrence of the following requisites is necessary: (a) identity of parties or at least such as represent the same
interest in both actions; (b) identity of rights asserted and reliefs prayed for, the reliefs being founded on the same facts; and, (c)
the identity in the two cases should be such that the judgment that may be rendered in one would, regardless of which party is
successful, amount to res judicata in the other.38
Applying the foregoing criteria in the instant case, litis pendentia does not obtain in this case because of the absence of the
second and third requisites. The issuance of the writ of possession being a ministerial function, and summary in nature, it
cannot be said to be a judgment on the merits, but simply an incident in the transfer of title. Hence, a separate case for
annulment of mortgage and foreclosure sale cannot be barred by litis pendentiaor res judicata.39 Thus, insofar as Spec. Proc.
No. 99-00988-D and Civil Case No. 99-03169-D pending before different branches of RTC Dagupan City are concerned, there
is no litis pendentia.
To sum up, the Court holds that the rule on indivisibility of the real estate mortgage cannot be equated with the venue of
foreclosure proceedings on mortgaged properties located in different provinces since these are two unrelated concepts. Also, no
prejudicial question can arise from the existence of a civil case for annulment of a certificate of sale and a petition for the
issuance of a writ of possession in a special proceeding since the two cases are both civil in nature which can proceed
separately and take their own direction independently of each other.
Furthermore, since the one-year period to redeem the foreclosed properties lapsed on October 1, 1999, title to the foreclosed
properties had already been consolidated under the name of the respondent. As the owner of the properties, respondent is
entitled to its possession as a matter of right.40 The issuance of a writ of possession over the properties by the trial court is
merely a ministerial function. As such, the trial court neither exercises its official discretion nor judgment. 41 Any question
regarding the validity of the mortgage or its foreclosure cannot be a legal ground for refusing the issuance of a writ of
possession.42 Regardless of the pending suit for annulment of the certificate of sale, respondent is entitled to a writ of
possession, without prejudice of course to the eventual outcome of said case. 43

G.R. No. L-53642 April 15, 1988


LEONILO C. DONATO, petitioners,
vs.
HON. ARTEMON D. LUNA, PRESIDING JUDGE, COURT OF FIRST INSTANCE OF MANIIA, BRANCH XXXII HON. JOSE
FLAMINIANO, CITY FISCAL OF MANILA; PAZ B. ABAYAN, respondents.
Leopoldo P. Dela Rosa for petitioner.
Emiterio C. Manibog for private respondent.
City Fiscal of Manila for public respondent.

GANCAYCO, J.:
In this petition for certiorari and prohibition with preliminary injunction, the question for the resolution of the Court is whether or
not a criminal case for bigamy pending before the Court of First Itance of Manila should be suspended in view of a civil case for
annulment of marriage pending before the Juvenile and Domestic Relations Court on the ground that the latter constitutes a
prejudicial question. The respondent judge ruled in the negative. We sustain him.
The pertinent facts as set forth in the records follow. On January 23, 1979, the City Fiscal of Manila acting thru Assistant City
Fiscal Amado N. Cantor filed an information for bigamy against herein petitioner, Leonilo C. Donato with the Court of First
Instance of Manila, docketed as Criminal Case No. 43554 and assigned to Branch XXXII of said court. The information was filed
based on the complaint of private respondent Paz B. Abayan.
On September 28, 1979, before the petitioner's arraignment, private respondent filed with the Juvenile and Domestic Relations
Court of Manila a civil action for declaration of nullity of her marriage with petitioner contracted on September 26, 1978, which
action was docketed as Civil Case No. E-02627. Said civil case was based on the ground that private respondent consented to
entering into the marriage, which was petitioner Donato's second one, since she had no previous knowledge that petitioner was
already married to a certain Rosalinda R. Maluping on June 30, 1978. Petitioner Donato's answer in the civil case for nullity
interposed the defense that his second marriage was void since it was solemnized without a marriage license and that force,
violence, intimidation and undue influence were employed by private respondent to obtain petitioner's consent to the marriage.
Prior to the solemnization of the subsequent or second marriage, petitioner and private respondent had lived together and
deported themselves as husband and wife without the benefit of wedlock for a period of at least five years as evidenced by a
joint affidavit executed by them on September 26, 1978, for which reason, the requisite marriage license was dispensed with
pursuant to Article 76 of the New Civil Code pertaining to marriages of exceptional character.
Prior to the date set for the trial on the merits of Criminal Case No. 43554, petitioner filed a motion to suspend the proceedings
of said case contending that Civil Case No. E-02627 seeking the annulment of his second marriage filed by private respondent
raises a prejudicial question which must first be determined or decided before the criminal case can proceed.
In an order dated April 7, 1980. Hon. Artemon D. Luna denied the motion to suspend the proceedings in Criminal Case No.
43554 for bigamy. Respondent judge's basis for denial is the ruling laid down in the case of Landicho vs. Relova. 1 The order
further directed that the proceedings in the criminal case can proceed as scheduled.
A motion for reconsideration was flied by herein petitioner thru counsel citing as one of his grounds for suspension of
proceedings the ruling laid down by this Court in the case of De la Cruz vs. Ejercito 2 which was a much later case than that
cited by respondent judge in his order of denial.
The motion for reconsideration of the said order was likewise denied in an order dated April 14, 1980, for lack of merit. Hence,
the present petition for certiorari and prohibition with preliminary injunction.
A prejudicial question has been defined to be one which arises in a case, the resolution of which question is a logical
antecedent of the issue involved in said case, and the cognizance of which pertains to another tribunal. 3 It is one based on a
fact distinct and separate from the crime but so intimately connected with it that it determines the guilt or innocence of the
accused, and for it to suspend the criminal action, it must appear not only that said case involves facts intimately related to
those upon which the criminal prosecution would be based but also that in the resolution of the issue or issues raised in the civil
case, the guilt or innocence of the accused would necessarily be determined. 4 A prejudicial question usually comes into play in
a situation where a civil action and a criminal action may proceed, because howsoever the issue raised in the civil action is
resolved would be determinative juris et de jure of the guilt or innocence of the accused in a criminal case. 5
The requisites of a prejudicial question do not obtain in the case at bar. It must be noted that the issue before the Juvenile and
Domestic Relations Court touching upon the nullity of the second marriage is not determinative of petitioner Donato's guilt or
innocence in the crime of bigamy. Furthermore, it was petitioner's second wife, the herein private respondent Paz B. Abayan
who filed the complaint for annulment of the second marriage on the ground that her consent was obtained through deceit.
Petitioner Donato raised the argument that the second marriage should have been declared null and void on the ground of
force, threats and intimidation allegedly employed against him by private respondent only sometime later when he was required
to answer the civil action for anulment of the second marriage. The doctrine elucidated upon by the case of Landicho vs.
Relova 6 may be applied to the present case. Said case states that:
The mere fact that there are actions to annul the marriages entered into by the accused in a bigamy case does not
mean that "prejudicial questions" are automatically raised in civil actions as to warrant the suspension of the case. In
order that the case of annulment of marriage be considered a prejudicial question to the bigamy case against the
accused, it must be shown that the petitioner's consent to such marriage must be the one that was obtained by
means of duress, force and intimidation to show that his act in the second marriage must be involuntary and cannot
be the basis of his conviction for the crime of bigamy. The situation in the present case is markedly different. At the
time the petitioner was indicted for bigamy on February 27, 1963, the fact that two marriage ceremonies had been
contracted appeared to be indisputable. And it was the second spouse, not the petitioner who filed the action for
nullity on the ground of force, threats and intimidation. And it was only on June 15, 1963, that petitioner, as
defendant in the civil action, filed a third-party complaint against the first spouse alleging that his marriage with her
should be declared null and void on the ground of force, threats and intimidation. Assuming that the first marriage
was null and void on the ground alleged by petitioner, the fact would not be material to the outcome of the case.
Parties to the marriage should not be permitted to judge for themselves its nullity, for the same must be submitted to
the judgment of the competent courts and only when the nullity of the marriage is so declared can it be held as void,
and so long as there is no such declaration the presumption is that the marriage exists. Therefore, he who contracts
a second marriage before the judicial declaration of nullity of the first marriage assumes the risk of being prosecuted
for bigamy. The lower court therefore, has not abused much less gravely abused, its discretion in failing to suspend
the hearing as sought by petitioner.
In the case at bar, petitioner has not even sufficiently shown that his consent to the second marriage has been obtained by the
use of threats, force and intimidation.

Petitioner calls the attention of this Court to the fact that the case of De la Cruz vs. Ejercito is a later case and as such it should
be the one applied to the case at bar. We cannot agree. The situation in the case at bar is markedly different. In the aforecited
case it was accused Milagros dela Cruz who was charged with bigamy for having contracted a second marriage while a
previous one existed. Likewise, Milagros dela Cruz was also the one who filed an action for annulment on the ground of duress,
as contra-distinguished from the present case wherein it was private respondent Paz B. Abayan, petitioner's second wife, who
filed a complaint for annulment of the second marriage on the ground that her consent was obtained through deceit since she
was not aware that petitioner's marriage was still subsisting. Moreover, in De la Cruz, a judgment was already rendered in the
civil case that the second marriage of De la Cruz was null and void, thus determinative of the guilt or innocence of the accused
in the criminal case. In the present case, there is as yet no such judgment in the civil case.
Pursuant to the doctrine discussed in Landicho vs. Relova, petitioner Donato cannot apply the rule on prejudicial questions
since a case for annulment of marriage can be considered as a prejudicial question to the bigamy case against the accused
only if it is proved that the petitioner's consent to such marriage was obtained by means of duress, violence and intimidation in
order to establish that his act in the subsequent marriage was an involuntary one and as such the same cannot be the basis for
conviction. The preceding elements do not exist in the case at bar.
Obviously, petitioner merely raised the issue of prejudicial question to evade the prosecution of the criminal case. The records
reveal that prior to petitioner's second marriage on September 26, 1978, he had been living with private respondent Paz B.
Abayan as husband and wife for more than five years without the benefit of marriage. Thus, petitioner's averments that his
consent was obtained by private respondent through force, violence, intimidation and undue influence in entering a subsequent
marriage is belled by the fact that both petitioner and private respondent executed an affidavit which stated that they had lived
together as husband and wife without benefit of marriage for five years, one month and one day until their marital union was
formally ratified by the second marriage and that it was private respondent who eventually filed the civil action for nullity.
Another event which militates against petitioner's contentions is the fact hat it was only when Civil Case No. E-02627 was filed
on September 28, 1979, or more than the lapse of one year from the solemnization of the second marriage that petitioner came
up with the story that his consent to the marriage was secured through the use of force, violence, intimidation and undue
influence. Petitioner also continued to live with private respondent until November 1978, when the latter left their abode upon
learning that Leonilo Donato was already previously married.
In the light of the preceding factual circumstances, it can be seen that the respondent Judge did not err in his earlier order.
There is no pivotal issue that must be pre-emptively resolved in Civil Case No. E-02627 before proceedings in the criminal
action for bigamy can be undertaken.
Accordingly, there being no prejudicial question shown to exit the order of denial issued by the respondent judge dated April 14,
1980 should be sustained.
WHEREFORE, in view of the foregoing, the instant petition is hereby DISMISSED for lack of merit. We make no
pronouncement as to costs.

Civil Personality
G.R. No. 26795 July 31, 1970
CARMEN QUIMIGUING, Suing through her parents, ANTONIO QUIMIGUING and JACOBA CABILIN,plaintiffs-appellants,
vs.
FELIX ICAO, defendant-appellee.
Torcuato L. Galon for plaintiffs-appellants.
Godardo Jacinto for defendant-appellee.

REYES, J.B.L., J.:


Appeal on points of law from an order of the Court of First Instance of Zamboanga del Norte (Judge Onofre Sison Abalos,
presiding), in its Civil Case No. 1590, dismissing a complaint for support and damages, and another order denying amendment
of the same pleading.
The events in the court of origin can be summarized as follows:
Appellant, Carmen Quimiguing, assisted by her parents, sued Felix Icao in the court below. In her complaint it was averred that
the parties were neighbors in Dapitan City, and had close and confidential relations; that defendant Icao, although married,
succeeded in having carnal intercourse with plaintiff several times by force and intimidation, and without her consent; that as a
result she became pregnant, despite efforts and drugs supplied by defendant, and plaintiff had to stop studying. Hence, she
claimed support at P120.00 per month, damages and attorney's fees.
Duly summoned, defendant Icao moved to dismiss for lack of cause of action since the complaint did not allege that the child
had been born; and after hearing arguments, the trial judge sustained defendant's motion and dismissed the complaint.
Thereafter, plaintiff moved to amend the complaint to allege that as a result of the intercourse, plaintiff had later given birth to a
baby girl; but the court, sustaining defendant's objection, ruled that no amendment was allowable, since the original complaint
averred no cause of action. Wherefore, the plaintiff appealed directly to this Court.
We find the appealed orders of the court below to be untenable. A conceived child, although as yet unborn, is given by law a
provisional personality of its own for all purposes favorable to it, as explicitly provided in Article 40 of the Civil Code of the
Philippines. The unborn child, therefore, has a right to support from its progenitors, particularly of the defendant-appellee
(whose paternity is deemed admitted for the purpose of the motion to dismiss), even if the said child is only "en ventre de sa
mere;" just as a conceived child, even if as yet unborn, may receive donations as prescribed by Article 742 of the same Code,
and its being ignored by the parent in his testament may result in preterition of a forced heir that annuls the institution of the
testamentary heir, even if such child should be born after the death of the testator Article 854, Civil Code).
ART. 742. Donations made to conceived and unborn children may be accepted by those persons who would legally
represent them if they were already born.
ART. 854. The preterition or omission of one, some, or all of the compulsory heirs in the direct line, whether living at
the time of the execution of the will or born after the death of the testator, shall annul the institution of heir; but the
devises and legacies shall be valid insofar as they are not inofficious.
If the omitted compulsory heirs should die before the testator, the institution shall be effectual, without prejudice to
the right of 'representation.
It is thus clear that the lower court's theory that Article 291 of the Civil Code declaring that support is an obligation of parents
and illegitimate children "does not contemplate support to children as yet unborn," violates Article 40 aforesaid, besides
imposing a condition that nowhere appears in the text of Article 291. It is true that Article 40 prescribing that "the conceived child
shall be considered born for all purposes that are favorable to it" adds further "provided it be born later with the conditions
specified in the following article" (i.e., that the foetus be alive at the time it is completely delivered from the mother's womb). This
proviso, however, is not a condition precedent to the right of the conceived child; for if it were, the first part of Article 40 would
become entirely useless and ineffective. Manresa, in his Commentaries (5th Ed.) to the corresponding Article 29 of the Spanish
Civil Code, clearly points this out:
Los derechos atribuidos al nasciturus no son simples expectativas, ni aun en el sentido tecnico que la moderna
doctrina da a esta figura juridica sino que constituyen un caso de los propiamente Ilamados 'derechos en estado de
pendenci'; el nacimiento del sujeto en las condiciones previstas por el art. 30, no determina el nacimiento de
aquellos derechos (que ya existian de antemano), sino que se trata de un hecho que tiene efectos declarativos. (1
Manresa, Op. cit., page 271)
A second reason for reversing the orders appealed from is that for a married man to force a woman not his wife to yield to his
lust (as averred in the original complaint in this case) constitutes a clear violation of the rights of his victim that entitles her to
claim compensation for the damage caused. Says Article 21 of the Civil Code of the Philippines:
ART. 21. Any person who wilfully causes loss or injury to another in a manner that is contrary to morals, good
customs or public policy shall compensate the latter for the damage.
The rule of Article 21 is supported by Article 2219 of the same Code:
ART 2219. Moral damages may be recovered in the following and analogous cases:
(3) Seduction, abduction, rape or other lascivious acts:
xxx xxx xxx
(10) Acts and actions referred to in Articles 21, 26, 27, 28 ....
Thus, independently of the right to Support of the child she was carrying, plaintiff herself had a cause of action for damages
under the terms of the complaint; and the order dismissing it for failure to state a cause of action was doubly in error.
WHEREFORE, the orders under appeal are reversed and set aside. Let the case be remanded to the court of origin for further
proceedings conformable to this decision. Costs against appellee Felix Icao. So ordered.

G.R. No. L-16439

July 20, 1961

ANTONIO GELUZ, petitioner,


vs.
THE HON. COURT OF APPEALS and OSCAR LAZO, respondents.
Mariano H. de Joya for petitioner.
A.P. Salvador for respondents.
REYES, J.B.L., J.:
This petition for certiorari brings up for review question whether the husband of a woman, who voluntarily procured her abortion,
could recover damages from physician who caused the same.
The litigation was commenced in the Court of First Instance of Manila by respondent Oscar Lazo, the of Nita Villanueva, against
petitioner Antonio Geluz, a physician. Convinced of the merits of the complaint upon the evidence adduced, the trial court
rendered judgment favor of plaintiff Lazo and against defendant Geluz, ordering the latter to pay P3,000.00 as damages,
P700.00 attorney's fees and the costs of the suit. On appeal, Court of Appeals, in a special division of five, sustained the award
by a majority vote of three justices as against two, who rendered a separate dissenting opinion.
The facts are set forth in the majority opinion as follows:
Nita Villanueva came to know the defendant (Antonio Geluz) for the first time in 1948 through her aunt Paula Yambot.
In 1950 she became pregnant by her present husband before they were legally married. Desiring to conceal her
pregnancy from her parent, and acting on the advice of her aunt, she had herself aborted by the defendant. After her
marriage with the plaintiff, she again became pregnant. As she was then employed in the Commission on Elections and
her pregnancy proved to be inconvenient, she had herself aborted again by the defendant in October 1953. Less than two
years later, she again became pregnant. On February 21, 1955, accompanied by her sister Purificacion and the latter's
daughter Lucida, she again repaired to the defendant's clinic on Carriedo and P. Gomez streets in Manila, where the three
met the defendant and his wife. Nita was again aborted, of a two-month old foetus, in consideration of the sum of fifty
pesos, Philippine currency. The plaintiff was at this time in the province of Cagayan, campaigning for his election to the
provincial board; he did not know of, nor gave his consent, to the abortion.
It is the third and last abortion that constitutes plaintiff's basis in filing this action and award of damages. Upon application of the
defendant Geluz we granted certiorari.
The Court of Appeals and the trial court predicated the award of damages in the sum of P3,000.06 upon the provisions of the
initial paragraph of Article 2206 of the Civil Code of the Philippines. This we believe to be error, for the said article, in fixing a
minimum award of P3,000.00 for the death of a person, does not cover the case of an unborn foetus that is not endowed with
personality. Under the system of our Civil Code, "la criatura abortiva no alcanza la categoria de persona natural y en
consscuencia es un ser no nacido a la vida del Derecho" (Casso-Cervera, "Diccionario de Derecho Privado", Vol. 1, p. 49),
being incapable of having rights and obligations.
Since an action for pecuniary damages on account of personal injury or death pertains primarily to the one injured, it is easy to
see that if no action for such damages could be instituted on behalf of the unborn child on account of the injuries it received, no
such right of action could derivatively accrue to its parents or heirs. In fact, even if a cause of action did accrue on behalf of the
unborn child, the same was extinguished by its pre-natal death, since no transmission to anyone can take place from on that
lacked juridical personality (or juridical capacity as distinguished from capacity to act). It is no answer to invoke the provisional
personality of a conceived child (conceptus pro nato habetur) under Article 40 of the Civil Code, because that same article
expressly limits such provisional personality by imposing the condition that the child should be subsequently born alive:
"provided it be born later with the condition specified in the following article". In the present case, there is no dispute that the
child was dead when separated from its mother's womb.
The prevailing American jurisprudence is to the same effect; and it is generally held that recovery can not had for the death of
an unborn child (Stafford vs. Roadway Transit Co., 70 F. Supp. 555; Dietrich vs. Northampton, 52 Am. Rep. 242; and numerous
cases collated in the editorial note, 10 ALR, (2d) 639).
This is not to say that the parents are not entitled to collect any damages at all. But such damages must be those inflicted
directly upon them, as distinguished from the injury or violation of the rights of the deceased, his right to life and physical
integrity. Because the parents can not expect either help, support or services from an unborn child, they would normally be
limited to moral damages for the illegal arrest of the normal development of the spes hominis that was the foetus, i.e., on
account of distress and anguish attendant to its loss, and the disappointment of their parental expectations (Civ. Code Art.
2217), as well as to exemplary damages, if the circumstances should warrant them (Art. 2230). But in the case before us, both
the trial court and the Court of Appeals have not found any basis for an award of moral damages, evidently because the
appellee's indifference to the previous abortions of his wife, also caused by the appellant herein, clearly indicates that he was
unconcerned with the frustration of his parental hopes and affections. The lower court expressly found, and the majority opinion
of the Court of Appeals did not contradict it, that the appellee was aware of the second abortion; and the probabilities are that he
was likewise aware of the first. Yet despite the suspicious repetition of the event, he appeared to have taken no steps to
investigate or pinpoint the causes thereof, and secure the punishment of the responsible practitioner. Even after learning of the
third abortion, the appellee does not seem to have taken interest in the administrative and criminal cases against the appellant.
His only concern appears to have been directed at obtaining from the doctor a large money payment, since he sued for
P50,000.00 damages and P3,000.00 attorney's fees, an "indemnity" claim that, under the circumstances of record, was clearly
exaggerated.
The dissenting Justices of the Court of Appeals have aptly remarked that:
It seems to us that the normal reaction of a husband who righteously feels outraged by the abortion which his wife has
deliberately sought at the hands of a physician would be highminded rather than mercenary; and that his primary concern
would be to see to it that the medical profession was purged of an unworthy member rather than turn his wife's
indiscretion to personal profit, and with that idea in mind to press either the administrative or the criminal cases he had
filed, or both, instead of abandoning them in favor of a civil action for damages of which not only he, but also his wife,
would be the beneficiaries.
It is unquestionable that the appellant's act in provoking the abortion of appellee's wife, without medical necessity to warrant it,
was a criminal and morally reprehensible act, that can not be too severely condemned; and the consent of the woman or that of
her husband does not excuse it. But the immorality or illegality of the act does not justify an award of damage that, under the
circumstances on record, have no factual or legal basis.
The decision appealed from is reversed, and the complaint ordered dismissed. Without costs.
Let a copy of this decision be furnished to the Department of Justice and the Board of Medical Examiners for their information
and such investigation and action against the appellee Antonio Geluz as the facts may warrant.

G.R. No. L-39110

November 28, 1933

ANTONIA L. DE JESUS, ET AL., plaintiff-appellant,


vs.
CESAR SYQUIA, defendant-appellant.
Jose Sotelo for plaintiffs-appellants.
Vicente J. Francisco for defendant-appellant.

STREET, J.:
This action was instituted in the Court of First Instance of Manila by Antonia Loanco de Jesus in her own right and by her
mother, Pilar Marquez, as next friend and representative of Ismael and Pacita Loanco, infants, children of the first-named
plaintiff, for the purpose of recovering from the defendant, Cesar Syquia, the sum of thirty thousand pesos as damages resulting
to the first-named plaintiff from breach of a marriage promise, to compel the defendant to recognize Ismael and Pacita as
natural children begotten by him with Antonia, and to pay for the maintenance of the three the amount of five hundred pesos per
month, together with costs. Upon hearing the cause, after answer of the defendant, the trial court erred a decree requiring the
defendant to recognize Ismael Loanco as his natural child and to pay maintenance for him at the rate of fifty pesos per month,
with costs, dismissing the action in other respects. From this judgment both parties appealed, the plaintiffs from so much of the
decision as denied part of the relief sought by them, and the defendant from that feature of the decision which required him to
recognize Ismael Loanco and to pay for his maintenance.
At the time with which we are here concerned, the defendant, Cesar Syquia was of the age of twenty-three years, and an
unmarried scion of the prominent family in Manila, being possessed of a considerable property in his own right. His brother-inlaw, Vicente Mendoza is the owner of a barber shop in Tondo, where the defendant was accustomed to go for tonsorial
attention. In the month of June Antonia Loanco, a likely unmarried girl of the age of twenty years, was taken on as cashier in this
barber shop. Syquia was not long in making her acquaintance and amorous relations resulted, as a consequence of which
Antonia was gotten with child and a baby boy was born on June 17, 1931. The defendant was a constant visitor at the home of
Antonia in the early months of her pregnancy, and in February, 1931, he wrote and placed in her hands a note directed to
the padre who has expected to christen the baby. This note was as follows:
Saturday, 1:30 p. m.
February 14, 1931
Rev. FATHER,
The baby due in June is mine and I should like for my name to be given to it.
CESAR SYQUIA
The occasion for writing this note was that the defendant was on the eve of his departure on a trip to China and Japan; and
while he was abroad on this visit he wrote several letters to Antonia showing a paternal interest in the situation that had
developed with her, and cautioning her to keep in good condition in order that "junior" (meaning the baby to be, "Syquia, Jr.")
might be strong, and promising to return to them soon. The baby arrived at the time expected, and all necessary anticipatory
preparations were made by the defendant. To this he employed his friend Dr. Crescenciano Talavera to attend at the birth, and
made arrangements for the hospitalization of the mother in Saint Joseph's Hospital of the City of Manila, where she was cared
for during confinement.
When Antonio was able to leave the hospital, Syquia took her, with her mother and the baby, to a house at No. 551 Camarines
Street, Manila, where they lived together for about a year in regular family style, all household expenses, including gas and
electric light, being defrayed by Syquia. In course of time, however, the defendant's ardor abated and, when Antonia began to
show signs of a second pregnancy the defendant decamped, and he is now married to another woman. A point that should here
be noted is that when the time came for christening the child, the defendant, who had charge of the arrangement for this
ceremony, caused the name Ismael Loanco to be given to him, instead of Cesar Syquia, Jr., as was at first planned.
The first question that is presented in the case is whether the note to the padre, quoted above, in connection with the letters
written by the defendant to the mother during pregnancy, proves an acknowledgment of paternity, within the meaning of
subsection 1 of article 135 of the Civil Code. Upon this point we have no hesitancy in holding that the acknowledgment thus
shown is sufficient. It is a universal rule of jurisprudence that a child, upon being conceived, becomes a bearer of legal rights
and capable of being dealt with as a living person. The fact that it is yet unborn is no impediment to the acquisition of rights. The
problem here presented of the recognition of unborn child is really not different from that presented in the ordinary case of the
recognition of a child already born and bearing a specific name. Only the means and resources of identification are different.
Even a bequest to a living child requires oral evidence to connect the particular individual intended with the name used.
It is contended however, in the present case that the words of description used in the writings before us are not legally sufficient
to indemnify the child now suing as Ismael Loanco. This contention is not, in our opinion, well founded. The words of recognition
contained in the note to the padre are not capable of two constructions. They refer to a baby then conceived which was
expected to be born in June and which would thereafter be presented for christening. The baby came, and though it was in the
end given the name of Ismael Loanco instead of Cesar Syquia, Jr., its identity as the child which the defendant intended to
acknowledge is clear. Any doubt that might arise on this point is removed by the letters Exhibit F, G, H, and J. In these letters
the defendant makes repeated reference to junior as the baby which Antonia, to whom the letters were addressed, was then
carrying in her womb, and the writer urged Antonia to eat with good appetite in order that junior might be vigorous. In the last
letter (Exhibit J) written only a few days before the birth of the child, the defendant urged her to take good care of herself and
of junior also.
It seems to us that the only legal question that can here arise as to the sufficiency of acknowledgment is whether the
acknowledgment contemplated in subsection 1 of article 135 of the Civil Code must be made in a single document or may be
made in more than one document, of indubitable authenticity, written by the recognizing father. Upon this point we are of the
opinion that the recognition can be made out by putting together the admissions of more than one document, supplementing the
admission made in one letter by an admission or admissions made in another. In the case before us the admission of paternity
is contained in the note to the padreand the other letters suffice to connect that admission with the child then being carried by
Antonia L. de Jesus. There is no requirement in the law that the writing shall be addressed to one, or any particular individual. It
is merely required that the writing shall be indubitable.
The second question that presents itself in this case is whether the trial court erred in holding that Ismael Loanco had been in
the uninterrupted possession of the status of a natural child, justified by the conduct of the father himself, and that as a
consequence, the defendant in this case should be compelled to acknowledge the said Ismael Loanco, under No. 2 of article
135 of the Civil Code. The facts already stated are sufficient, in our opinion, to justify the conclusion of the trial court on this
point, and we may add here that our conclusion upon the first branch of the case that the defendant had acknowledged this
child in writings above referred to must be taken in connection with the facts found by the court upon the second point. It is
undeniable that from the birth of this child the defendant supplied a home for it and the mother, in which they lived together with
the defendant. This situation continued for about a year, and until Antonia became enciente a second time, when the idea
entered the defendant's head of abandoning her. The law fixes no period during which a child must be in the continuous
possession of the status of a natural child; and the period in this case was long enough to evince the father's resolution to

concede the status. The circumstance that he abandoned the mother and child shortly before this action was started is
unimportant. The word "continuous" in subsection 2 of article 135 of the Civil Code does not mean that the concession of status
shall continue forever, but only that it shall not be of an intermittent character while it continues.
What has been said disposes of the principal feature of the defendant's appeal. With respect to the appeal of the plaintiffs, we
are of the opinion that the trial court was right in refusing to give damages to the plaintiff, Antonia Loanco, for supposed breach
of promise to marry. Such promise is not satisfactorily proved, and we may add that the action for breach of promise to marry
has no standing in the civil law, apart from the right to recover money or property advanced by the plaintiff upon the faith of such
promise. This case exhibits none of the features necessary to maintain such an action. Furthermore, there is no proof upon
which a judgment could be based requiring the defendant to recognize the second baby, Pacita Loanco.
Finally, we see no necessity or propriety in modifying the judgment as to the amount of the maintenance which the trial court
allowed to Ismael Loanco. And in this connection we merely point out that, as conditions change, the Court of First Instance will
have jurisdiction to modify the order as to the amount of the pension as circumstances will require.
The judgment appealed from is in all respects affirmed, without costs. So ordered.

CONTINENTAL
STEEL
MANUFACTURING
CORPORATION,
Petitioner,
- versus -

G.R. No. 182836


Present:
CARPIO, J.,
Chairperson,
CHICO-NAZARIO,
VELASCO, JR.,
NACHURA, and
PERALTA, JJ.

HON.
ACCREDITED
VOLUNTARY
ARBITRATOR
ALLAN S. MONTAO and
NAGKAKAISANG
MANGGAGAWA NG CENTRO
STEEL
CORPORATIONSOLIDARITY OF UNIONS IN
Promulgated:
THE
PHILIPPINES
FOR
EMPOWERMENT
AND
REFORMS (NMCSC-SUPER),
Respondents.
October 13, 2009
x- - - - - - - - - - - - - - - - - - - - - - - - - - - - - - - - - - - - - - - - - - - - - - - - - - -x

DECISION

CHICO-NAZARIO, J.:

Before Us is a Petition for Review on Certiorari, under Rule 45 of the Rules of Court, assailing the Decision [1] dated 27
February 2008 and the Resolution [2] dated 9 May 2008 of the Court of Appeals in CA-G.R. SP No. 101697, affirming the
Resolution[3] dated 20 November 2007 of respondent Accredited Voluntary Arbitrator Atty. Allan S. Montao (Montao) granting
bereavement leave and other death benefits to Rolando P. Hortillano (Hortillano), grounded on the death of his unborn child.
The antecedent facts of the case are as follows:
Hortillano, an employee of petitioner Continental Steel Manufacturing Corporation (Continental Steel) and a member of
respondent Nagkakaisang Manggagawa ng Centro Steel Corporation-Solidarity of Trade Unions in the Philippines for
Empowerment and Reforms (Union) filed on 9 January 2006, a claim for Paternity Leave, Bereavement Leave and Death and
Accident Insurance for dependent, pursuant to the Collective Bargaining Agreement (CBA) concluded between Continental and
the Union, which reads:
ARTICLE X: LEAVE OF ABSENCE
xxxx
Section 2. BEREAVEMENT LEAVEThe Company agrees to grant a bereavement leave with pay to any
employee in case of death of the employees legitimate dependent (parents, spouse, children, brothers and sisters)
based on the following:
2.1 Within Metro Manila up to Marilao, Bulacan - 7 days
2.2 Provincial/Outside Metro Manila - 11 days
xxxx
ARTICLE XVIII: OTHER BENEFITS
xxxx
Section 4. DEATH AND ACCIDENT INSURANCEThe Company shall grant death and accidental insurance
to the employee or his family in the following manner:
xxxx
4.3 DEPENDENTSEleven Thousand Five Hundred Fifty Pesos (Php11,550.00) in case of death of the
employees legitimate dependents (parents, spouse, and children). In case the employee is single, this benefit
covers the legitimate parents, brothers and sisters only with proper legal document to be presented ( e.g. death
certificate).[4]

The claim was based on the death of Hortillanos unborn child. Hortillanos wife, Marife V. Hortillano, had a premature
delivery on 5 January 2006 while she was in the 38th week of pregnancy.[5] According to the Certificate of Fetal Death dated 7
January 2006, the female fetus died during labor due to fetal Anoxia secondary to uteroplacental insufficiency. [6]
Continental Steel immediately granted Hortillanos claim for paternity leave but denied his claims for bereavement leave
and other death benefits, consisting of the death and accident insurance. [7]
Seeking the reversal of the denial by Continental Steel of Hortillanos claims for bereavement and other death benefits,
theUnion resorted to the grievance machinery provided in the CBA. Despite the series of conferences held, the parties still
failed to settle their dispute,[8] prompting the Union to file a Notice to Arbitrate before the National Conciliation and Mediation
Board (NCMB) of the Department of Labor and Employment (DOLE), National Capital Region (NCR). [9] In a Submission
Agreement dated 9 October 2006, the Union and Continental Steel submitted for voluntary arbitration the sole issue of whether
Hortillano was entitled to bereavement leave and other death benefits pursuant to Article X, Section 2

and Article XVIII, Section 4.3 of the CBA. [10] The parties mutually chose Atty. Montao, an Accredited Voluntary Arbitrator, to
resolve said issue.[11]
When the preliminary conferences again proved futile in amicably settling the dispute, the parties proceeded to submit
their respective Position Papers, [12] Replies,[13] and Rejoinders[14] to Atty. Montao.
The Union argued that Hortillano was entitled to bereavement leave and other death benefits pursuant to the CBA. The
Union maintained that Article X, Section 2 and Article XVIII, Section 4.3 of the CBA did not specifically state that
the dependent should have first been born alive or must have acquired juridical personality so that his/her subsequent death
could be covered by the CBA death benefits. The Union cited cases wherein employees of MKK Steel Corporation (MKK Steel)
and Mayer Steel Pipe Corporation (Mayer Steel), sister companies of Continental Steel, in similar situations as Hortillano were
able to receive death benefits under similar provisions of their CBAs.
The Union mentioned in particular the case of Steve L. Dugan (Dugan), an employee of Mayer Steel, whose wife also
prematurely delivered a fetus, which had already died prior to the delivery. Dugan was able to receive paternity leave,
bereavement leave, and voluntary contribution under the CBA between his union and Mayer Steel. [15] Dugans child was only 24
weeks in the womb and died before labor, as opposed to Hortillanos child who was already 37-38 weeks in the womb and only
died during labor.
The Union called attention to the fact that MKK Steel and Mayer Steel are located in the same compound as Continental
Steel; and the representatives of MKK Steel and Mayer Steel who signed the CBA with their respective employees unions were
the same as the representatives of Continental Steel who signed the existing CBA with the Union.
Finally, the Union invoked Article 1702 of the Civil Code, which provides that all doubts in labor legislations and labor
contracts shall be construed in favor of the safety of and decent living for the laborer.
On the other hand, Continental Steel posited that the express provision of the CBA did not contemplate the death of an
unborn child, a fetus, without legal personality. It claimed that there are two elements for the entitlement to the benefits, namely:
(1) deathand (2) status as legitimate dependent, none of which existed in Hortillanos case. Continental Steel, relying on
Articles 40, 41 and 42[16] of the Civil Code, contended that only one with civil personality could die. Hence, the unborn child
never died because it never acquired juridical personality. Proceeding from the same line of thought, Continental Steel
reasoned that a fetus that was dead from the moment of delivery was not a person at all. Hence, the term dependent could not
be applied to a fetus that never acquired juridical personality. A fetus that was delivered dead could not be considered
a dependent, since it never needed any support, nor did it ever acquire the right to be supported.
Continental Steel maintained that the wording of the CBA was clear and unambiguous. Since neither of the parties
qualified the terms used in the CBA, the legally accepted definitions thereof were deemed automatically accepted by both
parties. The failure of the Union to have unborn child included in the definition of dependent, as used in the CBA the death of
whom would have qualified the parent-employee for bereavement leave and other death benefits bound the Union to the
legally accepted definition of the latter term.
Continental Steel, lastly, averred that similar cases involving the employees of its sister companies, MKK Steel and Mayer
Steel, referred to by the Union, were irrelevant and incompetent evidence, given the separate and distinct personalities of the
companies. Neither could the Union sustain its claim that the grant of bereavement leave and other death benefits to the
parent-employee for the loss of an unborn child constituted company practice.
On 20 November 2007, Atty. Montao, the appointed Accredited Voluntary Arbitrator, issued a Resolution [17] ruling that
Hortillano was entitled to bereavement leave with pay and death benefits.
Atty. Montao identified the elements for entitlement to said benefits, thus:

This Office declares that for the entitlement of the benefit of bereavement leave with pay by the covered employees
as provided under Article X, Section 2 of the parties CBA, three (3) indispensable elements must be present: (1)
there is death; (2) such death must be of employees dependent; and (3) such dependent must be legitimate.
On the otherhand, for the entitlement to benefit for death and accident insurance as provided under Article
XVIII, Section 4, paragraph (4.3) of the parties CBA, four (4) indispensable elements must be present: (a) there is
death; (b) such death must be of employees dependent; (c) such dependent must be legitimate; and (d) proper
legal document to be presented.[18]

Atty. Montao found that there was no dispute that the death of an employees legitimate dependent occurred. The fetus
had the right to be supported by the parents from the very moment he/she was conceived. The fetus had to rely on another for
support; he/she could not have existed or sustained himself/herself without the power or aid of someone else, specifically,
his/her mother. Therefore, the fetus was already a dependent, although he/she died during the labor or delivery. There was also
no question that Hortillano and his wife were lawfully married, making their dependent, unborn child, legitimate.
In the end, Atty. Montao decreed:
WHEREFORE, premises considered, a resolution is hereby rendered ORDERING [herein petitioner
Continental Steel] to pay Rolando P. Hortillano the amount of Four Thousand Nine Hundred Thirty-Nine Pesos
(P4,939.00), representing his bereavement leave pay and the amount of Eleven Thousand Five Hundred Fifty Pesos
(P11,550.00) representing death benefits, or a total amount of P16,489.00
The complaint against Manuel Sy, however, is ORDERED DISMISSED for lack of merit.
All other claims are DISMISSED for lack of merit.

Further, parties are hereby ORDERED to faithfully abide with the herein dispositions.

Aggrieved, Continental Steel filed with the Court of Appeals a Petition for Review on Certiorari,[19] under Section 1, Rule 43
of the Rules of Court, docketed as CA-G.R. SP No. 101697.
Continental Steel claimed that Atty. Montao erred in granting Hortillanos claims for bereavement leave with pay and
other death benefits because no death of an employees dependent had occurred. The death of a fetus, at whatever stage of
pregnancy, was excluded from the coverage of the CBA since what was contemplated by the CBA was the death of a legal
person, and not that of a fetus, which did not acquire any juridical personality. Continental Steel pointed out that its contention
was bolstered by the fact that the term death was qualified by the phrase legitimate dependent. It asserted that the status of a
child could only be determined upon said childs birth, otherwise, no such appellation can be had. Hence, the conditions sine
qua non for Hortillanos entitlement to bereavement leave and other death benefits under the CBA were lacking.
The Court of Appeals, in its Decision dated 27 February 2008, affirmed Atty. Montaos Resolution dated 20 November
2007. The appellate court interpreted death to mean as follows:

[Herein petitioner Continental Steels] exposition on the legal sense in which the term death is used in the
CBA fails to impress the Court, and the same is irrelevant for ascertaining the purpose, which the grant of
bereavement leave and death benefits thereunder, is intended to serve. While there is no arguing with [Continental
Steel] that the acquisition of civil personality of a child or fetus is conditioned on being born alive upon delivery, it
does not follow that such event of premature delivery of a fetus could never be contemplated as a death as to be
covered by the CBA provision, undoubtedly an event causing loss and grief to the affected employee, with whom the
dead fetus stands in a legitimate relation. [Continental Steel] has proposed a narrow and technical significance to
the term death of a legitimate dependent as condition for granting bereavement leave and death benefits under the
CBA. Following [Continental Steels] theory, there can be no experience of death to speak of. The Court, however,
does not share this view. A dead fetus simply cannot be equated with anything less than loss of human life,
especially for the expectant parents. In this light, bereavement leave and death benefits are meant to assuage the
employee and the latters immediate family, extend to them solace and support, rather than an act conferring legal
status or personality upon the unborn child. [Continental Steels] insistence that the certificate of fetal death is for
statistical purposes only sadly misses this crucial point. [20]

Accordingly, the fallo of the 27 February 2008 Decision of the Court of Appeals reads:
WHEREFORE, premises considered, the present petition is hereby DENIED for lack of merit. The assailed
Resolution dated November 20, 2007 of Accredited Voluntary Arbitrator Atty. Allan S. Montao is hereby AFFIRMED
and UPHELD.
With costs against [herein petitioner Continental Steel]. [21]

In a Resolution[22] dated 9 May 2008, the Court of Appeals denied the Motion for Reconsideration [23] of Continental Steel.
Hence, this Petition, in which Continental Steel persistently argues that the CBA is clear and unambiguous, so that the
literal and legal meaning of death should be applied. Only one with juridical personality can die and a dead fetus never
acquired a juridical personality.
We are not persuaded.
As Atty. Montao identified, the elements for bereavement leave under Article X, Section 2 of the CBA are: (1) death; (2)
the death must be of a dependent, i.e., parent, spouse, child, brother, or sister, of an employee; and (3) legitimate relations of
the dependent to the employee. The requisites for death and accident insurance under Article XVIII, Section 4(3) of the CBA
are: (1) death; (2) the death must be of a dependent, who could be a parent, spouse, or child of a married employee; or a
parent, brother, or sister of a single employee; and (4) presentation of the proper legal document to prove such death, e.g.,
death certificate.
It is worthy to note that despite the repeated assertion of Continental Steel that the provisions of the CBA are clear and
unambiguous, its fundamental argument for denying Hortillanos claim for bereavement leave and other death benefits rests on
the purportedly proper interpretation of the terms death and dependent as used in the CBA. If the provisions of the CBA are
indeed clear and unambiguous, then there is no need to resort to the interpretation or construction of the same. Moreover,
Continental Steel itself admitted that neither management nor the Union sought to define the pertinent terms for bereavement
leave and other death benefits during the negotiation of the CBA.
The reliance of Continental Steel on Articles 40, 41 and 42 of the Civil Code for the legal definition of death is
misplaced. Article 40 provides that a conceived child acquires personality only when it is born, and Article 41 defines when a
child is considered born. Article 42 plainly states that civil personality is extinguished by death.
First, the issue of civil personality is not relevant herein. Articles 40, 41 and 42 of the Civil Code on natural persons, must
be applied in relation to Article 37 of the same Code, the very first of the general provisions on civil personality, which reads:
Art. 37. Juridical capacity, which is the fitness to be the subject of legal relations, is inherent in every natural
person and is lost only through death. Capacity to act, which is the power to do acts with legal effect, is acquired
and may be lost.

We need not establish civil personality of the unborn child herein since his/her juridical capacity and capacity to act as a person
are not in issue. It is not a question before us whether the unborn child acquired any rights or incurred any obligations prior to
his/her death that were passed on to or assumed by the childs parents. The rights to bereavement leave and other death
benefits in the instant case pertain directly to the parents of the unborn child upon the latters death.

Second, Sections 40, 41 and 42 of the Civil Code do not provide at all a definition of death. Moreover, while the Civil
Code expressly provides that civil personality may be extinguished by death, it does not explicitly state that only those who have
acquired juridical personality could die.
And third, death has been defined as the cessation of life. [24] Life is not synonymous with civil personality. One need not
acquire civil personality first before he/she could die. Even a child inside the womb already has life. No less than the
Constitution recognizes the life of the unborn from conception,[25] that the State must protect equally with the life of the
mother. If the unborn already has life, then the cessation thereof even prior to the child being delivered, qualifies as death.
Likewise, the unborn child can be considered a dependent under the CBA. As Continental Steel itself defines,
a dependent is one who relies on another for support; one not able to exist or sustain oneself without the power or aid of
someone else. Under said general definition,[26] even an unborn child is a dependent of its parents. Hortillanos child could not
have reached 38-39 weeks of its gestational life without depending upon its mother, Hortillanos wife, for
sustenance. Additionally, it is explicit in the CBA provisions in question that the dependent may be the parent, spouse,
or child of a married employee; or the parent, brother, or sister of a single employee. The CBA did not provide a qualification for
the child dependent, such that the child must have been born or must have acquired civil personality, as Continental Steel
avers. Without such qualification, then child shall be understood in its more general sense, which includes the unborn fetus in
the mothers womb.

[27]

The term legitimate merely addresses the dependent childs status in relation to his/her parents. In Angeles v. Maglaya,
we have expounded on who is a legitimate child, viz:
A legitimate child is a product of, and, therefore, implies a valid and lawful marriage. Remove the element of lawful
union and there is strictly no legitimate filiation between parents and child. Article 164 of the Family Code cannot be
more emphatic on the matter: Children conceived or born during the marriage of the parents are legitimate.
(Emphasis ours.)

Conversely, in Briones v. Miguel,[28] we identified an illegitimate child to be as follows:


The fine distinctions among the various types of illegitimate children have been eliminated in the Family Code.
Now, there are only two classes of children -- legitimate (and those who, like the legally adopted, have the rights of
legitimate children) and illegitimate. All children conceivedand born outside a valid marriage are illegitimate, unless
the law itself gives them legitimate status. (Emphasis ours.)

It is apparent that according to the Family Code and the afore-cited jurisprudence, the legitimacy or illegitimacy of a child
attaches upon his/her conception. In the present case, it was not disputed that Hortillano and his wife were validly married and
that their child was conceived during said marriage, hence, making said child legitimate upon her conception.
Also incontestable is the fact that Hortillano was able to comply with the fourth element entitling him to death and accident
insurance under the CBA, i.e., presentation of the death certificate of his unborn child.
Given the existence of all the requisites for bereavement leave and other death benefits under the CBA, Hortillanos
claims for the same should have been granted by Continental Steel.
We emphasize that bereavement leave and other death benefits are granted to an employee to give aid to, and if
possible, lessen the grief of, the said employee and his family who suffered the loss of a loved one. It cannot be said that the
parents grief and sense of loss arising from the death of their unborn child, who, in this case, had a gestational life of 38-39
weeks but died during delivery, is any less than that of parents whose child was born alive but died subsequently.
Being for the benefit of the employee, CBA provisions on bereavement leave and other death benefits should be
interpreted liberally to give life to the intentions thereof. Time and again, the Labor Code is specific in enunciating that in case
of doubt in the interpretation of any law or provision affecting labor, such should be interpreted in favor of labor. [29] In the same
way, the CBA and CBA provisions should be interpreted in favor of labor. In Marcopper Mining v. National Labor Relations
Commission,[30]we pronounced:
Finally, petitioner misinterprets the declaration of the Labor Arbiter in the assailed decision that "when the
pendulum of judgment swings to and fro and the forces are equal on both sides, the same must be stilled in favor of
labor." While petitioner acknowledges that all doubts in the interpretation of the Labor Code shall be resolved in
favor of labor, it insists that what is involved-here is the amended CBA which is essentially a contract between
private persons. What petitioner has lost sight of is the avowed policy of the State, enshrined in our Constitution, to
accord utmost protection and justice to labor, a policy, we are, likewise, sworn to uphold.
In Philippine Telegraph & Telephone Corporation v. NLRC [183 SCRA 451 (1990)], we categorically stated
that:
When conflicting interests of labor and capital are to be weighed on the scales of social justice, the
heavier influence of the latter should be counter-balanced by sympathy and compassion the law must
accord the underprivileged worker.
Likewise, in Terminal Facilities and Services Corporation v. NLRC [199 SCRA 265 (1991)], we declared:
Any doubt concerning the rights of labor should be resolved in its favor pursuant to the social
justice policy.

IN VIEW WHEREOF, the Petition is DENIED. The Decision dated 27 February 2008 and Resolution dated 9 May 2008 of
the Court of Appeals in CA-G.R. SP No. 101697, affirming the Resolution dated 20 November 2007 of Accredited Voluntary

Arbitrator Atty. Allan S. Montao, which granted to Rolando P. Hortillano bereavement leave pay and other death benefits in the
amounts of Four Thousand Nine Hundred Thirty-Nine Pesos (P4,939.00) and Eleven Thousand Five Hundred Fifty Pesos
(P11,550.00), respectively, grounded on the death of his unborn child, are AFFIRMED. Costs against Continental Steel
Manufacturing Corporation.
SO ORDERED.

G.R. No. L-770

April 27, 1948

ANGEL T. LIMJOCO, petitioner,


vs.
INTESTATE ESTATE OF PEDRO O. FRAGRANTE, deceased, respondent.
Angel Limjoco, Jr. and Delfin L. Gonzales for petitioner.
Bienvenido A. Tan for respondent.
HILADO, J.:
Under date of May 21, 1946, the Public Service Commission, through Deputy Commissioner Fidel Ibaez, rendered its decision
in case No. 4572 of Pedro O. Fragante, as applicant for a certificate of public convenience to install, maintain and operate an
ice plant in San Juan, Rizal, whereby said commission held that the evidence therein showed that the public interest and
convenience will be promoted in a proper and suitable manner "by authorizing the operation and maintenance of another ice
plant of two and one-half (2-) tons in the municipality of San Juan; that the original applicant Pedro O. Fragante was a Filipino
Citizen at the time of his death; and that his intestate estate is financially capable of maintaining the proposed service". The
commission, therefore, overruled the opposition filed in the case and ordered "that under the provisions of section 15 of
Commonwealth Act No. 146, as amended a certificate of public convenience be issued to the Intestate Estate of the deceased
Pedro Fragante, authorizing said Intestate Estate through its Special or Judicial Administrator, appointed by the proper court of
competent jurisdiction, to maintain and operate an ice plant with a daily productive capacity of two and one-half (2-1/2) tons in
the Municipality of San Juan and to sell the ice produced from said plant in the said Municipality of San Juan and in the
Municipality of Mandaluyong, Rizal, and in Quezon City", subject to the conditions therein set forth in detail (petitioner's brief,
pp. 33-34).
Petitioner makes four assignments of error in his brief as follows:
1. The decision of the Public Service Commission is not in accordance with law.
2. The decision of the Public Service Commission is not reasonably supported by evidence.
3. The Public Service Commission erred in not giving petitioner and the Ice and Cold Storage Industries of the Philippines,
Inc., as existing operators, a reasonable opportunity to meet the increased demand.
4. The decision of the Public Service Commission is an unwarranted departure from its announced policy with respect to
the establishment and operation of ice plant. (Pp. 1-2, petitioner's brief.)
In his argument petitioner contends that it was error on the part of the commission to allow the substitution of the legal
representative of the estate of Pedro O. Fragante for the latter as party applicant in the case then pending before the
commission, and in subsequently granting to said estate the certificate applied for, which is said to be in contravention of law.
If Pedro O. Fragante had not died, there can be no question that he would have had the right to prosecute his application before
the commission to its final conclusion. No one would have denied him that right. As declared by the commission in its decision,
he had invested in the ice plant in question P 35,000, and from what the commission said regarding his other properties and
business, he would certainly have been financially able to maintain and operate said plant had he not died. His transportation
business alone was netting him about P1,440 a month. He was a Filipino citizen and continued to be such till his demise. The
commission declared in its decision, in view of the evidence before it, that his estate was financially able to maintain and
operate the ice plant. The aforesaid right of Pedro O. Fragante to prosecute said application to its conclusion was one which by
its nature did not lapse through his death. Hence, it constitutes a part of the assets of his estate, for which a right was property
despite the possibility that in the end the commission might have denied application, although under the facts of the case, the
commission granted the application in view of the financial ability of the estate to maintain and operate the ice plant. Petitioner,
in his memorandum of March 19, 1947, admits (page 3) that the certificate of public convenience once granted "as a rule,
should descend to his estate as an asset". Such certificate would certainly be property, and the right to acquire such a
certificate, by complying with the requisites of the law, belonged to the decedent in his lifetime, and survived to his estate and
judicial administrator after his death.
If Pedro O. Fragrante had in his lifetime secured an option to buy a piece of land and during the life of the option he died, if the
option had been given him in the ordinary course of business and not out of special consideration for his person, there would be
no doubt that said option and the right to exercise it would have survived to his estate and legal representatives. In such a case
there would also be the possibility of failure to acquire the property should he or his estate or legal representative fail to comply
with the conditions of the option. In the case at bar Pedro O. Fragrante's undoubted right to apply for and acquire the desired
certificate of public convenience the evidence established that the public needed the ice plant was under the law
conditioned only upon the requisite citizenship and economic ability to maintain and operate the service. Of course, such right to
acquire or obtain such certificate of public convenience was subject to failure to secure its objective through nonfulfillment of the
legal conditions, but the situation here is no different from the legal standpoint from that of the option in the illustration just given.
Rule 88, section 2, provides that the executor or administrator may bring or defend actions, among other cases, for the
protection of the property or rights of the deceased which survive, and it says that such actions may be brought or defended "in
the right of the deceased".
Rule 82, section 1, paragraph (a), mentions among the duties of the executor or administrator, the making of an inventory of all
goods, chattels, rights, credits, and estate of the deceased which shall come to his possession or knowledge, or to the
possession of any other person for him.
In his commentaries on the Rules of Court (Volume II, 2nd ed., pages 366, 367) the present chief Justice of this Court draws the
following conclusion from the decisions cited by him:
Therefore, unless otherwise expressly provided by law, any action affecting the property or rights (emphasis supplied) of a
deceased person which may be brought by or against him if he were alive, may likewise be instituted and prosecuted by
or against the administrator, unless the action is for recovery of money, debt or interest thereon, or unless, by its very
nature, it cannot survive, because death extinguishes the right . . . .
It is true that a proceeding upon the application for a certificate of public convenience before the Public Service Commission is
not an "action". But the foregoing provisions and citations go to prove that the decedent's rights which by their nature are not
extinguished by death go to make up a part and parcel of the assets of his estate which, being placed under the control and
management of the executor or administrator, can not be exercised but by him in representation of the estate for the benefit of
the creditors, devisees or legatees, if any, and the heirs of the decedent. And if the right involved happens to consist in the
prosecution of an unfinished proceeding upon an application for a certificate of public convenience of the deceased before the
Public Service Commission, it is but logical that the legal representative be empowered and entitled in behalf of the estate to
make the right effective in that proceeding.
Manresa (Vol. III, 6th ed., p. 11) says that No. 10 of article 334 and article 336 of the Civil Code, respectively, consider
as immovable and movable things rights which are not material. The same eminent commentator says in the cited volume (p.
45) that article 336 of the Civil Code has been deficiently drafted in that it is not sufficiently expressive of all incorporeal rights
which are also property for juridical purposes.

Corpus Juris (Vol. 50, p. 737) states that in the broad sense of the term, property includes, among other things, "an option", and
"the certificate of the railroad commission permitting the operation of a bus line", and on page 748 of the same volume we read:
However, these terms (real property, as estate or interest) have also been declared to include every species of
title, inchoate or complete, and embrace rights which lie in contract, whether executory or executed. (Emphasis supplied.)
Another important question raised by petitioner is whether the estate of Pedro O. Fragrante is a "person" within the meaning of
the Public Service Act.
Words and Phrases, First Series, (Vol. 6, p, 5325), states the following doctrine in the jurisdiction of the State of Indiana:
As the estate of the decedent is in law regarded as a person, a forgery committed after the death of the man whose name
purports to be signed to the instrument may be prosecuted as with the intent to defraud the estate. Billings vs. State, 107
Ind., 54, 55, 6 N. E. 914, 7 N. E. 763, 57 Am. Rep. 77.
The Supreme Court of Indiana in the decision cited above had before it a case of forgery committed after the death of one
Morgan for the purpose of defrauding his estate. The objection was urged that the information did not aver that the forgery was
committed with the intent to defraud any person. The Court, per Elliott, J., disposed of this objection as follows:
. . . The reason advanced in support of this proposition is that the law does not regard the estate of a decedent as a
person. This intention (contention) cannot prevail. The estate of the decedent is a person in legal contemplation. "The
word "person" says Mr. Abbot, "in its legal signification, is a generic term, and includes artificial as well as natural
persons," 2 Abb. Dict. 271; Douglas vs. Pacific, etc. Co., 4 Cal. 304; Planters', etc., Bank vs. Andrews, 8 Port. (Ala.) 404. It
said in another work that 'persons are of two kinds: natural and artificial. A natural person is a human being. Artificial
persons include (1) a collection or succession of natural persons forming a corporation; (2) a collection of property to
which the law attributes the capacity of having rights and duties. The latter class of artificial persons is recognized only to
a limited extent in our law. "Examples are the estate of a bankrupt or deceased person." 2 Rapalje & L. Law Dict. 954. Our
own cases inferentially recognize the correctness of the definition given by the authors from whom we have quoted, for
they declare that it is sufficient, in pleading a claim against a decedent's estate, to designate the defendant as the estate
of the deceased person, naming him. Ginn vs. Collins, 43 Ind. 271. Unless we accept this definition as correct, there
would be a failure of justice in cases where, as here, the forgery is committed after the death of a person whose name is
forged; and this is a result to be avoided if it can be done consistent with principle. We perceive no difficulty in avoiding
such a result; for, to our minds, it seems reasonable that the estate of a decedent should be regarded as an artificial
person. It is the creation of law for the purpose of enabling a disposition of the assets to be properly made, and, although
natural persons as heirs, devises, or creditors, have an interest in the property, the artificial creature is a distinct legal
entity. The interest which natural persons have in it is not complete until there has been a due administration; and one
who forges the name of the decedent to an instrument purporting to be a promissory note must be regarded as having
intended to defraud the estate of the decedent, and not the natural persons having diverse interests in it, since ha cannot
be presumed to have known who those persons were, or what was the nature of their respective interest. The fraudulent
intent is against the artificial person, the estate and not the natural persons who have direct or contingent interest in
it. (107 Ind. 54, 55, 6 N.E. 914-915.)
In the instant case there would also be a failure of justice unless the estate of Pedro O. Fragrante is considered a "person", for
quashing of the proceedings for no other reason than his death would entail prejudicial results to his investment amounting to
P35,000.00 as found by the commission, not counting the expenses and disbursements which the proceeding can be presumed
to have occasioned him during his lifetime, let alone those defrayed by the estate thereafter. In this jurisdiction there are ample
precedents to show that the estate of a deceased person is also considered as having legal personality independent of their
heirs. Among the most recent cases may be mentioned that of "Estate of Mota vs. Concepcion, 56 Phil., 712, 717, wherein the
principal plaintiff was the estate of the deceased Lazaro Mota, and this Court gave judgment in favor of said estate along with
the other plaintiffs in these words:
. . . the judgment appealed from must be affirmed so far as it holds that defendants Concepcion and Whitaker are
indebted to he plaintiffs in the amount of P245,804.69 . . . .
Under the regime of the Civil Code and before the enactment of the Code of Civil Procedure, the heirs of a deceased person
were considered in contemplation of law as the continuation of his personality by virtue of the provision of article 661 of the first
Code that the heirs succeed to all the rights and obligations of the decedent by the mere fact of his death. It was so held by this
Court in Barrios vs. Dolor, 2 Phil., 44, 46. However, after the enactment of the Code of Civil Procedure, article 661 of the Civil
Code was abrogated, as held in Suiliong & Co. vs. Chio-Taysan, 12 Phil., 13, 22. In that case, as well as in many others decided
by this Court after the innovations introduced by the Code of Civil Procedure in the matter of estates of deceased persons, it
has been the constant doctrine that it is the estate or the mass of property, rights and assets left by the decedent, instead of the
heirs directly, that becomes vested and charged with his rights and obligations which survive after his demise.
The heirs were formerly considered as the continuation of the decedent's personality simply by legal fiction, for they might not
have been flesh and blood the reason was one in the nature of a legal exigency derived from the principle that the heirs
succeeded to the rights and obligations of the decedent. Under the present legal system, such rights and obligations as survive
after death have to be exercised and fulfilled only by the estate of the deceased. And if the same legal fiction were not indulged,
there would be no juridical basis for the estate, represented by the executor or administrator, to exercise those rights and to
fulfill those obligations of the deceased. The reason and purpose for indulging the fiction is identical and the same in both
cases. This is why according to the Supreme Court of Indiana in Billings vs. State, supra, citing 2 Rapalje & L. Dictionary, 954,
among the artificial persons recognized by law figures "a collection of property to which the law attributes the capacity of having
rights and duties", as for instance, the estate of a bankrupt or deceased person.
Petitioner raises the decisive question of whether or not the estate of Pedro O. Fragrante can be considered a "citizen of the
Philippines" within the meaning of section 16 of the Public Service Act, as amended, particularly the proviso thereof expressly
and categorically limiting the power of the commission to issue certificates of public convenience or certificates of public
convenience and necessity "only to citizens of the Philippines or of the United States or to corporations, copartnerships,
associations, or joint-stock companies constituted and organized under the laws of the Philippines", and the further proviso that
sixty per centum of the stock or paid-up capital of such entities must belong entirely to citizens of the Philippines or of the United
States.
Within the Philosophy of the present legal system, the underlying reason for the legal fiction by which, for certain purposes, the
estate of the deceased person is considered a "person" is the avoidance of injustice or prejudice resulting from the impossibility
of exercising such legal rights and fulfilling such legal obligations of the decedent as survived after his death unless the fiction is
indulged. Substantially the same reason is assigned to support the same rule in the jurisdiction of the State of Indiana, as
announced in Billings vs. State, supra, when the Supreme Court of said State said:
. . . It seems reasonable that the estate of a decedent should be regarded as an artificial person. it is the creation of law
for the purpose of enabling a disposition of the assets to be properly made . . . .
Within the framework and principles of the constitution itself, to cite just one example, under the bill of rights it seems clear that
while the civil rights guaranteed therein in the majority of cases relate to natural persons, the term "person" used in section 1 (1)
and (2) must be deemed to include artificial or juridical persons, for otherwise these latter would be without the constitutional
guarantee against being deprived of property without due process of law, or the immunity from unreasonable searches and
seizures. We take it that it was the intendment of the framers to include artificial or juridical, no less than natural, persons in

these constitutional immunities and in others of similar nature. Among these artificial or juridical persons figure estates of
deceased persons. Hence, we hold that within the framework of the Constitution, the estate of Pedro O. Fragrante should be
considered an artificial or juridical person for the purposes of the settlement and distribution of his estate which, of course,
include the exercise during the judicial administration thereof of those rights and the fulfillment of those obligations of his which
survived after his death. One of those rights was the one involved in his pending application before the Public Service
Commission in the instant case, consisting in the prosecution of said application to its final conclusion. As stated above, an
injustice would ensue from the opposite course.
How about the point of citizenship? If by legal fiction his personality is considered extended so that any debts or obligations left
by, and surviving, him may be paid, and any surviving rights may be exercised for the benefit of his creditors and heirs,
respectively, we find no sound and cogent reason for denying the application of the same fiction to his citizenship, and for not
considering it as likewise extended for the purposes of the aforesaid unfinished proceeding before the Public Service
Commission. The outcome of said proceeding, if successful, would in the end inure to the benefit of the same creditors and the
heirs. Even in that event petitioner could not allege any prejudice in the legal sense, any more than he could have done if
Fragrante had lived longer and obtained the desired certificate. The fiction of such extension of his citizenship is grounded upon
the same principle, and motivated by the same reason, as the fiction of the extension of personality. The fiction is made
necessary to avoid the injustice of subjecting his estate, creditors and heirs, solely by reason of his death to the loss of the
investment amounting to P35,000, which he has already made in the ice plant, not counting the other expenses occasioned by
the instant proceeding, from the Public Service Commission of this Court.
We can perceive no valid reason for holding that within the intent of the constitution (Article IV), its provisions on Philippine
citizenship exclude the legal principle of extension above adverted to. If for reasons already stated our law indulges the fiction of
extension of personality, if for such reasons the estate of Pedro O. Fragrante should be considered an artificial or juridical
person herein, we can find no justification for refusing to declare a like fiction as to the extension of his citizenship for the
purposes of this proceeding.
Pedro O. Fragrante was a Filipino citizen, and as such, if he had lived, in view of the evidence of record, he would have
obtained from the commission the certificate for which he was applying. The situation has suffered but one change, and that is,
his death. His estate was that of a Filipino citizen. And its economic ability to appropriately and adequately operate and maintain
the service of an ice plant was the same that it received from the decedent himself. In the absence of a contrary showing, which
does not exist here, his heirs may be assumed to be also Filipino citizens; and if they are not, there is the simple expedient of
revoking the certificate or enjoining them from inheriting it.
Upon the whole, we are of the opinion that for the purposes of the prosecution of said case No. 4572 of the Public Service
Commission to its final conclusion, both the personality and citizenship of Pedro O. Fragrante must be deemed extended, within
the meaning and intent of the Public Service Act, as amended, in harmony with the constitution: it is so adjudged and decreed.
Decision affirmed, without costs. So ordered.

G.R. No. L-27956 April 30, 1976


DIONISIO DUMLAO, in his own behalf and in his capacity as Administrator of the Testate Estate of the late Pedro Oria;
FAUSTA DUMLAO, AMADO DUMLAO, and BENJAMIN DUMLAO, plaintiffs-appellants,
vs.
QUALITY PLASTIC PRODUCTS, INC., defendant-appellee.
Castillo & Castillo for appellants.
Eugenio T. Estavillo for appellee.

AQUINO, J.:p
On February 28, 1962 the Court of First Instance of Pangasinan in Civil Case No.
T-662 rendered a judgment ordering defendants Vicente Soliven, Pedro Oria, Santiago Laurencio, Marcelino Sumalbag and
Juana Darang to pay solidarity Quality Plastic Products, Inc. the sum of P3,667.03 plus the legal rate of interest from November,
1958. The lower court directed that in case the defendants failed to pay the said amount before its decision became final, then
Quality Plastic Products, Inc. "is hereby authorized to foreclose the bond, Exhibit A, in accordance with law, for the satisfaction
of the judgment". (Under that bond the four sureties bound themselves to answer solidarity for the obligations of the principal,
Vicente Soliven and certain real properties of the sureties were "given as security for" their undertaking).
Upon defendants' failure to pay the amount of the judgment and after the decision had become final, the lower court, on motion
of Quality Plastic Products, Inc., ordered the "foreclosure" of the surety bond and the sale at public auction of the land of Pedro
Oria which he had given as security under the bond. Oria's land, which was covered by Original Certificate of Title No. 28732
and has an area of nine and six-tenths hectares, was levied upon and sold by the sheriff at public auction on September 24,
1962. The sale was confirmed by the lower court in its order of November 20, 1962.
It turned out that Oria died on April 23, 1959 or long before June 13, 1960 when the action was filed. Oria's death was not
known to Quality Plastic Products, Inc. Nor were the representatives of Quality Plastic Products, Inc. aware that in the same
Tayug court Special Proceeding No. T-212, Testate Estate of the deceased Pedro Oria, was pending.
The summons and copies of the complaint for the five defendants in Civil Case No.
T-662 had been personally served on June 24, 1960 by a deputy sheriff on Soliven, the principal in the bond, who
acknowledged such service by signing on the back of the original summons in his own behalf and again signing for his codefendants.
On March 1, 1963 Dionisio, Fausta, Amado and Benjamin, all surnamed Dumlao and all testamentary heirs in Oria's duly
probated will, sued Quality Plastic Products, Inc., also in the Tayug court for the annulment of the judgment against Oria and the
execution against his land. (Dionisio Dumlao also sued in his capacity as administrator of Oria's testate estate).
The ground for annulment was lack of jurisdiction over the person of the deceased Oria (Civil Case No. T- 873). It was only
when Quality Plastic Products, Inc. received the summons in Civil Case No. T-873 that it learned that Oria was already dead at
the time the prior case, Civil Case No. T-662, was filed.
Quality Plastic Products, Inc. in its answer alleged that Oria's heirs were aware of the suit against Soliven and his sureties and
that the said heirs were estopped to question the court's jurisdiction over Oria.
After hearing the lower court held that it acquired jurisdiction over Soliven and the other defendants in Civil Case No. T-662 by
reason of their voluntary appearance. It reasoned out that Soliven acted in bad faith because he did not apprise the court that
Oria was dead. It specifically ruled that "it had acquired jurisdiction over the person" of Oria and that the judgment was valid as
to him. From that decision the plaintiffs appealed.
The four assignments of error of appellants Dumlao may be boiled down to the issue as to the validity of the lower court's
judgment against the deceased Pedro Oria who, being already in the other world, was never served with summons.
There is no difficulty in resolving that issue. Since no jurisdiction was acquired over Oria, the judgment against him is a patent
nullity (Ang Lam vs. Rosillosa and Santiago, 86 Phil. 447; Asuncion vs. Nieto, 4 Phil. 97; Gorostiaga vs. Sarte, 68 Phil. 4).
As far as Oria was concerned, the lower court's judgment against him in Civil Case No. T-662 is void for lack of jurisdiction over
his person. He was not, and he could not have been, validly served with summons. He had no more civil personality. His
juridical capacity, which is the fitness to be the subject of legal relations, was lost through death. (Arts. 37 and 42, Civil Code).
The lower court erred in ruling that since Soliven's counsel also appeared as counsel for Oria, there was a voluntary
appearance which enabled the court to acquire jurisdiction over Oria, as contemplated in section 23, Rule 14 of the Revised
Rules of Court. Soliven's counsel could not have validly appeared for a dead co-defendant. Estoppel has no application to this
case.
But from the fact that appellants Dumlao had to sue Quality Plastic Products, Inc. in order to annul the judgment against Oria, it
does not follow that they are entitled to claim attorney's fees against that corporation. The parties herein agreed in their
stipulation of facts that Quality Plastic Products, Inc. was unaware of Oria's death. Appellants Dumlao in effect conceded that
the appellee acted in good faith in joining Oria as a co-defendant.
WHEREFORE, the lower court's decision is reversed and set aside. Its judgment in Civil Case No. T-662 against Pedro Oria is
declared void for lack of jurisdiction. The execution sale of Oria's land covered by OCT No. 28732 is also void. No costs.
SO ORDERED.

Family Code
Requisite of Marriage
Article 2 of the Family Code specifies the essential requisites, as opposed to the formal requisites, of marriage. It reads:
Art. 2. No marriage shall be valid, unless these essential requisites are present:
(1) Legal capacity of the contracting parties who must be a male and a female; and
(2) Consent freely given in the presence of the solemnizing officer.
The parties, who must be a male and a female, must have legal capacity and must freely give their consent.
On the other hand, Article 3 of the Family Code specifies the formal requisites, as opposed to the essential requisites, of
marriage. It reads:
Art. 3. The formal requisites of marriage are:
(1) Authority of the solemnizing officer;
(2) A valid marriage license except in the cases provided for in Chapter 2 of this Title; and
(3) A marriage ceremony which takes place with the appearance of the contracting parties before the solemnizing officer and
their personal declaration that they take each other as husband and wife in the presence of not less than two witnesses of legal
age.

G.R. No. 118978 May 23, 1997


PHILIPPINE TELEGRAPH AND TELEPHONE COMPANY, * petitioner,
vs.
NATIONAL LABOR RELATIONS COMMISSION and GRACE DE GUZMAN, respondents.

REGALADO, J.:
Seeking relief through the extraordinary writ of certiorari, petitioner Philippine Telegraph and Telephone Company (hereafter, PT
& T) invokes the alleged concealment of civil status and defalcation of company funds as grounds to terminate the services of
an employee. That employee, herein private respondent Grace de Guzman, contrarily argues that what really motivated PT & T
to terminate her services was her having contracted marriage during her employment, which is prohibited by petitioner in its
company policies. She thus claims that she was discriminated against in gross violation of law, such a proscription by an
employer being outlawed by Article 136 of the Labor Code.
Grace de Guzman was initially hired by petitioner as a reliever, specifically as a "Supernumerary Project Worker," for a fixed
period from November 21, 1990 until April 20, 1991 vice one C.F. Tenorio who went on maternity leave. 1 Under the Reliever
Agreement which she signed with petitioner company, her employment was to be immediately terminated upon expiration of the
agreed period. Thereafter, from June 10, 1991 to July 1, 1991, and from July 19, 1991 to August 8, 1991, private respondent's
services as reliever were again engaged by petitioner, this time in replacement of one Erlinda F. Dizon who went on leave
during both periods. 2 After August 8, 1991, and pursuant to their Reliever Agreement, her services were terminated.
On September 2, 1991, private respondent was once more asked to join petitioner company as a probationary employee, the
probationary period to cover 150 days. In the job application form that was furnished her to be filled up for the purpose, she
indicated in the portion for civil status therein that she was single although she had contracted marriage a few months earlier,
that is, on May 26, 1991. 3
It now appears that private respondent had made the same representation in the two successive reliever agreements which she
signed on June 10, 1991 and July 8, 1991. When petitioner supposedly learned about the same later, its branch supervisor in
Baguio City, Delia M. Oficial, sent to private respondent a memorandum dated January 15, 1992 requiring her to explain the
discrepancy. In that memorandum, she was reminded about the company's policy of not accepting married women for
employment. 4
In her reply letter dated January 17, 1992, private respondent stated that she was not aware of PT&T's policy regarding married
women at the time, and that all along she had not deliberately hidden her true civil status. 5Petitioner nonetheless remained
unconvinced by her explanations. Private respondent was dismissed from the company effective January 29, 1992, 6 which she
readily contested by initiating a complaint for illegal dismissal, coupled with a claim for non-payment of cost of living allowances
(COLA), before the Regional Arbitration Branch of the National Labor Relations Commission in Baguio City.
At the preliminary conference conducted in connection therewith, private respondent volunteered the information, and this was
incorporated in the stipulation of facts between the parties, that she had failed to remit the amount of P2,380.75 of her
collections. She then executed a promissory note for that amount in favor of petitioner 7. All of these took place in a formal
proceeding and with the agreement of the parties and/or their counsel.
On November 23, 1993, Labor Arbiter Irenarco R. Rimando handed down a decision declaring that private respondent, who had
already gained the status of a regular employee, was illegally dismissed by petitioner. Her reinstatement, plus payment of the
corresponding back wages and COLA, was correspondingly ordered, the labor arbiter being of the firmly expressed view that
the ground relied upon by petitioner in dismissing private respondent was clearly insufficient, and that it was apparent that she
had been discriminated against on account of her having contracted marriage in violation of company rules.
On appeal to the National Labor Relations Commission (NLRC), said public respondent upheld the labor arbiter and, in its
decision dated April 29, 1994, it ruled that private respondent had indeed been the subject of an unjust and unlawful
discrimination by her employer, PT & T. However, the decision of the labor arbiter was modified with the qualification that Grace
de Guzman deserved to be suspended for three months in view of the dishonest nature of her acts which should not be
condoned. In all other respects, the NLRC affirmed the decision of the labor arbiter, including the order for the reinstatement of
private respondent in her employment with PT & T.
The subsequent motion for reconsideration filed by petitioner was rebuffed by respondent NLRC in its resolution of November 9,
1994, hence this special civil action assailing the aforestated decisions of the labor arbiter and respondent NLRC, as well as the
denial resolution of the latter.
1. Decreed in the Bible itself is the universal norm that women should be regarded with love and respect but, through the ages,
men have responded to that injunction with indifference, on the hubristic conceit that women constitute the inferior sex.
Nowhere has that prejudice against womankind been so pervasive as in the field of labor, especially on the matter of equal
employment opportunities and standards. In the Philippine setting, women have traditionally been considered as falling within
the vulnerable groups or types of workers who must be safeguarded with preventive and remedial social legislation against
discriminatory and exploitative practices in hiring, training, benefits, promotion and retention.
The Constitution, cognizant of the disparity in rights between men and women in almost all phases of social and political life,
provides a gamut of protective provisions. To cite a few of the primordial ones, Section 14, Article II 8on the Declaration of
Principles and State Policies, expressly recognizes the role of women in nation-building and commands the State to ensure, at
all times, the fundamental equality before the law of women and men. Corollary thereto, Section 3 of Article XIII 9 (the progenitor
whereof dates back to both the 1935 and 1973 Constitution) pointedly requires the State to afford full protection to labor and to
promote full employment and equality of employment opportunities for all, including an assurance of entitlement to tenurial
security of all workers. Similarly, Section 14 of Article XIII 10 mandates that the State shall protect working women through
provisions for opportunities that would enable them to reach their full potential.
2. Corrective labor and social laws on gender inequality have emerged with more frequency in the years since the Labor Code
was enacted on May 1, 1974 as Presidential Decree No. 442, largely due to our country's commitment as a signatory to the
United Nations Convention on the Elimination of All Forms of Discrimination Against Women (CEDAW). 11
Principal among these laws are Republic Act No. 6727 12 which explicitly prohibits discrimination against women with respect to
terms and conditions of employment, promotion, and training opportunities; Republic Act No. 6955 13 which bans the "mailorder-bride" practice for a fee and the export of female labor to countries that cannot guarantee protection to the rights of
women workers; Republic Act No. 7192 14 also known as the "Women in Development and Nation Building Act," which affords
women equal opportunities with men to act and to enter into contracts, and for appointment, admission, training, graduation,
and commissioning in all military or similar schools of the Armed Forces of the Philippines and the Philippine National Police;
Republic Act No. 7322 15 increasing the maternity benefits granted to women in the private sector; Republic Act No.
7877 16 which outlaws and punishes sexual harassment in the workplace and in the education and training environment; and
Republic Act No. 8042, 17 or the "Migrant Workers and Overseas Filipinos Act of 1995," which prescribes as a matter of
policy, inter alia, the deployment of migrant workers, with emphasis on women, only in countries where their rights are secure.
Likewise, it would not be amiss to point out that in the Family Code, 18 women's rights in the field of civil law have been greatly
enhanced and expanded.

In the Labor Code, provisions governing the rights of women workers are found in Articles 130 to 138 thereof. Article 130
involves the right against particular kinds of night work while Article 132 ensures the right of women to be provided with facilities
and standards which the Secretary of Labor may establish to ensure their health and safety. For purposes of labor and social
legislation, a woman working in a nightclub, cocktail lounge, massage clinic, bar or other similar establishments shall be
considered as an employee under Article 138. Article 135, on the other hand, recognizes a woman's right against discrimination
with respect to terms and conditions of employment on account simply of sex. Finally, and this brings us to the issue at hand,
Article 136 explicitly prohibits discrimination merely by reason of the marriage of a female employee.
3. Acknowledged as paramount in the due process scheme is the constitutional guarantee of protection to labor and security of
tenure. Thus, an employer is required, as a condition sine qua non prior to severance of the employment ties of an individual
under his employ, to convincingly establish, through substantial evidence, the existence of a valid and just cause in dispensing
with the services of such employee, one's labor being regarded as constitutionally protected property.
On the other hand, it is recognized that regulation of manpower by the company falls within the so-called management
prerogatives, which prescriptions encompass the matter of hiring, supervision of workers, work assignments, working methods
and assignments, as well as regulations on the transfer of employees, lay-off of workers, and the discipline, dismissal, and
recall of employees. 19 As put in a case, an employer is free to regulate, according to his discretion and best business judgment,
all aspects of employment, "from hiring to firing," except in cases of unlawful discrimination or those which may be provided by
law. 20
In the case at bar, petitioner's policy of not accepting or considering as disqualified from work any woman worker who contracts
marriage runs afoul of the test of, and the right against, discrimination, afforded all women workers by our labor laws and by no
less than the Constitution. Contrary to petitioner's assertion that it dismissed private respondent from employment on account of
her dishonesty, the record discloses clearly that her ties with the company were dissolved principally because of the company's
policy that married women are not qualified for employment in PT & T, and not merely because of her supposed acts of
dishonesty.
That it was so can easily be seen from the memorandum sent to private respondent by Delia M. Oficial, the branch supervisor of
the company, with the reminder, in the words of the latter, that "you're fully aware that the company is not accepting married
women employee (sic), as it was verbally instructed to you." 21 Again, in the termination notice sent to her by the same branch
supervisor, private respondent was made to understand that her severance from the service was not only by reason of her
concealment of her married status but, over and on top of that, was her violation of the company's policy against marriage ("and
even told you that married women employees are not applicable [sic] or accepted in our company.") 22 Parenthetically, this
seems to be the curious reason why it was made to appear in the initiatory pleadings that petitioner was represented in this
case only by its said supervisor and not by its highest ranking officers who would otherwise be solidarily liable with the
corporation. 23
Verily, private respondent's act of concealing the true nature of her status from PT & T could not be properly characterized as
willful or in bad faith as she was moved to act the way she did mainly because she wanted to retain a permanent job in a stable
company. In other words, she was practically forced by that very same illegal company policy into misrepresenting her civil
status for fear of being disqualified from work. While loss of confidence is a just cause for termination of employment, it should
not be simulated. 24 It must rest on an actual breach of duty committed by the employee and not on the employer's
caprices. 25 Furthermore, it should never be used as a subterfuge for causes which are improper, illegal, or unjustified. 26
In the present controversy, petitioner's expostulations that it dismissed private respondent, not because the latter got married
but because she concealed that fact, does have a hollow ring. Her concealment, so it is claimed, bespeaks dishonesty hence
the consequent loss of confidence in her which justified her dismissal.
Petitioner would asseverate, therefore, that while it has nothing against marriage, it nonetheless takes umbrage over the
concealment of that fact. This improbable reasoning, with interstitial distinctions, perturbs the Court since private respondent
may well be minded to claim that the imputation of dishonesty should be the other way around.
Petitioner would have the Court believe that although private respondent defied its policy against its female employees
contracting marriage, what could be an act of insubordination was inconsequential. What it submits as unforgivable is her
concealment of that marriage yet, at the same time, declaring that marriage as a trivial matter to which it supposedly has no
objection. In other words, PT & T says it gives its blessings to its female employees contracting marriage, despite the maternity
leaves and other benefits it would consequently respond for and which obviously it would have wanted to avoid. If that
employee confesses such fact of marriage, there will be no sanction; but if such employee conceals the same instead of
proceeding to the confessional, she will be dismissed. This line of reasoning does not impress us as reflecting its true
management policy or that we are being regaled with responsible advocacy.
This Court should be spared the ennui of strained reasoning and the tedium of propositions which confuse through less than
candid arguments. Indeed, petitioner glosses over the fact that it was its unlawful policy against married women, both on the
aspects of qualification and retention, which compelled private respondent to conceal her supervenient marriage. It was,
however, that very policy alone which was the cause of private respondent's secretive conduct now complained of. It is then
apropos to recall the familiar saying that he who is the cause of the cause is the cause of the evil caused.
Finally, petitioner's collateral insistence on the admission of private respondent that she supposedly misappropriated company
funds, as an additional ground to dismiss her from employment, is somewhat insincere and self-serving. Concededly, private
respondent admitted in the course of the proceedings that she failed to remit some of her collections, but that is an altogether
different story. The fact is that she was dismissed solely because of her concealment of her marital status, and not on the basis
of that supposed defalcation of company funds. That the labor arbiter would thus consider petitioner's submissions on this
supposed dishonesty as a mere afterthought, just to bolster its case for dismissal, is a perceptive conclusion born of experience
in labor cases. For, there was no showing that private respondent deliberately misappropriated the amount or whether her
failure to remit the same was through negligence and, if so, whether the negligence was in nature simple or grave. In fact, it was
merely agreed that private respondent execute a promissory note to refund the same, which she did, and the matter was
deemed settled as a peripheral issue in the labor case.
Private respondent, it must be observed, had gained regular status at the time of her dismissal. When she was served her
walking papers on January 29, 1992, she was about to complete the probationary period of 150 days as she was contracted as
a probationary employee on September 2, 1991. That her dismissal would be effected just when her probationary period was
winding down clearly raises the plausible conclusion that it was done in order to prevent her from earning security of
tenure. 27 On the other hand, her earlier stints with the company as reliever were undoubtedly those of a regular employee, even
if the same were for fixed periods, as she performed activities which were essential or necessary in the usual trade and
business of PT & T. 28 The primary standard of determining regular employment is the reasonable connection between the
activity performed by the employee in relation to the business or trade of the employer. 29
As an employee who had therefore gained regular status, and as she had been dismissed without just cause, she is entitled to
reinstatement without loss of seniority rights and other privileges and to full back wages, inclusive of allowances and other
benefits or their monetary equivalent. 30 However, as she had undeniably committed an act of dishonesty in concealing her
status, albeit under the compulsion of an unlawful imposition of petitioner, the three-month suspension imposed by respondent
NLRC must be upheld to obviate the impression or inference that such act should be condoned. It would be unfair to the
employer if she were to return to its fold without any sanction whatsoever for her act which was not totally justified. Thus, her
entitlement to back wages, which shall be computed from the time her compensation was withheld up to the time of her actual
reinstatement, shall be reduced by deducting therefrom the amount corresponding to her three months suspension.

4. The government, to repeat, abhors any stipulation or policy in the nature of that adopted by petitioner PT & T. The Labor
Code state, in no uncertain terms, as follows:
Art. 136. Stipulation against marriage. It shall be unlawful for an employer to require as a condition of
employment or continuation of employment that a woman shall not get married, or to stipulate expressly or tacitly
that upon getting married, a woman employee shall be deemed resigned or separated, or to actually dismiss,
discharge, discriminate or otherwise prejudice a woman employee merely by reason of marriage.
This provision had a studied history for its origin can be traced to Section 8 of Presidential Decree No. 148, 31better known as
the "Women and
Child Labor Law," which amended paragraph (c), Section 12 of Republic Act No. 679, 32 entitled "An Act to Regulate the
Employment of Women and Children, to Provide Penalties for Violations Thereof, and for Other Purposes." The forerunner to
Republic Act No. 679, on the other hand, was Act No. 3071 which became law on March 16, 1923 and which regulated the
employment of women and children in shops, factories, industrial, agricultural, and mercantile establishments and other places
of labor in the then Philippine Islands.
It would be worthwhile to reflect upon and adopt here the rationalization in Zialcita, et al. vs. Philippine Air Lines, 33a decision
that emanated from the Office of the President. There, a policy of Philippine Air Lines requiring that prospective flight attendants
must be single and that they will be automatically separated from the service once they marry was declared void, it being
violative of the clear mandate in Article 136 of the Labor Code with regard to discrimination against married women. Thus:
Of first impression is the incompatibility of the respondent's policy or regulation with the codal provision of law.
Respondent is resolute in its contention that Article 136 of the Labor Code applies only to women employed in
ordinary occupations and that the prohibition against marriage of women engaged in extraordinary occupations, like
flight attendants, is fair and reasonable, considering the pecularities of their chosen profession.
We cannot subscribe to the line of reasoning pursued by respondent. All along, it knew that the controverted policy
has already met its doom as early as March 13, 1973 when Presidential Decree No. 148, otherwise known as the
Women and Child Labor Law, was promulgated. But for the timidity of those affected or their labor unions in
challenging the validity of the policy, the same was able to obtain a momentary reprieve. A close look at Section 8 of
said decree, which amended paragraph (c) of Section 12 of Republic Act No. 679, reveals that it is exactly the same
provision reproduced verbatim in Article 136 of the Labor Code, which was promulgated on May 1, 1974 to take
effect six (6) months later, or on November 1, 1974.
It cannot be gainsaid that, with the reiteration of the same provision in the new Labor Code, all policies and acts
against it are deemed illegal and therefore abrogated. True, Article 132 enjoins the Secretary of Labor to establish
standards that will ensure the safety and health of women employees and in appropriate cases shall by regulation
require employers to determine appropriate minimum standards for termination in special occupations, such as
those of flight attendants, but that is precisely the factor that militates against the policy of respondent. The
standards have not yet been established as set forth in the first paragraph, nor has the Secretary of Labor issued
any regulation affecting flight attendants.
It is logical to presume that, in the absence of said standards or regulations which are as yet to be established, the
policy of respondent against marriage is patently illegal. This finds support in Section 9 of the New Constitution,
which provides:
Sec. 9. The State shall afford protection to labor, promote full employment and equality in employment, ensure equal
work opportunities regardless of sex, race, or creed, and regulate the relations between workers and employees.
The State shall assure the rights of workers to self-organization, collective bargaining, security of tenure, and just
and humane conditions of work . . . .
Moreover, we cannot agree to the respondent's proposition that termination from employment of flight attendants on
account of marriage is a fair and reasonable standard designed for their own health, safety, protection and welfare,
as no basis has been laid therefor. Actually, respondent claims that its concern is not so much against the continued
employment of the flight attendant merely by reason of marriage as observed by the Secretary of Labor, but rather
on the consequence of marriage-pregnancy. Respondent discussed at length in the instant appeal the supposed ill
effects of pregnancy on flight attendants in the course of their employment. We feel that this needs no further
discussion as it had been adequately explained by the Secretary of Labor in his decision of May 2, 1976.
In a vain attempt to give meaning to its position, respondent went as far as invoking the provisions of Articles 52 and
216 of the New Civil Code on the preservation of marriage as an inviolable social institution and the family as a
basic social institution, respectively, as bases for its policy of non-marriage. In both instances, respondent predicates
absence of a flight attendant from her home for long periods of time as contributory to an unhappy married life. This
is pure conjecture not based on actual conditions, considering that, in this modern world, sophisticated technology
has narrowed the distance from one place to another. Moreover, respondent overlooked the fact that married flight
attendants can program their lives to adapt to prevailing circumstances and events.
Article 136 is not intended to apply only to women employed in ordinary occupations, or it should have categorically
expressed so. The sweeping intendment of the law, be it on special or ordinary occupations, is reflected in the whole
text and supported by Article 135 that speaks of non-discrimination on the employment of women.
The judgment of the Court of Appeals in Gualberto, et al. vs. Marinduque Mining & Industrial Corporation 34considered as void a
policy of the same nature. In said case, respondent, in dismissing from the service the complainant, invoked a policy of the firm
to consider female employees in the project it was undertaking as separated the moment they get married due to lack of
facilities for married women. Respondent further claimed that complainant was employed in the project with an oral
understanding that her services would be terminated when she gets married. Branding the policy of the employer as an
example of "discriminatory chauvinism" tantamount to denying equal employment opportunities to women simply on account of
their sex, the appellate court struck down said employer policy as unlawful in view of its repugnance to the Civil Code,
Presidential Decree No. 148 and the Constitution.
Under American jurisprudence, job requirements which establish employer preference or conditions relating to the marital status
of an employee are categorized as a "sex-plus" discrimination where it is imposed on one sex and not on the other. Further, the
same should be evenly applied and must not inflict adverse effects on a racial or sexual group which is protected by federal job
discrimination laws. Employment rules that forbid or restrict the employment of married women, but do not apply to married
men, have been held to violate Title VII of the United States Civil Rights Act of 1964, the main federal statute prohibiting job
discrimination against employees and applicants on the basis of, among other things, sex. 35
Further, it is not relevant that the rule is not directed against all women but just against married women. And, where the
employer discriminates against married women, but not against married men, the variable is sex and the discrimination is
unlawful. 36 Upon the other hand, a requirement that a woman employee must remain unmarried could be justified as a "bona
fide occupational qualification," or BFOQ, where the particular requirements of the job would justify the same, but not on the
ground of a general principle, such as the desirability of spreading work in the workplace. A requirement of that nature would be
valid provided it reflects an inherent quality reasonably necessary for satisfactory job performance. Thus, in one case, a nomarriage rule applicable to both male and female flight attendants, was regarded as unlawful since the restriction was not
related to the job performance of the flight attendants. 37

5. Petitioner's policy is not only in derogation of the provisions of Article 136 of the Labor Code on the right of a woman to be
free from any kind of stipulation against marriage in connection with her employment, but it likewise assaults good morals and
public policy, tending as it does to deprive a woman of the freedom to choose her status, a privilege that by all accounts inheres
in the individual as an intangible and inalienable right. 38 Hence, while it is true that the parties to a contract may establish any
agreements, terms, and conditions that they may deem convenient, the same should not be contrary to law, morals, good
customs, public order, or public policy. 39 Carried to its logical consequences, it may even be said that petitioner's policy against
legitimate marital bonds would encourage illicit or common-law relations and subvert the sacrament of marriage.
Parenthetically, the Civil Code provisions on the contract of labor state that the relations between the parties, that is, of capital
and labor, are not merely contractual, impressed as they are with so much public interest that the same should yield to the
common good. 40 It goes on to intone that neither capital nor labor should visit acts of oppression against the other, nor impair
the interest or convenience of the public. 41 In the final reckoning, the danger of just such a policy against marriage followed by
petitioner PT & T is that it strikes at the very essence, ideals and purpose of marriage as an inviolable social institution and,
ultimately, of the family as the foundation of the nation. 42 That it must be effectively interdicted here in all its indirect, disguised
or dissembled forms as discriminatory conduct derogatory of the laws of the land is not only in order but imperatively required.
ON THE FOREGOING PREMISES, the petition of Philippine Telegraph and Telephone Company is hereby DISMISSED for lack
of merit, with double costs against petitioner.
SO ORDERED.

G.R. No. 11263

November 2, 1916

ELOISA GOITIA DE LA CAMARA, plaintiff-appellant,


vs.
JOSE CAMPOS RUEDA, defendant-appellee.
Eduardo Gutierrez Repide and Felix Socias for appellant.
Sanz, Opisso and Luzuriaga for appellee.

TRENT, J.:
This is an action by the wife against her husband for support outside of the conjugal domicile. From a judgment sustaining the
defendant's demurrer upon the ground that the facts alleged in the complaint do not state a cause of action, followed by an
order dismissing the case after the plaintiff declined to amend, the latter appealed.
It was urged in the first instance, and the court so held, that the defendant cannot be compelled to support the plaintiff, except in
his own house, unless it be by virtue of a judicial decree granting her a divorce or separation from the defendant.
The parties were legally married in the city of Manila on January 7, 1915, and immediately thereafter established their residence
at 115 Calle San Marcelino, where they lived together for about a month, when the plaintiff returned to the home of her parents.
The pertinent allegations of the complaint are as follows:
That the defendant, one month after he had contracted marriage with the plaintiff, demanded of her that she perform
unchaste and lascivious acts on his genital organs; that the plaintiff spurned the obscene demands of the defendant and
refused to perform any act other than legal and valid cohabitation; that the defendant, since that date had continually on
other successive dates, made similar lewd and indecorous demands on his wife, the plaintiff, who always spurned them,
which just refusals of the plaintiff exasperated the defendant and induce him to maltreat her by word and deed and inflict
injuries upon her lips, her face and different parts of her body; and that, as the plaintiff was unable by any means to induce
the defendant to desist from his repugnant desires and cease from maltreating her, she was obliged to leave the conjugal
abode and take refuge in the home of her parents.
Marriage in this jurisdiction is a contract entered into in the manner and with the solemnities established by General Orders No.
68, in so far as its civil effects are concerned requiring the consent of the parties. (Garcia vs. Montague, 12 Phil. Rep., 480,
citing article 1261 of Civil Code.) Upon the termination of the marriage ceremony, a conjugal partnership is formed between the
parties. (Sy Joc Lieng vs. Encarnacion, 16 Phil. Rep., 137.) To this extent a marriage partakes of the nature of an ordinary
contract. But it is something more than a mere contract. It is a new relation, the rights, duties, and obligations of which rest not
upon the agreement of the parties but upon the general law which defines and prescribes those rights, duties, and
obligations .Marriage is an institution, in the maintenance of which in its purity the public is deeply interested. It is a relation for
life and the parties cannot terminate it at any shorter period by virtue of any contract they may make .The reciprocal rights
arising from this relation, so long as it continues, are such as the law determines from time to time, and none other. When the
legal existence of the parties is merged into one by marriage, the new relation is regulated and controlled by the state or
government upon principles of public policy for the benefit of society as well as the parties. And when the object of a marriage is
defeated by rendering its continuance intolerable to one of the parties and productive of no possible good to the community,
relief in some way should be obtainable. With these principles to guide us, we will inquire into the status of the law touching and
governing the question under consideration.
Articles 42 to 107 of the Civil Code are not in force in the Philippine Islands (Benedicto vs. De la Rama, 3 Phil .Rep., 34).
Articles 44 to 78 of the Law of Civil Marriage of 1870, in force in the Peninsula, were extended to the Philippine Islands by royal
decree on April 13, 1883 (Ebreo vs. Sichon, 4 Phil. Rep., 705). Articles 44, 45, and 48 of this law read:
ART. 44. The spouses are obliged to be faithful to each other and to mutually assist each other.
ART. 45. The husband must live with and protect his wife. (The second paragraph deals with the management of the
wife's property.)
ART. 48. The wife must obey her husband, live with him, and follow him when he charges his domicile or residence.
Notwithstanding the provisions of the foregoing paragraph, the court may for just cause relieve her from this duty when the
husband removes his residence to a foreign country.
And articles 143 and 149 of the Civil Code are as follows:
ART. 143. The following are obliged to support each other reciprocally to the whole extent specified in the preceding
article.
1. The consorts.
xxx

xxx

xxx

ART. (149) 49. The person obliged to give support may, at his option, satisfy it, either by paying the pension that may be
fixed or by receiving and maintaining in his own home the person having the right to the same.
Article 152 of the Civil Code gives the instances when the obligation to give support shall cease. The failure of the wife to live
with her husband is not one of them.
The above quoted provisions of the Law of Civil Marriage and the Civil Code fix the duties and obligations of the spouses. The
spouses must be faithful to, assist, and support each other. The husband must live with and protect his wife. The wife must obey
and live with her husband and follow him when he changes his domicile or residence, except when he removes to a foreign
country. But the husband who is obliged to support his wife may, at his option, do so by paying her a fixed pension or by
receiving and maintaining her in his own home. May the husband, on account of his conduct toward his wife, lose this option
and be compelled to pay the pension? Is the rule established by article 149 of the Civil Code absolute? The supreme court of
Spain in its decision of December 5, 1903, held:.
That in accordance with the ruling of the supreme court of Spain in its decisions dated May 11, 1897, November 25, 1899,
and July 5, 1901, the option which article 149 grants the person, obliged to furnish subsistence, between paying the
pension fixed or receiving and keeping in his own house the party who is entitled to the same, is not so absolute as to
prevent cases being considered wherein, either because this right would be opposed to the exercise of a preferential right
or because of the existence of some justifiable cause morally opposed to the removal of the party enjoying the
maintenance, the right of selection must be understood as being thereby restricted.
Whereas the only question discussed in the case which gave rise to this appeal was whether there was any reason to
prevent the exercise of the option granted by article 149 of the Civil Code to the person obliged to furnish subsistence, to
receive and maintain in his own house the one who is entitled to receive it; and inasmuch as nothing has been alleged or
discussed with regard to the parental authority of Pedro Alcantara Calvo, which he ha not exercised, and it having been

set forth that the natural father simply claims his child for the purpose of thus better attending to her maintenance, no
action having been taken by him toward providing the support until, owing to such negligence, the mother was obliged to
demand it; it is seen that these circumstances, together with the fact of the marriage of Pedro Alcantara, and that it would
be difficult for the mother to maintain relations with her daughter, all constitute an impediment of such a nature as to
prevent the exercise of the option in the present case, without prejudice to such decision as may be deemed proper with
regard to the other questions previously cited in respect to which no opinion should be expressed at this time.
The above was quoted with approval in United States and De Jesus vs. Alvir (9 Phil. Rep., 576), wherein the court held that the
rule laid down in article 149 of the Civil Code "is not absolute." but it is insisted that there existed a preexisting or preferential
right in each of these cases which was opposed to the removal of the one entitled to support. It is true that in the first the person
claiming the option was the natural father of the child and had married a woman other than the child's mother, and in the second
the right to support had already been established by a final judgment in a criminal case. Notwithstanding these facts the two
cases clearly established the proposition that the option given by article 149 of the Civil Code may not be exercised in any and
all cases.
Counsel for the defendant cite, in support of their contention, the decision of the supreme court of Spain, dated November 3,
1905. In this case Don Berno Comas, as a result of certain business reverses and in order no to prejudice his wife, conferred
upon her powers to administer and dispose of her property. When she left him he gave her all the muniments of title, mortgage
credits, notes, P10,000 in accounts receivable, and the key to the safe in which he kept a large amount of jewels, thus depriving
himself of all his possessions and being reduced in consequence to want. Subsequently he instituted this civil action against his
wife, who was then living in opulence, for support and the revocation of the powers heretofore granted in reference to the
administration and disposal of her property. In her answer the wife claimed that the plaintiff (her husband) was not legally in a
situation to claim support and that the powers voluntarily conferred and accepted by her were bilateral and could not be
canceled by the plaintiff. From a judgment in favor of the plaintiff the defendant wife appealed to the Audencia Territorialwherein,
after due trial, judgment was rendered in her favor dismissing the action upon the merits. The plaintiff appealed to the supreme
court and that high tribunal, in affirming the judgment of the Audencia Territorial, said:
Considering that article 143, No. 1, of the Civil Code, providing that the spouses are mutually obliged to provide each
other with support, cannot but be subordinate to the other provisions of said Code which regulates the family organization
and the duties of spouses not legally separated, among which duties are those of their living together and mutually helping
each other, as provided in article 56 of the aforementioned code; and taking this for granted, the obligation of the spouse
who has property to furnish support to the one who has no property and is in need of it for subsistence, is to be
understood as limited to the case where, in accordance with law, their separation has been decreed, either temporarily or
finally and this case, with respect to the husband, cannot occur until a judgment of divorce is rendered, since, until then, if
he is culpable, he is not deprived of the management of his wife's property and of the product of the other property
belonging to the conjugal partnership; and
Considering that, should the doctrine maintained in the appeal prevail, it would allow married persons to disregard the
marriage bond and separate from each other of their own free will, thus establishing, contrary to the legal provision
contained in said article 56 of the Civil Code, a legal status entirely incompatible with the nature and effects of marriage in
disregard of the duties inherent therein and disturbing the unity of the family, in opposition to what the law, in conformity
with good morals, has established; and.
Considering that, as the spouses D. Ramon Benso and Doa Adela Galindo are not legally separated, it is their duty to
live together and afford each other help and support; and for this reason, it cannot be held that the former has need of
support from his wife so that he may live apart from her without the conjugal abode where it is his place to be, nor of her
conferring power upon him to dispose even of the fruits of her property in order therewith to pay the matrimonial expenses
and, consequently, those of his own support without need of going to his wife; wherefore the judgment appealed from,
denying the petition of D. Ramon Benso for support, has not violated the articles of the Civil Code and the doctrine
invoked in the assignments of error 1 and 5 of the appeal.
From a careful reading of the case just cited and quoted from it appears quite clearly that the spouses separated voluntarily in
accordance with an agreement previously made. At least there are strong indications to this effect, for the court says, "should
the doctrine maintained in the appeal prevail, it would allow married persons to disregard the marriage bond and separate from
each other of their own free will." If this be the true basis upon which the supreme court of Spain rested its decision, then the
doctrine therein enunciated would not be controlling in cases where one of the spouses was compelled to leave the conjugal
abode by the other or where the husband voluntarily abandons such abode and the wife seeks to force him to furnish support.
That this is true appears from the decision of the same high tribunal, dated October 16, 1903. In this case the wife brought an
action for support against her husband who had willfully and voluntarily abandoned the conjugal abode without any cause
whatever. The supreme court, reversing the judgment absolving the defendant upon the ground that no action for divorce, etc.,
had been instituted, said:
In the case at bar, it has been proven that it was Don Teodoro Exposito who left the conjugal abode, although he claims,
without however proving his contention, that the person responsible for this situation was his wife, as she turned him out
of the house. From this state of affairs it results that it is the wife who is party abandoned, the husband not having
prosecuted any action to keep her in his company and he therefore finds himself, as long as he consents to the situation,
under the ineluctable obligation to support his wife in fulfillment of the natural duty sanctioned in article 56 of the Code in
relation with paragraph 1 of article 143. In not so holding, the trial court, on the mistaken ground that for the fulfillment of
this duty the situation or relation of the spouses should be regulated in the manner it indicates, has made the errors of law
assigned in the first three grounds alleged, because the nature of the duty of affording mutual support is compatible and
enforcible in all situations, so long as the needy spouse does not create any illicit situation of the court above
described.lawphil.net
If we are in error as to the doctrine enunciated by the supreme court of Spain in its decision of November 3, 1905, and if the
court did hold, as contended by counsel for the defendant in the case under consideration, that neither spouse can be
compelled to support the other outside of the conjugal abode, unless it be by virtue of a final judgment granting the injured one a
divorce or separation from the other, still such doctrine or holding would not necessarily control in this jurisdiction for the reason
that the substantive law is not in every particular the same here as it is in Spain. As we have already stated, articles 42 to 107 of
the Civil Code in force in the Peninsula are not in force in the Philippine Islands. The law governing the duties and obligations of
husband and wife in this country are articles 44 to 78 of the Law of Civil Marriage of 1870 .In Spain the complaining spouse has,
under article 105 of the Civil Code, various causes for divorce, such as adultery on the part of the wife in every case and on the
part of the husband when public scandal or disgrace of the wife results therefrom; personal violence actually inflicted or grave
insults: violence exercised by the husband toward the wife in order to force her to change her religion; the proposal of the
husband to prostitute his wife; the attempts of the husband or wife to corrupt their sons or to prostitute their daughters; the
connivance in their corruption or prostitution; and the condemnation of a spouse to perpetual chains or hard labor, while in this
jurisdiction the only ground for a divorce is adultery. (Benedicto vs. De la Rama, 3 Phil .Rep., 34, 45.) This positive and absolute
doctrine was announced by this court in the case just cited after an exhaustive examination of the entire subject. Although the
case was appealed to the Supreme Court of the United States and the judgment rendered by this court was there reversed, the
reversal did not affect in any way or weaken the doctrine in reference to adultery being the only ground for a divorce. And since
the decision was promulgated by this court in that case in December, 1903, no change or modification of the rule has been
announced. It is, therefore, the well settled and accepted doctrine in this jurisdiction.
But it is argued that to grant support in an independent suit is equivalent to granting divorce or separation, as it necessitates a
determination of the question whether the wife has a good and sufficient cause for living separate from her husband; and,
consequently, if a court lacks power to decree a divorce, as in the instant case, power to grant a separate maintenance must

also be lacking. The weakness of this argument lies in the assumption that the power to grant support in a separate action is
dependent upon a power to grant a divorce. That the one is not dependent upon the other is apparent from the very nature of
the marital obligations of the spouses. The mere act of marriage creates an obligation on the part of the husband to support his
wife. This obligation is founded not so much on the express or implied terms of the contract of marriage as on the natural and
legal duty of the husband; an obligation, the enforcement of which is of such vital concern to the state itself that the laws will not
permit him to terminate it by his own wrongful acts in driving his wife to seek protection in the parental home. A judgment for
separate maintenance is not due and payable either as damages or as a penalty; nor is it a debt in the strict legal sense of the
term, but rather a judgment calling for the performance of a duty made specific by the mandate of the sovereign. This is done
from necessity and with a view to preserve the public peace and the purity of the wife; as where the husband makes so base
demands upon his wife and indulges in the habit of assaulting her. The pro tanto separation resulting from a decree for separate
support is not an impeachment of that public policy by which marriage is regarded as so sacred and inviolable in its nature; it is
merely a stronger policy overruling a weaker one; and except in so far only as such separation is tolerated as a means of
preserving the public peace and morals may be considered, it does not in any respect whatever impair the marriage contract or
for any purpose place the wife in the situation of a feme sole.
The foregoing are the grounds upon which our short opinion and order for judgment, heretofore filed in this case, rest.

G.R. No. 83598 March 7, 1997


LEONCIA BALOGBOG and GAUDIOSO BALOGBOG, petitioners,
vs.
HONORABLE COURT OF APPEALS, RAMONITO BALOGBOG and GENEROSO BALOGBOG, respondents.

MENDOZA, J.:
This is a petition for review of the decision 1 of the Court of Appeals, affirming the decision of the Court of First Instance of Cebu
City (Branch IX), declaring private respondents heirs of the deceased Basilio and Genoveva Balogbog entitled to inherit from
them.
The facts are as follows. Petitioners Leoncia and Gaudioso Balogbog are the children of Basilio Balogbog and Genoveva
Arzibal who died intestate in 1951 and 1961, respectively. They had an older brother, Gavino, but he died in 1935, predeceasing
their parents.
In 1968, private respondents Ramonito and Generoso Balogbog brought an action for partition and accounting against
petitioners, claiming that they were the legitimate children of Gavino by Catalina Ubas and that, as such, they were entitled to
the one-third share of Gavino in the estate of their grandparents.
In their answer, petitioners denied knowing private respondents. They alleged that their brother Gavino died single and without
issue in their parents' residence at Tag-amakan, Asturias, Cebu. In the beginning they claimed that the properties of the estate
had been sold to them by their mother when she was still alive, but they later withdrew this allegation.
Private respondents presented Priscilo Y. Trazo, 2 then 81 years old, mayor of the municipality of Asturias from 1928 to 1934,
who testified that he knew Gavino and Catalina to be husband and wife and Ramonito to be their first child. On
crossexamination, Trazo explained that he knew Gavino and Catalina because they performed at his campaign rallies, Catalina
as "balitaw" dancer and Gavino Balogbog as her guitarist. Trazo said he attended the wedding of Gavino and Catalina
sometime in 1929, in which Rev. Father Emiliano Jomao-as officiated and Egmidio Manuel, then a municipal councilor, acted as
one of the witnesses.
The second witness presented was Matias Pogoy, 3 a family friend of private respondents, who testified that private respondents
are the children of Gavino and Catalina. According to him, the wedding of Gavino and Catalina was solemnized in the Catholic
Church of Asturias, Cebu and that he knew this because he attended their wedding and was in fact asked by Gavino to
accompany Catalina and carry her wedding dress from her residence in Camanaol to the poblacion of Asturias before the
wedding day. He testified that Gavino died in 1935 in his residence at Obogon, Balamban, Cebu, in the presence of his wife.
(This contradicts petitioners' claim made in their answer that Gavino died in the ancestral house at Tag-amakan, Asturias.)
Pogoy said he was a carpenter and he was the one who had made the coffin of Gavino. He also made the coffin of the couple's
son, Petronilo, who died when he was six.
Catalina Ubas testified concerning her marriage to Gavino. 4 She testified that after the wedding, she was handed a "receipt,"
presumably the marriage certificate, by Fr. Jomao-as, but it was burned during the war. She said that she and Gavino lived
together in Obogon and begot three children, namely, Ramonito, Petronilo, and Generoso. Petronilo died after an illness at the
age of six. On crossexamination, she stated that after the death of Gavino, she lived in common law relation with a man for a
year and then they separated.
Private respondents produced a certificate from the Office of the Local Civil Registrar (Exh. P) that the Register of Marriages did
not have a record of the marriage of Gavino and Catalina, another certificate from the Office of the Treasurer (Exh. L) that there
was no record of the birth of Ramonito in that office and, for this reason, the record must be presumed to have been lost or
destroyed during the war, and a certificate by the Parish Priest of Asturias that there was likewise no record of birth of Ramonito
in the church, the records of which were either lost or destroyed during the war. (Exh. M)
On the other hand, as defendant below, petitioner Leoncia Balogbog testified 5 that Gavino died single at the family residence in
Asturias. She denied that her brother had any legitimate children and stated that she did not know private respondents before
this case was filed. She obtained a certificate (Exh. 10) from the Local Civil Registrar of Asturias to the effect that that office did
not have a record of the names of Gavino and Catalina. The certificate was prepared by Assistant Municipal Treasurer Juan
Maranga, who testified that there was no record of the marriage of Gavino and Catalina in the Book of Marriages between 1925
to 1935. 6
Witness Jose Narvasa testified 7 that Gavino died single in 1935 and that Catalina lived with a certain Eleuterio Keriado after the
war, although he did not know whether they were legally married. He added, however, that Catalina had children by a man she
had married before the war, although he did not know the names of the children. On crossexamination, Narvasa stated that
Leoncia Balogbog, who requested him to testify, was also his bondsman in a criminal case filed by a certain Mr. Cuyos.
Ramonito Balogbog was presented 8 to rebut Leoncia Balogbog's testimony.
On June 15, 1973, the Court of First Instance of Cebu City rendered judgment for private respondents (plaintiffs below),
ordering petitioners to render an accounting from 1960 until the finality of its judgment, to partition the estate and deliver to
private respondents one-third of the estate of Basilio and Genoveva, and to pay attorney's fees and costs.
Petitioners filed a motion for new trial and/or reconsideration, contending that the trial court erred in not giving weight to the
certification of the Office of the Municipal Treasurer of Asturias (Exh. 10) to the effect that no marriage of Gavino and Catalina
was recorded in the Book of Marriages for the years 1925-1935. Their motion was denied by the trial court, as was their second
motion for new trial and/or reconsideration based on the church records of the parish of Asturias which did not contain the
record of the alleged marriage in that church.
On appeal, the Court of Appeals affirmed. It held that private respondents failed to overcome the legal presumption that a man
and a woman deporting themselves as husband and wife are in fact married, that a child is presumed to be legitimate, and that
things happen according to the ordinary course of nature and the ordinary habits of life. 9 Hence, this petition.
We find no reversible error committed by the Court of Appeals.
First. Petitioners contend that the marriage of Gavino and Catalina should have been proven in accordance with Arts. 53 and 54
of the Civil Code of 1889 because this was the law in force at the time the alleged marriage was celebrated. Art. 53 provides
that marriages celebrated under the Civil Code of 1889 should be proven only by a certified copy of the memorandum in the
Civil Registry, unless the books thereof have not been kept or have been lost, or unless they are questioned in the courts, in
which case any other proof, such as that of the continuous possession by parents of the status of husband and wife, may be
considered, provided that the registration of the birth of their children as their legitimate children is also submitted in evidence.
This Court noted long ago, however, that Arts. 42 to 107 of the Civil Code of 1889 of Spain did not take effect, having been
suspended by the Governor General of the Philippines shortly after the extension of that code to this
country. 10 Consequently, Arts. 53 and 54 never came into force. Since this case was brought in the lower court in 1968, the
existence of the marriage must be determined in accordance with the present Civil Code, which repealed the provisions of the
former Civil Code, except as they related to vested rights, 11 and the rules on evidence. Under the Rules of Court, the

presumption is that a man and a woman conducting themselves as husband and wife are legally married. 12 This presumption
may be rebutted only by cogent proof to the contrary. 13 In this case, petitioners' claim that the certification presented by private
respondents (to the effect that the record of the marriage had been lost or destroyed during the war) was belied by the
production of the Book of Marriages by the assistant municipal treasurer of Asturias. Petitioners argue that this book does not
contain any entry pertaining to the alleged marriage of private respondents' parents.
This contention has no merit. In Pugeda v. Trias, 14 the defendants, who questioned the marriage of the plaintiffs, produced a
photostatic copy of the record of marriages of the Municipality of Rosario, Cavite for the month of January, 1916, to show that
there was no record of the alleged marriage. Nonetheless, evidence consisting of the testimonies of witnesses was held
competent to prove the marriage. Indeed, although a marriage contract is considered primary evidence of marriage, 15 the failure
to present it is not proof that no marriage took place. Other evidence may be presented to prove
marriage. 16 Here, private respondents proved, through testimonial evidence, that Gavino and Catalina were married in 1929;
that they had three children, one of whom died in infancy; that their marriage subsisted until 1935 when Gavino died; and that
their children, private respondents herein, were recognized by Gavino's family and by the public as the legitimate children of
Gavino.
Neither is there merit in the argument that the existence of the marriage cannot be presumed because there was no evidence
showing in particular that Gavino and Catalina, in the presence of two witnesses, declared that they were taking each other as
husband and wife. 17 An exchange of vows can be presumed to have been made from the testimonies of the witnesses who
state that a wedding took place, since the very purpose for having a wedding is to exchange vows of marital commitment. It
would indeed be unusual to have a wedding without an exchange of vows and quite unnatural for people not to notice its
absence.
The law favors the validity of marriage, because the State is interested in the preservation of the family and the sanctity of the
family is a matter of constitutional concern. As stated in Adong v. Cheong Seng Gee: 18
The basis of human society throughout the civilized world is that of marriage. Marriage in this jurisdiction is not only
a civil contract, but it is a new relation, an institution in the maintenance of which the public is deeply interested.
Consequently, every intendment of the law leans toward legalizing matrimony. Persons dwelling together in apparent
matrimony are presumed, in the absence of any counter-presumption or evidence special to the case, to be in fact
married. The reason is that such is the common order of society, and if the parties were not what they thus hold
themselves out as being, they would be living in the constant violation of decency and of law. A presumption
established by our Code of Civil Procedure is "that a man and a woman deporting themselves as husband and wife
have entered into a lawful contract of marriage." (Sec. 334, No. 28) Semper praesumitur pro matrimonio Always
presume marriage. (U.S. vs. Villafuerte and Rabano [1905], 4 Phil., 476; Son Cui vs. Guepangco, supra; U.S. vs.
Memoracion and Uri [1916], 34 Phil., 633; Teter vs. Teter [1884], 101 Ind., 129.)
Second. Petitioners contend that private respondents' reliance solely on testimonial evidence to support their claim that private
respondents had been in the continuous possession of the status of legitimate children is contrary to Art. 265 of the Civil Code
which provides that such status shall be proven by the record of birth in the Civil Register, by an authentic document or by final
judgment. But in accordance with Arts. 266 and 267, in the absence of titles indicated in Art. 265, the filiation of children may be
proven by continuous possession of the status of a legitimate child and by any other means allowed by the Rules of Court or
special laws. Thus the Civil Code provides:
Art. 266. In the absence of the titles indicated in the preceding article, the filiation shall be proved by the continuous
possession of status of a legitimate child.
Art. 267. In the absence of a record of birth, authentic document, final judgment or possession of status, legitimate
filiation may be proved by any other means allowed by the Rules of Court and special laws.
Petitioners contend that there is no justification for presenting testimonies as to the possession by private respondents of the
status of legitimate children because the Book of Marriages for the years 1928-1929 is available.
What is in issue, however, is not the marriage of Gavino and Catalina but the filiation of private respondents as their children.
The marriage of Gavino and Catalina has already been shown in the preceding discussion. The treasurer of Asturias, Cebu
certified that the records of birth of that municipality for the year 1930 could not be found, presumably because they were lost or
destroyed during the war (Exh. L). But Matias Pogoy testified that Gavino and Catalina begot three children, one of whom,
Petronilo, died at the age of six. Catalina testified that private respondents Ramonito and Generoso are her children by Gavino
Balogbog. That private respondents are the children of Gavino and Catalina Balogbog cannot therefore be doubted.
Moreover, the evidence in the record shows that petitioner Gaudioso Balogbog admitted to the police of Balamban, Cebu that
Ramonito is his nephew. As the Court of Appeals found:
Ironically, it is appellant Gaudioso himself who supplies the clincher that tips the balance in favor of the appellees. In
an investigation before the Police Investigating Committee of Balamban, Cebu, held on March 8, 1968, conducted
for the purpose of inquiring into a complaint filed by Ramonito against a patrolman of the Balamban police force,
Gaudioso testified that the complainant in that administrative case is his nephew. Excerpts from the transcript of the
proceedings conducted on that date (Exhs. "N", "N-1", "N-2", "N-3" and "N-4") read:
Atty. Kiamco May it please this investigative body.
Q. Do you know the complainant in this Administrative Case No. 1?
A. Yes I know.
Q. Why do you know him?
A. I know because he is my nephew.
Q. Are you in good terms with your nephew, the complainant?
A. Yes.
Q. Do you mean to say that you are close to him?
A. Yes. We are close.
Q. Why do you say you are close?
A. We are close because aside from the fact that he is my nephew we were also leaving (sic) in the
same house in Butuan City, and I even barrow (sic) from him money in the amount of P300.00, when I
return to Balamban, Cebu.
xxx xxx xxx
Q. Why is Ramonito Balogbog your nephew?

A. Because he is the son of my elder brother.


This admission of relationship is admissible against Gaudioso although made in another case. It is considered as a
reliable declaration against interest (Rule 130, Section 22). Significantly, Gaudioso did not try to offer any
explanation to blunt the effects of that declaration. He did not even testify during the trial. Such silence can only
mean that Ramonito is indeed the nephew of Gaudioso, the former being the son of Gavino.
WHEREFORE, the decision appealed from is AFFIRMED.
SO ORDERED.

A.M. No. MTJ-92-721 September 30, 1994


JUVY N. COSCA, EDMUNDO B. PERALTA, RAMON C. SAMBO, and APOLLO A. VILLAMORA, complainants,
vs.
HON. LUCIO P. PALAYPAYON, JR., Presiding Judge, and NELIA B. ESMERALDA-BAROY, Clerk of Court II, both of the
Municipal Trial Court of Tinambac, Camarines Sur, respondents.
Esteban R. Abonal for complainants.
Haide B. Vista-Gumba for respondents.

PER CURIAM, J.:


Complainants Juvy N. Cosca, Edmundo B. Peralta, Ramon C. Sambo, and Apollo Villamora, are Stenographer I, Interpreter I,
Clerk II, and Process Server, respectively, of the Municipal Trial Court of Tinambac, Camarines Sur. Respondents Judge Lucio
P. Palaypayon, Jr. and Nelia B. Esmeralda-Baroy are respectively the Presiding Judge and Clerk of Court II of the same court.
In an administrative complaint filed with the Office of the Court Administrator on October 5, 1992, herein respondents were
charged with the following offenses, to wit: (1) illegal solemnization of marriage; (2) falsification of the monthly reports of cases;
(3) bribery in consideration of an appointment in the court; (4) non-issuance of receipt for cash bond received; (5) infidelity in the
custody of detained prisoners; and (6) requiring payment of filing fees from exempted entities. 1
Pursuant to a resolution issued by this Court respondents filed their respective Comments. 2 A Reply to Answers of
Respondents was filed by complainants. 3 The case was thereafter referred to Executive Judge David C. Naval of the Regional
Trial Court, Naga City, for investigation report and recommendation. The case was however transferred to First Assistant
Executive Judge Antonio N. Gerona when Judge Naval inhibited himself for the reason that his wife is a cousin of respondent
Judge Palaypayon, Jr. 4
The contending versions of the parties regarding the factual antecedents of this administrative matter, as culled from the records
thereof, are set out under each particular charge against respondents.
1. Illegal solemnization of marriage
Complainants allege that respondent judge solemnized marriages even without the requisite marriage license. Thus, the
following couples were able to get married by the simple expedient of paying the marriage fees to respondent Baroy, despite the
absence of a marriage license, viz.: Alano P. Abellano and Nelly Edralin, Francisco Selpo and Julieta Carrido, Eddie Terrobias
and Maria Gacer, Renato Gamay and Maricris Belga, Arsenio Sabater and Margarita Nacario, and Sammy Bocaya and Gina
Bismonte. As a consequence, their marriage contracts (Exhibits B, C, D, F, G, and A, respectively) did not reflect any marriage
license number. In addition, respondent judge did not sign their marriage contracts and did not indicate the date of
solemnization, the reason being that he allegedly had to wait for the marriage license to be submitted by the parties which was
usually several days after the ceremony. Indubitably, the marriage contracts were not filed with the local civil registrar.
Complainant Ramon Sambo, who prepares the marriage contracts, called the attention of respondents to the lack of marriage
licenses and its effect on the marriages involved, but the latter opted to proceed with the celebration of said marriages.
Respondent Nelia Baroy claims that when she was appointed Clerk of Court II, the employees of the court were already hostile
to her, especially complainant Ramon Sambo who told her that he was filing a protest against her appointment. She avers that it
was only lately when she discovered that the court had a marriage Register which is in the custody of Sambo; that it was
Sambo who failed to furnish the parties copies of the marriage contract and to register these with the local civil registrar; and
that apparently Sambo kept these marriage contracts in preparation for this administrative case. Complainant Sambo, however,
claims that all file copies of the marriage contracts were kept by respondent Baroy, but the latter insists that she had instructed
Sambo to follow up the submission by the contracting parties of their marriage licenses as part of his duties but he failed to do
so.
Respondent Judge Palaypayon, Jr. contends that the marriage between Alano P. Abellano and Nelly Edralin falls under Article
34 of the Civil Code, hence it is exempt from the marriage license requirement; that he gave strict instructions to complainant
Sambo to furnish the couple a copy of the marriage contract and to file the same with the civil registrar, but the latter failed to do
so; that in order to solve the problem, the spouses subsequently formalized their marriage by securing a marriage license and
executing their marriage contract, a copy of which was filed with the civil registrar; that the other five marriages alluded to in the
administrative complaint were not illegally solemnized because the marriage contracts were not signed by him and they did not
contain the date and place of marriage; that copies of these marriage contracts are in the custody of complainant Sambo; that
the alleged marriage of Francisco Selpo and Julieta Carrido, Eddie Terrobias and Maria Emma Gaor, Renato Gamay and
Maricris Belga, and of Arsenio Sabater and Margarita Nacario were not celebrated by him since he refused to solemnize them in
the absence of a marriage license; that the marriage of Samy Bocaya and Gina Bismonte was celebrated even without the
requisite license due to the insistence of the parties in order to avoid embarrassment to their guests but that, at any rate, he did
not sign their marriage contract which remains unsigned up to the present.
2. Falsification of monthly report for July, 1991 regarding the number of marriages solemnized and the number of
documents notarized.
It is alleged that respondent judge made it appear that he solemnized seven (7) marriages in the month of July, 1992, when in
truth he did not do so or at most those marriages were null and void; that respondents likewise made it appear that they have
notarized only six (6) documents for July, 1992, but the Notarial Register will show that there were one hundred thirteen (113)
documents which were notarized during that month; and that respondents reported a notarial fee of only P18.50 for each
document, although in fact they collected P20.00 therefor and failed to account for the difference.
Respondent Baroy contends, however, that the marriage registry where all marriages celebrated by respondent judge are
entered is under the exclusive control and custody of complainant Ramon Sambo, hence he is the only one who should be held
responsible for the entries made therein; that the reported marriages are merely based on the payments made as solemnization
fees which are in the custody of respondent Baroy. She further avers that it is Sambo who is likewise the custodian of the
Notarial Register; that she cannot be held accountable for whatever alleged difference there is in the notarial fees because she
is liable only for those payments tendered to her by Sambo himself; that the notarial fees she collects are duly covered by
receipts; that of the P20.00 charged, P18.50 is remitted directly to the Supreme Court as part of the Judiciary Development
Fund and P150 goes to the general fund of the Supreme Court which is paid to the Municipal Treasurer of Tinambac,
Camarines Sur. Respondent theorizes that the discrepancies in the monthly report were manipulated by complainant Sambo
considering that he is the one in charge of the preparation of the monthly report.
Respondent Judge Palaypayon avers that the erroneous number of marriages celebrated was intentionally placed by
complainant Sambo; that the number of marriages solemnized should not be based on solemnization fees paid for that month
since not all the marriages paid for are solemnized in the same month. He claims that there were actually only six (6)
documents notarized in the month of July, 1992 which tallied with the official receipts issued by the clerk of court; that it is
Sambo who should be held accountable for any unreceipted payment for notarial fees because he is the one in charge of the
Notarial Register; and that this case filed by complainant Sambo is merely in retaliation for his failure to be appointed as the
clerk of court. Furthermore, respondent judge contends that he is not the one supervising or preparing the monthly report, and
that he merely has the ministerial duty to sign the same.

3. Bribery in consideration of an appointment in the court


Complainants allege that because of the retirement of the clerk of court, respondent judge forwarded to the Supreme Court the
applications of Rodel Abogado, Ramon Sambo, and Jessell Abiog. However, they were surprised when respondent Baroy
reported for duty as clerk of court on October 21, 1991. They later found out that respondent Baroy was the one appointed
because she gave a brand-new air-conditioning unit to respondent judge.
Respondent Baroy claims that when she was still in Naga City she purchased an air-conditioning unit but when she was
appointed clerk of court she had to transfer to Tinambac and, since she no longer needed the air conditioner, she decided to sell
the same to respondent judge. The installation and use thereof by the latter in his office was with the consent of the Mayor of
Tinambac.
Respondent judge contends that he endorsed all the applications for the position of clerk of court to the Supreme Court which
has the sole authority over such appointments and that he had no hand in the appointment of respondent Baroy. He contends
that the air-conditioning unit was bought from his
co-respondent on installment basis on May 29, 1992, eight (8) months after Baroy had been appointed clerk of court. He claims
that he would not be that naive to exhibit to the public as item which could not be defended as a matter of honor and prestige.
4. Cash bond issued without a receipt
It is alleged that in Criminal Case No. 5438, entitled "People vs. Mendeza, et al., "bondswoman Januaria Dacara was allowed
by respondent judge to change her property bond to cash bond; that she paid the amount of P1,000.00 but was never issued a
receipt therefor nor was it made to appear in the records that the bond has been paid; that despite the lapse of two years, the
money was never returned to the bondswoman; and that it has not been shown that the money was turned over to the Municipal
Treasurer of Tinambac.
Respondent Baroy counters that the cash bond was deposited with the former clerk of court, then turned over to the acting clerk
of court and, later, given to her under a corresponding receipt; that the cash bond is deposited with the bank; and that should
the bondswoman desire to withdraw the same, she should follow the proper procedure therefor.
Respondent judge contends that Criminal Case No. 5438 was archieved for failure of the bondsman to deliver the body of the
accused in court despite notice; and that he has nothing to do with the payment of the cash bond as this is the duty of the clerk
of court.
5. Infidelity in the custody of prisoners
Complainants contend that respondent judge usually got detention prisoners to work in his house, one of whom was Alex Alano,
who is accused in Criminal Case No. 5647 for violation of the Dangerous Drugs Act; that while Alano was in the custody of
respondent judge, the former escaped and was never recaptured; that in order to conceal this fact, the case was archived
pursuant to an order issued by respondent judge dated April 6, 1992.
Respondent judge denied the accusation and claims that he never employed detention prisoners and that he has adequate
household help; and that he had to order the case archived because it had been pending for more than six (6) months and the
accused therein remained at large.
6. Unlawful collection of docket fees
Finally, respondents are charged with collecting docket fees from the Rural Bank of Tinambac, Camarines Sur, Inc. although
such entity is exempt by law from the payment of said fees, and that while the corresponding receipt was issued, respondent
Baroy failed to remit the amount to the Supreme Court and, instead, she deposited the same in her personal account.
Respondents Baroy contends that it was Judge-Designate Felimon Montenegro (because respondent judge was on sick leave)
who instructed her to demand payment of docket fees from said rural bank; that the bank issued a check for P800.00; that she
was not allowed by the Philippine National Bank to encash the check and, instead, was instructed to deposit the same in any
bank account for clearing; that respondent deposited the same in her account; and that after the check was cleared, she
remitted P400.00 to the Supreme Court and the other P400.00 was paid to the Municipal Treasurer of Tinambac.
On the basis of the foregoing contentions, First Vice-Executive Judge Antonio N. Gerona prepared and submitted to us his
Report and Recommendations dated May 20, 1994, together with the administrative matter. We have perspicaciously reviewed
the same and we are favorably impressed by the thorough and exhaustive presentation and analysis of the facts and evidence
in said report. We commend the investigating judge for his industry and perspicacity reflected by his findings in said report
which, being amply substantiated by the evidence and supported by logical illations, we hereby approve and hereunder
reproduce at length the material portions thereof.
xxx xxx xxx
The first charge against the respondents is illegal solemnization of marriage. Judge Palaypayon is charged with
having solemnized without a marriage license the marriage of Sammy Bocaya and Gina Besmonte (Exh. A). Alano
Abellano and Nelly Edralin (Exh. B), Francisco Selpo and Julieta Carrido (Exh. C), Eddie Terrobias and Maria Emma
Gaor (Exh. D), Renato Gamay and Maricris Belga (Exh. F) and Arsenio Sabater and Margarita Nacario (Exh. G).
In all these aforementioned marriages, the blank space in the marriage contracts to show the number of the
marriage was solemnized as required by Article 22 of the Family Code were not filled up. While the contracting
parties and their witnesses signed their marriage contracts, Judge Palaypayon did not affix his signature in the
marriage contracts, except that of Abellano and Edralin when Judge Palaypayon signed their marriage certificate as
he claims that he solemnized this marriage under Article 34 of the Family Code of the Philippines. In said marriages
the contracting parties were not furnished a copy of their marriage contract and the Local Civil Registrar was not
sent either a copy of the marriage certificate as required by Article 23 of the Family Code.
The marriage of Bocaya and Besmonte is shown to have been solemnized by Judge Palaypayon without a marriage
license. The testimonies of Bocay himself and Pompeo Ariola, one of the witnesses of the marriage of Bocaya and
Besmonte, and the photographs taken when Judge Palaypayon solemnized their marriage (Exhs. K-3 to K-9)
sufficiently show that Judge Palaypayon really solemnized their marriage. Bocaya declared that they were advised
by Judge Palaypayon to return after ten (10) days after their marriage was solemnized and bring with them their
marriage license. In the meantime, they already started living together as husband and wife believing that the formal
requisites of marriage were complied with.
Judge Palaypayon denied that he solemnized the marriage of Bocaya and Besmonte because the parties allegedly
did not have a marriage license. He declared that in fact he did not sign the marriage certificate, there was no date
stated on it and both the parties and the Local Civil Registrar did not have a copy of the marriage certificate.
With respect to the photographs which show that he solemnized the marriage of Bocaya and Besmonte, Judge
Palaypayon explains that they merely show as if he was solemnizing the marriage. It was actually a simulated
solemnization of marriage and not a real one. This happened because of the pleading of the mother of one of the
contracting parties that he consent to be photographed to show that as if he was solemnizing the marriage as he
was told that the food for the wedding reception was already prepared, visitors were already invited and the place of

the parties where the reception would be held was more than twenty (20) kilometers away from the poblacion of
Tinambac.
The denial made by Judge Palaypayon is difficult to believe. The fact alone that he did not sign the marriage
certificate or contract, the same did not bear a date and the parties and the Local Civil Registrar were not furnished
a copy of the marriage certificate, do not by themselves show that he did not solemnize the marriage. His
uncorroborated testimony cannot prevail over the testimony of Bocaya and Ariola who also declared, among others,
that Bocaya and his bride were advised by Judge Palaypayon to return after ten (10) days with their marriage
license and whose credibility had not been impeached.
The pictures taken also from the start of the wedding ceremony up to the signing of the marriage certificate in front
of Judge Palaypayon and on his table (Exhs. K-3, K-3-a, K-3-b, K-3-c, K-4, K-4-a, K-4-b, K-4-c,
K-4-d, K-5, K-5-a, K-5-b, K-6, K-7, K-8, K-8-a and K-9), cannot possibly be just to show a simulated solemnization of
marriage. One or two pictures may convince a person of the explanation of Judge Palaypayon, but not all those
pictures.
Besides, as a judge it is very difficult to believe that Judge Palaypayon would allows himself to be photographed as
if he was solemnizing a marriage on a mere pleading of a person whom he did not even know for the alleged
reasons given. It would be highly improper and unbecoming of him to allow himself to be used as an instrument of
deceit by making it appear that Bocaya and Besmonte were married by him when in truth and in fact he did not
solemnize their marriage.
With respect to the marriage of Abellano and Edralin (Exh. B), Judge Palaypayon admitted that he solemnized their
marriage, but he claims that it was under Article 34 of the Family Code, so a marriage license was not required. The
contracting parties here executed a joint affidavit that they have been living together as husband and wife for almost
six (6) years already (Exh. 12; Exh. AA).
In their marriage contract which did not bear any date either when it was solemnized, it was stated that Abellano was
only eighteen (18) years, two (2) months and seven (7) days old. If he and Edralin had been living together as
husband and wife for almost six (6) years already before they got married as they stated in their joint affidavit,
Abellano must ha(ve) been less than thirteen (13) years old when he started living with Edralin as his wife and this is
hard to believe. Judge Palaypayon should ha(ve) been aware of this when he solemnized their marriage as it was
his duty to ascertain the qualification of the contracting parties who might ha(ve) executed a false joint affidavit in
order to have an instant marriage by avoiding the marriage license requirement.
On May 23, 1992, however, after this case was already filed, Judge Palaypayon married again Abellano and Edralin,
this time with a marriage license (Exh. BB). The explanation given by Judge Palaypayon why he solemnized the
marriage of the same couple for the second time is that he did not consider the first marriage he solemnized under
Article 34 of the Family Code as (a) marriage at all because complainant Ramon Sambo did not follow his instruction
that the date should be placed in the marriage certificate to show when he solemnized the marriage and that the
contracting parties were not furnished a copy of their marriage certificate.
This act of Judge Palaypayon of solemnizing the marriage of Abellano and Edralin for the second time with a
marriage license already only gave rise to the suspicion that the first time he solemnized the marriage it was only
made to appear that it was solemnized under exceptional character as there was not marriage license and Judge
Palaypayon had already signed the marriage certificate. If it was true that he solemnized the first marriage under
exceptional character where a marriage license was not required, why did he already require the parties to have a
marriage license when he solemnized their marriage for the second time?
The explanation of Judge Palaypayon that the first marriage of Abellano and Edralin was not a marriage at all as the
marriage certificate did not state the date when the marriage was solemnized and that the contracting parties were
not furnished a copy of their marriage certificate, is not well taken as they are not any of those grounds under
Article(s) 35, 36, 37 and 38 of the Family Code which declare a marriage void from the beginning. Even if no one,
however, received a copy of the marriage certificate, the marriage is still valid (Jones vs. H(o)rtiguela, 64 Phil. 179).
Judge Palaypayon cannot just absolve himself from responsibility by blaming his personnel. They are not the
guardian(s) of his official function and under Article 23 of the Family Code it is his duty to furnish the contracting
parties (a) copy of their marriage contract.
With respect to the marriage of Francisco Selpo and Julieta Carrido (Exh. C), and Arsenio Sabater and Margarita
Nacario (Exh. G), Selpo and Carrido and Sabater and Nacarcio executed joint affidavits that Judge Palaypayon did
not solemnize their marriage (Exh. 13-A and Exh. 1). Both Carrido and Nacario testified for the respondents that
actually Judge Palaypayon did not solemnize their marriage as they did not have a marriage license. On crossexamination, however, both admitted that they did not know who prepared their affidavits. They were just told,
Carrido by a certain Charito Palaypayon, and Nacario by a certain Kagawad Encinas, to just go to the Municipal
building and sign their joint affidavits there which were already prepared before the Municipal Mayor of Tinambac,
Camarines Sur.
With respect to the marriage of Renato Gamay and Maricris Belga (Exh. f), their marriage contract was signed by
them and by their two (2) witnesses, Atty. Elmer Brioso and respondent Baroy (Exhs. F-1 and F-2). Like the other
aforementioned marriages, the solemnization fee was also paid as shown by a receipt dated June 7, 1992 and
signed by respondent Baroy (Exh. F-4).
Judge Palaypayon also denied having solemnized the marriage of Gamay and Belga allegedly because there was
no marriage license. On her part, respondent Baroy at first denied that the marriage was solemnized. When she was
asked, however, why did she sign the marriage contract as a witness she answered that she thought the marriage
was already solemnized (TSN, p. 14; 10-28-93).
Respondent Baroy was, and is, the clerk of court of Judge Palaypayon. She signed the marriage contract of Gamay
and Belga as one of the two principal sponsors. Yet, she wanted to give the impression that she did not even know
that the marriage was solemnized by Judge Palaypayon. This is found very difficult to believe.
Judge Palaypayon made the same denial of having solemnized also the marriage of Terrobias and Gaor (Exh. D).
The contracting parties and their witnesses also signed the marriage contract and paid the solemnization fee, but
Judge Palaypayon allegedly did not solemnize their marriage due to lack of marriage license. Judge Palaypayon
submitted the affidavit of William Medina, Vice-Mayor of Tinambac, to corroborate his testimony (Exh. 14). Medina,
however, did not testify in this case and so his affidavit has no probative value.
Judge Palaypayon testified that his procedure and practice have been that before the contracting parties and their
witnesses enter his chamber in order to get married, he already required complainant Ramon Sambo to whom he
assigned the task of preparing the marriage contract, to already let the parties and their witnesses sign their
marriage contracts, as what happened to Gamay and Belga, and Terrobias and Gaor, among others. His purpose
was to save his precious time as he has been solemnizing marriages at the rate of three (3) to four (4) times
everyday (TSN, p. 12;
2-1-94).

This alleged practice and procedure, if true, is highly improper and irregular, if not illegal, because the contracting
parties are supposed to be first asked by the solemnizing officer and declare that they take each other as husband
and wife before the solemnizing officer in the presence of at least two (2) witnesses before they are supposed to
sign their marriage contracts (Art. 6, Family Code).
The uncorroborated testimony, however, of Judge Palaypayon as to his alleged practice and procedure before
solemnizing a marriage, is not true as shown by the picture taken during the wedding of Bocaya and Besmonte
(Exhs. K-3 to K-9) and by the testimony of respondent Baroy herself who declared that the practice of Judge
Palaypayon ha(s) been to let the contracting parties and their witnesses sign the marriage contract only after Judge
Palaypayon has solemnized their marriage (TSN, p. 53;
10-28-93).
Judge Palaypayon did not present any evidence to show also that he was really solemnizing three (3) to four (4)
marriages everyday. On the contrary his monthly report of cases for July, 1992 shows that his court had only twentyseven (27) pending cases and he solemnized only seven (7) marriages for the whole month (Exh. E). His monthly
report of cases for September, 1992 shows also that he solemnized only four (4) marriages during the whole month
(Exh. 7).
In this first charge of having illegally solemnized marriages, respondent Judge Palaypayon has presented and
marked in evidence several marriage contracts of other persons, affidavits of persons and certification issued by the
Local Civil Registrar (Exhs. 12-B to 12-H). These persons who executed affidavits, however, did not testify in this
case. Besides, the marriage contracts and certification mentioned are immaterial as Judge Palaypayon is not
charged of having solemnized these marriages illegally also. He is not charged that the marriages he solemnized
were all illegal.
The second charge against herein respondents, that of having falsified the monthly report of cases submitted to the
Supreme Court and not stating in the monthly report the actual number of documents notarized and issuing the
corresponding receipts of the notarial fees, have been sufficiently proven by the complainants insofar as the monthly
report of cases for July and September, 1992 are concerned.
The monthly report of cases of the MTC of Tinambac, Camarines Sur for July, 1992 both signed by the respondents,
show that for said month there were six (6) documents notarized by Judge Palaypayon in his capacity as Ex-Officio
Notary Public (Exhs. H to H-1-b). The notarial register of the MTC of Tinambac, Camarines Sur, however, shows that
there were actually one hundred thirteen (113) documents notarized by Judge Palaypayon for the said month (Exhs.
Q to Q-45).
Judge Palaypayon claims that there was no falsification of the monthly report of cases for July, 1992 because there
were only six (6) notarized documents that were paid (for) as shown by official receipts. He did not, however,
present evidence of the alleged official receipts showing that the notarial fee for the six (6) documetns were paid.
Besides, the monthly report of cases with respect to the number of documents notarized should not be based on
how many notarized documents were paid of the notarial fees, but the number of documents placed or recorded in
the notarial register.
Judge Palaypayon admitted that he was not personally verifying and checking anymore the correctness of the
monthly reports because he relies on his co-respondent who is the Clerk of Court and whom he has assumed to
have checked and verified the records. He merely signs the monthly report when it is already signed by respondent
Baroy.
The explanation of Judge Palaypayon is not well taken because he is required to have close supervision in the
preparation of the monthly report of cases of which he certifies as to their correctness. As a judge he is personally
responsible for the proper discharge of his functions (The Phil. Trial Lawyer's Asso. Inc. vs. Agana, Sr., 102 SCRA
517). In Nidera vs. Lazaro, 174 SCRA 581, it was held that "A judge cannot take refuge behind the inefficiency or
mismanagement of his court personnel."
On the part of respondent Baroy, she puts the blame of the falsification of the monthly report of cases on
complainant Sambo whom she allegedly assigned to prepare not only the monthly report of cases, but the
preparation and custody of marriage contracts, notarized documents and the notarial register. By her own admission
she has assigned to complainant Sambo duties she was supposed to perform, yet according to her she never
bother(ed) to check the notarial register of the court to find out the number of documents notarized in a month (TSN,
p. 30; 11-23-93).
Assuming that respondent Baroy assigned the preparation of the monthly report of cases to Sambo, which was
denied by the latter as he claims that he only typed the monthly report based on the data given to him by her, still it
is her duty to verify and check whether the report is correct.
The explanation of respondent Baroy that Sambo was the one in custody of marriage contracts, notarized
documents and notarial register, among other things, is not acceptable not only because as clerk of court she was
supposed to be in custody, control and supervision of all court records including documents and other properties of
the court (p. 32, Manual for Clerks of Court), but she herself admitted that from January, 1992 she was already in full
control of all the records of the court including receipts (TSN, p. 11; 11-23-93).
The evidence adduced in this cases in connection with the charge of falsification, however, also shows that
respondent Baroy did not account for what happened to the notarial fees received for those documents notarized
during the month of July and September, 1992. The evidence adduced in this case also sufficiently show that she
received cash bond deposits and she did not deposit them to a bank or to the Municipal Treasurer; and that she only
issued temporary receipts for said cash bond deposits.
For July, 1992 there were only six (6) documents reported to have been notarized by Judge Palaypayon although
the documents notarized for said month were actually one hundred thirteen (113) as recorded in the notarial register.
For September, 1992, there were only five (5) documents reported as notarized for that month, though the notarial
register show(s) that there were fifty-six (56) documents actually notarized. The fee for each document notarized as
appearing in the notarial register was P18.50. Respondent Baroy and Sambo declared that what was actually being
charged was P20.00. Respondent Baroy declared that P18.50 went to the Supreme Court and P1.50 was being
turned over to the Municipal Treasurer.
Baroy, however, did not present any evidence to show that she really sent to the Supreme Court the notarial fees of
P18.50 for each document notarized and to the Municipal Treasurer the additional notarial fee of P1.50. This should
be fully accounted for considering that Baroy herself declared that some notarial fees were allowed by her at her
own discretion to be paid later. Similarly, the solemnization fees have not been accounted for by Baroy considering
that she admitted that even (i)n those instances where the marriages were not solemnized due to lack of marriage
license the solemnization fees were not returned anymore, unless the contracting parties made a demand for their
return. Judge Palaypayon declared that he did not know of any instance when solemnization fee was returned when
the marriage was not solemnized due to lack of marriage license.

Respondent Baroy also claims that Ramon Sambo did not turn over to her some of the notarial fees. This is difficult
to believe. It was not only because Sambo vehemently denied it, but the minutes of the conference of the personnel
of the MTC of Tinambac dated January 20, 1992 shows that on that date Baroy informed the personnel of the court
that she was taking over the functions she assigned to Sambo, particularly the collection of legal fees (Exh. 7). The
notarial fees she claims that Sambo did not turn over to her were for those documents notarized (i)n July and
September, 1992 already. Besides there never was any demand she made for Sambo to turn over some notarial
fees supposedly in his possession. Neither was there any memorandum she issued on this matter, in spite of the
fact that she has been holding meetings and issuing memoranda to the personnel of the court (Exhs. V, W, FF, FF-1,
FF-2, FF-3; Exhs. 4-A (supplement(s), 5-8, 6-S, 7-S and 8-S).
It is admitted by respondent Baroy that on October 29, 1991 a cash bond deposit of a certain Dacara in the amount
of One Thousand (P1,000.00) Pesos was turned over to her after she assumed office and for this cash bond she
issued only a temporary receipt (Exh. Y). She did not deposit this cash bond in any bank or to the Municipal
Treasurer. She just kept it in her own cash box on the alleged ground that the parties in that case where the cash
bond was deposited informed her that they would settle the case amicably.
Respondent Baroy declared that she finally deposited the aforementioned cash bond of One Thousand (P1,000.00)
Pesos with the Land Bank of the Philippines (LBP) in February, 1993, after this administrative case was already filed
(TSN, pp. 27-28; 12-22-93). The Pass Book, however, shows that actually Baroy opened an account with the LBP,
Naga Branch, only on March 26, 1993 when she deposited an amount of Two Thousand (P2,000.00) Pesos (Exhs. 8
to 8-1-a). She claims that One Thousand (P1,000.000) Pesos of the initial deposit was the cash bond of Dacara. If it
were true, it was only after keeping to herself the cash bond of One Thousand (P1,000.00) Pesos for around one
year and five months when she finally deposited it because of the filing of this case.
On April 29, 1993, or only one month and two days after she finally deposited the One Thousand (P1,000.00) Pesos
cash bond of Dacara, she withdrew it from the bank without any authority or order from the court. It was only on July
23, 1993, or after almost three (3) months after she withdrew it, when she redeposited said cash bond (TSN, p. 6; 14-94).
The evidence presented in this case also show that on February 28, 1993 respondent Baroy received also a cash
bond of Three Thousand (P3,000.00) Pesos from a certain Alfredo Seprones in Crim. Case No. 5180. For this cash
bond deposit, respondent Baroy issued only an annumbered temporary receipt (Exh. X and X-1). Again Baroy just
kept this Three Thousand (P3,000.00) Pesos cash bond to herself. She did not deposit it either (in) a bank or (with)
the Municipal Treasurer. Her explanation was that the parties in Crim. Case No. 5180 informed her that they would
settle the case amicably. It was on April 26, 1993, or almost two months later when Judge Palaypayon issued an
order for the release of said cash bond (Exh. 7).
Respondent Baroy also admitted that since she assumed office on October 21, 1991 she used to issue temporary
receipt only for cash bond deposits and other payments and collections she received. She further admitted that
some of these temporary receipts she issued she failed to place the number of the receipts such as that receipt
marked Exhibit X (TSN, p. 35; 11-23-93). Baroy claims that she did not know that she had to use the official receipts
of the Supreme Court. It was only from February, 1993, after this case was already filed, when she only started
issuing official receipts.
The next charge against the respondents is that in order to be appointed Clerk of Court, Baroy gave Judge
Palaypayon an air conditioner as a gift. The evidence adduced with respect to this charge, show that on August 24,
1991 Baroy bought an air conditioner for the sum of Seventeen Thousand Six Hundred (P17,600.00) Pesos (Exhs. I
and I-1). The same was paid partly in cash and in check (Exhs. I-2 and I-3). When the air conditioner was brought to
court in order to be installed in the chamber of Judge Palaypayon, it was still placed in the same box when it was
bought and was not used yet.
The respondents claim that Baroy sold it to Judge Palaypayon for Twenty Thousand (P20,00.00) Pesos on
installment basis with a down payment of Five Thousand (P5,000.00) Pesos and as proof thereof the respondents
presented a typewritten receipt dated May 29, 1993 (Exh. 22). The receipt was signed by both respondents and by
the Municipal Mayor of Tinambac, Camarines Sur and another person as witness.
The alleged sale between respondents is not beyond suspicion. It was bought by Baroy at a time when she was
applying for the vacant position of Clerk of Court (to) which she was eventually appointed in October, 1991. From
the time she bought the air conditioner on August 24, 1991 until it was installed in the office of Judge Palaypayon it
was not used yet. The sale to Judge Palaypayon was only evidenced by a mere typewritten receipt dated May 29,
1992 when this case was already filed. The receipt could have been easily prepared. The Municipal Mayor of
Tinambac who signed in the receipt as a witness did not testify in this case. The sale is between the Clerk of Court
and the Judge of the same court. All these circumstances give rise to suspicion of at least impropriety. Judges
should avoid such action as would subject (them) to suspicion and (their) conduct should be free from the
appearance of impropriety (Jaagueta vs. Boncasos, 60 SCRA 27).
With respect to the charge that Judge Palaypayon received a cash bond deposit of One Thousand (P1,000.00)
Pesos from Januaria Dacara without issuing a receipt, Dacara executed an affidavit regarding this charge that Judge
Palaypayon did not give her a receipt for the P1,000.00 cash bond she deposited (Exh. N). Her affidavit, however,
has no probative value as she did not show that this cash bond of P1,000.00 found its way into the hands of
respondent Baroy who issued only a temporary receipt for it and this has been discussed earlier.
Another charge against Judge Palaypayon is the getting of detention prisoners to work in his house and one of them
escaped while in his custody and was never found again. To hide this fact, the case against said accused was
ordered archived by Judge Palaypayon. The evidence adduced with respect to this particular charge, show that in
Crim. Case No. 5647 entitled People vs. Stephen Kalaw, Alex Alano and Allan Adupe, accused Alex Alano and Allan
Adupe were arrested on April 12, 1991 and placed in the municipal jail of Tinambac, Camarines Sur (Exhs. 0, 0-1, 02 and 0-3; Exh. 25). The evidence presented that Alex Alano was taken by Judge Palaypayon from the municipal jail
where said accused was confined and that he escaped while in custody of Judge Palaypayon is solely testimonial,
particularly that of David Ortiz, a former utility worker of the MTC of Tinambac.
Herein investigator finds said evidence not sufficient. The complainants should have presented records from the
police of Tinambac to show that Judge Palaypayon took out from the municipal jail Alex Alano where he was under
detention and said accused escaped while in the custody of Judge Palaypayon.
The order, however, of Judge Palaypayon dated April 6, 1992 in Crim. Case No. 5047 archiving said case appears
to be without basis. The order states: "this case was filed on April 12, 1991 and the records show that the warrant of
arrest (was) issued against the accused, but up to this moment there is no return of service for the warrant of arrest
issued against said accused" (Exh. 0-4). The records of said case, however, show that in fact there was a return of
the service of the warrant of arrest dated April 12, 1991 showing that Alano and Adupe were arrested (Exh. 0-3).
Judge Palaypayon explained that his order dated April 6, 1992 archiving Crim. Case No. 5047 referred only to one of
the accused who remained at large. The explanation cannot be accepted because the two other accused, Alano and
Adupe, were arrested. Judge Palaypayon should have issued an order for the arrest of Adupe who allegedly jumped

bail, but Alano was supposed to be confined in the municipal jail if his claim is true that he did not take custody of
Alano.
The explanation also of Judge Palaypayon why he ordered the case archived was because he heard from the police
that Alano escaped. This explanation is not acceptable either. He should ha(ve) set the case and if the police failed
to bring to court Alano, the former should have been required to explain in writing why Alano was not brought to
court. If the explanation was that Alano escaped from jail, he should have issued an order for his arrest. It is only
later on when he could not be arrested when the case should have been ordered archived. The order archiving this
case for the reason that he only heard that Alano escaped is another circumstance which gave rise to a suspicion
that Alano might have really escaped while in his custody only that the complainants could not present records or
other documentary evidence to prove the same.
The last charge against the respondents is that they collected filing fees on collection cases filed by the Rural Bank
of Tinambac, Camarines Sur which was supposed to be exempted in paying filing fees under existing laws and that
the filing fees received was deposited by respondent Baroy in her personal account in the bank. The evidence
presented show that on February 4, 1992 the Rural Bank of Tinambac filed ten (10) civil cases for collection against
farmers and it paid the total amount of Four Hundred (P400.00) Pesos representing filing fees. The complainants
cited Section 14 of Republic Act 720, as amended, which exempts Rural Banks (from) the payment of filing fees on
collection of sums of money cases filed against farmers on loans they obtained.
Judge Palaypayon, however, had nothing to do with the payment of the filing fees of the Rural Bank of Tinambac as
it was respondent Baroy who received them and besides, on February 4, 1992, he was on sick leave. On her part
Baroy claims that the bank paid voluntarily the filing fees. The records, however, shows that respondent Baroy sent
a letter to the manager of the bank dated January 28, 1992 to the effect that if the bank would not pay she would
submit all Rural Bank cases for dismissal (Annex 6, comment by respondent Baroy).
Respondent Baroy should have checked whether the Rural Bank of Tinambac was really exempt from the payment
of filing fees pursuant to Republic Act 720, as amended, instead of threatening the bank to have its cases be
submitted to the court in order to have them dismissed. Here the payment of the filing fees was made on February 4,
1992, but the Four Hundred (P400.00) Pesos was only turned over to the Municipal Treasurer on March 12, 1992.
Here, there is an undue delay again in complying with her obligation as accountable officer.
In view of the foregoing findings that the evidence presented by the complainants sufficiently show that respondent
Judge Lucio P. Palaypayon, Jr. had solemnized marriages, particularly that of Sammy Bocaya and Gina Besmonte,
without a marriage license, and that it having been shown that he did not comply with his duty in closely supervising
his clerk of court in the preparation of the monthly report of cases being submitted to the Supreme Court, particularly
for the months of July and September, 1992 where it has been proven that the reports for said two (2) months were
falsified with respect to the number of documents notarized, it is respectfully recommended that he be imposed a
fine of TEN THOUSAND (P10,000.00) PESOS with a warning that the same or similar offenses will be more
severely dealt with.
The fact that Judge Palaypayon did not sign the marriage contracts or certificates of those marriages he solemnized
without a marriage license, there were no dates placed in the marriage contracts to show when they were
solemnized, the contracting parties were not furnished their marriage contracts and the Local Civil Registrar was not
being sent any copy of the marriage contract, will not absolve him from liability. By solemnizing alone a marriage
without a marriage license he as the solemnizing officer is the one responsible for the irregularity in not complying
(with) the formal requ(i)sites of marriage and under Article 4(3) of the Family Code of the Philippines, he shall be
civilly, criminally and administratively liable.
Judge Palaypayon is likewise liable for his negligence or failure to comply with his duty of closely supervising his
clerk of court in the performance of the latter's duties and functions, particularly the preparation of the monthly report
of cases (Bendesula vs. Laya, 58 SCRA 16). His explanation that he only signed the monthly report of cases only
when his clerk of court already signed the same, cannot be accepted. It is his duty to closely supervise her, to check
and verify the records if the monthly reports prepared by his clerk of court do not contain false statements. It was
held that "A judge cannot take refuge behind the inefficiency or incompetence of court personnel (Nidua vs. Lazaro,
174 SCRA 158).
In view also of the foregoing finding that respondent Nelia Esmeralda-Baroy, the clerk of court of the Municipal Trial
Court of Tinambac, Camarines Sur, has been found to have falsified the monthly report of cases for the months of
July and September, 1992 with respect to the number of documents notarized, for having failed to account (for) the
notarial fees she received for said two (2) months period; for having failed to account (for) the solemnization fees of
those marriages allegedly not solemnized, but the solemnization fees were not returned; for unauthorized issuance
of temporary receipts, some of which were issued unnumbered; for receiving the cash bond of Dacara on October
29, 1991 in the amount of One Thousand (P1,000.00) Pesos for which she issued only a temporary receipt (Exh. Y)
and for depositing it with the Land Bank of the Philippines only on March 26, 1993, or after one year and five months
in her possession and after this case was already filed; for withdrawing said cash bond of One Thousand
(P1,000.00) Pesos on April 29, 1993 without any court order or authority and redepositing it only on July 23, 1993;
for receiving a cash bond of Three Thousand (P3,000.00) Pesos from Alfredo Seprones in Crim. Case No. 5180,
MTC, Tinambac, Camarines Sur, for which she issued only an unnumbered temporary receipt (Exhs. X and X-1) and
for not depositing it with a bank or with the Municipal Treasurer until it was ordered released; and for requiring the
Rural Bank of Tinambac, Camarines Sur to pay filing fees on February 4, 1992 for collection cases filed against
farmers in the amount of Four Hundred (P400.00) Pesos, but turning over said amount to the Municipal Treasurer
only on March 12, 1992, it is respectfully recommended that said respondent clerk of court Nelia Esmeralda-Baroy
be dismissed from the service.
It is provided that "Withdrawal of court deposits shall be by the clerk of court who shall issue official receipt to the
provincial, city or municipal treasurer for the amount withdrawn. Court deposits cannot be withdrawn except by order
of the court, . . . ." (Revised Manual of Instructions for Treasurers, Sec. 183, 184 and 626; p. 127, Manual for Clerks
of Court). A circular also provides that the Clerks of Court shall immediately issue an official receipt upon receipt of
deposits from party litigants and thereafter deposit intact the collection with the municipal, city or provincial treasurer
and their deposits, can only be withdrawn upon proper receipt and order of the Court (DOJ Circular No. 52, 26 April
1968; p. 136, Manual for Clerks of Court). Supreme Court Memorandum Circular No. 5, 25 November 1982, also
provides that "all collections of funds of fiduciary character including rental deposits, shall be deposited immediately
by the clerk of court concerned upon receipt thereof with City, Municipal or Provincial Treasurer where his court is
located" and that "no withdrawal of any of such deposits shall be made except upon lawful order of the court
exercising jurisdiction over the subject matter.
Respondent Baroy had either failed to comply with the foregoing circulars, or deliberately disregarded, or even
intentionally violated them. By her conduct, she demonstrated her callous unconcern for the obligations and
responsibility of her duties and functions as a clerk of court and accountable officer. The gross neglect of her duties
shown by her constitute(s) a serious misconduct which warrant(s) her removal from office. In the case of Belen P.
Ferriola vs. Norma Hiam, Clerk of Court, MTCC, Branch I, Batangas City; A.M. No. P-90-414; August 9, 1993, it was
held that "The clerk of court is not authorized to keep funds in his/her custody; monies received by him/her shall be
deposited immediately upon receipt thereof with the City, Municipal or Provincial Treasurer. Supreme Court Circular
Nos. 5 dated November 25, 1982 and 5-A dated December 3, 1982. Respondent Hiam's failure to remit the cash

bail bonds and fine she collected constitutes serious misconduct and her misappropriation of said funds constitutes
dishonesty. "Respondent Norma Hiam was found guilty of dishonesty and serious misconduct prejudicial to the best
interest of the service and (the Court) ordered her immediate dismissal (from) the service.
xxx xxx xxx
We here emphasize once again our adjuration that the conduct and behavior of everyone connected with an office charged with
the dispensation of justice, from the presiding judge to the lowliest clerk, should be circumscribed with the heavy burden of
responsibility. His conduct, at all times, must not only be characterized by propriety and decorum but, above all else, must be
beyond suspicion. Every employee should be an example of integrity, uprightness and honesty. 5 Integrity in a judicial office is
more than a virtue, it is a necessity. 6 It applies, without qualification as to rank or position, from the judge to the least of its
personnel, they being standard-bearers of the exacting norms of ethics and morality imposed upon a Court of justice.
On the charge regarding illegal marriages the Family Code pertinently provides that the formal requisites of marriage are, inter
alia, a valid marriage license except in the cases provided for therein. 7 Complementarily, it declares that the absence of any of
the essential or formal requisites shall generally render the marriage void ab initio and that, while an irregularity in the formal
requisites shall not affect the validity of the marriage, the party or parties responsible for the irregularity shall be civilly, criminally
and administratively liable. 8
The civil aspect is addressed to the contracting parties and those affected by the illegal marriages, and what we are providing
for herein pertains to the administrative liability of respondents, all without prejudice to their criminal responsibility. The Revised
Penal Code provides that "(p)riests or ministers of any religious denomination or sect, or civil authorities who shall perform or
authorize any illegal marriage ceremony shall be punished in accordance with the provisions of the Marriage Law." 9 This is of
course, within the province of the prosecutorial agencies of the Government.
The recommendation with respect to the administrative sanction to be imposed on respondent judge should, therefore, be
modified. For one, with respect to the charge of illegal solemnization of marriages, it does appear that he had not taken to heart,
but actually trifled with, the law's concern for the institution of marriage and the legal effects flowing from civil status. This, and
his undeniable participation in the other offenses charged as hereinbefore narrated in detail, approximate such serious degree
of misconduct and of gross negligence in the performance of judicial duties as to ineludibly require a higher penalty.
WHEREFORE, the Court hereby imposes a FINE of P20,000.00 on respondent Judge Lucio P. Palaypayon. Jr., with a stern
warning that any repetition of the same or similar offenses in the future will definitely be severely dealt with. Respondent Nelia
Esmeralda-Baroy is hereby DISMISSED from the service, with forfeiture of all retirement benefits and with prejudice to
employment in any branch, agency or instrumentality of the Government, including government-owned or controlled
corporations.
Let copies of this decision be spread on their records and furnished to the Office of the Ombudsman for appropriate action.
SO ORDERED.

G.R. No. 85140 May 17, 1990


TOMAS EUGENIO, SR., petitioner,
vs.
HON. ALEJANDRO M. VELEZ, Presiding Judge, Regional Trial Court, Branch 20, Cagayan de Oro City, DEPUTY
SHERIFF JOHNSON TAN, JR., Deputy Sheriff of Branch 20, Regional Trial Court, Cagayan de Oro City, and the Private
Respondents, the petitioners in Sp. Proc. No. 88-55, for "Habeas Corpus", namely: CRISANTA VARGAS-SANCHEZ,
SANTOS and NARCISA VARGAS-BENTULAN, respondents.
G.R. No. 86470 May 17, 1990.
TOMAS EUGENIO, petitioner-appellant,
vs.
HON. ALEJANDRO M. VELEZ, Presiding Judge, Regional Trial Court, Branch 20, Cagayan de Oro City, CRISANTA
VARGAS-SANCHEZ, FELIX VARGAS, ERNESTO VARGAS, NATIVIDAD VARGAS-CAGAPE, NENITA VARGAS-CADENAS,
LUDIVINA VARGAS-DE LOS SANTOS and NARCISA VARGAS-BENTULAN, respondents-appellees.
Maximo G. Rodriguez for petitioner.
Erasmo B. Damasing and Oliver Asis Improso for respondents.

PADILLA, J.:
On 5 October 1988, petitioner came to this Court with a petition for certiorari and prohibition with application for restraining order
and/or injunction (docketed as G.R. No. 85140) seeking to enjoin respondent Judge from proceeding with the Habeas
Corpus case (Sp. Proc. No. 88- 55, RTC, Branch 20, Cagayan de Oro City), * the respondent Sheriff from enforcing and
implementing the writ and orders of the respondent Judge dated 28, 29, and 30 September 1988, and to declare said writ and
orders as null and void. In a resolution issued on 11 October 1988, this Court required comment from the respondents on the
petition but denied the application for a temporary restraining order.
The records disclose the following:
Unaware of the death on 28 August 1988 of (Vitaliana Vargas Vitaliana for brevity), her full blood brothers and sisters, herein
private respondents (Vargases', for brevity) filed on 27 September 1988, a petition for habeas corpus before the RTC of
Misamis Oriental (Branch 20, Cagayan de Oro City) alleging that Vitaliana was forcibly taken from her residence sometime in
1987 and confined by herein petitioner in his palacial residence in Jasaan, Misamis Oriental. Despite her desire to escape,
Vitaliana was allegedly deprived of her liberty without any legal authority. At the time the petition was filed, it was alleged that
Vitaliana was 25 years of age, single, and living with petitioner Tomas Eugenio.
The respondent court in an order dated 28 September 1988 issued the writ of habeas corpus, but the writ was returned
unsatisfied. Petitioner refused to surrender the body of Vitaliana (who had died on 28 August 1988) to the respondent sheriff,
reasoning that a corpse cannot be the subject of habeas corpus proceedings; besides, according to petitioner, he had already
obtained a burial permit from the Undersecretary of the Department of Health, authorizing the burial at the palace quadrangle of
the Philippine Benevolent Christian Missionary, Inc. (PBCM), a registered religious sect, of which he (petitioner) is the Supreme
President and Founder.
Petitioner also alleged that Vitaliana died of heart failure due to toxemia of pregnancy in his residence on 28 August 1988. As
her common law husband, petitioner claimed legal custody of her body. These reasons were incorporated in an explanation filed
before the respondent court. Two (2) orders dated 29 and 30 September 1988 were then issued by respondent court, directing
delivery of the deceased's body to a funeral parlor in Cagayan de Oro City and its autopsy.
Petitioner (as respondent in the habeas corpus proceedings) filed an urgent motion to dismiss the petition therein, claiming lack
of jurisdiction of the court over the nature of the action under sec. 1(b) of Rule 16 in relation to sec. 2, Rule 72 of the Rules of
Court. 1 A special proceeding for habeas corpus, petitioner argued, is not applicable to a dead person but extends only to all
cases of illegal confinement or detention of a live person.
Before resolving the motion to dismiss, private respondents (as petitioners below) were granted leave to amend their
petition. 2 Claiming to have knowledge of the death of Vitaliana only on 28 September 1988 (or after the filing of thehabeas
corpus petition), private respondents (Vargases') alleged that petitioner Tomas Eugenia who is not in any way related to
Vitaliana was wrongfully interfering with their (Vargases') duty to bury her. Invoking Arts. 305 and 308 of the Civil Code, 3the
Vargases contended that, as the next of kin in the Philippines, they are the legal custodians of the dead body of their sister
Vitaliana. An exchange of pleadings followed. The motion to dismiss was finally submitted for resolution on 21 October 1988.
In the absence of a restraining order from this Court, proceedings continued before the respondent court; the body was placed
in a coffin, transferred to the Greenhills Memorial Homes in Cagayan de Oro City, viewed by the presiding Judge of respondent
court, and examined by a duly authorized government pathologist. 4
Denying the motion to dismiss filed by petitioner, the court a quo held in an order, 5 dated 17 November 1988, that:
It should be noted from the original petition, to the first amended petition, up to the second amended petition that the
ultimate facts show that if the person of Vitaliana Vargas turns out to be dead then this Court is being prayed to
declare the petitioners as the persons entitled to the custody, interment and/or burial of the body of said deceased.
The Court, considering the circumstance that Vitaliana Vargas was already dead on August 28, 1988 but only
revealed to the Court on September 29, 1988 by respondent's counsel, did not lose jurisdiction over the nature and
subject matter of this case because it may entertain this case thru the allegations in the body of the petition on the
determination as to who is entitled to the custody of the dead body of the late Vitaliana Vargas as well as the burial
or interment thereof, for the reason that under the provisions of Sec. 19 of Batas Pambansa Blg. 129, which reads
as follows:
Sec. 19. Jurisdiction in civil cases. Regional Trial Courts shall exercise exclusive original jurisdiction:
(1) In all civil actions in which the subject of the litigation is incapable of pecuniary estimation;
xxx xxx xxx
(5) In all actions involving the contract of marriage and marital relations;
(6) In all cases not within the exclusive jurisdiction of any court, tribunal, person or body exercising
judicial or quasi-judicial functions:
xxx xxx xxx
it so provides that the Regional Trial Court has exclusive original jurisdiction to try this case. The authority to try the
issue of custody and burial of a dead person is within the lawful jurisdiction of this Court because of Batas

Pambansa Blg. 129 and because of the allegations of the pleadings in this case, which are enumerated in Sec. 19,
pars. 1, 5 and 6 of Batas Pambansa Blg. 129.
Thereafter, the court a quo proceeded as in or civil cases and, in due course, rendered a decision on 17 January
1989, 6 resolving the main issue of whether or not said court acquired jurisdiction over the case by treating it as an action for
custody of a dead body, without the petitioners having to file a separate civil action for such relief, and without the Court first
dismissing the original petition for habeas corpus.
Citing Sections 19 and 20 of Batas Pambansa Blg. 129 (the Judiciary Reorganization Act of 1981), 7 Sections 5 and 6 of Rule
135 of the Rules of Court 8 Articles 305 and 308 in relation to Article 294 of the Civil Code and Section 1104 of the Revised
Administrative Code, 9 the decision stated:
. . . . By a mere reading of the petition the court observed that the allegations in the original petition as well as in the
two amended petitions show that Vitaliana Vargas has been restrained of her liberty and if she were dead then relief
was prayed for the custody and burial of said dead person. The amendments to the petition were but elaborations
but the ultimate facts remained the same, hence, this court strongly finds that this court has ample jurisdiction to
entertain and sit on this case as an action for custody and burial of the dead body because the body of the petition
controls and is binding and since this case was raffled to this court to the exclusion of all other courts, it is the
primary duty of this court to decide and dispose of this case. . . . . 10
Satisfied with its jurisdiction, the respondent court then proceeded to the matter of rightful custody over the dead body, (for
purposes of burial thereof). The order of preference to give support under Art. 294 was used as the basis of the award. Since
there was no surviving spouse, ascendants or descendants, the brothers and sisters were preferred over petitioner who was
merely a common law spouse, the latter being himself legally married to another woman. 11
On 23 January 1989, a new petition for review with application for a temporary restraining order and/or preliminary injunction
was filed with this Court (G.R. No. 86470). Raised therein were pure questions of law, basically Identical to those raised in the
earlier petition (G.R. No. 85140); hence, the consolidation of both cases. 12 On 7 February 1989, petitioner filed an urgent
motion for the issuance of an injunction to maintain status quo pending appeal, which this Court denied in a resolution dated 23
February 1989 stating that "Tomas Eugenio has so far failed to sufficiently establish a clear legal right to the custody of the dead
body of Vitaliana Vargas, which now needs a decent burial." The petitions were then submitted for decision without further
pleadings.
Between the two (2) consolidated petitions, the following issues are raised:
1. propriety of a habeas corpus proceeding under Rule 102 of the Rules of Court to recover custody of the dead
body of a 25 year old female, single, whose nearest surviving claimants are full blood brothers and sisters and a
common law husband.
2. jurisdiction of the RTC over such proceedings and/or its authority to treat the action as one for
custody/possession/authority to bury the deceased/recovery of the dead.
3. interpretation of par. 1, Art. 294 of the Civil Code (Art. 199 of the new Family Code) which states:
Art. 294. The claim for support, when proper and two or more persons are obliged to give it, shall be
made in the following order:
(1) From the spouse;
xxx xxx xxx
Section 19, Batas Pambansa Blg. 129 provides for the exclusive original jurisdiction of the Regional Trial Courts over civil
cases. Under Sec. 2, Rule 102 of the Rules of Court, the writ of habeas corpus may be granted by a Court of First Instance
(now Regional Trial Court). It is an elementary rule of procedure that what controls is not the caption of the complaint or petition;
but the allegations therein determine the nature of the action, and even without the prayer for a specific remedy, proper relief
may nevertheless be granted by the court if the facts alleged in the complaint and the evidence introduced so warrant. 13
When the petition for habeas corpus was filed before the court a quo, it was not certain whether Vitaliana was dead or alive.
While habeas corpus is a writ of right, it will not issue as a matter of course or as a mere perfimetory operation on the filing of
the petition. Judicial discretion is exercised in its issuance, and such facts must be made to appear to the judge to whom the
petition is presented as, in his judgment, prima facie entitle the petitioner to the writ. 14 While the court may refuse to grant the
writ if the petition is insufficient in form and substance, the writ should issue if the petition complies with the legal requirements
and its averments make a prima facie case for relief. However, a judge who is asked to issue a writ of habeas corpus need not
be very critical in looking into the petition for very clear grounds for the exercise of this jurisdiction. The latter's power to make
full inquiry into the cause of commitment or detention will enable him to correct any errors or defects in the petition. 15
In Macazo and Nunez vs. Nunez, 16 the Court frowned upon the dismissal of a habeas corpus petition filed by a brother to
obtain custody of a minor sister, stating:
All these circumstances notwithstanding, we believe that the case should not have been dismissed. The court below
should not have overlooked that by dismissing the petition, it was virtually sanctioning the continuance of an
adulterous and scandalous relation between the minor and her married employer, respondent Benildo Nunez
against all principles of law and morality. It is no excuse that the minor has expressed preference for remaining with
said respondent, because the minor may not chose to continue an illicit relation that morals and law repudiate.
xxx xxx xxx
The minor's welfare being the paramount consideration, the court below should not allow the technicality, that Teofilo
Macazo was not originally made a party, to stand in the way of its giving the child full protection. Even in a habeas
corpus proceeding the court had power to award temporary custody to the petitioner herein, or some other suitable
person, after summoning and hearing all parties concerned. What matters is that the immoral situation disclosed by
the records be not allowed to continue. 17
After the fact of Vitaliana's death was made known to the petitioners in the habeas corpus proceedings,amendment of the
petition for habeas corpus, not dismissal, was proper to avoid multiplicity of suits. Amendments to pleadings are generally
favored and should be liberally allowed in furtherance of justice in order that every case may so far as possible be determined
on its real facts and in order to expedite the trial of cases or prevent circuity of action and unnecessary expense, unless there
are circumstances such as inexcusable delay or the taking of the adverse party by surprise or the like, which justify a refusal of
permission to amend. 18 As correctly alleged by respondents, the writ of habeas corpus as a remedy became moot and
academic due to the death of the person allegedly restrained of liberty, but the issue of custody remained, which the court a quo
had to resolve.
Petitioner claims he is the spouse contemplated under Art. 294 of the Civil Code, the term spouse used therein not being
preceded by any qualification; hence, in the absence of such qualification, he is the rightful custodian of Vitaliana's body.
Vitaliana's brothers and sisters contend otherwise. Indeed, Philippine Law does not recognize common law marriages. A man
and woman not legally married who cohabit for many years as husband and wife, who represent themselves to the public as

husband and wife, and who are reputed to be husband and wife in the community where they live may be considered legally
mauled in common law jurisdictions but not in the Philippines. 19
While it is true that our laws do not just brush aside the fact that such relationships are present in our society, and that they
produce a community of properties and interests which is governed by law, 20 authority exists in case law to the effect that such
form of co-ownership requires that the man and woman living together must not in any way be incapacitated to contract
marriage. 21 In any case, herein petitioner has a subsisting marriage with another woman, a legal impediment which disqualified
him from even legally marrying Vitaliana. In Santero vs. CFI of Cavite, 22 ,the Court, thru Mr. Justice Paras, interpreting Art. 188
of the Civil Code (Support of Surviving Spouse and Children During Liquidation of Inventoried Property) stated: "Be it noted
however that with respect to 'spouse', the same must be the legitimate 'spouse' (not common-law spouses)."
There is a view that under Article 332 of the Revised Penal Code, the term "spouse" embraces common law relation for
purposes of exemption from criminal liability in cases of theft, swindling and malicious mischief committed or caused mutually by
spouses. The Penal Code article, it is said, makes no distinction between a couple whose cohabitation is sanctioned by a
sacrament or legal tie and another who are husband and wife de facto. 23 But this view cannot even apply to the facts of the
case at bar. We hold that the provisions of the Civil Code, unless expressly providing to the contrary as in Article 144, when
referring to a "spouse" contemplate a lawfully wedded spouse. Petitioner vis-a-vis Vitaliana was not a lawfully-wedded spouse
to her; in fact, he was not legally capacitated to marry her in her lifetime.
Custody of the dead body of Vitaliana was correctly awarded to her surviving brothers and sisters (the Vargases). Section 1103
of the Revised Administrative Code provides:
Sec. 1103. Persons charged with duty of burial. The immediate duty of burying the body of a deceased person,
regardless of the ultimate liability for the expense thereof, shall devolve upon the persons hereinbelow specified:
xxx xxx xxx
(b) If the deceased was an unmarried man or woman, or a child, and left any kin, the duty of burial shall
devolve upon the nearest of kin of the deceased, if they be adults and within the Philippines and in
possession of sufficient means to defray the necessary expenses.
WHEREFORE, the decision appealed from is AFFIRMED. Both petitions are hereby DISMISSED. No Costs.
SO ORDERED.

RESTITUTO M. ALCANTARA,
Petitioner,

G.R. No. 167746


Present:
YNARES-SANTIAGO, J.,
Chairperson,
AUSTRIA-MARTINEZ,
CHICO-NAZARIO,
NACHURA, and
REYES, JJ.

- versus -

ROSITA A. ALCANTARA and


Promulgated:
HON. COURT OF APPEALS,
Respondents
August 28, 2007
.
x- - - - - - - - - - - - - - - - - - - - - - - - - - - - - - - - - - - - - - - - - - - - - - - - - -x

DECISION
CHICO-NAZARIO, J.:

Before this Court is a Petition for Review on Certiorari filed by petitioner Restituto Alcantara assailing the Decision[1] of the
Court of Appeals dated 30 September 2004 in CA-G.R. CV No. 66724 denying petitioners appeal and affirming the decision [2] of
the Regional Trial Court (RTC) of Makati City, Branch 143, in Civil Case No. 97-1325 dated 14 February 2000, dismissing his
petition for annulment of marriage.
The antecedent facts are:
A petition for annulment of marriage [3] was filed by petitioner against respondent Rosita A. Alcantara alleging that on 8
December 1982 he and respondent, without securing the required marriage license, went to the Manila City Hall for the purpose
of looking for a person who could arrange a marriage for them. They met a person who, for a fee, arranged their wedding
before a certain Rev. Aquilino Navarro, a Minister of the Gospel of the CDCC BR Chapel. [4] They got married on the same
day, 8 December 1982. Petitioner and respondent went through another marriage ceremony at the San Jose
de Manuguit Church in Tondo,Manila, on 26 March 1983. The marriage was likewise celebrated without the parties securing a
marriage license. The alleged marriage license, procured in Carmona, Cavite, appearing on the marriage contract, is a sham,
as neither party was a resident ofCarmona, and they never went to Carmona to apply for a license with the local civil registrar of
the said place. On 14 October 1985, respondent gave birth to their child Rose Ann Alcantara. In 1988, they parted ways and
lived separate lives. Petitioner prayed that after due hearing, judgment be issued declaring their marriage void and ordering the
Civil Registrar to cancel the corresponding marriage contract [5] and its entry on file.[6]
Answering petitioners petition for annulment of marriage, respondent asserts the validity of their marriage and maintains
that there was a marriage license issued as evidenced by a certification from the Office of the Civil Registry
of Carmona, Cavite. Contrary to petitioners representation, respondent gave birth to their first child named Rose
Ann Alcantara on 14 October 1985 and to another daughter named Rachel Ann Alcantara on 27 October 1992.[7] Petitioner has
a mistress with whom he has three children. [8] Petitioner only filed the annulment of their marriage to evade prosecution
for concubinage.[9] Respondent,

in

fact,

has

filed

case

for concubinage against

petitioner

before

the Metropolitan Trial Court of Mandaluyong City, Branch 60.[10] Respondent prays that the petition for annulment of marriage
be denied for lack of merit.
On 14 February 2000, the RTC of Makati City, Branch 143, rendered its Decision disposing as follows:
The foregoing considered, judgment is rendered as follows:
1.

The Petition is dismissed for lack of merit;

2.
Petitioner is ordered to pay respondent the sum of twenty thousand pesos (P20,000.00) per month
as support for their two (2) children on the first five (5) days of each month; and
3.

To pay the costs.[11]

As earlier stated, the Court of Appeals rendered its Decision dismissing the petitioners appeal. His Motion for
Reconsideration was likewise denied in a resolution of the Court of Appeals dated 6 April 2005.[12]
The Court of Appeals held that the marriage license of the parties is presumed to be regularly issued and petitioner had
not presented any evidence to overcome the presumption. Moreover, the parties marriage contract being a public document is
a prima facie proof of the questioned marriage under Section 44, Rule 130 of the Rules of Court. [13]
In his Petition before this Court, petitioner raises the following issues for resolution:

a.

The Honorable Court of Appeals committed a reversible error when it ruled that the Petition for Annulment
has no legal and factual basis despite the evidence on record that there was no marriage license at the
precise moment of the solemnization of the marriage.

b.

The Honorable Court of Appeals committed a reversible error when it gave weight to the Marriage License
No. 7054133 despite the fact that the same was not identified and offered as evidence during the trial, and
was not the Marriage license number appearing on the face of the marriage contract.

c.

The Honorable Court of Appeals committed a reversible error when it failed to apply the ruling laid down by
this Honorable Court in the case of Sy vs. Court of Appeals. (G.R. No. 127263, 12 April 2000 [330 SCRA
550]).

d.

The Honorable Court of Appeals committed a reversible error when it failed to relax the observance of
procedural rules to protect and promote the substantial rights of the party litigants. [14]

We deny the petition.


Petitioner submits that at the precise time that his marriage with the respondent was celebrated, there was no marriage
license because he and respondent just went to the Manila City Hall and dealt with a fixer who arranged everything for them.
[15]

The wedding took place at the stairs in Manila City Hall and not in CDCC BR Chapel where Rev. Aquilino Navarro who

solemnized the marriage belongs.[16] He and respondent did not go to Carmona, Cavite, to apply for a marriage
license. Assuming a marriage license from Carmona, Cavite, was issued to them, neither he nor the respondent was a resident
of the place. The certification of the Municipal Civil Registrar of Carmona, Cavite, cannot be given weight because the
certification states that Marriage License number 7054133 was issued in favor of Mr. Restituto Alcantara and Miss
Rosita Almario[17] but their marriage contract bears the number 7054033 for their marriage license number.
The marriage involved herein having been solemnized on 8 December 1982, or prior to the effectivity of the Family Code,
the applicable law to determine its validity is the Civil Code which was the law in effect at the time of its celebration.
A valid marriage license is a requisite of marriage under Article 53 of the Civil Code, the absence of which renders the
marriagevoid ab initio pursuant to Article 80(3)[18] in relation to Article 58 of the same Code.[19]
Article 53 of the Civil Code[20] which was the law applicable at the time of the marriage of the parties states:
Art. 53. No marriage shall be solemnized unless all these requisites are complied with:
(1)

Legal capacity of the contracting parties;

(2)

Their consent, freely given;

(3)

Authority of the person performing the marriage; and

(4)

A marriage license, except in a marriage of exceptional character.

The requirement and issuance of a marriage license is the States demonstration of its involvement and participation in
every marriage, in the maintenance of which the general public is interested. [21]
Petitioner cannot insist on the absence of a marriage license to impugn the validity of his marriage. The cases where the
court considered the absence of a marriage license as a ground for considering the marriage void are clear-cut.
In Republic of the Philippines v. Court of Appeals,[22] the Local Civil Registrar issued a certification of due search and
inability to find a record or entry to the effect that Marriage License No. 3196182 was issued to the parties. The Court held that
the certification of due search and inability to find a record or entry as to the purported marriage license, issued by the Civil
Registrar of Pasig, enjoys probative value, he being the officer charged under the law to keep a record of all data relative to the
issuance of a marriage license. Based on said certification, the Court held that there is absence of a marriage license that
would render the marriage void ab initio.
In Cario v. Cario,[23] the Court considered the marriage of therein petitioner Susan Nicdao and the deceased Santiago
S.Carino as void ab initio. The records reveal that the marriage contract of petitioner and the deceased bears no marriage
license number and, as certified by the Local Civil Registrar of San Juan, Metro Manila, their office has no record of such
marriage license. The court held that the certification issued by the local civil registrar is adequate to prove the non-issuance of
the marriage license. Their marriage having been solemnized without the necessary marriage license and not being one of the
marriages exempt from the marriage license requirement, the marriage of the petitioner and the deceased is undoubtedly
void ab initio.
In Sy v. Court of Appeals,[24] the marriage license was issued on 17 September 1974, almost one year after the ceremony
took place on 15 November 1973. The Court held that the ineluctable conclusion is that the marriage was indeed contracted
without a marriage license.

In all these cases, there was clearly an absence of a marriage license which rendered the marriage void.

Clearly, from these cases, it can be deduced that to be considered void on the ground of absence of a marriage license,
the law requires that the absence of such marriage license must be apparent on the marriage contract, or at the very least,
supported by a certification from the local civil registrar that no such marriage license was issued to the parties. In this case, the
marriage contract between the petitioner and respondent reflects a marriage license number. A certification to this effect was
also issued by the local civil registrar of Carmona, Cavite.[25] The certification moreover is precise in that it specifically identified
the parties to whom the marriage license was issued, namely Restituto Alcantara and Rosita Almario, further validating the fact
that a license was in fact issued to the parties herein.
The certification of Municipal Civil Registrar Macrino L. Diaz of Carmona, Cavite, reads:
This is to certify that as per the registry Records of Marriage filed in this office, Marriage License No. 7054133
was issued in favor of Mr.Restituto Alcantara and Miss Rosita Almario on December 8, 1982.
This Certification is being issued upon the request of Mrs. Rosita A. Alcantara for whatever legal purpose or
intents it may serve.[26]

This certification enjoys the presumption that official duty has been regularly performed and the issuance of the marriage
license was done in the regular conduct of official business. [27] The presumption of regularity of official acts may be rebutted by
affirmative evidence of irregularity or failure to perform a duty. However, the presumption prevails until it is overcome by no less
than clear and convincing evidence to the contrary. Thus, unless the presumption is rebutted, it becomes conclusive. Every
reasonable intendment will be made in support of the presumption and, in case of doubt as to an officers act being lawful or
unlawful, construction should be in favor of its lawfulness. [28] Significantly, apart from these, petitioner, by counsel, admitted that
a marriage license was, indeed, issued in Carmona, Cavite.[29]
Petitioner, in a faint attempt to demolish the probative value of the marriage license, claims that neither he nor respondent
is a resident of Carmona, Cavite. Even then, we still hold that there is no sufficient basis to annul petitioner and respondents
marriage. Issuance of a marriage license in a city or municipality, not the residence of either of the contracting parties, and
issuance of a marriage license despite the absence of publication or prior to the completion of the 10-day period for publication
are considered mere irregularities that do not affect the validity of the marriage. [30] An irregularity in any of the formal requisites
of marriage does not affect its validity but the party or parties responsible for the irregularity are civilly, criminally and
administratively liable.[31]
Again, petitioner harps on the discrepancy between the marriage license number in the certification of the Municipal Civil
Registrar, which states that the marriage license issued to the parties is No. 7054133, while the marriage contract states that
the marriage license number of the parties is number 7054033. Once more, this argument fails to sway us. It is not impossible
to assume that the same is a mere a typographical error, as a closer scrutiny of the marriage contract reveals the overlapping of
the numbers 0 and 1, such that the marriage license may read either as 7054133 or 7054033. It therefore does not detract from
our conclusion regarding the existence and issuance of said marriage license to the parties.
Under the principle that he who comes to court must come with clean hands, [32] petitioner cannot pretend that he was not
responsible or a party to the marriage celebration which he now insists took place without the requisite marriage
license. Petitioner admitted that the civil marriage took place because he initiated it. [33] Petitioner is an educated person. He
is a mechanical engineer by profession. He knowingly and voluntarily went to the Manila City Hall and likewise, knowingly and
voluntarily, went through a marriage ceremony. He cannot benefit from his action and be allowed to extricate himself from the
marriage bond at his mere say-so when the situation is no longer palatable to his taste or suited to his lifestyle. We cannot
countenance such effrontery. His attempt to make a mockery of the institution of marriage betrays his bad faith. [34]
Petitioner and respondent went through a marriage ceremony twice in a span of less than one year utilizing the same
marriage license. There is no claim that he went through the second wedding ceremony in church under duress or with a gun to
his head. Everything was executed without nary a whimper on the part of the petitioner.
In fact, for the second wedding of petitioner and respondent, they presented to the San Jose de Manuguit Church the
marriage contract executed during the previous wedding ceremony before the Manila City Hall. This is confirmed in petitioners
testimony as follows
WITNESS
As I remember your honor, they asked us to get the necessary document prior to the wedding.
COURT
What particular document did the church asked you to produce? I am referring to the San Jose
de Manuguit church.
WITNESS

I dont remember your honor.


COURT
Were you asked by the church to present a Marriage License?
WITNESS
I think they asked us for documents and I said we have already a Marriage Contract and I dont know if it is
good enough for the marriage and they accepted it your honor.
COURT
In other words, you represented to the San Jose de Manuguit church that you have with you already a
Marriage Contract?
WITNESS
Yes your honor.
COURT
That is why the San Jose de Manuguit church copied the same marriage License in the Marriage Contract
issued which Marriage License is Number 7054033.
WITNESS
Yes your honor.[35]

The logical conclusion is that petitioner was amenable and a willing participant to all that took place at that
time. Obviously, the church ceremony was confirmatory of their civil marriage, thereby cleansing whatever irregularity or defect
attended the civil wedding.[36]
Likewise, the issue raised by petitioner -- that they appeared before a fixer who arranged everything for them and who
facilitated the ceremony before a certain Rev. Aquilino Navarro, a Minister of the Gospel of the CDCC Br Chapel -- will not
strengthen his posture. The authority of the officer or clergyman shown to have performed a marriage ceremony will be
presumed in the absence of any showing to the contrary.[37] Moreover, the solemnizing officer is not duty-bound to investigate
whether or not a marriage license has been duly and regularly issued by the local civil registrar. All the solemnizing officer
needs to know is that the license has been issued by the competent official, and it may be presumed from the issuance of the
license that said official has fulfilled the duty to ascertain whether the contracting parties had fulfilled the requirements of law. [38]
Semper praesumitur pro matrimonio. The presumption is always in favor of the validity of the marriage. [39] Every
intendment of the law or fact leans toward the validity of the marriage bonds. The Courts look upon this presumption with great
favor. It is not to be lightly repelled; on the contrary, the presumption is of great weight.
WHEREFORE, premises considered, the instant Petition is DENIED for lack of merit. The decision of the Court of
Appeals dated 30 September 2004 affirming the decision of the Regional Trial Court, Branch 143 of Makati City, dated 14
February 2000, are AFFIRMED. Costs against petitioner.
SO ORDERED.

[A.M. No. MTJ-96-1088. July 19, 1996]

RODOLFO G. NAVARRO, complainant, vs. JUDGE HERNANDO C. DOMAGTOY, respondent.


DECISION
ROMERO, J.:
The complainant in this administrative case is the Municipal Mayor of Dapa, Surigao del Norte, Rodolfo G. Navarro. He has
submitted evidence in relation to two specific acts committed by respondent Municipal Circuit Trial Court Judge Hernando
Domagtoy, which, he contends, exhibits gross misconduct as well as inefficiency in office and ignorance of the law.
First, on September 27, 1994, respondent judge solemnized the wedding between Gaspar A. Tagadan and Arlyn F. Borga,
despite the knowledge that the groom is merely separated from his first wife.
Second, it is alleged that he performed a marriage ceremony between Floriano Dador Sumaylo and Gemma G. del Rosario
outside his court's jurisdiction on October 27, 1994. Respondent judge holds office and has jurisdiction in the Municipal Circuit
Trial Court of Sta. Monica-Burgos, Surigao del Norte. The wedding was solemnized at the respondent judge's residence in the
municipality of Dapa, which does not fall within his jurisdictional area of the municipalities of Sta. Monica and Burgos, located
some 40 to 45 kilometers away from the municipality of Dapa, Surigao del Norte.
In his letter-comment to the Office of the Court Administrator, respondent judge avers that the office and name of the
Municipal Mayor of Dapa have been used by someone else, who, as the mayor's "lackey," is overly concerned with his
actuations both as judge and as a private person. The same person had earlier filed Administrative Matter No. 94-980-MTC,
which was dismissed for lack of merit on September 15, 1994, and Administrative Matter No. OCA-IPI-95-16, " Antonio Adapon
v. Judge Hernando C. Domagtoy," which is still pending.
In relation to the charges against him, respondent judge seeks exculpation from his act of having solemnized the marriage
between Gaspar Tagadan, a married man separated from his wife, and Arlyn F. Borga by stating that he merely relied on the
Affidavit issued by the Municipal Trial Judge of Basey, Samar, confirming the fact that Mr. Tagadan and his first wife have not
seen each other for almost seven years. [1] With respect to the second charge, he maintains that in solemnizing the marriage
between Sumaylo and del Rosario, he did not violate Article 7, paragraph 1 of the Family Code which states that: "Marriage
may be solemnized by: (1) Any incumbent member of the judiciary within the court's jurisdiction; and that Article 8 thereof
applies to the case in question.
The complaint was not referred, as is usual, for investigation, since the pleadings submitted were considered sufficient for a
resolution of the case.[2]
Since the countercharges of sinister motives and fraud on the part of complainant have not been sufficiently proven, they will
not be dwelt upon. The acts complained of and respondent judge's answer thereto will suffice and can be objectively assessed
by themselves to prove the latter's malfeasance.
The certified true copy of the marriage contract between Gaspar Tagadan and Arlyn Borga states that Tagadan's civil status
is "separated." Despite this declaration, the wedding ceremony was solemnized by respondent judge. He presented in
evidence a joint affidavit by Maurecio A. Labado, Sr. and Eugenio Bullecer, subscribed and sworn to before Judge
Demosthenes C. Duquilla, Municipal Trial Judge of Basey, Samar. [3]The affidavit was not issued by the latter judge, as claimed
by respondent judge, but merely acknowledged before him. In their affidavit, the affiants stated that they knew Gaspar Tagadan
to have been civilly married to Ida D. Pearanda in September 1983; that after thirteen years of cohabitation and having borne
five children, Ida Pearanda left the conjugal dwelling in Valencia, Bukidnon and that she has not returned nor been heard of for
almost seven years, thereby giving rise to the presumption that she is already dead.
In effect, Judge Domagtoy maintains that the aforementioned joint affidavit is sufficient proof of Ida Pearanda's presumptive
death, and ample reason for him to proceed with the marriage ceremony. We do not agree.
Article 41 of the Family Code expressly provides:
"A marriage contracted by any person during the subsistence of a previous marriage shall be null and void, unless before the
celebration of the subsequent marriage, the prior spouse had been absent for four consecutive years and the spouse present
had a well-founded belief that the absent spouse was already dead. In case of disappearance where there is danger of death
under the circumstances set forth in the provisions of Articles 391 of the Civil Code, an absence of only two years shall be
sufficient.
For the purpose of contracting the subsequent marriage under the preceding paragraph, the spouse present must institute a
summary proceeding as provided in this Code for the declaration of presumptive death of the absentee, without
prejudice to the effect of reappearance of the absent spouse." (Emphasis added.)
There is nothing ambiguous or difficult to comprehend in this provision. In fact, the law is clear and simple. Even if the
spouse present has a well-founded belief that the absent spouse was already dead, a summary proceeding for the declaration
of presumptive death is necessary in order to contract a subsequent marriage, a mandatory requirement which has been
precisely incorporated into the Family Code to discourage subsequent marriages where it is not proven that the previous
marriage has been dissolved or a missing spouse is factually or presumptively dead, in accordance with pertinent provisions of
law.
In the case at bar, Gaspar Tagadan did not institute a summary proceeding for the declaration of his first wife's presumptive
death. Absent this judicial declaration, he remains married to Ida Pearanda. Whether wittingly, or unwittingly, it was manifest
error on the part of respondent judge to have accepted the joint affidavit submitted by the groom. Such neglect or ignorance of
the law has resulted in a bigamous, and therefore void, marriage. Under Article 35 of the Family Code, "The following marriage
shall be void from the beginning: (4) Those bigamous x x x marriages not falling under Article 41."
The second issue involves the solemnization of a marriage ceremony outside the court's jurisdiction, covered by Articles 7
and 8 of the Family Code, thus:
"Art. 7.
(1)

Marriage may be solemnized by:

Any incumbent member of the judiciary within the court's jurisdiction;


xxx

xxx

xxx (Emphasis supplied.)

Art. 8. The marriage shall be solemnized publicly in the chambers of the judge or in open court, in the church, chapel or temple,
or in the office of the consul-general, consul or vice-consul, as the case may be, and not elsewhere, except in cases of
marriages contracted on the point of death or in remote places in accordance with Article 29 of this Code, or where
both parties request the solemnizing officer in writing in which case the marriage may be solemnized at a house or
place designated by them in a sworn statement to that effect."
Respondent judge points to Article 8 and its exceptions as the justifications for his having solemnized the marriage between
Floriano Sumaylo and Gemma del Rosario outside of his court's jurisdiction. As the aforequoted provision states, a marriage
can be held outside of the judge's chambers or courtroom only in the following instances: (1) at the point of death, (2) in remote
places in accordance with Article 29 or (3) upon request of both parties in writing in a sworn statement to this effect. There is no
pretense that either Sumaylo or del Rosario was at the point of death or in a remote place. Moreover, the written request
presented addressed to the respondent judge was made by only one party, Gemma del Rosario. [4]

More importantly, the elementary principle underlying this provision is the authority of the solemnizing judge. Under Article 3,
one of the formal requisites of marriage is the "authority of the solemnizing officer." Under Article 7, marriage may be
solemnized by, among others, "any incumbent member of the judiciary within the court's jurisdiction." Article 8, which is a
directory provision, refers only to the venue of the marriage ceremony and does not alter or qualify the authority of the
solemnizing officer as provided in the preceding provision. Non-compliance herewith will not invalidate the marriage.
A priest who is commissioned and allowed by his local ordinary to marry the faithful, is authorized to do so only within the
area of the diocese or place allowed by his Bishop. An appellate court Justice or a Justice of this Court has jurisdiction over the
entire Philippines to solemnize marriages, regardless of the venue, as long as the requisites of the law are complied
with. However, judges who are appointed to specific jurisdictions, may officiate in weddings only within said areas and not
beyond. Where a judge solemnizes a marriage outside his court's jurisdiction, there is a resultant irregularity in the formal
requisite laid down in Article 3, which while it may not affect the validity of the marriage, may subject the officiating official to
administrative liability.[5]
Inasmuch as respondent judge's jurisdiction covers the municipalities of Sta. Monica and Burgos, he was not clothed with
authority to solemnize a marriage in the municipality of Dapa, Surigao del Norte. By citing Article 8 and the exceptions therein
as grounds for the exercise of his misplaced authority, respondent judge again demonstrated a lack of understanding of the
basic principles of civil law.
Accordingly, the Court finds respondent to have acted in gross ignorance of the law. The legal principles applicable in the
cases brought to our attention are elementary and uncomplicated, prompting us to conclude that respondent's failure to apply
them is due to a lack of comprehension of the law.
The judiciary should be composed of persons who, if not experts, are at least, proficient in the law they are sworn to apply,
more than the ordinary laymen. They should be skilled and competent in understanding and applying the law. It is imperative
that they be conversant with basic legal principles like the ones involved in instant case. [6] It is not too much to expect them to
know and apply the law intelligently.[7] Otherwise, the system of justice rests on a shaky foundation indeed, compounded by the
errors committed by those not learned in the law. While magistrates may at times make mistakes in judgment, for which they
are not penalized, the respondent judge exhibited ignorance of elementary provisions of law, in an area which has greatly
prejudiced the status of married persons.
The marriage between Gaspar Tagadan and Arlyn Borga is considered bigamous and void, there being a subsisting
marriage between Gaspar Tagadan and Ida Pearanda.
The Office of the Court Administrator recommends, in its Memorandum to the Court, a six-month suspension and a stern
warning that a repetition of the same or similar acts will be dealt with more severely. Considering that one of the marriages in
question resulted in a bigamous union and therefore void, and the other lacked the necessary authority of respondent judge, the
Court adopts said recommendation. Respondent is advised to be more circumspect in applying the law and to cultivate a
deeper understanding of the law.
IN VIEW OF THE FOREGOING, respondent Judge Hernando C. Domagtoy is hereby SUSPENDED for a period of six (6)
months and given a STERN WARNING that a repetition of the same or similar acts will be dealt with more severely.
SO ORDERED.

[A.M. No. MTJ-02-1390. April 11, 2002]

MERCEDITA MATA ARAES, petitioner, vs. JUDGE SALVADOR M. OCCIANO, respondent.


DECISION
PUNO, J.:
Petitioner Mercedita Mata Araes charges respondent judge with Gross Ignorance of the Law via a sworn Letter-Complaint
dated 23 May 2001. Respondent is the Presiding Judge of the Municipal Trial Court of Balatan, Camarines Sur. Petitioner
alleges that on 17 February 2000, respondent judge solemnized her marriage to her late groom Dominador B. Orobia without
the requisite marriage license and at Nabua, Camarines Sur which is outside his territorial jurisdiction.
They lived together as husband and wife on the strength of this marriage until her husband passed away. However, since the
marriage was a nullity, petitioners right to inherit the vast properties left by Orobia was not recognized. She was likewise
deprived of receiving the pensions of Orobia, a retired Commodore of the Philippine Navy.
Petitioner prays that sanctions be imposed against respondent judge for his illegal acts and unethical misrepresentations
which allegedly caused her so much hardships, embarrassment and sufferings.
On 28 May 2001, the case was referred by the Office of the Chief Justice to then Acting Court Administrator Zenaida N.
Elepao for appropriate action. On 8 June 2001, the Office of the Court Administrator required respondent judge to comment.
In his Comment dated 5 July 2001, respondent judge averred that he was requested by a certain Juan Arroyo on 15
February 2000 to solemnize the marriage of the parties on 17 February 2000. Having been assured that all the documents to
the marriage were complete, he agreed to solemnize the marriage in his sala at the Municipal Trial Court of Balatan, Camarines
Sur. However, on 17 February 2000, Arroyo informed him that Orobia had a difficulty walking and could not stand the rigors of
travelling to Balatan which is located almost 25 kilometers from his residence in Nabua. Arroyo then requested if respondent
judge could solemnize the marriage in Nabua, to which request he acceded.
Respondent judge further avers that before he started the ceremony, he carefully examined the documents submitted to him
by petitioner. When he discovered that the parties did not possess the requisite marriage license, he refused to solemnize the
marriage and suggested its resetting to another date. However, due to the earnest pleas of the parties, the influx of visitors, and
the delivery of provisions for the occasion, he proceeded to solemnize the marriage out of human compassion. He also feared
that if he reset the wedding, it might aggravate the physical condition of Orobia who just suffered from a stroke. After the
solemnization, he reiterated the necessity for the marriage license and admonished the parties that their failure to give it would
render the marriage void. Petitioner and Orobia assured respondent judge that they would give the license to him in the
afternoon of that same day. When they failed to comply, respondent judge followed it up with Arroyo but the latter only gave him
the same reassurance that the marriage license would be delivered to his sala at the Municipal Trial Court of Balatan,
Camarines Sur.
Respondent judge vigorously denies that he told the contracting parties that their marriage is valid despite the absence of a
marriage license. He attributes the hardships and embarrassment suffered by the petitioner as due to her own fault and
negligence.
On 12 September 2001, petitioner filed her Affidavit of Desistance dated 28 August 2001 with the Office of the Court
Administrator. She attested that respondent judge initially refused to solemnize her marriage due to the want of a duly issued
marriage license and that it was because of her prodding and reassurances that he eventually solemnized the same. She
confessed that she filed this administrative case out of rage. However, after reading the Comment filed by respondent judge,
she realized her own shortcomings and is now bothered by her conscience.
Reviewing the records of the case, it appears that petitioner and Orobia filed their Application for Marriage License on 5
January 2000. It was stamped in this Application that the marriage license shall be issued on 17 January 2000. However,
neither petitioner nor Orobia claimed it.
It also appears that the Office of the Civil Registrar General issued a Certification that it has no record of such marriage that
allegedly took place on 17 February 2000. Likewise, the Office of the Local Civil Registrar of Nabua, Camarines Sur issued
another Certification dated 7 May 2001 that it cannot issue a true copy of the Marriage Contract of the parties since it has no
record of their marriage.
On 8 May 2001, petitioner sought the assistance of respondent judge so the latter could communicate with the Office of the
Local Civil Registrar of Nabua, Camarines Sur for the issuance of her marriage license. Respondent judge wrote the Local Civil
Registrar of Nabua, Camarines Sur. In a letter dated 9 May 2001, a Clerk of said office, Grace T. Escobal, informed respondent
judge that their office cannot issue the marriage license due to the failure of Orobia to submit the Death Certificate of his
previous spouse.
The Office of the Court Administrator, in its Report and Recommendation dated 15 November 2000, found the respondent
judge guilty of solemnizing a marriage without a duly issued marriage license and for doing so outside his territorial
jurisdiction. A fine of P5,000.00 was recommended to be imposed on respondent judge.
We agree.
Under the Judiciary Reorganization Act of 1980, or B.P.129, the authority of the regional trial court judges and judges of
inferior courts to solemnize marriages is confined to their territorial jurisdiction as defined by the Supreme Court.
The case at bar is not without precedent. In Navarro vs. Domagtoy,[1] respondent judge held office and had jurisdiction in
the Municipal Circuit Trial Court of Sta. Monica-Burgos, Surigao del Norte. However, he solemnized a wedding at his residence
in the municipality of Dapa, Surigao del Norte which did not fall within the jurisdictional area of the municipalities of Sta. Monica
and Burgos. We held that:
A priest who is commissioned and allowed by his local ordinance to marry the faithful is authorized to do so only within the area
or diocese or place allowed by his Bishop. An appellate court Justice or a Justice of this Court has jurisdiction over the entire
Philippines to solemnize marriages, regardless of the venue, as long as the requisites of the law are complied with. However,
judges who are appointed to specific jurisdictions, may officiate in weddings only within said areas and not beyond.
Where a judge solemnizes a marriage outside his courts jurisdiction, there is a resultant irregularity in the formal
requisite laid down in Article 3, which while it may not affect the validity of the marriage, may subject the officiating
official to administrative liability.[2] (Emphasis supplied.)
In said case, we suspended respondent judge for six (6) months on the ground that his act of solemnizing a marriage
outside his jurisdiction constitutes gross ignorance of the law. We further held that:
The judiciary should be composed of persons who, if not experts, are at least, proficient in the law they are sworn to apply,
more than the ordinary laymen. They should be skilled and competent in understanding and applying the law. It is imperative
that they be conversant with basic legal principles like the ones involved in the instant case. x x x While magistrates may at
times make mistakes in judgment, for which they are not penalized, the respondent judge exhibited ignorance of elementary
provisions of law, in an area which has greatly prejudiced the status of married persons. [3]
In the case at bar, the territorial jurisdiction of respondent judge is limited to the municipality of Balatan, Camarines Sur. His
act of solemnizing the marriage of petitioner and Orobia in Nabua, Camarines Sur therefore is contrary to law and subjects him
to administrative liability. His act may not amount to gross ignorance of the law for he allegedly solemnized the marriage out of
human compassion but nonetheless, he cannot avoid liability for violating the law on marriage.

Respondent judge should also be faulted for solemnizing a marriage without the requisite marriage license. In People vs.
Lara,[4] we held that a marriage which preceded the issuance of the marriage license is void, and that the subsequent issuance
of such license cannot render valid or even add an iota of validity to the marriage. Except in cases provided by law, it is the
marriage license that gives the solemnizing officer the authority to solemnize a marriage. Respondent judge did not possess
such authority when he solemnized the marriage of petitioner. In this respect, respondent judge acted in gross ignorance of the
law.
Respondent judge cannot be exculpated despite the Affidavit of Desistance filed by petitioner. This Court has consistently
held in a catena of cases that the withdrawal of the complaint does not necessarily have the legal effect of exonerating
respondent from disciplinary action. Otherwise, the prompt and fair administration of justice, as well as the discipline of court
personnel, would be undermined. [5] Disciplinary actions of this nature do not involve purely private or personal matters. They can
not be made to depend upon the will of every complainant who may, for one reason or another, condone a detestable act. We
cannot be bound by the unilateral act of a complainant in a matter which involves the Courts constitutional power to discipline
judges. Otherwise, that power may be put to naught, undermine the trust character of a public office and impair the integrity and
dignity of this Court as a disciplining authority.[6]
WHEREFORE, respondent Judge Salvador M. Occiano, Presiding Judge of the Municipal Trial Court of Balatan, Camarines
Sur, is fined P5,000.00 pesos with a STERN WARNING that a repetition of the same or similar offense in the future will be dealt
with more severely.
SO ORDERED.

G.R. No. L-5955

September 19, 1952

JOSE L. LAXAMANA, petitioner,


vs.
JOSE T. BALTAZAR, respondent.
Gerardo S. Limlingan and Jose L. Baltazar for petitioner.
Macapagal, Punzalan and Yabut for respondent.
Ramon Duterte and Pedro Lopez as amici curiae.
BENGZON, J.:
When in July 1952 the mayor of Sexmoan, Pampanga, was suspended, the vice-mayor Jose T. Salazar, assumed office as
mayor by virtue of section 2195 of the Revised Administrative Code. However, the provincial governor, acting under section 21
(a) of the Revised Election Code (R.A. 180), with the consent of the provincial board appointed Jose L. Laxamana, as mayor of
Sexmoan, who immediately took the corresponding official oath.
Result: this quo warranto proceeding, based solely on the petitioner's proposition that the section first mentioned has been
repealed by the subsequent provision of the Revised Election Code.
If there was such repeal, this petition should be granted, and Laxamana declared the lawful mayor of Sexmoan. Otherwise it
must be denied.1
The two statutory provisions read as follows:
SEC. 2195. Temporary disability of the mayor. Upon the occasion of the absence, suspension, or other temporary
disability of the Mayor, his duties shall be discharged by the Vice-Mayor, or if there be no Vice-Mayor, by the councilor who
at the last general election received the highest number of votes.
SEC. 21 (a). Vacancy in elective provincial, city or municipal office. Whenever a temporary vacancy in any elective
local office occurs, the same shall be filled by appointment by the President if it is a provincial or city office, and by the
provincial governor, with the consent of the Provincial Board, if it is a municipal office. (R.A. 180, the Revised Election
Code.
SEC. 21 (a) The portion relating to municipal offices was taken from section 2180 of the Revised Administrative Code,
which partly provided:
SEC. 2180. Vacancies in municipal office. (a) In case of a temporary vacancy in any municipal office, the same shall be
filled by appointment by the provincial governor, with the consent of the provincial board.
(b) In case of a permanent vacancy in any municipal office, the same shall be filled by appointment by the provincial
board, except in case of a municipal president, in which the permanent vacancy shall be filled by the municipal vicepresident. . . .
It will be seen that under this section, when the office of municipal president (now mayor) become permanently vacant the vicepresident stepped into the office. The section omitted reference to temporary vacancy of such office because section 2195
governed that contingency. In this regard sections 2180 and 2195 supplemented each other. Paragraph (a) of section 2180
applied to municipal offices in general, other than that of the municipal president.
Under the Revised Administrative Code, specially the two sections indicated there was no doubt in Government circles
that when the municipal president was suspended from office, the vice-president took his place.
Temporary vacancy in office of municipal president. Paragraph (a) of this section (2180) should be construed to cover
only municipal offices other than the office of president. Section 2195 of the Administrative Code should be applied in case
of the absence, suspension, or other temporary disability of the municipal president. (Op. Atty. Gen. Sept. 21, 1917; Ins.
Aud. Oct. 23, 1927.) (Araneta, Administrative Code Vol. IV p. 2838)
Municipal president cannot designate acting president. There is no provision of law expressly or implied authorizing the
municipal president to designate any person to act in his stead during his temporary absence or disability. From the
provision of section 2195 of the code, it is clear that the vice-president or, if there be no vice-president, the councilor who
at the last general election received the highest number of votes, should automatically (without any formal designation)
discharge the duties of the president. (Op. Ins. Aud. March 2, 1926) (Araneta Administrative Code Vol. IV, p. 2839)
Now it is reasonable to assume that the incorporation of the above section 2180 into the Revised Election Law as section 21 (a)
did not have the effect of enlarging its scope, 2 to supersede or repeal section 2195, what with the presumption against implied
repeals.3 "Where a statute has received a contemporaneous and practical interpretation and the statute as interpreted is reenacted, the practical interpretation is accorded greater weight than it ordinarily receives, and is regarded as presumptively the
correct interpretation of the law. The rule here is based upon the theory that the legislature is acquainted with the
contemporaneous interpretation of a statute, especially when made by an administrative body or executive officers charged with
the duty of administering or enforcing the law, and therefore impliedly adopts the interpretation upon re-enactment." (Sutherland
Statutory Construction, sec. 5109.)
Indeed, even disregarding their origin, the allegedly conflicting sections, could be interpreted in the light of the principle of
statutory construction that when a general and a particular provision are inconsistent the latter is paramount to the former (sec.
288, Act 190). In other words, section 2195 referring particularly to vacancy in the office of mayor, must prevail over the general
terms of section 21 (a) as to vacancies of municipal (local) offices. Otherwise stated, section 2195 may be deemed an exception
to or qualification of the latter.4 "Where one statute deals with a subject in general terms, and another deals with a part of the
same subject in a more detailed way, the two should be harmonized if possible; but if there is any conflict, the latter will prevail,
regardless of whether it was passed prior to the general statute." (Sutherland Statutory Construction, sec. 5204)
In a recent decision,5 we had occasion to pass on a similar situation repeal by subsequent general provision of a prior special
provision and we said,:
It is well-settled that a special and local statute, providing for a particular case or class of cases, is not repealed by a
subsequent statute, general in its terms, provisions and application, unless the intent to repeal or alter is manifest,
although the terms of the general act are broad enough to include the cases embraced in the special law. . . . It is a canon
of statutory construction that a later statute, general in its terms and not expressly repealing a prior special statute, will
ordinarily not affect the special provisions, of such earlier statute. (Steamboat Company vs. Collector, 18 Wall. (U.S.), 478;
Cass County vs. Gillett, 100 U.S. 585; Minnesota vs. Hitchcock, 185 U.S. 373, 396.)
Where there are two statutes, the earlier special and the later general the terms of the general brood enough to include
the matter provided for in the special the fact that one is special and the other is general creates a presumption that the
special is to be considered as remaining an exception to the general, one as a general law of the land, the other as the
law of a particular case. (State vs. Stoll, 17 Wall. (U.S.) 425)

In fact even after the Revised Election Code was enacted, the Department of the Interior and the office of executive Secretary
who are charged with the supervision of provincial and municipal governments have "consistently held that in case of the
suspension or other temporary disability of the mayor, the vice-mayor shall, by operation of law, assume the office of the mayor,
and if the vice-mayor is not available, the said office shall be discharged by the first councilor." (Annex 5 of the answer.)
Needless to say, the contemporaneous construction placed upon the statute by the executive officers charged with its execution
deserves great weight in the courts.6
Consequently it is our ruling that when the mayor of a municipality is suspended, absent or temporarily unable, his duties should
be discharged by the vice-mayor in accordance with sec. 2195 of the Revised Administrative Code.
This quo warranto petition is dismissed with costs. So ordered.1wphl.nt

G.R. No. 116835 March 5, 1998


ANTONIETTA GARCIA VDA. DE CHUA, petitioner,
vs.
COURT OF APPEALS (Special Eight Division), HON. JAPAL M. GUIANI, RTC, Branch 14, 12th Judicial Region, Cotabato
City, and FLORITA A. VALLEJO, as Administratrix of the Estate of the late Roberto L. Chua, respondents.

KAPUNAN, J.:
Assailed before us in this Appeal by Certiorari under Rule 45 of the Rules of Court is the decision of the Court of Appeals in CAGR Sp. No. 33101, promulgated on 19 April 1994 affirming the decision of the Regional Trial Court, Branch 14, of Cotabato City
in Special Procedure Case No. 331.
As culled from the records, the following facts have been established by evidence:
During his lifetime, Roberto Lim Chua lived out of wedlock with private respondent Florita A. Vallejo from 1970 up to 1981. Out
of this union, the couple begot two illegitimate children, namely, Roberto Rafson Alonzo and Rudyard Pride Alonzo.
On 28 May 1992, Roberto Chua died intestate in Davao City.
On 2 July 1992, private respondent filed with the Regional Trial Court of Cotabato City a Petition 1 which is reproduced
hereunder:
IN RE: PETITION FOR DECLARATION
OF HEIRSHIP, GUARDIANSHIP OVER
THE PERSONS AND PROPERTIES OF
MINORS ROBERT RAFSON ALONZO SP. PROC. NO/ 331
and RUDYARD PRIDE ALONZO, all
surnamed CHUA and ISSUANCE OF
LETTERS OF ADMINISTRATION.
FLORITA ALONZO VALLEJO,
Petitioner
PETITION
COMES NOW the petitioner assisted by counsel and unto this Honorable Court most respectfully states:
1. That she is of legal age, Filipino, married but separated from her husband and residing at Quezon Avenue, Cotabato
City, Philippines;
2. That sometime from 1970 up to and until late 1981 your petitioner lived with Roberto Lim Chua as husband and wife
and out of said union they begot two (2) children, namely, Robert Rafson Alonzo Chua who was born in General Santos
City on April 28, 1977 and Rudyard Pride Alonzo Chua who was born in Davao City on August 30, 1978. A xerox copy of
the birth certificate of each child is hereto attached as annex "A" and "B", respectively.
3. That the aforementioned children who are still minors today are both staying with herein petitioner at her address at
Quezon Avenue, Cotabato City;
4. That Roberto Lim Chua, father of the above-mentioned minors, died intestate on May 28, 1992 in Davao City.
5. That the aforementioned deceased left properties both real and personal worth P5,000,000.00 consisting of the
following:
a) Lot in Kakar, Cotabato City covered by TCT
No. T-12835 with an area of 290 sq. m. estimated at P50,000.00
b) Lot in Kakar, Cotabato City covered by TCT
No. T-12834 with an area of 323 sq. m. 50,000.00
c) Lot in Davao City covered by TCT
No. T-126583 with an area of 303 sq. m. 50,000.00
d) Lot in Davao City covered by TCT
No. T-126584 with an area of 303 sq. m. 50,000.00
e) Residential house in Cotabato City valued at 30,000.00
f) Residential house in Davao City valued at 600,000.00
g) Car, Colt Lancer with Motor No. 4G33-3 AF6393 210,000.00
h) Colt, Galant Super Saloon with Motor
No. 4G37-GB0165 545,000.00
i) Car, Colt Galant with Motor No. 4G52-52D75248 110,000.00
j) Reo Isuzu Dump Truck with Motor
No. DA640-838635 350,000.00
k) Hino Dump Truck with Motor No. ED100-T47148 350,000.00
l) Stockholdings in various corporations with par value
estimated at 3,335,000.00
Total P5,000,000.00

6. That deceased Roberto Lim Chua died single and without legitimate descendants or ascendants, hence, the above
named minors Robert Rafson Alonzo Chua and Rudyard Pride Alonzo Chua, his children with herein petitioner shall
succeed to the entire estate of the deceased. (Article 988 of the Civil Code of the Philippines).
7. That the names, ages and residences of the relatives of said minors are the following, to wit:
Names Relationship Ages Residence
1. Carlos Chua Uncle 60 Quezon Avenue,
Cotabato City
2. Aida Chua Auntie 55 Rosary Heights,
Cotabato City
3. Romulo Uy Uncle 40 c/o Overseas
Fishing Exporation
Co. Inc., Matina,
Davao City
6. That considering the fact that the aforementioned minors by operation of law are to succeed to the entire estate of
Roberto Lim Chua under the provisions of Article 988 of the New Civil Code of the Philippines, it is necessary that for the
protection of the rights and interest of Robert Rafson Alonzo Chua and Rudyard Pride Alonzo Chua, both minors and heirs
of deceased Roberto Lim Chua, a guardian over the persons and properties of said minors be appointed by this
Honorable Court.
7. That herein petitioner being the mother and natural guardian of said minors is also competent and willing to act as the
guardian of minors Robert Rafson Alonzo Chua and Rudyard Pride Alonzo Chua both staying and living with her; that
petitioner possesses all the qualifications and none of the disqualifications of a guardian.
WHEREFORE, premises considered, it is most respectfully prayed:
1. That, upon proper notice and hearing, an order be issued declaring minors ROBERTO RAFSON ALONZO CHUA and
RUDYARD PRIDE ALONZO CHUA as heirs to the intestate estate of deceased ROBERTO LIM CHUA;
2. That Letters of Administration be issued to herein petitioner for the administration of the estate of the deceased
ROBERTO LIM CHUA;
3. That the petitioner be also appointed the guardian of the persons and estate of minors ROBERT RAFSON ALONZO
CHUA and RUDYARD PRIDE ALONZO CHUA;
4. That after all the property of deceased Roberto Lim Chua have been inventoried and expenses and just debts, have
been paid, the intestate estate of Roberto Lim Chua be distributed to its rightful heirs, the minors in this case, pursuant to
the provisions of Article 988 of the New Civil Code of the Philippines.
5. And for such other reliefs and remedies this Honorable Court may consider fit and proper in the premises.
Cotabato City, Philippines, June 29, 1992.
(Sgd.) FLORITA ALONZO VALLEJO
(Petitioner)
The trial court issued an order setting the hearing of the petition on 14 August 1992 and directed that notice thereof be
published in a newspaper of general circulation in the province of Maguindanao and Cotabato City and or Davao City.
On 21 July 1992, herein petitioner Antonietta Garcia Vda. de Chua, representing to be the surviving spouse of Roberto Chua,
filed a Motion to Dismiss 2 on the ground of improper venue. Petitioner alleged that at the time of the decedent's death Davao
City was his residence, hence, the Regional Trial Court of Davao City is the proper forum.
Private respondent filed an opposition to the Motion to Dismiss 3 dated July 20, 1992 based on the following grounds:
(1) That this petition is for the guardianship of the minor children of the petitioner who are heirs to the estate of the late
Roberto L. Chua and under Section 1, Rule 92 of the Rules of Court the venue shall be at the place where the minor
resides;
(2) That the above-named minors are residents of Cotabato City:
(3) That the movant in this case has no personality to intervene nor oppose in the granting of this petition for the reason
that she is a total stranger to the minors Robert Rafson Alonzo and Rudyard Pride Alonzo, all surnamed Chua.
(4) That deceased Roberto L. Chua died a bachelor. He is the father of the above-named minors with the petitioner in this
case;
(5) That movant/oppositor Antonietta Chua is not the surviving spouse of the late Roberto L. Chua but a pretender to the
estate of the latter since the deceased never contracted marriage with any woman until he died.
On 6 August 1992, private respondent Vallejo filed a Motion for Admission of an Amended Petition 4 "in order that the
designation of the case title can properly and appropriately capture or capsulize in clear terms the material averments in the
body of the pleadings; thus avoiding any confusion or misconception of the nature and real intent and purpose of this petition."
The amended petition 5 contained identical material allegations but differed in its title, thus:.
IN RE: PETITION FOR THE SETTLEMENT OF THE INTESTATE ESTATE OF ROBERTO L. CHUA, DECLARATION OF
HEIRSHIP, GUARDIANSHIP OVER THE PERSONS AND PROPERTIES OF MINORS ROBERT AND RUDYARD, all
surnamed CHUA and ISSUANCE OF LETTERS OF ADMINISTRATION.
FLORITA ALONZO VALLEJO,
Petitioner.
Paragraph 4 of the original petition was also amended to read as follows:
4. That Roberto Lim Chua, father of the abovementioned minors is a resident of Cotabato City and died intestate on May
28, 1992 at Davao City.
The petition contained exactly the same prayers as the original petition.
Petitioner opposed the motion to amend petition alleging that at the hearing of said motion on 24 July 1992, private
respondent's counsel allegedly admitted that the sole intention of the original petition was to secure guardianship over the
persons and property of the minors. 6

On 21 August 1992, the trial court issued an Order 7 denying the motion to dismiss for lack of merit. The court ruled that
Antonietta Garcia had no personality to file the motion to dismiss not having proven her status as wife of the decedent. Further,
the court found that the actual residence of the deceased was Cotabato City, and even assuming that there was concurrent
venue among the Regional Trial Courts where the decedent had resided, the R.T.C. of Cotabato had already taken cognizance
of the settlement of the decedent's estate to the exclusion of all others. The pertinent portions of the order read:
At the hearing of the motion to dismiss on August 19, 1992, counsel for movant Antonietta G. Chua presented 18 Exhibits
in support of her allegation that she was the lawful wife of the decedent and that the latter resides in Davao City at the
time of his death. Exh. "1" was the xerox copy of the alleged marriage contract between the movant and the petitioner.
This cannot be admitted in evidence on the ground of the timely objection of the counsels for petitioner that the best
evidence is the original copy or authenticated copy which the movant cannot produce. Further, the counsels for petitioner
in opposition presented the following: a certification from the Local Civil Registrar concerned that no such marriage
contract was ever registered with them; a letter from Judge Augusto Banzali, the alleged person to have solemnized the
alleged marriage that he has not solemnized such alleged marriage. Exhibit "2" through "18" consist among others of
Transfer Certificate of Title issued in the name of Roberto L. Chua married to Antonietta Garcia, and a resident of Davao
City; Residence Certificates from 1988 and 1989 issued at Davao City indicating that he was married and was born in
Cotabato City; Income Tax Returns for 1990 and 1991 filed in Davao City where the status of the decedent was stated as
married; passport of the decedent specifying that he was married and his residence was Davao City. Petitioner through
counsels, objected to the admission in evidence of Exhibits "2" through "18" if the purpose is to establish the truth of the
alleged marriage between the decedent and Antonietta Garcia. The best evidence they said is the marriage contract. They
do not object to the admission of said exhibit if the purpose is to show that Davao City was the business residence of the
decedent.
Petitioner through counsels, presented Exhibit "A" through "K" to support her allegation that the decedent was a resident
of Cotabato City; that he died a bachelor; that he begot two illegitimate children with the petitioner as mother. Among
these exhibits are Income Tax Returns filed in Cotabato City from 1968 through 1979 indicating therein that he was single;
birth certificates of the alleged two illegitimate children of the decedent; Resident Certificates of the decedent issued in
Cotabato City; Registration Certificate of Vehicle of the decedent showing that his residence is Cotabato City.
It is clear from the foregoing that the movant failed to establish the truth of her allegation that she was the lawful wife of
the decedent. The best evidence is a valid marriage contract which the movant failed to produce. Transfer Certificates of
Title, Residence Certificates, passports and other similar documents cannot prove marriage especially so when the
petitioner has submitted a certification from the Local Civil Registrar concerned that the alleged marriage was not
registered and a letter from the judge alleged to have solemnized the marriage that he has not solemnized said alleged
marriage. Consequently, she has no personality to file the subject motion to dismiss.
On the issue of the residence of the decedent at the time of his death, the decedent as a businessman has many
business residences from different parts of the country where he usually stays to supervise and pursue his business
ventures. Davao City is one of them. It cannot be denied that Cotabato City is his actual residence where his alleged
illegitimate children also reside.
The place of residence of the deceased in settlement of estates, probate of will, and issuance of letters of administration
does not constitute an element of jurisdiction over the subject matter. It is merely constitutive of venue (Fule vs. CA, L40502, November 29, 1976). Even assuming that there is concurrent venue among the Regional Trial Courts of the places
where the decedent has residences, the Regional Trial Court first taking cognizance of the settlement of the estate of the
decedent, shall exercise jurisdiction to the exclusion of all other courts (Section 1, Rule 73). It was this Court which first
took cognizance of the case when the petition was filed on July 2, 1992, docketed as Special Proceeding No. 331 and an
order of publication issued by this Court on July 13, 1992.
WHEREFORE, in view of the foregoing, the motion to dismiss is hereby denied for lack of merit.
On 31 August 1992, upon motion of private respondent, the trial court issued an order appointing Romulo Lim Uy, a first cousin
of the deceased, as special administrator of the decedent's estate. 8
On the same day, the trial court, likewise, issued an Order appointing Florita Vallejo as guardian over the persons and
properties of the two minor children. 9
Thereafter, petitioner filed a Motion dated 25 October 1993 10 praying that the letters of administration issued to Vallejo be
recalled and that new letters of administration be issued to her. She, likewise, filed a Motion dated 5 November 1993 11 to
declare the proceedings a mistrial. Both motions were denied by the trial court in its Order dated 22 November
1993. 12 Petitioner's motion for reconsideration of the order was denied by the trial court in an Order dated 13 December 1993. 13
Assailing the last two orders of the trial court, petitioner filed a petition for certiorari and prohibition (Rule 65) with the respondent
Court of Appeals, docketed as CA G.R. No. Sp. 33101, alleging that the trial court acted with grave abuse of discretion in:
(1) unilaterally and summarily converting, if not treating, the guardianship proceedings into an intestate proceeding;
(2) summarily hearing the intestate proceedings without jurisdiction and without any notice to herein petitioner whatsoever;
and
(3) issuing the questioned order (sic) on the alleged pretension that herein petitioner has no personality to intervene in
SPL Proc. No. 331 questioning the highly anomalous orders precipitately issued ex-parte by the public respondent R.T.C.
without notice to the petitioners.
Petitioner in the main argued that private respondent herself admitted in her opposition to petitioner's motion to dismiss filed in
the trial court and in open court that the original petition she filed is one for guardianship; hence, the trial court acted beyond its
jurisdiction when it issued letters of administration over the estate of Roberto L. Chua, thereby converting the petition into an
intestate proceeding, without the amended petition being published in a newspaper of general circulation as required by Section
3, Rule 79.
The Court of Appeals, in its decision promulgated on 19 April 1994, 14 denied the petition ratiocinating that the original petition
filed was one for guardianship of the illegitimate children of the deceased as well as for administration of his intestate estate.
While private respondent may have alleged in her opposition to the motion to dismiss that petition was for guardianship, the fact
remains that the very allegations of the original petition unmistakably showed a twin purpose: (1) guardianship; and (2) issuance
of letters of administration. As such, it was unnecessary for her to republish the notice of hearing through a newspaper of
general circulation in the province. The amended petition was filed for the only reason stated in the motion for leave: so that the
"case title can properly and appropriately capture or capsulize in clear terms the material averments in the body of the
pleadings; thus avoiding any confusion or misconception of the nature and real intent and purpose of this petition," which was
for guardianship over the persons and properties of her minor children and for the settlement of the intestate estate of the
decedent who was their father. In other words, there being no change in the material allegations between the original and
amended petitions, the publication of the first in a newspaper of general circulation sufficed for purposes of compliance with the
legal requirements of notice.
Moreover, the appellate court ruled that the petitioner's remedy is appeal from the orders complained of under Section 1(f), Rule
109 of the Rules of Court, not certiorari and prohibition.

Not satisfied with the decision of the Court of Appeals, petitioner comes to this Court contending that the appellate court
committed the following errors:
I
THE PUBLIC RESPONDENT COURT OF APPEALS GRAVELY AND SERIOUSLY ERRED IN HOLDING THAT THE
ORIGINAL PETITION (Annex F, Petition) WAS FOR A TWIN PURPOSE, TO WIT: FOR GUARDIANSHIP AND FOR
INTESTATE ESTATE PROCEEDINGS;
II
THE PUBLIC RESPONDENT COURT APPEALS SERIOUSLY ERRED IN HOLDING THAT THERE IS NO NEED TO
PUBLISH THE AMENDED PETITION FOR ADMINISTRATION OF THE INTESTATE ESTATE THEREBY
CONTRAVENING THE RULES OF COURT AND THE RULINGS OF THE SUPREME COURT.
III
THE PUBLIC RESPONDENT COURT OF APPEALS SERIOUSLY ERRED IN NOT NULLIFYING THE ORDERS (Annex
"P" to "T") PRECIPITATELY ISSUED EX-PARTE BY THE PUBLIC RESPONDENT REGIONAL TRIAL COURT IN THE
INTESTATE PROCEEDINGS WITHOUT PRIOR HEARING OR NOTICE TO HEREIN PETITIONER THEREBY
DEPRIVING THE LATTER (ANTONIETTA GARCIA VDA. DE CHUA ) OF DUE PROCESS AND OPPORTUNITY TO BE
HEARD.
IV
THE PUBLIC RESPONDENT COURT OF APPEALS GRAVELY ERRED IN SWEEPINGLY HOLDING THAT
PETITIONER'S REMEDY IS APPEAL. 15
In support of her first assignment of error, petitioner submits that the Court of Appeals' conclusion that the original petition was
one for guardianship and administration of the intestate estate is contradicted by the evidence on hand, asserting that the
original petition failed to allege and state the jurisdictional facts required by the Rules of Court in petitions for administration of a
decedent's estate, such as: (a) the last actual residence of the decedent at the time of his death; (b) names, ages and
residences of the heirs; and (c) the names and residences of the creditors of the decedent. Petitioner also reiterates her
argument regarding private respondent's alleged admission that the original petition was one for guardianship and not for
issuance of letters of administration, pointing to the Opposition to the Motion to Dismiss dated 20 July 1992, where the private
respondent alleged.
1. That this petition is for guardianship of the minor children of the petitioner who are heirs to the estate of the late Roberto
L. Chua and under Section 1, Rule 92 of the Rules of Court the venue shall be at the place where the minor resides. 16
as well as to the statements made by counsel for the private respondent during the 24 July 1992 hearing on the motion to
dismiss:
ATTY. RENDON:
We filed our opposition to the motion to dismiss the petition because this is a petition for guardianship of minors, not for
intestate proceedings. So this is a case where the mother wanted to be appointed as guardian because she is also the
litigant here. Because whenever there is an intestate proceedings, she has to represent the minors, and under the Rules
of Court in any guardianship proceedings, the venue is at the place where the minor is actually residing. 17
The petition is devoid of merit.
The title alone of the original petition clearly shows that the petition is one which includes the issuance of letters of
administration. The title of said petition reads:
IN RE: PETITION FOR DECLARATION OF HEIRSHIPS, GUARDIANSHIP OVER THE PERSON AND PROPERTIES OF
MINORS ROBERTO ALONZO AND RUDYARD ALONZO, all surnamed CHUA and ISSUANCE OF LETTERS OF
ADMINISTRATION. 18
Likewise, the prayer of the petition states:
2. That Letters of Administration be issued to herein petition for the administration of the estate of the deceased
ROBERTO LIM CHUA.
The original petition also contains the jurisdictional facts required in a petition for the issuance of letters of administration.
Section 2, Rule 79 of the Rules of Court reads:
Sec. 2. Contents of petition for letters of administration A petition for letters of administration must be filed by an
interested person and must show, so far as known to the petitioner:
(a) jurisdictional facts;
(b) The names, ages, and residences of the heirs and the names and residences of the creditors, of the decedent'
(c) The probative value and character of the property of the estate;.
(d) The name of the person for whom letters of administration are prayed;
But no defect in the petition shall render void the issuance of letters of administration. (emphasis ours).
The jurisdictional facts required in a petition for issuance of letters of administration are: (1) the death of the testator; (2)
residence at the time of death in the province where the probate court is located; and (3) if the decedent was a non-resident, the
fact of being a resident of a foreign country and that the decedent has left an estate in the province where the court is sitting. 19
While paragraph 4 of the original petition stating:
(4) That Roberto Lim Chua, father of the above mentioned minors, died intestate on May 28, 1992 in Davao City.
failed to indicate the residence of the deceased at the time of his death, the omission was cured by the amended petitions
wherein the same paragraph now reads:
(4) That Roberto Lim Chua, father of the abovementioned minors is a resident of Cotabato City and died intestate on May
28, 1992 at Davao City. 20 (Emphasis in the original.)
All told the original petition alleged substantially all the facts required to be stated in the petition for letters of administration.
Consequently, there was no need to publish the amended petition as petitioner would insist in her second assignment of errors.

Be that as it may, petitioner has no legal standing to file the motion to dismiss as she is not related to the deceased, nor does
she have any interest in his estate as creditor or otherwise. The Rules are explicit on who may do so:
Sec. 4. Opposition to petition for administration Any interested person, may by filing a written opposition, contest the
petition on the ground of incompetency of the person for whom letters of administration are prayed therein, or on the
ground of the contestant's own right to the administration, and may pray that letters issue to himself, or to any competent
person or persons named in the opposition..
Only an interested person may oppose the petition for issuance of letters of administration. An interested person is one who
would be benefited by the estate such as an heir, or one who has a claim against the estate, such as a creditor; his interest is
material and direct, and not one that is only indirect or contingent. 21
Petitioner was not able to prove her status as the surviving wife of the decedent. The best proof of marriage between man and
wife is a marriage contract which Antonietta Chua failed to produce. The lower court correctly disregarded the photostat copy of
the marriage certificate which she presented, this being a violation of the best evidence rule, together with other worthless
pieces of evidence. The trial court correctly ruled in its 21 August 1992 Order that:
. . . Transfer Certificates of Title, Residence Certificates, passports and other similar documents cannot prove marriage
especially so when the petitioner has submitted a certification from the Local Civil Registrar concerned that the alleged
marriage was not registered and a letter from the judge alleged to have solemnized the marriage that he has not
solemnized said alleged marriage. . . . 22
Under her third assignment of error, petitioner claims that the trial court issued its orders, Annexes "P" to "T" without prior
hearing or notice to her, thus, depriving her of due process.
The orders referred to by petitioner are: Order dated 31 August 1992 appointing Romulo Lim Uy, first cousin of the deceased, as
special administrator of the estate; Order dated 31 August 1992 appointing private respondent as guardian over the person and
property of the minors; Order dated 5 August 1993, directing the transfer of the remains of the deceased from Davao City to
Cotabato City; Order dated 6 September 1993 directing petitioner to turn over a Mitsubishi Gallant car owned by the estate of
the deceased to the special administrator; and Order dated 28 September 1993, authorizing the sheriff to break open the
deceased's house for the purpose of conducting an inventory of the properties found therein, after the sheriff was refused entry
to the house by the driver and maid of petitioner.
Apart from the fact that petitioner was not entitled to notice of the proceedings of the trial court, not being able to establish proof
of her alleged marriage to the deceased, or of her interest in the estate as creditor or otherwise, petitioner categorically stated in
the instant petition that on 25 October 1993 she filed a motion praying for the recall of the letters of administration issued by the
trial court and another motion dated 5 August 1993 praying that the proceedings conducted by the trial court be declared as a
mistrial and the court orders relative thereto be set aside and nullified. Petitioner further stated that her motions were denied by
the trial court in its Order dated 22 November 21, 1993 and that on 30 November 1993 she filed a motion for reconsideration of
the order of denial which in turn was denied by the trial court on 13 December 1993.
Due process was designed to afford opportunity to be heard, not that an actual hearing should always and indispensably be
held. 23 The essence of due process is simply an opportunity to be heard. 24 Here, even granting that the petitioner was not
notified of the orders of the trial court marked as Exhibits "P" to "T," inclusive, nonetheless, she was duly heard in her motions to
recall letters of administration and to declare the proceedings of the court as a "mistrial," which motions were denied in the
Order dated 22 November 1993. 25 A motion for the reconsideration of this order of denial was also duly heard by the trial court
but was denied in its Order of 13 December 1993. 26
Denial of due process cannot be successfully invoked by a party who has had the opportunity to be heard on his motion for
reconsideration. 27
As to the last assignment of errors, we agree with the Court of Appeals that the proper remedy of the petitioner in said court was
an ordinary appeal and not a special civil action for certiorari; which can be availed of if a party has no plain, speedy and
adequate remedy in the ordinary course of law. Except for her bare allegation that an ordinary appeal would be inadequate,
nothing on record would indicate that extraordinary remedy of certiorari or prohibition is warranted.
Finally, petitioner further argues as supplement to her memorandum that the ruling of the Court of Appeals treating the Special
Proceeding No. 331 as one for both guardianship and settlement of estate is in contravention of our ruling in Gomez
vs. Imperial, 28 which the petitioner quotes:
The distribution of the residue of the estate of the deceased is a function pertaining property not to the guardianship
proceedings, but to another proceeding which the heirs are at liberty to initiate.
Petitioner's reliance on said case is misplaced. In the Gomez case, the action before the lower court was merely one for
guardianship. Therefore said court did not have the jurisdiction to distribute the estate of the deceased. While in the case at bar,
the petition filed before the court was both for guardianship and settlement of estate.
IN VIEW OF THE FOREGOING, the petition of petitioner Antonietta Chua is hereby denied.
SO ORDERED.

G.R. No. 103047 September 2, 1994

REPUBLIC OF THE PHILIPPINES, petitioner,


vs.
COURT OF APPEALS AND ANGELINA M. CASTRO, respondents.
Parungao, Abesamis, Eleazar & Pulgar Law Offices for private respondent.

PUNO, J.:
The case at bench originated from a petition filed by private respondent Angelina M. Castro in the Regional Trial Court of
Quezon City seeking a judicial declaration of nullity of her marriage to Edwin F. Cardenas. 1 As ground therefor, Castro claims
that no marriage license was ever issued to them prior to the solemnization of their marriage.
Despite notice, defendant Edwin F. Cardenas failed to file his answer. Consequently, he was declared in default. Trial proceeded
in his absence.
The controlling facts are undisputed:
On June 24, 1970, Angelina M. Castro and Edwin F. Cardenas were married in a civil ceremony performed by Judge Pablo M.
Malvar, City Court Judge of Pasay City. The marriage was celebrated without the knowledge of Castro's parents. Defendant
Cardenas personally attended to the processing of the documents required for the celebration of the marriage, including the
procurement of the marriage, license. In fact, the marriage contract itself states that marriage license no. 3196182 was issued in
the name of the contracting parties on June 24, 1970 in Pasig, Metro Manila.
The couple did not immediately live together as husband and wife since the marriage was unknown to Castro's parents. Thus, it
was only in March 1971, when Castro discovered she was pregnant, that the couple decided to live together. However, their
cohabitation lasted only for four (4) months. Thereafter, the couple parted ways. On October 19, 1971, Castro gave birth. The
baby was adopted by Castro's brother, with the consent of Cardenas.
The baby is now in the United States. Desiring to follow her daughter, Castro wanted to put in order her marital status before
leaving for the States. She thus consulted a lawyer, Atty. Frumencio E. Pulgar, regarding the possible annulment of her
marriage. Through her lawyer's efforts, they discovered that there was no marriage license issued to Cardenas prior to the
celebration of their marriage.
As proof, Angelina Castro offered in evidence a certification from the Civil Register of Pasig, Metro Manila. It reads:
February 20, 1987
TO WHOM IT MAY CONCERN:
This is to certify that the names EDWIN F. CARDENAS and ANGELINA M. CASTRO who were allegedly married in
the Pasay City Court on June 21, 1970 under an alleged (s)upportive marriage license
no. 3196182 allegedly issued in the municipality on June 20, 1970 cannot be located as said license no. 3196182
does not appear from our records.
Issued upon request of Mr. Ed Atanacio.
(Sgd) CENONA D. QUINTOS
Senior Civil Registry Officer
Castro testified that she did not go to the civil registrar of Pasig on or before June 24, 1970 in order to apply for a license.
Neither did she sign any application therefor. She affixed her signature only on the marriage contract on June 24, 1970 in Pasay
City.
The trial court denied the petition. 2 It held that the above certification was inadequate to establish the alleged non-issuance of a
marriage license prior to the celebration of the marriage between the parties. It ruled that the "inability of the certifying official to
locate the marriage license is not conclusive to show that there was no marriage license issued."
Unsatisfied with the decision, Castro appealed to respondent appellate court. She insisted that the certification from the local
civil registrar sufficiently established the absence of a marriage license.
As stated earlier, respondent appellate court reversed the Decision of the trial court. 3 It declared the marriage between the
contracting parties null and void and directed the Civil Registrar of Pasig to cancel the subject marriage contract.
Hence this petition for review on certiorari.
Petitioner Republic of the Philippines urges that respondent appellate court erred when it ruled that the certification issued by
the civil registrar that marriage license no. 3196182 was not in their record adequately proved that no such license was ever
issued. Petitioner also faults the respondent court for relying on the self-serving and uncorroborated testimony of private
respondent Castro that she had no part in the procurement of the subject marriage license. Petitioner thus insists that the
certification and the uncorroborated testimony of private respondent are insufficient to overthrow the legal presumption
regarding the validity of a marriage.
Petitioner also points that in declaring the marriage between the parties as null and void, respondent appellate court
disregarded the presumption that the solemnizing officer, Judge Pablo M. Malvar, regularly performed his duties when he
attested in the marriage contract that marriage license no. 3196182 was duly presented to him before the solemnization of the
subject marriage.
The issues, being interrelated, shall be discussed jointly.
The core issue presented by the case at bench is whether or not the documentary and testimonial evidence presented by
private respondent are sufficient to establish that no marriage license was issued by the Civil Registrar of Pasig prior to the
celebration of the marriage of private respondent to Edwin F. Cardenas.
We affirm the impugned Decision.
At the time the subject marriage was solemnized on June 24, 1970, the law governing marital relations was the New Civil Code.
The law 4 provides that no marriage shall be solemnized without a marriage license first issued by a local civil registrar. Being
one of the essential requisites of a valid marriage, absence of a license would render the marriage voidab initio. 5
Petitioner posits that the certification of the local civil registrar of due search and inability to find a record or entry to the effect
that marriage license no. 3196182 was issued to the parties is not adequate to prove its non-issuance.
We hold otherwise. The presentation of such certification in court is sanctioned by Section 29, Rule 132 of the Rules of
Court, viz.:

Sec. 29. Proof of lack of record. A written statement signed by an officer having custody of an official record or by
his deputy, that after diligent search, no record or entry of a specified tenor is found to exist in the records of his
office, accompanied by a certificate as above provided, is admissible as evidence that the records of his office
contain no such record or entry.
The above Rule authorized the custodian of documents to certify that despite diligent search, a particular document does not
exist in his office or that a particular entry of a specified tenor was not to be found in a register. As custodians of public
documents, civil registrars are public officers charged with the duty, inter alia, of maintaining a register book where they are
required to enter all applications for marriage licenses, including the names of the applicants, the date the marriage license was
issued and such other relevant data. 6
The certification of "due search and inability to find" issued by the civil registrar of Pasig enjoys probative value, he being the
officer charged under the law to keep a record of all data relative to the issuance of a marriage license. Unaccompanied by any
circumstance of suspicion and pursuant to Section 29, Rule 132 of the Rules of Court, a certificate of "due search and inability
to find" sufficiently proved that his office did not issue marriage license no. 3196182 to the contracting parties.
The fact that private respondent Castro offered only her testimony in support of her petition is, in itself, not a ground to deny her
petition. The failure to offer any other witness to corroborate her testimony is mainly due to the peculiar circumstances of the
case. It will be remembered that the subject marriage was a civil ceremony performed by a judge of a city court. The subject
marriage is one of those commonly known as a "secret marriage" a legally non-existent phrase but ordinarily used to refer to
a civil marriage celebrated without the knowledge of the relatives and/or friends of either or both of the contracting parties. The
records show that the marriage between Castro and Cardenas was initially unknown to the parents of the former.
Surely, the fact that only private respondent Castro testified during the trial cannot be held against her. Her husband, Edwin F.
Cardenas, was duly served with notice of the proceedings and a copy of the petition. Despite receipt thereof, he chose to ignore
the same. For failure to answer, he was properly declared in default. Private respondent cannot be faulted for her husband's
lack of interest to participate in the proceedings. There was absolutely no evidence on record to show that there was collusion
between private respondent and her husband Cardenas.
It is noteworthy to mention that the finding of the appellate court that the marriage between the contracting parties is null and
void for lack of a marriage license does not discount the fact that indeed, a spurious marriage license, purporting to be issued
by the civil registrar of Pasig, may have been presented by Cardenas to the solemnizing officer.
In fine, we hold that, under the circumstances of the case, the documentary and testimonial evidence presented by private
respondent Castro sufficiently established the absence of the subject marriage license.
IN VIEW WHEREOF, the petition is DENIED there being no showing of any reversible error committed by respondent appellate
court.

G.R. No. 138322

October 2, 2001

GRACE J. GARCIA, a.k.a. GRACE J. GARCIA-RECIO, petitioner,


vs.
REDERICK A. RECIO, respondents.
PANGANIBAN, J.:
A divorce obtained abroad by an alien may be recognized in our jurisdiction, provided such decree is valid according to the
national law of the foreigner. However, the divorce decree and the governing personal law of the alien spouse who obtained the
divorce must be proven. Our courts do not take judicial notice of foreign laws and judgment; hence, like any other facts, both the
divorce decree and the national law of the alien must be alleged and proven according to our law on evidence.
The Case
Before us is a Petition for Review under Rule 45 of the Rules of Court, seeking to nullify the January 7, 1999 Decision 1 and the
March 24, 1999 Order2 of the Regional Trial Court of Cabanatuan City, Branch 28, in Civil Case No. 3026-AF. The assailed
Decision disposed as follows:
"WHEREFORE, this Court declares the marriage between Grace J. Garcia and Rederick A. Recio solemnized on January
12, 1994 at Cabanatuan City as dissolved and both parties can now remarry under existing and applicable laws to any
and/or both parties."3
The assailed Order denied reconsideration of the above-quoted Decision.
The Facts
Rederick A. Recio, a Filipino, was married to Editha Samson, an Australian citizen, in Malabon, Rizal, on March 1, 1987. 4 They
lived together as husband and wife in Australia. On May 18, 1989, 5 a decree of divorce, purportedly dissolving the marriage,
was issued by an Australian family court.
On June 26, 1992, respondent became an Australian citizen, as shown by a "Certificate of Australian Citizenship" issued by the
Australian government.6 Petitioner a Filipina and respondent were married on January 12, 1994 in Our Lady of Perpetual
Help Church in Cabanatuan City.7 In their application for a marriage license, respondent was declared as "single" and "Filipino." 8
Starting October 22, 1995, petitioner and respondent lived separately without prior judicial dissolution of their marriage. While
the two were still in Australia, their conjugal assets were divided on May 16, 1996, in accordance with their Statutory
Declarations secured in Australia.9
On March 3, 1998, petitioner filed a Complaint for Declaration of Nullity of Marriage 10 in the court a quo, on the ground of
bigamy respondent allegedly had a prior subsisting marriage at the time he married her on January 12, 1994. She claimed
that she learned of respondent's marriage to Editha Samson only in November, 1997.
In his Answer, respondent averred that, as far back as 1993, he had revealed to petitioner his prior marriage andits subsequent
dissolution.11 He contended that his first marriage to an Australian citizen had been validly dissolved by a divorce decree
obtained in Australian in 1989;12 thus, he was legally capacitated to marry petitioner in 1994.1wphi1.nt
On July 7, 1998 or about five years after the couple's wedding and while the suit for the declaration of nullity was pending
respondent was able to secure a divorce decree from a family court in Sydney, Australia because the "marriage ha[d]
irretrievably broken down."13
Respondent prayed in his Answer that the Complained be dismissed on the ground that it stated no cause of action. 14 The Office
of the Solicitor General agreed with respondent. 15 The court marked and admitted the documentary evidence of both
parties.16 After they submitted their respective memoranda, the case was submitted for resolution. 17
Thereafter, the trial court rendered the assailed Decision and Order.
Ruling of the Trial Court
The trial court declared the marriage dissolved on the ground that the divorce issued in Australia was valid and recognized in
the Philippines. It deemed the marriage ended, but not on the basis of any defect in an essential element of the marriage; that
is, respondent's alleged lack of legal capacity to remarry. Rather, it based its Decision on the divorce decree obtained by
respondent. The Australian divorce had ended the marriage; thus, there was no more martial union to nullify or annual.
Hence, this Petition.18
Issues
Petitioner submits the following issues for our consideration:
"I
The trial court gravely erred in finding that the divorce decree obtained in Australia by the respondent ipso
facto terminated his first marriage to Editha Samson thereby capacitating him to contract a second marriage with the
petitioner.
"2
The failure of the respondent, who is now a naturalized Australian, to present a certificate of legal capacity to marry
constitutes absence of a substantial requisite voiding the petitioner' marriage to the respondent.
"3
The trial court seriously erred in the application of Art. 26 of the Family Code in this case.
"4
The trial court patently and grievously erred in disregarding Arts. 11, 13, 21, 35, 40, 52 and 53 of the Family Code as the
applicable provisions in this case.
"5
The trial court gravely erred in pronouncing that the divorce gravely erred in pronouncing that the divorce decree obtained
by the respondent in Australia ipso facto capacitated the parties to remarry, without first securing a recognition of the
judgment granting the divorce decree before our courts." 19
The Petition raises five issues, but for purposes of this Decision, we shall concentrate on two pivotal ones: (1) whether the
divorce between respondent and Editha Samson was proven, and (2) whether respondent was proven to be legally capacitated
to marry petitioner. Because of our ruling on these two, there is no more necessity to take up the rest.

The Court's Ruling


The Petition is partly meritorious.
First Issue:
Proving the Divorce Between Respondent and Editha Samson
Petitioner assails the trial court's recognition of the divorce between respondent and Editha Samson. Citing Adong v. Cheong
Seng Gee,20 petitioner argues that the divorce decree, like any other foreign judgment, may be given recognition in this
jurisdiction only upon proof of the existence of (1) the foreign law allowing absolute divorce and (2) the alleged divorce decree
itself. She adds that respondent miserably failed to establish these elements.
Petitioner adds that, based on the first paragraph of Article 26 of the Family Code, marriages solemnized abroad are governed
by the law of the place where they were celebrated (the lex loci celebrationist). In effect, the Code requires the presentation of
the foreign law to show the conformity of the marriage in question to the legal requirements of the place where the marriage was
performed.
At the outset, we lay the following basic legal principles as the take-off points for our discussion. Philippine law does not provide
for absolute divorce; hence, our courts cannot grant it. 21 A marriage between two Filipinos cannot be dissolved even by a
divorce obtained abroad, because of Articles 15 22 and 1723 of the Civil Code.24 In mixed marriages involving a Filipino and a
foreigner, Article 2625 of the Family Code allows the former to contract a subsequent marriage in case the divorce is "validly
obtained abroad by the alien spouse capacitating him or her to remarry." 26 A divorce obtained abroad by a couple, who are both
aliens, may be recognized in the Philippines, provided it is consistent with their respective national laws. 27
A comparison between marriage and divorce, as far as pleading and proof are concerned, can be made. Van Dorn v. Romillo
Jr. decrees that "aliens may obtain divorces abroad, which may be recognized in the Philippines, provided they are valid
according to their national law." 28 Therefore, before a foreign divorce decree can be recognized by our courts, the party pleading
it must prove the divorce as a fact and demonstrate its conformity to the foreign law allowing it. 29 Presentation solely of the
divorce decree is insufficient.
Divorce as a Question of Fact
Petitioner insists that before a divorce decree can be admitted in evidence, it must first comply with the registration
requirements under Articles 11, 13 and 52 of the Family Code. These articles read as follows:
"ART. 11. Where a marriage license is required, each of the contracting parties shall file separately a sworn application for
such license with the proper local civil registrar which shall specify the following:
xxx

xxx

xxx

"(5) If previously married, how, when and where the previous marriage was dissolved or annulled;
xxx

xxx

xxx

"ART. 13. In case either of the contracting parties has been previously married, the applicant shall be required to furnish,
instead of the birth of baptismal certificate required in the last preceding article, the death certificate of the deceased
spouse or the judicial decree of annulment or declaration of nullity of his or her previous marriage. x x x.
"ART. 52. The judgment of annulment or of absolute nullity of the marriage, the partition and distribution of the properties
of the spouses, and the delivery of the children's presumptive legitimes shall be recorded in the appropriate civil registry
and registries of property; otherwise, the same shall not affect their persons."
Respondent, on the other hand, argues that the Australian divorce decree is a public document a written official act of an
Australian family court. Therefore, it requires no further proof of its authenticity and due execution.
Respondent is getting ahead of himself. Before a foreign judgment is given presumptive evidentiary value, the document must
first be presented and admitted in evidence. 30 A divorce obtained abroad is proven by the divorce decree itself. Indeed the best
evidence of a judgment is the judgment itself. 31 The decree purports to be a written act or record of an act of an officially body or
tribunal of a foreign country.32
Under Sections 24 and 25 of Rule 132, on the other hand, a writing or document may be proven as a public or official record of
a foreign country by either (1) an official publication or (2) a copy thereof attested 33 by the officer having legal custody of the
document. If the record is not kept in the Philippines, such copy must be (a) accompanied by a certificate issued by the proper
diplomatic or consular officer in the Philippine foreign service stationed in the foreign country in which the record is kept and (b)
authenticated by the seal of his office. 34
The divorce decree between respondent and Editha Samson appears to be an authentic one issued by an Australian family
court.35 However, appearance is not sufficient; compliance with the aforemetioned rules on evidence must be demonstrated.
Fortunately for respondent's cause, when the divorce decree of May 18, 1989 was submitted in evidence, counsel for petitioner
objected, not to its admissibility, but only to the fact that it had not been registered in the Local Civil Registry of Cabanatuan
City.36 The trial court ruled that it was admissible, subject to petitioner's qualification. 37Hence, it was admitted in evidence and
accorded weight by the judge. Indeed, petitioner's failure to object properly rendered the divorce decree admissible as a written
act of the Family Court of Sydney, Australia.38
Compliance with the quoted articles (11, 13 and 52) of the Family Code is not necessary; respondent was no longer bound by
Philippine personal laws after he acquired Australian citizenship in 1992. 39 Naturalization is the legal act of adopting an alien
and clothing him with the political and civil rights belonging to a citizen. 40 Naturalized citizens, freed from the protective cloak of
their former states, don the attires of their adoptive countries. By becoming an Australian, respondent severed his allegiance to
the Philippines and the vinculum juris that had tied him to Philippine personal laws.
Burden of Proving Australian Law
Respondent contends that the burden to prove Australian divorce law falls upon petitioner, because she is the party challenging
the validity of a foreign judgment. He contends that petitioner was satisfied with the original of the divorce decree and was
cognizant of the marital laws of Australia, because she had lived and worked in that country for quite a long time. Besides, the
Australian divorce law is allegedly known by Philippine courts: thus, judges may take judicial notice of foreign laws in the
exercise of sound discretion.
We are not persuaded. The burden of proof lies with "the party who alleges the existence of a fact or thing necessary in the
prosecution or defense of an action."41 In civil cases, plaintiffs have the burden of proving the material allegations of the
complaint when those are denied by the answer; and defendants have the burden of proving the material allegations in their
answer when they introduce new matters.42 Since the divorce was a defense raised by respondent, the burden of proving the
pertinent Australian law validating it falls squarely upon him.

It is well-settled in our jurisdiction that our courts cannot take judicial notice of foreign laws. 43 Like any other facts, they must be
alleged and proved. Australian marital laws are not among those matters that judges are supposed to know by reason of their
judicial function.44 The power of judicial notice must be exercised with caution, and every reasonable doubt upon the subject
should be resolved in the negative.
Second Issue:
Respondent's Legal Capacity to Remarry
Petitioner contends that, in view of the insufficient proof of the divorce, respondent was legally incapacitated to marry her in
1994.
Hence, she concludes that their marriage was void ab initio.
Respondent replies that the Australian divorce decree, which was validly admitted in evidence, adequately established his legal
capacity to marry under Australian law.
Respondent's contention is untenable. In its strict legal sense, divorce means the legal dissolution of a lawful union for a cause
arising after marriage. But divorces are of different types. The two basic ones are (1) absolute divorce or a vinculo
matrimonii and (2) limited divorce or a mensa et thoro. The first kind terminates the marriage, while the second suspends it and
leaves the bond in full force.45 There is no showing in the case at bar which type of divorce was procured by respondent.
Respondent presented a decree nisi or an interlocutory decree a conditional or provisional judgment of divorce. It is in effect
the same as a separation from bed and board, although an absolute divorce may follow after the lapse of the prescribed period
during which no reconciliation is effected. 46
Even after the divorce becomes absolute, the court may under some foreign statutes and practices, still restrict remarriage.
Under some other jurisdictions, remarriage may be limited by statute; thus, the guilty party in a divorce which was granted on
the ground of adultery may be prohibited from remarrying again. The court may allow a remarriage only after proof of good
behavior.47
On its face, the herein Australian divorce decree contains a restriction that reads:
"1. A party to a marriage who marries again before this decree becomes absolute (unless the other party has died)
commits the offence of bigamy." 48
This quotation bolsters our contention that the divorce obtained by respondent may have been restricted. It did not absolutely
establish his legal capacity to remarry according to his national law. Hence, we find no basis for the ruling of the trial court,
which erroneously assumed that the Australian divorce ipso facto restored respondent's capacity to remarry despite the paucity
of evidence on this matter.
We also reject the claim of respondent that the divorce decree raises a disputable presumption or presumptive evidence as to
his civil status based on Section 48, Rule 3949 of the Rules of Court, for the simple reason that no proof has been presented on
the legal effects of the divorce decree obtained under Australian laws.
Significance of the Certificate of Legal Capacity
Petitioner argues that the certificate of legal capacity required by Article 21 of the Family Code was not submitted together with
the application for a marriage license. According to her, its absence is proof that respondent did not have legal capacity to
remarry.
We clarify. To repeat, the legal capacity to contract marriage is determined by the national law of the party concerned. The
certificate mentioned in Article 21 of the Family Code would have been sufficient to establish the legal capacity of respondent,
had he duly presented it in court. A duly authenticated and admitted certificate is prima facie evidence of legal capacity to marry
on the part of the alien applicant for a marriage license. 50
As it is, however, there is absolutely no evidence that proves respondent's legal capacity to marry petitioner. A review of the
records before this Court shows that only the following exhibits were presented before the lower court: (1) for petitioner: (a)
Exhibit "A" Complaint;51 (b) Exhibit "B" Certificate of Marriage Between Rederick A. Recto (Filipino-Australian) and Grace J.
Garcia (Filipino) on January 12, 1994 in Cabanatuan City, Nueva Ecija; 52(c) Exhibit "C" Certificate of Marriage Between
Rederick A. Recio (Filipino) and Editha D. Samson (Australian) on March 1, 1987 in Malabon, Metro Manila; 53 (d) Exhibit "D"
Office of the City Registrar of Cabanatuan City Certification that no information of annulment between Rederick A. Recto and
Editha D. Samson was in its records;54 and (e) Exhibit "E" Certificate of Australian Citizenship of Rederick A. Recto; 55 (2) for
respondent: (Exhibit "1" Amended Answer; 56 (b) Exhibit "S" Family Law Act 1975 Decree Nisi of Dissolution of Marriage in
the Family Court of Australia;57 (c) Exhibit "3" Certificate of Australian Citizenship of Rederick A. Recto; 58 (d) Exhibit "4"
Decree Nisi of Dissolution of Marriage in the Family Court of Australia Certificate; 59 and Exhibit "5" Statutory Declaration of the
Legal Separation Between Rederick A. Recto and Grace J. Garcia Recio since October 22, 1995. 60
Based on the above records, we cannot conclude that respondent, who was then a naturalized Australian citizen, was legally
capacitated to marry petitioner on January 12, 1994. We agree with petitioner's contention that the court a quo erred in finding
that the divorce decree ipso facto clothed respondent with the legal capacity to remarry without requiring him to adduce
sufficient evidence to show the Australian personal law governing his status; or at the very least, to prove his legal capacity to
contract the second marriage.
Neither can we grant petitioner's prayer to declare her marriage to respondent null and void on the ground of bigamy. After all, it
may turn out that under Australian law, he was really capacitated to marry petitioner as a direct result of the divorce decree.
Hence, we believe that the most judicious course is to remand this case to the trial court to receive evidence, if any, which show
petitioner's legal capacity to marry petitioner. Failing in that, then the court a quo may declare a nullity of the parties' marriage
on the ground of bigamy, there being already in evidence two existing marriage certificates, which were both obtained in the
Philippines, one in Malabon, Metro Manila dated March 1, 1987 and the other, in Cabanatuan City dated January 12, 1994.
WHEREFORE, in the interest of orderly procedure and substantial justice, we REMAND the case to the court a quo for the
purpose of receiving evidence which conclusively show respondent's legal capacity to marry petitioner; and failing in that, of
declaring the parties' marriage void on the ground of bigamy, as above discussed. No costs.
SO ORDERED.

G.R. No. 142820

June 20, 2003

WOLFGANG O. ROEHR, petitioner,


vs.
MARIA CARMEN D. RODRIGUEZ, HON. JUDGE JOSEFINA GUEVARA-SALONGA, Presiding Judge of Makati RTC,
Branch 149, respondents.
QUISUMBING, J.:
At the core of the present controversy are issues of (a) grave abuse of discretion allegedly committed by public respondent and
(b) lack of jurisdiction of the regional trial court, in matters that spring from a divorce decree obtained abroad by petitioner.
In this special civil action for certiorari, petitioner assails (a) the order 1 dated September 30, 1999 of public respondent Judge
Josefina Guevara-Salonga, Presiding Judge of Makati Regional Trial Court, 2 Branch 149, in Civil Case No. 96-1389 for
declaration of nullity of marriage, and (b) the order 3 dated March 31, 2000 denying his motion for reconsideration. The assailed
orders partially set aside the trial courts order dismissing Civil Case No. 96-1389, for the purpose of resolving issues relating to
the property settlement of the spouses and the custody of their children.
Petitioner Wolfgang O. Roehr, a German citizen and resident of Germany, married private respondent Carmen Rodriguez, a
Filipina, on December 11, 1980 in Hamburg, Germany. Their marriage was subsequently ratified on February 14, 1981 in
Tayasan, Negros Oriental.4 Out of their union were born Carolynne and Alexandra Kristine on November 18, 1981 and October
25, 1987, respectively.
On August 28, 1996, private respondent filed a petition 5 for declaration of nullity of marriage before the Regional Trial Court
(RTC) of Makati City. On February 6, 1997, petitioner filed a motion to dismiss, 6 but it was denied by the trial court in its
order7 dated May 28, 1997.
On June 5, 1997, petitioner filed a motion for reconsideration, but was also denied in an order 8 dated August 13, 1997. On
September 5, 1997, petitioner filed a petition for certiorari with the Court of Appeals. On November 27, 1998, the appellate court
denied the petition and remanded the case to the RTC.
Meanwhile, petitioner obtained a decree of divorce from the Court of First Instance of Hamburg-Blankenese, promulgated on
December 16, 1997.
The decree provides in part:
[T]he Court of First Instance, Hamburg-Blankenese, Branch 513, has ruled through Judge van Buiren of the Court of First
Instance on the basis of the oral proceedings held on 4 Nov. 1997:
The marriage of the Parties contracted on 11 December 1980 before the Civil Registrar of Hamburg-Altona is hereby
dissolved.
The parental custody for the children
Carolynne Roehr, born 18 November 1981
Alexandra Kristine Roehr, born on 25 October 1987
is granted to the father.
The litigation expenses shall be assumed by the Parties. 9
In view of said decree, petitioner filed a Second Motion to Dismiss on May 20, 1999 on the ground that the trial court had no
jurisdiction over the subject matter of the action or suit as a decree of divorce had already been promulgated dissolving the
marriage of petitioner and private respondent.
On July 14, 1999, Judge Guevara-Salonga issued an order granting petitioners motion to dismiss. Private respondent filed a
Motion for Partial Reconsideration, with a prayer that the case proceed for the purpose of determining the issues of custody of
children and the distribution of the properties between petitioner and private respondent.
On August 18, 1999, an Opposition to the Motion for Partial Reconsideration was filed by the petitioner on the ground that there
is nothing to be done anymore in the instant case as the marital tie between petitioner Wolfgang Roehr and respondent Ma.
Carmen D. Rodriguez had already been severed by the decree of divorce promulgated by the Court of First Instance of
Hamburg, Germany on December 16, 1997 and in view of the fact that said decree of divorce had already been recognized by
the RTC in its order of July 14, 1999, through the implementation of the mandate of Article 26 of the Family Code, 10 endowing
the petitioner with the capacity to remarry under the Philippine law.
On September 30, 1999, respondent judge issued the assailed order partially setting aside her order dated July 14, 1999 for the
purpose of tackling the issues of property relations of the spouses as well as support and custody of their children. The pertinent
portion of said order provides:
Acting on the Motion for Partial Reconsideration of the Order dated July 14, 1999 filed by petitioner thru counsel which
was opposed by respondent and considering that the second paragraph of Article 26 of the Family Code was included as
an amendment thru Executive Order 227, to avoid the absurd situation of a Filipino as being still married to his or her alien
spouse though the latter is no longer married to the Filipino spouse because he/she had obtained a divorce abroad which
is recognized by his/her national law, and considering further the effects of the termination of the marriage under Article 43
in relation to Article 50 and 52 of the same Code, which include the dissolution of the property relations of the spouses,
and the support and custody of their children, the Order dismissing this case is partially set aside with respect to these
matters which may be ventilated in this Court.
SO ORDERED.11 (Emphasis supplied.)
Petitioner filed a timely motion for reconsideration on October 19, 1999, which was denied by respondent judge in an order
dated March 31, 2000.12
Petitioner ascribes lack of jurisdiction of the trial court and grave abuse of discretion on the part of respondent judge. He cites
as grounds for his petition the following:
1. Partially setting aside the order dated July 14, 1999 dismissing the instant case is not allowed by 1997 Rules of Civil
Procedure.13
2. Respondent Maria Carmen Rodriguez by her motion for Partial Reconsideration had recognized and admitted the
Divorce Decision obtained by her ex-husband in Hamburg, Germany.14
3. There is nothing left to be tackled by the Honorable Court as there are no conjugal assets alleged in the Petition for
Annulment of Marriage and in the Divorce petition, and the custody of the children had already been awarded to Petitioner
Wolfgang Roehr.15
Pertinent in this case before us are the following issues:

1. Whether or not respondent judge gravely abused her discretion in issuing her order dated September 30, 1999, which
partially modified her order dated July 14, 1999; and
2. Whether or not respondent judge gravely abused her discretion when she assumed and retained jurisdiction over the
present case despite the fact that petitioner has already obtained a divorce decree from a German court.
On the first issue, petitioner asserts that the assailed order of respondent judge is completely inconsistent with her previous
order and is contrary to Section 3, Rule 16, Rules of Civil Procedure, which provides:
Sec. 3. Resolution of motion - After the hearing, the court may dismiss the action or claim, deny the motion, or order the
amendment of the pleading.
The court shall not defer the resolution of the motion for the reason that the ground relied upon is not indubitable.
In every case, the resolution shall state clearly and distinctly the reasons therefor. (Emphasis supplied.)
Petitioner avers that a courts action on a motion is limited to dismissing the action or claim, denying the motion, or ordering the
amendment of the pleading.
Private respondent, on her part, argues that the RTC can validly reconsider its order dated July 14, 1999 because it had not yet
attained finality, given the timely filing of respondents motion for reconsideration.
Pertinent to this issue is Section 3 in relation to Section 7, Rule 37 of the 1997 Rules of Civil Procedure, which provides:
Sec. 3. Action upon motion for new trial or reconsideration.The trial court may set aside the judgment or final order and
grant a new trial, upon such terms as may be just, or may deny the motion. If the court finds that excessive damages have
been awarded or that the judgment or final order is contrary to the evidence or law, it may amend such judgment or final
order accordingly.
Sec. 7. Partial new trial or reconsideration.If the grounds for a motion under this Rule appear to the court to affect the
issues as to only a part, or less than all of the matters in controversy, or only one, or less than all, of the parties to it, the
court may order a new trial or grant reconsideration as to such issues if severable without interfering with the judgment or
final order upon the rest. (Emphasis supplied.)
It is clear from the foregoing rules that a judge can order a partial reconsideration of a case that has not yet attained finality.
Considering that private respondent filed a motion for reconsideration within the reglementary period, the trial court's decision of
July 14, 1999 can still be modified. Moreover, in Saado v. Court of Appeals,16we held that the court could modify or alter a
judgment even after the same has become executory whenever circumstances transpire rendering its decision unjust and
inequitable, as where certain facts and circumstances justifying or requiring such modification or alteration transpired after the
judgment has become final and executory17 and when it becomes imperative in the higher interest of justice or when
supervening events warrant it.18 In our view, there are even more compelling reasons to do so when, as in this case, judgment
has not yet attained finality.
Anent the second issue, petitioner claims that respondent judge committed grave abuse of discretion when she partially set
aside her order dated July 14, 1999, despite the fact that petitioner has already obtained a divorce decree from the Court of
First Instance of Hamburg, Germany.
In Garcia v. Recio,19 Van Dorn v. Romillo, Jr.,20 and Llorente v. Court of Appeals,21 we consistently held that a divorce obtained
abroad by an alien may be recognized in our jurisdiction, provided such decree is valid according to the national law of the
foreigner. Relevant to the present case is Pilapil v. Ibay-Somera,22 where this Court specifically recognized the validity of a
divorce obtained by a German citizen in his country, the Federal Republic of Germany. We held in Pilapil that a foreign divorce
and its legal effects may be recognized in the Philippines insofar as respondent is concerned in view of the nationality principle
in our civil law on the status of persons.
In this case, the divorce decree issued by the German court dated December 16, 1997 has not been challenged by either of the
parties. In fact, save for the issue of parental custody, even the trial court recognized said decree to be valid and binding,
thereby endowing private respondent the capacity to remarry. Thus, the present controversy mainly relates to the award of the
custody of their two children, Carolynne and Alexandra Kristine, to petitioner.
As a general rule, divorce decrees obtained by foreigners in other countries are recognizable in our jurisdiction, but the legal
effects thereof, e.g. on custody, care and support of the children, must still be determined by our courts. 23 Before our courts can
give the effect of res judicata to a foreign judgment, such as the award of custody to petitioner by the German court, it must be
shown that the parties opposed to the judgment had been given ample opportunity to do so on grounds allowed under Rule 39,
Section 50 of the Rules of Court (now Rule 39, Section 48, 1997 Rules of Civil Procedure), to wit:
SEC. 50. Effect of foreign judgments. - The effect of a judgment of a tribunal of a foreign country, having jurisdiction to
pronounce the judgment is as follows:
(a) In case of a judgment upon a specific thing, the judgment is conclusive upon the title to the thing;
(b) In case of a judgment against a person, the judgment is presumptive evidence of a right as between the parties and
their successors in interest by a subsequent title; but the judgment may be repelled by evidence of a want of jurisdiction,
want of notice to the party, collusion, fraud, or clear mistake of law or fact.
It is essential that there should be an opportunity to challenge the foreign judgment, in order for the court in this jurisdiction to
properly determine its efficacy. In this jurisdiction, our Rules of Court clearly provide that with respect to actions in personam, as
distinguished from actions in rem, a foreign judgment merely constitutes prima facieevidence of the justness of the claim of a
party and, as such, is subject to proof to the contrary.24
In the present case, it cannot be said that private respondent was given the opportunity to challenge the judgment of the
German court so that there is basis for declaring that judgment as res judicata with regard to the rights of petitioner to have
parental custody of their two children. The proceedings in the German court were summary. As to what was the extent of private
respondents participation in the proceedings in the German court, the records remain unclear. The divorce decree itself states
that neither has she commented on the proceedings 25 nor has she given her opinion to the Social Services Office. 26 Unlike
petitioner who was represented by two lawyers, private respondent had no counsel to assist her in said proceedings. 27 More
importantly, the divorce judgment was issued to petitioner by virtue of the German Civil Code provision to the effect that when a
couple lived separately for three years, the marriage is deemed irrefutably dissolved. The decree did not touch on the issue as
to who the offending spouse was. Absent any finding that private respondent is unfit to obtain custody of the children, the trial
court was correct in setting the issue for hearing to determine the issue of parental custody, care, support and education mindful
of the best interests of the children. This is in consonance with the provision in the Child and Youth Welfare Code that the childs
welfare is always the paramount consideration in all questions concerning his care and custody. 28
On the matter of property relations, petitioner asserts that public respondent exceeded the bounds of her jurisdiction when she
claimed cognizance of the issue concerning property relations between petitioner and private respondent. Private respondent
herself has admitted in Par. 14 of her petition for declaration of nullity of marriage dated August 26, 1996 filed with the RTC of
Makati, subject of this case, that: "[p]etitioner and respondent have not acquired any conjugal or community property nor have

they incurred any debts during their marriage." 29Herein petitioner did not contest this averment. Basic is the rule that a court
shall grant relief warranted by the allegations and the proof. 30 Given the factual admission by the parties in their pleadings that
there is no property to be accounted for, respondent judge has no basis to assert jurisdiction in this case to resolve a matter no
longer deemed in controversy.
In sum, we find that respondent judge may proceed to determine the issue regarding the custody of the two children born of the
union between petitioner and private respondent. Private respondent erred, however, in claiming cognizance to settle the matter
of property relations of the parties, which is not at issue.
WHEREFORE, the orders of the Regional Trial Court of Makati, Branch 149, issued on September 30, 1999 and March 31,
2000 are AFFIRMED with MODIFICATION. We hereby declare that the trial court has jurisdiction over the issue between the
parties as to who has parental custody, including the care, support and education of the children, namely Carolynne and
Alexandra Kristine Roehr. Let the records of this case be remanded promptly to the trial court for continuation of appropriate
proceedings. No pronouncement as to costs.
SO ORDERED.

G.R. No. 80116 June 30, 1989

IMELDA MANALAYSAY PILAPIL, petitioner,


vs.
HON. CORONA IBAY-SOMERA, in her capacity as Presiding Judge of the Regional Trial Court of Manila, Branch XXVI;
HON. LUIS C. VICTOR, in his capacity as the City Fiscal of Manila; and ERICH EKKEHARD GEILING, respondents.

REGALADO, J.:
An ill-starred marriage of a Filipina and a foreigner which ended in a foreign absolute divorce, only to be followed by a criminal
infidelity suit of the latter against the former, provides Us the opportunity to lay down a decisional rule on what hitherto appears
to be an unresolved jurisdictional question.
On September 7, 1979, petitioner Imelda Manalaysay Pilapil, a Filipino citizen, and private respondent Erich Ekkehard Geiling,
a German national, were married before the Registrar of Births, Marriages and Deaths at Friedensweiler in the Federal Republic
of Germany. The marriage started auspiciously enough, and the couple lived together for some time in Malate, Manila where
their only child, Isabella Pilapil Geiling, was born on April 20, 1980. 1
Thereafter, marital discord set in, with mutual recriminations between the spouses, followed by a separation de facto between
them.
After about three and a half years of marriage, such connubial disharmony eventuated in private respondent initiating a divorce
proceeding against petitioner in Germany before the Schoneberg Local Court in January, 1983. He claimed that there was
failure of their marriage and that they had been living apart since April, 1982. 2
Petitioner, on the other hand, filed an action for legal separation, support and separation of property before the Regional Trial
Court of Manila, Branch XXXII, on January 23, 1983 where the same is still pending as Civil Case No. 83-15866. 3
On January 15, 1986, Division 20 of the Schoneberg Local Court, Federal Republic of Germany, promulgated a decree of
divorce on the ground of failure of marriage of the spouses. The custody of the child was granted to petitioner. The records
show that under German law said court was locally and internationally competent for the divorce proceeding and that the
dissolution of said marriage was legally founded on and authorized by the applicable law of that foreign jurisdiction. 4
On June 27, 1986, or more than five months after the issuance of the divorce decree, private respondent filed two complaints
for adultery before the City Fiscal of Manila alleging that, while still married to said respondent, petitioner "had an affair with a
certain William Chia as early as 1982 and with yet another man named Jesus Chua sometime in 1983". Assistant Fiscal Jacinto
A. de los Reyes, Jr., after the corresponding investigation, recommended the dismissal of the cases on the ground of
insufficiency of evidence. 5 However, upon review, the respondent city fiscal approved a resolution, dated January 8, 1986,
directing the filing of two complaints for adultery against the petitioner. 6 The complaints were accordingly filed and were
eventually raffled to two branches of the Regional Trial Court of Manila. The case entitled "People of the Philippines vs. Imelda
Pilapil and William Chia", docketed as Criminal Case No. 87-52435, was assigned to Branch XXVI presided by the respondent
judge; while the other case, "People of the Philippines vs. Imelda Pilapil and James Chua", docketed as Criminal Case No. 8752434 went to the sala of Judge Leonardo Cruz, Branch XXV, of the same court. 7
On March 14, 1987, petitioner filed a petition with the Secretary of Justice asking that the aforesaid resolution of respondent
fiscal be set aside and the cases against her be dismissed. 8 A similar petition was filed by James Chua, her co-accused in
Criminal Case No. 87-52434. The Secretary of Justice, through the Chief State Prosecutor, gave due course to both petitions
and directed the respondent city fiscal to inform the Department of Justice "if the accused have already been arraigned and if
not yet arraigned, to move to defer further proceedings" and to elevate the entire records of both cases to his office for review.

Petitioner thereafter filed a motion in both criminal cases to defer her arraignment and to suspend further proceedings
thereon. 10 As a consequence, Judge Leonardo Cruz suspended proceedings in Criminal Case No. 87-52434. On the other
hand, respondent judge merely reset the date of the arraignment in Criminal Case No. 87-52435 to April 6, 1987. Before such
scheduled date, petitioner moved for the cancellation of the arraignment and for the suspension of proceedings in said Criminal
Case No. 87-52435 until after the resolution of the petition for review then pending before the Secretary of Justice. 11 A motion to
quash was also filed in the same case on the ground of lack of jurisdiction, 12 which motion was denied by the respondent judge
in an order dated September 8, 1987. The same order also directed the arraignment of both accused therein, that is, petitioner
and William Chia. The latter entered a plea of not guilty while the petitioner refused to be arraigned. Such refusal of the
petitioner being considered by respondent judge as direct contempt, she and her counsel were fined and the former was
ordered detained until she submitted herself for arraignment. 13 Later, private respondent entered a plea of not guilty. 14
On October 27, 1987, petitioner filed this special civil action for certiorari and prohibition, with a prayer for a temporary
restraining order, seeking the annulment of the order of the lower court denying her motion to quash. The petition is anchored
on the main ground that the court is without jurisdiction "to try and decide the charge of adultery, which is a private offense that
cannot be prosecuted de officio (sic), since the purported complainant, a foreigner, does not qualify as an offended spouse
having obtained a final divorce decree under his national law prior to his filing the criminal complaint." 15
On October 21, 1987, this Court issued a temporary restraining order enjoining the respondents from implementing the
aforesaid order of September 8, 1987 and from further proceeding with Criminal Case No. 87-52435. Subsequently, on March
23, 1988 Secretary of Justice Sedfrey A. Ordoez acted on the aforesaid petitions for review and, upholding petitioner's
ratiocinations, issued a resolution directing the respondent city fiscal to move for the dismissal of the complaints against the
petitioner. 16
We find this petition meritorious. The writs prayed for shall accordingly issue.
Under Article 344 of the Revised Penal Code, 17 the crime of adultery, as well as four other crimes against chastity, cannot be
prosecuted except upon a sworn written complaint filed by the offended spouse. It has long since been established, with
unwavering consistency, that compliance with this rule is a jurisdictional, and not merely a formal, requirement. 18 While in point
of strict law the jurisdiction of the court over the offense is vested in it by the Judiciary Law, the requirement for a sworn written
complaint is just as jurisdictional a mandate since it is that complaint which starts the prosecutory proceeding 19 and without
which the court cannot exercise its jurisdiction to try the case.
Now, the law specifically provides that in prosecutions for adultery and concubinage the person who can legally file the
complaint should be the offended spouse, and nobody else. Unlike the offenses of seduction, abduction, rape and acts of
lasciviousness, no provision is made for the prosecution of the crimes of adultery and concubinage by the parents,
grandparents or guardian of the offended party. The so-called exclusive and successive rule in the prosecution of the first four
offenses above mentioned do not apply to adultery and concubinage. It is significant that while the State, as parens patriae, was
added and vested by the 1985 Rules of Criminal Procedure with the power to initiate the criminal action for a deceased or
incapacitated victim in the aforesaid offenses of seduction, abduction, rape and acts of lasciviousness, in default of her parents,
grandparents or guardian, such amendment did not include the crimes of adultery and concubinage. In other words, only the
offended spouse, and no other, is authorized by law to initiate the action therefor.
Corollary to such exclusive grant of power to the offended spouse to institute the action, it necessarily follows that such initiator
must have the status, capacity or legal representation to do so at the time of the filing of the criminal action. This is a familiar
and express rule in civil actions; in fact, lack of legal capacity to sue, as a ground for a motion to dismiss in civil cases, is
determined as of the filing of the complaint or petition.

The absence of an equivalent explicit rule in the prosecution of criminal cases does not mean that the same requirement and
rationale would not apply. Understandably, it may not have been found necessary since criminal actions are generally and
fundamentally commenced by the State, through the People of the Philippines, the offended party being merely the complaining
witness therein. However, in the so-called "private crimes" or those which cannot be prosecuted de oficio, and the present
prosecution for adultery is of such genre, the offended spouse assumes a more predominant role since the right to commence
the action, or to refrain therefrom, is a matter exclusively within his power and option.
This policy was adopted out of consideration for the aggrieved party who might prefer to suffer the outrage in silence rather than
go through the scandal of a public trial. 20 Hence, as cogently argued by petitioner, Article 344 of the Revised Penal Code thus
presupposes that the marital relationship is still subsisting at the time of the institution of the criminal action for, adultery. This is
a logical consequence since the raison d'etre of said provision of law would be absent where the supposed offended party had
ceased to be the spouse of the alleged offender at the time of the filing of the criminal case. 21
In these cases, therefore, it is indispensable that the status and capacity of the complainant to commence the action be
definitely established and, as already demonstrated, such status or capacity must indubitably exist as of the time he initiates the
action. It would be absurd if his capacity to bring the action would be determined by his status before or subsequent to the
commencement thereof, where such capacity or status existed prior to but ceased before, or was acquired subsequent to but
did not exist at the time of, the institution of the case. We would thereby have the anomalous spectacle of a party bringing suit at
the very time when he is without the legal capacity to do so.
To repeat, there does not appear to be any local precedential jurisprudence on the specific issue as to when precisely the status
of a complainant as an offended spouse must exist where a criminal prosecution can be commenced only by one who in law
can be categorized as possessed of such status. Stated differently and with reference to the present case, the inquiry ;would be
whether it is necessary in the commencement of a criminal action for adultery that the marital bonds between the complainant
and the accused be unsevered and existing at the time of the institution of the action by the former against the latter.
American jurisprudence, on cases involving statutes in that jurisdiction which are in pari materia with ours, yields the rule
that after a divorce has been decreed, the innocent spouse no longer has the right to institute proceedings against the
offenders where the statute provides that the innocent spouse shall have the exclusive right to institute a prosecution for
adultery. Where, however, proceedings have been properly commenced, a divorce subsequently granted can have no legal
effect on the prosecution of the criminal proceedings to a conclusion. 22
In the cited Loftus case, the Supreme Court of Iowa held that
'No prosecution for adultery can be commenced except on the complaint of the husband or wife.' Section 4932,
Code. Though Loftus was husband of defendant when the offense is said to have been committed, he had ceased
to be such when the prosecution was begun; and appellant insists that his status was not such as to entitle him to
make the complaint. We have repeatedly said that the offense is against the unoffending spouse, as well as the
state, in explaining the reason for this provision in the statute; and we are of the opinion that the unoffending spouse
must be such when the prosecution is commenced. (Emphasis supplied.)
We see no reason why the same doctrinal rule should not apply in this case and in our jurisdiction, considering our statutory law
and jural policy on the matter. We are convinced that in cases of such nature, the status of the complainant vis-a-vis the
accused must be determined as of the time the complaint was filed. Thus, the person who initiates the adultery case must be an
offended spouse, and by this is meant that he is still married to the accused spouse, at the time of the filing of the complaint.
In the present case, the fact that private respondent obtained a valid divorce in his country, the Federal Republic of Germany, is
admitted. Said divorce and its legal effects may be recognized in the Philippines insofar as private respondent is concerned 23 in
view of the nationality principle in our civil law on the matter of status of persons.
Thus, in the recent case of Van Dorn vs. Romillo, Jr., et al., 24 after a divorce was granted by a United States court between
Alice Van Dornja Filipina, and her American husband, the latter filed a civil case in a trial court here alleging that her business
concern was conjugal property and praying that she be ordered to render an accounting and that the plaintiff be granted the
right to manage the business. Rejecting his pretensions, this Court perspicuously demonstrated the error of such stance, thus:
There can be no question as to the validity of that Nevada divorce in any of the States of the United States. The
decree is binding on private respondent as an American citizen. For instance, private respondent cannot sue
petitioner, as her husband, in any State of the Union. ...
It is true that owing to the nationality principle embodied in Article 15 of the Civil Code, only Philippine nationals are
covered by the policy against absolute divorces the same being considered contrary to our concept of public policy
and morality. However, aliens may obtain divorces abroad, which may be recognized in the Philippines, provided
they are valid according to their national law. ...
Thus, pursuant to his national law, private respondent is no longer the husband of petitioner. He would have no
standing to sue in the case below as petitioner's husband entitled to exercise control over conjugal assets. ... 25
Under the same considerations and rationale, private respondent, being no longer the husband of petitioner, had no legal
standing to commence the adultery case under the imposture that he was the offended spouse at the time he filed suit.
The allegation of private respondent that he could not have brought this case before the decree of divorce for lack of
knowledge, even if true, is of no legal significance or consequence in this case. When said respondent initiated the divorce
proceeding, he obviously knew that there would no longer be a family nor marriage vows to protect once a dissolution of the
marriage is decreed. Neither would there be a danger of introducing spurious heirs into the family, which is said to be one of the
reasons for the particular formulation of our law on adultery, 26 since there would thenceforth be no spousal relationship to speak
of. The severance of the marital bond had the effect of dissociating the former spouses from each other, hence the actuations of
one would not affect or cast obloquy on the other.
The aforecited case of United States vs. Mata cannot be successfully relied upon by private respondent. In applying Article 433
of the old Penal Code, substantially the same as Article 333 of the Revised Penal Code, which punished adultery "although the
marriage be afterwards declared void", the Court merely stated that "the lawmakers intended to declare adulterous the infidelity
of a married woman to her marital vows, even though it should be made to appear that she is entitled to have her marriage
contract declared null and void, until and unless she actually secures a formal judicial declaration to that effect". Definitely, it
cannot be logically inferred therefrom that the complaint can still be filed after the declaration of nullity because such declaration
that the marriage is void ab initio is equivalent to stating that it never existed. There being no marriage from the beginning, any
complaint for adultery filed after said declaration of nullity would no longer have a leg to stand on. Moreover, what was
consequently contemplated and within the purview of the decision in said case is the situation where the criminal action for
adultery was filed before the termination of the marriage by a judicial declaration of its nullity ab initio. The same rule and
requisite would necessarily apply where the termination of the marriage was effected, as in this case, by a valid foreign divorce.
Private respondent's invocation of Donio-Teves, et al. vs. Vamenta, hereinbefore cited, 27 must suffer the same fate of
inapplicability. A cursory reading of said case reveals that the offended spouse therein had duly and seasonably filed a
complaint for adultery, although an issue was raised as to its sufficiency but which was resolved in favor of the complainant.
Said case did not involve a factual situation akin to the one at bar or any issue determinative of the controversy herein.

WHEREFORE, the questioned order denying petitioner's motion to quash is SET ASIDE and another one
enteredDISMISSING the complaint in Criminal Case No. 87-52435 for lack of jurisdiction. The temporary restraining order
issued in this case on October 21, 1987 is hereby made permanent.
SO ORDERED.

G.R. No. L-68470 October 8, 1985

ALICE REYES VAN DORN, petitioner,


vs.
HON. MANUEL V. ROMILLO, JR., as Presiding Judge of Branch CX, Regional Trial Court of the National Capital Region
Pasay City and RICHARD UPTON respondents.

MELENCIO-HERRERA, J.:\
In this Petition for certiorari and Prohibition, petitioner Alice Reyes Van Dorn seeks to set aside the Orders, dated September
15, 1983 and August 3, 1984, in Civil Case No. 1075-P, issued by respondent Judge, which denied her Motion to Dismiss said
case, and her Motion for Reconsideration of the Dismissal Order, respectively.
The basic background facts are that petitioner is a citizen of the Philippines while private respondent is a citizen of the United
States; that they were married in Hongkong in 1972; that, after the marriage, they established their residence in the Philippines;
that they begot two children born on April 4, 1973 and December 18, 1975, respectively; that the parties were divorced in
Nevada, United States, in 1982; and that petitioner has re-married also in Nevada, this time to Theodore Van Dorn.
Dated June 8, 1983, private respondent filed suit against petitioner in Civil Case No. 1075-P of the Regional Trial Court, Branch
CXV, in Pasay City, stating that petitioner's business in Ermita, Manila, (the Galleon Shop, for short), is conjugal property of the
parties, and asking that petitioner be ordered to render an accounting of that business, and that private respondent be declared
with right to manage the conjugal property. Petitioner moved to dismiss the case on the ground that the cause of action is
barred by previous judgment in the divorce proceedings before the Nevada Court wherein respondent had acknowledged that
he and petitioner had "no community property" as of June 11, 1982. The Court below denied the Motion to Dismiss in the
mentioned case on the ground that the property involved is located in the Philippines so that the Divorce Decree has no bearing
in the case. The denial is now the subject of this certiorari proceeding.
Generally, the denial of a Motion to Dismiss in a civil case is interlocutory and is not subject to appeal. certiorari and Prohibition
are neither the remedies to question the propriety of an interlocutory order of the trial Court. However, when a grave abuse of
discretion was patently committed, or the lower Court acted capriciously and whimsically, then it devolves upon this Court in a
certiorari proceeding to exercise its supervisory authority and to correct the error committed which, in such a case, is equivalent
to lack of jurisdiction. 1 Prohibition would then lie since it would be useless and a waste of time to go ahead with the
proceedings. 2 Weconsider the petition filed in this case within the exception, and we have given it due course.
For resolution is the effect of the foreign divorce on the parties and their alleged conjugal property in the Philippines.
Petitioner contends that respondent is estopped from laying claim on the alleged conjugal property because of the
representation he made in the divorce proceedings before the American Court that they had no community of property; that the
Galleon Shop was not established through conjugal funds, and that respondent's claim is barred by prior judgment.
For his part, respondent avers that the Divorce Decree issued by the Nevada Court cannot prevail over the prohibitive laws of
the Philippines and its declared national policy; that the acts and declaration of a foreign Court cannot, especially if the same is
contrary to public policy, divest Philippine Courts of jurisdiction to entertain matters within its jurisdiction.
For the resolution of this case, it is not necessary to determine whether the property relations between petitioner and private
respondent, after their marriage, were upon absolute or relative community property, upon complete separation of property, or
upon any other regime. The pivotal fact in this case is the Nevada divorce of the parties.
The Nevada District Court, which decreed the divorce, had obtained jurisdiction over petitioner who appeared in person before
the Court during the trial of the case. It also obtained jurisdiction over private respondent who, giving his address as No. 381
Bush Street, San Francisco, California, authorized his attorneys in the divorce case, Karp & Gradt Ltd., to agree to the divorce
on the ground of incompatibility in the understanding that there were neither community property nor community
obligations. 3 As explicitly stated in the Power of Attorney he executed in favor of the law firm of KARP & GRAD LTD., 336 W.
Liberty, Reno, Nevada, to represent him in the divorce proceedings:
xxx xxx xxx
You are hereby authorized to accept service of Summons, to file an Answer, appear on my behalf and do an things
necessary and proper to represent me, without further contesting, subject to the following:
1. That my spouse seeks a divorce on the ground of incompatibility.
2. That there is no community of property to be adjudicated by the Court.
3. 'I'hat there are no community obligations to be adjudicated by the court.
xxx xxx xxx 4
There can be no question as to the validity of that Nevada divorce in any of the States of the United States. The decree is
binding on private respondent as an American citizen. For instance, private respondent cannot sue petitioner, as her husband,
in any State of the Union. What he is contending in this case is that the divorce is not valid and binding in this jurisdiction, the
same being contrary to local law and public policy.
It is true that owing to the nationality principle embodied in Article 15 of the Civil Code, 5 only Philippine nationals are covered by
the policy against absolute divorces the same being considered contrary to our concept of public police and morality. However,
aliens may obtain divorces abroad, which may be recognized in the Philippines, provided they are valid according to their
national law. 6 In this case, the divorce in Nevada released private respondent from the marriage from the standards of
American law, under which divorce dissolves the marriage. As stated by the Federal Supreme Court of the United States in
Atherton vs. Atherton, 45 L. Ed. 794, 799:
The purpose and effect of a decree of divorce from the bond of matrimony by a court of competent jurisdiction are to
change the existing status or domestic relation of husband and wife, and to free them both from the bond. The
marriage tie when thus severed as to one party, ceases to bind either. A husband without a wife, or a wife without a
husband, is unknown to the law. When the law provides, in the nature of a penalty. that the guilty party shall not
marry again, that party, as well as the other, is still absolutely freed from the bond of the former marriage.
Thus, pursuant to his national law, private respondent is no longer the husband of petitioner. He would have no standing to sue
in the case below as petitioner's husband entitled to exercise control over conjugal assets. As he is bound by the Decision of his
own country's Court, which validly exercised jurisdiction over him, and whose decision he does not repudiate, he is estopped by
his own representation before said Court from asserting his right over the alleged conjugal property.
To maintain, as private respondent does, that, under our laws, petitioner has to be considered still married to private respondent
and still subject to a wife's obligations under Article 109, et. seq. of the Civil Code cannot be just. Petitioner should not be
obliged to live together with, observe respect and fidelity, and render support to private respondent. The latter should not
continue to be one of her heirs with possible rights to conjugal property. She should not be discriminated against in her own
country if the ends of justice are to be served.

WHEREFORE, the Petition is granted, and respondent Judge is hereby ordered to dismiss the Complaint filed in Civil Case No.
1075-P of his Court.
Without costs.
SO ORDERED

G.R. No. 105308. September 25, 1998]

HERBERT CANG, petitioner, vs. COURT OF APPEALS and Spouses RONALD V. CLAVANO and MARIA CLARA
CLAVANO, respondents.
DECISION
ROMERO, J.:
Can minor children be legally adopted without the written consent of a natural parent on the ground that the latter has
abandoned them? The answer to this interesting query, certainly not one of first impression, would have to be reached, not
solely on the basis of law and jurisprudence, but also the hard reality presented by the facts of the case.
This is the question posed before this Court in this petition for review on certiorari of the Decision[1] of the Court of Appeals
affirming the decree of adoption issued by the Regional Trial Court of Cebu City, Branch 14, [2] in Special Proceedings No. 1744CEB, In the Matter of the Petition for Adoption of the minors Keith, Charmaine and Joseph Anthony, all surnamed Cang,
Spouses Ronald V. Clavano and Maria Clara Diago Clavano, petitioners.
Petitioner Herbert Cang and Anna Marie Clavano who were married on January 27, 1973, begot three children, namely:
Keith, born on July 3, 1973; Charmaine, born on January 23, 1977, and Joseph Anthony, born on January 3, 1981.
During the early years of their marriage, the Cang couples relationship was undisturbed. Not long thereafter, however, Anna
Marie learned of her husbands alleged extramarital affair with Wilma Soco, a family friend of the Clavanos.
Upon learning of her husbands alleged illicit liaison, Anna Marie filed a petition for legal separation with alimony pendente
lite with the then Juvenile and Domestic Relations Court of Cebu [4] which rendered a decision[5] approving the joint
manifestation of the Cang spouses providing that they agreed to live separately and apart or from bed and board. They further
agreed:
[3]

(c) That the children of the parties shall be entitled to a monthly support of ONE THOUSAND PESOS (P1,000.00) effective
from the date of the filing of the complaint. This shall constitute a first lien on the net proceeds of the house and lot
jointly owned by the parties situated at Cinco Village, Mandaue City;
(d) That the plaintiff shall be entitled to enter into any contract or agreement with any person or persons, natural or juridical
without the written consent of the husband; or any undertaking or acts that ordinarily requires husbands consent as the
parties are by this agreement legally separated; [6]
Petitioner then left for the United States where he sought a divorce from Anna Marie before the Second Judicial District
Court of the State of Nevada. Said court issued the divorce decree that also granted sole custody of the three minor children to
Anna Marie, reserving rights of visitation at all reasonable times and places to petitioner. [7]
Thereafter, petitioner took an American wife and thus became a naturalized American citizen. In 1986, he divorced his
American wife and never remarried.
While in the United States, petitioner worked in Tablante Medical Clinic earning P18,000.00 to P20,000.00 a month[8] a
portion of which was remitted to the Philippines for his childrens expenses and another, deposited in the bank in the name of
his children.
Meanwhile, on September 25, 1987, private respondents Ronald V. Clavano and Maria Clara Diago Clavano, respectively
the brother and sister-in-law of Anna Marie, filed Special Proceedings No. 1744-CEB for the adoption of the three minor Cang
children before the Regional Trial Court of Cebu. The petition bears the signature of then 14-year-old Keith signifying consent
to his adoption. Anna Marie likewise filed an affidavit of consent alleging that her husband had evaded his legal obligation to
support his children; that her brothers and sisters including Ronald V. Clavano, had been helping her in taking care of the
children; that because she would be going to the United States to attend to a family business, leaving the children would be a
problem and would naturally hamper (her) job-seeking venture abroad; and that her husband had long forfeited his parental
rights over the children for the following reasons:
1. The decision in Civil Case No. JD-707 allowed her to enter into any contract without the written consent of her husband;
2. Her husband had left the Philippines to be an illegal alien in the United States and had been transferring from one place to
another to avoid detection by Immigration authorities, and
3. Her husband had divorced her.
Upon learning of the petition for adoption, petitioner immediately returned to the Philippines and filed an opposition thereto,
alleging that, although private respondents Ronald and Maria Clara Clavano were financially capable of supporting the children
while his finances were too meager compared to theirs, he could not in conscience, allow anybody to strip him of his parental
authority over his beloved children.
Pending resolution of the petition for adoption, petitioner moved to reacquire custody over his children alleging that Anna
Marie had transferred to the United States thereby leaving custody of their children to private respondents. On January 11,
1988, the Regional Trial Court of Cebu City, Branch 19, issued an order finding that Anna Marie had, in effect, relinquished
custody over the children and, therefore, such custody should be transferred to the father. The court then directed the Clavanos
to deliver custody over the minors to petitioner.
On March 27, 1990, the Regional Trial Court of Cebu City, Branch 14, issued a decree of adoption with a dispositive portion
reading as follows:
WHEREFORE, premises considered, the petition for adoption of the minors Keith, Charmaine and Joseph Anthony all
surnamed Cang, by the petitioners-spouses Ronald V. Clavano and Maria Clara Diago Clavano is hereby granted and
approved. These children shall henceforth be known and called as Keith D. Clavano, Charmaine D. Clavano and Joseph
Anthony D. Clavano respectively. Moreover, this Decree of Adoption shall:
(1)

Confer upon the adopted children the same rights and duties as though they were in fact the legitimate children of
the petitioners;

(2)

Dissolve the authority vested in the parents by nature, of the children; and,

(3)

Vest the same authority in the petitioners.

Furnish the Local Civil Registrar of Cebu City, Philippines with a copy of this Decree of Adoption for registration purposes.
SO ORDERED.
In so ruling, the lower court was impelled by these reasons:
(1) The Cang children had, since birth, developed close filial ties with the Clavano family, especially their maternal uncle,
petitioner Ronald Clavano.
(2) Ronald and Maria Clara Clavano were childless and, with their printing press, real estate business, export business and
gasoline station and mini-mart in Rosemead, California, U.S.A., had substantial assets and income.

(3) The natural mother of the children, Anna Marie, nicknamed Menchu, approved of the adoption because of her heart
ailment, near-fatal accident in 1981, and the fact that she could not provide them a secure and happy future as she
travels a lot.
(4) The Clavanos could provide the children moral and spiritual direction as they would go to church together and had sent
the children to Catholic schools.
(5) The children themselves manifested their desire to be adopted by the Clavanos Keith had testified and expressed the
wish to be adopted by the Clavanos while the two younger ones were observed by the court to have snuggled close to
Ronald even though their natural mother was around.
On the other hand, the lower court considered the opposition of petitioner to rest on a very shaky foundation because of its
findings that:
(1) Petitioner was morally unfit to be the father of his children on account of his being an improvident father of his family
and an undisguised Lothario. This conclusion is based on the testimony of his alleged paramour, mother of his two
sons and close friend of Anna Marie, Wilma Soco, who said that she and petitioner lived as husband and wife in the very
house of the Cangs in Opao, Mandaue City.
(2) The alleged deposits of around $10,000 that were of comparatively recent dates were attempts at verisimilitude as
these were joint deposits the authenticity of which could not be verified.
(3) Contrary to petitioners claim, the possibility of his reconciliation with Anna Marie was dim if not nil because it was
petitioner who devised, engineered and executed the divorce proceedings at the Nevada Washoe County court.
(4) By his naturalization as a U.S. citizen, petitioner is now an alien from the standpoint of Philippine laws and therefore,
how his new attachments and loyalties would sit with his (Filipino) children is an open question.
Quoting with approval the evaluation and recommendation of the RTC Social Worker in her Child Study Report, the lower
court concluded as follows:
Simply put, the oppositor Herbert Cang has abandoned his children. And abandonment of a child by its (sic) parent is
commonly specified by statute as a ground for dispensing with his consent to its (sic) adoption (Re Cozza, 163 Cal. 514 P. 161,
Ann. [As. 1914A, 214]). Indeed, in such case, adoption will be allowed not only without the consent of the parent, but even
against his opposition (Re McKeag, 141 Cal. 403, 74 P. 1039, 99 Am. St. Rep. 80; Re Camp. 131 Cal. 469, 63 P. 736, 82 Am.
St. Rep. 371; Graham v. Francis, 83 Colo. 346, 265 P. 690, citing R.C.L.; Seibert, 170 Iowa, 561, 153 N.W. 160, citing R.C.L.;
Stearns v. Allen, 183 Mass. 404, 67 N.E. 349; 97 Am. St. Rep. 441; Wilson v. Otis, 71 N.H. 483, 53 A. 439, 93 Am. St. Rep.
564; Nugent v. Powell, 4 Wyo. 173, 33 P. 23, 20 L.R.A. 199, 62 Am. St. Rep. 17.) [9]
Before the Court of Appeals, petitioner contended that the lower court erred in holding that it would be in the best interest of
the three children if they were adopted by private respondents Ronald and Maria Clara Clavano. He asserted that the petition
for adoption was fatally defective and tailored to divest him of parental authority because: (a) he did not have a written consent
to the adoption; (b) he never abandoned his children; (c) Keith and Charmaine did not properly give their written consent; and
(d) the petitioners for adoption did not present as witness the representative of the Department of Social Welfare and
Development who made the case study report required by law.
The Court of Appeals affirmed the decree of adoption stating:
Article 188 of the Family Code requires the written consent of the natural parents of the child to be adopted. It has been held
however that the consent of the parent who has abandoned the child is not necessary (Dayrit vs. Piccio, 92 Phil. 729; Santos
vs. Ananzanso, 16 SCRA 344). The question therefore is whether or not oppositor may be considered as having abandoned
the children. In adoption cases, abandonment connotes any conduct on the part of the parent to forego parental duties and
relinquish parental claims to the child, or the neglect or refusal to perform the natural and legal obligations which parents owe
their children (Santos vs. Ananzanso, supra), or the withholding of the parents presence, his care and the opportunity to display
voluntary affection. The issue of abandonment is amply covered by the discussion of the first error.
Oppositor argues that he has been sending dollar remittances to the children and has in fact even maintained bank accounts in
their names. His duty to provide support comes from two judicial pronouncements. The first, the decision in JD-707 CEB,
supra, obliges him to pay the children P1,000.00 a month. The second is mandated by the divorce decree of the Nevada,
U.S.A. Federal Court which orders him to pay monthly support of US$50.00 for each child. Oppositor has not submitted any
evidence to show compliance with the decision in JD-101 CEB, but he has submitted 22 cancelled dollar checks (Exhs. 24 to
45) drawn in the childrens names totalling $2,126.98. The last remittance was on October 6, 1987 (Exh. 45). His obligation to
provide support commenced under the divorce decree on May 5, 1982 so that as of October 6, 1987, oppositor should have
made 53 remittances of $150.00, or a total of $7,950.00. No other remittances were shown to have been made after October 6,
1987, so that as of this date, oppositor was woefully in arrears under the terms of the divorce decree. And since he was totally in
default of the judgment in JD-707 CEB, the inevitable conclusion is oppositor had not really been performing his duties as a
father, contrary to his protestations.
True, it has been shown that oppositor had opened three accounts in different banks, as follows
Acct. No.

Date Opened
July 23, 1985
Oct. 29, 1987

Balance Name of Bank


$5,018.50 Great Western Savings, Daly
City, Cal., U.S.A.

March 5, 1986
Oct. 26, 1987

3,129.00

December 31,
1986
3) 564-146883 Oct. 29, 1987

2,622.19

1) 118606437-4
2) 73-166-8

Matewan National Bank of


Williamson, West Virginia,
U.S.A.
Security Pacific National
Bank, Daly City, Cal., U.S.A.

The first and third accounts were opened however in oppositors name as trustee for Charmaine Cang and Joseph Anthony
Cang, respectively. In other words, the accounts are operated and the amounts withdrawable by oppositor himself and it cannot
be said that they belong to the minors. The second is an `or account, in the names of Herbert Cang or Keith Cang. Since Keith
is a minor and in the Philippines, said account is operable only by oppositor and the funds withdrawable by him alone.
The bank accounts do not really serve what oppositor claimed in his offer of evidence `the aim and purpose of providing for a
better future and security of his family. [10]
Petitioner moved to reconsider the decision of the Court of Appeals. He emphasized that the decree of legal separation was
not based on the merits of the case as it was based on a manifestation amounting to a compromise agreement between him
and Anna Marie. That he and his wife agreed upon the plan for him to leave for the United States was borne out by the fact that
prior to his departure to the United States, the family lived with petitioners parents. Moreover, he alone did not instigate the
divorce proceedings as he and his wife initiated the joint complaint for divorce.
Petitioner argued that the finding that he was not fit to rear and care for his children was belied by the award to him of
custody over the children in Civil Case No. JD-707. He took exception to the appellate courts findings that as an American

citizen he could no longer lay claim to custody over his children because his citizenship would not take away the fact that he is
still a father to his children. As regards his alleged illicit relationship with another woman, he had always denied the same both
in Civil Case No. JD-707 and the instant adoption case. Neither was it true that Wilma Soco was a neighbor and family friend of
the Clavanos as she was residing in Mandaue City seven (7) kilometers away from the Clavanos who were residents of Cebu
City. Petitioner insisted that the testimony of Wilma Soco should not have been given weight for it was only during the hearing
of the petition for adoption that Jose Clavano, a brother of Ronald, came to know her and went to her residence in Iligan City to
convince her to be a witness for monetary considerations. Lastly, petitioner averred that it would be hypocritical of the Clavanos
to claim that they could love the children much more than he could. [11]
His motion for reconsideration having been denied, petitioner is now before this Court, alleging that the petition for adoption
was fatally defective as it did not have his written consent as a natural father as required by Article 31 (2) of Presidential Decree
No. 603, the Child and Youth Welfare Code, and Article 188 (2) of the Family Code.
Article 31 of P.D. No. 603 provides ART. 31. Whose Consent is Necessary. The written consent of the following to the adoption shall be necessary:
(1) The person to be adopted, if fourteen years of age or over;
(2) The natural parents of the child or his legal guardian of the Department of Social Welfare or any duly licensed child
placement agency under whose care the child may be;
(3) The natural children, fourteen years and above, of the adopting parents. (Underscoring supplied)
On December 17, 1986, then President Corazon C. Aquino issued Executive Order No. 91 amending Articles 27, 28, 29, 31,
33 and 35 of the Child and Youth Welfare Code. As thus amended, Article 31 read:
ART. 31. Whose Consent is Necessary. The written consent of the following to the adoption shall be necessary:
(1)

The person to be adopted, if fourteen years of age or over;

(2) The natural parents of the child or his legal guardian after receiving counselling and appropriate social services from
the Ministry of Social Services and Development or from a duly licensed child-placement agency;
(3) The Ministry of Social Services and Development or any duly licensed child-placement agency under whose care
and legal custody the child may be;
(4) The natural children, fourteen years and above, of the adopting parents. (Underscoring supplied)
Jurisdiction being a matter of substantive law, the established rule is that the statute in force at the time of the
commencement of the action determines the jurisdiction of the court. [12] As such, when private respondents filed the petition for
adoption on September 25, 1987, the applicable law was the Child and Youth Welfare Code, as amended by Executive Order
No. 91.
During the pendency of the petition for adoption or on August 3, 1988, the Family Code which amended the Child and Youth
Welfare Code took effect. Article 256 of the Family Code provides for its retroactivity insofar as it does not prejudice or impair
vested or acquired rights in accordance with the Civil Code or other laws. As amended by the Family Code, the statutory
provision on consent for adoption now reads:
Art. 188. The written consent of the following to the adoption shall be necessary:
(1) The person to be adopted, if ten years of age or over;
(2) The parents by nature of the child, the legal guardian, or the proper government instrumentality;
(3) The legitimate and adopted children, ten years of age or over, of the adopting parent or parents;
(4) The illegitimate children, ten years of age or over, of the adopting parents, if living with said parent and the latters
spouse, if any; and
(5) The spouse, if any, of the person adopting or to be adopted. (Underscoring supplied)
Based on the foregoing, it is thus evident that notwithstanding the amendments to the law, the written consent of the natural
parent to the adoption has remained a requisite for its validity. Notably, such requirement is also embodied in Rule 99 of the
Rules of Court as follows:
SEC. 3. Consent to adoption. There shall be filed with the petition a written consent to the adoption signed by the child, if
fourteen years of age or over and not incompetent, and by the childs spouse, if any, and by each of its known living parents
who is not insane or hopelessly intemperate or has not abandoned the child, or if there are no such parents by the general
guardian or guardian ad litem of the child, or if the child is in the custody of an orphan asylum, childrens home, or benevolent
society or person, by the proper officer or officers of such asylum, home, or society, or by such persons; but if the child is
illegitimate and has not been recognized, the consent of its father to the adoption shall not be required. (Underscoring supplied)
As clearly inferred from the foregoing provisions of law, the written consent of the natural parent is indispensable for the
validity of the decree of adoption. Nevertheless, the requirement of written consent can be dispensed with if the parent has
abandoned the child[13] or that such parent is insane or hopelessly intemperate. The court may acquire jurisdiction over the
case even without the written consent of the parents or one of the parents provided that the petition for adoption alleges facts
sufficient to warrant exemption from compliance therewith. This is in consonance with the liberality with which this Court treats
the procedural aspect of adoption. Thus, the Court declared:
x x x. The technical rules of pleading should not be stringently applied to adoption proceedings, and it is deemed more
important that the petition should contain facts relating to the child and its parents, which may give information to those
interested, than that it should be formally correct as a pleading. Accordingly, it is generally held that a petition will confer
jurisdiction if it substantially complies with the adoption statute, alleging all facts necessary to give the court jurisdiction. [14]
In the instant case, only the affidavit of consent of the natural mother was attached to the petition for adoption. Petitioners
consent, as the natural father is lacking. Nonetheless, the petition sufficiently alleged the fact of abandonment of the minors for
adoption by the natural father as follows:
3. That the childrens mother, sister of petitioner RONALD V. CLAVANO, has given her express consent to this adoption, as
shown by Affidavit of Consent, Annex `A. Likewise, the written consent of Keith Cang, now 14 years of age appears on page 2
of this petition; However, the father of the children, Herbert Cang, had already left his wife and children and had already
divorced the former, as evidenced by the xerox copy of the DECREE OF DIVORCE issued by the County of Washoe, State of
Nevada, U.S.A. (Annex `B) which was filed at the instance of Mr. Cang, not long after he abandoned his family to live in the
United States as an illegal immigrant.[15]

The allegations of abandonment in the petition for adoption, even absent the written consent of petitioner, sufficiently vested
the lower court with jurisdiction since abandonment of the child by his natural parents is one of the circumstances under which
our statutes and jurisprudence[16] dispense with the requirement of written consent to the adoption of their minor children.
However, in cases where the father opposes the adoption primarily because his consent thereto was not sought, the matter
of whether he had abandoned his child becomes a proper issue for determination. The issue of abandonment by the oppositor
natural parent is a preliminary issue that an adoption court must first confront. Only upon failure of the oppositor natural father
to prove to the satisfaction of the court that he did not abandon his child may the petition for adoption be considered on its
merits.
As a rule, factual findings of the lower courts are final and binding upon this Court. [17] This Court is not expected nor required
to examine or contrast the oral and documentary evidence submitted by the parties. [18] However, although this Court is not a trier
of facts, it has the authority to review and reverse the factual findings of the lower courts if it finds that these do not conform to
the evidence on record.[19]
In Reyes v. Court of Appeals,[20] this Court has held that the exceptions to the rule that factual findings of the trial court are
final and conclusive and may not be reviewed on appeal are the following: (1) when the inference made is manifestly mistaken,
absurd or impossible; (2) when there is a grave abuse of discretion; (3) when the finding is grounded entirely on speculations,
surmises or conjectures; (4) when the judgment of the Court of Appeals is based on misapprehension of facts; (5) when the
findings of fact are conflicting; (6) when the Court of Appeals, in making its findings, went beyond the issues of the case and the
same is contrary to the admissions of both appellant and appellee; (7) when the findings of the Court of Appeals are contrary to
those of the trial court; (8) when the findings of fact are conclusions without citation of specific evidence on which they are
based; (9) when the Court of Appeals manifestly overlooked certain relevant facts not disputed by the parties and which, if
properly considered, would justify a different conclusion and (10) when the findings of fact of the Court of Appeals are premised
on the absence of evidence and are contradicted by the evidence on record.
This Court finds that both the lower court and the Court of Appeals failed to appreciate facts and circumstances that should
have elicited a different conclusion [21] on the issue of whether petitioner has so abandoned his children, thereby making his
consent to the adoption unnecessary.
In its ordinary sense, the word abandon means to forsake entirely, to forsake or renounce utterly. The dictionaries trace
this word to the root idea of putting under a ban. The emphasis is on the finality and publicity with which a thing or body is
thus put in the control of another, hence, the meaning of giving up absolutely, with intent never to resume or claim ones rights or
interests.[22] In reference to abandonment of a child by his parent, the act of abandonment imports any conduct of the parent
which evinces a settled purpose to forego all parental duties and relinquish all parental claims to the child. It means neglect or
refusal to perform the natural and legal obligations of care and support which parents owe their children. [23]
In the instant case, records disclose that petitioners conduct did not manifest a settled purpose to forego all parental duties
and relinquish all parental claims over his children as to constitute abandonment. Physical estrangement alone,
without financial and moral desertion, is not tantamount to abandonment. [24] While admittedly, petitioner was physically absent
as he was then in the United States, he was not remiss in his natural and legal obligations of love, care and support for his
children. He maintained regular communication with his wife and children through letters and telephone. He used to send
packages by mail and catered to their whims.
Petitioners testimony on the matter is supported by documentary evidence consisting of the following handwritten letters to
him of both his wife and children:
1. Exh. 1 a 4-page undated letter of Menchu (Anna Marie) addressed to Dear Bert on a C.Westates Carbon Phil. Corp.
stationery. Menchu stated therein that it had been a long time since the last time youve heard from me excluding that of the
phone conversation weve had. She discussed petitioners intention to buy a motorbike for Keith, expressing apprehension
over risks that could be engendered by Keiths use of it. She said that in the last phone conversation she had with
petitioner on the birthday of Ma, she forgot to tell petitioner that Keiths voice had changed; he had become a bagito or a
teen-ager with many fans who sent him Valentines cards. She told him how Charmaine had become quite a talkative
almost dalaga who could carry on a conversation with herangkong and how pretty she was in white dress when she won
among the candidates in the Flores de Mayo after she had prayed so hard for it. She informed him, however, that she was
worried because Charmaine was vain and wont to extravagance as she loved clothes. About Joeton (Joseph Anthony), she
told petitioner that the boy was smart for his age and quite spoiled being the youngest of the children in Lahug. Joeton was
mischievous but Keith was his idol with whom he would sleep anytime. She admitted having said so much about the children
because they might not have informed petitioner of some happenings and spices of life about themselves. She said that it
was just very exciting to know how theyve grown up and very pleasant, too, that each of them have (sic) different
characters. She ended the letter with the hope that petitioner was at the best of health. After extending her regards to all,
she signed her name after the word Love. This letter was mailed on July 9, 1986 from Cebu to petitioner whose address
was P.O. Box 2445, Williamson, West Virginia 25661 (Exh. 1-D).
2. Exh. 2 letter dated 11/13/84 on a green stationery with golden print of a note from Menchu on the left upper corner.
Anna Marie stated that we wrote to petitioner on Oct. 2, 1984 and that Keith and Joeton were very excited when petitioner
called up last time. She told him how Joeton would grab the phone from Keith just so petitioner would know what he
wanted to order. Charmaine, who was asleep, was so disappointed that she missed petitioners call because she also
wanted something that petitioner should buy. Menchu told petitioner that Charmaine wanted a pencil sharpener, light-colored
T-shirts for her walking shorts and a (k)nap sack. Anna Marie informed petitioner that the kids were growing up and so were
their needs. She told petitioner to be very fatherly about the childrens needs because those were expensive here. For
herself, Anna Marie asked for a subscription of Glamour and Vogue magazines and that whatever expenses he would incur,
she would replace these. As a postscript, she told petitioner that Keith wanted a size 6 khaki-colored Sperry topsider
shoes.
3. Exh. 3 an undated note on a yellow small piece of paper that reads:
Dear Herbert,
Hi, how was Christmas and New Year? Hope you had a wonderful one.
By the way thanks for the shoes, it was a nice one. Its nice to be thought of at Xmas. Thanks again.
Sincerely,
Menchu
4. Exh. 4 a two-page undated letter of Keith on stationery of Jose Clavano, Inc. addressed to Dear Dad. Keith told his
father that they tried to tell their mother to stay for a little while, just a few weeks after classes start(s) on June 16. He
informed petitioner that Joeton would be in Kinder I and that, about the motorbike, he had told his mother to write petitioner
about it and well see what youre (sic) decision will be. He asked for chocolates, nuts, basketball shirt and shorts, rubber
shoes, socks, headband, some clothes for outing and perfume. He told petitioner that they had been going to Lahug with
their mother picking them up after Angkong or Ama had prepared lunch or dinner. From her aerobics, his mother would go
for them in Lahug at about 9:30 or 10:00 oclock in the evening. He wished his father luck and the best of health and that
they prayed for him and their other relatives. The letter was ended with Love Keith.
5. Exh. 5 another undated long letter of Keith. He thanked his father for the Christmas card with $40.00, $30.00 and
$30.00 and the card of Joeton with $5.00 inside. He told petitioner the amounts following his fathers instructions and
promise to send money through the mail. He asked his father to address his letter directly to him because he wanted to

open his own letters. He informed petitioner of activities during the Christmas season that they enjoyed eating, playing and
giving surprises to their mother. He apprised him of his daily schedule and that their mother had been closely supervising
them, instructing them to fold their blankets and pile up their pillows. He informed petitioner that Joeton had become very
smart while Charmaine, who was also smart, was very demanding of their mother. Because their mother was leaving for the
United States on February 5, they would be missing her like they were missing petitioner. He asked for his things and
$200.00. He told petitioner more anecdotes about Joeton like he would make the sign of the cross even when they would
pass by the Iglesia ni Cristo church and his insistence that Aquino was not dead because he had seen him on the betamax
machine. For Keith, Charmaine had become very maldita who was not always satisfied with her dolls and things but
Joeton was full of surprises. He ended the letter with Love your son, Keith. The letter was mailed on February 6, 1985
(Exh. 5-D).
6. Exh. 6 an undated letter Charmaine. She thanked petitioner for the bathing suit, key chain, pencil box, socks, half shirt,
pencil sharpener and $50.00. She reminded him of of her birthday on January 23 when she would turn 9 years old. She
informed him that she wore size 10 and the size of her feet was IM. They had fun at Christmas in Lahug but classes would
start on January 9 although Keiths classes had started on January 6. They would feel sad again because Mommy would be
leaving soon. She hoped petitioner would keep writing them. She signed, Love, Charmaine.
7. Exh . 7 an undated letter of Keith. He explained to petitioner that they had not been remiss in writing letters to him. He
informed him of their trip to Manila they went to Malacaang, Tito Doy Laurels house, the Ministry of Foreign Affairs, the
executive house, Tagaytay for three days and Baguio for one week. He informed him that he got honors, Charmaine was
7th in her class and Joeton had excellent grades. Joeton would be enrolled in Sacred Heart soon and he was glad they
would be together in that school. He asked for his reward from petitioner and so with Charmaine and Joeton. He asked for
a motorbike and dollars that he could save. He told petitioner that he was saving the money he had been sending them. He
said he missed petitioner and wished him the best. He added that petitioner should call them on Sundays.
8. Exh. 8 a letter from Joeton and Charmaine but apparently written by the latter. She asked for money from petitioner to
buy something for the school and something else. She promised not to spend so much and to save some. She said she
loved petitioner and missed him. Joeton said hi! to petitioner. After ending the letter with Love, Joeton and Charmaine,
she asked for her prize for her grades as she got seventh place.
9. Exh. 9 undated letter of Keith. He assured petitioner that he had been writing him; that he would like to have some
money but he would save them; that he learned that petitioner had called them up but he was not around; that he would be
going to Manila but would be back home May 3; that his Mommy had just arrived Thursday afternoon, and that he would be
the official altar boy. He asked petitioner to write them soon.
10.
Exh. 10 Keith thanked petitioner for the money he sent. He told petitioner that he was saving some in the bank
and he was proud because he was the only one in his group who saved in the bank. He told him that Joeton had become
naughty and would claim as his own the shirts sent to Keith by petitioner. He advised petitioner to send pants and shirts to
Joeton, too, and asked for a pair of topsider shoes and candies. He informed petitioner that he was a member of the
basketball team and that his mom would drive for his group. He asked him to call them often like the father of Ana Christie
and to write them when he would call so that they could wait for it. He informed petitioner that they had all grown bigger and
heavier. He hoped petitioner would be happy with the letter that had taken him so long to write because he did not want to
commit any mistakes. He asked petitioner to buy him perfume (Drakkar) and, after thanking petitioner, added that the latter
should buy something for Mommy.
11.

Exh. 11 a Christmas card For My Wonderful Father dated October 8, 1984 from Keith, Charmaine and Joeton.

12.
Exh. 12 another Christmas card, Our Wish For You with the year 83 written on the upper right hand corner of
the inside page, from Keith, Charmaine and Joeton.
13.
Exh. 13 a letter of Keith telling petitioner that he had written him even when their Mom was there where she
bought them clothes and shoes. Keith asked petitioner for $300.00. Because his mother would not agree to buy him a
motorbike, he wanted a Karaoke unit that would cost P12,000.00. He informed petitioner that he would go to an afternoon
disco with friends but their grades were all good with Joeton receiving stars for excellence. Keith wanted a bow and arrow
Rambo toys and G.I. Joe. He expressed his desire that petitioner would come and visit them someday.
14.
Exh. 14 a letter of Keith with one of the four pages bearing the date January 1986. Keith told his father that they
had received the package that the latter sent them. The clothes he sent, however, fitted only Keith but not Charmaine and
Joeton who had both grown bigger. Keith asked for grocery items, toys and more clothes. He asked, in behalf of his mother,
for low-heeled shoes and a dress to match, jogging pants, tights and leotards that would make her look sexy. He intimated
to petitioner that he had grown taller and that he was already ashamed to be asking for things to buy in the grocery even
though his mother had told him not to be shy about it.
Aside from these letters, petitioner also presented certifications of banks in the U.S.A. showing that even prior to the filing of
the petition for adoption, he had deposited amounts for the benefit of his children. [25] Exhibits 24 to 45 are copies of checks sent
by petitioner to the children from 1985 to 1989.
These pieces of evidence are all on record. It is, therefore, quite surprising why the courts below simply glossed over these,
ignoring not only evidence on financial support but also the emotional exchange of sentiments between petitioner and his
family. Instead, the courts below emphasized the meagerness of the amounts he sent to his children and the fact that, as
regards the bank deposits, these were withdrawable by him alone. Simply put, the courts below attached a high premium to
the prospective adopters financial status but totally brushed aside the possible repercussion of the adoption on the emotional
and psychological well-being of the children.
True, Keith had expressed his desire to be adopted by his uncle and aunt. However, his seeming steadfastness on the
matter as shown by his testimony is contradicted by his feelings towards his father as revealed in his letters to him. It is not at
all farfetched to conclude that Keiths testimony was actually the effect of the filing of the petition for adoption that would
certainly have engendered confusion in his young mind as to the capability of his father to sustain the lifestyle he had been used
to.
The courts below emphasized respondents emotional attachment to the children. This is hardly surprising for, from the very
start of their young lives, the children were used to their presence. Such attachment had persisted and certainly, the young
ones act of snuggling close to private respondent Ronald Clavano was not indicative of their emotional detachment from their
father. Private respondents, being the uncle and aunt of the children, could not but come to their succor when they needed help
as when Keith got sick and private respondent Ronald spent for his hospital bills.
In a number of cases, this Court has held that parental authority cannot be entrusted to a person simply because he could
give the child a larger measure of material comfort than his natural parent. Thus, in David v. Court of Appeals,[26] the Court
awarded custody of a minor illegitimate child to his mother who was a mere secretary and market vendor instead of to his
affluent father who was a married man, not solely because the child opted to go with his mother. The Court said:
Daisie and her children may not be enjoying a life of affluence that private respondent promises if the child lives with him. It is
enough, however, that petitioner is earning a decent living and is able to support her children according to her means.
In Celis v. Cafuir[27] where the Court was confronted with the issue of whether to award custody of a child to the natural
mother or to a foster mother, this Court said:

This court should avert the tragedy in the years to come of having deprived mother and son of the beautiful associations and
tender, imperishable memories engendered by the relationship of parent and child. We should not take away from a mother the
opportunity of bringing up her own child even at the cost of extreme sacrifice due to poverty and lack of means; so that
afterwards, she may be able to look back with pride and a sense of satisfaction at her sacrifices and her efforts, however
humble, to make her dreams of her little boy come true. We should not forget that the relationship between a foster mother and
a child is not natural but artificial. If the child turns out to be a failure or forgetful of what its foster parents had done for him, said
parents might yet count and appraise (sic) all that they have done and spent for him and with regret consider all of it as a dead
loss, and even rue the day they committed the blunder of taking the child into their hearts and their home. Not so with a real
natural mother who never counts the cost and her sacrifices, ever treasuring memories of her associations with her child,
however unpleasant and disappointing. Flesh and blood count. x x x.
In Espiritu v. Court of Appeals,[28] the Court stated that (I)n ascertaining the welfare and best interests of the child, courts are
mandated by the Family Code to take into account all relevant considerations. Thus, in awarding custody of the child to the
father, the Court said:
A scrutiny of the pleadings in this case indicates that Teresita, or at least, her counsel are more intent on emphasizing the
`torture and agony of a mother separated from her children and the humiliation she suffered as a result of her character being
made a key issue in court rather than the feelings and future, the best interests and welfare of her children. While the bonds
between a mother and her small child are special in nature, either parent, whether father or mother, is bound to suffer agony
and pain if deprived of custody. One cannot say that his or her suffering is greater than that of the other parent. It is not so much
the suffering, pride, and other feelings of either parent but the welfare of the child which is the paramount consideration. (Italics
supplied)[29]
Indeed, it would be against the spirit of the law if financial consideration were to be the paramount consideration in deciding
whether to deprive a person of parental authority over his children. There should be a holistic approach to the matter, taking
into account the physical, emotional, psychological, mental, social and spiritual needs of the child. [30] The conclusion of the
courts below that petitioner abandoned his family needs more evidentiary support other than his inability to provide them the
material comfort that his admittedly affluent in-laws could provide. There should be proof that he had so emotionally
abandoned them that his children would not miss his guidance and counsel if they were given to adopting parents. The letters
he received from his children prove that petitioner maintained the more important emotional tie between him and his
children. The children needed him not only because he could cater to their whims but also because he was a person they could
share with their daily activities, problems and triumphs.
The Court is thus dismayed that the courts below did not look beyond petitioners meager financial support to ferret out
other indications on whether petitioner had in fact abandoned his family. The omission of said courts has led us to examine why
the children were subjected to the process of adoption, notwithstanding the proven ties that bound them to their father. To our
consternation, the record of the case bears out the fact that the welfare of the children was not exactly the paramount
consideration that impelled Anna Marie to consent to their adoption.
In her affidavit of consent, Anna Marie expressly said that leaving the children in the country, as she was wont to travel
abroad often, was a problem that would naturally hamper her job-seeking abroad. In other words, the adoption appears to be a
matter of convenience for her because Anna Marie herself is financially capable of supporting her children. [31] In his testimony,
private respondent Ronald swore that Anna Marie had been out of the country for two years and came home twice or three
times,[32] thereby manifesting the fact that it was she who actually left her children to the care of her relatives. It was bad enough
that their father left their children when he went abroad, but when their mother followed suit for her own reasons, the situation
worsened. The Clavano family must have realized this. Hence, when the family first discussed the adoption of the children, they
decided that the prospective adopter should be Anna Maries brother Jose. However, because he had children of his own, the
family decided to devolve the task upon private respondents. [33]
This couple, however, could not always be in Cebu to care for the children. A businessman, private respondent Ronald
Clavano commutes between Cebu and Manila while his wife, private respondent Maria Clara, is an international flight
stewardess.[34] Moreover, private respondent Ronald claimed that he could take care of the children while their parents are
away,[35] thereby indicating the evanescence of his intention. He wanted to have the childrens surname changed to Clavano
for the reason that he wanted to take them to the United States as it would be difficult for them to get a visa if their surname
were different from his.[36] To be sure, he also testified that he wanted to spare the children the stigma of being products of a
broken home.
Nevertheless, a close analysis of the testimonies of private respondent Ronald, his sister Anna Marie and their brother Jose
points to the inescapable conclusion that they just wanted to keep the children away from their father. One of the overriding
considerations for the adoption was allegedly the state of Anna Maries health she was a victim of an almost fatal accident and
suffers from a heart ailment. However, she herself admitted that her health condition was not that serious as she could still take
care of the children.[37] An eloquent evidence of her ability to physically care for them was her employment at the Philippine
Consulate in Los Angeles[38]- she could not have been employed if her health were endangered. It is thus clear that the
Clavanos attempt at depriving petitioner of parental authority apparently stemmed from their notion that he was an inveterate
womanizer. Anna Marie in fact expressed fear that her children would never be at ease with the wife of their father. [39]
Petitioner, who described himself as single in status, denied being a womanizer and father to the sons of Wilma Soco. [40] As
to whether he was telling the truth is beside the point. Philippine society, being comparatively conservative and traditional,
aside from being Catholic in orientation, it does not countenance womanizing on the part of a family man, considering the
baneful effects such irresponsible act visits on his family. Neither may the Court place a premium on the inability of a man to
distinguish between siring children and parenting them. Nonetheless, the actuality that petitioner carried on an affair with a
paramour cannot be taken as sufficient basis for the conclusion that petitioner was necessarily an unfit father. [41] Conventional
wisdom and common human experience show that a bad husband does not necessarily make a bad father. That a husband
is not exactly an upright man is not, strictly speaking, a sufficient ground to deprive him as a father of his inherent right to
parental authority over the children. [42] Petitioner has demonstrated his love and concern for his children when he took the
trouble of sending a telegram[43] to the lower court expressing his intention to oppose the adoption immediately after learning
about it. He traveled back to this country to attend to the case and to testify about his love for his children and his desire to
unite his family once more in the United States. [44]
Private respondents themselves explained why petitioner failed to abide by the agreement with his wife on the support of the
children. Petitioner was an illegal alien in the United States. As such, he could not have procured gainful employment. Private
respondents failed to refute petitioners testimony that he did not receive his share from the sale of the conjugal home,
[45]
pursuant to their manifestation/compromise agreement in the legal separation case. Hence, it can be reasonably presumed
that the proceeds of the sale redounded to the benefit of his family, particularly his children. The proceeds may not have lasted
long but there is ample evidence to show that thereafter, petitioner tried to abide by his agreement with his wife and sent his
family money, no matter how meager.
The liberality with which this Court treats matters leading to adoption insofar as it carries out the beneficent purposes of the
law to ensure the rights and privileges of the adopted child arising therefrom, ever mindful that the paramount consideration is
the overall benefit and interest of the adopted child, should be understood in its proper context and perspective. The Courts
position should not be misconstrued or misinterpreted as to extend to inferences beyond the contemplation of law and
jurisprudence.[46] The discretion to approve adoption proceedings is not to be anchored solely on best interests of the child but
likewise, with due regard to the natural rights of the parents over the child. [47]
In this regard, this Court notes private respondents reliance on the manifestation/compromise agreement between petitioner
and Anna Marie which became the basis of the decree of legal separation. According to private respondents counsel, [48] the
authority given to Anna Marie by that decree to enter into contracts as a result of the legal separation was all embracing [49] and,
therefore, included giving her sole consent to the adoption. This conclusion is however, anchored on the wrong premise that the

authority given to the innocent spouse to enter into contracts that obviously refer to their conjugal properties, shall include
entering into agreements leading to the adoption of the children. Such conclusion is as devoid of a legal basis as private
respondents apparent reliance on the decree of legal separation for doing away with petitioners consent to the adoption.
The transfer of custody over the children to Anna Marie by virtue of the decree of legal separation did not, of necessity,
deprive petitioner of parental authority for the purpose of placing the children up for adoption. Article 213 of the Family Code
states: . . . in case of legal separation of parents, parental authority shall be exercised by the parent designated by the
court. In awarding custody, the court shall take into account all relevant considerations, especially the choice of the child over
seven years of age, unless the parent chosen is unfit.
It should be noted, however, that the law only confers on the innocent spouse the exercise of parental authority. Having
custody of the child, the innocent spouse shall implement the sum of parental rights with respect to his rearing and care. The
innocent spouse shall have the right to the childs services and earnings, and the right to direct his activities and make decisions
regarding his care and control, education, health and religion. [50]
In a number of cases, this Court has considered parental authority, the joint exercise of which is vested by the law upon the
parents,[51] as
x x x a mass of rights and obligations which the law grants to parents for the purpose of the childrens physical preservation
and development, as well as the cultivation of their intellect and the education of their hearts and senses. As regards parental
authority, `there is no power, but a task; no complex of rights, but a sum of duties; no sovereignty but a sacred trust for the
welfare of the minor.
Parental authority and responsibility are inalienable and may not be transferred or renounced except in cases authorized by
law. The right attached to parental authority, being purely personal, the law allows a waiver of parental authority only in cases of
adoption, guardianship and surrender to a childrens home or an orphan institution. When a parent entrusts the custody of a
minor to another, such as a friend or godfather, even in a document, what is given is merely temporary custody and it does not
constitute a renunciation of parental authority. Even if a definite renunciation is manifest, the law still disallows the same.
The father and mother, being the natural guardians of unemancipated children, are duty-bound and entitled to keep them in their
custody and company.[52] (Italics supplied)
As such, in instant case, petitioner may not be deemed as having been completely deprived of parental authority,
notwithstanding the award of custody to Anna Marie in the legal separation case. To reiterate, that award was arrived at by the
lower court on the basis of the agreement of the spouses.
While parental authority may be waived, as in law it may be subject to a compromise, [53] there was no factual finding in the
legal separation case that petitioner was such an irresponsible person that he should be deprived of custody of his children or
that there are grounds under the law that could deprive him of parental authority. In fact, in the legal separation case, the court
thereafter ordered the transfer of custody over the children from Anna Marie back to petitioner. The order was not implemented
because of Anna Maries motion for reconsideration thereon. The Clavano family also vehemently objected to the transfer of
custody to the petitioner, such that the latter was forced to file a contempt charge against them. [54]
The law is clear that either parent may lose parental authority over the child only for a valid reason. No such reason was
established in the legal separation case. In the instant case for adoption, the issue is whether or not petitioner had abandoned
his children as to warrant dispensation of his consent to their adoption. Deprivation of parental authority is one of the effects
of a decree of adoption.[55] But there cannot be a valid decree of adoption in this case precisely because, as this Court has
demonstrated earlier, the finding of the courts below on the issue of petitioners abandonment of his family was based on a
misappreciation that was tantamount to non-appreciation, of facts on record.
As regards the divorce obtained in the United States, this Court has ruled in Tenchavez v. Escao [56] that a divorce obtained
by Filipino citizens after the effectivity of the Civil Code is not recognized in this jurisdiction as it is contrary to State
policy. While petitioner is now an American citizen, as regards Anna Marie who has apparently remained a Filipino citizen, the
divorce has no legal effect.
Parental authority is a constitutionally protected State policy borne out of established customs and tradition of our
people. Thus, in Silva v. Court of Appeals, [57] a case involving the visitorial rights of an illegitimate parent over his child, the
Court expressed the opinion that:
Parents have the natural right, as well as the moral and legal duty, to care for their children, see to their upbringing and
safeguard their best interest and welfare. This authority and responsibility may not be unduly denied the parents; neither may it
be renounced by them. Even when the parents are estranged and their affection for each other is lost, the attachment and
feeling for their offsprings invariably remain unchanged. Neither the law nor the courts allow this affinity to suffer absent, of
course, any real, grave and imminent threat to the well-being of the child.
Since the incorporation of the law concerning adoption in the Civil Code, there has been a pronounced trend to place
emphasis in adoption proceedings, not so much on the need of childless couples for a child, as on the paramount interest of a
child who needs the love and care of parents. After the passage of the Child and Youth Welfare Code and the Family Code, the
discernible trend has impelled the enactment of Republic Act No. 8043 on Intercountry Adoption [58] and Republic Act No. 8552
establishing the rules on the domestic adoption of Filipino children. [59]
The case at bar applies the relevant provisions of these recent laws, such as the following policies in the Domestic Adoption
Act of 1998:
(a) To ensure that every child remains under the care and custody of his/her parent(s) and be provided with love, care,
understanding and security towards the full and harmonious development of his/her personality. [60]
(b) In all matters relating to the care, custody and adoption of a child, his/her interest shall be the paramount consideration
in accordance with the tenets set forth in the United Nations (UN) Convention on the Rights of the Child. [61]
(c) To prevent the child from unnecessary separation from his/her biological parent(s). [62]
Inasmuch as the Philippines is a signatory to the United Nations Convention on the Rights of the Child, the government and
its officials are duty bound to comply with its mandates. Of particular relevance to instant case are the following provisions:
States Parties shall respect the responsibilities, rights and duties of parents . . . to provide, in a manner consistent with the
evolving capacities of the child, appropriate direction and guidance in the exercise by the child of the rights recognized in the
present Convention.[63]
States Parties shall respect the right of the child who is separated from one or both parents to maintain personal relations and
direct contact with both parents on a regular basis, except if it is contrary to the childs best interests. [64]
A child whose parents reside in different States shall have the right to maintain on a regular basis, save in exceptional
circumstances personal relations and direct contacts with both parents . . . [65]
States Parties shall respect the rights and duties of the parents . . . to provide direction to the child in the exercise of his or her
right in a manner consistent with the evolving capacities of the child. [66]
Underlying the policies and precepts in international conventions and the domestic statutes with respect to children is the
overriding principle that all actuations should be in the best interests of the child. This is not, however, to be implemented in

derogation of the primary right of the parent or parents to exercise parental authority over him. The rights of parents vis--vis
that of their children are not antithetical to each other, as in fact, they must be respected and harmonized to the fullest extent
possible.
Keith, Charmaine and Joseph Anthony have all grown up. Keith and Charmaine are now of legal age while Joseph Anthony
is approaching eighteen, the age of majority. For sure, they shall be endowed with the discretion to lead lives independent of
their parents. This is not to state that this case has been rendered moot and academic, for their welfare and best interests
regarding their adoption, must be determined as of the time that the petition for adoption was filed. [67] Said petition must be
denied as it was filed without the required consent of their father who, by law and under the facts of the case at bar, has not
abandoned them.
WHEREFORE, the instant petition for review on certiorari is hereby GRANTED. The questioned Decision and Resolution of
the Court of Appeals, as well as the decision of the Regional Trial Court of Cebu, are SET ASIDE thereby denying the petition
for adoption of Keith, Charmaine and Joseph Anthony, all surnamed Cang, by the spouse respondents Ronald and Maria Clara
Clavano. This Decision is immediately executory.
SO ORDERED.

G.R. No. L-19671

November 29, 1965

PASTOR B. TENCHAVEZ, plaintiff-appellant,


vs.
VICENTA F. ESCAO, ET AL., defendants-appellees.
I. V. Binamira & F. B. Barria for plaintiff-appellant.
Jalandoni & Jarnir for defendants-appellees.
REYES, J.B.L., J.:
Direct appeal, on factual and legal questions, from the judgment of the Court of First Instance of Cebu, in its Civil Case No. R4177, denying the claim of the plaintiff-appellant, Pastor B. Tenchavez, for legal separation and one million pesos in damages
against his wife and parents-in-law, the defendants-appellees, Vicente, Mamerto and Mena, 1 all surnamed "Escao,"
respectively.2
The facts, supported by the evidence of record, are the following:
Missing her late afternoon classes on 24 February 1948 in the University of San Carlos, Cebu City, where she was then enrolled
as a second year student of commerce, Vicenta Escao, 27 years of age (scion of a well-to-do and socially prominent Filipino
family of Spanish ancestry and a "sheltered colegiala"), exchanged marriage vows with Pastor Tenchavez, 32 years of age, an
engineer, ex-army officer and of undistinguished stock, without the knowledge of her parents, before a Catholic chaplain, Lt.
Moises Lavares, in the house of one Juan Alburo in the said city. The marriage was the culmination of a previous love affair and
was duly registered with the local civil register.
Vicenta's letters to Pastor, and his to her, before the marriage, indicate that the couple were deeply in love. Together with a
friend, Pacita Noel, their matchmaker and go-between, they had planned out their marital future whereby Pacita would be the
governess of their first-born; they started saving money in a piggy bank. A few weeks before their secret marriage, their
engagement was broken; Vicenta returned the engagement ring and accepted another suitor, Joseling Lao. Her love for Pastor
beckoned; she pleaded for his return, and they reconciled. This time they planned to get married and then elope. To facilitate the
elopement, Vicenta had brought some of her clothes to the room of Pacita Noel in St. Mary's Hall, which was their usual trysting
place.
Although planned for the midnight following their marriage, the elopement did not, however, materialize because when Vicente
went back to her classes after the marriage, her mother, who got wind of the intended nuptials, was already waiting for her at
the college. Vicenta was taken home where she admitted that she had already married Pastor. Mamerto and Mena Escao
were surprised, because Pastor never asked for the hand of Vicente, and were disgusted because of the great scandal that the
clandestine marriage would provoke (t.s.n., vol. III, pp. 1105-06). The following morning, the Escao spouses sought priestly
advice. Father Reynes suggested a recelebration to validate what he believed to be an invalid marriage, from the standpoint of
the Church, due to the lack of authority from the Archbishop or the parish priest for the officiating chaplain to celebrate the
marriage. The recelebration did not take place, because on 26 February 1948 Mamerto Escao was handed by a maid, whose
name he claims he does not remember, a letter purportedly coming from San Carlos college students and disclosing an
amorous relationship between Pastor Tenchavez and Pacita Noel; Vicenta translated the letter to her father, and thereafter
would not agree to a new marriage. Vicenta and Pastor met that day in the house of Mrs. Pilar Mendezona. Thereafter, Vicenta
continued living with her parents while Pastor returned to his job in Manila. Her letter of 22 March 1948 (Exh. "M"), while still
solicitous of her husband's welfare, was not as endearing as her previous letters when their love was aflame.
Vicenta was bred in Catholic ways but is of a changeable disposition, and Pastor knew it. She fondly accepted her being called
a "jellyfish." She was not prevented by her parents from communicating with Pastor (Exh. "1-Escao"), but her letters became
less frequent as the days passed. As of June, 1948 the newlyweds were already estranged (Exh. "2-Escao"). Vicenta had
gone to Jimenez, Misamis Occidental, to escape from the scandal that her marriage stirred in Cebu society. There, a lawyer
filed for her a petition, drafted by then Senator Emmanuel Pelaez, to annul her marriage. She did not sign the petition (Exh. "B5"). The case was dismissed without prejudice because of her non-appearance at the hearing (Exh. "B-4").
On 24 June 1950, without informing her husband, she applied for a passport, indicating in her application that she was single,
that her purpose was to study, and she was domiciled in Cebu City, and that she intended to return after two years. The
application was approved, and she left for the United States. On 22 August 1950, she filed a verified complaint for divorce
against the herein plaintiff in the Second Judicial District Court of the State of Nevada in and for the County of Washoe, on the
ground of "extreme cruelty, entirely mental in character." On 21 October 1950, a decree of divorce, "final and absolute", was
issued in open court by the said tribunal.
In 1951 Mamerto and Mena Escao filed a petition with the Archbishop of Cebu to annul their daughter's marriage to Pastor
(Exh. "D"). On 10 September 1954, Vicenta sought papal dispensation of her marriage (Exh. "D"-2).
On 13 September 1954, Vicenta married an American, Russell Leo Moran, in Nevada. She now lives with him in California, and,
by him, has begotten children. She acquired American citizenship on 8 August 1958.
But on 30 July 1955, Tenchavez had initiated the proceedings at bar by a complaint in the Court of First Instance of Cebu, and
amended on 31 May 1956, against Vicenta F. Escao, her parents, Mamerto and Mena Escao, whom he charged with having
dissuaded and discouraged Vicenta from joining her husband, and alienating her affections, and against the Roman Catholic
Church, for having, through its Diocesan Tribunal, decreed the annulment of the marriage, and asked for legal separation and
one million pesos in damages. Vicenta claimed a valid divorce from plaintiff and an equally valid marriage to her present
husband, Russell Leo Moran; while her parents denied that they had in any way influenced their daughter's acts, and
counterclaimed for moral damages.
The appealed judgment did not decree a legal separation, but freed the plaintiff from supporting his wife and to acquire property
to the exclusion of his wife. It allowed the counterclaim of Mamerto Escao and Mena Escao for moral and exemplary
damages and attorney's fees against the plaintiff-appellant, to the extent of P45,000.00, and plaintiff resorted directly to this
Court.
The appellant ascribes, as errors of the trial court, the following:
1. In not declaring legal separation; in not holding defendant Vicenta F. Escao liable for damages and in dismissing the
complaint;.
2. In not holding the defendant parents Mamerto Escano and the heirs of Doa Mena Escao liable for damages;.
3 In holding the plaintiff liable for and requiring him to pay the damages to the defendant parents on their counterclaims;
and.
4. In dismissing the complaint and in denying the relief sought by the plaintiff.
That on 24 February 1948 the plaintiff-appellant, Pastor Tenchavez, and the defendant-appellee, Vicenta Escao, were validly
married to each other, from the standpoint of our civil law, is clearly established by the record before us. Both parties were then
above the age of majority, and otherwise qualified; and both consented to the marriage, which was performed by a Catholic
priest (army chaplain Lavares) in the presence of competent witnesses. It is nowhere shown that said priest was not duly
authorized under civil law to solemnize marriages.

The chaplain's alleged lack of ecclesiastical authorization from the parish priest and the Ordinary, as required by Canon law, is
irrelevant in our civil law, not only because of the separation of Church and State but also because Act 3613 of the Philippine
Legislature (which was the marriage law in force at the time) expressly provided that
SEC. 1. Essential requisites. Essential requisites for marriage are the legal capacity of the contracting parties and
consent. (Emphasis supplied)
The actual authority of the solemnizing officer was thus only a formal requirement, and, therefore, not essential to give the
marriage civil effects,3 and this is emphasized by section 27 of said marriage act, which provided the following:
SEC. 27. Failure to comply with formal requirements. No marriage shall be declared invalid because of the absence of
one or several of the formal requirements of this Act if, when it was performed, the spouses or one of them believed in
good faith that the person who solemnized the marriage was actually empowered to do so, and that the marriage was
perfectly legal.
The good faith of all the parties to the marriage (and hence the validity of their marriage) will be presumed until the contrary is
positively proved (Lao vs. Dee Tim, 45 Phil. 739, 745; Francisco vs. Jason, 60 Phil. 442, 448). It is well to note here that in the
case at bar, doubts as to the authority of the solemnizing priest arose only after the marriage, when Vicenta's parents consulted
Father Reynes and the archbishop of Cebu. Moreover, the very act of Vicenta in abandoning her original action for annulment
and subsequently suing for divorce implies an admission that her marriage to plaintiff was valid and binding.
Defendant Vicenta Escao argues that when she contracted the marriage she was under the undue influence of Pacita Noel,
whom she charges to have been in conspiracy with appellant Tenchavez. Even granting, for argument's sake, the truth of that
contention, and assuming that Vicenta's consent was vitiated by fraud and undue influence, such vices did not render her
marriage ab initio void, but merely voidable, and the marriage remained valid until annulled by a competent civil court. This was
never done, and admittedly, Vicenta's suit for annulment in the Court of First Instance of Misamis was dismissed for nonprosecution.
It is equally clear from the record that the valid marriage between Pastor Tenchavez and Vicenta Escao remained subsisting
and undissolved under Philippine law, notwithstanding the decree of absolute divorce that the wife sought and obtained on 21
October 1950 from the Second Judicial District Court of Washoe County, State of Nevada, on grounds of "extreme cruelty,
entirely mental in character." At the time the divorce decree was issued, Vicenta Escao, like her husband, was still a Filipino
citizen.4 She was then subject to Philippine law, and Article 15 of the Civil Code of the Philippines (Rep. Act No. 386), already in
force at the time, expressly provided:
Laws relating to family rights and duties or to the status, condition and legal capacity of persons are binding upon the
citizens of the Philippines, even though living abroad.
The Civil Code of the Philippines, now in force, does not admit absolute divorce, quo ad vinculo matrimonii; and in fact does not
even use that term, to further emphasize its restrictive policy on the matter, in contrast to the preceding legislation that admitted
absolute divorce on grounds of adultery of the wife or concubinage of the husband (Act 2710). Instead of divorce, the present
Civil Code only provides for legal separation (Title IV, Book 1, Arts. 97 to 108), and, even in that case, it expressly prescribes
that "the marriage bonds shall not be severed" (Art. 106, subpar. 1).
For the Philippine courts to recognize and give recognition or effect to a foreign decree of absolute divorce betiveen Filipino
citizens could be a patent violation of the declared public policy of the state, specially in view of the third paragraph of Article 17
of the Civil Code that prescribes the following:
Prohibitive laws concerning persons, their acts or property, and those which have for their object public order, policy and
good customs, shall not be rendered ineffective by laws or judgments promulgated, or by determinations or conventions
agreed upon in a foreign country.
Even more, the grant of effectivity in this jurisdiction to such foreign divorce decrees would, in effect, give rise to an irritating and
scandalous discrimination in favor of wealthy citizens, to the detriment of those members of our polity whose means do not
permit them to sojourn abroad and obtain absolute divorces outside the Philippines.
From this point of view, it is irrelevant that appellant Pastor Tenchavez should have appeared in the Nevada divorce court.
Primarily because the policy of our law cannot be nullified by acts of private parties (Civil Code,Art. 17, jam quot.); and
additionally, because the mere appearance of a non-resident consort cannot confer jurisdiction where the court originally had
none (Area vs. Javier, 95 Phil. 579).
From the preceding facts and considerations, there flows as a necessary consequence that in this jurisdiction Vicenta Escao's
divorce and second marriage are not entitled to recognition as valid; for her previous union to plaintiff Tenchavez must be
declared to be existent and undissolved. It follows, likewise, that her refusal to perform her wifely duties, and her denial
of consortium and her desertion of her husband constitute in law a wrong caused through her fault, for which the husband is
entitled to the corresponding indemnity (Civil Code, Art. 2176). Neither an unsubstantiated charge of deceit nor an anonymous
letter charging immorality against the husband constitute, contrary to her claim, adequate excuse. Wherefore, her marriage and
cohabitation with Russell Leo Moran is technically "intercourse with a person not her husband" from the standpoint of Philippine
Law, and entitles plaintiff-appellant Tenchavez to a decree of "legal separation under our law, on the basis of adultery" (Revised
Penal Code, Art. 333).
The foregoing conclusions as to the untoward effect of a marriage after an invalid divorce are in accord with the previous
doctrines and rulings of this court on the subject, particularly those that were rendered under our laws prior to the approval of
the absolute divorce act (Act 2710 of the Philippine Legislature). As a matter of legal history, our statutes did not recognize
divorces a vinculo before 1917, when Act 2710 became effective; and the present Civil Code of the Philippines, in disregarding
absolute divorces, in effect merely reverted to the policies on the subject prevailing before Act 2710. The rulings, therefore,
under the Civil Code of 1889, prior to the Act above-mentioned, are now, fully applicable. Of these, the decision in Ramirez vs.
Gmur, 42 Phil. 855, is of particular interest. Said this Court in that case:
As the divorce granted by the French Court must be ignored, it results that the marriage of Dr. Mory and Leona Castro,
celebrated in London in 1905, could not legalize their relations; and the circumstance that they afterwards passed for
husband and wife in Switzerland until her death is wholly without legal significance. The claims of the very children to
participate in the estate of Samuel Bishop must therefore be rejected. The right to inherit is limited to legitimate,
legitimated and acknowledged natural children. The children of adulterous relations are wholly excluded. The word
"descendants" as used in Article 941 of the Civil Code cannot be interpreted to include illegitimates born
of adulterous relations. (Emphasis supplied)
Except for the fact that the successional rights of the children, begotten from Vicenta's marriage to Leo Moran after the invalid
divorce, are not involved in the case at bar, the Gmur case is authority for the proposition that such union is adulterous in this
jurisdiction, and, therefore, justifies an action for legal separation on the part of the innocent consort of the first marriage, that
stands undissolved in Philippine law. In not so declaring, the trial court committed error.
True it is that our ruling gives rise to anomalous situations where the status of a person (whether divorced or not) would depend
on the territory where the question arises. Anomalies of this kind are not new in the Philippines, and the answer to them was
given in Barretto vs. Gonzales, 58 Phil. 667:

The hardship of the existing divorce laws in the Philippine Islands are well known to the members of the Legislature. It is
the duty of the Courts to enforce the laws of divorce as written by Legislature if they are constitutional. Courts have no
right to say that such laws are too strict or too liberal. (p. 72)
The appellant's first assignment of error is, therefore, sustained.
However, the plaintiff-appellant's charge that his wife's parents, Dr. Mamerto Escao and his wife, the late Doa Mena Escao,
alienated the affections of their daughter and influenced her conduct toward her husband are not supported by credible
evidence. The testimony of Pastor Tenchavez about the Escao's animosity toward him strikes us to be merely conjecture and
exaggeration, and are belied by Pastor's own letters written before this suit was begun (Exh. "2-Escao" and "Vicenta," Rec. on
App., pp. 270-274). In these letters he expressly apologized to the defendants for "misjudging them" and for the "great
unhappiness" caused by his "impulsive blunders" and "sinful pride," "effrontery and audacity" [sic]. Plaintiff was admitted to the
Escao house to visit and court Vicenta, and the record shows nothing to prove that he would not have been accepted to marry
Vicente had he openly asked for her hand, as good manners and breeding demanded. Even after learning of the clandestine
marriage, and despite their shock at such unexpected event, the parents of Vicenta proposed and arranged that the marriage
be recelebrated in strict conformity with the canons of their religion upon advice that the previous one was canonically defective.
If no recelebration of the marriage ceremony was had it was not due to defendants Mamerto Escao and his wife, but to the
refusal of Vicenta to proceed with it. That the spouses Escao did not seek to compel or induce their daughter to assent to the
recelebration but respected her decision, or that they abided by her resolve, does not constitute in law an alienation of
affections. Neither does the fact that Vicenta's parents sent her money while she was in the United States; for it was natural that
they should not wish their daughter to live in penury even if they did not concur in her decision to divorce Tenchavez (27 Am.
Jur. 130-132).
There is no evidence that the parents of Vicenta, out of improper motives, aided and abetted her original suit for annulment, or
her subsequent divorce; she appears to have acted independently, and being of age, she was entitled to judge what was best
for her and ask that her decisions be respected. Her parents, in so doing, certainly cannot be charged with alienation of
affections in the absence of malice or unworthy motives, which have not been shown, good faith being always presumed until
the contrary is proved.
SEC. 529. Liability of Parents, Guardians or Kin. The law distinguishes between the right of a parent to interest himself
in the marital affairs of his child and the absence of rights in a stranger to intermeddle in such affairs. However, such
distinction between the liability of parents and that of strangers is only in regard to what will justify interference. A parent
isliable for alienation of affections resulting from his own malicious conduct, as where he wrongfully entices his son or
daughter to leave his or her spouse, but he is not liable unless he acts maliciously, without justification and from unworthy
motives. He is not liable where he acts and advises his child in good faith with respect to his child's marital relations in the
interest of his child as he sees it, the marriage of his child not terminating his right and liberty to interest himself in, and be
extremely solicitous for, his child's welfare and happiness, even where his conduct and advice suggest or result in the
separation of the spouses or the obtaining of a divorce or annulment, or where he acts under mistake or misinformation,
or where his advice or interference are indiscreet or unfortunate, although it has been held that the parent is liable for
consequences resulting from recklessness. He may in good faith take his child into his home and afford him or her
protection and support, so long as he has not maliciously enticed his child away, or does not maliciously entice or cause
him or her to stay away, from his or her spouse. This rule has more frequently been applied in the case of advice given to
a married daughter, but it is equally applicable in the case of advice given to a son.
Plaintiff Tenchavez, in falsely charging Vicenta's aged parents with racial or social discrimination and with having exerted efforts
and pressured her to seek annulment and divorce, unquestionably caused them unrest and anxiety, entitling them to recover
damages. While this suit may not have been impelled by actual malice, the charges were certainly reckless in the face of the
proven facts and circumstances. Court actions are not established for parties to give vent to their prejudices or spleen.
In the assessment of the moral damages recoverable by appellant Pastor Tenchavez from defendant Vicente Escao, it is
proper to take into account, against his patently unreasonable claim for a million pesos in damages, that (a) the marriage was
celebrated in secret, and its failure was not characterized by publicity or undue humiliation on appellant's part; (b) that the
parties never lived together; and (c) that there is evidence that appellant had originally agreed to the annulment of the marriage,
although such a promise was legally invalid, being against public policy (cf. Art. 88, Civ. Code). While appellant is unable to
remarry under our law, this fact is a consequence of the indissoluble character of the union that appellant entered into
voluntarily and with open eyes rather than of her divorce and her second marriage. All told, we are of the opinion that appellant
should recover P25,000 only by way of moral damages and attorney's fees.
With regard to the P45,000 damages awarded to the defendants, Dr. Mamerto Escao and Mena Escao, by the court below,
we opine that the same are excessive. While the filing of this unfounded suit must have wounded said defendants' feelings and
caused them anxiety, the same could in no way have seriously injured their reputation, or otherwise prejudiced them, lawsuits
having become a common occurrence in present society. What is important, and has been correctly established in the decision
of the court below, is that said defendants were not guilty of any improper conduct in the whole deplorable affair. This Court,
therefore, reduces the damages awarded to P5,000 only.
Summing up, the Court rules:
(1) That a foreign divorce between Filipino citizens, sought and decreed after the effectivity of the present Civil Code (Rep. Act
386), is not entitled to recognition as valid in this jurisdiction; and neither is the marriage contracted with another party by the
divorced consort, subsequently to the foreign decree of divorce, entitled to validity in the country;
(2) That the remarriage of divorced wife and her co-habitation with a person other than the lawful husband entitle the latter to a
decree of legal separation conformably to Philippine law;
(3) That the desertion and securing of an invalid divorce decree by one consort entitles the other to recover damages;
(4) That an action for alienation of affections against the parents of one consort does not lie in the absence of proof of malice or
unworthy motives on their part.
WHEREFORE, the decision under appeal is hereby modified as follows;
(1) Adjudging plaintiff-appellant Pastor Tenchavez entitled to a decree of legal separation from defendant Vicenta F. Escao;
(2) Sentencing defendant-appellee Vicenta Escao to pay plaintiff-appellant Tenchavez the amount of P25,000 for damages and
attorneys' fees;
(3) Sentencing appellant Pastor Tenchavez to pay the appellee, Mamerto Escao and the estate of his wife, the deceased Mena
Escao, P5,000 by way of damages and attorneys' fees.
Neither party to recover costs.
REPUBLIC OF THE PHILIPPINES,
Petitioner,

G.R. No. 154380


Present:

- versus -

Davide, Jr., C.J.,


(Chairman),
Quisumbing,
Ynares-Santiago,
Carpio, and
Azcuna, JJ.

CIPRIANO ORBECIDO III,


Respondent.

Promulgated:
October 5, 2005

x --------------------------------------------------x
DECISION
QUISUMBING, J.:

Given a valid marriage between two Filipino citizens, where one party is later naturalized as a foreign citizen and obtains a
valid divorce decree capacitating him or her to remarry, can the Filipino spouse likewise remarry under Philippine law?
Before us is a case of first impression that behooves the Court to make a definite ruling on this apparently novel question,
presented as a pure question of law.
In this petition for review, the Solicitor General assails the Decision[1] dated May 15, 2002, of the Regional Trial Court of
Molave, Zamboanga del Sur, Branch 23 and its Resolution[2] dated July 4, 2002 denying the motion for reconsideration. The
courta quo had declared that herein respondent Cipriano Orbecido III is capacitated to remarry. The fallo of the impugned
Decision reads:
WHEREFORE, by virtue of the provision of the second paragraph of Art. 26 of the Family Code and by
reason of the divorce decree obtained against him by his American wife, the petitioner is given the capacity to
remarry under the Philippine Law.
IT IS SO ORDERED.[3]

The factual antecedents, as narrated by the trial court, are as follows.


On May 24, 1981, Cipriano Orbecido III married Lady Myros M. Villanueva at the United Church of Christ in the
Philippines in Lam-an, Ozamis City. Their marriage was blessed with a son and a daughter, Kristoffer Simbortriz V. Orbecido
and Lady Kimberly V. Orbecido.
In 1986, Ciprianos wife left for the United States bringing along their son Kristoffer. A few years later, Cipriano discovered
that his wife had been naturalized as an American citizen.
Sometime in 2000, Cipriano learned from his son that his wife had obtained a divorce decree and then married a certain
Innocent Stanley. She, Stanley and her child by him currently live at 5566 A. Walnut Grove Avenue, San Gabriel, California.
Cipriano thereafter filed with the trial court a petition for authority to remarry invoking Paragraph 2 of Article 26 of the
Family Code. No opposition was filed. Finding merit in the petition, the court granted the same. The Republic, herein petitioner,
through the Office of the Solicitor General (OSG), sought reconsideration but it was denied.
In this petition, the OSG raises a pure question of law:
WHETHER OR NOT RESPONDENT CAN REMARRY UNDER ARTICLE 26 OF THE FAMILY CODE[4]

The OSG contends that Paragraph 2 of Article 26 of the Family Code is not applicable to the instant case because it only
applies to a valid mixed marriage; that is, a marriage celebrated between a Filipino citizen and an alien. The proper remedy,
according to the OSG, is to file a petition for annulment or for legal separation. [5] Furthermore, the OSG argues there is no law
that governs respondents situation. The OSG posits that this is a matter of legislation and not of judicial determination. [6]

For his part, respondent admits that Article 26 is not directly applicable to his case but insists that when his naturalized
alien wife obtained a divorce decree which capacitated her to remarry, he is likewise capacitated by operation of law pursuant to
Section 12, Article II of the Constitution.[7]
At the outset, we note that the petition for authority to remarry filed before the trial court actually constituted a petition for
declaratory relief. In this connection, Section 1, Rule 63 of the Rules of Court provides:
RULE 63
DECLARATORY RELIEF AND SIMILAR REMEDIES
Section 1. Who may file petitionAny person interested under a deed, will, contract or other written
instrument, or whose rights are affected by a statute, executive order or regulation, ordinance, or other
governmental regulation may, before breach or violation thereof, bring an action in the appropriate Regional Trial
Court to determine any question of construction or validity arising, and for a declaration of his rights or duties,
thereunder.
...

The requisites of a petition for declaratory relief are: (1) there must be a justiciable controversy; (2) the controversy must
be between persons whose interests are adverse; (3) that the party seeking the relief has a legal interest in the controversy; and
(4) that the issue is ripe for judicial determination. [8]
This case concerns the applicability of Paragraph 2 of Article 26 to a marriage between two Filipino citizens where one
later acquired alien citizenship, obtained a divorce decree, and remarried while in the U.S.A. The interests of the parties are
also adverse, as petitioner representing the State asserts its duty to protect the institution of marriage while respondent, a
private citizen, insists on a declaration of his capacity to remarry. Respondent, praying for relief, has legal interest in the
controversy. The issue raised is also ripe for judicial determination inasmuch as when respondent remarries, litigation ensues
and puts into question the validity of his second marriage.
Coming now to the substantive issue, does Paragraph 2 of Article 26 of the Family Code apply to the case of
respondent? Necessarily, we must dwell on how this provision had come about in the first place, and what was the intent of the
legislators in its enactment?

Brief Historical Background


On July 6, 1987, then President Corazon Aquino signed into law Executive Order No. 209, otherwise known as the
Family Code, which took effect on August 3, 1988. Article 26 thereof states:
All marriages solemnized outside the Philippines in accordance with the laws in force in the country where
they were solemnized, and valid there as such, shall also be valid in this country, except those prohibited under
Articles 35, 37, and 38.

On July 17, 1987, shortly after the signing of the original Family Code, Executive Order No. 227 was likewise signed into
law, amending Articles 26, 36, and 39 of the Family Code. A second paragraph was added to Article 26. As so amended, it now
provides:
ART. 26. All marriages solemnized outside the Philippines in accordance with the laws in force in the country
where they were solemnized, and valid there as such, shall also be valid in this country, except those prohibited
under Articles 35(1), (4), (5) and (6), 36, 37 and 38.
Where a marriage between a Filipino citizen and a foreigner is validly celebrated and a divorce is thereafter
validly obtained abroad by the alien spouse capacitating him or her to remarry, the Filipino spouse shall have
capacity to remarry under Philippine law. (Emphasis supplied)

On its face, the foregoing provision does not appear to govern the situation presented by the case at hand. It seems to
apply only to cases where at the time of the celebration of the marriage, the parties are a Filipino citizen and a foreigner. The
instant case is one where at the time the marriage was solemnized, the parties were two Filipino citizens, but later on, the wife
was naturalized as an American citizen and subsequently obtained a divorce granting her capacity to remarry, and indeed she
remarried an American citizen while residing in the U.S.A.

Noteworthy, in the Report of the Public Hearings [9] on the Family Code, the Catholic Bishops Conference of the
Philippines (CBCP) registered the following objections to Paragraph 2 of Article 26:
1.
The rule is discriminatory. It discriminates against those whose spouses are Filipinos who divorce them
abroad. These spouses who are divorced will not be able to re-marry, while the spouses of foreigners who
validly divorce them abroad can.
2.

This is the beginning of the recognition of the validity of divorce even for Filipino citizens. For those
whose foreign spouses validly divorce them abroad will also be considered to be validly divorced here and can
re-marry. We propose that this be deleted and made into law only after more widespread consultation.
(Emphasis supplied.)

Legislative Intent
Records of the proceedings of the Family Code deliberations showed that the intent of Paragraph 2 of Article 26,
according to Judge Alicia Sempio-Diy, a member of the Civil Code Revision Committee, is to avoid the absurd situation where
the Filipino spouse remains married to the alien spouse who, after obtaining a divorce, is no longer married to the Filipino
spouse.
Interestingly, Paragraph 2 of Article 26 traces its origin to the 1985 case of Van Dorn v. Romillo, Jr.[10] The Van Dorn case
involved a marriage between a Filipino citizen and a foreigner. The Court held therein that a divorce decree validly obtained by
the alien spouse is valid in the Philippines, and consequently, the Filipino spouse is capacitated to remarry under Philippine law.
Does the same principle apply to a case where at the time of the celebration of the marriage, the parties were Filipino
citizens, but later on, one of them obtains a foreign citizenship by naturalization?
The jurisprudential answer lies latent in the 1998 case of Quita v. Court of Appeals. [11] In Quita, the parties were, as in this
case, Filipino citizens when they got married. The wife became a naturalized American citizen in 1954 and obtained a divorce
in the same year. The Court therein hinted, by way of obiter dictum, that a Filipino divorced by his naturalized foreign spouse is
no longer married under Philippine law and can thus remarry.
Thus, taking into consideration the legislative intent and applying the rule of reason, we hold that Paragraph 2 of Article 26
should be interpreted to include cases involving parties who, at the time of the celebration of the marriage were Filipino citizens,
but later on, one of them becomes naturalized as a foreign citizen and obtains a divorce decree. The Filipino spouse should
likewise be allowed to remarry as if the other party were a foreigner at the time of the solemnization of the marriage. To rule
otherwise would be to sanction absurdity and injustice. Where the interpretation of a statute according to its exact and literal
import would lead to mischievous results or contravene the clear purpose of the legislature, it should be construed according to
its spirit and reason, disregarding as far as necessary the letter of the law. A statute may therefore be extended to cases not
within the literal meaning of its terms, so long as they come within its spirit or intent. [12]
If we are to give meaning to the legislative intent to avoid the absurd situation where the Filipino spouse remains married
to the alien spouse who, after obtaining a divorce is no longer married to the Filipino spouse, then the instant case must be
deemed as coming within the contemplation of Paragraph 2 of Article 26.
In view of the foregoing, we state the twin elements for the application of Paragraph 2 of Article 26 as follows:
1.

There is a valid marriage that has been celebrated between a Filipino citizen and a foreigner; and

2.

A valid divorce is obtained abroad by the alien spouse capacitating him or her to remarry.

The reckoning point is not the citizenship of the parties at the time of the celebration of the marriage, but their citizenship at
the time a valid divorce is obtained abroad by the alien spouse capacitating the latter to remarry.
In this case, when Ciprianos wife was naturalized as an American citizen, there was still a valid marriage that has been
celebrated between her and Cipriano. As fate would have it, the naturalized alien wife subsequently obtained a valid divorce

capacitating her to remarry. Clearly, the twin requisites for the application of Paragraph 2 of Article 26 are both present in this
case. Thus Cipriano, the divorced Filipino spouse, should be allowed to remarry.
We are also unable to sustain the OSGs theory that the proper remedy of the Filipino spouse is to file either a petition for
annulment or a petition for legal separation. Annulment would be a long and tedious process, and in this particular case, not
even feasible, considering that the marriage of the parties appears to have all the badges of validity. On the other hand, legal
separation would not be a sufficient remedy for it would not sever the marriage tie; hence, the legally separated Filipino spouse
would still remain married to the naturalized alien spouse.
However, we note that the records are bereft of competent evidence duly submitted by respondent concerning the divorce
decree and the naturalization of respondents wife. It is settled rule that one who alleges a fact has the burden of proving it and
mere allegation is not evidence.[13]
Accordingly, for his plea to prosper, respondent herein must prove his allegation that his wife was naturalized as an
American citizen. Likewise, before a foreign divorce decree can be recognized by our own courts, the party pleading it must
prove the divorce as a fact and demonstrate its conformity to the foreign law allowing it. [14] Such foreign law must also be
proved as our courts cannot take judicial notice of foreign laws. Like any other fact, such laws must be alleged and proved. [15]
Furthermore, respondent must also show that the divorce decree allows his former wife to remarry as specifically required in
Article 26. Otherwise, there would be no evidence sufficient to declare that he is capacitated to enter into another marriage.
Nevertheless, we are unanimous in our holding that Paragraph 2 of Article 26 of the Family Code (E.O. No. 209, as
amended by E.O. No. 227), should be interpreted to allow a Filipino citizen, who has been divorced by a spouse who had
acquired foreign citizenship and remarried, also to remarry. However, considering that in the present petition there is no
sufficient evidence submitted and on record, we are unable to declare, based on respondents bare allegations that his wife,
who was naturalized as an American citizen, had obtained a divorce decree and had remarried an American, that respondent is
now capacitated to remarry. Such declaration could only be made properly upon respondents submission of the aforecited
evidence in his favor.
ACCORDINGLY, the petition by the Republic of the Philippines is GRANTED. The assailed Decision dated May 15,
2002, and Resolution dated July 4, 2002, of the Regional Trial Court of Molave, Zamboanga del Sur, Branch 23, are
hereby SET ASIDE.
No pronouncement as to costs.
SO ORDERED.

Marriage Exempt from License Requiremnt


G.R. No. 133778. March 14, 2000]

ENGRACE NIAL for Herself and as Guardian ad Litem of the minors BABYLINE NIAL, INGRID NIAL, ARCHIE NIAL
& PEPITO NIAL, JR., petitioners, vs. NORMA BAYADOG, respondent. Ncmmis
DECISION
YNARES_SANTIAGO, J.:
May the heirs of a deceased person file a petition for the declaration of nullity of his marriage after his death?
Pepito Nial was married to Teodulfa Bellones on September 26, 1974. Out of their marriage were born herein petitioners.
Teodulfa was shot by Pepito resulting in her death on April 24, 1985. One year and 8 months thereafter or on December 11,
1986, Pepito and respondent Norma Badayog got married without any marriage license. In lieu thereof, Pepito and Norma
executed an affidavit dated December 11, 1986 stating that they had lived together as husband and wife for at least five years
and were thus exempt from securing a marriage license. On February 19, 1997, Pepito died in a car accident. After their fathers
death, petitioners filed a petition for declaration of nullity of the marriage of Pepito to Norma alleging that the said marriage was
void for lack of a marriage license. The case was filed under the assumption that the validity or invalidity of the second marriage
would affect petitioners successional rights. Norma filed a motion to dismiss on the ground that petitioners have no cause of
action since they are not among the persons who could file an action for "annulment of marriage" under Article 47 of the Family
Code.
Judge Ferdinand J. Marcos of the Regional Trial Court of Toledo City, Cebu, Branch 59, dismissed the petition after finding that
the Family Code is "rather silent, obscure, insufficient" to resolve the following issues:
(1) Whether or not plaintiffs have a cause of action against defendant in asking for the declaration of the nullity of
marriage of their deceased father, Pepito G. Nial, with her specially so when at the time of the filing of this instant
suit, their father Pepito G. Nial is already dead;
(2) Whether or not the second marriage of plaintiffs deceased father with defendant is null and void ab initio;
(3) Whether or not plaintiffs are estopped from assailing the validity of the second marriage after it was dissolved
due to their fathers death.[1]
Thus, the lower court ruled that petitioners should have filed the action to declare null and void their fathers marriage to
respondent before his death, applying by analogy Article 47 of the Family Code which enumerates the time and the persons
who could initiate an action for annulment of marriage. [2] Hence, this petition for review with this Court grounded on a pure
question of law. Scnc m
This petition was originally dismissed for non-compliance with Section 11, Rule 13 of the 1997 Rules of Civil Procedure, and
because "the verification failed to state the basis of petitioners averment that the allegations in the petition are true and
correct." It was thus treated as an unsigned pleading which produces no legal effect under Section 3, Rule 7, of the 1997 Rules.
[3]
However, upon motion of petitioners, this Court reconsidered the dismissal and reinstated the petition for review. [4]
The two marriages involved herein having been solemnized prior to the effectivity of the Family Code (FC), the applicable law to
determine their validity is the Civil Code which was the law in effect at the time of their celebration. [5] A valid marriage license is
a requisite of marriage under Article 53 of the Civil Code, [6] the absence of which renders the marriage void ab initio pursuant to
Article 80(3)[7] in relation to Article 58.[8] The requirement and issuance of marriage license is the States demonstration of its
involvement and participation in every marriage, in the maintenance of which the general public is interested. [9] This interest
proceeds from the constitutional mandate that the State recognizes the sanctity of family life and of affording protection to the
family as a basic "autonomous social institution." [10] Specifically, the Constitution considers marriage as an "inviolable social
institution," and is the foundation of family life which shall be protected by the State. [11] This is why the Family Code considers
marriage as "a special contract of permanent union" [12] and case law considers it "not just an adventure but a lifetime
commitment."[13]
However, there are several instances recognized by the Civil Code wherein a marriage license is dispensed with, one of which
is that provided in Article 76,[14] referring to the marriage of a man and a woman who have lived together and exclusively with
each other as husband and wife for a continuous and unbroken period of at least five years before the marriage. The rationale
why no license is required in such case is to avoid exposing the parties to humiliation, shame and embarrassment concomitant
with the scandalous cohabitation of persons outside a valid marriage due to the publication of every applicants name for a
marriage license. The publicity attending the marriage license may discourage such persons from legitimizing their status. [15] To
preserve peace in the family, avoid the peeping and suspicious eye of public exposure and contain the source of gossip arising
from the publication of their names, the law deemed it wise to preserve their privacy and exempt them from that
requirement. Sdaa miso
There is no dispute that the marriage of petitioners father to respondent Norma was celebrated without any marriage license. In
lieu thereof, they executed an affidavit stating that "they have attained the age of majority, and, being unmarried, have lived
together as husband and wife for at least five years, and that we now desire to marry each other." [16] The only issue that needs
to be resolved pertains to what nature of cohabitation is contemplated under Article 76 of the Civil Code to warrant the counting
of the five year period in order to exempt the future spouses from securing a marriage license. Should it be a cohabitation
wherein both parties are capacitated to marry each other during the entire five-year continuous period or should it be a
cohabitation wherein both parties have lived together and exclusively with each other as husband and wife during the entire
five-year continuous period regardless of whether there is a legal impediment to their being lawfully married, which impediment
may have either disappeared or intervened sometime during the cohabitation period?
Working on the assumption that Pepito and Norma have lived together as husband and wife for five years without the benefit of
marriage, that five-year period should be computed on the basis of a cohabitation as "husband and wife" where the only missing
factor is the special contract of marriage to validate the union. In other words, the five-year common-law cohabitation period,
which is counted back from the date of celebration of marriage, should be a period of legal union had it not been for the
absence of the marriage. This 5-year period should be the years immediately before the day of the marriage and it should be a
period of cohabitation characterized by exclusivity meaning no third party was involved at any time within the 5 years and
continuity that is unbroken. Otherwise, if that continuous 5-year cohabitation is computed without any distinction as to whether
the parties were capacitated to marry each other during the entire five years, then the law would be sanctioning immorality and
encouraging parties to have common law relationships and placing them on the same footing with those who lived faithfully with
their spouse. Marriage being a special relationship must be respected as such and its requirements must be strictly observed.
The presumption that a man and a woman deporting themselves as husband and wife is based on the approximation of the
requirements of the law. The parties should not be afforded any excuse to not comply with every single requirement and later
use the same missing element as a pre-conceived escape ground to nullify their marriage. There should be no exemption from
securing a marriage license unless the circumstances clearly fall within the ambit of the exception. It should be noted that a
license is required in order to notify the public that two persons are about to be united in matrimony and that anyone who is
aware or has knowledge of any impediment to the union of the two shall make it known to the local civil registrar. [17] The Civil
Code provides:
Article 63: "x x x. This notice shall request all persons having knowledge of any impediment to the marriage to
advice the local civil registrar thereof. x x x."
Article 64: "Upon being advised of any alleged impediment to the marriage, the local civil registrar shall forthwith
make an investigation, examining persons under oath. x x x" Sdaad

This is reiterated in the Family Code thus:


Article 17 provides in part: "x x x. This notice shall request all persons having knowledge of any impediment to the
marriage to advise the local civil registrar thereof. x x x."
Article 18 reads in part: "x x x. In case of any impediment known to the local civil registrar or brought to his attention,
he shall note down the particulars thereof and his findings thereon in the application for a marriage license. x x x."
This is the same reason why our civil laws, past or present, absolutely prohibited the concurrence of multiple marriages by the
same person during the same period. Thus, any marriage subsequently contracted during the lifetime of the first spouse shall
be illegal and void,[18] subject only to the exception in cases of absence or where the prior marriage was dissolved or annulled.
The Revised Penal Code complements the civil law in that the contracting of two or more marriages and the having of
extramarital affairs are considered felonies, i.e., bigamy and concubinage and adultery.[19] The law sanctions monogamy.
In this case, at the time of Pepito and respondents marriage, it cannot be said that they have lived with each other as husband
and wife for at least five years prior to their wedding day. From the time Pepitos first marriage was dissolved to the time of his
marriage with respondent, only about twenty months had elapsed. Even assuming that Pepito and his first wife had separated in
fact, and thereafter both Pepito and respondent had started living with each other that has already lasted for five years, the fact
remains that their five-year period cohabitation was not the cohabitation contemplated by law. It should be in the nature of a
perfect union that is valid under the law but rendered imperfect only by the absence of the marriage contract. Pepito had a
subsisting marriage at the time when he started cohabiting with respondent. It is immaterial that when they lived with each other,
Pepito had already been separated in fact from his lawful spouse. The subsistence of the marriage even where there was actual
severance of the filial companionship between the spouses cannot make any cohabitation by either spouse with any third party
as being one as "husband and wife". Scs daad
Having determined that the second marriage involved in this case is not covered by the exception to the requirement of a
marriage license, it is void ab initio because of the absence of such element.
The next issue to be resolved is: do petitioners have the personality to file a petition to declare their fathers marriage void after
his death?
Contrary to respondent judges ruling, Article 47 of the Family Code [20] cannot be applied even by analogy to petitions for
declaration of nullity of marriage. The second ground for annulment of marriage relied upon by the trial court, which allows "the
sane spouse" to file an annulment suit "at any time before the death of either party" is inapplicable. Article 47 pertains to the
grounds, periods and persons who can file an annulment suit, not a suit for declaration of nullity of marriage. The Code is silent
as to who can file a petition to declare the nullity of a marriage. Voidable and void marriages are not identical. A marriage that is
annulable is valid until otherwise declared by the court; whereas a marriage that is void ab initio is considered as having never
to have taken place[21] and cannot be the source of rights. The first can be generally ratified or confirmed by free cohabitation or
prescription while the other can never be ratified. A voidable marriage cannot be assailed collaterally except in a direct
proceeding while a void marriage can be attacked collaterally. Consequently, void marriages can be questioned even after the
death of either party but voidable marriages can be assailed only during the lifetime of the parties and not after death of either,
in which case the parties and their offspring will be left as if the marriage had been perfectly valid. [22] That is why the action or
defense for nullity is imprescriptible, unlike voidable marriages where the action prescribes. Only the parties to a voidable
marriage can assail it but any proper interested party may attack a void marriage. Void marriages have no legal effects except
those declared by law concerning the properties of the alleged spouses, regarding co-ownership or ownership through actual
joint contribution,[23] and its effect on the children born to such void marriages as provided in Article 50 in relation to Article 43
and 44 as well as Article 51, 53 and 54 of the Family Code. On the contrary, the property regime governing voidable marriages
is generally conjugal partnership and the children conceived before its annulment are legitimate. Sup rema
Contrary to the trial courts ruling, the death of petitioners father extinguished the alleged marital bond between him and
respondent. The conclusion is erroneous and proceeds from a wrong premise that there was a marriage bond that was
dissolved between the two. It should be noted that their marriage was void hence it is deemed as if it never existed at all and the
death of either extinguished nothing.
Jurisprudence under the Civil Code states that no judicial decree is necessary in order to establish the nullity of a marriage. [24] "A
void marriage does not require a judicial decree to restore the parties to their original rights or to make the marriage void but
though no sentence of avoidance be absolutely necessary, yet as well for the sake of good order of society as for the peace of
mind of all concerned, it is expedient that the nullity of the marriage should be ascertained and declared by the decree of a court
of competent jurisdiction."[25] "Under ordinary circumstances, the effect of a void marriage, so far as concerns the conferring of
legal rights upon the parties, is as though no marriage had ever taken place. And therefore, being good for no legal purpose, its
invalidity can be maintained in any proceeding in which the fact of marriage may be material, either direct or collateral, in any
civil court between any parties at any time, whether before or after the death of either or both the husband and the wife, and
upon mere proof of the facts rendering such marriage void, it will be disregarded or treated as non-existent by the courts." It is
not like a voidable marriage which cannot be collaterally attacked except in direct proceeding instituted during the lifetime of the
parties so that on the death of either, the marriage cannot be impeached, and is made good ab initio.[26] But Article 40 of the
Family Code expressly provides that there must be a judicial declaration of the nullity of a previous marriage, though void,
before a party can enter into a second marriage [27] and such absolute nullity can be based only on a final judgment to that effect.
[28]
For the same reason, the law makes either the action or defense for the declaration of absolute nullity of marriage
imprescriptible.[29] Corollarily, if the death of either party would extinguish the cause of action or the ground for defense, then the
same cannot be considered imprescriptible. Juris
However, other than for purposes of remarriage, no judicial action is necessary to declare a marriage an absolute nullity. For
other purposes, such as but not limited to determination of heirship, legitimacy or illegitimacy of a child, settlement of estate,
dissolution of property regime, or a criminal case for that matter, the court may pass upon the validity of marriage even in a suit
not directly instituted to question the same so long as it is essential to the determination of the case. This is without prejudice to
any issue that may arise in the case. When such need arises, a final judgment of declaration of nullity is necessary even if the
purpose is other than to remarry. The clause "on the basis of a final judgment declaring such previous marriage void" in Article
40 of the Family Code connotes that such final judgment need not be obtained only for purpose of remarriage.
WHEREFORE, the petition is GRANTED. The assailed Order of the Regional Trial Court, Toledo City, Cebu, Branch 59,
dismissing Civil Case No. T-639, is REVERSED and SET ASIDE. The said case is ordered REINSTATED.
SO ORDERED.

A.M. No. MTJ-00-1329. March 8, 2001]

HERMINIA BORJA-MANZANO, petitioner, vs. JUDGE ROQUE R. SANCHEZ, MTC, Infanta, Pangasinan, respondent.
RESOLUTION
DAVIDE, JR., C.J.:
The solemnization of a marriage between two contracting parties who were both bound by a prior existing marriage is the
bone of contention of the instant complaint against respondent Judge Roque R. Sanchez, Municipal Trial Court, Infanta,
Pangasinan. For this act, complainant Herminia Borja-Manzano charges respondent Judge with gross ignorance of the law in a
sworn Complaint-Affidavit filed with the Office of the Court Administrator on 12 May 1999.
Complainant avers that she was the lawful wife of the late David Manzano, having been married to him on 21 May 1966 in
San Gabriel Archangel Parish, Araneta Avenue, Caloocan City. [1] Four children were born out of that marriage. [2] On 22 March
1993, however, her husband contracted another marriage with one Luzviminda Payao before respondent Judge. [3] When
respondent Judge solemnized said marriage, he knew or ought to know that the same was void and bigamous, as the marriage
contract clearly stated that both contracting parties were separated.
Respondent Judge, on the other hand, claims in his Comment that when he officiated the marriage between Manzano and
Payao he did not know that Manzano was legally married. What he knew was that the two had been living together as husband
and wife for seven years already without the benefit of marriage, as manifested in their joint affidavit. [4] According to him, had he
known that the late Manzano was married, he would have advised the latter not to marry again; otherwise, he (Manzano) could
be charged with bigamy. He then prayed that the complaint be dismissed for lack of merit and for being designed merely to
harass him.
After an evaluation of the Complaint and the Comment, the Court Administrator recommended that respondent Judge be
found guilty of gross ignorance of the law and be ordered to pay a fine of P2,000, with a warning that a repetition of the same or
similar act would be dealt with more severely.
On 25 October 2000, this Court required the parties to manifest whether they were willing to submit the case for resolution
on the basis of the pleadings thus filed. Complainant answered in the affirmative.
For his part, respondent Judge filed a Manifestation reiterating his plea for the dismissal of the complaint and setting aside
his earlier Comment. He therein invites the attention of the Court to two separate affidavits [5] of the late Manzano and of Payao,
which were allegedly unearthed by a member of his staff upon his instruction. In those affidavits, both David Manzano and
Luzviminda Payao expressly stated that they were married to Herminia Borja and Domingo Relos, respectively; and that since
their respective marriages had been marked by constant quarrels, they had both left their families and had never cohabited or
communicated with their spouses anymore. Respondent Judge alleges that on the basis of those affidavits, he agreed to
solemnize the marriage in question in accordance with Article 34 of the Family Code.
We find merit in the complaint.
Article 34 of the Family Code provides:
No license shall be necessary for the marriage of a man and a woman who have lived together as husband and wife for at least
five years and without any legal impediment to marry each other. The contracting parties shall state the foregoing facts in an
affidavit before any person authorized by law to administer oaths. The solemnizing officer shall also state under oath that he
ascertained the qualifications of the contracting parties and found no legal impediment to the marriage.
For this provision on legal ratification of marital cohabitation to apply, the following requisites must concur:
1. The man and woman must have been living together as husband and wife for at least five years before the marriage;
2. The parties must have no legal impediment to marry each other;
3. The fact of absence of legal impediment between the parties must be present at the time of marriage;
4. The parties must execute an affidavit stating that they have lived together for at least five years [and are without legal
impediment to marry each other]; and
5. The solemnizing officer must execute a sworn statement that he had ascertained the qualifications of the parties and
that he had found no legal impediment to their marriage. [6]
Not all of these requirements are present in the case at bar. It is significant to note that in their separate affidavits executed
on 22 March 1993 and sworn to before respondent Judge himself, David Manzano and Luzviminda Payao expressly stated the
fact of their prior existing marriage. Also, in their marriage contract, it was indicated that both were separated.
Respondent Judge knew or ought to know that a subsisting previous marriage is a diriment impediment, which would make
the subsequent marriage null and void. [7] In fact, in his Comment, he stated that had he known that the late Manzano was
married he would have discouraged him from contracting another marriage. And respondent Judge cannot deny knowledge of
Manzanos and Payaos subsisting previous marriage, as the same was clearly stated in their separate affidavits which were
subscribed and sworn to before him.
The fact that Manzano and Payao had been living apart from their respective spouses for a long time already is
immaterial. Article 63(1) of the Family Code allows spouses who have obtained a decree of legal separation to live separately
from each other, but in such a case the marriage bonds are not severed. Elsewise stated, legal separation does not dissolve
the marriage tie, much less authorize the parties to remarry. This holds true all the more when the separation is merely de
facto, as in the case at bar.
Neither can respondent Judge take refuge on the Joint Affidavit of David Manzano and Luzviminda Payao stating that they
had been cohabiting as husband and wife for seven years. Just like separation, free and voluntary cohabitation with another
person for at least five years does not severe the tie of a subsisting previous marriage. Marital cohabitation for a long period of
time between two individuals who are legally capacitated to marry each other is merely a ground for exemption from marriage
license. It could not serve as a justification for respondent Judge to solemnize a subsequent marriage vitiated by the
impediment of a prior existing marriage.
Clearly, respondent Judge demonstrated gross ignorance of the law when he solemnized a void and bigamous
marriage. The maxim ignorance of the law excuses no one has special application to judges, [8] who, under Rule 1.01 of the
Code of Judicial Conduct, should be the embodiment of competence, integrity, and independence. It is highly imperative that
judges be conversant with the law and basic legal principles. [9] And when the law transgressed is simple and elementary, the
failure to know it constitutes gross ignorance of the law.[10]
ACCORDINGLY, the recommendation of the Court Administrator is hereby ADOPTED, with the MODIFICATION that the
amount of fine to be imposed upon respondent Judge Roque Sanchez is increased to P20,000.
SO ORDERED.

A.M. No. MTJ-92-721 September 30, 1994


JUVY N. COSCA, EDMUNDO B. PERALTA, RAMON C. SAMBO, and APOLLO A. VILLAMORA, complainants,
vs.

HON. LUCIO P. PALAYPAYON, JR., Presiding Judge, and NELIA B. ESMERALDA-BAROY, Clerk of Court II, both of the
Municipal Trial Court of Tinambac, Camarines Sur, respondents.
Esteban R. Abonal for complainants.
Haide B. Vista-Gumba for respondents.

PER CURIAM, J.:


Complainants Juvy N. Cosca, Edmundo B. Peralta, Ramon C. Sambo, and Apollo Villamora, are Stenographer I, Interpreter I,
Clerk II, and Process Server, respectively, of the Municipal Trial Court of Tinambac, Camarines Sur. Respondents Judge Lucio
P. Palaypayon, Jr. and Nelia B. Esmeralda-Baroy are respectively the Presiding Judge and Clerk of Court II of the same court.
In an administrative complaint filed with the Office of the Court Administrator on October 5, 1992, herein respondents were
charged with the following offenses, to wit: (1) illegal solemnization of marriage; (2) falsification of the monthly reports of cases;
(3) bribery in consideration of an appointment in the court; (4) non-issuance of receipt for cash bond received; (5) infidelity in the
custody of detained prisoners; and (6) requiring payment of filing fees from exempted entities. 1
Pursuant to a resolution issued by this Court respondents filed their respective Comments. 2 A Reply to Answers of
Respondents was filed by complainants. 3 The case was thereafter referred to Executive Judge David C. Naval of the Regional
Trial Court, Naga City, for investigation report and recommendation. The case was however transferred to First Assistant
Executive Judge Antonio N. Gerona when Judge Naval inhibited himself for the reason that his wife is a cousin of respondent
Judge Palaypayon, Jr. 4
The contending versions of the parties regarding the factual antecedents of this administrative matter, as culled from the records
thereof, are set out under each particular charge against respondents.
1. Illegal solemnization of marriage
Complainants allege that respondent judge solemnized marriages even without the requisite marriage license. Thus, the
following couples were able to get married by the simple expedient of paying the marriage fees to respondent Baroy, despite the
absence of a marriage license, viz.: Alano P. Abellano and Nelly Edralin, Francisco Selpo and Julieta Carrido, Eddie Terrobias
and Maria Gacer, Renato Gamay and Maricris Belga, Arsenio Sabater and Margarita Nacario, and Sammy Bocaya and Gina
Bismonte. As a consequence, their marriage contracts (Exhibits B, C, D, F, G, and A, respectively) did not reflect any marriage
license number. In addition, respondent judge did not sign their marriage contracts and did not indicate the date of
solemnization, the reason being that he allegedly had to wait for the marriage license to be submitted by the parties which was
usually several days after the ceremony. Indubitably, the marriage contracts were not filed with the local civil registrar.
Complainant Ramon Sambo, who prepares the marriage contracts, called the attention of respondents to the lack of marriage
licenses and its effect on the marriages involved, but the latter opted to proceed with the celebration of said marriages.
Respondent Nelia Baroy claims that when she was appointed Clerk of Court II, the employees of the court were already hostile
to her, especially complainant Ramon Sambo who told her that he was filing a protest against her appointment. She avers that it
was only lately when she discovered that the court had a marriage Register which is in the custody of Sambo; that it was
Sambo who failed to furnish the parties copies of the marriage contract and to register these with the local civil registrar; and
that apparently Sambo kept these marriage contracts in preparation for this administrative case. Complainant Sambo, however,
claims that all file copies of the marriage contracts were kept by respondent Baroy, but the latter insists that she had instructed
Sambo to follow up the submission by the contracting parties of their marriage licenses as part of his duties but he failed to do
so.
Respondent Judge Palaypayon, Jr. contends that the marriage between Alano P. Abellano and Nelly Edralin falls under Article
34 of the Civil Code, hence it is exempt from the marriage license requirement; that he gave strict instructions to complainant
Sambo to furnish the couple a copy of the marriage contract and to file the same with the civil registrar, but the latter failed to do
so; that in order to solve the problem, the spouses subsequently formalized their marriage by securing a marriage license and
executing their marriage contract, a copy of which was filed with the civil registrar; that the other five marriages alluded to in the
administrative complaint were not illegally solemnized because the marriage contracts were not signed by him and they did not
contain the date and place of marriage; that copies of these marriage contracts are in the custody of complainant Sambo; that
the alleged marriage of Francisco Selpo and Julieta Carrido, Eddie Terrobias and Maria Emma Gaor, Renato Gamay and
Maricris Belga, and of Arsenio Sabater and Margarita Nacario were not celebrated by him since he refused to solemnize them in
the absence of a marriage license; that the marriage of Samy Bocaya and Gina Bismonte was celebrated even without the
requisite license due to the insistence of the parties in order to avoid embarrassment to their guests but that, at any rate, he did
not sign their marriage contract which remains unsigned up to the present.
2. Falsification of monthly report for July, 1991 regarding the number of marriages solemnized and the number of
documents notarized.
It is alleged that respondent judge made it appear that he solemnized seven (7) marriages in the month of July, 1992, when in
truth he did not do so or at most those marriages were null and void; that respondents likewise made it appear that they have
notarized only six (6) documents for July, 1992, but the Notarial Register will show that there were one hundred thirteen (113)
documents which were notarized during that month; and that respondents reported a notarial fee of only P18.50 for each
document, although in fact they collected P20.00 therefor and failed to account for the difference.
Respondent Baroy contends, however, that the marriage registry where all marriages celebrated by respondent judge are
entered is under the exclusive control and custody of complainant Ramon Sambo, hence he is the only one who should be held
responsible for the entries made therein; that the reported marriages are merely based on the payments made as solemnization
fees which are in the custody of respondent Baroy. She further avers that it is Sambo who is likewise the custodian of the
Notarial Register; that she cannot be held accountable for whatever alleged difference there is in the notarial fees because she
is liable only for those payments tendered to her by Sambo himself; that the notarial fees she collects are duly covered by
receipts; that of the P20.00 charged, P18.50 is remitted directly to the Supreme Court as part of the Judiciary Development
Fund and P150 goes to the general fund of the Supreme Court which is paid to the Municipal Treasurer of Tinambac,
Camarines Sur. Respondent theorizes that the discrepancies in the monthly report were manipulated by complainant Sambo
considering that he is the one in charge of the preparation of the monthly report.
Respondent Judge Palaypayon avers that the erroneous number of marriages celebrated was intentionally placed by
complainant Sambo; that the number of marriages solemnized should not be based on solemnization fees paid for that month
since not all the marriages paid for are solemnized in the same month. He claims that there were actually only six (6)
documents notarized in the month of July, 1992 which tallied with the official receipts issued by the clerk of court; that it is
Sambo who should be held accountable for any unreceipted payment for notarial fees because he is the one in charge of the
Notarial Register; and that this case filed by complainant Sambo is merely in retaliation for his failure to be appointed as the
clerk of court. Furthermore, respondent judge contends that he is not the one supervising or preparing the monthly report, and
that he merely has the ministerial duty to sign the same.
3. Bribery in consideration of an appointment in the court

Complainants allege that because of the retirement of the clerk of court, respondent judge forwarded to the Supreme Court the
applications of Rodel Abogado, Ramon Sambo, and Jessell Abiog. However, they were surprised when respondent Baroy
reported for duty as clerk of court on October 21, 1991. They later found out that respondent Baroy was the one appointed
because she gave a brand-new air-conditioning unit to respondent judge.
Respondent Baroy claims that when she was still in Naga City she purchased an air-conditioning unit but when she was
appointed clerk of court she had to transfer to Tinambac and, since she no longer needed the air conditioner, she decided to sell
the same to respondent judge. The installation and use thereof by the latter in his office was with the consent of the Mayor of
Tinambac.
Respondent judge contends that he endorsed all the applications for the position of clerk of court to the Supreme Court which
has the sole authority over such appointments and that he had no hand in the appointment of respondent Baroy. He contends
that the air-conditioning unit was bought from his
co-respondent on installment basis on May 29, 1992, eight (8) months after Baroy had been appointed clerk of court. He claims
that he would not be that naive to exhibit to the public as item which could not be defended as a matter of honor and prestige.
4. Cash bond issued without a receipt
It is alleged that in Criminal Case No. 5438, entitled "People vs. Mendeza, et al., "bondswoman Januaria Dacara was allowed
by respondent judge to change her property bond to cash bond; that she paid the amount of P1,000.00 but was never issued a
receipt therefor nor was it made to appear in the records that the bond has been paid; that despite the lapse of two years, the
money was never returned to the bondswoman; and that it has not been shown that the money was turned over to the Municipal
Treasurer of Tinambac.
Respondent Baroy counters that the cash bond was deposited with the former clerk of court, then turned over to the acting clerk
of court and, later, given to her under a corresponding receipt; that the cash bond is deposited with the bank; and that should
the bondswoman desire to withdraw the same, she should follow the proper procedure therefor.
Respondent judge contends that Criminal Case No. 5438 was archieved for failure of the bondsman to deliver the body of the
accused in court despite notice; and that he has nothing to do with the payment of the cash bond as this is the duty of the clerk
of court.
5. Infidelity in the custody of prisoners
Complainants contend that respondent judge usually got detention prisoners to work in his house, one of whom was Alex Alano,
who is accused in Criminal Case No. 5647 for violation of the Dangerous Drugs Act; that while Alano was in the custody of
respondent judge, the former escaped and was never recaptured; that in order to conceal this fact, the case was archived
pursuant to an order issued by respondent judge dated April 6, 1992.
Respondent judge denied the accusation and claims that he never employed detention prisoners and that he has adequate
household help; and that he had to order the case archived because it had been pending for more than six (6) months and the
accused therein remained at large.
6. Unlawful collection of docket fees
Finally, respondents are charged with collecting docket fees from the Rural Bank of Tinambac, Camarines Sur, Inc. although
such entity is exempt by law from the payment of said fees, and that while the corresponding receipt was issued, respondent
Baroy failed to remit the amount to the Supreme Court and, instead, she deposited the same in her personal account.
Respondents Baroy contends that it was Judge-Designate Felimon Montenegro (because respondent judge was on sick leave)
who instructed her to demand payment of docket fees from said rural bank; that the bank issued a check for P800.00; that she
was not allowed by the Philippine National Bank to encash the check and, instead, was instructed to deposit the same in any
bank account for clearing; that respondent deposited the same in her account; and that after the check was cleared, she
remitted P400.00 to the Supreme Court and the other P400.00 was paid to the Municipal Treasurer of Tinambac.
On the basis of the foregoing contentions, First Vice-Executive Judge Antonio N. Gerona prepared and submitted to us his
Report and Recommendations dated May 20, 1994, together with the administrative matter. We have perspicaciously reviewed
the same and we are favorably impressed by the thorough and exhaustive presentation and analysis of the facts and evidence
in said report. We commend the investigating judge for his industry and perspicacity reflected by his findings in said report
which, being amply substantiated by the evidence and supported by logical illations, we hereby approve and hereunder
reproduce at length the material portions thereof.
xxx xxx xxx
The first charge against the respondents is illegal solemnization of marriage. Judge Palaypayon is charged with
having solemnized without a marriage license the marriage of Sammy Bocaya and Gina Besmonte (Exh. A). Alano
Abellano and Nelly Edralin (Exh. B), Francisco Selpo and Julieta Carrido (Exh. C), Eddie Terrobias and Maria Emma
Gaor (Exh. D), Renato Gamay and Maricris Belga (Exh. F) and Arsenio Sabater and Margarita Nacario (Exh. G).
In all these aforementioned marriages, the blank space in the marriage contracts to show the number of the
marriage was solemnized as required by Article 22 of the Family Code were not filled up. While the contracting
parties and their witnesses signed their marriage contracts, Judge Palaypayon did not affix his signature in the
marriage contracts, except that of Abellano and Edralin when Judge Palaypayon signed their marriage certificate as
he claims that he solemnized this marriage under Article 34 of the Family Code of the Philippines. In said marriages
the contracting parties were not furnished a copy of their marriage contract and the Local Civil Registrar was not
sent either a copy of the marriage certificate as required by Article 23 of the Family Code.
The marriage of Bocaya and Besmonte is shown to have been solemnized by Judge Palaypayon without a marriage
license. The testimonies of Bocay himself and Pompeo Ariola, one of the witnesses of the marriage of Bocaya and
Besmonte, and the photographs taken when Judge Palaypayon solemnized their marriage (Exhs. K-3 to K-9)
sufficiently show that Judge Palaypayon really solemnized their marriage. Bocaya declared that they were advised
by Judge Palaypayon to return after ten (10) days after their marriage was solemnized and bring with them their
marriage license. In the meantime, they already started living together as husband and wife believing that the formal
requisites of marriage were complied with.
Judge Palaypayon denied that he solemnized the marriage of Bocaya and Besmonte because the parties allegedly
did not have a marriage license. He declared that in fact he did not sign the marriage certificate, there was no date
stated on it and both the parties and the Local Civil Registrar did not have a copy of the marriage certificate.
With respect to the photographs which show that he solemnized the marriage of Bocaya and Besmonte, Judge
Palaypayon explains that they merely show as if he was solemnizing the marriage. It was actually a simulated
solemnization of marriage and not a real one. This happened because of the pleading of the mother of one of the
contracting parties that he consent to be photographed to show that as if he was solemnizing the marriage as he
was told that the food for the wedding reception was already prepared, visitors were already invited and the place of
the parties where the reception would be held was more than twenty (20) kilometers away from the poblacion of
Tinambac.

The denial made by Judge Palaypayon is difficult to believe. The fact alone that he did not sign the marriage
certificate or contract, the same did not bear a date and the parties and the Local Civil Registrar were not furnished
a copy of the marriage certificate, do not by themselves show that he did not solemnize the marriage. His
uncorroborated testimony cannot prevail over the testimony of Bocaya and Ariola who also declared, among others,
that Bocaya and his bride were advised by Judge Palaypayon to return after ten (10) days with their marriage
license and whose credibility had not been impeached.
The pictures taken also from the start of the wedding ceremony up to the signing of the marriage certificate in front
of Judge Palaypayon and on his table (Exhs. K-3, K-3-a, K-3-b, K-3-c, K-4, K-4-a, K-4-b, K-4-c,
K-4-d, K-5, K-5-a, K-5-b, K-6, K-7, K-8, K-8-a and K-9), cannot possibly be just to show a simulated solemnization of
marriage. One or two pictures may convince a person of the explanation of Judge Palaypayon, but not all those
pictures.
Besides, as a judge it is very difficult to believe that Judge Palaypayon would allows himself to be photographed as
if he was solemnizing a marriage on a mere pleading of a person whom he did not even know for the alleged
reasons given. It would be highly improper and unbecoming of him to allow himself to be used as an instrument of
deceit by making it appear that Bocaya and Besmonte were married by him when in truth and in fact he did not
solemnize their marriage.
With respect to the marriage of Abellano and Edralin (Exh. B), Judge Palaypayon admitted that he solemnized their
marriage, but he claims that it was under Article 34 of the Family Code, so a marriage license was not required. The
contracting parties here executed a joint affidavit that they have been living together as husband and wife for almost
six (6) years already (Exh. 12; Exh. AA).
In their marriage contract which did not bear any date either when it was solemnized, it was stated that Abellano was
only eighteen (18) years, two (2) months and seven (7) days old. If he and Edralin had been living together as
husband and wife for almost six (6) years already before they got married as they stated in their joint affidavit,
Abellano must ha(ve) been less than thirteen (13) years old when he started living with Edralin as his wife and this is
hard to believe. Judge Palaypayon should ha(ve) been aware of this when he solemnized their marriage as it was
his duty to ascertain the qualification of the contracting parties who might ha(ve) executed a false joint affidavit in
order to have an instant marriage by avoiding the marriage license requirement.
On May 23, 1992, however, after this case was already filed, Judge Palaypayon married again Abellano and Edralin,
this time with a marriage license (Exh. BB). The explanation given by Judge Palaypayon why he solemnized the
marriage of the same couple for the second time is that he did not consider the first marriage he solemnized under
Article 34 of the Family Code as (a) marriage at all because complainant Ramon Sambo did not follow his instruction
that the date should be placed in the marriage certificate to show when he solemnized the marriage and that the
contracting parties were not furnished a copy of their marriage certificate.
This act of Judge Palaypayon of solemnizing the marriage of Abellano and Edralin for the second time with a
marriage license already only gave rise to the suspicion that the first time he solemnized the marriage it was only
made to appear that it was solemnized under exceptional character as there was not marriage license and Judge
Palaypayon had already signed the marriage certificate. If it was true that he solemnized the first marriage under
exceptional character where a marriage license was not required, why did he already require the parties to have a
marriage license when he solemnized their marriage for the second time?
The explanation of Judge Palaypayon that the first marriage of Abellano and Edralin was not a marriage at all as the
marriage certificate did not state the date when the marriage was solemnized and that the contracting parties were
not furnished a copy of their marriage certificate, is not well taken as they are not any of those grounds under
Article(s) 35, 36, 37 and 38 of the Family Code which declare a marriage void from the beginning. Even if no one,
however, received a copy of the marriage certificate, the marriage is still valid (Jones vs. H(o)rtiguela, 64 Phil. 179).
Judge Palaypayon cannot just absolve himself from responsibility by blaming his personnel. They are not the
guardian(s) of his official function and under Article 23 of the Family Code it is his duty to furnish the contracting
parties (a) copy of their marriage contract.
With respect to the marriage of Francisco Selpo and Julieta Carrido (Exh. C), and Arsenio Sabater and Margarita
Nacario (Exh. G), Selpo and Carrido and Sabater and Nacarcio executed joint affidavits that Judge Palaypayon did
not solemnize their marriage (Exh. 13-A and Exh. 1). Both Carrido and Nacario testified for the respondents that
actually Judge Palaypayon did not solemnize their marriage as they did not have a marriage license. On crossexamination, however, both admitted that they did not know who prepared their affidavits. They were just told,
Carrido by a certain Charito Palaypayon, and Nacario by a certain Kagawad Encinas, to just go to the Municipal
building and sign their joint affidavits there which were already prepared before the Municipal Mayor of Tinambac,
Camarines Sur.
With respect to the marriage of Renato Gamay and Maricris Belga (Exh. f), their marriage contract was signed by
them and by their two (2) witnesses, Atty. Elmer Brioso and respondent Baroy (Exhs. F-1 and F-2). Like the other
aforementioned marriages, the solemnization fee was also paid as shown by a receipt dated June 7, 1992 and
signed by respondent Baroy (Exh. F-4).
Judge Palaypayon also denied having solemnized the marriage of Gamay and Belga allegedly because there was
no marriage license. On her part, respondent Baroy at first denied that the marriage was solemnized. When she was
asked, however, why did she sign the marriage contract as a witness she answered that she thought the marriage
was already solemnized (TSN, p. 14; 10-28-93).
Respondent Baroy was, and is, the clerk of court of Judge Palaypayon. She signed the marriage contract of Gamay
and Belga as one of the two principal sponsors. Yet, she wanted to give the impression that she did not even know
that the marriage was solemnized by Judge Palaypayon. This is found very difficult to believe.
Judge Palaypayon made the same denial of having solemnized also the marriage of Terrobias and Gaor (Exh. D).
The contracting parties and their witnesses also signed the marriage contract and paid the solemnization fee, but
Judge Palaypayon allegedly did not solemnize their marriage due to lack of marriage license. Judge Palaypayon
submitted the affidavit of William Medina, Vice-Mayor of Tinambac, to corroborate his testimony (Exh. 14). Medina,
however, did not testify in this case and so his affidavit has no probative value.
Judge Palaypayon testified that his procedure and practice have been that before the contracting parties and their
witnesses enter his chamber in order to get married, he already required complainant Ramon Sambo to whom he
assigned the task of preparing the marriage contract, to already let the parties and their witnesses sign their
marriage contracts, as what happened to Gamay and Belga, and Terrobias and Gaor, among others. His purpose
was to save his precious time as he has been solemnizing marriages at the rate of three (3) to four (4) times
everyday (TSN, p. 12;
2-1-94).
This alleged practice and procedure, if true, is highly improper and irregular, if not illegal, because the contracting
parties are supposed to be first asked by the solemnizing officer and declare that they take each other as husband

and wife before the solemnizing officer in the presence of at least two (2) witnesses before they are supposed to
sign their marriage contracts (Art. 6, Family Code).
The uncorroborated testimony, however, of Judge Palaypayon as to his alleged practice and procedure before
solemnizing a marriage, is not true as shown by the picture taken during the wedding of Bocaya and Besmonte
(Exhs. K-3 to K-9) and by the testimony of respondent Baroy herself who declared that the practice of Judge
Palaypayon ha(s) been to let the contracting parties and their witnesses sign the marriage contract only after Judge
Palaypayon has solemnized their marriage (TSN, p. 53;
10-28-93).
Judge Palaypayon did not present any evidence to show also that he was really solemnizing three (3) to four (4)
marriages everyday. On the contrary his monthly report of cases for July, 1992 shows that his court had only twentyseven (27) pending cases and he solemnized only seven (7) marriages for the whole month (Exh. E). His monthly
report of cases for September, 1992 shows also that he solemnized only four (4) marriages during the whole month
(Exh. 7).
In this first charge of having illegally solemnized marriages, respondent Judge Palaypayon has presented and
marked in evidence several marriage contracts of other persons, affidavits of persons and certification issued by the
Local Civil Registrar (Exhs. 12-B to 12-H). These persons who executed affidavits, however, did not testify in this
case. Besides, the marriage contracts and certification mentioned are immaterial as Judge Palaypayon is not
charged of having solemnized these marriages illegally also. He is not charged that the marriages he solemnized
were all illegal.
The second charge against herein respondents, that of having falsified the monthly report of cases submitted to the
Supreme Court and not stating in the monthly report the actual number of documents notarized and issuing the
corresponding receipts of the notarial fees, have been sufficiently proven by the complainants insofar as the monthly
report of cases for July and September, 1992 are concerned.
The monthly report of cases of the MTC of Tinambac, Camarines Sur for July, 1992 both signed by the respondents,
show that for said month there were six (6) documents notarized by Judge Palaypayon in his capacity as Ex-Officio
Notary Public (Exhs. H to H-1-b). The notarial register of the MTC of Tinambac, Camarines Sur, however, shows that
there were actually one hundred thirteen (113) documents notarized by Judge Palaypayon for the said month (Exhs.
Q to Q-45).
Judge Palaypayon claims that there was no falsification of the monthly report of cases for July, 1992 because there
were only six (6) notarized documents that were paid (for) as shown by official receipts. He did not, however,
present evidence of the alleged official receipts showing that the notarial fee for the six (6) documetns were paid.
Besides, the monthly report of cases with respect to the number of documents notarized should not be based on
how many notarized documents were paid of the notarial fees, but the number of documents placed or recorded in
the notarial register.
Judge Palaypayon admitted that he was not personally verifying and checking anymore the correctness of the
monthly reports because he relies on his co-respondent who is the Clerk of Court and whom he has assumed to
have checked and verified the records. He merely signs the monthly report when it is already signed by respondent
Baroy.
The explanation of Judge Palaypayon is not well taken because he is required to have close supervision in the
preparation of the monthly report of cases of which he certifies as to their correctness. As a judge he is personally
responsible for the proper discharge of his functions (The Phil. Trial Lawyer's Asso. Inc. vs. Agana, Sr., 102 SCRA
517). In Nidera vs. Lazaro, 174 SCRA 581, it was held that "A judge cannot take refuge behind the inefficiency or
mismanagement of his court personnel."
On the part of respondent Baroy, she puts the blame of the falsification of the monthly report of cases on
complainant Sambo whom she allegedly assigned to prepare not only the monthly report of cases, but the
preparation and custody of marriage contracts, notarized documents and the notarial register. By her own admission
she has assigned to complainant Sambo duties she was supposed to perform, yet according to her she never
bother(ed) to check the notarial register of the court to find out the number of documents notarized in a month (TSN,
p. 30; 11-23-93).
Assuming that respondent Baroy assigned the preparation of the monthly report of cases to Sambo, which was
denied by the latter as he claims that he only typed the monthly report based on the data given to him by her, still it
is her duty to verify and check whether the report is correct.
The explanation of respondent Baroy that Sambo was the one in custody of marriage contracts, notarized
documents and notarial register, among other things, is not acceptable not only because as clerk of court she was
supposed to be in custody, control and supervision of all court records including documents and other properties of
the court (p. 32, Manual for Clerks of Court), but she herself admitted that from January, 1992 she was already in full
control of all the records of the court including receipts (TSN, p. 11; 11-23-93).
The evidence adduced in this cases in connection with the charge of falsification, however, also shows that
respondent Baroy did not account for what happened to the notarial fees received for those documents notarized
during the month of July and September, 1992. The evidence adduced in this case also sufficiently show that she
received cash bond deposits and she did not deposit them to a bank or to the Municipal Treasurer; and that she only
issued temporary receipts for said cash bond deposits.
For July, 1992 there were only six (6) documents reported to have been notarized by Judge Palaypayon although
the documents notarized for said month were actually one hundred thirteen (113) as recorded in the notarial register.
For September, 1992, there were only five (5) documents reported as notarized for that month, though the notarial
register show(s) that there were fifty-six (56) documents actually notarized. The fee for each document notarized as
appearing in the notarial register was P18.50. Respondent Baroy and Sambo declared that what was actually being
charged was P20.00. Respondent Baroy declared that P18.50 went to the Supreme Court and P1.50 was being
turned over to the Municipal Treasurer.
Baroy, however, did not present any evidence to show that she really sent to the Supreme Court the notarial fees of
P18.50 for each document notarized and to the Municipal Treasurer the additional notarial fee of P1.50. This should
be fully accounted for considering that Baroy herself declared that some notarial fees were allowed by her at her
own discretion to be paid later. Similarly, the solemnization fees have not been accounted for by Baroy considering
that she admitted that even (i)n those instances where the marriages were not solemnized due to lack of marriage
license the solemnization fees were not returned anymore, unless the contracting parties made a demand for their
return. Judge Palaypayon declared that he did not know of any instance when solemnization fee was returned when
the marriage was not solemnized due to lack of marriage license.
Respondent Baroy also claims that Ramon Sambo did not turn over to her some of the notarial fees. This is difficult
to believe. It was not only because Sambo vehemently denied it, but the minutes of the conference of the personnel
of the MTC of Tinambac dated January 20, 1992 shows that on that date Baroy informed the personnel of the court

that she was taking over the functions she assigned to Sambo, particularly the collection of legal fees (Exh. 7). The
notarial fees she claims that Sambo did not turn over to her were for those documents notarized (i)n July and
September, 1992 already. Besides there never was any demand she made for Sambo to turn over some notarial
fees supposedly in his possession. Neither was there any memorandum she issued on this matter, in spite of the
fact that she has been holding meetings and issuing memoranda to the personnel of the court (Exhs. V, W, FF, FF-1,
FF-2, FF-3; Exhs. 4-A (supplement(s), 5-8, 6-S, 7-S and 8-S).
It is admitted by respondent Baroy that on October 29, 1991 a cash bond deposit of a certain Dacara in the amount
of One Thousand (P1,000.00) Pesos was turned over to her after she assumed office and for this cash bond she
issued only a temporary receipt (Exh. Y). She did not deposit this cash bond in any bank or to the Municipal
Treasurer. She just kept it in her own cash box on the alleged ground that the parties in that case where the cash
bond was deposited informed her that they would settle the case amicably.
Respondent Baroy declared that she finally deposited the aforementioned cash bond of One Thousand (P1,000.00)
Pesos with the Land Bank of the Philippines (LBP) in February, 1993, after this administrative case was already filed
(TSN, pp. 27-28; 12-22-93). The Pass Book, however, shows that actually Baroy opened an account with the LBP,
Naga Branch, only on March 26, 1993 when she deposited an amount of Two Thousand (P2,000.00) Pesos (Exhs. 8
to 8-1-a). She claims that One Thousand (P1,000.000) Pesos of the initial deposit was the cash bond of Dacara. If it
were true, it was only after keeping to herself the cash bond of One Thousand (P1,000.00) Pesos for around one
year and five months when she finally deposited it because of the filing of this case.
On April 29, 1993, or only one month and two days after she finally deposited the One Thousand (P1,000.00) Pesos
cash bond of Dacara, she withdrew it from the bank without any authority or order from the court. It was only on July
23, 1993, or after almost three (3) months after she withdrew it, when she redeposited said cash bond (TSN, p. 6; 14-94).
The evidence presented in this case also show that on February 28, 1993 respondent Baroy received also a cash
bond of Three Thousand (P3,000.00) Pesos from a certain Alfredo Seprones in Crim. Case No. 5180. For this cash
bond deposit, respondent Baroy issued only an annumbered temporary receipt (Exh. X and X-1). Again Baroy just
kept this Three Thousand (P3,000.00) Pesos cash bond to herself. She did not deposit it either (in) a bank or (with)
the Municipal Treasurer. Her explanation was that the parties in Crim. Case No. 5180 informed her that they would
settle the case amicably. It was on April 26, 1993, or almost two months later when Judge Palaypayon issued an
order for the release of said cash bond (Exh. 7).
Respondent Baroy also admitted that since she assumed office on October 21, 1991 she used to issue temporary
receipt only for cash bond deposits and other payments and collections she received. She further admitted that
some of these temporary receipts she issued she failed to place the number of the receipts such as that receipt
marked Exhibit X (TSN, p. 35; 11-23-93). Baroy claims that she did not know that she had to use the official receipts
of the Supreme Court. It was only from February, 1993, after this case was already filed, when she only started
issuing official receipts.
The next charge against the respondents is that in order to be appointed Clerk of Court, Baroy gave Judge
Palaypayon an air conditioner as a gift. The evidence adduced with respect to this charge, show that on August 24,
1991 Baroy bought an air conditioner for the sum of Seventeen Thousand Six Hundred (P17,600.00) Pesos (Exhs. I
and I-1). The same was paid partly in cash and in check (Exhs. I-2 and I-3). When the air conditioner was brought to
court in order to be installed in the chamber of Judge Palaypayon, it was still placed in the same box when it was
bought and was not used yet.
The respondents claim that Baroy sold it to Judge Palaypayon for Twenty Thousand (P20,00.00) Pesos on
installment basis with a down payment of Five Thousand (P5,000.00) Pesos and as proof thereof the respondents
presented a typewritten receipt dated May 29, 1993 (Exh. 22). The receipt was signed by both respondents and by
the Municipal Mayor of Tinambac, Camarines Sur and another person as witness.
The alleged sale between respondents is not beyond suspicion. It was bought by Baroy at a time when she was
applying for the vacant position of Clerk of Court (to) which she was eventually appointed in October, 1991. From
the time she bought the air conditioner on August 24, 1991 until it was installed in the office of Judge Palaypayon it
was not used yet. The sale to Judge Palaypayon was only evidenced by a mere typewritten receipt dated May 29,
1992 when this case was already filed. The receipt could have been easily prepared. The Municipal Mayor of
Tinambac who signed in the receipt as a witness did not testify in this case. The sale is between the Clerk of Court
and the Judge of the same court. All these circumstances give rise to suspicion of at least impropriety. Judges
should avoid such action as would subject (them) to suspicion and (their) conduct should be free from the
appearance of impropriety (Jaagueta vs. Boncasos, 60 SCRA 27).
With respect to the charge that Judge Palaypayon received a cash bond deposit of One Thousand (P1,000.00)
Pesos from Januaria Dacara without issuing a receipt, Dacara executed an affidavit regarding this charge that Judge
Palaypayon did not give her a receipt for the P1,000.00 cash bond she deposited (Exh. N). Her affidavit, however,
has no probative value as she did not show that this cash bond of P1,000.00 found its way into the hands of
respondent Baroy who issued only a temporary receipt for it and this has been discussed earlier.
Another charge against Judge Palaypayon is the getting of detention prisoners to work in his house and one of them
escaped while in his custody and was never found again. To hide this fact, the case against said accused was
ordered archived by Judge Palaypayon. The evidence adduced with respect to this particular charge, show that in
Crim. Case No. 5647 entitled People vs. Stephen Kalaw, Alex Alano and Allan Adupe, accused Alex Alano and Allan
Adupe were arrested on April 12, 1991 and placed in the municipal jail of Tinambac, Camarines Sur (Exhs. 0, 0-1, 02 and 0-3; Exh. 25). The evidence presented that Alex Alano was taken by Judge Palaypayon from the municipal jail
where said accused was confined and that he escaped while in custody of Judge Palaypayon is solely testimonial,
particularly that of David Ortiz, a former utility worker of the MTC of Tinambac.
Herein investigator finds said evidence not sufficient. The complainants should have presented records from the
police of Tinambac to show that Judge Palaypayon took out from the municipal jail Alex Alano where he was under
detention and said accused escaped while in the custody of Judge Palaypayon.
The order, however, of Judge Palaypayon dated April 6, 1992 in Crim. Case No. 5047 archiving said case appears
to be without basis. The order states: "this case was filed on April 12, 1991 and the records show that the warrant of
arrest (was) issued against the accused, but up to this moment there is no return of service for the warrant of arrest
issued against said accused" (Exh. 0-4). The records of said case, however, show that in fact there was a return of
the service of the warrant of arrest dated April 12, 1991 showing that Alano and Adupe were arrested (Exh. 0-3).
Judge Palaypayon explained that his order dated April 6, 1992 archiving Crim. Case No. 5047 referred only to one of
the accused who remained at large. The explanation cannot be accepted because the two other accused, Alano and
Adupe, were arrested. Judge Palaypayon should have issued an order for the arrest of Adupe who allegedly jumped
bail, but Alano was supposed to be confined in the municipal jail if his claim is true that he did not take custody of
Alano.

The explanation also of Judge Palaypayon why he ordered the case archived was because he heard from the police
that Alano escaped. This explanation is not acceptable either. He should ha(ve) set the case and if the police failed
to bring to court Alano, the former should have been required to explain in writing why Alano was not brought to
court. If the explanation was that Alano escaped from jail, he should have issued an order for his arrest. It is only
later on when he could not be arrested when the case should have been ordered archived. The order archiving this
case for the reason that he only heard that Alano escaped is another circumstance which gave rise to a suspicion
that Alano might have really escaped while in his custody only that the complainants could not present records or
other documentary evidence to prove the same.
The last charge against the respondents is that they collected filing fees on collection cases filed by the Rural Bank
of Tinambac, Camarines Sur which was supposed to be exempted in paying filing fees under existing laws and that
the filing fees received was deposited by respondent Baroy in her personal account in the bank. The evidence
presented show that on February 4, 1992 the Rural Bank of Tinambac filed ten (10) civil cases for collection against
farmers and it paid the total amount of Four Hundred (P400.00) Pesos representing filing fees. The complainants
cited Section 14 of Republic Act 720, as amended, which exempts Rural Banks (from) the payment of filing fees on
collection of sums of money cases filed against farmers on loans they obtained.
Judge Palaypayon, however, had nothing to do with the payment of the filing fees of the Rural Bank of Tinambac as
it was respondent Baroy who received them and besides, on February 4, 1992, he was on sick leave. On her part
Baroy claims that the bank paid voluntarily the filing fees. The records, however, shows that respondent Baroy sent
a letter to the manager of the bank dated January 28, 1992 to the effect that if the bank would not pay she would
submit all Rural Bank cases for dismissal (Annex 6, comment by respondent Baroy).
Respondent Baroy should have checked whether the Rural Bank of Tinambac was really exempt from the payment
of filing fees pursuant to Republic Act 720, as amended, instead of threatening the bank to have its cases be
submitted to the court in order to have them dismissed. Here the payment of the filing fees was made on February 4,
1992, but the Four Hundred (P400.00) Pesos was only turned over to the Municipal Treasurer on March 12, 1992.
Here, there is an undue delay again in complying with her obligation as accountable officer.
In view of the foregoing findings that the evidence presented by the complainants sufficiently show that respondent
Judge Lucio P. Palaypayon, Jr. had solemnized marriages, particularly that of Sammy Bocaya and Gina Besmonte,
without a marriage license, and that it having been shown that he did not comply with his duty in closely supervising
his clerk of court in the preparation of the monthly report of cases being submitted to the Supreme Court, particularly
for the months of July and September, 1992 where it has been proven that the reports for said two (2) months were
falsified with respect to the number of documents notarized, it is respectfully recommended that he be imposed a
fine of TEN THOUSAND (P10,000.00) PESOS with a warning that the same or similar offenses will be more
severely dealt with.
The fact that Judge Palaypayon did not sign the marriage contracts or certificates of those marriages he solemnized
without a marriage license, there were no dates placed in the marriage contracts to show when they were
solemnized, the contracting parties were not furnished their marriage contracts and the Local Civil Registrar was not
being sent any copy of the marriage contract, will not absolve him from liability. By solemnizing alone a marriage
without a marriage license he as the solemnizing officer is the one responsible for the irregularity in not complying
(with) the formal requ(i)sites of marriage and under Article 4(3) of the Family Code of the Philippines, he shall be
civilly, criminally and administratively liable.
Judge Palaypayon is likewise liable for his negligence or failure to comply with his duty of closely supervising his
clerk of court in the performance of the latter's duties and functions, particularly the preparation of the monthly report
of cases (Bendesula vs. Laya, 58 SCRA 16). His explanation that he only signed the monthly report of cases only
when his clerk of court already signed the same, cannot be accepted. It is his duty to closely supervise her, to check
and verify the records if the monthly reports prepared by his clerk of court do not contain false statements. It was
held that "A judge cannot take refuge behind the inefficiency or incompetence of court personnel (Nidua vs. Lazaro,
174 SCRA 158).
In view also of the foregoing finding that respondent Nelia Esmeralda-Baroy, the clerk of court of the Municipal Trial
Court of Tinambac, Camarines Sur, has been found to have falsified the monthly report of cases for the months of
July and September, 1992 with respect to the number of documents notarized, for having failed to account (for) the
notarial fees she received for said two (2) months period; for having failed to account (for) the solemnization fees of
those marriages allegedly not solemnized, but the solemnization fees were not returned; for unauthorized issuance
of temporary receipts, some of which were issued unnumbered; for receiving the cash bond of Dacara on October
29, 1991 in the amount of One Thousand (P1,000.00) Pesos for which she issued only a temporary receipt (Exh. Y)
and for depositing it with the Land Bank of the Philippines only on March 26, 1993, or after one year and five months
in her possession and after this case was already filed; for withdrawing said cash bond of One Thousand
(P1,000.00) Pesos on April 29, 1993 without any court order or authority and redepositing it only on July 23, 1993;
for receiving a cash bond of Three Thousand (P3,000.00) Pesos from Alfredo Seprones in Crim. Case No. 5180,
MTC, Tinambac, Camarines Sur, for which she issued only an unnumbered temporary receipt (Exhs. X and X-1) and
for not depositing it with a bank or with the Municipal Treasurer until it was ordered released; and for requiring the
Rural Bank of Tinambac, Camarines Sur to pay filing fees on February 4, 1992 for collection cases filed against
farmers in the amount of Four Hundred (P400.00) Pesos, but turning over said amount to the Municipal Treasurer
only on March 12, 1992, it is respectfully recommended that said respondent clerk of court Nelia Esmeralda-Baroy
be dismissed from the service.
It is provided that "Withdrawal of court deposits shall be by the clerk of court who shall issue official receipt to the
provincial, city or municipal treasurer for the amount withdrawn. Court deposits cannot be withdrawn except by order
of the court, . . . ." (Revised Manual of Instructions for Treasurers, Sec. 183, 184 and 626; p. 127, Manual for Clerks
of Court). A circular also provides that the Clerks of Court shall immediately issue an official receipt upon receipt of
deposits from party litigants and thereafter deposit intact the collection with the municipal, city or provincial treasurer
and their deposits, can only be withdrawn upon proper receipt and order of the Court (DOJ Circular No. 52, 26 April
1968; p. 136, Manual for Clerks of Court). Supreme Court Memorandum Circular No. 5, 25 November 1982, also
provides that "all collections of funds of fiduciary character including rental deposits, shall be deposited immediately
by the clerk of court concerned upon receipt thereof with City, Municipal or Provincial Treasurer where his court is
located" and that "no withdrawal of any of such deposits shall be made except upon lawful order of the court
exercising jurisdiction over the subject matter.
Respondent Baroy had either failed to comply with the foregoing circulars, or deliberately disregarded, or even
intentionally violated them. By her conduct, she demonstrated her callous unconcern for the obligations and
responsibility of her duties and functions as a clerk of court and accountable officer. The gross neglect of her duties
shown by her constitute(s) a serious misconduct which warrant(s) her removal from office. In the case of Belen P.
Ferriola vs. Norma Hiam, Clerk of Court, MTCC, Branch I, Batangas City; A.M. No. P-90-414; August 9, 1993, it was
held that "The clerk of court is not authorized to keep funds in his/her custody; monies received by him/her shall be
deposited immediately upon receipt thereof with the City, Municipal or Provincial Treasurer. Supreme Court Circular
Nos. 5 dated November 25, 1982 and 5-A dated December 3, 1982. Respondent Hiam's failure to remit the cash
bail bonds and fine she collected constitutes serious misconduct and her misappropriation of said funds constitutes
dishonesty. "Respondent Norma Hiam was found guilty of dishonesty and serious misconduct prejudicial to the best
interest of the service and (the Court) ordered her immediate dismissal (from) the service.

xxx xxx xxx


We here emphasize once again our adjuration that the conduct and behavior of everyone connected with an office charged with
the dispensation of justice, from the presiding judge to the lowliest clerk, should be circumscribed with the heavy burden of
responsibility. His conduct, at all times, must not only be characterized by propriety and decorum but, above all else, must be
beyond suspicion. Every employee should be an example of integrity, uprightness and honesty. 5 Integrity in a judicial office is
more than a virtue, it is a necessity. 6 It applies, without qualification as to rank or position, from the judge to the least of its
personnel, they being standard-bearers of the exacting norms of ethics and morality imposed upon a Court of justice.
On the charge regarding illegal marriages the Family Code pertinently provides that the formal requisites of marriage are, inter
alia, a valid marriage license except in the cases provided for therein. 7 Complementarily, it declares that the absence of any of
the essential or formal requisites shall generally render the marriage void ab initio and that, while an irregularity in the formal
requisites shall not affect the validity of the marriage, the party or parties responsible for the irregularity shall be civilly, criminally
and administratively liable. 8
The civil aspect is addressed to the contracting parties and those affected by the illegal marriages, and what we are providing
for herein pertains to the administrative liability of respondents, all without prejudice to their criminal responsibility. The Revised
Penal Code provides that "(p)riests or ministers of any religious denomination or sect, or civil authorities who shall perform or
authorize any illegal marriage ceremony shall be punished in accordance with the provisions of the Marriage Law." 9 This is of
course, within the province of the prosecutorial agencies of the Government.
The recommendation with respect to the administrative sanction to be imposed on respondent judge should, therefore, be
modified. For one, with respect to the charge of illegal solemnization of marriages, it does appear that he had not taken to heart,
but actually trifled with, the law's concern for the institution of marriage and the legal effects flowing from civil status. This, and
his undeniable participation in the other offenses charged as hereinbefore narrated in detail, approximate such serious degree
of misconduct and of gross negligence in the performance of judicial duties as to ineludibly require a higher penalty.
WHEREFORE, the Court hereby imposes a FINE of P20,000.00 on respondent Judge Lucio P. Palaypayon. Jr., with a stern
warning that any repetition of the same or similar offenses in the future will definitely be severely dealt with. Respondent Nelia
Esmeralda-Baroy is hereby DISMISSED from the service, with forfeiture of all retirement benefits and with prejudice to
employment in any branch, agency or instrumentality of the Government, including government-owned or controlled
corporations.
Let copies of this decision be spread on their records and furnished to the Office of the Ombudsman for appropriate action.
SO ORDERED.

G.R. No. L-57062 January 24, 1992

MARIA DEL ROSARIO MARIATEGUI, ET AL., petitioners,


vs.
HON. COURT OF APPEALS, JACINTO MARIATEGUI, JULIAN MARIATEGUI and PAULINA MARIATEGUI,respondents.
Montesa, Albon & Associates for petitioners.
Parmenio B. Patacsil, Patacsil Twins Law Office for the heirs of the late Maria del Rosario Mariategui.
Tinga, Fuentes & Tagle Firm for private respondents.

BIDIN, J.:
This is a petition for review on certiorari of the decision * of the Court of Appeals dated December 24, 1980 in CA-G.R. No.
61841, entitled "Jacinto Mariategui, et al. v. Maria del Rosario Mariategui, et al.," reversing the judgment of the then Court of
First Instance of Rizal, Branch VIII ** at Pasig, Metro Manila.
The undisputed facts are as follows:
Lupo Mariategui died without a will on June 26, 1953 (Brief for respondents, Rollo, pp. 116; 8). During his lifetime, Lupo
Mariategui contracted three (3) marriages. With his first wife, Eusebia Montellano, who died on November 8, 1904, he begot
four (4) children, namely: Baldomera, Maria del Rosario, Urbana and Ireneo. Baldomera died and was survived by her children
named Antero, Rufina, Catalino, Maria, Gerardo, Virginia and Federico, all surnamed Espina. Ireneo also died and left a son
named Ruperto. With his second wife, Flaviana Montellano, he begot a daughter named Cresenciana who was born on May 8,
1910 (Rollo, Annex "A", p. 36).
Lupo Mariategui and Felipa Velasco (Lupo's third wife) got married sometime in 1930. They had three children, namely: Jacinto,
born on July 3, 1929, Julian, born on February 16, 1931 and Paulina, born on April 19, 1938. Felipa Velasco Mariategui died in
1941 (Rollo, Ibid).
At the time of his death, Lupo Mariategui left certain properties which he acquired when he was still unmarried (Brief for
respondents, Rollo, pp. 116; 4). These properties are described in the complaint as Lots Nos. 163, 66, 1346 and 156 of the
Muntinglupa Estate (Rollo, Annex "A", p. 39).
On December 2, 1967, Lupo's descendants by his first and second marriages, namely, Maria del Rosario, Urbana, Ruperto,
Cresencia, all surnamed Mariategui and Antero, Rufina, Catalino, Maria, Gerardo, Virginia and Federico, all surnamed Espina,
executed a deed of extrajudicial partition whereby they adjudicated unto themselves Lot No. 163 of the Muntinglupa Estate.
Thereafter, Lot No. 163 was the subject of a voluntary registration proceedings filed by the adjudicatees under Act No. 496, and
the land registration court issued a decree ordering the registration of the lot. Thus, on April 1, 1971, OCT No. 8828 was issued
in the name of the above-mentioned heirs. Subsequently, the registered owners caused the subdivision of the said lot into Lots
Nos. 163-A to 163-H, for which separate transfer certificates of title were issued to the respective parties (Rollo, ibid).
On April 23, 1973, Lupo's children by his third marriage with Felipa Velasco (Jacinto, Julian and Paulina) filed with the lower
court an amended complaint claiming that Lot No. 163 together with Lots Nos. 669, 1346 and 154 were owned by their common
father, Lupo Mariategui, and that, with the adjudication of Lot No. 163 to their co-heirs, they (children of the third marriage) were
deprived of their respective shares in the lots. Plaintiffs pray for partition of the estate of their deceased father and annulment of
the deed of extrajudicial partition dated December 2, 1967 (Petition, Rollo, p. 10). Cresencia Mariategui Abas, Flaviana
Mariategui Cabrera and Isabel Santos were impleaded in the complaint as unwilling defendants as they would not like to join the
suit as plaintiffs although they acknowledged the status and rights of the plaintiffs and agreed to the partition of the parcels of
land as well as the accounting of their fruits (Ibid., Rollo, p. 8; Record on Appeal, p. 4).
The defendants (now petitioners) filed an answer with counterclaim (Amended Record on Appeal, p. 13). Thereafter, they filed a
motion to dismiss on the grounds of lack of cause of action and prescription. They specifically contended that the complaint was
one for recognition of natural children. On August 14, 1974, the motion to dismiss was denied by the trial court, in an order the
dispositive portion of which reads:
It is therefore the opinion of the Court that Articles 278 and 285 of the Civil Code cited by counsel for the defendants
are of erroneous application to this case. The motion to dismiss is therefore denied for lack of merit.
SO ORDERED. (Ibid, p. 37).
However, on February 16, 1977, the complaint as well as petitioners' counterclaim were dismissed by the trial court, in its
decision stating thus:
The plaintiffs' right to inherit depends upon the acknowledgment or recognition of their continuous enjoyment and
possession of status of children of their supposed father. The evidence fails to sustain either premise, and it is clear
that this action cannot be sustained. (Ibid, Rollo, pp. 67-68)
The plaintiffs elevated the case to the Court of Appeals on the ground that the trial court committed an error ". . . in not finding
that the parents of the appellants, Lupo Mariategui and Felipa Velasco (were) lawfully married, and in holding (that) they
(appellants) are not legitimate children of their said parents, thereby divesting them of their inheritance . . . " (Rollo, pp. 14-15).
On December 24, 1980, the Court of Appeals rendered a decision declaring all the children and descendants of Lupo
Mariategui, including appellants Jacinto, Julian and Paulina (children of the third marriage) as entitled to equal shares in the
estate of Lupo Mariategui; directing the adjudicatees in the extrajudicial partition of real properties who eventually acquired
transfer certificates of title thereto, to execute deeds of reconveyance in favor, and for the shares, of Jacinto, Julian and Paulina
provided rights of innocent third persons are not prejudiced otherwise the said adjudicatees shall reimburse the said heirs the
fair market value of their shares; and directing all the parties to submit to the lower court a project of partition in the net estate of
Lupo Mariategui after payment of taxes, other government charges and outstanding legal obligations.
The defendants-appellees filed a motion for reconsideration of said decision but it was denied for lack of merit. Hence, this
petition which was given due course by the court on December 7, 1981.
The petitioners submit to the Court the following issues: (a) whether or not prescription barred private respondents' right to
demand the partition of the estate of Lupo Mariategui, and (b) whether or not the private respondents, who belatedly filed the
action for recognition, were able to prove their successional rights over said estate. The resolution of these issues hinges,
however, on the resolution of the preliminary matter, i.e., the nature of the complaint filed by the private respondents.
The complaint alleged, among other things, that "plaintiffs are the children of the deceased spouses Lupo Mariategui . . . and
Felipa Velasco"; that "during his lifetime, Lupo Mariategui had repeatedly acknowledged and confirmed plaintiffs as his children
and the latter, in turn, have continuously enjoyed such status since their birth"; and "on the basis of their relationship to the
deceased Lupo Mariategui and in accordance with the law on intestate succession, plaintiffs are entitled to inherit shares in the
foregoing estate (Record on Appeal, pp. 5 & 6). It prayed, among others, that plaintiffs be declared as children and heirs of Lupo
Mariategui and adjudication in favor of plaintiffs their lawful shares in the estate of the decedent (Ibid, p. 10).

A perusal of the entire allegations of the complaint, however, shows that the action is principally one of partition. The allegation
with respect to the status of the private respondents was raised only collaterally to assert their rights in the estate of the
deceased. Hence, the Court of Appeals correctly adopted the settled rule that the nature of an action filed in court is determined
by the facts alleged in the complaint constituting the cause of action (Republic vs. Estenzo, 158 SCRA 282 [1988]).
It has been held that, if the relief demanded is not the proper one which may be granted under the law, it does not characterize
or determine the nature of plaintiffs' action, and the relief to which plaintiff is entitled based on the facts alleged by him in his
complaint, although it is not the relief demanded, is what determines the nature of the action (1 Moran, p. 127, 1979 ed., citing
Baguioro vs. Barrios, et al., 77 Phil. 120).
With respect to the legal basis of private respondents' demand for partition of the estate of Lupo Mariategui, the Court of
Appeals aptly held that the private respondents are legitimate children of the deceased.
Lupo Mariategui and Felipa Velasco were alleged to have been lawfully married in or about 1930. This fact is based on the
declaration communicated by Lupo Mariategui to Jacinto who testified that "when (his) father was still living, he was able to
mention to (him) that he and (his) mother were able to get married before a Justice of the Peace of Taguig, Rizal." The spouses
deported themselves as husband and wife, and were known in the community to be such. Although no marriage certificate was
introduced to this effect, no evidence was likewise offered to controvert these facts. Moreover, the mere fact that no record of
the marriage exists does not invalidate the marriage, provided all requisites for its validity are present (People vs. Borromeo,
133 SCRA 106 [1984]).
Under these circumstances, a marriage may be presumed to have taken place between Lupo and Felipa. The laws presume
that a man and a woman, deporting themselves as husband and wife, have entered into a lawful contract of marriage; that a
child born in lawful wedlock, there being no divorce, absolute or from bed and board is legitimate; and that things have
happened according to the ordinary course of nature and the ordinary habits of life (Section 5 (z), (bb), (cc), Rule 131, Rules of
Court; Corpus v. Corpus, 85 SCRA 567 [1978]; Saurnaba v. Workmen's Compensation, 85 SCRA 502 [1978]; Alavado v. City
Gov't. of Tacloban, 139 SCRA 230 [1985]; Reyes v. Court of Appeals, 135 SCRA 439 [1985]).
Courts look upon the presumption of marriage with great favor as it is founded on the following rationale:
The basis of human society throughout the civilized world is that of marriage. Marriage in this jurisdiction is not only
a civil contract, but it is a new relation, an institution in the maintenance of which the public is deeply interested.
Consequently, every intendment of the law leans toward legalizing matrimony. Persons dwelling together in apparent
matrimony are presumed, in the absence of any counterpresumption or evidence special to that case, to be in fact
married. The reason is that such is the common order of society and if the parties were not what they thus hold
themselves out as being, they would be living in the constant violation of decency and of
law . . . (Adong vs. Cheong Seng Gee, 43 Phil. 43, 56 [1922] quoted in Alavado vs. City Government of Tacloban,
139 SCRA 230 [1985]).
So much so that once a man and a woman have lived as husband and wife and such relationship is not denied nor
contradicted, the presumption of their being married must be admitted as a fact (Alavado v. City Gov't. of Tacloban, supra).
The Civil Code provides for the manner under which legitimate filiation may be proven. However, considering the effectivity of
the Family Code of the Philippines, the case at bar must be decided under a new if not entirely dissimilar set of rules because
the parties have been overtaken by events, to use the popular phrase (Uyguangco vs. Court of Appeals, G.R. No. 76873,
October 26, 1989). Thus, under Title VI of the Family Code, there are only two classes of children legitimate and illegitimate.
The fine distinctions among various types of illegitimate children have been eliminated (Castro vs. Court of Appeals, 173 SCRA
656 [1989]).
Article 172 of the said Code provides that the filiation of legitimate children may be established by the record of birth appearing
in the civil register or a final judgment or by the open and continuous possession of the status of a legitimate child.
Evidence on record proves the legitimate filiation of the private respondents. Jacinto's birth certificate is a record of birth referred
to in the said article. Again, no evidence which tends to disprove facts contained therein was adduced before the lower court. In
the case of the two other private respondents, Julian and Paulina, they may not have presented in evidence any of the
documents required by Article 172 but they continuously enjoyed the status of children of Lupo Mariategui in the same manner
as their brother Jacinto.
While the trial court found Jacinto's testimonies to be inconsequential and lacking in substance as to certain dates and names of
relatives with whom their family resided, these are but minor details. The nagging fact is that for a considerable length of time
and despite the death of Felipa in 1941, the private respondents and Lupo lived together until Lupo's death in 1953. It should be
noted that even the trial court mentioned in its decision the admission made in the affidavit of Cresenciana Mariategui Abas, one
of the petitioners herein, that " . . . Jacinto, Julian and Paulina Mariategui ay pawang mga kapatid ko sa
ama . . ." (Exh. M, Record on Appeal, pp. 65-66).
In view of the foregoing, there can be no other conclusion than that private respondents are legitimate children and heirs of
Lupo Mariategui and therefore, the time limitation prescribed in Article 285 for filing an action for recognition is inapplicable to
this case. Corollarily, prescription does not run against private respondents with respect to the filing of the action for partition so
long as the heirs for whose benefit prescription is invoked, have not expressly or impliedly repudiated the co-ownership. In other
words, prescription of an action for partition does not lie except when the co-ownership is properly repudiated by the co-owner
(Del Banco vs. Intermediate Appellate Court, 156 SCRA 55 [1987] citing Jardin vs. Hollasco, 117 SCRA 532 [1982]).
Otherwise stated, a co-owner cannot acquire by prescription the share of the other co-owners absent a clear repudiation of coownership duly communicated to the other co-owners (Mariano vs. De Vega, 148 SCRA 342 [1987]). Furthermore, an action to
demand partition is imprescriptible and cannot be barred by laches (Del Banco vs. IAC, 156 SCRA 55 [1987]). On the other
hand, an action for partition may be seen to be at once an action for declaration of co-ownership and for segregation and
conveyance of a determinate portion of the property involved (Roque vs. IAC, 165 SCRA 118 [1988]).
Petitioners contend that they have repudiated the co-ownership when they executed the extrajudicial partition excluding the
private respondents and registered the properties in their own names (Petition, p. 16; Rollo, p. 20). However, no valid
repudiation was made by petitioners to the prejudice of private respondents. Assuming petitioners' registration of the subject lot
in 1971 was an act of repudiation of the co-ownership, prescription had not yet set in when private respondents filed in 1973 the
present action for partition (Ceniza vs. C.A., 181 SCRA 552 [1990]).
In their complaint, private respondents averred that in spite of their demands, petitioners, except the unwilling defendants in the
lower court, failed and refused to acknowledge and convey their lawful shares in the estate of their father (Record on Appeal, p.
6). This allegation, though denied by the petitioners in their answer (Ibid, p. 14), was never successfully refuted by them. Put
differently, in spite of petitioners' undisputed knowledge of their relationship to private respondents who are therefore their coheirs, petitioners fraudulently withheld private respondent's share in the estate of Lupo Mariategui. According to respondent
Jacinto, since 1962, he had been inquiring from petitioner Maria del Rosario about their (respondents) share in the property left
by their deceased father and had been assured by the latter (Maria del Rosario) not to worry because they will get some shares.
As a matter of fact, sometime in 1969, Jacinto constructed a house where he now resides on Lot No. 163 without any complaint
from petitioners.
Petitioners' registration of the properties in their names in 1971 did not operate as a valid repudiation of the co-ownership.
In Adille vs. Court of Appeals (157 SCRA 455, 461-462 [1988]), the Court held:

Prescription, as a mode of terminating a relation of co-ownership, must have been preceded by repudiation (of the
co-ownership). The act of repudiation, in turn, is subject to certain conditions: (1) a co-owner repudiates the coownership; (2) such an act of repudiation is clearly made known to the other co-owners; (3) the evidence thereon is
clear and conclusive; and (4) he has been in possession through open, continuous, exclusive, and notorious
possession of the property for the period required by law.
xxx xxx xxx
It is true that registration under the Torrens system is constructive notice of title, but it has likewise been our holding
that the Torrens title does not furnish shield for fraud. It is therefore no argument to say that the act of registration is
equivalent to notice of repudiation, assuming there was one, notwithstanding the long-standing rule that registration
operates as a universal notice of title.
Inasmuch as petitioners registered the properties in their names in fraud of their co-heirs prescription can only be deemed to
have commenced from the time private respondents discovered the petitioners' act of defraudation (Adille vs. Court of
Appeals, supra). Hence, prescription definitely may not be invoked by petitioners because private respondents commenced the
instant action barely two months after learning that petitioners had registered in their names the lots involved.
WHEREFORE, the petition is DENIED and the assailed decision of the Court of Appeals dated December 24, 1980 is Affirmed.
SO ORDERED.

G.R. No. 175581

March 28, 2008

REPUBLIC OF THE PHILIPPINES, Petitioner,


vs.
JOSE A. DAYOT, Respondent.
x - - - - - - - - - - - - - - - - - - - - - - -x
G.R. No. 179474
FELISA TECSON-DAYOT, Petitioner,
vs.
JOSE A. DAYOT, Respondent.
DECISION
CHICO-NAZARIO, J.:
Before us are two consolidated petitions. G.R. No. 175581 and G.R. No. 179474 are Petitions for Review under Rule 45 of the
Rules of Court filed by the Republic of the Philippines and Felisa Tecson-Dayot (Felisa), respectively, both challenging the
Amended Decision1 of the Court of Appeals, dated 7 November 2006, in CA-G.R. CV No. 68759, which declared the marriage
between Jose Dayot (Jose) and Felisa void ab initio.
The records disclose that on 24 November 1986, Jose and Felisa were married at the Pasay City Hall. The marriage was
solemnized by Rev. Tomas V. Atienza.2 In lieu of a marriage license, Jose and Felisa executed a sworn affidavit, 3 also dated 24
November 1986, attesting that both of them had attained the age of maturity, and that being unmarried, they had lived together
as husband and wife for at least five years.
On 7 July 1993, Jose filed a Complaint4 for Annulment and/or Declaration of Nullity of Marriage with the Regional Trial Court
(RTC), Bian, Laguna, Branch 25. He contended that his marriage with Felisa was a sham, as no marriage ceremony was
celebrated between the parties; that he did not execute the sworn affidavit stating that he and Felisa had lived as husband and
wife for at least five years; and that his consent to the marriage was secured through fraud.
In his Complaint, Jose gave his version of the events which led to his filing of the same. According to Jose, he was introduced to
Felisa in 1986. Immediately thereafter, he came to live as a boarder in Felisas house, the latter being his landlady. Some three
weeks later, Felisa requested him to accompany her to the Pasay City Hall, ostensibly so she could claim a package sent to her
by her brother from Saudi Arabia. At the Pasay City Hall, upon a pre-arranged signal from Felisa, a man bearing three folded
pieces of paper approached them. They were told that Jose needed to sign the papers so that the package could be released to
Felisa. He initially refused to do so. However, Felisa cajoled him, and told him that his refusal could get both of them killed by
her brother who had learned about their relationship. Reluctantly, he signed the pieces of paper, and gave them to the man who
immediately left. It was in February 1987 when he discovered that he had contracted marriage with Felisa. He alleged that he
saw a piece of paper lying on top of the table at the sala of Felisas house. When he perused the same, he discovered that it
was a copy of his marriage contract with Felisa. When he confronted Felisa, the latter feigned ignorance.
In opposing the Complaint, Felisa denied Joses allegations and defended the validity of their marriage. She declared that they
had maintained their relationship as man and wife absent the legality of marriage in the early part of 1980, but that she had
deferred contracting marriage with him on account of their age difference. 5 In her pre-trial brief, Felisa expounded that while her
marriage to Jose was subsisting, the latter contracted marriage with a certain Rufina Pascual (Rufina) on 31 August 1990. On 3
June 1993, Felisa filed an action for bigamy against Jose. Subsequently, she filed an administrative complaint against Jose with
the Office of the Ombudsman, since Jose and Rufina were both employees of the National Statistics and Coordinating
Board.6 The Ombudsman found Jose administratively liable for disgraceful and immoral conduct, and meted out to him the
penalty of suspension from service for one year without emolument. 7
On 26 July 2000, the RTC rendered a Decision 8 dismissing the Complaint. It disposed:
WHEREFORE, after a careful evaluation and analysis of the evidence presented by both parties, this Court finds and so holds
that the [C]omplaint does not deserve a favorable consideration. Accordingly, the above-entitled case is hereby ordered
DISMISSED with costs against [Jose].9
The RTC ruled that from the testimonies and evidence presented, the marriage celebrated between Jose and Felisa on 24
November 1986 was valid. It dismissed Joses version of the story as implausible, and rationalized that:
Any person in his right frame of mind would easily suspect any attempt to make him or her sign a blank sheet of paper. [Jose]
could have already detected that something was amiss, unusual, as they were at Pasay City Hall to get a package for [Felisa]
but it [was] he who was made to sign the pieces of paper for the release of the said package. Another indirect suggestion that
could have put him on guard was the fact that, by his own admission, [Felisa] told him that her brother would kill them if he will
not sign the papers. And yet it took him, more or less, three months to "discover" that the pieces of paper that he signed was
[sic] purportedly the marriage contract. [Jose] does not seem to be that ignorant, as perceived by this Court, to be "taken in for a
ride" by [Felisa.]
[Joses] claim that he did not consent to the marriage was belied by the fact that he acknowledged Felisa Tecson as his wife
when he wrote [Felisas] name in the duly notarized statement of assets and liabilities he filled up on May 12, 1988, one year
after he discovered the marriage contract he is now claiming to be sham and false. [Jose], again, in his company I.D., wrote the
name of [Felisa] as the person to be contacted in case of emergency. This Court does not believe that the only reason why her
name was written in his company I.D. was because he was residing there then. This is just but a lame excuse because if he
really considers her not his lawfully wedded wife, he would have written instead the name of his sister.
When [Joses] sister was put into the witness stand, under oath, she testified that she signed her name voluntarily as a witness
to the marriage in the marriage certificate (T.S.N., page 25, November 29, 1996) and she further testified that the signature
appearing over the name of Jose Dayot was the signature of his [sic] brother that he voluntarily affixed in the marriage contract
(page 26 of T.S.N. taken on November 29, 1996), and when she was asked by the Honorable Court if indeed she believed that
Felisa Tecson was really chosen by her brother she answered yes. The testimony of his sister all the more belied his claim that
his consent was procured through fraud.10
Moreover, on the matter of fraud, the RTC ruled that Joses action had prescribed. It cited Article 87 11 of the New Civil Code
which requires that the action for annulment of marriage must be commenced by the injured party within four years after the
discovery of the fraud. Thus:
That granting even for the sake of argument that his consent was obtained by [Felisa] through fraud, trickery and machinations,
he could have filed an annulment or declaration of nullity of marriage at the earliest possible opportunity, the time when he
discovered the alleged sham and false marriage contract. [Jose] did not take any action to void the marriage at the earliest
instance. x x x.12
Undeterred, Jose filed an appeal from the foregoing RTC Decision to the Court of Appeals. In a Decision dated 11 August 2005,
the Court of Appeals found the appeal to be without merit. The dispositive portion of the appellate courts Decision reads:
WHEREFORE, the Decision appealed from is AFFIRMED. 13

The Court of Appeals applied the Civil Code to the marriage between Jose and Felisa as it was solemnized prior to the
effectivity of the Family Code. The appellate court observed that the circumstances constituting fraud as a ground for annulment
of marriage under Article 8614 of the Civil Code did not exist in the marriage between the parties. Further, it ruled that the action
for annulment of marriage on the ground of fraud was filed beyond the prescriptive period provided by law. The Court of Appeals
struck down Joses appeal in the following manner:
Nonetheless, even if we consider that fraud or intimidation was employed on Jose in giving his consent to the marriage, the
action for the annulment thereof had already prescribed. Article 87 (4) and (5) of the Civil Code provides that the action for
annulment of marriage on the ground that the consent of a party was obtained by fraud, force or intimidation must be
commenced by said party within four (4) years after the discovery of the fraud and within four (4) years from the time the force
or intimidation ceased. Inasmuch as the fraud was allegedly discovered by Jose in February, 1987 then he had only until
February, 1991 within which to file an action for annulment of marriage. However, it was only on July 7, 1993 that Jose filed the
complaint for annulment of his marriage to Felisa. 15
Likewise, the Court of Appeals did not accept Joses assertion that his marriage to Felisa was void ab initio for lack of a
marriage license. It ruled that the marriage was solemnized under Article 76 16 of the Civil Code as one of exceptional character,
with the parties executing an affidavit of marriage between man and woman who have lived together as husband and wife for at
least five years. The Court of Appeals concluded that the falsity in the affidavit to the effect that Jose and Felisa had lived
together as husband and wife for the period required by Article 76 did not affect the validity of the marriage, seeing that the
solemnizing officer was misled by the statements contained therein. In this manner, the Court of Appeals gave credence to the
good-faith reliance of the solemnizing officer over the falsity of the affidavit. The appellate court further noted that on the dorsal
side of said affidavit of marriage, Rev. Tomas V. Atienza, the solemnizing officer, stated that he took steps to ascertain the ages
and other qualifications of the contracting parties and found no legal impediment to their marriage. Finally, the Court of Appeals
dismissed Joses argument that neither he nor Felisa was a member of the sect to which Rev. Tomas V. Atienza belonged.
According to the Court of Appeals, Article 5617 of the Civil Code did not require that either one of the contracting parties to the
marriage must belong to the solemnizing officers church or religious sect. The prescription was established only in Article 7 18 of
the Family Code which does not govern the parties marriage.
Differing with the ruling of the Court of Appeals, Jose filed a Motion for Reconsideration thereof.1avvphi1 His central opposition
was that the requisites for the proper application of the exemption from a marriage license under Article 76 of the Civil Code
were not fully attendant in the case at bar. In particular, Jose cited the legal condition that the man and the woman must have
been living together as husband and wife for at least five years before the marriage. Essentially, he maintained that the affidavit
of marital cohabitation executed by him and Felisa was false.
The Court of Appeals granted Joses Motion for Reconsideration and reversed itself. Accordingly, it rendered an Amended
Decision, dated 7 November 2006, the fallo of which reads:
WHEREFORE, the Decision dated August 11, 2005 is RECALLED and SET ASIDE and another one entered declaring the
marriage between Jose A. Dayot and Felisa C. Tecson void ab initio.
Furnish a copy of this Amended Decision to the Local Civil Registrar of Pasay City.19
In its Amended Decision, the Court of Appeals relied on the ruling of this Court in Nial v. Bayadog, 20 and reasoned that:
In Nial v. Bayadog, where the contracting parties to a marriage solemnized without a marriage license on the basis of their
affidavit that they had attained the age of majority, that being unmarried, they had lived together for at least five (5) years and
that they desired to marry each other, the Supreme Court ruled as follows:
"x x x In other words, the five-year common-law cohabitation period, which is counted back from the date of celebration of
marriage, should be a period of legal union had it not been for the absence of the marriage. This 5-year period should be the
years immediately before the day of the marriage and it should be a period of cohabitation characterized by exclusivity
meaning no third party was involved at any time within the 5 years and continuity that is unbroken. Otherwise, if that
continuous 5-year cohabitation is computed without any distinction as to whether the parties were capacitated to marry each
other during the entire five years, then the law would be sanctioning immorality and encouraging parties to have common law
relationships and placing them on the same footing with those who lived faithfully with their spouse. Marriage being a special
relationship must be respected as such and its requirements must be strictly observed. The presumption that a man and a
woman deporting themselves as husband and wife is based on the approximation of the requirements of the law. The parties
should not be afforded any excuse to not comply with every single requirement and later use the same missing element as a
pre-conceived escape ground to nullify their marriage. There should be no exemption from securing a marriage license unless
the circumstances clearly fall within the ambit of the exception. It should be noted that a license is required in order to notify the
public that two persons are about to be united in matrimony and that anyone who is aware or has knowledge of any impediment
to the union of the two shall make it known to the local civil registrar.
Article 80(3) of the Civil Code provides that a marriage solemnized without a marriage license, save marriages of exceptional
character, shall be void from the beginning. Inasmuch as the marriage between Jose and Felisa is not covered by the exception
to the requirement of a marriage license, it is, therefore, void ab initio because of the absence of a marriage license. 21
Felisa sought reconsideration of the Amended Decision, but to no avail. The appellate court rendered a Resolution 22 dated 10
May 2007, denying Felisas motion.
Meanwhile, the Republic of the Philippines, through the Office of the Solicitor General (OSG), filed a Petition for Review before
this Court in G.R. No. 175581, praying that the Court of Appeals Amended Decision dated 7 November 2006 be reversed and
set aside for lack of merit, and that the marriage between Jose and Felisa be declared valid and subsisting. Felisa filed a
separate Petition for Review, docketed as G.R. No. 179474, similarly assailing the appellate courts Amended Decision. On 1
August 2007, this Court resolved to consolidate the two Petitions in the interest of uniformity of the Court rulings in similar cases
brought before it for resolution.23
The Republic of the Philippines propounds the following arguments for the allowance of its Petition, to wit:
I
RESPONDENT FAILED TO OVERTHROW THE PRESUMPTION OF THE VALIDITY OF HIS MARRIAGE TO FELISA.
II
RESPONDENT DID NOT COME TO THE COURT WITH CLEAN HANDS AND SHOULD NOT BE ALLOWED TO PROFIT
FROM HIS OWN FRAUDULENT CONDUCT.
III
RESPONDENT IS ESTOPPED FROM ASSAILING THE LEGALITY OF HIS MARRIAGE FOR LACK OF MARRIAGE
LICEN[S]E.24
Correlative to the above, Felisa submits that the Court of Appeals misapplied Nial. 25 She differentiates the case at bar from
Nial by reasoning that one of the parties therein had an existing prior marriage, a circumstance which does not obtain in her
cohabitation with Jose. Finally, Felisa adduces that Jose only sought the annulment of their marriage after a criminal case for

bigamy and an administrative case had been filed against him in order to avoid liability. Felisa surmises that the declaration of
nullity of their marriage would exonerate Jose from any liability.
For our resolution is the validity of the marriage between Jose and Felisa. To reach a considered ruling on the issue, we shall
jointly tackle the related arguments vented by petitioners Republic of the Philippines and Felisa.
The Republic of the Philippines asserts that several circumstances give rise to the presumption that a valid marriage exists
between Jose and Felisa. For her part, Felisa echoes the claim that any doubt should be resolved in favor of the validity of the
marriage by citing this Courts ruling in Hernandez v. Court of Appeals. 26 To buttress its assertion, the Republic points to the
affidavit executed by Jose and Felisa, dated 24 November 1986, attesting that they have lived together as husband and wife for
at least five years, which they used in lieu of a marriage license. It is the Republics position that the falsity of the statements in
the affidavit does not affect the validity of the marriage, as the essential and formal requisites were complied with; and the
solemnizing officer was not required to investigate as to whether the said affidavit was legally obtained. The Republic opines
that as a marriage under a license is not invalidated by the fact that the license was wrongfully obtained, so must a marriage not
be invalidated by the fact that the parties incorporated a fabricated statement in their affidavit that they cohabited as husband
and wife for at least five years. In addition, the Republic posits that the parties marriage contract states that their marriage was
solemnized under Article 76 of the Civil Code. It also bears the signature of the parties and their witnesses, and must be
considered a primary evidence of marriage. To further fortify its Petition, the Republic adduces the following documents: (1)
Joses notarized Statement of Assets and Liabilities, dated 12 May 1988 wherein he wrote Felisas name as his wife; (2)
Certification dated 25 July 1993 issued by the Barangay Chairman 192, Zone ZZ, District 24 of Pasay City, attesting that Jose
and Felisa had lived together as husband and wife in said barangay; and (3) Joses company ID card, dated 2 May 1988,
indicating Felisas name as his wife.
The first assignment of error compels this Court to rule on the issue of the effect of a false affidavit under Article 76 of the Civil
Code. A survey of the prevailing rules is in order.
It is beyond dispute that the marriage of Jose and Felisa was celebrated on 24 November 1986, prior to the effectivity of the
Family Code. Accordingly, the Civil Code governs their union. Article 53 of the Civil Code spells out the essential requisites of
marriage as a contract:
ART. 53. No marriage shall be solemnized unless all these requisites are complied with:
(1) Legal capacity of the contracting parties;
(2) Their consent, freely given;
(3) Authority of the person performing the marriage; and
(4) A marriage license, except in a marriage of exceptional character. (Emphasis ours.)
Article 5827 makes explicit that no marriage shall be solemnized without a license first being issued by the local civil registrar of
the municipality where either contracting party habitually resides, save marriages of an exceptional character authorized by the
Civil Code, but not those under Article 75. 28 Article 80(3)29 of the Civil Code makes it clear that a marriage performed without the
corresponding marriage license is void, this being nothing more than the legitimate consequence flowing from the fact that the
license is the essence of the marriage contract. 30 This is in stark contrast to the old Marriage Law,31 whereby the absence of a
marriage license did not make the marriage void. The rationale for the compulsory character of a marriage license under the
Civil Code is that it is the authority granted by the State to the contracting parties, after the proper government official has
inquired into their capacity to contract marriage. 32
Under the Civil Code, marriages of exceptional character are covered by Chapter 2, Title III, comprising Articles 72 to 79. To wit,
these marriages are: (1) marriages in articulo mortis or at the point of death during peace or war, (2) marriages in remote
places, (2) consular marriages,33 (3) ratification of marital cohabitation, (4) religious ratification of a civil marriage, (5)
Mohammedan or pagan marriages, and (6) mixed marriages. 34
The instant case pertains to a ratification of marital cohabitation under Article 76 of the Civil Code, which provides:
ART. 76. No marriage license shall be necessary when a man and a woman who have attained the age of majority and who,
being unmarried, have lived together as husband and wife for at least five years, desire to marry each other. The contracting
parties shall state the foregoing facts in an affidavit before any person authorized by law to administer oaths. The official, priest
or minister who solemnized the marriage shall also state in an affidavit that he took steps to ascertain the ages and other
qualifications of the contracting parties and that he found no legal impediment to the marriage.
The reason for the law,35 as espoused by the Code Commission, is that the publicity attending a marriage license may
discourage such persons who have lived in a state of cohabitation from legalizing their status. 36
It is not contested herein that the marriage of Jose and Felisa was performed without a marriage license. In lieu thereof, they
executed an affidavit declaring that "they have attained the age of maturity; that being unmarried, they have lived together as
husband and wife for at least five years; and that because of this union, they desire to marry each other." 37 One of the central
issues in the Petition at bar is thus: whether the falsity of an affidavit of marital cohabitation, where the parties have in truth
fallen short of the minimum five-year requirement, effectively renders the marriage void ab initio for lack of a marriage license.
We answer in the affirmative.
Marriages of exceptional character are, doubtless, the exceptions to the rule on the indispensability of the formal requisite of a
marriage license. Under the rules of statutory construction, exceptions, as a general rule, should be strictly 38 but reasonably
construed.39 They extend only so far as their language fairly warrants, and all doubts should be resolved in favor of the general
provisions rather than the exception.40 Where a general rule is established by statute with exceptions, the court will not curtail
the former or add to the latter by implication. 41 For the exception in Article 76 to apply, it is a sine qua non thereto that the man
and the woman must have attained the age of majority, and that, being unmarried, they have lived together as husband and wife
for at least five years.
A strict but reasonable construction of Article 76 leaves us with no other expediency but to read the law as it is plainly written.
The exception of a marriage license under Article 76 applies only to those who have lived together as husband and wife for at
least five years and desire to marry each other. The Civil Code, in no ambiguous terms, places a minimum period requirement
of five years of cohabitation. No other reading of the law can be had, since the language of Article 76 is precise. The minimum
requisite of five years of cohabitation is an indispensability carved in the language of the law. For a marriage celebrated under
Article 76 to be valid, this material fact cannot be dispensed with. It is embodied in the law not as a directory requirement, but as
one that partakes of a mandatory character. It is worthy to mention that Article 76 also prescribes that the contracting parties
shall state the requisite facts42 in an affidavit before any person authorized by law to administer oaths; and that the official, priest
or minister who solemnized the marriage shall also state in an affidavit that he took steps to ascertain the ages and other
qualifications of the contracting parties and that he found no legal impediment to the marriage.
It is indubitably established that Jose and Felisa have not lived together for five years at the time they executed their sworn
affidavit and contracted marriage. The Republic admitted that Jose and Felisa started living together only in June 1986, or
barely five months before the celebration of their marriage. 43 The Court of Appeals also noted Felisas testimony that Jose was

introduced to her by her neighbor, Teresita Perwel, sometime in February or March 1986 after the EDSA Revolution. 44 The
appellate court also cited Felisas own testimony that it was only in June 1986 when Jose commenced to live in her house. 45
Moreover, it is noteworthy that the question as to whether they satisfied the minimum five-year requisite is factual in nature. A
question of fact arises when there is a need to decide on the truth or falsehood of the alleged facts. 46Under Rule 45, factual
findings are ordinarily not subject to this Courts review.47 It is already well-settled that:
The general rule is that the findings of facts of the Court of Appeals are binding on this Court. A recognized exception to this rule
is when the Court of Appeals and the trial court, or in this case the administrative body, make contradictory findings. However,
the exception does not apply in every instance that the Court of Appeals and the trial court or administrative body disagree. The
factual findings of the Court of Appeals remain conclusive on this Court if such findings are supported by the record or based on
substantial evidence.48
Therefore, the falsity of the affidavit dated 24 November 1986, executed by Jose and Felisa to exempt them from the
requirement of a marriage license, is beyond question.
We cannot accept the insistence of the Republic that the falsity of the statements in the parties affidavit will not affect the
validity of marriage, since all the essential and formal requisites were complied with. The argument deserves scant merit.
Patently, it cannot be denied that the marriage between Jose and Felisa was celebrated without the formal requisite of a
marriage license. Neither did Jose and Felisa meet the explicit legal requirement in Article 76, that they should have lived
together as husband and wife for at least five years, so as to be excepted from the requirement of a marriage license.
Anent petitioners reliance on the presumption of marriage, this Court holds that the same finds no applicability to the case at
bar. Essentially, when we speak of a presumption of marriage, it is with reference to the prima facie presumption that a man and
a woman deporting themselves as husband and wife have entered into a lawful contract of marriage. 49 Restated more explicitly,
persons dwelling together in apparent matrimony are presumed, in the absence of any counter-presumption or evidence special
to the case, to be in fact married.50 The present case does not involve an apparent marriage to which the presumption still
needs to be applied. There is no question that Jose and Felisa actually entered into a contract of marriage on 24 November
1986, hence, compelling Jose to institute a Complaint for Annulment and/or Declaration of Nullity of Marriage, which spawned
the instant consolidated Petitions.
In the same vein, the declaration of the Civil Code 51 that every intendment of law or fact leans towards the validity of marriage
will not salvage the parties marriage, and extricate them from the effect of a violation of the law. The marriage of Jose and
Felisa was entered into without the requisite marriage license or compliance with the stringent requirements of a marriage under
exceptional circumstance. The solemnization of a marriage without prior license is a clear violation of the law and would lead or
could be used, at least, for the perpetration of fraud against innocent and unwary parties, which was one of the evils that the law
sought to prevent by making a prior license a prerequisite for a valid marriage. 52 The protection of marriage as a sacred
institution requires not just the defense of a true and genuine union but the exposure of an invalid one as well. 53 To permit a
false affidavit to take the place of a marriage license is to allow an abject circumvention of the law. If this Court is to protect the
fabric of the institution of marriage, we must be wary of deceptive schemes that violate the legal measures set forth in our laws.
Similarly, we are not impressed by the ratiocination of the Republic that as a marriage under a license is not invalidated by the
fact that the license was wrongfully obtained, so must a marriage not be invalidated by a fabricated statement that the parties
have cohabited for at least five years as required by law. The contrast is flagrant. The former is with reference to an irregularity
of the marriage license, and not to the absence of one. Here, there is no marriage license at all. Furthermore, the falsity of the
allegation in the sworn affidavit relating to the period of Jose and Felisas cohabitation, which would have qualified their
marriage as an exception to the requirement for a marriage license, cannot be a mere irregularity, for it refers to a quintessential
fact that the law precisely required to be deposed and attested to by the parties under oath. If the essential matter in the sworn
affidavit is a lie, then it is but a mere scrap of paper, without force and effect. Hence, it is as if there was no affidavit at all.
In its second assignment of error, the Republic puts forth the argument that based on equity, Jose should be denied relief
because he perpetrated the fabrication, and cannot thereby profit from his wrongdoing. This is a misplaced invocation. It must
be stated that equity finds no room for application where there is a law.54 There is a law on the ratification of marital cohabitation,
which is set in precise terms under Article 76 of the Civil Code. Nonetheless, the authorities are consistent that the declaration
of nullity of the parties marriage is without prejudice to their criminal liability. 55
The Republic further avers in its third assignment of error that Jose is deemed estopped from assailing the legality of his
marriage for lack of a marriage license. It is claimed that Jose and Felisa had lived together from 1986 to 1990, notwithstanding
Joses subsequent marriage to Rufina Pascual on 31 August 1990, and that it took Jose seven years before he sought the
declaration of nullity; hence, estoppel had set in.
This is erroneous. An action for nullity of marriage is imprescriptible. 56 Jose and Felisas marriage was celebrated sans a
marriage license. No other conclusion can be reached except that it is void ab initio. In this case, the right to impugn a void
marriage does not prescribe, and may be raised any time.
Lastly, to settle all doubts, jurisprudence has laid down the rule that the five-year common-law cohabitation period under Article
76 means a five-year period computed back from the date of celebration of marriage, and refers to a period of legal union had it
not been for the absence of a marriage.57 It covers the years immediately preceding the day of the marriage, characterized by
exclusivity - meaning no third party was involved at any time within the five years - and continuity that is unbroken. 58
WHEREFORE, the Petitions are DENIED. The Amended Decision of the Court of Appeals, dated 7 November 2006 in CA-G.R.
CV No. 68759, declaring the marriage of Jose Dayot to Felisa Tecson-Dayot void ab initio, is AFFIRMED, without prejudice to
their criminal liability, if any. No costs.
SO ORDERED.

Void and Voidable Marriage

G.R. No. 104818 September 17, 1993


ROBERTO DOMINGO, petitioner,
vs.
COURT OF APPEALS and DELIA SOLEDAD AVERA represented by her Attorney-in-Fact MOISES R.
AVERA,respondents.
Jose P.O. Aliling IV for petitioner.
De Guzman, Meneses & Associates for private respondent.

ROMERO, J.:
The instant petition seeks the reversal of respondent court's ruling finding no grave abuse of discretion in the lower court's order
denying petitioner's motion to dismiss the petition for declaration of nullity of marriage and separation of property.
On May 29, 1991, private respondent Delia Soledad A. Domingo filed a petition before the Regional Trial Court of Pasig entitled
"Declaration of Nullity of Marriage and Separation of Property" against petitioner Roberto Domingo. The petition which was
docketed as Special Proceedings No. 1989-J alleged among others that: they were married on November 29, 1976 at the
YMCA Youth Center Bldg., as evidenced by a Marriage Contract Registry No. 1277K-76 with Marriage License No. 4999036
issued at Carmona, Cavite; unknown to her, he had a previous marriage with one Emerlina dela Paz on April 25, 1969 which
marriage is valid and still existing; she came to know of the prior marriage only sometime in 1983 when Emerlina dela Paz sued
them for bigamy; from January 23 1979 up to the present, she has been working in Saudi Arabia and she used to come to the
Philippines only when she would avail of the one-month annual vacation leave granted by her foreign employer since 1983 up to
the present, he has been unemployed and completely dependent upon her for support and subsistence; out of her personal
earnings, she purchased real and personal properties with a total amount of approximately P350,000.00, which are under the
possession and administration of Roberto; sometime in June 1989, while on her one-month vacation, she discovered that he
was cohabiting with another woman; she further discovered that he had been disposing of some of her properties without her
knowledge or consent; she confronted him about this and thereafter appointed her brother Moises R. Avera as her attorney-infact to take care of her properties; he failed and refused to turn over the possession and administration of said properties to her
brother/attorney-in-fact; and he is not authorized to administer and possess the same on account of the nullity of their marriage.
The petition prayed that a temporary restraining order or a writ of preliminary injunction be issued enjoining Roberto from
exercising any act of administration and ownership over said properties; their marriage be declared null and void and of no force
and effect; and Delia Soledad be declared the sole and exclusive owner of all properties acquired at the time of their void
marriage and such properties be placed under the proper management and administration of the attorney-in-fact.
Petitioner filed a Motion to Dismiss on the ground that the petition stated no cause of action. The marriage being void ab initio,
the petition for the declaration of its nullity is, therefore, superfluous and unnecessary. It added that private respondent has no
property which is in his possession.
On August 20, 1991, Judge Maria Alicia M. Austria issued an Order denying the motion to dismiss for lack of merit. She
explained:
Movant argues that a second marriage contracted after a first marriage by a man with another woman is illegal and
void (citing the case of Yap v. Court of Appeals, 145 SCRA 229) and no judicial decree is necessary to establish the
invalidity of a void marriage (citing the cases of People v. Aragon, 100 Phil. 1033; People v. Mendoza, 95 Phil. 845).
Indeed, under the Yap case there is no dispute that the second marriage contracted by respondent with herein
petitioner after a first marriage with another woman is illegal and void. However, as to whether or not the second
marriage should first be judicially declared a nullity is not an issue in said case. In the case of Vda. de Consuegra
v. GSIS, the Supreme Court ruled in explicit terms, thus:
And with respect to the right of the second wife, this Court observed that although the second marriage
can be presumed to be void ab initio as it was celebrated while the first marriage was still subsisting, still
there is need for judicial declaration of its nullity. (37 SCRA 316, 326)
The above ruling which is of later vintage deviated from the previous rulings of the Supreme Court in the
aforecited cases of Aragon and Mendoza.
Finally, the contention of respondent movant that petitioner has no property in his possession is an issue
that may be determined only after trial on the merits. 1
A motion for reconsideration was filed stressing the erroneous application of Vda. de Consuegra v. GSIS 2 and the absence of
justiciable controversy as to the nullity of the marriage. On September 11, 1991, Judge Austria denied the motion for
reconsideration and gave petitioner fifteen (15) days from receipt within which to file his answer.
Instead of filing the required answer, petitioner filed a special civil action of certiorari and mandamus on the ground that the
lower court acted with grave abuse of discretion amounting to lack of jurisdiction in denying the motion to dismiss.
On February 7, 1992, the Court of Appeals 3 dismissed the petition. It explained that the case of Yap v. CA 4 cited by petitioner
and that of Consuegra v. GSIS relied upon by the lower court do not have relevance in the case at bar, there being no identity of
facts because these cases dealt with the successional rights of the second wife while the instant case prays for separation of
property corollary with the declaration of nullity of marriage. It observed that the separation and subsequent distribution of the
properties acquired during the union can be had only upon proper determination of the status of the marital relationship between
said parties, whether or not the validity of the first marriage is denied by petitioner. Furthermore, in order to avoid duplication
and multiplicity of suits, the declaration of nullity of marriage may be invoked in this proceeding together with the partition and
distribution of the properties involved. Citing Articles 48, 50 and 52 of the Family Code, it held that private respondent's prayer
for declaration of absolute nullity of their marriage may be raised together with other incidents of their marriage such as the
separation of their properties. Lastly, it noted that since the Court has jurisdiction, the alleged error in refusing to grant the
motion to dismiss is merely one of law for which the remedy ordinarily would have been to file an answer, proceed with the trial
and in case of an adverse decision, reiterate the issue on appeal. The motion for reconsideration was subsequently denied for
lack of merit. 5
Hence, this petition.
The two basic issues confronting the Court in the instant case are the following.
First, whether or not a petition for judicial declaration of a void marriage is necessary. If in the affirmative, whether the same
should be filed only for purposes of remarriage.
Second, whether or not SP No. 1989-J is the proper remedy of private respondent to recover certain real and personal
properties allegedly belonging to her exclusively.
Petitioner, invoking the ruling in People v. Aragon 6 and People v. Mendoza, 7 contends that SP. No. 1989-J for Declaration of
Nullity of Marriage and Separation of Property filed by private respondent must be dismissed for being unnecessary and

superfluous. Furthermore, under his own interpretation of Article 40 of the Family Code, he submits that a petition for declaration
of absolute nullity of marriage is required only for purposes of remarriage. Since the petition in SP No. 1989-J contains no
allegation of private respondent's intention to remarry, said petition should therefore, be dismissed.
On the other hand, private respondent insists on the necessity of a judicial declaration of the nullity of their marriage, not for
purposes of remarriage, but in order to provide a basis for the separation and distribution of the properties acquired during
coverture.
There is no question that the marriage of petitioner and private respondent celebrated while the former's previous marriage with
one Emerlina de la Paz was still subsisting, is bigamous. As such, it is from the beginning. 8 Petitioner himself does not dispute
the absolute nullity of their marriage. 9
The cases of People v. Aragon and People v. Mendoza relied upon by petitioner are cases where the Court had earlier ruled
that no judicial decree is necessary to establish the invalidity of a void, bigamous marriage. It is noteworthy to observe that
Justice Alex Reyes, however, dissented on these occasions stating that:
Though the logician may say that where the former marriage was void there would be nothing to dissolve, still it is
not for the spouses to judge whether that marriage was void or not. That judgment is reserved to the courts. . . . 10
This dissenting opinion was adopted as the majority position in subsequent cases involving the same issue. Thus, in Gomez
v. Lipana, 11 the Court abandoned its earlier ruling in the Aragon and Mendoza cases. In reversing the lower court's order
forfeiting the husband's share of the disputed property acquired during the second marriage, the Court stated that "if the nullity,
or annulment of the marriage is the basis for the application of Article 1417, there is need for a judicial declaration thereof, which
of course contemplates an action for that purpose."
Citing Gomez v. Lipana, the Court subsequently held in Vda. de Consuegra v. Government Service Insurance System, that
"although the second marriage can be presumed to be void ab initio as it was celebrated while the first marriage was still
subsisting, still there is need for judicial declaration of such nullity."
In Tolentino v. Paras, 12 however, the Court turned around and applied the Aragon and Mendoza ruling once again. In granting
the prayer of the first wife asking for a declaration as the lawful surviving spouse and the correction of the death certificate of
her deceased husband, it explained that "(t)he second marriage that he contracted with private respondent during the lifetime of
his first spouse is null and void from the beginning and of no force and effect. No judicial decree is necessary to establish the
invalidity of a void marriage."
However, in the more recent case of Wiegel v. Sempio-Diy 13 the Court reverted to the Consuegra case and held that there was
"no need of introducing evidence about the existing prior marriage of her first husband at the time they married each other, for
then such a marriage though void still needs according to this Court a judicial declaration of such fact and for all legal intents
and purposes she would still be regarded as a married woman at the time she contracted her marriage with respondent Karl
Heinz Wiegel."
Came the Family Code which settled once and for all the conflicting jurisprudence on the matter. A declaration of the absolute
nullity of a marriage is now explicitly required either as a cause of action or a ground for defense. 14Where the absolute nullity of
a previous marriage is sought to be invoked for purposes of contracting a second marriage, the sole basis acceptable in law for
said projected marriage be free from legal infirmity is a final judgment declaring the previous marriage void. 15
The Family Law Revision Committee and the Civil Code Revision Committee 16 which drafted what is now the Family Code of
the Philippines took the position that parties to a marriage should not be allowed to assume that their marriage is void even if
such be the fact but must first secure a judicial declaration of the nullity of their marriage before they can be allowed to marry
again. This is borne out by the following minutes of the 152nd Joint Meeting of the Civil Code and Family Law Committees
where the present Article 40, then Art. 39, was discussed.
B. Article 39.
The absolute nullity of a marriage may be invoked only on the basis of a final judgment declaring the
marriage void, except as provided in Article 41.
Justice Caguioa remarked that the above provision should include not only void but also voidable marriages. He
then suggested that the above provision be modified as follows:
The validity of a marriage may be invoked only . . .
Justice Reyes (J.B.L. Reyes), however, proposed that they say:
The validity or invalidity of a marriage may be invoked
only . . .
On the other hand, Justice Puno suggested that they say:
The invalidity of a marriage may be invoked only . . .
Justice Caguioa explained that his idea is that one cannot determine for himself whether or not his marriage is valid
and that a court action is needed. Justice Puno accordingly proposed that the provision be modified to read:
The invalidity of a marriage may be invoked only on the basis of a final judgment annulling the marriage
or declaring the marriage void, except as provided in Article 41.
Justice Caguioa remarked that in annulment, there is no question. Justice Puno, however, pointed out that, even if it
is a judgment of annulment, they still have to produce the judgment.
Justice Caguioa suggested that they say:
The invalidity of a marriage may be invoked only on the basis of a final judgment declaring the marriage
invalid, except as provided in Article 41.
Justice Puno raised the question: When a marriage is declared invalid, does it include the annulment of a marriage
and the declaration that the marriage is void? Justice Caguioa replied in the affirmative. Dean Gupit added that in
some judgments, even if the marriage is annulled, it is declared void. Justice Puno suggested that this matter be
made clear in the provision.
Prof. Baviera remarked that the original idea in the provision is to require first a judicial declaration of a void
marriage and not annullable marriages, with which the other members concurred. Judge Diy added that annullable
marriages are presumed valid until a direct action is filed to annul it, which the other members affirmed. Justice
Puno remarked that if this is so, then the phrase "absolute nullity" can stand since it might result in confusion if they
change the phrase to "invalidity" if what they are referring to in the provision is the declaration that the marriage is
void.

Prof. Bautista commented that they will be doing away with collateral defense as well as collateral attack. Justice
Caguioa explained that the idea in the provision is that there should be a final judgment declaring the marriage void
and a party should not declare for himself whether or not the marriage is void, while the other members affirmed.
Justice Caguioa added that they are, therefore, trying to avoid a collateral attack on that point. Prof. Bautista stated
that there are actions which are brought on the assumption that the marriage is valid. He then asked: Are they
depriving one of the right to raise the defense that he has no liability because the basis of the liability is void? Prof.
Bautista added that they cannot say that there will be no judgment on the validity or invalidity of the marriage
because it will be taken up in the same proceeding. It will not be a unilateral declaration that, it is a void
marriage. Justice Caguioa saw the point of Prof. Bautista and suggested that they limit the provision to remarriage.
He then proposed that Article 39 be reworded as follows:
The absolute nullity of a marriage for purposes of remarriage may be invoked only on the basis of final
judgment . . .
Justice Puno suggested that the above be modified as follows:
The absolute nullity of a previous marriage may be invoked for purposes of establishing the validity of a
subsequent marriage only on the basis of a final judgment declaring such previous marriage void, except
as provided in Article 41.
Justice Puno later modified the above as follows:
For the purpose of establishing the validity of a subsequent marriage, the absolute nullity of a previous
marriage may only be invoked on the basis of a final judgment declaring such nullity, except as provided
in Article 41.
Justice Caguioa commented that the above provision is too broad and will not solve the objection of Prof. Bautista.
He proposed that they say:
For the purpose of entering into a subsequent marriage, the absolute nullity of a previous marriage may
only be invoked on the basis of a final judgment declaring such nullity, except as provided in Article 41.
Justice Caguioa explained that the idea in the above provision is that if one enters into a subsequent marriage
without obtaining a final judgment declaring the nullity of a previous marriage, said subsequent marriage is void ab
initio.
After further deliberation, Justice Puno suggested that they go back to the original wording of the provision as
follows:
The absolute nullity of a previous marriage may be invoked for purposes of remarriage only on the basis
of a final judgment declaring such previous marriage void, except as provided in Article 41. 17
In fact, the requirement for a declaration of absolute nullity of a marriage is also for the protection of the spouse who, believing
that his or her marriage is illegal and void, marries again. With the judicial declaration of the nullity of his or her first marriage,
the person who marries again cannot be charged with bigamy. 18
Just over a year ago, the Court made the pronouncement that there is a necessity for a declaration of absolute nullity of a prior
subsisting marriage before contracting another in the recent case of Terre v. Terre. 19 The Court, in turning down the defense of
respondent Terre who was charged with grossly immoral conduct consisting of contracting a second marriage and living with
another woman other than complainant while his prior marriage with the latter remained subsisting, said that "for purposes of
determining whether a person is legally free to contract a second marriage, a judicial declaration that the first marriage was null
and void ab initio is essential."
As regards the necessity for a judicial declaration of absolute nullity of marriage, petitioner submits that the same can be
maintained only if it is for the purpose of remarriage. Failure to allege this purpose, according to petitioner's theory, will warrant
dismissal of the same.
Article 40 of the Family Code provides:
Art. 40. The absolute nullity of a previous marriage may be invoked for purposes of remarriage on the basis solely of
a final judgment declaring such previous marriage void. (n)
Crucial to the proper interpretation of Article 40 is the position in the provision of the word "solely." As it is placed, the same
shows that it is meant to qualify "final judgment declaring such previous marriage void." Realizing the need for careful
craftsmanship in conveying the precise intent of the Committee members, the provision in question, as it finally emerged, did
not state "The absolute nullity of a previous marriage may be invoked solely for purposes of remarriage . . .," in which case
"solely" would clearly qualify the phrase "for purposes of remarriage." Had the phraseology been such, the interpretation of
petitioner would have been correct and, that is, that the absolute nullity of a previous marriage may be invoked solely for
purposes of remarriage, thus rendering irrelevant the clause "on the basis solely of a final judgment declaring such previous
marriage void."
That Article 40 as finally formulated included the significant clause denotes that such final judgment declaring the previous
marriage void need not be obtained only for purposes of remarriage. Undoubtedly, one can conceive of other instances where a
party might well invoke the absolute nullity of a previous marriage for purposes other than remarriage, such as in case of an
action for liquidation, partition, distribution and separation of property between the erstwhile spouses, as well as an action for
the custody and support of their common children and the delivery of the latters' presumptive legitimes. In such cases, evidence
needs must be adduced, testimonial or documentary, to prove the existence of grounds rendering such a previous marriage an
absolute nullity. These need not be limited solely to an earlier final judgment of a court declaring such previous marriage void.
Hence, in the instance where a party who has previously contracted a marriage which remains subsisting desires to enter into
another marriage which is legally unassailable, he is required by law to prove that the previous one was an absolute nullity. But
this he may do on the basis solely of a final judgment declaring such previous marriage void.
This leads us to the question: Why the distinction? In other words, for purposes of remarriage, why should the only legally
acceptable basis for declaring a previous marriage an absolute nullity be a final judgment declaring such previous marriage
void? Whereas, for purposes other than remarriage, other evidence is acceptable?
Marriage, a sacrosanct institution, declared by the Constitution as an "inviolable social institution, is the foundation of the
family;" as such, it "shall be protected by the State." 20 In more explicit terms, the Family Code characterizes it as "a special
contract of permanent union between a man and a woman entered into in accordance with law for the establishment of
conjugal, and family life." 21 So crucial are marriage and the family to the stability and peace of the nation that their "nature,
consequences, and incidents are governed by law and not subject to stipulation . . ." 22 As a matter of policy, therefore, the
nullification of a marriage for the purpose of contracting another cannot be accomplished merely on the basis of the perception
of both parties or of one that their union is so defective with respect to the essential requisites of a contract of marriage as to
render it void ipso jure and with no legal effect and nothing more. Were this so, this inviolable social institution would be
reduced to a mockery and would rest on very shaky foundations indeed. And the grounds for nullifying marriage would be as
diverse and far-ranging as human ingenuity and fancy could conceive. For such a social significant institution, an official state

pronouncement through the courts, and nothing less, will satisfy the exacting norms of society. Not only would such an open
and public declaration by the courts definitively confirm the nullity of the contract of marriage, but the same would be easily
verifiable through records accessible to everyone.
That the law seeks to ensure that a prior marriage is no impediment to a second sought to be contracted by one of the parties
may be gleaned from new information required in the Family Code to be included in the application for a marriage license, viz,
"If previously married, how, when and where the previous marriage was dissolved and annulled." 23
Reverting to the case before us, petitioner's interpretation of Art. 40 of the Family Code is, undoubtedly, quite restrictive. Thus,
his position that private respondent's failure to state in the petition that the same is filed to enable her to remarry will result in the
dismissal of SP No. 1989-J is untenable. His misconstruction of Art. 40 resulting from the misplaced emphasis on the term
"solely" was in fact anticipated by the members of the Committee.
Dean Gupit commented the word "only" may be misconstrued to refer to "for purposes of remarriage." Judge Diy
stated that "only" refers to "final judgment." Justice Puno suggested that they say "on the basis only of a final
judgment." Prof. Baviera suggested that they use the legal term "solely" instead of "only," which the Committee
approved. 24 (Emphasis supplied)
Pursuing his previous argument that the declaration for absolute nullity of marriage is unnecessary, petitioner suggests that
private respondent should have filed an ordinary civil action for the recovery of the properties alleged to have been acquired
during their union. In such an eventuality, the lower court would not be acting as a mere special court but would be clothed with
jurisdiction to rule on the issues of possession and ownership. In addition, he pointed out that there is actually nothing to
separate or partition as the petition admits that all the properties were acquired with private respondent's money.
The Court of Appeals disregarded this argument and concluded that "the prayer for declaration of absolute nullity of marriage
may be raised together with the other incident of their marriage such as the separation of their properties."
When a marriage is declared void ab initio, the law states that the final judgment therein shall provide for "the liquidation,
partition and distribution of the properties of the spouses, the custody and support of the common children, and the delivery of
their presumptive legitimes, unless such matters had been adjudicated in previous judicial proceedings." 25 Other specific effects
flowing therefrom, in proper cases, are the following:
Art. 43. xxx xxx xxx
(2) The absolute community of property or the conjugal partnership, as the case may be, shall be dissolved and
liquidated, but if either spouse contracted said marriage in bad faith, his or her share of the net profits of the
community property or conjugal partnership property shall be forfeited in favor of the common children or, if there are
none, the children of the guilty spouse by a previous marriage or, in default of children, the innocent spouse;
(3) Donations by reason of marriage shall remain valid, except that if the donee contracted the marriage in bad faith,
such donations made to said donee are revoked by operation of law;
(4) The innocent spouse may revoke the designation of the other spouse who acted in bad faith as a beneficiary in
any insurance policy, even if such designation be stipulated as irrevocable; and
(5) The spouse who contracted the subsequent marriage in bad faith shall be disqualified to inherit from the innocent
spouse by testate and intestate succession. (n)
Art. 44. If both spouses of the subsequent marriage acted in bad faith, said marriage shall be void ab initio and all
donations by reason of marriage and testamentary disposition made by one in favor of the other are revoked by
operation of law. (n) 26
Based on the foregoing provisions, private respondent's ultimate prayer for separation of property will simply be one of the
necessary consequences of the judicial declaration of absolute nullity of their marriage. Thus, petitioner's suggestion that in
order for their properties to be separated, an ordinary civil action has to be instituted for that purpose is baseless. The Family
Code has clearly provided the effects of the declaration of nullity of marriage, one of which is the separation of property
according to the regime of property relations governing them. It stands to reason that the lower court before whom the issue of
nullity of a first marriage is brought is likewise clothed with jurisdiction to decide the incidental questions regarding the couple's
properties. Accordingly, the respondent court committed no reversible error in finding that the lower court committed no grave
abuse of discretion in denying petitioner's motion to dismiss SP No. 1989-J.
WHEREFORE, the instant petition is hereby DENIED. The decision of respondent Court dated February 7, 1992 and the
Resolution dated March 20, 1992 are AFFIRMED.
SO ORDERED.

G.R. No. 108763 February 13, 1997

REPUBLIC OF THE PHILIPPINES,


vs.
COURT OF APPEALS and RORIDEL OLAVIANO MOLINA, respondents.

PANGANIBAN, J.:
The Family Code of the Philippines provides an entirely new ground (in addition to those enumerated in the Civil Code) to assail
the validity of a marriage, namely, "psychological incapacity." Since the Code's effectivity, our courts have been swamped with
various petitions to declare marriages void based on this ground. Although this Court had interpreted the meaning of
psychological incapacity in the recent case of Santos vs. Court of Appeals, still many judges and lawyers find difficulty in
applying said novel provision in specific cases. In the present case and in the context of the herein assailed Decision of the
Court of Appeals, the Solicitor General has labelled exaggerated to be sure but nonetheless expressive of his frustration
Article 36 as the "most liberal divorce procedure in the world." Hence, this Court in addition to resolving the present case, finds
the need to lay down specific guidelines in the interpretation and application of Article 36 of the Family Code.
Before us is a petition for review on certiorari under Rule 45 challenging the January 25, 1993 Decision 1 of the Court of
Appeals 2 in CA-G.R. CV No. 34858 affirming in toto the May 14, 1991 decision of the Regional Trial Court of La
Trinidad, 3 Benguet, which declared the marriage of respondent Roridel Olaviano Molina to Reynaldo Molina void ab initio, on
the ground of "psychological incapacity" under Article 36 of the Family Code.
The Facts
This case was commenced on August 16, 1990 with the filing by respondent Roridel O. Molina of a verified petition for
declaration of nullity of her marriage to Reynaldo Molina. Essentially, the petition alleged that Roridel and Reynaldo were
married on April 14, 1985 at the San Agustin Church 4 in Manila; that a son, Andre O. Molina was born; that after a year of
marriage, Reynaldo showed signs of "immaturity and irresponsibility" as a husband and a father since he preferred to spend
more time with his peers and friends on whom he squandered his money; that he depended on his parents for aid and
assistance, and was never honest with his wife in regard to their finances, resulting in frequent quarrels between them; that
sometime in February 1986, Reynaldo was relieved of his job in Manila, and since then Roridel had been the sole breadwinner
of the family; that in October 1986 the couple had a very intense quarrel, as a result of which their relationship was estranged;
that in March 1987, Roridel resigned from her job in Manila and went to live with her parents in Baguio City; that a few weeks
later, Reynaldo left Roridel and their child, and had since then abandoned them; that Reynaldo had thus shown that he was
psychologically incapable of complying with essential marital obligations and was a highly immature and habitually quarrel some
individual who thought of himself as a king to be served; and that it would be to the couple's best interest to have their marriage
declared null and void in order to free them from what appeared to be an incompatible marriage from the start.
In his Answer filed on August 28, 1989, Reynaldo admitted that he and Roridel could no longer live together as husband and
wife, but contended that their misunderstandings and frequent quarrels were due to (1) Roridel's strange behavior of insisting on
maintaining her group of friends even after their marriage; (2) Roridel's refusal to perform some of her marital duties such as
cooking meals; and (3) Roridel's failure to run the household and handle their finances.
During the pre-trial on October 17, 1990, the following were stipulated:
1. That the parties herein were legally married on April 14, 1985 at the Church of St. Augustine, Manila;
2. That out of their marriage, a child named Albert Andre Olaviano Molina was born on July 29, 1986;
3. That the parties are separated-in-fact for more than three years;
4. That petitioner is not asking support for her and her child;
5. That the respondent is not asking for damages;
6. That the common child of the parties is in the custody of the petitioner wife.
Evidence for herein respondent wife consisted of her own testimony and that of her friends Rosemarie Ventura and Maria
Leonora Padilla as well as of Ruth G. Lalas, a social worker, and of Dr. Teresita Hidalgo-Sison, a psychiatrist of the Baguio
General Hospital and Medical Center. She also submitted documents marked as Exhibits "A" to "E-1." Reynaldo did not present
any evidence as he appeared only during the pre-trial conference.
On May 14, 1991, the trial court rendered judgment declaring the marriage void. The appeal of petitioner was denied by the
Court of Appeals which affirmed in toto the RTC's decision. Hence, the present recourse.
The Issue
In his petition, the Solicitor General insists that "the Court of Appeals made an erroneous and incorrect interpretation of the
phrase 'psychological incapacity' (as provided under Art. 36 of the Family Code) and made an incorrect application thereof to
the facts of the case," adding that the appealed Decision tended "to establish in effect the most liberal divorce procedure in the
world which is anathema to our culture."
In denying the Solicitor General's appeal, the respondent Court relied 5 heavily on the trial court's findings "that the marriage
between the parties broke up because of their opposing and conflicting personalities." Then, it added it sown opinion that "the
Civil Code Revision Committee (hereinafter referred to as Committee) intended to liberalize the application of our civil laws on
personal and family rights. . . ." It concluded that:
As ground for annulment of marriage, We view psychologically incapacity as a broad range of mental and behavioral
conduct on the part of one spouse indicative of how he or she regards the marital union, his or her personal
relationship with the other spouse, as well as his or her conduct in the long haul for the attainment of the principal
objectives of marriage. If said conduct, observed and considered as a whole, tends to cause the union to selfdestruct because it defeats the very objectives of marriage, then there is enough reason to leave the spouses to
their individual fates.
In the case at bar, We find that the trial judge committed no indiscretion in analyzing and deciding the instant case,
as it did, hence, We find no cogent reason to disturb the findings and conclusions thus made.
Respondent, in her Memorandum, adopts these discussions of the Court of Appeals.
The petitioner, on the other hand, argues that "opposing and conflicting personalities" is not equivalent to psychological
incapacity, explaining that such ground "is not simply the neglect by the parties to the marriage of their responsibilities and
duties, but a defect in their psychological nature which renders them incapable of performing such marital responsibilities and
duties."
The Court's Ruling
The petition is meritorious.

In Leouel Santos vs. Court of Appeals 6 this Court, speaking thru Mr. Justice Jose C. Vitug, ruled that "psychological incapacity
should refer to no less than a mental (nor physical) incapacity . . . and that (t)here is hardly any doubt that the intendment of the
law has been to confine the meaning of 'psychological incapacity' to the most serious cases of personality disorders clearly
demonstrative of an utter insensitivity or inability to give meaning and significance to the marriage. This psychologic condition
must exist at the time the marriage is celebrated." Citing Dr. Gerardo Veloso, a former presiding judge of the Metropolitan
Marriage Tribunal of the Catholic Archdiocese of Manila, 7 Justice Vitug wrote that "the psychological incapacity must be
characterized by (a) gravity, (b) juridical antecedence, and (c) incurability."
On the other hand, in the present case, there is no clear showing to us that the psychological defect spoken of is an incapacity.
It appears to us to be more of a "difficulty," if not outright "refusal" or "neglect" in the performance of some marital obligations.
Mere showing of "irreconciliable differences" and "conflicting personalities" in no wise constitutes psychological incapacity. It is
not enough to prove that the parties failed to meet their responsibilities and duties as married persons; it is essential that they
must be shown to be incapable of doing so, due to some psychological (nor physical) illness.
The evidence adduced by respondent merely showed that she and her husband could nor get along with each other. There had
been no showing of the gravity of the problem; neither its juridical antecedence nor its incurability. The expert testimony of Dr.
Sison showed no incurable psychiatric disorder but only incompatibility, not psychological incapacity. Dr. Sison testified: 8
COURT
Q It is therefore the recommendation of the psychiatrist based on your findings that it is better for the
Court to annul (sic) the marriage?
A Yes, Your Honor.
Q There is no hope for the marriage?
A There is no hope, the man is also living with another woman.
Q Is it also the stand of the psychiatrist that the parties are psychologically unfit for each other but they
are psychologically fit with other parties?
A Yes, Your Honor.
Q Neither are they psychologically unfit for their professions?
A Yes, Your Honor.
The Court has no more questions.
In the case of Reynaldo, there is no showing that his alleged personality traits were constitutive of psychological incapacity
existing at the time of marriage celebration. While some effort was made to prove that there was a failure to fulfill pre-nuptial
impressions of "thoughtfulness and gentleness" on Reynaldo's part of being "conservative, homely and intelligent" on the part of
Roridel, such failure of expectation is nor indicative of antecedent psychological incapacity. If at all, it merely shows love's
temporary blindness to the faults and blemishes of the beloved.
During its deliberations, the Court decided to go beyond merely ruling on the facts of this case vis-a-vis existing law and
jurisprudence. In view of the novelty of Art. 36 of the Family Code and the difficulty experienced by many trial courts interpreting
and applying it, the Court decided to invite two amici curiae, namely, the Most Reverend Oscar V. Cruz, 9 Vicar
Judicial (Presiding Judge) of the National Appellate Matrimonial Tribunal of the Catholic Church in the Philippines, and Justice
Ricardo C. Puno, 10 a member of the Family Code Revision Committee. The Court takes this occasion to thank these friends of
the Court for their informative and interesting discussions during the oral argument on December 3, 1996, which they followed
up with written memoranda.
From their submissions and the Court's own deliberations, the following guidelines in the interpretation and application of Art. 36
of the Family Code are hereby handed down for the guidance of the bench and the bar:
(1) The burden of proof to show the nullity of the marriage belongs to the plaintiff. Any doubt should be resolved in favor of the
existence and continuation of the marriage and against its dissolution and nullity. This is rooted in the fact that both our
Constitution and our laws cherish the validity of marriage and unity of the family. Thus, our Constitution devotes an entire Article
on the Family, 11 recognizing it "as the foundation of the nation." It decrees marriage as legally "inviolable," thereby protecting it
from dissolution at the whim of the parties. Both the family and marriage are to be "protected" by the state.
The Family Code 12 echoes this constitutional edict on marriage and the family and emphasizes thepermanence,
inviolability and solidarity
(2) The root cause of the psychological incapacity must be (a) medically or clinically identified, (b) alleged in the complaint, (c)
sufficiently proven by experts and (d) clearly explained in the decision. Article 36 of the Family Code requires that the incapacity
must be psychological not physical. although its manifestations and/or symptoms may be physical. The evidence must
convince the court that the parties, or one of them, was mentally or physically ill to such an extent that the person could not
have known the obligations he was assuming, or knowing them, could not have given valid assumption thereof. Although no
example of such incapacity need be given here so as not to limit the application of the provision under the principle of ejusdem
generis, 13 nevertheless such root cause must be identified as a psychological illness and its incapacitating nature explained.
Expert evidence may be given qualified psychiatrist and clinical psychologists.
(3) The incapacity must be proven to be existing at "the time of the celebration" of the marriage. The evidence must show that
the illness was existing when the parties exchanged their "I do's." The manifestation of the illness need not be perceivable at
such time, but the illness itself must have attached at such moment, or prior thereto.
(4) Such incapacity must also be shown to be medically or clinically permanent or incurable. Such incurability may be absolute
or even relative only in regard to the other spouse, not necessarily absolutely against everyone of the same sex. Furthermore,
such incapacity must be relevant to the assumption of marriage obligations, not necessarily to those not related to marriage, like
the exercise of a profession or employment in a job. Hence, a pediatrician may be effective in diagnosing illnesses of children
and prescribing medicine to cure them but may not be psychologically capacitated to procreate, bear and raise his/her own
children as an essential obligation of marriage.
(5) Such illness must be grave enough to bring about the disability of the party to assume the essential obligations of marriage.
Thus, "mild characteriological peculiarities, mood changes, occasional emotional outbursts" cannot be accepted as root causes.
The illness must be shown as downright incapacity or inability, nor a refusal, neglect or difficulty, much less ill will. In other
words, there is a natal or supervening disabling factor in the person, an adverse integral element in the personality structure that
effectively incapacitates the person from really accepting and thereby complying with the obligations essential to marriage.
(6) The essential marital obligations must be those embraced by Articles 68 up to 71 of the Family Code as regards the
husband and wife as well as Articles 220, 221 and 225 of the same Code in regard to parents and their children. Such noncomplied marital obligation(s) must also be stated in the petition, proven by evidence and included in the text of the decision.

(7) Interpretations given by the National Appellate Matrimonial Tribunal of the Catholic Church in the Philippines, while not
controlling or decisive, should be given great respect by our courts. It is clear that Article 36 was taken by the Family Code
Revision Committee from Canon 1095 of the New Code of Canon Law, which became effective in 1983 and which provides:
The following are incapable of contracting marriage: Those who are unable to assume the essential obligations of
marriage due to causes of psychological nature. 14
Since the purpose of including such provision in our Family Code is to harmonize our civil laws with the religious faith of our
people, it stands to reason that to achieve such harmonization, great persuasive weight should be given to decision of such
appellate tribunal. Ideally subject to our law on evidence what is decreed as canonically invalid should also be decreed
civilly void.
This is one instance where, in view of the evident source and purpose of the Family Code provision, contemporaneous religious
interpretation is to be given persuasive effect. Here, the State and the Church while remaining independent, separate and
apart from each other shall walk together in synodal cadence towards the same goal of protecting and cherishing marriage
and the family as the inviolable base of the nation.
(8) The trial court must order the prosecuting attorney or fiscal and the Solicitor General to appear as counsel for the state. No
decision shall he handed down unless the Solicitor General issues a certification, which will be quoted in the decision, briefly
staring therein his reasons for his agreement or opposition, as the case may be, to the petition. The Solicitor General, along with
the prosecuting attorney, shall submit to the court such certification within fifteen (15) days from the date the case is deemed
submitted for resolution of the court. The Solicitor General shall discharge the equivalent function of the defensor
vinculi contemplated under Canon 1095.
In the instant case and applying Leouel Santos, we have already ruled to grant the petition. Such ruling becomes even more
cogent with the use of the foregoing guidelines.
WHEREFORE, the petition is GRANTED. The assailed Decision is REVERSED and SET ASIDE. The marriage of Roridel
Olaviano to Reynaldo Molina subsists and remains valid.
SO ORDERED.

G.R. No. 112019 January 4, 1995

LEOUEL SANTOS, petitioner,


vs.
THE HONORABLE COURT OF APPEALS AND JULIA ROSARIO BEDIA-SANTOS, respondents.

VITUG, J.:
Concededly a highly, if not indeed the most likely, controversial provision introduced by the Family Code is Article 36 (as
amended by E.O. No. 227 dated 17 July 1987), which declares:
Art. 36. A marriage contracted by any party who, at the time of the celebration, was psychologically incapacitated to
comply with the essential marital obligations of marriage, shall likewise be void even if such incapacity becomes
manifest only after its solemnization.
The present petition for review on certiorari, at the instance of Leouel Santos ("Leouel"), brings into fore the above
provision which is now invoked by him. Undaunted by the decisions of the court a quo 1 and the Court of Appeal, 2 Leouel
persists in beseeching its application in his attempt to have his marriage with herein private respondent, Julia Rosario
Bedia-Santos ("Julia"), declared a nullity.
It was in Iloilo City where Leouel, who then held the rank of First Lieutenant in the Philippine Army, first met Julia. The meeting
later proved to be an eventful day for Leouel and Julia. On 20 September 1986, the two exchanged vows before Municipal Trial
Court Judge Cornelio G. Lazaro of Iloilo City, followed, shortly thereafter, by a church wedding. Leouel and Julia lived with the
latter's parents at the J. Bedia Compound, La Paz, Iloilo City. On 18 July 1987, Julia gave birth to a baby boy, and he was
christened Leouel Santos, Jr. The ecstasy, however, did not last long. It was bound to happen, Leouel averred, because of the
frequent interference by Julia's parents into the young spouses family affairs. Occasionally, the couple would also start a
"quarrel" over a number of other things, like when and where the couple should start living independently from Julia's parents or
whenever Julia would express resentment on Leouel's spending a few days with his own parents.
On 18 May 1988, Julia finally left for the United Sates of America to work as a nurse despite Leouel's pleas to so dissuade her.
Seven months after her departure, or on 01 January 1989, Julia called up Leouel for the first time by long distance telephone.
She promised to return home upon the expiration of her contract in July 1989. She never did. When Leouel got a chance to visit
the United States, where he underwent a training program under the auspices of the Armed Forces of the Philippines from 01
April up to 25 August 1990, he desperately tried to locate, or to somehow get in touch with, Julia but all his efforts were of no
avail.
Having failed to get Julia to somehow come home, Leouel filed with the regional trial Court of Negros Oriental, Branch 30, a
complaint for "Voiding of marriage Under Article 36 of the Family Code" (docketed, Civil Case No. 9814). Summons was served
by publication in a newspaper of general circulation in Negros Oriental.
On 31 May 1991, respondent Julia, in her answer (through counsel), opposed the complaint and denied its allegations, claiming,
in main, that it was the petitioner who had, in fact, been irresponsible and incompetent.
A possible collusion between the parties to obtain a decree of nullity of their marriage was ruled out by the Office of the
Provincial Prosecutor (in its report to the court).
On 25 October 1991, after pre-trial conferences had repeatedly been set, albeit unsuccessfully, by the court, Julia ultimately
filed a manifestation, stating that she would neither appear nor submit evidence.
On 06 November 1991, the court a quo finally dismissed the complaint for lack of merit. 3
Leouel appealed to the Court of Appeal. The latter affirmed the decision of the trial court. 4
The petition should be denied not only because of its non-compliance with Circular 28-91, which requires a certification of nonshopping, but also for its lack of merit.
Leouel argues that the failure of Julia to return home, or at the very least to communicate with him, for more than five years are
circumstances that clearly show her being psychologically incapacitated to enter into married life. In his own words, Leouel
asserts:
. . . (T)here is no leave, there is no affection for (him) because respondent Julia Rosario Bedia-Santos failed all
these years to communicate with the petitioner. A wife who does not care to inform her husband about her
whereabouts for a period of five years, more or less, is psychologically incapacitated.
The family Code did not define the term "psychological incapacity." The deliberations during the sessions of the Family Code
Revision Committee, which has drafted the Code, can, however, provide an insight on the import of the provision.
Art. 35. The following marriages shall be void from the beginning:
xxx xxx xxx
Art. 36. . . .
(7) Those marriages contracted by any party who, at the time of the celebration, was wanting in the sufficient use of
reason or judgment to understand the essential nature of marriage or was psychologically or mentally incapacitated
to discharge the essential marital obligations, even if such lack of incapacity is made manifest after the celebration.
On subparagraph (7), which as lifted from the Canon Law, Justice (Jose B.L.) Reyes suggested that they say
"wanting in sufficient use," but Justice (Eduardo) Caguioa preferred to say "wanting in the sufficient use." On the
other hand, Justice Reyes proposed that they say "wanting in sufficient reason." Justice Caguioa, however, pointed
out that the idea is that one is not lacking in judgment but that he is lacking in the exercise of judgment. He added
that lack of judgment would make the marriage voidable. Judge (Alicia Sempio-) Diy remarked that lack of judgment
is more serious than insufficient use of judgment and yet the latter would make the marriage null and void and the
former only voidable. Justice Caguioa suggested that subparagraph (7) be modified to read:
"That contracted by any party who, at the time of the celebration, was psychologically incapacitated to
discharge the essential marital obligations, even if such lack of incapacity is made manifest after the
celebration."
Justice Caguioa explained that the phrase "was wanting in sufficient use of reason of judgment to understand the
essential nature of marriage" refers to defects in the mental faculties vitiating consent, which is not the idea in
subparagraph (7), but lack of appreciation of one's marital obligations.
Judge Diy raised the question: Since "insanity" is also a psychological or mental incapacity, why is "insanity" only a
ground for annulment and not for declaration or nullity? In reply, Justice Caguioa explained that in insanity, there is
the appearance of consent, which is the reason why it is a ground for voidable marriages, while subparagraph (7)
does not refer to consent but to the very essence of marital obligations.

Prof. (Araceli) Baviera suggested that, in subparagraph (7), the word "mentally" be deleted, with which Justice
Caguioa concurred. Judge Diy, however, prefers to retain the word "mentally."
Justice Caguioa remarked that subparagraph (7) refers to psychological impotence. Justice (Ricardo) Puno stated
that sometimes a person may be psychologically impotent with one but not with another. Justice (Leonor Ines-)
Luciano said that it is called selective impotency.
Dean (Fortunato) Gupit stated that the confusion lies in the fact that in inserting the Canon Law annulment in the
Family Code, the Committee used a language which describes a ground for voidable marriages under the Civil
Code. Justice Caguioa added that in Canon Law, there are voidable marriages under the Canon Law, there are no
voidable marriages Dean Gupit said that this is precisely the reason why they should make a distinction.
Justice Puno remarked that in Canon Law, the defects in marriage cannot be cured.
Justice Reyes pointed out that the problem is: Why is "insanity" a ground for void ab initio marriages? In reply,
Justice Caguioa explained that insanity is curable and there are lucid intervals, while psychological incapacity is not.
On another point, Justice Puno suggested that the phrase "even if such lack or incapacity is made manifest" be
modified to read "even if such lack or incapacity becomes manifest."
Justice Reyes remarked that in insanity, at the time of the marriage, it is not apparent.
Justice Caguioa stated that there are two interpretations of the phrase "psychological or mentally incapacitated"
in the first one, there is vitiation of consent because one does not know all the consequences of the marriages, and
if he had known these completely, he might not have consented to the marriage.
xxx xxx xxx
Prof. Bautista stated that he is in favor of making psychological incapacity a ground for voidable marriages since
otherwise it will encourage one who really understood the consequences of marriage to claim that he did not and to
make excuses for invalidating the marriage by acting as if he did not understand the obligations of marriage. Dean
Gupit added that it is a loose way of providing for divorce.
xxx xxx xxx
Justice Caguioa explained that his point is that in the case of incapacity by reason of defects in the mental faculties,
which is less than insanity, there is a defect in consent and, therefore, it is clear that it should be a ground for
voidable marriage because there is the appearance of consent and it is capable of convalidation for the simple
reason that there are lucid intervals and there are cases when the insanity is curable. He emphasized that
psychological incapacity does not refer to mental faculties and has nothing to do with consent; it refers to obligations
attendant to marriage.
xxx xxx xxx
On psychological incapacity, Prof. (Flerida Ruth P.) Romero inquired if they do not consider it as going to the very
essence of consent. She asked if they are really removing it from consent. In reply, Justice Caguioa explained that,
ultimately, consent in general is effected but he stressed that his point is that it is not principally a vitiation of consent
since there is a valid consent. He objected to the lumping together of the validity of the marriage celebration and the
obligations attendant to marriage, which are completely different from each other, because they require a different
capacity, which is eighteen years of age, for marriage but in contract, it is different. Justice Puno, however, felt that
psychological incapacity is still a kind of vice of consent and that it should not be classified as a voidable marriage
which is incapable of convalidation; it should be convalidated but there should be no prescription. In other words, as
long as the defect has not been cured, there is always a right to annul the marriage and if the defect has been really
cured, it should be a defense in the action for annulment so that when the action for annulment is instituted, the
issue can be raised that actually, although one might have been psychologically incapacitated, at the time the action
is brought, it is no longer true that he has no concept of the consequence of marriage.
Prof. (Esteban) Bautista raised the question: Will not cohabitation be a defense? In response, Justice Puno stated
that even the bearing of children and cohabitation should not be a sign that psychological incapacity has been
cured.
Prof. Romero opined that psychological incapacity is still insanity of a lesser degree. Justice Luciano suggested that
they invite a psychiatrist, who is the expert on this matter. Justice Caguioa, however, reiterated that psychological
incapacity is not a defect in the mind but in the understanding of the consequences of marriage, and therefore, a
psychiatrist will not be a help.
Prof. Bautista stated that, in the same manner that there is a lucid interval in insanity, there are also momentary
periods when there is an understanding of the consequences of marriage. Justice Reyes and Dean Gupit remarked
that the ground of psychological incapacity will not apply if the marriage was contracted at the time when there is
understanding of the consequences of marriage. 5
xxx xxx xxx
Judge Diy proposed that they include physical incapacity to copulate among the grounds for void marriages. Justice
Reyes commented that in some instances the impotence that in some instances the impotence is only temporary
and only with respect to a particular person. Judge Diy stated that they can specify that it is incurable. Justice
Caguioa remarked that the term "incurable" has a different meaning in law and in medicine. Judge Diy stated that
"psychological incapacity" can also be cured. Justice Caguioa, however, pointed out that "psychological incapacity"
is incurable.
Justice Puno observed that under the present draft provision, it is enough to show that at the time of the celebration
of the marriage, one was psychologically incapacitated so that later on if already he can comply with the essential
marital obligations, the marriage is still void ab initio. Justice Caguioa explained that since in divorce, the
psychological incapacity may occur after the marriage, in void marriages, it has to be at the time of the celebration of
marriage. He, however, stressed that the idea in the provision is that at the time of the celebration of the marriage,
one is psychologically incapacitated to comply with the essential marital obligations, which incapacity continues and
later becomes manifest.
Justice Puno and Judge Diy, however, pointed out that it is possible that after the marriage, one's psychological
incapacity become manifest but later on he is cured. Justice Reyes and Justice Caguioa opined that the remedy in
this case is to allow him to remarry. 6
xxx xxx xxx
Justice Puno formulated the next Article as follows:

Art. 37. A marriage contracted by any party who, at the time of the celebration, was psychologically
incapacitated, to comply with the essential obligations of marriage shall likewise be void from the
beginning even if such incapacity becomes manifest after its solemnization.
Justice Caguioa suggested that "even if" be substituted with "although." On the other hand, Prof. Bautista proposed
that the clause "although such incapacity becomes manifest after its solemnization" be deleted since it may
encourage one to create the manifestation of psychological incapacity. Justice Caguioa pointed out that, as in other
provisions, they cannot argue on the basis of abuse.
Judge Diy suggested that they also include mental and physical incapacities, which are lesser in degree than
psychological incapacity. Justice Caguioa explained that mental and physical incapacities are vices of consent while
psychological incapacity is not a species of vice or consent.
Dean Gupit read what Bishop Cruz said on the matter in the minutes of their February 9, 1984 meeting:
"On the third ground, Bishop Cruz indicated that the phrase "psychological or mental impotence" is an
invention of some churchmen who are moralists but not canonists, that is why it is considered a weak
phrase. He said that the Code of Canon Law would rather express it as "psychological or mental
incapacity to discharge . . ."
Justice Caguioa remarked that they deleted the word "mental" precisely to distinguish it from vice of consent. He
explained that "psychological incapacity" refers to lack of understanding of the essential obligations of marriage.
Justice Puno reminded the members that, at the last meeting, they have decided not to go into the classification of
"psychological incapacity" because there was a lot of debate on it and that this is precisely the reason why they
classified it as a special case.
At this point, Justice Puno, remarked that, since there having been annulments of marriages arising from
psychological incapacity, Civil Law should not reconcile with Canon Law because it is a new ground even under
Canon Law.
Prof. Romero raised the question: With this common provision in Civil Law and in Canon Law, are they going to
have a provision in the Family Code to the effect that marriages annulled or declared void by the church on the
ground of psychological incapacity is automatically annulled in Civil Law? The other members replied negatively.
Justice Puno and Prof. Romero inquired if Article 37 should be retroactive or prospective in application.
Justice Diy opined that she was for its retroactivity because it is their answer to the problem of church annulments of
marriages, which are still valid under the Civil Law. On the other hand, Justice Reyes and Justice Puno were
concerned about the avalanche of cases.
Dean Gupit suggested that they put the issue to a vote, which the Committee approved.
The members voted as follows:
(1) Justice Reyes, Justice Puno and Prof. Romero were for prospectivity.
(2) Justice Caguioa, Judge Diy, Dean Gupit, Prof. Bautista and Director Eufemio were for retroactivity.
(3) Prof. Baviera abstained.
Justice Caguioa suggested that they put in the prescriptive period of ten years within which the action for declaration
of nullity of the marriage should be filed in court. The Committee approved the suggestion. 7
It could well be that, in sum, the Family Code Revision Committee in ultimately deciding to adopt the provision with less
specificity than expected, has in fact, so designed the law as to allow some resiliency in its application. Mme. Justice Alicia V.
Sempio-Diy, a member of the Code Committee, has been quoted by Mr. Justice Josue N. Bellosillo in Salita
vs. Hon. Magtolis (G.R. No. 106429, 13 June 1994); thus: 8
The Committee did not give any examples of psychological incapacity for fear that the giving of examples would limit
the applicability of the provision under the principle of ejusdem generis. Rather, the Committee would like the judge
to interpret the provision on a case-to-case basis, guided by experience, the findings of experts and researchers in
psychological disciplines, and by decisions of church tribunals which, although not binding on the civil courts, may
be given persuasive effect since the provision was taken from Canon Law.
A part of the provision is similar to Canon 1095 of the New Code of Canon Law, 9 which reads:
Canon 1095. They are incapable of contracting marriage:
1. who lack sufficient use of reason;
2. who suffer from a grave defect of discretion of judgment concerning essentila matrimonial rights and duties, to be
given and accepted mutually;
3. who for causes of psychological nature are unable to assume the essential obligations of marriage. (Emphasis
supplied.)
Accordingly, although neither decisive nor even perhaps all that persuasive for having no juridical or secular effect, the
jurisprudence under Canon Law prevailing at the time of the code's enactment, nevertheless, cannot be dismissed as
impertinent for its value as an aid, at least, to the interpretation or construction of the codal provision.
One author, Ladislas Orsy, S.J., in his treaties,
states:

10

giving an account on how the third paragraph of Canon 1095 has been framed,

The history of the drafting of this canon does not leave any doubt that the legislator intended, indeed, to broaden the
rule. A strict and narrow norm was proposed first:
Those who cannot assume the essential obligations of marriage because of a grave psycho-sexual
anomaly (ob gravem anomaliam psychosexualem) are unable to contract marriage (cf. SCH/1975,
canon 297, a new canon, novus);
then a broader one followed:
. . . because of a grave psychological anomaly (ob gravem anomaliam psychicam) . . . (cf. SCH/1980, canon 1049);
then the same wording was retained in the text submitted to the pope (cf. SCH/1982, canon 1095, 3);

finally, a new version was promulgated:


because of causes of a psychological nature (ob causas naturae psychiae).
So the progress was from psycho-sexual to psychological anomaly, then the term anomaly was altogether
eliminated. it would be, however, incorrect to draw the conclusion that the cause of the incapacity need not be some
kind of psychological disorder; after all, normal and healthy person should be able to assume the ordinary
obligations of marriage.
Fr. Orsy concedes that the term "psychological incapacity" defies any precise definition since psychological causes can be of an
infinite variety.
In a book, entitled "Canons and Commentaries on Marriage," written by Ignatius Gramunt, Javier Hervada and LeRoy Wauck,
the following explanation appears:
This incapacity consists of the following: (a) a true inability to commit oneself to the essentials of marriage. Some
psychosexual disorders and other disorders of personality can be the psychic cause of this defect, which is here
described in legal terms. This particular type of incapacity consists of a real inability to render what is due by the
contract. This could be compared to the incapacity of a farmer to enter a binding contract to deliver the crops which
he cannot possibly reap; (b) this inability to commit oneself must refer to the essential obligations of marriage: the
conjugal act, the community of life and love, the rendering of mutual help, the procreation and education of offspring;
(c) the inability must be tantamount to a psychological abnormality. The mere difficulty of assuming these
obligations, which could be overcome by normal effort, obviously does not constitute incapacity. The canon
contemplates a true psychological disorder which incapacitates a person from giving what is due (cf. John Paul II,
Address to R. Rota, Feb. 5, 1987). However, if the marriage is to be declared invalid under this incapacity, it must be
proved not only that the person is afflicted by a psychological defect, but that the defect did in fact deprive the
person, at the moment of giving consent, of the ability to assume the essential duties of marriage and consequently
of the possibility of being bound by these duties.
Justice Sempio-Diy 11 cites with approval the work of Dr. Gerardo Veloso, a former Presiding Judge of the Metropolitan Marriage
Tribunal of the Catholic Archdiocese of Manila (Branch 1), who opines that psychological incapacity must be characterized by
(a) gravity, (b) juridical antecedence, and (c) incurability. The incapacity must be grave or serious such that the party would be
incapable of carrying out the ordinary duties required in marriage; it must be rooted in the history of the party antedating the
marriage, although the overt manifestations may emerge only after the marriage; and it must be incurable or, even if it were
otherwise, the cure would be beyond the means of the party involved.
It should be obvious, looking at all the foregoing disquisitions, including, and most importantly, the deliberations of the Family
Code Revision Committee itself, that the use of the phrase "psychological incapacity" under Article 36 of the Code has not been
meant to comprehend all such possible cases of psychoses as, likewise mentioned by some ecclesiastical authorities,
extremely low intelligence, immaturity, and like circumstances (cited in Fr. Artemio Baluma's "Void and Voidable Marriages in the
Family Code and their Parallels in Canon Law," quoting from the Diagnostic Statistical Manual of Mental Disorder by the
American Psychiatric Association; Edward Hudson's "Handbook II for Marriage Nullity Cases"). Article 36 of the Family Code
cannot be taken and construed independently of, but must stand in conjunction with, existing precepts in our law on marriage.
Thus correlated, "psychological incapacity" should refer to no less than a mental (not physical) incapacity that causes a party to
be truly incognitive of the basic marital covenants that concomitantly must be assumed and discharged by the parties to the
marriage which, as so expressed by Article 68 of the Family Code, include their mutual obligations to live together, observe love,
respect and fidelity and render help and support. There is hardly any doubt that the intendment of the law has been to confine
the meaning of "psychological incapacity" to the most serious cases of personality disorders clearly demonstrative of an utter
intensitivity or inability to give meaning and significance to the marriage. This pschologic condition must exist at the time the
marriage is celebrated. The law does not evidently envision, upon the other hand, an inability of the spouse to have sexual
relations with the other. This conclusion is implicit under Article 54 of the Family Code which considers children conceived prior
to the judicial declaration of nullity of the void marriage to be "legitimate."
The other forms of psychoses, if existing at the inception of marriage, like the state of a party being of unsound mind or
concealment of drug addiction, habitual alcoholism, homosexuality or lesbianism, merely renders the marriage
contract voidable pursuant to Article 46, Family Code. If drug addiction, habitual alcholism, lesbianism or homosexuality should
occur only during the marriage, they become mere grounds for legal separation under Article 55 of the Family Code. These
provisions of the Code, however, do not necessarily preclude the possibility of these various circumstances being themselves,
depending on the degree and severity of the disorder, indicia of psychological incapacity.
Until further statutory and jurisprudential parameters are established, every circumstance that may have some bearing on the
degree, extent, and other conditions of that incapacity must, in every case, be carefully examined and evaluated so that no
precipitate and indiscriminate nullity is peremptorily decreed. The well-considered opinions of psychiatrists, psychologists, and
persons with expertise in psychological disciplines might be helpful or even desirable.
Marriage is not an adventure but a lifetime commitment. We should continue to be reminded that innate in our society, then
enshrined in our Civil Code, and even now still indelible in Article 1 of the Family Code, is that
Art. 1. Marriage is a special contract of permanent union between a man a woman entered into in accordance with
law for the establishment of conjugal and family life. It is the foundation of the family and an inviolable social
institution whose nature, consequences, and incidents are governed by law and not subject to stipulation, except
that marriage settlements may fix the property relations during the marriage within the limits provided by this Code.
(Emphasis supplied.)
Our Constitution is no less emphatic:
Sec. 1. The State recognizes the Filipino family as the foundation of the nation. Accordingly, it shall strengthen its
solidarity and actively promote its total development.
Sec. 2. Marriage, as an inviolable social institution, is the foundation of the family and shall be protected by the
State. (Article XV, 1987 Constitution).
The above provisions express so well and so distinctly the basic nucleus of our laws on marriage and the family, and they are
doubt the tenets we still hold on to.
The factual settings in the case at bench, in no measure at all, can come close to the standards required to decree a nullity of
marriage. Undeniably and understandably, Leouel stands aggrieved, even desperate, in his present situation. Regrettably,
neither law nor society itself can always provide all the specific answers to every individual problem.
WHEREFORE, the petition is DENIED.

[G.R. No. 136490. October 19, 2000]

BRENDA B. MARCOS, petitioner, vs. WILSON G. MARCOS, respondent.


DECISION
PANGANIBAN, J.:
Psychological incapacity, as a ground for declaring the nullity of a marriage, may be established by the totality of evidence
presented. There is no requirement, however, that the respondent should be examined by a physician or a psychologist as
a conditio sine qua non for such declaration.
The Case

Before us is a Petition for Review on Certiorari under Rule 45 of the Rules of Court, assailing the July 24, 1998 Decision [1] of
the Court of Appeals (CA) in CA-GR CV No. 55588, which disposed as follows:
"WHEREFORE, the contested decision is set aside and the marriage between the parties is hereby declared valid." [2]
Also challenged by petitioner is the December 3, 1998 CA Resolution denying her Motion for Reconsideration.
Earlier, the Regional Trial Court (RTC) had ruled thus:
"WHEREFORE, the marriage between petitioner Brenda B. Marcos and respondent Wilson G. Marcos, solemnized on
September 6, 1982 in Pasig City is declared null and void ab initio pursuant to Art. 36 of the Family Code. The conjugal
properties, if any, is dissolved [sic] in accordance with Articles 126 and 129 of the same Code in relation to Articles 50, 51 and
52 relative to the delivery of the legitime of [the] parties' children. In the best interest and welfare of the minor children, their
custody is granted to petitioner subject to the visitation rights of respondent.
"Upon finality of this Decision, furnish copy each to the Office of the Civil Registrar of Pasig City where the marriage was
solemnized, the National Census and Statistics Office, Manila and the Register of Deeds of Mandaluyong City for their
appropriate action consistent with this Decision.
"SO ORDERED."
The Facts

The facts as found by the Court of Appeals are as follows:


"It was established during the trial that the parties were married twice: (1) on September 6, 1982 which was solemnized by
Judge Eriberto H. Espiritu at the Municipal Court of Pasig (Exh. A); and (2) on May 8, 1983 which was solemnized by Rev.
Eduardo L. Eleazar, Command Chaplain, at the Presidential Security Command Chapel in Malacaang Park, Manila (Exh. A1). Out of their marriage, five (5) children were born (Exhs. B, C, D, E and F).
"Appellant Wilson G. Marcos joined the Armed Forces of the Philippines in 1973. Later on, he was transferred to the Presidential
Security Command in Malacaang during the Marcos Regime. Appellee Brenda B. Marcos, on the other hand, joined the
Women's Auxilliary Corps under the Philippine Air Force in 1978. After the Edsa Revolution, both of them sought a discharge
from the military service.
"They first met sometime in 1980 when both of them were assigned at the Malacaang Palace, she as an escort of Imee
Marcos and he as a Presidential Guard of President Ferdinand Marcos. Through telephone conversations, they became
acquainted and eventually became sweethearts.
"After their marriage on September 6, 1982, they resided at No. 1702 Daisy Street, Hulo Bliss, Mandaluyong, a housing unit
which she acquired from the Bliss Development Corporation when she was still single.
"After the downfall of President Marcos, he left the military service in 1987 and then engaged in different business ventures that
did not however prosper. As a wife, she always urged him to look for work so that their children would see him, instead of her,
as the head of the family and a good provider. Due to his failure to engage in any gainful employment, they would often quarrel
and as a consequence, he would hit and beat her. He would even force her to have sex with him despite her weariness. He
would also inflict physical harm on their children for a slight mistake and was so severe in the way he chastised them. Thus, for
several times during their cohabitation, he would leave their house. In 1992, they were already living separately.
"All the while, she was engrossed in the business of selling "magic uling" and chickens. While she was still in the military, she
would first make deliveries early in the morning before going to Malacaang. When she was discharged from the military
service, she concentrated on her business. Then, she became a supplier in the Armed Forces of the Philippines until she was
able to put up a trading and construction company, NS Ness Trading and Construction Development Corporation.
"The 'straw that broke the camel's back' took place on October 16, 1994, when they had a bitter quarrel. As they were already
living separately, she did not want him to stay in their house anymore. On that day, when she saw him in their house, she was
so angry that she lambasted him. He then turned violent, inflicting physical harm on her and even on her mother who came to
her aid. The following day, October 17, 1994, she and their children left the house and sought refuge in her sister's house.
"On October 19, 1994, she submitted herself [to] medical examination at the Mandaluyong Medical Center where her injuries
were diagnosed as contusions (Exh. G, Records, 153).
"Sometime in August 1995, she together with her two sisters and driver, went to him at the Bliss unit in Mandaluyong to look for
their missing child, Niko. Upon seeing them, he got mad. After knowing the reason for their unexpected presence, he ran after
them with a samurai and even [beat] her driver.
"At the time of the filing of this case, she and their children were renting a house in Camella, Paraaque, while the appellant
was residing at the Bliss unit in Mandaluyong.
"In the case study conducted by Social Worker Sonia C. Millan, the children described their father as cruel and physically
abusive to them (Exh. UU, Records, pp. 85-100).
"The appellee submitted herself to psychologist Natividad A. Dayan, Ph.D., for psychological evaluation (Exh. YY, Records, pp.
207-216), while the appellant on the other hand, did not.
"The court a quo found the appellant to be psychologically incapacitated to perform his marital obligations mainly because of his
failure to find work to support his family and his violent attitude towards appellee and their children, x x x."[3]
Ruling of the Court of Appeals

Reversing the RTC, the CA held that psychological incapacity had not been established by the totality of the evidence
presented. It ratiocinated in this wise:

"Essential in a petition for annulment is the allegation of the root cause of the spouse's psychological incapacity which should
also be medically or clinically identified, sufficiently proven by experts and clearly explained in the decision. The incapacity must
be proven to be existing at the time of the celebration of the marriage and shown to be medically or clinically permanent or
incurable. It must also be grave enough to bring about the disability of the parties to assume the essential obligations of
marriage as set forth in Articles 68 to 71 and Articles 220 to 225 of the Family Code and such non-complied marital obligations
must similarly be alleged in the petition, established by evidence and explained in the decision.
"In the case before us, the appellant was not subjected to any psychological or psychiatric evaluation. The psychological
findings about the appellant by psychiatrist Natividad Dayan were based only on the interviews conducted with the
appellee. Expert evidence by qualified psychiatrists and clinical psychologists is essential if only to prove that the parties were
or any one of them was mentally or psychically ill to be truly incognitive of the marital obligations he or she was assuming, or as
would make him or her x x x unable to assume them. In fact, he offered testimonial evidence to show that he [was] not
psychologically incapacitated. The root cause of his supposed incapacity was not alleged in the petition, nor medically or
clinically identified as a psychological illness or sufficiently proven by an expert. Similarly, there is no evidence at all that would
show that the appellant was suffering from an incapacity which [was] psychological or mental - not physical to the extent that he
could not have known the obligations he was assuming: that the incapacity [was] grave, ha[d] preceded the marriage and [was]
incurable."[4]
Hence, this Petition.[5]
Issues

In her Memorandum,[6] petitioner presents for this Court's consideration the following issues:
"I. Whether or not the Honorable Court of Appeals could set aside the findings by the Regional Trial Court of
psychological incapacity of a respondent in a Petition for declaration of nullity of marriage simply because the
respondent did not subject himself to psychological evaluation.
II. Whether or not the totality of evidence presented and the demeanor of all the witnesses should be the basis of the
determination of the merits of the Petition." [7]
The Court's Ruling

We agree with petitioner that the personal medical or psychological examination of respondent is not a requirement for a
declaration ofpsychological incapacity. Nevertheless, the totality of the evidence she presented does not show such incapacity.
Preliminary Issue: Need for Personal Medical Examination

Petitioner contends that the testimonies and the results of various tests that were submitted to determine respondent's
psychological incapacity to perform the obligations of marriage should not have been brushed aside by the Court of Appeals,
simply because respondent had not taken those tests himself. Petitioner adds that the CA should have realized that under the
circumstances, she had no choice but to rely on other sources of information in order to determine the psychological capacity of
respondent, who had refused to submit himself to such tests.
In Republic v. CA and Molina, [8] the guidelines governing the application and the interpretation of psychological
incapacity referred to in Article 36 of the Family Code[9] were laid down by this Court as follows:
"1) The burden of proof to show the nullity of the marriage belongs to the plaintiff. Any doubt should be resolved in favor
of the existence and continuation of the marriage and against its dissolution and nullity. This is rooted in the fact that
both our Constitution and our laws cherish the validity of marriage and unity of the family. Thus, our Constitution
devotes an entire Article on the Family, recognizing it 'as the foundation of the nation.' It decrees marriage as legally
'inviolable,' thereby protecting it from dissolution at the whim of the parties. Both the family and marriage are to be
'protected' by the state.
xxxxxxxxx
2) The root cause of the psychological incapacity must be: (a) medically or clinically identified, (b) alleged in the
complaint, (c) sufficiently proven by experts and (d) clearly explained in the decision. Article 36 of the Family Code
requires that the incapacity must be psychological - not physical, although its manifestations and/or symptoms may be
physical. The evidence must convince the court that the parties, or one of them, was mentally or psychically ill to such
an extent that the person could not have known the obligations he was assuming, or knowing them, could not have
given valid assumption thereof. Although no example of such incapacity need be given here so as not to limit the
application of the provision under the principle of ejusdem generis, nevertheless such root cause must be identified as
a psychological illness and its incapacitating nature fully explained. Expert evidence may be given by qualified
psychiatrists and clinical psychologists.
3) The incapacity must be proven to be existing at 'the time of the celebration' of the marriage. The evidence must show
that the illness was existing when the parties exchanged their 'I do's.' The manifestation of the illness need not be
perceivable at such time, but the illness itself must have attached at such moment, or prior thereto.
4) Such incapacity must also be shown to be medically or clinically permanent or incurable. Such incurability may be
absolute or even relative only in regard to the other spouse, not necessarily absolutely against everyone of the same
sex. Furthermore, such incapacity must be relevant to the assumption of marriage obligations, not necessarily to those
not related to marriage, like the exercise of a profession or employment in a job. Hence, a pediatrician may be
effective in diagnosing illnesses of children and prescribing medicine to cure them but not be psychologically
capacitated to procreate, bear and raise his/her own children as an essential obligation of marriage.
5) Such illness must be grave enough to bring about the disability of the party to assume the essential obligations of
marriage. Thus, 'mild characteriological peculiarities, mood changes, occasional emotional outbursts cannot be
accepted as root causes. The illness must be shown as downright incapacity or inability, not a refusal, neglect or
difficulty, much less ill will. In other words, there is a natal or supervening disabling factor in the person, an adverse
integral element in the personality structure that effectively incapacitates the person from really accepting and thereby
complying with the obligations essential to marriage.
6) The essential marital obligations must be those embraced by Articles 68 up to 71 of the Family Code as regards the
husband and wife as well as Articles 220, 221 and 225 of the same Code in regard to parents and their children. Such
non-complied marital obligation(s) must also be stated in the petition, proven by evidence and included in the text of
the decision.
7) Interpretations given by the National Appellate Matrimonial Tribunal of the Catholic Church in the Philippines, while not
controlling or decisive, should be given great respect by our courts.
xxxxxxxxx
(8) The trial court must order the prosecuting attorney or fiscal and the Solicitor General to appear as counsel for the
state. No decision shall be handed down unless the Solicitor General issues a certification, which will be quoted in the
decision, briefly stating therein his reasons for his agreement or opposition, as the case may be, to the petition. The

Solicitor General, along with the prosecuting attorney, shall submit to the court such certification within fifteen (15)
days from the date the case is deemed submitted for resolution of the court. The Solicitor General shall discharge the
equivalent function of the defensor vinculi contemplated under Canon 1095."[10]
The guidelines incorporate the three basic requirements earlier mandated by the Court in Santos v. Court of Appeals:
"psychological incapacity must be characterized by (a) gravity (b) juridical antecedence, and (c) incurability." The foregoing
guidelines do not require that a physician examine the person to be declared psychologically incapacitated. In fact, the root
cause may be "medically or clinically identified." What is important is the presence of evidence that can adequately establish the
party's psychological condition. For indeed, if the totality of evidence presented is enough to sustain a finding of psychological
incapacity, then actual medical examination of the person concerned need not be resorted to.
[11]

Main Issue: Totality of Evidence Presented

The main question, then, is whether the totality of the evidence presented in the present case -- including the testimonies of
petitioner, the common children, petitioner's sister and the social worker -- was enough to sustain a finding that respondent was
psychologically incapacitated.
We rule in the negative. Although this Court is sufficiently convinced that respondent failed to provide material support to the
family and may have resorted to physical abuse and abandonment, the totality of his acts does not lead to a conclusion of
psychological incapacity on his part.There is absolutely no showing that his "defects" were already present at the inception of
the marriage or that they are incurable.
Verily, the behavior of respondent can be attributed to the fact that he had lost his job and was not gainfully employed for a
period of more than six years. It was during this period that he became intermittently drunk, failed to give material and moral
support, and even left the family home.
Thus, his alleged psychological illness was traced only to said period and not to the inception of the marriage. Equally
important, there is no evidence showing that his condition is incurable, especially now that he is gainfully employed as a taxi
driver.
Article 36 of the Family Code, we stress, is not to be confused with a divorce law that cuts the marital bond at the time the
causes therefor manifest themselves. It refers to a serious psychological illness afflicting a party even before the celebration of
the marriage. It is a malady so grave and so permanent as to deprive one of awareness of the duties and responsibilities of the
matrimonial bond one is about to assume.These marital obligations are those provided under Articles 68 to 71, 220, 221 and
225 of the Family Code.
Neither is Article 36 to be equated with legal separation, in which the grounds need not be rooted in psychological incapacity
but on physical violence, moral pressure, moral corruption, civil interdiction, drug addiction, habitual alcoholism, sexual infidelity,
abandonment and the like.[12]At best, the evidence presented by petitioner refers only to grounds for legal separation, not for
declaring a marriage void.
Because Article 36 has been abused as a convenient divorce law, this Court laid down the procedural requirements for its
invocation inMolina. Petitioner, however, has not faithfully observed them.
In sum, this Court cannot declare the dissolution of the marriage for failure of petitioner to show that the alleged
psychological incapacity is characterized by gravity, juridical antecedence and incurability; and for her failure to observe the
guidelines outlined in Molina.
WHEREFORE, the Petition is DENIED and assailed Decision AFFIRMED, except that portion requiring personal medical
examination as a conditio sine qua non to a finding of psychological incapacity. No costs.
SO ORDERED.

G.R. No. 149498

May 20, 2004

REPUBLIC OF THE PHILIPPINES, petitioner,


vs.
LOLITA QUINTERO-HAMANO, respondent.
DECISION
CORONA, J.:
Before us is a petition for review of the decision 1 dated August 20, 2001 of the Court of Appeals 2 affirming the decision3 dated
August 28, 1997 of the Regional Trial Court of Rizal, Branch 72, declaring as null and void the marriage contracted between
herein respondent Lolita M. Quintero-Hamano and her husband Toshio Hamano.
On June 17, 1996, respondent Lolita Quintero-Hamano filed a complaint for declaration of nullity of her marriage to her husband
Toshio Hamano, a Japanese national, on the ground of psychological incapacity.
Respondent alleged that in October 1986, she and Toshio started a common-law relationship in Japan. They later lived in the
Philippines for a month. Thereafter, Toshio went back to Japan and stayed there for half of 1987. On November 16, 1987, she
gave birth to their child.
On January 14, 1988, she and Toshio were married by Judge Isauro M. Balderia of the Municipal Trial Court of Bacoor, Cavite.
Unknown to respondent, Toshio was psychologically incapacitated to assume his marital responsibilities, which incapacity
became manifest only after the marriage. One month after their marriage, Toshio returned to Japan and promised to return by
Christmas to celebrate the holidays with his family. After sending money to respondent for two months, Toshio stopped giving
financial support. She wrote him several times but he never responded. Sometime in 1991, respondent learned from her friends
that Toshio visited the Philippines but he did not bother to see her and their child.
The summons issued to Toshio remained unserved because he was no longer residing at his given address. Consequently, on
July 8, 1996, respondent filed an ex parte motion for leave to effect service of summons by publication. The trial court granted
the motion on July 12, 1996. In August 1996, the summons, accompanied by a copy of the petition, was published in a
newspaper of general circulation giving Toshio 15 days to file his answer. Because Toshio failed to file a responsive pleading
after the lapse of 60 days from publication, respondent filed a motion dated November 5, 1996 to refer the case to the
prosecutor for investigation. The trial court granted the motion on November 7, 1996.
On November 20, 1996, prosecutor Rolando I. Gonzales filed a report finding that no collusion existed between the parties. He
prayed that the Office of the Provincial Prosecutor be allowed to intervene to ensure that the evidence submitted was not
fabricated. On February 13, 1997, the trial court granted respondents motion to present her evidence ex parte. She then
testified on how Toshio abandoned his family. She thereafter offered documentary evidence to support her testimony.
On August 28, 1997, the trial court rendered a decision, the dispositive portion of which read:
WHEREFORE, premises considered, the marriage between petitioner Lolita M. Quintero-Hamano and Toshio Hamano, is
hereby declared NULL and VOID.
The Civil Register of Bacoor, Cavite and the National Statistics Office are ordered to make proper entries into the records
of the afore-named parties pursuant to this judgment of the Court.
SO ORDERED.4
In declaring the nullity of the marriage on the ground of Toshios psychological incapacity, the trial court held that:
It is clear from the records of the case that respondent spouses failed to fulfill his obligations as husband of the petitioner
and father to his daughter. Respondent remained irresponsible and unconcerned over the needs and welfare of his family.
Such indifference, to the mind of the Court, is a clear manifestation of insensitivity and lack of respect for his wife and child
which characterizes a very immature person. Certainly, such behavior could be traced to respondents mental incapacity
and disability of entering into marital life. 5
The Office of the Solicitor General, representing herein petitioner Republic of the Philippines, appealed to the Court of Appeals
but the same was denied in a decision dated August 28, 1997, the dispositive portion of which read:
WHEREFORE, in view of the foregoing, and pursuant to applicable law and jurisprudence on the matter and evidence on
hand, judgment is hereby rendered denying the instant appeal. The decision of the court a quo is AFFIRMED. No costs.
SO ORDERED.6
The appellate court found that Toshio left respondent and their daughter a month after the celebration of the marriage, and
returned to Japan with the promise to support his family and take steps to make them Japanese citizens. But except for two
months, he never sent any support to nor communicated with them despite the letters respondent sent. He even visited the
Philippines but he did not bother to see them. Respondent, on the other hand, exerted all efforts to contact Toshio, to no avail.
The appellate court thus concluded that respondent was psychologically incapacitated to perform his marital obligations to his
family, and to "observe mutual love, respect and fidelity, and render mutual help and support" pursuant to Article 68 of the
Family Code of the Philippines. The appellate court rhetorically asked:
But what is there to preserve when the other spouse is an unwilling party to the cohesion and creation of a family as a
social inviolable institution? Why should petitioner be made to suffer in a marriage where the other spouse is not around
and worse, left them without even helping them cope up with family life and assist in the upbringing of their daughter as
required under Articles 68 to 71 of the Family Code? 7
The appellate court emphasized that this case could not be equated with Republic vs. Court of Appeals and Molina 8 and Santos
vs. Court of Appeals.9 In those cases, the spouses were Filipinos while this case involved a "mixed marriage," the husband
being a Japanese national.
Hence, this appeal by petitioner Republic based on this lone assignment of error:
I
The Court of Appeals erred in holding that respondent was able to prove the psychological incapacity of Toshio Hamano to
perform his marital obligations, despite respondents failure to comply with the guidelines laid down in the Molina case.10
According to petitioner, mere abandonment by Toshio of his family and his insensitivity to them did not automatically constitute
psychological incapacity. His behavior merely indicated simple inadequacy in the personality of a spouse falling short of
reasonable expectations. Respondent failed to prove any severe and incurable personality disorder on the part of Toshio, in
accordance with the guidelines set in Molina.
The Office of the Public Attorney, representing respondent, reiterated the ruling of the courts a quo and sought the denial of the
instant petition.

We rule in favor of petitioner.


The Court is mindful of the policy of the 1987 Constitution to protect and strengthen the family as the basic autonomous social
institution and marriage as the foundation of the family.11 Thus, any doubt should be resolved in favor of the validity of the
marriage.12
Respondent seeks to annul her marriage with Toshio on the ground of psychological incapacity. Article 36 of the Family Code of
the Philippines provides that:
Art. 36. A marriage contracted by any party who, at the time of the celebration, was psychologically incapacitated to comply with
the essential marital obligations of marriage, shall likewise be void even if such incapacity becomes manifest only after its
solemnization.
In Molina, we came up with the following guidelines in the interpretation and application of Article 36 for the guidance of the
bench and the bar:
(1) The burden of proof to show the nullity of the marriage belongs to the plaintiff. Any doubt should be resolved in favor of
the existence and continuation of the marriage and against its dissolution and nullity. This is rooted in the fact that both our
Constitution and our laws cherish the validity of marriage and unity of the family. x x x
(2) The root cause of the psychological incapacity must be: (a) medically or clinically identified, (b) alleged in the
complaint, (c) sufficiently proven by experts and (d) clearly explained in the decision. Article 36 of the Family Code
requires that the incapacity must be psychological - not physical, although its manifestations and/or symptoms may be
physical. The evidence must convince the court that the parties, or one of them, was mentally or psychically ill to such an
extent that the person could not have known the obligations he was assuming, or knowing them, could not have given
valid assumption thereof. Although no example of such incapacity need be given here so as not to limit the application of
the provision under the principle of ejusdem generis (Salita vs. Magtolis, 233 SCRA 100, June 13, 1994), nevertheless
such root cause must be identified as a psychological illness and its incapacitating nature fully explained. Expert evidence
may be given by qualified psychiatrists and clinical psychologists.
(3) The incapacity must be proven to be existing at "the time of the celebration" of the marriage. The evidence must show
that the illness was existing when the parties exchanged their "I dos." The manifestation of the illness need not be
perceivable at such time, but the illness itself must have attached at such moment, or prior thereto.
(4) Such incapacity must also be shown to be medically or clinically permanent or incurable. Such incurability may be
absolute or even relative only in regard to the other spouse, not necessarily absolutely against everyone of the same sex.
Furthermore, such incapacity must be relevant to the assumption of marriage obligations, not necessarily to those not
related to marriage, like the exercise of a profession or employment in a job. Hence, a pediatrician may be effective in
diagnosing illnesses of children and prescribing medicine to cure them but may not be psychologically capacitated to
procreate, bear and raise his/her own children as an essential obligation of marriage.
(5) Such illness must be grave enough to bring about the disability of the party to assume the essential obligations of
marriage. Thus, "mild characteriological peculiarities, mood changes, occasional emotional outbursts" cannot be accepted
as root causes. The illness must be shown as downright incapacity or inability, not a refusal, neglect or difficulty, much
less ill will. In other words, there is a natal or supervening disabling factor in the person, an adverse integral element in the
personality structure that effectively incapacitates the person from really accepting and thereby complying with the
obligations essential to marriage.
(6) The essential marital obligations must be those embraced by Articles 68 up to 71 of the Family Code as regards the
husband and wife as well as Articles 220, 221 and 225 of the same Code in regard to parents and their children. Such
non-complied marital obligation(s) must also be stated in the petition, proven by evidence and included in the text of the
decision.
(7) Interpretations given by the National Appellate Matrimonial Tribunal of the Catholic Church in the Philippines, while not
controlling or decisive, should be given great respect by our courts. x x x
(8) The trial court must order the prosecuting attorney or fiscal and the Solicitor General to appear as counsel for the
state. No decision shall be handed down unless the Solicitor General issues a certification, which will be quoted in the
decision, briefly stating therein his reasons for his agreement or opposition, as the case may be, to the petition. The
Solicitor-General, along with the prosecuting attorney, shall submit to the court such certification within fifteen (15) days
from the date the case is deemed submitted for resolution of the court. The Solicitor-General shall discharge the
equivalent function of the defensor vinculicontemplated under Canon 1095.13 (emphasis supplied)
The guidelines incorporate the three basic requirements earlier mandated by the Court in Santos: "psychological incapacity
must be characterized by (a) gravity (b) juridical antecedence and (c) incurability." 14 The foregoing guidelines do not require that
a physician examine the person to be declared psychologically incapacitated. In fact, the root cause may be "medically or
clinically identified." What is important is the presence of evidence that can adequately establish the partys psychological
condition. For indeed, if the totality of evidence presented is enough to sustain a finding of psychological incapacity, then actual
medical examination of the person concerned need not be resorted to. 15
We now proceed to determine whether respondent successfully proved Toshios psychological incapacity to fulfill his marital
responsibilities.
Petitioner showed that Toshio failed to meet his duty to live with, care for and support his family. He abandoned them a month
after his marriage to respondent. Respondent sent him several letters but he never replied. He made a trip to the Philippines but
did not care at all to see his family.
We find that the totality of evidence presented fell short of proving that Toshio was psychologically incapacitated to assume his
marital responsibilities. Toshios act of abandonment was doubtlessly irresponsible but it was never alleged nor proven to be
due to some kind of psychological illness. After respondent testified on how Toshio abandoned his family, no other evidence was
presented showing that his behavior was caused by a psychological disorder. Although, as a rule, there was no need for an
actual medical examination, it would have greatly helped respondents case had she presented evidence that medically or
clinically identified his illness. This could have been done through an expert witness. This respondent did not do.
We must remember that abandonment is also a ground for legal separation. 16 There was no showing that the case at bar was
not just an instance of abandonment in the context of legal separation. We cannot presume psychological defect from the mere
fact that Toshio abandoned his family immediately after the celebration of the marriage. As we ruled in Molina, it is not enough
to prove that a spouse failed to meet his responsibility and duty as a married person; it is essential that he must be shown to be
incapable of doing so due to some psychological,not physical, illness.17 There was no proof of a natal or supervening disabling
factor in the person, an adverse integral element in the personality structure that effectively incapacitates a person from
accepting and complying with the obligations essential to marriage. 18
According to the appellate court, the requirements in Molina and Santos do not apply here because the present case involves a
"mixed marriage," the husband being a Japanese national. We disagree. In proving psychological incapacity, we find no
distinction between an alien spouse and a Filipino spouse. We cannot be lenient in the application of the rules merely because
the spouse alleged to be psychologically incapacitated happens to be a foreign national. The medical and clinical rules to

determine psychological incapacity were formulated on the basis of studies of human behavior in general. Hence, the norms
used for determining psychological incapacity should apply to any person regardless of nationality.
In Pesca vs. Pesca,19 this Court declared that marriage is an inviolable social institution that the State cherishes and protects.
While we commiserate with respondent, terminating her marriage to her husband may not necessarily be the fitting
denouement.
WHEREFORE, the petition for review is hereby GRANTED. The decision dated August 28, 1997 of the Court of Appeals is
hereby REVERSED and SET ASIDE.
SO ORDERED.

[G.R. No. 143376. November 26, 2002]

LENI O. CHOA, petitioner, vs. ALFONSO C. CHOA, respondent.


DECISION
PANGANIBAN, J.:
Though interlocutory in character, an order denying a demurrer to evidence may be the subject of a certiorari proceeding,
provided the petitioner can show that it was issued with grave abuse of discretion; and that appeal in due course is not plain,
adequate or speedy under the circumstances. Indeed, when the plaintiffs evidence is utterly and patently insufficient to prove
the complaint, it would be capricious for a trial judge to deny the demurrer and to require the defendant to present evidence to
controvert a nonexisting case. Verily, the denial constitutes an unwelcome imposition on the courts docket and an assault on
the defendants resources and peace of mind. In short, such denial needlessly delays and, thus, effectively denies justice.
The Case
Before us is a Petition for Review on Certiorari under Rule 45 of the Rules of Court, assailing the March 16, 2000
Decision[1] and the May 22, 2000 Resolution[2] of the Court of Appeals (CA) in CA-GR SP No. 53100. The decretal portion of the
Decision reads as follows:
WHEREFORE, the instant Petition is hereby DISMISSED for lack of merit. [3]
The assailed Resolution denied petitioners Motion for Reconsideration. [4]
The Facts
Petitioner and respondent were married on March 15, 1981. Out of this union, two children were born, Cheryl Lynne and
Albryan. On October 27, 1993, respondent filed before the Regional Trial Court (RTC) of Negros Occidental, Branch 51, a
Complaint[5] for the annulment of his marriage to petitioner. The Complaint was docketed as Civil Case No. 938098. Afterwards he filed an Amended Complaint [6] dated November 8, 1993 for the declaration of nullity of his marriage to
petitioner based on her alleged psychological incapacity.
The case went to trial with respondent presenting his evidence in chief. After his last witness testified, he submitted his
Formal Offer of Exhibits[7] dated February 20, 1998. Instead of offering any objection to it, petitioner filed a Motion to Dismiss
(Demurrer to Evidence)[8] dated May 11, 1998. The lower court then allowed a number of pleadings to be filed thereafter.
Finally, the RTC issued its December 2, 1998 Order[9] denying petitioners Demurrer to Evidence. It held that [respondent]
established a quantum of evidence that the [petitioner] must controvert. [10] After her Motion for Reconsideration [11] was denied in
the March 22, 1999 Order,[12] petitioner elevated the case to the CA by way of a Petition for Certiorari, [13] docketed as CA-GR No.
53100.
Ruling of the Court of Appeals
The CA held that the denial of the demurrer was merely interlocutory; hence, certiorari under Rule 65 of the Rules of Court
was not available. The proper remedy was for the defense to present evidence; and if an unfavorable decision was handed
down later, to take an appeal therefrom. [14] In any event, no grave abuse of discretion was committed by respondent judge in
issuing the assailed Orders.[15]
The CA also ruled that the propriety of granting or denying a demurrer to evidence rests on the sound exercise of the [trial]
courts discretion.[16] Further, the [p]etitioner failed to show that the issues in the court below [had] been resolved arbitrarily or
without basis.[17]
Hence, this Petition.[18]
The Issues
In her Memorandum,[19] petitioner submits the following issues for our consideration:
1)
Upon the denial of petitioners demurrer to evidence under Rule 33 of the 1997 Rules of Civil Procedure, is she under
obligation, as a matter of inflexible rule, as what the Court of Appeals required of her, to present her evidence, and when an
unfavorable [verdict] is handed down, appeal therefrom in the manner authorized by law, despite the palpably and patently weak
and grossly insufficient or so inadequate evidence of the private respondent as plaintiff in the annulment of marriage case,
grounded on psychological incapacity under Art. 36 of The Family Code? Or under such circumstances, can the extraordinary
remedy of certiorari be directly and immediately resorted to by the petitioner; and
2)
In upholding the lower courts denial of petitioners demurrer to evidence, did the Court of Appeals wantonly violate,
ignore or disregard in a whimsical manner the doctrinal pronouncements of this Court in Molina (G.R. No. 108763, February 13,
1997, 268 SCRA 198) and Santos (G.R. No. 112019, January 14, 1995, 58 SCRA 17)? [20]
Simply stated, the issues are: (1) is certiorari available to correct an order denying a demurrer to evidence? and (2) in its
denial, did the RTC commit grave abuse of discretion by violating or ignoring the applicable law and jurisprudence?
The Courts Ruling
The Petition is meritorious.
First Issue:
Resort to Certiorari
Petitioner argues that the RTC denied her Demurrer to Evidence despite the patent weakness and gross insufficiency of
respondents evidence. Thus, she was entitled to the immediate recourse of the extraordinary remedy of certiorari. Echoing the
CA, respondent counters that appeal in due course, not certiorari, is the proper remedy.
We clarify. In general, interlocutory orders are neither appealable nor subject to certiorari proceedings.
However, this rule is not absolute. In Tadeo v. People,[21] this Court declared that appeal -- not certiorari -- in due time was
indeed the proper remedy, provided there was no grave abuse of discretion or excess of jurisdiction or oppressive exercise of
judicial authority.
In fact, Rules 41 and 65 of the Rules of Court expressly recognize this exception and allow certiorari when the lower court
acts with grave abuse of discretion in the issuance of an interlocutory order. Rule 41 provides:
No appeal may be taken from:

xxx

xxx

xxx

(c) An interlocutory order;


xxx

xxx

xxx

In all the above instances where the judgment or final order is not appealable, the aggrieved party may file an appropriate
special civil action under Rule 65. [22]
In turn, Section 1 of Rule 65 reads as follows:
SEC. 1. Petition for certiorari -- When any tribunal, board or officer exercising judicial or quasi-judicial functions has acted
without or in excess of its or his jurisdiction, or with grave abuse of discretion amounting to lack or excess of jurisdiction, and
there is no appeal, nor any plain, speedy, and adequate remedy in the ordinary course of law, a person aggrieved thereby may
file a verified petition in the proper court, alleging the facts with certainty and praying that judgment be rendered annulling or
modifying the proceedings of such tribunal, board or officer, and granting such incidental reliefs as law and justice may
require.[23]
Thus, a denial of a demurrer that is tainted with grave abuse of discretion amounting to lack or excess of jurisdiction may be
assailed through a petition for certiorari. [24] In Cruz v. People, this exception was stressed by the Court in this wise:
Admittedly, the general rule that the extraordinary writ of certiorari is not available to challenge interlocutory orders of the trial
court may be subject to exceptions. When the assailed interlocutory orders are patently erroneous or issued with grave abuse of
discretion, the remedy of certiorari lies.[25]
Second Issue:
Denial of Demurrer to Evidence
Having established that a writ of certiorari may be issued in exceptional circumstances, this Court is now tasked to
determine whether the present case falls under the exception; that is, whether the RTC indeed committed a patent error or
grave abuse of discretion in denying petitioners Demurrer to Evidence.
A demurrer to evidence is defined as an objection or exception by one of the parties in an action at law, to the effect that the
evidence which his adversary produced is insufficient in point of law (whether true or not) to make out his case or sustain the
issue.[26] The demurrer challenges the sufficiency of the plaintiffs evidence to sustain a verdict. [27] In passing upon the
sufficiency of the evidence raised in a demurrer, the court is merely required to ascertain whether there is competent or
sufficient proof to sustain the indictment or to support a verdict of guilt. [28]
We have thoroughly reviewed the records of the present case, and we are convinced that the evidence against respondent
(herein petitioner) is grossly insufficient to support any finding of psychological incapacity that would warrant a declaration of
nullity of the parties marriage.
First. Respondent claims that the filing by petitioner of a series of charges against him are proof of the latters psychological
incapacity to comply with the essential obligations of marriage. These charges included Complaints for perjury,[29] false
testimony,[30] concubinage[31] and deportation.[32] According to him, the filing and the prosecution of these cases clearly showed
that his wife (herein petitioner) wanted not only to put him behind bars, but also to banish him from the country. He contends
that this is very abnormal for a wife who, instead of protecting the name and integrity of her husband as the father of her
children, had acted to the contrary. [33]
We do not agree. The documents presented by respondent during the trial do not in any way show the alleged
psychological incapacity of his wife. It is the height of absurdity and inequity to condemn her as psychologically incapacitated to
fulfill her marital obligations, simply because she filed cases against him. The evidence presented, even if taken as true, merely
establishes the prosecution of the cases against him. To rule that the filings are sufficient to establish her psychological
incapacity is not only totally erroneous, but also grave abuse of discretion bordering on absurdity.
Second. Neither is the testimony of respondent, taken by itself or in conjunction with his documentary offerings, sufficient to
prove petitioners alleged psychological incapacity. He testified in these words:
Q

Will you please tell us or explain to the Court what do you mean by psychologically incapacitated to comply with the
essential obligations of marriage. What do you mean by that?

Because before our marriage she was already on the family way, so at that time she even want it aborted by taking
pills. She was even immature, carefree, and she lacked the intention of procreative sexuality. [34]

xxx

xxx

xxx

ATTY. CHUA:
And you consider her that she was carefree, she is psychologically incapacitated? Will you please elaborate on this
what you mean by carefree approximating psychologically incapacitated?
ATTY. MIRANO:
I think we better ask the witness what he means by carefree.
ATTY. CHUA:
Okay.
COURT:
Witness may answer.
WITNESS:
She does not help in the household chores, she does not take care of the child, she wants me to hire an attendant in
order to take care of the child. Even when the children were sick she does not bother to let the children see a doctor.
[35]

xxx

xxx

xxx

STENOGRAPHER (reads back the question of Atty. Chua):


ATTY. CHUA:
Now. From the time of courtship up to the time of your marriage to the defendant, did you notice any characteristic or
traits which you consider as psychological incapacity?
WITNESS:
Sometimes when I cannot visit at her house she gets mad at me, and she wont talk to me when I call her up by
telephone. So, all she wanted for me to visit her everytime and even at the time when I am busy with some other
things. So, I think that is all.[36]
Even if taken as true, the testimony of respondent basically complains about three aspects of petitioners personality;
namely, her alleged (1) lack of attention to their children, (2) immaturity and (3) lack of an intention of procreative
sexuality. None of these three, singly or collectively, constitutes psychological incapacity. Far from it.

In Santos v. CA,[37] this Court clearly explained that psychological incapacity must be characterized by (a) gravity, (b)
juridical antecedence and (c) incurability. [38] Said the Court:
It should be obvious, looking at all the foregoing disquisitions, including, and most importantly, the deliberations of the Family
Code Revision Committee itself, that the use of the phrase psychological incapacity under Article 36 of the Code has not been
meant to comprehend all such possible cases of psychoses as, likewise mentioned by some ecclesiastical authorities,
extremely low intelligence, immaturity, and like circumstances (cited in Fr. Artemio Baluma's Void and Voidable Marriages in the
Family Code and their Parallels in Canon Law, quoting from the Diagnostic Statistical Manual of Mental Disorder by the
American Psychiatric Association; Edward Hudson's Handbook II for Marriage Nullity Cases). Article 36 of the Family Code
cannot be taken and construed independently of but must stand in conjunction with, existing precepts in our law on marriage.
Thus correlated, psychological incapacity should refer to no less than a mental (not physical) incapacity that causes a party to
be truly incognitive of the basic marital covenants that concomitantly must be assumed and discharged by the parties to the
marriage which, as so expressed by Article 68 of the Family Code, include their mutual obligations to live together, observe love,
respect and fidelity and render help and support. There is hardly any doubt that the intendment of the law has been to confine
the meaning of psychological incapacity to the most serious cases of personality disorders clearly demonstrative of an utter
insensitivity or inability to give meaning and significance to the marriage. This psychologic condition must exist at the time the
marriage is celebrated.[39]
Furthermore, in Republic v. Molina,[40] we ruled that the psychological incapacity must be more than just a difficulty, a
refusal or a neglect in the performance of some marital obligations. We stressed that a mere showing of irreconcilable
differences and conflicting personalities in no wise constitutes psychological incapacity.
In the case at bar, the evidence adduced by respondent merely shows that he and his wife could not get along with each
other. There was absolutely no showing of the gravity or juridical antecedence or incurability of the problems besetting their
marital union.
Sorely lacking in respondents evidence is proof that the psychological incapacity was grave enough to bring about the
disability of a party to assume the essential obligations of marriage. In Molina, we affirmed that mild characterological
peculiarities, mood changes and occasional emotional outbursts cannot be accepted as root causes of psychological
incapacity. The illness must be shown as downright incapacity or inability, not a refusal, neglect or difficulty, much less ill will. In
other words, there should be a natal or supervening disabling factor in the person, an adverse integral element in the
personality structure that effectively incapacitates the person from really accepting and thereby complying with the obligations
essential to marriage.[41]
Respondents pious peroration that petitioner lacked the intention of procreative sexuality is easily belied by the fact that
two children were born during their union. Moreover, there is absolutely no showing that the alleged defect was already
existing at the time of the celebration of the marriage.
Third. Most telling is the insufficiency, if not incompetency, of the supposed expert testimony presented by respondent. His
witness, Dr. Antonio M. Gauzon, utterly failed to identify and prove the root cause of the alleged psychological
incapacity. Specifically, his testimony did not show that the incapacity, if true, was medically or clinically permanent or
incurable. Neither did he testify that it was grave enough to bring about the disability of the party to assume the essential
obligations of marriage. The pertinent portions of his testimony are quoted thus:
ATTY. CHUA:
And then finally and ultimately you reached the conclusion that both parties, meaning the husband and the wife in the
present case have a personality which is normal. That is your conclusion?
WITNESS:
They are normal, but they cannot mix together.
Q.

So as a general proposition, both of them are of normal personality, only that they are not compatible with each
other?

A.
Q.

Yes.
And by normal personality, you mean that neither of them suffer from any personality disorder, bordering on
abnormality?

A.

Yes.

Q.

But Doctor, is not a fact or a fact of life, that no couple could be or are perfectly match?

A.

Precisely, if there is a problem, marital problem, there should be somebody who knows how to handle marriage, that
should try to intervene.

Q.

You mean expert advise or services should be needed by the couple?

A.

Yes.

Q.

Now, if the couple are mature enough and each of them practises what we call maximum tolerance and give and
take, will that serve the purpose?

A.

That would served the purpose of getting well.

Q.

Yes?

A.

Yes.

Q.

Meaning to say that the incompatibility could be harmonized?

A.

Yes, because they are supposedly normal, but both of them are personally disordered. It cannot be
harmonized. So this case, if only they have tried professional help to take care of their marital problem, it could have
been solved.

Q.
A.

Or the situation could have been remedied?


Yes. But I would like to say that it must be somebody who is an expert. Not just any from Tom, Dick and Harry
could handle this. That means from the very beginning they have personalities which they were incompatible. So if
anybody would handle that, they will not mix, they will be always quarreling with each other. They should not have
got married.[42]

xxx
Q.

xxx

xxx

Yes. So in this present case, your expert opinion was sought by the plaintiff, and you found out that both are
normal?

A.

With different personalities. So that they were incompatible.

Q.

Normal, simply incompatible.

A.

Yes, with personalities different from each other, which I mentioned there in my last page. That they are like oil and
water, immiscible. Like oil and water, they will not mix.

Q.

You also mentioned that the plaintiff. Meaning to say the husband told you about the frequent quarrels had with the
wife. Did he ever tell you that was a serious or major quarrel?

A.

Actually there was no major quarrel. It was all petty quarrels.[43]

xxx

xxx

Q.

So the problem of this couple is fundamentally a conflicting personalities?

A.

Yes.[44]

xxx

xxx

Q.

Now, you mentioned that you maybe able to make them reconcile?

A.

Yes.

Q.

You mean that given the time and opportunity, things could be worked out?

A.

Yes.

Q.

xxx

xxx

You mean reconciliation at this stage with expert services, and the advise of those who possess the necessary
[expertise] could be worked out?
Yes, as I said it can be done by therapy. Family therapy.[45]

A.
xxx

xxx

Q.

Doctor, you draw your conclusion that there is psychological inc[a]pacity existing in this case?

A.

Yes.

Q.

Because of the

A.

The incompatibility.

Q.

Incompatibility.

A.

Yes.[46]

xxx

His testimony established merely that the spouses had an incompatibility, a defect that could possibly be treated or
alleviated through psychotherapy. We need not expound further on the patent insufficiency of the expert testimony to establish
the psychological incapacity of petitioner.
Furthermore, the assessment of petitioner by Dr. Gauzon was based merely on descriptions communicated to him by
respondent. The doctor never conducted any psychological examination of her. Neither did he ever claim to have done so. In
fact, his Professional Opinion [47]began with the statement [I]f what Alfonso Choa said about his wife Leni is true, x x x. [48] The
expert witness testified thus:
ATTY. CHUA
Q

Doctor, in this professional opinion of yours, you gathered most of your material data from the plaintiff who is the
husband?

WITNESS
A

Yes. By the way, I requested the husband Alfonso, if it was possible for me to interview Leni, and he said, he doesnt
know.

ATTY. CHUA
Q

He doesnt know. Now, Doctor if we were to request you to conduct the same personal interview and written
psychological examination on the part of the wife, [w]ould you be willing to do that?

WITNESS
A

Sure for a fee. I maybe able to make them reconcile. [49]

Obviously, Dr. Gauzon had no personal knowledge of the facts he testified to, as these had merely been relayed to him by
respondent. The former was working on pure suppositions and secondhand information fed to him by one side. Consequently,
his testimony can be dismissed as unscientific and unreliable.
Dr. Gauzon tried to save his credibility by asserting that he was able to assess petitioners character, not only through the
descriptions given by respondent, but also through the formers at least fifteen hours [50] of study of the voluminous transcript of
records of this case. Even if it took the good doctor a whole day or a whole week to examine the records of this case, we still
find his assessment of petitioners psychological state sorely insufficient and methodologically flawed.
As to respondents argument -- that because Dr. Gauzons testimony had never been objected to, the objection raised
thereafter was deemed waived -- the Supreme Court has already ruled on the matter. It held that although the question of
admissibility of evidence could not be raised for the first time on appeal, hearsay or unreliable evidence should be disregarded
whether objected to or not, because it has no probative value. [51]
We are, of course, mindful of the ruling that a medical examination is not a conditio sine qua non to a finding of
psychological incapacity, so long as the totality of evidence presented is enough to establish the incapacity adequately. [52] Here,
however, the totality of evidence presented by respondent was completely insufficient to sustain a finding of psychological
incapacity -- more so without any medical, psychiatric or psychological examination.
The trial court should have carefully studied and assessed the evidence presented by respondent and taken into account the
prevailing jurisprudence on the matter. It could then have easily concluded, as we conclude now, that it was useless to proceed
further with the tedious process of hearing contravening proof. His evidence was obviously, grossly and clearly insufficient to
support a declaration of nullity of marriage based on psychological incapacity. Withal, it was grave abuse of discretion for the
RTC to deny the Demurrer and to violate or ignore this Courts rulings in point. Indeed, continuing the process of litigation would
have been a total waste of time and money for the parties and an unwelcome imposition on the trial courts docket.
We have already ruled that grave abuse of discretion may arise when a lower court or tribunal violates or contravenes the
Constitution, the law or existing jurisprudence. [53] Any decision, order or resolution of a lower court tantamount to overruling a
judicial pronouncement of the highest Court is unmistakably a very grave abuse of discretion. [54]
There is no reason to believe that an appeal would prove to be a plain, speedy or adequate remedy in the case at bar. An
appeal would not promptly relieve petitioner from the injurious effects of the patently mistaken Orders maintaining the baseless
action of respondent. It would only compel her to go needlessly through a protracted trial, which would further clog the court
dockets with another futile case.[55]
WHEREFORE, the Petition is hereby GRANTED and the assailed CA Decision REVERSED and SET ASIDE. Respondents
Demurrer to Evidence is GRANTED, and the case for declaration of nullity of marriage based on the alleged psychological
incapacity of petitioner isDISMISSED. No pronouncement as to costs.
SO ORDERED.

G.R. No. 155800

March 10, 2006

LEONILO ANTONIO Petitioner,


vs.
MARIE IVONNE F. REYES, Respondent.
DECISION

TINGA, J.:
Statistics never lie, but lovers often do, quipped a sage. This sad truth has unsettled many a love transformed into matrimony.
Any sort of deception between spouses, no matter the gravity, is always disquieting. Deceit to the depth and breadth unveiled in
the following pages, dark and irrational as in the modern noir tale, dims any trace of certitude on the guilty spouses capability to
fulfill the marital obligations even more.
The Petition for Review on Certiorari assails the Decision1 and Resolution2 of the Court of Appeals dated 29 November 2001
and 24 October 2002. The Court of Appeals had reversed the judgment 3 of the Regional Trial Court (RTC) of Makati declaring
the marriage of Leonilo N. Antonio (petitioner) and Marie Ivonne F. Reyes (respondent), null and void. After careful
consideration, we reverse and affirm instead the trial court.
Antecedent Facts
Petitioner and respondent met in August 1989 when petitioner was 26 years old and respondent was 36 years of age. Barely a
year after their first meeting, they got married before a minister of the Gospel 4 at the Manila City Hall, and through a subsequent
church wedding5 at the Sta. Rosa de Lima Parish, Bagong Ilog, Pasig, Metro Manila on 6 December 1990. 6 Out of their union, a
child was born on 19 April 1991, who sadly died five (5) months later.
On 8 March 1993,7 petitioner filed a petition to have his marriage to respondent declared null and void. He anchored his petition
for nullity on Article 36 of the Family Code alleging that respondent was psychologically incapacitated to comply with the
essential obligations of marriage. He asserted that respondents incapacity existed at the time their marriage was celebrated
and still subsists up to the present.8
As manifestations of respondents alleged psychological incapacity, petitioner claimed that respondent persistently lied about
herself, the people around her, her occupation, income, educational attainment and other events or things, 9 to wit:
(1) She concealed the fact that she previously gave birth to an illegitimate son, 10 and instead introduced the boy to petitioner as
the adopted child of her family. She only confessed the truth about the boys parentage when petitioner learned about it from
other sources after their marriage.11
(2) She fabricated a story that her brother-in-law, Edwin David, attempted to rape and kill her when in fact, no such incident
occurred.12
(3) She misrepresented herself as a psychiatrist to her obstetrician, Dr. Consuelo Gardiner, and told some of her friends that
she graduated with a degree in psychology, when she was neither.13
(4) She claimed to be a singer or a free-lance voice talent affiliated with Blackgold Recording Company (Blackgold); yet, not a
single member of her family ever witnessed her alleged singing activities with the group. In the same vein, she postulated that a
luncheon show was held at the Philippine Village Hotel in her honor and even presented an invitation to that effect 14 but
petitioner discovered per certification by the Director of Sales of said hotel that no such occasion had taken place. 15
(5) She invented friends named Babes Santos and Via Marquez, and under those names, sent lengthy letters to petitioner
claiming to be from Blackgold and touting her as the "number one moneymaker" in the commercial industry worth P2
million.16 Petitioner later found out that respondent herself was the one who wrote and sent the letters to him when she admitted
the truth in one of their quarrels.17 He likewise realized that Babes Santos and Via Marquez were only figments of her
imagination when he discovered they were not known in or connected with Blackgold. 18
(6) She represented herself as a person of greater means, thus, she altered her payslip to make it appear that she earned a
higher income. She bought a sala set from a public market but told petitioner that she acquired it from a famous furniture
dealer.19 She spent lavishly on unnecessary items and ended up borrowing money from other people on false pretexts. 20
(7) She exhibited insecurities and jealousies over him to the extent of calling up his officemates to monitor his whereabouts.
When he could no longer take her unusual behavior, he separated from her in August 1991. He tried to attempt a reconciliation
but since her behavior did not change, he finally left her for good in November 1991. 21
In support of his petition, petitioner presented Dr. Dante Herrera Abcede (Dr. Abcede), a psychiatrist, and Dr. Arnulfo V.
Lopez (Dr. Lopez), a clinical psychologist, who stated, based on the tests they conducted, that petitioner was essentially a
normal, introspective, shy and conservative type of person. On the other hand, they observed that respondents persistent and
constant lying
to petitioner was abnormal or pathological. It undermined the basic relationship that should be based on love, trust and
respect.22 They further asserted that respondents extreme jealousy was also pathological. It reached the point of paranoia since
there was no actual basis for her to suspect that petitioner was having an affair with another woman. They concluded based on
the foregoing that respondent was psychologically incapacitated to perform her essential marital obligations. 23
In opposing the petition, respondent claimed that she performed her marital obligations by attending to all the needs of her
husband. She asserted that there was no truth to the allegation that she fabricated stories, told lies and invented
personalities.24 She presented her version, thus:
(1) She concealed her child by another man from petitioner because she was afraid of losing her husband. 25
(2) She told petitioner about Davids attempt to rape and kill her because she surmised such intent from Davids act of touching
her back and ogling her from head to foot. 26
(3) She was actually a BS Banking and Finance graduate and had been teaching psychology at the Pasig Catholic School for
two (2) years.27
(4) She was a free-lance voice talent of Aris de las Alas, an executive producer of Channel 9 and she had done three (3)
commercials with McCann Erickson for the advertisement of Coca-cola, Johnson & Johnson, and Traders Royal Bank. She told
petitioner she was a Blackgold recording artist although she was not under contract with the company, yet she reported to the
Blackgold office after office hours. She claimed that a luncheon show was indeed held in her honor at the Philippine Village
Hotel on 8 December 1979.28
(5) She vowed that the letters sent to petitioner were not written by her and the writers thereof were not fictitious. Bea Marquez
Recto of the Recto political clan was a resident of the United States while Babes Santos was employed with Saniwares. 29
(6) She admitted that she called up an officemate of her husband but averred that she merely asked the latter in a diplomatic
matter if she was the one asking for chocolates from petitioner, and not to monitor her husbands whereabouts. 30
(7) She belied the allegation that she spent lavishly as she supported almost ten people from her monthly budget
of P7,000.00.31
In fine, respondent argued that apart from her non-disclosure of a child prior to their marriage, the other lies attributed to her by
petitioner were mostly hearsay and unconvincing. Her stance was that the totality of the evidence presented is not sufficient for
a finding of psychological incapacity on her part. 32

In addition, respondent presented Dr. Antonio Efren Reyes (Dr. Reyes), a psychiatrist, to refute the allegations anent her
psychological condition. Dr. Reyes testified that the series of tests conducted by his assistant, 33together with the screening
procedures and the Comprehensive Psycho-Pathological Rating Scale (CPRS) he himself conducted, led him to conclude that
respondent was not psychologically incapacitated to perform the essential marital obligations. He postulated that regressive
behavior, gross neuroticism, psychotic tendencies, and poor control of impulses, which are signs that might point to the
presence of disabling trends, were not elicited from respondent. 34
In rebuttal, Dr. Lopez asseverated that there were flaws in the evaluation conducted by Dr. Reyes as (i) he was not the one who
administered and interpreted respondents psychological evaluation, and (ii) he made use of only one instrument called CPRS
which was not reliable because a good liar can fake the results of such test. 35
After trial, the lower court gave credence to petitioners evidence and held that respondents propensity to lying about almost
anythingher occupation, state of health, singing abilities and her income, among othershad been duly established. According
to the trial court, respondents fantastic ability to invent and fabricate stories and personalities enabled her to live in a world of
make-believe. This made her psychologically incapacitated as it rendered her incapable of giving meaning and significance to
her marriage.36 The trial court thus declared the marriage between petitioner and respondent null and void.
Shortly before the trial court rendered its decision, the Metropolitan Tribunal of the Archdiocese of Manila annulled the Catholic
marriage of the parties, on the ground of lack of due discretion on the part of the parties. 37 During the pendency of the appeal
before the Court of Appeals, the Metropolitan Tribunals ruling was affirmed with modification by both the National Appellate
Matrimonial Tribunal, which held instead that only respondent was impaired by a lack of due discretion. 38 Subsequently, the
decision of the National Appellate Matrimonial Tribunal was upheld by the Roman Rota of the Vatican. 39
Petitioner duly alerted the Court of Appeals of these rulings by the Catholic tribunals. Still, the appellate court reversed the
RTCs judgment. While conceding that respondent may not have been completely honest with petitioner, the Court of Appeals
nevertheless held that the totality of the evidence presented was insufficient to establish respondents psychological incapacity.
It declared that the requirements in the case of Republic v. Court of Appeals40 governing the application and interpretation of
psychological incapacity had not been satisfied.
Taking exception to the appellate courts pronouncement, petitioner elevated the case to this Court. He contends herein that the
evidence conclusively establish respondents psychological incapacity.
In considering the merit of this petition, the Court is heavily influenced by the credence accorded by the RTC to the factual
allegations of petitioner.41 It is a settled principle of civil procedure that the conclusions of the trial court regarding the credibility
of witnesses are entitled to great respect from the appellate courts because the trial court had an opportunity to observe the
demeanor of witnesses while giving testimony which may indicate their candor or lack thereof. 42 The Court is likewise guided by
the fact that the Court of Appeals did not dispute the veracity of the evidence presented by petitioner. Instead, the appellate
court concluded that such evidence was not sufficient to establish the psychological incapacity of respondent. 43
Thus, the Court is impelled to accept the factual version of petitioner as the operative facts. Still, the crucial question remains as
to whether the state of facts as presented by petitioner sufficiently meets the standards set for the declaration of nullity of a
marriage under Article 36 of the Family Code. These standards were definitively laid down in the Courts 1997 ruling in Republic
v. Court of Appeals44 (also known as the Molina case45), and indeed the Court of Appeals cited the Molina guidelines in reversing
the RTC in the case at bar.46 Since Molinawas decided in 1997, the Supreme Court has yet to squarely affirm the declaration of
nullity of marriage under Article 36 of the Family Code. 47 In fact, even before Molina was handed down, there was only one
case, Chi Ming Tsoi v. Court of Appeals,48 wherein the Court definitively concluded that a spouse was psychologically
incapacitated under Article 36.
This state of jurisprudential affairs may have led to the misperception that the remedy afforded by Article 36 of the Family Code
is hollow, insofar as the Supreme Court is concerned. 49 Yet what Molina and the succeeding cases did ordain was a set of
guidelines which, while undoubtedly onerous on the petitioner seeking the declaration of nullity, still leave room for a decree of
nullity under the proper circumstances. Molina did not foreclose the grant of a decree of nullity under Article 36, even as it raised
the bar for its allowance.
Legal Guides to Understanding Article 36
Article 36 of the Family Code states that "[a] marriage contracted by any party who, at the time of the celebration, was
psychologically incapacitated to comply with the essential marital obligations of marriage, shall likewise be void even if such
incapacity becomes manifest only after its solemnization." 50 The concept of psychological incapacity as a ground for nullity of
marriage is novel in our body of laws, although mental incapacity has long been recognized as a ground for the dissolution of a
marriage.
The Spanish Civil Code of 1889 prohibited from contracting marriage persons "who are not in the full enjoyment of their reason
at the time of contracting marriage."51 Marriages with such persons were ordained as void, 52 in the same class as marriages with
underage parties and persons already married, among others. A partys mental capacity was not a ground for divorce under the
Divorce Law of 1917,53 but a marriage where "either party was of unsound mind" at the time of its celebration was cited as an
"annullable marriage" under the Marriage Law of 1929. 54 Divorce on the ground of a spouses incurable insanity was permitted
under the divorce law enacted during the Japanese occupation. 55 Upon the enactment of the Civil Code in 1950, a marriage
contracted by a party of "unsound mind" was classified under Article 85 of the Civil Code as a voidable marriage. 56 The mental
capacity, or lack thereof, of the marrying spouse was not among the grounds for declaring a marriage void ab initio.57 Similarly,
among the marriages classified as voidable under Article 45 (2) of the Family Code is one contracted by a party of unsound
mind.58
Such cause for the annulment of marriage is recognized as a vice of consent, just like insanity impinges on consent freely given
which is one of the essential requisites of a contract. 59 The initial common consensus on psychological incapacity under Article
36 of the Family Code was that it did not constitute a specie of vice of consent. Justices Sempio-Diy and Caguioa, both
members of the Family Code revision committee that drafted the Code, have opined that psychological incapacity is not a vice
of consent, and conceded that the spouse may have given free and voluntary consent to a marriage but was nonetheless
incapable of fulfilling such rights and obligations. 60 Dr. Tolentino likewise stated in the 1990 edition of his commentaries on the
Family Code that this "psychological incapacity to comply with the essential marital obligations does not affect the consent to
the marriage."61
There were initial criticisms of this original understanding of Article 36 as phrased by the Family Code committee. Tolentino
opined that "psychologically incapacity to comply would not be
juridically different from physical incapacity of consummating the marriage, which makes the marriage only voidable under
Article 45 (5) of the Civil Code x x x [and thus] should have been a cause for annulment of the marriage only." 62 At the same
time, Tolentino noted "[it] would be different if it were psychological incapacity to understand the essential marital obligations,
because then this would amount to lack of consent to the marriage." 63 These concerns though were answered, beginning
with Santos v. Court of Appeals,64 wherein the Court, through Justice Vitug, acknowledged that "psychological incapacity should
refer to no less than a mental (not physical) incapacity that causes a party to be truly incognitive of the basic marital
covenants that concomitantly must be assumed and discharged by the parties to the marriage." 65
The notion that psychological incapacity pertains to the inability to understand the obligations of marriage, as opposed to a mere
inability to comply with them, was further affirmed in the Molina66 case. Therein, the Court, through then Justice (now Chief

Justice) Panganiban observed that "[t]he evidence [to establish psychological incapacity] must convince the court that the
parties, or one of them, was mentally or psychically ill to such extent that the person could not have known the obligations he
was assuming, or knowing them, could not have given valid assumption thereto." 67 Jurisprudence since then has recognized
that psychological incapacity "is a malady so grave and permanent as to deprive one of awareness of the duties and
responsibilities of the matrimonial bond one is about to assume." 68
It might seem that this present understanding of psychological incapacity deviates from the literal wording of Article 36, with its
central phase reading "psychologically incapacitated to comply
with the essential marital obligations of marriage." 69 At the same time, it has been consistently recognized by this Court that the
intent of the Family Code committee was to design the law as to allow some resiliency in its application, by avoiding specific
examples that would limit the applicability of the provision under the principle ofejusdem generis. Rather, the preference of the
revision committee was for "the judge to interpret the provision ona case-to-case basis, guided by experience, in the
findings of experts and researchers in psychological disciplines, and by decisions of church tribunals which, although
not binding on
the civil courts, may be given persuasive effect since the provision was taken from Canon Law." 70
We likewise observed in Republic v. Dagdag:71
Whether or not psychological incapacity exists in a given case calling for annulment of a marriage, depends crucially, more than
in any field of the law, on the facts of the case. Each case must be judged, not on the basis of a priori assumptions,
predilections or generalizations but according to its own facts. In regard to psychological incapacity as a ground for annulment
of marriage, it is trite to say that no case is on "all fours" with another case. The trial judge must take pains in examining the
factual milieu and the appellate court must, as much as possible, avoid substituting its own judgment for that of the trial court. 72
The Court thus acknowledges that the definition of psychological incapacity, as intended by the revision committee, was not
cast in intractable specifics. Judicial understanding of psychological incapacity may be informed by evolving standards, taking
into account the particulars of each case, current trends in psychological and even canonical thought, and experience. It is
under the auspices of the deliberate ambiguity of the framers that the Court has developed the Molina rules, which have been
consistently applied since 1997. Molina has proven indubitably useful in providing a unitary framework that guides courts in
adjudicating petitions for declaration of nullity under Article 36. At the same time, the Molina guidelines are not set in stone, the
clear legislative intent mandating a case-to-case perception of each situation, and Molina itself arising from this evolutionary
understanding of Article 36. There is no cause to disavow Molina at present, and indeed the disposition of this case shall rely
primarily on that precedent. There is need though to emphasize other perspectives as well which should govern the disposition
of petitions for declaration of nullity under Article 36.
Of particular notice has been the citation of the Court, first in Santos then in Molina, of the considered opinion of canon law
experts in the interpretation of psychological incapacity. This is but unavoidable, considering that the Family Code committee
had bluntly acknowledged that the concept of psychological incapacity was derived from canon law, 73 and as one member
admitted, enacted as a solution to the problem of marriages already annulled by the Catholic Church but still existent under civil
law.74 It would be disingenuous to disregard the influence of Catholic Church doctrine in the formulation and subsequent
understanding of Article 36, and the Court has expressly acknowledged that interpretations given by the National Appellate
Matrimonial Tribunal of the local Church, while not controlling or decisive, should be given great respect by our courts. 75 Still, it
must be emphasized that the Catholic Church is hardly the sole source of influence in the interpretation of Article 36. Even
though the concept may have been derived from canon law, its incorporation into the Family Code and subsequent judicial
interpretation occurred in wholly secular progression. Indeed, while Church thought on psychological incapacity is merely
persuasive on the trial courts, judicial decisions of this Court interpreting psychological incapacity are binding on lower courts. 76
Now is also opportune time to comment on another common legal guide utilized in the adjudication of petitions for declaration of
nullity under Article 36. All too frequently, this Court and lower courts, in denying petitions of the kind, have favorably cited
Sections 1 and 2, Article XV of the Constitution, which respectively state that "[t]he State recognizes the Filipino family as the
foundation of the nation. Accordingly, it shall strengthen its solidarity and actively promote its total developmen[t]," and that
"[m]arriage, as an inviolable social institution, is the foundation of the family and shall be protected by the State." These
provisions highlight the importance of the family and the constitutional protection accorded to the institution of marriage.
But the Constitution itself does not establish the parameters of state protection to marriage as a social institution and the
foundation of the family. It remains the province of the legislature to define all legal aspects of marriage and prescribe the
strategy and the modalities to protect it, based on whatever socio-political influences it deems proper, and subject of course to
the qualification that such legislative enactment itself adheres to the Constitution and the Bill of Rights. This being the case, it
also falls on the legislature to put into operation the constitutional provisions that protect marriage and the family. This has been
accomplished at present through the enactment of the Family Code, which defines marriage and the family, spells out the
corresponding legal effects, imposes the limitations that affect married and family life, as well as prescribes the grounds for
declaration of nullity and those for legal separation. While it may appear that the judicial denial of a petition for declaration of
nullity is reflective of the constitutional mandate to protect marriage, such action in fact merely enforces a statutory definition of
marriage, not a constitutionally ordained decree of what marriage is. Indeed, if circumstances warrant, Sections 1 and 2 of
Article XV need not be the only constitutional considerations to be taken into account in resolving a petition for declaration of
nullity.
Indeed, Article 36 of the Family Code, in classifying marriages contracted by a psychologically incapacitated person as a nullity,
should be deemed as an implement of this constitutional protection of marriage. Given the avowed State interest in promoting
marriage as the foundation of the family, which in turn serves as the foundation of the nation, there is a corresponding interest
for the State to defend against marriages ill-equipped to promote family life. Void ab initio marriages under Article 36 do not
further the initiatives of the State concerning marriage and family, as they promote wedlock among persons who, for reasons
independent of their will, are not capacitated to understand or comply with the essential obligations of marriage.
These are the legal premises that inform us as we decide the present petition.
Molina Guidelines As Applied in This Case
As stated earlier, Molina established the guidelines presently recognized in the judicial disposition of petitions for nullity under
Article 36. The Court has consistently applied Molina since its promulgation in 1997, and the guidelines therein operate as the
general rules. They warrant citation in full:
1) The burden of proof to show the nullity of the marriage belongs to the plaintiff. Any doubt should be resolved in favor of
the existence and continuation of the marriage and against its dissolution and nullity. This is rooted in the fact that both our
Constitution and our laws cherish the validity of marriage and unity of the family. Thus, our Constitution devotes an entire
Article on the Family, recognizing it "as the foundation of the nation." It decrees marriage as legally "inviolable," thereby
protecting it from dissolution at the whim of the parties. Both the family and marriage are to be "protected" by the state.
The Family Code echoes this constitutional edict on marriage and the family and emphasizes their permanence,
inviolability and solidarity.
2) The root cause of the psychological incapacity must be: (a) medically or clinically identified, (b) alleged in the complaint,
(c) sufficiently proven by experts and (d) clearly explained in the decision. Article 36 of the Family Code requires that the
incapacity must be psychologicalnot physical, although its manifestations and/or symptoms may be physical. The

evidence must convince the court that the parties, or one of them, was mentally or psychically ill to such an extent that the
person could not have known the obligations he was assuming, or knowing them, could not have given valid assumption
thereof. Although no example of such incapacity need be given here so as not to limit the application of the provision
under the principle ofejusdem generis, nevertheless such root cause must be identified as a psychological illness and its
incapacitating nature fully explained. Expert evidence may be given by qualified psychiatrists and clinical psychologists.
3) The incapacity must be proven to be existing at "the time of the celebration" of the marriage. The evidence must show
that the illness was existing when the parties exchanged their "I dos." The manifestation of the illness need not be
perceivable at such time, but the illness itself must have attached at such moment, or prior thereto.
4) Such incapacity must also be shown to be medically or clinically permanent or incurable. Such incurability may be
absolute or even relative only in regard to the other spouse, not necessarily absolutely against everyone of the same sex.
Furthermore, such incapacity must be relevant to the assumption of marriage obligations, not necessarily to those not
related to marriage, like the exercise of a profession or employment in a job. Hence, a pediatrician may be effective in
diagnosing illnesses of children and prescribing medicine to cure them but not be psychologically capacitated to
procreate, bear and raise his/her own children as an essential obligation of marriage.
5) Such illness must be grave enough to bring about the disability of the party to assume the essential obligations of
marriage. Thus, "mild characteriological peculiarities, mood changes, occasional emotional outbursts" cannot be accepted
as root causes. The illness must be shown as downright incapacity or inability, not a refusal, neglect or difficulty, much
less ill will. In other words, there is a natal or supervening disabling factor in the person, an adverse integral element in the
personality structure that effectively incapacitates the person from really accepting and thereby complying with the
obligations essential to marriage.
6) The essential marital obligations must be those embraced by Articles 68 up to 71 of the Family Code as regards the
husband and wife as well as Articles 220, 221 and 225 of the same Code in regard to parents and their children. Such
non-complied marital obligation(s) must also be stated in the petition, proven by evidence and included in the text of the
decision.
7) Interpretations given by the National Appellate Matrimonial Tribunal of the Catholic Church in the Philippines, while not
controlling or decisive, should be given great respect by our courts. It is clear that Article 36 was taken by the Family Code
Revision Committee from Canon 1095 of the New Code of Canon Law, which became effective in 1983 and which
provides:
"The following are incapable of contracting marriage: Those who are unable to assume the essential obligations of marriage due
to causes of psychological nature."
Since the purpose of including such provision in our Family Code is to harmonize our civil laws with the religious faith of our
people, it stands to reason that to achieve such harmonization, great persuasive weight should be given to decisions of such
appellate tribunal. Ideallysubject to our law on evidencewhat is decreed as canonically invalid should also be decreed civilly
void.77
Molina had provided for an additional requirement that the Solicitor General issue a certification stating his reasons for his
agreement or opposition to the petition. 78 This requirement however was dispensed with following the implementation of A.M.
No. 02-11-10-SC, or the Rule on Declaration of Absolute Nullity of Void Marriages and Annulment of Voidable Marriages. 79 Still,
Article 48 of the Family Code mandates that the appearance of the prosecuting attorney or fiscal assigned be on behalf of the
State to take steps to prevent collusion between the parties and to take care that evidence is not fabricated or suppressed.
Obviously, collusion is not an issue in this case, considering the consistent vigorous opposition of respondent to the petition for
declaration of nullity. In any event, the fiscals participation in the hearings before the trial court is extant from the records of this
case.
As earlier noted, the factual findings of the RTC are now deemed binding on this Court, owing to the great weight accorded to
the opinion of the primary trier of facts, and the refusal of the Court of Appeals to dispute the veracity of these facts. As such, it
must be considered that respondent had consistently lied about many material aspects as to her character and personality. The
question remains whether her pattern of fabrication sufficiently establishes her psychological incapacity, consistent with Article
36 and generally, the Molina guidelines.
We find that the present case sufficiently satisfies the guidelines in Molina.
First. Petitioner had sufficiently overcome his burden in proving the psychological incapacity of his spouse. Apart from his own
testimony, he presented witnesses who corroborated his allegations on his wifes behavior, and certifications from Blackgold
Records and the Philippine Village Hotel Pavillon which disputed respondents claims pertinent to her alleged singing career. He
also presented two (2) expert witnesses from the field of psychology who testified that the aberrant behavior of respondent was
tantamount to psychological incapacity. In any event, both courts below considered petitioners evidence as credible enough.
Even the appellate court acknowledged that respondent was not totally honest with petitioner. 80
As in all civil matters, the petitioner in an action for declaration of nullity under Article 36 must be able to establish the cause of
action with a preponderance of evidence. However, since the action cannot be considered as a non-public matter between
private parties, but is impressed with State interest, the Family Code likewise requires the participation of the State, through the
prosecuting attorney, fiscal, or Solicitor General, to take steps to prevent collusion between the parties and to take care that
evidence is not fabricated or suppressed. Thus, even if the petitioner is able establish the psychological incapacity of
respondent with preponderant evidence, any finding of collusion among the parties would necessarily negate such proofs.
Second. The root cause of respondents psychological incapacity has been medically or clinically identified, alleged in the
complaint, sufficiently proven by experts, and clearly explained in the trial courts decision. The initiatory complaint alleged that
respondent, from the start, had exhibited unusual and abnormal behavior "of peren[n]ially telling lies, fabricating ridiculous
stories, and inventing personalities and situations," of writing letters to petitioner using fictitious names, and of lying about her
actual occupation, income, educational attainment, and family background, among others. 81
These allegations, initially characterized in generalities, were further linked to medical or clinical causes by expert witnesses
from the field of psychology. Petitioner presented two (2) such witnesses in particular. Dr. Abcede, a psychiatrist who had
headed the department of psychiatry of at least two (2) major hospitals, 82 testified as follows:
WITNESS:
Given that as a fact, which is only based on the affidavit provided to me, I can say that there are a couple of things that [are]
terribly wrong with the standards. There are a couple of things that seems (sic) to be repeated over and over again in the
affidavit. One of which is the persistent, constant and repeated lying of the "respondent"; which, I think, based on assessment of
normal behavior of an individual, is abnormal or pathological. x x x
ATTY. RAZ: (Back to the witness)
Q- Would you say then, Mr. witness, that because of these actuations of the respondent she is then incapable of performing the
basic obligations of her marriage?

A- Well, persistent lying violates the respect that one owes towards another. The lack of concern, the lack of love towards the
person, and it is also something that endangers human relationship. You see, relationship is based on communication between
individuals and what we generally communicate are our thoughts and feelings. But then when one talks and expresse[s] their
feelings, [you] are expected to tell the truth. And therefore, if you constantly lie, what do you think is going to happen as far as
this relationship is concerned. Therefore, it undermines that basic relationship that should be based on love, trust and respect.
Q- Would you say then, Mr. witness, that due to the behavior of the respondent in constantly lying and fabricating stories, she is
then incapable of performing the basic obligations of the marriage?
xxx
ATTY. RAZ: (Back to the witness)
Q- Mr. witness, based on the testimony of Mr. Levy Mendoza, who is the third witness for the petitioner, testified that the
respondent has been calling up the petitioners officemates and ask him (sic) on the activities of the petitioner and ask him on
the behavior of the petitioner. And this is specifically stated on page six (6) of the transcript of stenographic notes, what can you
say about this, Mr. witness?
A- If an individual is jealous enough to the point that he is paranoid, which means that there is no actual basis on her suspect
(sic) that her husband is having an affair with a woman, if carried on to the extreme, then that is pathological. That is not
abnormal. We all feel jealous, in the same way as we also lie every now and then; but everything that is carried out in extreme
is abnormal or pathological. If there is no basis in reality to the fact that the husband is having an affair with another woman and
if she persistently believes that the husband is having an affair with different women, then that is pathological and we call that
paranoid jealousy.
Q- Now, if a person is in paranoid jealousy, would she be considered psychologically incapacitated to perform the basic
obligations of the marriage?
A- Yes, Maam.83
The other witness, Dr. Lopez, was presented to establish not only the psychological incapacity of respondent, but also the
psychological capacity of petitioner. He concluded that respondent "is [a] pathological liar, that [she continues] to lie [and] she
loves to fabricate about herself."84
These two witnesses based their conclusions of psychological incapacity on the case record, particularly the trial transcripts of
respondents testimony, as well as the supporting affidavits of petitioner. While these witnesses did not personally examine
respondent, the Court had already held in Marcos v. Marcos85 that personal examination of the subject by the physician is not
required for the spouse to be declared psychologically incapacitated. 86 We deem the methodology utilized by petitioners
witnesses as sufficient basis for their medical conclusions. Admittedly, Drs. Abcede and Lopezs common conclusion of
respondents psychological incapacity hinged heavily on their own acceptance of petitioners version as the true set of facts.
However, since the trial court itself accepted the veracity of petitioners factual premises, there is no cause to dispute the
conclusion of psychological incapacity drawn therefrom by petitioners expert witnesses.
Also, with the totality of the evidence presented as basis, the trial court explicated its finding of psychological incapacity in its
decision in this wise:
To the mind of the Court, all of the above are indications that respondent is psychologically incapacitated to perform the
essential obligations of marriage. It has been shown clearly from her actuations that respondent has that propensity for telling
lies about almost anything, be it her occupation, her state of health, her singing abilities, her income, etc. She has this fantastic
ability to invent and fabricate stories and personalities. She practically lived in a world of make believe making her therefore not
in a position to give meaning and significance to her marriage to petitioner. In persistently and constantly lying to petitioner,
respondent undermined the basic tenets of relationship between spouses that is based on love, trust and respect. As concluded
by the psychiatrist presented by petitioner, such repeated lying is abnormal and pathological and amounts to psychological
incapacity.87
Third. Respondents psychological incapacity was established to have clearly existed at the time of and even before the
celebration of marriage. She fabricated friends and made up letters from fictitious characters well before she married petitioner.
Likewise, she kept petitioner in the dark about her natural childs real parentage as she only confessed when the latter had
found out the truth after their marriage.
Fourth. The gravity of respondents psychological incapacity is sufficient to prove her disability to assume the essential
obligations of marriage. It is immediately discernible that the parties had shared only a little over a year of cohabitation before
the exasperated petitioner left his wife. Whatever such circumstance speaks of the degree of tolerance of petitioner, it likewise
supports the belief that respondents psychological incapacity, as borne by the record, was so grave in extent that any
prolonged marital life was dubitable.
It should be noted that the lies attributed to respondent were not adopted as false pretenses in order to induce petitioner into
marriage. More disturbingly, they indicate a failure on the part of respondent to distinguish truth from fiction, or at least abide by
the truth. Petitioners witnesses and the trial court were emphatic on respondents inveterate proclivity to telling lies and the
pathologic nature of her mistruths, which according to them, were revelatory of respondents inability to understand and perform
the essential obligations of marriage. Indeed, a person unable to distinguish between fantasy and reality would similarly be
unable to comprehend the legal nature of the marital bond, much less its psychic meaning, and the corresponding obligations
attached to marriage, including parenting. One unable to adhere to reality cannot be expected to adhere as well to any legal or
emotional commitments.
The Court of Appeals somehow concluded that since respondent allegedly tried her best to effect a reconciliation, she had
amply exhibited her ability to perform her marital obligations. We are not convinced. Given the nature of her psychological
condition, her willingness to remain in the marriage hardly banishes nay extenuates her lack of capacity to fulfill the essential
marital obligations. Respondents ability to even comprehend what the essential marital obligations are is impaired at best.
Considering that the evidence convincingly disputes respondents ability to adhere to the truth, her avowals as to her
commitment to the marriage cannot be accorded much credence.
At this point, it is worth considering Article 45(3) of the Family Code which states that a marriage may be annulled if the consent
of either party was obtained by fraud, and Article 46 which enumerates the circumstances constituting fraud under the previous
article, clarifies that "no other misrepresentation or deceit as to character, health, rank, fortune or chastity shall constitute such
fraud as will give grounds for action for the annulment of marriage." It would be improper to draw linkages between
misrepresentations made by respondent and the misrepresentations under Articles 45 (3) and 46. The fraud under Article 45(3)
vitiates the consent of the spouse who is lied to, and does not allude to vitiated consent of the lying spouse. In this case, the
misrepresentations of respondent point to her own inadequacy to cope with her marital obligations, kindred to psychological
incapacity under Article 36.
Fifth. Respondent is evidently unable to comply with the essential marital obligations as embraced by Articles 68 to 71 of the
Family Code. Article 68, in particular, enjoins the spouses to live together, observe mutual love, respect and fidelity, and render
mutual help and support. As noted by the trial court, it is difficult to see how an inveterate pathological liar would be able to
commit to the basic tenets of relationship between spouses based on love, trust and respect.

Sixth. The Court of Appeals clearly erred when it failed to take into consideration the fact that the marriage of the parties was
annulled by the Catholic Church. The appellate court apparently deemed this detail totally inconsequential as no reference was
made to it anywhere in the assailed decision despite petitioners efforts to bring the matter to its attention. 88 Such deliberate
ignorance is in contravention of Molina, which held that interpretations given by the National Appellate Matrimonial Tribunal of
the Catholic Church in the Philippines, while not controlling or decisive, should be given great respect by our courts.
As noted earlier, the Metropolitan Tribunal of the Archdiocese of Manila decreed the invalidity of the marriage in question in
a Conclusion89 dated 30 March 1995, citing the "lack of due discretion" on the part of respondent. 90Such decree of nullity was
affirmed by both the National Appellate Matrimonial Tribunal, 91 and the Roman Rota of the Vatican.92 In fact, respondents
psychological incapacity was considered so grave that a restrictive clause 93was appended to the sentence of nullity prohibiting
respondent from contracting another marriage without the Tribunals consent.
In its Decision dated 4 June 1995, the National Appellate Matrimonial Tribunal pronounced:
The JURISRPRUDENCE in the Case maintains that matrimonial consent is considered ontologically defective and wherefore
judicially ineffective when elicited by a Part Contractant in possession and employ of a discretionary judgment faculty with a
perceptive vigor markedly inadequate for the practical understanding of the conjugal Covenant or serious impaired from the
correct appreciation of the integral significance and implications of the marriage vows.
The FACTS in the Case sufficiently prove with the certitude required by law that based on the depositions of the Partes in
Causa and premised on the testimonies of the Common and Expert Witnesse[s], the Respondent made the marriage option
in tenure of adverse personality constracts that were markedly antithetical to the substantive content and implications
of the Marriage Covenant, and that seriously undermined the integrality of her matrimonial consent in terms of its
deliberative component. In other words, afflicted with a discretionary faculty impaired in its practico-concrete
judgment formation on account of an adverse action and reaction pattern, the Respondent was impaired from eliciting
a judicially binding matrimonial consent. There is no sufficient evidence in the Case however to prove as well the fact of
grave lack of due discretion on the part of the Petitioner.94
Evidently, the conclusion of psychological incapacity was arrived at not only by the trial court, but also by canonical bodies. Yet,
we must clarify the proper import of the Church rulings annulling the marriage in this case. They hold sway since they are drawn
from a similar recognition, as the trial court, of the veracity of petitioners allegations. Had the trial court instead appreciated
respondents version as correct, and the appellate court affirmed such conclusion, the rulings of the Catholic Church on this
matter would have diminished persuasive value. After all, it is the factual findings of the judicial trier of facts, and not that of the
canonical courts, that are accorded significant recognition by this Court.
Seventh. The final point of contention is the requirement in Molina that such psychological incapacity be shown to be medically
or clinically permanent or incurable. It was on this score that the Court of Appeals reversed the judgment of the trial court, the
appellate court noting that it did not appear certain that respondents condition was incurable and that Dr. Abcede did not testify
to such effect.95
Petitioner points out that one month after he and his wife initially separated, he returned to her, desiring to make their marriage
work. However, respondents aberrant behavior remained unchanged, as she continued to lie, fabricate stories, and maintained
her excessive jealousy. From this fact, he draws the conclusion that respondents condition is incurable.
From the totality of the evidence, can it be definitively concluded that respondents condition is incurable? It would seem, at
least, that respondents psychosis is quite grave, and a cure thereof a remarkable feat. Certainly, it would have been easier had
petitioners expert witnesses characterized respondents condition as incurable. Instead, they remained silent on whether the
psychological incapacity was curable or incurable.
But on careful examination, there was good reason for the experts taciturnity on this point.
The petitioners expert witnesses testified in 1994 and 1995, and the trial court rendered its decision on 10 August 1995. These
events transpired well before Molina was promulgated in 1997 and made explicit the requirement that the psychological
incapacity must be shown to be medically or clinically permanent or incurable. Such requirement was not expressly stated in
Article 36 or any other provision of the Family Code.
On the other hand, the Court in Santos, which was decided in January 1995, began its discussion by first citing the deliberations
of the Family Code committee,96 then the opinion of canonical scholars,97 before arriving at its formulation of the doctrinal
definition of psychological incapacity.98 Santos did refer to Justice Caguioas opinion expressed during the deliberations that
"psychological incapacity is incurable,"99 and the view of a former presiding judge of the Metropolitan Marriage Tribunal of the
Archdiocese of Manila that psychological incapacity must be characterized "by (a) gravity, (b) juridical antecedence, and (c)
incurability."100 However, in formulating the doctrinal rule on psychological incapacity, the Court in Santos omitted any reference
to incurability as a characteristic of psychological incapacity.101
This disquisition is material as Santos was decided months before the trial court came out with its own ruling that remained
silent on whether respondents psychological incapacity was incurable. Certainly, Santos did not clearly mandate that the
incurability of the psychological incapacity be established in an action for declaration of nullity. At least, there was no
jurisprudential clarity at the time of the trial of this case and the subsequent promulgation of the trial courts decision that
required a medical finding of incurability. Such requisite arose only with Molina in 1997, at a time when this case was on
appellate review, or after the reception of evidence.
We are aware that in Pesca v. Pesca,102 the Court countered an argument that Molina and Santos should not apply retroactively
with the observation that the interpretation or construction placed by the courts of a law constitutes a part of that law as of the
date the statute in enacted.103 Yet we approach this present case from utterly practical considerations. The requirement that
psychological incapacity must be shown to be medically or clinically permanent or incurable is one that necessarily cannot be
divined without expert opinion. Clearly in this case, there was no categorical averment from the expert witnesses that
respondents psychological incapacity was curable or incurable simply because there was no legal necessity yet to elicit such a
declaration and the appropriate question was not accordingly propounded to him. If we apply Pesca without deep reflection,
there would be undue prejudice to those cases tried before Molina or Santos, especially those presently on appellate review,
where presumably the respective petitioners and their expert witnesses would not have seen the need to adduce a diagnosis of
incurability. It may hold in those cases, as in this case, that the psychological incapacity of a spouse is actually incurable, even if
not pronounced as such at the trial court level.
We stated earlier that Molina is not set in stone, and that the interpretation of Article 36 relies heavily on a case-to-case
perception. It would be insensate to reason to mandate in this case an expert medical or clinical diagnosis of incurability, since
the parties would have had no impelling cause to present evidence to that effect at the time this case was tried by the RTC more
than ten (10) years ago. From the totality of the evidence, we are sufficiently convinced that the incurability of respondents
psychological incapacity has been established by the petitioner. Any lingering doubts are further dispelled by the fact that the
Catholic Church tribunals, which indubitably consider incurability as an integral requisite of psychological incapacity, were
sufficiently convinced that respondent was so incapacitated to contract marriage to the degree that annulment was warranted.
All told, we conclude that petitioner has established his cause of action for declaration of nullity under Article 36 of the Family
Code. The RTC correctly ruled, and the Court of Appeals erred in reversing the trial court.

There is little relish in deciding this present petition, pronouncing as it does the marital bond as having been inexistent in the first
place. It is possible that respondent, despite her psychological state, remains in love with petitioner, as exhibited by her
persistent challenge to the petition for nullity. In fact, the appellate court placed undue emphasis on respondents avowed
commitment to remain in the marriage. Yet the Court decides these cases on legal reasons and not vapid sentimentality.
Marriage, in legal contemplation, is more than the legitimatization of a desire of people in love to live together.
WHEREFORE, the petition is GRANTED. The decision of the RTC dated 10 August 1995, declaring the marriage between
petitioner and respondent NULL and VOID under Article 36 of the Family Code, is REINSTATED. No costs.
SO ORDERED.

G.R. No. 119190 January 16, 1997


CHI MING TSOI, petitioner,
vs.
COURT OF APPEALS and GINA LAO-TSOI, respondents.

TORRES, JR., J.:

Man has not invented a reliable compass by which to steer a marriage in its journey over troubled waters. Laws are seemingly
inadequate. Over time, much reliance has been placed in the works of the unseen hand of Him who created all things.
Who is to blame when a marriage fails?
This case was originally commenced by a distraught wife against her uncaring husband in the Regional Trial Court of Quezon
City (Branch 89) which decreed the annulment of the marriage on the ground of psychological incapacity. Petitioner appealed
the decision of the trial court to respondent Court of Appeals (CA-G.R. CV No. 42758) which affirmed the Trial Court's decision
November 29, 1994 and correspondingly denied the motion for reconsideration in a resolution dated February 14, 1995.
The statement of the case and of the facts made by the trial court and reproduced by the Court of Appeals 1 its decision are as
follows:
From the evidence adduced, the following acts were preponderantly established:
Sometime on May 22, 1988, the plaintiff married the defendant at the Manila Cathedral, . . . Intramuros Manila, as
evidenced by their Marriage Contract. (Exh. "A")
After the celebration of their marriage and wedding reception at the South Villa, Makati, they went and proceeded to the
house of defendant's mother.
There, they slept together on the same bed in the same room for the first night of their married life.
It is the version of the plaintiff, that contrary to her expectations, that as newlyweds they were supposed to enjoy making
love, or having sexual intercourse, with each other, the defendant just went to bed, slept on one side thereof, then turned
his back and went to sleep . There was no sexual intercourse between them during the first night. The same thing
happened on the second, third and fourth nights.
In an effort to have their honeymoon in a private place where they can enjoy together during their first week as husband
and wife, they went to Baguio City. But, they did so together with her mother, an uncle, his mother and his nephew. They
were all invited by the defendant to join them. [T]hey stayed in Baguio City for four (4) days. But, during this period, there
was no sexual intercourse between them, since the defendant avoided her by taking a long walk during siesta time or by
just sleeping on a rocking chair located at the living room. They slept together in the same room and on the same bed
since May 22, 1988 until March 15, 1989. But during this period, there was no attempt of sexual intercourse between
them. [S]he claims, that she did not: even see her husband's private parts nor did he see hers.
Because of this, they submitted themselves for medical examinations to Dr. Eufemio Macalalag, a urologist at the Chinese
General Hospital, on January 20, 1989.
The results of their physical examinations were that she is healthy, normal and still a virgin, while that of her husband's
examination was kept confidential up to this time. While no medicine was prescribed for her, the doctor prescribed
medications for her husband which was also kept confidential. No treatment was given to her. For her husband, he was
asked by the doctor to return but he never did.
The plaintiff claims, that the defendant is impotent, a closet homosexual as he did not show his penis. She said, that she
had observed the defendant using an eyebrow pencil and sometimes the cleansing cream of his mother. And that,
according to her, the defendant married her, a Filipino citizen, to acquire or maintain his residency status here in the
country and to publicly maintain the appearance of a normal man.
The plaintiff is not willing to reconcile with her husband.
On the other hand, it is the claim of the defendant that if their marriage shall be annulled by reason of psychological
incapacity, the fault lies with his wife.
But, he said that he does not want his marriage with his wife annulled for several reasons, viz: (1) that he loves her very
much; (2) that he has no defect on his part and he is physically and psychologically capable; and, (3) since the
relationship is still very young and if there is any differences between the two of them, it can still be reconciled and that,
according to him, if either one of them has some incapabilities, there is no certainty that this will not be cured. He further
claims, that if there is any defect, it can be cured by the intervention of medical technology or science.
The defendant admitted that since their marriage on May 22, 1988, until their separation on March 15, 1989, there was no
sexual contact between them. But, the reason for this, according to the defendant, was that everytime he wants to have
sexual intercourse with his wife, she always avoided him and whenever he caresses her private parts, she always
removed his hands. The defendant claims, that he forced his wife to have sex with him only once but he did not continue
because she was shaking and she did not like it. So he stopped.
There are two (2) reasons, according to the defendant , why the plaintiff filed this case against him, and these are: (1) that
she is afraid that she will be forced to return the pieces of jewelry of his mother, and, (2) that her husband, the defendant,
will consummate their marriage.
The defendant insisted that their marriage will remain valid because they are still very young and there is still a chance to
overcome their differences.
The defendant submitted himself to a physical examination. His penis was examined by Dr. Sergio Alteza, Jr., for the
purpose of finding out whether he is impotent . As a result thereof, Dr. Alteza submitted his Doctor's Medical Report. (Exh.
"2"). It is stated there, that there is no evidence of impotency (Exh. "2-B"), and he is capable of erection. (Exh. "2-C")
The doctor said, that he asked the defendant to masturbate to find out whether or not he has an erection and he found out
that from the original size of two (2) inches, or five (5) centimeters, the penis of the defendant lengthened by one (1) inch
and one centimeter. Dr. Alteza said, that the defendant had only a soft erection which is why his penis is not in its full
length. But, still is capable of further erection, in that with his soft erection, the defendant is capable of having sexual
intercourse with a woman.
In open Court, the Trial Prosecutor manifested that there is no collusion between the parties and that the evidence is not
fabricated." 2
After trial, the court rendered judgment, the dispositive portion of which reads:
ACCORDINGLY, judgment is hereby rendered declaring as VOID the marriage entered into by the plaintiff with the
defendant on May 22, 1988 at the Manila Cathedral, Basilica of the Immaculate Conception, Intramuros, Manila, before
the Rt. Rev. Msgr. Melencio de Vera. Without costs. Let a copy of this decision be furnished the Local Civil Registrar of
Quezon City. Let another copy be furnished the Local Civil Registrar of Manila.
SO ORDERED.
On appeal, the Court of Appeals affirmed the trial court's decision.

Hence, the instant petition.


Petitioner alleges that the respondent Court of Appeals erred:
I
in affirming the conclusions of the lower court that there was no sexual intercourse between the parties without making
any findings of fact.
II
in holding that the refusal of private respondent to have sexual communion with petitioner is a psychological incapacity
inasmuch as proof thereof is totally absent.
III
in holding that the alleged refusal of both the petitioner and the private respondent to have sex with each other constitutes
psychological incapacity of both.
IV
in affirming the annulment of the marriage between the parties decreed by the lower court without fully satisfying itself that
there was no collusion between them.
We find the petition to be bereft of merit.
Petitioner contends that being the plaintiff in Civil Case No. Q-89-3141, private respondent has the burden of proving the
allegations in her complaint; that since there was no independent evidence to prove the alleged non-coitus between the parties,
there remains no other basis for the court's conclusion except the admission of petitioner; that public policy should aid acts
intended to validate marriage and should retard acts intended to invalidate them; that the conclusion drawn by the trial court on
the admissions and confessions of the parties in their pleadings and in the course of the trial is misplaced since it could have
been a product of collusion; and that in actions for annulment of marriage, the material facts alleged in the complaint shall
always be proved. 3
Section 1, Rule 19 of the Rules of Court reads:
Section 1. Judgment on the pleadings. Where an answer fails to tender an issue, or otherwise admits the material
allegations of the adverse party's pleading, the court may, on motion of that party, direct judgment on such pleading. But in
actions for annulment of marriage or for legal separation the material facts alleged in the complaint shall always be
proved.
The foregoing provision pertains to a judgment on the pleadings. What said provision seeks to prevent is annulment of marriage
without trial. The assailed decision was not based on such a judgment on the pleadings. When private respondent testified
under oath before the trial court and was cross-examined by oath before the trial court and was cross-examined by the adverse
party, she thereby presented evidence in form of a testimony. After such evidence was presented, it be came incumbent upon
petitioner to present his side. He admitted that since their marriage on May 22, 1988, until their separation on March 15, 1989,
there was no sexual intercourse between them.
To prevent collusion between the parties is the reason why, as stated by the petitioner, the Civil Code provides that no judgment
annulling a marriage shall be promulgated upon a stipulation of facts or by confession of judgment (Arts. 88 and 101[par. 2]) and
the Rules of Court prohibit such annulment without trial (Sec. 1, Rule 19).
The case has reached this Court because petitioner does not want their marriage to be annulled. This only shows that there is
no collusion between the parties. When petitioner admitted that he and his wife (private respondent) have never had sexual
contact with each other, he must have been only telling the truth. We are reproducing the relevant portion of the challenged
resolution denying petitioner's Motion for Reconsideration, penned with magisterial lucidity by Associate Justice Minerva
Gonzaga-Reyes, viz:
The judgment of the trial court which was affirmed by this Court is not based on a stipulation of facts. The issue of whether
or not the appellant is psychologically incapacitated to discharge a basic marital obligation was resolved upon a review of
both the documentary and testimonial evidence on record. Appellant admitted that he did not have sexual relations with
his wife after almost ten months of cohabitation, and it appears that he is not suffering from any physical disability. Such
abnormal reluctance or unwillingness to consummate his marriage is strongly indicative of a serious personality disorder
which to the mind of this Court clearly demonstrates an 'utter insensitivity or inability to give meaning and significance to
the marriage' within the meaning of Article 36 of the Family Code (See Santos vs. Court of Appeals, G.R. No. 112019,
January 4, 1995). 4
Petitioner further contends that respondent court erred in holding that the alleged refusal of both the petitioner and the private
respondent to have sex with each other constitutes psychological incapacity of both. He points out as error the failure of the trial
court to make "a categorical finding about the alleged psychological incapacity and an in-depth analysis of the reasons for such
refusal which may not be necessarily due to physchological disorders" because there might have been other reasons, i.e.,
physical disorders, such as aches, pains or other discomforts, why private respondent would not want to have sexual
intercourse from May 22, 1988 to March 15, 1989, in a short span of 10 months.
First, it must be stated that neither the trial court nor the respondent court made a finding on who between petitioner and private
respondent refuses to have sexual contact with the other. The fact remains, however, that there has never been coitus between
them. At any rate, since the action to declare the marriage void may be filed by either party, i.e., even the psychologically
incapacitated, the question of who refuses to have sex with the other becomes immaterial.
Petitioner claims that there is no independent evidence on record to show that any of the parties is suffering from phychological
incapacity. Petitioner also claims that he wanted to have sex with private respondent; that the reason for private respondent's
refusal may not be psychological but physical disorder as stated above.
We do not agree. Assuming it to be so, petitioner could have discussed with private respondent or asked her what is ailing her,
and why she balks and avoids him everytime he wanted to have sexual intercourse with her. He never did. At least, there is
nothing in the record to show that he had tried to find out or discover what the problem with his wife could be. What he
presented in evidence is his doctor's Medical Report that there is no evidence of his impotency and he is capable of
erection. 5 Since it is petitioner's claim that the reason is not psychological but perhaps physical disorder on the part of private
respondent, it became incumbent upon him to prove such a claim.
If a spouse, although physically capable but simply refuses to perform his or her essential marriage obligations, and the
refusal is senseless and constant, Catholic marriage tribunals attribute the causes to psychological incapacity than to
stubborn refusal. Senseless and protracted refusal is equivalent to psychological incapacity. Thus, the prolonged refusal of
a spouse to have sexual intercourse with his or her spouse is considered a sign of psychological incapacity. 6

Evidently, one of the essential marital obligations under the Family Code is "To procreate children based on the universal
principle that procreation of children through sexual cooperation is the basic end of marriage." Constant non- fulfillment of this
obligation will finally destroy the integrity or wholeness of the marriage. In the case at bar, the senseless and protracted refusal
of one of the parties to fulfill the above marital obligation is equivalent to psychological incapacity.
As aptly stated by the respondent court,
An examination of the evidence convinces Us that the husband's plea that the wife did not want carnal intercourse with
him does not inspire belief. Since he was not physically impotent, but he refrained from sexual intercourse during the
entire time (from May 22, 1988 to March 15, 1989) that he occupied the same bed with his wife, purely out of symphaty for
her feelings, he deserves to be doubted for not having asserted his right seven though she balked (Tompkins vs.
Tompkins, 111 Atl. 599, cited in I Paras, Civil Code, at p. 330). Besides, if it were true that it is the wife was suffering from
incapacity, the fact that defendant did not go to court and seek the declaration of nullity weakens his claim. This case was
instituted by the wife whose normal expectations of her marriage were frustrated by her husband's inadequacy.
Considering the innate modesty of the Filipino woman, it is hard to believe that she would expose her private life to public
scrutiny and fabricate testimony against her husband if it were not necessary to put her life in order and put to rest her
marital status.
We are not impressed by defendant's claim that what the evidence proved is the unwillingness or lack of intention to
perform the sexual act, which is not phychological incapacity, and which can be achieved "through proper motivation."
After almost ten months of cohabitation, the admission that the husband is reluctant or unwilling to perform the sexual act
with his wife whom he professes to love very dearly, and who has not posed any insurmountable resistance to his alleged
approaches, is indicative of a hopeless situation, and of a serious personality disorder that constitutes psychological
incapacity to discharge the basic marital covenants within the contemplation of the Family Code. 7
While the law provides that the husband and the wife are obliged to live together, observe mutual love, respect and fidelity (Art.
68, Family Code), the sanction therefor is actually the "spontaneous, mutual affection between husband and wife and not any
legal mandate or court order" (Cuaderno vs. Cuaderno 120 Phil. 1298). Love is useless unless it is shared with another. Indeed,
no man is an island, the cruelest act of a partner in marriage is to say "I could not have cared less." This is so because an
ungiven self is an unfulfilled self. The egoist has nothing but himself. In the natural order, it is sexual intimacy which brings
spouses wholeness and oneness. Sexual intimacy is a gift and a participation in the mystery of creation. It is a function which
enlivens the hope of procreation and ensures the continuation of family relations.
It appears that there is absence of empathy between petitioner and private respondent. That is a shared feeling which
between husband and wife must be experienced not only by having spontaneous sexual intimacy but a deep sense of spiritual
communion. Marital union is a two-way process. An expressive interest in each other's feelings at a time it is needed by the
other can go a long way in deepening the marital relationship. Marriage is definitely not for children but for two consenting adults
who view the relationship with love amor gignit amorem, respect, sacrifice and a continuing commitment to compromise,
conscious of its value as a sublime social institution.
This Court, finding the gravity of the failed relationship in which the parties found themselves trapped in its mire of unfulfilled
vows and unconsummated marital obligations, can do no less but sustain the studied judgment of respondent appellate court.
IN VIEW OF THE FOREGOING PREMISES , the assailed decision of the Court of Appeals dated November 29, 1994 is hereby
AFFIRMED in all respects and the petition is hereby DENIED for lack of merit.
SO ORDERED.

MA. ARMIDA PEREZ-FERRARIS,


Petitioner,

G.R. No. 162368


Present:

- versus -

Panganiban, C.J. (Chairperson),


Ynares-Santiago,
Austria-Martinez,
Callejo, Sr., and

Chico-Nazario, JJ.
BRIX FERRARIS,
Respondent.

Promulgated:

July 17, 2006


x ---------------------------------------------------------------------------------------- x
RESOLUTION
YNARES-SANTIAGO, J.:

This resolves the motion for reconsideration filed by petitioner Ma. Armida Perez-Ferraris of the Resolution dated June 9,
2004 denying the petition for review on certiorari of the Decision and Resolution of the Court of Appeals dated April 30,
2003 andFebruary 24, 2004, respectively, for failure of the petitioner to sufficiently show that the Court of Appeals committed
any reversible error.

On February 20, 2001, the Regional Trial Court of Pasig City, Branch 151 rendered a Decision[1] denying the petition for
declaration of nullity of petitioners marriage with Brix Ferraris. The trial court noted that suffering from epilepsy does not
amount to psychological incapacity under Article 36 of the Civil Code and the evidence on record were insufficient to prove
infidelity. Petitioners motion for reconsideration was denied in an Order [2] dated April 20, 2001 where the trial court reiterated
that there was no evidence that respondent is mentally or physically ill to such an extent that he could not have known the
obligations he was assuming, or knowing them, could not have given valid assumption thereof.

Petitioner appealed to the Court of Appeals which affirmed [3] in toto the judgment of the trial court. It held that the
evidence on record did not convincingly establish that respondent was suffering from psychological incapacity or that his
defects were incurable and already present at the inception of the marriage. [4] The Court of Appeals also found that Dr.
Dayans testimony failed to establish the substance of respondents psychological incapacity; that she failed to explain how she
arrived at the conclusion that the respondent has a mixed personality disorder; that she failed to clearly demonstrate that there
was a natal or supervening disabling factor or an adverse integral element in respondents character that effectively
incapacitated him from accepting and complying with the essential marital obligations. [5]

Petitioners motion for reconsideration was denied [6] for lack of merit; thus, she filed a petition for review on certiorari with
this Court. As already stated, the petition for review was denied for failure of petitioner to show that the appellate tribunal
committed any reversible error.

Petitioner filed the instant motion for reconsideration. [7] The Court required respondent Brix Ferraris to file comment [8] but
failed to comply; thus, he is deemed to have waived the opportunity to file comment. Further, the Court directed the Office of
the Solicitor General (OSG) to comment on petitioners motion for reconsideration which it complied on March 2, 2006.

After considering the arguments of both the petitioner and the OSG, the Court resolves to deny petitioners motion for
reconsideration.

The issue of whether or not psychological incapacity exists in a given case calling for annulment of marriage depends
crucially, more than in any field of the law, on the facts of the case. [9] Such factual issue, however, is beyond the province of this
Court to review. It is not the function of the Court to analyze or weigh all over again the evidence or premises supportive of
such factual determination.[10] It is a well-established principle that factual findings of the trial court, when affirmed by the Court
of Appeals, are binding on this Court, [11] save for the most compelling and cogent reasons, like when the findings of the
appellate court go beyond the issues of the case, run contrary to the admissions of the parties to the case, or fail to notice
certain relevant facts which, if properly considered, will justify a different conclusion; or when there is a misappreciation of facts,
[12]

which are unavailing in the instant case.


The term psychological incapacity to be a ground for the nullity of marriage under Article 36 of the Family Code, refers to

a serious psychological illness afflicting a party even before the celebration of the marriage. It is a malady so grave and so
permanent as to deprive one of awareness of the duties and responsibilities of the matrimonial bond one is about to assume.
[13]

As all people may have certain quirks and idiosyncrasies, or isolated characteristics associated with certain personality

disorders, there is hardly any doubt that the intendment of the law has been to confine the meaning of psychological

incapacity to the most serious cases of personality disorders clearly demonstrative of an utter insensitivity or inability to give
meaning and significance to the marriage. [14] It is for this reason that the Court relies heavily on psychological experts for its
understanding of the human personality. However, the root cause must be identified as a psychological illness and its
incapacitating nature must be fully explained, [15] which petitioner failed to convincingly demonstrate.
As aptly held by the Court of Appeals:
Simply put, the chief and basic consideration in the resolution of marital annulment cases is the presence of
evidence that can adequately establish respondents psychological condition. Here, appellant contends that there is
such evidence. We do not agree. Indeed, the evidence on record did not convincingly establish that respondent
was suffering from psychological incapacity. There is absolutely no showing that his defects were already present
at the inception of the marriage, or that those are incurable.
Quite apart from being plainly self-serving, petitioners evidence showed that respondents alleged failure to
perform his so-called marital obligations was not at all a manifestation of some deep-seated, grave, permanent and
incurable psychological malady. To be sure, the couples relationship before the marriage and even during their brief
union (for well about a year or so) was not all bad. During that relatively short period of time, petitioner was happy
and contented with her life in the company of respondent. In fact, by petitioners own reckoning, respondent was a
responsible and loving husband. x x x. Their problems began when petitioner started doubting respondents
fidelity. It was only when they started fighting about the calls from women that respondent began to withdraw into
his shell and corner, and failed to perform his so-called marital obligations. Respondent could not understand
petitioners lack of trust in him and her constant naggings. He thought her suspicions irrational. Respondent could
not relate to her anger, temper and jealousy. x x x.
xxxx
At any rate, Dr. Dayan did not explain how she arrived at her diagnosis that respondent has a mixed
personality disorder called schizoid, and why he is the dependent and avoidant type. In fact, Dr. Dayans
statement that one suffering from such mixed personality disorder is dependent on others for decision x x x lacks
specificity; it seems to belong to the realm of theoretical speculation. Also, Dr. Dayans information that respondent
had extramarital affairs was supplied by the petitioner herself. Notably, when asked as to the root cause of
respondents alleged psychological incapacity, Dr. Dayans answer was vague, evasive and inconclusive. She
replied that such disorder can be part of his family upbringing x x x. She stated that there was a history of
respondents parents having difficulties in their relationship. But this input on the supposed problematic history of
respondents parents also came from petitioner. Nor did Dr. Dayan clearly demonstrate that there was really a natal
or supervening disabling factor on the part of respondent, or an adverse integral element in respondents
character that effectively incapacitated him from accepting, and, thereby complying with, the essential marital
obligations. Of course, petitioner likewise failed to prove that respondents supposed psychological or mental
malady existed even before the marriage. All these omissions must be held up against petitioner, for the reason that
upon her devolved the onus of establishing nullity of the marriage. Indeed, any doubt should be resolved in favor of
the validity of the marriage and the indissolubility of the marital vinculum. [16]

We find respondents alleged mixed personality disorder, the leaving-the-house attitude whenever they quarreled, the
violent tendencies during epileptic attacks, the sexual infidelity, the abandonment and lack of support, and his preference to
spend more time with his band mates than his family, are not rooted on some debilitating psychological condition but a mere
refusal or unwillingness to assume the essential obligations of marriage.

In Republic v. Court of Appeals,[17] where therein respondent preferred to spend more time with his friends than his family
on whom he squandered his money, depended on his parents for aid and assistance, and was dishonest to his wife regarding
his

finances,

the

Court

held

that

the

psychological

defects

spoken

of

were

more

of

a difficulty, if

not

outright refusal or neglect in the performance of some marital obligations and that a mere showing of irreconcilable
differences and conflicting personalities in no wise constitute psychological incapacity; it is not enough to prove that the parties
failed to meet their responsibilities and duties as married persons; it is essential that they must be shown to be incapable of
doing so, due to some psychological, not physical, illness.
Also, we held in Hernandez v. Court of Appeals[18] that habitual alcoholism, sexual infidelity or perversion, and
abandonment do not by themselves constitute grounds for declaring a marriage void based on psychological incapacity.
While petitioners marriage with the respondent failed and appears to be without hope of reconciliation, the remedy
however is not always to have it declared void ab initio on the ground of psychological incapacity. An unsatisfactory marriage,
however, is not a null and void marriage. [19] No less than the Constitution recognizes the sanctity of marriage and the unity of
the family; it decrees marriage as legally inviolable and protects it from dissolution at the whim of the parties. Both the family
and marriage are to beprotected by the state.[20]
Thus, in determining the import of psychological incapacity under Article 36, it must be read in conjunction with, although
to be taken as distinct from Articles 35, [21] 37,[22] 38,[23] and 41[24] that would likewise, but for different reasons, render the
marriage void ab initio, or Article 45[25] that would make the marriage merely voidable, or Article 55 that could justify a petition for
legal separation. Care must be observed so that these various circumstances are not applied so indiscriminately as if the law
were indifferent on the matter.[26] Article 36 should not to be confused with a divorce law that cuts the marital bond at the time
the causes therefor manifest themselves. [27] Neither it is to be equated with legal separation, in which the grounds need not be
rooted in psychological incapacity but on physical violence, moral pressure, moral corruption, civil interdiction, drug addiction,
habitual alcoholism, sexual infidelity, abandonment and the like. [28]

WHEREFORE, in view of the foregoing, the motion for reconsideration of the Resolution dated June 9, 2004 denying the
petition for review on certiorari for failure of the petitioner to sufficiently show that the Court of Appeals committed any reversible
error, is DENIED WITH FINALITY.

NARCISO S. NAVARRO, JR.,


Petitioner,

G.R. No. 162049


Present:
QUISUMBING, J., Chairperson,
CARPIO,

- versus -

CARPIO MORALES,
TINGA, and
VELASCO, JR., JJ.
Promulgated:

CYNTHIA CECILIO-NAVARRO,
Respondent
.

April 13, 2007

x- - - - - - - - - - - - - - - - - - - - - - - - - - - - - - - - - - - - - - - - - - - - - - - - - - -x
DECISION
QUISUMBING, J.:
For review is the Decision[1] dated January 8, 2003 of the Court of Appeals in CA-G.R. CV No. 65677, reversing the
Regional Trial Courts declaration of nullity of the marriage of petitioner and respondent. Likewise assailed is the Court of
Appeals Resolution dated February 4, 2004 denying reconsideration.
In Civil Case No. 94-70727, filed by petitioner Narciso Navarro, Jr. with the Regional Trial Court of Manila, Branch 37, he
sought the declaration of nullity of his marriage to respondent.
As culled from the records, the facts of the case are as follows:
Petitioner and respondent were college sweethearts. At the time they got married, both in civil and church ceremonies,
they were awaiting their first child. Since petitioner was still a medical student, while respondent was a student of pharmacy,
they lived with petitioners parents, on whom they were financially dependent. Eventually, their union bore four children.
Petitioner alleged that respondent constantly complained that he didnt have time for her; and that she constantly
quarreled with him even before marriage when he could not give her the things she wanted. He added that she was not
supportive of his career. Even marriage counseling did not work. Petitioner stated that when they quarreled, she refused to
have sex with him and even told him to look for other women. He filed the petition for nullification of their marriage when he
found out their eldest daughter had been made pregnant by a man whom respondent hired to follow him.
Abdona T. de Castro, a marriage counselor duly accredited by the Department of Social Welfare and Development,
testified that when petitioner saw her on April 6, 1994, he was distraught, harassed, and unhappy. She concluded from
meetings with the petitioner that the marriage was dysfunctional, destructive, and reconciliation was out of the question since he
claims he would go insane if he were to go back to his wife. Relying on the view of another expert, one Dr. Gerardo Velasco,
witness de Castro opined that professionals are per se incapacitated to perform the essential obligations of marriage because
they spend a lot of time in the pursuit of their profession and have very little time to spend with their family. She concluded that
respondent was also psychologically incapacitated to perform the marital obligations because she knew, from the start, that her
husband was going to be a doctor, yet she did not give him the support and understanding that was expected of a doctors wife.
Lilia Tayco, the housemaid of petitioners parents also testified that petitioner and respondent were always quarreling
because respondent was always jealous of petitioners classmates.
A psychologist, Dr. Natividad Dayan, who conducted a psychiatric test on petitioner, testified that tests showed that
petitioner was a perfectionist, short-tempered, critical, argumentative and irritable when people do not meet his
expectations. He married Cynthia only after he got her pregnant. He had depressions and tended to escapism when beset with
problems. He was vocal about his marital problems. He believed that the lack of communication, absence of quality time,
inadequacy in problem-solving, and many problems caused the failure of the marriage.
For her part, respondent refused to submit to the psychiatric examination asked by the petitioner, but said she would do so
only when her defense requires it. She averred that she had no marital problems, not until petitioner had an illicit affair with a
certain Dr. Lucila Posadas. Petitioner denied the affair. Respondent narrated that early 1984, she caught petitioner and Lucila
inside the Harana Motel in Sta. Mesa where a confrontation ensued. After the incident, petitioner seldom went home until he
permanently left his family sometime in 1986. Respondent claimed petitioner and Lucila continued to see each other and had
gone abroad together several times. She explained that she uttered she would not make love with her husband and dared him
to look for other women only out of frustration and anger upon discovery of the affair. She admitted hiring someone to spy on
petitioner, but added that she still loved her husband.
Cynthias friend since high school, Miraflor Respicio testified that Cynthia was a good, stable, and mature person; that she
was a loving and caring mother who gave up her career to take care of her children; and that petitioner and respondent were
happy during the early days of the marriage.
On August 21, 1998, the trial court held that petitioner and respondent were both psychologically incapacitated to perform
their marital obligations. The dispositive portion of the courts decision reads:

WHEREFORE, the marriage between the parties is (sic) dated June 2, 1973 is hereby declared null and void
with the following effects:
1.

The Plaintiff is hereby directed to support his children with the Defendant in the amount of forty
thousand pesos (P40,000.00) a month, which sum shall be payable on or before the 5 th day of each month,
effective September, 1998;

2.

The parties are hereby disqualified from inheriting from each other by way of testate or intestate
succession;

3.

Either of the parties may revoke the designation of the other as beneficiary in a life insurance policy;

4.

The parties children are hereby declared legitimate, and the custody of the parties minor children is
hereby awarded to the Defendant with the Plaintiff exercising his right to visit them at least once a week;

5.

The properties in the name of the parties consisting of a house and lot located at 15 Bronze Street,
Filinvest, Quezon City are hereby deemed as their advance legitime to their children.

SO ORDERED.[2]
Respondent appealed the case to the Court of Appeals. She averred that the trial court erred when it annulled their
marriage instead of decreeing their legal separation, with the ruling that petitioner was the guilty spouse.
In a Decision dated January 8, 2003, the Court of Appeals held that the constant arguments, bickerings and conflicts
between the spouses did not constitute psychological incapacity. It ruled that petitioner failed to show that any psychological
incapacity in either of the two parties existed at the time of the celebration of marriage. The appellate court reversed the
decision of the trial court and declared that the marriage still subsists.
Petitioner now comes before us raising the following as issues:
(1)
Are the decision and resolution of the Honorable Court of Appeals proper subject for review by the
Honorable Court under Rule 45 of the 1997 Rules of Civil Procedure?
(2) Is the conclusion of the Honorable Court of Appeals that the lower court (RTC) erred in finding the
parties (petitioner and respondent) both psychologically incapacitated under Article 36 of The Family Code correct
or not?
(3) Is the conclusion of the Honorable Court of Appeals that the evidence failed to show that the parties
(petitioner and respondent) were completely unable to discharge the essential obligations of marriage correct or
not? and
(4) Which is more in accord with existing law and settled jurisprudence, the decision of the Court of Appeals
or the decision of the trial court?[3]
Simply stated, the issue before us is whether the marriage is void on the ground of the parties psychological incapacity.
Petitioner contends that the decision of the trial court was well-founded, based on the evidence indicating that the
marriage was beyond reconciliation, and allowing the marriage to subsist would only prolong the spouses agony. Respondent
counters that petitioner failed to prove psychological incapacity, and that their psychological incapacities existed as early as the
time of the celebration of their marriage.
We shall now resolve the issue.
Article 36 of the Family Code states:
A marriage contracted by any party who, at the time of the celebration, was psychologically incapacitated to
comply with the essential marital obligations of marriage, shall likewise be void even if such incapacity becomes
manifest only after its solemnization.
In addition, as early as 1995, in Santos v. Court of Appeals,[4] we categorically said that psychological incapacity required
by Art. 36 must be characterized by (a) gravity, (b) juridical antecedence, and (c) incurability. Psychological incapacity should
refer to no less than a mental (not physical) incapacity that causes a party to be truly incognitive of the basic marital covenants
that concomitantly must be assumed and discharged by the parties to the marriage. These include the obligations to live
together, observe mutual love, respect and fidelity, and render mutual help and support. [5]
We likewise have repeatedly reminded that the intention of the law is to confine the meaning of psychological incapacity
to the most serious cases of personality disorders clearly demonstrative of an utter insensitivity or inability to give meaning and
significance to the marriage.[6] In Republic v. Court of Appeals,[7] the Court gave the guidelines in the interpretation and
application of Art. 36 which are as follows:
(1)

The burden of proof to show the nullity of the marriage belongs to the plaintiff. Any doubt should be resolved
in favor of the existence and continuation of the marriage and against its dissolution and nullity...

(2)

The root cause of the psychological incapacity must be: (a) medically or clinically identified, (b) alleged in the
complaint, (c) sufficiently proven by experts and (d) clearly explained in the decision...

(3)

The incapacity must be proven to be existing at the time of the celebration of the marriage.

(4)

Such incapacity must also be shown to be medically or clinically permanent or incurable...

(5)

Such illness must be grave enough to bring about the disability of the party to assume the essential
obligations of marriage...

(6)

The essential marital obligations must be those embraced by Articles 68 up to 71 of the Family Code as
regards the husband and wife as well as Articles 220, 221 and 225 of the same Code in regard to parents and
their children. Such non-complied marital obligation(s) must also be stated in the petition, proven by evidence
and included in the text of the decision.

(7)

Interpretations given by the National Appellate Matrimonial Tribunal of the Catholic Church in the Philippines,
while not controlling or decisive, should be given great respect by our courts...

(8)

The trial court must order the prosecuting attorney or fiscal and the Solicitor General to appear as counsel for
the state. No decision shall be handed down unless the Solicitor General issues a certification, which will be
quoted in the decision, briefly stating therein his reasons for his agreement or opposition, as the case may be,
to the petition...[8]

In the present case, the spouses frequent squabbles and respondents refusal to sleep with petitioner and be supportive
to him do not constitute psychological incapacity. The records show that petitioner and respondent were living in harmony in the
first few years of their marriage, which bore them four children. Psychological incapacity must be more than just a difficulty,
refusal or neglect in the performance of some marital obligations, [9] it is essential that they must be shown to be incapable of
doing so, due to some psychological illness[10] existing at the time of the celebration of the marriage.
It will be noted that respondent did not undergo psychological tests. Witness de Castros diagnosis was based solely on
petitioners avowals and not on personal knowledge of the spouses relationship. Hence, de Castros diagnosis is based on
hearsay and has no probative value.[11]
Further, de Castros statement that professionals are per se incapacitated to perform the essential obligations of marriage
because their profession allows them little time for family life is highly debatable.
Lastly, petitioner failed to show that grave and incurable incapacity, on the part of both spouses, existed at the time of the
celebration of the marriage. Their bickerings and arguments even before their marriage and respondents scandalous outbursts
in public, at most, show their immaturity, and immaturity does not constitute psychological incapacity. [12] Thus so far, both
petitioner and respondent have not shown proof of a natal or supervening disabling factor, an adverse integral element in their
personality structure that effectively incapacitates them from accepting and complying with the obligations essential to marriage.
[13]

WHEREFORE,

the

petition

is DENIED for

lack

of

merit. The Decision

dated January

Resolution datedFebruary 4, 2004 of the Court of Appeals in CA-GR CV No. 65677 are hereby AFFIRMED.
No pronouncement as to costs.

EDWARD KENNETH NGO TE,


Petitioner,

G.R. No. 161793

8,

2003 and

the

Present:
- versus ROWENA ONG GUTIERREZ
YU-TE,
Respondent,
REPUBLIC OF
THEPHILIPPINES,
Oppositor.

YNARES-SANTIAGO, J.,
Chairperson,
AUSTRIA-MARTINEZ,
CHICO-NAZARIO,
NACHURA, and
PERALTA, JJ.
Promulgated:
February 13, 2009

x------------------------------------------------------------------------------------x

DECISION
NACHURA, J.:

Far from novel is the issue involved in this petition. Psychological incapacity, since its incorporation in our laws, has
become a clichd subject of discussion in our jurisprudence. The Court treats this case, however, with much ado, it having
realized that current jurisprudential doctrine has unnecessarily imposed a perspective by which psychological incapacity should
be viewed, totally inconsistent with the way the concept was formulatedfree in form and devoid of any definition.
For the resolution of the Court is a petition for review on certiorari under Rule 45 of the Rules of Court assailing the
August 5, 2003 Decision[1] of the Court of Appeals (CA) in CA-G.R. CV No. 71867. The petition further assails the January 19,
2004 Resolution[2] denying the motion for the reconsideration of the challenged decision.
The relevant facts and proceedings follow.
Petitioner Edward Kenneth Ngo Te first got a glimpse of respondent Rowena Ong Gutierrez Yu-Te in a gathering
organized by the Filipino-Chinese association in their college. Edward was then initially attracted to Rowenas close friend; but,
as the latter already had a boyfriend, the young man decided to court Rowena. That was in January 1996, when petitioner was
a sophomore student and respondent, a freshman. [3]
Sharing similar angst towards their families, the two understood one another and developed a certain degree of closeness
towards each other. In March 1996, or around three months after their first meeting, Rowena asked Edward that they elope. At
first, he refused, bickering that he was young and jobless. Her persistence, however, made him relent. Thus, they
left Manila and sailed toCebu that month; he, providing their travel money and she, purchasing the boat ticket. [4]
However, Edwards P80,000.00 lasted for only a month. Their pension house accommodation and daily sustenance fast
depleted it. And they could not find a job. In April 1996, they decided to go back to Manila. Rowena proceeded to her uncles
house and Edward to his parents home. As his family was abroad, and Rowena kept on telephoning him, threatening him that
she would commit suicide, Edward agreed to stay with Rowena at her uncles place. [5]
On April 23, 1996, Rowenas uncle brought the two to a court to get married. He was then 25 years old, and she, 20.
[6]

The two then continued to stay at her uncles place where Edward was treated like a prisonerhe was not allowed to go out

unaccompanied. Her uncle also showed Edward his guns and warned the latter not to leave Rowena. [7] At one point, Edward
was able to call home and talk to his brother who suggested that they should stay at their parents home and live with
them. Edward relayed this to Rowena who, however, suggested that he should get his inheritance so that they could live on
their own. Edward talked to his father about this, but the patriarch got mad, told Edward that he would be disinherited, and
insisted that Edward must go home.[8]
After a month, Edward escaped from the house of Rowenas uncle, and stayed with his parents. His family then hid him
from Rowena and her family whenever they telephoned to ask for him. [9]
In June 1996, Edward was able to talk to Rowena. Unmoved by his persistence that they should live with his parents, she
said that it was better for them to live separate lives. They then parted ways. [10]
After almost four years, or on January 18, 2000, Edward filed a petition before the Regional Trial Court (RTC) of Quezon
City, Branch 106, for the annulment of his marriage to Rowena on the basis of the latters psychological incapacity. This was
docketed as Civil Case No. Q-00-39720.[11]

As Rowena did not file an answer, the trial court, on July 11, 2000, ordered the Office of the City Prosecutor (OCP)
ofQuezon City to investigate whether there was collusion between the parties. [12] In the meantime, on July 27, 2000, the Office
of the Solicitor General (OSG) entered its appearance and deputized the OCP to appear on its behalf and assist it in the
scheduled hearings.[13]
On August 23, 2000, the OCP submitted an investigation report stating that it could not determine if there was collusion
between the parties; thus, it recommended trial on the merits. [14]
The clinical psychologist who examined petitioner found both parties psychologically incapacitated, and made the
following findings and conclusions:
BACKGROUND DATA & BRIEF MARITAL HISTORY:
EDWARD KENNETH NGO TE is a [29-year-old] Filipino male adult born and baptized Born Again Christian
at Manila. He finished two years in college at AMA Computer College last 1994 and is currently unemployed. He is
married to and separated from ROWENA GUTIERREZ YU-TE. He presented himself at my office for a
psychological evaluation in relation to his petition for Nullification of Marriage against the latter by the grounds of
psychological incapacity. He is now residing at 181 P. Tuazon Street, Quezon City.
Petitioner got himself three siblings who are now in business and one deceased sister. Both his parents are
also in the business world by whom he [considers] as generous, hospitable, and patient. This said virtues are said
to be handed to each of the family member. He generally considers himself to be quiet and simple. He clearly
remembers himself to be afraid of meeting people. After 1994, he tried his luck in being a Sales Executive of
Mansfield International Incorporated. And because of job incompetence, as well as being quiet and loner, he did not
stay long in the job until 1996. His interest lie[s] on becoming a full servant of God by being a priest or a pastor. He
[is] said to isolate himself from his friends even during his childhood days as he only loves to read the Bible and hear
its message.
Respondent is said to come from a fine family despite having a lazy father and a disobedient wife. She is
said to have not finish[ed] her collegiate degree and shared intimate sexual moments with her boyfriend prior to that
with petitioner.
In January of 1996, respondent showed her kindness to petitioner and this became the foundation of their
intimate relationship. After a month of dating, petitioner mentioned to respondent that he is having problems with his
family. Respondent surprisingly retorted that she also hates her family and that she actually wanted to get out of
their lives. From that [time on], respondent had insisted to petitioner that they should elope and live
together. Petitioner hesitated because he is not prepared as they are both young and inexperienced, but she
insisted that they would somehow manage because petitioner is rich. In the last week of March 1996, respondent
seriously brought the idea of eloping and she already bought tickets for the boat going to Cebu. Petitioner
reluctantly agreed to the idea and so they eloped to Cebu. The parties are supposed to stay at the house of a friend
of respondent, but they were not able to locate her, so petitioner was compelled to rent an apartment. The parties
tried to look for a job but could not find any so it was suggested by respondent that they should go back and seek
help from petitioners parents. When the parties arrived at the house of petitioner, all of his whole family was all out
of the country so respondent decided to go back to her home for the meantime while petitioner stayed behind at
their home. After a few days of separation, respondent called petitioner by phone and said she wanted to talk to
him. Petitioner responded immediately and when he arrived at their house, respondent confronted petitioner as to
why he appeared to be cold, respondent acted irrationally and even threatened to commit suicide. Petitioner got
scared so he went home again. Respondent would call by phone every now and then and became angry as
petitioner does not know what to do. Respondent went to the extent of threatening to file a case against petitioner
and scandalize his family in the newspaper. Petitioner asked her how he would be able to make amends and at this
point in time[,] respondent brought the idea of marriage. Petitioner[,] out of frustration in life[,] agreed to her to pacify
her. And so on April 23, 1996, respondents uncle brought the parties to Valenzuela[,] and on that very same day[,]
petitioner was made to sign the Marriage Contract before the Judge. Petitioner actually never applied for any
Marriage License.
Respondent decided that they should stay first at their house until after arrival of the parents of
petitioner. But when the parents of petitioner arrived, respondent refused to allow petitioner to go home. Petitioner
was threatened in so many ways with her uncle showing to him many guns. Respondent even threatened that if he
should persist in going home, they will commission their military friends to harm his family. Respondent even made
petitioner sign a declaration that if he should perish, the authorities should look for him at his parents[ ]and relatives[
]houses. Sometime in June of 1996, petitioner was able to escape and he went home. He told his parents about
his predicament and they forgave him and supported him by giving him military escort. Petitioner, however, did not
inform them that he signed a marriage contract with respondent. When they knew about it[,] petitioner was referred
for counseling. Petitioner[,] after the counseling[,] tried to contact respondent. Petitioner offered her to live instead
to[sic] the home of petitioners parents while they are still studying. Respondent refused the idea and claimed that
she would only live with him if they will have a separate home of their own and be away from his parents. She also
intimated to petitioner that he should already get his share of whatever he would inherit from his parents so they can
start a new life. Respondent demanded these not knowing [that] the petitioner already settled his differences with
his own family. When respondent refused to live with petitioner where he chose for them to stay, petitioner decided
to tell her to stop harassing the home of his parents. He told her already that he was disinherited and since he also
does not have a job, he would not be able to support her. After knowing that petitioner does not have any money
anymore, respondent stopped tormenting petitioner and informed petitioner that they should live separate lives.
The said relationship between Edward and Rowena is said to be undoubtedly in the wreck and weaklyfounded. The break-up was caused by both parties[] unreadiness to commitment and their young age. He was still
in the state of finding his fate and fighting boredom, while she was still egocentrically involved with herself.
TESTS ADMINISTERED:
Revised Beta Examination
Bender Visual Motor Gestalt Test
Draw A Person Test
Rorschach Psychodiagnostic Test
Sachs Sentence Completion Test
MMPI
TEST RESULTS & EVALUATION:

Both petitioner and respondent are dubbed to be emotionally immature and recklessly impulsive upon
swearing to their marital vows as each of them was motivated by different notions on marriage.
Edward Kenneth Ngo Te, the petitioner in this case[,] is said to be still unsure and unready so as to commit
himself to marriage. He is still founded to be on the search of what he wants in life. He is absconded as an introvert
as he is not really sociable and displays a lack of interest in social interactions and mingling with other
individuals. He is seen too akin to this kind of lifestyle that he finds it boring and uninteresting to commit himself to a
relationship especially to that of respondent, as aggravated by her dangerously aggressive moves. As he is more of
the reserved and timid type of person, as he prefer to be religiously attached and spend a solemn time alone.
ROWENA GUTIERREZ YU-TE, the respondent, is said to be of the aggressive-rebellious type of woman. She
is seen to be somewhat exploitative in her [plight] for a life of wealth and glamour. She is seen to take move on
marriage as she thought that her marriage with petitioner will bring her good fortune because he is part of a rich
family. In order to have her dreams realized, she used force and threats knowing that [her] husband is somehow
weak-willed. Upon the realization that there is really no chance for wealth, she gladly finds her way out of the
relationship.
REMARKS:
Before going to marriage, one should really get to know himself and marry himself before submitting to marital
vows. Marriage should not be taken out of intuition as it is profoundly a serious institution solemnized by religious
and law. In the case presented by petitioner and respondent[,] (sic) it is evidently clear that both parties have
impulsively taken marriage for granted as they are still unaware of their own selves. He is extremely introvert to the
point of weakening their relationship by his weak behavioral disposition. She, on the other hand[,] is extremely
exploitative and aggressive so as to be unlawful, insincere and undoubtedly uncaring in her strides toward
convenience. It is apparent that she is suffering the grave, severe, and incurable presence of Narcissistic and
Antisocial Personality Disorder that started since childhood and only manifested during marriage. Both parties
display psychological incapacities that made marriage a big mistake for them to take. [15]

The trial court, on July 30, 2001, rendered its Decision [16] declaring the marriage of the parties null and void on the ground
that both parties were psychologically incapacitated to comply with the essential marital obligations. [17] The Republic,
represented by the OSG, timely filed its notice of appeal. [18]
On review, the appellate court, in the assailed August 5, 2003 Decision [19] in CA-G.R. CV No. 71867, reversed and set
aside the trial courts ruling.[20] It ruled that petitioner failed to prove the psychological incapacity of respondent. The clinical
psychologist did not personally examine respondent, and relied only on the information provided by petitioner. Further, the
psychological incapacity was not shown to be attended by gravity, juridical antecedence and incurability. In sum, the evidence
adduced fell short of the requirements stated in Republic v. Court of Appeals and Molina [21] needed for the declaration of nullity
of the marriage under Article 36 of the Family Code. [22] The CA faulted the lower court for rendering the decision without the
required certification of the OSG briefly stating therein the OSGs reasons for its agreement with or opposition to, as the case
may be, the petition.[23] The CA later denied petitioners motion for reconsideration in the likewise assailed January 19, 2004
Resolution.[24]
Dissatisfied, petitioner filed before this Court the instant petition for review on certiorari. On June 15, 2005, the Court gave
due course to the petition and required the parties to submit their respective memoranda. [25]
In his memorandum,[26] petitioner argues that the CA erred in substituting its own judgment for that of the trial court. He
posits that the RTC declared the marriage void, not only because of respondents psychological incapacity, but rather due to
both parties psychological incapacity. Petitioner also points out that there is no requirement for the psychologist to personally
examine respondent. Further, he avers that the OSG is bound by the actions of the OCP because the latter represented it
during the trial; and it had been furnished copies of all the pleadings, the trial court orders and notices. [27]
For its part, the OSG contends in its memorandum, [28] that the annulment petition filed before the RTC contains no
statement of the essential marital obligations that the parties failed to comply with. The root cause of the psychological
incapacity was likewise not alleged in the petition; neither was it medically or clinically identified. The purported incapacity of
both parties was not shown to be medically or clinically permanent or incurable. And the clinical psychologist did not personally
examine the respondent. Thus, the OSG concludes that the requirements in Molina[29] were not satisfied.[30]
The Court now resolves the singular issue of whether, based on Article 36 of the Family Code, the marriage between the
parties is null and void.[31]
I.
We begin by examining the provision, tracing its origin and charting the development of jurisprudence interpreting it.
Article 36 of the Family Code[32] provides:
Article 36. A marriage contracted by any party who, at the time of the celebration, was psychologically
incapacitated to comply with the essential marital obligations of marriage, shall likewise be void even if such
incapacity becomes manifest only after its solemnization.

As borne out by the deliberations of the Civil Code Revision Committee that drafted the Family Code, Article 36 was
based on grounds available in the Canon Law. Thus, Justice Flerida Ruth P. Romero elucidated in her separate opinion
in Santos v. Court of Appeals:[33]

However, as a member of both the Family Law Revision Committee of the Integrated Bar of
the Philippines and the Civil Code Revision Commission of the UP Law Center, I wish to add some observations.
The letter dated April 15, 1985 of then Judge Alicia V. Sempio-Diy written in behalf of the Family Law and Civil Code
Revision Committee to then Assemblywoman Mercedes Cojuangco-Teodoro traced the background of the inclusion
of the present Article 36 in the Family Code.
During its early meetings, the Family Law Committee had thought of including a chapter on
absolute divorce in the draft of a new Family Code (Book I of the Civil Code) that it had been tasked by
the IBP and the UP Law Center to prepare. In fact, some members of the Committee were in favor of a
no-fault divorce between the spouses after a number of years of separation, legal or de facto. Justice
J.B.L. Reyes was then requested to prepare a proposal for an action for dissolution of marriage and the
effects thereof based on two grounds: (a) five continuous years of separation between the spouses, with
or without a judicial decree of legal separation, and (b) whenever a married person would have obtained
a decree of absolute divorce in another country. Actually, such a proposal is one for absolute divorce but
called by another name. Later, even the Civil Code Revision Committee took time to discuss the
proposal of Justice Reyes on this matter.
Subsequently, however, when the Civil Code Revision Committee and Family Law Committee
started holding joint meetings on the preparation of the draft of the New Family Code, they agreed and
formulated the definition of marriage as
a special contract of permanent partnership between a man and a woman entered into
in accordance with law for the establishment of conjugal and family life. It is an inviolable
social institution whose nature, consequences, and incidents are governed by law and not
subject to stipulation, except that marriage settlements may fix the property relations during
the marriage within the limits provided by law.
With the above definition, and considering the Christian traditional concept of marriage of the
Filipino people as a permanent, inviolable, indissoluble social institution upon which the family and
society are founded, and also realizing the strong opposition that any provision on absolute divorce
would encounter from the Catholic Church and the Catholic sector of our citizenry to whom the great
majority of our people belong, the two Committees in their joint meetings did not pursue the idea of
absolute divorce and, instead, opted for an action for judicial declaration of invalidity of marriage based
on grounds available in the Canon Law. It was thought that such an action would not only be an
acceptable alternative to divorce but would also solve the nagging problem of church annulments of
marriages on grounds not recognized by the civil law of the State. Justice Reyes was, thus, requested to
again prepare a draft of provisions on such action for celebration of invalidity of marriage. Still later, to
avoid the overlapping of provisions on void marriages as found in the present Civil Code and those
proposed by Justice Reyes on judicial declaration of invalidity of marriage on grounds similar to the
Canon Law, the two Committees now working as a Joint Committee in the preparation of a New Family
Code decided to consolidate the present provisions on void marriages with the proposals of Justice
Reyes. The result was the inclusion of an additional kind of void marriage in the enumeration of void
marriages in the present Civil Code, to wit:
(7) those marriages contracted by any party who, at the time of the celebration, was
wanting in the sufficient use of reason or judgment to understand the essential nature of
marriage or was psychologically or mentally incapacitated to discharge the essential marital
obligations, even if such lack or incapacity is made manifest after the celebration.
as well as the following implementing provisions:
Art. 32. The absolute nullity of a marriage may be invoked or pleaded only on the basis
of a final judgment declaring the marriage void, without prejudice to the provision of Article
34.
Art. 33. The action or defense for the declaration of the absolute nullity of a marriage
shall not prescribe.
xxx

xxx

xxx

It is believed that many hopelessly broken marriages in our country today may already be dissolved or annulled on
the grounds proposed by the Joint Committee on declaration of nullity as well as annulment of marriages, thus
rendering an absolute divorce law unnecessary. In fact, during a conference with Father Gerald Healy of the Ateneo
University, as well as another meeting with Archbishop Oscar Cruz of the Archdiocese of Pampanga, the Joint
Committee was informed that since Vatican II, the Catholic Church has been declaring marriages null and void on
the ground of lack of due discretion for causes that, in other jurisdictions, would be clear grounds for divorce, like
teen-age or premature marriages; marriage to a man who, because of some personality disorder or disturbance,
cannot support a family; the foolish or ridiculous choice of a spouse by an otherwise perfectly normal person;
marriage to a woman who refuses to cohabit with her husband or who refuses to have children. Bishop Cruz also
informed the Committee that they have found out in tribunal work that a lot of machismo among husbands are
manifestations of their sociopathic personality anomaly, like inflicting physical violence upon their wives,
constitutional indolence or laziness, drug dependence or addiction, and psychosexual anomaly. [34]

In her separate opinion in Molina,[35] she expounded:


At the Committee meeting of July 26, 1986, the draft provision read:
(7)
Those marriages contracted by any party who, at the time of the celebration, was wanting in the
sufficient use of reason or judgment to understand the essential nature of marriage or was psychologically or
mentally incapacitated to discharge the essential marital obligations, even if such lack of incapacity is made manifest
after the celebration.
The twists and turns which the ensuing discussion took finally produced the following revised provision even
before the session was over:
(7)
That contracted by any party who, at the time of the celebration, was psychologically incapacitated to
discharge the essential marital obligations, even if such lack or incapacity becomes manifest after the celebration.
Noticeably, the immediately preceding formulation above has dropped any reference to wanting in the
sufficient use of reason or judgment to understand the essential nature of marriage and to mentally incapacitated.

It was explained that these phrases refer to defects in the mental faculties vitiating consent, which is not the
idea . . . but lack of appreciation of one's marital obligation. There being a defect in consent, it is clear that it should
be a ground for voidable marriage because there is the appearance of consent and it is capable of convalidation for
the simple reason that there are lucid intervals and there are cases when the insanity is curable . . . Psychological
incapacity does not refer to mental faculties and has nothing to do with consent; it refers to obligations attendant to
marriage.
My own position as a member of the Committee then was that psychological incapacity is, in a sense, insanity
of a lesser degree.
As to the proposal of Justice Caguioa to use the term psychological or mental impotence, Archbishop Oscar
Cruz opined in the earlier February 9, 1984 session that this term is an invention of some churchmen who are
moralists but not canonists, that is why it is considered a weak phrase. He said that the Code of Canon Law would
rather express it as psychological or mental incapacity to discharge . . . Justice Ricardo C. Puno opined that
sometimes a person may be psychologically impotent with one but not with another.
One of the guidelines enumerated in the majority opinion for the interpretation and application of Art. 36 is:
Such incapacity must also be shown to be medically or clinically permanent or incurable. Such incurability may be
absolute or even relative only in regard to the other spouse, not necessarily absolutely against everyone of the same
sex.
The Committee, through Prof. Araceli T. Barrera, considered the inclusion of the phrase and is incurable but
Prof. Esteban B. Bautista commented that this would give rise to the question of how they will determine curability
and Justice Caguioa agreed that it would be more problematic. Yet, the possibility that one may be cured after the
psychological incapacity becomes manifest after the marriage was not ruled out by Justice Puno and Justice Alice
Sempio-Diy. Justice Caguioa suggested that the remedy was to allow the afflicted spouse to remarry.
For clarity, the Committee classified the bases for determining void marriages, viz.:
1.
2.
3.

lack of one or more of the essential requisites of marriage as contract;


reasons of public policy;
special cases and special situations.

The ground of psychological incapacity was subsumed under special cases and special situations, hence, its
special treatment in Art. 36 in the Family Code as finally enacted.
Nowhere in the Civil Code provisions on Marriage is there a ground for avoiding or annulling marriages that
even comes close to being psychological in nature.
Where consent is vitiated due to circumstances existing at the time of the marriage, such marriage which
stands valid until annulled is capable of ratification or convalidation.
On the other hand, for reasons of public policy or lack of essential requisites, some marriages are void from
the beginning.
With the revision of Book I of the Civil Code, particularly the provisions on Marriage, the drafters, now open to
fresh winds of change in keeping with the more permissive mores and practices of the time, took a leaf from the
relatively liberal provisions of Canon Law.
Canon 1095 which states, inter alia, that the following persons are incapable of contracting marriage: 3.
(those) who, because of causes of a psychological nature, are unable to assume the essential obligations of
marriage provided the model for what is now Art. 36 of the Family Code: A marriage contracted by any party who,
at the time of the celebration, was psychologically incapacitated to comply with the essential marital obligations of
marriage, shall likewise be void even if such incapacity becomes manifest only after its solemnization.
It bears stressing that unlike in Civil Law, Canon Law recognizes only two types of marriages with respect to
their validity: valid and void. Civil Law, however, recognizes an intermediate state, the voidable or annullable
marriages. When the Ecclesiastical Tribunal annuls a marriage, it actually declares the marriage null and void, i.e.,
it never really existed in the first place, for a valid sacramental marriage can never be dissolved. Hence, a properly
performed and consummated marriage between two living Roman Catholics can only be nullified by the formal
annulment process which entails a full tribunal procedure with a Court selection and a formal hearing.
Such so-called church annulments are not recognized by Civil Law as severing the marriage ties as to
capacitate the parties to enter lawfully into another marriage. The grounds for nullifying civil marriage, not being
congruent with those laid down by Canon Law, the former being more strict, quite a number of married couples have
found themselves in limbofreed from the marriage bonds in the eyes of the Catholic Church but yet unable to
contract a valid civil marriage under state laws. Heedless of civil law sanctions, some persons contract new
marriages or enter into live-in relationships.
It was precisely to provide a satisfactory solution to such anomalous situations that the Civil Law Revision
Committee decided to engraft the Canon Law concept of psychological incapacity into the Family Codeand
classified the same as a ground for declaring marriages void ab initio or totally inexistent from the beginning.
A brief historical note on the Old Canon Law (1917). This Old Code, while it did not provide directly for
psychological incapacity, in effect, recognized the same indirectly from a combination of three old canons: Canon
#1081 required persons to be capable according to law in order to give valid consent; Canon #1082 required that
persons be at least not ignorant of the major elements required in marriage; and Canon #1087 (the force and fear
category) required that internal and external freedom be present in order for consent to be valid. This line of
interpretation produced two distinct but related grounds for annulment called lack of due discretion and lack of due
competence. Lack of due discretion means that the person did not have the ability to give valid consent at the time
of the wedding and, therefore, the union is invalid. Lack of due competence means that the person was incapable of
carrying out the obligations of the promise he or she made during the wedding ceremony.
Favorable annulment decisions by the Roman Rota in the 1950s and 1960s involving sexual disorders such
as homosexuality and nymphomania laid the foundation for a broader approach to the kind of proof necessary for
psychological grounds for annulment. TheRota had reasoned for the first time in several cases that the capacity to
give valid consent at the time of marriage was probably not present in persons who had displayed such problems
shortly after the marriage. The nature of this change was nothing short of revolutionary. Once the Rotaitself had
demonstrated a cautious willingness to use this kind of hindsight, the way was paved for what came after
1970. Diocesan Tribunals began to accept proof of serious psychological problems that manifested themselves
shortly after the ceremony as proof of an inability to give valid consent at the time of the ceremony.[36]

Interestingly, the Committee did not give any examples of psychological incapacity for fear that by so doing, it might limit
the applicability of the provision under the principle of ejusdem generis. The Committee desired that the courts should interpret
the provision on a case-to-case basis; guided by experience, the findings of experts and researchers in psychological
disciplines, and by decisions of church tribunals which, although not binding on the civil courts, may be given persuasive effect
since the provision itself was taken from the Canon Law.[37] The law is then so designed as to allow some resiliency in its
application.[38]
Yet, as held in Santos,[39] the phrase psychological incapacity is not meant to comprehend all possible cases of
psychoses. It refers to no less than a mental (not physical) incapacity that causes a party to be truly noncognitive of the basic
marital covenants that concomitantly must be assumed and discharged by the parties to the marriage which, as expressed by
Article 68[40] of the Family Code, include their mutual obligations to live together, observe love, respect and fidelity; and render
help and support. The intendment of the law has been to confine it to the most serious of cases of personality disorders clearly
demonstrative of an utter insensitivity or inability to give meaning and significance to the marriage. [41] This interpretation is, in
fact, consistent with that in Canon Law, thus:
3.5.3.1. The Meaning of Incapacity to Assume. A sharp conceptual distinction must be made between the
second and third paragraphs of C.1095, namely between the grave lack of discretionary judgment and the incapacity
to assume the essential obligation. Mario Pompedda, a rotal judge, explains the difference by an ordinary, if
somewhat banal, example. Jose wishes to sell a house to Carmela, and on the assumption that they are capable
according to positive law to enter such contract, there remains the object of the contract, viz, the house. The house
is located in a different locality, and prior to the conclusion of the contract, the house was gutted down by fire
unbeknown to both of them. This is the hypothesis contemplated by the third paragraph of the canon. The third
paragraph does not deal with the psychological process of giving consent because it has been established a priori
that both have such a capacity to give consent, and they both know well the object of their consent [the house and
its particulars]. Rather, C.1095.3 deals with the object of the consent/contract which does not exist. The contract is
invalid because it lacks its formal object. The consent as a psychological act is both valid and sufficient. The
psychological act, however, is directed towards an object which is not available. Urbano Navarrete summarizes this
distinction: the third paragraph deals not with the positing of consent but with positing the object of consent. The
person may be capable of positing a free act of consent, but he is not capable of fulfilling the responsibilities he
assumes as a result of the consent he elicits.
Since the address of Pius XII to the auditors of the Roman Rota in 1941 regarding psychic incapacity with respect to
marriage arising from pathological conditions, there has been an increasing trend to understand as ground of nullity
different from others, the incapacity to assume the essential obligations of marriage, especially the incapacity which
arises from sexual anomalies. Nymphomania is a sample which ecclesiastical jurisprudence has studied under this
rubric.
The problem as treated can be summarized, thus: do sexual anomalies always and in every case imply a grave
psychopathological condition which affects the higher faculties of intellect, discernment, and freedom; or are there
sexual anomalies that are purely so that is to say, they arise from certain physiological dysfunction of the hormonal
system, and they affect the sexual condition, leaving intact the higher faculties however, so that these persons are
still capable of free human acts. The evidence from the empirical sciences is abundant that there are certain
anomalies of a sexual nature which may impel a person towards sexual activities which are not normal, either with
respect to its frequency [nymphomania, satyriasis] or to the nature of the activity itself [sadism, masochism,
homosexuality]. However, these anomalies notwithstanding, it is altogether possible that the higher faculties remain
intact such that a person so afflicted continues to have an adequate understanding of what marriage is and of the
gravity of its responsibilities. In fact, he can choose marriage freely. The question though is whether such a person
can assume those responsibilities which he cannot fulfill, although he may be able to understand them. In this latter
hypothesis, the incapacity to assume the essential obligations of marriage issues from the incapacity to posit the
object of consent, rather than the incapacity to posit consent itself.
Ecclesiastical jurisprudence has been hesitant, if not actually confused, in this regard. The initial steps taken by
church courts were not too clear whether this incapacity is incapacity to posit consent or incapacity to posit the
object of consent. A case c. Pinna, for example, arrives at the conclusion that the intellect, under such an irresistible
impulse, is prevented from properly deliberating and its judgment lacks freedom. This line of reasoning supposes
that the intellect, at the moment of consent, is under the influence of this irresistible compulsion, with the inevitable
conclusion that such a decision, made as it was under these circumstances, lacks the necessary freedom. It would
be incontrovertible that a decision made under duress, such as this irresistible impulse, would not be a free act. But
this is precisely the question: is it, as a matter of fact, true that the intellect is always and continuously under such an
irresistible compulsion? It would seem entirely possible, and certainly more reasonable, to think that there are
certain cases in which one who is sexually hyperaesthetic can understand perfectly and evaluate quite maturely
what marriage is and what it implies; his consent would be juridically ineffective for this one reason that he cannot
posit the object of consent, the exclusive jus in corpus to be exercised in a normal way and with usually regularity. It
would seem more correct to say that the consent may indeed be free, but is juridically ineffective because the party
is consenting to an object that he cannot deliver. The house he is selling was gutted down by fire.
3.5.3.2. Incapacity as an Autonomous Ground. Sabattani seems to have seen his way more clearly through this
tangled mess, proposing as he did a clear conceptual distinction between the inability to give consent on the one
hand, and the inability to fulfill the object of consent, on the other. It is his opinion that nymphomaniacs usually
understand the meaning of marriage, and they are usually able to evaluate its implications. They would have no
difficulty with positing a free and intelligent consent. However, such persons, capable as they are of eliciting an
intelligent and free consent, experience difficulty in another sphere: delivering the object of the consent. Anne,
another rotal judge, had likewise treated the difference between the act of consenting and the act of positing the
object of consent from the point of view of a person afflicted with nymphomania. According to him, such an affliction
usually leaves the process of knowing and understanding and evaluating intact. What it affects is the object of
consent: the delivering of the goods.
3.5.3.3 Incapacity as Incapacity to Posit the Object of Consent. From the selected rotal jurisprudence cited,
supra, it is possible to see a certain progress towards a consensus doctrine that the incapacity to assume the
essential obligations of marriage (that is to say, the formal object of consent) can coexist in the same person with the
ability to make a free decision, an intelligent judgment, and a mature evaluation and weighing of things. The
decision coram Sabattani concerning a nymphomaniac affirmed that such a spouse can have difficulty not only with
regard to the moment of consent but also, and especially, with regard to the matrimonium in facto esse. The
decision concludes that a person in such a condition is incapable of assuming the conjugal obligation of fidelity,
although she may have no difficulty in understanding what the obligations of marriage are, nor in the weighing and
evaluating of those same obligations.

Prior to the promulgation of the Code of Canon Law in 1983, it was not unusual to refer to this ground as moral
impotence or psychic impotence, or similar expressions to express a specific incapacity rooted in some anomalies
and disorders in the personality. These anomalies leave intact the faculties of the will and the intellect. It is qualified
as moral or psychic, obviously to distinguish it from the impotence that constitutes the impediment dealt with by
C.1084. Nonetheless, the anomalies render the subject incapable of binding himself in a valid matrimonial pact, to
the extent that the anomaly renders that person incapable of fulfilling the essential obligations. According to the
principle affirmed by the long tradition of moral theology: nemo ad impossibile tenetur.
xxxx
3.5.3.5 Indications of Incapacity. There is incapacity when either or both of the contractants are not capable of
initiating or maintaining this consortium. One immediately thinks of those cases where one of the parties is so selfcentered [e.g., a narcissistic personality] that he does not even know how to begin a union with the other, let alone
how to maintain and sustain such a relationship. A second incapacity could be due to the fact that the spouses are
incapable of beginning or maintaining a heterosexual consortium, which goes to the very substance of
matrimony. Another incapacity could arise when a spouse is unable to concretize the good of himself or of the other
party. The canon speaks, not of the bonum partium,but of the bonum conjugum. A spouse who is capable only of
realizing or contributing to the good of the other party qua persona rather than qua conjunx would be deemed
incapable of contracting marriage. Such would be the case of a person who may be quite capable of procuring the
economic good and the financial security of the other, but not capable of realizing the bonum conjugale of the
other. These are general strokes and this is not the place for detained and individual description.
A rotal decision c. Pinto resolved a petition where the concrete circumstances of the case concerns a person
diagnosed to be suffering from serious sociopathy. He concluded that while the respondent may have understood,
on the level of the intellect, the essential obligations of marriage, he was not capable of assuming them because of
his constitutional immorality.
Stankiewicz clarifies that the maturity and capacity of the person as regards the fulfillment of responsibilities is
determined not only at the moment of decision but also and especially during the moment of execution of
decision. And when this is applied to constitution of the marital consent, it means that the actual fulfillment of the
essential obligations of marriage is a pertinent consideration that must be factored into the question of whether a
person was in a position to assume the obligations of marriage in the first place. When one speaks of the inability of
the party to assume and fulfill the obligations, one is not looking at matrimonium in fieri, but also and especially
at matrimonium in facto esse. In [the] decision of 19 Dec. 1985, Stankiewicz collocated the incapacity of the
respondent to assume the essential obligations of marriage in the psychic constitution of the person, precisely on
the basis of his irresponsibility as regards money and his apathy as regards the rights of others that he had
violated. Interpersonal relationships are invariably disturbed in the presence of this personality disorder. A lack of
empathy (inability to recognize and experience how others feel) is common. A sense of entitlement, unreasonable
expectation, especially favorable treatment, is usually present. Likewise common is interpersonal exploitativeness,
in which others are taken advantage of in order to achieve ones ends.
Authors have made listings of obligations considered as essential matrimonial obligations. One of them is the right
to the communio vitae. This and their corresponding obligations are basically centered around the good of the
spouses and of the children. Serious psychic anomalies, which do not have to be necessarily incurable, may give
rise to the incapacity to assume any, or several, or even all of these rights. There are some cases in which
interpersonal relationship is impossible. Some characteristic features of inability for interpersonal relationships in
marriage include affective immaturity, narcissism, and antisocial traits.
Marriage and Homosexuality. Until 1967, it was not very clear under what rubric homosexuality was understood to
be invalidating of marriage that is to say, is homosexuality invalidating because of the inability to evaluate the
responsibilities of marriage, or because of the inability to fulfill its obligations. Progressively, however, rotal
jurisprudence began to understand it as incapacity to assume the obligations of marriage so that by 1978, Parisella
was able to consider, with charity, homosexuality as an autonomous ground of nullity. This is to say that a person so
afflicted is said to be unable to assume the essential obligations of marriage. In this same rotal decision, the object
of matrimonial consent is understood to refer not only to the jus in corpus but also the consortium totius vitae. The
third paragraph of C.1095 [incapacity to assume the essential obligations of marriage] certainly seems to be the
more adequate juridical structure to account for the complex phenomenon that homosexuality is. The homosexual
is not necessarily impotent because, except in very few exceptional cases, such a person is usually capable of full
sexual relations with the spouse. Neither is it a mental infirmity, and a person so afflicted does not necessarily suffer
from a grave lack of due discretion because this sexual anomaly does not by itself affect the critical, volitive, and
intellectual faculties. Rather, the homosexual person is unable to assume the responsibilities of marriage because
he is unable to fulfill this object of the matrimonial contract. In other words, the invalidity lies, not so much in the
defect of consent, as in the defect of the object of consent.
3.5.3.6 Causes of Incapacity. A last point that needs to be addressed is the source of incapacity specified by the
canon: causes of a psychological nature. Pompedda proffers the opinion that the clause is a reference to the
personality of the contractant. In other words, there must be a reference to the psychic part of the person. It is only
when there is something in the psyche or in the psychic constitution of the person which impedes his capacity that
one can then affirm that the person is incapable according to the hypothesis contemplated by C.1095.3. A person is
judged incapable in this juridical sense only to the extent that he is found to have something rooted in his psychic
constitution which impedes the assumption of these obligations. A bad habit deeply engrained in ones
consciousness would not seem to qualify to be a source of this invalidating incapacity. The difference being that
there seems to be some freedom, however remote, in the development of the habit, while one accepts as given
ones psychic constitution. It would seem then that the law insists that the source of the incapacity must be one
which is not the fruit of some degree of freedom. [42]

Conscious of the laws intention that it is the courts, on a case-to-case basis, that should determine whether a party to a
marriage is psychologically incapacitated, the Court, in sustaining the lower courts judgment of annulment in Tuason v. Court of
Appeals,[43] ruled that the findings of the trial court are final and binding on the appellate courts. [44]
Again, upholding the trial courts findings and declaring that its decision was not a judgment on the pleadings, the Court,
inTsoi v. Court of Appeals,[45] explained that when private respondent testified under oath before the lower court and was crossexamined by the adverse party, she thereby presented evidence in the form of testimony. Importantly, the Court, aware of
parallel decisions of Catholic marriage tribunals, ruled that the senseless and protracted refusal of one of the parties to fulfill the
marital obligation of procreating children is equivalent to psychological incapacity.

The resiliency with which the concept should be applied and the case-to-case basis by which the provision should be
interpreted, as so intended by its framers, had, somehow, been rendered ineffectual by the imposition of a set of strict
standards inMolina,[46] thus:
From their submissions and the Court's own deliberations, the following guidelines in the interpretation and
application of Art. 36 of the Family Code are hereby handed down for the guidance of the bench and the bar:
(1)
The burden of proof to show the nullity of the marriage belongs to the plaintiff. Any doubt should
be resolved in favor of the existence and continuation of the marriage and against its dissolution and nullity. This is
rooted in the fact that both our Constitution and our laws cherish the validity of marriage and unity of the family.
Thus, our Constitution devotes an entire Article on the Family, recognizing it as the foundation of the nation. It
decrees marriage as legally inviolable, thereby protecting it from dissolution at the whim of the parties. Both the
family and marriage are to be protected by the state.
The Family Code echoes this constitutional edict on marriage and the family and emphasizes their
permanence, inviolability and solidarity.
(2)
The root cause of the psychological incapacity must be (a) medically or clinically identified, (b) alleged
in the complaint, (c) sufficiently proven by experts and (d) clearly explained in the decision. Article 36 of the Family
Code requires that the incapacity must be psychologicalnot physical, although its manifestations and/or symptoms
may be physical. The evidence must convince the court that the parties, or one of them, was mentally or psychically
ill to such an extent that the person could not have known the obligations he was assuming, or knowing them, could
not have given valid assumption thereof. Although no example of such incapacity need be given here so as not to
limit the application of the provision under the principle of ejusdem generis, nevertheless such root cause must be
identified as a psychological illness and its incapacitating nature fully explained. Expert evidence may be given by
qualified psychiatrists and clinical psychologists.
(3)
The incapacity must be proven to be existing at the time of the celebration of the marriage. The
evidence must show that the illness was existing when the parties exchanged their I do's. The manifestation of the
illness need not be perceivable at such time, but the illness itself must have attached at such moment, or prior
thereto.
(4)
Such incapacity must also be shown to be medically or clinically permanent or incurable. Such
incurability may be absolute or even relative only in regard to the other spouse, not necessarily absolutely against
everyone of the same sex. Furthermore, such incapacity must be relevant to the assumption of marriage obligations,
not necessarily to those not related to marriage, like the exercise of a profession or employment in a job. Hence, a
pediatrician may be effective in diagnosing illnesses of children and prescribing medicine to cure them but may not
be psychologically capacitated to procreate, bear and raise his/her own children as an essential obligation of
marriage.
(5)
Such illness must be grave enough to bring about the disability of the party to assume the essential
obligations of marriage. Thus, mild characterological peculiarities, mood changes, occasional emotional outbursts
cannot be accepted as root causes. The illness must be shown as downright incapacity or inability, not a refusal,
neglect or difficulty, much less ill will. In other words, there is a natal or supervening disabling factor in the person,
an adverse integral element in the personality structure that effectively incapacitates the person from really
accepting and thereby complying with the obligations essential to marriage.
(6)
The essential marital obligations must be those embraced by Articles 68 up to 71 of the Family Code
as regards the husband and wife as well as Articles 220, 221 and 225 of the same Code in regard to parents and
their children. Such non-complied marital obligation(s) must also be stated in the petition, proven by evidence and
included in the text of the decision.
(7)
Interpretations given by the National Appellate Matrimonial Tribunal of the Catholic Church in
the Philippines, while not controlling or decisive, should be given great respect by our courts. It is clear that Article
36 was taken by the Family Code Revision Committee from Canon 1095 of the New Code of Canon Law, which
became effective in 1983 and which provides:
The following are incapable of contracting marriage: Those who are unable to assume the essential
obligations of marriage due to causes of psychological nature.
Since the purpose of including such provision in our Family Code is to harmonize our civil laws with the
religious faith of our people, it stands to reason that to achieve such harmonization, great persuasive weight should
be given to decisions of such appellate tribunal. Ideally subject to our law on evidencewhat is decreed as
canonically invalid should also be decreed civilly void.
This is one instance where, in view of the evident source and purpose of the Family Code provision,
contemporaneous religious interpretation is to be given persuasive effect. Here, the State and the Churchwhile
remaining independent, separate and apart from each othershall walk together in synodal cadence towards the
same goal of protecting and cherishing marriage and the family as the inviolable base of the nation.
(8)
The trial court must order the prosecuting attorney or fiscal and the Solicitor General to appear as
counsel for the state. No decision shall be handed down unless the Solicitor General issues a certification, which will
be quoted in the decision, briefly stating therein his reasons for his agreement or opposition, as the case may be, to
the petition. The Solicitor General, along with the prosecuting attorney, shall submit to the court such certification
within fifteen (15) days from the date the case is deemed submitted for resolution of the court. The Solicitor General
shall discharge the equivalent function of the defensor vinculi contemplated under Canon 1095.[47]

Noteworthy is that in Molina, while the majority of the Courts membership concurred in the ponencia of then Associate
Justice (later Chief Justice) Artemio V. Panganiban, three justices concurred in the result and another threeincluding, as
aforesaid, Justice Romerotook pains to compose their individual separate opinions. Then Justice Teodoro R. Padilla even
emphasized that each case must be judged, not on the basis of a priori assumptions, predelictions or generalizations, but
according to its own facts. In the field of psychological incapacity as a ground for annulment of marriage, it is trite to say that no
case is on all fours with another case. The trial judge must take pains in examining the factual milieu and the appellate court
must, as much as possible, avoid substituting its own judgment for that of the trial court. [48]

Predictably, however, in resolving subsequent cases, [49] the Court has applied the aforesaid standards, without too much
regard for the laws clear intention that each case is to be treated differently, as courts should interpret the provision on a
case-to-case basis; guided by experience, the findings of experts and researchers in psychological disciplines, and by decisions
of church tribunals.
In hindsight, it may have been inappropriate for the Court to impose a rigid set of rules, as the one in Molina, in resolving
all cases of psychological incapacity. Understandably, the Court was then alarmed by the deluge of petitions for the dissolution
of marital bonds, and was sensitive to the OSGs exaggeration of Article 36 as the most liberal divorce procedure in the
world.[50]The unintended consequences of Molina, however, has taken its toll on people who have to live with deviant behavior,
moral insanity and sociopathic personality anomaly, which, like termites, consume little by little the very foundation of their
families, our basic social institutions. Far from what was intended by the Court, Molina has become a strait-jacket, forcing all
sizes to fit into and be bound by it. Wittingly or unwittingly, the Court, in conveniently applying Molina, has allowed diagnosed
sociopaths, schizophrenics, nymphomaniacs, narcissists and the like, to continuously debase and pervert the sanctity of
marriage. Ironically, the Roman Rota has annulled marriages on account of the personality disorders of the said individuals. [51]
The Court need not worry about the possible abuse of the remedy provided by Article 36, for there are ample safeguards
against this contingency, among which is the intervention by the State, through the public prosecutor, to guard against collusion
between the parties and/or fabrication of evidence. [52] The Court should rather be alarmed by the rising number of cases
involving marital abuse, child abuse, domestic violence and incestuous rape.
In dissolving marital bonds on account of either partys psychological incapacity, the Court is not demolishing the
foundation of families, but it is actually protecting the sanctity of marriage, because it refuses to allow a person afflicted with a
psychological disorder, who cannot comply with or assume the essential marital obligations, from remaining in that sacred bond.
It may be stressed that the infliction of physical violence, constitutional indolence or laziness, drug dependence or addiction,
and psychosexual anomaly are manifestations of a sociopathic personality anomaly. [53] Let it be noted that in Article 36, there is
no marriage to speak of in the first place, as the same is void from the very beginning. [54] To indulge in imagery, the declaration
of nullity under Article 36 will simply provide a decent burial to a stillborn marriage.
The prospect of a possible remarriage by the freed spouses should not pose too much of a concern for the Court. First
and foremost, because it is none of its business. And second, because the judicial declaration of psychological incapacity
operates as a warning or a lesson learned. On one hand, the normal spouse would have become vigilant, and never
again marry a person with a personality disorder. On the other hand, a would-be spouse of the psychologically incapacitated
runs the risk of the latters disorder recurring in their marriage.
Lest it be misunderstood, we are not suggesting the abandonment of Molina in this case. We simply declare that, as aptly
stated by Justice Dante O. Tinga in Antonio v. Reyes,[55] there is need to emphasize other perspectives as well which should
govern the disposition of petitions for declaration of nullity under Article 36. At the risk of being redundant, we reiterate once
more the principle that each case must be judged, not on the basis of a priori assumptions, predilections or generalizations but
according to its own facts. And, to repeat for emphasis, courts should interpret the provision on a case-to-case basis; guided by
experience, the findings of experts and researchers in psychological disciplines, and by decisions of church tribunals.
II.
We now examine the instant case.
The parties whirlwind relationship lasted more or less six (6) months. They met in January 1996, eloped in March,
exchanged marital vows in May, and parted ways in June. The psychologist who provided expert testimony found both parties
psychologically incapacitated. Petitioners behavioral pattern falls under the classification of dependent personality disorder, and
respondents, that of the narcissistic and antisocial personality disorder.[56]
By the very nature of Article 36, courts, despite having the primary task and burden of decision-making, must not
discount but, instead, must consider as decisive evidence the expert opinion on the psychological and mental
temperaments of the parties.[57]
Justice Romero explained this in Molina, as follows:
Furthermore, and equally significant, the professional opinion of a psychological expert became increasingly
important in such cases. Data about the person's entire life, both before and after the ceremony, were presented to
these experts and they were asked to give professional opinions about a party's mental capacity at the time of the
wedding. These opinions were rarely challenged and tended to be accepted as decisive evidence of lack of valid
consent.
The Church took pains to point out that its new openness in this area did not amount to the addition of new
grounds for annulment, but rather was an accommodation by the Church to the advances made in psychology
during the past decades. There was now the expertise to provide the all-important connecting link between a
marriage breakdown and premarital causes.

During the 1970s, the Church broadened its whole idea of marriage from that of a legal contract to that of a
covenant. The result of this was that it could no longer be assumed in annulment cases that a person who could
intellectually understand the concept of marriage could necessarily give valid consent to marry. The ability to both
grasp and assume the real obligations of a mature, lifelong commitment are now considered a necessary
prerequisite to valid matrimonial consent.
Rotal decisions continued applying the concept of incipient psychological incapacity, not only to sexual
anomalies but to all kinds of personality disorders that incapacitate a spouse or both spouses from assuming or
carrying out the essential obligations of marriage. For marriage . . . is not merely cohabitation or the right of the
spouses to each other's body for heterosexual acts, but is, in its totality the right to the community of the whole of
life; i.e., the right to a developing lifelong relationship. Rotal decisions since 1973 have refined the meaning of
psychological or psychic capacity for marriage as presupposing the development of an adult personality;
as meaning the capacity of the spouses to give themselves to each other and to accept the other as a distinct
person; that the spouses must be other oriented since the obligations of marriage are rooted in a self-giving love;
and that the spouses must have the capacity for interpersonal relationship because marriage is more than just a
physical reality but involves a true intertwining of personalities. The fulfillment of the obligations of marriage
depends, according to Church decisions, on the strength of this interpersonal relationship. A serious incapacity for
interpersonal sharing and support is held to impair the relationship and consequently, the ability to fulfill the essential
marital obligations. The marital capacity of one spouse is not considered in isolation but in reference to the
fundamental relationship to the other spouse.
Fr. Green, in an article in Catholic Mind, lists six elements necessary to the mature marital relationship:
The courts consider the following elements crucial to the marital commitment: (1) a permanent
and faithful commitment to the marriage partner; (2) openness to children and partner; (3) stability; (4)
emotional maturity; (5) financial responsibility; (6) an ability to cope with the ordinary stresses and strains
of marriage, etc.
Fr. Green goes on to speak about some of the psychological conditions that might lead to the failure of a marriage:
At stake is a type of constitutional impairment precluding conjugal communion even with the best
intentions of the parties. Among the psychic factors possibly giving rise to his or her inability to fulfill
marital obligations are the following: (1) antisocial personality with its fundamental lack of loyalty to
persons or sense of moral values; (2) hyperesthesia, where the individual has no real freedom of sexual
choice; (3) the inadequate personality where personal responses consistently fall short of reasonable
expectations.
xxxx
The psychological grounds are the best approach for anyone who doubts whether he or she has a
case for an annulment on any other terms. A situation that does not fit into any of the more traditional
categories often fits very easily into the psychological category.
As new as the psychological grounds are, experts are already detecting a shift in their use.
Whereas originally the emphasis was on the parties' inability to exercise proper judgment at the time of
the marriage (lack of due discretion), recent cases seem to be concentrating on the parties' incapacity to
assume or carry out their responsibilities and obligations as promised (lack of due competence). An
advantage to using the ground of lack of due competence is that at the time the marriage was entered
into civil divorce and breakup of the family almost always is proof of someone's failure to carry out
marital responsibilities as promised at the time the marriage was entered into. [58]

Hernandez v. Court of Appeals [59] emphasizes the importance of presenting expert testimony to establish the precise
cause of a partys psychological incapacity, and to show that it existed at the inception of the marriage. And as Marcos v.
Marcos[60]asserts, there is no requirement that the person to be declared psychologically incapacitated be personally examined
by a physician, if the totality of evidence presented is enough to sustain a finding of psychological incapacity. [61] Verily, the
evidence must show a link, medical or the like, between the acts that manifest psychological incapacity and the psychological
disorder itself.
This is not to mention, but we mention nevertheless for emphasis, that the presentation of expert proof presupposes a
thorough and in-depth assessment of the parties by the psychologist or expert, for a conclusive diagnosis of a grave, severe
and incurable presence of psychological incapacity.[62] Parenthetically, the Court, at this point, finds it fitting to suggest the
inclusion in the Rule on Declaration of Absolute Nullity of Void Marriages and Annulment of Voidable Marriages,[63] an option for
the trial judge to refer the case to a court-appointed psychologist/expert for an independent assessment and evaluation of the
psychological state of the parties. This will assist the courts, who are no experts in the field of psychology, to arrive at an
intelligent and judicious determination of the case. The rule, however, does not dispense with the parties prerogative to present
their own expert witnesses.
Going back, in the case at bench, the psychological assessment, which we consider as adequate, produced the findings
that both parties are afflicted with personality disordersto repeat, dependent personality disorder for petitioner, and narcissistic
and antisocial personality disorder for respondent. We note that The Encyclopedia of Mental Health discusses personality
disorders as follows
A group of disorders involving behaviors or traits that are characteristic of a persons recent and long-term
functioning. Patterns of perceiving and thinking are not usually limited to isolated episodes but are deeply ingrained,
inflexible, maladaptive and severe enough to cause the individual mental stress or anxieties or to interfere with
interpersonal relationships and normal functioning. Personality disorders are often recognizable by adolescence or
earlier, continue through adulthood and become less obvious in middle or old age. An individual may have more
than one personality disorder at a time.
The common factor among individuals who have personality disorders, despite a variety of character traits, is
the way in which the disorder leads to pervasive problems in social and occupational adjustment. Some individuals

with personality disorders are perceived by others as overdramatic, paranoid, obnoxious or even criminal, without an
awareness of their behaviors. Such qualities may lead to trouble getting along with other people, as well as
difficulties in other areas of life and often a tendency to blame others for their problems. Other individuals with
personality disorders are not unpleasant or difficult to work with but tend to be lonely, isolated or dependent. Such
traits can lead to interpersonal difficulties, reduced self-esteem and dissatisfaction with life.
Causes of Personality Disorders Different mental health viewpoints propose a variety of causes of
personality disorders. These include Freudian, genetic factors, neurobiologic theories and brain wave activity.
Freudian Sigmund Freud believed that fixation at certain stages of development led to certain personality
types. Thus, some disorders as described in the Diagnostic and Statistical Manual of Mental Disorders (3d ed., rev.)
are derived from his oral, anal and phallic character types. Demanding and dependent behavior (dependent and
passive-aggressive) was thought to derive from fixation at the oral stage. Characteristics of obsessionality, rigidity
and emotional aloofness were thought to derive from fixation at the anal stage; fixation at the phallic stage was
thought to lead to shallowness and an inability to engage in intimate relationships. However, later researchers have
found little evidence that early childhood events or fixation at certain stages of development lead to specific
personality patterns.
Genetic Factors Researchers have found that there may be a genetic factor involved in the etiology of
antisocial and borderline personality disorders; there is less evidence of inheritance of other personality
disorders. Some family, adoption and twin studies suggest that schizotypal personality may be related to genetic
factors.
Neurobiologic Theories In individuals who have borderline personality, researchers have found that low
cerebrospinal fluid 5-hydroxyindoleacetic acid (5-HIAA) negatively correlated with measures of aggression and a
past history of suicide attempts. Schizotypal personality has been associated with low platelet monoamine oxidase
(MAO) activity and impaired smooth pursuit eye movement.
Brain Wave Activity Abnormalities in electroencephalograph (EEG) have been reported in antisocial
personality for many years; slow wave is the most widely reported abnormality. A study of borderline patients
reported that 38 percent had at least marginal EEG abnormalities, compared with 19 percent in a control group.
Types of Disorders According to the American Psychiatric Associations Diagnostic and Statistical Manual
of Mental Disorders (3d ed., rev., 1987), or DSM-III-R, personality disorders are categorized into three major
clusters:
Cluster A: Paranoid, schizoid and schizotypal personality disorders. Individuals who have these disorders
often appear to have odd or eccentric habits and traits.
Cluster B: Antisocial, borderline, histrionic and narcissistic personality disorders. Individuals who have
these disorders often appear overly emotional, erratic and dramatic.
Cluster
C: Avoidant,
dependent,
obsessive-compulsive
and
disorders. Individuals who have these disorders often appear anxious or fearful.

passive-aggressive

personality

The DSM-III-R also lists another category, personality disorder not otherwise specified, that can be used for
other specific personality disorders or for mixed conditions that do not qualify as any of the specific personality
disorders.
Individuals with diagnosable personality disorders usually have long-term concerns, and thus therapy may be
long-term.[64]
Dependent personality disorder is characterized in the following manner
A personality disorder characterized by a pattern of dependent and submissive behavior. Such individuals usually
lack self-esteem and frequently belittle their capabilities; they fear criticism and are easily hurt by others
comments. At times they actually bring about dominance by others through a quest for overprotection.
Dependent personality disorder usually begins in early adulthood. Individuals who have this disorder may be
unable to make everyday decisions without advice or reassurance from others, may allow others to make most of
their important decisions (such as where to live), tend to agree with people even when they believe they are wrong,
have difficulty starting projects or doing things on their own, volunteer to do things that are demeaning in order to get
approval from other people, feel uncomfortable or helpless when alone and are often preoccupied with fears of
being abandoned.[65]
and antisocial personality disorder described, as follows
Characteristics include a consistent pattern of behavior that is intolerant of the conventional behavioral limitations
imposed by a society, an inability to sustain a job over a period of years, disregard for the rights of others (either
through exploitiveness or criminal behavior), frequent physical fights and, quite commonly, child or spouse abuse
without remorse and a tendency to blame others. There is often a faade of charm and even sophistication that
masks disregard, lack of remorse for mistreatment of others and the need to control others.
Although characteristics of this disorder describe criminals, they also may befit some individuals who are
prominent in business or politics whose habits of self-centeredness and disregard for the rights of others may be
hidden prior to a public scandal.
During the 19th century, this type of personality disorder was referred to as moral insanity. The term
described immoral, guiltless behavior that was not accompanied by impairments in reasoning.
According to the classification system used in the Diagnostic and Statistical Manual of Mental Disorders (3d
ed., rev. 1987), anti-social personality disorder is one of the four dramatic personality disorders, the others being
borderline, histrionic and narcissistic.[66]

The seriousness of the diagnosis and the gravity of the disorders considered, the Court, in this case, finds as decisive the
psychological evaluation made by the expert witness; and, thus, rules that the marriage of the parties is null and void on ground

of both parties psychological incapacity. We further consider that the trial court, which had a first-hand view of the witnesses
deportment, arrived at the same conclusion.
Indeed, petitioner, who is afflicted with dependent personality disorder, cannot assume the essential marital obligations of
living together, observing love, respect and fidelity and rendering help and support, for he is unable to make everyday decisions
without advice from others, allows others to make most of his important decisions (such as where to live), tends to agree with
people even when he believes they are wrong, has difficulty doing things on his own, volunteers to do things that are
demeaning in order to get approval from other people, feels uncomfortable or helpless when alone and is often preoccupied
with fears of being abandoned. [67] As clearly shown in this case, petitioner followed everything dictated to him by the persons
around him. He is insecure, weak and gullible, has no sense of his identity as a person, has no cohesive self to speak of, and
has no goals and clear direction in life.
Although on a different plane, the same may also be said of the respondent. Her being afflicted with antisocial personality
disorder makes her unable to assume the essential marital obligations. This finding takes into account her disregard for the
rights of others, her abuse, mistreatment and control of others without remorse, her tendency to blame others, and her
intolerance of the conventional behavioral limitations imposed by society. [68] Moreover, as shown in this case, respondent is
impulsive and domineering; she had no qualms in manipulating petitioner with her threats of blackmail and of committing
suicide.
Both parties being afflicted with grave, severe and incurable psychological incapacity, the precipitous marriage which they
contracted on April 23, 1996 is thus, declared null and void.
WHEREFORE, premises considered, the petition for review on certiorari is GRANTED. The August 5, 2003 Decision and
the January 19, 2004 Resolution of the Court of Appeals in CA-G.R. CV No. 71867 are REVERSED and SET ASIDE, and the
Decision, dated July 30, 2001, REINSTATED.

G.R. No. 150758

February 18, 2004

VERONICO TENEBRO, petitioner


vs.
THE HONORABLE COURT OF APPEALS, respondent.
DECISION
YNARES-SANTIAGO, J.:
We are called on to decide the novel issue concerning the effect of the judicial declaration of the nullity of a second or
subsequent marriage, on the ground of psychological incapacity, on an individuals criminal liability for bigamy. We hold that the
subsequent judicial declaration of nullity of marriage on the ground of psychological incapacity does not retroact to the date of
the celebration of the marriage insofar as the Philippines penal laws are concerned. As such, an individual who contracts a
second or subsequent marriage during the subsistence of a valid marriage is criminally liable for bigamy, notwithstanding the
subsequent declaration that the second marriage is void ab initio on the ground of psychological incapacity.
Petitioner in this case, Veronico Tenebro, contracted marriage with private complainant Leticia Ancajas on April 10, 1990. The
two were wed by Judge Alfredo B. Perez, Jr. of the City Trial Court of Lapu-lapu City. Tenebro and Ancajas lived together
continuously and without interruption until the latter part of 1991, when Tenebro informed Ancajas that he had been previously
married to a certain Hilda Villareyes on November 10, 1986. Tenebro showed Ancajas a photocopy of a marriage contract
between him and Villareyes. Invoking this previous marriage, petitioner thereafter left the conjugal dwelling which he shared
with Ancajas, stating that he was going to cohabit with Villareyes. 1
On January 25, 1993, petitioner contracted yet another marriage, this one with a certain Nilda Villegas, before Judge German
Lee, Jr. of the Regional Trial Court of Cebu City, Branch 15. 2 When Ancajas learned of this third marriage, she verified from
Villareyes whether the latter was indeed married to petitioner. In a handwritten letter, 3Villareyes confirmed that petitioner,
Veronico Tenebro, was indeed her husband.
Ancajas thereafter filed a complaint for bigamy against petitioner.4 The Information,5 which was docketed as Criminal Case No.
013095-L, reads:
That on the 10th day of April 1990, in the City of Lapu-lapu, Philippines, and within the jurisdiction of this Honorable Court, the
aforenamed accused, having been previously united in lawful marriage with Hilda Villareyes, and without the said marriage
having been legally dissolved, did then and there willfully, unlawfully and feloniously contract a second marriage with LETICIA
ANCAJAS, which second or subsequent marriage of the accused has all the essential requisites for validity were it not for the
subsisting first marriage.
CONTRARY TO LAW.
When arraigned, petitioner entered a plea of "not guilty". 6
During the trial, petitioner admitted having cohabited with Villareyes from 1984-1988, with whom he sired two children. However,
he denied that he and Villareyes were validly married to each other, claiming that no marriage ceremony took place to
solemnize their union.7 He alleged that he signed a marriage contract merely to enable her to get the allotment from his office in
connection with his work as a seaman. 8 He further testified that he requested his brother to verify from the Civil Register in
Manila whether there was any marriage at all between him and Villareyes, but there was no record of said marriage. 9
On November 10, 1997, the Regional Trial Court of Lapu-lapu City, Branch 54, rendered a decision finding the accused guilty
beyond reasonable doubt of the crime of bigamy under Article 349 of the Revised Penal Code, and sentencing him to four (4)
years and two (2) months of prision correccional, as minimum, to eight (8) years and one (1) day of prision mayor, as
maximum.10 On appeal, the Court of Appeals affirmed the decision of the trial court. Petitioners motion for reconsideration was
denied for lack of merit.
Hence, the instant petition for review on the following assignment of errors:
I. THE HONORABLE COURT OF APPEALS GRAVELY ERRED, AND THIS ERROR IS CORRECTIBLE IN THIS APPEAL
WHEN IT AFFIRMED THE DECISION OF THE HONORABLE COURT A QUO CONVICTING THE ACCUSED FOR (sic)
THE CRIME OF BIGAMY, DESPITE THE NON-EXISTENCE OF THE FIRST MARRIAGE AND INSUFFICIENCY OF
EVIDENCE.
II. THE COURT ERRED IN CONVICTING THE ACCUSED FOR (sic) THE CRIME OF BIGAMY DESPITE CLEAR PROOF
THAT THE MARRIAGE BETWEEN THE ACCUSED AND PRIVATE COMPLAINANT HAD BEEN DECLARED NULL AND
VOID AB INITIO AND WITHOUT LEGAL FORCE AND EFFECT.11
After a careful review of the evidence on record, we find no cogent reason to disturb the assailed judgment.
Under Article 349 of the Revised Penal Code, the elements of the crime of Bigamy are:
(1) that the offender has been legally married;
(2) that the first marriage has not been legally dissolved or, in case his or her spouse is absent, the absent spouse could
not yet be presumed dead according to the Civil Code;
(3) that he contracts a second or subsequent marriage; and
(4) that the second or subsequent marriage has all the essential requisites for validity.12
Petitioners assignment of errors presents a two-tiered defense, in which he (1) denies the existence of his first marriage to
Villareyes, and (2) argues that the declaration of the nullity of the second marriage on the ground of psychological incapacity,
which is an alleged indicator that his marriage to Ancajas lacks the essential requisites for validity, retroacts to the date on which
the second marriage was celebrated.13 Hence, petitioner argues that all four of the elements of the crime of bigamy are absent,
and prays for his acquittal.14
Petitioners defense must fail on both counts.
First, the prosecution presented sufficient evidence, both documentary and oral, to prove the existence of the first marriage
between petitioner and Villareyes. Documentary evidence presented was in the form of: (1) a copy of a marriage contract
between Tenebro and Villareyes, dated November 10, 1986, which, as seen on the document, was solemnized at the Manila
City Hall before Rev. Julieto Torres, a Minister of the Gospel, and certified to by the Office of the Civil Registrar of Manila; 15 and
(2) a handwritten letter from Villareyes to Ancajas dated July 12, 1994, informing Ancajas that Villareyes and Tenebro were
legally married.16
To assail the veracity of the marriage contract, petitioner presented (1) a certification issued by the National Statistics Office
dated October 7, 1995;17 and (2) a certification issued by the City Civil Registry of Manila, dated February 3, 1997. 18 Both these
documents attest that the respective issuing offices have no record of a marriage celebrated between Veronico B. Tenebro and
Hilda B. Villareyes on November 10, 1986.

To our mind, the documents presented by the defense cannot adequately assail the marriage contract, which in itself would
already have been sufficient to establish the existence of a marriage between Tenebro and Villareyes.
All three of these documents fall in the category of public documents, and the Rules of Court provisions relevant to public
documents are applicable to all. Pertinent to the marriage contract, Section 7 of Rule 130 of the Rules of Court reads as follows:
Sec. 7. Evidence admissible when original document is a public record. When the original of a document is in the custody of a
public officer or is recorded in a public office, its contents may be proved by a certified copy issued by the public officer in
custody thereof (Emphasis ours).
This being the case, the certified copy of the marriage contract, issued by a public officer in custody thereof, was admissible as
the best evidence of its contents. The marriage contract plainly indicates that a marriage was celebrated between petitioner and
Villareyes on November 10, 1986, and it should be accorded the full faith and credence given to public documents.
Moreover, an examination of the wordings of the certification issued by the National Statistics Office on October 7, 1995 and
that issued by the City Civil Registry of Manila on February 3, 1997 would plainly show that neither document attests as a
positive fact that there was no marriage celebrated between Veronico B. Tenebro and Hilda B. Villareyes on November 10,
1986. Rather, the documents merely attest that the respective issuing offices have no record of such a marriage. Documentary
evidence as to the absence of a record is quite different from documentary evidence as to the absence of a marriage ceremony,
or documentary evidence as to the invalidity of the marriage between Tenebro and Villareyes.
The marriage contract presented by the prosecution serves as positive evidence as to the existence of the marriage between
Tenebro and Villareyes, which should be given greater credence than documents testifying merely as to absence of any record
of the marriage, especially considering that there is absolutely no requirement in the law that a marriage contract needs to be
submitted to the civil registrar as a condition precedent for the validity of a marriage. The mere fact that no record of a marriage
exists does not invalidate the marriage, provided all requisites for its validity are present. 19 There is no evidence presented by
the defense that would indicate that the marriage between Tenebro and Villareyes lacked any requisite for validity, apart from
the self-serving testimony of the accused himself. Balanced against this testimony are Villareyes letter, Ancajas testimony that
petitioner informed her of the existence of the valid first marriage, and petitioners own conduct, which would all tend to indicate
that the first marriage had all the requisites for validity.
Finally, although the accused claims that he took steps to verify the non-existence of the first marriage to Villareyes by
requesting his brother to validate such purported non-existence, it is significant to note that the certifications issued by the
National Statistics Office and the City Civil Registry of Manila are dated October 7, 1995 and February 3, 1997, respectively.
Both documents, therefore, are dated after the accuseds marriage to his second wife, private respondent in this case.
As such, this Court rules that there was sufficient evidence presented by the prosecution to prove the first and second requisites
for the crime of bigamy.
The second tier of petitioners defense hinges on the effects of the subsequent judicial declaration 20 of the nullity of the second
marriage on the ground of psychological incapacity.
Petitioner argues that this subsequent judicial declaration retroacts to the date of the celebration of the marriage to Ancajas. As
such, he argues that, since his marriage to Ancajas was subsequently declared void ab initio, the crime of bigamy was not
committed.21
This argument is not impressed with merit.
Petitioner makes much of the judicial declaration of the nullity of the second marriage on the ground of psychological incapacity,
invoking Article 36 of the Family Code. What petitioner fails to realize is that a declaration of the nullity of the second marriage
on the ground of psychological incapacity is of absolutely no moment insofar as the States penal laws are concerned.
As a second or subsequent marriage contracted during the subsistence of petitioners valid marriage to Villareyes, petitioners
marriage to Ancajas would be null and void ab initio completely regardless of petitioners psychological capacity or
incapacity.22 Since a marriage contracted during the subsistence of a valid marriage is automatically void, the nullity of this
second marriage is not per se an argument for the avoidance of criminal liability for bigamy. Pertinently, Article 349 of the
Revised Penal Code criminalizes "any person who shall contract a second or subsequent marriage before the former marriage
has been legally dissolved, or before the absent spouse has been declared presumptively dead by means of a judgment
rendered in the proper proceedings". A plain reading of the law, therefore, would indicate that the provision penalizes the mere
act of contracting a second or a subsequent marriage during the subsistence of a valid marriage.
Thus, as soon as the second marriage to Ancajas was celebrated on April 10, 1990, during the subsistence of the valid first
marriage, the crime of bigamy had already been consummated. To our mind, there is no cogent reason for distinguishing
between a subsequent marriage that is null and void purely because it is a second or subsequent marriage, and a subsequent
marriage that is null and void on the ground of psychological incapacity, at least insofar as criminal liability for bigamy is
concerned. The States penal laws protecting the institution of marriage are in recognition of the sacrosanct character of this
special contract between spouses, and punish an individuals deliberate disregard of the permanent character of the special
bond between spouses, which petitioner has undoubtedly done.
Moreover, the declaration of the nullity of the second marriage on the ground of psychological incapacity is not an indicator that
petitioners marriage to Ancajas lacks the essential requisites for validity. The requisites for the validity of a marriage are
classified by the Family Code into essential (legal capacity of the contracting parties and their consent freely given in the
presence of the solemnizing officer)23 and formal (authority of the solemnizing officer, marriage license, and marriage ceremony
wherein the parties personally declare their agreement to marry before the solemnizing officer in the presence of at least two
witnesses).24 Under Article 5 of the Family Code, any male or female of the age of eighteen years or upwards not under any of
the impediments mentioned in Articles 3725 and 3826 may contract marriage.27
In this case, all the essential and formal requisites for the validity of marriage were satisfied by petitioner and Ancajas. Both
were over eighteen years of age, and they voluntarily contracted the second marriage with the required license before Judge
Alfredo B. Perez, Jr. of the City Trial Court of Lapu-lapu City, in the presence of at least two witnesses.
Although the judicial declaration of the nullity of a marriage on the ground of psychological incapacity retroacts to the date of the
celebration of the marriage insofar as the vinculum between the spouses is concerned, it is significant to note that said marriage
is not without legal effects. Among these effects is that children conceived or born before the judgment of absolute nullity of the
marriage shall be considered legitimate. 28 There is therefore a recognition written into the law itself that such a marriage,
although void ab initio, may still produce legal consequences. Among these legal consequences is incurring criminal liability for
bigamy. To hold otherwise would render the States penal laws on bigamy completely nugatory, and allow individuals to
deliberately ensure that each marital contract be flawed in some manner, and to thus escape the consequences of contracting
multiple marriages, while beguiling throngs of hapless women with the promise of futurity and commitment.
As such, we rule that the third and fourth requisites for the crime of bigamy are present in this case, and affirm the judgment of
the Court of Appeals.
As a final point, we note that based on the evidence on record, petitioner contracted marriage a third time, while his marriages
to Villareyes and Ancajas were both still subsisting. Although this is irrelevant in the determination of the accuseds guilt for
purposes of this particular case, the act of the accused displays a deliberate disregard for the sanctity of marriage, and the

State does not look kindly on such activities. Marriage is a special contract, the key characteristic of which is its permanence.
When an individual manifests a deliberate pattern of flouting the foundation of the States basic social institution, the States
criminal laws on bigamy step in.
Under Article 349 of the Revised Penal Code, as amended, the penalty for the crime of bigamy is prision mayor, which has a
duration of six (6) years and one (1) day to twelve (12) years. There being neither aggravating nor mitigating circumstance, the
same shall be imposed in its medium period. Applying the Indeterminate Sentence Law, petitioner shall be entitled to a minimum
term, to be taken from the penalty next lower in degree, i.e., prision correccional which has a duration of six (6) months and one
(1) day to six (6) years. Hence, the Court of Appeals correctly affirmed the decision of the trial court which sentenced petitioner
to suffer an indeterminate penalty of four (4) years and two (2) months of prision correccional, as minimum, to eight (8) years
and one (1) day of prision mayor, as maximum.
WHEREFORE, in view of all the foregoing, the instant petition for review is DENIED. The assailed decision of the Court of
Appeals in CA-G.R. CR No. 21636, convicting petitioner Veronico Tenebro of the crime of Bigamy and sentencing him to suffer
the indeterminate penalty of four (4) years and two (2) months of prision correccional, as minimum, to eight (8) years and one
(1) day of prision mayor, as maximum, is AFFIRMED in toto.
SO ORDERED.

G.R. No. 145226

February 06, 2004

LUCIO MORIGO y CACHO, petitioner,


vs.
PEOPLE OF THE PHILIPPINES, respondent.

DECISION

QUISUMBING, J.:
This petition for review on certiorari seeks to reverse the decision1 dated October 21, 1999 of the Court of Appeals in CA-G.R.
CR No. 20700, which affirmed the judgment 2 dated August 5, 1996 of the Regional Trial Court (RTC) of Bohol, Branch 4, in
Criminal Case No. 8688. The trial court found herein petitioner Lucio Morigo y Cacho guilty beyond reasonable doubt of bigamy
and sentenced him to a prison term of seven (7) months of prision correccional as minimum to six (6) years and one (1) day
of prision mayor as maximum. Also assailed in this petition is the resolution 3 of the appellate court, dated September 25, 2000,
denying Morigos motion for reconsideration.
The facts of this case, as found by the court a quo, are as follows:
Appellant Lucio Morigo and Lucia Barrete were boardmates at the house of Catalina Tortor at Tagbilaran City, Province of
Bohol, for a period of four (4) years (from 1974-1978).
After school year 1977-78, Lucio Morigo and Lucia Barrete lost contact with each other.
In 1984, Lucio Morigo was surprised to receive a card from Lucia Barrete from Singapore. The former replied and after an
exchange of letters, they became sweethearts.
In 1986, Lucia returned to the Philippines but left again for Canada to work there. While in Canada, they maintained
constant communication.
In 1990, Lucia came back to the Philippines and proposed to petition appellant to join her in Canada. Both agreed to get
married, thus they were married on August 30, 1990 at the Iglesia de Filipina Nacional at Catagdaan, Pilar, Bohol.
On September 8, 1990, Lucia reported back to her work in Canada leaving appellant Lucio behind.
On August 19, 1991, Lucia filed with the Ontario Court (General Division) a petition for divorce against appellant which
was granted by the court on January 17, 1992 and to take effect on February 17, 1992.
On October 4, 1992, appellant Lucio Morigo married Maria Jececha Lumbago 4 at the Virgen sa Barangay Parish,
Tagbilaran City, Bohol.
On September 21, 1993, accused filed a complaint for judicial declaration of nullity of marriage in the Regional Trial Court
of Bohol, docketed as Civil Case No. 6020. The complaint seek (sic) among others, the declaration of nullity of accuseds
marriage with Lucia, on the ground that no marriage ceremony actually took place.
On October 19, 1993, appellant was charged with Bigamy in an Information 5 filed by the City Prosecutor of Tagbilaran
[City], with the Regional Trial Court of Bohol. 6
The petitioner moved for suspension of the arraignment on the ground that the civil case for judicial nullification of his marriage
with Lucia posed a prejudicial question in the bigamy case. His motion was granted, but subsequently denied upon motion for
reconsideration by the prosecution. When arraigned in the bigamy case, which was docketed as Criminal Case No. 8688,
herein petitioner pleaded not guilty to the charge. Trial thereafter ensued.
On August 5, 1996, the RTC of Bohol handed down its judgment in Criminal Case No. 8688, as follows:
WHEREFORE, foregoing premises considered, the Court finds accused Lucio Morigo y Cacho guilty beyond reasonable
doubt of the crime of Bigamy and sentences him to suffer the penalty of imprisonment ranging from Seven (7) Months
of Prision Correccional as minimum to Six (6) Years and One (1) Day of Prision Mayor as maximum.
SO ORDERED.7
In convicting herein petitioner, the trial court discounted petitioners claim that his first marriage to Lucia was null and
void ab initio. Following Domingo v. Court of Appeals,8 the trial court ruled that want of a valid marriage ceremony is not a
defense in a charge of bigamy. The parties to a marriage should not be allowed to assume that their marriage is void even if
such be the fact but must first secure a judicial declaration of the nullity of their marriage before they can be allowed to marry
again.
Anent the Canadian divorce obtained by Lucia, the trial court cited Ramirez v. Gmur,9 which held that the court of a country in
which neither of the spouses is domiciled and in which one or both spouses may resort merely for the purpose of obtaining a
divorce, has no jurisdiction to determine the matrimonial status of the parties. As such, a divorce granted by said court is not
entitled to recognition anywhere. Debunking Lucios defense of good faith in contracting the second marriage, the trial court
stressed that following People v. Bitdu,10 everyone is presumed to know the law, and the fact that one does not know that his act
constitutes a violation of the law does not exempt him from the consequences thereof.
Seasonably, petitioner filed an appeal with the Court of Appeals, docketed as CA-G.R. CR No. 20700.
Meanwhile, on October 23, 1997, or while CA-G.R. CR No. 20700 was pending before the appellate court, the trial court
rendered a decision in Civil Case No. 6020 declaring the marriage between Lucio and Lucia void ab initiosince no marriage
ceremony actually took place. No appeal was taken from this decision, which then became final and executory.
On October 21, 1999, the appellate court decided CA-G.R. CR No. 20700 as follows:
WHEREFORE, finding no error in the appealed decision, the same is hereby AFFIRMED in toto.
SO ORDERED.11
In affirming the assailed judgment of conviction, the appellate court stressed that the subsequent declaration of nullity of Lucios
marriage to Lucia in Civil Case No. 6020 could not acquit Lucio. The reason is that what is sought to be punished by Article
34912 of the Revised Penal Code is the act of contracting a second marriage before the first marriage had been dissolved.
Hence, the CA held, the fact that the first marriage was void from the beginning is not a valid defense in a bigamy case.

The Court of Appeals also pointed out that the divorce decree obtained by Lucia from the Canadian court could not be accorded
validity in the Philippines, pursuant to Article 15 13 of the Civil Code and given the fact that it is contrary to public policy in this
jurisdiction. Under Article 1714 of the Civil Code, a declaration of public policy cannot be rendered ineffectual by a judgment
promulgated in a foreign jurisdiction.
Petitioner moved for reconsideration of the appellate courts decision, contending that the doctrine in Mendiola v.
People,15 allows mistake upon a difficult question of law (such as the effect of a foreign divorce decree) to be a basis for good
faith.
On September 25, 2000, the appellate court denied the motion for lack of merit. 16 However, the denial was by a split vote.
The ponente of the appellate courts original decision in CA-G.R. CR No. 20700, Justice Eugenio S. Labitoria, joined in the
opinion prepared by Justice Bernardo P. Abesamis. The dissent observed that as the first marriage was validly declared
void ab initio, then there was no first marriage to speak of. Since the date of the nullity retroacts to the date of the first marriage
and since herein petitioner was, in the eyes of the law, never married, he cannot be convicted beyond reasonable doubt of
bigamy.
The present petition raises the following issues for our resolution:
A.
WHETHER OR NOT THE COURT OF APPEALS ERRED IN FAILING TO APPLY THE RULE THAT IN CRIMES
PENALIZED UNDER THE REVISED PENAL CODE, CRIMINAL INTENT IS AN INDISPENSABLE REQUISITE.
COROLLARILY, WHETHER OR NOT THE COURT OF APPEALS ERRED IN FAILING TO APPRECIATE [THE]
PETITIONERS LACK OF CRIMINAL INTENT WHEN HE CONTRACTED THE SECOND MARRIAGE.
B.
WHETHER OR NOT THE COURT OF APPEALS ERRED IN HOLDING THAT THE RULING IN PEOPLE VS. BITDU (58
PHIL. 817) IS APPLICABLE TO THE CASE AT BAR.
C.
WHETHER OR NOT THE COURT OF APPEALS ERRED IN FAILING TO APPLY THE RULE THAT EACH AND EVERY
CIRCUMSTANCE FAVORING THE INNOCENCE OF THE ACCUSED MUST BE TAKEN INTO ACCOUNT.17
To our mind, the primordial issue should be whether or not petitioner committed bigamy and if so, whether his defense of good
faith is valid.
The petitioner submits that he should not be faulted for relying in good faith upon the divorce decree of the Ontario court. He
highlights the fact that he contracted the second marriage openly and publicly, which a person intent upon bigamy would not be
doing. The petitioner further argues that his lack of criminal intent is material to a conviction or acquittal in the instant case. The
crime of bigamy, just like other felonies punished under the Revised Penal Code, is mala in se, and hence, good faith and lack
of criminal intent are allowed as a complete defense. He stresses that there is a difference between the intent to commit the
crime and the intent to perpetrate the act. Hence, it does not necessarily follow that his intention to contract a second marriage
is tantamount to an intent to commit bigamy.
For the respondent, the Office of the Solicitor General (OSG) submits that good faith in the instant case is a convenient but
flimsy excuse. The Solicitor General relies upon our ruling in Marbella-Bobis v. Bobis,18 which held that bigamy can be
successfully prosecuted provided all the elements concur, stressing that under Article 40 19 of the Family Code, a judicial
declaration of nullity is a must before a party may re-marry. Whether or not the petitioner was aware of said Article 40 is of no
account as everyone is presumed to know the law. The OSG counters that petitioners contention that he was in good faith
because he relied on the divorce decree of the Ontario court is negated by his act of filing Civil Case No. 6020, seeking a
judicial declaration of nullity of his marriage to Lucia.
Before we delve into petitioners defense of good faith and lack of criminal intent, we must first determine whether all the
elements of bigamy are present in this case. In Marbella-Bobis v. Bobis,20 we laid down the elements of bigamy thus:
(1) the offender has been legally married;
(2) the first marriage has not been legally dissolved, or in case his or her spouse is absent, the absent spouse has not
been judicially declared presumptively dead;
(3) he contracts a subsequent marriage; and
(4) the subsequent marriage would have been valid had it not been for the existence of the first.
Applying the foregoing test to the instant case, we note that during the pendency of CA-G.R. CR No. 20700, the RTC of Bohol
Branch 1, handed down the following decision in Civil Case No. 6020, to wit:
WHEREFORE, premises considered, judgment is hereby rendered decreeing the annulment of the marriage entered into
by petitioner Lucio Morigo and Lucia Barrete on August 23, 1990 in Pilar, Bohol and further directing the Local Civil
Registrar of Pilar, Bohol to effect the cancellation of the marriage contract.
SO ORDERED.21
The trial court found that there was no actual marriage ceremony performed between Lucio and Lucia by a solemnizing officer.
Instead, what transpired was a mere signing of the marriage contract by the two, without the presence of a solemnizing officer.
The trial court thus held that the marriage is void ab initio, in accordance with Articles 322 and 423 of the Family Code. As the
dissenting opinion in CA-G.R. CR No. 20700, correctly puts it, "This simply means that there was no marriage to begin with; and
that such declaration of nullity retroacts to the date of the first marriage. In other words, for all intents and purposes, reckoned
from the date of the declaration of the first marriage as void ab initio to the date of the celebration of the first marriage, the
accused was, under the eyes of the law, never married." 24 The records show that no appeal was taken from the decision of the
trial court in Civil Case No. 6020, hence, the decision had long become final and executory.
The first element of bigamy as a crime requires that the accused must have been legally married. But in this case, legally
speaking, the petitioner was never married to Lucia Barrete. Thus, there is no first marriage to speak of. Under the principle of
retroactivity of a marriage being declared void ab initio, the two were never married "from the beginning." The contract of
marriage is null; it bears no legal effect. Taking this argument to its logical conclusion, for legal purposes, petitioner was not
married to Lucia at the time he contracted the marriage with Maria Jececha. The existence and the validity of the first marriage
being an essential element of the crime of bigamy, it is but logical that a conviction for said offense cannot be sustained where
there is no first marriage to speak of. The petitioner, must, perforce be acquitted of the instant charge.
The present case is analogous to, but must be distinguished from Mercado v. Tan.25 In the latter case, the judicial declaration of
nullity of the first marriage was likewise obtained after the second marriage was already celebrated. We held therein that:

A judicial declaration of nullity of a previous marriage is necessary before a subsequent one can be legally contracted.
One who enters into a subsequent marriage without first obtaining such judicial declaration is guilty of bigamy. This
principle applies even if the earlier union is characterized by statutes as "void." 26
It bears stressing though that in Mercado, the first marriage was actually solemnized not just once, but twice: first before a judge
where a marriage certificate was duly issued and then again six months later before a priest in religious rites. Ostensibly, at
least, the first marriage appeared to have transpired, although later declared void ab initio.
In the instant case, however, no marriage ceremony at all was performed by a duly authorized solemnizing officer. Petitioner
and Lucia Barrete merely signed a marriage contract on their own. The mere private act of signing a marriage contract bears no
semblance to a valid marriage and thus, needs no judicial declaration of nullity. Such act alone, without more, cannot be
deemed to constitute an ostensibly valid marriage for which petitioner might be held liable for bigamy unless he first secures a
judicial declaration of nullity before he contracts a subsequent marriage.
The law abhors an injustice and the Court is mandated to liberally construe a penal statute in favor of an accused and weigh
every circumstance in favor of the presumption of innocence to ensure that justice is done. Under the circumstances of the
present case, we held that petitioner has not committed bigamy. Further, we also find that we need not tarry on the issue of the
validity of his defense of good faith or lack of criminal intent, which is now moot and academic.
WHEREFORE, the instant petition is GRANTED. The assailed decision, dated October 21, 1999 of the Court of Appeals in CAG.R. CR No. 20700, as well as the resolution of the appellate court dated September 25, 2000, denying herein petitioners
motion for reconsideration, is REVERSED and SET ASIDE. The petitioner Lucio Morigo y Cacho is ACQUITTED from the
charge of BIGAMY on the ground that his guilt has not been proven with moral certainty.
SO ORDERED.

G.R. No. L-53703 August 19, 1986


LILIA OLIVA WIEGEL, petitioner,
vs.
THE HONORABLE ALICIA V. SEMPIO-DIY (as presiding judge of the Juvenile and Domestic Relations Court of
Caloocan City) and KARL HEINZ WIEGEL, respondents.
Dapucanta, Dulay & Associates for petitioner.
Siguion Reyna, Montecillo and Ongsiako Law Office for private respondent.

PARAS, J.:
In an action (Family Case No. 483) filed before the erstwhile Juvenile and Domestic Relations Court of Caloocan City, herein
respondent Karl Heinz Wiegel (plaintiff therein) asked for the declaration of Nullity of his marriage (celebrated on July, 1978 at
the Holy Catholic Apostolic Christian Church Branch in Makati, Metro Manila) with herein petitioner Lilia Oliva Wiegel (Lilia, for
short, and defendant therein) on the ground of Lilia's previous existing marriage to one Eduardo A. Maxion, the ceremony
having been performed on June 25, 1972 at our Lady of Lourdes Church in Quezon City. Lilia, while admitting the existence of
said prior subsisting marriage claimed that said marriage was null and void, she and the first husband Eduardo A. Maxion
having been allegedly forced to enter said marital union. In the pre-trial that ensued, the issue agreed upon by both parties was
the status of the first marriage (assuming the presence of force exerted against both parties): was said prior marriage void or
was it merely voidable? Contesting the validity of the pre-trial order, Lilia asked the respondent court for an opportunity to
present evidence(1) that the first marriage was vitiated by force exercised upon both her and the first husband; and
(2) that the first husband was at the time of the marriage in 1972 already married to someone else.
Respondent judge ruled against the presentation of evidence because the existence of force exerted on both parties of the first
marriage had already been agreed upon. Hence, the present petition for certiorari assailing the following Orders of
therespondent Judge(1) the Order dated March 17, 1980 in which the parties were compelled to submit the case for resolution based on "agreed
facts;" and
(2) the Order dated April 14, 1980, denying petitioner's motion to allow her to present evidence in her favor.
We find the petition devoid of merit.
There is no need for petitioner to prove that her first marriage was vitiated by force committed against both parties because
assuming this to be so, the marriage will not be void but merely viodable (Art. 85, Civil Code), and therefore valid until annulled.
Since no annulment has yet been made, it is clear that when she married respondent she was still validly married to her first
husband, consequently, her marriage to respondent is VOID (Art. 80, Civil Code).
There is likewise no need of introducing evidence about the existing prior marriage of her first husband at the time they married
each other, for then such a marriage though void still needs according to this Court a judicial declaration 1 of such fact and for all
legal intents and purposes she would still be regarded as a married woman at the time she contracted her marriage with
respondent Karl Heinz Wiegel); accordingly, the marriage of petitioner and respondent would be regarded VOID under the law.
WHEREFORE, this petition is hereby DISMISSED, for lack of merit, and the Orders complained of are hereby AFFIRMED.
Costs against petitioner.
SO ORDERED.

G.R. No. 122749 July 31, 1996


ANTONIO A. S. VALDEZ, petitioner,
vs.
REGIONAL TRIAL COURT, BRANCH 102, QUEZON CITY, and CONSUELO M. GOMEZ-VALDEZ, respondents.

VITUG, J.:p
The petition for new bewails, purely on the question of law, an alleged error committed by the Regional Trial Court in Civil Case
No. Q-92-12539. Petitioner avers that the court a quo has failed to apply the correct law that should govern the disposition of a
family dwelling in a situation where a marriage is declared void ab initio because of psychological incapacity on the part of either
or both parties in the contract.
The pertinent facts giving rise to this incident are, by large, not in dispute.
Antonio Valdez and Consuelo Gomez were married on 05 January 1971. Begotten during the marriage were five children. In a
petition, dated 22 June 1992, Valdez sought the declaration of nullity of the marriage pursuant to Article 36 of the Family code
(docketed Civil Case No. Q-92-12539, Regional Trial Court of Quezon City, Branch 102). After the hearing the parties following
the joinder of issues, the trial court, 1 in its decision of 29 July 1994, granted the petition, viz:
WHEREFORE, judgment is hereby rendered as follows:
(1) The marriage of petitioner Antonio Valdez and respondent Consuelo Gomez-Valdez is hereby declared null and void
under Article 36 of the Family Code on the ground of their mutual psychological incapacity to comply with their essential
marital obligations;
(2) The three older children, Carlos Enrique III, Antonio Quintin and Angela Rosario shall choose which parent they would
want to stay with.
Stella Eloisa and Joaquin Pedro shall be placed in the custody of their mother, herein respondent Consuelo GomezValdes.
The petitioner and respondent shall have visitation rights over the children who are in the custody of the other.
(3) The petitioner and the respondent are directed to start proceedings on the liquidation of their common properties as
defined by Article 147 of the Family Code, and to comply with the provisions of Articles 50, 51, and 52 of the same code,
within thirty (30) days from notice of this decision.
Let a copy of this decision be furnished the Local Civil Registrar of Mandaluyong, Metro Manila, for proper recording in the
registry of marriages. 2 (Emphasis ours.)
Consuelo Gomez sought a clarification of that portion of the decision directing compliance with Articles 50, 51 and 52 of the
Family Code. She asserted that the Family Code contained no provisions on the procedure for the liquidation of common
property in "unions without marriage." Parenthetically, during the hearing of the motion, the children filed a joint affidavit
expressing their desire to remain with their father, Antonio Valdez, herein petitioner.
In an order, dated 05 May 1995, the trial court made the following clarification:
Consequently, considering that Article 147 of the Family Code explicitly provides that the property acquired by both parties
during their union, in the absence of proof to the contrary, are presumed to have been obtained through the joint efforts of
the parties and will be owned by them in equal shares, plaintiff and defendant will own their "family home" and all their
properties for that matter in equal shares.
In the liquidation and partition of properties owned in common by the plaintiff and defendant, the provisions on ownership
found in the Civil Code shall apply. 3 (Emphasis supplied.)
In addressing specifically the issue regarding the disposition of the family dwelling, the trial court said:
Considering that this Court has already declared the marriage between petitioner and respondent as null and void ab
initio, pursuant to Art. 147, the property regime of petitioner and respondent shall be governed by the rules on ownership.
The provisions of Articles 102 and 129 of the Family Code finds no application since Article 102 refers to the procedure for
the liquidation of the conjugal partnership property and Article 129 refers to the procedure for the liquidation of
the absolute community of property. 4
Petitioner moved for a reconsideration of the order. The motion was denied on 30 October 1995.
In his recourse to this Court, petitioner submits that Articles 50, 51 and 52 of the Family Code should be held controlling: he
argues that:
I
Article 147 of the Family Code does not apply to cases where the parties are psychologically incapacitated.
II
Articles 50, 51 and 52 in relation to Articles 102 and 129 of the Family Code govern the disposition of the family dwelling in
cases where a marriage is declared void ab initio, including a marriage declared void by reason of the psychological
incapacity of the spouses.
III
Assuming arguendo that Article 147 applies to marriages declared void ab initio on the ground of the psychological
incapacity of a spouse, the same may be read consistently with Article 129.
IV
It is necessary to determine the parent with whom majority of the children wish to stay. 5
The trial court correctly applied the law. In a void marriage, regardless of the cause thereof, the property relations of the parties
during the period of cohabitation is governed by the provisions of Article 147 or Article 148, such as the case may be, of the
Family Code. Article 147 is a remake of Article 144 of the Civil Code as interpreted and so applied in previous cases; 6 it
provides:
Art. 147. When a man and a woman who are capacitated to marry each other, live exclusively with each other as husband
and wife without the benefit of marriage or under a void marriage, their wages and salaries shall be owned by them in

equal shares and the property acquired by both of them through their work or industry shall be governed by the rules on
co-ownership.
In the absence of proof to the contrary, properties acquired while they lived together shall be presumed to have been
obtained by their joint efforts, work or industry, and shall be owned by them in equal shares. For purposes of this Article, a
party who did not participate in the acquisition by the other party of any property shall be deemed to have contributed
jointly in the acquisition thereof in the former's efforts consisted in the care and maintenance of the family and of the
household.
Neither party can encumber or dispose by acts inter vivos of his or her share in the property acquired during cohabitation
and owned in common, without the consent of the other, until after the termination of their cohabitation.
When only one of the parties to a void marriage is in good faith, the share of the party in bad faith in the ownership shall
be forfeited in favor of their common children. In case of default of or waiver by any or all of the common children or their
descendants, each vacant share shall belong to the innocent party. In all cases, the forfeiture shall take place upon the
termination of the cohabitation.
This particular kind of co-ownership applies when a man and a woman, suffering no illegal impediment to marry each other, so
exclusively live together as husband and wife under a void marriage or without the benefit of marriage. The term "capacitated" in
the provision (in the first paragraph of the law) refers to the legal capacity of a party to contract marriage, i.e., any "male or
female of the age of eighteen years or upwards not under any of the impediments mentioned in Articles 37 and 38" 7 of the
Code.
Under this property regime, property acquired by both spouses through their work and industry shall be governed by the rules
on equal co-ownership. Any property acquired during the union is prima facie presumed to have been obtained through their
joint efforts. A party who did not participate in the acquisition of the property shall be considered as having contributed thereto
jointly if said party's "efforts consisted in the care and maintenance of the family household." 8 Unlike the conjugal partnership of
gains, the fruits of the couple's separate property are not included in the co-ownership.
Article 147 of the Family Code, in the substance and to the above extent, has clarified Article 144 of the Civil Code; in addition,
the law now expressly provides that
(a) Neither party can dispose or encumber by act intervivos his or her share in co-ownership property, without consent of the
other, during the period of cohabitation; and
(b) In the case of a void marriage, any party in bad faith shall forfeit his or her share in the co-ownership in favor of their
common children; in default thereof or waiver by any or all of the common children, each vacant share shall belong to the
respective surviving descendants, or still in default thereof, to the innocent party. The forfeiture shall take place upon the
termination of the cohabitation 9 or declaration of nullity of the marriage. 10
When the common-law spouses suffer from a legal impediment to marry or when they do not live exclusively with each other (as
husband and wife), only the property acquired by both of them through their actual joint contribution of money, property or
industry shall be owned in common and in proportion to their respective contributions. Such contributions and corresponding
shares, however, are prima facie presumed to be equal. The share of any party who is married to another shall accrue to the
absolute community or conjugal partnership, as the case may be, if so existing under a valid marriage. If the party who has
acted in bad faith is not validly married to another, his or her share shall be forfeited in the manner already heretofore
expressed. 11
In deciding to take further cognizance of the issue on the settlement of the parties' common property, the trial court acted neither
imprudently nor precipitately; a court which has jurisdiction to declare the marriage a nullity must be deemed likewise clothed in
authority to resolve incidental and consequential matters. Nor did it commit a reversible error in ruling that petitioner and private
respondent own the "family home" and all their common property in equal shares, as well as in concluding that, in the liquidation
and partition of the property owned in common by them, the provisions on co-ownership under the Civil Code, not Articles 50,
51 and 52, in relation to Articles 102 and 129, 12 of the Family Code, should aptly prevail. The rules set up to govern the
liquidation of either the absolute community or the conjugal partnership of gains, the property regimes recognized for valid and
voidable marriages (in the latter case until the contract is annulled), are irrelevant to the liquidation of the co-ownership that
exists between common-law spouses. The first paragraph of Articles 50 of the Family Code, applying paragraphs (2), (3), (4)
and 95) of Article 43, 13 relates only, by its explicit terms, to voidable marriages and, exceptionally, to void marriages under
Article 4014 of the Code, i.e., the declaration of nullity of a subsequent marriage contracted by a spouse of a prior void marriage
before the latter is judicially declared void. The latter is a special rule that somehow recognizes the philosophy and an old
doctrine that void marriages are inexistent from the very beginning and no judicial decree is necessary to establish their nullity.
In now requiring for purposes of remarriage, the declaration of nullity by final judgment of the previously contracted void
marriage, the present law aims to do away with any continuing uncertainty on the status of the second marriage. It is not then
illogical for the provisions of Article 43, in relation to Articles 41 15 and 42, 16 of the Family Code, on the effects of the termination
of a subsequent marriage contracted during the subsistence of a previous marriage to be made applicable pro hac vice. In all
other cases, it is not to be assumed that the law has also meant to have coincident property relations, on the one hand,
between spouses in valid and voidable marriages (before annulment) and, on the other, between common-law spouses or
spouses of void marriages, leaving to ordain, on the latter case, the ordinary rules on co-ownership subject to the provisions of
the Family Code on the "family home," i.e., the provisions found in Title V, Chapter 2, of the Family Code, remain in force and
effect regardless of the property regime of the spouses.
WHEREFORE, the questioned orders, dated 05 May 1995 and 30 October 1995, of the trial court are AFFIRMED. No costs.

G.R. No. L-10016

February 28, 1957

THE PEOPLE OF THE PHILIPPINES, plaintiff-appelle,


vs.
PROCESO S. ARAGON, defendant-appellant.
LABRADOR, J.:
Appeal from a judgment of the Court of First Instance of Cebu finding appellant guilty of bigamy. The facts are not disputed and,
as found by the trial court, are as follows:
On September 28, 1925, the accused, under the name of Proceso Rosima, contracted marriage with a certain Maria Gorrea in
the Philippine Independent Church in Cebu (Exhibits "1" and "1-A"). While his marriage with Maria Gorrea was subsisting, the
accused under the name of Proceso Aragon, contracted a canonical marriage with Maria Faicol on August 27, 1934, in the
Santa Teresita Church in Iloilo City.
The sponsors of the accused and Maria Faicol were Eulogio Giroy, who was then an employee of the Office of the Municipal
Treasurer of Iloilo, and a certain Emilio Tomesa, a clerk in the said office (Exhibit "A" and testimonies of Eulogio Giroy and
complainant Maria Faicol). After the said marriage, the accused and Maria Faicol established residence in Iloilo. As the accused
was then a traveling salesman, he commuted between Iloilo where he maintained Maria Faicol, and Cebu where he maintained
his first wife, Maria Gorrea died in Cebu City on August 5, 1939 (Exhibit "2"). After Maria Gorrea's death, and seeing that the
coast was dear in Cebu, the accused brought Maria Faicol to Cebu City in 1940, where she worked as a teacher-nurse.
It would seem that the accused and Maria Faicol did not live a happy marital life in Cebu, for it appears that in 1949 and 1950,
Maria Faicol suffered injuries to her eyes because of physical maltreatment in the hands of the accused. On January 22, 1953,
the accused sent Maria Faicol to Iloilo, allegedly for the purpose of undergoing treatment of her eyesight. During her absence,
the accused contracted a third marriage with a certain Jesusa C. Maglasang on October 3, 1953, in Sibonga, Cebu. (See
Exhibits "C", "D", "E" and "F")
The accused admitted having contracted marriage with Jesusa C. Maglasangin Sibonga, Cebu, on October 3, 1953, Although
the accused made an attempt to deny his previous marriage with Maria Faicol, the Court, however, believes that the attempt is
futile for the fact of the said second marriage was fully established not only by the certificate of the said marriage, but also by
the testimony of Maria Faicol and of Eulogio Giroy, one of the sponsors of the wedding, and the identification of the accused
made by Maria Faicol. (See Exhibits "A" and "B"; t.s.n. pp. 32-33, 40, 41, hearing of April 27, 1954).
The Court of First Instance of Cebu held that even in the absence of an express provision in Act No. 3613 authorizing the filing
of an action for judicial declaration of nullity of a marriage void ab initio, defendant could not legally contract marriage with
Jesusa C. Maglasang without the dissolution of his marriage to Maria Faicol, either by the death of the latter or by the judicial
declaration of the nullity of such marriage, at the instance of the latter. Authorities given for this ruling are 5 Viada, 5th edition,
651; 35 American Jurisprudence, Marriage, Sec. 46, p. 212; Bickford vs. Bickford, 74 N. H. 466, 69 A. 579.
Appellant in this Court relies on the case of People vs. Mendoza, (95 Phil., 845; 50 Off. Gaz., [10] 4767). In this case the
majority of this Court declared:
The statutory provision (section 29 of the Marriage Law or Act No. 3613) plainly makes a subsequent marriage contracted by
any person during the lifetime of his first spouse illegal and void from its performance, and no judicial decree is necessary to
establish its invalidity, as distinguished from mere annullable marriages. There is here no pretense that appellant's second
marriage with Olga Lema was contracted in the belief that the first spouse, Jovita de Asis, had been absent for seven
consecutive years or generally considered as dead, so as to render said marriage valid until declared null and void by a
subsequent court.
We are of the very weighty reasons by Justice Alex Reyes in the dissent in the case above-quoted But this weighty reasons
notwithstanding, the very fundamental principle of strict construction of penal laws in favor of the accused, which principle we
may not ignore, seems to justify our stand in the above-cited case of People vs. Mendoza. Our Revised Penal Code is of recent
enactment and had the rule enunciated in Spain and in America requiring judicial declaration of nullity of ab initio void marriages
been within the contemplation of the legislature, an express provision to that effect would or should have been inserted in the
law. In its absence, we are bound by said rule of strict interpretation already adverted to.
It is to be noted that the action was instituted upon complaint of the second wife, whose marriage with the appellant was not
renewed after the death of the first wife and before the third marriage was entered into. Hence, the last marriage was a valid
one and appellant's prosecution for contracting this marriage can not prosper.
For the foregoing considerations, the judgment appealed from is hereby reversed and the defendant-appellant acquitted, with
costs de oficio, without prejudice to his prosecution for having contracted the second bigamous marriage. So ordered.

G.R. No. 138509

July 31, 2000

IMELDA MARBELLA-BOBIS, petitioner,


vs.
ISAGANI D. BOBIS, respondent.
YNARES-SANTIAGO, J.:
On October 21, 1985, respondent contracted a first marriage with one Maria Dulce B. Javier. Without said marriage having been
annulled, nullified or terminated, the same respondent contracted a second marriage with petitioner Imelda Marbella-Bobis on
January 25, 1996 and allegedly a third marriage with a certain Julia Sally Hernandez. Based on petitioner's complaint-affidavit,
an information for bigamy was filed against respondent on February 25, 1998, which was docketed as Criminal Case No. Q9875611 of the Regional Trial Court, Branch 226, Quezon City. Sometime thereafter, respondent initiated a civil action for the
judicial declaration of absolute nullity of his first marriage on the ground that it was celebrated without a marriage license.
Respondent then filed a motion to suspend the proceedings in the criminal case for bigamy invoking the pending civil case for
nullity of the first marriage as a prejudicial question to the criminal case. The trial judge granted the motion to suspend the
criminal case in an Order dated December 29, 1998. 1 Petitioner filed a motion for reconsideration, but the same was denied.
Hence, this petition for review on certiorari. Petitioner argues that respondent should have first obtained a judicial declaration of
nullity of his first marriage before entering into the second marriage, inasmuch as the alleged prejudicial question justifying
suspension of the bigamy case is no longer a legal truism pursuant to Article 40 of the Family Code. 2
The issue to be resolved in this petition is whether the subsequent filing of a civil action for declaration of nullity of a previous
marriage constitutes a prejudicial question to a criminal case for bigamy.
A prejudicial question is one which arises in a case the resolution of which is a logical antecedent of the issue involved
therein.3It is a question based on a fact distinct and separate from the crime but so intimately connected with it that it determines
the guilt or innocence of the accused.4 It must appear not only that the civil case involves facts upon which the criminal action is
based, but also that the resolution of the issues raised in the civil action would necessarily be determinative of the criminal
case.5 Consequently, the defense must involve an issue similar or intimately related to the same issue raised in the criminal
action and its resolution determinative of whether or not the latter action may proceed. 6 Its two essential elements are:7
(a) the civil action involves an issue similar or intimately related to the issue raised in the criminal action; and
(b) the resolution of such issue determines whether or not the criminal action may proceed.
A prejudicial question does not conclusively resolve the guilt or innocence of the accused but simply tests the sufficiency of the
allegations in the information in order to sustain the further prosecution of the criminal case. A party who raises a prejudicial
question is deemed to have hypothetically admitted that all the essential elements of a crime have been adequately alleged in
the information, considering that the prosecution has not yet presented a single evidence on the indictment or may not yet have
rested its case. A challenge of the allegations in the information on the ground of prejudicial question is in effect a question on
the merits of the criminal charge through a non-criminal suit.
Article 40 of the Family Code, which was effective at the time of celebration of the second marriage, requires a prior judicial
declaration of nullity of a previous marriage before a party may remarry. The clear implication of this is that it is not for the
parties, particularly the accused, to determine the validity or invalidity of the marriage. 8Whether or not the first marriage was
void for lack of a license is a matter of defense because there is still no judicial declaration of its nullity at the time the second
marriage was contracted. It should be remembered that bigamy can successfully be prosecuted provided all its elements concur
two of which are a previous marriage and a subsequent marriage which would have been valid had it not been for the
existence at the material time of the first marriage. 9
In the case at bar, respondent's clear intent is to obtain a judicial declaration of nullity of his first marriage and thereafter to
invoke that very same judgment to prevent his prosecution for bigamy. He cannot have his cake and eat it too. Otherwise, all
that an adventurous bigamist has to do is to disregard Article 40 of the Family Code, contract a subsequent marriage and
escape a bigamy charge by simply claiming that the first marriage is void and that the subsequent marriage is equally void for
lack of a prior judicial declaration of nullity of the first. A party may even enter into a marriage aware of the absence of a
requisite - usually the marriage license - and thereafter contract a subsequent marriage without obtaining a declaration of nullity
of the first on the assumption that the first marriage is void. Such scenario would render nugatory the provisions on bigamy. As
succinctly held in Landicho v. Relova:10
(P)arties to a marriage should not be permitted to judge for themselves its nullity, only competent courts having such
authority. Prior to such declaration of nullity, the validity of the first marriage is beyond question. A party who contracts a
second marriage then assumes the risk of being prosecuted for bigamy.
Respondent alleges that the first marriage in the case before us was void for lack of a marriage license. Petitioner, on the other
hand, argues that her marriage to respondent was exempt from the requirement of a marriage license. More specifically,
petitioner claims that prior to their marriage, they had already attained the age of majority and had been living together as
husband and wife for at least five years. 11 The issue in this case is limited to the existence of a prejudicial question, and we are
not called upon to resolve the validity of the first marriage. Be that as it may, suffice it to state that the Civil Code, under which
the first marriage was celebrated, provides that "every intendment of law or fact leans toward the validity of marriage, the
indissolubility of the marriage bonds."12 [] Hence, parties should not be permitted to judge for themselves the nullity of their
marriage, for the same must be submitted to the determination of competent courts. Only when the nullity of the marriage is so
declared can it be held as void, and so long as there is no such declaration the presumption is that the marriage exists. 13 No
matter how obvious, manifest or patent the absence of an element is, the intervention of the courts must always be resorted to.
That is why Article 40 of the Family Code requires a "final judgment," which only the courts can render. Thus, as ruled
in Landicho v. Relova,14 he who contracts a second marriage before the judicial declaration of nullity of the first marriage
assumes the risk of being prosecuted for bigamy, and in such a case the criminal case may not be suspended on the ground of
the pendency of a civil case for declaration of nullity. In a recent case for concubinage, we held that the pendency of a civil case
for declaration of nullity of marriage is not a prejudicial question. 15 This ruling applies here by analogy since both crimes
presuppose the subsistence of a marriage.
Ignorance of the existence of Article 40 of the Family Code cannot even be successfully invoked as an excuse. 16The contracting
of a marriage knowing that the requirements of the law have not been complied with or that the marriage is in disregard of a
legal impediment is an act penalized by the Revised Penal Code. 17 The legality of a marriage is a matter of law and every
person is presumed to know the law. As respondent did not obtain the judicial declaration of nullity when he entered into the
second marriage, why should he be allowed to belatedly obtain that judicial declaration in order to delay his criminal prosecution
and subsequently defeat it by his own disobedience of the law? If he wants to raise the nullity of the previous marriage, he can
do it as a matter of defense when he presents his evidence during the trial proper in the criminal case.
The burden of proof to show the dissolution of the first marriage before the second marriage was contracted rests upon the
defense,18 but that is a matter that can be raised in the trial of the bigamy case. In the meantime, it should be stressed that not
every defense raised in the civil action may be used as a prejudicial question to obtain the suspension of the criminal action.
The lower court, therefore, erred in suspending the criminal case for bigamy. Moreover, when respondent was indicted for
bigamy, the fact that he entered into two marriage ceremonies appeared indubitable. It was only after he was sued by petitioner
for bigamy that he thought of seeking a judicial declaration of nullity of his first marriage. The obvious intent, therefore, is that

respondent merely resorted to the civil action as a potential prejudicial question for the purpose of frustrating or delaying his
criminal prosecution. As has been discussed above, this cannot be done.1awphi1
In the light of Article 40 of the Family Code, respondent, without first having obtained the judicial declaration of nullity of the first
marriage, can not be said to have validly entered into the second marriage. Per current jurisprudence, a marriage though void
still needs a judicial declaration of such fact before any party can marry again; otherwise the second marriage will also be
void.19 The reason is that, without a judicial declaration of its nullity, the first marriage is presumed to be subsisting. In the case
at bar, respondent was for all legal intents and purposes regarded as a married man at the time he contracted his second
marriage with petitioner.20 Against this legal backdrop, any decision in the civil action for nullity would not erase the fact that
respondent entered into a second marriage during the subsistence of a first marriage. Thus, a decision in the civil case is not
essential to the determination of the criminal charge. It is, therefore, not a prejudicial question. As stated above, respondent
cannot be permitted to use his own malfeasance to defeat the criminal action against him. 21
WHEREFORE, the petition is GRANTED. The order dated December 29, 1998 of the Regional Trial Court, Branch 226 of
Quezon City is REVERSED and SET ASIDE and the trial court is ordered to IMMEDIATELY proceed with Criminal Case No.
Q98-75611.

[G.R. No. 132529. February 2, 2001]

SUSAN NICDAO CARIO, petitioner, vs. SUSAN YEE CARIO, respondent.


DECISION
YNARES-SANTIAGO, J.:
The issue for resolution in the case at bar hinges on the validity of the two marriages contracted by the deceased SPO4
Santiago S. Cario, whose death benefits is now the subject of the controversy between the two Susans whom he married.
Before this Court is a petition for review on certiorari seeking to set aside the decision [1] of the Court of Appeals in CA-G.R.
CV No. 51263, which affirmedin toto the decision[2] of the Regional Trial Court of Quezon City, Branch 87, in Civil Case No. Q93-18632.
During the lifetime of the late SPO4 Santiago S. Cario, he contracted two marriages, the first was on June 20, 1969, with
petitioner Susan Nicdao Cario (hereafter referred to as Susan Nicdao), with whom he had two offsprings, namely, Sahlee and
Sandee Cario; and the second was on November 10, 1992, with respondent Susan Yee Cario (hereafter referred to as Susan
Yee), with whom he had no children in their almost ten year cohabitation starting way back in 1982.
In 1988, SPO4 Santiago S. Cario became ill and bedridden due to diabetes complicated by pulmonary tuberculosis. He
passed away on November 23, 1992, under the care of Susan Yee, who spent for his medical and burial expenses. Both
petitioner and respondent filed claims for monetary benefits and financial assistance pertaining to the deceased from various
government agencies. Petitioner Susan Nicdao was able to collect a total of P146,000.00 from MBAI, PCCUI, Commutation,
NAPOLCOM, [and] Pag-ibig,[3] while respondent Susan Yee received a total of P21,000.00 from GSIS Life, Burial (GSIS) and
burial (SSS).[4]
On December 14, 1993, respondent Susan Yee filed the instant case for collection of sum of money against petitioner Susan
Nicdao praying, inter alia, that petitioner be ordered to return to her at least one-half of the one hundred forty-six thousand
pesos (P146,000.00) collectively denominated as death benefits which she (petitioner) received from MBAI, PCCUI,
Commutation, NAPOLCOM, [and] Pag-ibig. Despite service of summons, petitioner failed to file her answer, prompting the trial
court to declare her in default.
Respondent Susan Yee admitted that her marriage to the deceased took place during the subsistence of, and without first
obtaining a judicial declaration of nullity of, the marriage between petitioner and the deceased. She, however, claimed that she
had no knowledge of the previous marriage and that she became aware of it only at the funeral of the deceased, where she met
petitioner who introduced herself as the wife of the deceased. To bolster her action for collection of sum of money, respondent
contended that the marriage of petitioner and the deceased is void ab initio because the same was solemnized without the
required marriage license. In support thereof, respondent presented: 1) the marriage certificate of the deceased and the
petitioner which bears no marriage license number; [5] and 2) a certification dated March 9, 1994, from the Local Civil Registrar of
San Juan, Metro Manila, which reads
This is to certify that this Office has no record of marriage license of the spouses SANTIAGO CARINO (sic) and SUSAN
NICDAO, who are married in this municipality on June 20, 1969. Hence, we cannot issue as requested a true copy or
transcription of Marriage License number from the records of this archives.
This certification is issued upon the request of Mrs. Susan Yee Cario for whatever legal purpose it may serve. [6]
On August 28, 1995, the trial court ruled in favor of respondent, Susan Yee, holding as follows:
WHEREFORE, the defendant is hereby ordered to pay the plaintiff the sum of P73,000.00, half of the amount which was paid to
her in the form of death benefits arising from the death of SPO4 Santiago S. Cario, plus attorneys fees in the amount
of P5,000.00, and costs of suit.
IT IS SO ORDERED.[7]
On appeal by petitioner to the Court of Appeals, the latter affirmed in toto the decision of the trial court. Hence, the instant
petition, contending that:
I.
THE HONORABLE COURT OF APPEALS GRAVELY ERRED IN AFFIRMING THE FINDINGS OF THE LOWER COURT
THAT VDA. DE CONSUEGRA VS. GSIS IS APPLICABLE TO THE CASE AT BAR.
II.
THE HONORABLE COURT OF APPEALS GRAVELY ERRED IN APPLYING EQUITY IN THE INSTANT CASE INSTEAD
OF THE CLEAR AND UNEQUIVOCAL MANDATE OF THE FAMILY CODE.
III.
THE HONORABLE COURT OF APPEALS GRAVELY ERRED IN NOT FINDING THE CASE OF VDA. DE CONSUEGRA VS
GSIS TO HAVE BEEN MODIFIED, AMENDED AND EVEN ABANDONED BY THE ENACTMENT OF THE FAMILY CODE. [8]
Under Article 40 of the Family Code, the absolute nullity of a previous marriage may be invoked for purposes of remarriage
on the basis solely of a final judgment declaring such previous marriage void. Meaning, where the absolute nullity of a previous
marriage is sought to be invoked for purposes of contracting a second marriage, the sole basis acceptable in law, for said
projected marriage to be free from legal infirmity, is a final judgment declaring the previous marriage void. [9] However, for
purposes other than remarriage, no judicial action is necessary to declare a marriage an absolute nullity. For other purposes,
such as but not limited to the determination of heirship, legitimacy or illegitimacy of a child, settlement of estate, dissolution of
property regime, or a criminal case for that matter, the court may pass upon the validity of marriage even after the death of the
parties thereto, and even in a suit not directly instituted to question the validity of said marriage, so long as it is essential to the
determination of the case.[10] In such instances, evidence must be adduced, testimonial or documentary, to prove the existence
of grounds rendering such a previous marriage an absolute nullity. These need not be limited solely to an earlier final judgment
of a court declaring such previous marriage void. [11]
It is clear therefore that the Court is clothed with sufficient authority to pass upon the validity of the two marriages in this
case, as the same is essential to the determination of who is rightfully entitled to the subject death benefits of the deceased.
Under the Civil Code, which was the law in force when the marriage of petitioner Susan Nicdao and the deceased was
solemnized in 1969, a valid marriage license is a requisite of marriage,[12] and the absence thereof, subject to certain
exceptions,[13] renders the marriage void ab initio.[14]
In the case at bar, there is no question that the marriage of petitioner and the deceased does not fall within the marriages
exempt from the license requirement. A marriage license, therefore, was indispensable to the validity of their marriage. This
notwithstanding, the records reveal that the marriage contract of petitioner and the deceased bears no marriage license number
and, as certified by the Local Civil Registrar of San Juan, Metro Manila, their office has no record of such marriage
license. In Republic v. Court of Appeals,[15] the Court held that such a certification is adequate to prove the non-issuance of a
marriage license. Absent any circumstance of suspicion, as in the present case, the certification issued by the local civil

registrar enjoys probative value, he being the officer charged under the law to keep a record of all data relative to the issuance
of a marriage license.
Such being the case, the presumed validity of the marriage of petitioner and the deceased has been sufficiently
overcome. It then became the burden of petitioner to prove that their marriage is valid and that they secured the required
marriage license. Although she was declared in default before the trial court, petitioner could have squarely met the issue and
explained the absence of a marriage license in her pleadings before the Court of Appeals and this Court. But petitioner
conveniently avoided the issue and chose to refrain from pursuing an argument that will put her case in jeopardy. Hence, the
presumed validity of their marriage cannot stand.
It is beyond cavil, therefore, that the marriage between petitioner Susan Nicdao and the deceased, having been solemnized
without the necessary marriage license, and not being one of the marriages exempt from the marriage license requirement, is
undoubtedly void ab initio.
It does not follow from the foregoing disquisition, however, that since the marriage of petitioner and the deceased is declared
void ab initio, the death benefits under scrutiny would now be awarded to respondent Susan Yee. To reiterate, under Article 40
of the Family Code, for purposes of remarriage, there must first be a prior judicial declaration of the nullity of a previous
marriage, though void, before a party can enter into a second marriage, otherwise, the second marriage would also be void.
Accordingly, the declaration in the instant case of nullity of the previous marriage of the deceased and petitioner Susan
Nicdao does not validate the second marriage of the deceased with respondent Susan Yee. The fact remains that their
marriage was solemnized without first obtaining a judicial decree declaring the marriage of petitioner Susan Nicdao and the
deceased void. Hence, the marriage of respondent Susan Yee and the deceased is, likewise, void ab initio.
One of the effects of the declaration of nullity of marriage is the separation of the property of the spouses according to the
applicable property regime.[16]Considering that the two marriages are void ab initio, the applicable property regime would not be
absolute community or conjugal partnership of property, but rather, be governed by the provisions of Articles 147 and 148 of the
Family Code on Property Regime of Unions Without Marriage.
Under Article 148 of the Family Code, which refers to the property regime of bigamous marriages, adulterous relationships,
relationships in a state of concubine, relationships where both man and woman are married to other persons, multiple alliances
of the same married man,[17] ... [O]nly the properties acquired by both of the parties through their actual joint contribution of money, property, or industry
shall be owned by them in common in proportion to their respective contributions ...
In this property regime, the properties acquired by the parties through their actual joint contribution shall belong to the coownership. Wages and salaries earned by each party belong to him or her exclusively. Then too, contributions in the form of
care of the home, children and household, or spiritual or moral inspiration, are excluded in this regime. [18]
Considering that the marriage of respondent Susan Yee and the deceased is a bigamous marriage, having been solemnized
during the subsistence of a previous marriage then presumed to be valid (between petitioner and the deceased), the application
of Article 148 is therefore in order.
The disputed P146,000.00 from MBAI [AFP Mutual Benefit Association, Inc.], NAPOLCOM, Commutation, Pag-ibig, and
PCCUI, are clearly renumerations, incentives and benefits from governmental agencies earned by the deceased as a police
officer. Unless respondent Susan Yee presents proof to the contrary, it could not be said that she contributed money, property
or industry in the acquisition of these monetary benefits. Hence, they are not owned in common by respondent and the
deceased, but belong to the deceased alone and respondent has no right whatsoever to claim the same. By intestate
succession, the said death benefits of the deceased shall pass to his legal heirs. And, respondent, not being the legal wife of
the deceased is not one of them.
As to the property regime of petitioner Susan Nicdao and the deceased, Article 147 of the Family Code governs. This article
applies to unions of parties who are legally capacitated and not barred by any impediment to contract marriage, but whose
marriage is nonetheless void for other reasons, like the absence of a marriage license. Article 147 of the Family Code reads Art. 147. When a man and a woman who are capacitated to marry each other, live exclusively with each other as husband and
wife without the benefit of marriage or under a void marriage, their wages and salaries shall be owned by them in equal shares
and the property acquired by both of them through their work or industry shall be governed by the rules on co-ownership.
In the absence of proof to the contrary, properties acquired while they lived together shall be presumed to have been obtained
by their joint efforts, work or industry, and shall be owned by them in equal shares. For purposes of this Article, a party who did
not participate in the acquisition by the other party of any property shall be deemed to have contributed jointly in the acquisition
thereof if the formers efforts consisted in the care and maintenance of the family and of the household.
xxx

xxx

xxx

When only one of the parties to a void marriage is in good faith, the share of the party in bad faith in the co-ownership shall be
forfeited in favor of their common children. In case of default of or waiver by any or all of the common children or their
descendants, each vacant share shall belong to the respective surviving descendants. In the absence of descendants, such
share shall belong to the innocent party. In all cases, the forfeiture shall take place upon termination of the cohabitation.
In contrast to Article 148, under the foregoing article, wages and salaries earned by either party during the cohabitation shall
be owned by the parties in equal shares and will be divided equally between them, even if only one party earned the wages and
the other did not contribute thereto. [19] Conformably, even if the disputed death benefits were earned by the deceased alone as
a government employee, Article 147 creates a co-ownership in respect thereto, entitling the petitioner to share one-half
thereof. As there is no allegation of bad faith in the present case, both parties of the first marriage are presumed to be in good
faith. Thus, one-half of the subject death benefits under scrutiny shall go to the petitioner as her share in the property regime,
and the other half pertaining to the deceased shall pass by, intestate succession, to his legal heirs, namely, his children with
Susan Nicdao.
In affirming the decision of the trial court, the Court of Appeals relied on the case of Vda. de Consuegra v. Government
Service Insurance System,[20] where the Court awarded one-half of the retirement benefits of the deceased to the first wife and
the other half, to the second wife, holding that:
... [S]ince the defendants first marriage has not been dissolved or declared void the conjugal partnership established by that
marriage has not ceased. Nor has the first wife lost or relinquished her status as putative heir of her husband under the new
Civil Code, entitled to share in his estate upon his death should she survive him. Consequently, whether as conjugal partner in
a still subsisting marriage or as such putative heir she has an interest in the husbands share in the property here in dispute....
And with respect to the right of the second wife, this Court observed that although the second marriage can be presumed to be
void ab initio as it was celebrated while the first marriage was still subsisting, still there is need for judicial declaration of such
nullity. And inasmuch as the conjugal partnership formed by the second marriage was dissolved before judicial declaration of
its nullity, [t]he only just and equitable solution in this case would be to recognize the right of the second wife to her share of
one-half in the property acquired by her and her husband, and consider the other half as pertaining to the conjugal partnership
of the first marriage.[21]
It should be stressed, however, that the aforecited decision is premised on the rule which requires a prior and separate
judicial declaration of nullity of marriage. This is the reason why in the said case, the Court determined the rights of the parties
in accordance with their existing property regime.

In Domingo v. Court of Appeals,[22] however, the Court, construing Article 40 of the Family Code, clarified that a prior and
separate declaration of nullity of a marriage is an all important condition precedent only for purposes of remarriage. That is, if a
party who is previously married wishes to contract a second marriage, he or she has to obtain first a judicial decree declaring
the first marriage void, before he or she could contract said second marriage, otherwise the second marriage would be
void. The same rule applies even if the first marriage is patently void because the parties are not free to determine for
themselves the validity or invalidity or their marriage. However, for purposes other than to remarry, like for filing a case for
collection of sum of money anchored on a marriage claimed to be valid, no prior and separate judicial declaration of nullity is
necessary. All that a party has to do is to present evidence, testimonial or documentary, that would prove that the marriage from
which his or her rights flow is in fact valid. Thereupon, the court, if material to the determination of the issues before it, will rule
on the status of the marriage involved and proceed to determine the rights of the parties in accordance with the applicable laws
and jurisprudence. Thus, in Nial v. Bayadog,[23] the Court explained:
[T]he court may pass upon the validity of marriage even in a suit not directly instituted to question the same so long as it is
essential to the determination of the case. This is without prejudice to any issue that may arise in the case. When such need
arises, a final judgment of declaration of nullity is necessary even if the purpose is other than to remarry. The clause on the
basis of a final judgment declaring such previous marriage void in Article 40 of the Family Code connoted that such final
judgment need not be obtained only for purpose of remarriage.
WHEREFORE, the petition is GRANTED, and the decision of the Court of Appeals in CA-G.R. CV No. 51263 which affirmed
the decision of the Regional Trial Court of Quezon City ordering petitioner to pay respondent the sum of P73,000.00 plus
attorneys fees in the amount of P5,000.00, is REVERSED and SET ASIDE. The complaint in Civil Case No. Q-93-18632, is
hereby DISMISSED. No pronouncement as to costs.
SO ORDERED.

REINEL ANTHONY B. DE CASTRO,


Petitioner,

G.R. No. 160172


Present:

QUISUMBING, J.,
Chairperson,
CARPIO,
CARPIO MORALES,
TINGA, and
VELASCO, JR., JJ.

- versus -

ANNABELLE ASSIDAO-DE CASTRO,


Respondent.
Promulgated:
February 13, 2008
x---------------------------------------------------------------------------x
DECISION
TINGA, J.:
This is a petition for review of the Decision[1] of the Court of Appeals in CA-GR CV. No. 69166, [2] declaring that (1) Reianna
Tricia A. De Castro is the legitimate child of the petitioner; and (2) that the marriage between petitioner and respondent is valid
until properly nullified by a competent court in a proceeding instituted for that purpose.
The facts of the case, as culled from the records, follow.
Petitioner and respondent met and became sweethearts in 1991. They planned to get married, thus they applied for a
marriage license with the Office of the Civil Registrar of Pasig City in September 1994. They had their first sexual relation
sometime in October 1994, and had regularly engaged in sex thereafter. When the couple went back to the Office of the Civil
Registrar, the marriage license had already expired. Thus, in order to push through with the plan, in lieu of a marriage license,
they executed an affidavit dated 13 March 1995 stating that they had been living together as husband and wife for at least five
years. The

couple

got

married

on

the

same

date,

with

Judge

Jose

C.

Bernabe,

presiding

judge

of

the Metropolitan Trial Court of Pasig City, administering the civil rites. Nevertheless, after the ceremony, petitioner and
respondent went back to their respective homes and did not live together as husband and wife.
On 13 November 1995, respondent gave birth to a child named Reinna Tricia A. De Castro. Since the childs birth,
respondent has been the one supporting her out of her income as a government dentist and from her private practice.
On 4

June

1998,

respondent

filed

complaint

for

support

against

petitioner

before

the Regional Trial Court of Pasig City (trial court.[3] In her complaint, respondent alleged that she is married to petitioner and
that the latter has reneged on his responsibility/obligation to financially support her as his wife and Reinna Tricia as his child. [4]
Petitioner denied that he is married to respondent, claiming that their marriage is void ab initio since the marriage was
facilitated by a fake affidavit; and that he was merely prevailed upon by respondent to sign the marriage contract to save her
from embarrassment and possible administrative prosecution due to her pregnant state; and that he was not able to get parental
advice from his parents before he got married. He also averred that they never lived together as husband and wife and that he
has never seen nor acknowledged the child.
In its Decision dated 16 October 2000,[5] the trial court ruled that the marriage between petitioner and respondent is not
valid because it was solemnized without a marriage license. However, it declared petitioner as the natural father of the child,
and thus obliged to give her support. Petitioner elevated the case to the Court of Appeals, arguing that the lower court
committed grave abuse of discretion when, on the basis of mere belief and conjecture, it ordered him to provide support to the
child when the latter is not, and could not have been, his own child.
The Court of Appeals denied the appeal. Prompted by the rule that a marriage is presumed to be subsisting until a judicial
declaration of nullity has been made, the appellate court declared that the child was born during the subsistence and validity of
the parties marriage. In addition, the Court of Appeals frowned upon petitioners refusal to undergo DNA testing to prove the
paternity and filiation, as well as his refusal to state with certainty the last time he had carnal knowledge with respondent, saying
that petitioners forgetfulness should not be used as a vehicle to relieve him of his obligation and reward him of his being
irresponsible.[6] Moreover, the Court of Appeals noted the affidavit dated 7 April 1998 executed by petitioner, wherein he
voluntarily admitted that he is the legitimate father of the child.
The appellate court also ruled that since this case is an action for support, it was improper for the trial court to declare the
marriage of petitioner and respondent as null and void in the very same case. There was no participation of the State, through
the prosecuting attorney or fiscal, to see to it that there is no collusion between the parties, as required by the Family Code in
actions for declaration of nullity of a marriage. The burden of proof to show that the marriage is void rests upon petitioner, but it
is a matter that can be raised in an action for declaration of nullity, and not in the instant proceedings. The proceedings before
the trial court should have been limited to the obligation of petitioner to support the child and his wife on the basis of the
marriage apparently and voluntarily entered into by petitioner and respondent. [7] The dispositive portion of the decision reads:

WHEREFORE, premises considered, the Decision dated 16 October 2000, of the Regional Trial Court of Pasig
City, National Capital Judicial Region, Brach 70, in JDRC No. 4626, is AFFIRMED with the MODIFICATIONS (1)
declaring Reianna Tricia A. De Castro, as the legitimate child of the appellant and the appellee and (2) declaring the
marriage on 13 March 1995 between the appellant and the appellee valid until properly annulled by a competent
court in a proceeding instituted for that purpose. Costs against the appellant.[8]

Petitioner filed a motion for reconsideration, but the motion was denied by the Court of Appeals. [9] Hence this petition.
Before us, petitioner contends that the trial court properly annulled his marriage with respondent because as shown by
the evidence and admissions of the parties, the marriage was celebrated without a marriage license. He stresses that the
affidavit they executed, in lieu of a marriage license, contained a false narration of facts, the truth being that he and respondent
never lived together as husband and wife. The false affidavit should never be allowed or admitted as a substitute to fill the
absence of a marriage license.[10] Petitioner additionally argues that there was no need for the appearance of a prosecuting
attorney in this case because it is only an ordinary action for support and not an action for annulment or declaration of absolute
nullity of marriage. In any case, petitioner argues that the trial court had jurisdiction to determine the invalidity of their marriage
since it was validly invoked as an affirmative defense in the instant action for support. Citing several authorities,[11] petitioner
claims that a void marriage can be the subject of a collateral attack. Thus, there is no necessity to institute another
independent proceeding for the declaration of nullity of the marriage between the parties. The refiling of another case for
declaration of nullity where the same evidence and parties would be presented would entail enormous expenses and anxieties,
would be time-consuming for the parties, and would increase the burden of the courts. [12] Finally, petitioner claims that in view of
the nullity of his marriage with respondent and his vigorous denial of the childs paternity and filiation, the Court of Appeals
gravely erred in declaring the child as his legitimate child.

In a resolution dated 16 February 2004, the Court required respondent and the Office of the Solicitor General (OSG) to
file their respective comments on the petition. [13]

In her Comment,[14] respondent claims that the instant petition is a mere dilatory tactic to thwart the finality of the decision
of the Court of Appeals. Echoing the findings and rulings of the appellate court, she argues that the legitimacy of their marriage
cannot be attacked collaterally, but can only be repudiated or contested in a direct suit specifically brought for that
purpose. With regard to the filiation of her child, she pointed out that compared to her candid and straightforward testimony,
petitioner was uncertain, if not evasive in answering questions about their sexual encounters. Moreover, she adds that despite
the challenge from her and from the trial court, petitioner strongly objected to being subjected to DNA testing to prove paternity
and filiation.[15]
For its part, the OSG avers that the Court of Appeals erred in holding that it was improper for the trial court to declare null
and void the marriage of petitioner and respondent in the action for support. Citing the case of Nial v. Bayadog,[16] it states
that courts may pass upon the validity of a marriage in an action for support, since the right to support from petitioner hinges on
the existence of a valid marriage. Moreover, the evidence presented during the proceedings in the trial court showed that the
marriage between petitioner and respondent was solemnized without a marriage license, and that their affidavit (of a man and
woman who have lived together and exclusively with each other as husband and wife for at least five years) was false. Thus, it
concludes the trial court correctly held that the marriage between petitioner and respondent is not valid.[17] In addit
ion, the OSG agrees with the findings of the trial court that the child is an illegitimate child of petitioner and thus entitled to
support.[18]
Two key issues are presented before us. First, whether the trial court had the jurisdiction to determine the validity of the
marriage between petitioner and respondent in an action for support and second, whether the child is the daughter of petitioner.
Anent the first issue, the Court holds that the trial court had jurisdiction to determine the validity of the marriage between
petitioner and respondent. The validity of a void marriage may be collaterally attacked. [19] Thus, in Nial v. Bayadog, we held:
However, other than for purposes of remarriage, no judicial action is necessary to declare a marriage an
absolute nullity. For other purposes, such as but not limited to determination of heirship, legitimacy or illegitimacy of a
child, settlement of estate, dissolution of property regime, or a criminal case for that matter, the court may pass upon
the validity of marriage even in a suit not directly instituted to question the same so long as it is essential to the
determination of the case. This is without prejudice to any issue that may arise in the case. When such need arises,
a final judgment of declaration of nullity is necessary even if the purpose is other than to remarry. The clause on the
basis of a final judgment declaring such previous marriage void in Article 40 of the Family Code connotes that such
final judgment need not be obtained only for purpose of remarriage. [20]

Likewise, in Nicdao Cario v. Yee Cario,[21] the Court ruled that it is clothed with sufficient authority to pass upon the
validity of two marriages despite the main case being a claim for death benefits. Reiterating Nial, we held that the Court may
pass upon the validity of a marriage even in a suit not directly instituted to question the validity of said marriage, so long as it is
essential to the determination of the case. However, evidence must be adduced, testimonial or documentary, to prove the
existence of grounds rendering such a marriage an absolute nullity.[22]

Under the Family Code, the absence of any of the essential or formal requisites shall render the marriage void ab
initio,whereas a defect in any of the essential requisites shall render the marriage voidable. [23] In the instant case, it is clear
from the evidence presented that petitioner and respondent did not have a marriage license when they contracted their
marriage. Instead, they presented an affidavit stating that they had been living together for more than five years. [24] However,
respondent herself in effect admitted the falsity of the affidavit when she was asked during cross-examination, thus
ATTY. CARPIO:
Q

But despite of (sic) the fact that you have not been living together as husband and wife for the last five years
on or before March 13, 1995, you signed the Affidavit, is that correct?
Yes, sir.[25]

The falsity of the affidavit cannot be considered as a mere irregularity in the formal requisites of marriage. The law dispenses
with the marriage license requirement for a man and a woman who have lived together and exclusively with each other as
husband and wife for a continuous and unbroken period of at least five years before the marriage. The aim of this provision is to
avoid exposing the parties to humiliation, shame and embarrassment concomitant with the scandalous cohabitation of persons
outside a valid marriage due to the publication of every applicants name for a marriage license. [26] In the instant case, there
was no scandalous cohabitation to protect; in fact, there was no cohabitation at all. The false affidavit which petitioner and
respondent executed so they could push through with the marriage has no value whatsoever; it is a mere scrap of paper. They
were not exempt from the marriage license requirement. Their failure to obtain and present a marriage license renders their
marriage void ab initio.

Anent the second issue, we find that the child is petitioners illegitimate daughter, and therefore entitled to support.

Illegitimate children may establish their illegitimate filiation in the same way and on the same evidence as legitimate
children.[27]

Thus, one can prove illegitimate filiation through the record of birth appearing in the civil register or a final

judgment, an admission of legitimate filiation in a public document or a private handwritten instrument and signed by the parent
concerned, or the open and continuous possession of the status of a legitimate child, or any other means allowed by the Rules
of Court and special laws.[28]

The Certificate of Live Birth[29] of the child lists petitioner as the father. In addition, petitioner, in an affidavit waiving
additional tax exemption in favor of respondent, admitted that he is the father of the child, thus stating:
1.

I am the legitimate father of REIANNA TRICIA A. DE CASTRO who was born on November 3, 1995 at Better
Living, Paraaque, Metro Manila; [30]

We are likewise inclined to agree with the following findings of the trial court:
That Reinna Tricia is the child of the respondent with the petitioner is supported not only by the testimony of
the latter, but also by respondents own admission in the course of his testimony wherein he conceded that petitioner
was his former girlfriend. While they were sweethearts, he used to visit petitioner at the latters house or clinic. At
times, they would go to a motel to have sex. As a result of their sexual dalliances, petitioner became pregnant which
ultimately led to their marriage, though invalid, as earlier ruled. While respondent claims that he was merely forced to
undergo the marriage ceremony, the pictures taken of the occasion reveal otherwise (Exhs. B, B-1, to B-3, C,
C-1 and C-2, D, D-1 and D-2, E, E-1 and E-2, F, F-1 and F-2, G, G-1 and G-2 and H, H-1 to
H-3). In one of the pictures (Exhs. D, D-1 and D-2), defendant is seen putting the wedding ring on petitioners
finger and in another picture (Exhs. E, E-1 and E-2) respondent is seen in the act of kissing the petitioner. [31]

WHEREFORE, the petition is granted in part. The assailed Decision and Resolution of the Court of Appeals in CAGR CV No. 69166 are SET ASIDE and the decision of the Regional Trial Court Branch 70 of Pasig City in JDRC No. 4626
dated 16 October 2000 is hereby REINSTATED.
SO ORDERED.

G.R. No. 94053 March 17, 1993


REPUBLIC OF THE PHILIPPINES, petitioner,
vs.
GREGORIO NOLASCO, respondent.
The Solicitor General for plaintiff-appellee.

Warloo G. Cardenal for respondent.


RESOLUTION

FELICIANO, J.:
On 5 August 1988, respondent Gregorio Nolasco filed before the Regional Trial Court of Antique, Branch 10, a petition for the
declaration of presumptive death of his wife Janet Monica Parker, invoking Article 41 of the Family Code. The petition prayed
that respondent's wife be declared presumptively dead or, in the alternative, that the marriage be declared null and void. 1
The Republic of the Philippines opposed the petition through the Provincial Prosecutor of Antique who had been deputized to
assist the Solicitor-General in the instant case. The Republic argued, first, that Nolasco did not possess a "well-founded belief
that the absent spouse was already dead," 2 and second, Nolasco's attempt to have his marriage annulled in the same
proceeding was a "cunning attempt" to circumvent the law on marriage. 3
During trial, respondent Nolasco testified that he was a seaman and that he had first met Janet Monica Parker, a British subject,
in a bar in England during one of his ship's port calls. From that chance meeting onwards, Janet Monica Parker lived with
respondent Nolasco on his ship for six (6) months until they returned to respondent's hometown of San Jose, Antique on 19
November 1980 after his seaman's contract expired. On 15 January 1982, respondent married Janet Monica Parker in San
Jose, Antique, in Catholic rites officiated by Fr. Henry van Tilborg in the Cathedral of San Jose.
Respondent Nolasco further testified that after the marriage celebration, he obtained another employment contract as a seaman
and left his wife with his parents in San Jose, Antique. Sometime in January 1983, while working overseas, respondent received
a letter from his mother informing him that Janet Monica had given birth to his son. The same letter informed him that Janet
Monica had left Antique. Respondent claimed he then immediately asked permission to leave his ship to return home. He
arrived in Antique in November 1983.
Respondent further testified that his efforts to look for her himself whenever his ship docked in England proved fruitless. He also
stated that all the letters he had sent to his missing spouse at No. 38 Ravena Road, Allerton, Liverpool, England, the address of
the bar where he and Janet Monica first met, were all returned to him. He also claimed that he inquired from among friends but
they too had no news of Janet Monica.
On cross-examination, respondent stated that he had lived with and later married Janet Monica Parker despite his lack of
knowledge as to her family background. He insisted that his wife continued to refuse to give him such information even after
they were married. He also testified that he did not report the matter of Janet Monica's disappearance to the Philippine
government authorities.
Respondent Nolasco presented his mother, Alicia Nolasco, as his witness. She testified that her daughter-in-law Janet Monica
had expressed a desire to return to England even before she had given birth to Gerry Nolasco on 7 December 1982. When
asked why her daughter-in-law might have wished to leave Antique, respondent's mother replied that Janet Monica never got
used to the rural way of life in San Jose, Antique. Alicia Nolasco also said that she had tried to dissuade Janet Monica from
leaving as she had given birth to her son just fifteen days before, but when she (Alicia) failed to do so, she gave Janet Monica
P22,000.00 for her expenses before she left on 22 December 1982 for England. She further claimed that she had no
information as to the missing person's present whereabouts.
The trial court granted Nolasco's petition in a Judgment dated 12 October 1988 the dispositive portion of which reads:
Wherefore, under Article 41, paragraph 2 of the Family Code of the Philippines (Executive Order No. 209, July 6,
1987, as amended by Executive Order No. 227, July 17, 1987) this Court hereby declares as presumptively dead
Janet Monica Parker Nolasco, without prejudice to her reappearance. 4
The Republic appealed to the Court of Appeals contending that the trial court erred in declaring Janet Monica Parker
presumptively dead because respondent Nolasco had failed to show that there existed a well founded belief for such
declaration.
The Court of Appeals affirmed the trial court's decision, holding that respondent had sufficiently established a basis to form a
belief that his absent spouse had already died.
The Republic, through the Solicitor-General, is now before this Court on a Petition for Review where the following allegations
are made:
1. The Court of Appeals erred in affirming the trial court's finding that there existed a well-founded belief on the part
of Nolasco that Janet Monica Parker was already dead; and
2. The Court of Appeals erred in affirming the trial Court's declaration that the petition was a proper case of the
declaration of presumptive death under Article 41, Family Code. 5
The issue before this Court, as formulated by petitioner is "[w]hether or not Nolasco has a well-founded belief that his wife is
already dead." 6
The present case was filed before the trial court pursuant to Article 41 of the Family Code which provides that:
Art. 41. A marriage contracted by any person during the subsistence of a previous marriage shall be null and void,
unless before the celebration of the subsequent marriage, the prior spouse had been absent for four consecutive
years and the spouse present had a well-founded belief that the absent spouse was already dead. In case of
disappearance where there is danger of death under the circumstances set forth in the provision of Article 391 of the
Civil Code, an absence of only two years shall be sufficient.
For the purpose of contracting the subsequent marriage under the preceding paragraph, the spouse present must
institute a summary proceeding as provided in this Code for the declaration of presumptive death of the absentee,
without prejudice to the effect of reappearance of the absent spouse. (Emphasis supplied).
When Article 41 is compared with the old provision of the Civil Code, which it superseded, 7 the following crucial differences
emerge. Under Article 41, the time required for the presumption to arise has been shortened to four (4) years; however, there is
need for a judicial declaration of presumptive death to enable the spouse present to remarry. 8 Also, Article 41 of the Family
Code imposes a stricter standard than the Civil Code: Article 83 of the Civil Code merely requires either that there be no news
that such absentee is still alive; or the absentee is generally considered to be dead and believed to be so by the spouse
present, or is presumed dead under Article 390 and 391 of the Civil Code. 9 The Family Code, upon the other hand, prescribes
as "well founded belief" that the absentee is already dead before a petition for declaration of presumptive death can be granted.
As pointed out by the Solicitor-General, there are four (4) requisites for the declaration of presumptive death under Article 41 of
the Family Code:

1. That the absent spouse has been missing for four consecutive years, or two consecutive years if the
disappearance occurred where there is danger of death under the circumstances laid down in Article 391, Civil
Code;
2. That the present spouse wishes to remarry;
3. That the present spouse has a well-founded belief that the absentee is dead; and
4. That the present spouse files a summary proceeding for the declaration of presumptive death of the absentee.

10

Respondent naturally asserts that he had complied with all these requirements. 11
Petitioner's argument, upon the other hand, boils down to this: that respondent failed to prove that he had complied with the
third requirement, i.e., the existence of a "well-founded belief" that the absent spouse is already dead.
The Court believes that respondent Nolasco failed to conduct a search for his missing wife with such diligence as to give rise to
a "well-founded belief" that she is dead.
United States v. Biasbas, 12 is instructive as to degree of diligence required in searching for a missing spouse. In that case,
defendant Macario Biasbas was charged with the crime of bigamy. He set-up the defense of a good faith belief that his first wife
had already died. The Court held that defendant had not exercised due diligence to ascertain the whereabouts of his first wife,
noting that:
While the defendant testified that he had made inquiries concerning the whereabouts of his wife, he fails to state of
whom he made such inquiries. He did not even write to the parents of his first wife, who lived in the Province of
Pampanga, for the purpose of securing information concerning her whereabouts. He admits that he had a suspicion
only that his first wife was dead. He admits that the only basis of his suspicion was the fact that she had been
absent. . . . 13
In the case at bar, the Court considers that the investigation allegedly conducted by respondent in his attempt to ascertain Janet
Monica Parker's whereabouts is too sketchy to form the basis of a reasonable or well-founded belief that she was already dead.
When he arrived in San Jose, Antique after learning of Janet Monica's departure, instead of seeking the help of local authorities
or of the British Embassy, 14 he secured another seaman's contract and went to London, a vast city of many millions of
inhabitants, to look for her there.
Q After arriving here in San Jose, Antique, did you exert efforts to inquire the whereabouts of your wife?
A Yes, Sir.
Court:
How did you do that?
A I secured another contract with the ship and we had a trip to London and I went to London to look for
her I could not find her (sic). 15 (Emphasis supplied)
Respondent's testimony, however, showed that he confused London for Liverpool and this casts doubt on his supposed efforts
to locate his wife in England. The Court of Appeal's justification of the mistake, to wit:
. . . Well, while the cognoscente (sic) would readily know the geographical difference between London and
Liverpool, for a humble seaman like Gregorio the two places could mean one place in England, the port where his
ship docked and where he found Janet. Our own provincial folks, every time they leave home to visit relatives in
Pasay City, Kalookan City, or Paraaque, would announce to friends and relatives, "We're going to Manila." This
apparent error in naming of places of destination does not appear to be fatal. 16
is not well taken. There is no analogy between Manila and its neighboring cities, on one hand, and London and Liverpool, on the
other, which, as pointed out by the Solicitor-General, are around three hundred fifty (350) kilometers apart. We do not consider
that walking into a major city like Liverpool or London with a simple hope of somehow bumping into one particular person there
which is in effect what Nolasco says he did can be regarded as a reasonably diligent search.
The Court also views respondent's claim that Janet Monica declined to give any information as to her personal background
even after she had married respondent 17 too convenient an excuse to justify his failure to locate her. The same can be said of
the loss of the alleged letters respondent had sent to his wife which respondent claims were all returned to him. Respondent
said he had lost these returned letters, under unspecified circumstances.
Neither can this Court give much credence to respondent's bare assertion that he had inquired from their friends of her
whereabouts, considering that respondent did not identify those friends in his testimony. The Court of Appeals ruled that since
the prosecutor failed to rebut this evidence during trial, it is good evidence. But this kind of evidence cannot, by its nature, be
rebutted. In any case, admissibility is not synonymous with credibility. 18 As noted before, there are serious doubts to
respondent's credibility. Moreover, even if admitted as evidence, said testimony merely tended to show that the missing spouse
had chosen not to communicate with their common acquaintances, and not that she was dead.
Respondent testified that immediately after receiving his mother's letter sometime in January 1983, he cut short his employment
contract to return to San Jose, Antique. However, he did not explain the delay of nine (9) months from January 1983, when he
allegedly asked leave from his captain, to November 1983 when be finally reached San Jose. Respondent, moreover, claimed
he married Janet Monica Parker without inquiring about her parents and their place of residence. 19 Also, respondent failed to
explain why he did not even try to get the help of the police or other authorities in London and Liverpool in his effort to find his
wife. The circumstances of Janet Monica's departure and respondent's subsequent behavior make it very difficult to regard the
claimed belief that Janet Monica was dead a well-founded one.
In Goitia v. Campos-Rueda, 20 the Court stressed that:
. . . Marriage is an institution, the maintenance of which in its purity the public is deeply interested. It is a relationship
for life and the parties cannot terminate it at any shorter period by virtue of any contract they
make. . . . . 21 (Emphasis supplied)
By the same token, the spouses should not be allowed, by the simple expedient of agreeing that one of them leave the conjugal
abode and never to return again, to circumvent the policy of the laws on marriage. The Court notes that respondent even tried
to have his marriage annulled before the trial court in the same proceeding.
In In Re Szatraw, 22 the Court warned against such collusion between the parties when they find it impossible to dissolve the
marital bonds through existing legal means.
While the Court understands the need of respondent's young son, Gerry Nolasco, for maternal care, still the requirements of the
law must prevail. Since respondent failed to satisfy the clear requirements of the law, his petition for a judicial declaration of
presumptive death must be denied. The law does not view marriage like an ordinary contract. Article 1 of the Family Code
emphasizes that.

. . . Marriage is a special contract of permanent union between a man and a woman entered into in accordance with
law for the establishment of conjugal and family life. It is the foundation of the familyand an inviolable social
institution whose nature, consequences, and incidents are governed by law and not subject to stipulation, except
that marriage settlements may fix the property relations during the marriage within the limits provided by this Code.
(Emphasis supplied)
In Arroyo, Jr. v. Court of Appeals, 23 the Court stressed strongly the need to protect.
. . . the basic social institutions of marriage and the family in the preservation of which the State bas the strongest
interest; the public policy here involved is of the most fundamental kind. In Article II, Section 12 of the Constitution
there is set forth the following basic state policy:
The State recognizes the sanctity of family life and shall protect and strengthen the family as a basic
autonomous social institution. . . .
The same sentiment bas been expressed in the Family Code of the Philippines in Article 149:
The family, being the foundation of the nation, is a basic social institution which public policy cherishes
and protects. Consequently, family relations are governed by law and no custom, practice or agreement
destructive of the family shall be recognized or given effect. 24
In fine, respondent failed to establish that he had the well-founded belief required by law that his absent wife was already dead
that would sustain the issuance of a court order declaring Janet Monica Parker presumptively dead.
WHEREFORE, the Decision of the Court of Appeals dated 23 February 1990, affirming the trial court's decision declaring Janet
Monica Parker presumptively dead is hereby REVERSED and both Decisions are hereby NULLIFIED and SET ASIDE. Costs
against respondent.

[G.R. No. L-8492. February 29, 1956.]


In the Matter of the Declaration of the Civil Status of: LOURDES G. LUKBAN,Petitioner-Appellant, vs. REPUBLIC OF
THE PHILIPPINES, Oppositor-Appellee.
DECISION
BAUTISTA ANGELO, J.:

This is a petition filed in the Court of First Instance of Rizal for a declaration that Petitioner is a widow of her husband Francisco
Chuidian who is presumed to be dead and has no legal impediment to contract a subsequent marriage.
The Solicitor General opposed the petition on the ground that the same is not authorized by law. After Petitioner had presented
her evidence, the court sustained the opposition and dismissed the petition. Hence this appeal.
Lourdes G. Lukban, Petitioner herein, contracted marriage with Francisco Chuidian on December 10, 1933 at the Paco Catholic
Church, Manila. On December 27, of the same year, Francisco left Lourdes after a violent quarrel and since then he has not
been heard from despite diligent search made by her. She also inquired about him from his parents and friends but no one was
able to indicate his whereabouts. She has no knowledge if he is still alive, his last known address being Calle Merced, Paco,
Manila. She believes that he is already dead because he had been absent for more than twenty years, and because she intends
to marry again, she desires that her civil status be defined in order that she may be relieved of any liability under the law.
We believe that the petition at bar comes within the purview of our decision in the case of Nicolai Szartraw, 46 Off. Gaz., 1st
Sup., 243, wherein it was held that a petition for judicial declaration thatPetitioners husband is presumed to be dead cannot be
entertained because it is not authorized by law, and if such declaration cannot be made in a special proceeding similar to the
present, much less can the court determine the status of Petitioner as a widow since this matter must of necessity depend upon
the fact of death of the husband. This the court can declare upon proper evidence, but not to decree that he is merely presumed
to be dead. (Nicolai Szartraw, 46 Off. Gaz., 1st sup. 243).
The philosophy behind the ruling that such judicial pronouncement cannot be made in a proceeding of this nature is well
expressed in the case above-cited. Thus, we there said that A judicial pronouncement to that effect, even if final and executory,
would still be a prima facie presumption only. It is still disputable. It is for that reason that it cannot be the subject of a judicial
pronouncement or declaration, if it is the only question or matter involved in a case, or upon which a competent court has to
pass cralaw. It is, therefore, clear that a judicial declaration that a person is presumptively dead, because he had been unheard
from in seven years, being a presumption juris tantum only, subject to contrary proof, cannot reach the stage of finality or
become final.
Appellant claims that the remedy she is seeking for can be granted in the present proceedings because in the case of Hagans
vs. Wislizenus, 42 Phil., 880, it was declared that a special proceeding is an application or proceeding to establish the status or
right of a party, or a particular fact; chan roblesvirtualawlibrarybut, as already said, that remedy can be invoked if the purpose is
to seek the declaration of death of the husband, and not, as in the present case, to establish a presumption of death. If it can be
satisfactorily proven that the husband is dead, the court would not certainly deny a declaration to that effect as has been
intimated in the case of Nicolas Szartraw, supra.
Appellant also claims that the present petition can be entertained because article 349 of the Revised Penal Code, in defining
bigamy, provides that a person commits that crime if he contracts a second marriage before the absent spouse has been
declared presumptively dead by means of a judgment rendered in the proper proceedings and, it is claimed, the present
petition comes within the purview of this legal provision. The argument is untenable for the words proper proceedings used in
said article can only refer to those authorized by law such as those which refer to the administration or settlement of the estate
of a deceased person (Articles 390 and 391, new Civil Code). That such is the correct interpretation of the provision in question
finds support in the case of Jones vs. Hortiguela, 64 Phil., 179, wherein this Court made the following
comment:chanroblesvirtuallawlibrary
For the purposes of the civil marriage law, it is not necessary to have the former spouse judicially declared an absentee. The
declaration of absence made in accordance with the provisions of the Civil Code has for its sole purpose to enable the taking of
the necessary precautions for the administration of the estate of the absentee. For the celebration of civil marriage, however,
the law only requires that the former spouse has been absent for seven consecutive years at the time of the second marriage,
that the spouse present does not know his or her former spouse to be living, that each former spouse is generally reputed to be
dead and the spouse present so believes at the time of the celebration of the marriage (section III, paragraph 2, General
Orders, No. 68).
The decision appealed from is affirmed, without pronouncement as to costs.

G.R. No. L-14058

March 24, 1960

In the matter of the petition for the declaration of William Gue, presumptively dead. ANGELINA L. GUE,petitionerappellant,
vs.
THE REPUBLIC OF THE PHILIPPINES, oppositor-appellee.

Virgilio V. David for appellant.


Office of the Solicitor General Edilberto Barot and Solicitor E. M. Salva for appellee.
MONTEMAYOR, J.:
This is an appeal from the order of the Court of First Instance of Manila, presided by Judge Bonifacio Ysip, dismissing the
petition of Angelina Gue. Involving as it does only question of law, the appeal was taken directly to us.
On November 20, 1957, Angelina L. Gue filed a petition in the Court of First Instance of Manila, Civil Case No. 34303, alleging
that she was married to William Gue; that they had a child named Anthony L. Gue; that January 5, 1946, her husband left
Manila where they were residing and went to Shanghai, China, but since then, he had not been heard of, neither had he written
to her, nor in anyway communicated with her as to his whereabouts; that despite her efforts and diligence, she failed to locate
him; and that they had not acquired any property during the marriage. She asked the court for a declaration of the presumption
of death of William Gue, pursuant to the provisions of Article 390 of the Civil Code of the Philippines. After due publication and
hearing, the trial court issued the order of dismissal, which we reproduce below:
This is a petition filed by Angelina L. Gue to declare her husband. William Gue, presumptively dead. During the hearing of
this petition, it was established by the testimony of the petitioner that she and her husband were married on October 11,
1944 in the City of Manila before the parish priest of Tondo, Manila, as shows in Exhibit B, the marriage contract. Her
husband, who is a Chinese citizen, left the Philippines for Shanghai on January, 1946. The petitioner joined him in
Shanghai in August of the same year. In January, 1949, the petitioner came back to the Philippines alone with her
children, on which occasion her husband promised to follow her. However, up to the present time, said William Gue has
not returned to the Philippines. From January, 1949, the petitioner had sent letters to her husband in Shanghai, but she
never received any reply thereto. She made inquiries from the Bureau of Immigration in 1955 and 1958 as to whether her
husband had already returned to the Philippines and she received Exhibit D and Exhibit E from said Office, which gave no
information as to the whereabouts of her husband. It was also established by petitioner's testimony that no properties
have been acquired by said spouses during their union, and during which they begot two children, named Eugeni and
Anthony, surnamed Gue.
With this evidence on record and considering the allegations in the petition, it is clear that no right had been established
by the petitioner upon which a judicial decree may be predicated, and this action is not for the settlement of the estate of
the absentee, as it is clear that he did not leave any.
In the case of "Petition for the Presumption of Death of Nicolai Szatraw", 81 Phil., 461, a case similar to the present, the
Supreme Court held:
The petition is not for the settlement of the estate of Nicolai Szatraw, because it does not appear that he possessed
property brought to the marriage and because he had acquired no property during his married life with the petitioner. The
rule invoked by the latter is merely one of evidence which permits the court to presume that a person is dead after the fact
that such person had been unheard from in seven years had been established. This presumption may arise and be
invoked and made in a case, whether in an action or in a special proceeding, which is tried or heard by, and submitted for
decision to, a special proceeding. In this case, there is no right to be enforced nor is there a remedy prayed for by the
petitioner for the final determination of his right or status or for the ascertainment of a particular fact (Hagans vs.
Wislizenus, 42 Phil., 880), for the petition does not pray for a declaration that the petitioner's husband is dead, but merely
asks for a declaration that he be presumed dead because he had been unheard from in seven years. If there is any
pretense at securing a declaration that the petitioner's husband is dead, such a pretension cannot be granted because it is
unauthorized. The petition is for a declaration that the petitioner's husband is presumptively dead. But this declaration,
even if judicially made, would not improve the petitioner's situation, because such a presumption is established by law. A
judicial pronouncement to that effect, even if final and executory, would still be a prima facie presumption only. It is still
disputable. It is for that reason that it cannot be the subject of a judicial pronouncement or declaration, if it is the only
question or matter involved in a case, or upon which a competent court has to pass. The latter must decide finally the
controversy the right or status of a party or established finally a particular fact, out of which certain rights and obligations
arise or may arise; and once such controversy is decided by a final judgment or such right or status is determined, then
the judgment on the subject of the controversy, or the decree upon the right or status of a party or upon the existence of a
particular fact, becomes res judicata, subject to no collateral attack, except in a few rare instances especially provided by
law. It is, therefore, clear that a judicial declaration that a person is presumptively dead, because he had been unheard
from in seven years, being a presumption juris tantum only, subject to contrary proof cannot reach the state of finality or
become final. Proof of actual death of the person presumed dead because he had been unheard from in seven years,
would have to be made in another proceeding to have such particular fact finally determined. If a judicial decree declaring
a person presumptively dead, because he had not been heard from in seven years, cannot become final and executory
even after the lapse of the reglementary period within which an appeal may be taken, for such a presumption is still
disputable and remains subject to contrary proof, then a petition for such a declaration is useless, unnecessary,
superfluous and of no benefit to the petitioner. The Court should not waste its valuable time and be made to perform a
superfluous and meaningless act.
"Little effort is necessary to perceive that a declaration such as the one prayed for by the petitioner, if granted, may make
or lead her to believe that the marital bonds which binds her to her husband are torn asunder, and that for that reason she
is or may feel free to enter into a new marriage contract. The framers of the rules of court, by the presumption provided for
in the rule of evidence in question, did not intend and mean that a judicial declaration based solely upon that presumption
may be made. A petition for a declaration such as the one filed in this case may be made in collusion with the other
spouse. If that were the case, then a decree of divorce that cannot be obtained or granted under the provisions of the
Divorce Law (Act No. 2710) could easily be secured by means of a judicial decree declaring a person unheard from in
seven years to be presumptively dead. This is another strong reason why a petition such as the one presented in this
case should not be countenanced and allowed. What cannot be obtained directly under the provisions of the Divorce Law
could indirectly be secured under the provisions of Rule 123, section 69 (x). Obviously, the latter must not be made to
prevail over the former."
In view of the foregoing and the doctrine of the Supreme Court laid down in the case above-cited, the Court hereby orders
that this case be, as it is hereby dismissed, without pronouncement as the costs.
In her appeal, Angelina invoked the provisions of the Article 390 of the New Civil Code, which for purpose of reference, we
reproduce below.
ART. 390. After an absence of seven years, it being unknown whether or not the absentee still lives, he shall be presumed
dead for all purposes, except for those of succession.
The absentee shall not be presumed dead for the purpose of opening this succession till after an absence of ten years. If
he disappeared after the of seventy-five years, an absence of five years shall be sufficient in order that his succession
may be opened.
She contends that under Article 191 of the Old Civil Code, which reads:
After thirty years have elapsed since disappearance of the absentee, or since he was last heard from, or ninety years from
his birth, the judgment upon the petition of any party lawfully interested, shall make an order declaring that such absentee
is presumed to be dead.

a person could be declared presumptively dead, but that said legal provision was repealed by the Code of Civil Procedure and
continued to be repealed by the Rules of Court. Consequently, only a mere disputable presumption of death was available to
any party, and that the case of Nicolai Szatraw, cited by the trial court, was decided on the law then existing, namely, the Code
of Civil Procedure, and later the new Rules of Court. However, according to appellant, with the promulgation of the New Civil
Code in 1950, particularly, Article 390 thereof, the Courts are now authorized to declare persons presumptively dead.
In answer to her contention, the Solicitor General, as appellee herein, correctly cites our decision in the recent case of Lourdes
G. Lukban vs. Republic of the Philippines, 98 Phil., 574; 52 Off. Gaz., No. 3, 1441, decided long after the New Civil Code went
into effect, wherein we reiterated the doctrine laid own in Nicolai Szatraw, supra. We quote the pertinent portions of our decision
in that case:
This is a petition filed in the Court of First Instance of Rizal for a declaration that petitioner is a widow of her husband
Francisco Chuidian who is presumed to be dead and has no legal impediment to contract a subsequent marriage.
The Solicitor General opposed the petition on the ground that the same is not authorized by law. After petitioner had
presented her evidence, the court sustained the opposition and dismissed the petition. Hence this appeal.
Lourdes G. Lukban, petitioner herein, contracted marriage with Francisco Chuidian on December 10, 1933 at the Paco
Catholic Church, Manila. On December 27, of the same year, Francisco left Lourdes after a violent quarrel and since then
he has not been heard from despite diligent search made by her. She also inquired about him from his parents and friends
but no one was able to indicate his whereabouts. She has no knowledge if he is still alive, his last known address being
Calle Merced, Paco, Manila. She believes that he is already dead because he had been absent for more than twenty
years, and because she intends to marry again, she desires that her civil status be defined in order that she may be
relieved of any liability under the law.
We believe that the petition at bar comes within the purview of our decision in the case of Nicolai Szatraw, 46 Off. Gaz. 1st
Sup. 243, wherein it was held that a petition for judicial declaration that petitioner's husband is presumed to be dead
cannot be entertained because it is not authorized by law, and if such declaration cannot be made in a special
proceedings similar to the present, much less can the court determine the status of petitioner as a widow since this matter
must of necessity depend upon the fact of death of the husband. This the court can declare upon proper evidence, but not
to decree that he is merely presumed to be dead. (Nicolai Szatraw, 48 Off. Gaz., 1st Sup. 243).
The philosophy behind the ruling that such judicial pronouncement cannot be made in a proceeding of this nature is well
expressed in the case above-cited. Thus, we there said that "A judicial pronouncement to that effect, even if final and
executory, would still be a prima facie presumption only. It is still disputable. It is for that reason that it cannot be the
subject of a judicial pronouncement or declaration, if it is the only question or matter involved in a case, or upon which a
competent court has to pass .. It is, therefore, clear that a judicial declaration that a person is presumptively dead,
because he had been unheard from in seven years, being a presumption juris tantum only, subject to contrary proof,
cannot reach the stage of finality or become final."

G.R. No. 136467

April 6, 2000

ANTONIA ARMAS Y CALISTERIO, petitioner,


vs.
MARIETTA CALISTERIO, respondent.

VITUG, J.:

On 24 April 1992, Teodorico Calisterio died intestate, leaving several parcels of land with an estimated value of P604,750.00.
Teodorico was survived by his wife, herein respondent Marietta Calisterio.
Teodorico was the second husband of Marietta who had previously been married to James William Bounds on 13 January 1946
at Caloocan City. James Bounds disappeared without a trace on 11 February 1947. Teodorico and Marietta were married eleven
years later, or on 08 May 1958, without Marietta having priorly secured a court declaration that James was presumptively dead.
On 09 October 1992, herein petitioner Antonia Armas y Calisterio, a surviving sister of Teodorico, filed with the Regional Trial
Court ("RTC") of Quezon City, Branch 104, a petition entitled, "In the Matter of Intestate Estate of the Deceased Teodorico
Calisterio y Cacabelos, Antonia Armas, Petitioner," claiming to be inter alia, the sole surviving heir of Teodorico Calisterio, the
marriage between the latter and respondent Marietta Espinosa Calisterio being allegedly bigamous and thereby null and void.
She prayed that her son Sinfroniano C. Armas, Jr., be appointed administrator, without bond, of the estate of the deceased and
that the inheritance be adjudicated to her after all the obligations of the estate would have been settled.
Respondent Marietta opposed the petition. Marietta stated that her first marriage with James Bounds had been dissolved due to
the latter's absence, his whereabouts being unknown, for more than eleven years before she contracted her second marriage
with Teodorico. Contending to be the surviving spouse of Teodorico, she sought priority in the administration of the estate of the
decedent.
On 05 February 1993, the trial court issued an order appointing jointly Sinfroniano C. Armas, Jr., and respondent Marietta
administrator and administratrix, respectively, of the intestate estate of Teodorico.
On 17 January 1996, the lower court handed down its decision in favor of petitioner Antonia; it adjudged:
WHEREFORE, judgment is hereby rendered finding for the petitioner and against the oppositor whereby herein petitioner,
Antonia Armas y Calisterio, is declared as the sole heir of the estate of Teodorico Calisterio y Cacabelos. 1
Respondent Marietta appealed the decision of the trial court to the Court of Appeals, formulating that
1. The trial court erred in applying the provisions of the Family Code in the instant case despite the fact that the
controversy arose when the New Civil Code was the law in force.
2. The trial court erred in holding that the marriage between oppositor-appellant and the deceased Teodorico Calisterio is
bigamous for failure of the former to secure a decree of the presumptive death of her first spouse.
3. The trial court erred in not holding that the property situated at No. 32 Batangas Street, San Francisco del Monte,
Quezon City, is the conjugal property of the oppositor-appellant and the deceased Teodorico Calisterio.
4. The trial court erred in holding that oppositor-appellant is not a legal heir of deceased Teodorico Calisterio.
5. The trial court erred in not holding that letters of administration should be granted solely in favor of oppositorappellant. 2
On 31 August 1998, the appellate court, through Mr. Justice Conrado M. Vasquez, Jr., promulgated its now assailed decision,
thus:
IN VIEW OF ALL THE FOREGOING, the Decision appealed from is REVERSED AND SET ASIDE, and a new one
entered declaring as follows:
(a) Marietta Calisterio's marriage to Teodorico remains valid;
(b) The house and lot situated at #32 Batangas Street, San Francisco del Monte, Quezon City, belong to the
conjugal partnership property with the concomitant obligation of the partnership to pay the value of the land to
Teodorico's estate as of the time of the taking;
(c) Marietta Calisterio, being Teodorico's compulsory heir, is entitled to one half of her husband's estate, and
Teodorico's sister, herein petitioner Antonia Armas and her children, to the other half;
(d) The trial court is ordered to determine the competence of Marietta E. Calisterio to act as administrator of
Teodorico's estate, and if so found competent and willing, that she be appointed as such; otherwise, to determine
who among the deceased's next of kin is competent and willing to become the administrator of the estate. 3
On 23 November 1998, the Court of Appeals denied petitioner's motion for reconsideration, prompting her to interpose the
present appeal. Petitioner asseverates:
It is respectfully submitted that the decision of the Court of Appeals reversing and setting aside the decision of the trial
court is not in accord with the law or with the applicable decisions of this Honorable Court. 4
It is evident that the basic issue focuses on the validity of the marriage between the deceased Teodorico and respondent
Marietta, that, in turn, would be determinative of her right as a surviving spouse.
The marriage between the deceased Teodorico and respondent Marietta was solemnized on 08 May 1958. The law in force at
that time was the Civil Code, not the Family Code which took effect only on 03 August 1988. Article 256 of the Family
Code 5 itself limited its retroactive governance only to cases where it thereby would not prejudice or impair vested or acquired
rights in accordance with the Civil Code or other laws.
Verily, the applicable specific provision in the instant controversy is Article 83 of the New Civil Code which provides:
Art. 83. Any marriage subsequently contracted by any person during the lifetime of the first spouse of such person with
any person other than such first spouse shall be illegal and void from its performance, unless:
(1) The first marriage was annulled or dissolved; or
(2) The first spouse had been absent for seven consecutive years at the time of the second marriage without the spouse
present having news of the absentee being alive, or if the absentee, though he has been absent for less than seven years,
is generally considered as dead and believed to be so by the spouse present at the time of contracting such subsequent
marriage, or if the absentee is presumed dead according to articles 390 and 391. The marriage so contracted shall be
valid in any of the three cases until declared null and void by a competent court.
Under the foregoing provisions, a subsequent marriage contracted during the lifetime of the first spouse is illegal and void ab
initio unless the prior marriage is first annulled or dissolved. Paragraph (2) of the law gives exceptions from the above rule. For
the subsequent marriage referred to in the three exceptional cases therein provided, to be held valid, the spouse present (not
the absentee spouse) so contracting the later marriage must have done so in good faith. 6 Bad faith imports a dishonest
purpose or some moral obliquity and conscious doing of wrong it partakes of the nature of fraud, a breach of a known duty
through some motive of interest or ill will. 7 The Court does not find these circumstances to be here extant.

A judicial declaration of absence of the absentee spouse is not necessary 8 as long as the prescribed period of absence is met. It
is equally noteworthy that the marriage in these exceptional cases are, by the explicit mandate of Article 83, to be deemed valid
"until declared null and void by a competent court." It follows that the burden of proof would be, in these cases, on the party
assailing the second marriage.
In contrast, under the 1988 Family Code, in order that a subsequent bigamous marriage may exceptionally be considered valid,
the following conditions must concur; viz.: (a) The prior spouse of the contracting party must have been absent for four
consecutive years, or two years where there is danger of death under the circumstances stated in Article 391 of the Civil Code
at the time of disappearance; (b) the spouse present has a well-founded belief that the absent spouse is already dead; and (c)
there is, unlike the old rule, a judicial declaration of presumptive death of the absentee for which purpose the spouse present
can institute a summary proceeding in court to ask for that declaration. The last condition is consistent and in consonance with
the requirement of judicial intervention in subsequent marriages as so provided in Article 41 9 , in relation to Article 40,10 of the
Family Code.
In the case at bar, it remained undisputed that respondent Marietta's first husband, James William Bounds, had been absent or
had disappeared for more than eleven years before she entered into a second marriage in 1958 with the deceased Teodorico
Calisterio. This second marriage, having been contracted during the regime of the Civil Code, should thus be deemed valid
notwithstanding the absence of a judicial declaration of presumptive death of James Bounds.
The conjugal property of Teodorico and Marietta, no evidence having been adduced to indicate another property regime
between the spouses, pertains to them in common. Upon its dissolution with the death of Teodorico, the property should rightly
be divided in two equal portions one portion going to the surviving spouse and the other portion to the estate of the deceased
spouse. The successional right in intestacy of a surviving spouse over the net estate 11 of the deceased, concurring with
legitimate brothers and sisters or nephews and nieces (the latter by right of representation), is one-half of the inheritance, the
brothers and sisters or nephews and nieces, being entitled to the other half. Nephews and nieces, however, can only succeed
by right of representation in the presence of uncles and aunts; alone, upon the other hand, nephews and nieces can succeed in
their own right which is to say that brothers or sisters exclude nephews and nieces except only in representation by the latter of
their parents who predecease or are incapacitated to succeed. The appellate court has thus erred in granting, in paragraph (c)
of the dispositive portion of its judgment, successional rights, to petitioner's children, along with their own mother Antonia who
herself is invoking successional rights over the estate of her deceased brother.1wphi1
WHEREFORE, the assailed judgment of the Court of Appeals in CA G.R. CV No. 51574 is AFFIRMED except insofar only as it
decreed in paragraph (c) of the dispositive portion thereof that the children of petitioner are likewise entitled, along with her, to
the other half of the inheritance, in lieu of which, it is hereby DECLARED that said one-half share of the decedent's estate
pertains solely to petitioner to the exclusion of her own children. No costs.

REPUBLIC OF THE PHILIPPINES,


Petitioner,

G.R. No. 159614


Present:

- versus THE HONORABLE COURT OF


APPEALS (TENTH DIVISION)
and ALAN B. ALEGRO,

PUNO, J., Chairman,


AUSTRIA-MARTINEZ,
CALLEJO, SR.,
TINGA, and
CHICO-NAZARIO, JJ.

Respondents.
Promulgated:
December 9, 2005
x--------------------------------------------------x
DECISION
CALLEJO, SR., J.:
On March 29, 2001, Alan B. Alegro filed a petition in the Regional Trial Court (RTC) of Catbalogan, Samar, Branch 27, for
the declaration of presumptive death of his wife, Rosalia (Lea) A. Julaton.
In an Order[1] dated April 16, 2001, the court set the petition for hearing on May 30, 2001 at 8:30 a.m. and directed that a copy
of the said order be published once a week for three (3) consecutive weeks in the Samar Reporter, a newspaper of general
circulation in the Province of Samar, and

that a copy be posted in the courts bulletin board for at least three weeks before the next scheduled hearing. The court also
directed that copies of the order be served on the Solicitor General, the Provincial Prosecutor of Samar, and Alan, through
counsel, and that copies be sent to Lea by registered mail. Alan complied with all the foregoing jurisdictional requirements.[2]
On May 28, 2001, the Republic of the Philippines, through the Office of the Solicitor General (OSG), filed a Motion to
Dismiss[3] the petition, which was, however, denied by the court for failure to comply with Rule 15 of the Rules of Court.[4]
At the hearing, Alan adduced evidence that he and Lea were married on January 20, 1995 in Catbalogan, Samar.[5] He
testified that, on February 6, 1995, Lea arrived home late in the evening and he berated her for being always out of their house.
He told her that if she enjoyed the life of a single person, it would be better for her to go back to her parents.[6] Lea did not
reply. Alan narrated that, when he reported for work the following day, Lea was still in the house, but when he arrived home
later in the day, Lea was nowhere to be found.[7] Alan thought that Lea merely went to her parents house in Bliss, Sto. Nio,
Catbalogan, Samar.[8]However, Lea did not return to their house anymore.
Alan further testified that, on February 14, 1995, after his work, he went to the house of Leas parents to see if she was there,
but he was told that she was not there. He also went to the house of Leas friend, Janeth Bautista, at Barangay Canlapwas,
but he was informed by Janettes brother-in-law, Nelson Abaenza, that Janeth had left for Manila.[9] When Alan went back to the
house of his parents-in-law, he learned from his father-in-law that Lea had been to their house but that she left without notice.
[10] Alan sought the help of Barangay Captain Juan Magat, who promised to help him locate his wife. He also inquired from his
friends of Leas whereabouts but to no avail.[11]
Sometime in June 1995, he decided to go to Manila to look for Lea, but his mother asked him to leave after the town fiesta of
Catbalogan, hoping that Lea may come home for the fiesta. Alan agreed.[12] However, Lea did not show up. Alan then left for
Manila on August 27, 1995. He went to a house in Navotas where Janeth, Leas friend, was staying. When asked where Lea
was, Janeth told him that she had not seen her.[13] He failed to find out Leas whereabouts despite his repeated talks with
Janeth. Alan decided to work as a part-time taxi driver. On his free time, he would look for Lea in the malls but still to no avail.
He returned to Catbalogan in 1997 and again looked for his wife but failed.[14]
On June 20, 2001, Alan reported Leas disappearance to the local police station.[15] The police authorities issued an Alarm
Notice on July 4, 2001.[16] Alan also reported Leas disappearance to the National Bureau of Investigation (NBI) on July 9,
2001.[17]
Barangay Captain Juan Magat corroborated the testimony of Alan. He declared that on February 14, 1995, at 2:00 p.m., Alan
inquired from him if Lea passed by his house and he told Alan that she did not. Alan also told him that Lea had disappeared.
He had not seen Lea in the barangay ever since.[18] Leas father, who was his compadre and the owner of Radio DYMS, told
him that he did not know where Lea was.[19]
After Alan rested his case, neither the Office of the Provincial Prosecutor nor the Solicitor General adduced evidence in
opposition to the petition.
On January 8, 2002, the court rendered judgment granting the petition. The fallo of the decision reads:
WHEREFORE, and in view of all the foregoing, petitioners absent spouse ROSALIA JULATON is hereby declared
PRESUMPTIVELY DEAD for the purpose of the petitioners subsequent marriage under Article 41 of the Family Code of the
Philippines, without prejudice to the effect of reappearance of the said absent spouse.
SO ORDERED.[20]
The OSG appealed the decision to the Court of Appeals (CA) which rendered judgment on August 4, 2003, affirming the
decision of the RTC.[21] The CA cited the ruling of this Court in Republic v. Nolasco.[22]
The OSG filed a petition for review on certiorari of the CAs decision alleging that respondent Alan B. Alegro failed to
prove that he had a well-founded belief that Lea was already dead.[23] It averred that the respondent failed to exercise
reasonable and diligent efforts to locate his wife. The respondent even admitted that Leas father told him on February 14, 1995
that Lea had been to their house but left without notice. The OSG pointed out that the respondent reported his wifes
disappearance to the local police and also to the NBI only after the petitioner filed a motion to dismiss the petition. The
petitioner avers that, as gleaned from the evidence, the respondent did not really want to find and locate Lea. Finally, the
petitioner averred:
In view of the summary nature of proceedings under Article 41 of the Family Code for the declaration of presumptive death of
ones spouse, the degree of due diligence set by this Honorable Court in the above-mentioned cases in locating the
whereabouts of a missing spouse must be strictly complied with. There have been times when Article 41 of the Family Code
had been resorted to by parties wishing to remarry knowing fully well that their alleged missing spouses are alive and well. It is
even possible that those who cannot have their marriages x x x declared null and void under Article 36 of the Family Code
resort to Article 41 of the Family Code for relief because of the x x x summary nature of its proceedings.
It is the policy of the State to protect and strengthen the family as a basic social institution. Marriage is the foundation of the
family. Since marriage is an inviolable social institution that the 1987 Constitution seeks to protect from dissolution at the whim
of the parties. For respondents failure to prove that he had a well-founded belief that his wife is already

dead and that he exerted the required amount of diligence in searching for his missing wife, the petition for declaration of
presumptive death should have been denied by the trial court and the Honorable Court of Appeals.[24]
The petition is meritorious.
Article 41 of the Family Code of the Philippines reads:
Art. 41. A marriage contracted by any person during the subsistence of a previous marriage shall be null and void, unless
before the celebration of the subsequent marriage, the prior spouse had been absent for four consecutive years and the spouse
present had a well-founded belief that the absent spouse was already dead. In case of disappearance where there is danger
under the circumstances set forth in the provisions of Article 391 of the Civil Code, an absence of only two years shall be
sufficient.
For the purpose of contracting the subsequent marriage under the preceding paragraph, the spouse present must
institute a summary proceeding as provided in this Code for the declaration of presumptive death of the absentee, without
prejudice to the effect of reappearance of the absent spouse.[25]
The spouse present is, thus, burdened to prove that his spouse has been absent and that he has a well-founded belief
that the absent spouse is already dead before the present spouse may contract a subsequent marriage. The law does not
define what is meant by a well-grounded belief. Cuello Callon writes that es menester que su creencia sea firme se funde en
motivos racionales.[26]
Belief is a state of the mind or condition prompting the doing of an overt act. It may be proved by direct evidence or
circumstantial evidence which may tend, even in a slight degree, to elucidate the inquiry or assist to a determination probably
founded in truth. Any fact or circumstance relating to the character, habits, conditions, attachments, prosperity and objects of
life which usually control the conduct of men, and are the motives of their actions, was, so far as it tends to explain or
characterize their disappearance or throw light on their intentions,[27] competence evidence on the ultimate question of his
death.
The belief of the present spouse must be the result of proper and honest to goodness inquiries and efforts to ascertain the
whereabouts of the absent spouse and whether the absent spouse is still alive or is already dead. Whether or not the spouse
present acted on a well-founded belief of death of the absent spouse depends upon the inquiries to be drawn from a great many
circumstances occurring before and after the disappearance of the absent spouse and the nature and extent of the inquiries
made by present spouse.[28]
Although testimonial evidence may suffice to prove the well-founded belief of the present spouse that the absent spouse
is already dead, in Republic v. Nolasco,[29] the Court warned against collusion between the parties when they find it impossible
to dissolve the marital bonds through existing legal means. It is also the maxim that men readily believe what they wish to be
true.
In this case, the respondent failed to present a witness other than Barangay Captain Juan Magat. The respondent even
failed to present Janeth Bautista or Nelson Abaenza or any other person from whom he allegedly made inquiries about Lea to
corroborate his testimony. On the other hand, the respondent admitted that when he returned to the house of his parents-in-law
on February 14, 1995, his father-in-law told him that Lea had just been there but that she left without notice.
The respondent declared that Lea left their abode on February 7, 1995 after he chided her for coming home late and for
being always out of their house, and told her that it would be better for her to go home to her parents if she enjoyed the life of a
single person. Lea, thus, left their conjugal abode and never returned. Neither did she communicate with the respondent after
leaving the conjugal abode because of her resentment to the chastisement she received from him barely a month after their
marriage. What is so worrisome is that, the respondent failed to make inquiries from his parents-in-law regarding Leas
whereabouts before filing his petition in the RTC. It could have enhanced the credibility of the respondent had he made
inquiries from his parents-in-law about Leas whereabouts considering that Leas father was the owner of Radio DYMS.
The respondent did report and seek the help of the local police authorities and the NBI to locate Lea, but it was only an
afterthought. He did so only after the OSG filed its notice to dismiss his petition in the RTC.
In sum, the Court finds and so holds that the respondent failed to prove that he had a well-founded belief, before he filed his
petition in the RTC, that his spouse Rosalia (Lea) Julaton was already dead.
IN LIGHT OF ALL THE FOREGOING, the petition is GRANTED. The Decision of the Court of Appeals in CA-G.R. CV No.
73749 is REVERSED and SET ASIDE. Consequently, the Regional Trial Court of Catbalogan, Samar, Branch 27,
isORDERED to DISMISS the respondents petition.
SO ORDERED.

G.R. No. 165545

March 24, 2006

SOCIAL SECURITY SYSTEM, Petitioner,


vs.
TERESITA JARQUE VDA. DE BAILON, Respondent.
DECISION
CARPIO MORALES,J.:
The Court of Appeals Decision1 dated June 23, 20042 and Resolution dated September 28, 2004 3 reversing the Resolution
dated April 2, 20034 and Order dated June 4, 20035 of the Social Security Commission (SSC) in SSC Case No. 4-15149-01 are
challenged in the present petition for review on certiorari.
On April 25, 1955, Clemente G. Bailon (Bailon) and Alice P. Diaz (Alice) contracted marriage in Barcelona, Sorsogon. 6
More than 15 years later or on October 9, 1970, Bailon filed before the then Court of First Instance (CFI) of Sorsogon a
petition7 to declare Alice presumptively dead.
By Order of December 10, 1970,8 the CFI granted the petition, disposing as follows:
WHEREFORE, there being no opposition filed against the petition notwithstanding the publication of the Notice of Hearing in a
newspaper of general circulation in the country, Alice Diaz is hereby declared to [sic] all legal intents and purposes, except for
those of succession, presumptively dead.
SO ORDERED.9 (Underscoring supplied)
Close to 13 years after his wife Alice was declared presumptively dead or on August 8, 1983, Bailon contracted marriage with
Teresita Jarque (respondent) in Casiguran, Sorsogon. 10
On January 30, 1998, Bailon, who was a member of the Social Security System (SSS) since 1960 and a retiree pensioner
thereof effective July 1994, died.11
Respondent thereupon filed a claim for funeral benefits, and was granted P12,00012 by the SSS.
Respondent filed on March 11, 1998 an additional claim for death benefits 13 which was also granted by the SSS on April 6,
1998.14
Cecilia Bailon-Yap (Cecilia), who claimed to be a daughter of Bailon and one Elisa Jayona (Elisa) contested before the SSS the
release to respondent of the death and funeral benefits. She claimed that Bailon contracted three marriages in his lifetime, the
first with Alice, the second with her mother Elisa, and the third with respondent, all of whom are still alive; she, together with her
siblings, paid for Bailons medical and funeral expenses; and all the documents submitted by respondent to the SSS in support
of her claims are spurious.
In support of her claim, Cecilia and her sister Norma Bailon Chavez (Norma) submitted an Affidavit dated February 13,
199915 averring that they are two of nine children of Bailon and Elisa who cohabited as husband and wife as early as 1958; and
they were reserving their right to file the necessary court action to contest the marriage between Bailon and respondent as they
personally know that Alice is "still very much alive." 16
In the meantime, on April 5, 1999, a certain Hermes P. Diaz, claiming to be the brother and guardian of "Aliz P. Diaz," filed
before the SSS a claim for death benefits accruing from Bailons death, 17 he further attesting in a sworn statement18 that it was
Norma who defrayed Bailons funeral expenses.
Elisa and seven of her children19 subsequently filed claims for death benefits as Bailons beneficiaries before the SSS. 20
Atty. Marites C. de la Torre of the Legal Unit of the SSS Bicol Cluster, Naga City recommended the cancellation of payment of
death pension benefits to respondent and the issuance of an order for the refund of the amount paid to her from February 1998
to May 1999 representing such benefits; the denial of the claim of Alice on the ground that she was not dependent upon Bailon
for support during his lifetime; and the payment of the balance of the five-year guaranteed pension to Bailons beneficiaries
according to the order of preference provided under the law, after the amount erroneously paid to respondent has been
collected. The pertinent portions of the Memorandum read:
1. Aliz [sic] Diaz never disappeared. The court must have been misled by misrepresentation in declaring the first wife, Aliz
[sic] Diaz, as presumptively dead.
xxxx
x x x the Order of the court in the "Petition to Declare Alice Diaz Presumptively Dead," did not become final. The presence
of Aliz [sic] Diaz, is contrary proof that rendered it invalid.
xxxx
3. It was the deceased member who abandoned his wife, Aliz [sic] Diaz. He, being in bad faith, and is the deserting
spouse, his remarriage is void, being bigamous.
xxxx
In this case, it is the deceased member who was the deserting spouse and who remarried, thus his marriage to Teresita Jarque,
for the second time was void as it was bigamous. To require affidavit of reappearance to terminate the second marriage is not
necessary as there is no disappearance of Aliz [sic] Diaz, the first wife, and a voidable marriage [sic], to speak
of.21 (Underscoring supplied)
In the meantime, the SSS Sorsogon Branch, by letter of August 16, 2000, 22 advised respondent that as Cecilia and Norma were
the ones who defrayed Bailons funeral expenses, she should return the P12,000 paid to her.
In a separate letter dated September 7, 1999, 23 the SSS advised respondent of the cancellation of her monthly pension for
death benefits in view of the opinion rendered by its legal department that her marriage with Bailon was void as it was
contracted while the latters marriage with Alice was still subsisting; and the December 10, 1970 CFI Order declaring Alice
presumptively dead did not become final, her "presence" being "contrary proof" against the validity of the order. It thus
requested respondent to return the amount of P24,000 representing the total amount of monthly pension she had received from
the SSS from February 1998 to May 1999.
Respondent protested the cancellation of her monthly pension for death benefits by letter to the SSS dated October 12,
1999.24 In a subsequent letter dated November 27, 1999 25 to the SSC, she reiterated her request for the release of her monthly
pension, asserting that her marriage with Bailon was not declared before any court of justice as bigamous or unlawful, hence, it
remained valid and subsisting for all legal intents and purposes as in fact Bailon designated her as his beneficiary.

The SSS, however, by letter to respondent dated January 21, 2000, 26 maintained the denial of her claim for and the
discontinuance of payment of monthly pension. It advised her, however, that she was not deprived of her right to file a petition
with the SSC.
Respondent thus filed a petition27 against the SSS before the SSC for the restoration to her of her entitlement to monthly
pension.
In the meantime, respondent informed the SSS that she was returning, under protest, the amount of P12,000 representing the
funeral benefits she received, she alleging that Norma and her siblings "forcibly and coercively prevented her from spending any
amount during Bailons wake."28
After the SSS filed its Answer29 to respondents petition, and the parties filed their respective Position Papers, one Alicia P. Diaz
filed an Affidavit30 dated August 14, 2002 with the SSS Naga Branch attesting that she is the widow of Bailon; she had only
recently come to know of the petition filed by Bailon to declare her presumptively dead; it is not true that she disappeared as
Bailon could have easily located her, she having stayed at her parents residence in Barcelona, Sorsogon after she found out
that Bailon was having an extramarital affair; and Bailon used to visit her even after their separation.
By Resolution of April 2, 2003, the SSC found that the marriage of respondent to Bailon was void and, therefore, she was "just a
common-law-wife." Accordingly it disposed as follows, quoted verbatim:
WHEREFORE, this Commission finds, and so holds, that petitioner Teresita Jarque-Bailon is not the legitimate spouse and
primary beneficiary of SSS member Clemente Bailon.
Accordingly, the petitioner is hereby ordered to refund to the SSS the amount of P24,000.00 representing the death benefit she
received therefrom for the period February 1998 until May 1999 as well as P12,000.00 representing the funeral benefit.
The SSS is hereby ordered to pay Alice (a.k.a. Aliz) Diaz-Bailon the appropriate death benefit arising from the demise of SSS
member Clemente Bailon in accordance with Section 8(e) and (k) as well as Section 13 of the SS Law, as amended, and its
prevailing rules and regulations and to inform this Commission of its compliance herewith.
SO ORDERED.31 (Underscoring supplied)
In so ruling against respondent, the SSC ratiocinated.
After a thorough examination of the evidence at hand, this Commission comes to the inevitable conclusion that thepetitioner is
not the legitimate wife of the deceased member.
xxxx
There is x x x ample evidence pointing to the fact that, contrary to the declaration of the then CFI of Sorsogon (10th Judicial
District), the first wife never disappeared as the deceased member represented in bad faith. This Commission accords credence
to the findings of the SSS contained in its Memorandum dated August 9, 1999,32revealing that Alice (a.k.a. Aliz) Diaz never left
Barcelona, Sorsogon, after her separation from Clemente Bailon x x x.
As the declaration of presumptive death was extracted by the deceased member using artifice and by exerting fraud upon the
unsuspecting court of law, x x x it never had the effect of giving the deceased member the right to marry anew. x x x [I]t is clear
that the marriage to the petitioner is void, considering that the first marriage on April 25, 1955 to Alice Diaz was not previously
annulled, invalidated or otherwise dissolved during the lifetime of the parties thereto. x x x as determined through the
investigation conducted by the SSS, Clemente Bailon was the abandoning spouse, not Alice Diaz Bailon.
xxxx
It having been established, by substantial evidence, that the petitioner was just a common-law wife of the deceased member, it
necessarily follows that she is not entitled as a primary beneficiary, to the latters death benefit. x x x
xxxx
It having been determined that Teresita Jarque was not the legitimate surviving spouse and primary beneficiary of Clemente
Bailon, it behooves her to refund the total amount of death benefit she received from the SSS for the period from February 1998
until May 1999 pursuant to the principle of solutio indebiti x x x
Likewise, it appearing that she was not the one who actually defrayed the cost of the wake and burial of Clemente Bailon, she
must return the amount of P12,000.00 which was earlier given to her by the SSS as funeral benefit. 33(Underscoring supplied)
Respondents Motion for Reconsideration34 having been denied by Order of June 4, 2003, she filed a petition for review 35 before
the Court of Appeals (CA).
By Decision of June 23, 2004, the CA reversed and set aside the April 2, 2003 Resolution and June 4, 2003 Order of the SSC
and thus ordered the SSS to pay respondent all the pension benefits due her. Held the CA:
x x x [T]he paramount concern in this case transcends the issue of whether or not the decision of the then CFI, now RTC,
declaring Alice Diaz presumptively dead has attained finality but, more importantly, whether or not the respondents SSS and
Commission can validly re-evaluate the findings of the RTC, and on its own, declare the latters decision to be bereft of any
basis. On similar import, can respondents SSS and Commission validly declare the first marriage subsisting and the second
marriage null and void?
xxxx
x x x while it is true that a judgment declaring a person presumptively dead never attains finality as the finding that "the person
is unheard of in seven years is merely a presumption juris tantum," the second marriage contracted by a person with an absent
spouse endures until annulled. It is only the competent court that can nullify the second marriage pursuant to Article 87 of the
Civil Code and upon the reappearance of the missing spouse, which action for annulment may be filed. Nowhere does the law
contemplates [sic] the possibility that respondent SSS may validly declare the second marriage null and void on the basis alone
of its own investigation and declare that the decision of the RTC declaring one to be presumptively dead is without basis.
Respondent SSS cannot arrogate upon itself the authority to review the decision of the regular courts under the pretext of
determining the actual and lawful beneficiaries of its members. Notwithstanding its opinion as to the soundness of the findings
of the RTC, it should extend due credence to the decision of the RTC absent of [sic] any judicial pronouncement to the contrary.
xxx
x x x [A]ssuming arguendo that respondent SSS actually possesses the authority to declare the decision of the RTC to be
without basis, the procedure it followed was offensive to the principle of fair play and thus its findings are of doubtful quality
considering that petitioner Teresita was not given ample opportunity to present evidence for and her behalf.
xxxx

Respondent SSS is correct in stating that the filing of an Affidavit of Reappearance with the Civil Registry is no longer practical
under the premises. Indeed, there is no more first marriage to restore as the marital bond between Alice Diaz and Clemente
Bailon was already terminated upon the latters death. Neither is there a second marriage to terminate because the second
marriage was likewise dissolved by the death of Clemente Bailon.
However, it is not correct to conclude that simply because the filing of the Affidavit of Reappearance with the Civil Registry
where parties to the subsequent marriage reside is already inutile, the respondent SSS has now the authority to review the
decision of the RTC and consequently declare the second marriage null and void.36(Emphasis and underscoring supplied)
The SSC and the SSS separately filed their Motions for Reconsideration 37 which were both denied for lack of merit.
Hence, the SSS present petition for review on certiorari 38 anchored on the following grounds:
I
THE DECISION OF THE HONORABLE COURT OF APPEALS IS CONTRARY TO LAW.
II
THE HONORABLE COURT OF APPEALS GRAVELY ABUSED ITS DISCRETION AMOUNTING TO LACK OF
JURISDICTION.39
The SSS faults the CA for failing to give due consideration to the findings of facts of the SSC on the prior and subsisting
marriage between Bailon and Alice; in disregarding the authority of the SSC to determine to whom, between Alice and
respondent, the death benefits should be awarded pursuant to Section 5 40 of the Social Security Law; and in declaring that the
SSS did not give respondent due process or ample opportunity to present evidence in her behalf.
The SSS submits that "the observations and findings relative to the CFI proceedings are of no moment to the present
controversy, as the same may be considered only as obiter dicta in view of the SSCs finding of the existence of a prior and
subsisting marriage between Bailon and Alice by virtue of which Alice has a better right to the death benefits." 41
The petition fails.
That the SSC is empowered to settle any dispute with respect to SSS coverage, benefits and contributions, there is no doubt. In
so exercising such power, however, it cannot review, much less reverse, decisions rendered by courts of law as it did in the
case at bar when it declared that the December 10, 1970 CFI Order was obtained through fraud and subsequently disregarded
the same, making its own findings with respect to the validity of Bailon and Alices marriage on the one hand and the invalidity of
Bailon and respondents marriage on the other.
In interfering with and passing upon the CFI Order, the SSC virtually acted as an appellate court. The law does not give the
SSC unfettered discretion to trifle with orders of regular courts in the exercise of its authority to determine the beneficiaries of
the SSS.
The two marriages involved herein having been solemnized prior to the effectivity on August 3, 1988 of the Family Code, the
applicable law to determine their validity is the Civil Code which was the law in effect at the time of their celebration. 42
Article 83 of the Civil Code43 provides:
Art. 83. Any marriage subsequently contracted by any person during the lifetime of the first spouse of such person with any
person other than such first spouse shall be illegal and void from its performance, unless:
(1) The first marriage was annulled or dissolved; or
(2) The first spouse had been absent for seven consecutive years at the time of the second marriage without the spouse
present having news of the absentee being alive, or if the absentee, though he has been absent for less than seven years,
is generally considered as dead and believed to be so by the spouse present at the time of contracting such subsequent
marriage, or if the absentee is presumed dead according to Articles 390 and 391. The marriage so contracted shall be
valid in any of the three cases until declared null and void by a competent court. (Emphasis and underscoring supplied)
Under the foregoing provision of the Civil Code, a subsequent marriage contracted during the lifetime of the first spouse is
illegal and void ab initio unless the prior marriage is first annulled or dissolved or contracted under any of the three exceptional
circumstances. It bears noting that the marriage under any of these exceptional cases is deemed valid "until declared null and
void by a competent court." It follows that the onus probandi in these cases rests on the party assailing the second marriage. 44
In the case at bar, as found by the CFI, Alice had been absent for 15 consecutive years 45 when Bailon sought the declaration of
her presumptive death, which judicial declaration was not even a requirement then for purposes of remarriage. 46
Eminent jurist Arturo M. Tolentino (now deceased) commented:
Where a person has entered into two successive marriages, a presumption arises in favor of the validity of the second marriage,
and the burden is on the party attacking the validity of the second marriage to prove that the first marriage had not been
dissolved; it is not enough to prove the first marriage, for it must also be shown that it had not ended when the second marriage
was contracted. The presumption in favor of the innocence of the defendant from crime or wrong and of the legality of his
second marriage, will prevail over the presumption of the continuance of life of the first spouse or of the continuance of the
marital relation with such first spouse.47(Underscoring supplied)
Under the Civil Code, a subsequent marriage being voidable,48 it is terminated by final judgment of annulment in a case
instituted by the absent spouse who reappears or by either of the spouses in the subsequent marriage.
Under the Family Code, no judicial proceeding to annul a subsequent marriage is necessary. Thus Article 42 thereof provides:
Art. 42. The subsequent marriage referred to in the preceding Article shall be automatically terminated by therecording of the
affidavit of reappearance of the absent spouse, unless there is a judgment annulling the previous marriage or declaring it
void ab initio.
A sworn statement of the fact and circumstances of reappearance shall be recorded in the civil registry of the residence of the
parties to the subsequent marriage at the instance of any interested person, with due notice to the spouses of the
subsequent marriage and without prejudice to the fact of reappearance being judicially determined in case such fact is
disputed. (Emphasis and underscoring supplied)
The termination of the subsequent marriage by affidavit provided by the above-quoted provision of the Family Code does not
preclude the filing of an action in court to prove the reappearance of the absentee and obtain a declaration of dissolution or
termination of the subsequent marriage.49
If the absentee reappears, but no step is taken to terminate the subsequent marriage, either by affidavit or by court action,
such absentees mere reappearance, even if made known to the spouses in the subsequent marriage, will not terminate such
marriage.50 Since the second marriage has been contracted because of a presumption that the former spouse is dead, such

presumption continues inspite of the spouses physical reappearance, and by fiction of law, he or she must still be regarded as
legally an absentee until the subsequent marriage is terminated as provided by law.51
If the subsequent marriage is not terminated by registration of an affidavit of reappearance or by judicial declaration but
by death of either spouse as in the case at bar, Tolentino submits:
x x x [G]enerally if a subsequent marriage is dissolved by the death of either spouse, the effects of dissolution of valid marriages
shall arise. The good or bad faith of either spouse can no longer be raised, because, as in annullable or voidable marriages, the
marriage cannot be questioned except in a direct action for annulment.52(Underscoring supplied)
Similarly, Lapuz v. Eufemio53 instructs:
In fact, even if the bigamous marriage had not been void ab initio but only voidable under Article 83, paragraph 2, of the Civil
Code, because the second marriage had been contracted with the first wife having been an absentee for seven consecutive
years, or when she had been generally believed dead, still the action for annulment became extinguished as soon as one of the
three persons involved had died, as provided in Article 87, paragraph 2, of the Code, requiring that the action for annulment
should be brought during the lifetime of any one of the parties involved. And furthermore, the liquidation of any conjugal
partnership that might have resulted from such voidable marriage must be carried out "in the testate or intestate proceedings of
the deceased spouse," as expressly provided in Section 2 of the Revised Rule 73, and not in the annulment
proceeding.54 (Emphasis and underscoring supplied)
It bears reiterating that a voidable marriage cannot be assailed collaterally except in a direct proceeding. Consequently, such
marriages can be assailed only during the lifetime of the parties and not after the death of either, in which case the parties and
their offspring will be left as if the marriage had been perfectly valid. 55 Upon the death of either, the marriage cannot be
impeached, and is made good ab initio.56
In the case at bar, as no step was taken to nullify, in accordance with law, Bailons and respondents marriage prior to the
formers death in 1998, respondent is rightfully the dependent spouse-beneficiary of Bailon.
In light of the foregoing discussions, consideration of the other issues raised has been rendered unnecessary.
WHEREFORE, the petition is DENIED..

ANGELITA VALDEZ,
Petitioner,

G.R. No. 180863


Present:

- versus -

YNARES-SANTIAGO, J.,
Chairperson,
CHICO-NAZARIO,
VELASCO, JR.,
NACHURA, and
PERALTA, JJ.
Promulgated:

REPUBLIC OF THE PHILIPPINES,


Respondent.

September 8, 2009

x------------------------------------------------------------------------------------x
Before this Court is a Petition for Review on Certiorari under Rule 45 of the Rules of Court assailing the Decision of the
Regional Trial Court (RTC) of Camiling, Tarlac dated November 12, 2007 dismissing petitioner Angelita Valdezs petition for the
declaration of presumptive death of her husband, Sofio Polborosa (Sofio).
The facts of the case are as follows:
Petitioner married Sofio on January 11, 1971 in Pateros, Rizal. On December 13, 1971, petitioner gave birth to the
spouses only child, Nancy. According to petitioner, she and Sofio argued constantly because the latter was unemployed and did
not bring home any money. In March 1972, Sofio left their conjugal dwelling. Petitioner and their child waited for him to return
but, finally, in May 1972, petitioner decided to go back to her parents home in Bancay 1 st, Camiling, Tarlac. Three years passed
without any word from Sofio. In October 1975, Sofio showed up at Bancay 1 st. He and petitioner talked for several hours and
they agreed to separate. They executed a document to that effect. [1] That was the last time petitioner saw him. After that,
petitioner didnt hear any news of Sofio, his whereabouts or even if he was alive or not. [2]
Believing that Sofio was already dead, petitioner married Virgilio Reyes on June 20, 1985. [3] Subsequently, however,
Virgilios application for naturalization filed with the United States Department of Homeland Security was denied because
petitioners marriage to Sofio was subsisting. [4] Hence, on March 29, 2007, petitioner filed a Petition before the RTC of Camiling,
Tarlac seeking the declaration of presumptive death of Sofio.
The RTC rendered its Decision[5] on November 12, 2007, dismissing the Petition for lack of merit. The RTC held that
Angelita was not able to prove the well-grounded belief that her husband Sofio Polborosa was already dead. It said that under
Article 41 of the Family Code, the present spouse is burdened to prove that her spouse has been absent and that she has a
well-founded belief that the absent spouse is already dead before the present spouse may contract a subsequent marriage.
This belief, the RTC said, must be the result of proper and honest-to-goodness inquiries and efforts to ascertain the
whereabouts of the absent spouse.
The RTC found that, by petitioners own admission, she did not try to find her husband anymore in light of their mutual
agreement to live separately. Likewise, petitioners daughter testified that her mother prevented her from looking for her father.
The RTC also said there is a strong possibility that Sofio is still alive, considering that he would have been only 61 years old by
then, and people who have reached their 60s have not become increasingly low in health and spirits, and, even assuming as
true petitioners testimony that Sofio was a chain smoker and a drunkard, there is no evidence that he continues to drink and
smoke until now.
Petitioner filed a motion for reconsideration. [6] She argued that it is the Civil Code that applies in this case and not the
Family Code since petitioners marriage to Sofio was celebrated on January 11, 1971, long before the Family Code took effect.
Petitioner further argued that she had acquired a vested right under the provisions of the Civil Code and the stricter provisions
of the Family Code should not be applied against her because Title XIV of the Civil Code, where Articles 384 and 390 on
declaration of absence and presumption of death, respectively, can be found, was not expressly repealed by the Family Code.
To apply the stricter provisions of the Family Code will impair the rights petitioner had acquired under the Civil Code.
The RTC denied the Motion for Reconsideration in a Resolution dated December 10, 2007. [7]
Petitioner now comes before this Court seeking the reversal of the RTC Decision and Motion for Reconsideration.
In its Manifestation and Motion, [8] the Office of the Solicitor General (OSG) recommended that the Court set aside the
assailed RTC Decision and grant the Petition to declare Sofio presumptively dead. The OSG argues that the requirement of
well-founded belief under Article 41 of the Family Code is not applicable to the instant case. It said that petitioner could not be
expected to comply with this requirement because it was not yet in existence during her marriage to Virgilio Reyes in 1985. The
OSG further argues that before the effectivity of the Family Code, petitioner already acquired a vested right as to the validity of
her marriage to Virgilio Reyes based on the presumed death of Sofio under the Civil Code. This vested right and the
presumption of Sofios death, the OSG posits, could not be affected by the obligations created under the Family Code. [9]

Next, the OSG contends that Article 390 of the Civil Code was not repealed by Article 41 of the Family Code. [10] Title XIV of
the Civil Code, the OSG said, was not one of those expressly repealed by the Family Code. Moreover, Article 256 of the Family
Code provides that its provisions shall not be retroactively applied if they will prejudice or impair vested or acquired rights. [11]
The RTC Decision, insofar as it dismissed the Petition, is affirmed. However, we must state that we are denying the
Petition on grounds different from those cited in the RTC Decision.
Initially, we discuss a procedural issue. Under the Rules of Court, a party may directly appeal to this Court from a decision
of the trial court only on pure questions of law. A question of law lies, on one hand, when the doubt or difference arises as to
what the law is on a certain set of facts; on the other hand, a question of fact exists when the doubt or difference arises as to the
truth or falsehood of the alleged facts. Here, the facts are not disputed; the controversy merely relates to the correct application
of the law or jurisprudence to the undisputed facts. [12]
The RTC erred in applying the provisions of the Family Code and holding that petitioner needed to prove a well-founded
belief that Sofio was already dead. The RTC applied Article 41 of the Family Code, to wit:
Art. 41. A marriage contracted by any person during subsistence of a previous marriage shall be null and
void, unless before the celebration of the subsequent marriage, the prior spouse had been absent for four
consecutive years and the spouse present has a well-founded belief that the absent spouse was already dead. In
case of disappearance where there is danger under the circumstances set forth in the provisions of Article 391 of the
Civil Code, an absence of only two years shall be sufficient.
For the purpose of contracting a subsequent marriage under the preceding paragraph, the spouse present
must institute a summary proceeding as provided in this Code for the declaration of presumptive death of the
absentee, without prejudice to the effect of reappearance of the absent spouse.

It is readily apparent, however, that the marriages of petitioner to Sofio and Virgilio on January 11, 1971 and June 20,
1985, respectively, were both celebrated under the auspices of the Civil Code.

The pertinent provision of the Civil Code is Article 83:


Art. 83. Any marriage subsequently contracted by any person during the lifetime of the first spouse of such
person with any person other than such first spouse shall be illegal and void from its performance, unless:
(1) The first marriage was annulled or dissolved; or
(2) The first spouse had been absent for seven consecutive years at the time of the second marriage without
the spouse present having news of the absentee being alive, of if the absentee, though he has been absent for less
than seven years, is generally considered as dead and believed to be so by the spouse present at the time of
contracting such subsequent marriage, or if the absentee is presumed dead according to Articles 390 and 391. The
marriage so contracted shall be valid in any of the three cases until declared null and void by a competent court.

Article 390 of the Civil Code states:


Art. 390. After an absence of seven years, it being unknown whether or not the absentee still lives, he shall be
presumed dead for all purposes, except for those of succession.
The absentee shall not be presumed dead for the purpose of opening his succession till after an absence of
ten years. If he disappeared after the age of seventy-five years, an absence of five years shall be sufficient in order
that his succession may be opened.

The Court, on several occasions, had interpreted the above-quoted provision in this wise:
For the purposes of the civil marriage law, it is not necessary to have the former spouse judicially declared an
absentee. The declaration of absence made in accordance with the provisions of the Civil Code has for its sole
purpose to enable the taking of the necessary precautions for the administration of the estate of the absentee. For
the celebration of civil marriage, however, the law only requires that the former spouse has been absent for seven
consecutive years at the time of the second marriage, that the spouse present does not know his or her former
spouse to be living, that such former spouse is generally reputed to be dead and the spouse present so believes at
the time of the celebration of the marriage. [13]

Further, the Court explained that presumption of death cannot be the subject of court proceedings independent of the settlement
of the absentees estate.
In re Szatraw[14] is instructive. In that case, petitioner contracted marriage with a Polish national in 1937. They lived
together as husband and wife for three years. Sometime in 1940, the husband, on the pretext of visiting some friends, left the
conjugal abode with their child and never returned. After inquiring from friends, petitioner found that her husband went
to Shanghai, China. However, friends who came from Shanghai told her that the husband was not seen there. In 1948,
petitioner filed a petition for the declaration of presumptive death of her husband arguing that since the latter had been absent
for more than seven years and she had not heard any news from him and about her child, she believes that he is dead. In
deciding the case, the Court said:
The petition is not for the settlement of the estate of Nicolai Szatraw, because it does not appear that he
possessed property brought to the marriage and because he had acquired no property during his married life with
the petitioner. The rule invoked by the latter is merely one of evidence which permits the court to presume that a
person is dead after the fact that such person had been unheard from in seven years had been established. This
presumption may arise and be invoked and made in a case, either in an action or in a special proceeding, which is
tried or heard by, and submitted for decision to, a competent court. Independently of such an action or special
proceeding, the presumption of death cannot be invoked, nor can it be made the subject of an action or
special proceeding. In this case, there is no right to be enforced nor is there a remedy prayed for by the petitioner
against her absent husband. Neither is there a prayer for the final determination of his right or status or for the
ascertainment of a particular fact (Hagans v. Wislizenus, 42 Phil. 880), for the petition does not pray for a declaration
that the petitioner's husband is dead, but merely asks for a declaration that he be presumed dead because he had
been unheard from in seven years. If there is any pretense at securing a declaration that the petitioner's husband is
dead, such a pretension cannot be granted because it is unauthorized. The petition is for a declaration that the
petitioner's husband is presumptively dead. But this declaration, even if judicially made, would not improve the
petitioner's situation, because such a presumption is already established by law. A judicial pronouncement to that
effect, even if final and executory, would still be a prima facie presumption only. It is still disputable. It is for
that reason that it cannot be the subject of a judicial pronouncement or declaration, if it is the only question
or matter involved in a case, or upon which a competent court has to pass. The latter must decide finally the
controversy between the parties, or determine finally the right or status of a party or establish finally a particular fact,
out of which certain rights and obligations arise or may arise; and once such controversy is decided by a final
judgment, or such right or status determined, or such particular fact established, by a final decree, then the judgment
on the subject of the controversy, or the decree upon the right or status of a party or upon the existence of a
particular fact, becomes res judicata, subject to no collateral attack, except in a few rare instances especially
provided by law. It is, therefore, clear that a judicial declaration that a person is presumptively dead, because
he had been unheard from in seven years, being a presumption juris tantum only, subject to contrary proof,
cannot reach the stage of finality or become final. Proof of actual death of the person presumed dead because
he had been unheard from in seven years, would have to be made in another proceeding to have such particular
fact finally determined. If a judicial decree declaring a person presumptively dead, because he had not been heard
from in seven years, cannot become final and executory even after the lapse of the reglementary period within which
an appeal may be taken, for such presumption is still disputable and remains subject to contrary proof, then a
petition for such a declaration is useless, unnecessary, superfluous and of no benefit to the petitioner. [15]

In Lukban v. Republic,[16] petitioner Lourdes G. Lukban contracted marriage with Francisco Chuidian on December 10,
1933. A few days later, on December 27, Francisco left Lourdes after a violent quarrel. She did not hear from him after that
day. Her diligent search, inquiries from his parents and friends, and search in his last known address, proved futile. Believing
her husband was already dead since he had been absent for more than twenty years, petitioner filed a petition in 1956 for a

declaration that she is a widow of her husband who is presumed to be dead and has no legal impediment to contract a
subsequent marriage. On the other hand, the antecedents in Gue v. Republic[17] are similar to Szatraw. On January 5, 1946,
Angelina Gues husband left Manila where they were residing and went to Shanghai, China. From that day on, he had not been
heard of, had not written to her, nor in anyway communicated with her as to his whereabouts. Despite her efforts and diligence,
she failed to locate him. After 11 years, she asked the court for a declaration of the presumption of death of Willian Gue,
pursuant to the provisions of Article 390 of the Civil Code of the Philippines.
In both cases, the Court reiterated its ruling in Szatraw. It held that a petition for judicial declaration that petitioner's
husband is presumed to be dead cannot be entertained because it is not authorized by law. [18]
From the foregoing, it can be gleaned that, under the Civil Code, the presumption of death is established by law [19] and no
court declaration is needed for the presumption to arise. Since death is presumed to have taken place by the seventh year of
absence,[20] Sofio is to be presumed dead starting October 1982.
Consequently, at the time of petitioners marriage to Virgilio, there existed no impediment to petitioners capacity to marry,
and the marriage is valid under paragraph 2 of Article 83 of the Civil Code.
Further, considering that it is the Civil Code that applies, proof of well-founded belief is not required. Petitioner could not
have been expected to comply with this requirement since the Family Code was not yet in effect at the time of her marriage to
Virgilio. The enactment of the Family Code in 1988 does not change this conclusion. The Family Code itself states:
Art. 256. This Code shall have retroactive effect insofar as it does not prejudice or impair vested or acquired
rights in accordance with the Civil Code or other laws.

To retroactively apply the provisions of the Family Code requiring petitioner to exhibit well-founded belief will, ultimately,
result in the invalidation of her second marriage, which was valid at the time it was celebrated. Such a situation would be
untenable and would go against the objectives that the Family Code wishes to achieve.
In sum, we hold that the Petition must be dismissed since no decree on the presumption of Sofios death can be granted
under the Civil Code, the same presumption having arisen by operation of law. However, we declare that petitioner was
capacitated to marry Virgilio at the time their marriage was celebrated in 1985 and, therefore, the said marriage is legal and
valid.
WHEREFORE, the foregoing premises considered, the Petition is DENIED.

G.R. No. L-27930 November 26, 1970

AURORA A. ANAYA, plaintiff-appellant,


vs.
FERNANDO O. PALAROAN, defendant-appellee.
Isabelo V. Castro for plaintiff-appellant.
Arturo A. Romero for defendant-appellee.

REYES, J.B.L., J.:


Appeal from an order of dismissal, issued motu proprio by the Juvenile & Domestic Relations Court, Manila, of a complaint for
annulment of marriage, docketed therein as Civil Case No. E-00431, entitled "Aurora A. Anaya, plaintiff vs. Fernando O.
Palaroan, defendant."
The complaint in said Civil Case No. E-00431 alleged, inter alia, that plaintiff Aurora and defendant Fernando were married on 4
December 1953; that defendant Fernando filed an action for annulment of the marriage on 7 January 1954 on the ground that
his consent was obtained through force and intimidation, which action was docketed in the Court of First Instance of Manila as
Civil Case No. 21589; that judgment was rendered therein on 23 September 1959 dismissing the complaint of Fernando,
upholding the validity of the marriage and granting Aurora's counterclaim; that (per paragraph IV) while the amount of the
counterclaim was being negotiated "to settle the judgment," Fernando had divulged to Aurora that several months prior to their
marriage he had pre-marital relationship with a close relative of his; and that "the non-divulgement to her of the aforementioned
pre-marital secret on the part of defendant that definitely wrecked their marriage, which apparently doomed to fail even before it
had hardly commenced ... frank disclosure of which, certitude precisely precluded her, the Plaintiff herein from going thru the
marriage that was solemnized between them constituted 'FRAUD', in obtaining her consent, within the contemplation of No. 4 of
Article 85 of the Civil Code" (sic) (Record on Appeal, page 3). She prayed for the annulment of the marriage and for moral
damages.
Defendant Fernando, in his answer, denied the allegation in paragraph IV of the complaint and denied having had pre-marital
relationship with a close relative; he averred that under no circumstance would he live with Aurora, as he had escaped from her
and from her relatives the day following their marriage on 4 December 1953; that he denied having committed any fraud against
her. He set up the defenses of lack of cause of action and estoppel, for her having prayed in Civil Case No. 21589 for the
validity of the marriage and her having enjoyed the support that had been granted her. He counterclaimed for damages for the
malicious filing of the suit. Defendant Fernando did not pray for the dismissal of the complaint but for its dismissal "with respect
to the alleged moral damages."
Plaintiff Aurora filed a reply with answer to the counterclaim, wherein she alleged:
(1) that prior to their marriage on 4 December 1953, he paid court to her, and pretended to shower her with love and
affection not because he really felt so but because she merely happened to be the first girl available to marry so he
could evade marrying the close relative of his whose immediate members of her family were threatening him to force
him to marry her (the close relative);
(2) that since he contracted the marriage for the reason intimated by him, and not because he loved her, he secretly
intended from the very beginning not to perform the marital duties and obligations appurtenant thereto, and
furthermore, he covertly made up his mind not to live with her;
(3) that the foregoing clandestine intentions intimated by him were prematurely concretized for him, when in order to
placate and appease the immediate members of the family of the first girl (referent being the close relative) and to
convince them of his intention not to live with plaintiff, carried on a courtship with a third girl with whom, after gaining
the latter's love cohabited and had several children during the whole range of nine years that Civil Case No. 21589,
had been litigated between them (parties); (Record on Appeal, pages 10-11)
Failing in its attempt to have the parties reconciled, the court set the case for trial on 26 August 1966 but it was postponed.
Thereafter, while reviewing the expendiente, the court realized that Aurora's allegation of the fraud was legally insufficient to
invalidate her marriage, and, on the authority of Brown vs. Yambao, 102 Phil. 168, holding:
It is true that the wife has not interposed prescription as a defense. Nevertheless, the courts can take cognizance
thereof, because actions seeking a decree of legal separation, or annulment of marriage, involve public interest, and
it is the policy of our law that no such decree be issued if any legal obstacles thereto appear upon the record.
the court a quo required plaintiff to show cause why her complaint should not be dismissed. Plaintiff Aurora
submitted a memorandum in compliance therewith, but the court found it inadequate and thereby issued an order,
dated 7 October 1966, for the dismissal of the complaint; it also denied reconsideration.
The main issue is whether or not the non-disclosure to a wife by her husband of his pre-marital relationship with another woman
is a ground for annulment of marriage.
We must agree with the lower court that it is not. For fraud as a vice of consent in marriage, which may be a cause for its
annulment, comes under Article 85, No. 4, of the Civil Code, which provides:
ART. 85. A marriage may be annulled for any of the following causes, existing at the time of the marriage:
xxx xxx xxx
(4) That the consent of either party was obtained by fraud, unless such party afterwards, with full knowledge of the
facts constituting the fraud, freely cohabited with the other as her husband or his wife, as the case may be;
This fraud, as vice of consent, is limited exclusively by law to those kinds or species of fraud enumerated in Article
86, as follows:
ART. 86. Any of the following circumstances shall constitute fraud referred to in number 4 of the preceding article:
(1) Misrepresentation as to the identity of one of the contracting parties;
(2) Non-disclosure of the previous conviction of the other party of a crime involving moral turpitude, and
the penalty imposed was imprisonment for two years or more;
(3) Concealment by the wife of the fact that at the time of the marriage, she was pregnant by a man
other than her husband.
No other misrepresentation or deceit as to character, rank, fortune or chastity shall constitute such fraud as will give
grounds for action for the annulment of marriage.
The intention of Congress to confine the circumstances that can constitute fraud as ground for annulment of marriage to the
foregoing three cases may be deduced from the fact that, of all the causes of nullity enumerated in Article 85, fraud is the only

one given special treatment in a subsequent article within the chapter on void and voidable marriages. If its intention were
otherwise, Congress would have stopped at Article 85, for, anyway, fraud in general is already mentioned therein as a cause for
annulment. But Article 86 was also enacted, expressly and specifically dealing with "fraud referred to in number 4 of the
preceding article," and proceeds by enumerating the specific frauds (misrepresentation as to identity, non-disclosure of a
previous conviction, and concealment of pregnancy), making it clear that Congress intended to exclude all other frauds or
deceits. To stress further such intention, the enumeration of the specific frauds was followed by the interdiction: "No other
misrepresentation or deceit as to character, rank, fortune or chastity shall constitute such fraud as will give grounds for action for
the annulment of marriage."
Non-disclosure of a husband's pre-marital relationship with another woman is not one of the enumerated circumstances that
would constitute a ground for annulment; and it is further excluded by the last paragraph of the article, providing that "no other
misrepresentation or deceit as to ... chastity" shall give ground for an action to annul a marriage. While a woman may detest
such non-disclosure of premarital lewdness or feel having been thereby cheated into giving her consent to the marriage,
nevertheless the law does not assuage her grief after her consent was solemnly given, for upon marriage she entered into an
institution in which society, and not herself alone, is interested. The lawmaker's intent being plain, the Court's duty is to give
effect to the same, whether it agrees with the rule or not.
But plaintiff-appellant Anaya emphasizes that not only has she alleged "non-divulgement" (the word chosen by her) of the premarital relationship of her husband with another woman as her cause of action, but that she has, likewise, alleged in her reply
that defendant Fernando paid court to her without any intention of complying with his marital duties and obligations and covertly
made up his mind not to live with her. Plaintiff-appellant contends that the lower court erred in ignoring these allegations in her
reply.
This second set of averments which were made in the reply (pretended love and absence of intention to perform duties of
consortium) is an entirely new and additional "cause of action." According to the plaintiff herself, the second set of allegations is
"apart, distinct and separate from that earlier averred in the Complaint ..." (Record on Appeal, page 76). Said allegations were,
therefore, improperly alleged in the reply, because if in a reply a party-plaintiff is not permitted to amend or change the cause of
action as set forth in his complaint (Calo vs. Roldan, 76 Phil. 445), there is more reason not to allow such party to allege a new
and additional cause of action in the reply. Otherwise, the series of pleadings of the parties could become interminable.
On the merits of this second fraud charge, it is enough to point out that any secret intention on the husband's part not to perform
his marital duties must have been discovered by the wife soon after the marriage: hence her action for annulment based on that
fraud should have been brought within four years after the marriage. Since appellant's wedding was celebrated in December of
1953, and this ground was only pleaded in 1966, it must be declared already barred.
FOR THE FOREGOING REASONS, the appealed order is hereby affirmed. No costs.

G.R. No. L-15853

July 27, 1960

FERNANDO AQUINO, petitioner,


vs.
CONCHITA DELIZO, respondent.
GUTIERREZ DAVID, J.:
This is a petition for certiorari to review a decision of the Court of Appeals affirming that of the Court of First Instance of Rizal
which dismissed petitioner's complaint for annulment of his marriage with respondent Conchita Delizo.
The dismissed complaint, which was filed on September 6, 1955, was based on the ground of fraud, it being alleged, among
other things, that defendant Conchita Delizo, herein respondent, at the date of her marriage to plaintiff, herein petitioner
Fernando Aquino, on December 27, 1954, concealed from the latter that fact that she was pregnant by another man, and
sometime in April, 1955, or about four months after their marriage, gave birth to a child. In her answer, defendant claimed that
the child was conceived out of lawful wedlock between her and the plaintiff.
At the trial, the attorney's for both parties appeared and the court a quo ordered Assistant Provincial Fiscal Jose Goco to
represent the State in the proceedings to prevent collusion. Only the plaintiff however, testified and the only documentary
evidence presented was the marriage contract between the parties. Defendant neither appeared nor presented any evidence
despite the reservation made by her counsel that he would present evidence on a later date.
On June 16, 1956, the trial court noting that no birth certificate was presented to show that the child was born within 180
days after the marriage between the parties, and holding that concealment of pregnancy as alleged by the plaintiff does not
constitute such fraud sa would annul a marriage dismissed the complaint. Through a verified "petition to reopen for reception
of additional evidence", plaintiff tried to present the certificates of birth and delivery of the child born of the defendant on April 26,
1955, which documents, according to him, he had failed to secure earlier and produce before the trial court thru excusable
negligence. The petition, however, was denied.
On appeal to the Court of Appeals, that court held that there has been excusable neglect in plaintiff's inability to present the
proof of the child's birth, through her birth certificate, and for that reason the court a quo erred in denying the motion for
reception of additional evidence. On the theory, however, that it was not impossible for plaintiff and defendant to have had
sexual intercourse during their engagement so that the child could be their own, and finding unbelievable plaintiff's claim that he
did not notice or even suspect that defendant was pregnant when he married her, the appellate court, nevertheless, affirmed the
dismissal of the complaint.
On March 17, 1959, plaintiff filed a motion praying that the decision be reconsidered, or, if such reconsideration be denied, that
the case be remanded to the lower court for new trial. In support of the motion, plaintiff attached as annexes thereof the
following documents:
1. Affidavit of Cesar Aquino (Annex A) (defendant's brother-in-law and plaintiff's brother, with whom defendant was living at
the time plaintiff met, courted and married her, and with whom defendant has begotten two more children, aside from her
first born, in common-law relationship) admitting that he is the father of defendant's first born, Catherine Bess Aquino, and
that he and defendant hid her pregnancy from plaintiff at the time of plaintiff's marriage to defendant;
2. Affidavit of defendant, Conchita Delizo (Annex "B") admitting her pregnancy by Cesar Aquino, her brother-in-law and
plaintiff's own brother, at the time of her marriage to plaintiff and her having hidden this fact from plaintiff before and up to
the time of their marriage;
3. Affidavit of Albert Powell (Annex "C") stating that he knew Cesar Aquino and defendant lived together as husband and
wife before December 27, 1954, the date of plaintiff's marriage to defendant;
4. Birth Certificate of defendant's first born, Catherine Bess Aquino showing her date of birth to be April 26, 1955;
5. Birth Certificate (Annex "D") of Carolle Ann Aquino, the second child of defendant with Cesar Aquino, her brother-in-law;
6. Birth Certificate (Annex "E") of Chris Charibel Aquino, the third child of Cesar Aquino and defendant; and
7. Pictures of defendant showing her natural plumpness as early as 1952 to as late as November, 1954, the November,
1954 photo itself does not show defendant's pregnancy which must have been almost four months old at the time the
picture was taken.
Acting upon the motion, the Court of Appeals ordered the defendant Conchita Delizo and Assistant Provincial Fiscal of Rizal,
who was representing the Government, to answer the motion for reconsideration, and deferred action on the prayer for new trial
until after the case is disposed of. As both the defendant and the fiscal failed to file an answer, and stating that it "does not
believe the veracity of the contents of the motion and its annexes", the Court of Appeals, on August 6, 1959, denied the motion.
From that order, the plaintiff brought the case to this Court thru the present petition for certiorari.
After going over the record of the case, we find that the dismissal of plaintiff's complaint cannot be sustained.
Under the new Civil Code, concealment by the wife of the fact that at the time of the marriage, she was pregnant by a man other
than her husband constitutes fraud and is ground for annulment of marriage. (Art. 85, par. (4) in relation to Art. 86, par. (3). In
the case of Buccat vs. Buccat (72 Phil., 19) cited in the decision sought to be reviewed, which was also an action for the
annulment of marriage on the ground of fraud, plaintiff's claim that he did not even suspect the pregnancy of the defendant was
held to be unbelievable, it having been proven that the latter was already in an advanced stage of pregnancy (7th month) at the
time of their marriage. That pronouncement, however, cannot apply to the case at bar. Here the defendant wife was alleged to
be only more than four months pregnant at the time of her marriage to plaintiff. At that stage, we are not prepared to say that her
pregnancy was readily apparent, especially since she was "naturally plump" or fat as alleged by plaintiff. According to medical
authorities, even on the 5th month of pregnancy, the enlargement of a woman's abdomen is still below the umbilicus, that is to
say, the enlargement is limited to the lower part of the abdomen so that it is hardly noticeable and may, if noticed, be attributed
only to fat formation on the lower part of the abdomen. It is only on the 6th month of pregnancy that the enlargement of the
woman's abdomen reaches a height above the umbilicus, making the roundness of the abdomen more general and apparent.
(See Lull, Clinical Obstetrics, p. 122) If, as claimed by plaintiff, defendant is "naturally plump", he could hardly be expected to
know, merely by looking, whether or not she was pregnant at the time of their marriage more so because she must have
attempted to conceal the true state of affairs. Even physicians and surgeons, with the aid of the woman herself who shows and
gives her subjective and objective symptoms, can only claim positive diagnosis of pregnancy in 33% at five months. and 50% at
six months. (XI Cyclopedia of Medicine, Surgery, etc. Pregnancy, p. 10).
The appellate court also said that it was not impossible for plaintiff and defendant to have had sexual intercourse before they
got married and therefore the child could be their own. This statement, however, is purely conjectural and finds no support or
justification in the record.
Upon the other hand, the evidence sought to be introduced at the new trial, taken together with what has already been adduced
would, in our opinion, be sufficient to sustain the fraud alleged by plaintiff. The Court of Appeals should, therefore, not have
denied the motion praying for new trial simply because defendant failed to file her answer thereto. Such failure of the defendant
cannot be taken as evidence of collusion, especially since a provincial fiscal has been ordered of represent the Government
precisely to prevent such collusion. As to the veracity of the contents of the motion and its annexes, the same can best be

determined only after hearing evidence. In the circumstance, we think that justice would be better served if a new trial were
ordered.
Wherefore, the decision complained of is set aside and the case remanded to the court a quo for new trial. Without costs.

G.R. No. L-12790

August 31, 1960

JOEL JIMENEZ, plaintiff-appellee,


vs.
REMEDIOS CAIZARES, defendant.
Republic of the Philippines, intervenor-appellant.
Acting Solicitor General Guillermo E. Torres and Solicitor Pacifico P. de Castro for appellant.
Climaco, Ascarraga and Silang for appellee.
PADILLA, J.:
In a complaint filed on 7 June 1955 in the Court of First Instance of Zamboanga the plaintiff Joel Jimenez prays for a decree
annulling his marriage to the defendant Remedios Caizares contracted on 3 August 1950 before a judge of the municipal court
of Zamboanga City, upon the ground that the office of her genitals or vagina was to small to allow the penetration of a male
organ or penis for copulation; that the condition of her genitals as described above existed at the time of marriage and continues
to exist; and that for that reason he left the conjugal home two nights and one day after they had been married. On 14 June
1955 the wife was summoned and served a copy of the complaint. She did not file an answer. On 29 September 1956, pursuant
to the provisions of article 88 of the Civil Code, the Court directed the city attorney of Zamboanga to inquire whether there was a
collusion, to intervene for the State to see that the evidence for the plaintiff is not a frame-up, concocted or fabricated. On 17
December 1956 the Court entered an order requiring the defendant to submit to a physical examination by a competent lady
physician to determine her physical capacity for copulation and to submit, within ten days from receipt of the order, a medical
certificate on the result thereof. On 14 March 1957 the defendant was granted additional five days from notice to comply with
the order of 17 December 1956 with warning that her failure to undergo medical examination and submit the required doctor's
certificate would be deemed lack of interest on her part in the case and that judgment upon the evidence presented by her
husband would be rendered.
After hearing, at which the defendant was not present, on 11 April 1957 the Court entered a decree annulling the marriage
between the plaintiff and the defendant. On 26 April 1957 the city attorney filed a motion for reconsideration of the decree thus
entered, upon the ground, among others, that the defendant's impotency has not been satisfactorily established as required by
law; that she had not been physically examined because she had refused to be examined; that instead of annulling the marriage
the Court should have punished her for contempt of court and compelled her to undergo a physical examination and submit a
medical certificate; and that the decree sought to be reconsidered would open the door to married couples, who want to end
their marriage to collude or connive with each other by just alleging impotency of one of them. He prayed that the complaint be
dismissed or that the wife be subjected to a physical examination. Pending resolution of his motion, the city attorney timely
appealed from the decree. On 13 May 1957 the motion for reconsideration was denied.
The question to determine is whether the marriage in question may be annulled on the strength only of the lone testimony of the
husband who claimed and testified that his wife was and is impotent. The latter did not answer the complaint, was absent during
the hearing, and refused to submit to a medical examination.
Marriage in this country is an institution in which the community is deeply interested. The state has surrounded it with
safeguards to maintain its purity, continuity and permanence. The security and stability of the state are largely dependent upon
it. It is the interest of each and every member of the community to prevent the bringing about of a condition that would shake its
foundation and ultimately lead to its destruction. The incidents of the status are governed by law, not by will of the parties. The
law specifically enumerates the legal grounds, that must be proved to exist by indubitable evidence, to annul a marriage. In the
case at bar, the annulment of the marriage in question was decreed upon the sole testimony of the husband who was expected
to give testimony tending or aiming at securing the annulment of his marriage he sought and seeks. Whether the wife is really
impotent cannot be deemed to have been satisfactorily established, becase from the commencement of the proceedings until
the entry of the decree she had abstained from taking part therein. Although her refusal to be examined or failure to appear in
court show indifference on her part, yet from such attitude the presumption arising out of the suppression of evidence could not
arise or be inferred because women of this country are by nature coy, bashful and shy and would not submit to a physical
examination unless compelled to by competent authority. This the Court may do without doing violence to and infringing in this
case is not self-incrimination. She is not charged with any offense. She is not being compelled to be a witness against
herself.1 "Impotency being an abnormal condition should not be presumed. The presumption is in favor of potency." 2 The lone
testimony of the husband that his wife is physically incapable of sexual intercourse is insufficient to tear asunder the ties that
have bound them together as husband and wife.
The decree appealed from is set aside and the case remanded to the lower court for further proceedings in accordance with this
decision, without pronouncement as to costs.

MANUEL G. ALMELOR,
Petitioner,

G.R. No. 179620


Present:

- versus -

THE HON. REGIONAL TRIAL


COURT OF LAS PIAS CITY,
BRANCH 254, and
LEONIDA T. ALMELOR,
Respondent.

YNARES-SANTIAGO, J.,
Chairperson,
AUSTRIA-MARTINEZ,
CHICO-NAZARIO,
NACHURA, and
REYES, JJ.
Promulgated:
August 26, 2008

x--------------------------------------------------x
DECISION
REYES, R.T., J.:

MARRIAGE, in its totality, involves the spouses right to the community of their whole lives. It likewise involves a true
intertwining of personalities.[1]
This is a petition for review on certiorari of the Decision[2] of the Court of Appeals (CA) denying the petition for annulment
of judgment and affirming in toto the decision of the Regional Trial Court (RTC), Las Pias, Branch 254. The CA dismissed
outright the Rule 47 petition for being the wrong remedy.
The Facts
Petitioner Manuel G. Almelor (Manuel) and respondent Leonida Trinidad (Leonida) were married on January 29, 1989 at
the Manila Cathedral.[3] Their union bore three children: (1) Maria Paulina Corinne, born on October 20, 1989; (2) Napoleon
Manuel,born on August 9, 1991; and (3) Manuel Homer, born on July 4, 1994.[4] Manuel and Leonida are both medical
practitioners, an anesthesiologist and a pediatrician, respectively.[5]
After eleven (11) years of marriage, Leonida filed a petition with the RTC in Las Pias City to annul their marriage on the
ground that Manuel was psychologically incapacitated to perform his marital obligations. The case, docketed as LP-000132 was raffled off to Branch 254.
During the trial, Leonida testified that she first met Manuel in 1981 at the San Lazaro Hospital where they worked as
medical student clerks. At that time, she regarded Manuel as a very thoughtful person who got along well with other
people. They soon became sweethearts. Three years after, they got married.[6]
Leonida averred that Manuels kind and gentle demeanor did not last long. In the public eye, Manuel was the picture of a
perfect husband and father. This was not the case in his private life. At home, Leonida described Manuel as a harsh
disciplinarian,unreasonably meticulous, easily angered. Manuels unreasonable way of imposing discipline on their children
was the cause of their frequent fights as a couple.[7] Leonida complained that this was in stark contrast to the alleged lavish
affection Manuel has for his mother. Manuels deep attachment to his mother and his dependence on her decision-making were
incomprehensible to Leonida.[8]
Further adding to her woes was his concealment to her of his homosexuality. Her suspicions were first aroused when she
noticed Manuels peculiar closeness to his male companions. For instance, she caught him in an indiscreet telephone
conversation manifesting his affection for a male caller.[9] She also found several pornographic homosexual materials in his
possession.[10] Her worse fears were confirmed when she saw Manuel kissed another man on the lips. The man was a certain
Dr. Nogales.[11] When she confronted Manuel, he denied everything. At this point, Leonida took her children and left their
conjugal abode. Since then, Manuel stopped giving support to their children. [12]
Dr. Valentina del Fonso Garcia, a clinical psychologist, was presented to prove Leonidas claim. Dr. del Fonso Garcia
testified that she conducted evaluative interviews and a battery of psychiatric tests on Leonida. She also had a one-time
interview with Manuel and face-to-face interviews with Ma. Paulina Corrinne (the eldest child). [13] She concluded that Manuel is
psychologically incapacitated.[14] Such incapacity is marked by antecedence; it existed even before the marriage and appeared
to be incurable.
Manuel, for his part, admitted that he and Leonida had some petty arguments here and there. He, however, maintained
thattheir marital relationship was generally harmonious. The petition for annulment filed by Leonida came as a surprise to him.

Manuel countered that the true cause of Leonidas hostility against him was their professional rivalry. It began
when herefused to heed the memorandum[15] released by Christ the King Hospital. The memorandum ordered him to desist
from

converting

his

own

lying-in

clinic

to

primary

or

secondary

hospital. [16] Leonidas

family

owns

Christ

the King Hospital which is situated in the same subdivision as Manuels clinic and residence. [17] In other words, he and her
family have competing or rival hospitals in the same vicinity.
Manuel belied her allegation that he was a cruel father to their children. He denied maltreating them. At most,
he only imposedthe necessary discipline on the children.
He also defended his show of affection for his mother. He said there was nothing wrong for him to return the love and
affection of the person who reared and looked after him and his siblings. This is especially apt now that his mother is in her
twilight years.[18] Manuel pointed out that Leonida found fault in this otherwise healthy relationship because of her very jealous
andpossessive nature.[19]
This same overly jealous behavior of Leonida drove Manuel to avoid the company of female friends. He wanted to avoid
anyfurther misunderstanding with his wife. But, Leonida instead conjured up stories about his sexual preference. She also
fabricatedtales about pornographic materials found in his possession to cast doubt on his masculinity.[20]

To corroborate his version, he presented his brother, Jesus G. Almelor. Jesus narrated that he usually stayed at Manuels
house during his weekly trips to Manila from Iriga City. He was a witness to the generally harmonious relationship between his
brother Manuel and sister-in-law, Leonida. True, they had some quarrels typical of a husband and wife relationship. But there
wasnothing similar to what Leonida described in her testimony.[21]
Jesus further testified that he was with his brother on the day Leonida allegedly saw Manuel kissed another man. He
deniedthat such an incident occurred. On that particular date,[22] he and Manuel went straight home from a trip to Bicol. There
was no other person with them at that time, except their driver.[23]
Manuel expressed his intention to refute Dr. del Fonso Garcias findings by presenting his own expert witness. However,
no psychiatrist was presented.
RTC Disposition
By decision dated November 25, 2005, the RTC granted the petition for annulment, with the following disposition:
WHEREFORE, premised on the foregoing, judgment is hereby rendered:
1.

Declaring the marriage contracted by herein parties on 29 January 1989 and all its effects under the
law null and void from the beginning;

2.

Dissolving the regime of community property between the same parties with forfeiture of defendants
share thereon in favor of the same parties children whose

legal custody is awarded to plaintiff with visitorial right afforded to defendant;


3.

Ordering the defendant to give monthly financial support to all the children; and

4.

Pursuant to the provisions of A.M. No. 02-11-10-SC:


a.

Directing the Branch Clerk of this Court to enter this Judgment upon its finality in the Book of
Entry of Judgment and to issue an Entry of Judgment in accordance thereto; and

b.

Directing the Local Civil Registrars of Las Pias City and Manila City to cause the registration
of the said Entry of Judgment in their respective Books of Marriages.

Upon compliance, a decree of nullity of marriage shall be issued.


SO ORDERED.[24] (Emphasis supplied)
The trial court nullified the marriage, not on the ground of Article 36, but Article 45 of the Family Code. It ratiocinated:
x x x a careful evaluation and in-depth analysis of the surrounding circumstances of the allegations in the
complaint and of the evidence presented in support thereof (sic) reveals that in this case (sic) there is more than
meets the eyes (sic).
Both legally and biologically, homosexuality x x x is, indeed, generally incompatible with hetero sexual
marriage. This is reason enough that in this jurisdiction (sic) the law recognizes marriage as a special contract
exclusively only between a man and a woman x x x and thus when homosexuality has trespassed into marriage, the
same law provides ample remedies to correct the situation [Article 45(3) in relation to Article 46(4) or Article 55, par.
6, Family Code]. This is of course in recognition of the biological fact that no matter how a man cheats himself that
he is not a homosexual and forces himself to live a normal heterosexual life, there will surely come a time when his
true sexual preference as a homosexual shall prevail in haunting him and thus jeopardizing the solidity, honor, and
welfare of his own family.[25]
Manuel filed a notice of appeal which was, however, denied due course. Undaunted, he filed a petition for annulment of
judgment with the CA.[26]
Manuel contended that the assailed decision was issued in excess of the lower courts jurisdiction; that it had no
jurisdiction to dissolve the absolute community of property and forfeit his conjugal share in favor of his children.
CA Disposition
On July 31, 2007, the CA denied the petition, disposing as follows:
WHEREFORE, the present Petition for Annulment of Judgment is hereby DENIED. The Court AFFIRMS in
toto the Decision (datedNovember 25, 2005) of the Regional Trial Court (Branch 254), in Las Pias City, in Civil
Case No. LP-00-0132. No costs.[27]
The CA stated that petitioner pursued the wrong remedy by filing the extraordinary remedy of petition for annulment
ofjudgment. Said the appellate court:
It is obvious that the petitioner is questioning the propriety of the decision rendered by the lower Court. But
the remedy assuming there was a mistake is not a Petition for Annulment of Judgment but an ordinary appeal. An
error of judgment may be reversed or corrected only by appeal.
What petitioner is ascribing is an error of judgment, not of jurisdiction, which is properly the subject of an
ordinary appeal.
In short, petitioner admits the jurisdiction of the lower court but he claims excess in the exercise
thereof. Excess assuming there was is not covered by Rule 47 of the 1997 Rules of Civil Procedure. The Rule
refers the lack of jurisdiction and not the exercise thereof. [28]
Issues
Petitioner Manuel takes the present recourse via Rule 45, assigning to the CA the following errors:

I
THE HONORABLE COURT OF APPEALS ERRED IN NOT TREATING THE PETITION FOR ANNULMENT OF
JUDGMENT AS A PETITION FOR REVIEW IN VIEW OF THE IMPORTANCE OF THE ISSUES INVOLVED AND IN
THE INTEREST OF JUSTICE;
II
THE HONORABLE COURT OF APPEALS ERRED IN UPHOLDING THE DECISION OF THE TRIAL COURT AS
REGARDS THE ORDER DECLARING THE MARRIAGE AS NULL AND VOID ON THE GROUND OF
PETITIONERS PSYCHOLOGICAL INCAPACITY;
III
THE HONORABLE COURT OF APPEALS ERRED IN UPHOLDING THE DECISION OF THE TRIAL COURT AS
REGARDS THE ORDER TO FORFEIT THE SHARE OF PETITIONER IN HIS SHARE OF THE CONJUGAL
ASSETS.[29]
Our Ruling

I.

The stringent rules of procedures may be relaxed to serve the demands of substantial justice and in

the Courts exercise of equity jurisdiction.


Generally, an appeal taken either to the Supreme Court or the CA by the wrong or inappropriate mode shall be dismissed.
[30]

This is to prevent the party from benefiting from ones neglect and mistakes. However, like most rules, it

carries certainexceptions. After all, the ultimate purpose of all rules of procedures is to achieve substantial justice as
expeditiously as possible.[31]
Annulment of judgment under Rule 47 is a last remedy. It can not be resorted to if the ordinary remedies are available or
no longer available through no fault of petitioner. [32] However, in Buenaflor v. Court of Appeals,[33] this Court clarified the proper
appreciation for technical rules of procedure, in this wise:
Rules of procedures are intended to promote, not to defeat, substantial justice and, therefore, they
should not be applied in a very rigid and technical sense. The exception is that while the Rules are liberally
construed, the provisions with respect to the rules on the manner and periods for perfecting appeals are
strictly applied. As an exception to the exception, these rules have sometimes been relaxed on equitable
considerations. Also, in some cases the Supreme Court has given due course to an appeal perfected out of time
where a stringent application of the rules would have denied it, but only when to do so would serve the demands of
substantial justice and in the exercise of equity jurisdiction of the Supreme Court.[34] (Emphasis and underscoring
supplied)
For reasons of justice and equity, this Court has allowed exceptions to the stringent rules governing appeals. [35] It has, in
the past, refused to sacrifice justice for technicality.[36]
After

discovering

the

palpable

error

of

his

petition,

Manuel seeks the

indulgence

of this

Court to consider his petition before the CA instead as a petition for certiorari under Rule 65.
A perusal of the said petition reveals that Manuel imputed grave abuse of discretion to the lower court for annulling his
marriage on account of his alleged homosexuality. This is not the first time that this Court is faced with a similar
situation. In Nerves v. Civil Service Commission,[37] petitioner Delia R. Nerves elevated to the CA a Civil Service Commission
(CSC) decisionsuspending her for six (6) months. The CSC ruled Nerves, a public school teacher, is deemed to have already
served her six-month suspension during the pendency of the case. Nevertheless, she is ordered reinstated without back
wages. On appeal, Nerves stated in her petition, inter alia:
1.

This is a petition for certiorari filed pursuant to Article IX-A, Section 7 of the Constitution of
the Philippines and under Rule 65 of the Rules of Court.

2.

But per Supreme Court Revised Administrative Circular No. 1-95 (Revised Circular No. 1-91) petitioner is
filing the instant petition with this Honorable Court instead of the Supreme Court. [38] (Underscoring supplied)

The CA dismissed Nerves petition for certiorari for being the wrong remedy or the inappropriate mode of appeal. [39] The
CA opined that under the Supreme Court Revised Administrative Circular No. 1-95 x x x appeals from judgments or final orders
or resolutions of CSC is by a petition for review.[40]
This Court granted Nerves petition and held that she had substantially complied with the Administrative Circular. The
Courtstated:
That it was erroneously labeled as a petition for certiorari under Rule 65 of the Rules of Court is only a minor
procedural lapse, not fatal to the appeal. x x x
More importantly, the appeal on its face appears to be impressed with merit. Hence, the Court of Appeals
should have overlooked the insubstantial defects of the petition x x x in order to do justice to the parties
concerned. There is, indeed, nothing sacrosanct about procedural rules, which should be liberally construed in
order to promote their object and assist the parties in obtaining just, speedy, and inexpensive determination of every
action or proceeding. As it has been said, where the rigid application of the rules would frustrate substantial justice,
or bar the vindication of a legitimate grievance, the courts are justified in exempting a particular case from the
operation of the rules.[41] (Underscoring supplied)
Similarly, in the more recent case of Tan v. Dumarpa,[42] petitioner Joy G. Tan availed of a wrong remedy by filing a petition
for review on certiorari instead of a motion for new trial or an ordinary appeal. In the interest of justice, this Court considered the
petition, pro hac vice, as a petition for certiorari under Rule 65.
This Court found that based on Tans allegations, the trial court prima facie committed grave abuse of discretion in
rendering a judgment by default. If uncorrected, it will cause petitioner great injustice. The Court elucidated in this wise:
Indeed, where as here, there is a strong showing that grave miscarriage of justice would result from the strict
application of the Rules, we will not hesitate to relax the same in the interest of substantial justice .[43] (Underscoring
supplied)

Measured by the foregoing yardstick, justice will be better served by giving due course to the present petition and
treatingpetitioners CA petition as one for certiorari under Rule 65, considering that what is at stake is the validity or non-validity
of a marriage.
In Salazar v. Court of Appeals,[44] citing Labad v. University of Southeastern Philippines, this Court reiterated:
x x x The dismissal of appeals on purely technical grounds is frowned upon. While the right to appeal is a
statutory, not a natural right, nonetheless it is an essential part of our judicial system and courts should proceed with
caution so as not to deprive a party of the right to appeal, but rather, ensure that every party-litigant has the amplest
opportunity for the proper and just disposition of his cause, free from the constraints of technicalities. [45]
Indeed, it is far better and more prudent for a court to excuse a technical lapse and afford the parties a review of the case
on the merits to attain the ends of justice. [46]

Furthermore, it

was

the negligence

and

incompetence of Manuels

counsel that prejudiced his

right

to

appeal. His counsel,Atty. Christine Dugenio, repeatedly availed of inappropriate remedies. After the denial of her notice of
appeal, she failed to move for reconsideration or new trial at the first instance. She also erroneously filed a petition for
annulment of judgment rather than pursue an ordinary appeal.
These manifest errors were clearly indicative of counsels incompetence. These gravely worked to the detriment
of Manuels appeal. True it is that the negligence of counsel binds the client. Still, this Court has recognized certain exceptions:
(1) where reckless or gross negligence of counsel deprives the client of due process of law; (2) when its application will result in
outright deprivation of the clients liberty and property; or (3) where the interest of justice so require. [47]
The negligence of Manuels counsel falls under the exceptions. Ultimately, the reckless or gross negligence of
petitioners

former

counsel led

to

the

loss of his right

to

appeal. He

should

not be

made

to

suffer

for

his

counsels grave mistakes. Higher interests of justice and equity demand that he be allowed to ventilate his case in a higher
court.
In Apex Mining, Inc. v. Court of Appeals,[48] this Court explained thus:
It is settled that the negligence of counsel binds the client. This is based on the rule that any act performed by a
counsel within the scope of his general or implied authority is regarded as an act of his client. However, where
counsel is guilty of gross ignorance, negligence and dereliction of duty, which resulted in the clients being held liable
for damages in a damage suit, the client is deprived of his day in court and the judgment may be set aside on such
ground. In the instant case, higher interests of justice and equity demand that petitioners be allowed to present
evidence on their defense. Petitioners may not be made to suffer for the lawyers mistakes. This Court will always
be disposed to grant relief to parties aggrieved by perfidy, fraud, reckless inattention and downright
incompetence of lawyers, which has the consequence of depriving their clients, of their day in court .
[49]
(Emphasis supplied)
Clearly, this Court has the power to except a particular case from the operation of the rule whenever the demands of
justice require it. With more conviction should it wield such power in a case involving the sacrosanct institution of
marriage. This Court is guided with the thrust of giving a party the fullest opportunity to establish the merits of ones action. [50]
The client was likewise spared from counsels negligence in Government Service Insurance System v. Bengson
Commercial Buildings, Inc.[51] and Ancheta v. Guersey-Dalaygon.[52] Said the Court in Bengson:
But if under the circumstances of the case, the rule deserts its proper office as an aid to justice and becomes a
great hindrance and chief enemy, its rigors must be relaxed to admit exceptions thereto and to prevent a miscarriage
of justice. In other words, the court has the power to except a particular case from the operation of the rule whenever
the purposes of justice require it.[53]
II.

Concealment of homosexuality is the proper ground to annul a marriage, not homosexuality per se.
Manuel is a desperate man determined to salvage what remains of his marriage. Persistent in his quest, he fought back

all the heavy accusations of incapacity, cruelty, and doubted masculinity thrown at him.
The trial court declared that Leonidas petition for nullity had no basis at all because the supporting grounds relied
upon can not legally make a case under Article 36 of the Family Code. It went further by citing Republic v. Molina:[54]

Indeed, mere allegations of conflicting personalities, irreconcilable differences, incessant quarrels and/or
beatings, unpredictable mood swings, infidelities, vices, abandonment, and difficulty, neglect, or failure in the
performance of some marital obligations do not suffice to establish psychological incapacity. [55]
If so, the lower court should have dismissed outright the petition for not meeting the guidelines set in Molina. What
Leonidaattempted to demonstrate were Manuels homosexual tendencies by citing overt acts generally predominant among

homosexualindividuals.[56] She wanted to prove that the perceived homosexuality rendered Manuel incapable of fulfilling
the essential marital obligations.
But instead of dismissing the petition, the trial court nullified the marriage between Manuel and Leonida on the ground of
vitiated consent by virtue of fraud. In support of its conclusion, the lower court reasoned out:
As insinuated by the State (p. 75, TSN, 15 December 2003), when there is smoke surely there is
fire. Although vehemently denied by defendant, there is preponderant evidence enough to establish with certainty
that defendant is really a homosexual. This is the fact that can bededuced from the totality of the marriage life
scenario of herein parties.
Before his marriage, defendant knew very well that people around him even including his own close
friends doubted his true sexual preference (TSN, pp. 35-36, 13 December 2000; pp. 73-75, 15 December
2003). After receiving many forewarnings, plaintiff told defendant about the rumor she heard but defendant did not
do anything to prove to the whole world once and for all the truth of all his denials. Defendant threatened to sue
those people but nothing happened after that. There may have been more important matters to attend to than to
waste time and effort filing cases against and be effected by these people and so, putting more premiums on
defendants denials, plaintiff just the same married him. Reasons upon reasons may be advanced to either
exculpate or nail to the cross defendant for his act of initially concealing his homosexuality to plaintiff, but in the end,
only one thing is certain even during his marriage with plaintiff, the smoke of doubt about his real preference
continued and even got thicker, reason why obviously defendant failed to establish a happy and solid family; and in
so failing, plaintiff and their children became his innocent and unwilling victims.

Yes, there is nothing untoward of a man if, like herein defendant, he is meticulous over even small details in
the house (sic) like wrongly folded bed sheets, etc. or if a man is more authoritative in knowing what clothes or
jewelry shall fit his wife (pp. 77-81, TSN, 15 December 2003); but these admissions of defendant taken in the light of
evidence presented apparently showing that he had extra fondness of his male friends ( sic) to the extent that twice
on separate occasions (pp. 4-7, TSN, 14 February 2001) he was allegedly seen by plaintiff kissing another man lipsto-lips plus the homosexual magazines and tapes likewise allegedly discovered underneath his bed (Exhibits L
and M), the doubt as to his real sex identity becomes stronger. The accusation of plaintiff versus thereof of
defendant may be the name of the game in this case; but the simple reason of professional rivalry advanced by the
defendant is certainly not enough to justify and obscure the question why plaintiff should accuse him of such a very
untoward infidelity at the expense and humiliation of their children and family as a whole. [57]
Evidently, no sufficient proof was presented to substantiate the allegations that Manuel is a homosexual and that he
concealed this to Leonida at the time of their marriage. The lower court considered the public perception of Manuels sexual
preference without the corroboration of witnesses. Also, it took cognizance of Manuels peculiarities and interpreted it against
his sexuality.
Even assuming, ex gratia argumenti, that Manuel is a homosexual, the lower court cannot appreciate it as a ground to
annul his marriage with Leonida. The law is clear a marriage may be annulled when the consent of either party was obtained
by fraud,[58]such as concealment of homosexuality.[59] Nowhere in the said decision was it proven by preponderance of evidence
that Manuel was a homosexual at the onset of his marriage and that he deliberately hid such fact to his wife. [60] It is the
concealment of homosexuality, and not homosexuality per se, that vitiates the consent of the innocent party. Such concealment
presupposes bad faith and intent to defraud the other party in giving consent to the marriage.
Consent is an essential requisite of a valid marriage. To be valid, it must be freely given by both parties. An allegation of
vitiated consent must be proven by preponderance of evidence. The Family Code has enumerated an exclusive list of
circumstances[61] constituting fraud. Homosexuality per se is not among those cited, but its concealment.
This distinction becomes more apparent when we go over the deliberations [62] of the Committees on the Civil Code and
Family Law, to wit:
Justice Caguioa remarked that this ground should be eliminated in the provision on the grounds for legal
separation. Dean Gupit, however, pointed out that in Article 46, they are talking only of concealment, while in the
article on legal separation, there is actuality. Judge Diy added that in legal separation, the ground existed after the
marriage, while in Article 46, the ground existed at the time of the marriage. Justice Reyes suggested that, for clarity,
they add the phrase existing at the time of the marriage at the end of subparagraph (4). The Committee approved
the suggestion.[63]
To reiterate, homosexuality per se is only a ground for legal separation. It is its concealment that serves as a valid ground
to annul a marriage.[64] Concealment in this case is not simply a blanket denial, but one that is constitutive of fraud. It is this
fundamental element that respondent failed to prove.
In the United States, homosexuality has been considered as a basis for divorce. It indicates that questions of sexual
identity strike so deeply at one of the basic elements of marriage, which is the exclusive sexual bond between the spouses.
[65]

In Crutcher v. Crutcher,[66] the Court held:


Unnatural practices of the kind charged here are an infamous indignity to the wife, and which would make the
marriage relation so revolting to her that it would become impossible for her to discharge the duties of a wife, and
would defeat the whole purpose of the relation. In the natural course of things, they would cause mental suffering to
the extent of affecting her health.[67]

However, although there may be similar sentiments here in the Philippines, the legal overtones are significantly
different. Divorce is not recognized in the country. Homosexuality and its alleged incompatibility to a healthy heterosexual life
are not sanctioned as grounds to sever the marriage bond in our jurisdiction. At most, it is only a ground to separate from bed
and board.
What was proven in the hearings a quo was a relatively blissful marital union for more than eleven (11) years, which
produced three (3) children. The burden of proof to show the nullity of the marriage rests on Leonida. Sadly, she failed to
discharge this onus.
The same failure to prove fraud which purportedly resulted to a vitiated marital consent was found in Villanueva v. Court of
Appeals.[68] In Villanueva, instead of proving vitiation of consent, appellant resorted to baseless portrayals of his wife as a
perpetrator of fraudulent schemes. Said the Court:
Factual findings of the Court of Appeals, especially if they coincide with those of the trial court, as in the instant
case, are generally binding on this Court. We affirm the findings of the Court of Appeals that petitioner freely and
voluntarily married private respondent and that no threats or intimidation, duress or violence compelled him to do so,
thus
Appellant anchored his prayer for the annulment of his marriage on the ground that he did not freely consent
to be married to the appellee. He cited several incidents that created on his mind a reasonable and well-grounded
fear of an imminent and grave danger to his life and safety. x x x
The Court is not convinced that appellants apprehension of danger to his person is so overwhelming as to
deprive him of the will to enter voluntarily to a contract of marriage. It is not disputed that at the time he was
allegedly being harassed, appellant worked as a security guard in a bank. Given the rudiments of self-defense, or,
at the very least, the proper way to keep himself out of harms way. x x x
Appellant also invoked fraud to annul his marriage, as he was made to believe by appellee that the latter was
pregnant with his child when they were married. Appellants excuse that he could not have impregnated the
appellee because he did not have an erection during their tryst is flimsy at best, and an outright lie at worst. The
complaint is bereft of any reference to his inability to copulate with the appellee. x x x
xxxx
x x x The failure to cohabit becomes relevant only if it arises as a result of the perpetration of any of the
grounds for annulling the marriage, such as lack of parental consent, insanity, fraud, intimidation, or undue influence
x x x. Since the appellant failed to justify his failure to cohabit with the appellee on any of these grounds, the validity
of his marriage must be upheld.[69]
Verily, the lower court committed grave abuse of discretion, not only by solely taking into account petitioners
homosexuality per se and not its concealment, but by declaring the marriage void from its existence.
This Court is mindful of the constitutional policy to protect and strengthen the family as the basic autonomous social
institution and marriage as the foundation of the family. [70] The State and the public have vital interest in the maintenance and
preservation of these social institutions against desecration by fabricated evidence. [71] Thus, any doubt should be resolved in
favor of the validity of marriage.
III.

In a valid marriage, the husband and wife jointly administer and enjoy their community or conjugal property.
Article 96 of the Family Code, on regimes of absolute community property, provides:
Art. 96. The administration and enjoyment of the community property shall belong to both spouses jointly. In
case of disagreement, the husbands decision shall prevail, subject to recourse to the court by the wife for a proper
remedy, which must be availed of within five years from the date of the contract implementing such decision.
In the event that one spouse is incapacitated or otherwise unable to participate in the administration of the
common properties, the other spouse may assume sole powers of administration. These powers do not include the
powers of disposition or encumbrance without the authority of the court or the written consent of the other
spouse. In the absence of such authority or consent, the disposition or encumbrance shall be void. However, the
transaction shall be construed as a continuing offer on the part of the consenting spouse and the third person, and
may be perfected as a binding contract upon the acceptance by the other spouse or authorization by the court
before the offer is withdrawn by either or both offerors.
A similar provision, Article 124[72] prescribes joint administration and enjoyment in a regime of conjugal partnership. In a

valid marriage, both spouses exercise administration and enjoyment of the property regime, jointly.
In the case under review, the RTC decreed a dissolution of the community property of Manuel and Leonida. In the same
breath, the trial court forfeited Manuels share in favor of the children. Considering that the marriage is upheld valid and
subsisting, the dissolution and forfeiture of Manuels share in the property regime is unwarranted. They remain the joint
administrators of the community property.
WHEREFORE, the petition is GRANTED. The appealed Decision is REVERSED and SET ASIDE and the petition in the
trial court to annul the marriage is DISMISSED.
SO ORDERED.

G.R. No. 137590

March 26, 2001

FLORENCE MALCAMPO-SIN, petitioner,


vs.
PHILIPP T. SIN, respondent.
PARDO, J.:
The Family Code emphasizes the permanent nature of marriage, hailing it as the foundation of the family. 1 It is this inviolability
which is central to our traditional and religious concepts of morality and provides the very bedrock on which our society finds
stability.2 Marriage is immutable and when both spouses give their consent to enter it, their consent becomes irrevocable,
unchanged even by their independent wills.
However, this inviolability depends on whether the marriage exists and is valid. If it is void ab initio, the "permanence" of the
union becomes irrelevant, and the Court can step in to declare it so. Article 36 of the Family Code is the justification. 3 Where it
applies and is duly proven, a judicial declaration can free the parties from the rights, obligations, burdens and consequences
stemming from their marriage.
A declaration of nullity of marriage under Article 36 of the Family Code requires the application of procedural and substantive
guidelines. While compliance with these requirements mostly devolves upon petitioner, the State is likewise mandated to
actively intervene in the procedure. Should there be non-compliance by the State with its statutory duty, there is a need to
remand the case to the lower court for proper trial.
The Case
What is before the Court4 is an appeal from a decision of the Court of Appeals 5 which affirmed the decision of the Regional Trial
Court, Branch 158, Pasig City6 dismissing petitioner Florence Malcampo-Sin's (hereafter "Florence") petition for declaration of
nullity of marriage due to psychological incapacity for insufficiency of evidence.
The Facts
On January 4, 1987, after a two-year courtship and engagement, Florence and respondent Philipp T. Sin (hereafter "Philipp"), a
Portugese citizen, were married at St. Jude Catholic Parish in San Miguel, Manila. 7
On September 20, 1994, Florence filed with the Regional Trial Court, Branch 158, Pasig City, a complaint for "declaration of
nullity of marriage" against Philipp.8 Trial ensued and the parties presented their respective documentary and testimonial
evidence.
On June 16, 1995, the trial court dismissed Florence's petition. 9
On December 19, 1995, Florence filed with the trial court a notice of appeal to the Court of Appeals. 10
After due proceedings, on April 30, 1998, the Court of Appeals promulgated its decision, the dispositive portion of which reads:
"IN THE LIGHT OF ALL THE FOREGOING, the Appeal is DISMISSED. The Decision appealed from is AFFIRMED. Cost
against the Appellant."11
On June 23, 1998, petitioner filed with the Court of Appeals a motion for reconsideration of the aforequoted decision. 12
On January 19, 1999, the Court of Appeals denied petitioner's motion for reconsideration. 13
Hence, this appeal.14
The Court's Ruling
We note that throughout the trial in the lower court, the State did not participate in the proceedings. While Fiscal Jose Danilo C.
Jabson15 filed with the trial court a manifestation dated November 16, 1994, stating that he found no collusion between the
parties,16 he did not actively participate therein. Other than entering his appearance at certain hearings of the case, nothing
more was heard from him. Neither did the presiding Judge take any step to encourage the fiscal to contribute to the
proceedings.
The Family Code mandates:
"ARTICLE 48. In all cases of annulment or declaration of absolute nullity of marriage, the Court shall orderthe prosecuting
attorney or fiscal assigned to it to appear on behalf of the State to take steps to prevent collusion between the parties
and to take care that evidence is not fabricated or suppressed (italics ours).
"In the cases referred to in the preceding paragraph, no judgment shall be based upon a stipulation of facts or confession
of judgment."
It can be argued that since the lower court dismissed the petition, the evil sought to be prevented (i.e., dissolution of the
marriage) did not come about, hence, the lack of participation of the State was cured. Not so. The task of protecting marriage as
an inviolable social institution requires vigilant and zealous participation and not mere pro-forma compliance. The protection of
marriage as a sacred institution requires not just the defense of a true and genuine union but the exposure of an invalid one as
well. This is made clear by the following pronouncement:
"(8) The trial court must order the prosecuting attorney or fiscal and the Solicitor General to appear as counsel for the
state. No decision shall be handed down unless the Solicitor General issues a certification, which will be quoted in the
decision,17 briefly stating therein his reasons for his agreement or opposition as the case may be, to the petition. The
Solicitor-General shall discharge the equivalent function of the defensor vinculi contemplated under Canon 1095 (italics
ours)."18
The records are bereft of any evidence that the State participated in the prosecution of the case not just at the trial level but on
appeal with the Court of Appeals as well. Other than the "manifestation" filed with the trial court on November 16, 1994, the
State did not file any pleading, motion or position paper, at any stage of the proceedings.
In Republic of the Philippines v. Erlinda Matias Dagdag,19 while we upheld the validity of the marriage, we nevertheless
characterized the decision of the trial court as "prematurely rendered" since the investigating prosecutor was not given an
opportunity to present controverting evidence before the judgment was rendered. This stresses the importance of the
participation of the State.
Having so ruled, we decline to rule on the factual disputes of the case, this being within the province of the trial court upon
proper re-trial.
Obiter Dictum

For purposes of re-trial, we guide the parties thus: In Republic vs. Court of Appeals,20 the guidelines in the interpretation and
application of Article 36 of the Family Code are as follows (omitting guideline [8] in the enumeration as it was already earlier
quoted):
"(1) The burden of proof to show the nullity of the marriage belongs to the plaintiff. Any doubt should be resolved in favor
of the existence and continuation of the marriage and against its dissolution and nullity. This is rooted in the fact that both
our Constitution and our laws cherish the validity of marriage and unity of the family. Thus, our Constitution devotes an
entire Article on the Family, recognizing it "as the foundation of the nation." It decrees marriage as legally "inviolable,"
thereby protecting it from dissolution at the whim of the parties. Both the family and marriage are to be "protected" by the
state. The Family Code echoes this constitutional edict on marriage and the family and emphasizes their permanence,
inviolability and solidarity.
"(2) The root cause of the psychological incapacity must be: a) medically or clinically identified, b) alleged in the complaint,
c) sufficiently proven by experts and d) clearly explained in the decision. Article 36 of the Family Code requires that the
incapacity must be psychological not physical, although its manifestations and/or symptoms may be physical. The
evidence must convince the court that the parties, or one of them, was mentally or psychically (sic) ill to such an extent
that the person could not have known the obligations he was assuming, or knowing them, could not have given valid
assumption thereof. Although no example of such incapacity need be given here so as not to limit the application of the
provision under the principle ofejusdem generis, nevertheless such root cause must be identified as a psychological
illness and its incapacitating nature fully explained. Expert evidence may be given by qualified psychiatrists and clinical
psychologists.
"(3) The incapacity must be proven to be existing at "the time of the celebration" of the marriage. The evidence must show
that the illness was existing when the parties exchanged their "I do's." The manifestation of the illness need not be
perceivable at such time, but the illness itself must have attached at such moment, or prior thereto.
"(4) Such incapacity must also be shown to be medically or clinically permanent or incurable. Such incurability may be
absolute or even relative only in regard to the other spouse, not necessarily absolutely against everyone of the same sex.
Furthermore, such incapacity must be relevant to the assumption of marriage obligations, not necessarily to those not
related to marriage, like the exercise of a profession or employment in a job. Hence, a pediatrician may be effective in
diagnosing illnesses of children and prescribing medicine to cure them but may not be psychologically capacitated to
procreate, bear and raise his/her own children as an essential obligation of marriage.
"(5) Such illness must be grave enough to bring about the disability of the party to assume the essential obligations of
marriage. Thus, "mild characteriological peculiarities, mood changes, occasional emotional outbursts" cannot be accepted
as root causes. The illness must be shown as downright incapacity or inability, not refusal, neglect or difficulty, much less
ill will. In other words, there is a natal or supervening disabling factor in the person, an adverse integral element in the
personality structure that effectively incapacitates the person from really accepting and thereby complying with the
obligations essential to marriage.
"(6) The essential marital obligations must be those embraced by Articles 68 up to 71 of the Family Code as regards the
husband and wife as well as Articles 220, 221 and 225 of the same Code in regard to parents and their children. Such
non-complied marital obligation(s) must also be stated in the petition, proven by evidence and included in the text of the
decision.
"(7) Interpretations given by the National Appellate Matrimonial Tribunal of the Catholic Church in the Philippines, while
not controlling or decisive, should be given great respect by our courts."
The Fallo
WHEREFORE, the Court REVERSES and SETS ASIDE the appealed decision of the Court of Appeals in CA-G.R. CV No.
51304, promulgated on April 30, 1998 and the decision of the Regional Trial Court, Branch 158, Pasig City in Civil Case No.
3190, dated June 16, 1995.
Let the case be REMANDED to the trial court for proper trial.
No costs.
SO ORDERED.

G.R. No. L-13553

February 23, 1960

JOSE DE OCAMPO, petitioner,


vs.
SERAFINA FLORENCIANO, respondent.
Joselito J. Coloma for petitioner.
BENGZON, J.:
Action for legal separation by Jose de Ocampo against his wife Serafina, on the ground of adultery. The court of first instance of
Nueva Ecija dismissed it. The Court of Appeals affirmed, holding there was confession of judgment, plus condonation or
consent to the adultery and prescription.
We granted certiorari to consider the application of articles 100 and 101 of the New Civil Code, which for convenience are
quoted herewith:
ART. 100.The legal separation may be claimed only by the innocent spouse, provided there has been no condonation of
or consent to the adultery or concubinage. Where both spouses are offenders, a legal separation cannot be claimed by
either of them. Collusion between the parties to obtain legal separation shall cause the dismissal of the petition.
ART. 101.No decree of legal separation shall be promulgated upon a stipulation of facts or by confession of judgment.
In case of non-appearance of the defendant, the court shall order the prosecuting attorney to inquire whether or not a
collusion between the parties exists. If there is no collusion, the prosecuting attorney shall intervene for the State in order
to take care that the evidence for the plaintiff is not fabricated.
The record shows that on July 5, 1955, the complaint for legal separation was filed. As amended, it described their marriage
performed in 1938, and the commission of adultery by Serafina, in March 1951 with Jose Arcalas, and in June 1955 with Nelson
Orzame.
Because the defendant made no answer, the court defaulted her, and pursuant to Art. 101 above, directed the provincial fiscal to
investigate whether or not collusion existed between the parties. The fiscal examined the defendant under oath, and then
reported to the Court that there was no collusion. The plaintiff presented his evidence consisting of the testimony of Vicente
Medina, Ernesto de Ocampo, Cesar Enriquez, Mateo Damo, Jose de Ocampo and Capt. Serafin Gubat.
According to the Court of Appeals, the evidence thus presented shows that "plaintiff and defendant were married in April 5, 1938
by a religious ceremony in Guimba, Nueva Ecija, and had lived thereafter as husband and wife. They begot several children
who are now living with plaintiff. In March, 1951, plaintiff discovered on several occasions that his wife was betraying his trust by
maintaining illicit relations with one Jose Arcalas. Having found the defendant carrying marital relations with another man
plaintiff sent her to Manila in June 1951 to study beauty culture, where she stayed for one year. Again, plaintiff discovered that
while in the said city defendant was going out with several other men, aside from Jose Arcalas. Towards the end of June, 1952,
when defendant had finished studying her course, she left plaintiff and since then they had lived separately.
"On June 18, 1955, plaintiff surprised his wife in the act of having illicit relations with another man by the name of Nelson
Orzame. Plaintiff signified his intention of filing a petition for legal separation, to which defendant manifested her conformity
provided she is not charged with adultery in a criminal action. Accordingly, plaintiff filed on July 5, 1955, a petition for legal
separation."
The Court of Appeals held that the husband's right to legal separation on account of the defendant's adultery with Jose Arcalas
had prescribed, because his action was not filed within one year from March 1951 when plaintiff discovered her infidelity. (Art.
102, New Civil Code) We must agree with the Court of Appeals on this point. 1
As to the adultery with Nelson Orzame, the appellate court found that in the night of June 18, 1955, the husband upon
discovering the illicit connection, expressed his wish to file a petition for legal separation and defendant readily agreed to such
filing. And when she was questioned by the Fiscal upon orders of the court, she reiterated her conformity to the legal separation
even as she admitted having had sexual relations with Nelson Orzame. Interpreting these facts virtually to mean a confession of
judgment the Appellate Court declared that under Art. 101, legal separation could not be decreed.
As we understand the article, it does not exclude, as evidence, any admission or confession made by the defendant outside of
the court. It merely prohibits a decree of separation upon a confession of judgment. Confession of judgment usually happens
when the defendant appears in court and confesses the right of plaintiff to judgment or files a pleading expressly agreeing to the
plaintiff's demand.2 This is not occur.
Yet, even supposing that the above statement of defendant constituted practically a confession of judgment, inasmuch as there
is evidence of the adultery independently of such statement, the decree may and should be granted, since it would not be based
on her confession, but upon evidence presented by the plaintiff. What the law prohibits is a judgment based exclusively or
mainly on defendant's confession. If a confession defeats the actionipso facto, any defendant who opposes the separation will
immediately confess judgment, purposely to prevent it.
The mere circumstance that defendants told the Fiscal that she "like also" to be legally separated from her husband, is no
obstacle to the successful prosecution of the action. When she refused to answer the complaint, she indicated her willingness to
be separated. Yet, the law does not order the dismissal. Allowing the proceeding to continue, it takes precautions against
collusion, which implies more than consent or lack of opposition to the agreement.
Needless to say, when the court is informed that defendant equally desires the separation and admitted the commission of the
offense, it should be doubly careful lest a collusion exists. (The Court of Appeals did not find collusion.)
Collusion in divorce or legal separation means the agreement.
. . . between husband and wife for one of them to commit, or to appear to commit, or to be represented in court as having
committed, a matrimonial offense, or to suppress evidence of a valid defense, for the purpose of enabling the other to
obtain a divorce. This agreement, if not express, may be implied from the acts of the parties. It is a ground for denying the
divorce. (Griffiths vs. Griffiths, 69 N. J. Eq. 689 60 Atl. 1099; Sandoz vs. Sandoz, 107 Ore. 282, 214 Pas. 590.).
In this case, there would be collusion if the parties had arranged to make it appear that a matrimonial offense had been
committed although it was not, or if the parties had connived to bring about a legal separation even in the absence of grounds
therefor.
Here, the offense of adultery had really taking place, according to the evidence. The defendant could not havefalsely told the
adulterous acts to the Fiscal, because her story might send her to jail the moment her husband requests the Fiscal to prosecute.
She could not have practiced deception at such a personal risk.
In this connection, it has been held that collusion may not be inferred from the mere fact that the guilty party confesses to the
offense and thus enables the other party to procure evidence necessary to prove it. (Williams vs. Williams, [N. Y.] 40 N. E. (2d)
1017; Rosenweig vs. Rosenweig, 246 N. Y. Suppl. 231; Conyers, vs. Conyers, 224 S. W. [2d] 688.).

And proof that the defendant desires the divorce and makes no defense, is not by itself collusion. (Pohlman vs. Pohlman, [N. J.]
46 Atl. Rep. 658.).
We do not think plaintiff's failure actively to search for defendant and take her home (after the latter had left him in 1952)
constituted condonation or consent to her adulterous relations with Orzame. It will be remembered that she "left" him after
having sinned with Arcalas and after he had discovered her dates with other men. Consequently, it was not his duty to search
for her to bring her home. Hers was the obligation to return.
Two decisions3 are cited wherein from apparently similar circumstances, this Court inferred the husband's consent to or
condonation of his wife's misconduct. However, upon careful examination, a vital difference will be found: in both instances, the
husband had abandoned his wife; here it was the wife who "left" her husband.
Wherefore, finding no obstacles to the aggrieved husband's petition we hereby reverse the appealed decision and decree a
legal separation between these spouse, all the consequent effects. Costs of all instances against Serafina Florenciano. So
ordered.

G.R. No. 116607 April 10, 1996


EMILIO R. TUASON, petitioner,
vs.
COURT OF APPEALS and MARIA VICTORIA L. TUASON, respondents.

PUNO, J.:p
This petition for review on certiorari seeks to annul and set aside the decision dated July 29, 1994 of the Court of Appeals in
CA-G.R. CV No. 37925 denying petitioner's appeal from an order of the Regional Trial Court, Branch 149, Makati in Civil Case
No. 3769.
This case arose from the following facts:
In 1989, private respondent Maria Victoria Lopez Tuason filed with the Regional Trial Court, Branch 149, Makati a petition for
annulment or declaration of nullity of her marriage to petitioner Emilio R. Tuason. In her complaint, private respondent alleged
that she and petitioner were married on June 3, 1972 and from this union, begot two children; that at the time of the marriage,
petitioner was already psychologically incapacitated to comply with his essential marital obligations which became manifest
afterward and resulted in violent fights between husband and wife; that in one of their fights, petitioner inflicted physical injuries
on private respondent which impelled her to file a criminal case for physical injuries against him; that petitioner used prohibited
drugs, was apprehended by the authorities and sentenced to a one-year suspended penalty and has not been rehabilitated; that
petitioner was a womanizer, and in 1984, he left the conjugal home and cohabited with three women in succession, one of
whom he presented to the public as his wife; that after he left the conjugal dwelling, petitioner gave minimal support to the family
and even refused to pay for the tuition fees of their children compelling private respondent to accept donations and dole-outs
from her family and friends; that petitioner likewise became a spendthrift and abused his administration of the conjugal
partnership by alienating some of their assets and incurring large obligations with banks, credit card companies and other
financial institutions, without private respondent's consent; that attempts at reconciliation were made but they all failed because
of petitioner's refusal to reform. In addition to her prayer for annulment of marriage, private respondent prayed for powers of
administration to save the conjugal properties from further dissipation. 1
Petitioner answered denying the imputations against him. As affirmative defense, he claimed that he and private respondent
were a normal married couple during the first ten years of their marriage and actually begot two children during this period; that
it was only in 1982 that they began to have serious personal differences when his wife did not accord the respect and dignity
due him as a husband but treated him like a persona non grata; that due to the "extreme animosities " between them, he
temporarily left the conjugal home for a "cooling-off period" in 1984; that it is private respondent who had been taking prohibited
drugs and had a serious affair with another man; that petitioner's work as owner and operator of a radio and television station
exposed him to malicious gossip linking him to various women in media and the entertainment world; and that since 1984, he
experienced financial reverses in his business and was compelled, with the knowledge of his wife, to dispose of some of the
conjugal shares in exclusive golf and country clubs. Petitioner petitioned the court to allow him to return to the conjugal home
and continue his administration of the conjugal partnership.
After the issues were joined, trial commenced on March 30, 1990. Private respondent presented four witnesses, namely,
herself; Dr. Samuel Wiley, a Canon Law expert and marriage counselor of both private respondent and petitioner; Ms. Adelita
Prieto, a close friend of the spouses, and Atty. Jose F. Racela IV, private respondent's counsel. Private respondent likewise
submitted documentary evidence consisting of newspaper articles of her husband's relationship with other women, his
apprehension by the authorities for illegal possession of drugs; and copies of a prior a church annulment decree. 2 The parties'
marriage was clerically annulled by the Tribunal Metropolitanum Matrimonial which was affirmed by the National Appellate
Matrimonial Tribunal in 1986. 3
During presentation of private respondent's evidence, petitioner, on April 18, 1990, filed his Opposition to private respondent's
petition for appointment as administratrix of the conjugal partnership of gains.
After private respondent rested her case, the trial court scheduled the reception of petitioner's evidence on May 11, 1990.
On May 8, 1990, two days before the scheduled hearing , a counsel for petitioner moved for a postponement on the ground that
the principal counsel was out of the country and due to return on the first week of June. 4 The court granted the motion and reset
the hearing to June 8, 1990. 5
On June 8, 1990, petitioner failed to appear. On oral motion of private respondent, the court declared petitioner to have waived
his right to present evidence and deemed the case submitted for decision on the basis of the evidence presented.
On June 29, 1990, the trial court rendered judgment declaring the nullity of private respondent's marriage to petitioner and
awarding custody of the children to private respondent. The court ruled:
WHEREFORE, in view of the foregoing, the marriage contracted by Ma. Victoria L. Tuason and Emilio R. Tuason on
June 3, 1972 is declared null and void ab initio on the ground of psychological incapacity on the part of the
defendant under Sec. 36 of the Family Code. Let herein judgment of annulment be recorded in the registry of
Mandaluyong, Metro Manila where the marriage was contracted and in the registry of Makati, Metro Manila where
the marriage is annulled.
The custody of the two (2) legitimate children of the plaintiff and the defendant is hereby awarded to the plaintiff.
The foregoing judgment is without prejudice to the application of the other effects of annulment as provided for
under Arts . 50 and 51 of the Family Code of the Philippines. 6
Counsel for petitioner received a copy of this decision on August 24, 1990. No appeal was taken from the decision.
On September 24, 1990, private respondent filed a "Motion for Dissolution of Conjugal Partnership of Gains and Adjudication to
Plaintiff of the Conjugal Properties." 7 Petitioner opposed the motion on October 17, 1990. 8
Also on the same day, October 17, 1990, petitioner, through new counsel, filed with the trial court a petition for relief from
judgment of the June 29, 1990 decision.
The trial court denied the petition on August 8, 1991. 9
Petitioner appealed before the Court of Appeals the order of the trial court denying his petition for relief from judgment. On July
29, 1994, the Court of Appeals dismissed the appeal and affirmed the order of the trial court. 10
Hence this petition.
The threshold issue is whether a petition for relief from judgment is warranted under the circumstances of the case.
We rule in the negative.
A petition for relief from judgment is governed by Rule 38, Section 2 of the Revised Rules of Court which provides:

Sec. 2. Petition to Court of First Instance for relief from judgment or other proceeding thereof. When a judgment
or order is entered, or any other proceeding is taken, against a party in a Court of First Instance through fraud,
accident, mistake, or excusable negligence, he may file a petition in such court and in the same cause praying that
the judgment, order or proceeding be set aside.
Under the rules, a final and executory judgment or order of the Regional Trial Court may be set aside on the ground of fraud,
accident, mistake or excusable negligence. In addition, the petitioner must assert facts showing that he has a good, substantial
and meritorious defense or cause of action. 11 If the petition is granted, the court shall proceed to hear and determine the case
as if a timely motion for new trial had been granted therein. 12
In the case at bar, the decision annulling petitioner's marriage to private respondent had already become final and executory
when petitioner failed to appeal during the reglementary period. Petitioner however claims that the decision of the trial court was
null and void for violation of his right to due process. He contends he was denied due process when, after failing to appear on
two scheduled hearings, the trial court deemed him to have waived his right to present evidence and rendered judgment on the
basis of the evidence for private respondent. Petitioner justifies his absence at the hearings on the ground that he was then
"confined for medical and/or rehabilitation reason." 13 In his affidavit of merit before the trial court, he attached a certification by
Lt. Col. Plaridel F. Vidal, Director of the Narcotics Command, Drug Rehabilitation Center which states that on March 27, 1990
petitioner was admitted for treatment of drug dependency at the Drug Rehabilitation Center at Camp Bagong Diwa, Bicutan,
Taguig, Metro Manila of the Philippine Constabulary-Integrated National Police. 14 The records, however, show that the former
counsel of petitioner did not inform the trial court of this confinement. And when the court rendered its decision, the same
counsel was out of the country for which reason the decision became final and executory as no appeal was taken therefrom. 15
The failure of petitioner's counsel to notify him on time of the adverse judgment to enable him to appeal therefrom is negligence
which is not excusable. Notice sent to counsel of record is binding upon the client and the neglect or failure of counsel to inform
him of an adverse judgment resulting in the loss of his right to appeal is not a ground for setting aside a judgment valid and
regular on its face. 16
Similarly inexcusable was the failure of his former counsel to inform the trial court of petitioner's confinement and medical
treatment as the reason for his non-appearance at the scheduled hearings. Petitioner has not given any reason why his former
counsel, intentionally or unintentionally, did not inform the court of this fact. This led the trial court to order the case deemed
submitted for decision on the basis of the evidence presented by the private respondent alone. To compound the negligence of
petitioner's counsel, the order of the trial court was never assailed via a motion for reconsideration.
Clearly, petitioner cannot now claim that he was deprived of due process. He may have lost his right to present evidence but he
was not denied his day in court. As the record show, petitioner, through counsel, actively participated in the proceedings below.
He filed his answer to the petition, cross-examined private respondent's witnesses and even submitted his opposition to private
respondent's motion for dissolution of the conjugal partnership of gains. 17
A petition for relief from judgment is an equitable remedy; it is allowed only in exception cases where there is no other available
or adequate remedy. When a party has another remedy available or adequate remedy. When a party has another remedy
available to him, which may be either a motion for new trial or appeal from an adverse decision of the trial or appeal from an
adverse decision of the trial court, and he was not prevented by fraud, accident, mistake or excusable negligence from filing
such motion or taking such appeal, he cannot avail himself of this petition. 18 Indeed, relief will not be granted to a party who
seeks avoidance from the effects of the judgment when the loss of the remedy at law was due to his own negligence; otherwise
the petition for relief can be used to revive the right to appeal which had been lost thru inexcusable negligence. 19
Petitioner also insists that he has a valid and meritorious defense. He cites the Family Code which provides that in actions for
annulment of marriage or legal separation, the prosecuting officer should intervene for the state because the law "looks with
disfavor upon the haphazard declaration of annulment of marriages by default." He contends that when he failed to appear at
the scheduled hearings, the trial court should have ordered the prosecuting officer to intervene for the state and inquire as to the
reason for his non-appearance. 20
Articles 48 and 60 of the Family Code read as follows:
Art. 48. In all cases of annulment or declaration of absolute nullity of marriage, the Court shall order the prosecution
attorney or fiscal assigned to it to appear on behalf of the State to take steps to prevent collusion between the
parties and to take care that evidence is not fabricated or suppressed.
In the cases referred to in the preceding paragraph, no judgment shall be based upon a stipulation of facts or
confession of judgment.
xxx xxx xxx
Art. 60. No decree of legal separation shall be based upon a stipulation of facts or a confession of judgment.
In any case, the Court shall order the prosecuting attorney or fiscal assigned to it to take steps to prevent collusion
between the parties and to take care that the evidence is not fabricated or suppressed. 21
A grant of annulment of marriage or legal separation by default is fraught with the danger of collusion. 22 Hence, in all cases for
annulment, declaration of nullity of marriage and legal separation, the prosecuting attorney or fiscal is ordered to appear on
behalf of the state for the purpose of preventing any collusion between the parties and to take care that their evidence is not
fabricated or suppressed. If the defendant spouse fails to answer the complaint, the court cannot declare him or her in default
but instead, should order the prosecuting attorney to determine if collusion exists between the parties. 23The prosecuting
attorney or fiscal may oppose the application for legal separation or annulment through the presentation of his own evidence, if
in his opinion, the proof adduced is dubious and fabricated. 24 Our Constitution is committed to the policy of strengthening the
family as a basic social institution. 25 Our family law is based on the policy that marriage is not a mere contract, but a social
institution in which the state is vitally interested. The state can find no stronger anchor than on good, solid and happy families.
The break up of families weakens our social and moral fabric and, hence, their preservation is not the concern alone of the
family members.
The facts in the case at bar do not call for the strict application of Articles 48 and 60 of the Family Code. For one, petitioner was
not declared in default by the trial court for failure to answer. Petitioner filed his answer to the complaint and contested the
cause of action alleged by private respondent. He actively participated in the proceedings below by filing several pleadings and
cross-examining the witnesses of private respondent. It is crystal clear that every stage of the litigation was characterized by a
no-holds barred contest and not by collusion.
The role of the prosecuting attorney or fiscal in annulment of marriage and legal separation proceedings is to determine whether
collusion exists between the parties and to take care that the evidence is not suppressed or fabricated. Petitioner's vehement
opposition to the annulment proceedings negates the conclusion that collusion existed between the parties. There is no
allegation by the petitioner that evidence was suppressed or fabricated by any of the parties. Under these circumstances, we
are convinced that the non-intervention of a prosecuting attorney to assure lack of collusion between the contending parties is
not fatal to the validity of the proceedings in the trial court.
Petitioner also refutes the testimonies of private respondent's witnesses, particularly Dr. Samuel Wiley and Ms. Adelita Prieto,
as biased, incredible and hearsay. Petitioner alleges that if he were able to present his evidence, he could have testified that he
was not psychologically incapacitated at the time of the marriage as indicated by the fact that during their first ten years, he and

private respondent lived together with their children as one normal and happy family, that he continued supporting his family
even after he left the conjugal dwelling and that his work as owner and operator of a radio and television corporation places him
in the public eye and makes him a good subject for malicious gossip linking him with various women. These facts, according to
petitioner, should disprove the ground for annulment of his marriage to petitioner.
Suffice it to state that the finding of the trial court as to the existence or non-existence of petitioner's psychological incapacity at
the time of the marriage is final and binding on us. 26 Petitioner has not sufficiently shown that the trial court's factual findings
and evaluation of the testimonies of private respondent's witnesses vis-a-vis petitioner's defenses are clearly and manifestly
erroneous. 27
IN VIEW WHEREOF, the petition is denied and the decision dated July 29, 1994 of the Court of Appeals in CA-G.R. CV No.
37925 is affirmed.

ROWENA PADILLA-RUMBAUA,
Petitioner,

G.R. No. 166738


Present:
*

- versus

EDWARD RUMBAUA,
Respondent.

CARPIOMORALES, J.,
Acting
Chairperson,
**
CARPIO,
***
CHICONAZARIO,
****
LEONARDODE CASTRO, and
BRION, JJ.
Promulgated:
August 14,

2009
x --------------------------------------------------------------------------------------- x
DECISION
BRION, J.:

Petitioner Rowena Padilla-Rumbaua (petitioner) challenges, through her petition for review on certiorari,[1] the decision
dated June 25, 2004[2] and the resolution dated January 18, 2005 [3] of the Court of Appeals (CA) in CA-G.R. CV No. 75095. The
challenged decision reversed the decision [4] of the Regional Trial Court (RTC) declaring the marriage of the petitioner and
respondent Edward Rumbaua (respondent) null and void on the ground of the latters psychological incapacity. The assailed
resolution, on the other hand, denied the petitioners motion for reconsideration.

ANTECEDENT FACTS

The present petition traces its roots to the petitioners complaint for the declaration of nullity of marriage against the
respondent before the RTC, docketed as Civil Case No. 767. The petitioner alleged that the respondent was psychologically
incapacitated to exercise the essential obligations of marriage as shown by the following circumstances: the respondent
reneged on his promise to live with her under one roof after finding work; he failed to extend financial support to her; he blamed
her for his mothers death; he represented himself as single in his transactions; and he pretended to be working in Davao,
although he was cohabiting with another woman in Novaliches, Quezon City.

Summons was served on the respondent through substituted service, as personal service proved futile.[5] The RTC
ordered the provincial prosecutor to investigate if collusion existed between the parties and to ensure that no fabrication or
suppression of evidence would take place. [6] Prosecutor Melvin P. Tiongsons report negated the presence of collusion between
the parties.[7]

The Republic of the Philippines (Republic), through the office of the Solicitor General (OSG), opposed the petition.[8] The
OSG entered its appearance and deputized the Provincial Prosecutor of Nueva Vizcaya to assist in all hearings of the case. [9]

The petitioner presented testimonial and documentary evidence to substantiate her charges.

The petitioner related that she and the respondent were childhood neighbors in Dupax del Norte, Nueva
Vizcaya. Sometime in 1987, they met again and became sweethearts but the respondents family did not approve of their
relationship. After graduation from college in 1991, the respondent promised to marry the petitioner as soon as he found a job.
The job came in 1993, when the Philippine Air Lines (PAL) accepted the respondent as a computer engineer. The respondent
proposed to the petitioner that they first have a secret marriage in order not to antagonize his parents. The petitioner agreed;
they were married in Manila on February 23, 1993. The petitioner and the respondent, however, never lived together; the
petitioner stayed with her sister in Fairview, Quezon City, while the respondent lived with his parents in Novaliches.

The petitioner and respondent saw each other every day during the first six months of their marriage. At that point, the
respondent refused to live with the petitioner for fear that public knowledge of their marriage would affect his application for a
PAL scholarship. Seven months into their marriage, the couples daily meetings became occasional visits to the petitioners
house inFairview; they would have sexual trysts in motels. Later that year, the respondent enrolled at FEATI University after he
lost his employment with PAL.[10]

In 1994, the parties respective families discovered their secret marriage. The respondents mother tried to convince him
to go to the United States, but he refused. To appease his mother, he continued living separately from the petitioner. The
respondent forgot to greet the petitioner during her birthday in 1992 and likewise failed to send her greeting cards on special
occasions. The respondent indicated as well in his visa application that he was single.

In April 1995, the respondents mother died. The respondent blamed the petitioner, associating his mothers death to the
pain that the discovery of his secret marriage brought. Pained by the respondents action, the petitioner severed her relationship
with the respondent. They eventually reconciled through the help of the petitioners father, although they still lived separately.

In 1997, the respondent informed the petitioner that he had found a job in Davao. A year later, the petitioner and her
mother went to the respondents house in Novaliches and found him cohabiting with one Cynthia Villanueva ( Cynthia). When
she confronted the respondent about it, he denied having an affair with Cynthia. [11] The petitioner apparently did not believe the
respondents and moved to to Nueva Vizcaya to recover from the pain and anguish that her discovery brought. [12]

The petitioner disclosed during her cross-examination that communication between her and respondent had
ceased. Aside from her oral testimony, the petitioner also presented a certified true copy of their marriage contract; [13] and the
testimony,curriculum vitae,[14] and psychological report[15] of clinical psychologist Dr. Nedy Lorenzo Tayag (Dr. Tayag).

Dr. Tayag declared on the witness stand that she administered the following tests on the petitioner: a Revised Beta
Examination; a Bender Visual Motor Gestalt Test; a Rorschach Psychodiagnostic Test; a Draw a Person Test; a Sachs
Sentence Completion Test; and MMPI. [16] She thereafter prepared a psychological report with the following findings:
TEST RESULTS AND EVALUATION
Psychometric tests data reveal petitioner to operate in an average intellectual level. Logic and reasoning
remained intact. She is seen to be the type of woman who adjusts fairly well into most situations especially if it
is within her interests. She is pictured to be faithful to her commitments and had reservations from negative
criticisms such that she normally adheres to social norms, behavior-wise. Her age speaks of maturity, both
intellectually and emotionally. Her one fault lies in her compliant attitude which makes her a subject for
manipulation and deception such that of respondent. In all the years of their relationship, she opted to endure
his irresponsibility largely because of the mere belief that someday things will be much better for them. But
upon the advent of her husbands infidelity, she gradually lost hope as well as the sense of self-respect, that
she has finally taken her tool to be assertive to the point of being aggressive and very cautious at times so
as to fight with the frustration and insecurity she had especially regarding her failed marriage.
Respondent in this case, is revealed to operate in a very self-centered manner as he believes that
the world revolves around him. His egocentrism made it so easy for him to deceitfully use others for
his own advancement with an extreme air of confidence and dominance. He would do actions without
any remorse or guilt feelings towards others especially to that of petitioner.
REMARKS
Love happens to everyone. It is dubbed to be boundless as it goes beyond the expectations people
tagged with it. In love, age does matter. People love in order to be secure that one will share his/her life with
another and that he/she will not die alone. Individuals who are in love had the power to let love grow or let love
die it is a choice one had to face when love is not the love he/she expected.
In the case presented by petitioner, it is very apparent that love really happened for her towards the
young respondent who used love as a disguise or deceptive tactic for exploiting the confidence she
extended towards him. He made her believe that he is responsible, true, caring and thoughtful only to reveal
himself contrary to what was mentioned. He lacked the commitment, faithfulness, and remorse that he was
able to engage himself to promiscuous acts that made petitioner look like an innocent fool. His character traits
reveal him to suffer Narcissistic Personality Disorder - declared to be grave, severe and incurable.
[17]
[Emphasis supplied.]

The RTC Ruling

The RTC nullified the parties marriage in its decision of April 19, 2002. The trial court saw merit in the testimonies of the
petitioner and Dr. Tayag, and concluded as follows:
xxxx
Respondent was never solicitous of the welfare and wishes of his wife. Respondent imposed limited or block
[sic] out communication with his wife, forgetting special occasions, like petitioners birthdays and Valentines Day;
going out only on occasions despite their living separately and to go to a motel to have sexual intercourse.
It would appear that the foregoing narration are the attendant facts in this case which show the psychological
incapacity of respondent, at the time of the celebration of the marriage of the parties, to enter into lawful marriage
and to discharge his marital responsibilities (See Articles 68 to 71, Family Code). This incapacity is declared grave,
severe and incurable.
WHEREFORE, in view of the foregoing, the marriage between petitioner Rowena Padilla Rumbaua and
respondent Edwin Rumbaua is hereby declared annulled.

SO ORDERED.[18]

The CA Decision

The Republic, through the OSG, appealed the RTC decision to the CA. [19] The CA decision of June 25, 2004 reversed
and set aside the RTC decision, and denied the nullification of the parties marriage. [20]

In its ruling, the CA observed that Dr. Tayags psychiatric report did not mention the cause of the respondents so-called
narcissistic personality disorder; it did not discuss the respondents childhood and thus failed to give the court an insight into
the respondents developmental years. Dr. Tayag likewise failed to explain why she came to the conclusion that the
respondents incapacity was deep-seated and incurable.

The CA held that Article 36 of the Family Code requires the incapacity to be psychological, although its manifestations
may be physical. Moreover, the evidence presented must show that the incapacitated party was mentally or physically ill so that
he or she could not have known the marital obligations assumed, knowing them, could not have assumed them. In other words,
the illness must be shown as downright incapacity or inability, not a refusal, neglect, or difficulty to perform the essential
obligations of marriage. In the present case, the petitioner suffered because the respondent adamantly refused to live with her
because of his parents objection to their marriage.

The petitioner moved to reconsider the decision, but the CA denied her motion in its resolution of January 18, 2005. [21]

The Petition and the Issues

The petitioner argues in the present petition that


1.

the OSG certification requirement under Republic v. Molina[22] (the Molina case) cannot be dispensed with because
A.M. No. 02-11-10-SC, which relaxed the requirement, took effect only on March 15, 2003;

2.

vacating the decision of the courts a quo and remanding the case to the RTC to recall her expert witness and cure
the defects in her testimony, as well as to present additional evidence, would temper justice with mercy; and

3.

Dr. Tayags testimony in court cured the deficiencies in her psychiatric report.

The petitioner prays that the RTCs and the CAs decisions be reversed and set aside, and the case be remanded to the
RTC for further proceedings; in the event we cannot grant this prayer, that the CAs decision be set aside and the RTCs
decision be reinstated.
The Republic maintained in its comment that: (a) A.M. No. 02-11-10-SC was applicable although it took effect after the
promulgation of Molina; (b) invalidating the trial courts decision and remanding the case for further proceedings were not
proper; and (c) the petitioner failed to establish respondents psychological incapacity. [23]

The parties simply reiterated their arguments in the memoranda they filed.

THE COURTS RULING

We resolve to deny the petition for lack of merit.


A.M. No. 02-11-10-SC is applicable

In Molina, the Court emphasized the role of the prosecuting attorney or fiscal and the OSG; they are to appear as counsel
for the State in proceedings for annulment and declaration of nullity of marriages:
(8) The trial court must order the prosecuting attorney or fiscal and the Solicitor General to appear as counsel
for the state. No decision shall be handed down unless the Solicitor General issues a certification, which will
be quoted in the decision, briefly stating therein his reasons for his agreement or opposition, as the case
may be, to the petition. The Solicitor General, along with the prosecuting attorney, shall submit to the court
such certification within fifteen (15) days from the date the case is deemed submitted for resolution of the court. The
Solicitor General shall discharge the equivalent function of the defensor vinculi contemplated under Canon 1095.
[Emphasis supplied.]

A.M. No. 02-11-10-SC[24] -- which this Court promulgated on March 15, 2003 and duly published -- is geared towards the
relaxation of the OSG certification that Molina required. Section 18 of this remedial regulation provides:
SEC. 18. Memoranda. The court may require the parties and the public prosecutor, in consultation with the
Office of the Solicitor General, to file their respective memoranda in support of their claims within fifteen days from
the date the trial is terminated. It may require the Office of the Solicitor General to file its own memorandum if the
case is of significant interest to the State. No other pleadings or papers may be submitted without leave of court.
After the lapse of the period herein provided, the case will be considered submitted for decision, with or without the
memoranda.

The petitioner argues that the RTC decision of April 19, 2002 should be vacated for prematurity, as it was rendered
despite the absence of the required OSG certification specified in Molina. According to the petitioner, A.M. No. 02-11-10-SC,
which took effect only on March 15, 2003, cannot overturn the requirements of Molina that was promulgated as early
as February 13, 1997.
The petitioners argument lacks merit.
The amendment introduced under A.M. No. 02-11-10-SC is procedural or remedial in character; it does not create or
remove any vested right, but only operates as a remedy in aid of or confirmation of already existing rights. The settled rule is
that procedural laws may be given retroactive effect, [25] as we held in De Los Santos v. Vda. de Mangubat:[26]
Procedural Laws do not come within the legal conception of a retroactive law, or the general rule against the
retroactive operation of statues - they may be given retroactive effect on actions pending and undetermined at the
time of their passage and this will not violate any right of a person who may feel that he is adversely affected,
insomuch as there are no vested rights in rules of procedure.

A.M. No. 02-11-10-SC, as a remedial measure, removed the mandatory nature of an OSG certification and may be
applied retroactively to pending matters. In effect, the measure cures in any pending matter any procedural lapse on the
certification prior to its promulgation. Our rulings in Antonio v. Reyes[27] and Navales v. Navales[28] have since confirmed and
clarified that A.M. No. 02-11-10-SC has dispensed with the Molina guideline on the matter of certification, although Article 48
mandates the appearance of the prosecuting attorney or fiscal to ensure that no collusion between the parties would take place. Thus,
what is important is the presence of the prosecutor in the case, not the remedial requirement that he be certified to be present. From this
perspective, the petitioners objection regarding the Molina guideline on certification lacks merit.

A Remand of the Case to the RTC is Improper

The petitioner maintains that vacating the lower courts decisions and the remand of the case to the RTC for further reception of
evidence are procedurally permissible. She argues that the inadequacy of her evidence during the trial was the fault of her former counsel,
Atty. Richard Tabago, and asserts that remanding the case to the RTC would allow her to cure the evidentiary insufficiencies. She posits in
this regard that while mistakes of counsel bind a party, the rule should be liberally construed in her favor to serve the ends of justice.
We do not find her arguments convincing.
A remand of the case to the RTC for further proceedings amounts to the grant of a new trial that is not procedurally proper at this
stage. Section 1 of Rule 37 provides that an aggrieved party may move the trial court to set aside a judgment or final
order already rendered and to grant a new trial within the period for taking an appeal. In addition, a motion for new trial may be filed
only on the grounds of (1) fraud, accident, mistake or excusable negligence that could not have been guarded against by
ordinary prudence, and by reason of which the aggrieved partys rights have probably been impaired; or (2) newly discovered
evidence that, with reasonable diligence, the aggrieved party could not have discovered and produced at the trial, and that
would probably alter the result if presented.
In the present case, the petitioner cites the inadequacy of the evidence presented by her former counsel as basis for a
remand. She did not, however, specify the inadequacy. That the RTC granted the petition for declaration of nullity prima
facie shows that the petitioners counsel had not been negligent in handling the case. Granting arguendo that the petitioners
counsel had been negligent, the negligence that would justify a new trial must be excusable, i.e. one that ordinary diligence and
prudence could not have guarded against. The negligence that the petitioner apparently adverts to is that cited in Uy v. First
Metro Integrated Steel Corporation where we explained:[29]
Blunders and mistakes in the conduct of the proceedings in the trial court as a result of the ignorance,
inexperience or incompetence of counsel do not qualify as a ground for new trial. If such were to be admitted as
valid reasons for re-opening cases, there would never be an end to litigation so long as a new counsel could be
employed to allege and show that the prior counsel had not been sufficiently diligent, experienced or learned. This

will put a premium on the willful and intentional commission of errors by counsel, with a view to securing new trials in
the event of conviction, or an adverse decision, as in the instant case.
Thus, we find no justifiable reason to grant the petitioners requested remand.
Petitioner failed to establish the
respondents psychological incapacity

A petition for declaration of nullity of marriage is anchored on

Article 36 of the Family Code which provides that a marriage contracted by any party who, at the time of its celebration,
was psychologically incapacitated to comply with the essential marital obligations of marriage, shall likewise be void even if such
incapacity becomes manifest only after its solemnization. In Santos v. Court of Appeals,[30] the Court first declared that
psychological incapacity must be characterized by (a) gravity; (b) juridical antecedence; and (c) incurability. The defect should
refer to no less than a mental (not physical) incapacity that causes a party to be truly incognitive of the basic marital covenants
that concomitantly must be assumed and discharged by the parties to the marriage. It must be confined to the most serious
cases of personality disorders clearly demonstrative of an utter insensitivity or inability to give meaning and significance to the
marriage.

We laid down more definitive guidelines in the interpretation and application of Article 36 of the Family Code in Republic v.
Court of Appeals where we said:
(1) The burden of proof to show the nullity of the marriage belongs to the plaintiff. Any doubt should be
resolved in favor of the existence and continuation of the marriage and against its dissolution and nullity. This is
rooted in the fact that both our Constitution and our laws cherish the validity of marriage and unity of the family.
Thus, our Constitution devotes an entire Article on the Family, recognizing it as the foundation of the nation. It
decrees marriage as legally inviolable, thereby protecting it from dissolution at the whim of the parties. Both the
family and marriage are to be protected by the state.
The Family Code echoes this constitutional edict on marriage and the family and emphasizes their
permanence, inviolability and solidarity.
(2) The root cause of the psychological incapacity must be
(a) medically or clinically identified, (b)
alleged in the complaint,
(c) sufficiently proven by experts and (d) clearly explained in the decision. Article
36 of the Family Code requires that the incapacity must bepsychological - not physical, although its manifestations
and/or symptoms may be physical. The evidence must convince the court that the parties, or one of them, was
mentally or psychically ill to such an extent that the person could not have known the obligations he was assuming,
or knowing them, could not have given valid assumption thereof. Although no example of such incapacity need be
given here so as not to limit the application of the provision under the principle of ejusdem generis, nevertheless
such root cause must be identified as a psychological illness and its incapacitating nature fully explained. Expert
evidence may be given by qualified psychiatrists and clinical psychologists.
(3) The incapacity must be proven to be existing at the time of the celebration of the marriage. The evidence
must show that the illness was existing when the parties exchanged their I do's. The manifestation of the illness
need not be perceivable at such time, but the illness itself must have attached at such moment, or prior thereto.
(4) Such incapacity must also be shown to be medically or clinically permanent or incurable. Such incurability
may be absolute or even relative only in regard to the other spouse, not necessarily absolutely against everyone of
the same sex. Furthermore, such incapacity must be relevant to the assumption of marriage obligations, not
necessarily to those not related to marriage, like the exercise of a profession or employment in a job. x x x
(5) Such illness must be grave enough to bring about the disability of the party to assume the essential
obligations of marriage. Thus, mild characteriological peculiarities, mood changes, occasional emotional outbursts
cannot be accepted as root causes. The illness must be shown as downright incapacity or inability, not a refusal,
neglect or difficulty, much less ill will. In other words, there is a natal or supervening disabling factor in the person,
an adverse integral element in the personality structure that effectively incapacitates the person from really
accepting and thereby complying with the obligations essential to marriage.
(6) The essential marital obligations must be those embraced by Articles 68 up to 71 of the Family Code as
regards the husband and wife as well as Articles 220, 221 and 225 of the same Code in regard to parents and their
children. Such non-complied marital obligation(s) must also be stated in the petition, proven by evidence and
included in the text of the decision.
(7) Interpretations given by the National Appellate Matrimonial Tribunal of the Catholic Church in
the Philippines, while not controlling or decisive, should be given great respect by our courts
(8) The trial court must order the prosecuting attorney or fiscal and the Solicitor General to appear as counsel
for the state. No decision shall be handed down unless the Solicitor General issues a certification, which will be
quoted in the decision, briefly stating therein his reasons for his agreement or opposition, as the case may be, to the
petition. The Solicitor General, along with the prosecuting attorney, shall submit to the court such certification within
fifteen (15) days from the date the case is deemed submitted for resolution of the court. The Solicitor General shall
discharge the equivalent function of the defensor vinculi contemplated under Canon 1095.

These Guidelines incorporate the basic requirements we established in Santos. To reiterate, psychological incapacity
must be characterized by:

(a) gravity; (b) juridical antecedence; and (c) incurability.[31] These requisites must be strictly

complied with, as the grant of a petition for nullity of marriage based on psychological incapacity must be confined only to the
most serious cases of personality disorders clearly demonstrative of an utter insensitivity or inability to give meaning and
significance to the marriage. Furthermore, since the Family Code does not define psychological incapacity, fleshing out its
terms is left to us to do so on a case-to-case basis through jurisprudence. [32] We emphasized this approach in the recent case
of Ting v. Velez-Ting[33] when we explained:
It was for this reason that we found it necessary to emphasize in Ngo Te that each case involving the
application of Article 36 must be treated distinctly and judged not on the basis of a priori assumptions, predilections
or generalizations but according to its own attendant facts. Courts should interpret the provision on a case-to-case
basis, guided by experience, the findings of experts and researchers in psychological disciplines, and by decisions
of church tribunals.

In the present case and using the above standards and approach, we find the totality of the petitioners evidence
insufficient to prove that the respondent is psychologically unfit to discharge the duties expected of him as a husband.
a.

Petitioners testimony did not prove the root cause, gravity and incurability of respondents condition

The petitioners evidence merely showed that the respondent:

(a) reneged on his promise to cohabit with her; (b)

visited her occasionally from 1993 to 1997; (c) forgot her birthday in 1992, and did not send her greeting cards during special
occasions; (d) represented himself as single in his visa application; (e) blamed her for the death of his mother; and (f) told her
he was working in Davao when in fact he was cohabiting with another woman in 1997.

These acts, in our view, do not rise to the level of the psychological incapacity that the law requires, and should be
distinguished from the difficulty, if not outright refusal or neglect in the performance of some marital obligations that
characterize some marriages. In Bier v. Bier,[34] we ruled that it was not enough that respondent, alleged to be psychologically
incapacitated, had difficulty in complying with his marital obligations, or was unwilling to perform these obligations. Proof of a
natal or supervening disabling factor an adverse integral element in the respondent's personality structure that effectively
incapacitated him from complying with his essential marital obligations had to be shown and was not shown in this cited case.
In the present case, the respondents stubborn refusal to cohabit with the petitioner was doubtlessly irresponsible, but it
was never proven to be rooted in some psychological illness. As the petitioners testimony reveals, respondent merely
refused to cohabit with her for fear of jeopardizing his application for a scholarship, and later due to his fear of antagonizing his
family. The respondents failure to greet the petitioner on her birthday and to send her cards during special occasions, as well as
his acts of blaming petitioner for his mothers death and of representing himself as single in his visa application, could only at
best amount to forgetfulness, insensitivity or emotional immaturity, not necessarily psychological incapacity. Likewise, the
respondents act of living with another woman four years into the marriage cannot automatically be equated with a psychological
disorder, especially when no specific evidence was shown that promiscuity was a trait already existing at the inception of
marriage. In fact, petitioner herself admitted that respondent was caring and faithful when they were going steady and for a time
after their marriage; their problems only came in later.
To be sure, the respondent was far from perfect and had some character flaws. The presence of these imperfections,
however, does not necessarily warrant a conclusion that he had a psychological malady at the time of the marriage that
rendered him incapable of fulfilling his duties and obligations. To use the words of Navales v. Navales:[35]
Article 36 contemplates downright incapacity or inability to take cognizance of and to assume basic marital obligations.
Mere difficulty, refusal or neglect in the performance of marital obligations or ill will on the part of the spouse is different
from incapacity rooted on some debilitating psychological condition or illness.Indeed, irreconcilable differences, sexual
infidelity or perversion, emotional immaturity and irresponsibility, and the like, do not by themselves warrant a
finding of psychological incapacity under Article 36, as the same may only be due to a person's refusal or
unwillingness to assume the essential obligations of marriage and not due to some psychological illness that is
contemplated by said rule.

b.

Dr. Tayags psychological report and court testimony

We cannot help but note that Dr. Tayags conclusions about the respondents psychological incapacity were based on the
information fed to her by only one side the petitioner whose bias in favor of her cause cannot be doubted. While this
circumstance alone does not disqualify the psychologist for reasons of bias, her report, testimony and conclusions deserve the
application of a more rigid and stringent set of standards in the manner we discussed above. [36] For, effectively, Dr. Tayag only
diagnosed the respondent from the prism of a third party account; she did not actually hear, see and evaluate the respondent
and how he would have reacted and responded to the doctors probes.

Dr. Tayag, in her report, merely summarized the petitioners narrations, and on this basis characterized the respondent to
be a self-centered, egocentric, and unremorseful person who believes that the world revolves around him; and who used love
as adeceptive tactic for exploiting the confidence [petitioner] extended towards him. Dr. Tayag then incorporated her own
idea of love; made a generalization that respondent was a person who lacked commitment, faithfulness, and remorse, and
who engaged in promiscuous acts that made the petitioner look like a fool; and finally concluded that the respondents
character traits reveal him to suffer Narcissistic Personality Disorder with traces of Antisocial Personality Disorder declared to
be grave and incurable.

We find these observations and conclusions insufficiently in-depth and comprehensive to warrant the conclusion that a
psychological incapacity existed that prevented the respondent from complying with the essential obligations of marriage. It
failed to identify the root cause of the respondent's narcissistic personality disorder and to prove that it existed at the inception
of the marriage. Neither did it explain the incapacitating nature of the alleged disorder, nor show that the respondent was really
incapable of fulfilling his duties due to some incapacity of a psychological, not physical, nature. Thus, we cannot avoid but

conclude that Dr. Tayags conclusion in her Report i.e., that the respondent suffered Narcissistic Personality Disorder with
traces of Antisocial Personality Disorder declared to be grave and incurable is an unfounded statement, not a necessary
inference from her previous characterization and portrayal of the respondent. While the various tests administered on the
petitioner could have been used as a fair gauge to assess her own psychological condition, this same statement cannot be
made with respect to the respondents condition. To make conclusions and generalizations on the respondents psychological
condition based on the information fed by only one side is, to our mind, not different from admitting hearsay evidence as proof of
the truthfulness of the content of such evidence.

Petitioner nonetheless contends that Dr. Tayags subsequent testimony in court cured whatever deficiencies attended her
psychological report.

We do not share this view.

A careful reading of Dr. Tayags testimony reveals that she failed to establish the fact that at the time the parties were
married, respondent was already suffering from a psychological defect that deprived him of the ability to assume the essential
duties and responsibilities of marriage. Neither did she adequately explain how she came to the conclusion that respondents
condition was grave and incurable. To directly quote from the records:
ATTY. RICHARD TABAGO:
Q:
I would like to call your attention to the Report already marked as Exh. E-7, there is a statement to
the effect that his character traits begin to suffer narcissistic personality disorder with traces of antisocial
personality disorder. What do you mean? Can you please explain in laymans word, Madam Witness?
DR. NEDY LORENZO TAYAG:
A:

Actually, in a laymans term, narcissistic personality disorder cannot accept that there is something
wrong with his own behavioral manifestation. [sic] They feel that they can rule the world; they are
eccentric; they are exemplary, demanding financial and emotional support, and this is clearly manifested
by the fact that respondent abused and used petitioners love. Along the line, a narcissistic person
cannot give empathy; cannot give love simply because they love themselves more than anybody else;
and thirdly, narcissistic person cannot support his own personal need and gratification without the help of
others and this is where the petitioner set in.

Q:
A:

Clinically, considering that label, the respondent behavioral manifestation under personality disorder
[sic] this is already considered grave, serious, and treatment will be impossible [sic]. As I say this, a kind
of developmental disorder wherein it all started during the early formative years and brought about by
one familiar relationship the way he was reared and cared by the family. Environmental exposure is also
part and parcel of the child disorder. [sic]

Q:
A:

Can you please describe the personal [sic] disorder?

You mean to say, from the formative [years] up to the present?

Actually, the respondent behavioral manner was [present] long before he entered marriage.
[Un]fortunately, on the part of the petitioner, she never realized that such behavioral manifestation of the
respondent connotes pathology. [sic]
xxxx

Q:
So in the representation of the petitioner that the respondent is now lying [sic] with somebody else,
how will you describe the character of this respondent who is living with somebody else?
A:

This is where the antisocial personality trait of the respondent [sic] because an antisocial person is
one who indulge in philandering activities, who do not have any feeling of guilt at the expense of another
person, and this [is] again a buy-product of deep seated psychological incapacity.

Q:
And this psychological incapacity based on this particular deep seated [sic], how would you
describe the psychological incapacity? [sic]
A:

As I said there is a deep seated psychological dilemma, so I would say incurable in nature and at
this time and again [sic] the psychological pathology of the respondent. One plays a major factor of not
being able to give meaning to a relationship in terms of sincerity and endurance.

Q:
And if this psychological disorder exists before the marriage of the respondent and the petitioner,
Madam Witness?
A:

Clinically, any disorder are usually rooted from the early formative years and so if it takes enough
that such psychological incapacity of respondent already existed long before he entered marriage,
because if you analyze how he was reared by her parents particularly by the mother, there is already an
unhealthy symbiosis developed between the two, and this creates a major emotional havoc when he
reached adult age.

Q:
A:

How about the gravity?

This is already grave simply because from the very start respondent never had an inkling that his
behavioral manifestation connotes pathology and second ground [sic], respondent will never admit again
that such behavior of his connotes again pathology simply because the disorder of the respondent is not
detrimental to himself but, more often than not, it is detrimental to other party involved.
xxxx

PROSECUTOR MELVIN TIONGSON:


Q:

You were not able to personally examine the respondent here?

DR. NEDY TAYAG:


A:

Efforts were made by the psychologist but unfortunately, the respondent never appeared at my
clinic.

Q:
On the basis of those examinations conducted with the petitioning wife to annul their marriage with
her husband in general, what can you say about the respondent?
A:

That from the very start respondent has no emotional intent to give meaning to their relationship. If
you analyze their marital relationship they never lived under one room. From the very start of the
[marriage], the respondent to have petitioner to engage in secret marriage until that time their family
knew of their marriage [sic]. Respondent completely refused, completely relinquished his marital
obligation to the petitioner.
xxxx

COURT:
Q:
Because you have interviewed or you have questioned the petitioner, can you really enumerate the
specific traits of the respondent?
DR. NEDY TAYAG:
A:

One is the happy-go-lucky attitude of the respondent and the dependent attitude of the respondent.

Q:

Even if he is already eligible for employment?

A:

He remains to be at the mercy of his mother. He is a happy-go-lucky simply because he never had
a set of responsibility. I think that he finished his education but he never had a stable job because he
completely relied on the support of his mother.

Q:
A:

You give a more thorough interview so I am asking you something specific?

The happy-go-lucky attitude; the overly dependent attitude on the part of the mother merely
because respondent happened to be the only son. I said that there is a unhealthy symbiosis relationship
[sic] developed between the son and the mother simply because the mother always pampered
completely, pampered to the point that respondent failed to develop his own sense of assertion or
responsibility particularly during that stage and there is also presence of the simple lying act particularly
his responsibility in terms of handling emotional imbalance and it is clearly manifested by the fact that
respondent refused to build a home together with the petitioner when in fact they are legally married.
Thirdly, respondent never felt or completely ignored the feelings of the petitioner; he never felt guilty
hurting the petitioner because on the part of the petitioner, knowing that respondent indulge with another
woman it is very, very traumatic on her part yet respondent never had the guts to feel guilty or to atone
said act he committed in their relationship, and clinically this falls under antisocial personality. [37]

In terms of incurability, Dr. Tayags answer was very vague and inconclusive, thus:
xxxx
ATTY. RICHARD TABAGO
Q:

Can this personally be cured, madam witness?


DR. NEDY TAYAG
A:

Clinically, if persons suffering from personality disorder curable, up to this very moment, no scientific
could be upheld to alleviate their kind of personality disorder; Secondly, again respondent or other
person suffering from any kind of disorder particularly narcissistic personality will never admit that they
are suffering from this kind of disorder, and then again curability will always be a question.[sic][38]

This testimony shows that while Dr. Tayag initially described the general characteristics of a person suffering from a
narcissistic personality disorder, she did not really show how and to what extent the respondent exhibited these traits. She
mentioned the buzz words that jurisprudence requires for the nullity of a marriage namely, gravity, incurability, existence at the
time of the marriage, psychological incapacity relating to marriage and in her own limited way, related these to the medical
condition she generally described. The testimony, together with her report, however, suffers from very basic flaws.

First, what she medically described was not related or linked to the respondents exact condition except in a very general
way. In short, her testimony and report were rich in generalities but disastrously short on particulars, most notably on how the
respondent can be said to be suffering from narcissistic personality disorder; why and to what extent the disorder is grave and
incurable; how and why it was already present at the time of the marriage; and the effects of the disorder on the respondents
awareness of and his capability to undertake the duties and responsibilities of marriage. All these are critical to the success of
the petitioners case.

Second, her testimony was short on factual basis for her diagnosis because it was wholly based on what the petitioner
related to her. As the doctor admitted to the prosecutor, she did not at all examine the respondent, only the petitioner. Neither

the law nor jurisprudence requires, of course, that the person sought to be declared psychologically incapacitated should be
personally examined by a physician or psychologist as a condition sine qua non to arrive at such declaration.[39] If a
psychological disorder can be proven by independent means, no reason exists why such independent proof cannot be admitted
and given credit.[40] No such independent evidence, however, appears on record to have been gathered in this case, particularly
about the respondents early life and associations, and about events on or about the time of the marriage and immediately
thereafter. Thus, the testimony and report appear to us to be no more than a diagnosis that revolves around the one-sided and
meager facts that the petitioner related, and were all slanted to support the conclusion that a ground exists to justify the
nullification of the marriage. We say this because only the baser qualities of the respondents life were examined and given
focus; none of these qualities were weighed and balanced with the better qualities, such as his focus on having a job, his
determination to improve himself through studies, his care and attention in the first six months of the marriage, among
others. The evidence fails to mention also what character and qualities the petitioner brought into her marriage, for example,
why the respondents family opposed the marriage and what events led the respondent to blame the petitioner for the death of
his mother, if this allegation is at all correct. To be sure, these are important because not a few marriages have failed, not
because of psychological incapacity of either or both of the spouses, but because of basic incompatibilities and marital
developments that do not amount to psychological incapacity. The continued separation of the spouses likewise never
appeared to have been factored in. Not a few married couples have likewise permanently separated simply because they have
fallen out of love, or have outgrown the attraction that drew them together in their younger years.

Thus, on the whole, we do not blame the petitioner for the move to secure a remand of this case to the trial courts for the
introduction of additional evidence; the petitioners evidence in its present state is woefully insufficient to support the conclusion
that the petitioners marriage to the respondent should be nullified on the ground of the respondents psychological incapacity.

The Court commiserates with the petitioners marital predicament. The respondent may indeed be unwilling to discharge
his marital obligations, particularly the obligation to live with ones spouse. Nonetheless, we cannot presume psychological
defect from the mere fact that respondent refuses to comply with his marital duties. As we ruled in Molina, it is not enough to
prove that a spouse failed to meet his responsibility and duty as a married person; it is essential that he must be
shown to be incapable of doing so due to some psychological illness. The psychological illness that must afflict a
party at the inception of the marriage should be a malady so grave and permanent as to deprive the party of his or her
awareness of the duties and responsibilities of the matrimonial bond he or she was then about to assume.[41]

WHEREFORE, in view of these considerations, we DENY the petition and AFFIRM the decision and resolution of the
Court of Appeals dated June 25, 2004 and January 18, 2005, respectively, in CA-G.R. CV No. 75095.
SO ORDERED.

Legal Separation
G.R. No. L-30977 January 31, 1972
CARMEN LAPUZ SY, represented by her substitute MACARIO LAPUZ, petitioner-appellant,
vs.
EUFEMIO S. EUFEMIO alias EUFEMIO SY UY, respondent-appellee.
Jose W. Diokno for petitioner-appellant.
D. G. Eufemio for respondent-appellee.

REYES J.B.L., J.:p


Petition, filed after the effectivity of Republic Act 5440, for review by certiorari of an order, dated 29 July 1969, of the Juvenile
and Domestic Relations Court of Manila, in its Civil Case No. 20387, dismissing said case for legal separation on the ground
that the death of the therein plaintiff, Carmen O. Lapuz Sy, which occurred during the pendency of the case, abated the cause of
action as well as the action itself. The dismissal order was issued over the objection of Macario Lapuz, the heir of the deceased
plaintiff (and petitioner herein) who sought to substitute the deceased and to have the case prosecuted to final judgment.
On 18 August 1953, Carmen O. Lapuz Sy filed a petition for legal separation against Eufemio S. Eufemio, alleging, in the main,
that they were married civilly on 21 September 1934 and canonically on 30 September 1934; that they had lived together as
husband and wife continuously until 1943 when her husband abandoned her; that they had no child; that they acquired
properties during their marriage; and that she discovered her husband cohabiting with a Chinese woman named Go Hiok at
1319 Sisa Street, Manila, on or about March 1949. She prayed for the issuance of a decree of legal separation, which, among
others, would order that the defendant Eufemio S. Eufemio should be deprived of his share of the conjugal partnership profits.
In his second amended answer to the petition, herein respondent Eufemio S. Eufemio alleged affirmative and special defenses,
and, along with several other claims involving money and other properties, counter-claimed for the declaration of nullity ab
initio of his marriage with Carmen O. Lapuz Sy, on the ground of his prior and subsisting marriage, celebrated according to
Chinese law and customs, with one Go Hiok, alias Ngo Hiok.
Issues having been joined, trial proceeded and the parties adduced their respective evidence. But before the trial could be
completed (the respondent was already scheduled to present surrebuttal evidence on 9 and 18 June 1969), petitioner Carmen
O. Lapuz Sy died in a vehicular accident on 31 May 1969. Counsel for petitioner duly notified the court of her death.
On 9 June 1969, respondent Eufemio moved to dismiss the "petition for legal separation" 1 on two (2) grounds, namely: that the
petition for legal separation was filed beyond the one-year period provided for in Article 102 of the Civil Code; and that the death
of Carmen abated the action for legal separation.
On 26 June 1969, counsel for deceased petitioner moved to substitute the deceased Carmen by her father, Macario Lapuz.
Counsel for Eufemio opposed the motion.
On 29 July 1969, the court issued the order under review, dismissing the case. 2 In the body of the order, the court stated that
the motion to dismiss and the motion for substitution had to be resolved on the question of whether or not the plaintiff's cause of
action has survived, which the court resolved in the negative. Petitioner's moved to reconsider but the motion was denied on 15
September 1969.
After first securing an extension of time to file a petition for review of the order of dismissal issued by the juvenile and domestic
relations court, the petitioner filed the present petition on 14 October 1969. The same was given due course and answer thereto
was filed by respondent, who prayed for the affirmance of the said order. 3
Although the defendant below, the herein respondent Eufemio S. Eufemio, filed counterclaims, he did not pursue them after the
court below dismissed the case. He acquiesced in the dismissal of said counterclaims by praying for the affirmance of the order
that dismissed not only the petition for legal separation but also his counterclaim to declare the Eufemio-Lapuz marriage to be
null and void ab initio.
But petitioner Carmen O. Lapuz Sy (through her self-assumed substitute for the lower court did not act on the motion for
substitution) stated the principal issue to be as follows:
When an action for legal separation is converted by the counterclaim into one for a declaration of nullity of a
marriage, does the death of a party abate the proceedings?
The issue as framed by petitioner injects into it a supposed conversion of a legal separation suit to one for declaration of nullity
of a marriage, which is without basis, for even petitioner asserted that "the respondent has acquiesced to the dismissal of his
counterclaim" (Petitioner's Brief, page 22). Not only this. The petition for legal separation and the counterclaim to declare the
nullity of the self same marriage can stand independent and separate adjudication. They are not inseparable nor was the action
for legal separation converted into one for a declaration of nullity by the counterclaim, for legal separation pre-supposes a valid
marriage, while the petition for nullity has a voidable marriage as a pre-condition.
The first real issue in this case is: Does the death of the plaintiff before final decree, in an action for legal separation, abate the
action? If it does, will abatement also apply if the action involves property rights? .
An action for legal separation which involves nothing more than the bed-and-board separation of the spouses (there being no
absolute divorce in this jurisdiction) is purely personal. The Civil Code of the Philippines recognizes this in its Article 100, by
allowing only the innocent spouse (and no one else) to claim legal separation; and in its Article 108, by providing that the
spouses can, by their reconciliation, stop or abate the proceedings and even rescind a decree of legal separation already
rendered. Being personal in character, it follows that the death of one party to the action causes the death of the action itself
actio personalis moritur cum persona.
... When one of the spouses is dead, there is no need for divorce, because the marriage is dissolved. The heirs
cannot even continue the suit, if the death of the spouse takes place during the course of the suit (Article 244,
Section 3). The action is absolutely dead (Cass., July 27, 1871, D. 71. 1. 81; Cass. req., May 8, 1933, D. H. 1933,
332.") 4 .
Marriage is a personal relation or status, created under the sanction of law, and an action for divorce is a proceeding
brought for the purpose of effecting a dissolution of that relation. The action is one of a personal nature. In the
absence of a statute to the contrary, the death of one of the parties to such action abates the action, for the reason
that death has settled the question of separation beyond all controversy and deprived the court of jurisdiction, both
over the persons of the parties to the action and of the subject-matter of the action itself. For this reason the courts
are almost unanimous in holding that the death of either party to a divorce proceeding, before final decree, abates
the action. 1 Corpus Juris, 208; Wren v. Moss, 2 Gilman, 72; Danforth v. Danforth, 111 Ill. 236; Matter of Grandall,
196 N.Y. 127, 89 N.E. 578; 134 Am St. Rep. 830; 17 Ann. Cas. 874; Wilcon v. Wilson, 73 Mich, 620, 41 N.W. 817;

Strickland v. Strickland, 80 Ark. 452, 97 S. W. 659; McCurley v. McCurley, 60 Md. 185, 45 Am. Rep. 717; Begbie v.
Begbie, 128 Cal. 155, 60 Pac. 667, 49 L.R.A. 141. 5
The same rule is true of causes of action and suits for separation and maintenance (Johnson vs. Bates, Ark. 101 SW 412; 1
Corpus Juris 208).
A review of the resulting changes in property relations between spouses shows that they are solely the effect of the decree of
legal separation; hence, they can not survive the death of the plaintiff if it occurs prior to the decree. On the point, Article 106 of
the Civil Code provides: .
Art. 106. The decree of legal separation shall have the following effects:
(1) The spouses shall be entitled to live separately from each other, but the marriage bonds shall not be severed; .
(2) The conjugal partnership of gains or the absolute conjugal community of property shall be dissolved and
liquidated, but the offending spouse shall have no right to any share of the profits earned by the partnership or
community, without prejudice to the provisions of article 176;
(3) The custody of the minor children shall be awarded to the innocent spouse, unless otherwise directed by the
court in the interest of said minors, for whom said court may appoint a guardian;
(4) The offending spouse shall be disqualified from inheriting from the innocent spouse by intestate succession.
Moreover, provisions in favor of the offending spouse made in the will of the innocent one shall be revoked by
operation of law.
From this article it is apparent that the right to the dissolution of the conjugal partnership of gains (or of the absolute community
of property), the loss of right by the offending spouse to any share of the profits earned by the partnership or community, or his
disqualification to inherit by intestacy from the innocent spouse as well as the revocation of testamentary provisions in favor of
the offending spouse made by the innocent one, are all rights and disabilities that, by the very terms of the Civil Code article,
are vested exclusively in the persons of the spouses; and by their nature and intent, such claims and disabilities are difficult to
conceive as assignable or transmissible. Hence, a claim to said rights is not a claim that "is not thereby extinguished" after a
party dies, under Section 17, Rule 3, of the Rules of Court, to warrant continuation of the action through a substitute of the
deceased party.
Sec. 17. Death of party. After a party dies and the claim is not thereby extinguished, the court shall order, upon
proper notice, the legal representative of the deceased to appear and to be substituted for the deceased, within a
period of thirty (30) days, or within such time as may be granted...
The same result flows from a consideration of the enumeration of the actions that survive for or against administrators in
Section 1, Rule 87, of the Revised Rules of Court:
SECTION 1. Actions which may and which may not be brought against executor or administrator. No action upon a
claim for the recovery of money or debt or interest thereon shall be commenced against the executor or
administrator; but actions to recover real or personal property, or an interest therein, from the estate, or to enforce a
lien thereon, and actions to recover damages for an injury to person or property, real or personal, may be
commenced against him.
Neither actions for legal separation or for annulment of marriage can be deemed fairly included in the enumeration..
A further reason why an action for legal separation is abated by the death of the plaintiff, even if property rights are involved, is
that these rights are mere effects of decree of separation, their source being the decree itself; without the decree such rights do
not come into existence, so that before the finality of a decree, these claims are merely rights in expectation. If death
supervenes during the pendency of the action, no decree can be forthcoming, death producing a more radical and definitive
separation; and the expected consequential rights and claims would necessarily remain unborn.
As to the petition of respondent-appellee Eufemio for a declaration of nullity ab initio of his marriage to Carmen Lapuz, it is
apparent that such action became moot and academic upon the death of the latter, and there could be no further interest in
continuing the same after her demise, that automatically dissolved the questioned union. Any property rights acquired by either
party as a result of Article 144 of the Civil Code of the Philippines 6 could be resolved and determined in a proper action for
partition by either the appellee or by the heirs of the appellant.
In fact, even if the bigamous marriage had not been void ab initio but only voidable under Article 83, paragraph 2, of the Civil
Code, because the second marriage had been contracted with the first wife having been an absentee for seven consecutive
years, or when she had been generally believed dead, still the action for annulment became extinguished as soon as one of the
three persons involved had died, as provided in Article 87, paragraph 2, of the Code, requiring that the action for annulment
should be brought during the lifetime of any one of the parties involved. And furthermore, the liquidation of any conjugal
partnership that might have resulted from such voidable marriage must be carried out "in the testate or intestate proceedings of
the deceased spouse", as expressly provided in Section 2 of the Revised Rule 73, and not in the annulment proceeding.
ACCORDINGLY, the appealed judgment of the Manila Court of Juvenile and Domestic Relations is hereby affirmed. No special
pronouncement as to costs.

G.R. No. 79284 November 27, 1987


FROILAN C. GANDIONCO, petitioner,
vs.
HON. SENEN C. PEARANDA, as Presiding Judge of the Regional Trial Court of Misamis Oriental, Branch 18, Cagayan
de Oro City, and TERESITA S. GANDIONCO, respondents.

PADILLA, J.:
A special civil action for certiorari, with application for injunction, to annul (1) the Order of the respondent Judge, dated 10
December 1986, ordering petitioner to pay support pendente lite to private respondent (his wife) and their child, and (2) the
Order of the same respondent Judge, dated 5 August 1987, denying petitioner's motion to suspend hearings in the action for
legal separation filed against him by private respondent as well as his motion to inhibit respondent Judge from further hearing
and trying the case.
On 29 May 1986, private respondent, the legal wife of the petitioner, filed with the Regional Trial Court of Misamis Oriental, 10th
Judicial District, Branch 18, in Cagayan de Oro City, presided over by respondent Judge, a complaint against petitioner for legal
separation, on the ground of concubinage, with a petition for support and payment of damages. This case was docketed as Civil
Case No. 10636. On 13 October 1986, private respondent also filed with the Municipal Trial Court, General Santos City, a
complaint against petitioner for concubinage, which was docketed on 23 October 1986 as Criminal Case No. 15437111. On 14
November 1986, application for the provisional remedy of support pendente lite, pending a decision in the action for legal
separation, was filed by private respondent in the civil case for legal separation. The respondent judge, as already stated, on 10
December 1986, ordered The payment of support pendente lite.
In this recourse, petitioner contends that the civil action for legal separation and the incidents consequent thereto, such as,
application for support pendente lite, should be suspended in view of the criminal case for concubinage filed against him the
private respondent. In support of his contention, petitioner cites Art. III. Sec. 3 of the 1985 Rules on Criminal Procedure, which
states:
SEC. 3. Other Civil action arising from offenses. Whenever the offended party shall have instituted the civil action
to enforce the civil liability arising from the offense. as contemplated in the first Section 1 hereof, the following rules
shall be observed:
(a) After a criminal action has been commenced the pending civil action arising from the same offense shall be
suspended, in whatever stage it may be found, until final judgment in the criminal proceeding has been
rendered. . . .
The civil action for legal separation, grounded as it is on concubinage, it is petitioner's position that such civil action arises from,
or is inextricably tied to the criminal action for concubinage, so that all proceedings related to legal separation will have to be
suspended to await conviction or acquittal for concubinage in the criminal case. Authority for this position is this Court's decision
in the case of Jerusalem vs. Hon. Roberto Zurbano. 1
Petitioner's contention is not correct.
In Jerusalem, the Court's statement to the effect that suspension of an action for legal separation would be proper if an
allegation of concubinage is made therein, relied solely on Sec. 1 of Rule 107 of the then provisions of the Rules of Court on
criminal procedure, to wit:
Sec. 1. Rules governing civil actions arising from offenses.-Except as otherwise provided by law, the following rules
shall he observed:
(a) When a criminal action is instituted, the civil action for recovery of civil liability arising from the offense charged is
impliedly instituted with the criminal action, unless the offended party expressly waives the civil action or reserves
his right to institute it separately;
(b) Criminal and civil actions arising from the same offense may be instituted separately, but after the criminal action
has been commenced the civil action can not be instituted until final judgment has been rendered in the criminal
action;
(c) After a criminal action has been commenced, no civil action arising from the same offense can be prosecuted
and the same shall be suspended in whatever stage it may be found until final judgment in the criminal proceeding
has been rendered ... (Emphasis supplied)
The provisions last quoted did not clearly state, as the 1985 Rules do, that the civil action to be suspended, with or upon the
filing of a criminal action, is one which is "to enforce the civil liability arising from the offense". In other words, in view of the
amendment under the 1985 Rules on Criminal Procedure, a civil action for legal separation, based on concubinage, may
proceed ahead of, or simultaneously with, a criminal action for concubinage, because said civil action is not one "to enforce the
civil liability arising from the offense" even if both the civil and criminal actions arise from or are related to the same offense.
Such civil action is one intended to obtain the right to live separately, with the legal consequences thereof, such as, the
dissolution of the conjugal partnership of gains, custody of offsprings, support, and disqualification from inheriting from the
innocent spouse, among others. As correctly pointed out by the respondent Judge in his Order dated 5 August 1987:
The unreported case of JERUSALEM vs. Hon. Roberto Zurbano, Judge of CFI of Antique, et al., L-11935, April 24,
1959 (105 Phil. 1277) is not controlling. It applied paragraph C of Sec. 1, of then Rule 107 of the Rules of Court,
which reads:
After a criminal action has been commenced, no civil action arising from the same offense can be
prosecuted and the same shall be suspended, in whatever stage it may be found, until final judgment in
the criminal proceeding has been rendered. (Emphasis supplied)
The governing rule is now Sec. 3, Rule 111, 1985 Rules on Criminal Procedure which refers to "civil actions to enforce the civil
liability arising from the offense" as contemplated in the first paragraph of Section 1 of Rule 111-which is a civil action "for
recovery of civil liability arising from the offense charged." Sec. 1, Rule 111, (1985) is specific that it refers to civil action for the
recovery of civil liability arising from the offense charged. Whereas, the old Sec. 1 (c), Rule 107 simply referred to "Civil action
arising from the offense."
As earlier noted this action for legal separation is not to recover civil liability, in the main, but is aimed at the conjugal rights of
the spouses and their relations to each other, within the contemplation of Articles 7 to 108, of the Civil Code." 2
Petitioner also argues that his conviction for concubinage will have to be first secured before the action for legal separation can
prosper or succeed, as the basis of the action for legal separation is his alleged offense of concubinage.
Petitioner's assumption is erroneous.

A decree of legal separation, on the ground of concubinage, may be issued upon proof by preponderance of evidence in the
action for legal separation. 3 No criminal proceeding or conviction is necessary. To this end, the doctrine in Francisco vs.
Tayao 4 has been modified, as that case was decided under Act. No. 2710, when absolute divorce was then allowed and had for
its grounds the same grounds for legal separation under the New Civil Code, with the requirement, under such former law, that
the guilt of defendant spouses had to be established by final judgment in a criminal action. That requirement has not been
reproduced or adopted by the framers of the present Civil Code, and the omission has been uniformly accepted as a
modification of the stringent rule in Francisco v. Tayao. 5
Petitioner's attempt to resist payment of support pendente lite to his wife must also fail, as we find no proof of grave abuse of
discretion on the part of the respondent Judge in ordering the same. Support pendente lite, as a remedy, can be availed of in an
action for legal separation, and granted at the discretion of the judge. 6 If petitioner finds the amount of support pendente
lite ordered as too onerous, he can always file a motion to modify or reduce the same. 7
Petitioner lastly seeks to have the respondent Judge disqualified from hearing the case, as the grant of supportpendente
lite and the denial of the motion to suspend hearings in the case, are taken by the petitioner as a disregard of applicable laws
and existing doctrines, thereby showing the respondent Judge's alleged manifest partiality to private respondent.
Petitioner's contention is without merit. Divergence of opinions between a judge hearing a case and a party's counsel, as to
applicable laws and jurisprudence, is not a sufficient ground to disqualify the judge from hearing the case, on the ground of bias
and manifest partiality. This is more so, in this case, where we find the judge's disposition of petitioner's motions to be sound
and well-taken.
WHEREFORE, the instant petition is hereby DISMISSED. Costs against petitioner.
SO ORDERED.

G.R. No. L-10033

December 28, 1956

BENJAMIN BUGAYONG, plaintiff-appellant,


vs.
LEONILA GINEZ, defendant-appellee.
Florencio Dumapias for appellant.
Numeriano Tanopo, Jr. for appellee.

FELIX, J.:
This is a case for legal separation filed in the Court of First Instance of Pangasinan wherein on motion of the defendant, the
case was dismissed. The order of dismissal was appealed to the Court of Appeals, but said Tribunal certified the case to the
Court on the ground that there is absolutely no question of fact involved, the motion being predicated on the assumption as true
of the very facts testified to by plaintiff-husband.
The facts of the case abridgedly stated are as follows: Benjamin Bugayong, a serviceman in the United States Navy, was
married to defendant Leonila Ginez on August 27, 1949, at Asingan, Pangasinan, while on furlough leave. Immediately after
their marriage, the couple lived with their sisters who later moved to Sampaloc, Manila. After some time, or about July, 1951,
Leonila Ginez left the dwelling of her sister-in-law and informed her husband by letter that she had gone to reside with her
mother in Asingan, Pangasinan, from which place she later moved to Dagupan City to study in a local college there.
As early as July, 1951, Benjamin Bugayong began receiving letters from Valeriana Polangco (plaintiff's sister-in-law) and some
from anonymous writers(which were not produced at the hearing) informing him of alleged acts of infidelity of his wife which he
did not even care to mention. On cross-examination, plaintiff admitted that his wife also informed him by letter, which she claims
to have destroyed, that a certain "Eliong" kissed her. All these communications prompted him in October, 1951 to seek the
advice of the Navy Chaplain as to the propriety of a legal separation between him and his wife on account of the latter's alleged
acts of infidelity, and he was directed to consult instead the navy legal department.
In August, 1952, plaintiff went to Asingan, Pangasinan, and sought for his wife whom he met in the house of one Mrs. Malalang,
defendant's godmother. She came along with him and both proceeded to the house of Pedro Bugayong, a cousin of the plaintiffhusband, where they stayed and lived for 2 nights and 1 day as husband and wife. Then they repaired to the plaintiff's house
and again passed the night therein as husband and wife. On the second day, Benjamin Bugayong tried to verify from his wife
the truth of the information he received that she had committed adultery but Leonila, instead of answering his query, merely
packed up and left, which he took as a confirmation of the acts of infidelity imputed on her. After that and despite such belief,
plaintiff exerted efforts to locate her and failing to find her, he went to Bacarra, Ilocos Norte, "to soothe his wounded feelings".
On November 18, 1952, Benjamin Bugayong filed in the Court of First Instance of Pangasinan a complaint for legal separation
against his wife, Leonila Ginez, who timely filed an answer vehemently denying the averments of the complaint and setting up
affirmative defenses. After the issues were joined and convinced that a reconciliation was not possible, the court set the case for
hearing on June 9, 1953. Plaintiff's counsel announced that he was to present 6 witnesses but after plaintiff-husband finished
testifying in his favor, counsel for the defendant orally moved for the dismissal of the complaint, but the Court ordered him to file
a written motion to that effect and gave plaintiff 10 days to answer the same.
The motion to dismiss was predicted on the following grounds: (1) Assuming arguendo the truth of the allegations of the
commission of "acts of rank infidelity amounting to adultery", the cause of action, if any, is barred by the statute of limitations; (2)
That under the same assumption, the act charged have been condoned by the plaintiff-husband; and (3) That the complaint
failed to state a cause of action sufficient for this court to render a valid judgment.
The motion to dismiss was answered by plaintiff and the Court, considering only the second ground of the motion to dismiss i.
e., condonation, ordered the dismissal of the action. After the motion for reconsideration filed by plaintiff was denied, the case
was taken up for review to the Court of Appeals, appellant's counsel maintaining that the lower court erred:
(a) In so prematurely dismissing the case;
(b) In finding that there were condonation on the part of plaintiff-appellant; and
(c) In entertaining condonation as a ground for dismissal inasmuch as same was not raised in the answer or in a motion to
dismiss.
As the questions raised in the brief were merely questions of law, the Court of Appeals certified the case to Superiority.
The Civil Code provides:
ART. 97. A petition for legal separation may be filed:
(1) For adultery on the part of the wife and for concubinage for the part of the husband as defined on the Penal Code; or
(2) An attempt by one spouse against the life of the other.
ART. 100. The legal separation may be claimed only by the innocent spouse, provided there has been no condonation of
or consent to the adultery or concubinage. Where both spouses are offenders, a legal separation cannot by either of them.
Collusion between the parties to obtain legal separation shall cause the dismissal of the petition.
ART. 102. An action for legal separation cannot be filed except within one year from and after the date on which the
plaintiff became cognizant of the cause and within five years from and after the date when such cause occurred.
As the only reason of the lower Court for dismissing the action was the alleged condonation of the charges of adultery that the
plaintiff-husband had preferred in the complaint against his wife, We will disregard the other 2 grounds of the motion to dismiss,
as anyway they have not been raised in appellant's assignment of errors.
Condonation is the forgiveness of a marital offense constituting a ground for legal separation or, as stated in I Bouver's Law
Dictionary, p. 585, condonation is the "conditional forgiveness or remission, by a husband or wife of a matrimonial offense which
the latter has committed". It is to be noted, however, that in defendant's answer she vehemently and vigorously denies having
committed any act of infidelity against her husband, and even if We were to give full weight to the testimony of the plaintiff, who
was the only one that had the chance of testifying in Court and link such evidence with the averments of the complaint, We
would have to conclude that the facts appearing on the record are far from sufficient to establish the charge of adultery, or, as
the complaint states, of "acts of rank infidelity amounting to adultery" preferred against the defendant. Certainly, the letter that
plaintiff claims to have received from his sister-in-law Valeriana Polangco, which must have been too vague and indefinite as to
defendant's infidelity to deserve its production in evidence; nor the anonymous letters which plaintiff also failed to present; nor
the alleged letter that, according to plaintiff, his wife addressed to him admitting that she had been kissed by one Eliong, whose
identity was not established and which admission defendant had no opportunity to deny because the motion to dismiss was filed
soon after plaintiff finished his testimony in Court, do not amount to anything that can be relied upon.

But this is not a question at issue. In this appeal, We have to consider plaintiff's line of conduct under the assumption that he
really believed his wife guilty of adultery. What did he do in such state of mind. In August, 1952, he went to Pangasinan and
looked for his wife and after finding her they lived together as husband and wife for 2 nights and 1 day, after which he says that
he tried to verify from her the truth of the news he had about her infidelity, but failed to attain his purpose because his wife,
instead of answering his query on the matter, preferred to desert him, probably enraged for being subjected to such humiliation.
And yet he tried to locate her, though in vain. Now, do the husband's attitude of sleeping with his wife for 2 nights despite his
alleged belief that she was unfaithful to him, amount to a condonation of her previous and supposed adulterous acts? In the
order appealed from, the Court a quo had the following to say on this point:
In the hearing of the case, the plaintiff further testified as follows:
Q. Now Mr. Bugayong, you have filed this action for legal separation from your wife. Please tell this Hon. Court why you
want to separate from your wife? A. I came to know that my wife is committing adultery, I consulted the chaplain and he
told me to consult the legal adviser. (p. 11, t.s.n.)
Q. Did you finally locate her?--A. Four days later or on the fifth day since my arrival she went to the house of our godmother, and as a husband I went to her to come along with me in our house but she refused. (p. 12, t.s.n.)lawphil.net
Q. What happened next? A. I persuaded her to come along with me. She consented but I did not bring her home but
brought her to the house of my cousin Pedro Bugayong. (p. 12, t.s.n.)
Q. How long did you remain in the house of your cousin Pedro Bugayong? A. One day and one night. (p. 12. t.s.n.)
Q. That night when you stayed in the house of your cousin Pedro Bugayong as husband and wife, did you slept together?
A. Yes, sir. (p. 19, t.s.n.)
Q. On the next night, when you slept in your own house, did you sleep together also as husband and wife? A. Yes, sir.
(p. 19. t.s.n.)
Q. When was that? A. That was in August, 1952. (p. 19 t.s.n.)
Q. How many nights did you sleep together as husband and wife? A. Only two nights. (p. 19, t.s.n.)
The New Civil Code of the Philippines, in its Art. 97, says:
A petition for legal separation may be filed:
(1) For adultery on the part of the wife and concubinage on the part of the husband as defined on the Penal Code.
and in its Art. 100 it says:lawphil.net
The legal separation may be claimed only by the innocent spouse, provided there has been no condonation of or consent
to the adultery or concubinage. Where both spouses are offenders, legal separation cannot be claimed by either of them.
Collusion between the parties to obtain legal separation shall cause the dismissal of the petition.
A detailed examination of the testimony of the plaintiff-husband, especially those portions quoted above, clearly shows
that there was a condonation on the part of the husband for the supposed "acts of rank infidelity amounting to adultery"
committed by defendant-wife. Admitting for the sake of argument that the infidelities amounting to adultery were
committed by the defendant, a reconciliation was effected between her and the plaintiff. The act of the latter in persuading
her to come along with him, and the fact that she went with him and consented to be brought to the house of his cousin
Pedro Bugayong and together they slept there as husband and wife for one day and one night, and the further fact that in
the second night they again slept together in their house likewise as husband and wife all these facts have no other
meaning in the opinion of this court than that a reconciliation between them was effected and that there was a
condonation of the wife by the husband. The reconciliation occurred almost ten months after he came to know of the acts
of infidelity amounting to adultery.
In Shackleton vs. Shackleton, 48 N. J. Eq. 364; 21 Atl. 935, it has been held that "condonation is implied from sexual
intercourse after knowledge of the other infidelity. such acts necessary implied forgiveness. It is entirely consonant with
reason and justice that if the wife freely consents to sexual intercourse after she has full knowledge of the husband's guilt,
her consent should operate as a pardon of his wrong."
In Tiffany's Domestic and Family Relations, section 107 says:
Condonation. Is the forgiveness of a marital offense constituting a ground for divorce and bars the right to a divorce.
But it is on the condition, implied by the law when not express, that the wrongdoer shall not again commit the
offense; and also that he shall thereafter treat the other spouse with conjugal kindness. A breach of the condition will
revive the original offense as a ground for divorce. Condonation may be express or implied.
It has been held in a long line of decisions of the various supreme courts of the different states of the U. S. that 'a single
voluntary act of sexual intercourse by the innocent spouse after discovery of the offense is ordinarily sufficient to
constitute condonation, especially as against the husband'. (27 Corpus Juris Secundum, section 61 and cases cited
therein).
In the lights of the facts testified to by the plaintiff-husband, of the legal provisions above quoted, and of the various
decisions above-cited, the inevitable conclusion is that the present action is untenable.
Although no acts of infidelity might have been committed by the wife, We agree with the trial judge that the conduct of the
plaintiff-husband above narrated despite his belief that his wife was unfaithful, deprives him, as alleged the offended spouse, of
any action for legal separation against the offending wife, because his said conduct comes within the restriction of Article 100 of
the Civil Code.
The only general rule in American jurisprudence is that any cohabitation with the guilty party, after the commission of the
offense, and with the knowledge or belief on the part of the injured party of its commission, will amount to conclusive evidence
of condonation; but this presumption may be rebutted by evidence (60 L. J. Prob. 73).
If there had been cohabitation, to what extent must it be to constitute condonation?
Single voluntary act of marital intercourse between the parties ordinarily is sufficient to constitute condonation, and where
the parties live in the same house, it is presumed that they live on terms of matrimonial cohabitation (27 C. J. S., section
6-d).
A divorce suit will not be granted for adultery where the parties continue to live together after it was known
(Land vs. Martin, 15 South 657; Day vs. Day, 80 Pac. 974) or there is sexual intercourse after knowledge of adultery
(Rogers vs. Rogers, 67 N. J. Eq. 534) or sleeping together for a single night (Toulson vs. Toulson, 50 Atl. 401, citing
Phinizy vs. Phinizy, 114 S. E. 185, 154 Ga. 199; Collins vs. Collins, 193 So. 702), and many others. The resumption of
marital cohabitation as a basis of condonation will generally be inferred, nothing appearing to the contrary, from the fact of
the living together as husband and wife, especially as against the husband (Marsh vs. Marsh, 14 N. J. Eq. 315).

There is no ruling on this matter in our jurisprudence but we have no reason to depart from the doctrines laid down in the
decisions of the various supreme courts of the United States above quoted.
There is no merit in the contention of appellant that the lower court erred in entertaining condonation as a ground for dismissal
inasmuch as same was not raised in the answer or in a motion to dismiss, because in the second ground of the motion to
dismiss. It is true that it was filed after the answer and after the hearing had been commenced, yet that motion serves to
supplement the averments of defendant's answer and to adjust the issues to the testimony of plaintiff himself (section 4, Rule 17
of the Rules of Court).
Wherefore, and on the strength of the foregoing, the order appealed from is hereby affirmed, with costs against appellant. It is
so ordered.

G.R. No. L-10699

October 18, 1957

WILLIAM H. BROWN, plaintiff-appellant,


vs.
JUANITA YAMBAO, defendant-appellee.
Jimenez B. Buendia for appellant.
Assistant City Fiscal Rafel A. Jose for appellee.
REYES, J.B.L., J.:
On July 14, 1955, William H. Brown filed suit in the Court of First Instance of Manila to obtain legal separation from his lawful
wife Juanita Yambao. He alleged under oath that while interned by the Japanese invaders, from 1942 to 1945, at the University
of Sto. Tomas internment camp, his wife engaged in adulterous relations with one Carlos Field of whom she begot a baby girl
that Brown learned of his wifes misconduct only in 1945, upon his release from internment; that thereafter the spouse lived
separately and later executed a document (Annex A ) liquidating their conjugal partnership and assigning certain properties to
the erring wife as her share. The complaint prayed for confirmation of the liquidation agreement; for custody of the children
issued of the marriage; that the defendant be declared disqualified to succeed the plaintiff; and for their remedy as might be just
and equitable.
Upon petition of the plaintiff, the court subsequently declared the wife in default, for failure to answer in due time, despite
service of summons; and directed the City Fiscal or his representatives to
investigate, in accordance with Article 101 of the Civil Code, whether or not a collusion exists between the parties and to
report to this Court the result of his investigation within fifteen (15) days from receipt of copy of this order. The City Fiscal
or his representative is also directed to intervene in the case in behalf of the State. (Rec. App. p. 9).
As ordered, Assistant City Fiscal Rafael Jose appeared at the trial, and cross-examined plaintiff Brown. His questions
(strenuously objected to by Brown's counsel) elicited the fact that after liberation, Brown had lived maritally with another woman
and had begotten children by her. Thereafter, the court rendered judgment denying the legal separation asked, on the ground
that, while the wife's adultery was established, Brown had incurred in a misconduct of similar nature that barred his right of
action under Article 100 of the new Civil Code, providing:
ART. 100. The legal separation may be claimed only by the innocent spouse, provided there has been no condonation or
of consent to the adultery or concubinage. Where both spouses are offenders, a legal separation cannot be claimed by
either of them. Collusion between the parties to obtain legal separation shall cause the dismissal of the petition.
that there had been consent and connivance, and because Brown's action had prescribed under Article 102 of the same Code:
ART. 102 An action for legal separation cannot be filed except within one year from and after the date on which the plaintiff
became cognizant of the cause and within five years from and after date when such cause occurred.
since the evidence showed that the learned of his wife's infidelity in 1945 but only filed action in 1945.
Brown appeared to this Court, assigning the following errors:
The court erred in permitting the Assistant Fiscal Rafel Jose of Manila to act as counsel for the defendant, who defaulted.
The court erred in declaring that there was condonation of or consent to the adultery.
The court erred in dismissing the plaintiff's complaint.
Appellant Brown argues that in cross-examining him with regard to his marital relation with Lilia Deito, who was not his wife, the
Assistant Fiscal acted as consel for the defaulting wife, "when the power of the prosecuting officer is limited to finding out
whether or not there is collusion, and if there is no collusion, which is the fact in the case at bar, to intervene for the state which
is not the fact in the instant case, the truth of the matter being that he intervened for Juanita Yambao, the defendant-appellee,
who is private citizen and who is far from being the state.".
The argument is untenable. Collusion in matrimonial cases being "the act of married persons in procuring a divorce by mutual
consent, whether by preconcerted commission by one of a matrimonial offense, or by failure, in pursuance of agreement to
defend divorce proceedings" (Cyclopedia Law Dictionary; Nelson, Divorce and Separation, Section 500), it was legitimate for
the Fiscal to bring to light any circumstances that could give rise to the inference that the wife's default was calculated, or
agreed upon, to enable appellant to obtain the decree of legal separation that he sought without regard to the legal merits of his
case. One such circumstance is obviously the fact of Brown's cohabitation with a woman other than his wife, since it bars him
from claiming legal separation by express provision of Article 100 of the new Civil Code. Wherefore, such evidence of such
misconduct, were proper subject of inquiry as they may justifiably be considered circumstantial evidence of collusion between
the spouses.
The policy of Article 101 of the new Civil Code, calling for the intervention of the state attorneys in case of uncontested
proceedings for legal separation (and of annulment of marriages, under Article 88), is to emphasize that marriage is more than a
mere contract; that it is a social institution in which the state is vitally interested, so that its continuation or interruption cannot be
made depend upon the parties themselves (Civil Code, Article 52; Adong vs, Cheong Gee, 43 Phil, 43; Ramirez vs. Gmur 42
Phil. 855; Goitia vs. Campos, 35 Phil. 252). It is consonant with this policy that the injury by the Fiscal should be allowed to
focus upon any relevant matter that may indicate whether the proceedings for separation or annulment are fully justified or not.
The court below also found, and correctly held that the appellant's action was already barred, because Brown did not petition for
legal separation proceedings until ten years after he learned of his wife's adultery, which was upon his release from internment
in 1945. Under Article 102 of the new Civil Code, action for legal separation can not be filed except within one (1) year from and
after the plaintiff became cognizant of the cause and within five years from and after the date when such cause occurred.
Appellant's brief does not even contest the correctness of such findings and conclusion.
It is true that the wife has not interposed prescription as a defense. Nevertheless, the courts can take cognizance thereof,
because actions seeking a decree of legal separation, or annulment of marriage, involve public interest and it is the policy of our
law that no such decree be issued if any legal obstacles thereto appear upon the record.
Hence, there being at least two well established statutory grounds for denying the remedy sought (commission of similar
offense by petitioner and prescription of the action), it becomes unnecesary to delve further into the case and ascertain if
Brown's inaction for ten years also evidences condonation or connivance on his part. Even if it did not, his situation would not be
improved. It is thus needless to discuss the second assignment of error.
The third assignment of error being a mere consequence of the others must necessarily fail with them.
The decision appealed from is affirmed, with costs against appellant. So ordered.

Rights and Obligations Beteen Husband and Wife


G.R. No. 139789

July 19, 2001

IN THE MATTER OF THE PETITION FOR HABEAS CORPUS OF POTENCIANO ILUSORIO, ERLINDA K.
ILUSORIO, petitioner,
vs.
ERLINDA K. ILUSORIO-BILDNER, SYLVIA K. ILUSORIO-YAP, JOHN DOES and JANE DOES, respondents.
x---------------------------------------------------------x
G.R. No. 139808 July 19, 2001
POTENCIANO ILUSORIO, MA. ERLINDA I. BILDNER and SYLVIA K. ILUSORIO, petitioners,
vs.
HON. COURT OF APPEALS and ERLINDA K. ILUSORIO, respondents.
RESOLUTION
PARDO, J.:
Once again we see the sad tale of a prominent family shattered by conflicts on expectancy in fabled fortune.
On March 11, 1999, Erlinda K. Ilusorio, the matriarch who was so lovingly inseparable from her husband some years ago, filed a
petition with the Court of Appeals1 for habeas corpus to have custody of her husband in consortium.
On April 5, 1999, the Court of Appeals promulgated its decision dismissing the petition for lack of unlawful restraint or detention
of the subject, Potenciano Ilusorio.
Thus, on October 11, 1999, Erlinda K. Ilusorio filed with the Supreme Court an appeal via certiorari pursuing her desire to have
custody of her husband Potenciano Ilusorio. 2 This case was consolidated with another case3 filed by Potenciano Ilusorio and his
children, Erlinda I. Bildner and Sylvia K. Ilusorio appealing from the order giving visitation rights to his wife, asserting that he
never refused to see her.
On May 12, 2000, we dismissed the petition for habeas corpus 4 for lack of merit, and granted the petition5 to nullify the Court of
Appeals' ruling6 giving visitation rights to Erlinda K. Ilusorio. 7
What is now before the Court is Erlinda's motion to reconsider the decision. 8
On September 20, 2000, we set the case for preliminary conference on October 11, 2000, at 10:00 a. m., without requiring the
mandatory presence of the parties.
In that conference, the Court laid down the issues to be resolved, to wit:
(a) To determine the propriety of a physical and medical examination of petitioner Potenciano Ilusorio;
(b) Whether the same is relevant; and
(c) If relevant, how the Court will conduct the same. 9
The parties extensively discussed the issues. The Court, in its resolution, enjoined the parties and their lawyers to initiate steps
towards an amicable settlement of the case through mediation and other means.
On November 29, 2000, the Court noted the manifestation and compliance of the parties with the resolution of October 11,
2000.10
On January 31, 2001, the Court denied Erlinda Ilusorio's manifestation and motion praying that Potenciano Ilusorio be produced
before the Court and be medically examined by a team of medical experts appointed by the Court. 11
On March 27, 2001, we denied with finality Erlinda's motion to reconsider the Court's order of January 31 , 2001. 12
The issues raised by Erlinda K. Ilusorio in her motion for reconsideration are mere reiterations of her arguments that have been
resolved in the decision.
Nevertheless, for emphasis, we shall discuss the issues thus:
First. Erlinda K. Ilusorio claimed that she was not compelling Potenciano to live with her in consortium and that Potenciano's
mental state was not an issue. However, the very root cause of the entire petition is her desire to have her
husband's custody.13 Clearly, Erlinda cannot now deny that she wanted Potenciano Ilusorio to live with her.
Second. One reason why Erlinda K. Ilusorio sought custody of her husband was that respondents Lin and Sylvia were illegally
restraining Potenciano Ilusorio to fraudulently deprive her of property rights out of pure greed. 14 She claimed that her two
children were using their sick and frail father to sign away Potenciano and Erlinda's property to companies controlled by Lin and
Sylvia. She also argued that since Potenciano retired as director and officer of Baguio Country Club and Philippine Oversees
Telecommunications, she would logically assume his position and control. Yet, Lin and Sylvia were the ones controlling the
corporations.15
The fact of illegal restraint has not been proved during the hearing at the Court of Appeals on March 23, 1999. 16Potenciano
himself declared that he was not prevented by his children from seeing anybody and that he had no objection to seeing his wife
and other children whom he loved.
Erlinda highlighted that her husband suffered from various ailments. Thus, Potenciano Ilusorio did not have the mental capacity
to decide for himself. Hence, Erlinda argued that Potenciano be brought before the Supreme Court so that we could determine
his mental state.
We were not convinced that Potenciano Ilusorio was mentally incapacitated to choose whether to see his wife or not. Again, this
is a question of fact that has been decided in the Court of Appeals.
As to whether the children were in fact taking control of the corporation, these are matters that may be threshed out in a
separate proceeding, irrelevant in habeas corpus.
Third. Petitioner failed to sufficiently convince the Court why we should not rely on the facts found by the Court of Appeals.
Erlinda claimed that the facts mentioned in the decision were erroneous and incomplete. We see no reason why the High Court
of the land need go to such length. The hornbook doctrine states that findings of fact of the lower courts are conclusive on the
Supreme Court.17 We emphasize, it is not for the Court to weigh evidence all over again. 18 Although there are exceptions to the
rule,19 Erlinda failed to show that this is an exceptional instance.

Fourth. Erlinda states that Article XII of the 1987 Constitution and Articles 68 and 69 of the Family Code support her position
that as spouses, they (Potenciano and Erlinda) are duty bound to live together and care for each other. We agree.
The law provides that the husband and the wife are obliged to live together, observe mutual love, respect and fidelity. 20 The
sanction therefor is the "spontaneous, mutual affection between husband and wife and not any legal mandate or court order" to
enforce consortium.21
Obviously, there was absence of empathy between spouses Erlinda and Potenciano, having separated from bed and board
since 1972. We defined empathy as a shared feeling between husband and wife experienced not only by having spontaneous
sexual intimacy but a deep sense of spiritual communion. Marital union is a two-way process.
Marriage is definitely for two loving adults who view the relationship with "amor gignit amorem" respect, sacrifice and a
continuing commitment to togetherness, conscious of its value as a sublime social institution. 22
On June 28, 2001, Potenciano Ilusorio gave his soul to the Almighty, his Creator and Supreme Judge. Let his soul rest in peace
and his survivors continue the much prolonged fracas ex aequo et bono.
IN VIEW WHEREOF, we DENY Erlinda's motion for reconsideration. At any rate, the case has been rendered moot by the death
of subject.

G.R. No. 11263

November 2, 1916

ELOISA GOITIA DE LA CAMARA, plaintiff-appellant,


vs.
JOSE CAMPOS RUEDA, defendant-appellee.
Eduardo Gutierrez Repide and Felix Socias for appellant.
Sanz, Opisso and Luzuriaga for appellee.

TRENT, J.:
This is an action by the wife against her husband for support outside of the conjugal domicile. From a judgment sustaining the
defendant's demurrer upon the ground that the facts alleged in the complaint do not state a cause of action, followed by an
order dismissing the case after the plaintiff declined to amend, the latter appealed.
It was urged in the first instance, and the court so held, that the defendant cannot be compelled to support the plaintiff, except in
his own house, unless it be by virtue of a judicial decree granting her a divorce or separation from the defendant.
The parties were legally married in the city of Manila on January 7, 1915, and immediately thereafter established their residence
at 115 Calle San Marcelino, where they lived together for about a month, when the plaintiff returned to the home of her parents.
The pertinent allegations of the complaint are as follows:
That the defendant, one month after he had contracted marriage with the plaintiff, demanded of her that she perform
unchaste and lascivious acts on his genital organs; that the plaintiff spurned the obscene demands of the defendant and
refused to perform any act other than legal and valid cohabitation; that the defendant, since that date had continually on
other successive dates, made similar lewd and indecorous demands on his wife, the plaintiff, who always spurned them,
which just refusals of the plaintiff exasperated the defendant and induce him to maltreat her by word and deed and inflict
injuries upon her lips, her face and different parts of her body; and that, as the plaintiff was unable by any means to induce
the defendant to desist from his repugnant desires and cease from maltreating her, she was obliged to leave the conjugal
abode and take refuge in the home of her parents.
Marriage in this jurisdiction is a contract entered into in the manner and with the solemnities established by General Orders No.
68, in so far as its civil effects are concerned requiring the consent of the parties. (Garcia vs. Montague, 12 Phil. Rep., 480,
citing article 1261 of Civil Code.) Upon the termination of the marriage ceremony, a conjugal partnership is formed between the
parties. (Sy Joc Lieng vs. Encarnacion, 16 Phil. Rep., 137.) To this extent a marriage partakes of the nature of an ordinary
contract. But it is something more than a mere contract. It is a new relation, the rights, duties, and obligations of which rest not
upon the agreement of the parties but upon the general law which defines and prescribes those rights, duties, and
obligations .Marriage is an institution, in the maintenance of which in its purity the public is deeply interested. It is a relation for
life and the parties cannot terminate it at any shorter period by virtue of any contract they may make .The reciprocal rights
arising from this relation, so long as it continues, are such as the law determines from time to time, and none other. When the
legal existence of the parties is merged into one by marriage, the new relation is regulated and controlled by the state or
government upon principles of public policy for the benefit of society as well as the parties. And when the object of a marriage is
defeated by rendering its continuance intolerable to one of the parties and productive of no possible good to the community,
relief in some way should be obtainable. With these principles to guide us, we will inquire into the status of the law touching and
governing the question under consideration.
Articles 42 to 107 of the Civil Code are not in force in the Philippine Islands (Benedicto vs. De la Rama, 3 Phil .Rep., 34).
Articles 44 to 78 of the Law of Civil Marriage of 1870, in force in the Peninsula, were extended to the Philippine Islands by royal
decree on April 13, 1883 (Ebreo vs. Sichon, 4 Phil. Rep., 705). Articles 44, 45, and 48 of this law read:
ART. 44. The spouses are obliged to be faithful to each other and to mutually assist each other.
ART. 45. The husband must live with and protect his wife. (The second paragraph deals with the management of the
wife's property.)
ART. 48. The wife must obey her husband, live with him, and follow him when he charges his domicile or residence.
Notwithstanding the provisions of the foregoing paragraph, the court may for just cause relieve her from this duty when the
husband removes his residence to a foreign country.
And articles 143 and 149 of the Civil Code are as follows:
ART. 143. The following are obliged to support each other reciprocally to the whole extent specified in the preceding
article.
1. The consorts.
xxx

xxx

xxx

ART. (149) 49. The person obliged to give support may, at his option, satisfy it, either by paying the pension that may be
fixed or by receiving and maintaining in his own home the person having the right to the same.
Article 152 of the Civil Code gives the instances when the obligation to give support shall cease. The failure of the wife to live
with her husband is not one of them.
The above quoted provisions of the Law of Civil Marriage and the Civil Code fix the duties and obligations of the spouses. The
spouses must be faithful to, assist, and support each other. The husband must live with and protect his wife. The wife must obey
and live with her husband and follow him when he changes his domicile or residence, except when he removes to a foreign
country. But the husband who is obliged to support his wife may, at his option, do so by paying her a fixed pension or by
receiving and maintaining her in his own home. May the husband, on account of his conduct toward his wife, lose this option
and be compelled to pay the pension? Is the rule established by article 149 of the Civil Code absolute? The supreme court of
Spain in its decision of December 5, 1903, held:.
That in accordance with the ruling of the supreme court of Spain in its decisions dated May 11, 1897, November 25, 1899,
and July 5, 1901, the option which article 149 grants the person, obliged to furnish subsistence, between paying the
pension fixed or receiving and keeping in his own house the party who is entitled to the same, is not so absolute as to
prevent cases being considered wherein, either because this right would be opposed to the exercise of a preferential right
or because of the existence of some justifiable cause morally opposed to the removal of the party enjoying the
maintenance, the right of selection must be understood as being thereby restricted.
Whereas the only question discussed in the case which gave rise to this appeal was whether there was any reason to
prevent the exercise of the option granted by article 149 of the Civil Code to the person obliged to furnish subsistence, to
receive and maintain in his own house the one who is entitled to receive it; and inasmuch as nothing has been alleged or
discussed with regard to the parental authority of Pedro Alcantara Calvo, which he ha not exercised, and it having been

set forth that the natural father simply claims his child for the purpose of thus better attending to her maintenance, no
action having been taken by him toward providing the support until, owing to such negligence, the mother was obliged to
demand it; it is seen that these circumstances, together with the fact of the marriage of Pedro Alcantara, and that it would
be difficult for the mother to maintain relations with her daughter, all constitute an impediment of such a nature as to
prevent the exercise of the option in the present case, without prejudice to such decision as may be deemed proper with
regard to the other questions previously cited in respect to which no opinion should be expressed at this time.
The above was quoted with approval in United States and De Jesus vs. Alvir (9 Phil. Rep., 576), wherein the court held that the
rule laid down in article 149 of the Civil Code "is not absolute." but it is insisted that there existed a preexisting or preferential
right in each of these cases which was opposed to the removal of the one entitled to support. It is true that in the first the person
claiming the option was the natural father of the child and had married a woman other than the child's mother, and in the second
the right to support had already been established by a final judgment in a criminal case. Notwithstanding these facts the two
cases clearly established the proposition that the option given by article 149 of the Civil Code may not be exercised in any and
all cases.
Counsel for the defendant cite, in support of their contention, the decision of the supreme court of Spain, dated November 3,
1905. In this case Don Berno Comas, as a result of certain business reverses and in order no to prejudice his wife, conferred
upon her powers to administer and dispose of her property. When she left him he gave her all the muniments of title, mortgage
credits, notes, P10,000 in accounts receivable, and the key to the safe in which he kept a large amount of jewels, thus depriving
himself of all his possessions and being reduced in consequence to want. Subsequently he instituted this civil action against his
wife, who was then living in opulence, for support and the revocation of the powers heretofore granted in reference to the
administration and disposal of her property. In her answer the wife claimed that the plaintiff (her husband) was not legally in a
situation to claim support and that the powers voluntarily conferred and accepted by her were bilateral and could not be
canceled by the plaintiff. From a judgment in favor of the plaintiff the defendant wife appealed to the Audencia Territorialwherein,
after due trial, judgment was rendered in her favor dismissing the action upon the merits. The plaintiff appealed to the supreme
court and that high tribunal, in affirming the judgment of the Audencia Territorial, said:
Considering that article 143, No. 1, of the Civil Code, providing that the spouses are mutually obliged to provide each
other with support, cannot but be subordinate to the other provisions of said Code which regulates the family organization
and the duties of spouses not legally separated, among which duties are those of their living together and mutually helping
each other, as provided in article 56 of the aforementioned code; and taking this for granted, the obligation of the spouse
who has property to furnish support to the one who has no property and is in need of it for subsistence, is to be
understood as limited to the case where, in accordance with law, their separation has been decreed, either temporarily or
finally and this case, with respect to the husband, cannot occur until a judgment of divorce is rendered, since, until then, if
he is culpable, he is not deprived of the management of his wife's property and of the product of the other property
belonging to the conjugal partnership; and
Considering that, should the doctrine maintained in the appeal prevail, it would allow married persons to disregard the
marriage bond and separate from each other of their own free will, thus establishing, contrary to the legal provision
contained in said article 56 of the Civil Code, a legal status entirely incompatible with the nature and effects of marriage in
disregard of the duties inherent therein and disturbing the unity of the family, in opposition to what the law, in conformity
with good morals, has established; and.
Considering that, as the spouses D. Ramon Benso and Doa Adela Galindo are not legally separated, it is their duty to
live together and afford each other help and support; and for this reason, it cannot be held that the former has need of
support from his wife so that he may live apart from her without the conjugal abode where it is his place to be, nor of her
conferring power upon him to dispose even of the fruits of her property in order therewith to pay the matrimonial expenses
and, consequently, those of his own support without need of going to his wife; wherefore the judgment appealed from,
denying the petition of D. Ramon Benso for support, has not violated the articles of the Civil Code and the doctrine
invoked in the assignments of error 1 and 5 of the appeal.
From a careful reading of the case just cited and quoted from it appears quite clearly that the spouses separated voluntarily in
accordance with an agreement previously made. At least there are strong indications to this effect, for the court says, "should
the doctrine maintained in the appeal prevail, it would allow married persons to disregard the marriage bond and separate from
each other of their own free will." If this be the true basis upon which the supreme court of Spain rested its decision, then the
doctrine therein enunciated would not be controlling in cases where one of the spouses was compelled to leave the conjugal
abode by the other or where the husband voluntarily abandons such abode and the wife seeks to force him to furnish support.
That this is true appears from the decision of the same high tribunal, dated October 16, 1903. In this case the wife brought an
action for support against her husband who had willfully and voluntarily abandoned the conjugal abode without any cause
whatever. The supreme court, reversing the judgment absolving the defendant upon the ground that no action for divorce, etc.,
had been instituted, said:
In the case at bar, it has been proven that it was Don Teodoro Exposito who left the conjugal abode, although he claims,
without however proving his contention, that the person responsible for this situation was his wife, as she turned him out
of the house. From this state of affairs it results that it is the wife who is party abandoned, the husband not having
prosecuted any action to keep her in his company and he therefore finds himself, as long as he consents to the situation,
under the ineluctable obligation to support his wife in fulfillment of the natural duty sanctioned in article 56 of the Code in
relation with paragraph 1 of article 143. In not so holding, the trial court, on the mistaken ground that for the fulfillment of
this duty the situation or relation of the spouses should be regulated in the manner it indicates, has made the errors of law
assigned in the first three grounds alleged, because the nature of the duty of affording mutual support is compatible and
enforcible in all situations, so long as the needy spouse does not create any illicit situation of the court above
described.lawphil.net
If we are in error as to the doctrine enunciated by the supreme court of Spain in its decision of November 3, 1905, and if the
court did hold, as contended by counsel for the defendant in the case under consideration, that neither spouse can be
compelled to support the other outside of the conjugal abode, unless it be by virtue of a final judgment granting the injured one a
divorce or separation from the other, still such doctrine or holding would not necessarily control in this jurisdiction for the reason
that the substantive law is not in every particular the same here as it is in Spain. As we have already stated, articles 42 to 107 of
the Civil Code in force in the Peninsula are not in force in the Philippine Islands. The law governing the duties and obligations of
husband and wife in this country are articles 44 to 78 of the Law of Civil Marriage of 1870 .In Spain the complaining spouse has,
under article 105 of the Civil Code, various causes for divorce, such as adultery on the part of the wife in every case and on the
part of the husband when public scandal or disgrace of the wife results therefrom; personal violence actually inflicted or grave
insults: violence exercised by the husband toward the wife in order to force her to change her religion; the proposal of the
husband to prostitute his wife; the attempts of the husband or wife to corrupt their sons or to prostitute their daughters; the
connivance in their corruption or prostitution; and the condemnation of a spouse to perpetual chains or hard labor, while in this
jurisdiction the only ground for a divorce is adultery. (Benedicto vs. De la Rama, 3 Phil .Rep., 34, 45.) This positive and absolute
doctrine was announced by this court in the case just cited after an exhaustive examination of the entire subject. Although the
case was appealed to the Supreme Court of the United States and the judgment rendered by this court was there reversed, the
reversal did not affect in any way or weaken the doctrine in reference to adultery being the only ground for a divorce. And since
the decision was promulgated by this court in that case in December, 1903, no change or modification of the rule has been
announced. It is, therefore, the well settled and accepted doctrine in this jurisdiction.
But it is argued that to grant support in an independent suit is equivalent to granting divorce or separation, as it necessitates a
determination of the question whether the wife has a good and sufficient cause for living separate from her husband; and,
consequently, if a court lacks power to decree a divorce, as in the instant case, power to grant a separate maintenance must

also be lacking. The weakness of this argument lies in the assumption that the power to grant support in a separate action is
dependent upon a power to grant a divorce. That the one is not dependent upon the other is apparent from the very nature of
the marital obligations of the spouses. The mere act of marriage creates an obligation on the part of the husband to support his
wife. This obligation is founded not so much on the express or implied terms of the contract of marriage as on the natural and
legal duty of the husband; an obligation, the enforcement of which is of such vital concern to the state itself that the laws will not
permit him to terminate it by his own wrongful acts in driving his wife to seek protection in the parental home. A judgment for
separate maintenance is not due and payable either as damages or as a penalty; nor is it a debt in the strict legal sense of the
term, but rather a judgment calling for the performance of a duty made specific by the mandate of the sovereign. This is done
from necessity and with a view to preserve the public peace and the purity of the wife; as where the husband makes so base
demands upon his wife and indulges in the habit of assaulting her. The pro tanto separation resulting from a decree for separate
support is not an impeachment of that public policy by which marriage is regarded as so sacred and inviolable in its nature; it is
merely a stronger policy overruling a weaker one; and except in so far only as such separation is tolerated as a means of
preserving the public peace and morals may be considered, it does not in any respect whatever impair the marriage contract or
for any purpose place the wife in the situation of a feme sole.
The foregoing are the grounds upon which our short opinion and order for judgment, heretofore filed in this case, rest.

[G.R. No. 127406. November 27, 2000]

OFELIA P. TY, petitioner, vs. THE COURT OF APPEALS, and EDGARDO M. REYES, respondents.
DECISION
QUISUMBING, J.:
This appeal seeks the reversal of the decision dated July 24, 1996, of the Court of Appeals in C.A. G.R. CV 37897, which
affirmed the decision of the Regional Trial Court of Pasig, Branch 160, declaring the marriage contract between private
respondent Edgardo M. Reyes and petitioner Ofelia P. Ty null and void ab initio. It also ordered private respondent to pay
P15,000.00 as monthly support for their children Faye Eloise Reyes and Rachel Anne Reyes.
As shown in the records of the case, private respondent married Anna Maria Regina Villanueva in a civil ceremony on March
29, 1977, in Manila. Then they had a church wedding on August 27, 1977. However, on August 4, 1980, the Juvenile and
Domestic Relations Court of Quezon City declared their marriage null and void ab initio for lack of a valid marriage license. The
church wedding on August 27, 1977, was also declared null and void ab initio for lack of consent of the parties.
Even before the decree was issued nullifying his marriage to Anna Maria, private respondent wed Ofelia P. Ty, herein
petitioner, on April 4, 1979, in ceremonies officiated by the judge of the City Court of Pasay. On April 4, 1982, they also had a
church wedding in Makati, Metro Manila.
On January 3, 1991, private respondent filed a Civil Case 1853-J with the RTC of Pasig, Branch 160, praying that his
marriage to petitioner be declared null and void. He alleged that they had no marriage license when they got married. He also
averred that at the time he married petitioner, he was still married to Anna Maria. He stated that at the time he married
petitioner the decree of nullity of his marriage to Anna Maria had not been issued. The decree of nullity of his marriage to Anna
Maria was rendered only on August 4, 1980, while his civil marriage to petitioner took place on April 4, 1979.
Petitioner, in defending her marriage to private respondent, pointed out that his claim that their marriage was contracted
without a valid license is untrue. She submitted their Marriage License No. 5739990 issued at Rosario, Cavite on April 3, 1979,
as Exh. 11, 12 and 12-A. He did not question this document when it was submitted in evidence. Petitioner also submitted the
decision of the Juvenile and Domestic Relations Court of Quezon City dated August 4, 1980, which declared null and void his
civil marriage to Anna Maria Regina Villanueva celebrated on March 29, 1977, and his church marriage to said Anna Maria on
August 27, 1977. These documents were submitted as evidence during trial and, according to petitioner, are therefore deemed
sufficient proof of the facts therein. The fact that the civil marriage of private respondent and petitioner took place on April 4,
1979, before the judgment declaring his prior marriage as null and void is undisputed. It also appears indisputable that private
respondent and petitioner had a church wedding ceremony on April 4, 1982. [1]
The Pasig RTC sustained private respondents civil suit and declared his marriage to herein petitioner null and void ab
initio in its decision dated November 4, 1991. Both parties appealed to respondent Court of Appeals. On July 24, 1996, the
appellate court affirmed the trial courts decision. It ruled that a judicial declaration of nullity of the first marriage (to Anna Maria)
must first be secured before a subsequent marriage could be validly contracted. Said the appellate court:
We can accept, without difficulty, the doctrine cited by defendants counsel that no judicial decree is necessary to establish the
invalidity of void marriages. It does not say, however, that a second marriage may proceed even without a judicial
decree. While it is true that if a marriage is null and void, ab initio, there is in fact no subsisting marriage, we are unwilling to
rule that the matter of whether a marriage is valid or not is for each married spouse to determine for himself for this would be
the consequence of allowing a spouse to proceed to a second marriage even before a competent court issues a judicial decree
of nullity of his first marriage. The results would be disquieting, to say the least, and could not have been the intendment of
even the now-repealed provisions of the Civil Code on marriage.
xxx
WHEREFORE, upon the foregoing ratiocination, We modify the appealed Decision in this wise:
1. The marriage contracted by plaintiff-appellant [herein private respondent] Eduardo M. Reyes and defendant-appellant
[herein petitioner] Ofelia P. Ty is declared null and void ab initio;
2. Plaintiff-appellant Eduardo M. Reyes is ordered to give monthly support in the amount of P15,000.00 to his children
Faye Eloise Reyes and Rachel Anne Reyes from November 4, 1991; and
3. Cost against plaintiff-appellant Eduardo M. Reyes.
SO ORDERED.[2]
Petitioners motion for reconsideration was denied. Hence, this instant petition asserting that the Court of Appeals erred:
I.
BOTH IN THE DECISION AND THE RESOLUTION, IN REQUIRING FOR THE VALIDITY OF PETITIONERS MARRIAGE TO
RESPONDENT, A JUDICIAL DECREE NOT REQUIRED BY LAW.
II
IN THE RESOLUTION, IN APPLYING THE RULING IN DOMINGO VS. COURT OF APPEALS.
III
IN BOTH THE DECISION AND RESOLUTION IN NOT CONSIDERING THE CIVIL EFFECTS OF THE RELIGIOUS
RATIFICATION WHICH USED THE SAME MARRIAGE LICENSE.
IV
IN THE DECISION NOT GRANTING MORAL AND EXEMPLARY DAMAGES TO THE DEFENDANT-APPELLANT.
The principal issue in this case is whether the decree of nullity of the first marriage is required before a subsequent marriage
can be entered into validly? To resolve this question, we shall go over applicable laws and pertinent cases to shed light on the
assigned errors, particularly the first and the second which we shall discuss jointly.
In sustaining the trial court, the Court of Appeals declared the marriage of petitioner to private respondent null and void for
lack of a prior judicial decree of nullity of the marriage between private respondent and Villanueva. The appellate court rejected
petitioners claim that People v. Mendoza [3] and People v. Aragon[4] are applicable in this case. For these cases held that where
a marriage is void from its performance, no judicial decree is necessary to establish its invalidity. But the appellate court said
these cases, decided before the enactment of the Family Code (E.O. No. 209 as amended by E.O No. 227), no longer
control. A binding decree is now needed and must be read into the provisions of law previously obtaining. [5]
In refusing to consider petitioners appeal favorably, the appellate court also said:
Terre v. Attorney Terre, Adm. Case No. 2349, 3 July 1992 is mandatory precedent for this case. Although decided by the High
Court in 1992, the facts situate it within the regime of the now-repealed provisions of the Civil Code, as in the instant case.

xxx
For purposes of determining whether a person is legally free to contract a second marriage, a judicial declaration that the first
marriage was null and void ab initio is essential. . . .[6]
At the outset, we must note that private respondents first and second marriages contracted in 1977 and 1979, respectively,
are governed by the provisions of the Civil Code. The present case differs significantly from the recent cases of Bobis v.
Bobis[7] and Mercado v. Tan,[8] both involving a criminal case for bigamy where the bigamous marriage was contracted during the
effectivity of the Family Code,[9] under which a judicial declaration of nullity of marriage is clearly required.
Pertinent to the present controversy, Article 83 of the Civil Code provides that:
Art. 83. Any marriage subsequently contracted by any person during the lifetime of the first spouse of such person with any
person other than such first spouse shall be illegal and void from its performance, unless:
(1) The first marriage was annulled or dissolved; or
(2) The first spouse had been absent for seven consecutive years at the time of the second marriage without the spouse
present having news of the absentee being alive, or if the absentee, though he has been absent for less than seven years, is
generally considered as dead and before any person believed to be so by the spouse present at the time of contracting such
subsequent marriage, or if the absentee is presumed dead according to articles 390 and 391. The marriage so contracted shall
be valid in any of the three cases until declared null and void by a competent court.
As to whether a judicial declaration of nullity of a void marriage is necessary, the Civil Code contains no express provision to
that effect. Jurisprudence on the matter, however, appears to be conflicting.
Originally, in People v. Mendoza,[10] and People v. Aragon,[11] this Court held that no judicial decree is necessary to establish
the nullity of a void marriage. Both cases involved the same factual milieu. Accused contracted a second marriage during the
subsistence of his first marriage. After the death of his first wife, accused contracted a third marriage during the subsistence of
the second marriage. The second wife initiated a complaint for bigamy. The Court acquitted accused on the ground that the
second marriage is void, having been contracted during the existence of the first marriage. There is no need for a judicial
declaration that said second marriage is void. Since the second marriage is void, and the first one terminated by the death of
his wife, there are no two subsisting valid marriages. Hence, there can be no bigamy. Justice Alex Reyes dissented in both
cases, saying that it is not for the spouses but the court to judge whether a marriage is void or not.
In Gomez v. Lipana,[12] and Consuegra v. Consuegra,[13] however, we recognized the right of the second wife who entered
into the marriage in good faith, to share in their acquired estate and in proceeds of the retirement insurance of the
husband. The Court observed that although the second marriage can be presumed to be void ab initio as it was celebrated
while the first marriage was still subsisting, still there was a need for judicial declaration of such nullity (of the second
marriage). And since the death of the husband supervened before such declaration, we upheld the right of the second wife to
share in the estate they acquired, on grounds of justice and equity.[14]
But in Odayat v. Amante (1977),[15] the Court adverted to Aragon and Mendoza as precedents. We exonerated a clerk of
court of the charge of immorality on the ground that his marriage to Filomena Abella in October of 1948 was void, since she was
already previously married to one Eliseo Portales in February of the same year. The Court held that no judicial decree is
necessary to establish the invalidity of void marriages. This ruling was affirmed in Tolentino v. Paras.[16]
Yet again in Wiegel v. Sempio-Diy (1986),[17] the Court held that there is a need for a judicial declaration of nullity of a void
marriage. InWiegel, Lilia married Maxion in 1972. In 1978, she married another man, Wiegel. Wiegel filed a petition with the
Juvenile Domestic Relations Court to declare his marriage to Lilia as void on the ground of her previous valid marriage. The
Court, expressly relying on Consuegra, concluded that:[18]
There is likewise no need of introducing evidence about the existing prior marriage of her first husband at the time they
married each other, for then such a marriage though void still needs according to this Court a judicial declaration
(citing Consuegra) of such fact and for all legal intents and purposes she would still be regarded as a married woman at the
time she contracted her marriage with respondent Karl Heinz Wiegel; accordingly, the marriage of petitioner and respondent
would be regarded VOID under the law. (Emphasis supplied).
In Yap v. Court of Appeals,[19] however, the Court found the second marriage void without need of judicial declaration, thus
reverting to theOdayat, Mendoza and Aragon rulings.
At any rate, the confusion under the Civil Code was put to rest under the Family Code. Our rulings in Gomez, Consuegra,
and Wiegel were eventually embodied in Article 40 of the Family Code. [20] Article 40 of said Code expressly required a judicial
declaration of nullity of marriage
Art. 40. The absolute nullity of a previous marriage may be invoked for purposes of remarriage on the basis solely of a final
judgment declaring such previous marriage void.
In Terre v. Terre (1992)[21] the Court, applying Gomez, Consuegra and Wiegel, categorically stated that a judicial declaration
of nullity of a void marriage is necessary. Thus, we disbarred a lawyer for contracting a bigamous marriage during the
subsistence of his first marriage. He claimed that his first marriage in 1977 was void since his first wife was already married in
1968. We held that Atty. Terre should have known that the prevailing case law is that for purposes of determining whether a
person is legally free to contract a second marriage, a judicial declaration that the first marriage was null and void ab initio is
essential.
The Court applied this ruling in subsequent cases. In Domingo v. Court of Appeals (1993),[22] the Court held:
Came the Family Code which settled once and for all the conflicting jurisprudence on the matter. A declaration of absolute
nullity of marriage is now explicitly required either as a cause of action or a ground for defense. (Art. 39 of the Family
Code). Where the absolute nullity of a previous marriage is sought to be invoked for purposes of contracting a second
marriage, the sole basis acceptable in law for said projected marriage to be free from legal infirmity is a final judgment declaring
the previous marriage void. (Family Code, Art. 40; See also arts. 11, 13, 42, 44, 48, 50, 52, 54, 86, 99, 147, 148). [23]
However, a recent case applied the old rule because of the peculiar circumstances of the case. In Apiag v. Cantero, (1997)
the first wife charged a municipal trial judge of immorality for entering into a second marriage. The judge claimed that his first
marriage was void since he was merely forced into marrying his first wife whom he got pregnant. On the issue of nullity of the
first marriage, we applied Odayat, Mendozaand Aragon. We held that since the second marriage took place and all the children
thereunder were born before the promulgation of Wiegeland the effectivity of the Family Code, there is no need for a judicial
declaration of nullity of the first marriage pursuant to prevailing jurisprudence at that time.
[24]

Similarly, in the present case, the second marriage of private respondent was entered into in 1979, before Wiegel. At that
time, the prevailing rule was found in Odayat, Mendoza and Aragon. The first marriage of private respondent being void for lack
of license and consent, there was no need for judicial declaration of its nullity before he could contract a second marriage. In
this case, therefore, we conclude that private respondents second marriage to petitioner is valid.
Moreover, we find that the provisions of the Family Code cannot be retroactively applied to the present case, for to do so
would prejudice the vested rights of petitioner and of her children. As held in Jison v. Court of Appeals,[25] the Family Code has
retroactive effect unless there be impairment of vested rights. In the present case, that impairment of vested rights of petitioner
and the children is patent. Additionally, we are not quite prepared to give assent to the appellate courts finding that despite
private respondents deceit and perfidy in contracting marriage with petitioner, he could benefit from her silence on the
issue. Thus, coming now to the civil effects of the church ceremony wherein petitioner married private respondent using the

marriage license used three years earlier in the civil ceremony, we find that petitioner now has raised this matter
properly. Earlier petitioner claimed as untruthful private respondents allegation that he wed petitioner but they lacked a
marriage license. Indeed we find there was a marriage license, though it was the same license issued on April 3, 1979 and used
in both the civil and the church rites. Obviously, the church ceremony was confirmatory of their civil marriage. As petitioner
contends, the appellate court erred when it refused to recognize the validity and salutary effects of said canonical marriage on a
technicality, i.e. that petitioner had failed to raise this matter as affirmative defense during trial. She argues that such failure
does not prevent the appellate court from giving her defense due consideration and weight. She adds that the interest of the
State in protecting the inviolability of marriage, as a legal and social institution, outweighs such technicality. In our view,
petitioner and private respondent had complied with all the essential and formal requisites for a valid marriage, including the
requirement of a valid license in the first of the two ceremonies. That this license was used legally in the celebration of the civil
ceremony does not detract from the ceremonial use thereof in the church wedding of the same parties to the marriage, for we
hold that the latter rites served not only to ratify but also to fortify the first. The appellate court might have its reasons for
brushing aside this possible defense of the defendant below which undoubtedly could have tendered a valid issue, but which
was not timely interposed by her before the trial court. But we are now persuaded we cannot play blind to the absurdity, if not
inequity, of letting the wrongdoer profit from what the CA calls his own deceit and perfidy.
On the matter of petitioners counterclaim for damages and attorneys fees. Although the appellate court admitted that they
found private respondent acted duplicitously and craftily in marrying petitioner, it did not award moral damages because the
latter did not adduce evidence to support her claim. [26]
Like the lower courts, we are also of the view that no damages should be awarded in the present case, but for another
reason. Petitioner wants her marriage to private respondent held valid and subsisting. She is suing to maintain her status as
legitimate wife. In the same breath, she asks for damages from her husband for filing a baseless complaint for annulment of
their marriage which caused her mental anguish, anxiety, besmirched reputation, social humiliation and alienation from her
parents. Should we grant her prayer, we would have a situation where the husband pays the wife damages from conjugal or
common funds. To do so, would make the application of the law absurd. Logic, if not common sense, militates against such
incongruity. Moreover, our laws do not comprehend an action for damages between husband and wife merely because of
breach of a marital obligation.[27] There are other remedies.[28]
WHEREFORE, the petition is GRANTED. The assailed Decision of the Court of Appeals dated July 24, 1996 and its
Resolution dated November 7, 1996, are reversed partially, so that the marriage of petitioner Ofelia P. Ty and private respondent
Edgardo M. Reyes is hereby DECLARED VALID AND SUBSISTING; and the award of the amount of P15,000.00 is RATIFIED
and MAINTAINED as monthly support to their two children, Faye Eloise Reyes and Rachel Anne Reyes, for as long as they are
of minor age or otherwise legally entitled thereto. Costs against private respondent.

G.R. No. 118305 February 12, 1998


AYALA INVESTMENT & DEVELOPMENT CORP. and ABELARDO MAGSAJO, petitioners,
vs.
COURT OF APPEALS and SPOUSES ALFREDO & ENCARNACION CHING, respondents.

MARTINEZ, J.:
Under Article 161 of the Civil Code, what debts and obligations contracted by the husband alone are considered "for the benefit
of the conjugal partnership" which are chargeable against the conjugal partnership? Is a surety agreement or an
accommodation contract entered into by the husband in favor of his employer within the contemplation of the said provision?
These are the issues which we will resolve in this petition for review.
The petitioner assails the decision dated April 14, 1994 of the respondent Court of Appeals in "Spouses Alfredo and
Encarnacion Ching vs. Ayala Investment and Development Corporation, et. al.," docketed as CA-G.R. CV No.
29632, 1 upholding the decision of the Regional Trial Court of Pasig, Branch 168, which ruled that the conjugal partnership of
gains of respondents-spouses Alfredo and Encarnacion Ching is not liable for the payment of the debts secured by respondenthusband Alfredo Ching.
A chronology of the essential antecedent facts is necessary for a clear understanding of the case at bar.
Philippine Blooming Mills (hereinafter referred to as PBM) obtained a P50,300,000.00 loan from petitioner Ayala Investment and
Development Corporation (hereinafter referred to as AIDC). As added security for the credit line extended to PBM, respondent
Alfredo Ching, Executive Vice President of PBM, executed security agreements on December 10, 1980 and on March 20, 1981
making himself jointly and severally answerable with PBM's indebtedness to AIDC.
PBM failed to pay the loan. Thus, on July 30, 1981, AIDC filed a case for sum of money against PBM and respondent-husband
Alfredo Ching with the then Court of First Instance of Rizal (Pasig), Branch VIII, entitled "Ayala Investment and Development
Corporation vs. Philippine Blooming Mills and Alfredo Ching," docketed as Civil Case No. 42228.
After trial, the court rendered judgment ordering PBM and respondent-husband Alfredo Ching to jointly and severally pay AIDC
the principal amount of P50,300,000.00 with interests.
Pending appeal of the judgment in Civil Case No. 42228, upon motion of AIDC, the lower court issued a writ of execution
pending appeal. Upon AIDC's putting up of an P8,000,000.00 bond, a writ of execution dated May 12, 1982 was issued.
Thereafter, petitioner Abelardo Magsajo, Sr., Deputy Sheriff of Rizal and appointed sheriff in Civil Case No. 42228, caused the
issuance and service upon respondents-spouses of a notice of sheriff sale dated May 20, 1982 on three (3) of their conjugal
properties. Petitioner Magsajo then scheduled the auction sale of the properties levied.
On June 9, 1982, private respondents filed a case of injunction against petitioners with the then Court of First Instance of Rizal
(Pasig), Branch XIII, to enjoin the auction sale alleging that petitioners cannot enforce the judgment against the conjugal
partnership levied on the ground that, among others, the subject loan did not redound to the benefit of the said conjugal
partnership. 2 Upon application of private respondents, the lower court issued a temporary restraining order to prevent petitioner
Magsajo from proceeding with the enforcement of the writ of execution and with the sale of the said properties at public auction.
AIDC filed a petition for certiorari before the Court of Appeals, 3 questioning the order of the lower court enjoining the sale.
Respondent Court of Appeals issued a Temporary Restraining Order on June 25, 1982, enjoining the lower court 4 from
enforcing its Order of June 14, 1982, thus paving the way for the scheduled auction sale of respondents-spouses conjugal
properties.
On June 25, 1982, the auction sale took place. AIDC being the only bidder, was issued a Certificate of Sale by petitioner
Magsajo, which was registered on July 2, 1982. Upon expiration of the redemption period, petitioner sheriff issued the final deed
of sale on August 4, 1982 which was registered on August 9, 1983.
In the meantime, the respondent court, on August 4, 1982, decided CA-G.R. SP No. 14404, in this manner:
WHEREFORE, the petition for certiorari in this case is granted and the challenged order of the respondent Judge
dated June 14, 1982 in Civil Case No. 46309 is hereby set aside and nullified. The same petition insofar as it seeks
to enjoin the respondent Judge from proceeding with Civil Case No. 46309 is, however, denied. No pronouncement
is here made as to costs. . . . 5
On September 3, 1983, AIDC filed a motion to dismiss the petition for injunction filed before Branch XIII of the CFI of Rizal
(Pasig) on the ground that the same had become moot and academic with the consummation of the sale. Respondents filed
their opposition to the motion arguing, among others, that where a third party who claim is ownership of the property attached or
levied upon, a different legal situation is presented; and that in this case, two (2) of the real properties are actually in the name
of Encarnacion Ching, a non-party to Civil Case No. 42228.
The lower court denied the motion to dismiss. Hence, trial on the merits proceeded. Private respondents presented several
witnesses. On the other hand, petitioners did not present any evidence.
On September 18, 1991, the trial court promulgated its decision declaring the sale on execution null and void. Petitioners
appealed to the respondent court, which was docketed as CA-G.R. CV No. 29632.
On April 14, 1994, the respondent court promulgated the assailed decision, affirming the decision of the regional trial court. It
held that:
The loan procured from respondent-appellant AIDC was for the advancement and benefit of Philippine Blooming
Mills and not for the benefit of the conjugal partnership of petitioners-appellees.
xxx xxx xxx
As to the applicable law, whether it is Article 161 of the New Civil Code or Article 1211 of the Family Code-suffice it
to say that the two provisions are substantially the same. Nevertheless, We agree with the trial court that the Family
Code is the applicable law on the matter . . . . . . .
Article 121 of the Family Code provides that "The conjugal partnership shall be liable for: . . . (2) All debts and
obligations contracted during the marriage by the designated Administrator-Spouse for the benefit of the conjugal
partnership of gains . . . ." The burden of proof that the debt was contracted for the benefit of the conjugal
partnership of gains, lies with the creditor-party litigant claiming as such. In the case at bar, respondent-appellant
AIDC failed to prove that the debt was contracted by appellee-husband, for the benefit of the conjugal partnership of
gains.
The dispositive portion of the decision reads:

WHEREFORE, in view of all the foregoing, judgment is hereby rendered DISMISSING the appeal. The decision of
the Regional Trial Court is AFFIRMED in toto. 6
Petitioner filed a Motion for Reconsideration which was denied by the respondent court in a Resolution dated November 28,
1994. 7
Hence, this petition for review. Petitioner contends that the "respondent court erred in ruling that the conjugal partnership of
private respondents is not liable for the obligation by the respondent-husband."
Specifically, the errors allegedly committed by the respondent court are as follows:
I. RESPONDENT COURT ERRED IN RULING THAT THE OBLIGATION INCURRED RESPONDENT
HUSBAND DID NOT REDOUND TO THE BENEFIT OF THE CONJUGAL PARTNERSHIP OF THE
PRIVATE RESPONDENT.
II. RESPONDENT COURT ERRED IN RULING THAT THE ACT OF RESPONDENT HUSBAND IN
SECURING THE SUBJECT LOAN IS NOT PART OF HIS INDUSTRY, BUSINESS OR CAREER FROM
WHICH HE SUPPORTS HIS FAMILY.
Petitioners in their appeal point out that there is no need to prove that actual benefit redounded to the benefit of the partnership;
all that is necessary, they say, is that the transaction was entered into for the benefit of the conjugal partnership. Thus,
petitioners aver that:
The wordings of Article 161 of the Civil Code is very clear: for the partnership to be held liable, the husband must
have contracted the debt "for the benefit of the partnership, thus:
Art. 161. The conjugal partnership shall be liable for:
1) all debts and obligations contracted by the husband for the benefit of the conjugal
partnership . . . .
There is a difference between the phrases: "redounded to the benefit of" or "benefited from" (on the one hand) and
"for the benefit of (on the other). The former require that actual benefit must have been realized; the latter requires
only that the transaction should be one which normally would produce benefit to the partnership, regardless of
whether or not actual benefit accrued. 8
We do not agree with petitioners that there is a difference between the terms "redounded to the benefit of" or "benefited from"
on the one hand; and "for the benefit of" on the other. They mean one and the same thing. Article 161 (1) of the Civil Code and
Article 121 (2) of the Family Code are similarly worded, i.e., both use the term "for the benefit of." On the other hand, Article 122
of the Family Code provides that "The payment of personal debts by the husband or the wife before or during the marriage shall
not be charged to the conjugal partnership except insofar as they redounded to the benefit of the family." As can be seen, the
terms are used interchangeably.
Petitioners further contend that the ruling of the respondent court runs counter to the pronouncement of this Court in the case
of Cobb-Perez vs. Lantin, 9 that the husband as head of the family and as administrator of the conjugal partnership is presumed
to have contracted obligations for the benefit of the family or the conjugal partnership.
Contrary to the contention of the petitioners, the case of Cobb-Perez is not applicable in the case at bar. This Court has, on
several instances, interpreted the term "for the benefit of the conjugal partnership."
In the cases of Javier vs. Osmea, 10 Abella de Diaz vs. Erlanger & Galinger, Inc., 11 Cobb-Perez vs. Lantin 12 and G-Tractors,
Inc. vs. Court of Appeals, 13 cited by the petitioners, we held that:
The debts contracted by the husband during the marriage relation, for and in the exercise of the industry or
profession by which he contributes toward the support of his family, are not his personal and private debts, and the
products or income from the wife's own property, which, like those of her husband's, are liable for the payment of the
marriage expenses, cannot be excepted from the payment of such debts. (Javier)
The husband, as the manager of the partnership (Article 1412, Civil Code), has a right to embark the partnership in
an ordinary commercial enterprise for gain, and the fact that the wife may not approve of a venture does not make it
a private and personal one of the husband. (Abella de Diaz)
Debts contracted by the husband for and in the exercise of the industry or profession by which he contributes to the
support of the family, cannot be deemed to be his exclusive and private debts. (Cobb-Perez).
. . . if he incurs an indebtedness in the legitimate pursuit of his career or profession or suffers losses in a legitimate
business, the conjugal partnership must equally bear the indebtedness and the losses, unless he deliberately acted
to the prejudice of his family. (G-Tractors)
However, in the cases of Ansaldo vs. Sheriff of Manila, Fidelity Insurance & Luzon Insurance Co., 14 Liberty Insurance
Corporation vs. Banuelos, 15 and Luzon Surety Inc. vs. De Garcia, 16 cited by the respondents, we ruled that:
The fruits of the paraphernal property which form part of the assets of the conjugal partnership, are subject to the
payment of the debts and expenses of the spouses, but not to the payment of the personal obligations (guaranty
agreements) of the husband, unless it be proved that such obligations were productive of some benefit to the
family." (Ansaldo; parenthetical phrase ours.)
When there is no showing that the execution of an indemnity agreement by the husband redounded to the benefit of
his family, the undertaking is not a conjugal debt but an obligation personal to him. (Liberty Insurance)
In the most categorical language, a conjugal partnership under Article 161 of the new Civil Code is liable only for
such "debts and obligations contracted by the husband for the benefit of the conjugal partnership." There must be
the requisite showing then of some advantage which clearly accrued to the welfare of the spouses. Certainly, to
make a conjugal partnership respond for a liability that should appertain to the husband alone is to defeat and
frustrate the avowed objective of the new Civil Code to show the utmost concern for the solidarity and well-being of
the family as a unit. The husband, therefore, is denied the power to assume unnecessary and unwarranted risks to
the financial stability of the conjugal partnership. (Luzon Surety, Inc.)
From the foregoing jurisprudential rulings of this Court, we can derive the following conclusions:
(A) If the husband himself is the principal obligor in the contract, i.e., he directly received the money and services to be used in
or for his own business or his own profession, that contract falls within the term . . . . obligations for the benefit of the conjugal
partnership." Here, no actual benefit may be proved. It is enough that the benefit to the family is apparent at the time of the
signing of the contract. From the very nature of the contract of loan or services, the family stands to benefit from the loan facility
or services to be rendered to the business or profession of the husband. It is immaterial, if in the end, his business or profession
fails or does not succeed. Simply stated, where the husband contracts obligations on behalf of the family business, the law
presumes, and rightly so, that such obligation will redound to the benefit of the conjugal partnership.

(B) On the other hand, if the money or services are given to another person or entity, and the husband acted only as
a surety or guarantor, that contract cannot, by itself, alone be categorized as falling within the context of "obligations for the
benefit of the conjugal partnership." The contract of loan or services is clearly for the benefit of the principal debtor and not for
the surety or his family. No presumption can be inferred that, when a husband enters into a contract of surety or accommodation
agreement, it is "for the benefit of the conjugal partnership." Proof must be presented to establish benefit redounding to the
conjugal partnership.
Thus, the distinction between the Cobb-Perez case, and we add, that of the three other companion cases, on the one hand, and
that of Ansaldo, Liberty Insurance and Luzon Surety, is that in the former, the husband contracted the obligation for his own
business; while in the latter, the husband merely acted as a surety for the loan contracted by another for the latter's business.
The evidence of petitioner indubitably show that co-respondent Alfredo Ching signed as surety for the P50M loan contracted on
behalf of PBM. petitioner should have adduced evidence to prove that Alfredo Ching's acting as surety redounded to the benefit
of the conjugal partnership. The reason for this is as lucidly explained by the respondent court:
The loan procured from respondent-appellant AIDC was for the advancement and benefit of Philippine Blooming
Mills and not for the benefit of the conjugal partnership of petitioners-appellees. Philippine Blooming Mills has a
personality distinct and separate from the family of petitioners-appellees this despite the fact that the members of
the said family happened to be stockholders of said corporate entity.
xxx xxx xxx
. . . . The burden of proof that the debt was contracted for the benefit of the conjugal partnership of gains, lies with
the creditor-party litigant claiming as such. In the case at bar, respondent-appellant AIDC failed to prove that the
debt was contracted by appellee-husband, for the benefit of the conjugal partnership of gains. What is apparent from
the facts of the case is that the judgment debt was contracted by or in the name of the Corporation Philippine
Blooming Mills and appellee-husband only signed as surety thereof. The debt is clearly a corporate debt and
respondent-appellant's right of recourse against appellee-husband as surety is only to the extent of his corporate
stockholdings. It does not extend to the conjugal partnership of gains of the family of petitioners-appellees. . . . . . . 17
Petitioners contend that no actual benefit need accrue to the conjugal partnership. To support this contention, they cite Justice
J.B.L. Reyes' authoritative opinion in the Luzon Surety Company case:
I concur in the result, but would like to make of record that, in my opinion, the words "all debts and obligations
contracted by the husband for the benefit of the conjugal partnership" used in Article 161 of the Civil Code of the
Philippines in describing the charges and obligations for which the conjugal partnership is liable do not require that
actual profit or benefit must accrue to the conjugal partnership from the husband's transaction; but it suffices that the
transaction should be one that normally would produce such benefit for the partnership. This is the ratio behind our
ruling in Javier vs. Osmea, 34 Phil. 336, that obligations incurred by the husband in the practice of his profession
are collectible from the conjugal partnership.
The aforequoted concurring opinion agreed with the majority decision that the conjugal partnership should not be made liable
for the surety agreement which was clearly for the benefit of a third party. Such opinion merely registered an exception to what
may be construed as a sweeping statement that in all cases actual profit or benefit must accrue to the conjugal partnership. The
opinion merely made it clear that no actual benefits to the family need be proved in some cases such as in the Javier case.
There, the husband was the principal obligor himself. Thus, said transaction was found to be "one that would normally produce .
. . benefit for the partnership." In the later case of G-Tractors, Inc., the husband was also the principal obligor not merely the
surety. This latter case, therefore, did not create any precedent. It did not also supersede the Luzon Surety Company case, nor
any of the previous accommodation contract cases, where this Court ruled that they were for the benefit of third parties.
But it could be argued, as the petitioner suggests, that even in such kind of contract of accommodation, a benefit for the family
may also result, when the guarantee is in favor of the husband's employer.
In the case at bar, petitioner claims that the benefits the respondent family would reasonably anticipate were the following:
(a) The employment of co-respondent Alfredo Ching would be prolonged and he would be entitled to his
monthly salary of P20,000.00 for an extended length of time because of the loan he guaranteed;
(b) The shares of stock of the members of his family would appreciate if the PBM could be rehabilitated
through the loan obtained;
(c) His prestige in the corporation would be enhanced and his career would be boosted should PBM
survive because of the loan.
However, these are not the benefits contemplated by Article 161 of the Civil Code. The benefits must be one directly resulting
from the loan. It cannot merely be a by-product or a spin-off of the loan itself.
In all our decisions involving accommodation contracts of the husband, 18 we underscored the requirement that: "there must be
the requisite showing . . . of some advantage which clearly accrued to the welfare of the spouses" or "benefits to his family" or
"that such obligations are productive of some benefit to the family." Unfortunately, the petition did not present any proof to show:
(a) Whether or not the corporate existence of PBM was prolonged and for how many months or years; and/or (b) Whether or not
the PBM was saved by the loan and its shares of stock appreciated, if so, how much and how substantial was the holdings of
the Ching family.
Such benefits (prospects of longer employment and probable increase in the value of stocks) might have been already apparent
or could be anticipated at the time the accommodation agreement was entered into. But would those "benefits" qualify the
transaction as one of the "obligations . . . for the benefit of the conjugal partnership"? Are indirect and remote probable benefits,
the ones referred to in Article 161 of the Civil Code? The Court of Appeals in denying the motion for reconsideration, disposed of
these questions in the following manner:
No matter how one looks at it, the debt/credit respondents-appellants is purely a corporate debt granted to PBM,
with petitioner-appellee-husband merely signing as surety. While such petitioner-appellee-husband, as such surety,
is solidarily liable with the principal debtor AIDC, such liability under the Civil Code provisions is specifically
restricted by Article 122 (par. 1) of the Family Code, so that debts for which the husband is liable may not be
charged against conjugal partnership properties. Article 122 of the Family Code is explicit "The payment of
personal debts contracted by the husband or the wife before or during the marriage shall not be charged to the
conjugal partnership except insofar as they redounded to the benefit of the family.
Respondents-appellants insist that the corporate debt in question falls under the exception laid down in said Article
122 (par. one). We do not agree. The loan procured from respondent-appellant AIDC was for the sole advancement
and benefit of Philippine Blooming Mills and not for the benefit of the conjugal partnership of petitioners-appellees.
. . . appellee-husband derives salaries, dividends benefits from Philippine Blooming Mills (the debtor corporation),
only because said husband is an employee of said PBM. These salaries and benefits, are not the "benefits"
contemplated by Articles 121 and 122 of the Family Code. The "benefits" contemplated by the exception in Article
122 (Family Code) is that benefit derived directly from the use of the loan. In the case at bar, the loan is a corporate

loan extended to PBM and used by PBM itself, not by petitioner-appellee-husband or his family. The alleged benefit,
if any, continuously harped by respondents-appellants, are not only incidental but also speculative. 19
We agree with the respondent court. Indeed, considering the odds involved in guaranteeing a large amount (P50,000,000.00) of
loan, the probable prolongation of employment in PBM and increase in value of its stocks, would be too small to qualify the
transaction as one "for the benefit" of the surety's family. Verily, no one could say, with a degree of certainty, that the said
contract is even "productive of some benefits" to the conjugal partnership.
We likewise agree with the respondent court (and this view is not contested by the petitioners) that the provisions of the Family
Code is applicable in this case. These provisions highlight the underlying concern of the law for the conservation of the conjugal
partnership; for the husband's duty to protect and safeguard, if not augment, not to dissipate it.
This is the underlying reason why the Family Code clarifies that the obligations entered into by one of the spouses must be
those that redounded to the benefit of the family and that the measure of the partnership's liability is to "the extent that the family
is benefited." 20
These are all in keeping with the spirit and intent of the other provisions of the Civil Code which prohibits any of the spouses to
donate or convey gratuitously any part of the conjugal property. 21 Thus, when co-respondent Alfredo Ching entered into a surety
agreement he, from then on, definitely put in peril the conjugal property (in this case, including the family home) and placed it in
danger of being taken gratuitously as in cases of donation.
In the second assignment of error, the petitioner advances the view that acting as surety is part of the business or profession of
the respondent-husband.
This theory is new as it is novel.
The respondent court correctly observed that:
Signing as a surety is certainly not an exercise of an industry or profession, hence the cited cases ofCobb-Perez
vs. Lantin; Abella de Diaz vs. Erlanger & Galinger; G-Tractors, Inc. vs. CA do not apply in the instant case. Signing
as a surety is not embarking in a business. 22
We are likewise of the view that no matter how often an executive acted or was persuaded to act, as a surety for his own
employer, this should not be taken to mean that he had thereby embarked in the business of suretyship or guaranty.
This is not to say, however, that we are unaware that executives are often asked to stand as surety for their company's loan
obligations. This is especially true if the corporate officials have sufficient property of their own; otherwise, their spouses'
signatures are required in order to bind the conjugal partnerships.
The fact that on several occasions the lending institutions did not require the signature of the wife and the husband signed alone
does not mean that being a surety became part of his profession. Neither could he be presumed to have acted for the conjugal
partnership.
Article 121, paragraph 3, of the Family Code is emphatic that the payment of personal debts contracted by the husband or the
wife before or during the marriage shall not be charged to the conjugal partnership except to the extent that they redounded to
the benefit of the family.
Here, the property in dispute also involves the family home. The loan is a corporate loan not a personal one. Signing as a surety
is certainly not an exercise of an industry or profession nor an act of administration for the benefit of the family.
On the basis of the facts, the rules, the law and equity, the assailed decision should be upheld as we now uphold it. This is, of
course, without prejudice to petitioner's right to enforce the obligation in its favor against the PBM receiver in accordance with
the rehabilitation program and payment schedule approved or to be approved by the Securities & Exchange Commission.
WHEREFORE, the petition for review should be, as it is hereby, DENIED for lack of merit.

Property Relations

[G.R. No. 116668. July 28, 1997]

ERLINDA A. AGAPAY, petitioner, vs. CARLINA (CORNELIA) V. PALANG and HERMINIA P. DELA CRUZ,respondents.
DECISION
ROMERO, J.:
Before us is a petition for review of the decision of the Court of Appeals in CA-G.R. CV No. 24199 entitled Erlinda Agapay v.
Carlina (Cornelia) Palang and Herminia P. Dela Cruz dated June 22, 1994 involving the ownership of two parcels of land
acquired during the cohabitation of petitioner and private respondents legitimate spouse.
Miguel Palang contracted his first marriage on July 16, 1949 when he took private respondent Carlina (or Cornelia)
Vallesterol as a wife at the Pozorrubio Roman Catholic Church in Pangasinan. A few months after the wedding, in October
1949, he left to work in Hawaii. Miguel and Carlinas only child, Herminia Palang, was born on May 12, 1950.
Miguel returned in 1954 for a year. His next visit to the Philippines was in 1964 and during the entire duration of his yearlong sojourn he stayed in Zambales with his brother, not in Pangasinan with his wife and child. The trial court found evidence
that as early as 1957, Miguel had attempted to divorce Carlina in Hawaii. [1] When he returned for good in 1972, he refused to
live with private respondents, but stayed alone in a house in Pozorrubio, Pangasinan.
On July 15, 1973, the then sixty-three-year-old Miguel contracted his second marriage with nineteen-year-old Erlinda
Agapay, herein petitioner.[2] Two months earlier, on May 17, 1973, Miguel and Erlinda, as evidenced by the Deed of Sale, jointly
purchased a parcel of agricultural land located at San Felipe, Binalonan, Pangasinan with an area of 10,080 square meters.
Consequently, Transfer Certificate of Title No. 101736 covering said rice land was issued in their names.
A house and lot in Binalonan, Pangasinan was likewise purchased on September 23, 1975, allegedly by Erlinda as the sole
vendee. TCT No. 143120 covering said property was later issued in her name.
On October 30, 1975, Miguel and Cornelia Palang executed a Deed of Donation as a form of compromise agreement to
settle and end a case filed by the latter.[3] The parties therein agreed to donate their conjugal property consisting of six parcels of
land to their only child, Herminia Palang.[4]
Miguel and Erlindas cohabitation produced a son, Kristopher A. Palang, born on December 6, 1977. In 1979, Miguel and
Erlinda were convicted of Concubinage upon Carlinas complaint. [5] Two years later, on February 15, 1981, Miguel died.
On July 11, 1981, Carlina Palang and her daughter Herminia Palang de la Cruz, herein private respondents, instituted the
case at bar, an action for recovery of ownership and possession with damages against petitioner before the Regional Trial Court
in Urdaneta, Pangasinan (Civil Case No. U-4265). Private respondents sought to get back the riceland and the house and lot
both located at Binalonan, Pangasinan allegedly purchased by Miguel during his cohabitation with petitioner.
Petitioner, as defendant below, contended that while the riceland covered by TCT No. 101736 is registered in their names
(Miguel and Erlinda), she had already given her half of the property to their son Kristopher Palang. She added that the house
and lot covered by TCT No. 143120 is her sole property, having bought the same with her own money. Erlinda added that
Carlina is precluded from claiming aforesaid properties since the latter had already donated their conjugal estate to Herminia.
After trial on the merits, the lower court rendered its decision on June 30, 1989 dismissing the complaint after declaring that
there was little evidence to prove that the subject properties pertained to the conjugal property of Carlina and Miguel
Palang. The lower court went on to provide for the intestate shares of the parties, particularly of Kristopher Palang, Miguels
illegitimate son. The dispositive portion of the decision reads:
WHEREFORE, premises considered, judgment is hereby rendered1)

Dismissing the complaint, with costs against plaintiffs;

2)
Confirming the ownership of defendant Erlinda Agapay of the residential lot located at Poblacion, Binalonan,
Pangasinan, as evidenced by TCT No. 143120, Lot 290-B including the old house standing therein;
3)
Confirming the ownership of one-half (1/2) portion of that piece of agricultural land situated at Balisa, San Felipe,
Binalonan, Pangasinan, consisting of 10,080 square meters and as evidenced by TCT No. 101736, Lot 1123-A to Erlinda
Agapay;
4)
Adjudicating to Kristopher Palang as his inheritance from his deceased father, Miguel Palang, the one-half (1/2) of the
agricultural land situated at Balisa, San Felipe, Binalonan, Pangasinan, under TCT No. 101736 in the name of Miguel Palang,
provided that the former (Kristopher) executes, within 15 days after this decision becomes final and executory, a quit-claim
forever renouncing any claims to annul/reduce the donation to Herminia Palang de la Cruz of all conjugal properties of her
parents, Miguel Palang and Carlina Vallesterol Palang, dated October 30, 1975, otherwise, the estate of deceased Miguel
Palang will have to be settled in another separate action;
5)

No pronouncement as to damages and attorneys fees.

SO ORDERED.[6]
On appeal, respondent court reversed the trial courts decision. The Court of Appeals rendered its decision on July 22, 1994
with the following dispositive portion:
WHEREFORE, PREMISES CONSIDERED, the appealed decision is hereby REVERSED and another one entered:
1.

Declaring plaintiffs-appellants the owners of the properties in question;

2.

Ordering defendant-appellee to vacate and deliver the properties in question to herein plaintiffs-appellants;

3.
Ordering the Register of Deeds of Pangasinan to cancel Transfer Certificate of Title Nos. 143120 and 101736 and to
issue in lieu thereof another certificate of title in the name of plaintiffs-appellants.
No pronouncement as to costs.[7]
Hence, this petition.
Petitioner claims that the Court of Appeals erred in not sustaining the validity of two deeds of absolute sale covering the
riceland and the house and lot, the first in favor of Miguel Palang and Erlinda Agapay and the second, in favor of Erlinda Agapay
alone. Second, petitioner contends that respondent appellate court erred in not declaring Kristopher A. Palang as Miguel
Palangs illegitimate son and thus entitled to inherit from Miguels estate. Third, respondent court erred, according to petitioner,
in not finding that there is sufficient pleading and evidence that Kristoffer A. Palang or Christopher A. Palang should be
considered as party-defendant in Civil Case No. U-4625 before the trial court and in CA-G.R. No. 24199. [8]
After studying the merits of the instant case, as well as the pertinent provisions of law and jurisprudence, the Court denies
the petition and affirms the questioned decision of the Court of Appeals.

The first and principal issue is the ownership of the two pieces of property subject of this action. Petitioner assails the
validity of the deeds of conveyance over the same parcels of land. There is no dispute that the transfers of ownership from the
original owners of the riceland and the house and lot, Corazon Ilomin and the spouses Cespedes, respectively, were valid.
The sale of the riceland on May 17, 1973, was made in favor of Miguel and Erlinda. The provision of law applicable here is
Article 148 of the Family Code providing for cases of cohabitation when a man and a woman who are not capacitated to marry
each other live exclusively with each other as husband and wife without the benefit of marriage or under a void marriage. While
Miguel and Erlinda contracted marriage on July 15, 1973, said union was patently void because the earlier marriage of Miguel
and Carlina was still susbsisting and unaffected by the latters de facto separation.
Under Article 148, only the properties acquired by both of the parties through their actual joint contribution of money,
property or industry shall be owned by them in common in proportion to their respective contributions. It must be stressed that
actual contribution is required by this provision, in contrast to Article 147 which states that efforts in the care and maintenance of
the family and household, are regarded as contributions to the acquisition of common property by one who has no salary or
income or work or industry. If the actual contribution of the party is not proved, there will be no co-ownership and no
presumption of equal shares.[9]
In the case at bar, Erlinda tried to establish by her testimony that she is engaged in the business of buy and sell and had
a sari-sari store[10]but failed to persuade us that she actually contributed money to buy the subject riceland. Worth noting is the
fact that on the date of conveyance, May 17, 1973, petitioner was only around twenty years of age and Miguel Palang was
already sixty-four and a pensioner of the U.S. Government. Considering her youthfulness, it is unrealistic to conclude that in
1973 she contributed P3,750.00 as her share in the purchase price of subject property,[11] there being no proof of the same.
Petitioner now claims that the riceland was bought two months before Miguel and Erlinda actually cohabited. In the nature
of an afterthought, said added assertion was intended to exclude their case from the operation of Article 148 of the Family
Code. Proof of the precise date when they commenced their adulterous cohabitation not having been adduced, we cannot state
definitively that the riceland was purchased even before they started living together. In any case, even assuming that the
subject property was bought before cohabitation, the rules of co-ownership would still apply and proof of actual contribution
would still be essential.
Since petitioner failed to prove that she contributed money to the purchase price of the riceland in Binalonan, Pangasinan,
we find no basis to justify her co-ownership with Miguel over the same. Consequently, the riceland should, as correctly held by
the Court of Appeals, revert to the conjugal partnership property of the deceased Miguel and private respondent Carlina Palang.
Furthermore, it is immaterial that Miguel and Carlina previously agreed to donate their conjugal property in favor of their
daughter Herminia in 1975. The trial court erred in holding that the decision adopting their compromise agreement in effect
partakes the nature of judicial confirmation of the separation of property between spouses and the termination of the conjugal
partnership.[12] Separation of property between spouses during the marriage shall not take place except by judicial order or
without judicial conferment when there is an express stipulation in the marriage settlements. [13] The judgment which resulted
from the parties compromise was not specifically and expressly for separation of property and should not be so inferred.
With respect to the house and lot, Erlinda allegedly bought the same for P20,000.00 on September 23, 1975 when she was
only 22 years old. The testimony of the notary public who prepared the deed of conveyance for the property reveals the
falsehood of this claim. Atty. Constantino Sagun testified that Miguel Palang provided the money for the purchase price and
directed that Erlindas name alone be placed as the vendee. [14]
The transaction was properly a donation made by Miguel to Erlinda, but one which was clearly void and inexistent by
express provision of law because it was made between persons guilty of adultery or concubinage at the time of the donation,
under Article 739 of the Civil Code. Moreover, Article 87 of the Family Code expressly provides that the prohibition against
donations between spouses now applies to donations between persons living together as husband and wife without a valid
marriage,[15] for otherwise, the condition of those who incurred guilt would turn out to be better than those in legal union. [16]
The second issue concerning Kristopher Palangs status and claim as an illegitimate son and heir to Miguels estate is here
resolved in favor of respondent courts correct assessment that the trial court erred in making pronouncements regarding
Kristophers heirship and filiation inasmuch as questions as to who are the heirs of the decedent, proof of filiation of illegitimate
children and the determination of the estate of the latter and claims thereto should be ventilated in the proper probate court or in
a special proceeding instituted for the purpose and cannot be adjudicated in the instant ordinary civil action which is for recovery
of ownership and possession.[17]
As regards the third issue, petitioner contends that Kristopher Palang should be considered as party-defendant in the case
at bar following the trial courts decision which expressly found that Kristopher had not been impleaded as party defendant but
theorized that he had submitted to the courts jurisdiction through his mother/guardian ad litem.[18] The trial court erred
gravely. Kristopher, not having been impleaded, was, therefore, not a party to the case at bar. His mother, Erlinda, cannot be
called his guardian ad litem for he was not involved in the case at bar. Petitioner adds that there is no need for Kristopher to file
another action to prove that he is the illegitimate son of Miguel, in order to avoid multiplicity of suits. [19] Petitioners grave error
has been discussed in the preceeding paragraph where the need for probate proceedings to resolve the settlement of Miguels
estate and Kristophers successional rights has been pointed out.
WHEREFORE, the instant petition is hereby DENIED. The questioned decision of the Court of Appeals is
AFFIRMED. Costs against petitioner.

G.R. No. 146683

November 22, 2001

CIRILA ARCABA, petitioner,


vs.
ERLINDA TABANCURA VDA. DE BATOCAEL, SEIGFREDO C. TABANCURA, DORIS C. TABANCURA, LUZELLI C.
TABANCURA, BELEN C. TABANCURA, RAUL A. COMILLE, BERNADETTE A. COMILLE, and ABNER A.
COMILLE, respondents.
MENDOZA, J.:
Petitioner Cirila Arcaba seeks review on certiorari of the decision 1 of the Court of Appeals, which affirmed with modification the
decision2 of the Regional Trial Court, Branch 10, Dipolog City, Zamboanga del Norte in Civil Case No. 4593, declaring as void a
deed of donation inter vivos executed by the late Francisco T. Comille in her favor and its subsequent resolution 3 denying
reconsideration.
The facts are as follows:
On January 16, 1956, Francisco Comille and his wife Zosima Montallana became the registered owners of Lot No. 437-A
located at the corner of Calle Santa Rosa (now Balintawak Street) and Calle Rosario (now Rizal Avenue) in Dipolog City,
Zamboanga del Norte. The total area of the lot was 418 square meters. 4 After the death of Zosima on October 3, 1980,
Francisco and his mother-in-law, Juliana Bustalino Montallana, executed a deed of extrajudicial partition with waiver of rights, in
which the latter waived her share consisting of one-fourth (1/4) of the property to Francisco. 5 On June 27, 1916, Francisco
registered the lot in his name with the Registry of Deeds. 6
Having no children to take care of him after his retirement, Francisco asked his niece Leticia Bellosillo, 7 the latter's cousin,
Luzviminda Paghacian,8 and petitioner Cirila Arcaba, then a widow, to take care of his house, as well as the store inside. 9
Conflicting testimonies were offered as to the nature of the relationship between Cirila and Francisco. Leticia Bellosillo said
Francisco and Cirila were lovers since they slept in the same room, 10 while Erlinda Tabancura,11another niece of Francisco,
claimed that the latter had told her that Cirila was his mistress. 12 On the other hand, Cirila said she was a mere helper who could
enter the master's bedroom only when the old man asked her to and that Francisco in any case was too old for her. She denied
they ever had sexual intercourse.13
It appears that when Leticia and Luzviminda were married, only Cirila was left to take care of Francisco. 14 Cirila testified that she
was a 34-year old widow while Francisco was a 75-year old widower when she began working for the latter; that he could still
walk with her assistance at that time; 15 and that his health eventually deteriorated and he became bedridden. 16 Erlinda
Tabancura testified that Francisco's sole source of income consisted of rentals from his lot near the public streets. 17 He did not
pay Cirila a regular cash wage as a househelper , though he provided her family with food and lodging. 18
On January 24, 1991, a few months before his death, Francisco executed an instrument denominated "Deed of Donation Inter
Vivos," in which he ceded a portion of Lot 437-A, consisting of 150 square meters, together with his house, to Cirila, who
accepted the donation in the same instrument. Francisco left the larger portion of 268 square meters in his name. The deed
stated that the donation was being made in consideration of "the faithful services [Cirila Arcaba] had rendered over the past ten
(10) years." The deed was notarized by Atty. Vic T. Lacaya, Sr.19and later registered by Cirila as its absolute owner . 20
On October 4, 1991, Francisco died without any children. In 1993, the lot which Cirila received from Francisco had a market
value of P57,105.00 and an assessed value of P28,550.00. 21
On February 18, 1993, respondents filed a complaint against petitioner 'for declaration of nullity of a deed of donation inter
vivos, recovery of possession, and damages. Respondents, who are the decedent's nephews and nieces and his heirs by
intestate succession, alleged that Cirila was the common-law wife of Francisco and the donation inter vivos made by Francisco
in her favor is void under Article 87 of the Family Code, which provides:
Every donation or grant of gratuitous advantage, direct or indirect, between the spouses during the marriage shall be void,
except moderate gifts which the spouses may give each other on the occasion of any family rejoicing. The prohibition shall
also apply to persons living together as husband and wife without a valid marriage.
On February 25, 1999, the trial court rendered judgment in favor of respondents, holding the donation void under this provision
of the Family Code. The trial court reached this conclusion based on the testimony of Erlinda Tabancura and certain documents
bearing the signature of one "Cirila Comille." The documents were (1) an application for a business permit to operate as real
estate lessor, dated January 8, 1991, with a carbon copy of the signature "Cirila Comille"; 22 (2) a sanitary permit to operate as
real estate lessor with a health certificate showing the signature "Cirila Comille" in black ink; 23 and (3) the death certificate of the
decedent with the signature "Cirila A. Comille" written in black ink. 24 The dispositive portion of the trial court's decision states:
WHEREFORE, in view of the foregoing, judgment is rendered:
1. Declaring the Deed of Donation Inter Vivos executed by the late Francisco Comille recorded as Doc. No. 7; Page No. 3;
Book No. V; Series of 1991 in the Notarial Register of Notary Public Vic T. Lacaya (Annex " A " to the Complaint) null and
void;
2. Ordering the defendant to deliver possession of the house and lot subject of the deed unto the plaintiffs within thirty (30)
days after finality of this decision; and finally
3. Ordering the defendant to pay attorney's fees in the sum of P10,000.00.
SO ORDERED.25
Petitioner appealed to the Court of Appeals, which rendered on June 19, 2000 the decision subject of this appeal. As already
stated, the appeals court denied reconsideration. Its conclusion was based on (1) the testimonies of Leticia, Erlinda, and Cirila;
(2) the copies of documents purportedly showing Cirila's use of Francisco's surname; (3) a pleading in another civil case
mentioning payment of rentals to Cirila as Francisco's common-law wife; and (4) the fact that Cirila did not receive a regular
cash wage.
Petitioner assigns the following errors as having been committed by the Court of Appeals:
(a) The judgment of the Court of Appeals that petitioner was the common-law wife of the late Francisco Comille is not
correct and is a reversible error because it is based on a misapprehension of facts, and unduly breaks the chain of
circumstances detailed by the totality of the evidence, its findings being predicated on totally incompetent or hearsay
evidence, and grounded on mere speculation, conjecture or possibility. (Salazar v. Gutierrez, 33 SCRA 243 and other
cases; cited in Quiason, Philippine Courts and their J urisdictions, 1993 ed., p. 604)
(b) The Court of Appeals erred in shifting the burden of evidence from the plaintiff to defendant. (Bunyi v. Reyes, 39 SCRA
504; Quiason, id.)
(c) The Court of Appeals decided the case in away probably not in accord with law or with the applicable jurisprudence in
Rodriguez v. Rodriguez, 20 SCRA 908, and Liguez v. CA, 102 Phil. 577, 584. 26

The issue in this case is whether the Court of Appeals correctly applied Art. 87 of the Family Code to the circumstances of this
case. After a review of the records, we rule in the affirmative.
The general rule is that only questions of law may be raised in a petition for review under Rule 45 of the Rules of Court, subject
only to certain exceptions: (a) when the conclusion is a finding grounded entirely on speculations, surmises, or conjectures; (b)
when the inference made is manifestly mistaken, absurd, or impossible; (c) where there is grave abuse of discretion; (d) when
the judgment is based on a misapprehension of facts; (e) when the findings of fact are conflicting; (f) when the Court of Appeals,
in making its findings, went beyond the issues of the case and the same are contrary to the admissions of both appellant and
appellee; (g) when the findings of the Court of Appeals are contrary to those of the trial court; (h) when the findings of fact are
conclusions without citation of specific evidence on which they are based; (i) when the finding of fact of the Court of Appeals is
premised on the supposed absence of evidence but is contradicted by the evidence on record; and G) when the Court of
Appeals manifestly overlooked certain relevant facts not disputed by the parties and which, if properly considered, would justify
a different conclusion.27 It appearing that the Court of Appeals based its findings on evidence presented by both parties, the
general rule should apply.
In Bitangcor v. Tan,28 we held that the term "cohabitation" or "living together as husband and wife" means not only residing
under one roof, but also having repeated sexual intercourse. Cohabitation, of course, means more than sexual intercourse,
especially when one of the parties is already old and may no longer be interested in sex. At the very least, cohabitation
is public assumption by a man and a woman of the marital relation, and dwelling together as man and wife, thereby holding
themselves out to the public as such. Secret meetings or nights clandestinely spent together, even if often repeated, do not
constitute such kind of cohabitation; they are merely meretricious. 29In this jurisdiction, this Court has considered as sufficient
proof of common-law relationship the stipulations between the parties, 30 a conviction of concubinage,31 or the existence of
legitimate children.32
Was Cirila Francisco's employee or his common-law wife? Cirila admitted that she and Francisco resided under one roof for a
long time, It is very possible that the two consummated their relationship, since Cirila gave Francisco therapeutic massage and
Leticia said they slept in the same bedroom. At the very least, their public conduct indicated that theirs was not just a
relationship of caregiver and patient, but that of exclusive partners akin to husband and wife.
Aside from Erlinda Tabancura's testimony that her uncle told her that Cirila was his mistress, there are other indications that
Cirila and Francisco were common-law spouses. Seigfredo Tabancura presented documents apparently signed by Cirila using
the surname "Comille." As previously stated, these are an application for a business permit to operate as a real estate lessor, 33 a
sanitary permit to operate as real estate lessor with a health certificate, 34 and the death certificate of Francisco.35 These
documents show that Cirila saw herself as Francisco's common-law wife, otherwise, she would not have used his last name.
Similarly, in the answer filed by Francisco's lessees in "Erlinda Tabancura, et al. vs. Gracia Adriatico Sy and Antonio Sy," RTC
Civil Case No.4719 (for collection of rentals), these lessees referred to Cirila as "the common-law spouse of Francisco." Finally,
the fact that Cirila did not demand from Francisco a regular cash wage is an indication that she was not simply a caregiveremployee, but Francisco's common law spouse. She was, after all, entitled to a regular cash wage under the law. 36 It is difficult
to believe that she stayed with Francisco and served him out of pure beneficence. Human reason would thus lead to the
conclusion that she was Francisco's common-law spouse.
Respondents having proven by a preponderance of evidence that Cirila and Francisco lived together as husband and wife
without a valid marriage, the inescapable conclusion is that the donation made by Francisco in favor of Cirila is void under Art.
87 of the Family Code.1wphi1.nt
WHEREFORE, the decision of the Court of Appeals affirming the decision of the trial court is hereby AFFIRMED.

G.R. No. 133743

February 6, 2007

EDGAR SAN LUIS, Petitioner,


vs.
FELICIDAD SAN LUIS, Respondent.
x ---------------------------------------------------- x
G.R. No. 134029

February 6, 2007

RODOLFO SAN LUIS, Petitioner,


vs.
FELICIDAD SAGALONGOS alias FELICIDAD SAN LUIS, Respondent.
DECISION
YNARES-SANTIAGO, J.:
Before us are consolidated petitions for review assailing the February 4, 1998 Decision 1 of the Court of Appeals in CA-G.R. CV
No. 52647, which reversed and set aside the September 12, 1995 2 and January 31, 1996 3Resolutions of the Regional Trial
Court of Makati City, Branch 134 in SP. Proc. No. M-3708; and its May 15, 1998 Resolution 4 denying petitioners motion for
reconsideration.
The instant case involves the settlement of the estate of Felicisimo T. San Luis (Felicisimo), who was the former governor of the
Province of Laguna. During his lifetime, Felicisimo contracted three marriages. His first marriage was with Virginia Sulit on
March 17, 1942 out of which were born six children, namely: Rodolfo, Mila, Edgar, Linda, Emilita and Manuel. On August 11,
1963, Virginia predeceased Felicisimo.
Five years later, on May 1, 1968, Felicisimo married Merry Lee Corwin, with whom he had a son, Tobias. However, on October
15, 1971, Merry Lee, an American citizen, filed a Complaint for Divorce 5 before the Family Court of the First Circuit, State of
Hawaii, United States of America (U.S.A.), which issued a Decree Granting Absolute Divorce and Awarding Child Custody on
December 14, 1973. 6
On June 20, 1974, Felicisimo married respondent Felicidad San Luis, then surnamed Sagalongos, before Rev. Fr. William
Meyer, Minister of the United Presbyterian at Wilshire Boulevard, Los Angeles, California, U.S.A. 7 He had no children with
respondent but lived with her for 18 years from the time of their marriage up to his death on December 18, 1992.
Thereafter, respondent sought the dissolution of their conjugal partnership assets and the settlement of Felicisimos estate. On
December 17, 1993, she filed a petition for letters of administration 8 before the Regional Trial Court of Makati City, docketed as
SP. Proc. No. M-3708 which was raffled to Branch 146 thereof.
Respondent alleged that she is the widow of Felicisimo; that, at the time of his death, the decedent was residing at 100 San
Juanico Street, New Alabang Village, Alabang, Metro Manila; that the decedents surviving heirs are respondent as legal
spouse, his six children by his first marriage, and son by his second marriage; that the decedent left real properties, both
conjugal and exclusive, valued at P30,304,178.00 more or less; that the decedent does not have any unpaid debts. Respondent
prayed that the conjugal partnership assets be liquidated and that letters of administration be issued to her.
On February 4, 1994, petitioner Rodolfo San Luis, one of the children of Felicisimo by his first marriage, filed a motion to
dismiss 9 on the grounds of improper venue and failure to state a cause of action. Rodolfo claimed that the petition for letters of
administration should have been filed in the Province of Laguna because this was Felicisimos place of residence prior to his
death. He further claimed that respondent has no legal personality to file the petition because she was only a mistress of
Felicisimo since the latter, at the time of his death, was still legally married to Merry Lee.
On February 15, 1994, Linda invoked the same grounds and joined her brother Rodolfo in seeking the dismissal 10 of the
petition. On February 28, 1994, the trial court issued an Order 11 denying the two motions to dismiss.
Unaware of the denial of the motions to dismiss, respondent filed on March 5, 1994 her opposition 12 thereto. She submitted
documentary evidence showing that while Felicisimo exercised the powers of his public office in Laguna, he regularly went
home to their house in New Alabang Village, Alabang, Metro Manila which they bought sometime in 1982. Further, she
presented the decree of absolute divorce issued by the Family Court of the First Circuit, State of Hawaii to prove that the
marriage of Felicisimo to Merry Lee had already been dissolved. Thus, she claimed that Felicisimo had the legal capacity to
marry her by virtue of paragraph 2, 13 Article 26 of the Family Code and the doctrine laid down in Van Dorn v. Romillo, Jr. 14
Thereafter, Linda, Rodolfo and herein petitioner Edgar San Luis, separately filed motions for reconsideration from the Order
denying their motions to dismiss. 15 They asserted that paragraph 2, Article 26 of the Family Code cannot be given retroactive
effect to validate respondents bigamous marriage with Felicisimo because this would impair vested rights in derogation of
Article 256 16 of the Family Code.
On April 21, 1994, Mila, another daughter of Felicisimo from his first marriage, filed a motion to disqualify Acting Presiding Judge
Anthony E. Santos from hearing the case.
On October 24, 1994, the trial court issued an Order 17 denying the motions for reconsideration. It ruled that respondent, as
widow of the decedent, possessed the legal standing to file the petition and that venue was properly laid. Meanwhile, the motion
for disqualification was deemed moot and academic 18 because then Acting Presiding Judge Santos was substituted by Judge
Salvador S. Tensuan pending the resolution of said motion.
Mila filed a motion for inhibition 19 against Judge Tensuan on November 16, 1994. On even date, Edgar also filed a motion for
reconsideration 20 from the Order denying their motion for reconsideration arguing that it does not state the facts and law on
which it was based.
On November 25, 1994, Judge Tensuan issued an Order
134 presided by Judge Paul T. Arcangel.

21

granting the motion for inhibition. The case was re-raffled to Branch

On April 24, 1995, 22 the trial court required the parties to submit their respective position papers on the twin issues of venue and
legal capacity of respondent to file the petition. On May 5, 1995, Edgar manifested 23 that he is adopting the arguments and
evidence set forth in his previous motion for reconsideration as his position paper. Respondent and Rodolfo filed their position
papers on June 14, 24 and June 20, 25 1995, respectively.
On September 12, 1995, the trial court dismissed the petition for letters of administration. It held that, at the time of his death,
Felicisimo was the duly elected governor and a resident of the Province of Laguna. Hence, the petition should have been filed in
Sta. Cruz, Laguna and not in Makati City. It also ruled that respondent was without legal capacity to file the petition for letters of
administration because her marriage with Felicisimo was bigamous, thus, void ab initio. It found that the decree of absolute
divorce dissolving Felicisimos marriage to Merry Lee was not valid in the Philippines and did not bind Felicisimo who was a
Filipino citizen. It also ruled that paragraph 2, Article 26 of the Family Code cannot be retroactively applied because it would
impair the vested rights of Felicisimos legitimate children.

Respondent moved for reconsideration

26

and for the disqualification

27

of Judge Arcangel but said motions were denied.

28

Respondent appealed to the Court of Appeals which reversed and set aside the orders of the trial court in its assailed Decision
dated February 4, 1998, the dispositive portion of which states:
WHEREFORE, the Orders dated September 12, 1995 and January 31, 1996 are hereby REVERSED and SET ASIDE; the
Orders dated February 28 and October 24, 1994 are REINSTATED; and the records of the case is REMANDED to the trial court
for further proceedings. 29
The appellante court ruled that under Section 1, Rule 73 of the Rules of Court, the term "place of residence" of the decedent, for
purposes of fixing the venue of the settlement of his estate, refers to the personal, actual or physical habitation, or actual
residence or place of abode of a person as distinguished from legal residence or domicile. It noted that although Felicisimo
discharged his functions as governor in Laguna, he actually resided in Alabang, Muntinlupa. Thus, the petition for letters of
administration was properly filed in Makati City.
The Court of Appeals also held that Felicisimo had legal capacity to marry respondent by virtue of paragraph 2, Article 26 of the
Family Code and the rulings in Van Dorn v. Romillo, Jr. 30 and Pilapil v. Ibay-Somera. 31 It found that the marriage between
Felicisimo and Merry Lee was validly dissolved by virtue of the decree of absolute divorce issued by the Family Court of the
First Circuit, State of Hawaii. As a result, under paragraph 2, Article 26, Felicisimo was capacitated to contract a subsequent
marriage with respondent. Thus
With the well-known rule express mandate of paragraph 2, Article 26, of the Family Code of the Philippines, the doctrines in
Van Dorn, Pilapil, and the reason and philosophy behind the enactment of E.O. No. 227, there is no justiciable reason to
sustain the individual view sweeping statement of Judge Arc[h]angel, that "Article 26, par. 2 of the Family Code,
contravenes the basic policy of our state against divorce in any form whatsoever." Indeed, courts cannot deny what the law
grants. All that the courts should do is to give force and effect to the express mandate of the law. The foreign divorce having
been obtained by the Foreigner on December 14, 1992,32 the Filipino divorcee, "shall x x x have capacity to remarry under
Philippine laws". For this reason, the marriage between the deceased and petitioner should not be denominated as "a bigamous
marriage.
Therefore, under Article 130 of the Family Code, the petitioner as the surviving spouse can institute the judicial proceeding for
the settlement of the estate of the deceased. x x x 33
Edgar, Linda, and Rodolfo filed separate motions for reconsideration

34

which were denied by the Court of Appeals.

On July 2, 1998, Edgar appealed to this Court via the instant petition for review on certiorari.
manifestation and motion to adopt the said petition which was granted. 36

35

Rodolfo later filed a

In the instant consolidated petitions, Edgar and Rodolfo insist that the venue of the subject petition for letters of administration
was improperly laid because at the time of his death, Felicisimo was a resident of Sta. Cruz, Laguna. They contend that
pursuant to our rulings in Nuval v. Guray 37 and Romualdez v. RTC, Br. 7, Tacloban City, 38 "residence" is synonymous with
"domicile" which denotes a fixed permanent residence to which when absent, one intends to return. They claim that a person
can only have one domicile at any given time. Since Felicisimo never changed his domicile, the petition for letters of
administration should have been filed in Sta. Cruz, Laguna.
Petitioners also contend that respondents marriage to Felicisimo was void and bigamous because it was performed during the
subsistence of the latters marriage to Merry Lee. They argue that paragraph 2, Article 26 cannot be retroactively applied
because it would impair vested rights and ratify the void bigamous marriage. As such, respondent cannot be considered the
surviving wife of Felicisimo; hence, she has no legal capacity to file the petition for letters of administration.
The issues for resolution: (1) whether venue was properly laid, and (2) whether respondent has legal capacity to file the subject
petition for letters of administration.
The petition lacks merit.
Under Section 1, 39 Rule 73 of the Rules of Court, the petition for letters of administration of the estate of Felicisimo should be
filed in the Regional Trial Court of the province "in which he resides at the time of his death." In the case of Garcia Fule v. Court
of Appeals, 40 we laid down the doctrinal rule for determining the residence as contradistinguished from domicile of the
decedent for purposes of fixing the venue of the settlement of his estate:
[T]he term "resides" connotes ex vi termini "actual residence" as distinguished from "legal residence or domicile." This term
"resides," like the terms "residing" and "residence," is elastic and should be interpreted in the light of the object or purpose of the
statute or rule in which it is employed. In the application of venue statutes and rules Section 1, Rule 73 of the Revised Rules
of Court is of such nature residence rather than domicile is the significant factor. Even where the statute uses the word
"domicile" still it is construed as meaning residence and not domicile in the technical sense. Some cases make a distinction
between the terms "residence" and "domicile" but as generally used in statutes fixing venue, the terms are synonymous, and
convey the same meaning as the term "inhabitant." In other words, "resides" should be viewed or understood in its popular
sense, meaning, the personal, actual or physical habitation of a person, actual residence or place of abode. It signifies physical
presence in a place and actual stay thereat. In this popular sense, the term means merely residence, that is, personal
residence, not legal residence or domicile. Residence simply requires bodily presence as an inhabitant in a given place, while
domicile requires bodily presence in that place and also an intention to make it ones domicile. No particular length of time of
residence is required though; however, the residence must be more than temporary.41 (Emphasis supplied)
It is incorrect for petitioners to argue that "residence," for purposes of fixing the venue of the settlement of the estate of
Felicisimo, is synonymous with "domicile." The rulings in Nuval and Romualdez are inapplicable to the instant case because
they involve election cases. Needless to say, there is a distinction between "residence" for purposes of election laws and
"residence" for purposes of fixing the venue of actions. In election cases, "residence" and "domicile" are treated as synonymous
terms, that is, the fixed permanent residence to which when absent, one has the intention of returning. 42 However, for purposes
of fixing venue under the Rules of Court, the "residence" of a person is his personal, actual or physical habitation, or actual
residence or place of abode, which may not necessarily be his legal residence or domicile provided he resides therein with
continuity and consistency.43 Hence, it is possible that a person may have his residence in one place and domicile in another.
In the instant case, while petitioners established that Felicisimo was domiciled in Sta. Cruz, Laguna, respondent proved that he
also maintained a residence in Alabang, Muntinlupa from 1982 up to the time of his death. Respondent submitted in evidence
the Deed of Absolute Sale 44 dated January 5, 1983 showing that the deceased purchased the aforesaid property. She also
presented billing statements 45 from the Philippine Heart Center and Chinese General Hospital for the period August to
December 1992 indicating the address of Felicisimo at "100 San Juanico, Ayala Alabang, Muntinlupa." Respondent also
presented proof of membership of the deceased in the Ayala Alabang Village Association 46 and Ayala Country Club,
Inc., 47 letter-envelopes 48 from 1988 to 1990 sent by the deceaseds children to him at his Alabang address, and the deceaseds
calling cards 49stating that his home/city address is at "100 San Juanico, Ayala Alabang Village, Muntinlupa" while his
office/provincial address is in "Provincial Capitol, Sta. Cruz, Laguna."
From the foregoing, we find that Felicisimo was a resident of Alabang, Muntinlupa for purposes of fixing the venue of the
settlement of his estate. Consequently, the subject petition for letters of administration was validly filed in the Regional Trial
Court 50 which has territorial jurisdiction over Alabang, Muntinlupa. The subject petition was filed on December 17, 1993. At that
time, Muntinlupa was still a municipality and the branches of the Regional Trial Court of the National Capital Judicial Region

which had territorial jurisdiction over Muntinlupa were then seated in Makati City as per Supreme Court Administrative Order No.
3. 51 Thus, the subject petition was validly filed before the Regional Trial Court of Makati City.
Anent the issue of respondent Felicidads legal personality to file the petition for letters of administration, we must first resolve
the issue of whether a Filipino who is divorced by his alien spouse abroad may validly remarry under the Civil Code, considering
that Felicidads marriage to Felicisimo was solemnized on June 20, 1974, or before the Family Code took effect on August 3,
1988. In resolving this issue, we need not retroactively apply the provisions of the Family Code, particularly Art. 26, par. (2)
considering that there is sufficient jurisprudential basis allowing us to rule in the affirmative.
The case of Van Dorn v. Romillo, Jr. 52 involved a marriage between a foreigner and his Filipino wife, which marriage was
subsequently dissolved through a divorce obtained abroad by the latter. Claiming that the divorce was not valid under Philippine
law, the alien spouse alleged that his interest in the properties from their conjugal partnership should be protected. The Court,
however, recognized the validity of the divorce and held that the alien spouse had no interest in the properties acquired by the
Filipino wife after the divorce. Thus:
In this case, the divorce in Nevada released private respondent from the marriage from the standards of American law, under
which divorce dissolves the marriage. As stated by the Federal Supreme Court of the United States in Atherton vs. Atherton, 45
L. Ed. 794, 799:
"The purpose and effect of a decree of divorce from the bond of matrimony by a competent jurisdiction are to change the
existing status or domestic relation of husband and wife, and to free them both from the bond. The marriage tie, when thus
severed as to one party, ceases to bind either. A husband without a wife, or a wife without a husband, is unknown to the law.
When the law provides, in the nature of a penalty, that the guilty party shall not marry again, that party, as well as the other, is
still absolutely freed from the bond of the former marriage."
Thus, pursuant to his national law, private respondent is no longer the husband of petitioner. He would have no standing to sue
in the case below as petitioners husband entitled to exercise control over conjugal assets. As he is bound by the Decision of his
own countrys Court, which validly exercised jurisdiction over him, and whose decision he does not repudiate, he is estopped by
his own representation before said Court from asserting his right over the alleged conjugal property. 53
As to the effect of the divorce on the Filipino wife, the Court ruled that she should no longer be considered married to the alien
spouse. Further, she should not be required to perform her marital duties and obligations. It held:
To maintain, as private respondent does, that, under our laws, petitioner has to be considered still married to private
respondent and still subject to a wife's obligations under Article 109, et. seq. of the Civil Code cannot be just. Petitioner
should not be obliged to live together with, observe respect and fidelity, and render support to private respondent. The latter
should not continue to be one of her heirs with possible rights to conjugal property. She should not be discriminated against
in her own country if the ends of justice are to be served. 54 (Emphasis added)
This principle was thereafter applied in Pilapil v. Ibay-Somera 55 where the Court recognized the validity of a divorce obtained
abroad. In the said case, it was held that the alien spouse is not a proper party in filing the adultery suit against his Filipino wife.
The Court stated that "the severance of the marital bond had the effect of dissociating the former spouses from each
other, hence the actuations of one would not affect or cast obloquy on the other." 56
Likewise, in Quita v. Court of Appeals, 57 the Court stated that where a Filipino is divorced by his naturalized foreign spouse, the
ruling in Van Dorn applies. 58 Although decided on December 22, 1998, the divorce in the said case was obtained in 1954 when
the Civil Code provisions were still in effect.
The significance of the Van Dorn case to the development of limited recognition of divorce in the Philippines cannot be denied.
The ruling has long been interpreted as severing marital ties between parties in a mixed marriage and capacitating the Filipino
spouse to remarry as a necessary consequence of upholding the validity of a divorce obtained abroad by the alien spouse. In
his treatise, Dr. Arturo M. Tolentino cited Van Dorn stating that "if the foreigner obtains a valid foreign divorce, the Filipino
spouse shall have capacity to remarry under Philippine law." 59 In Garcia v. Recio, 60 the Court likewise cited the aforementioned
case in relation to Article 26. 61
In the recent case of Republic v. Orbecido III, 62 the historical background and legislative intent behind paragraph 2, Article 26 of
the Family Code were discussed, to wit:
Brief Historical Background
On July 6, 1987, then President Corazon Aquino signed into law Executive Order No. 209, otherwise known as the "Family
Code," which took effect on August 3, 1988. Article 26 thereof states:
All marriages solemnized outside the Philippines in accordance with the laws in force in the country where they were
solemnized, and valid there as such, shall also be valid in this country, except those prohibited under Articles 35, 37, and 38.
On July 17, 1987, shortly after the signing of the original Family Code, Executive Order No. 227 was likewise signed into law,
amending Articles 26, 36, and 39 of the Family Code. A second paragraph was added to Article 26. As so amended, it now
provides:
ART. 26. All marriages solemnized outside the Philippines in accordance with the laws in force in the country where they were
solemnized, and valid there as such, shall also be valid in this country, except those prohibited under Articles 35(1), (4), (5) and
(6), 36, 37 and 38.
Where a marriage between a Filipino citizen and a foreigner is validly celebrated and a divorce is thereafter validly obtained
abroad by the alien spouse capacitating him or her to remarry, the Filipino spouse shall have capacity to remarry under
Philippine law. (Emphasis supplied)
xxxx
Legislative Intent
Records of the proceedings of the Family Code deliberations showed that the intent of Paragraph 2 of Article 26, according to
Judge Alicia Sempio-Diy, a member of the Civil Code Revision Committee, is to avoid the absurd situation where the Filipino
spouse remains married to the alien spouse who, after obtaining a divorce, is no longer married to the Filipino spouse.
Interestingly, Paragraph 2 of Article 26 traces its origin to the 1985 case of Van Dorn v. Romillo, Jr. TheVan Dorn case
involved a marriage between a Filipino citizen and a foreigner. The Court held therein that a divorce decree validly
obtained by the alien spouse is valid in the Philippines, and consequently, the Filipino spouse is capacitated to
remarry under Philippine law. 63 (Emphasis added)
As such, the Van Dorn case is sufficient basis in resolving a situation where a divorce is validly obtained abroad by the alien
spouse. With the enactment of the Family Code and paragraph 2, Article 26 thereof, our lawmakers codified the law already
established through judicial precedent.1awphi1.net
Indeed, when the object of a marriage is defeated by rendering its continuance intolerable to one of the parties and productive
of no possible good to the community, relief in some way should be obtainable. 64 Marriage, being a mutual and shared

commitment between two parties, cannot possibly be productive of any good to the society where one is considered released
from the marital bond while the other remains bound to it. Such is the state of affairs where the alien spouse obtains a valid
divorce abroad against the Filipino spouse, as in this case.
Petitioners cite Articles 15 65 and 17 66 of the Civil Code in stating that the divorce is void under Philippine law insofar as Filipinos
are concerned. However, in light of this Courts rulings in the cases discussed above, the Filipino spouse should not be
discriminated against in his own country if the ends of justice are to be served. 67 In Alonzo v. Intermediate Appellate
Court, 68 the Court stated:
But as has also been aptly observed, we test a law by its results; and likewise, we may add, by its purposes. It is a cardinal rule
that, in seeking the meaning of the law, the first concern of the judge should be to discover in its provisions the intent of the
lawmaker. Unquestionably, the law should never be interpreted in such a way as to cause injustice as this is never within the
legislative intent. An indispensable part of that intent, in fact, for we presume the good motives of the legislature, is to render
justice.
Thus, we interpret and apply the law not independently of but in consonance with justice. Law and justice are inseparable, and
we must keep them so. To be sure, there are some laws that, while generally valid, may seem arbitrary when applied in a
particular case because of its peculiar circumstances. In such a situation, we are not bound, because only of our nature and
functions, to apply them just the same, in slavish obedience to their language. What we do instead is find a balance between
the word and the will, that justice may be done even as the law is obeyed.
As judges, we are not automatons. We do not and must not unfeelingly apply the law as it is worded, yielding like robots to the
literal command without regard to its cause and consequence. "Courts are apt to err by sticking too closely to the words of a
law," so we are warned, by Justice Holmes again, "where these words import a policy that goes beyond them."
xxxx
More than twenty centuries ago, Justinian defined justice "as the constant and perpetual wish to render every one his due." That
wish continues to motivate this Court when it assesses the facts and the law in every case brought to it for decision. Justice is
always an essential ingredient of its decisions. Thus when the facts warrants, we interpret the law in a way that will render
justice, presuming that it was the intention of the lawmaker, to begin with, that the law be dispensed with justice. 69
Applying the above doctrine in the instant case, the divorce decree allegedly obtained by Merry Lee which absolutely allowed
Felicisimo to remarry, would have vested Felicidad with the legal personality to file the present petition as Felicisimos surviving
spouse. However, the records show that there is insufficient evidence to prove the validity of the divorce obtained by Merry Lee
as well as the marriage of respondent and Felicisimo under the laws of the U.S.A. In Garcia v. Recio, 70 the Court laid down the
specific guidelines for pleading and proving foreign law and divorce judgments. It held that presentation solely of the divorce
decree is insufficient and that proof of its authenticity and due execution must be presented. Under Sections 24 and 25 of Rule
132, a writing or document may be proven as a public or official record of a foreign country by either (1) an official publication or
(2) a copy thereof attested by the officer having legal custody of the document. If the record is not kept in the Philippines, such
copy must be (a) accompanied by a certificate issued by the proper diplomatic or consular officer in the Philippine foreign
service stationed in the foreign country in which the record is kept and (b) authenticated by the seal of his office. 71
With regard to respondents marriage to Felicisimo allegedly solemnized in California, U.S.A., she submitted photocopies of the
Marriage Certificate and the annotated text 72 of the Family Law Act of California which purportedly show that their marriage was
done in accordance with the said law. As stated in Garcia, however, the Court cannot take judicial notice of foreign laws as they
must be alleged and proved. 73
Therefore, this case should be remanded to the trial court for further reception of evidence on the divorce decree obtained by
Merry Lee and the marriage of respondent and Felicisimo.
Even assuming that Felicisimo was not capacitated to marry respondent in 1974, nevertheless, we find that the latter has the
legal personality to file the subject petition for letters of administration, as she may be considered the co-owner of Felicisimo as
regards the properties that were acquired through their joint efforts during their cohabitation.
Section 6, 74 Rule 78 of the Rules of Court states that letters of administration may be granted to the surviving spouse of the
decedent. However, Section 2, Rule 79 thereof also provides in part:
SEC. 2. Contents of petition for letters of administration. A petition for letters of administration must be filed by an interested
person and must show, as far as known to the petitioner: x x x.
An "interested person" has been defined as one who would be benefited by the estate, such as an heir, or one who has a claim
against the estate, such as a creditor. The interest must be material and direct, and not merely indirect or contingent. 75
In the instant case, respondent would qualify as an interested person who has a direct interest in the estate of Felicisimo by
virtue of their cohabitation, the existence of which was not denied by petitioners. If she proves the validity of the divorce and
Felicisimos capacity to remarry, but fails to prove that her marriage with him was validly performed under the laws of the U.S.A.,
then she may be considered as a co-owner under Article 144 76 of the Civil Code. This provision governs the property relations
between parties who live together as husband and wife without the benefit of marriage, or their marriage is void from the
beginning. It provides that the property acquired by either or both of them through their work or industry or their wages and
salaries shall be governed by the rules on co-ownership. In a co-ownership, it is not necessary that the property be acquired
through their joint labor, efforts and industry. Any property acquired during the union is prima facie presumed to have been
obtained through their joint efforts. Hence, the portions belonging to the co-owners shall be presumed equal, unless the contrary
is proven. 77
Meanwhile, if respondent fails to prove the validity of both the divorce and the marriage, the applicable provision would be
Article 148 of the Family Code which has filled the hiatus in Article 144 of the Civil Code by expressly regulating the property
relations of couples living together as husband and wife but are incapacitated to marry. 78In Saguid v. Court of Appeals, 79 we
held that even if the cohabitation or the acquisition of property occurred before the Family Code took effect, Article 148
governs. 80 The Court described the property regime under this provision as follows:
The regime of limited co-ownership of property governing the union of parties who are not legally capacitated to marry each
other, but who nonetheless live together as husband and wife, applies to properties acquired during said cohabitation in
proportion to their respective contributions. Co-ownership will only be up to the extent of the proven actual contribution of
money, property or industry. Absent proof of the extent thereof, their contributions and corresponding shares shall be presumed
to be equal.
xxxx
In the cases of Agapay v. Palang, and Tumlos v. Fernandez, which involved the issue of co-ownership of properties acquired by
the parties to a bigamous marriage and an adulterous relationship, respectively, we ruled that proof of actual contribution in the
acquisition of the property is essential. x x x
As in other civil cases, the burden of proof rests upon the party who, as determined by the pleadings or the nature of the case,
asserts an affirmative issue. Contentions must be proved by competent evidence and reliance must be had on the strength of
the partys own evidence and not upon the weakness of the opponents defense. x x x 81

In view of the foregoing, we find that respondents legal capacity to file the subject petition for letters of administration may arise
from her status as the surviving wife of Felicisimo or as his co-owner under Article 144 of the Civil Code or Article 148 of the
Family Code.
WHEREFORE, the petition is DENIED. The Decision of the Court of Appeals reinstating and affirming the February 28, 1994
Order of the Regional Trial Court which denied petitioners motion to dismiss and its October 24, 1994 Order which dismissed
petitioners motion for reconsideration is AFFIRMED. Let this case be REMANDED to the trial court for further proceedings.

[G.R. No. 119064. August 22, 2000]

NENG KAGUI KADIGUIA MALANG, petitioner, vs. HON. COROCOY MOSON, Presiding Judge of 5th Sharia District
Court, Cotabato City, HADJI MOHAMMAD ULYSSIS MALANG, HADJI ISMAEL MALINDATU MALANG, FATIMA
MALANG, DATULNA MALANG, LAWANBAI MALANG, JUBAIDA KADO MALANG, NAYO OMAL MALANG and
MABAY GANAP MALANG, respondents.
DECISION
GONZAGA-REYES, J.:
Presented for resolution in this special civil action of certiorari is the issue of whether or not the regime of conjugal
partnership of gains governed the property relationship of two Muslims who contracted marriage prior to the effectivity of the
Code of Muslim Personal Laws of the Philippines (hereafter, P.D. 1083 or Muslim Code). The question is raised in connection
with the settlement of the estate of the deceased husband.
Hadji Abdula Malang, a Muslim, contracted marriage with Aida (Kenanday) Limba. They begot three sons named Hadji
Mohammad Ulyssis, Hadji Ismael Malindatu and Datulna, and a daughter named Lawanbai. Hadji Abdula Malang was engaged
in farming, tilling the land that was Aidas dowry (mahr or majar). Thereafter, he bought a parcel of land in Sousa,
Cotabato. Hadji Abdula and Aida already had two children when he married for the second time another Muslim named Jubaida
Kado in Kalumamis, Talayan, Maguindanao. No child was born out of Hadji Abdulas second marriage. When Aida, the first
wife, was pregnant with their fourth child, Hadji Abdula divorced her.
In 1965, Hadji Abdula married another Muslim, Nayo H. Omar but they were childless. Thereafter, Hadji Abdula contracted
marriage with Hadji Mabai (Mabay) H. Adziz in Kalumamis, Talayan, Maguindanao and soon they had a daughter named Fatima
(Kueng). Hadji Abdula and Hadji Mabai stayed in that place to farm while Hadji Abdula engaged in the business of buying and
selling of rice, corn and other agricultural products. Not long after, Hadji Abdula married three other Muslim women named
Saaga, Mayumbai and Sabai but he eventually divorced them.
Hadji Abdula then migrated to Tambunan where, in 1972, he married petitioner Neng Kagui Kadiguia Malang, his fourth
wife, excluding the wives he had divorced. They established residence in Cotabato City but they were childless. For a living,
they relied on farming and on the business of buying and selling of agricultural products. Hadji Abdula acquired vast tracts of
land in Sousa and Talumanis, Cotabato City, some of which were cultivated by tenants. He deposited money in such banks as
United Coconut Planters Bank, Metrobank and Philippine Commercial and Industrial Bank.
On December 18, 1993, while he was living with petitioner in Cotabato City, Hadji Abdula died without leaving a will. On
January 21, 1994, petitioner filed with the Sharia District Court in Cotabato City a petition for the settlement of his estate with a
prayer that letters of administration be issued in the name of her niece, Tarhata Lauban.
Petitioner claimed in that petition that she was the wife of Hadji Abdula; that his other legal heirs are his three children
named Teng Abdula, Keto Abdula and Kueng Malang, and that he left seven (7) parcels of land, five (5) of which are titled in
Hadji Abdulas name married to Neng P. Malang, and a pick-up jeepney.
On February 7, 1994, the Sharia District Court ordered the publication of the petition. [1] After such publication[2] or on March
16, 1994, Hadji Mohammad Ulyssis Malang (Hadji Mohammad, for brevity), the eldest son of Hadji Abdula, filed his opposition
to the petition. He alleged among other matters that his fathers surviving heirs are as follows: (a) Jubaida Malang, surviving
spouse; (b) Nayo Malang, surviving spouse; (c) Mabay Malang, surviving spouse; (d) petitioner Neng Malang, surviving
spouse; (e) oppositor Hadji Mohammad Ulyssis Malang who is also known as Teng Abdula, son; (f) Hadji Ismael Malindatu
Malang, also known as Keto Abdula, son, (g) Fatima Malang, also known as Kueng Malang, daughter; (h) Datulna Malang,
son, and (i) Lawanbai Malang, daughter. Oppositor Hadji Mohammad Ulyssis Malang alleged that since he and his brother,
Hadji Ismael Malindatu Malang, had helped their father in his business, then they were more competent to be administrators of
his estate.[3]
On March 30, 1994, Jubaida Malang, Ismael Malindatu Malang, Nayo Malang, Fatima Malang, Mabay Malang, Datulna
Malang and Lawanbai Malang filed an opposition to the petition, adopting as their own the written opposition of Hadji
Mohammad.[4]
On April 7, 1994, the Sharia District Court issued an Order appointing Hadji Mohammad administrator of his fathers
properties outside Cotabato City. The same order named petitioner and Hadji Ismael Malindatu Malang as joint administrators
of the estate in Cotabato City. Each administrator was required to post a bond in the amount of P100,000.00. [5] On April 13,
1994, letters of administration were issued to Hadji Mohammad after he had posted the required bond. He took his oath on the
same day.[6] The following day, Hadji Ismael and petitioner likewise filed their respective bonds and hence, they were allowed to
take their oath as administrators.[7]
On April 25, 1994 and May 3, 1994, petitioner filed two motions informing the court that Hadji Abdula had outstanding
deposits with nine (9) major banks. [8] Petitioner prayed that the managers of each of those banks be ordered to submit a bank
statement of the outstanding deposit of Hadji Abdula. [9] The Sharia District Court having granted the motions, [10] Assistant Vice
President Rockman O. Sampuha of United Coconut Planters Bank informed the court that as of April 24, 1994, the outstanding
deposit of Hadji Abdula amounted to one million five hundred twenty thousand four hundred pesos and forty-eight centavos
(P1,520,400.48).[11] The Senior Manager of the Cotabato branch of Metrobank also certified that as of December 18, 1993,
Hadji Abdula Malang or Malindatu Malang had on savings deposit the balance of three hundred seventy-eight thousand four
hundred ninety-three pesos and 32/100 centavos (P378,493.32). [12] PCIB likewise issued a certification that Hadji Abdula had a
balance of eight hundred fifty pesos (P850.00) in his current account as of August 11, 1994. [13]
During the pendency of the case, petitioner suffered a congestive heart failure that required immediate medical
treatment. On May 5, 1994, she filed a motion praying that on account of her ailment, she be allowed to withdraw from UCPB
the amount of three hundred thousand pesos (P300,000.00) that shall constitute her advance share in the estate of Hadji
Abdula.[14] After due hearing, the Sharia District Court allowed petitioner to withdraw the sum of two hundred fifty thousand
pesos (P250,000.00).[15]
On May 12, 1994, the Sharia District Court required petitioner and Hadji Ismael as joint administrators to submit an
inventory and appraisal of all properties of Hadji Abdula. [16] In compliance therewith, Hadji Ismael submitted an inventory
showing that in Cotabato City, Hadji Abdula had seven (7) residential lots with assessed value ranging from P5,020.00 to
P25,800.00, an agricultural land with assessed value of P860.00, three (3) one-storey residential buildings, and one (1) twostorey residential building.[17] All these properties were declared for taxation purposes in Hadji Abdulas name.
For her part, petitioner submitted an inventory showing that Hadji Abdula married to Neng Malang had seven (7) residential
lots with a total assessed value of P243,840.00 in Cotabato City, an Isuzu pick-up jeepney valued at P30,000.00 and bank
deposits.[18]
In the Memorandum that she filed with the Sharia District Court, petitioner asserted that all the properties located in
Cotabato City, including the vehicle and bank deposits, were conjugal properties in accordance with Article 160 of the Civil Code
and Article 116 of the Family Code while properties located outside of Cotabato City were exclusive properties of the decedent.
[19]

On the other hand, the oppositors contended in their own Memorandum that all the properties left by Hadji Abdula were his
exclusive properties for various reasons. First, Hadji Abdula had no conjugal partnership with petitioner because his having
contracted eight (8) marriages with different Muslim women was in violation of the Civil Code that provided for a monogamous
marriage; a conjugal partnership presupposes a valid civil marriage, not a bigamous marriage or a common-law
relationship. Second, the decedent adopted a complete separation of property regime in his marital relations; while his wives
Jubaida Kado, Nayo Hadji Omal and Mabay Ganap Hadji Adzis contributed to the decedents properties, there is no

evidence that petitioner had contributed funds for the acquisition of such properties. Third, the presumption that properties
acquired during the marriage are conjugal properties is inapplicable because at the time he acquired the properties, the
decedent was married to four (4) women. Fourth, the properties are not conjugal in nature notwithstanding that some of these
properties were titled in the name of the decedent married to Neng Malang because such description is not conclusive of the
conjugal nature of the property. Furthermore, because petitioner admitted in her verified petition that the properties belonged to
the estate of decedent, she was estopped from claiming, after formal offer of evidence, that the properties were conjugal in
nature just because some of the properties were titled in Hadji Abdulas name married to Neng Malang. Fifth, if it is true that
the properties were conjugal properties, then these should have been registered in the names of both petitioner and the
decedent.[20]
In its Order of September 26, 1994, the Sharia District Court presided by Judge Corocoy D. Moson held that there was no
conjugal partnership of gains between petitioner and the decedent primarily because the latter married eight times. The Civil
Code provision on conjugal partnership cannot be applied if there is more than one wife because conjugal partnership
presupposes a valid civil marriage, not a plural marriage or a common-law relationship. The court further found that the
decedent was the chief, if not the sole, breadwinner of his families and that petitioner did not contribute to the properties unlike
the other wives named Jubaida, Nayo and Mabay. The description married to Neng Malang in the titles to the real properties
is no more than that -- the description of the relationship between petitioner and the decedent. Such description is insufficient
to prove that the properties belong to the conjugal partnership of gains. The court stated:
In the instant case, decedent had four (4) wives at the time he acquired the properties in question. To sustain the contention of
the petitioner that the properties are her conjugal property with the decedent is doing violence to the provisions of the Civil
Code. Be it noted that at the time of the marriage of the petitioner with the decedent, there were already three (3) existing
marriages. Assuming for the moment that petitioner and the decedent had agreed that the property regime between them will
be governed by the regime of conjugal partnership property, that agreement is null and void for it is against the law, public
policy, public order, good moral(s) and customs.
Under Islamic law, the regime of property relationship is complete separation of property, in the absence of any stipulation to the
contrary in the marriage settlements or any other contract (Article 38, P.D. 1083). There being no evidence of such contrary
stipulation or contract, this Court concludes as it had begun, that the properties in question, both real and personal, are not
conjugal, but rather, exclusive property of the decedent. [21]
Thus, the Sharia District Court held that the Islamic law should be applied in the distribution of the estate of Hadji Abdula
and accordingly disposed of the case as follows:
WHEREFORE, premises considered, the Court orders the following:
1) That the estate shall pay the corresponding estate tax, reimburse the funeral expenses in the amount of P50,000.00,
and the judicial expenses in the amount of P2,040.80;
2) That the net estate, consisting of real and personal properties, located in Talayan, Maguindanao and in Cotabato City,
is hereby ordered to be distributed and adjudicated as follows:
a) Jubaida Kado Malang

------------------------- 2/64 of the estate

b) Nayo Omar Malang

------------------------- 2/64

- do -

c) Mabai Aziz Malang

------------------------- 2/64

- do -

d) Neng Kagui Kadiguia Malang ------------------- 2/64

- do -

e) Mohammad Ulyssis Malang-------------------------14/64

- do -

f) Ismael Malindatu Malang---------------------------14/64

- do -

g) Datulna Malang
h) Lawanbai Malang

------------------------- 14/64

- do -

------------------------- 7/64

i) Fatima (Kueng) Malang ------------------------- 7/64

- do -

- do -

Total------------------------ 64/64
3) That the amount of P250,000.00 given to Neng Kagui Kadiguia Malang by way of advance be charged against her
share and if her share is not sufficient, to return the excess; and
4) That the heirs are hereby ordered to submit to this court their Project of Partition for approval, not later than three (3)
months from receipt of this order.
SO ORDERED.
On October 4, 1994, petitioner filed a motion for the reconsideration of that Order. The oppositors objected to that
motion. On January 10, 1995, the Sharia District Court denied petitioners motion for reconsideration. [22] Unsatisfied, petitioner
filed a notice of appeal.[23] However, on January 19, 1995, she filed a manifestation withdrawing the notice of appeal on the
strength of the following provisions of P.D. No. 1083:
Art. 145. Finality of Decisions The decisions of the Sharia District Courts whether on appeal from the Sharia Circuit Court or
not shall be final. Nothing herein contained shall affect the original and appellate jurisdiction of the Supreme Court as provided
in the Constitution.
Petitioner accordingly informed the court that she would be filing an original action of certiorari with the Supreme Court.[24]
On March 1, 1995, petitioner filed the instant petition for certiorari with preliminary injunction and/or restraining order. She
contends that the Sharia District Court gravely erred in: (a) ruling that when she married Hadji Abdula Malang, the latter had
three existing marriages with Jubaida Kado Malang, Nayo Omar Malang and Mabay Ganap Malang and therefore the properties
acquired during her marriage could not be considered conjugal, and (b) holding that said properties are not conjugal because
under Islamic Law, the regime of relationship is complete separation of property, in the absence of stipulation to the contrary in
the marriage settlement or any other contract. [25]
As petitioner sees it, the law applicable on issues of marriage and property regime is the New Civil Code, under which all
property of the marriage is presumed to belong to the conjugal partnership. The Sharia Court, meanwhile, viewed the Civil
Code provisions on conjugal partnership as incompatible with plural marriage, which is permitted under Muslim law, and held
the applicable property regime to be complete separation of property under P.D. 1083.
Owing to the complexity of the issue presented, and the fact that the case is one of first impression --- this is a singular
situation where the issue on what law governs the property regime of a Muslim marriage celebrated prior to the passage of the
Muslim Code has been elevated from a Sharia court for the Courts resolution --- the Court decided to solicit the opinions of
two amici curiae, Justice Ricardo C. Puno[26] and former Congressman Michael O. Mastura [27]. The Court extends its warmest
thanks to the amici curiae for their valuable inputs in their written memoranda [28] and in the hearing of June 27, 2000.

Resolution of the instant case is made more difficult by the fact that very few of the pertinent dates of birth, death, marriage
and divorce are established by the record. This is because, traditionally, Muslims do not register acts, events or judicial decrees
affecting civil status.[29] It also explains why the evidence in the instant case consisted substantially of oral testimonies.
What is not disputed is that: Hadji Abdula contracted a total of eight marriages, counting the three which terminated in
divorce; all eight marriages were celebrated during the effectivity of the Civil Code and before the enactment of the Muslim
Code; Hadji Abdula divorced four wives --- namely, Aida, Saaga, Mayumbai and Sabai --- all divorces of which took place before
the enactment of the Muslim Code; and, Hadji Abdula died on December 18, 1993, after the Muslim Code and Family Code took
effect, survived by four wives (Jubaida, Nayo, Mabay and Neng) and five children, four of whom he begot with Aida and one
with Mabay. It is also clear that the following laws were in force, at some point or other, during the marriages of Hadji Abdula:
the Civil Code, which took effect on August 30, 1950; Republic Act No. 394 (R.A. 394), authorizing Muslim divorces, which was
effective from June 18, 1949 to June 13, 1969; the Muslim Code, which took effect February 4, 1977; and the Family Code,
effective August 3, 1988.
Proceeding upon the foregoing, the Court has concluded that the record of the case is simply inadequate for purposes of
arriving at a fair and complete resolution of the petition. To our mind, any attempt at this point to dispense with the basic issue
given the scantiness of the evidence before us could result in grave injustice to the parties in this case, as well as cast profound
implications on Muslim families similarly or analogously situated to the parties herein. Justice and accountability dictate a
remand; trial must reopen in order to supply the factual gaps or, in Congressman Masturas words, missing links, that would be
the bases for judgment and accordingly, allow respondent court to resolve the instant case. In ordering thus, however, we take
it as an imperative on our part to set out certain guidelines in the interpretation and application of pertinent laws to facilitate the
task of respondent court.
It will also be recalled that the main issue presented by the petition --- concerning the property regime applicable to two
Muslims married prior to the effectivity of the Muslim Code --- was interposed in relation to the settlement of the estate of the
deceased husband. Settlement of estates of Muslims whose civil acts predate the enactment of the Muslim Code may easily
result in the application of the Civil Code and other personal laws, thus convincing the Court that it is but propitious to go
beyond the issue squarely presented and identify such collateral issues as are required to be resolved in a settlement of estate
case. As amicus curiae Congressman Mastura puts it, the Court does not often come by a case as the one herein, and
jurisprudence will be greatly enriched by a discussion of the watershed of collateral issues that this case presents. [30]
The Court has identified the following collateral issues, which we hereby present in question form: (1) What law governs the
validity of a Muslim marriage celebrated under Muslim rites before the effectivity of the Muslim Code? (2) Are multiple marriages
celebrated before the effectivity of the Muslim Code valid? (3) How do the Courts pronouncements in People vs. Subano, 73
Phil. 692 (1942), and People vs. Dumpo, 62 Phil. 246 (1935), affect Muslim marriages celebrated before the effectivity of the
Muslim Code? (4) What laws govern the property relationship of Muslim multiple marriages celebrated before the Muslim Code?
(5) What law governs the succession to the estate of a Muslim who died after the Muslim Code and the Family Code took
effect? (6) What laws apply to the dissolution of property regimes in the cases of multiple marriages entered into before the
Muslim Code but dissolved (by the husbands death) after the effectivity of the Muslim Code? and (7) Are Muslim divorces
effected before the enactment of the Muslim Code valid?
The succeeding guidelines, which derive mainly from the Compliance of amicus curiae Justice Puno, are hereby laid down
by the Court for the reference of respondent court, and for the direction of the bench and bar:
First Collateral Issue: The Law(s) Governing Validity of Muslim Marriages Celebrated Before the Muslim Code

The time frame in which all eight marriages of Hadji Abdula were celebrated was during the effectivity of the Civil Code
which, accordingly, governs the marriages. Article 78 of the Civil Code[31] recognized the right of Muslims to contract marriage
in accordance with their customs and rites, by providing that --Marriages between Mohammedans or pagans who live in the non-Christian provinces may be performed in accordance with
their customs, rites or practices. No marriage license or formal requisites shall be necessary. Nor shall the persons
solemnizing these marriages be obliged to comply with article 92.
However, thirty years after the approval of this Code, all marriages performed between Muslims or other non-Christians shall be
solemnized in accordance with the provisions of this Code. But the President of the Philippines, upon recommendation of the
Commissioner of National Integration, may at any time before the expiration of said period, by proclamation, make any of said
provisions applicable to the Muslims and non-Christian inhabitants of any of the non-Christian provinces.
Notably, before the expiration of the thirty-year period after which Muslims are enjoined to solemnize their marriages in
accordance with the Civil Code, P.D. 1083 or the Muslim Code was passed into law. The enactment of the Muslim Code on
February 4, 1977 rendered nugatory the second paragraph of Article 78 of the Civil Code which provides that marriages
between Muslims thirty years after the approval of the Civil Code shall be solemnized in accordance with said Code.
Second and Third Collateral Issues: The Validity of Muslim Multiple Marriages Celebrated Before the Muslim Code; The Effect of People vs. Subano and People vs. Dumpo

Prior to the enactment of P.D. 1083, there was no law in this jurisdiction which sanctioned multiple marriages. [32] It is also not
to be disputed that the only law in force governing marriage relations between Muslims and non-Muslims alike was the Civil
Code of 1950.
The Muslim Code, which is the first comprehensive codification [33] of Muslim personal laws, [34] also provides in respect of acts
that transpired prior to its enactment:
Art. 186. Effect of code on past acts. --- (1) Acts executed prior to the effectivity of this Code shall be governed by the laws in
force at the time of their execution, and nothing herein except as otherwise specifically provided, shall affect their validity or
legality or operate to extinguish any right acquired or liability incurred thereby.
The foregoing provisions are consistent with the principle that all laws operate prospectively, unless the contrary appears or is
clearly, plainly and unequivocably expressed or necessarily implied; [35] accordingly, every case of doubt will be resolved against
the retroactive opertion of laws. [36]Article 186 aforecited enunciates the general rule of the Muslim Code to have its provisions
applied prospectively, and implicitly upholds the force and effect of a pre-existing body of law, specifically, the Civil Code --- in
respect of civil acts that took place before the Muslim Codes enactment.
Admittedly, an apparent antagonism arises when we consider that what the provisions of the Civil Code contemplate and
nurture is a monogamous marriage. Bigamous or polygamous marriages are considered void and inexistent from the time of
their performance.[37] The Family Code which superseded the Civil Code provisions on marriage emphasizes that a subsequent
marriage celebrated before the registration of the judgment declaring a prior marriage void shall likewise be void. [38] These
provisions illustrate that the marital relation perceived by the Civil Code is one that is monogamous, and that subsequent
marriages entered into by a person with others while the first one is subsisting is by no means countenanced.
Thus, when the validity of Muslim plural marriages celebrated before the enactment of the Muslim Code was touched upon
in two criminal cases, the Court applied the perspective in the Civil Code that only one valid marriage can exist at any given
time.
In People vs. Subano, supra, the Court convicted the accused of homicide, not parricide, since ---

(f)rom the testimony of Ebol Subano, father of the deceased, it appears that the defendant has three wives and that the
deceased was the last in point of time. Although the practice of polygamy is approved by custom among these nonChristians, polygamy, however, is not sanctioned by the Marriage Law [39], which merely recognizes tribal marriage
rituals. The deceased, under our law, is not thus the lawful wife of the defendant and this precludes conviction for the crime
of parricide.
In People vs. Dumpo, supra, Mora Dumpo was prosecuted for bigamy when, legally married to Moro Hassan, she allegedly
contracted a second marriage with Moro Sabdapal. The Court acquitted her on the ground that it was not duly proved that the
alleged second marriage had all the essential requisites to make it valid were it not for the subsistence of the first marriage. As
it appears that the consent of the brides father is an indispensable requisite to the validity of a Muslim marriage, and as Mora
Dumpos father categorically affirmed that he did not give his consent to her union with Moro Sabdapal, the Court held that such
union could not be a marriage otherwise valid were it not for the existence of the first one, and resolved to acquit her of the
charge of bigamy.
The ruling in Dumpo indicates that, had it been proven as a fact that the second marriage contained all the essential
requisites to make it valid, a conviction for bigamy would have prospered. [40]
Fourth Collateral Issue: Law(s) Governing Property Relations of Muslim Marriages Celebrated Before the Muslim Code

This is the main issue presented by the instant petition. In keeping with our holding that the validity of the marriages in the
instant case is determined by the Civil Code, we hold that it is the same Code that determines and governs the property
relations of the marriages in this case, for the reason that at the time of the celebration of the marriages in question the Civil
Code was the only law on marriage relations, including property relations between spouses, whether Muslim or nonMuslim. Inasmuch as the Family Code makes substantial amendments to the Civil Code provisions on property relations, some
of its provisions are also material, particularly to property acquired from and after August 3, 1988.
Which law would govern depends upon: (1) when the marriages took place; (2) whether the parties lived together as
husband and wife; and (3) when and how the subject properties were acquired.
Following are the pertinent provisions of the Civil Code:
Art. 119. The future spouses may in the marriage settlements agree upon absolute or relative community of property, or upon
complete separation of property, or upon any other regime. In the absence of marriage settlements, or when the same are void,
the system of relative community or conjugal partnership of gains as established in this Code shall govern the property relations
between husband and wife.
Art. 135. All property brought by the wife to the marriage, as well as all property she acquires during the marriage, in
accordance with article 148, is paraphernal.
Art. 136. The wife retains the ownership of the paraphernal property.
Art. 142. By means of the conjugal partnership of gains the husband and wife place in a common fund the fruits of their
separate property and the income from their work or industry, and divide equally, upon the dissolution of the marriage or of the
partnership, the net gains or benefits obtained indiscriminately by either spouse during the marriage.
Art. 143. All property of the conjugal partnership of gains is owned in common by the husband and wife.
The Civil Code also provides in Article 144:
When a man and a woman live together as husband and wife, but they are not married, or their marriage is void from the
beginning, the property acquired by either or both of them through their work or industry or their wages and salaries shall be
governed by the rules on co-ownership.
In a long line of cases, this Court has interpreted the co-ownership provided in Article 144 of the Civil Code to require that
the man and woman living together as husband and wife without the benefit of marriage or under a void marriage must not in
any way be incapacitated to marry.[41] Situating these rulings to the instant case, therefore, the co-ownership contemplated in
Article 144 of the Civil Code cannot apply to Hadji Abdulas marriages celebrated subsequent to a valid and legally existing
marriage, since from the point of view of the Civil Code Hadji Abdula is not capacitated to marry. However, the wives in such
marriages are not precluded from proving that property acquired during their cohabitation with Hadji Abdula is
their exclusive property, respectively.[42] Absent such proof, however, the presumption is that property acquired during the
subsistence of a valid marriage --- and in the Civil Code, there can only be one validly existing marriage at any given time --- is
conjugal property of such subsisting marriage. [43]
With the effectivity of the Family Code on August 3, 1988, the following provisions of the said Code are pertinent:
Art. 147. When a man and a woman who are capacitated to marry each other live exclusively with each other as husband and
wife without the benefit of marriage or under a void marriage, their wages and salaries shall be owned by them in equal shares
and the property acquired by both of them through their work or industry shall be governed by the rules on co-ownership.
In the absence of proof to the contrary, properties acquired while they lived together shall be presumed to have been obtained
by their joint efforts, work or industry, and shall be owned by them in equal shares. For purposes of this Article, a party who did
not participate in the acquisition of the other party of any property shall be deemed to have contributed jointly in the acquisition
thereof if the formers efforts consisted in the care and maintenance of the family and of the household.
Neither party can encumber or dispose by acts inter vivos of his or her share in the property acquired during cohabitation
and owned in common, without the consent of the other, until after the termination of the cohabitation.
When only one of the parties to a void marriage is in good faith, the share of the party in bad faith in the co-ownership shall be
forfeited in favor of their common children. In case of default or of waiver by any or all of the common children or their
descendants, each vacant share shall belong to the respective surviving descendants. In the absence of descendants, such
share shall belong to the innocent party. In all cases, the forfeiture shall take place upon termination of the cohabitation.
Art. 148. In cases of cohabitation not falling under the preceding Article, only the properties acquired by both of the parties
through their actual joint contribution of money, property, or industry shall be owned by them in common in proportion to their
respective contributions. In the absence of proof to the contrary, their contributions and corresponding shares are presumed to
be equal. The same rule and presumption shall apply to joint deposits of money and evidences of credit.
If one of the parties is validly married to another, his or her share in the co-ownership shall accrue to the absolute community or
conjugal partnership existing in such valid marriage. If the party who acted in bad faith is not validly married to another, his or
her share shall be forfeited in the manner provided in the last paragraph of the preceding Article.
The foregoing rules on forfeiture shall likewise apply even if both parties are in bad faith.
It will be noted that while the Civil Code merely requires that the parties live together as husband and wife the Family Code
in Article 147 specifies that they live exclusively with each other as husband and wife. Also, in contrast to Article 144 of the
Civil Code as interpreted by jurisprudence, Article 148 of the Family Code allows for co-ownership in cases of cohabitation
where, for instance, one party has a pre-existing valid marriage, provided that the parties prove their actual joint contribution of

money, property, or industry and only to the extent of their proportionate interest therein. The rulings in Juaniza vs. Jose, 89
SCRA 306, Camporodendo vs. Garcia, 102 Phil. 1055, and related cases are embodied in the second paragraph of Article 148,
which declares that the share of the party validly married to another shall accrue to the property regime of such existing
marriage.
Fifth and Sixth Collateral Issues: Law(s) on Succession and Dissolution of Property Regimes

Hadji Abdula died intestate on December 16, 1993. Thus, it is the Muslim Code which should determine the identification of
the heirs in the order of intestate succession and the respective shares of the heirs.
Meanwhile, the status and capacity to succeed on the part of the individual parties who entered into each and every
marriage ceremony will depend upon the law in force at the time of the performance of the marriage rite.
The status and capacity to succeed of the children will depend upon the law in force at the time of conception or birth of the
child. If the child was conceived or born during the period covered by the governance of the Civil Code, the Civil Code
provisions on the determination of the legitimacy or illegitimacy of the child would appear to be in point. Thus, the Civil Code
provides:
Art. 255. Children born after one hundred and eighty days following the celebration of the marriage, and before three hundred
days following its dissolution or the separation of the spouses shall be presumed to be legitimate.
Against this presumption no evidence shall be admitted other than that of the physical impossibility of the husbands having
access to his wife within the first one hundred and twenty days of the three hundred which preceded the birth of the child.
This physical impossibility may be caused:
(1) By the impotence of the husband;
(2) By the fact that the husband and wife were living separately, in such a way that access was not possible;
(3) By the serious illness of the husband.
Art. 256. The child shall be presumed legitimate, although the mother may have declared against its legitimacy or may have
been sentenced as an adulteress.
If the child was conceived or born during the period covered by the governance of the Muslim Code, i.e., from February 4,
1977 up to the death of Hadji Abdula on December 18, 1993, the Muslim Code determines the legitimacy or illegitimacy of the
child. Under the Muslim Code:
Art. 58. Legitimacy, how established. --- Legitimacy of filiation is established by the evidence of valid marriage between the
father and the mother at the time of the conception of the child.
Art. 59. Legitimate children. --(1) Children conceived in lawful wedlock shall be presumed to be legitimate. Whoever claims illegitimacy of or impugns
such filiation must prove his allegation.
(2) Children born after six months following the consummation of marriage or within two years after the dissolution of the
marriage shall be presumed to be legitimate. Against this presumption no evidence shall be admitted other than that
of physical impossibility of access between the parents at or about the time of the conception of the child.
Art. 60. Children of subsequent marriage. --- Should the marriage be dissolved and the wife contracts another marriage after
the expiration of her idda, the child born within six months from the dissolution of the prior marriage shall be presumed to have
been conceived during the former marriage, and if born thereafter, during the latter.
Art. 61. Pregnancy after dissolution. --- If, after the dissolution of marriage, the wife believes that she is pregnant by her former
husband, she shall, within thirty days from the time she became aware of her pregnancy, notify the former husband or his heirs
of that fact. The husband or his heirs may ask the court to take measures to prevent a simulation of birth.
Upon determination of status and capacity to succeed based on the foregoing provisions, the provisions on legal succession
in the Muslim Code will apply. Under Article 110 of the said Code, the sharers to an inheritance include:
(a) The husband, the wife;
(b) The father, the mother, the grandfather, the grandmother;
(c) The daughter and the sons daughter in the direct line;
(d) The full sister, the consanguine sister, the uterine sister and the uterine brother.
When the wife survives with a legitimate child or a child of the decedents son, she is entitled to one-eighth of the hereditary
estate; in the absence of such descendants, she shall inherit one-fourth of the estate. [44] The respective shares of the other
sharers, as set out in Article 110 abovecited, are provided for in Articles 113 to 122 of P.D. 1083.
Seventh Collateral Issue: Muslim Divorces Before the Effectivity of the Muslim Code

R.A. 394 authorized absolute divorce among Muslims residing in non-Christian provinces, in accordance with Muslim
custom, for a period of 20 years from June 18, 1949 (the date of approval of R.A. 394) to June 13, 1969. [45] Thus, a Muslim
divorce under R.A. 394 is valid if it took place from June 18, 1949 to June 13, 1969.
From the seven collateral issues that we discussed, we identify four corollary issues as to further situate the points of
controversy in the instant case for the guidance of the lower court. Thus:
1. Which of the several marriages was validly and legally existing at the time of the opening of the succession of Hadji
Abdula when he died in 1993? The validly and legally existing marriage would be that marriage which was celebrated at a time
when there was no other subsisting marriage standing undissolved by a valid divorce or by death. This is because all of the
marriages were celebrated during the governance of the Civil Code, under the rules of which only one marriage can exist at any
given time.
Whether or not the marriage was validly dissolved by a Muslim divorce depends upon the time frame and the applicable
law. A Muslim divorce under R.A. No. 394 is valid if it took place from June 18, 1949 to June 13, 1969, and void if it took place
from June 14, 1969. [46]
2. There being a dispute between the petitioner and the oppositors as regards the heirship of the children begotten from
different marriages, who among the surviving children are legitimate and who are illegitimate? The children conceived and born
of a validly existing marriage as determined by the first corollary issue are legitimate. The fact and time of conception or birth
may be determined by proof orpresumption depending upon the time frame and the applicable law.
3. What properties constituted the estate of Hadji Abdula at the time of his death on December 18, 1993? The estate of
Hadji Abdula consists of the following:

a. Properties acquired during the existence of a valid marriage as determined by the first corollary issue are conjugal
properties and should be liquidated and divided between the spouses under the Muslim Code, this being the law in force at the
time of Hadji Abdulas death.
b. Properties acquired under the conditions prescribed in Article 144 of the Civil Code during the period August 30, 1950 to
August 2, 1988 are conjugal properties and should be liquidated and divided between the spouses under the Muslim
Code. However, the wives other than the lawful wife as determined under the first corollary issue may submit their respective
evidence to prove that any of such property is theirs exclusively.
c. Properties acquired under the conditions set out in Articles 147 and 148 of the Family Code during the period from and
after August 3, 1988 are governed by the rules on co-ownership.
d. Properties acquired under conditions not covered by the preceding paragraphs and obtained from the exclusive efforts or
assets of Hadji Abdula are his exclusive properties.
4. Who are the legal heirs of Hadji Abdula, and what are their shares in intestacy? The following are Hadji Abdulas legal
heirs: (a) the lawful wife, as determined under the first corollary issue, and (2) the children, as determined under the second
corollary issue. The Muslim Code, which was already in force at the time of Hadji Abdulas death, will govern the determination
of their respective shares.
As we have indicated early on, the evidence in this case is inadequate to resolve in its entirety the main, collateral and
corollary issues herein presented and a remand to the lower court is in order. Accordingly, evidence should be received to
supply the following proofs: (1) the exact dates of the marriages performed in accordance with Muslim rites or practices; (2) the
exact dates of the dissolutions of the marriages terminated by death or by divorce in accordance with Muslim rites and
practices, thus indicating which marriage resulted in a conjugal partnership under the criteria prescribed by the first, second,
and third collateral issues and the first corollary issue; (3) the exact periods of actual cohabitation (common life under a
common roof) of each of the marriages during which time the parties lived together; (4) the identification of specific properties
acquired during each of the periods of cohabitation referred to in paragraph 3 above, and the manner and source of acquisition,
indicating joint or individual effort, thus showing the asset as owned separately, conjugally or in co-ownership; and (5) the
identities of the children (legitimate or illegitimate) begotten from the several unions, the dates of their respective conceptions or
births in relation to paragraphs 1 and 2 above, thereby indicating their status as lawful heirs.
Amicus curiae Congressman Mastura agrees that since the marriage of petitioner to decedent took place in 1972 the Civil
Code is the law applicable on the issue of marriage settlement, [47] but espouses that customs or established practices among
Muslims in Mindanao must also be applied with the force of law to the instant case. [48] Congressman Masturas disquisition has
proven extremely helpful in impressing upon us the background in which Islamic law and the Muslim Code need to be
interpreted, particularly the interconnectedness of law and religion for Muslims [49] and the impracticability of a strict application of
the Civil Code to plural marriages recognized under Muslim law. [50] Regrettably, the Court is duty-bound to resolve the instant
case applying such laws and rights as are in existence at the time the pertinent civil acts took place. Corollarily, we are unable
to supplant governing law with customs, albeit how widely observed. In the same manner, we cannot supply a perceived hiatus
in P.D. 1083 concerning the distribution of property between divorced spouses upon one of the spouses death. 51
WHEREFORE, the decision dated September 26, 1994 of the Fifth Sharia District Court of Cotabato City in Special
Proceeding No. 94-40 is SET ASIDE, and the instant petition is REMANDED for the reception of additional evidence and the
resolution of the issues of the case based on the guidelines set out in this Decision.

[G.R. No. 109557. November 29, 2000]

JOSE UY and his Spouse GLENDA J. UY and GILDA L. JARDELEZA, petitioners, vs. COURT OF APPEALS and
TEODORO L. JARDELEZA, respondents.
DECISION
PARDO, J.:
The case is an appeal via certiorari from the decision[1] of the Court of Appeals and its resolution denying
reconsideration[2] reversing that of the Regional Trial Court, Iloilo, Branch 32[3] and declaring void the special
proceedings instituted therein by petitioners to authorize petitioner Gilda L. Jardeleza, in view of the comatose condition of her
husband, Ernesto Jardeleza, Sr., with the approval of the court, to dispose of their conjugal property in favor of co-petitioners,
their daughter and son in law, for the ostensible purpose of financial need in the personal, business and medical expenses of
her incapacitated husband.
The facts, as found by the Court of Appeals, are as follows:
This case is a dispute between Teodoro L. Jardeleza (herein respondent) on the one hand, against his mother Gilda L.
Jardeleza, and sister and brother-in-law, the spouses Jose Uy and Glenda Jardeleza (herein petitioners) on the other
hand. The controversy came about as a result of Dr. Ernesto Jardeleza, Sr.s suffering of a stroke on March 25, 1991, which left
him comatose and bereft of any motor or mental faculties. Said Ernesto Jardeleza, Sr. is the father of herein respondent
Teodoro Jardeleza and husband of herein private respondent Gilda Jardeleza.
Upon learning that one piece of real property belonging to the senior Jardeleza spouses was about to be sold, petitioner
Teodoro Jardeleza, on June 6, 1991, filed a petition (Annex A) before the R.T.C. of Iloilo City, Branch 25, where it was
docketed as Special Proceeding No. 4689, in the matter of the guardianship of Dr. Ernesto Jardeleza, Sr. The petitioner
averred therein that the present physical and mental incapacity of Dr. Ernesto Jardeleza, Sr. prevent him from competently
administering his properties, and in order to prevent the loss and dissipation of the Jardelezas real and personal assets, there
was a need for a court-appointed guardian to administer said properties. It was prayed therein that Letters of Guardianship be
issued in favor of herein private respondent Gilda Ledesma Jardeleza, wife of Dr. Ernesto Jardeleza, Sr. It was further prayed
that in the meantime, no property of Dr. Ernesto Jardeleza, Sr. be negotiated, mortgaged or otherwise alienated to third
persons, particularly Lot No. 4291 and all the improvements thereon, located along Bonifacio Drive, Iloilo City, and covered by
T.C.T. No. 47337.
A few days later, or on June 13, 1991, respondent Gilda L. Jardeleza herself filed a petition docketed as Special Proceeding
NO. 4691, before Branch 32 of the R.T.C. of Iloilo City, regarding the declaration of incapacity of Ernesto Jardeleza, Sr.,
assumption of sole powers of administration of conjugal properties, and authorization to sell the same (Annex B). Therein, the
petitioner Gilda L. Jardeleza averred the physical and mental incapacity of her husband, who was then confined for intensive
medical care and treatment at the Iloilo Doctors Hospital. She signified to the court her desire to assume sole powers of
administration of their conjugal properties. She also alleged that her husbands medical treatment and hospitalization expenses
were piling up, accumulating to several hundred thousands of pesos already. For this, she urgently needed to sell one piece of
real property, specifically Lot No. 4291 and its improvements. Thus, she prayed for authorization from the court to sell said
property.
The following day, June 14, 1991, Branch 32 of the R.T.C. of Iloilo City issued an Order (Annex C) finding the petition in
Spec. Proc. No. 4691 to be sufficient in form and substance, and setting the hearing thereof for June 20, 1991. The scheduled
hearing of the petition proceeded, attended by therein petitioner Gilda Jardeleza, her counsel, her two children, namely Ernesto
Jardeleza, Jr., and Glenda Jardeleza Uy, and Dr. Rolando Padilla, one of Ernesto Jardeleza, Sr.s attending physicians.
On that same day, June 20, 1991, Branch 32 of the RTC of Iloilo City rendered its Decision (Annex D), finding that it was
convinced that Ernesto Jardeleza, Sr. was truly incapacitated to participate in the administration of the conjugal properties, and
that the sale of Lot No. 4291 and the improvements thereon was necessary to defray the mounting expenses for treatment and
Hospitalization. The said court also made the pronouncement that the petition filed by Gilda L. Jardeleza was pursuant to
Article 124 of the Family Code, and that the proceedings thereon are governed by the rules on summary proceedings
sanctioned under Article 253 of the same Code x x x.
The said court then disposed as follows:
WHEREFORE, there being factual and legal bases to the petition dated June 13, 1991, the Court hereby renders judgment as
follows:
1) declaring Ernesto Jardeleza, Sr., petitioners husband, to be incapacitated and unable to participate in the administration of
conjugal properties;
2) authorizing petitioner Gilda L. Jardeleza to assume sole powers of administration of their conjugal properties; and
3) authorizing aforesaid petitioner to sell Lot No. 4291 of the Cadastral Survey of Iloilo, situated in Iloilo City and covered by
TCT No. 47337 issued in the names of Ernesto Jardeleza, Sr. and Gilda L. Jardeleza and the buildings standing thereof.
SO ORDERED.
On June 24, 1991, herein petitioner Teodoro Jardeleza filed his Opposition to the proceedings before Branch 32 in Spec. Proc.
Case No. 4691, said petitioner being unaware and not knowing that a decision has already been rendered on the case by public
respondent.
On July 3, 1991, herein petitioner Teodoro Jardeleza filed a motion for reconsideration of the judgment in Spec. Proc. No. 4691
and a motion for consolidation of the two cases (Annex F). He propounded the argument that the petition for declaration of
incapacity, assumption of sole powers of administration, and authority to sell the conjugal properties was essentially a petition
for guardianship of the person and properties of Ernesto Jardeleza, Sr. As such, it cannot be prosecuted in accordance with the
provisions on summary proceedings set out in Article 253 of the Family Code. It should follow the rules governing special
proceedings in the Revised Rules of Court which require procedural due process, particularly the need for notice and a hearing
on the merits. On the other hand, even if Gilda Jardelezas petition can be prosecuted by summary proceedings, there was
still a failure to comply with the basic requirements thereof, making the decision in Spec. Proc. No. 4691 a defective one. He
further alleged that under the New Civil Code, Ernesto Jardeleza, Sr. had acquired vested rights as a conjugal partner, and that
these rights cannot be impaired or prejudiced without his consent. Neither can he be deprived of his share in the conjugal
properties through mere summary proceedings. He then restated his position that Spec. Proc. No. 4691 should be consolidated
with Spec. Proc. No. 4689 which was filed earlier and pending before Branch 25.
Teodoro Jardeleza also questioned the propriety of the sale of Lot No. 4291 and the improvements thereon supposedly to pay
the accumulated financial obligations arising from Ernesto Jardeleza, Sr.s hospitalization. He alleged that the market value of
the property would be around Twelve to Fifteen Million Pesos, but that he had been informed that it would be sold for much
less. He also pointed out that the building thereon which houses the Jardeleza Clinic is a monument to Ernesto Jardeleza Sr.s
industry, labor and service to his fellowmen. Hence, the said property has a lot of sentimental value to his family. Besides,
argued Teodoro Jardeleza, then conjugal partnership had other liquid assets to pay off all financial obligations. He mentioned

that apart from sufficient cash, Jardeleza, Sr. owned stocks of Iloilo Doctors Hospital which can be off-set against the cost of
medical and hospital bills. Furthermore, Ernesto Jardeleza, Sr. enjoys certain privileges at the said hospital which allows him to
pay on installment basis. Moreover, two of Ernesto Jardeleza Sr.s attending physicians are his own sons who do not charge
anything for their professional services.
On July 4, 1991, Teodoro Jardeleza filed in Spec. Proc. No. 4691 a supplement to his motion for reconsideration (Annex
G). He reiterated his contention that summary proceedings was irregularly applied. He also noted that the provisions on
summary proceedings found in Chapter 2 of the Family Code comes under the heading on Separation in Fact Between
Husband and Wife which contemplates of a situation where both spouses are of disposing mind. Thus, he argued that were
one spouse is comatose without motor and mental faculties, the said provisions cannot be made to apply.
While the motion for reconsideration was pending, Gilda Jardeleza disposed by absolute sale Lot No. 4291 and all its
improvements to her daughter, Ma. Glenda Jardeleza Uy, for Eight Million Pesos (P8,000,000.00), as evidenced by a Deed
Absolute Sale dated July 8, 1991 executed between them (p. 111, Rollo). Under date of July 23, 1991, Gilda Jardeleza filed an
urgent ex-parte motion for approval of the deed of absolute sale.
On August 12, 1991 Teodoro Jardeleza filed his Opposition to the motion for approval of the deed of sale on the grounds that:
(1) the motion was prematurely filed and should be held in abeyance until the final resolution of the petition; (2) the motion does
not allege nor prove the justifications for the sale; and (3) the motion does not allege that had Ernesto Jardeleza, Sr. been
competent, he would have given his consent to the sale.
Judge Amelita K. del Rosario-Benedicto of Branch 32 of the respondent Court, who had penned the decision in Spec. Proc. No.
4691 had in the meantime formally inhibited herself from further acting in this case (Annex I). The case was then reraffled to
Branch 28 of the said court.
On December 19, 1991, the said court issued an Order (Annex M) denying herein petitioners motion for reconsideration and
approving respondent Jardelezas motion for approval of the deed of absolute sale. The said court ruled that:
After a careful and thorough perusal of the decision, dated June 20, 1991, the Motion for Reconsideration, as well as its
supplements filed by oppositor, Teodoro L. Jardeleza, through counsel, and the opposition to the Motion for Reconsideration,
including its supplements, filed by petitioner, through counsel, this Court is of the opinion and so holds, that her Honor, Amelita
K. del Rosario-Benedicto, Presiding Judge of Branch 32, of this Court, has properly observed the procedure embodied under
Article 253, in relation to Article 124, of the Family Code, in rendering her decision dated June 20, 1991.
Also, as correctly stated by petitioner, through counsel, that oppositor Teodor L. Jardeleza does not have the personality to
oppose the instant petition considering that the property or properties, subject of the petition, belongs to the conjugal
partnership of the spouses Ernesto and Gilda Jardeleza, who are both still alive.
In view thereof, the Motion for Reconsideration of oppositor Teodoro L. Jardeleza, is hereby denied for lack of merit.
Considering the validity of the decision dated June 20, 1991, which among others, authorized Gilda L. Jardeleza to sell Lot No.
4291 of the Cadastral Survey of Iloilo, covered by Transfer Certificate of Title No. 47337 issued in the names of Ernesto
Jardeleza, Sr., and Gilda L. Jardeleza and the building standing thereon, the Urgent Ex-Parte Motion for Approval of Deed of
Absolute Sale dated July 23, 1991, filed by petitioner, through counsel, is hereby granted and the deed of absolute sale,
executed and notarized on July 8, 1991, by and between Gilda L. Jardeleza, as vendor, and Ma. Glenda Jardeleza, as vendee,
is hereby approved, and the Register of Deeds of Iloilo City, is directed to register the sale and issue the corresponding transfer
certificate of title to the vendee.
SO ORDERED.[4]
On December 9, 1992, the Court of Appeals promulgated its decision reversing the appealed decision and ordering the trial
court to dismiss the special proceedings to approve the deed of sale, which was also declared void. [5]
On December 29, 1992, petitioners filed a motion for reconsideration, [6] however, on March 29, 1993, the Court of Appeals
denied the motion, finding no cogent and compelling reason to disturb the decision. [7]
Hence, this appeal.[8]
The issue raised is whether petitioner Gilda L. Jardeleza as the wife of Ernesto Jardeleza, Sr. who suffered a stroke, a
cerebrovascular accident, rendering him comatose, without motor and mental faculties, and could not manage their conjugal
partnership property may assume sole powers of administration of the conjugal property under Article 124 of the Family Code
and dispose of a parcel of land with its improvements, worth more than twelve million pesos, with the approval of the court in a
summary proceedings, to her co-petitioners, her own daughter and son-in-law, for the amount of eight million pesos.
The Court of Appeals ruled that in the condition of Dr. Ernesto Jardeleza, Sr., the procedural rules on summary proceedings
in relation to Article 124 of the Family Code are not applicable. Because Dr. Jardeleza, Sr. was unable to take care of himself
and manage the conjugal property due to illness that had rendered him comatose, the proper remedy was the appointment of a
judicial guardian of the person or estate or both of such incompetent, under Rule 93, Section 1, 1964 Revised Rules of
Court. Indeed, petitioner earlier had filed such a petition for judicial guardianship.
Article 124 of the Family Code provides as follows:
ART. 124. The administration and enjoyment of the conjugal partnership property shall belong to both spouses jointly. In case
of disagreement, the husbands decision shall prevail, subject to recourse to the court by the wife for a proper remedy which
must be availed of within five years from the date of the contract implementing such decision.
In the event that one spouse is incapacitated or otherwise unable to participate in the administration of the conjugal properties,
the other spouse may assume sole powers of administration. These powers do not include the powers of disposition or
encumbrance which must have the authority of the court or the written consent of the other spouse. In the absence of such
authority or consent, the disposition or encumbrance shall be void. However, the transaction shall be construed as a continuing
offer on the part of the consenting spouse and the third person, and may be perfected as a binding contract upon the
acceptance by the other spouse or authorization by the court before the offer is withdrawn by either or both offerors. (165a).
In regular manner, the rules on summary judicial proceedings under the Family Code govern the proceedings under Article
124 of the Family Code. The situation contemplated is one where the spouse is absent, or separated in fact or has abandoned
the other or consent is withheld or cannot be obtained. Such rules do not apply to cases where the non-consenting spouse is
incapacitated or incompetent to give consent. In this case, the trial court found that the subject spouse "is an incompetent" who
was in comatose or semi-comatose condition, a victim of stroke, cerebrovascular accident, without motor and mental faculties,
and with a diagnosis of brain stem infarct. [9] In such case, the proper remedy is a judicial guardianship proceedings under Rule
93 of the 1964 Revised Rules of Court.
Even assuming that the rules of summary judicial proceedings under the Family Code may apply to the wife's administration
of the conjugal property, the law provides that the wife who assumes sole powers of administration has the same powers and
duties as a guardian under the Rules of Court. [10]
Consequently, a spouse who desires to sell real property as such administrator of the conjugal property must observe the
procedure for the sale of the wards estate required of judicial guardians under Rule 95, 1964 Revised Rules of Court, not the
summary judicial proceedings under the Family Code.

In the case at bar, the trial court did not comply with the procedure under the Revised Rules of Court. Indeed, the trial court
did not even observe the requirements of the summary judicial proceedings under the Family Code. Thus, the trial court did not
serve notice of the petition to the incapacitated spouse; it did not require him to show cause why the petition should not be
granted.
Hence, we agree with the Court of Appeals that absent an opportunity to be heard, the decision rendered by the trial court is
void for lack of due process. The doctrine consistently adhered to by this Court is that a denial of due process suffices to cast on
the official act taken by whatever branch of the government the impress of nullity. [11] A decision rendered without due process is
void ab initio and may be attacked directly or collaterally.[12] A decision is void for lack of due process if, as a result, a party is
deprived of the opportunity of being heard. [13] A void decision may be assailed or impugned at any time either directly or
collaterally, by means of a separate action, or by resisting such decision in any action or proceeding where it is invoked. [14]

SPS. LITA DE LEON and

G.R. No. 185063

FELIX RIO TARROSA,


Petitioners,
- versus ANITA B. DE LEON, DANILO B.
DE LEON, and VILMA B. DELEON,
Respondents
.

Present:
YNARES-SANTIAGO, Chairperson,
CHICO-NAZARIO,
VELASCO, JR.,
NACHURA, and
PERALTA, JJ.
Promulgated:

July 23, 2009


x-----------------------------------------------------------------------------------------x
DECISION
VELASCO, JR., J.:
The Case
Before us is a Petition for Review on Certiorari under Rule 45 assailing and seeking to set aside the Decision [1] and
Resolution[2] dated August 27, 2008 and October 20, 2008, respectively, of the Court of Appeals (CA) in CA-G.R. CV No.
88571. The CA affirmed with modification the October 4, 2006 Decision [3] in Civil Case No. Q04-51595 of the Regional Trial
Court (RTC), Branch 22 in Quezon City.
The Facts
On July 20, 1965, Bonifacio O. De Leon, then single, and the Peoples Homesite and Housing Corporation (PHHC)
entered into a Conditional Contract to Sell for the purchase on installment of a 191.30 square-meter lot situated
in Fairview, Quezon City. Subsequently, on April 24, 1968, Bonifacio married Anita de Leon in a civil rite officiated by the
Municipal Mayor of Zaragosa, Nueva Ecija. To this union were born Danilo and Vilma.
Following the full payment of the cost price for the lot thus purchased, PHHC executed, on June 22, 1970, a Final Deed of
Sale in favor of Bonifacio. Accordingly, Transfer Certificate of Title (TCT) No. 173677 was issued on February 24, 1972 in the
name of Bonifacio, single.
Subsequently, Bonifacio, for PhP 19,000, sold the subject lot to her sister, Lita, and husband Felix Rio Tarrosa (Tarrosas),
petitioners herein. The conveying Deed of Sale dated January 12, 1974 (Deed of Sale) did not bear the written consent and
signature of Anita.
Thereafter, or on May 23, 1977, Bonifacio and Anita renewed their vows in a church wedding at St. John the Baptist
Parish inSan Juan, Manila.
On February 29, 1996, Bonifacio died.
Three months later, the Tarrosas registered the Deed of Sale and had TCT No. 173677 canceled. They secured the
issuance in their names of TCT No. N-173911 from the Quezon City Register of Deeds.
Getting wind of the cancellation of their fathers title and the issuance of TCT No. N-173911, Danilo and Vilma filed on
May 19, 2003 a Notice of Adverse Claim before the Register of Deeds of Quezon City to protect their rights over the subject
property. Very much later, Anita, Danilo, and Vilma filed a reconveyance suit before the RTC in Quezon City. In their complaint,
Anita and her children alleged, among other things, that fraud attended the execution of the Deed of Sale and that subsequent
acts of Bonifacio would show that he was still the owner of the parcel of land. In support of their case, they presented, inter alia,
the following documents:
a.
A Real Estate Mortgage execution by Bonifacio in favor of spouses Cesar Diankinay and Filomena
Almero on July 22, 1977.
b.
A Civil Complaint filed by Bonifacio against spouses Cesar Diankinay and Filomena Almero on
November 27, 1979 for nullification of the Real Estate Mortgage.
c.
The Decision issued by the Court of First Instance of Rizal, Quezon City, promulgated on July 30,
1982, nullifying the Real Estate Mortgage. [4]

The Tarrosas, in their Answer with Compulsory Counterclaim, averred that the lot Bonifacio sold to them was his exclusive
property inasmuch as he was still single when he acquired it from PHHC. As further alleged, they were not aware of the
supposed marriage between Bonifacio and Anita at the time of the execution of the Deed of Sale.
After several scheduled hearings, both parties, assisted by their respective counsels, submitted a Joint Stipulation of
Facts with Motion, to wit:
1.
The parties have agreed to admit the following facts:
a.
Bonifacio O. De Leon, while still single x x x, purchased from the [PHHC] through a Conditional
Contract to Sell on July 20, 1965 a parcel of land with an area of 191.30 square meters situated in Fairview, Quezon
City for P841.72;
b.
On April 24, 1968, Bonifacio O. De Leon married plaintiff Anita B. De Leon before the Municipal Mayor
of Zaragosa, Nueva Ecija. Both parties stipulate that said marriage is valid and binding under the laws of
the Philippines;
c.
On June 22, 1970, Bonifacio O. De Leon paid [PHHC] the total amount of P1,023.74 x x x. The right of
ownership over the subject parcel of land was transferred to the late Bonifacio O. De Leon on June 22, 1970, upon
the full payment of the total [price] of P1,023.74 and upon execution of the Final Deed of Sale;

d.

After full payment, Bonifacio O. De Leon was issued [TCT] No. 173677 on February 24, 1972;

e.
On January 12, 1974, Bonifacio O. De Leon executed a Deed of Sale in favor of defendants-spouses
Felix Rio Tarrosa and Lita O. De Leon disposing the parcel of land under TCT No. 173677 for valuable consideration
amount of P19,000.00 and subscribed before Atty. Salvador R. Aguinaldo who was commissioned to [notarize]
documents on said date. The parties stipulate that the Deed of Sale is valid and genuine. However, plaintiff Anita
De Leon was not a signatory to the Deed of Sale executed on January 12, 1974;
f.
That plaintiff Anita B. De Leon and the late Bonifacio O. De Leon were married in church rites on May
23, 1977 x x x;
g.

The late Bonifacio O. De Leon died on February 29, 1996 at the UST Hospital, Espaa, Manila;

h.
The said Deed of Sale executed on January 12, 1974 was registered on May 8, 1996 before the
Office of the Register of Deeds of Quezon City and [TCT] No. N-173911 was issued to Lita O. De Leon and Felix Rio
Tarrosa.[5]

The Ruling of the Trial Court


On October 4, 2006, the RTC, on the finding that the lot in question was the conjugal property of Bonifacio and Anita,
rendered judgment in favor of Anita and her children. The dispositive portion of the decision reads:
WHEREFORE, premises considered, judgment is hereby rendered in favor of plaintiffs and against
defendants in the following manner:
(1)
Declaring the Deed of Sale dated January 12, 1974 executed by the late Bonifacio O. De Leon in favor
of defendants-spouses Lita De Leon and Felix Rio Tarrosa void ab initio;
(2)
Directing the Register of Deed of Quezon City to cancel Transfer Certificate of Title No. N-173911 in
the name of Lita O. De Leon, married to Felix Rio Tarrosa and restore Transfer Certificate of Title No. 173667 in
the name of Bonifacio O. De Leon;
(3)

Ordering the defendants-spouses to pay plaintiffs the following sums:

(a)
(b)
(c)
(d)

P25,000.00 as moral damages;


P20,000.00 as exemplary damages;
P50,000.00 as attorneys fees plus appearance fee of P2,500.00 per court appearance;
Costs of this suit.

SO ORDERED.
Aggrieved, the Tarrosas appealed to the CA. As they would submit, the RTC erred:
(1)
(2)
(3)
(4)
(5)
(6)

in finding for the plaintiffs-appellees by declaring that the land subject matter of the case is conjugal
property;
in not declaring the land as the exclusive property of Bonifacio O. De Leon when sold to defendantappellants;
in ruling that defendant-appellants did not adduce any proof that the property was acquired solely by the
efforts of Bonifacio O. De Leon;
in declaring that one-half of the conjugal assets does not vest to Bonifacio O. De Leon because of the
absence of liquidation;
in cancelling TCT No. N-173911 and restored TCT No. [173677] in the name of Bonifacio O. De Leon;
in awarding moral and exemplary damages and attorneys fees to the plaintiffs-appellees. [6]

The Ruling of the Appellate Court


On August 27, 2008, the CA rendered a decision affirmatory of that of the RTC, save for the award of damages, attorneys
fees, and costs of suit which the appellate court ordered deleted. The fallo of the CA decision reads:
WHEREFORE, in view of the foregoing, the assailed decision dated October 4, 2006, of the Regional Trial
Court, Branch 22, Quezon Cityin Civil Case No. Q-04-51595 is hereby AFFIRMED with MODIFICATION, in that the
award of moral and exemplary damages as well as attorneys fees, appearance fee and costs of suit are hereby
DELETED.
SO ORDERED.

Just like the RTC, the CA held that the Tarrosas failed to overthrow the legal presumption that the parcel of land in dispute
was conjugal. The appellate court held further that the cases they cited were inapplicable.
As to the deletion of the grant of moral and exemplary damages, the CA, in gist, held that no evidence was adduced to
justify the award. Based on the same reason, it also deleted the award of attorneys fees and costs of suit.
The Tarrosas moved but was denied reconsideration by the CA in its equally assailed resolution of October 20, 2008.
Hence, they filed this petition.
The Issues
I
Whether the [CA] gravely erred in concluding that the land purchased on installment by Bonifacio O. De Leon before
marriage although some installments were paid during the marriage is conjugal and not his exclusive property.
II

Whether the [CA] gravely erred in ruling that the Lorenzo, et al. vs. Nicolas, et al., and Alvarez vs. Espiritu cases do
not apply in the case at bar because in the latter the land involved is not a friar land unlike in the
former.
III
Whether the [CA] gravely erred in affirming the decision of the trial court a quo which ruled that petitioners did not
adduce any proof that the land was acquired solely by the efforts of Bonifacio O. De Leon.
IV
Whether the court of appeals gravely erred in affirming the decision of the trial court which ruled that one-half (1/2)
of the conjugal assets do not vest to Bonifacio O. De Leon because of the absence of liquidation.

Our Ruling
The petition lacks merit.

The Subject Property is the


Conjugal Property of Bonifacio and Anita
The first three issues thus raised can be summed up to the question of whether or not the subject property is conjugal.
Petitioners assert that, since Bonifacio purchased the lot from PHHC on installment before he married Anita, the land was
Bonifacios exclusive property and not conjugal, even though some installments were paid and the title was issued to Bonifacio
during the marriage. In support of their position, petitioners cite Lorenzo v. Nicolas[7] and Alvarez v. Espiritu.[8]
We disagree.
Article 160 of the 1950 Civil Code, the governing provision in effect at the time Bonifacio and Anita contracted marriage,
provides that all property of the marriage is presumed to belong to the conjugal partnership unless it is proved that it pertains
exclusively to the husband or the wife. For the presumption to arise, it is not, as Tan v. Court of Appeals[9] teaches, even
necessary to prove that the property was acquired with funds of the partnership. Only proof of acquisition during the marriage is
needed to raise the presumption that the property is conjugal. In fact, even when the manner in which the properties were
acquired does not appear, the presumption will still apply, and the properties will still be considered conjugal. [10]
In the case at bar, ownership over what was once a PHHC lot and covered by the PHHC-Bonifacio Conditional Contract
to Sell was only transferred during the marriage of Bonifacio and Anita. It is well settled that a conditional sale is akin, if not
equivalent, to a contract to sell. In both types of contract, the efficacy or obligatory force of the vendors obligation to transfer
title is subordinated to the happening of a future and uncertain event, usually the full payment of the purchase price, so that if
the suspensive condition does not take place, the parties would stand as if the conditional obligation had never existed. [11] In
other words, in a contract to sell ownership is retained by the seller and is not passed to the buyer until full payment of the price,
unlike in a contract of sale where title passes upon delivery of the thing sold. [12]
Such is the situation obtaining in the instant case. The conditional contract to sell executed by and between Bonifacio
and PHHC on July 20, 1965 provided that ownership over and title to the property will vest on Bonifacio only upon execution of
the final deed of sale which, in turn, will be effected upon payment of the full purchase price, to wit:
14.
Titles to the property subject of this contract remains with the CORPORATION and shall pass to, and be
transferred in the name of the APPLICANT only upon the execution of the final Deed of Sale provided for in the next
succeeding paragraph.
15.
Upon the full payment by the APPLICANT of the price of the lot above referred to together with all the
interest due thereon, taxes and other charges, and upon his faithful compliance with all the conditions of this
contract the CORPORATION agrees to execute in favor of the APPLICANT a final deed of sale of the aforesaid
land, and the APPLICANT agrees to accept said deed, as full performance by the CORPORATION of its covenants
and undertakings hereunder.[13] x x x

Evidently, title to the property in question only passed to Bonifacio after he had fully paid the purchase price on June 22,
1970. This full payment, to stress, was made more than two (2) years after his marriage to Anita on April 24, 1968. In net
effect, the property was acquired during the existence of the marriage; as such, ownership to the property is, by law, presumed
to belong to the conjugal partnership.
Such presumption is rebuttable only with strong, clear, categorical, and convincing evidence. [14] There must be clear
evidence of the exclusive ownership of one of the spouses, [15] and the burden of proof rests upon the party asserting it. [16]
Petitioners argument that the disputed lot was Bonifacios exclusive property, since it was registered solely in his name, is
untenable. The mere registration of a property in the name of one spouse does not destroy its conjugal nature. [17] What is
material is the time when the property was acquired.
Thus, the question of whether petitioners were able to adduce proof to overthrow the presumption is a factual issue best
addressed by the trial court. As a matter of long and sound practice, factual determinations of the trial courts, [18] especially when
confirmed by the appellate court, are accorded great weight by the Court and, as rule, will not be disturbed on appeal, except for

the most compelling reasons.[19] Petitioners have not, as they really cannot, rebut the presumptive conjugal nature of the lot in
question. In this regard, the Court notes and quotes with approval the following excerpts from the trial courts disposition:
The defendants, however, did not adduce any proof that the property in question was acquired solely by the
efforts of [Bonifacio]. The established jurisprudence on the matter leads this Court to the conclusion that the
property involved in this dispute is indeed the conjugal property of the deceased [Bonifacio] De Leon.
In fact, defendant even admitted that [Bonifacio] brought into his marriage with plaintiff Anita the said land,
albeit in the concept of a possessor only as it was not yet registered in his name. The property was registered only
in 1972 during the existence of the marriage. However, the absence of evidence on the source of funding has called
for the application of the presumption under Article 160 in favor of the plaintiffs. [20]

The cases petitioners cited are without governing applicability to this case simply because they involved a law specifically
enacted to govern the disposition of and ownership of friar lands. In Lorenzo, the Court held that the pervading legislative intent
of Act No. 1120 is to sell the friar lands acquired by the Government to actual settlers and occupants of the same. [21] The
Court went on further to say in Alvarez that under the Friar Lands Act of 1120, the equitable and beneficial title to the land
passes to the purchaser the moment the first installment is paid and a certificate of sale is issued. [22] Plainly, the said cases are
not applicable here considering that the disputed property is not friar land.
There can be no quibbling that Anitas conformity to the sale of the disputed lot to petitioners was never obtained or at
least not formally expressed in the conveying deed. The parties admitted as much in their Joint Stipulation of Facts with Motion
earlier reproduced. Not lost on the Court of course is the fact that petitioners went to the process of registering the deed after
Bonifacios death in 1996, some 22 years after its execution. In the interim, petitioners could have had workbut did not
towards securing Anitas marital consent to the sale.
It cannot be over-emphasized that the 1950 Civil Code is very explicit on the consequence of the husband alienating or
encumbering any real property of the conjugal partnership without the wifes consent. [23] To a specific point, the sale of a
conjugal piece of land by the husband, as administrator, must, as a rule, be with the wifes consent. Else, the sale is not
valid. So it is that in several cases we ruled that the sale by the husband of property belonging to the conjugal partnership
without the consent of the wife is void ab initio, absent any showing that the latter is incapacitated, under civil interdiction, or like
causes. The nullity, as we have explained, proceeds from the fact that sale is in contravention of the mandatory requirements of
Art. 166 of the Code.[24] Since Art. 166 of the Code requires the consent of the wife before the husband may alienate or
encumber any real property of the conjugal partnership, it follows that the acts or transactions executed against this mandatory
provision are void except when the law itself authorized their validity.[25]
Accordingly, the Deed of Sale executed on January 12, 1974 between Bonifacio and the Tarrosas covering the PHHC lot
is void.
Interest in the Conjugal Partnership Is
Merely Inchoate until Liquidation
As a final consideration, the Court agrees with the CA that the sale of one-half of the conjugal property without liquidation
of the partnership is void. Prior to the liquidation of the conjugal partnership, the interest of each spouse in the conjugal assets
isinchoate, a mere expectancy, which constitutes neither a legal nor an equitable estate, and does not ripen into a title until it
appears that there are assets in the community as a result of the liquidation and settlement. [26] The interest of each spouse is
limited to the net remainder or remanente liquido (haber ganancial) resulting from the liquidation of the affairs of the
partnership after its dissolution.[27] Thus, the right of the husband or wife to one-half of the conjugal assets does not vest until
the dissolution and liquidation of the conjugal partnership, or after dissolution of the marriage, when it is finally determined that,
after settlement of conjugal obligations, there are net assets left which can be divided between the spouses or their respective
heirs.[28]
Therefore, even on the supposition that Bonifacio only sold his portion of the conjugal partnership, the sale is still
theoretically void, for, as previously stated, the right of the husband or the wife to one-half of the conjugal assets does not vest
until the liquidation of the conjugal partnership.
Nevertheless, this Court is mindful of the fact that the Tarrosas paid a valuable consideration in the amount of PhP 19,000
for the property in question. Thus, as a matter of fairness and equity, the share of Bonifacio after the liquidation of the
partnership should be liable to reimburse the amount paid by the Tarrosas. It is a well-settled principle that no person should
unjustly enrich himself at the expense of another.[29]
WHEREFORE, the petition is DENIED. The CA Decision in CA-G.R. CV No. 88571 is AFFIRMED. Costs against
petitioners.

G.R. No. L-19565

January 30, 1968

ESTRELLA DE LA CRUZ, plaintiff-appellee,


vs.
SEVERINO DE LA CRUZ, defendant-appellant.
Estacion and Paltriguera for plaintiff-appellee.
Manuel O. Soriano and Pio G. Villoso for defendant-appellant.
CASTRO, J.:
The plaintiff Estrella de la Cruz filed a complaint on July 22, 1958 with the Court of First Instance of Negros Occidental, alleging
in essence that her husband, the defendant Severino de la Cruz, had not only abandoned her but as well was mismanaging
their conjugal partnership properties, and praying for (1) separation of property, (2) monthly support of P2,500 during the
pendency of the action, and (3) payment of P20,000 as attorney's fees, and costs.
The court a quo forthwith issued an order allowing the plaintiff the amount prayed for as alimony pendente lite, which however,
upon defendant's motion, was reduced to P2,000.
On June 1, 1961 the trial court rendered judgment ordering separation and division of the conjugal assets, and directing the
defendant to pay to the plaintiff the sum of P20,000 as attorney's fees, with legal interest from the date of the original complaint,
that is, from July 22, 1958, until fully paid, plus costs. From this judgment the defendant appealed to the Court of Appeals, which
certified the case to us, "it appearing that the total value of the conjugal assets is over P500,000".
The basic facts are not controverted. The plaintiff and the defendant were married in Bacolod City on February 1, 1938. Six
children were born to them, namely, Zenia (1939), Ronnie (1942), Victoria (1944), Jessie 1945), Bella (1946), and Felipe (1948).
During their coverture they acquired seven parcels of land of the Bacolod Cadastre, all assessed at P45,429, and three parcels
of the Silay Cadastre, all assessed at P43,580. All these parcels are registered in their names. The hacienda in Silay yielded for
the year 1957 a net profit of P3,390.49.
They are also engaged in varied business ventures with fixed assets valued as of December 31, 1956 at P496,006.92, from
which they obtained for that year a net profit of P75,655.78. The net gain of the Philippine Texboard Factory, the principal
business of the spouses, was P90,454.48 for the year 1957. As of December 31, 1959, the total assets of the various
enterprises of the conjugal partnership were valued at P1,021,407.68, not including those of the Top Service Inc., of which firm
the defendant has been the president since its organization in 1959 in Manila with a paid-up capital of P50,000, P10,000 of
which was contributed by him. This corporation was the Beverly Hills Subdivision in Antipolo, Rizal, the Golden Acres
Subdivision and the Green Valley Subdivision in Las Pias, Rizal, and a lot and building located at M. H. del Pilar, Manila
purchased for P285,000, an amount borrowed from the Manufacturer's Bank and Trust Company.
The spouses are indebted to the Philippine National Bank and the Development Bank of the Philippines for loans obtained, to
secure which they mortgaged the Philippine Texboard Factory, the Silay hacienda, their conjugal house, and all their parcels of
land located in Bacolod City.
The essential issues of fact may be gleaned from the nine errors the defendant imputes to the court a quo, namely,
1. In finding that the only visit, from May 15, 1955 to the rendition of the decision, made by the defendant to the conjugal
abode to see his wife was on June 15, 1955;
2. In finding that the letter exh. 3 was written by one Nenita Hernandez and that she and the defendant are living as
husband and wife;
3. In finding that since 1951 the relations between the plaintiff and the defendant were far from cordial, and that it was
from 1948 that the former has been receiving an allowance from the latter;
4. In finding that the defendant has abandoned the plaintiff;
5. In finding that the defendant since 1956 has not discussed with his wife the business activities of the partnership, and
that this silence constituted "abuse of administration of the conjugal partnerships";
6. In declaring that the defendant mortgaged the conjugal assets without the knowledge of the plaintiff and thru false
pretences to which the latter was prey;
7. In allowing the plaintiff, on the one hand, to testify on facts not actually known by her, and, on the other hand, in not
allowing the defendant to establish his special defenses;
8. In ordering separation of the conjugal partnership properties; and
9. In sentencing the defendant to pay to the plaintiff attorney's fees in the amount of P20,000, with interest at the legal
rate.
1wph1.t

Two issues of law as well emerge, requiring resolution petition: (1) Did the separation of the defendant from the plaintiff
constitute abandonment in law that would justify a separation of the conjugal partnership properties? (2) Was the defendant's
failure and/or refusal to inform the plaintiff of the state of their business enterprises such an abuse of his powers of
administration of the conjugal partnership as to warrant a division of the matrimonial assets?
The plaintiff's evidence may be summarized briefly. The defendant started living in Manila in 1955, although he occasionally
returned to Bacolod City, sleeping in his office at the Philippine Texboard Factory in Mandalagan, instead of in the conjugal
home at 2nd Street, Bacolod City. Since 1955 the defendant had not slept in the conjugal dwelling, although in the said year he
paid short visits during which they engaged in brief conversations. After 1955 up to the time of the trial, the defendant had never
visited the conjugal abode, and when he was in Bacolod, she was denied communication with him. He has abandoned her and
their children, to live in Manila with his concubine, Nenita Hernandez. In 1949 she began to suspect the existence of illicit
relations between her husband and Nenita. This suspicion was confirmed in 1951 when she found an unsigned note in a pocket
of one of her husband's polo shirt which was written by Nenita and in which she asked "Bering" to meet her near the church.
She confronted her husband who forthwith tore the note even as he admitted his amorous liaison with Nenita. He then allayed
her fears by vowing to forsake his mistress. Subsequently, in November 1951, she found in the iron safe of her husband a letter,
exh. C, also written by Nenita. In this letter the sender (who signed as "D") apologized for her conduct, and expressed the hope
that the addressee ("Darling") could join her in Baguio as she was alone in the Patria Inn and lonely in "a place for
honeymooners". Immediately after her husband departed for Manila the following morning, the plaintiff enplaned for Baguio,
where she learned that Nenita had actually stayed at the Patria Inn, but had already left for Manila before her arrival. Later she
met her husband in the house of a relative in Manila from whence they proceeded to the Avenue Hotel where she again
confronted him about Nenita. He denied having further relations with this woman.
Celia Baez, testifying for the plaintiff, declared that she was employed as a cook in the home of the spouses from May 15,
1955 to August 15, 1958, and that during the entire period of her employment she saw the defendant in the place only once.
This declaration is contradicted, however, by the plaintiff herself who testified that in 1955 the defendant "used to have a short
visit there," which statement implies more than one visit.

The defendant, for his part, denied having abandoned his wife and children, but admitted that in 1957, or a year before the filing
of the action, he started to live separately from his wife. When he transferred his living quarters to his office in Mandalagan,
Bacolod City, his intention was not, as it never has been, to abandon his wife and children, but only to teach her a lesson as she
was quarrelsome and extremely jealous of every woman. He decided to live apart from his wife temporarily because at home he
could not concentrate on his work as she always quarreled with him, while in Mandalagan he could pass the nights in peace.
Since 1953 he stayed in Manila for some duration of time to manage their expanding business and look for market outlets for
their texboard products. Even the plaintiff admitted in both her original and amended complaints that "sometime in 1953,
because of the expanding business of the herein parties, the defendant established an office in the City of Manila, wherein
some of the goods, effects and merchandise manufactured or produced in the business enterprises of the parties were sold or
disposed of". From the time he started living separately in Mandalagan up to the filing of the complaint, the plaintiff herself
furnished him food and took care of his laundry. This latter declaration was not rebutted by the plaintiff.
The defendant, with vehemence, denied that he has abandoned his wife and family, averring that he has never failed, even for a
single month, to give them financial support, as witnessed by the plaintiff's admission in her original and amended complaints as
well as in open court that during the entire period of their estrangement, he was giving her around P500 a month for support. In
point of fact, his wife and children continued to draw allowances from his office of a total ranging from P1,200 to P1,500 a
month. He financed the education of their children, two of whom were studying in Manila at the time of the trial and were not
living with the plaintiff. While in Bacolod City, he never failed to visit his family, particularly the children. His wife was always in
bad need of money because she played mahjong, an accusation which she did not traverse, explaining that she
played mahjong to entertain herself and forget the infidelities of her husband.
Marcos V. Ganaban, the manager of the Philippine Texboard Factory, corroborated the testimony of the defendant on the matter
of the support the latter gave to his family, by declaring in court that since the start of his employment in 1950 as assistant
general manager, the plaintiff has been drawing an allowance of P1,000 to P1,500 monthly, which amount was given personally
by the defendant or, in his absence, by the witness himself.
The defendant denied that he ever maintained a mistress in Manila. He came to know Nenita Hernandez when she was barely
12 years old, but had lost track of her thereafter. His constant presence in Manila was required by the pressing demands of an
expanding business. He denied having destroyed the alleged note which the plaintiff claimed to have come from Nenita, nor
having seen, previous to the trial, the letter exh. C. The allegation of his wife that he had a concubine is based on mere
suspicion. He had always been faithful to his wife, and not for a single instance had he been caught or surprised by her with
another woman.
On the matter of the alleged abuse by the defendant of his powers of administration of the conjugal partnership, the plaintiff
declared that the defendant refused and failed to inform her of the progress of their various business concerns. Although she did
not allege, much less prove, that her husband had dissipated the conjugal properties, she averred nevertheless that her
husband might squander and dispose of the conjugal assets in favor of his concubine. Hence, the urgency of separation of
property.
The defendant's answer to the charge of mismanagement is that he has applied his industry, channeled his ingenuity, and
devoted his time, to the management, maintenance and expansion of their business concerns, even as his wife threw money
away at the mahjong tables. Tangible proof of his endeavors is that from a single cargo truck which he himself drove at the time
of their marriage, he had built up one business after another, the Speedway Trucking Service, the Negros Shipping Service, the
Bacolod Press, the Philippine Texboard Factory, and miscellaneous other business enterprises worth over a million pesos; that
all that the spouses now own have been acquired through his diligence, intelligence and industry; that he has steadily expanded
the income and assets of said business enterprises from year to year, contrary to the allegations of the complainant, as proved
by his balance sheet and profit and loss statements for the year 1958 and 1959 (exhibits 1 and 2); and that out of the income of
their enterprises he had purchased additional equipment and machineries and has partially paid their indebtedness to the
Philippine National Bank and the Development Bank of the Philippines.
It will be noted that the plaintiff does not ask for legal separation. The evidence presented by her to prove concubinage on the
part of the defendant, while pertinent and material in the determination of the merits of a petition for legal separation, must in
this case be regarded merely as an attempt to bolster her claim that the defendant had abandoned her, which abandonment, if it
constitutes abandonment in law, would justify separation of the conjugal assets under the applicable provisions of article 178 of
the new Civil Code which read: "The separation in fact between husband and wife without judicial approval, shall not affect the
conjugal partnership, except that . . . if the husband has abandoned the wife without just cause for at least one year, she may
petition the court for a receivership, or administration by her of the conjugal partnership property, or separation of property". In
addition to abandonment as a ground, the plaintiff also invokes article 167 of the new Civil Code in support of her prayer for
division of the matrimonial assets. This article provides that "In case of abuse of powers of administration of the conjugal
partnership property by the husband, the courts, on the petition of the wife, may provide for a receivership, or administration by
the wife, or separation of property". It behooves us, therefore, to inquire, in the case at bar, whether there has been
abandonment, in the legal sense, by the defendant of the plaintiff, and/or whether the defendant has abused his powers of
administration of the conjugal partnership property, so as to justify the plaintiff's plea for separation of property.
We have made a searching scrutiny of the record, and it is our considered view that the defendant is not guilty of abandonment
of his wife, nor of such abuse of his powers of administration of the conjugal partnership, as to warrant division of the conjugal
assets.
The extraordinary remedies afforded to the wife by article 178 when she has been abandoned by the husband for at least one
year are the same as those granted to her by article 167 in case of abuse of the powers of administration by the husband. To
entitle her to any of these remedies, under article 178, there must be real abandonment, and not mere separation. 1 The
abandonment must not only be physical estrangement but also amount to financial and moral desertion.
Although an all-embracing definition of the term "abandonment " is yet to be spelled out in explicit words, we nevertheless can
determine its meaning from the context of the Law as well as from its ordinary usage. The concept of abandonment in article
178 may be established in relation to the alternative remedies granted to the wife when she has been abandoned by the
husband, namely, receivership, administration by her, or separation of property, all of which are designed to protect the conjugal
assets from waste and dissipation rendered imminent by the husband's continued absence from the conjugal abode, and to
assure the wife of a ready and steady source of support. Therefore, physical separation alone is not the full meaning of the term
"abandonment", if the husband, despite his voluntary departure from the society of his spouse, neither neglects the
management of the conjugal partnership nor ceases to give support to his wife.
The word "abandon", in its ordinary sense, means to forsake entirely; to forsake or renounce utterly. 2 The dictionaries trace this
word to the root idea of "putting under a bar". The emphasis is on the finality and the publicity with which some thing or body is
thus put in the control of another, and hence the meaning of giving up absolutely, with intent never again to resume or claim
one's rights or interests. 3 When referring to desertion of a wife by a husband, the word has been defined as "the act of a
husband in voluntarily leaving his wife with intention to forsake her entirely, never to return to her, and never to resume his
marital duties towards her, or to claim his marital rights; such neglect as either leaves the wife destitute of the common
necessaries of life, or would leave her destitute but for the charity of others." 4 The word "abandonment", when referring to the
act of one consort of leaving the other, is "the act of the husband or the wife who leaves his or her consort wilfully, and with an
intention of causing per perpetual separation." 5 Giving to the word "abandoned", as used in article 178, the meaning drawn from
the definitions above reproduced, it seems rather clear that to constitute abandonment of the wife by the husband, there must
be absolute cessation of marital relations and duties and rights, with the intention of perpetual separation.

Coming back to the case at bar, we believe that the defendant did not intend to leave his wife and children permanently. The
record conclusively shows that he continued to give support to his family despite his absence from the conjugal home. This fact
is admitted by the complainant, although she minimized the amount of support given, saying that it was only P500 monthly.
There is good reason to believe, however, that she and the children received more than this amount, as the defendant's claim
that his wife and children continued to draw from his office more than P500 monthly was substantially corroborated by Marcos
Ganaban, whose declarations were not rebutted by the plaintiff. And then there is at all no showing that the plaintiff and the
children were living in want. On the contrary, the plaintiff admitted, albeit reluctantly, that she frequently played mahjong, from
which we can infer that she had money; to spare.
The fact that the defendant never ceased to give support to his wife and children negatives any intent on his part not to return to
the conjugal abode and resume his marital duties and rights. In People v. Schelske, 6 it was held that where a husband, after
leaving his wife, continued to make small contributions at intervals to her support and that of their minor child, he was not guilty
of their "abandonment", which is an act of separation with intent that it shall be perpetual, since contributing to their support
negatived such intent. In re Hoss' Estate, supra, it was ruled that a father did not abandon his family where the evidence
disclosed that he almost always did give his wife part of his earnings during the period of their separation and that he gradually
paid some old rental and grocery bills.
With respect to the allegation that the defendant maintained a concubine, we believe, contrary to the findings of the court a quo,
that the evidence on record fails to preponderate in favor of the plaintiff's thesis. The proof that Nenita Hernandez was the
concubine of the defendant and that they were living as husband and wife in Manila, is altogether too indefinite. Aside from the
uncorroborated statement of the plaintiff that she knew that Nenita Hernandez was her husband's concubine, without
demonstrating by credible evidence the existence of illicit relations between Nenita and the defendant, the only evidence on
record offered to link the defendant to his alleged mistress is exh. C. The plaintiff however failed to connect authorship of the
said letter with Nenita, on the face whereof the sender merely signed as "D" and the addressee was one unidentified "Darling".
The plaintiff's testimony on cross-examination, hereunder quoted, underscores such failure:
Q. You personally never received any letter from Nenita?
A. No.
Q. Neither have you received on any time until today from 1949 from Nenita?
A. No.
Q. Neither have you written to her any letter yourself until now?
A. Why should I write a letter to her.
Q. In that case, Mrs. De la Cruz, you are not familiar with the handwriting of Nenita. Is that right?
A. I can say that Nenita writes very well.
Q. I am not asking you whether she writes very well or not but, my question is this: In view of the fact that you have never
received a letter from Nenita, you have ot sent any letter to her, you are not familiar with her handwriting?
A. Yes.
Q. You have not seen her writing anybody?
A. Yes.
Anent the allegation that the defendant had mismanaged the conjugal partnership property, the record presents a different
picture. There is absolutely no evidence to show that he has squandered the conjugal assets. Upon the contrary, he proved that
through his industry and zeal, the conjugal assets at the time of the trial had increased to a value of over a million pesos.
The lower court likewise erred in holding that mere refusal or failure of the husband as administrator of the conjugal partnership
to inform the wife of the progress of the family businesses constitutes abuse of administration. For "abuse" to exist, it is not
enough that the husband perform an act or acts prejudicial to the wife. Nor is it sufficient that he commits acts injurious to the
partnership, for these may be the result of mere inefficient or negligent administration. Abuse connotes willful and utter
disregard of the interests of the partnership, evidenced by a repetition of deliberate acts and/or omissions prejudicial to the
latter. 7
If there is only physical separation between the spouses (and nothing more), engendered by the husband's leaving the conjugal
abode, but the husband continues to manage the conjugal properties with the same zeal, industry, and efficiency as he did prior
to the separation, and religiously gives support to his wife and children, as in the case at bar, we are not disposed to grant the
wife's petition for separation of property. This decision may appear to condone the husband's separation from his wife; however,
the remedies granted to the wife by articles 167 and 178 are not to be construed as condonation of the husband's act but are
designed to protect the conjugal partnership from waste and shield the wife from want. Therefore, a denial of the wife's prayer
does not imply a condonation of the husband's act but merely points up the insufficiency or absence of a cause of action.
1wph1.t

Courts must need exercise judicial restraint and reasoned hesitance in ordering a separation of conjugal properties because the
basic policy of the law is homiletic, to promote healthy family life and to preserve the union of the spouses, in person, in spirit
and in property.
Consistent with its policy of discouraging a regime of separation as not in harmony with the unity of the family and the
mutual affection and help expected of the spouses, the Civil Code (both old and new) requires that separation of property
shall not prevail unless expressly stipulated in marriage settlements before the union is solemnized or by formal judicial
decree during the existence of the marriage (Article 190, new Civil Code, Article 1432, old Civil Code): and in the latter
case, it may only be ordered by the court for causes specified in Article 191 of the new Civil Code. 8
Furthermore, a judgment ordering the division of conjugal assets where there has been no real abandonment, the separation
not being wanton and absolute, may altogether slam shut the door for possible reconciliation. The estranged spouses may drift
irreversibly further apart; the already broken family solidarity may be irretrievably shattered; and any flickering hope for a new
life together may be completely and finally extinguished.
The monthly alimony in the sum of P2,000 which was allowed to the wife in 1958, long before the devaluation of the Philippine
peso in 1962, should be increased to P3,000.
On the matter of attorney's fees, it is our view that because the defendant, by leaving the conjugal abode, has given cause for
the plaintiff to seek redress in the courts, and ask for adequate support, an award of attorney's fees to the plaintiff must be
made. Ample authority for such award is found in paragraphs 6 and 11 of article 2208 of the new Civil Code which empower
courts to grant counsel's fees "in actions for legal support" and in cases "where the court deems it just and equitable that
attorney's fees . . . should be recovered." However, an award of P10,000, in our opinion, is, under the environmental
circumstances, sufficient.

This Court would be remiss if it did not, firstly, remind the plaintiff and the defendant that the law enjoins husband and wife to
live together, and, secondly, exhort them to avail of mutually, earnestly and steadfastly all opportunities for reconciliation to
the end that their marital differences may be happily resolved, and conjugal harmony may return and, on the basis of mutual
respect and understanding, endure.
ACCORDINGLY, the judgment a quo, insofar as it decrees separation of the conjugal properties, is reversed and set aside.
Conformably to our observations, however, the defendant is ordered to pay to the plaintiff, in the concept of support, the amount
of P3,000 per month, until he shall have rejoined her in the conjugal home, which amount may, in the meantime, be reduced or
increased in the discretion of the court a quo as circumstances warrant. The award of attorney's fees to the plaintiff is reduced
to P10,000, without interest. No pronouncement as to costs.

G.R. No. 82606 December 18, 1992


PRIMA PARTOSA-JO, petitioner,
vs.
THE HONORABLE COURT OF APPEALS and HO HANG (with aliases JOSE JO and CONSING), respondents.

CRUZ, J.:
The herein private respondent, Jose Jo, admits to having cohabited with three women and fathered fifteen children. The first of
these women, the herein petitioner, claims to be his legal wife whom he begot a daughter, Monina Jo. The other women and
their respective offspring are not parties of these case.
In 1980, the petitioner filed a complaint against Jo for judicial separation of conjugal property, docketed as Civil Case No. 51, in
addition to an earlier action for support, also against him and docketed as Civil Case No. 36, in the Regional Trial Court of
Negros Oriental, Branch 35.
The two cases were consolidated and tried jointly. On November 29, 1983, Judge German G. Lee, Jr. rendered an extensive
decision, the dispositive portion of which read:
WHEREFORE, in view of all the foregoing arguments and considerations, this court hereby holds that the plaintiff
Prima Partosa was legally married to Jose Jo alias Ho Hang, alias Consing, and, therefore, is entitled to support as
the lawfully wedded wife and the defendant is hereby ordered to give a monthly support of P500.00 to the plaintiff
Prima Partosa, to be paid on or before the 5th day of every month, and to give to the plaintiff the amount of
P40,000.00 for the construction of the house in Zamboanguita, Negros Oriental where she may live separately from
the defendant being entitled under the law to separate maintenance being the innocent spouse and to pay the
amount of P19,200.00 to the plaintiff by way of support in arrears and to pay the plaintiff the amount of P3,000.00 in
the concept of attorney's fees.
As will be noticed, there was a definite disposition of the complaint for support but none of the complaint for judicial separation
of conjugal property.
Jo elevated the decision to the Court of Appeals, which affirmed the ruling of the trial court in the complaint for support.

The
complaint for judicial separation of conjugal property was dismissed for lack of a cause of action and on the ground that separation by agreement was not covered by Article 178 of
the Civil Code.

When their motions for reconsideration were denied, both parties came to this Court for relief. The private respondent's petition
for review on certiorari was dismissed for tardiness in our resolution dated February 17, 1988, where we also affirmed the
legality of the marriage between Jose and Prima and the obligation of the former to support her and her daughter.
This petition deals only with the complaint for judicial separation of conjugal property.
It is here submitted that the Court of Appeals erred in holding that: a) the judicial separation of conjugal property sought was not
allowed under Articles 175, 178 and 191 of the Civil Code; and b) no such separation was decreed by the trial court in the
dispositive portion of its decision.
The private respondent contends that the decision of the trial court can longer be reviewed at this time because it has a long
since become final and executory. As the decretal portion clearly made no disposition of Civil Case No. 51, that case should be
considered impliedly dismissed. The petitioner should have called the attention of the trial court to the omission so that the
proper rectification could be made on time. Not having done so, she is now concluded by the said decision, which can no longer
be corrected at this late hour.
We deal first with the second ground.
While admitting that no mention was made of Civil Case No. 51 in the dispositive portion of the decision of the trial court, the
petitioner argues that a disposition of the case was nonetheless made in the penultimate paragraph of the decision reading as
follows:
It is, therefore, hereby ordered that all properties in question are considered properties of Jose Jo, the defendant in
this case, subject to separation of property under Article 178, third paragraph of the Civil Code, which is subject of
separate proceedings as enunciated herein.
The petitioner says she believed this to be disposition enough and so did not feel it was necessary for her to appeal, particularly
since the order embodied in that paragraph was in her favor. It was only when the respondent court observed that there was no
dispositive portion regarding that case and so ordered its dismissal that she found it necessary to come to this Court for relief.
The petitioner has a point.
The dispositive portion of the decision in question was incomplete insofar as it carried no ruling on the complaint for judicial
separation of conjugal property although it was extensively discussed in the body of the decision. The drafting of the decision
was indeed not exactly careful. The petitioner's counsel, noting this, should have taken immediate steps for the rectification for
the omission so that the ruling expressed in the text of the decision could have been embodied in the decretal portion. Such
alertness could have avoided this litigation on a purely technical issue.
Nevertheless, the technicality invoked in this case should not be allowed to prevail over considerations of substantive justive.
After all, the technical defect is not insuperable. We have said time and again that where there is an ambiguity caused by an
omission or a mistake in the dispositive portion of the decision, this Court may clarify such an ambiguity by an amendment even
after the judgment have become final. 2 In doing so, the Court may resort to the pleading filed by the parties and the findings of fact and the conclusions of law
expressed in the text or body of the decision.

The trial court made definite findings on the complaint for judicial separation of conjugal property, holding that the petitioner and
the private respondent were legally married and that the properties mentioned by the petitioner were acquired by Jo during their
marriage although they were registered in the name of the apparent dummy.
There is no question therefore that the penultimate paragraph of the decision of the trial court was a ruling based upon such
findings and so should have been embodied in the dispositive portion. The respondent court should have made the necessary
modification instead of dismissing Civil Case No. 51 and thus upholding mere form over substance.
In the interest of substantive justice, and to expedite these proceedings, we hereby make such modification.
And now to the merits of Civil Case No. 51.
The Court of Appeals dismissed the complaint on the ground that the separation of the parties was due to their agreement and
not because of abondonment. The respondent court relied mainly on the testimony of the petitioner, who declared under oath
that she left Dumaguete City, where she and Jo were living together "because that was our agreement." It held that a agreement
to live separately without just cause was void under Article 221 of the Civil Code and could not sustain any claim of

abandonment by the aggrieved spouse. Its conclusion was that the only remedy availabe to the petitioner was legal separation
under Article 175 of the Civil Code, 4 by virtue of which the conjugal partnership of property would be terminated.
The petitioner contends that the respondent court has misinterpreted Articles 175, 178 and 191 of the Civil Code. She submits
that the agreement between her and the private respondent was for her to temporarily live with her parents during the initial
period of her pregnancy and for him to visit and support her. They never agreed to separate permanently. And even if they did,
this arrangement was repudiated and ended in 1942, when she returned to him at Dumaguete City and he refused to accept
her.
The petitioner invokes Article 178 (3) of the Civil Code, which reads:
Art. 178. The separation in fact between husband and wife without judicial approval, shall not affect the conjugal
partnership, except that:
xxx xxx xxx
(3) If the husband has abandoned the wife without just cause for at least one year, she may petition the court for a
receivership, or administration by her of the conjugal partnership property or separation of property.
The above-quoted provision has been superseded by Article 128 of the Family Code, which states:
Art. 128. If a spouse without just cause abandons the other or fails to comply with his or her obligations to the family,
the aggrieved spouse may petition the court for receivership, for judicial separation of property, of for authority to be
the sole administrator of the conjugal partnership property, subject to such precautionary conditions as the court
may impose.
The obligations to the family mentioned in the preceding paragraph refer to martial, parental or property relations.
A spouse is deemed to have abondoned the other when he or she has left the conjugal dwelling without any
intention of returning. The spouse who has left the conjugal dwelling for a period of three months or has failed within
the same period to give any information as to his or her whereabouts shall be prima facie presumed to have no
intention of returning to the conjugal dwelling.
Under the this provision, the aggrieved spouse may petition for judicial separation on either of these grounds:
1. Abondonment by a spouse of the other without just cause; and
2. Failure of one spouse to comply with his or her obligations to the family without just cause, even if she said
spouse does not leave the other spouse.
Abandonment implies a departure by one spouse with the avowed intent never to return, followed by prolonged absence without
just cause, and without in the meantime providing in the least for one's family although able to do so. 5 There must be absolute cessation
of marital relations, duties and rights, with the intention of perpetual separation. 6This idea is clearly expressed in the above-quoted provision, which states that "a spouse is
deemed to have abandoned the other when he or she has left the conjugal dwelling without any intention of returning."

The record shows that as early as 1942, the private respondent had already rejected the petitioner, whom he denied admission
to their conjugal home in Dumaguete City when she returned from Zamboanguita. The fact that she was not accepted by Jo
demonstrates all too clearly that he had no intention of resuming their conjugal relationship. Moreover, beginning 1968 until the
determination by this Court of the action for support in 1988, the private respondent refused to give financial support to the
petitioner. The physical separation of the parties, coupled with the refusal by the private respondent to give support to the
petitioner, sufficed to constitute abandonment as a ground for the judicial separation of their conjugal property.
In addition, the petitioner may also invoke the second ground allowed by Article 128, for the fact is that he has failed without just
cause to comply with his obligations to the family as husband or parent. Apart form refusing to admit his lawful wife to their
conjugal home in Dumaguete City, Jo has freely admitted to cohabiting with other women and siring many children by them. It
was his refusal to provide for the petitioner and their daughter that prompted her to file the actions against him for support and
later for separation of the conjugal property, in which actions, significantly, he even denied being married to her. The private
respondent has not established any just cause for his refusal to comply with his obligations to his wife as dutiful husband.
Their separation thus falls also squarely under Article 135 of the Family Code, providing as follows:
Art. 135. Any of the following shall be considered sufficient cause for judicial separation of property:
xxx xxx xxx
(6) That at the time of the petition, the spouse have been separated in fact for at least one year and reconciliation is
highly improbable.
The amendments introduced in the Family Code are applicable to the case before us although they became effective only on
August 3, 1988. As we held in Ramirez v. Court of Appeals: 7
The greater weight of authority is inclined to the view that an appellate court, in reviewing a judgment on appeal, will
dispose of a question according to the law prevailing at the term of such disposition, and not according to the law
prevailing at the time of rendition of the appealed judgement. The court will therefore reverse a judgement which
was correct at the time it was originally rendered where, by statute, there has been an intermediate change in the
law which renders such judgement erroneous at the time the case was finally disposed of on appeal.
The order of judicial separation of the properties in question is based on the finding of both the trial and respondent courts that
the private respondent is indeed their real owner. It is these properties that should now be divided between him and the
petitioner, on the assumption that they were acquired during coverture and so belong to the spouses half and half. As the
private respondent is a Chinese citizen, the division must include such properties properly belonging to the conjugal partnership
as may have been registered in the name of other persons in violation of the Anti-Dummy Law.
The past has caught up with the private respondent. After his extramarital flings and a succession of illegitimate children, he
must now make an accounting to his lawful wife of the properties he denied her despite his promise to their of his eternal love
and care.
WHEREFORE, the petition is GRANTED and the assailed decision of the respondent court is MODIFIED. Civil Case No. 51 is
hereby decided in favor the plaintiff, the petitioner herein, and the conjugal property of the petitioner and the private respondent
is hereby ordered divided between them, share and share alike. This division shall be implemented by the trial court after
determination of all the properties pertaining to the said conjugal partnership, including those that may have been illegally
registered in the name of the persons.

G.R. No. L-61464 May 28, 1988


BA FINANCE CORPORATION, petitioner,
vs.
THE HONORABLE COURT OF APPEALS, AUGUSTO YULO, LILY YULO (doing business under the name and style of A
& L INDUSTRIES), respondents.

GUTIERREZ, JR., J.:


This is a petition for review seeking to set aside the decision of the Court of Appeals which affirmed the decision of the then
Court of First Instance of Manila, dismissing the complaint instituted by the petitioner and ordering it to pay damages on the
basis of the private respondent's counterclaim.
On July 1, 1975, private respondent Augusto Yulo secured a loan from the petitioner in the amount of P591,003.59 as
evidenced by a promissory note he signed in his own behalf and as representative of the A & L Industries. Respondent Yulo
presented an alleged special power of attorney executed by his wife, respondent Lily Yulo, who manages A & L Industries and
under whose name the said business is registered, purportedly authorizing Augusto Yulo to procure the loan and sign the
promissory note. About two months prior to the loan, however, Augusto Yulo had already left Lily Yulo and their children and had
abandoned their conjugal home. When the obligation became due and demandable, Augusto Yulo failed to pay the same.
On October 7, 1975, the petitioner filed its amended complaint against the spouses Augusto and Lily Yulo on the basis of the
promissory note. It also prayed for the issuance of a writ of attatchment alleging that the said spouses were guilty of fraud in
contracting the debt upon which the action was brought and that the fraud consisted of the spouses' inducing the petitioner to
enter into a contract with them by executing a Deed of Assignment in favor of the petitioner, assigning all their rights, titles and
interests over a construction contract executed by and between the spouses and A. Soriano Corporation on June 19, 1974 for a
consideration of P615,732.50 when, in truth, the spouses did not have any intention of remitting the proceeds of the said
construction contract to the petitioner because despite the provisions in the Deed of Assignment that the spouses shall, without
compensation or costs, collect and receive in trust for the petitioner all payments made upon the construction contract and shall
remit to the petitioner all collections therefrom, the said spouses failed and refuse to remit the collections and instead,
misappropriated the proceeds for their own use and benefit, without the knowledge or consent of the petitioner.
The trial court issued the writ of attachment prayed for thereby enabling the petitioner to attach the properties of A & L
Industries. Apparently not contented with the order, the petitioner filed another motion for the examination of attachment debtor,
alleging that the properties attached by the sheriff were not sufficient to secure the satisfaction of any judgment that may be
recovered by it in the case. This was likewise granted by the court.
Private respondent Lily Yulo filed her answer with counterclaim, alleging that although Augusta Yulo and she are husband and
wife, the former had abandoned her and their children five (5) months before the filing of the complaint; that they were already
separated when the promissory note was executed; that her signature in the special power of attorney was forged because she
had never authorized Augusto Yulo in any capacity to transact any business for and in behalf of A & L Industries, which is owned
by her as a single proprietor, that she never got a single centavo from the proceeds of the loan mentioned in the promissory
note; and that as a result of the illegal attachment of her properties, which constituted the assets of the A & L Industries, the
latter closed its business and was taken over by the new owner.
After hearing, the trial court rendered judgment dismissing the petitioner's complaint against the private respondent Lily Yulo
and A & L Industries and ordering the petitioner to pay the respondent Lily Yulo P660,000.00 as actual damages; P500,000.00
as unrealized profits; P300,000.00 as exemplary damages; P30,000.00 as and for attorney's fees; and to pay the costs.
The petitioner appealed. The Court of Appeals affirmed the trial court's decision except for the exemplary damages which it
reduced from P300,000.00 to P150,000.00 and the attorney's fees which were reduced from P30,000.00 to P20,000.00.
In resolving the question of whether or not the trial court erred in holding that the signature of respondent Lily Yulo in the special
power of attorney was forged, the Court of Appeals said:
The crucial issue to be determined is whether or not the signatures of the appellee Lily Yulo in Exhibits B and B-1
are forged. Atty. Crispin Ordoa, the Notary Public, admitted in open court that the parties in the subject documents
did not sign their signatures in his presence. The same were already signed by the supposed parties and their
supposed witnesses at the time they were brought to him for ratification. We quote from the records the pertinent
testimony of Atty. Ordoa, thus:
Q. This document marked as Exhibit B-1, when this was presented to you by that common friend, June
Enriquez, it was already typewritten, it was already accomplished, all typewritten.?
A. Yes, sir.
Q And the parties had already affixed their signatures in this document?
A. Yes, sir.
Q. In this document marked as Exhibit B although it appears here that this is an acknowledgment, you
have not stated here that the principal actually acknowledged this document to be her voluntary act and
deed?
A This in one of those things that escaped my attention. Actually I have not gone over the second page. I
believed it was in order I signed it. (TSN pp. 13-14, Hearing of Nov. 26, 1976).
The glaring admission by the Notary Public that he failed to state in the acknowledgment portion of Exhibit B-1 that
the appellee Lily Yulo acknowledged the said document to be her own voluntary act and deed, is a very strong and
commanding circumstance to show that she did not appear personally before the said Notary Public and did not sign
the document.
Additionally, the Notary Public admitted that, while June Enriquez is admittedly a mutual friend of his and the
defendant Augusta Yulo, and who is also an instrumental witness in said Exhibit B-1., he could not recognize or tell
which of the two signatures appearing therein, was the signature of this June Enriquez.
Furthermore, as the issue is one of credibility of a witness, the findings and conclusions of the trial court before
whom said witness, Atty. Crispin Ordoa, the Notary Public before whom the questioned document was supposedly
ratified and acknowledged, deserve great respect and are seldom disturbed on appeal by appellate tribunals, since it
is in the best and peculiar advantage of determining and observing the conduct, demeanor and deportment of a
particular witness while he is testifying in court, an opportunity not enjoyed by the appellate courts who merely have
to rely on the recorded proceedings which transpired in the court below, and the records are bare of any
circumstance of weight, which the trial court had overlooked and which if duly considered, may radically affect the
outcome of the case.

On the other hand, the appellee Lily Yulo, to back up her claim of forgery of her signature in Exhibit B-1, presented
in court a handwriting expert witness in the person of Police Captain Yakal Giron of the Integrated National Police
Training Command, and who is also a Document Examiner of the same Command's Crime Laboratory at Fort
Bonifacio, Metro Manila. His experience as an examiner of questioned and disputed documents, in our mind, is quite
impressive. To qualify him as a handwriting expert, he declared that he underwent extensive and actual studies and
examination of disputed or questioned document, both at the National Bureau of Investigation Academy and
National Bureau of Investigation Questioned Document Laboratory, respectively, from July 1964, up to his
appointment as Document Examiner in June, 1975, and, to further his experience along this line, he attended the
297th Annual Conference of the American Society of Questioned Docurnent Examiners held at Seattle, Washington,
in August 1971, as a representative of the Philippines, and likewise conducted an observation of the present and
modern trends of crime laboratories in the West Coast, U.S.A., in 1971; that he likewise had conducted actual tests
and examination of about 100,000 documents, as requested by the different courts, administrative, and
governmental agencies of the Government, substantial portions of which relate to actual court cases.
In concluding that the signatures of the appellee Lily Yulo, in the disputed document in question (Exh. B-1), were all
forgeries, and not her genuine signature, the expert witness categorically recited and specified in open court what he
observed to be about twelve (12) glaring and material significant differences, in his comparison of the signatures
appearing in the genuine specimen signatures of the said appellee and with those appearing in the questioned
document (Exhibit B-1). Indeed, we have likewise seen the supposed notable differences, found in the standard or
genuine signatures of the appellee which were lifted and obtained in the official files of the government, such as the
Bureau of Internal Revenue on her income tax returns, as compared to the pretended signature of the appellee
appearing in Exhibits B, B-1. It is also noteworthy to mention that the appellant did not even bother to conduct a
cross-examination of the handwriting expert witness, Capt. Giron, neither did the appellant present another
handwriting expert, at least to counter-act or balance the appellee's handwriting expert.
Prescinding from the foregoing facts, we subscribe fully to the lower court's observations that the signatures of the
appellee Lily Yulo in the questioned document (Exh. B-1) were forged. Hence, we find no factual basis to disagree.
(pp. 28-30, Rollo)
As to the petitioner's contention that even if the signature of Lily Yulo was forged or even if the attached properties were her
exclusive property, the same can be made answerable to the obligation because the said properties form part of the conjugal
partnership of the spouses Yulo, the appellate court held that these contentions are without merit because there is strong
preponderant evidence to show that A & L Industries belongs exclusively to respondent Lily Yulo, namely: a) The Certificate of
Registration of A & L Industries, issued by the Bureau of Commerce, showing that said business is a single proprietorship, and
that the registered owner thereof is only Lily Yulo; b) The Mayor's Permit issued in favor of A & L Industries, by the Caloocan
City Mayor's Office showing compliance by said single proprietorship company with the City Ordinance governing business
establishments; and c) The Special Power of Attorney itself, assuming but without admitting its due execution, is tangible proof
that Augusto Yulo has no interest whatsoever in the A & L Industries, otherwise, there would have been no necessity for the
Special Power of Attorney if he is a part owner of said single proprietorship.
With regard to the award of damages, the Court of Appeals affirmed the findings of the trial court that there was bad faith on the
part of the petitioner as to entitle the private respondent to damages as shown not only by the fact that the petitioner did not
present the Deed of Assignment or the construction agreement or any evidence whatsoever to support its claim of fraud on the
part of the private respondent and to justify the issuance of a preliminary attachment, but also by the following findings:
Continuing and elaborating further on the appellant's mala fide actuations in securing the writ of attachment, the
lower court stated as follows:
Plaintiff not satisfied with the instant case where an order for attachment has already been issued and
enforced, on the strength of the same Promissory Note (Exhibit"A"), utilizing the Deed of Chattel
Mortgage (Exhibit "4"), filed a foreclosure proceedings before the Office of the Sheriff of Caloocan
(Exhibit"6") foreclosing the remaining properties found inside the premises formerly occupied by the A &
L Industries. A minute examination of Exhibit "4" will show that the contracting parties thereto, as
appearing in par. 1 thereof, are Augusto Yulo, doing business under the style of A & L Industries (should
be A & L Glass Industries Corporation), as mortgagor and BA Finance Corporation as mortgagee, thus
the enforcement of the Chattel Mortgage against the property of A & L Industries exclusively owned by
Lily T. Yulo appears to be without any factual or legal basis whatsoever. The chattel mortgage, Exhibit "4"
and the Promissory Note, Exhibit A, are based on one and the same obligation. Plaintiff tried to enforce
as it did enforce its claim into two different modes a single obligation.
Aware that defendant Lily Yulo, filed a Motion to Suspend Proceedings by virtue of a complaint she filed
with the Court of First Instance of Caloocan, seeking annulment of the Promissory Note, the very basis
of the plaintiff in filing this complaint, immediately after the day it filed a Motion for the Issuance of an
Alias Writ of Preliminary Attachment . . .Yet, inspite of the knowledge and the filing of this Motion to
Suspend Proceedings, the Plaintiff still filed a Motion for the Issuance of a Writ of Attachment dated
February 6, 1976 before this court. To add insult to injury, plaintiff even filed a Motion for Examination of
the Attachment Debtor, although aware that Lily Yulo had already denied participation in the execution of
Exhibits "A" and "B". These incidents and actions taken by plaintiff, to the thinking of the court, are
sufficient to prove and establish the element of bad faith and malice on the part of plaintiff which may
warrant the award of damages in favor of defendant Lily Yulo. (Ibid., pp. 102-103).
<re||an1w>

Indeed, the existence of evident bad faith on the appellant's part in proceeding against the appellee Lily
Yulo in the present case, may likewise be distressed on the fact that its officer Mr. Abraham Co, did not
even bother to demand the production of at least the duplicate original of the Special Power of Attorney
(Exhibit B) and merely contended himself with a mere xerox copy thereof, neither did he require a more
specific authority from the A & L Industries to contract the loan in question, since from the very content
and recitals of the disputed document, no authority, express or implied, has been delegated or granted to
August Yulo to contract a loan, especially with the appellant. (pp. 33-34, Rollo)
Concerning the actual damages, the appellate court ruled that the petitioner should have presented evidence to disprove or
rebut the private respondent's claim but it remained quiet and chose not to disturb the testimony and the evidence presented by
the private respondent to prove her claim.
In this petition for certiorari, the petitioner raises three issues. The first issue deals with the appellate court's affirmance of the
trial court's findings that the signature of the private respondent on the Special Power of Attorney was forged. According to the
petitioner, the Court of Appeals disregarded the direct mandate of Section 23, Rule 132 of the Rules of Court which states in
part that evidence of handwriting by comparison may be made "with writings admitted or treated as genuine by the party against
whom the evidence is offered, or proved to be genuine to the satisfaction of the judge," and that there is no evidence on record
which proves or tends to prove the genuineness of the standards used.
There is no merit in this contention.
The records show that the signatures which were used as "standards" for comparison with the alleged signature of the private
respondent in the Special Power of Attorney were those from the latter's residence certificates in the years 1973, 1974 and
1975, her income tax returns for the years 1973 and 1975 and from a document on long bond paper dated May 18, 1977. Not

only were the signatures in the foregoing documents admitted by the private respondent as hers but most of the said documents
were used by the private respondent in her transactions with the government. As was held in the case of Plymouth Saving &
Loan Assn. No. 2 v. Kassing (125 NE 488, 494):
We believe the true rule deduced from the authorities to be that the genuineness of a "standard" writing may be
established (1) by the admission of the person sought to be charged with the disputed writing made at or for the
purposes of the trial or by his testimony; (2) by witnesses who saw the standards written or to whom or in whose
hearing the person sought to be charged acknowledged the writing thereof; (3) by evidence showing that the
reputed writer of the standard has acquiesced in or recognized the same, or that it has been adopted and acted
upon by him his business transactions or other concerns....
Furthermore, the judge found such signatures to be sufficient as standards. In the case of Taylor-Wharton Iron & Steel Co. v.
Earnshaw (156 N.E. 855, 856), it was held:
When a writing is offered as a standard of comparison it is for the presiding judge to decide whether it is the
handwriting of the party to be charged. Unless his finding is founded upon error of law, or upon evidence which is, as
matter of law, insufficient to justify the finding, this court will not revise it upon exceptions." (Costelo v. Crowell, 139
Mass. 588, 590, 2 N.E. 648; Nuez v. Perry, 113 Mass, 274, 276.)
We cannot find any error on the part of the trial judge in using the above documents as standards and also in giving credence to
the expert witness presented by the private respondent whose testimony the petitioner failed to rebut and whose credibility it
likewise failed to impeach. But more important is the fact that the unrebutted handwriting expert's testimony noted twelve (12)
glaring and material differences in the alleged signature of the private respondent in the Special Power of Attorney as compared
with the specimen signatures, something which the appellate court also took into account. In Cesar v. Sandiganbayan (134
SCRA 105, 132), we ruled:
Mr. Maniwang pointed to other significant divergences and distinctive characteristics between the sample signatures
and the signatures on the questioned checks in his report which the court's Presiding Justice kept mentioning during
Maniwang's testimony.
In the course of his cross-examination, NBI expert Tabayoyong admitted that he saw the differences between the
exemplars used and the questioned signatures but he dismissed the differences because he did not consider them
fundamental. We rule that significant differences are more fundamental than a few similarities. A forger always
strives to master some similarities.
The second issue raised by the petitioner is that while it is true that A & L Industries is a single proprietorship and the registered
owner thereof is private respondent Lily Yulo, the said proprietorship was established during the marriage and its assets were
also acquired during the same. Therefore, it is presumed that this property forms part of the conjugal partnership of the spouses
Augusto and Lily Yulo and thus, could be held liable for the obligations contracted by Augusto Yulo, as administrator of the
partnership.
There is no dispute that A & L Industries was established during the marriage of Augusta and Lily Yulo and therefore the same is
presumed conjugal and the fact that it was registered in the name of only one of the spouses does not destroy its conjugal
nature (See Mendoza v. Reyes, 124 SCRA 161, 165). However, for the said property to be held liable, the obligation contracted
by the husband must have redounded to the benefit of the conjugal partnership under Article 161 of the Civil Code. In the
present case, the obligation which the petitioner is seeking to enforce against the conjugal property managed by the private
respondent Lily Yulo was undoubtedly contracted by Augusto Yulo for his own benefit because at the time he incurred the
obligation he had already abandoned his family and had left their conjugal home. Worse, he made it appear that he was duly
authorized by his wife in behalf of A & L Industries, to procure such loan from the petitioner. Clearly, to make A & L Industries
liable now for the said loan would be unjust and contrary to the express provision of the Civil Code. As we have ruled in Luzon
Surety Co., Inc. v. De Gracia (30 SCRA 111, 115-117):
As explained in the decision now under review: "It is true that the husband is the administrator of the conjugal
property pursuant to the provisions of Art. 163 of the new Civil Code. However, as such administrator the only
obligations incurred by the husband that are chargeable against the conjugal property are those incurred in the
legitimate pursuit of his career, profession or business with the honest belief that he is doing right for the benefit of
the family. This is not true in the case at bar for we believe that the husband in acting as guarantor or surety for
another in an indemnity agreement as that involved in this case did not act for the benefit of the conjugal
partnership. Such inference is more emphatic in this case, when no proof is presented that Vicente Garcia in acting
as surety or guarantor received consideration therefore, which may redound to the benefit of the conjugal
partnership.(Ibid, pp. 46-47).
xxx xxx xxx
xxx xxx xxx
In the most categorical language, a conjugal partnership under that provision is liable only for such "debts and
obligations contracted by the husband for the benefit of the conjugal partnership." There must be the requisite
showing then of some advantage which clearly accrued to the welfare of the spouses. There is none in this case.
xxx xxx xxx
Moreover, it would negate the plain object of the additional requirement in the present Civil Code that a debt
contracted by the husband to bind a conjugal partnership must redound to its benefit. That is still another provision
indicative of the solicitude and tender regard that the law manifests for the family as a unit. Its interest is paramount;
its welfare uppermost in the minds of the codifiers and legislators.
We, therefore, rule that the petitioner cannot enforce the obligation contracted by Augusto Yulo against his conjugal properties
with respondent Lily Yulo. Thus, it follows that the writ of attachment cannot issue against the said properties.
Finally, the third issue assails the award of actual damages according to the petitioner, both the lower court and the appellate
court overlooked the fact that the properties referred to are still subject to a levy on attachment. They are, therefore, still
under custodia legis and thus, the assailed decision should have included a declaration as to who is entitled to the attached
properties and that assuming arguendo that the attachment was erroneous, the lower court should have ordered the sheriff to
return to the private respondent the attached properties instead of condemning the petitioner to pay the value thereof by way of
actual damages.
In the case of Lazatin v. Twao (2 SCRA 842, 847), we ruled:
xxx xxx xxx
... It should be observed that Sec. 4 of Rule 59, does not prescribed the remedies available to the attachment
defendant in case of a wrongful attachment, but merely provides an action for recovery upon the bond, based on the
undertaking therein made and not upon the liability arising from a tortuous act, like the malicious suing out of an
attachment. Under the first, where malice is not essential, the attachment defendant, is entitled to recover only the
actual damages sustained by him by reason of the attachment. Under the second, where the attachment is

maliciously sued out, the damages recoverable may include a compensation for every injury to his credit, business
or feed (Tyler v. Mahoney, 168 NC 237, 84 SE 362; Pittsburg etc. 5 Wakefield, etc., 135 NC 73, 47 SE 234). ...
The question before us, therefore, is whether the attachment of the properties of A & L Industries was wrongful so as to entitle
the petitioner to actual damages only or whether the said attachment was made in bad faith and with malice to warrant the
award of other kinds of damages. Moreover, if the private respondent is entitled only to actual damages, was the court justified
in ordering the petitioner to pay for the value of the attached properties instead of ordering the return of the said properties to
the private respondent Yulo ?
Both the trial and appellate courts found that there was bad faith on the part of the petitioner in securing the writ of attachment.
We do not think so. "An attachment may be said to be wrongful when, for instance, the plaintiff has no cause of action, or that
there is no true ground therefore, or that the plaintiff has a sufficient security other than the property attached, which is
tantamout to saying that the plaintiff is not entitled to attachment because the requirements of entitling him to the writ are
wanting. (7 C.J.S., 664)" (p. 48, Section 4, Rule 57, Francisco, Revised Rules of Court).
Although the petitioner failed to prove the ground relied upon for the issuance of the writ of attachment, this failure cannot be
equated with bad faith or malicious intent. The steps which were taken by the petitioner to ensure the security of its claim were
premised, on the firm belief that the properties involved could be made answerable for the unpaid obligation due it. There is no
question that a loan in the amount of P591,003.59 was borrowed from the bank.
We, thus, find that the petitioner is liable only for actual damages and not for exemplary damages and attorney's fees.
Respondent Lily Yulo has manifested before this Court that she no longer desires the return of the attached properties since the
said attachment caused her to close down the business. From that time she has become a mere employee of the new owner of
the premises. She has grave doubts as to the running condition of the attached machineries and equipments considering that
the attachment was effected way back in 1975. She states as a matter of fact that the petitioner has already caused the sale of
the machineries for fear that they might be destroyed due to prolonged litigation. We, therefore, deem it just and equitable to
allow private respondent Lily Yulo to recover actual damages based on the value of the attached properties as proven in the trial
court, in the amount of P660,000.00. In turn, if there are any remaining attached properties, they should be permanently
released to herein petitioner.
We cannot, however, sustain the award of P500,000.00 representing unrealized profits because this amount was not proved or
justified before the trial court. The basis of the alleged unearned profits is too speculative and conjectural to show actual
damages for a future period. The private respondent failed to present reports on the average actual profits earned by her
business and other evidence of profitability which are necessary to prove her claim for the said amount (See G. A. Machineries,
Inc. v. Yaptinchay, 126 SCRA 78, 88).
The judgment is therefore set aside insofar as it holds the petitioner liable for P500,000.00 actual damages representing
unrealized profits, P150,000.00 for exemplary damages and P20,000.00 for attorney's fees. As stated earlier, the attached
properties, should be released in favor of the petitioner.
WHEREFORE, the decision of the Court of Appeals is hereby SET ASIDE and the petitioner is ordered to pay the private
respondent Lily Yulo the amount of SIX HUNDRED SIXTY THOUSAND PESOS (P660,000.00) as actual damages. The
remaining properties subject of the attachment are ordered released in favor of the petitioner.

G.R. No. 102692 September 23, 1996


JOHNSON & JOHNSON (PHILS.), INC., petitioner,
vs.
COURT OF APPEALS and ALEJO M. VINLUAN, respondents.

PANGANIBAN, J.:
May a husband be held liable for the debts of his wife which were incurred without his consent and which did not benefit
the conjugal partnership? May a judgment declaring a wife solely liable, be executed uponconjugal property, over the
objection of the husband?
These are the main questions raised in the instant petition for review on certiorari under Rule 45 of the Rules of Court
which seeks nullification of the Decision 1 in CA-G.R. SP No. 19178 of the Court of Appeals, 2 the dispositive portion of which reads:
WHEREFORE, in view of all the foregoing, the instant petition is hereby GRANTED, and the orders dated July 24,
1989 and October 4, 1989 of the Regional Trial Court of Makati, Branch 137, in Civil Case No. 4186, as well as the
notices of levy issued by the Provincial Sheriff of Rizal dated February 8, 1989, are hereby declared null and void
and set aside. No costs.
The Facts
This case was initiated in the trial court by a complaint

filed by petitioner against spouses Delilah A. Vinluan, owner of Vinluan Enterprises, and her
husband Capt. Alejo M. Vinluan (the private respondent before us), for collection of a sum of money with damages, which was docketed as Civil Case No. 4186 and tried in
the Regional Trial Court of Makati, Branch 137. 4 The respondent appellate Court found the antecedent facts, to be as follows: 5
The plaintiff-respondent Johnson & Johnson (Phils.), Incorporated (hereinafter referred to as the corporation) is engaged in the manufacturing and selling of
various cosmetics, health, and body care products, as well as medical drugs. On several occasions in the year 1982, the defendant, Delilah Vinluan, purchased
products of the plaintiff-respondent corporation, as she was also engaged in the business of retailing Johnson products, among others. The defendants, under the
name and style of "Vinluan Enterprises," thus incurred an obligation of Two Hundred Thirty-Five Thousand Eight Hundred Eighty Pesos and Eighty-Nine
(P235,880.89) Centavos, for which she issued seven (7) Philippine Banking Corporation checks of varying amounts and due dates. When presented on their
respective due dates, however, the checks given in payment of the obligation bounced and were dishonored for having been drawn against insufficient funds.

Several demands thereafter for payment were to no avail, despite the accommodations given by the plaintiffrespondent corporation by granting several extensions to the defendant spouses to settle the obligation. It was only
on January 5, 1983 that the defendants made a partial payment of Five Thousand (P5,000.00) Pesos, thereby
reducing their principal obligation to P230,880.89. When no further payments were made to settle the obligation
despite repeated demands, the plaintiff-respondent corporation was constrained to file a complaint (Annex "A") on
June 8, 1983 against defendant spouses Vinluan, for collection of the principal obligation plus interest, with
damages. Filed before the respondent Regional Trial Court of Makati, Branch 137, it was docketed as Civil Case No.
4186.
After trial on the merits, on February 5, 1985, the respondent court rendered its Decision (Annex "C"), the dispositive
portion of which reads:
WHEREFORE, judgment is hereby rendered sentencing the defendant DELILAH A. VINLUAN to
pay plaintiff Johnson & Johnson (Phils.), Inc, the sum of P242,482.40 with interest and penalty
charges at the rate of 2% per month from 30 January 1983 until fully paid, and the sum of
P30,000.00 as attorney's fees, and to pay the costs.
Defendants' counterclaim is hereby dismissed for lack of sufficient merit.
In arriving at the sole liability of defendant Delilah A. Vinluan, the trial court found after "meticulous scrutiny and careful
evaluation of the evidence on record" that there was "no privity of contract, whether direct or indirect, between plaintiff and
defendant-husband regarding the obligations incurred by defendant-wife." According to the trial court, "(i)n fact, the acts
performed, and the statements made, by defendant-husband, and from which plaintiff derived the notion that said
defendant is a co-owner of VINLUAN ENTERPRISES, took place after the obligation involved in this action had been
incurred or contracted by the defendant-wife, albeit without the husband's knowledge or consent, as there was no
allegation in the complaint that said obligations were incurred by defendant-wife with her husband's consent, or that it was
incurred for the benefit of the family. . . ." 6
The trial court also found that private respondent never intimated in his conversations or meetings with, or in any of his
letters to, petitioner that "he was a co-owner of VINLUAN ENTERPRISES, much less did he represent himself as such coowner, to the plaintiff and to plaintiff's counsel . . . ." When private respondent personally negotiated with petitioner and
proposed a settlement of the subject obligations, these actuations were not to be considered as admission of coownership of VINLUAN ENTERPRISES for "(a)fter all, common sense and our inborn mores of conduct dictate that a
husband must give aid and comfort to his distressed wife." 7 The trial court further held that the defendant spouses had sufficiently established that
the defendant wife was sole owner of the business venture, that the conjugal partnership never derived any benefit therefrom, and that the same closed due to continued
losses. In sum, the court a quo held that private respondent could not legally be held liable for the obligations contracted by the wife.

Thus, the court below issued a writ of execution

on February 3, 1989, directing the Provincial Sheriff of Rizal to execute the judgment on
the properties of the defendant-wife. However, the two notices of levy on execution 9 issued on February 8, 1989 covered not only her exclusive or paraphernal properties,
but also the real and personal properties of the conjugal partnership of the spouses Vinluan. The next day, her husband (herein private respondent) filed a third-party
claim 10seeking the lifting of the levy on the conjugal properties, followed by another third-party claim reiterating the same demand with threat of possible lawsuit.
Subsequently, petitioner corporation filed a motion dated February 14, 1989 asking the court to fix the value of the properties levied upon by the sheriff. In response to the
third-party claims of private respondent, a comment and/or opposition dated March 6, 1989 was filed by petitioner.

Private respondent moved on July 1, 1989 to quash the levy on execution on the ground that the notices of levy on
execution did not conform to the final decision of the court and to the writ of execution. As expected, petitioner opposed
the motion. On July 24, 1989, the trial court issued the first assailed Order fixing the value of the levied personal
properties at P300,000.00, and denying the third-party claim and the motion to quash the levy on execution. Citing the last
sentence of Article 117 11 of the Civil Code, the court a quo ruled that: 12
Since Alejo Vinluan did not seek the intervention of the Court to air his objections in his wife's engaging in business, coupled by the fact that he made several
representations for the settlement of his wife's account, Alejo Vinluan's consent thereto became evident. As such, even his own capital may be liable, together with
the conjugal and paraphernal property (I Paras 363, 1987 ed., p. 6; Art. 6-10, Code of Commerce). Withal, Article 172 of the New Civil Code categorically declares
that

The wife cannot bind the conjugal partnership without the husband's consent, except in the cases
provided by law.
Granting arguendo that Alejo Vinluan did not give his consent, expressly or impliedly, the paraphernal and conjugal
property may still be held liable but not his capital (I Paras 363, 1978 ed.).
Petitioner's motion for reconsideration of the abovequoted first order (on the ground that it directly contravened the
decision itself which had already become final and executory) was denied via the second contested Order dated October
4, 1989, where the trial court ruled: 13

The Court finds untenable movant-defendant's assertion that Art. 172 of the New Civil Code is not in point. The consent of the husband is indeed vital in
determining what properties shall be subsidiarily liable in the event the paraphernal properties of Delilah Vinluan should turn out to be insufficient to cover the
judgment debt, as fully explained in the Order dated 24 July 1989.

Art. 122 of the Family Code which party provides that


Art. 122. The payment of personal debts contracted by the husband or the wife before or during the
marriage shall not be charged to the conjugal partnership except insofar as they redounded to the
benefit of the family.
xxx xxx xxx
is not applicable in that
This Code (Family Code) shall have retroactive effect insofar as it does not prejudice or impair
vested or acquired rights in accordance with the Civil Code or other laws (Art. 255, Family Code;
emphasis supplied).
Plaintiff (petitioner herein), having acquired a vested right prior to the effectivity of the Family Code, said code is
not a propos (sic). Even granting arguendo that the same is befitting, movant defendant failed to realize that
although Delilah Vinluan suffered losses in her legitimate business, the experience she has gained redounded to the
benefit of the family, and as such, the conjugal partnership must bear the indebtedness and losses (I Paras 464,
1981 ed.). Moreover, had the business Delilah Vinluan engaged in been a success, all profits would have been
considered conjugal; it is therefore but fair that the risks of the business should be borne by the conjugal partnership
(Miravite, Bar Review Materials in Commercial Law, 1986 ed., p. 89; J.N. Nolledo, Commercial Law Reviewer, 1986
ed., pp. 6, 7; U.P. Law Complex, Answers to Bar Questions In Commercial Law, 1986 ed., pp. 174, 175; Vitug,
Commercial Law Reviewer, 1984, ed., p. 5).
There is a wide-embracing oversight when movant-defendant asserted that to hold the conjugal partnership property
liable for the indebtedness incurred solely by his wife would in effect modify the Decision dated 5 Feb 1985 which is
now final and executory. As afore-discussed, the conjugal property is subsidiarily liable.
As indicated above, the private respondent elevated the matter to the respondent appellate Court, charging the trial court
with grave abuse of discretion for effectively reversing its own final judgment. The respondent Court upheld the private
respondent in its now-assailed Decision, and denied herein petitioner's subsequent motion for reconsideration. Thus,
petitioner is now before us seeking review under Rule 45.
The Issues
Petitioner raised the following "issues of law" for consideration of this Court, to wit:

14

1. Whether or not the decision of the honorable trial court dated February 5, 1985 exonerating (sic) defendant husband, private respondent herein, from the
obligation contracted by the wife in the pursuit of her business also absolves the conjugal partnership from liability.

2. Whether or not the subsequent order of the honorable trial court dated July 24, 1989 and October 4, 1989 is a
reversal of its own original decision as found out by the honorable public respondent.
The pivotal issues in this case may be re-stated thus: whether or not the order of the trial court denying private
respondent's third-party claim and motion to quash levy on execution in effect amended the dispositive portion of the trial
court's decision which had long become final and executory, and if so, whether same is proper or not. These issues shall
be ruled upon together.
The Court's Ruling
Petitioner contends 15 that the purpose of impleading private respondent as co-defendant in petitioner's complaint was to bind not only the defendant-spouses'
conjugal partnership but also private respondent's capital. The trial court resolved that it was not necessary that private respondent (as husband) be joined as partydefendant in the suit below. Inasmuch as it appeared from the allegations in the complaint that private respondent may be a co-owner of Vinluan Enterprises, the trial court
nonetheless did not exclude private respondent but passed upon the issue of such co-ownership to determine whether he may be held liable in the same manner as his
wife. Petitioner insists that the trial court in its decision merely made a finding that the private respondent husband was not a co-owner of the business venture of his wife,
which conclusion ("exoneration") only exempted his capital from the adjudged liability, but not the conjugal properties of the spouses. Petitioner further argues that nowhere
in the trial court's decision can there be found any pronouncement absolving the conjugal property from liability, contrary to the findings of the respondent Court.

Also, petitioner reasons that the enforcement of the decision against the conjugal property is merely compliance with law,
and that this Court in a long line of cases 16 held that a judgment is not confined to what appears upon the face of the decision but also those necessarily
included therein or necessary thereto. 17Additionally, petitioner pleads that the trial court's order did not modify its final and executory decision but only clarified an
ambiguity in the decision as to what properties are liable. As authority, it cites Republic vs. De los Angeles. 18

Petitioner's contentions are devoid of merit.


Respondent Court correctly ruled that the trial court cannot, in the guise of deciding the third-party claim, reverse its final
decision. Commenting on the trial court's very patent "about-face" on the issues of consent of the husband, benefit to the
family, and the husband's liability for obligations contracted by his wife, the appellate Court held, and we quote: 19
We see in these stark contradictions an attempt by the respondent Court to reverse itself, even when the decision sought to be executed had already become final.
The respondent Court has no authority to modify or vary the terms and conditions of a final and executory judgment (Vda. de Nabong vs. Sadang, 167 SCRA 232)
and this attempt to thwart the rules cannot be allowed to pass. Even if the respondent Court feels that it needed to reverse its findings to correct itself, the decision,
whether erroneous or not, has become the law of the case between the parties upon attaining finality (Balais vs. Balais, 159 SCRA 37). the respondent Court has
no choice but to order the execution of the final decision according to what is ordained and decreed in the dispositive portion of the decision (National Steel Corp.
vs. NLRC, 165 SCRA 452).

The dispositive portion of the decision charges the defendant Delilah Vinluan alone to pay the plaintiff corporation,
having already declared that the defendant-husband cannot be held legally liable for his wife's obligations. Perhaps,
when it was later discovered that the defendant Delilah Vinluan did not have sufficient property of her own to settle
the obligation, the conjugal properties of the defendant-spouses became the object of the levy. But in order to bind
the conjugal partnership and its properties, the New Civil Code provides that the debts and obligations contracted by
the husband (or the wife) must be for the benefit of the conjugal partnership (Article 161, par. 1); and that the
husband must consent to his wife's engaging in business (Article 117).
Thus, we see a belated effort on the part of the respondent Court to reverse itself by declaring that the obligations
incurred by the defendant wife redounded to the benefit of the family and that the defendant husband had given his
consent, in order to bind the conjugal partnership.
As We stated earlier, this cannot be done because the decision, along with the respondent Court's original findings,
had already become final and indisputable. The respondent Court already found that the defendant husband did not
give his consent; neither did the obligation incurred by the defendant wife redound to the benefit of the family.
Hence, the conjugal partnership, as well as the defendant husband, cannot be held liable. As originally decreed by
the Court, only the defendant wife and her paraphernal property can be held liable. Since the power of the court in
execution of judgments extends only to properties unquestionably belonging to the judgment debtor alone (Republic
vs. Enriquez, 166 SCRA 608), the conjugal properties and the capital of the defendant husband cannot be levied
upon.

The settled rule is that a judgment which has acquired finality becomes immutable and unalterable, and hence may no
longer be modified in any respect except only to correct clerical errors or mistakes all the issues between the parties
being deemed resolved and laid to rest. 20 This is meant to preserve the stability of decisions rendered by the courts, and to dissuade parties from trifling
with court processes. One who has submitted his case to a regular court necessarily commits himself to abide by whatever decision the court may render. Any error in the
decision which has not been considered in a timely motion for reconsideration or appeal cannot be impugned when such error becomes apparent only during execution.
This rule applies with more force in the case of to decision judge who has limited prerogative during execution of the judgment. For as correctly held by herein public
respondent, aside from ordering the enforcement of the dispositive portion of the decision, the trial judge can do nothing about the errors in the ratiocination of the decision
or even alter the dispositive portion by mere order issued subsequent to the finality of the decision. The issue having been laid to rest, the court cannot on the pretext of
determining the validity of the third-party claim and the motion to quash levy on execution alter the scope of the dispositive portion of the decision sought to be
implemented.

Petitioner's arguments notwithstanding, the trial court's order cannot be said to be merely clarificatory in nature. There is
no ambiguity at all in the decision, for it categorically declared defendant Delilah A. Vinluan solely liable, without any
recourse provided against her husband. Thus, the case of Republic vs. Delos Angeles, 21 holding that doubtful or ambiguous judgments
are to have a reasonable intendment to do justice and avoid wrong, does not apply here. as was later held in Filinvest Credit Corporation vs. Court of Appeals, 22 "(w)here
there is an ambiguity, a judgment shall be read in connection with the entire record and construed accordingly. In such a case, it is proper to consider the pleadings and the
evidence." (Emphasis supplied). But the text of the trial court's decision points to no other person liable but Delilah Vinluan, and in fact made a rather lengthy discussion on
the exemption from liability of the conjugal partnership; hence, there can be no ambiguity to speak of in the decision. And even more clearly, the body of the decision of the
trial court expressly exempted private respondent from liability by categorically ruling that "the defendant-husband cannot, together, with co-defendant, legally be held liable
for the obligations contracted by the wife." 23 Further, the trial court expounded: 24
. . . . What is more, it is an admitted fact that the subject obligations had partially been paid by the defendant-wife herself. Thus, plaintiff implicitly averred that
"defendant Delilah Vinluan, ding business under the name and style of VINLUAN ENTERPRISES is one of the various customers of the plaintiff's products' (Cf. p.
1, Plaintiff's Pre-Trial Brief); that "Delilah Vinluan . . . purchased different Johnson products . . . , thus incurring an obligation of P235,880.89" (Cf. par. III,
Complaint); that "defendant Delilah Vinluan tried to pay (her) obligations . . . when she issued Philippine Banking Checks . . . , but which checks upon presentment
to the Bank were dishonored for the reason "Drawn Against Insufficient Funds" (Cf. par. V, id.); that " . . . , defendant Delilah A. Vinluan appealed to the company
and also represented that she be given an opportunity to settle the accountability" (Cf. par. VI, id.); that "defendant sent a letter to the company where she alleged
that payment cannot be made because they are "victims of some bad practices in the trade and that they are working on some means to settle their accounts and
all that they ask is time to settle." (Cf. par. VI, id.).

We take this occasion to reiterate the ruling of this Court in an early


case 25 that litigations must end and terminate sometime and somewhere, it being essential to the effective and efficient administration of justice that once a judgment
has become final, the winning party be not, through a mere subterfuge, deprived of the fruits of the verdict. Hence, courts must guard against any scheme calculated to
bring about that result, for, constituted as they are to put an end to controversies, courts should frown upon any attempt to prolong them. Furthermore, public policy and
sound practice demand that at the risk of occasional errors, judgments of courts should become final and irrevocable at some definite date fixed by law. And this is better
observed if the court executing the judgment would refrain from creating further controversy by effectively modifying and altering the dispositive portion of the decision, thus
further delaying the satisfaction of the judgment. No matter how just the intention of the trial court, it cannot legally reverse what has already been settled. Holding the
conjugal partnership liable in the order after the finality of the decision is evidently not just correcting a mere clerical error; it goes into the merits of the case. And this is
prohibited by the rules and jurisprudence.

We have elsewhere ruled that "should judgment of lower courts which may normally be subject to review by higher
tribunals become final and executory before, or without, exhaustion of all recourse of appeal, they, too, become
inviolable, impervious to modification. They may, then, no longer be reviewed, or in any way modified directly or indirectly,
by a higher court, not even by the Supreme Court, much less by any other official, branch or department of
Government." 26
. . . (N)othing is more settled in the law than that when a final judgment becomes executory, it thereby becomes immutable and unalterable. The judgment may no
longer be modified in any respect, even if the modification is meant to correct what is perceived to be an erroneous conclusion of fact or law, and regardless of
whether the modification is attempted to be made by the Court rendering it or by the highest Court of land. They only recognized exceptions are the correction of
clerical errors or the making of so-called nunc pro tuncentries which cause no prejudice to any party, and, of course, where the judgment is void.

Furthermore, "(a)ny amendment or alteration which substantially affects a final and executory judgment is null and
void for lack of jurisdiction, including the entire proceedings held for that purpose." 27
The respondent Court also commented on the sheriff's actuations as follows:

28

Furthermore, it is the duty of the sheriff to ensure that only that portion of the decision ordained and decreed in the dispositive part should be the subject of the
execution (Cunanan vs. Cruz, 167 SCRA 674). The writ of execution itself states that only the properties of the defendant wife were to be levied upon. There was
no mention even of conjugal properties. Hence, in levying on the properties that did not exclusively belong to the judgment debtor, the notices of levy failed to
conform to the decree of the decision, and are, therefore, irregular and contrary to the Rules (Canlas vs. CA, 164 SCRA 160).

It is a rule firmly established in our jurisprudence that a sheriff is not authorized to attach or levy on property not belonging
to the judgment debtor. 29 A sheriff even incurs liability if he wrongfully levies upon the property of a third person. 30 A sheriff has no authority to attach the
property of any person under execution except that of the judgment debtor. The sheriff maybe liable for enforcing execution on property belonging to a third party.
does so, the writ of execution affords him no justification, for the action is not in obedience to the mandate of the writ.

31

WHEREFORE, in view of the foregoing considerations, the herein petition is hereby DENIED, and the Decision of the
respondent Court is AFFIRMED. Costs against petitioner.

If he

G.R. No. L-16991

March 31, 1964

ROBERTO LAPERAL, JR., ET AL., plaintiffs-appellants,


vs.
RAMON L. KATIGBAK, ET AL., defendants-appellees.
William H. Quasha and Associates for plaintiffs-appellants.
Bausa, Ampil and Suarez for defendant-appellee Evelina Kalaw Katigbak.
Ramon L. Katigbak in his own behalf as defendant-appellee.
REGALA, J.:
This is an appeal from a decision of the Court of First Instance of Manila declaring the property covered by TCT No. 57626 in
the City of Manila to be the separate or paraphernal property of the defendant-appellee Evelina Kalaw. Plaintiffs-appellants, the
spouses Laperal, disagree with the said finding. Hence they appealed from the said decision. They maintain that the realty in
question, with its improvements and income, are conjugal assets of the spouses Evelina Kalaw and Ramon Katigbak.
This litigation is a sequel to the one instituted by the Laperals against Katigbak and Kalaw way back in August, 1950. In that
case, Civil Case No. 11767 of the Court of First Instance of Manila, the Laperals sought from the therein defendants "recovery
of P14,000 evidenced by various promissory notes executed in favor of the Laperals by Katigbak, and for the return of jewelry
valued at P97,500.00, delivered by the Laperals to Katigbak for sale on commission, or a total of P111,500.00." On November 1,
1950, upon a confession of judgment by Katigbak, the trial court rendered judgment against him to pay the Laperals the sum of
P14,000.00, and to return the jewelry involved, or in lieu thereof, to pay plaintiffs P97,500.00, with interest from August 8, 1950.
About a month after this decision was rendered, Kalaw filed a complaint against her husband Katigbak, for "judicial separation
of property and separate administration," docketed as Civil Case No. 12860, of the Court of First Instance of Manila. Prior to the
trial, Katigbak and Kalaw submitted an agreement or stipulation of facts on the basis of which, the court granted the prayer for
the "judicial separation of property and separate administration."
On February 1, 1955, the Laperals filed another complaint against Kalaw and Katigbak, Civil Case No. 25235 in the Court of
First Instance of Manila, seeking among other things, annulment of the proceedings had in Civil Case No. 12860 for "judicial
separation of property and separate administration," to enforce the judgment secured by the Laperals in Civil Case No. 11767
on the fruits of Kalaw's paraphernal property, and to secure a ruling declaring the real property covered by TCT No. 57626 as
conjugal property of Katigbak and Kalaw. After trial, the court dismissed the complaint, which dismissal the Laperals appealed to
this Court. Acting on the same, We rendered judgment under G.R. No. L-11418, promulgated on December 27, 1958, the
pertinent portion of which reads:
However, our holding does not write a finis to the case. Because the trial court held that the conjugal partnership was not
liable, it naturally, saw no reason or necessity for ruling upon the other issues involved, such as the legality of the
proceedings in Civil Case No. 12860 for the dissolution of the conjugal partnership and whether or not the property
covered by Transfer Certificate of Title No. 57626 belongs to the conjugal partnership.
In conclusion we hold that while the fruits of the paraphernal property of Kalaw are not liable for the enforcement of the
obligations contracted by Katigbak, nevertheless, the conjugal properties are.
1wph1.t

The case is hereby ordered remanded for further proceedings to make trial court, the latter to make the necessary findings
indicated and then render a decision on the basis of said findings in accordance with our decision. No costs.
In compliance with the above endorsement, the trial court, after submission by the parties of their respective memoranda, there
being already sufficient evidence in the record rendered judgment declaring the property covered by TCT No. 57626 as
paraphernal. Hence, this appeal.
The facts upon which trial court predicated its conclusion that the realty in question is paraphernal are as follows:
For the reason that it established, without contradiction in the records that the spouses Ramon Katigbak and Evelina Kalaw
were married in 1938 (tsn, p. 20, hearing of Jan. 31, 1956) and neither of them had brought properties unto the marriage that
Ramon's occupation was that of Asst. Atty. of the Bank of the Phil. Islands wherein his monthly salary P200.00 (id); that the
property under TCT No. 57626 was registered in the name of "Evelina Kalaw-Katigbak, married to Ramon Katigbak" on
December 6, 1939, only two years after marriage and the property was and is in Calle Evangelista, which was and is a business
district; the Court, not withstanding the presumption that all properties acquired during the marriage are conjugal, is led to
believe that, as Evelina declares, her mother Pura Villanueva was the one that had bought property for her and had placed it
only in her name as the practice of her mother; that is, buying properties placing them directly in the names of her children; and
this is recognized by Article 1448; and the Court is all the more led to the conclusion when it sees that Ramon Katigbak, in the
same year 1939, that is, long before the spouses had come to the parting of ways, made a manifestation that he had no interest
in the properties
Ramon Katigbak, the husband of the vendor signed this document only for the purpose of assisting his wife but he has no
interest in the property.
(Sgd.) Ramon L. Katigbak
(Exh. 5-a, p. 189, Record)
As this case is before Us now, therefore, the issue is whether or not the above findings warrant a rejection of the presumption
that the property disputed, for the reason that it was acquired during the marriage, is conjugal.
We find for the appellee.
There is no denying that all properties acquired during the marriage are, by law, presumed conjugal. (Art. 160, Civil Code) The
presumption, however, is not conclusive but merely rebuttable, for the same law is un equivocal that it exists only "unless it be
proved that it (the property) belongs exclusively to the husband or the wife." And, examining the records and evidence in this
suit, We hold that this is a case where the presumption has been sufficiently and convincingly disproven.
The facts recited by the trial judge in explanation of his view that the property in dispute is paraphernal despite its having been
acquired during coverture, impress Us as adequate and conclusive. As a matter of fact, the factors he took into account in
rejecting the presumption, on the whole, tally with Our own views in the cases of Casiano v. Samaniego, 30 Phil. 135
and Coingco v. Flores, 82 Phil. 284.
In the Casiano case, the deeds to the property in question were in the name of the defendant who testified that they were
"purchased by her mother for herself" and that the purchase price was paid with money furnished by her mother. On the
foregoing, the trial judge rendered judgment in favor of the defendant, and declared the real properties to be paraphernal. On
appeal to this Court, declaring the ruling as essentially factual, We said:
We do not question the correctness of the doctrine contended for, but we think it is sufficient to say that the legal
presumption established by article 1407 of the Civil Code has been overcame by the evidence of record. There is nothing
in the record which would justify us in disturbing the findings, of the trial judge as to the credibility of the witnesses called

by the defense, and if we believe the defendant herself there can be no doubt the land in question was purchased for the
wife with her own separate funds.
It should be further noted that the husband in the aforecited case, apart from relying on the presumption established by the Civil
Code, sought to show the conjugal nature of the disputed property by presenting a number of documentary evidence. He
exhibited, for instance, "certified copies of reports on file in the City Assessor's Office showing that the land was assessed in his
name; a certified copy of an inspector's report in which the name of the husband appears as the owner; and, a tax declaration
made in November, 1905, relating to the property in dispute, in the name of the husband." Yet, even then, this Court declined to
give effect to the presumption as the wife's evidence to the contrary were more preponderant. In the present case, on the other
hand, We note that other than invoking the presumption, the burden of denying the evidence so presented was shifted to the
appellant. In this latter task, the appellant failed completely.
Again, in the Coingco case, We ruled:
The second question raised in the motion for reconsideration is, whether the presumption that the properties in litigation
are conjugal properties because they were acquired during the coverture may be sufficiently rebutted by any one of the
following facts: (1) the titles to them are in the name of wife alone; (2) that the husband gave his marital consent to their
being mortgaged by the wife; (3) that the wife financially able to buy those properties. While it is true that each one of
them, taken separately, may not be sufficient to overcome the above-quoted presumption established by Art. 14 of the
Civil Code, it is nonetheless true that all of them taken together, with all the other facts and circumstances established by
the evidence, might be, and were, considered by the lower court as sufficient to rebut the same presumption.
In the case before Us now for review, the deed to the disputed land is in the name of the wife. At the time of its purchase, the
property was already of such substantial value as admittedly, the husband, by himself could not have afforded to buy,
considering that singular source of income then was his P200.00 a month salary from a Manila Bank. As in the Casiano
case, supra, the defendant herein testified, and was believe by the trial court, that the purchase price was furnish by her mother
so she could buy the property for herself. Furthermore, it was established during the trial that it was a practice of defendant's
parents to so provide their children with money to purchase realties for themselves.
These facts, quite obviously, more than measure up to the circumstances obtaining in the two cases previous cited wherein We
held the conjugal presumption to have been rebutted.
IN VIEW OF THE FOREGOING, the judgment of the lower Court declaring the property covered by TCT No. 57626 of the
Register of Deeds of Manila as paraphernal is hereby affirmed, with costs against the appellants.

G.R. No. L-67742 October 29, 1987


MELITON GALLARDO and TERESA VILLANUEVA, petitioners,
vs.
HONORABLE INTERMEDIATE APPELLATE COURT, MARTA VILLANUEVA VDA. DE AGANA, VISITACION AGANA
KIPPING, PEDRO V. AGANA, MARCELO V. AGANA, JR., TERESITA AGANA SANTOS and JESUS V.
AGANA, respondents.
This is a petition for review on certiorari seeking to set aside or reverse the decision

* of the Intermediate Appellate Court (now Court of Appeals) promulgated on May


22, 1984 in AC-G.R. CV No. 69946 entitled Meliton Gallardo and Teresa Villanueva v. Marta Villanueva vda. de Agana, et al. (Rollo, p. 37) affirming the decision ** of the Court of First Instance of Laguna 8th Judicial District, Branch II, Sta.
Cruz, Laguna (now Regional Trial Court, Sta. Cruz, Laguna) dated January 20, 1982, which dismissed the complaint for Quieting of Title in Civil Case No. SC-1492 and declared the plaintiff's (petitioner's herein) Re-constituted Transfer
Certificate of Title RT-6293 (No. 23350) as null and void (Record on Appeal, pp. 215-216).

The dispositive portion of the questioned decision reads as follows:


WHEREFORE, the appealed judgment is in full accord with the evidence and the law and is hereby therefore
affirmed in all its part. Costs against plaintiff -appellants
SO ORDERED.
The subject matter of this controversy involves a parcel of land situated in Cavinti, Laguna consisting of 81,300 square meters,
more or less, initially covered by an original Certificate of Title No. 2262, issued on April 2, 1924 owned and registered in the
name of the late Pedro Villanueva (former Justice of the Peace of the Municipal Court, Cavinti, Laguna), pursuant to Decree No.
150562 issued in L.R.C. Cadastral Record No. 136, Cad. Case No. 1 (Record on Appeal; Answer, p. 28).
Petitioners were nephew and niece of the late Pedro Villanueva and first cousin of the private respondent Marta Villanueva vda.
de Agana, the latter being the daughter of Pedro Villanueva.
On August 10, 1937, petitioner claimed that the aforestated land was sold to them in a private document, an unnotarized deed
of sale written in Tagalog (Annex "B" of the complaint) that was allegedly signed by the late Pedro Villanueva conveying and
transfering the property in question in favor of the petitioners (Record on Appeal, Exhibit "B", pp. 9-10) which deed is
reproduced as follows.
Ako, Pedro Villanueva, 66 taong gulang, balo at nananahanan sa municipio ng Cavinti, lalawigang Laguna at
Kapuluang Pilipinas, alang-alang sa halagang LIMANG DAANG PISO (P500.00) salaping filipino, na sa akin
ibinayad ng mag-asawa ni Meliton Gallardo at Teresa Villanueva, tagarito rin sa nasabing municipio, lalawigang at
kapulwan sa hinaharap ng kasulatan ay sinasaysay ko na aking inilillwat at pinagbili ng biling patuluyan sa nasabing
mag-asawa Meliton Gallardo at Teresa Villanueva, sampo na sa kanilay mangagmamana at hahalili, ang aking
isang palagay na lupa na nabubuo sa limang luang na tubigan, punlang kalahating kabang palay at saka dalatan o
katihan na may isang kabang palay na hasik, tumatayo sa nayon ng Kanlurang Talaongan, sakop nitong municipio
ng Cavinti at napapaloob sa mga hangganang sumusunod:
HILAGAAN, Braulio Villanueva at Modesto Ribera
SILANGAN, Braulio Villanueva.
TIMUGAN, Braulio Villanueva, Ilog Kaliraya at Jacinto Toque
KANLURAN, Jacinto Toque.
Ang pagaaring ito ay tunay kong pananarili sapagkat aking nabili sa magkakapatid na Aniano Gallardo, Zacarias
Gallardo at Perfecto Gallardo at natatala sa Registro ng Amillarmiento dito sa Cavinti sa ilalim ng Blg. 22888, at
walang ano mang ipinagkakautang ni pinanagutan kaya at magagamit na nitong aking pinagbilhan ang kanilang
matuwid na maipamana at mailiwa sa iba. Gayon ding sinasaysay ko na akoy umaakong mananagutan dito sa
aking pinagbilhan, tungkol sa pagaaring ito na ang katibay ay aking ipagsasanggalang laban sa kanino mang
maghahabol.
Dapat tantoin, gayon man, na ang pagaaring ito ay registrado na sa Registro de la Propiedad nitong lalawigang
Laguna, subalit at sa isang kamalian ng pagkakasukat tungkol sa lawak at laki, ay hindi pa natutubos ang kanyang
titulo, kaya at kung maisaayos na ang nasabing titulo ay saka na ipatatala sa pangalan nitong aking pinagbilhan
upang lalong malagay sa katahimikan itong aking pinagbilhan.
At sa katunayan ay nilagdaan ko ang kasulatang ito dito sa municipio ng Cavinti, Laguna, ngayong ika sampung
araw ng Agosto taong isanglibo siyam na daan at tatlompu at pito (1937).
(LGD) PEDRO VILLANUEVA
Nagfirma sa hinaharap ni
(LGD) BALTAZAR VILLANUEVA
JUAN VILLANUEVA
Subsequently, the Original Certificate of Title was cancelled on the basis of the private document of sale (Exhibit "B") and a new
certificate of title was issued in the name of the petitioners covered by Transfer Certificate of Title No. RT- 6293 (No. 23350) on
January 4, 1944, particularly describing the land as follows:
A parcel of land (Lot No. 401 of the Cadastral Survey of Cavinti) with the improvements thereon, situated in the
municipality of Cavinti, Bounded on the N and NE., by Lot No. 403; on the SE by Lot No. 393 and the Caliraya River;
and on the SW by Lot No. 515. Area Eighty One Thousand and Three Hundred (81,300) Square Meters, more or
less. (Record on Appeal, Annex "A," pp. 7 and 9).
During the Second World War, the records as well as the Office of the Register of Deeds of Laguna, where the original of their
new transfer certificate of title was kept, were completely burned. Accordingly, by virtue of an Affidavit of Reconstitution dated
December 2, 1958 (Record on Appeal, Annex "DD," pp. 41-42) and upon presentation of the Owner's Duplicate Certificate of
Title, the title was administratively reconstituted and the Register of Deeds of Laguna issued Transfer Certificate of Title No. RT6293 (No. 23350) in the name of the petitioners (Record on Appeal, Annex "B", pp. 7).
On November 17, 1976, defendant Marta Villanueva together with Pedro Villanueva, Jr., and Restituto R. Villanueva executed
and filed an Affidavit of Adverse Claim with the Office of the Register of Deeds of Laguna (Record on Appeal, Annex "C", pp. 1013). However, on December 6, 1976 a joint affidavit was filed by Pedro G. Villanueva, Jr. and Restituto Villanueva withdrawing
their adverse claim on the said parcel of land, with the Office of the Register of Deeds of Laguna (Record on Appeal, Annex " D,
" pp. 13-14).
When petitioners learned of this Affidavit of Adverse Claim, attempt was made to settle said controversy amicably. Several
demands made by herein petitioners upon private respondents Marta Vda. de Agana to withdraw her adverse claim, failed.

On December 9, 1976, said private respondent executed a Deed of Conveyance and Release of Claim (Record on Appeal and
Annex "AA", p. 35) wherein the parties agreed, among other things, to the following:
That in consideration of the said transfer and conveyance over a 1,000 square meter portion mentioned in the next
preceding paragraph, the VENDEE (Marta V. Agana) does hereby withdraw the adverse claim mentioned above;
(Rollo, p. 119).
However, when private respondent Marta Villanueva vda. de Agana refused to sign an Affidavit of Quit-claim (Exhibit "9; "
Record on appeal, p. 195), petitioners instituted court suit against the private respondent and her husband, Dr. Marcelo S.
Agana, Sr. by filing a complaint for Quieting of Title and Damages with the Court of First Instance of Laguna on February 3,
1977, demanding that their title over the questioned land be fortified by a declaration of ownership in their favor and avoiding the
af/recited Deed of Conveyance and Release of Claim (Record on Appeal, pp. 1-7). Accordingly, private respondents in their
answer countered that the Deed of Sale in Tagalog and petitioners' title over the land be declared void ab initio, among other
demands (Record on Appeal, pp. 16-35).
On January 20, 1982, the Court of First Instance of Laguna rendered its decision declaring the deed of sale of August 10, 1937,
as well as the reconstituted transfer certificate of title of petitioners, void ab initio Record on Appeal, pp. 208-216).
The dispositive portion of said decision (Record on Appeal, pp. 215-216) reads as follows:
WHEREFORE, judgment is hereby rendered in favor of the defendants and against the plaintiffs, as follows:
a. declaring as null and void the private document dated August 10, 1937 written in Tagalog (Exhibit B);
b. declaring as null and void plaintiffs' reconstituted Transfer Certificate of Title RT-6293 (No. 23350)
(Exhibit F) and ordering the Register of Deeds of Laguna to issue a new reconstituted or to reinstate
Original Certificate of Title No. 2262 issued on April 2, 1924 in the name of Pedro Villanueva within thirty
(30) days from the finality of this decision;
c. declaring the heirs of Pedro Villanueva as the owners of the property in litigation and ordering the
plaintiffs and her agents and those acting for in their behalf to vacate the land in question and surrender
the possession of the same to the heirs of the late Pedro Villanueva thru Marta V. Agana;
d. declaring all buildings; plantings and improvements introduced by the plaintiffs forfeited in favor of' the
defendants:
e. ordering plaintiffs, jointly and severally, to pay the defendants the sum of P10,000.00 as moral and
exemplary damages;
f. ordering plaintiffs, jointly and severally, to pay defendants the sum of P5,000.00 as and for attorney's
fees: and
g. ordering plaintiffs, jointly and severally, to pay defendants the sum of P5,000.00 as litigation expenses;
and costs of suit.
SO ORDERED.
Thus, petitioners filed notice of appeal on February 10, 1982, followed by an appeal made to the Intermediate Appellate Court.
However, the Intermediate Appellate Court, on May 22, 1984, affirmed in toto the decision of the trial court.
Hence, this petition.
On August 30, 1984, the Court in its Resolution without giving due course to the petition required the respondents to comment
on the said petition (Rollo, p. 50). However, the counsel for private respondents failed to file comment on the petition for review
on certiorari within the period which expired on September 17, 1984. Thus, in the Resolution of January 7, 1985 the Court,
required counsel for petitioners to show cause why disciplinary action should not be taken against him (Rollo, p. 51).
On February 23, 1985 respondents filed their comment (Rollo, p. 57). Considering respondents' comment as answer the petition
was given due course and the parties were required to submit their respective memoranda (Rollo, p. 104).
Private respondents and petitioners filed their respective memoranda on May 18, 1985 (Rollo, p. 117) and on June 7, 1985
(Rollo, p. 143) respectively. On July 1, 1985, the Court resolved to consider the case submitted for deliberation (Rollo, p. 168).
Petitioners, however filed a Supplemental Memorandum, with leave of court on May 18, 1987 (Rollo, p. 169) which was noted
by the court in its resolution dated June 19, 1987 (Rollo, p. 188).
In its petition petitioners raised the following assignment of errors, to wit:
I
THE TRIAL COURT ERRED IN HOLDING THAT EXHIBIT B DOES NOT TRANSFER OWNERSHIP, THE SAME BEING NULL
AND VOID.
II
THE TRIAL COURT ERRED IN HOLDING THAT DEFENDANTS-APPELLANTS ARE NOT GUILTY OF LACHES.
III
THE TRIAL COURT ERRED IN HOLDING THAT PLAINTIFF- APPELLANTS CANNOT ACQUIRE OWNERSHIP OF SUBJECT
LAND BY PRESCRIPTION UPON THE PRINCIPLE THAT NO TITLE TO REGISTERED LAND IN DEROGATION OF THAT OF
THE REGISTERED OWNER SHALL BE ACQUIRED BY PRESCRIPTION.
IV
THE TRIAL COURT ERRED IN NOT HOLDING THAT STATUTE OF LIMITATION HAS SET INTO THIS CASE; AND,
V
THE TRIAL COURT ERRED IN DECLARING TRANSFER CERTIFlCATE OF TITLE NO. RT-6293 AS NULL AND VOID.
The pivotal issue in this case is whether or not there was a valid reconstitution of Transfer Certificate of Title No. RT-6293 (No.
23350) issued in the names of petitioners.
It is admitted that the land in question is formerly covered by Original Certificate of Title No. 2262, issued in the name of Pedro
Villanueva and that the cancellation of said OCT No. 2262 and the issuance of the reconstituted Transfer Certificate of Title No.
RT-6293 (No. 23350) are based either on the Affidavit for Reconstitution of Teresa Villanueva and not of Pedro Villanueva, or

the unnotarized deed of sale of August 10, 1937 (Annex "B" for plaintiffs), held void by the lower court and by the Court of
Appeals. As a consequence TCT No. RT-6293 (No. 23350) was likewise held void ab initio. (Record on Appeal, p. 20).
As to the validity of the Affidavit for Reconstitution, affiant Teresa Villanueva testified on December 19, 1980, that she did not
know anything about the reconstitution of their title as it was their children who took charge of the same and that she never
participated in the said reconstitution. In fact she never appeared before the Notary Public and this testimony was corroborated
by the testimony of Eleuterio Rebenque, entry clerk in the Office of the Register of Deeds who never made any categorical
affirmation that said Teresa Villanueva appeared at said office. (Rollo, p. 43).
Consequently, the crux of the matter now centers on whether or not the unnotarized deed of sale purportedly executed on
August 10, 1937 by the primitive owner Pedro Villanueva, in favor of petitioners, can be considered as a valid instrument for
effecting the alienation by way of sale of a parcel of land registerd under the Torrens System. Corollary thereto, it becomes
necessary to examine other matters surrounding the execution of the alleged document of sale (Exhibit B).
Petitioners claim that the sale although not in a public document, is nevertheless valid and binding citing this Court's rulings in
the cases of Cauto v. Cortes, 8 Phil. 459, 460; Guerrero v. Miguel, 10 Phil. 52, 53; Bucton v. Gabar 55 SCRA 499 wherein this
Court ruled that even a verbal contract of sale of real estate produces legal effects between the parties.
The contention is unmeritorious.
As the respondent court aptly stated in its decision:
True, as argued by appellants, a private conveyance of registered property is valid as between the parties. However,
the only right the vendee of registered property in a private document is to compel through court processes the
vendor to execute a deed of conveyance sufficient in law for purposes of registration. Plaintiffs-appellants' reliance
on Article 1356 of the Civil Code is unfortunate. The general rule enunciated in said Art. 1356 is that contracts are
obligatory, in whatever form they may have been entered, provided all the essential requisites for their validity are
present. The next sentence provides the exception, requiring a contract to be in some form when the law so requires
for validity or enforceability. Said law is Section 127 of Act 496 which requires, among other things, that the
conveyance be executed "before the judge of a court of record or clerk of a court of record or a notary public or a
justice of the peace, who shall certify such acknowledgment substantially in form next hereinafter stated."
Such law was violated in this case. The action of the Register of Deeds of Laguna in allowing the registration of the
private deed of sale was unauthorized and did not lend a bit of validity to the defective private document of sale.
With reference to the special law, Section 127 of the Land Registration Act, Act 496 (now Sec. 112 of P.D. No. 1529) provides:
Sec. 127. Deeds of Conveyance, ... affecting lands, whether registered under this act or unregistered shall be
sufficient in law when made substantially in accordance with the following forms, and shall be as effective to convey,
encumber, ... or bind the lands as though made in accordance with the more prolix forms heretofore in use:
Provided, That every such instrument shall be signed by the person or persons executing the same, in the presence
of two witnesses, who shall sign the instrument as witnesses to the execution thereof, and shall be acknowledged to
be his or their free act and deed by the person or persons executing the same, before the judge of a court of record
or clerk of a court of record, or a notary public, or a justice of the peace, who shall certify to such acknowledgement
substantially in the form next hereinafter stated. (Emphasis supplied).
It is therefore evident that Exhibit " E " in the case at bar is definitely not registerable under the Land Registration Act.
Likewise noteworthy is the case of Pornellosa and Angels v. Land Tenure Administration and Guzman, 110 Phil. 986, where the
Court ruled:
The deed of sale (Exhibit A), allegedly executed by Vicente San Jose in favor of Pornellosa is a mere private
document and does not conclusively establish their right to the parcel of land. WhiIe it is valid and binding upon the
parties with respect to the sale of the house erected thereon, yet it is not sufficient to convey title or any right to the
residential lot in litigation. Acts and contracts which have for their object the creation, transmission, modification or
extinguishment of real rights over immovable property must appear in a public document.
Upon consideration of the facts and circumstances surrounding the execution of the assailed document, the trial court found
that said private document (Exhibit "B") was null and void and that it was signed by somebody else not Pedro Villanueva. Such
findings of fact besides being based on the records, were sustained by the Court of Appeals.
The contention that ownership over registered property may be acquired by prescription or adverse possession is absolutely
without merit. No title to registered land in derogation of that of the registered owner shall be acquired by prescription or adverse
possession. Prescription is unavailing not only against the registered owner but also against his hereditary successors (Umbay
vs. Alecha, 135 SCRA 427 [1985]). The right to recover possession of registered land is imprescriptible because possession is a
mere consequence of ownership (Umbay vs. Alecha,supra, citing Atun v. Nuuz 97 Phil. 762; Manlapas and Tolentino v.
Llorente, 48 Phil. 298, 308: J.M. Tuazon & Co., Inc. v. Aguirre, 117 Phil. 110, 113-114) where land has been registered under the
Torrens System (Alarcon v. Bidin, 120 SCRA 390; Umbay v. Alecha, supra) because the efficacy and integrity of the Torrens
System must be protected (Director of Lands v. CA, 120 SCRA 370). As prescription is rightly regarded as a statute of repose
whose objective is to suppress fraudulent and stale claims from springing up at great distances of time and suprising the parties
or their representatives when the facts have become obscure from the lapse of time or the defective memory or death or
removal of witnesses ( Senoan v. Sorongon, 136 SCRA 407 [1985]).
In the matter of laches, the Court aptly stated in the case of Marcelo Sotto v. Pilar Teves, et al., 86 SCRA 155 [1978] that "in
determining whether a delay in seeking to enforce a right constitutes laches, the existence of a confidential relationship between
the parties is an important circumstance for consideration. A delay under such circumstance is not as strictly regarded as where
the parties are strangers to each other. The doctrine of laches is not strictly applied between near relatives, and the fact that the
parties are connected by ties of blood or marriage tends to excuse an otherwise unreasonable delay."
In the case of Esso Standard Eastern, Inc. v. Alfonso Lim, 123 SCRA 464, 480 [1983]), the Court ruled that laches cannot be
asserted by a mere possessor without claim of title, legal or equitable because for laches to exist, there should be a showing of
delay in asserting the complainant's right. The complainant should have knowledge or notice of the defendant's conduct and an
opportunity to institute a suit. Delay is not counted from the date the lot was sold to the buyer but from the time of entry of the
defendant or from the time the complainant came to know of the occupancy for that is the only time it could possibly have
demanded that he get out of the premises or could have instituted a suit. In the case at bar, it will be noted that what transpired
was an administrative reconstitution, essentially ex-parte and without notice, thereby lending credence to the claim that private
respondent Marta Agana was unaware of such reconstitution and possession until she discovered the same in the Office of the
Register of Deeds in 1976. As such it cannot be claimed that she slept on her right as from that time on, it is undeniable that she
filed her adverse claim on the said lot.
After a careful perusal of the case, there appears to be no cogent reason to disturb the findings of fact of the Court of Appeals
which affirmed the findings of the trial court.
PREMISES CONSIDERED, the petition is DENIED and the assailed decision of the Intermediate Appellate Court is AFFIRMED.
G.R. No. L-34583

October 22, 1931

THE BANK OF THE PHILIPPINE ISLANDS, administrator of the estate of the late Adolphe Oscar Schuetze,plaintiffappellant,
vs.
JUAN POSADAS, JR., Collector of Internal Revenue, defendant-appellee.
Araneta, De Joya, Zaragoza and Araneta for appellant.
Attorney-General Jaranilla for appellee.

VILLA-REAL, J.:
The Bank of the Philippine Islands, as administrator of the estate of the deceased Adolphe Oscar Schuetze, has appealed to
this court from the judgment of the Court of First Instance of Manila absolving the defendant Juan Posadas, Jr., Collector of
Internal Revenue, from the complaint filed against him by said plaintiff bank, and dismissing the complaint with costs.
The appellant has assigned the following alleged errors as committed by the trial court in its judgment, to wit:
1. The lower court erred in holding that the testimony of Mrs. Schuetze was inefficient to established the domicile of her
husband.
2. The lower court erred in holding that under section 1536 of the Administrative Code the tax imposed by the defendant is
lawful and valid.
3. The lower court erred in not holding that one-half () of the proceeds of the policy in question is community property
and that therefore no inheritance tax can be levied, at least on one-half () of the said proceeds.
4. The lower court erred in not declaring that it would be unconstitutional to impose an inheritance tax upon the insurance
policy here in question as it would be a taking of property without due process of law.
The present complaint seeks to recover from the defendant Juan Posadas, Jr., Collector of Internal Revenue, the amount of
P1,209 paid by the plaintiff under protest, in its capacity of administrator of the estate of the late Adolphe Oscar Schuetze, as
inheritance tax upon the sum of P20,150, which is the amount of an insurance policy on the deceased's life, wherein his own
estate was named the beneficiary.
At the hearing, in addition to documentary and parol evidence, both parties submitted the following agreed statement of facts of
the court for consideration:
It is hereby stipulated and agreed by and between the parties in the above-entitled action through their respective
undersigned attorneys:
1. That the plaintiff, Rosario Gelano Vda. de Schuetze, window of the late Adolphe Oscar Schuetze, is of legal age, a
native of Manila, Philippine Islands, and is and was at all times hereinafter mentioned a resident of Germany, and at the
time of the death of her husband, the late Adolphe Oscar Schuetze, she was actually residing and living in Germany;
2. That the Bank of the Philippine Islands, is and was at all times hereinafter mentioned a banking institution duly
organized and existing under and by virtue of the laws of the Philippine Islands;
3. That on or about August 23, 1928, the herein plaintiff before notary public Salvador Zaragoza, drew a general power
appointing the above-mentioned Bank of the Philippine Islands as her attorney-in-fact, and among the powers conferred to
said attorney-in-fact was the power to represent her in all legal actions instituted by or against her;
4. That the defendant, of legal age, is and at all times hereinafter mentioned the duly appointed Collector of Internal
Revenue with offices at Manila, Philippine Islands;
5. That the deceased Adolphe Oscar Schuetze came to the Philippine Islands for the first time of March 31, 1890, and
worked in the several German firms as a mere employee and that from the year 1903 until the year 1918 he was partner
in the business of Alfredo Roensch;
6. That from 1903 to 1922 the said Adolphe Oscar Schuetze was in the habit of making various trips to Europe;
7. That on December 3, 1927, the late Adolphe Oscar Schuetze coming from Java, and with the intention of going to
Bremen, landed in the Philippine Islands where he met his death on February 2, 1928;
8. That on March 31, 1926, the said Adolphe Oscar Schuetze, while in Germany, executed a will, in accordance with its
law, wherein plaintiff was named his universal heir;
9. That the Bank of the Philippine Islands by order of the Court of First Instance of Manila under date of May 24, 1928,
was appointed administrator of the estate of the deceased Adolphe Oscar Schuetze;
10. That, according to the testamentary proceedings instituted in the Court of First Instance of Manila, civil case No.
33089, the deceased at the time of his death was possessed of not only real property situated in the Philippine Islands,
but also personal property consisting of shares of stock in nineteen (19) domestic corporations;
11. That the fair market value of all the property in the Philippine Islands left by the deceased at the time of his death in
accordance with the inventory submitted to the Court of First Instance of Manila, civil case No. 33089, was P217,560.38;
12. That the Bank of the Philippine Islands, as administrator of the estate of the deceased rendered its final account on
June 19, 1929, and that said estate was closed on July 16, 1929;
13. That among the personal property of the deceased was found life-insurance policy No. 194538 issued at Manila,
Philippine Islands, on January 14, 1913, for the sum of $10,000 by the Sun Life Assurance Company of Canada, Manila
branch, a foreign corporation duly organized and existing under and by virtue of the laws of Canada, and duly authorized
to transact business in the Philippine Islands;
14. That in the insurance policy the estate of the said Adolphe Oscar Schuetze was named the beneficiary without any
qualification whatsoever;
15. That for five consecutive years, the deceased Adolphe Oscar Schuetze paid the premiums of said policy to the Sun
Life Assurance Company of Canada, Manila branch;
16. That on or about the year 1918, the Sun Life Assurance Company of Canada, Manila branch, transferred said policy to
the Sun Life Assurance Company of Canada, London branch;
17. That due to said transfer the said Adolphe Oscar Schuetze from 1918 to the time of his death paid the premiums of
said policy to the Sun Life Assurance Company of Canada, London Branch;

18. That the sole and only heir of the deceased Adolphe Oscar Schuetze is his widow, the plaintiff herein;
19. That at the time of the death of the deceased and at all times thereafter including the date when the said insurance
policy was paid, the insurance policy was not in the hands or possession of the Manila office of the Sun Life Assurance
Company of Canada, nor in the possession of the herein plaintiff, nor in the possession of her attorney-in-fact the Bank of
the Philippine Islands, but the same was in the hands of the Head Office of the Sun Life Assurance Company of Canada,
at Montreal, Canada;
20. That on July 13, 1928, the Bank of the Philippine Islands as administrator of the decedent's estate received from the
Sun Life Assurance Company of Canada, Manila branch, the sum of P20,150 representing the proceeds of the insurance
policy, as shown in the statement of income and expenses of the estate of the deceased submitted on June 18, 1929, by
the administrator to the Court of First Instance of Manila, civil case No. 33089;
21. That the Bank of the Philippine Islands delivered to the plaintiff herein the said sum of P20,150;
22. That the herein defendant on or about July 5, 1929, imposed an inheritance tax upon the transmission of the proceeds
of the policy in question in the sum of P20,150 from the estate of the late Adolphe Oscar Schuetze to the sole heir of the
deceased, or the plaintiff herein, which inheritance tax amounted to the sum of P1,209;
23. That the Bank of the Philippine Islands as administrator of the decedent's estate and as attorney-in-fact of the herein
plaintiff, having been demanded by the herein defendant to pay inheritance tax amounting to the sum of P1,209, paid to
the defendant under protest the above-mentioned sum;
24. That notwithstanding the various demands made by plaintiff to the defendant, said defendant has refused and refuses
to refund to plaintiff the above mentioned sum of P1,209;
25. That plaintiff reserves the right to adduce evidence as regards the domicile of the deceased, and so the defendant, the
right to present rebuttal evidence;
26. That both plaintiff and defendant submit this stipulation of facts without prejudice to their right to introduce such
evidence, on points not covered by the agreement, which they may deem proper and necessary to support their
respective contentions.
In as much as one of the question raised in the appeal is whether an insurance policy on said Adolphe Oscar Schuetze's life
was, by reason of its ownership, subject to the inheritance tax, it would be well to decide first whether the amount thereof is
paraphernal or community property.
According to the foregoing agreed statement of facts, the estate of Adolphe Oscar Schuetze is the sole beneficiary named in the
life-insurance policy for $10,000, issued by the Sun Life Assurance Company of Canada on January 14, 1913. During the
following five years the insured paid the premiums at the Manila branch of the company, and in 1918 the policy was transferred
to the London branch.
The record shows that the deceased Adolphe Oscar Schuetze married the plaintiff-appellant Rosario Gelano on January 16,
1914.
With the exception of the premium for the first year covering the period from January 14, 1913 to January 14, 1914, all the
money used for paying the premiums, i. e., from the second year, or January 16, 1914, or when the deceased Adolphe Oscar
Schuetze married the plaintiff-appellant Rosario Gelano, until his death on February 2, 1929, is conjugal property inasmuch as it
does not appear to have exclusively belonged to him or to his wife (art. 1407, Civil Code). As the sum of P20,150 here in
controversy is a product of such premium it must also be deemed community property, because it was acquired for a valuable
consideration, during said Adolphe Oscar Schuetze's marriage with Rosario Gelano at the expense of the common fund (art.
1401, No. 1, Civil Code), except for the small part corresponding to the first premium paid with the deceased's own money.
In his Commentaries on the Civil Code, volume 9, page 589, second edition, Manresa treats of life insurance in the following
terms, to wit:
The amount of the policy represents the premiums to be paid, and the right to it arises the moment the contract is
perfected, for at the moment the power of disposing of it may be exercised, and if death occurs payment may be
demanded. It is therefore something acquired for a valuable consideration during the marriage, though the period of its
fulfillment, depend upon the death of one of the spouses, which terminates the partnership. So considered, the question
may be said to be decided by articles 1396 and 1401: if the premiums are paid with the exclusive property of husband or
wife, the policy belongs to the owner; if with conjugal property, or if the money cannot be proved as coming from one or
the other of the spouses, the policy is community property.
The Supreme Court of Texas, United States, in the case of Martin vs. Moran (11 Tex. Civ. A., 509) laid down the following
doctrine:
COMMUNITY PROPERTY LIFE INSURANCE POLICY. A husband took out an endowment life insurance policy on
his life, payable "as directed by will." He paid the premiums thereon out of community funds, and by his will made the
proceeds of the policy payable to his own estate. Held, that the proceeds were community estate, one-half of which
belonged to the wife.
In In re Stan's Estate, Myr. Prob. (Cal.), 5, the Supreme Court of California laid down the following doctrine:
A testator, after marriage, took out an insurance policy, on which he paid the premiums from his salary. Held that the
insurance money was community property, to one-half of which, the wife was entitled as survivor.
In In re Webb's Estate, Myr. Prob. (Cal.), 93, the same court laid down the following doctrine:
A decedent paid the first third of the amount of the premiums on his life-insurance policy out of his earnings before
marriage, and the remainder from his earnings received after marriage. Held, that one-third of the policy belonged to his
separate estate, and the remainder to the community property.
Thus both according to our Civil Code and to the ruling of those North American States where the Spanish Civil Code once
governed, the proceeds of a life-insurance policy whereon the premiums were paid with conjugal money, belong to the conjugal
partnership.
The appellee alleges that it is a fundamental principle that a life-insurance policy belongs exclusively to the beneficiary upon the
death of the person insured, and that in the present case, as the late Adolphe Oscar Schuetze named his own estate as the
sole beneficiary of the insurance on his life, upon his death the latter became the sole owner of the proceeds, which therefore
became subject to the inheritance tax, citing Del Val vs. Del Val (29 Phil., 534), where the doctrine was laid down that an heir
appointed beneficiary to a life-insurance policy taken out by the deceased, becomes the absolute owner of the proceeds of such
policy upon the death of the insured.
The estate of a deceased person cannot be placed on the same footing as an individual heir. The proceeds of a life-insurance
policy payable to the estate of the insured passed to the executor or administrator of such estate, and forms part of its assets

(37 Corpus Juris, 565, sec. 322); whereas the proceeds of a life-insurance policy payable to an heir of the insured as
beneficiary belongs exclusively to said heir and does not form part of the deceased's estate subject to administrator. (Del Val vs.
Del Val, supra; 37 Corpus Juris, 566, sec. 323, and articles 419 and 428 of the Code of Commerce.)
Just as an individual beneficiary of a life-insurance policy taken out by a married person becomes the exclusive owner of the
proceeds upon the death of the insured even if the premiums were paid by the conjugal partnership, so, it is argued, where the
beneficiary named is the estate of the deceased whose life is insured, the proceeds of the policy become a part of said estate
upon the death of the insured even if the premiums have been paid with conjugal funds.
In a conjugal partnership the husband is the manager, empowered to alienate the partnership property without the wife's
consent (art. 1413, Civil Code), a third person, therefore, named beneficiary in a life-insurance policy becomes the absolute
owner of its proceeds upon the death of the insured even if the premiums should have been paid with money belonging to the
community property. When a married man has his life insured and names his own estate after death, beneficiary, he makes no
alienation of the proceeds of conjugal funds to a third person, but appropriates them himself, adding them to the assets of his
estate, in contravention of the provisions of article 1401, paragraph 1, of the Civil Code cited above, which provides that "To the
conjugal partnership belongs" (1) Property acquired for a valuable consideration during the marriage at the expense of the
common fund, whether the acquisition is made for the partnership or for one of the spouses only." Furthermore, such
appropriation is a fraud practised upon the wife, which cannot be allowed to prejudice her, according to article 1413, paragraph
2, of said Code. Although the husband is the manager of the conjugal partnership, he cannot of his own free will convert the
partnership property into his own exclusive property.
As all the premiums on the life-insurance policy taken out by the late Adolphe Oscar Schuetze, were paid out of the conjugal
funds, with the exceptions of the first, the proceeds of the policy, excluding the proportional part corresponding to the first
premium, constitute community property, notwithstanding the fact that the policy was made payable to the deceased's estate, so
that one-half of said proceeds belongs to the estate, and the other half to the deceased's widow, the plaintiff-appellant Rosario
Gelano Vda. de Schuetze.
The second point to decide in this appeal is whether the Collector of Internal Revenue has authority, under the law, to collect the
inheritance tax upon one-half of the life-insurance policy taken out by the late Adolphe Oscar Schuetze, which belongs to him
and is made payable to his estate.
According to the agreed statement of facts mentioned above, the plaintiff-appellant, the Bank of the Philippine Islands, was
appointed administrator of the late Adolphe Oscar Schuetze's testamentary estate by an order dated March 24, 1928, entered
by the Court of First Instance of Manila. On July 13, 1928, the Sun Life Assurance Company of Canada, whose main office is in
Montreal, Canada, paid Rosario Gelano Vda. de Schuetze upon her arrival at Manila, the sum of P20,150, which was the
amount of the insurance policy on the life of said deceased, payable to the latter's estate. On the same date Rosario Gelano
Vda. de Schuetze delivered the money to said Bank of the Philippine Islands, as administrator of the deceased's estate, which
entered it in the inventory of the testamentary estate, and then returned the money to said widow.
Section 1536 of the Administrative Code, as amended by section 10 of Act No. 2835 and section 1 of Act No. 3031, contains the
following relevant provision:
SEC. 1536. Conditions and rate of taxation. Every transmission by virtue of inheritance, devise, bequest, gift mortis
causa or advance in anticipation of inheritance, devise, or bequest of real property located in the Philippine Islands and
real rights in such property; of any franchise which must be exercised in the Philippine Islands; of any shares, obligations,
or bonds issued by any corporation or sociedad anonima organized or constituted in the Philippine Islands in accordance
with its laws; of any shares or rights in any partnership, business or industry established in the Philippine Islands or of any
personal property located in the Philippine Islands shall be subject to the following tax:
xxx

xxx

xxx

In as much as the proceeds of the insurance policy on the life of the late Adolphe Oscar Schuetze were paid to the Bank of the
Philippine Islands, as administrator of the deceased's estate, for management and partition, and as such proceeds were turned
over to the sole and universal testamentary heiress Rosario Gelano Vda. de Schuetze, the plaintiff-appellant, here in Manila, the
situs of said proceeds is the Philippine Islands.
In his work "The Law of Taxation," Cooley enunciates the general rule governing the levying of taxes upon tangible personal
property, in the following words:
GENERAL RULE. The suits of tangible personal property, for purposes of taxation may be where the owner is
domiciled but is not necessarily so. Unlike intangible personal property, it may acquire a taxation situs in a state other than
the one where the owner is domiciled, merely because it is located there. Its taxable situs is where it is more or less
permanently located, regardless of the domicile of the owner. It is well settled that the state where it is more or less
permanently located has the power to tax it although the owner resides out of the state, regardless of whether it has been
taxed for the same period at the domicile of the owner, provided there is statutory authority for taxing such property. It is
equally well settled that the state where the owner is domiciled has no power to tax it where the property has acquired an
actual situs in another state by reason of its more or less permanent location in that state. ... (2 Cooley, The Law of
Taxation, 4th ed., p. 975, par. 451.)
With reference to the meaning of the words "permanent" and "in transit," he has the following to say:
PERMANENCY OF LOCATION; PROPERTY IN TRANSIT. In order to acquire a situs in a state or taxing district so as
to be taxable in the state or district regardless of the domicile of the owner and not taxable in another state or district at
the domicile of the owner, tangible personal property must be more or less permanently located in the state or district. In
other words, the situs of tangible personal property is where it is more or less permanently located rather than where it is
merely in transit or temporarily and for no considerable length of time. If tangible personal property is more or less
permanently located in a state other than the one where the owner is domiciled, it is not taxable in the latter state but is
taxable in the state where it is located. If tangible personal property belonging to one domiciled in one state is in another
state merely in transitu or for a short time, it is taxable in the former state, and is not taxable in the state where it is for the
time being. . . . .
Property merely in transit through a state ordinarily is not taxable there. Transit begins when an article is committed to a
carrier for transportation to the state of its destination, or started on its ultimate passage. Transit ends when the goods
arrive at their destination. But intermediate these points questions may arise as to when a temporary stop in transit is such
as to make the property taxable at the place of stoppage. Whether the property is taxable in such a case usually depends
on the length of time and the purpose of the interruption of transit. . . . .
. . . It has been held that property of a construction company, used in construction of a railroad, acquires a situs at the
place where used for an indefinite period. So tangible personal property in the state for the purpose of undergoing a partial
finishing process is not to be regarded as in the course of transit nor as in the state for a mere temporary purpose. (2
Cooley, The Law of Taxation, 4th ed., pp. 982, 983 and 988, par. 452.)
If the proceeds of the life-insurance policy taken out by the late Adolphe Oscar Schuetze and made payable to his estate, were
delivered to the Bank of the Philippine Islands for administration and distribution, they were not in transit but were more or less
permanently located in the Philippine Islands, according to the foregoing rules. If this be so, half of the proceeds which is
community property, belongs to the estate of the deceased and is subject to the inheritance tax, in accordance with the legal

provision quoted above, irrespective of whether or not the late Adolphe Oscar Schuetze was domiciled in the Philippine Islands
at the time of his death.
By virtue of the foregoing, we are of opinion and so hold: (1) That the proceeds of a life-insurance policy payable to the
insured's estate, on which the premiums were paid by the conjugal partnership, constitute community property, and belong onehalf to the husband and the other half to the wife, exclusively; (2) that if the premiums were paid partly with paraphernal and
partly conjugal funds, the proceeds are likewise in like proportion paraphernal in part and conjugal in part; and (3) that the
proceeds of a life-insurance policy payable to the insured's estate as the beneficiary, if delivered to the testamentary
administrator of the former as part of the assets of said estate under probate administration, are subject to the inheritance tax
according to the law on the matter, if they belong to the assured exclusively, and it is immaterial that the insured was domiciled
in these Islands or outside.
1awphil.net

Wherefore, the judgment appealed from is reversed, and the defendant is ordered to return to the plaintiff the one-half of the tax
collected upon the amount of P20,150, being the proceeds of the insurance policy on the life of the late Adolphe Oscar
Schuetze, after deducting the proportional part corresponding to the first premium, without special pronouncement of costs. So
ordered.

G.R. No. 70082 August 19, 1991

SPOUSES RICKY WONG and ANITA CHAN, LEONARDO JOSON, JUANITO SANTOS, EMERITO SICAT and CONRADO
LAGMAN, petitioners,
vs.
HON. INTERMEDIATE APPELLATE COURT and ROMARICO HENSON, respondents.
Feliciano C. Tumale for petitioners.
Benjamin Dadios and Bausa, Ampil, Suarez, Paredes & Bausa for private respondent.

FERNAN, C.J.:p
Submitted for adjudication in the instant petition for review on certiorari is the issue of whether or not the execution of a decision
in an action for collection of a sum of money may be nullified on the ground that the real properties levied upon and sold at
public auction are the alleged exclusive properties of a husband who did not participate in his wife's business transaction from
which said action stemmed.
Private respondent Romarico Henson married Katrina Pineda on January 6, 1964.

They have three children but even during the early years of
their marriage, Romarico and Katrina had been most of the time living separately. The former stayed in Angeles City while the latter lived in Manila. During the marriage or on
January 6, 1971, Romarico bought a 1,787 square-meter parcel of land in Angeles City for P11,492 from his father, Dr. Celestino L. Henson 2 with money borrowed from an
officemate. His father need the amount for investments in Angeles City and Palawan. 3

Meanwhile, in Hongkong sometime in June 1972, Katrina entered into an agreement with Anita Chan whereby the latter
consigned to Katrina pieces of jewelry for sale valued at 199,895 Hongkong dollars or P321,830.95. 4 When Katrina failed to return the
pieces of jewelry within the 20-day period agreed upon, Anita Chan demanded payment of their value.

On September 18, 1972, Katrina issued in favor of Anita Chan a check for P55,000 which, however, was dishonored for lack of
funds. Hence, Katrina was charged with estafa before the then Court of First Instance of Pampanga and Angeles City, Branch
IV. 5 After trial, the lower court rendered a decision dismissing the case on the ground that Katrina's liability was not criminal but civil in nature as no estafa was committed by the
issuance of the check in payment of a pre-existing obligation.

In view of said decision, Anita Chan and her husband Ricky Wong filed against Katrina and her husband Romarico Henson, an
action for collection of a sum of money also in the same branch of the aforesaid court. 7 The records of the case show that Atty. Gregorio Albino,
Jr. filed an answer with counterclaim but only in behalf of Katrina. When the case was called for pre-trial, Atty. Albino once again appeared as counsel for Katrina only. While it is
true that during subsequent hearings, Atty. Expedite Yumul, who collaborated with Atty. Albino, appeared for the defendants, it is not shown on record that said counsel also
represented Romarico. In fact, a power of attorney which Atty. Albino produced during the trial, showed that the same was executed solely by Katrina. 8

After trial, the court promulgated a decisions

in favor of the Wongs. It ordered Katrina and Romarico Henson to pay the Wongs HK$199,895.00 or P321,830.95
with legal interest from May 27, 1975, the date of filing of the complaint, until fully paid; P20,000 as expenses for litigation; P15,000 as attorney's fees, and the costs of the suit.

A writ of execution was thereafter issued. Levied upon were four lots in Angeles City covered by Transfer Certificates of Title
Nos. 30950, 30951, 30952 and 30953 all in the name of Romarico Henson ... married to Katrina Henson. 10
The public auction sale was first set for October 30, 1977 but since said date was declared a public holiday, Deputy Sheriff
Emerito Sicat reset the sale to November 11, 1977. On said date, the following properties registered in the name of Romarico
Henson "married to Katrina Henson" were sold at public auction: (a) two parcels of land covered by Transfer Certificates of Title
Nos. 30950 and 30951 with respective areas of 293 and 289 square meters at P145,000 each to Juanito L. Santos, 11 and (b) two
parcels of land covered by Transfer Certificates of Title Nos. 30952 and 30953 with respective areas of 289 and 916 square meters in the amount of P119,000.00 to Leonardo B.
Joson. 12

After the inscription on Transfer Certificate of Title No. 30951 of the levy on execution of the judgment in Civil Case No. 2224,
the property covered by said title was extrajudicially foreclosed by the Rural Bank of Porac, Pampanga on account of the
mortgage loan of P8,000 which Romarico and Katrina had obtained from said bank. The property was sold by the sheriff to the
highest bidder for P57,000 on September 9, 1977. On September 14, 1978, Juanito Santos, who had earlier bought the same
property at public auction on November 11, 1977, redeemed it by paying the sum of P57,000 plus the legal interest of P6,840.00
or a total amount of P63,840.00. 13
About a month before such redemption or on August 8, 1 978, Romarico filed an action for the annulment of the decision in Civil
Case No. 2224 as well as the writ of execution, levy on execution and the auction sale therein in the same Court of First
Instance. 14 Romarico alleged that he was "not given his day in court" because he was not represented by counsel as Attys. Albino and Yumul appeared solely for Katrina; that
although he did not file an answer to the complaint, he was not declared in default in the case; that while Atty. Albino received a copy of the decision, he and his wife were never
personally served a copy thereof; that he had nothing to do with the business transactions of Katrina as he did not authorize her to enter into such transactions; and that the
properties levied on execution and sold at public auction by the sheriff were his capital properties and therefore, as to him, all the proceedings had in the case were null and void.

On November 10, 1978, the lower court issued an order restraining the Register of Deeds of Angeles City from issuing the final
bill of sale of Transfer Certificates of Title Nos. 30950 and 30951 in favor of Juanito Santos and Transfer Certificates of Title
Nos. 30952 and 30953 in favor of Leonardo Joson until further orders of the court. 15On January 22, 1979, upon motion of Romarico, the court
issued a writ of preliminary injunction enjoining the sheriff from approving the final bill of sale of the land covered by the aforementioned certificates of title and the Register of
Deeds of Angeles City from registering said certificates of title in the names of Santos and Joson until the final outcome of the case subject to Romarico's posting of a bond in the
amount of P321,831.00. 16

After trial on the merits, the lower court

17

rendered a decision holding that Romarico was indeed not given his day in court as he was not represented by counsel
nor was he notified of the hearings therein although he was never declared in default. Noting that the complaint in Civil Case No. 2224 as well as the testimonial and documentary
evidence adduced at the trial in said case do not show that Romarico had anything to do with the transactions between Katrina and Anita Chan, the court ruled that the judgment in
Civil Case No. 2224 "is devoid of legal or factual basis which is not even supported by a finding of fact or ratio decidendi in the body of the decision, and may be declared null and
void ... pursuant to a doctrine laid down by the Supreme Court to the effect that the Court of First Instance or a branch thereof, has authority and jurisdiction to try and decide an
action for annulment of a final and executory judgment or order rendered by another court of first instance or of a branch thereof (Gianan vs. Imperial, 55 SCRA 755)." 18

On whether or not the properties lenied upon and sold at public auction may be reconveyed to Romarico, the court, finding that
there was no basis for holding the conjugal partnership liable for the personal indebtedness of Katrina, ruled in favor of
reconveyance in view of the jurisprudence that the interest of the wife in the conjugal partnership property being inchoate and
therefore merely an expectancy, the same may not be sold or disposed of for value until after the liquidation and settlement of
the community assets. The dispositive portion of the decision reads:
WHEREFORE, and in view of the foregoing, judgment is hereby rendered in favor of the plaintiff and against all the
defendants, as follows:
(a) The Decision of the Court of First Instance of Pampanga and Angeles City, Branch IV, rendered in Civil Case No.
2224, entitled "RICKY WONG, ET AL. vs. KATRINA PINEDA HENSON and ROMARICO HENSON", is hereby
declared null and void, only as far as it affects plaintiff herein Romarico Henson;
(b) The Writ of Execution, levy in execution and auction sale of the conjugal property of the spouses Romarico
Henson and Katrina Pineda Henson which were sold at public auction on November 11, 1977, without notice to
plaintiff herein, by Deputy Sheriff Emerito Sicat, are likewise declared null and void and of no force and effect;
(c) Defendants Emerito Sicat and Conrado Lagman, in their official capacity as Sheriff and Register of Deeds,
respectively, are enjoined permanently from issuing and/or registering the corresponding deeds of sale affecting the
property;
(d) The aforementioned buyers are directed to reconvey the property they have thus purchased at public auction to
plaintiff Romarico Henson;

(e) As far as the claim for reimbursement filed by Juanito Santos concerning the redemption of the property covered
by Transfer Certificate of Title No. 30951 from the Rural Bank of Porac, which foreclosed the same extrajudicially, is
concerned, plaintiff Romarico Henson may redeem the same within the period and in the manner prescribed by law,
after the corresponding deed of redemption shall have been registered in the Office of the Registry of Deeds for
Angeles City;
(f) Defendants Spouses Ricky Wong and Anita Chan are, with the exception of the defendants Juanito Santos,
Leonardo Joson, Sheriff and Register of Deeds, are ordered jointly and severally, to pay the plaintiff Romarico
Henson the sum of P10,000.00, corresponding to the expenses of litigation, with legal interest thereon from the time
this suit was filed up to the time the same shall have been paid, plus P5,000.00 for and as attorney's fees, and the
costs of suit; and
(g) The counterclaims respectively filed on behalf of all the defendants in the above-entitled case are hereby
DISMISSED.
SO ORDERED.
The defendants appealed to the then Intermediate Appellate Court. In its decision of January 22, 1985

19

the said court affirmed in toto the


decision of the lower court. It added that as to Romarico, the judgment in Civil Case No. 2224 had not attained finality as the decision therein was not served on him and that he
was not represented by counsel. Therefore, estoppel may not be applied against him as, not having been served with the decision, Romarico did not know anything about it.
Corollarily, there can be no valid writ of execution inasmuch as the decision had not become final as far as Romarico is concerned.

On whether the properties may be levied upon as conjugal properties, the appellate court ruled in the negative. It noted that the
properties are Romarico' s exclusive capital having been bought by him with his own funds. But granting that the properties are
conjugal, they cannot answer for Katrina's obligations as the latter were exclusively hers because they were incurred without the
consent of her husband, they were not for the daily expenses of the family and they did not redound to the benefit of the family.
The court underscored the fact that no evidence has been submitted that the administration of the conjugal partnership had
been transferred to Katrina either by Romarico or by the court before said obligations were incurred.
The appellants filed a motion for reconsideration of the decision of the appellate court but the same was denied for lack of merit
on February 6, 1985. 20
Hence, the instant petition for review on certiorari. Petitioners contend that, inasmuch as the Henson spouses were duly
represented by Atty. Albino as shown by their affidavit of August 25, 1977 wherein they admitted that they were represented by
said counsel until Atty. Yumul took over the actual management and conduct of the case and that Atty. Albino had not withdrawn
as their counsel, the lower court "did not commit an error" in serving a copy of the decision in Civil Case No. 2224 only on Atty.
Albino. Moreover, during the 2-year period between the filing of the complaint in Civil Case No. 2224 and the public auction sale
on November 11, 1977, Romarico remained silent thereby making him in estoppel and guilty of laches.
Petitioners further aver that there being sufficient evidence that the auction sale was conducted in accordance with law, the acts
of the sheriffs concerned are presumed to be regular and valid. But granting that an irregularity consisting of the non-notification
of Romarico attended the conduct of the auction sale, the rights of Santos and Joson who were "mere strangers who
participated as the highest bidders" therein, may not be prejudiced. Santos and Joson bought the properties sincerely believing
that the sheriff was regularly performing his duties and no evidence was presented to the effect that they acted with fraud or that
they connived with the sheriff. However, should the auction sale be nullified, petitioners assert that Romarico should not be
unduly enriched at the expense of Santos and Joson.
The petitioners' theory is that Romarico Henson was guilty of laches and may not now belatedly assert his rights over the
properties because he and Katrina were represented by counsel in Civil Case No. 2224. Said theory is allegedly founded on the
perception that the Hensons were like any other ordinary couple wherein a spouse knows or should know the transactions of
the other spouse which necessarily must be in interest of the family. The factual background of this case, however, takes it out
of said ideal situation.
Romarico and Katrina had in fact been separated when Katrina entered into a business deal with Anita Wong. Thus, when that
business transaction eventually resulted in the filing of Civil Case No. 2224, Romarico acted, or, as charged by petitioners,
failed to act, in the belief that he was not involved in the personal dealings of his estranged wife. That belief was buttressed by
the fact that the complaint itself did not mention or implicate him other than as the husband of Katrina. On whether Romarico
was also represented by Atty. Albino, Katrina's counsel, the courts below found that:
... Atty. Albino filed an Answer with Counterclaims dated July 25, 1975 solely on behalf of defendant Katrina Henson.
The salutary statement in that Answer categorically reads: ... COMES NOW THE DEFENDANT KATRINA HENSON
by and through undersigned counsel, in answer to plaintiffs' complaint respectfully alleges: ... .
That Answer was signed by GREGORIO ALBINO, JR., over the phrase COUNSEL FOR DEFENDANT KATRINA
HENSON.
Again, when Civil Case No. 2224 was called for pre-trial on November 27, 1975, before then Presiding Judge
Bienvenido Ejercito, it is clearly stated on page 2 of the day's stenographic notes, under "APPEARANCES that Atty.
Albino, Jr. appeared as COUNSEL FOR DEFENDANT KATRINA HENSON". And when the case was called, Atty.
Jose Baltazar, Sr. appeared for the plaintiffs while Atty. Albino categorically appeared "FOR DEFENDANT KATRINA
HENSON".
It might be true that in subsequent hearings, Atty. Expedito Yumul 'appeared as counsel for the defendants,' but the
whole trouble is that he never expressly manifested to the Court that he was likewise actually representing
defendant "ROMARICO HENSON", for it cannot be disputed that Atty. Yumul only entered his appearance in
collaboration with Atty. Albino (see p. 2 tsn, January 26, 1976, Espinosa), who in turn entered his initial appearance
during the pre- trial, and through the filing of an Answer, for defendant KATRINA HENSON. As a matter of fact, the
Power of Attorney which Atty. Albino produced during the pre-trial was executed solely by defendant KATRINA
HENSON. Accordingly, as collaborating counsel, Atty. Yumul cannot, by any stretch of the imagination, be
considered as duly authorized to formally appear likewise on behalf of defendant ROMARICO HENSON for whom
principal counsel of record Atty. Gregorio Albino, Jr. never made any formal appearance. On this score, it is not
amiss to state that "A spring cannot rise higher than its source:.
Now, what about that statement in the aforementioned joint affidavit of the spouses KATRINA HENSON and
ROMARICO HENSON, to the effect that our first lawyer in said case was Atty. Gregorio Albino, Jr., and sometime
later Atty. Expedito B. Yumul took over ...
That statement which plaintiff ROMARICO HENSON was made to sign by Atty. Yumul on August 25,1977, after the
filing of this case, allegedly for the purpose of dissolving the writ of execution, as claimed in paragraph XIV of the
complaint herein, and is satisfactorily explained by both plaintiff herein and his wife, while on cross-examination by
Atty. Baltazar, Sr., and We quote:
Q So, the summons directed your filing of your Answer for both of you, your wife and your good self?
A Yes, sir but may I add, I received the summons but I did not file an answer because my wife took a
lawyer and that lawyer I think will protect her interest and my interest being so I did not have nothing to
do in the transaction which is attached to the complaint.' (TSN, Jan. 14, 1980, pp. 52-53).

That plaintiff never appeared in Civil Case No. 2224, nor was he therein represented by counsel was
impliedly admitted by defendants' counsel of records thru a question he propounded on cross, and the
answer given by Katrina Pineda, to wit:
Q How about your husband, do you remember if he physically appeared in that Civil Case No. 2224, will
you tell us if he was represented by counsel as a party defendant?
A No, sir, he did not appear.
Q You are husband and wife, please tell us the reason why you have your own counsel in that case
whereas Romarico Henson did not appear nor a counsel did not appear in that proceedings (TSN, Feb.
25,1980, pp. 6-7).
xxx xxx xxx
A Because that case is my exclusive and personal case, he has nothing to do with that, sir. (TSN, Feb.
25, 1980, p. 9). (Rollo, pp. 17-20)
Hence, laches may not be charged against Romarico because, aside from the fact that he had no knowledge of the transactions
of his estranged wife, he was also not afforded an opportunity to defend himself in Civil Case No. 2224. 21 There is no laches or even
finality of decision to speak of with respect to Romarico since the decision in Civil Case No. 2224 is null and void for having been rendered without jurisdiction for failure to observe
the notice requirements prescribed by law. 22 Failure to notify Romarico may not be attributed to the fact that the plaintiffs in Civil Case No. 2224 acted on the presumption that the
Hensons were still happily married because the complaint itself shows that they did not consider Romarico as a party to the transaction which Katrina undertook with Anita Wong.
In all likelihood, the plaintiffs merely impleaded Romarico as a nominal party in the case pursuant to the provisions of Rule 3, Section 4 of the Rules of Court.

Consequently, the writ of execution cannot be issued against Romarico as he has not yet had his day in court
public auction sale is null and void.
debtor. 25

24

23

and, necessarily, the


Moreover, the power of the court in the execution of judgments extends only over properties unquestionably belonging to the judgment

On the matter of ownership of the properties involved, however, the Court disagrees with the appellate court that the said
properties are exclusively owned by Romarico. Having been acquired during the marriage, they are still presumed to belong to
the conjugal partnership 26 even though Romarico and Katrina had been living
separately. 27

The presumption of the conjugal nature of the properties subsists in the absence of clear, satisfactory and convincing evidence
to overcome said presumption or to prove that the properties are exclusively owned by Romarico. 28 While there is proof that Romarico
acquired the properties with money he had borrowed from an officemate, it is unclear where he obtained the money to repay the loan. If he paid it out of his salaries, then the
money is part of the conjugal assets 29 and not exclusively his. Proof on this matter is of paramount importance considering that in the determination of the nature of a property
acquired by a person during covertrue, the controlling factor is the source of the money utilized in the purchase.

The conjugal nature of the properties notwithstanding, Katrina's indebtedness may not be paid for with them her obligation not
having been shown by the petitioners to be one of the charges against the conjugal partnership. 30In addition to the fact that her rights over
the properties are merely inchoate prior to the liquidation of the conjugal partnership, the consent of her husband and her authority to incur such indebtedness had not been
alleged in the complaint and proven at the trial. 31

Furthermore, under the Civil Code (before the effectivity of the Family Code on August 3, 1988), a wife may bind the conjugal
partnership only when she purchases things necessary for the support of the family or when she borrows money for the purpose
of purchasing things necessary for the support of the family if the husband fails to deliver the proper sum; 32 when the administration of
the conjugal partnership is transferred to the wife by the courts 33 or by the husband 34 and when the wife gives moderate donations for charity. 35 Having failed to establish that any
of these circumstances occurred, the Wongs may not bind the conjugal assets to answer for Katrina's personal obligation to them.

Petitioners' contention that the rights of Santos and Joson as innocent buyers at the public auction sale may not be prejudiced,
is, to a certain extent, valid. After all, in the absence of proof that irregularities attended the sale, the same must be presumed to
have been conducted in accordance with law. There is, however, a peculiar factual circumstance that goes against the grain of
that general presumption the properties levied upon and sold at the public auction do not exclusively belong to the judgment
debtor. Thus, the guiding jurisprudence is as follows:
The rule in execution sales is that an execution creditor acquires no higher or better right than what the execution
debtor has in the property levied upon. The purchaser of property on sale under execution and levy takes as
assignee, only as the judicial seller possesses no title other than that which would pass by an assignment by the
owner. "An execution purchaser generally acquires such estate or interest as was vested in the execution debtor at
the time of the seizure on execution, and only such interest, taking merely a quit-claim of the execution debtor's title,
without warranty on the part of either the execution officer or of the parties, whether the property is realty or
personalty. This rule prevails even if a larger interest in the property was intended to be sold. Accordingly, if the
judgment debtor had no interest in the property, the execution purchaser acquires no interest therein." (Pacheco vs.
Court of Appeals, L-48689, August 31, 1987, 153 SCRA 382, 388-389 quoting Laureano vs. Stevenson, 45 Phil.
252; Cabuhat vs. Ansery, 42 Phil. 170; Fore v. Manove, 18 Cal. 436 and 21 Am. Jur., 140-141. Emphasis supplied.)
Applying this jurisprudence, execution purchasers Santos and Joson possess no rights which may rise above judgment debtor
Katrina's inchoate proprietary rights over the properties sold at public auction. After all, a person can sell only what he owns or is
authorized to sell and the buyer can, as a consequence, acquire no more that what the seller can legally transfer. 36 But, inasmuch as
the decision in Civil Case No. 2224 is void only as far as Romarico and the conjugal properties are concerned, the same may still be executed by the Spouses Wong against
Katrina Henson personally and exclusively. The Spouses Wong must return to Juanito Santos and Leonardo Joson the purchase prices of P145,000 and P119,000 respectively,
received by said spouse from the public auction sale.

The redemption made by Santos in the foreclosure proceeding against Romarico and Katrina Henson filed by the Rural Bank of
Porac, should, however, be respected unless Romarico exercises his right of redemption over the property covered by Transfer
Certificate of Title No. 30951 in accordance with law.
WHEREFORE, the decisions of the appellate court and the lower court in Civil Case No. 28-09 are hereby AFFIRMED subject
to the modifications above stated. No costs.
SO ORDERED.

G.R. No. 146504

April 9, 2002

HONORIO L. CARLOS, petitioner,


vs.
MANUEL T. ABELARDO, respondent.
KAPUNAN, J.:
Assailed in this petition for review on certiorari under Rule 45 of the Rules of Court is the decision of the Court of Appeals dated
November 10, 2000 in CA-G.R. CV No. 54464 which reversed and set aside the decision of the Regional Trial Court of
Valenzuela, Branch 172, and dismissed for insufficiency of evidence the complaint for a sum of money and damages filed by
herein petitioner Honorio Carlos against respondent Manuel Abelardo, his son-in-law, and the latters wife, Maria Theresa
Carlos-Abelardo.
Petitioner averred in his complaint filed on October 13, 1994 that in October 1989, respondent and his wife Maria Theresa
Carlos-Abelardo approached him and requested him to advance the amount of US$25,000.00 for the purchase of a house and
lot located at #19952 Chestnut Street, Executive Heights Village, Paranaque, Metro Manila. To enable and assist the spouses
conduct their married life independently and on their own, petitioner, in October 31, 1989, issued a check in the name of a
certain Pura Vallejo, seller of the property, who acknowledged receipt thereof. 1 The amount was in full payment of the property.
When petitioner inquired from the spouses in July 1991 as to the status of the amount he loaned to them, the latter
acknowledged their obligation but pleaded that they were not yet in a position to make a definite settlement of the
same.2Thereafter, respondent expressed violent resistance to petitioners inquiries on the amount to the extent of making
various death threats against petitioner.3
On August 24, 1994, petitioner made a formal demand for the payment of the amount of US$25,000.00 but the spouses failed to
comply with their obligation.4Thus, on October 13, 1994, petitioner filed a complaint for collection of a sum of money and
damages against respondent and his wife before the Regional Trial Court of Valenzuela, Branch 172, docketed as Civil Case
No. 4490-V-94. In the complaint, petitioner asked for the payment of the US$25,000.00 or P625,000.00, its equivalent in
Philippine currency plus legal interest from date of extra-judicial demand. 5Petitioner likewise claimed moral and exemplary
damages, attorneys fees and costs of suit from respondent. 6
As they were separated in fact for more than a year prior to the filing of the complaint, respondent and his wife filed separate
answers. Maria Theresa Carlos-Abelardo admitted securing a loan together with her husband, from petitioner. 7She claimed,
however, that said loan was payable on a staggered basis so she was surprised when petitioner demanded immediate payment
of the full amount.8
In his separate Answer, respondent admitted receiving the amount of US$25,000.00 but claimed that:
xxx
a. Defendant (respondent) xxx revived that otherwise dormant construction firm H.L. CARLOS CONSTRUCTION of herein
plaintiff which suffered tremendous setback after the assassination of Senator Benigno Aquino;
b. Working day and night and almost beyond human endurance, defendant devoted all his efforts and skill, used all his
business and personal connection to be able to revive the construction business of plaintiff;
c. Little-by-little, starting with small construction business, defendant was able to obtain various construction jobs using the
name H.L. CARLOS CONSTRUCTION and the income derived therefrom were deposited in the name of such firm of
plaintiff,
d. Defendant xxx was made to believe that the earnings derived from such construction will be for him and his family since
he was the one working to secure the contract and its completion, he was allowed to use the facilities of the plaintiff;
e. The plaintiff seeing the progress brought about by defendant xxx to his company proposed a profit sharing scheme to
the effect that all projects amounting to more than P10 million shall be for the account of plaintiff; lower amount shall be for
defendants account but still using H.L. CARLOS CONSTRUCTION.
f. But, to clear account on previous construction contracts that brought income to H.L.CARLOS CONSTRUCTION, out of
which defendant derived his income, plaintiff gave the amount of US$25,000.00 to defendant to square off account and to
start the arrangement in paragraph (e) supra;
g. That, the said US$25,000.00 was never intended as loan of defendant. It was his share of income on contracts
obtained by defendant;
xxx 9
Respondent denied having made death threats to petitioner and by way of compulsory counterclaim, he asked for moral
damages from petitioner for causing the alienation of his wifes love and affection, attorneys fees and costs of suit. 10
On June 26, 1996, the Regional Trial Court rendered a decision in favor of petitioner, the dispositive portion of which reads:
WHEREFORE, judgment is hereby rendered as follows:
1. Ordering the defendants to pay plaintiff the amount of US$25,000.00 or its equivalent in Philippine Currency at the time
of its payment, plus legal interest thereon from August 24, 1994 until fully paid;
2. Ordering the defendant Manuel T. Abelardo to pay the plaintiff the amount of P500,000.00 representing moral damages
and the further amount of P50,000.00 as exemplary damages; and
3. Ordering the defendants to pay the plaintiff the amount of P100,000.00 as attorneys fees, plus the costs of suit.
SO ORDERED.11
Respondent appealed the decision of the trial court to the Court of Appeals. On November 10, 2000, the Court of Appeals
reversed and set aside the trial courts decision and dismissed the complaint for insufficiency of evidence to show that the
subject amount was indeed loaned by petitioner to respondent and his wife. The Court of Appeals found that the amount of
US$25,000.00 was respondents share in the profits of H.L. Carlos Construction. The dispositive portion of the Court of Appeals
decision states:
WHEREFORE, premises considered, the Decision of the Regional Trial Court of Valenzuela, Branch 172 in Civil Case No.
4490-V-94 is hereby REVERSED and SET ASIDE and a new one entered DISMISSING the Complaint for insufficiency of
evidence.
The claim for damages by defendant-appellant is likewise DISMISSED, also for insufficiency of evidence, because of his
failure to present substantial evidence to prove that plaintiff-appellee caused the defendant-spouses separation.
Costs against the plaintiff-appellee.

SO ORDERED.12
A motion for reconsideration of the above decision having been denied on, petitioner brought this appeal assigning the following
errors:
THE COURT OF APPEALS ERRED IN FINDING INSUFFICIENT EVIDENCE TO PROVE THAT THE AMOUNT OF
US$25,000.00 WAS A LOAN OBTAINED BY PRIVATE RESPONDENT AND HIS WIFE FROM PETITIONER.
THE COURT OF APPEALS ERRED IN HOLDING THAT THE US$25,000.00 WAS GIVEN AS PRIVATE RESPONDENTS
SHARE IN THE PROFITS OF H.L. CARLOS CONSTRUCTION, INC. AND THAT THE FILING OF THE COMPLAINT IS A
HOAX.
THE COURT OF APPEALS ERRED IN NULLIFYING THE AWARD OF DAMAGES FOR LACK OF PROOF THEREOF.
We find merit in the petition.
As gleaned from the records, the following facts are undisputed: (1) there was a check in the amount of US$25,000.00 issued
by petitioner; (2) this amount was received by respondent and his wife and given to a certain Pura Vallejo for the full payment of
a house and lot located at #19952 Chestnut Street, Executive Heights Village, Paranaque, Metro Manila; (3) this house and lot
became the conjugal dwelling of respondent and his wife; and (4) respondents wife executed an instrument acknowledging the
loan but which respondent did not sign.
To prove his claim that the amount was in the nature of a loan or an advance he extended to respondent and his wife, petitioner
presented Bankers Trust Check No. 337 in the amount of US$25,000.00 he issued on October 31, 1989 to Pura Vallejo. 13 He
also introduced in evidence an instrument executed by respondents wife on July 31, 1991 acknowledging her and her
husbands accountability to petitioner for the said amount which was advanced in payment of a house and lot located at #19952
Chestnut Street, Executive Heights Subdivision, Paranaque. 14 A formal demand letter by counsel for petitioner dated August 24,
1994 sent to and received by respondent was also on record. 15
All these pieces of evidence, taken together with respondents admission that he and his wife received the subject amount and
used the same to purchase their house and lot, sufficiently prove by a preponderance of evidence petitioners claim that the
amount of US$25,000.00 was really in the nature of a loan.
Respondent tried to rebut petitioners evidence by claiming that the US$25,000.00 was not a loan but his share in the profits of
H.L. Carlos Construction. He alleged that he received money from petitioner amounting to almost P3 million as his share in the
profits of the corporation. To prove this, he presented ten (10) Bank of the Philippine Islands (BPI) checks allegedly given to him
by petitioner.16He argued that if indeed, he and his wife were indebted to petitioner, the latter could have easily deducted the
amount of the said loan from his share of the profits.
Respondent fails to convince this Court.
All the checks presented by respondent, which he claims to be his share in the profits of petitioners company, were all in the
account of H.L. Carlos Construction.17 On the other hand, the Bankers Trust Check in the amount of US$25,000.00 was drawn
from the personal account of petitioner.18Assuming to be true that the checks presented by respondent were his profits from the
corporation, then all the more does this prove that the amount of US$25,000.00 was not part of such profits because it was
issued by petitioner from his own account. Indeed, if such amount was respondents share of the profits, then the same should
have been issued under the account of H.L. Carlos Construction.
Moreover, respondent failed to substantiate his claim that he is entitled to the profits and income of the corporation. There was
no showing that respondent was a stockholder of H.L. Carlos Construction. His name does not appear in the Articles of
Incorporation as well as the Organizational Profile of said company either as stockholder or officer. 19 Not being a stockholder, he
cannot be entitled to the profits or income of said corporation. Neither did respondent prove that he was an employee or an
agent so as to be entitled to salaries or commissions from the corporation.
We quote with favor the disquisition of the trial court on this point:
Early in time, it must be noted that payment of personal debts contracted by the husband or the wife before or during the
marriage shall not be charged to the conjugal partnership except insofar as they redounded to the benefit of the family.
The defendants never denied that the check of US$25,000.00 was used to purchase the subject house and lot. They do
not deny that the same served as their conjugal home, thus benefiting the family. On the same principle, acknowledgment
of the loan made by the defendant-wife binds the conjugal partnership since its proceeds redounded to the benefit of the
family. Hence, defendant-husband and defendant-wife are jointly and severally liable in the payment of the loan.
Defendant-husband cannot allege as a defense that the amount of US $25,000.00 was received as his share in the
income or profits of the corporation and not as a loan. Firstly, defendant-husband does not appear to be a stockholder nor
an employee nor an agent of the corporation, H. L. Carlos Construction, Inc. Since he is not a stockholder, he has no right
to participate in the income or profits thereof. In the same manner that as he is not an employee nor an agent of H. L.
Carlos Construction, Inc., he has no right to receive any salary or commission therefrom. Secondly, the amount advanced
for the purchase of the house and lot came from the personal account of the plaintiff. If, indeed, it was to be construed as
defendant-husbands share in the profits of the corporation, the checks should come from the corporations account and
not from the plaintiffs personal account, considering that the corporation has a personality separate and distinct from that
of its stockholders and officers.
1wphi1.nt

Even granting that the checks amount to US $3,000.000.00 given by the plaintiff to the defendant-spouses was their share
in the profits of the corporation, still there is no sufficient evidence to establish that the US $25,000.00 is to be treated
similarly. Defendant-husband in invoking the defense of compensation argued that if indeed they were indebted to the
plaintiff, the latter could have applied their share in the proceeds or income of the corporation to the concurrent amount of
the alleged loan, instead of giving the amount of P3,000,000.00 to them. This argument is untenable. Article 1278 of the
Civil Code provides that compensation shall take place when two persons, in their own right, are debtors and creditors of
each other. As its indicates, compensation is a sort of balancing between two obligations. In the instant case, the plaintiff
and the defendant-husband are not debtors and creditors of each other. Even granting that the defendant-husbands claim
to the profits of the corporation is justified, still compensation cannot extinguish his loan obligation to the plaintiff because
under such assumption, the defendant is dealing with the corporation and not with the plaintiff in his personal capacity.
Hence, compensation cannot take place.
The Court of Appeals, thus, erred in finding that respondents liability was not proved by preponderance of evidence. On the
contrary, the evidence adduced by petitioner sufficiently established his claim that the US$25,000.00 he advanced to
respondent and his wife was a loan.
The loan is the liability of the conjugal partnership pursuant to Article 121 of the Family Code:
Article 121. The conjugal partnership shall be liable for:
xxx

(2) All debts and obligations contracted during the marriage by the designated administrator-spouse for the benefit of the
conjugal partnership of gains, or by both spouses or by one of them with the consent of the other;
(3) Debts and obligations contracted by either spouse without the consent of the other to the extent that the family may
have been benefited;
If the conjugal partnership is insufficient to cover the foregoing liabilities, the spouses shall be solidarily liable for the
unpaid balance with their separate properties.
xxx
While respondent did not and refused to sign the acknowledgment executed and signed by his wife, undoubtedly, the loan
redounded to the benefit of the family because it was used to purchase the house and lot which became the conjugal home of
respondent and his family. Hence, notwithstanding the alleged lack of consent of respondent, under Art. 21 of the Family Code,
he shall be solidarily liable for such loan together with his wife.
We also find sufficient basis for the award of damages to petitioner, contrary to the findings of the Court of Appeals that
petitioner is not entitled thereto.
Petitioners allegations of verbal and written threats directed against him by respondent is duly supported by evidence on
record. He presented two witnesses, Irineo Pajarin and Randy Rosal, who testified on separate incidents where threats were
made by respondent against petitioner.
Randy Rosal, driver of petitioner, declared that around three o clock in the afternoon of September 15, 1991, he was sent by
respondents wife on an errand to deliver the acknowledgment letter to respondent for him to sign. Respondent did not sign the
acknowledgment and instead, wrote a letter addressed to petitioner threatening him. He narrated what took place thereafter:
xxx
Q
When you were requested by Ma. Theresa C. Abelardo to bring a letter to herein defendant Manuel Abelardo for
him to sign the same, do you know whether that letter was actually signed by Manuel Abelardo?
A

No, sir.
xxx

And what happened when Manuel Abelardo refused to sign that letter coming from the other defendant?

He made me wait and he prepared a letter to Mr. Honorio Carlos, sir.


xxx

Where were you at the time when this defendant Manuel Abelardo prepared this letter?

In his house, sir.

And where did he actually prepare that letter?

At the dining table, sir.

How far were you from Manuel Abelardo from the dining table at the time when he was preparing a letter.

Around 1 meter, sir.

And do you know where in, what particular paper did Mr. Abelardo prepare or write this letter?

He wrote it in a Manila envelope, sir.


xxx

What happened after Manuel Abelardo prepared this letter in a Manila envelope?

He got a small envelope and placed there the name of Mr. Carlos as the addressee, sir.
xxx

Q
After preparing this letter on a Manila envelope and then getting another envelope and writing on it the address of
herein plaintiff, what did the defendant Manuel Abelardo do, if any?
A

He instructed me to mail the letter which he prepared, sir.


xxx

Q
And did you actually accede to the request of herein defendant Manuel Abelardo for you to mail that letter to Engr.
Carlos?
A

I got the envelope but I did not mail it, sir.


xxx

May we know from you the reason why you did not mail said letter?

Because Engr. Carlos might become frightened, sir.

What did you do with that letter, although you did not mail it?

I kept it, sir.


xxx

And what did you do next after keeping the letter for several days?

I gave the letter personally to Engr. Carlos, sir.

What prompted you to give that letter to Engr. Carlos instead of mailing it?

So that Engr. Carlos can prepare, sir.


x

x20

This incident was duly entered and recorded in the Police Blotter on October 7, 1991 by a certain Sgt. Casile of the Valenzuela
Police Station.21 A photocopy of this written threat was also attached to the Police Report and presented in evidence. 22
Another witness, Irineo Pajarin, recounted an incident which occurred in the afternoon of May 25, 1994, to wit:
xxx
Q
Now Mr. Witness, on May 25, 1994 at around 2:30 in the afternoon do you recall where you were on that particular
date and time?
A

I was at B.F. Homes, Paranaque, sir.

What were you doing at that time?

I was waiting for Sargie Cornista, sir.


xxx

Will you please narrate to this Honorable Court that unusual incident?

A
Manuel Abelardo passed by and when he saw me he called me. I approached him while he was then on board his
car and asked me who was my companion, sir.
Q

And what was your answer to him?

I told him it was Sargie, sir.

And what was his reply if any?

He again asked me if I have in my company one of his children, sir.

What was your reply?

I answered none, sir.

Q
Incidentally Mr. Witness, where or in what particular place did this conversation between you and Manuel T.
Abelardo take place?
A

Parking Area of Academy I, Gov. Santos corner Aguirre St., sir.

Now, what else happened after you talk[ed] with this Manuel T. Abelardo?

A
He said I may be fooling him because he said I once fooled him when I ran away with his children which he is going
to take back, sir.
Q

And what was your reply to that?

I answered I did not do that and he said that once he discovered that I did it he would box me, sir.

What else if any did he tell you at that time?

He asked me who instructed me, sir.

Instructed you about what?

To run away with the children, sir.

And what was your reply?

None, he was the one who said "was it your Ate Puppet?" But I did not answer, sir.

What happened next when you failed to answer?

"Or my father in law?"

And when he said his father in law to whom was he referring at that time?

Mr. Honorio Carlos, sir.

After mentioning the name of his father-in-law Mr. Honorio Carlos what happened next?

He told me "Sabihin mo sa biyenan ko babarilin ko siya pag nakita ko siya."

Where was Manuel Abelardo at that particular time when he told this threatening remark against Honorio Carlos?

He was inside his car in Aguirre St., sir.

Q
How about you where were you approximately at that particular time when he narrated that message to you
threatening the herein plaintiff?
A

I was outside looking in his vehicle at Aguirre St., sir.


xxx

And what was your reply or reaction when he made this threatening remarks?

None, because he left. I was left behind, sir.23

This testimony was in part corroborated by an entry dated May 28, 1994 in the Police Blotter of the Paranaque Police Station
narrating the aforementioned incident.24
The testimonies of these witnesses on the two separate incidents of threat are positive, direct and straightforward. Petitioner
also declared on the witness stand that on several occasions, he received telephone calls from respondent cursing and
threatening him.25These incidents of threat were also evidenced by a letter written by respondents wife and addressed to her
father-in-law (father of respondent).26The letter recounted the instances when threats were made by her husband against
petitioner, particularly, the incident reported by Pajarin and the threats made by respondent through the telephone. 27
All these circumstances sufficiently establish that threats were directed by respondent against petitioner justifying the award of
moral damages in favor of petitioner. However, the Court finds the amount of P500,000.00 as moral damages too exorbitant

under the circumstances and the same is reduced to P50,000.00. The exemplary damages and attorneys fees are likewise
reduced to P20,000.00 and P50,000.00, respectively.
WHEREFORE, the petition is hereby GRANTED and the decision of the Court of Appeals in CA GR-CV No. 54464
is MODIFIED in that respondent is ordered to pay petitioner the amounts of (1) US$25,000 or its equivalent in Philippine
currency at the time of payment, plus legal interest from August 4, 1994, until fully paid; (2) P50,000.00 as moral damages;
(3) P20,000.00 as exemplary damages; and (4) P50,000.00 as attorneys fees.
1wphi1.nt

SO ORDERED.

G.R. No. L-51283 June 7, 1989


LOURDES MARIANO, petitioner,
vs.
COURT OF APPEALS, and DANIEL SANCHEZ, respondents.
Jose V. Natividad & Associates for petitioner .
Arturo S. Santos for respondents.

NARVASA, J.:
The proceedings at bar concern (1) an attempt by a married man to prevent execution against conjugal property of a judgment
rendered against his wife, for obligations incurred by the latter while engaged in a business that had admittedly redounded to
the benefit of the family, and (2) the interference by a court with the proceedings on execution of a co-equal or coordinate court.
Both acts being proscribed by law, correction is called for and will hereby be effected.
The proceedings originated from a suit filed by Esther Sanchez against Lourdes Mariano in the Court of First Instance at
Caloocan City, 1 for recovery of the value of ladies' ready made dresses allegedly purchased by and delivered to the latter. 2
A writ of preliminary attachment issued at Esther Sanchez' instance, upon a bond posted by Veritas Insurance Company in the
amount of P 11,000.00, and resulted in the seizure of Lourdes Mariano's property worth P 15,000.00 or so. 3 Her motion for the
discharge of the attachment having been denied, 4 Lourdes Mariano went up to the Court of Appeals on certiorari. That Court ordered 5 the Trial Court to receive evidence on
whether or not the attachment had been improvidently or irregularly issued. 6 The Trial Court did so, came to the conclusion that the attachment had indeed been improperly
issued, and consequently dissolved it. 7

Trial then ensued upon the issues arising from the complaint as well as Lourdes Mariano's answer with counterclaim-which
included a claim for damages resulting from wrongful attachment. Thereafter judgment was rendered in favor of defendant
Lourdes Mariano and against plaintiff Esther Sanchez containing the following dispositions, to wit: 8
1. On the complaint, defendant is ordered to pay unto the plaintiff for the value of the dishonored check (Exhs. G-1,
H and I) in the total amount of P 1,512.00.
2. On the counterclaim, the plaintiff is ordered to pay unto defendant the following, as follows:
a) P 7,500.00 for loss of income of the defendant for 75 days;
b) P 16,000.00 for the value of attached goods;
c) P 25,000.00 for moral and exemplary damages;
d) P 5,000.00 as attorney's fees plus costs of suit.
The Veritas Insurance Company which issued the attachment bond is ordered to pay unto the defendant the full insurance
coverage of P 11,000.00 to answer for the total liability of the plaintiff thereof
Esther Sanchez sought to perfect an appeal by filing a notice of appeal, an appeal bond and a record on appeal.

Pending approval of the record on appeal, Lourdes Mariano filed a motion for the immediate execution of the judgment which
the Court granted. 10 In virtue of the writ of execution which afterwards issued in due course, the sheriff garnished the sum of P 11,000.00 from Veritas Insurance Company,
and levied on real and personal property belonging to the conjugal partnership of Esther Sanchez and her husband, Daniel Sanchez. Esther Sanchez then filed a petition for
certiorari with the Court of Appeals, praying for the annulment of the execution pending appeal authorized by the Trial Court; but her petition was adjudged to be without merit and
was accordingly dismissed. 11

Daniel Sanchez, Esther's husband, now made his move. He filed a complaint for annulment of the execution in the Court of First
Instance at Quezon City in his capacity as administrator of the conjugal partnership. 12 He alleged that the conjugal assets could not validly be
made to answer for obligations exclusively contracted by his wife, and that, moreover, some of the personal property levied on, such as household appliances and utensils
necessarily used in the conjugal dwelling, were exempt from execution. He also applied for a preliminary injunction pending adjudication of the case on the merits. 13

The Quezon City Court issued an order setting the matter of the injunction for hearing, and commanding the sheriff, in the
meantime, to desist from proceeding with the auction sale of the property subject of Daniel Sanchez' claim. 14 Lourdes Mariano filed a
motion to dismiss the action; this, the Court denied. 15 She then instituted a special civil action of certiorari in the Court of Appeals 16 where she initially enjoyed some measure of
success: her petition was given due course, and the Quezon City Court was restrained by the Appellate Court's Seventh Division 17 from further proceeding with the
case. 18 Eventually, however, the Eighth Division 19 came to the conclusion that there was no merit in her cause and dismissed her petition. 20 It ruled that the Quezon City Court
had not interfered with the execution process of the Caloocan Court because Daniel Sanchez's action in the former court raised an issue-the validity of the sheriffs levy on the
conjugal partnership assets of the Sanchez spouses different from those adjudicated in the Caloocan Court, and Sanchez was not a party to the case tried by the latter.

From this verdict Lourdes Mariano has appealed to this Court, contending that the Appellate Court committed reversible error1) in ruling that the conjugal partnership of Daniel and Esther Sanchez could not be made liable for Esther's
judgment obligation arising from the spouses' joint business with Lourdes Mariano;
2) in ruling that the Quezon City Court of First Instance had not interfered with the execution process of the
Caloocan Court of First Instance; and
3) when its Eighth Division decided the petition of Lourdes Mariano although the case had been raffled to the
Seventh Division and the latter had in fact given due course to the petition.
1. There is no dispute about the fact that Esther Sanchez was engaged in business not only without objection on the
part of her husband, Daniel, but in truth with his consent and approval. 21 It is also established that, as expressly acknowledged by
Esther herself and never denied by Daniel, the profits from the business had been used to meet, in part at least, expenses for the support of her family, i.e., the
schooling of the children, food and other household expenses. 22 Under the circumstances, Lourdes Mariano action against Esther Sanchez was justified, the
litigation being "incidental to the ... business in which she is engaged 23 and consequently, the conjugal partnership of Daniel and Esther Sanchez was liable for the
debts and obligations contracted by Esther in her business since the income derived therefrom, having been used to defray some of the expenses for the
maintenance of the family and the education of the children, had redounded to the benefit of the partnership. 24 It was therefore error for the Court of Appeals to
have ruled otherwise.

2. It was also error for the Court of Appeals to have held that there was no interference by the Quezon City Court of
First Instance with the execution process of the Caloocan Court.
The rule, one of great importance in the administration of justice, is that a Court of First Instance has no power to restrain by
means of injunction the execution of a judgment or decree of another judge of concurrent or coordinate jurisdictions. 25 But this is
precisely what was done by the Quezon City Court of First Instance: it enjoined the execution of a judgment authorized and directed by a co-equal and coordinate court, the
Caloocan City Court of First Instance. It did so on the claim of Daniel Sanchez that the property being levied on belonged to the conjugal partnership and could not be made liable
for the wife's obligations.

The question that arises is whether such a claim that property levied on in execution of a judgment is not property of the
judgment debtor, Daniel Sanchez's wife, but of the conjugal partnership of the Sanchez Spouses is properly cognizable by a
Court other than that which rendered judgment adversely to the wife.

To be sure, Section 17, Rule 39 of the Rules of Court, authorizes a "third person," i.e., "any other person than the judgment
debtor or his agent," to vindicate "his claim to the property by any proper action." The section reads as follows: 26
SEC. 17. Proceedings where property claimed by third person.-If property levied on be claimed by any other person
than the judgment debtor or his agent, and such person make an affidavit of his title thereto or right to the
possession thereof, stating the grounds of such right or title, and serve the same upon the officer making the levy,
and a copy thereof upon the judgment creditor, the officer shall not be bound to keep the property, unless such
judgment creditor or his agent, on demand of the officer, indemnify the officer against such claim by a bond in a sum
not greater than the value of the property levied on. In case of disagreement as to such value, the same shall be
determined by the court issuing the writ of execution.
The officer is not liable for damages, for the taking or keeping of the property, to any third-party claimant, unless a
claim is made by the latter and unless an action for damages is brought by him against the officer within one
hundred twenty (120) days from the date of the filing of the bond. But nothing herein contained shall prevent such
claimant or any third person from vindicating his claim to the property by any proper action.
xxx xxx xxx
The "proper action" referred to in the section "is and should be an entirely separate and distinct action from that in which
execution has issued, if instituted by a stranger to the latter suit:" 27 and in "such separate action, the court may issue a writ of preliminary injunction
against the sheriff enjoining him from proceeding with the execution sale." 28 "Upon the other hand, if the claim of impropriety on the part of the sheriff in the execution proceedings
is made by a party to the action, not a stranger thereto, any relief therefrom may be applied for with, and obtained from, only the executing court; and this is true even if a new party
has been impleaded in the suit." 29

In the case at bar, the husband of the judgment debtor cannot be deemed a "stranger" to the case prosecuted and adjudged
against his wife. A strikingly similar situation was presented in a case decided by this Court as early as 1976, Rejuso v.
Estipona. 30 There, the sheriff tried to evict petitioner Rejuso and his family from their house and lot which had been sold in execution of a money judgment rendered by the
Court of First Instance of Davao against Rejuso. What Rejuso did was to institute, together with his wife, Felisa, a separate suit in the same court against the sheriff and the
judgment creditor, Estipona, for the purpose of annulling the levy, execution sale, and writ of possession issued in the first action in respect of their residential house and lot, on the
theory that that property was conjugal in character and "hence, not subject to such proceedings considering that Felisa was not a party to the previous case." The action was
however dismissed by the court on the ground that it had "no jurisdiction over the subject matter of the action or the nature of the action and of the relief sought." 31 The dismissal
was had on motion of Estipona who argued that the court had no jurisdiction to "vacate or annul and/or enjoin the enforcement of the process issued by another branch in another
case," and since Rejuso had already raised the same issues in the first case, without success, he should not be allowed to "get from another branch ... what he failed to get ...
(from) Branch l." This Court affirmed that judgment of dismissal, 32 holding that Rejuso's action was barred by res adjudicata; and "(a)s regards Felisa Rejuso, who is a new party in
Civil Case No. 5102" (the second action) it was ruled that-

... her remedy, if it has not yet been barred by the statute of limitations or become stale in some other way, is within
Civil Case No. 4435 (the first suit). Indeed, it is superfluous to start a new action on a matter which can be more
simply and conveniently litigated within a former proceeding of which it is more logically and legally an integral part.
(Ipekdjian Merchandising Co., Inc, v. CTA, 8 SCRA 59 [1963]). Actually, the court in which the former proceeding
was pending has exclusive jurisdiction thereof (De Leon vs. Salvador, 36 SCRA 567), the fact that the two cases are
in the same Branch of the same Court of First Instance and presided over by the same Judge notwithstanding. After
all, it is simpler and more convenient to observe such practice, which insures also consistency in the resolutions of
related questions because they are to be determined in most if not all instances by the same judge.
In any case, whether by intervention in the court issuing the writ, or by separate action, it is unavailing for either Esther Sanchez
or her husband, Daniel, to seek preclusion of the enforcement of the writ of possession against their conjugal assets. For it
being established, as aforestated, that Esther had engaged in business with her husband's consent, and the income derived
therefrom had been expended, in part at least, for the support of her family, the liability of the conjugal assets to respond for the
wife's obligations in the premises cannot be disputed.
The petitioner's appeal must therefore be sustained.
However, the petitioner's theory that the Eighth Division of the Appellate Court had improperly taken cognizance of the case
which had been raffled to the Seventh Division, must be rejected. It is without foundation, and was evidently made without
attempt to ascertain the relevant facts and applicable rules. The case had originally been assigned to Mr. Justice Isidro C.
Borromeo for study and report while he was still a member of the Seventh Division. The case was brought by him to the Eighth
Division when he was subsequently transferred thereto; and he had ultimately written the opinion for the division after due
deliberation with his colleagues. All of this took place in accordance with the Rules of the Court of Appeals.
WHEREFORE, the Decision of the Court of Appeals subject of the petition is REVERSED AND SET ASIDE, and the Regional
Trial Court (formerly Court of First Instance) at Quezon City is ORDERED to dismiss Civil Case No. 20415 entitled "Daniel P.
Sanchez v. Deputy Sheriff Mariano V. Cachero, et al.," with prejudice. Costs against private respondents.

G.R. No. 124642

February 23, 2004

ALFREDO CHING and ENCARNACION CHING, petitioners


vs.
THE HON. COURT OF APPEALS and ALLIED BANKING CORPORATION, respondents.
DECISION
CALLEJO, SR., J.:
This petition for review, under Rule 45 of the Revised Rules of Court, assails the Decision 1 of the Court of Appeals (CA) dated
November 27, 1995 in CA-G.R. SP No. 33585, as well as the Resolution 2 on April 2, 1996 denying the petitioners motion for
reconsideration. The impugned decision granted the private respondents petition forcertiorari and set aside the Orders of the
trial court dated December 15, 19933 and February 17, 19944 nullifying the attachment of 100,000 shares of stocks of the
Citycorp Investment Philippines under the name of petitioner Alfredo Ching.
The following facts are undisputed:
On September 26, 1978, the Philippine Blooming Mills Company, Inc. (PBMCI) obtained a loan of P9,000,000.00 from the Allied
Banking Corporation (ABC). By virtue of this loan, the PBMCI, through its Executive Vice-President Alfredo Ching, executed a
promissory note for the said amount promising to pay on December 22, 1978 at an interest rate of 14% per annum. 5 As added
security for the said loan, on September 28, 1978, Alfredo Ching, together with Emilio Taedo and Chung Kiat Hua, executed a
continuing guaranty with the ABC binding themselves to jointly and severally guarantee the payment of all the PBMCI
obligations owing the ABC to the extent of P38,000,000.00.6 The loan was subsequently renewed on various dates, the last
renewal having been made on December 4, 1980. 7
Earlier, on December 28, 1979, the ABC extended another loan to the PBMCI in the amount of P13,000,000.00 payable in
eighteen months at 16% interest per annum. As in the previous loan, the PBMCI, through Alfredo Ching, executed a promissory
note to evidence the loan maturing on June 29, 1981. 8 This was renewed once for a period of one month. 9
The PBMCI defaulted in the payment of all its loans. Hence, on August 21, 1981, the ABC filed a complaint for sum of money
with prayer for a writ of preliminary attachment against the PBMCI to collect the P12,612,972.88 exclusive of interests, penalties
and other bank charges. Impleaded as co-defendants in the complaint were Alfredo Ching, Emilio Taedo and Chung Kiat Hua
in their capacity as sureties of the PBMCI.
The case was docketed as Civil Case No. 142729 in the Regional Trial Court of Manila, Branch XVIII. 10 In its application for a
writ of preliminary attachment, the ABC averred that the "defendants are guilty of fraud in incurring the obligations upon which
the present action is brought11 in that they falsely represented themselves to be in a financial position to pay their obligation
upon maturity thereof."12 Its supporting affidavit stated, inter alia, that the "[d]efendants have removed or disposed of their
properties, or [are] ABOUT to do so, with intent to defraud their creditors." 13
On August 26, 1981, after an ex-parte hearing, the trial court issued an Order denying the ABCs application for a writ of
preliminary attachment. The trial court decreed that the grounds alleged in the application and that of its supporting affidavit "are
all conclusions of fact and of law" which do not warrant the issuance of the writ prayed for. 14 On motion for reconsideration,
however, the trial court, in an Order dated September 14, 1981, reconsidered its previous order and granted the ABCs
application for a writ of preliminary attachment on a bond of P12,700,000. The order, in relevant part, stated:
With respect to the second ground relied upon for the grant of the writ of preliminary attachment ex-parte, which is the alleged
disposal of properties by the defendants with intent to defraud creditors as provided in Sec. 1(e) of Rule 57 of the Rules of
Court, the affidavits can only barely justify the issuance of said writ as against the defendant Alfredo Ching who has allegedly
bound himself jointly and severally to pay plaintiff the defendant corporations obligation to the plaintiff as a surety thereof.
WHEREFORE, let a writ of preliminary attachment issue as against the defendant Alfredo Ching requiring the sheriff of this
Court to attach all the properties of said Alfredo Ching not exceeding P12,612,972.82 in value, which are within the jurisdiction
of this Court and not exempt from execution upon, the filing by plaintiff of a bond duly approved by this Court in the sum of
Twelve Million Seven Hundred Thousand Pesos (P12,700,000.00) executed in favor of the defendant Alfredo Ching to secure
the payment by plaintiff to him of all the costs which may be adjudged in his favor and all damages he may sustain by reason of
the attachment if the court shall finally adjudge that the plaintiff was not entitled thereto.
SO ORDERED.15
Upon the ABCs posting of the requisite bond, the trial court issued a writ of preliminary attachment. Subsequently, summonses
were served on the defendants,16 save Chung Kiat Hua who could not be found.
Meanwhile, on April 1, 1982, the PBMCI and Alfredo Ching jointly filed a petition for suspension of payments with the Securities
and Exchange Commission (SEC), docketed as SEC Case No. 2250, at the same time seeking the PBMCIs rehabilitation. 17
On July 9, 1982, the SEC issued an Order placing the PBMCIs business, including its assets and liabilities, under rehabilitation
receivership, and ordered that "all actions for claims listed in Schedule "A" of the petition pending before any court or tribunal
are hereby suspended in whatever stage the same may be until further orders from the Commission." 18 The ABC was among
the PBMCIs creditors named in the said schedule.
Subsequently, on January 31, 1983, the PBMCI and Alfredo Ching jointly filed a Motion to Dismiss and/or motion to suspend the
proceedings in Civil Case No. 142729 invoking the PBMCIs pending application for suspension of payments (which Ching cosigned) and over which the SEC had already assumed jurisdiction. 19 On February 4, 1983, the ABC filed its Opposition thereto. 20
In the meantime, on July 26, 1983, the deputy sheriff of the trial court levied on attachment the 100,000 common shares of
Citycorp stocks in the name of Alfredo Ching.21
Thereafter, in an Order dated September 16, 1983, the trial court partially granted the aforementioned motion by suspending the
proceedings only with respect to the PBMCI. It denied Chings motion to dismiss the complaint/or suspend the proceedings and
pointed out that P.D. No. 1758 only concerns the activities of corporations, partnerships and associations and was never
intended to regulate and/or control activities of individuals. Thus, it directed the individual defendants to file their answers. 22
Instead of filing an answer, Ching filed on January 14, 1984 a Motion to Suspend Proceedings on the same ground of the
pendency of SEC Case No. 2250. This motion met the opposition from the ABC. 23
On January 20, 1984, Taedo filed his Answer with counterclaim and cross-claim. 24 Ching eventually filed his Answer on July 12,
1984.25
On October 25, 1984, long after submitting their answers, Ching filed an Omnibus Motion, 26 again praying for the dismissal of
the complaint or suspension of the proceedings on the ground of the July 9, 1982 Injunctive Order issued in SEC Case No.
2250. He averred that as a surety of the PBMCI, he must also necessarily benefit from the defenses of his principal. The ABC
opposed Chings omnibus motion.

Emilio Y. Taedo, thereafter, filed his own Omnibus Motion 27 praying for the dismissal of the complaint, arguing that the ABC had
"abandoned and waived" its right to proceed against the continuing guaranty by its act of resorting to preliminary attachment.
On December 17, 1986, the ABC filed a Motion to Reduce the amount of his preliminary attachment bond fromP12,700,000
to P6,350,000.28 Alfredo Ching opposed the motion,29 but on April 2, 1987, the court issued an Order setting the incident for
further hearing on May 28, 1987 at 8:30 a.m. for the parties to adduce evidence on the actual value of the properties of Alfredo
Ching levied on by the sheriff.30
On March 2, 1988, the trial court issued an Order granting the motion of the ABC and rendered the attachment bond
of P6,350,000.31
On November 16, 1993, Encarnacion T. Ching, assisted by her husband Alfredo Ching, filed a Motion to Set Aside the levy on
attachment. She alleged inter alia that the 100,000 shares of stocks levied on by the sheriff were acquired by her and her
husband during their marriage out of conjugal funds after the Citycorp Investment Philippines was established in 1974.
Furthermore, the indebtedness covered by the continuing guaranty/comprehensive suretyship contract executed by petitioner
Alfredo Ching for the account of PBMCI did not redound to the benefit of the conjugal partnership. She, likewise, alleged that
being the wife of Alfredo Ching, she was a third-party claimant entitled to file a motion for the release of the properties. 32 She
attached therewith a copy of her marriage contract with Alfredo Ching. 33
The ABC filed a comment on the motion to quash preliminary attachment and/or motion to expunge records, contending that:
2.1 The supposed movant, Encarnacion T. Ching, is not a party to this present case; thus, she has no personality to file
any motion before this Honorable Court;
2.2 Said supposed movant did not file any Motion for Intervention pursuant to Section 2, Rule 12 of the Rules of Court;
2.3 Said Motion cannot even be construed to be in the nature of a Third-Party Claim conformably with Sec. 14, Rule 57 of
the Rules of Court.
3. Furthermore, assuming in gracia argumenti that the supposed movant has the required personality, her Motion cannot be
acted upon by this Honorable Court as the above-entitled case is still in the archives and the proceedings thereon still remains
suspended. And there is no previous Motion to revive the same. 34
The ABC also alleged that the motion was barred by prescription or by laches because the shares of stocks were in custodia
legis.
During the hearing of the motion, Encarnacion T. Ching adduced in evidence her marriage contract to Alfredo Ching to prove
that they were married on January 8, 1960; 35 the articles of incorporation of Citycorp Investment Philippines dated May 14,
1979;36 and, the General Information Sheet of the corporation showing that petitioner Alfredo Ching was a member of the Board
of Directors of the said corporation and was one of its top twenty stockholders.
On December 10, 1993, the Spouses Ching filed their Reply/Opposition to the motion to expunge records.
Acting on the aforementioned motion, the trial court issued on December 15, 1993 an Order 37 lifting the writ of preliminary
attachment on the shares of stocks and ordering the sheriff to return the said stocks to the petitioners. The dispositive portion
reads:
WHEREFORE, the instant Motion to Quash Preliminary Attachment, dated November 9, 1993, is hereby granted. Let the writ of
preliminary attachment subject matter of said motion, be quashed and lifted with respect to the attached 100,000 common
shares of stock of Citycorp Investment Philippines in the name of the defendant Alfredo Ching, the said shares of stock to be
returned to him and his movant-spouse by Deputy Sheriff Apolonio A. Golfo who effected the levy thereon on July 26, 1983, or
by whoever may be presently in possession thereof.
SO ORDERED.38
The plaintiff Allied Banking Corporation filed a motion for the reconsideration of the order but denied the same on February 17,
1994. The petitioner bank forthwith filed a petition for certiorari with the CA, docketed as CA-G.R. SP No. 33585, for the
nullification of the said order of the court, contending that:
1. The respondent Judge exceeded his authority thereby acted without jurisdiction in taking cognizance of, and granting a
"Motion" filed by a complete stranger to the case.
2. The respondent Judge committed a grave abuse of discretion in lifting the writ of preliminary attachment without any
basis in fact and in law, and contrary to established jurisprudence on the matter.39
On November 27, 1995, the CA rendered judgment granting the petition and setting aside the assailed orders of the trial court,
thus:
WHEREFORE, premises considered, the petition is GRANTED, hereby setting aside the questioned orders (dated December
15, 1993 and February 17, 1994) for being null and void.
SO ORDERED.40
The CA sustained the contention of the private respondent and set aside the assailed orders. According to the CA, the RTC
deprived the private respondent of its right to file a bond under Section 14, Rule 57 of the Rules of Court. The petitioner
Encarnacion T. Ching was not a party in the trial court; hence, she had no right of action to have the levy annulled with a motion
for that purpose. Her remedy in such case was to file a separate action against the private respondent to nullify the levy on the
100,000 Citycorp shares of stocks. The court stated that even assuming that Encarnacion T. Ching had the right to file the said
motion, the same was barred by laches.
Citing Wong v. Intermediate Appellate Court,41 the CA ruled that the presumption in Article 160 of the New Civil Code shall not
apply where, as in this case, the petitioner-spouses failed to prove the source of the money used to acquire the shares of stock.
It held that the levied shares of stocks belonged to Alfredo Ching, as evidenced by the fact that the said shares were registered
in the corporate books of Citycorp solely under his name. Thus, according to the appellate court, the RTC committed a grave
abuse of its discretion amounting to excess or lack of jurisdiction in issuing the assailed orders. The petitioners motion for
reconsideration was denied by the CA in a Resolution dated April 2, 1996.
The petitioner-spouses filed the instant petition for review on certiorari, asserting that the RTC did not commit any grave abuse
of discretion amounting to excess or lack of jurisdiction in issuing the assailed orders in their favor; hence, the CA erred in
reversing the same. They aver that the source of funds in the acquisition of the levied shares of stocks is not the controlling
factor when invoking the presumption of the conjugal nature of stocks under Art. 160, 42 and that such presumption subsists even
if the property is registered only in the name of one of the spouses, in this case, petitioner Alfredo Ching. 43 According to the
petitioners, the suretyship obligation was not contracted in the pursuit of the petitioner-husbands profession or business. 44 And,
contrary to the ruling of the CA, where conjugal assets are attached in a collection suit on an obligation contracted by the
husband, the wife should exhaust her motion to quash in the main case and not file a separate suit. 45 Furthermore, the
petitioners contend that under Art. 125 of the Family Code, the petitioner-husbands gratuitous suretyship is null and void ab

initio,46 and that the share of one of the spouses in the conjugal partnership remains inchoate until the dissolution and liquidation
of the partnership.47
In its comment on the petition, the private respondent asserts that the CA correctly granted its petition for certiorari nullifying the
assailed order. It contends that the CA correctly relied on the ruling of this Court in Wong v. Intermediate Appellate Court. Citing
Cobb-Perez v. Lantin and G-Tractors, Inc. v. Court of Appeals, the private respondent alleges that the continuing guaranty and
suretyship executed by petitioner Alfredo Ching in pursuit of his profession or business. Furthermore, according to the private
respondent, the right of the petitioner-wife to a share in the conjugal partnership property is merely inchoate before the
dissolution of the partnership; as such, she had no right to file the said motion to quash the levy on attachment of the shares of
stocks.
The issues for resolution are as follows: (a) whether the petitioner-wife has the right to file the motion to quash the levy on
attachment on the 100,000 shares of stocks in the Citycorp Investment Philippines; (b) whether or not the RTC committed a
grave abuse of its discretion amounting to excess or lack of jurisdiction in issuing the assailed orders.
On the first issue, we agree with the petitioners that the petitioner-wife had the right to file the said motion, although she was not
a party in Civil Case No. 142729.48
In Ong v. Tating,49 we held that the sheriff may attach only those properties of the defendant against whom a writ of attachment
has been issued by the court. When the sheriff erroneously levies on attachment and seizes the property of a third person in
which the said defendant holds no right or interest, the superior authority of the court which has authorized the execution may
be invoked by the aggrieved third person in the same case. Upon application of the third person, the court shall order a
summary hearing for the purpose of determining whether the sheriff has acted rightly or wrongly in the performance of his duties
in the execution of the writ of attachment, more specifically if he has indeed levied on attachment and taken hold of property not
belonging to the plaintiff. If so, the court may then order the sheriff to release the property from the erroneous levy and to return
the same to the third person. In resolving the motion of the third party, the court does not and cannot pass upon the question of
the title to the property with any character of finality. It can treat the matter only insofar as may be necessary to decide if the
sheriff has acted correctly or not. If the claimants proof does not persuade the court of the validity of the title, or right of
possession thereto, the claim will be denied by the court. The aggrieved third party may also avail himself of the remedy of
"terceria" by executing an affidavit of his title or right of possession over the property levied on attachment and serving the same
to the office making the levy and the adverse party. Such party may also file an action to nullify the levy with damages resulting
from the unlawful levy and seizure, which should be a totally separate and distinct action from the former case. The abovementioned remedies are cumulative and any one of them may be resorted to by one third-party claimant without availing of the
other remedies.50
In this case, the petitioner-wife filed her motion to set aside the levy on attachment of the 100,000 shares of stocks in the name
of petitioner-husband claiming that the said shares of stocks were conjugal in nature; hence, not liable for the account of her
husband under his continuing guaranty and suretyship agreement with the PBMCI. The petitioner-wife had the right to file the
motion for said relief.
On the second issue, we find and so hold that the CA erred in setting aside and reversing the orders of the RTC. The private
respondent, the petitioner in the CA, was burdened to prove that the RTC committed a grave abuse of its discretion amounting
to excess or lack of jurisdiction. The tribunal acts without jurisdiction if it does not have the legal purpose to determine the case;
there is excess of jurisdiction where the tribunal, being clothed with the power to determine the case, oversteps its authority as
determined by law. There is grave abuse of discretion where the tribunal acts in a capricious, whimsical, arbitrary or despotic
manner in the exercise of its judgment and is equivalent to lack of jurisdiction. 51
It was incumbent upon the private respondent to adduce a sufficiently strong demonstration that the RTC acted whimsically in
total disregard of evidence material to, and even decide of, the controversy before certiorari will lie. A special civil action for
certiorari is a remedy designed for the correction of errors of jurisdiction and not errors of judgment. When a court exercises its
jurisdiction, an error committed while so engaged does not deprive it of its jurisdiction being exercised when the error is
committed.52
After a comprehensive review of the records of the RTC and of the CA, we find and so hold that the RTC did not commit any
grave abuse of its discretion amounting to excess or lack of jurisdiction in issuing the assailed orders.
Article 160 of the New Civil Code provides that all the properties acquired during the marriage are presumed to belong to the
conjugal partnership, unless it be proved that it pertains exclusively to the husband, or to the wife. InTan v. Court of
Appeals,53 we held that it is not even necessary to prove that the properties were acquired with funds of the partnership. As long
as the properties were acquired by the parties during the marriage, they are presumed to be conjugal in nature. In fact, even
when the manner in which the properties were acquired does not appear, the presumption will still apply, and the properties will
still be considered conjugal. The presumption of the conjugal nature of the properties acquired during the marriage subsists in
the absence of clear, satisfactory and convincing evidence to overcome the same. 54
In this case, the evidence adduced by the petitioners in the RTC is that the 100,000 shares of stocks in the Citycorp Investment
Philippines were issued to and registered in its corporate books in the name of the petitioner-husband when the said corporation
was incorporated on May 14, 1979. This was done during the subsistence of the marriage of the petitioner-spouses. The shares
of stocks are, thus, presumed to be the conjugal partnership property of the petitioners. The private respondent failed to adduce
evidence that the petitioner-husband acquired the stocks with his exclusive money.55 The barefaced fact that the shares of
stocks were registered in the corporate books of Citycorp Investment Philippines solely in the name of the petitioner-husband
does not constitute proof that the petitioner-husband, not the conjugal partnership, owned the same. 56 The private respondents
reliance on the rulings of this Court in Maramba v. Lozano 57 and Associated Insurance & Surety Co., Inc. v. Banzon 58 is
misplaced. In the Maramba case, we held that where there is no showing as to when the property was acquired, the fact that
the title is in the wifes name alone is determinative of the ownership of the property. The principle was reiterated in the
Associated Insurance case where the uncontroverted evidence showed that the shares of stocks were acquired during the
marriage of the petitioners.
Instead of fortifying the contention of the respondents, the ruling of this Court in Wong v. Intermediate Appellate
Court59 buttresses the case for the petitioners. In that case, we ruled that he who claims that property acquired by the spouses
during their marriage is not conjugal partnership property but belongs to one of them as his personal property is burdened to
prove the source of the money utilized to purchase the same. In this case, the private respondent claimed that the petitionerhusband acquired the shares of stocks from the Citycorp Investment Philippines in his own name as the owner thereof. It was,
thus, the burden of the private respondent to prove that the source of the money utilized in the acquisition of the shares of
stocks was that of the petitioner-husband alone. As held by the trial court, the private respondent failed to adduce evidence to
prove this assertion.
The CA, likewise, erred in holding that by executing a continuing guaranty and suretyship agreement with the private
respondent for the payment of the PBMCI loans, the petitioner-husband was in the exercise of his profession, pursuing a
legitimate business. The appellate court erred in concluding that the conjugal partnership is liable for the said account of PBMCI
under Article 161(1) of the New Civil Code.
Article 161(1) of the New Civil Code (now Article 121[2 and 3] 60 of the Family Code of the Philippines) provides:
Art. 161. The conjugal partnership shall be liable for:

(1) All debts and obligations contracted by the husband for the benefit of the conjugal partnership, and those contracted by the
wife, also for the same purpose, in the cases where she may legally bind the partnership.
The petitioner-husband signed the continuing guaranty and suretyship agreement as security for the payment of the loan
obtained by the PBMCI from the private respondent in the amount of P38,000,000. In Ayala Investment and Development Corp.
v. Court of Appeals,61 this Court ruled "that the signing as surety is certainly not an exercise of an industry or profession. It is not
embarking in a business. No matter how often an executive acted on or was persuaded to act as surety for his own employer,
this should not be taken to mean that he thereby embarked in the business of suretyship or guaranty."
For the conjugal partnership to be liable for a liability that should appertain to the husband alone, there must be a showing that
some advantages accrued to the spouses. Certainly, to make a conjugal partnership responsible for a liability that should
appertain alone to one of the spouses is to frustrate the objective of the New Civil Code to show the utmost concern for the
solidarity and well being of the family as a unit. The husband, therefore, is denied the power to assume unnecessary and
unwarranted risks to the financial stability of the conjugal partnership. 62
In this case, the private respondent failed to prove that the conjugal partnership of the petitioners was benefited by the
petitioner-husbands act of executing a continuing guaranty and suretyship agreement with the private respondent for and in
behalf of PBMCI. The contract of loan was between the private respondent and the PBMCI, solely for the benefit of the latter.
No presumption can be inferred from the fact that when the petitioner-husband entered into an accommodation agreement or a
contract of surety, the conjugal partnership would thereby be benefited. The private respondent was burdened to establish that
such benefit redounded to the conjugal partnership. 63
It could be argued that the petitioner-husband was a member of the Board of Directors of PBMCI and was one of its top twenty
stockholders, and that the shares of stocks of the petitioner-husband and his family would appreciate if the PBMCI could be
rehabilitated through the loans obtained; that the petitioner-husbands career would be enhanced should PBMCI survive
because of the infusion of fresh capital. However, these are not the benefits contemplated by Article 161 of the New Civil Code.
The benefits must be those directly resulting from the loan. They cannot merely be a by-product or a spin-off of the loan itself. 64
This is different from the situation where the husband borrows money or receives services to be used for his own business or
profession. In the Ayala case, we ruled that it is such a contract that is one within the term "obligation for the benefit of the
conjugal partnership." Thus:
(A) If the husband himself is the principal obligor in the contract, i.e., he directly received the money and services to be used in
or for his own business or his own profession, that contract falls within the term " obligations for the benefit of the conjugal
partnership." Here, no actual benefit may be proved. It is enough that the benefit to the family is apparent at the time of the
signing of the contract. From the very nature of the contract of loan or services, the family stands to benefit from the loan facility
or services to be rendered to the business or profession of the husband. It is immaterial, if in the end, his business or profession
fails or does not succeed. Simply stated, where the husband contracts obligations on behalf of the family business, the law
presumes, and rightly so, that such obligation will redound to the benefit of the conjugal partnership. 65
The Court held in the same case that the rulings of the Court in Cobb-Perez and G-Tractors, Inc. are not controlling because the
husband, in those cases, contracted the obligation for his own business. In this case, the petitioner-husband acted merely as a
surety for the loan contracted by the PBMCI from the private respondent.
IN LIGHT OF ALL THE FOREGOING, the petition is GRANTED. The Decision and Resolution of the Court of Appeals are SET
ASIDE AND REVERSED. The assailed orders of the RTC are AFFIRMED.
SO ORDERED.

G.R. No. 145222

April 24, 2009

SPOUSES ROBERTO BUADO and VENUS BUADO, Petitioners,


vs.
THE HONORABLE COURT OF APPEALS, Former Division, and ROMULO NICOL, Respondents.
DECISION
TINGA, J.:
Before this Court is a petition for certiorari assailing the Decision 1 of the Court of Appeals in CA-G.R. CV No. 47029 and its
Resolution denying the motion for reconsideration thereof.
The case stemmed from the following factual backdrop:
On 30 April 1984, Spouses Roberto and Venus Buado (petitioners) filed a complaint for damages against Erlinda Nicol (Erlinda)
with Branch 19 of the Regional Trial Court (RTC) of Bacoor, Cavite, docketed as Civil Case No. 84-33. Said action originated
from Erlinda Nicols civil liability arising from the criminal offense of slander filed against her by petitioners.
On 6 April 1987, the trial court rendered a decision ordering Erlinda to pay damages. The dispositive portion reads:
Wherefore, judgment is hereby rendered in favor of the plaintiff[s] and against defendant ordering the latter to pay the former the
amount of thirty thousand (P30,000.00) pesos as moral damages, five thousand (P5,000.00) pesos as attorneys fees and
litigation expenses, another five thousand (P5,000.00) pesos as exemplary damages and the cost of suit. 2
Said decision was affirmed, successively, by the Court of Appeals and this Court. It became final and executory on 5 March
1992.
On 14 October 1992, the trial court issued a writ of execution, a portion of which provides:
Now, therefore, you are commanded that of the goods and chattels of the defendant Erlinda Nicol, or from her estates or legal
heirs, you cause the sum in the amount of forty thousand pesos (P40,000.00), Philippine Currency, representing the moral
damages, attorneys fees and litigation expenses and exemplary damages and the cost of suit of the plaintiff aside from your
lawful fees on this execution and do likewise return this writ into court within sixty (60) days from date, with your proceedings
endorsed hereon.
But if sufficient personal property cannot be found whereof to satisfy this execution and lawful fees thereon, then you are
commanded that of the lands and buildings of said defendant you make the said sum of money in the manner required by the
Rules of Court, and make return of your proceedings with this writ within sixty (60) days from date. 3
Finding Erlinda Nicols personal properties insufficient to satisfy the judgment, the Deputy Sheriff issued a notice of levy on real
property on execution addressed to the Register of Deeds of Cavite. The notice of levy was annotated on the Transfer
Certificate of Title No. T-125322.
On 20 November 1992, a notice of sheriffs sale was issued.
Two (2) days before the public auction sale on 28 January 1993, an affidavit of third-party claim from one Arnulfo F. Fulo was
received by the deputy sheriff prompting petitioners to put up a sheriffs indemnity bond. The auction sale proceeded with
petitioners as the highest bidder.
On 4 February 1993, a certificate of sale was issued in favor of petitioners.
Almost a year later on 2 February 1994, Romulo Nicol (respondent), the husband of Erlinda Nicol, filed a complaint for
annulment of certificate of sale and damages with preliminary injunction against petitioners and the deputy sheriff. Respondent,
as plaintiff therein, alleged that the defendants, now petitioners, connived and directly levied upon and execute his real property
without exhausting the personal properties of Erlinda Nicol. Respondent averred that there was no proper publication and
posting of the notice of sale. Furthermore, respondent claimed that his property which was valued at P500,000.00 was only sold
at a "very low price" of P51,685.00, whereas the judgment obligation of Erlinda Nicol was only P40,000.00. The case was
assigned to Branch 21 of the RTC of Imus, Cavite.
In response, petitioners filed a motion to dismiss on the grounds of lack of jurisdiction and that they had acted on the basis of a
valid writ of execution. Citing De Leon v. Salvador,4 petitioners claimed that respondent should have filed the case with Branch
19 where the judgment originated and which issued the order of execution, writ of execution, notice of levy and notice of
sheriffs sale.
In an Order5 dated 18 April 1994, the RTC dismissed respondents complaint and ruled that Branch 19 has jurisdiction over the
case, thus:
As correctly pointed out by the defendants, any flaw in the implementation of the writ of execution by the implementing sheriff
must be brought before the court issuing the writ of execution. Besides, there are two (2) remedies open to the plaintiff, if he
feels that the property being levied on belongs to him and not to the judgment debtor. The first remedy is to file a third-party
claim. If he fails to do this, a right is reserved to him to vindicate his claim over the property by any proper action. But certainly,
this is not the proper action reserved to the plaintiff to vindicate his claim over the property in question to be ventilated before
this court. As earlier stated, this case should have been addressed to Branch 19, RTC Bacoor as it was that court which issued
the writ of execution.6
Respondent moved for reconsideration but it was denied on 26 July 1994.
On appeal, the Court of Appeals reversed the trial court and held that Branch 21 has jurisdiction to act on the complaint filed by
appellant. The dispositive portion reads:
WHEREFORE, the Orders appealed from are hereby REVERSED and SET ASIDE. This case is REMANDED to the Regional
Trial Court of Imus, Cavite, Branch 21 for further proceedings.
SO ORDERED.7
Petitioners motion for reconsideration was denied on 23 August 2000. Hence, the instant petition attributing grave abuse of
discretion on the part of the Court of Appeals.
A petition for certiorari is an extraordinary remedy that is adopted to correct errors of jurisdiction committed by the lower court or
quasi-judicial agency, or when there is grave abuse of discretion on the part of such court or agency amounting to lack or
excess of jurisdiction. Where the error is not one of jurisdiction, but of law or fact which is a mistake of judgment, the proper
remedy should be appeal. In addition, an independent action for certiorari may be availed of only when there is no appeal or any
plain, speedy and adequate remedy in the ordinary course of law.8
Nowhere in the petition was it shown that the jurisdiction of the Court of Appeals was questioned. The issue devolves on
whether the husband of the judgment debtor may file an independent action to protect the conjugal property subject to
execution. The alleged error therefore is an error of judgment which is a proper subject of an appeal.

Nevertheless, even if we were to treat this petition as one for review, the case should still be dismissed on substantive grounds.
Petitioners maintain that Branch 19 retained jurisdiction over its judgment to the exclusion of all other co-ordinate courts for its
execution and all incidents thereof, in line with De Leon v. Salvador. Petitioners insist that respondent, who is the husband of the
judgment debtor, is not the "third party" contemplated in Section 17 (now Section 16), Rule 39 of the Rules of Court, hence a
separate action need not be filed. Furthermore, petitioners assert that the obligation of the wife redounded to the benefit of the
conjugal partnership and cited authorities to the effect that the husband is liable for the tort committed by his wife.
Respondent on the other hand merely avers that the decision of the Court of Appeals is supported by substantial evidence and
in accord with law and jurisprudence.9
Verily, the question of jurisdiction could be resolved through a proper interpretation of Section 16, Rule 39 of the Rules of Court,
which reads:
Sec. 16. Proceedings where property claimed by third person.
If the property levied on is claimed by any person other than the judgment obligor or his agent, and such person makes an
affidavit of his title thereto or right to the possession thereof, stating the grounds of such right or title, and serves the same upon
the officer making the levy and a copy thereof upon the judgment obligee, the officer shall not be bound to keep the property,
unless such judgment obligee, on demand of the officer, files a bond approved by the court to indemnify the third-party claimant
in a sum not less than the value of the property levied on. In case of disagreement as to such value, the same shall be
determined by the court issuing the writ of execution. No claim for damages for the taking or keeping of the property may be
enforced against the bond unless the action therefor is filed within one hundred twenty (120) days from the date of the filing of
the bond.
The officer shall not be liable for damages for the taking or keeping of the property, to any third-party claimant if such bond is
filed. Nothing herein contained shall prevent such claimant or any third person from vindicating his claim to the property in a
separate action, or prevent the judgment obligee from claiming damages in the same or a separate action against a third-party
claimant who filed a frivolous or plainly spurious claim.
When the writ of execution is issued in favor of the Republic of the Philippines, or any officer duly representing it, the filing of
such bond shall not be required, and in case the sheriff or levying officer is sued for damages as a result of the levy, he shall be
represented by the Solicitor General and if held liable therefor, the actual damages adjudged by the court shall be paid by the
National Treasurer out of such funds as may be appropriated for the purpose. (Emphasis Supplied)
Apart from the remedy of terceria available to a third-party claimant or to a stranger to the foreclosure suit against the sheriff or
officer effecting the writ by serving on him an affidavit of his title and a copy thereof upon the judgment creditor, a third-party
claimant may also resort to an independent separate action, the object of which is the recovery of ownership or possession of
the property seized by the sheriff, as well as damages arising from wrongful seizure and detention of the property. If a separate
action is the recourse, the third-party claimant must institute in a forum of competent jurisdiction an action, distinct and separate
from the action in which the judgment is being enforced, even before or without need of filing a claim in the court that issued the
writ.10
1awphi1.zw+

A third-party claim must be filed a person other than the judgment debtor or his agent. In other words, only a stranger to the
case may file a third-party claim.
This leads us to the question: Is the husband, who was not a party to the suit but whose conjugal property is being executed on
account of the other spouse being the judgment obligor, considered a "stranger?"
In determining whether the husband is a stranger to the suit, the character of the property must be taken into account. In
Mariano v. Court of Appeals,11 which was later adopted in Spouses Ching v. Court of Appeals, 12 this Court held that the husband
of the judgment debtor cannot be deemed a "stranger" to the case prosecuted and adjudged against his wife for an obligation
that has redounded to the benefit of the conjugal partnership. 13 On the other hand, in Naguit v. Court of Appeals 14 and Sy v.
Discaya,15 the Court stated that a spouse is deemed a stranger to the action wherein the writ of execution was issued and is
therefore justified in bringing an independent action to vindicate her right of ownership over his exclusive or paraphernal
property.
lawphil.net

Pursuant to Mariano however, it must further be settled whether the obligation of the judgment debtor redounded to the benefit
of the conjugal partnership or not.
Petitioners argue that the obligation of the wife arising from her criminal liability is chargeable to the conjugal partnership. We do
not agree.
There is no dispute that contested property is conjugal in nature. Article 122 of the Family Code 16 explicitly provides that
payment of personal debts contracted by the husband or the wife before or during the marriage shall not be charged to the
conjugal partnership except insofar as they redounded to the benefit of the family.
Unlike in the system of absolute community where liabilities incurred by either spouse by reason of a crime orquasi-delict is
chargeable to the absolute community of property, in the absence or insufficiency of the exclusive property of the debtorspouse, the same advantage is not accorded in the system of conjugal partnership of gains. The conjugal partnership of gains
has no duty to make advance payments for the liability of the debtor-spouse.
Parenthetically, by no stretch of imagination can it be concluded that the civil obligation arising from the crime of slander
committed by Erlinda redounded to the benefit of the conjugal partnership.
To reiterate, conjugal property cannot be held liable for the personal obligation contracted by one spouse, unless some
advantage or benefit is shown to have accrued to the conjugal partnership. 17
In Guadalupe v. Tronco,18 this Court held that the car which was claimed by the third party complainant to be conjugal property
was being levied upon to enforce "a judgment for support" filed by a third person, the third-party claim of the wife is proper since
the obligation which is personal to the husband is chargeable not on the conjugal property but on his separate property.
Hence, the filing of a separate action by respondent is proper and jurisdiction is thus vested on Branch 21. Petitioners failed to
show that the Court of Appeals committed grave abuse of discretion in remanding the case toBranch 21 for further
proceedings.
WHEREFORE, the petition is DISMISSED. The Decision of the Court of Appeals is AFFIRMED. Costs against petitioners.

Property Regime of Unions without Marriage


G.R. No. 136803

June 16, 2000

EUSTAQUIO MALLILIN, JR., petitioner,


vs.
MA. ELVIRA CASTILLO, respondent.
MENDOZA, J.:
This is a petition for review of the amended decision 1 of the Court of Appeals dated May 7, 1998 in CA G.R. CV No. 48443
granting respondent's motion for reconsideration of its decision dated November 7, 1996, and of the resolution dated December
21, 1998 denying petitioner's motion for reconsideration.
The factual and procedural antecedents are as follows:
On February 24, 1993, petitioner Eustaquio Mallilin, Jr. filed a complaint 2 for "Partition and/or Payment of Co-Ownership Share,
Accounting and Damages" against respondent Ma. Elvira Castillo. The complaint, docketed as Civil Case No. 93-656 at the
Regional Trial Court in Makati City, alleged that petitioner and respondent, both married and with children, but separated from
their respective spouses, cohabited after a brief courtship sometime in 1979 while their respective marriages still subsisted.
During their union, they set up the Superfreight Customs Brokerage Corporation, with petitioner as president and chairman of
the board of directors, and respondent as vice-president and treasurer. The business flourished and petitioner and respondent
acquired real and personal properties which were registered solely in respondent's name. In 1992, due to irreconcilable
differences, the couple separated. Petitioner demanded from respondent his share in the subject properties, but respondent
refused alleging that said properties had been registered solely in her name.
In her Amended Answer,3 respondent admitted that she engaged in the customs brokerage business with petitioner but alleged
that the Superfreight Customs Brokerage Corporation was organized with other individuals and duly registered with the
Securities and Exchange Commission in 1987. She denied that she and petitioner lived as husband and wife because the fact
was that they were still legally married to their respective spouses. She claimed to be the exclusive owner of all real personal
properties involved in petitioner's action for partition on the ground that they were acquired entirely out of her own money and
registered solely in her name.
On November 25, 1994, respondent filed a Motion for Summary Judgment, 4 in accordance with Rule 34 of the Rules of
Court.5 She contended that summary judgment was proper, because the issues raised in the pleadings were sham and not
genuine, to wit:
A.
The main issue is Can plaintiff validly claim the partition and/or payment of co-ownership share, accounting and
damages, considering that plaintiff and defendant are admittedly both married to their respective spouses under still valid
and subsisting marriages, even assuming as claimed by plaintiff, that they lived together as husband and wife without
benefit of marriage? In other words, can the parties be considered as co-owners of the properties, under the law,
considering the present status of the parties as both married and incapable of marrying each other, even assuming that
they lived together as husband and wife (?)
B.
As a collateral issue, can the plaintiff be considered as an unregistered co-owner of the real properties under the Transfer
Certificates of Title duly registered solely in the name of defendant Ma. Elvira Castillo? This issue is also true as far as the
motor vehicles in question are concerned which are also registered in the name of defendant. 6
On the first point, respondent contended that even if she and petitioner actually cohabited, petitioner could not validly claim a
part of the subject real and personal properties because Art. 144 of the Civil Code, which provides that the rules on coownership shall govern the properties acquired by a man and a woman living together as husband and wife but not married, or
under a marriage which is void ab initio, applies only if the parties are not in any way incapacitated to contract marriage. 7 In the
parties' case, their union suffered the legal impediment of a prior subsisting marriage. Thus, the question of fact being raised by
petitioner, i.e., whether they lived together as husband and wife, was irrelevant as no co-ownership could exist between them.
As to the second issue, respondent maintained that petitioner cannot be considered an unregistered co-owner of the subject
properties on the ground that, since titles to the land are solely in her name, to grant petitioner's prayer would be to allow a
collateral attack on the validity of such titles.
Petitioner opposed respondent's Motion for Summary Judgment. 8 He contended that the case presented genuine factual issues
and that Art. 144 of the Civil Code had been repealed by the Family Code which now allows, under Art. 148, a limited coownership even though a man and a woman living together are not capacitated to marry each other. Petitioner also asserted
that an implied trust was constituted when he and respondent agreed to register the properties solely in the latter's name
although the same were acquired out of the profits made from their brokerage business. Petitioner invoked the following
provisions of the Civil Code:
Art. 1452. If two or more persons agree to purchase property and by common consent the legal title is taken in the name
of one of them for the benefit of all, a trust is created by force of law in favor of the others in proportion to the interest of
each.
Art. 1453. When the property is conveyed to a person in reliance upon his declared intention to hold it for, or transfer it to
another grantor, there is an implied trust in favor of the person whose benefit is contemplated.
On January 30, 1995, the trial court rendered its decision 9 granting respondent's motion for summary judgment. It ruled that an
examination of the pleadings shows that the issues involved were purely legal. The trial court also sustained respondent's
contention that petitioner's action for partition amounted to a collateral attack on the validity of the certificates of title covering
the subject properties. It held that even if the parties really had cohabited, the action for partition could not be allowed because
an action for partition among co-owners ceases to be so and becomes one for title if the defendant, as in the present case,
alleges exclusive ownership of the properties in question. For these reasons, the trial court dismissed Civil Case No. 93-656.
On appeals, the Court of Appeals on November 7, 1996, ordered the case remanded to the court of origin for trial on the merits.
It cited the decision in Roque v. Intermediate Appellate Court 10 to the effect that an action for partition is at once an action for
declaration of co-ownership and for segregation and conveyance of a determinate portion of the properties involved. If the
defendant asserts exclusive title over the property, the action for partition should not be dismissed. Rather, the court should
resolve the case and if the plaintiff is unable to sustain his claimed status as a co-owner, the court should dismiss the action, not
because the wrong remedy was availed of, but because no basis exists for requiring the defendant to submit to partition.
Resolving the issue whether petitioner's action for partition was a collateral attack on the validity of the certificates of title, the
Court of Appeals held that since petitioner sought to compel respondent to execute documents necessary to effect transfer of
what he claimed was his share, petitioner was not actually attacking the validity of the titles but in fact, recognized their validity.
Finally, the appellate court upheld petitioner's position that Art. 144 of the Civil Code had been repealed by Art. 148 of the
Family Code.
Respondent moved for reconsideration of the decision of Court of Appeals. On May 7, 1998, nearly two years after its first
decision, the Court of Appeals granted respondent's motion and reconsidered its prior decision. In its decision now challenged in
the present petition, it held

Prefatorily, and to better clarify the controversy on whether this suit is a collateral attack on the titles in issue, it must be
underscored that plaintiff-appellant alleged in his complaint that all the nine (9) titles are registered in the name of defendantappellee, Ma. Elvira T. Castillo, except one which appears in the name of Eloisa Castillo (seepar. 9, Complaint). However, a
verification of the annexes of such initiatory pleading shows some discrepancies, to wit:
1. TCT No. 149046 (Annex A) = Elvira T. Castillo, single
2. TCT No. 168208 (Annex B) = do
3. TCT No. 37046 (Annex C) = do
4. TCT No. 37047 (Annex D) = do
5. TCT No. 37048 (Annex E) = do
6. TCT No. 30368 (Annex F) = Steelhaus Realty & Dev. Corp.
7. TCT No. 30369 (Annex G) = do
8. TCT No. 30371 (Annex F) = do
9. TCT No. (92323) 67881 (Annex I) = Eloisa Castillo
In this action, plaintiff-appellant seeks to be declared as 1/2 co-owner of the real properties covered by the above listed titles
and eventually for their partition [par. (a), Prayer; p. 4 Records]. Notably, in order to achieve such prayer for a joint co-ownership
declaration, it is unavoidable that the individual titles involved be altered, changed, cancelled or modified to include therein the
name of the appellee as a registered 1/2 co-owner. Yet, no cause of action or even a prayer is contained
filed. Manifestly, absent any cause or prayer for the alteration, cancellation, modification or changing of the titles involved, the
desired declaration of co-ownership and eventual partition will utterly be an indirect or collateral attack on the subject titled in
this suit.
It is here that We fell into error, such that, if not rectified will surely lead to a procedural lapse and a possible injustice. Well
settled is the rules that a certificate of title cannot be altered, modified or cancelled except in a direct proceeding in accordance
with law.
In this jurisdiction, the remedy of the landowner whose property has been wrongfully or erroneously registered in another name
is, after one year from the date of the decree, not to set aside the decree, but respecting it as incontrovertible and no longer
open to review, to bring an action for reconveyance or, if the property had passed into the hands of an innocent purchaser for
value, for damages. Verily, plaintiff-appellant should have first pursued such remedy or any other relief directly attacking the
subject titles before instituting the present partition suit.Apropos, the case at bench appears to have been prematurely filed.
Lastly, to grant the partition prayed for by the appellant will in effect rule and decide against the properties registered in the
names of Steelhouse Realty and Development Corporation and Eloisa Castillo, who are not parties in the case. To allow this to
happen will surely result to injustice and denial of due process of law. . . . 11
Petitioner moved for reconsideration but his motion was denied by the Court of Appeals in its resolution dated December 21,
1998. Hence this petition.
Petitioner contends that: (1) the Court of Appeals, in its first decision of November 7, 1996, was correct in applying the Roque
ruling and in rejecting respondent's claim that she was the sole owner of the subject properties and that the partition suit was a
collateral attack on the titles; (2) the Court of Appeals correctly rules in its first decision that Art. 148 of the Family Code governs
the co-ownership between the parties, hence, the complaint for partition is proper; (3) with respect to the properties registered in
the name of Steelhouse Realty, respondent admitted ownership thereof and, at the very least, these properties could simply be
excluded and the partition limited to the remaining real and personal properties; and (4) the Court of Appeals erred in not
holding that under the Civil Code, there is an implied trust in his favor. 12
The issue in this case is really whether summary judgment, in accordance with Rule 35 of the Rules of Court, is proper. We rule
in the negative.
First. Rule 35, 3 of the Rules of Court provides that summary judgment is proper only when, based on the pleadings,
depositions, and admissions on file, and after summary hearing, it is shown that except as to the amount of damages, there is
no veritable issue regarding any material fact in the action and the movant is entitled to judgment as a matter of
law. 1 Conversely, where the pleadings tender a genuine issue, i.e., an issue of fact the resolution of which calls for the
presentation of evidence, as distinguished from an issue which is sham, fictitious, contrived, set-up in bad faith, or patently
unsubstantial, summary judgment is not proper. 14
In the present case, we are convinced that genuine issues exist. Petitioner anchors his claim of co-ownership on two factual
grounds: first, that said properties were acquired by him and respondent during their union from 1979 to 1992 from profits
derived from their brokerage business; and second, that said properties were registered solely in respondent's name only
because they agreed to that arrangement, thereby giving rise to an implied trust in accordance with Art. 1452 and Art. 1453 of
the Civil Code. These allegations are denied by respondent. She denies that she and petitioner lived together as husband and
wife. She also claims that the properties in question were acquired solely by her with her own money and resources. With such
conflicting positions, the only way to ascertain the truth is obviously through the presentation of evidence by the parties.
The trial court ruled that it is immaterial whether the parties actually lived together as husband and wife because Art. 144 of the
Civil Code can not be made to apply to them as they were both incapacitated to marry each other. Hence, it was impossible for
a co-ownership to exist between them.
We disagree.
Art. 144 of the Civil Code provides:
When a man and a woman live together as husband and wife, but they are not married, or their marriage is void from the
beginning, the property acquired by either or both of them through their work or industry or their wages and salaries shall
be governed by the rules on co-ownership.
This provision of the Civil Code, applies only to cases in which a man and a woman live together as husband and wife without
the benefit of marriage provided they are not incapacitated or are without impediment to marry each other, 15 or in which the
marriage is void ab initio, provided it is not bigamous. Art. 144, therefore, does not cover parties living in an adulterous
relationship. However, Art. 148 of the Family Code now provides for a limited co-ownership in cases where the parties in union
are incapacitated to marry each other. It states:
In cases of cohabitation not falling under the preceding article, 16 only the properties acquired by both of the parties
through their actual joint contribution of money, property or industry shall be owned by them in common in proportion to
their respective contributions. In the absence of proof to the contrary, their contributions and corresponding shares are
presumed to be equal. The same rule and presumption shall apply to joint deposits of money and evidences of credits.

If one of the parties is validly married to another, his or her share in the co-ownership shall accrue to the absolute
community or conjugal partnership existing in such valid marriage. If the party who acted in bad faith is not validly married
to another, his or her share shall be forfeited in the manner provided in the last paragraph of the preceding article.
The foregoing rules on forfeiture shall likewise apply even if both parties are in bad faith.
It was error for the trial court to rule that, because the parties in this case were not capacitated to marry each other at the time
that they were alleged to have been living together, they could not have owned properties in common. The Family Code, in
addition to providing that a co-ownership exists between a man and a woman who live together as husband and wife without the
benefit of marriage, likewise provides that, if the parties are incapacitated to marry each other, properties acquired by them
through their joint contribution of money, property or industry shall be owned by them in common in proportion to their
contributions which, in the absence of proof to the contrary, is presumed to be equal. There is thus co-ownership eventhough
the couple are not capacitated to marry each other.
In this case, there may be a co-ownership between the parties herein. Consequently, whether petitioner and respondent
cohabited and whether the properties involved in the case are part of the alleged co-ownership are genuine and material. All but
one of the properties involved were alleged to have been acquired after the Family Code took effect on August 3, 1988. With
respect to the property acquired before the Family Code took effect if it is shown that it was really acquired under the regime of
the Civil Code, then it should be excluded.
Petitioner also alleged in paragraph 7 of his complaint that:
Due to the effective management, hardwork and enterprise of plaintiff assisted by defendant, their customs brokerage
business grew and out of the profits therefrom, the parties acquired real and personal properties which were, upon
agreement of the parties, listed and registered in defendant's name with plaintiff as the unregistered co-owner of all said
properties. 17
On the basis of this, he contends that an implied trust existed pursuant to Art. 1452 of the Civil Code which provides that "(I)f
two or more persons agree to purchase property and by common consent the legal title is taken in the name of one of them for
the benefit of all, a trust is created by force of law in favor of the others in proportion to the interest of each." We do not think this
is correct. The legal relation of the parties is already specifically covered by Art. 148 of the Family Code under which all the
properties acquired by the parties out of their actual joint contributions of money, property or industry shall constitute a coownership. Co-ownership is a form of trust and every co-owner is a trustee for the other. 18 The provisions of Art. 1452 and Art.
1453 of the Civil Code, then are no longer material since a trust relation already inheres in a co-ownership which is governed
under Title III, Book II of the Civil Code.
Second. The trial court likewise dismissed petitioner's action on the ground that the same amounted to a collateral attack on the
certificates of title involved. As already noted, at first, the Court of Appeals ruled that petitioner's action does not challenge the
validity of respondent's titles. However, on reconsideration, it reversed itself and affirmed the trial court. It noted that petitioner's
complaint failed to include a prayer for the alteration, cancellation, modification, or changing of the titles involved. Absent such
prayer, the appellate court ruled that a declaration of co-ownership and eventual partition would involve an indirect or collateral
attack on the titles. We disagree.
A torrens title, as a rule, is conclusive and indefeasible. Proceeding from this, P.D. No. 1529, 19 48 provides that a certificate of
title shall not be subject to collateral attack and can not be altered, modified, or canceled except in a direct proceeding. When is
an action an attack on a title? It is when the object of the action or proceeding is to nullify the title, and thus challenge the
judgment pursuant to which the title was decreed. The attack is direct when the object of an action or proceeding is to annul or
set aside such judgment, or enjoin its enforcement. On the other hand, the attack is indirect or collateral when, in an action to
obtain a different relief, an attack on the judgment is nevertheless made as an incident thereof. 20
In his complaint for partition, consistent with our ruling in Roque regarding the nature of an action for partition, petitioner seeks
first, a declaration that he is a co-owner of the subject properties; and second, the conveyance of his lawful shares. He does not
attack respondent's titles. Petitioner alleges no fraud, mistake, or any other irregularity that would justify a review of the
registration decree in respondent's favor. His theory is that although the subject properties were registered solely in
respondent's name, but since by agreement between them as well as under the Family Code, he is co-owner of these
properties and as such is entitled to the conveyance of his shares. On the premise that he is a co-owner, he can validly seek the
partition of the properties in co-ownership and the conveyance to him of his share.
Thus, in Guevara v. Guevara, 21 in which a parcel of land bequeathed in a last will and testament was registered in the name of
only one of the heirs, with the understanding that he would deliver to the others their shares after the debts of the original owner
had been paid, this Court ruled that notwithstanding the registration of the land in the name of only one of the heirs, the other
heirs can claim their shares in "such action, judicial or extrajudicial, as may be necessary to partition the estate of the
testator." 22
Third. The Court of Appeals also reversed its first decision on the ground that to order partition will, in effect, rule and decide
against Steelhouse Realty Development Corporation and Eloisa Castillo, both strangers to the present case, as to the
properties registered in their names. This reasoning, however, ignores the fact that the majority of the properties involved in the
present case are registered in respondent's name, over which petitioner claims rights as a co-owner. Besides, other than the
real properties, petitioner also seeks partition of a substantial amount of personal properties consisting of motor vehicles and
several pieces of jewelry. By dismissing petitioner's complaint for partition on grounds of due process and equity, the appellate
court unwittingly denied petitioner his right to prove ownership over the claimed real and personal properties. The dismissal of
petitioner's complaint is unjustified since both ends may be amply served by simply excluding from the action for partition the
properties registered in the name of Steelhouse Realty and Eloisa Castillo.
WHEREFORE, the amended decision of the Court of Appeals, dated May 7, 1998, is REVERSED and the case is REMANDED
to the Regional Trial Court, Branch 59, Makati City for further proceedings on the merits.

G.R. No. 151967

February 16, 2005

JOSEFINA C. FRANCISCO, petitioner,


vs.

MASTER IRON WORKS & CONSTRUCTION CORPORATION and ROBERTO V. ALEJO, Sheriff IV, Regional Trial Court of
Makati City, Branch 142, respondents.
DECISION
CALLEJO, SR., J.:
Before us is a petition for review on certiorari of the Decision1 of the Court of Appeals (CA) in CA-G.R. No. CV No. 59045, which
reversed and set aside the Decision2 of the Regional Trial Court (RTC) of Paraaque, Metro Manila, Branch 260, in Civil Case
No. 94-2260 and the Resolution of the CA denying the petitioners motion for reconsideration of the said decision.
Josefina Castillo was only 24 years old when she and Eduardo G. Francisco were married on January 15, 1983. 3Eduardo was
then employed as the vice president in a private corporation. A little more than a year and seven months thereafter, or on August
31, 1984, the Imus Rural Bank, Inc. (Imus Bank) executed a deed of absolute sale for P320,000.00 in favor of Josefina Castillo
Francisco, married to Eduardo Francisco, covering two parcels of residential land with a house thereon located at St. Martin de
Porres Street, San Antonio Valley I, Sucat, Paraaque, Metro Manila. One of the lots was covered by Transfer Certificate of Title
(TCT) No. 36519, with an area of 342 square meters, while the other lot, with an area of 360 square meters, was covered by
TCT No. 36518.4 The purchase price of the property was paid to the Bank via Check No. 002334 in the amount ofP320,000.00
drawn and issued by the Commercial Bank of Manila, for which the Imus Bank issued Official Receipt No. 121408 on August 31,
1984.5 On the basis of the said deed of sale, TCT Nos. 36518 and 36519 were cancelled and, on September 4, 1984, the
Register of Deeds issued TCT Nos. 87976 (60550) and 87977 (60551) in the name of "Josefina Castillo Francisco married to
Eduardo G. Francisco."6
On February 15, 1985, the Register of Deeds made of record Entry No. 85-18003 at the dorsal portion of the said titles. This
referred to an Affidavit of Waiver executed by Eduardo where he declared that before his marriage to Josefina, the latter
purchased two parcels of land, including the house constructed thereon, with her own savings, and that he was waiving
whatever claims he had over the property.7 On January 13, 1986, Josefina mortgaged the said property to Leonila Cando for a
loan of P157,000.00.8 It appears that Eduardo affixed his marital conformity to the deed. 9
On June 11, 1990, Eduardo, who was then the General Manager and President of Reach Out Trading International, bought
7,500 bags of cement worth P768,750.00 from Master Iron Works & Construction Corporation (MIWCC) but failed to pay for the
same. On November 27, 1990, MIWCC filed a complaint against him in the RTC of Makati City for the return of the said
commodities, or the value thereof in the amount ofP768,750.00. The case was docketed as Civil Case No. 90-3251. On January
8, 1992, the trial court rendered judgment in favor of MIWCC and against Eduardo. The fallo of the decision reads:
Accordingly, the Court renders judgment in favor of the plaintiff Master Iron Works And Construction Corporation against the
defendant [Eduardo] Francisco ordering the latter as follows:
1. To replace to plaintiff 7,500 bags at 50 kilos/bag of Portland cement or, in the alternative, to pay the plaintiff the amount
of P768,750.00;
2. In either case, to pay liquidated damages by way of interest at 12% per annum from June 21, 1990 until fully paid;
3. To pay P50,000.00 as actual damages; and
4. To pay attorneys fees of P153,750.00 and litigation expenses of P20,000.00.
SO ORDERED.10
The decision in Civil Case No. 90-3251 became final and executory and, on June 7, 1994, the court issued a writ of
execution.11 On June 14, 1994, Sheriff Roberto Alejo sold at a public auction one stainless, owner-type jeep forP10,000.00 to
MIWCC.12 Sheriff Alejo issued a Notice of Levy on Execution/Attachment over the lots covered by TCT No. 87976 (60550) and
87977 (60551) for the recovery of the balance of the amount due under the decision of the trial court in Civil Case No. 903251.13 On June 24, 1994, the sale of the property at a public auction was set to August 5, 1994. 14
On July 3, 1994, Josefina executed an Affidavit of Third Party Claim 15 over the two parcels of land in which she claimed that they
were her paraphernal property, and that her husband Eduardo had no proprietary right or interest over them as evidenced by his
affidavit of waiver, a copy of which she attached to her affidavit. She, likewise, requested Sheriff Alejo to cause the cancellation
of the notice of levy on execution/attachment earlier issued by him.
On July 7, 1994, Josefina filed the said Affidavit of Third Party Claim in the trial court and served a copy thereof to the sheriff.
MIWCC then submitted an indemnity bond16 in the amount of P1,361,500.00 issued by the Prudential Guarantee and
Assurance, Inc. The sale at public auction proceeded. MIWCC made a bid for the property for the price of P1,350,000.00.17
On July 28, 1994, Josefina filed a Complaint against MIWCC and Sheriff Alejo in the RTC of Paraaque for damages with a
prayer for a writ of preliminary injunction or temporary restraining order, docketed as Civil Case No. 94-2260. She alleged then
that she was the sole owner of the property levied on execution by Sheriff Alejo in Civil Case No. 90-3251; hence, the levy on
execution of the property was null and void. She reiterated that her husband, the defendant in Civil Case No. 90-3251, had no
right or proprietary interest over the said property as evidenced by his affidavit of waiver annotated at the dorsal portion of the
said title. Josefina prayed that the court issue a temporary restraining order/writ of preliminary injunction to enjoin MIWCC from
causing the sale of the said property at public auction. Considering that no temporary restraining order had as yet been issued
by the trial court, the sheriff sold the subject property at public auction to MIWCC for P1,350,000.00 on August 5,
1994.18However, upon the failure of MIWCC to remit the sheriffs commission on the sale, the latter did not execute a sheriffs
certificate of sale over the property. The RTC of Paraaque, thereafter, issued a temporary restraining order 19 on August 16,
1994.
When Josefina learned of the said sale at public auction, she filed an amended complaint impleading MIWCC, with the following
prayer:
WHEREFORE, premises considered, it is most respectfully prayed to this Honorable Court that, after hearing, judgment be
rendered in favor of the plaintiff and against the defendants and the same be in the following tenor:
1. Ordering the defendants, jointly and severally, to pay the plaintiff the following amounts:
A. The sum of P50,000.00 representing as actual damages;
B. The sum of P200,000.00 representing as moral damages;
C. The sum of P50,000.00 or such amount which this Honorable Court deems just as exemplary damages;
D. The sum of P60,000.00 as and for attorneys fees.
2. Declaring the levying and sale at public auction of the plaintiffs properties null and void;
3. To issue writ of preliminary injunction and makes it permanent;

4. Order the cancellation of whatever entries appearing at the titles as a result of the enforcement of the writ of execution
issued in Civil Case No. 90-3251.
Plaintiff further prays for such other reliefs as may be just under the premises. 20
In its answer to the complaint, MIWCC cited Article 116 of the Family Code of the Philippines and averred that the property was
the conjugal property of Josefina and her husband Eduardo, who purchased the same on August 31, 1984 after their marriage
on January 14, 1983. MIWCC asserted that Eduardo executed the affidavit of waiver to evade the satisfaction of the decision in
Civil Case No. 90-3251 and to place the property beyond the reach of creditors; hence, the said affidavit was null and void.
Before she could commence presenting her evidence, Josefina filed a petition to annul her marriage to Eduardo in the RTC of
Paraaque, Metro Manila, on the ground that when they were married on January 15, 1983, Eduardo was already married to
one Carmelita Carpio. The case was docketed as Civil Case No. 95-0169.
Josefina and Carmelita testified in Civil Case No. 95-0169. Josefina declared that during her marriage to Eduardo, she acquired
the property covered by TCT Nos. 87976 (60550) and 87977 (60551), through the help of her sisters and brother, and that
Eduardo had no participation whatsoever in the said acquisition. She added that Eduardo had five children, namely, Mary Jane,
Dianne, Mary Grace Jo, Mark Joseph and Mary Cecille, all surnamed Francisco.
On September 9, 1996, the RTC of Paraaque rendered judgment 21 in Civil Case No. 95-0169, declaring the marriage between
Josefina and Eduardo as null and void for being bigamous.
In the meantime, Josefina testified in Civil Case No. 94-2260, declaring, inter alia, that she was able to purchase the property
from the Bank when she was still single with her mothers financial assistance; she was then engaged in recruitment when
Eduardo executed an affidavit of waiver; she learned that he was previously married when they already had two children;
nevertheless, she continued cohabiting with him and had three more children by him; and because of Eduardos first marriage,
she decided to have him execute the affidavit of waiver.
Eduardo testified that when his wife bought the property in 1984, he was in Davao City and had no knowledge of the said
purchases; he came to know of the purchase only when Josefina informed him a week after his arrival from Davao; 22 Josefinas
sister, Lolita Castillo, told him that she would collect from him the money his wife borrowed from her and their mother to buy the
property;23 when he told Lolita that he had no money, she said that she would no longer collect from him, on the condition that
he would have no participation over the property,24which angered Eduardo;25 when Josefina purchased the property, he had a
gross monthly income of P10,000.00 and gave P5,000.00 to Josefina for the support of his family; 26 Josefina decided that he
execute the affidavit of waiver because her mother and sister gave the property to her. 27
On December 20, 1997, the trial court rendered judgment finding the levy on the subject property and the sale thereof at public
auction to be null and void. The fallo of the decision reads:
WHEREFORE, PREMISES CONSIDERED, THIS COURT finds the Levying and sale at public auction of the plaintiffs
properties null and void.
The court orders the defendants to, jointly and severally, pay plaintiff the following amounts:
a. The sum of P50,000.00 as actual damages;
b. The sum of P50,000.00 representing as moral damages;
c. The sum of P50,000.00 as exemplary damages;
d. The sum of P60,000.00 as and for attorneys fees.
The court orders the cancellation of whatever entries appearing at the Titles as a result of the enforcement of the writ of
execution issued in Civil Case No. 90-3251.
SO ORDERED.28
The trial court held that the property levied by Sheriff Alejo was the sole and exclusive property of Josefina, applying Articles
144, 160, 175 and 485 of the New Civil Code. The trial court also held that MIWCC failed to prove that Eduardo Francisco
contributed to the acquisition of the property.
MIWCC appealed the decision to the CA in which it alleged that:
I. THE TRIAL COURT ERRED IN RULING THAT THE REAL ESTATE PROPERTIES SUBJECT OF THE AUCTION SALE ARE
PARAPHERNAL PROPERTIES OWNED BY PLAINTIFF-APPELLEE JOSEFINA FRANCISCO;
II. THE TRIAL COURT ERRED IN ALLOWING THE RECEPTION OF REBUTTAL EVIDENCE WITH REGARD TO THE
ANNULMENT OF PLAINTIFF-APPELLEES MARRIAGE WITH EDUARDO FRANCISCO;
III. THE TRIAL COURT ERRED IN RULING THAT THE LEVY ON EXECUTION OF PLAINTIFF-APPELLEES PROPERTIES
SUBJECT OF THE PRESENT CONTROVERSY IS NULL AND VOID;
IV. THE TRIAL COURT ERRED IN ORDERING DEFENDANT-APPELLANT TO PAY DAMAGES TO PLAINTIFF-APPELLEE
FOR ALLEGED IMPROPER LEVY ON EXECUTION. 29
The CA rendered judgment setting aside and reversing the decision of the RTC on September 20, 2001. The falloof the decision
reads:
WHEREFORE, premises considered, the Decision, dated 20 December 1997, of the Regional Trial Court of Paraaque, Branch
260, is hereby REVERSED and SETASIDE and a new one entered dismissing Civil Case No. 94-0126.
SO ORDERED.30
The CA ruled that the property was presumed to be the conjugal property of Eduardo and Josefina, and that the latter failed to
rebut such presumption. It also held that the affidavit of waiver executed by Eduardo was contrary to Article 146 of the New Civil
Code and, as such, had no force and effect. Josefina filed a motion for reconsideration of the decision, which was, likewise,
denied by the CA.
Josefina, now the petitioner, filed the present petition for review, alleging that:
A. THE HONORABLE COURT OF APPEALS ERRED IN FINDING THAT THERE EXISTS A CONJUGAL PARTNERSHIP
BETWEEN PETITIONER AND EDUARDO FRANCISCO;
B. THE HONORABLE COURT OF APPEALS ERRED IN DECLARING THAT THE SUBJECT PROPERTIES WERE NOT
PARAPHERNAL PROPERTIES OF PETITIONER;

C. THE HONORABLE COURT OF APPEALS ERRED IN DISTURBING THE FINDINGS OF FACTS AND CONCLUSION BY
THE TRIAL COURT IN ITS DECISION OF DECEMBER 20, 1997, THE SAME BEING IN ACCORDANCE WITH LAW AND
JURISPRUDENCE.31
The threshold issues for resolution are as follows: (a) whether or not the subject property is the conjugal property of Josefina
Castillo and Eduardo Francisco; and (b) whether or not the subject properties may be held to answer for the personal
obligations of Eduardo.
We shall deal with the issues simultaneously as they are closely related.
The petitioner asserts that inasmuch as her marriage to Eduardo is void ab initio, there is no occasion that would give rise to a
regime of conjugal partnership of gains. The petitioner adds that to rule otherwise would render moot and irrelevant the
provisions on the regime of special co-ownership under Articles 147 and 148 of the Family Code of the Philippines, in relation to
Article 144 of the New Civil Code.
The petitioner avers that since Article 148 of the Family Code governs their property relationship, the respondents must adduce
evidence to show that Eduardo actually contributed to the acquisition of the subject properties. The petitioner asserts that she
purchased the property before her marriage to Eduardo with her own money without any contribution from him; hence, the
subject property is her paraphernal property. Consequently, such property is not liable for the debts of Eduardo to private
respondent MIWCC.
l^vvphi1.net

The respondents, on the other hand, contend that the appellate court was correct in ruling that the properties are conjugal in
nature because there is nothing in the records to support the petitioners uncorroborated claim that the funds she used to
purchase the subject properties were her personal funds or came from her mother and sister. The respondents point out that if,
as claimed by the petitioner, the subject properties were, indeed, not conjugal in nature, then, there was no need for her to
obtain marital (Eduardos) consent when she mortgaged the properties to two different parties sometime in the first quarter of
1986, or after Eduardo executed the affidavit of waiver.
We note that the only questions raised in this case are questions of facts. Under Rule 45 of the Rules of Court, only questions of
law may be raised in and resolved by the Court. The Court may, however, determine and resolve questions of facts in cases
where the findings of facts of the trial court and those of the CA are inconsistent, where highly meritorious circumstances are
present, and where it is necessary to give substantial justice to the parties. In the present action, the findings of facts and the
conclusions of the trial court and those of the CA are opposite. There is thus an imperative need for the Court to delve into and
resolve the factual issues, in tandem with the questions of law raised by the parties.
The petition has no merit.
The petitioner failed to prove that she acquired the property with her personal funds before her cohabitation with Eduardo and
that she is the sole owner of the property. The evidence on record shows that the Imus Bank executed a deed of absolute sale
over the property to the petitioner on August 31, 1984 and titles over the property were, thereafter, issued to the latter as vendee
on September 4, 1984 after her marriage to Eduardo on January 15, 1983.
1vvphi1.nt

We agree with the petitioner that Article 144 of the New Civil Code does not apply in the present case. This Court in Tumlos v.
Fernandez32 held that Article 144 of the New Civil Code applies only to a relationship between a man and a woman who are not
incapacitated to marry each other, or to one in which the marriage of the parties is void from the very beginning. It does not
apply to a cohabitation that is adulterous or amounts to concubinage, for it would be absurd to create a co-ownership where
there exists a prior conjugal partnership or absolute community between the man and his lawful wife. In this case, the petitioner
admitted that when she and Eduardo cohabited, the latter was incapacitated to marry her.
Article 148 of the Family Code of the Philippines, on which the petitioner anchors her claims, provides as follows:
Art. 148. In cases of cohabitation not falling under the preceding Article, only the properties acquired by both of the parties
through their actual joint contribution of money, property, or industry shall be owned by them in common in proportion to their
respective contributions. In the absence of proof to the contrary, their contributions and corresponding shares are presumed to
be equal. The same rule and presumption shall apply to joint deposits of money and evidences of credit.
If one of the parties is validly married to another, his or her share in the co-ownership shall accrue to the absolute community or
conjugal partnership existing in such valid marriage. If the party who acted in bad faith is not validly married to another, his or
her share shall be forfeited in the manner provided in the last paragraph of the preceding Article.
The foregoing rules on forfeiture shall, likewise, apply even if both parties are in bad faith.
Indeed, the Family Code has filled the hiatus in Article 144 of the New Civil Code by expressly regulating in Article 148 the
property relations of couples living in a state of adultery or concubinage. Under Article 256 of the Family Code, the law can be
applied retroactively if it does not prejudice vested or acquired rights. The petitioner failed to prove that she had any vested right
over the property in question.33
Since the subject property was acquired during the subsistence of the marriage of Eduardo and Carmelita, under normal
circumstances, the same should be presumed to be conjugal property.34 Article 105 of the Family Code of the Philippines
provides that the Code shall apply to conjugal partnership established before the code took effect,without prejudice to vested
rights already acquired under the New Civil Code or other laws.35 Thus, even if Eduardo and Carmelita were married before the
effectivity of the Family Code of the Philippines, the property still cannot be considered conjugal property because there can
only be but one valid existing marriage at any given time. 36 Article 148 of the Family Code also debilitates against the petitioners
claim since, according to the said article, a co-ownership may ensue in case of cohabitation where, for instance, one party has a
pre-existing valid marriage provided that the parents prove their actual joint contribution of money, property or industry and only
to the extent of their proportionate interest thereon. 37
We agree with the findings of the appellate court that the petitioner failed to adduce preponderance of evidence that she
contributed money, property or industry in the acquisition of the subject property and, hence, is not a co-owner of the property:
First of all, other than plaintiff-appellees bare testimony, there is nothing in the record to support her claim that the funds she
used to purchase the subject properties came from her mother and sister. She did not, for instance, present the testimonies of
her mother and sister who could have corroborated her claim. Furthermore, in her Affidavit of Third-Party Claim (Exh. "C"), she
stated that the subject properties "are my own paraphernal properties, including the improvements thereon, as such are the
fruits of my own exclusive efforts ," clearly implying that she used her own money and contradicting her later claim that the
funds were provided by her mother and sister. She also stated in her affidavit that she acquired the subject properties before her
marriage to Eduardo Francisco on 15 January 1983, a claim later belied by the presentation of the Deed of Absolute Sale
clearly indicating that she bought the properties from Imus Rural Bank on 31 August 1984, or one year and seven months after
her marriage (Exh. "D"). In the face of all these contradictions, plaintiff-appellees uncorroborated testimony that she acquired
the subject properties with funds provided by her mother and sister should not have been given any weight by the lower court.
It is to be noted that plaintiff-appellee got married at the age of 23. At that age, it is doubtful if she had enough funds of her own
to purchase the subject properties as she claimed in her Affidavit of Third Party Claim. Confronted with this reality, she later
claimed that the funds were provided by her mother and sister, clearly an afterthought in a desperate effort to shield the subject
properties from appellant Master Iron as judgment creditor.38

Aside from her bare claims, the petitioner offered nothing to prove her allegation that she borrowed the amount ofP320,000.00
from her mother and her sister, which she paid to the Imus Bank on August 31, 1984 to purchase the subject property. The
petitioner even failed to divulge the name of her mother and the sources of her income, if any, and that of her sister. When she
testified in Civil Case No. 95-0169, the petitioner declared that she borrowed part of the purchase price of the property from her
brother,39 but failed to divulge the latters name, let alone reveal how much money she borrowed and when. The petitioner even
failed to adduce any evidence to prove that her mother and sister had P320,000.00 in 1984, which, considering the times, was
then quite a substantial amount. Moreover, the petitioners third-party-claim affidavit stating that the properties "are the fruits of
my own exclusive effort before I married Eduardo Francisco" belies her testimony in the trial court and in Civil Case No. 950169.
1awphi1.nt

We note that, as gleaned from the receipt issued by the Imus Bank, the payment for the subject property was drawn via Check
No. 002334 and issued by the Commercial Bank of Manila in the amount of P320,000.00.40 The petitioner failed to testify against
whose account the check was drawn and issued, and whether the said account was owned by her and/or Eduardo Francisco or
her mother, sister or brother. She even failed to testify whether the check was a managers check and, if so, whose money was
used to purchase the same.
We also agree with the findings of the CA that the affidavit of waiver executed by Eduardo on February 15, 1985, stating that the
property is owned by the petitioner, is barren of probative weight. We are convinced that he executed the said affidavit in
anticipation of claims by third parties against him and hold the property liable for the said claims. First, the petitioner failed to
prove that she had any savings before her cohabitation with Eduardo. Second, despite Eduardos affidavit of waiver, he
nevertheless affixed his marital conformity to the real estate mortgage executed by the petitioner over the property in favor of
Leonila on January 13, 1986.41 Third, the petitioner testified that she borrowed the funds for the purchase of the property from
her mother and sister.42Fourth, the petitioner testified that Eduardo executed the affidavit of waiver because she discovered that
he had a first marriage.43 Lastly, Eduardo belied the petitioners testimony when he testified that he executed the affidavit of
waiver because his mother-in-law and sister-in-law had given the property to the petitioner. 44
IN LIGHT OF ALL THE FOREGOING, the petition is DENIED for lack of merit. The Decision of the Court of Appeals reversing
the decision of the Regional Trial Court is AFFIRMED. No pronouncement as to costs.

G.R. No. 146294

July 31, 2006

JOHN ABING, petitioner,


vs.
JULIET WAEYAN, respondent.
DECISION
GARCIA, J.:
In this appeal by way of a petition for review under Rule 45 of the Rules of Court, petitioner John Abing (John, hereafter) seeks
to set aside the Decision1 dated October 24, 2000 of the Court of Appeals (CA) in CA-G.R. SP No. 48675, reversing that of the
Regional Trial Court (RTC) of Benguet, Branch 64, which affirmed an earlier decision of the Municipal Trial Court (MTC) of
Mankayan, Benguet in an ejectment suit thereat commenced by the petitioner against the respondent.
In the main, the controversy is between a man and a woman who, during the good old days, lived together as husband and wife
without the benefit of marriage. During their cohabitation, they acquired properties. Later, they parted ways, and with it this
litigation between them involving one of their common properties.
The facts:
Sometime in 1986, John and respondent Juliet Waeyan (Juliet, for short) met and fell in love with each other. In time, the duo
cohabited as husband and wife without the benefit of marriage. Together, the couple bought a 2-storey residential house from
one Benjamin Macua which was erected on a lot owned by a certain Alejandro Dio on Aurora Street, Mankayan, Benguet.
Consequent to the purchase, the tax declaration of the 2-storey house was transferred in the name of Juliet.
On December 2, 1991, Juliet left for overseas employment in Korea. She would send money to John who deposited the same in
their joint bank account.
In 1992, the original 2-storey residential house underwent renovation. To it was annexed a new structure which housed a sarisari store. This new structure and the sari-sari store thereat are the properties involved in this case.
In 1994, Juliet returned from Korea and continued to live with John. She managed the sari-sari store while John worked as a
mine employee of the Lepanto Consolidated Mining, Inc.
In 1995, the relationship between the two turned from bad to worse. Hence, they decided to partition their properties. For the
purpose, they executed on October 7, 1995 a Memorandum of Agreement. Unfortunately, the document was left unsigned by
the parties although signed by the witnesses thereto. Under their unsigned agreement, John shall leave the couples' dwelling
with Juliet paying him the amount of P428,870.00 representing John's share in all their properties. On the same date October
7, 1995 Juliet paid John the sum ofP232,397.66 by way of partial payment of his share, with the balance of P196,472.34 to be
paid by Juliet in twelve monthly installment beginning November 1995.
Juliet, however, failed to make good the balance. On account thereof, John demanded of her to vacate the annex structure
housing the sari-sari store. Juliet refused, prompting John to file an ejectment suit against her before the MTC of Mankayan,
Benguet.
In his complaint, John alleged that he alone spent for the construction of the annex structure with his own funds and thru money
he borrowed from his relatives. In fact, he added that the tax declaration for the structure was under his name. On this premise,
John claimed exclusive ownership of the subject structure, which thereby gave him the right to eject Juliet therefrom upon the
latter's failure to pay the agreed balance due him under the aforementioned Memorandum of Agreement.
In her answer, Juliet countered that their original house was renovated thru their common funds and that the subject structure
annexed thereto was merely an attachment or an extension of their original residential house, hence the same pertained to the
two of them in common.
In a decision2 dated March 15, 1997, the MTC, on its finding that the money used in the construction of the structure in question
solely came from John, ruled that the same exclusively pertained to the latter, and accordingly ordered Juliet's eviction
therefrom, including the sari-sari store thereat, and required her to surrender possession thereof to John, thus:
WHEREFORE, judgment is rendered in favor of the plaintiff (John) and against the defendant (Juliet).
Defendant is hereby ordered to vacate the premises of the store in litigation covered by Tax Declaration No. 96-001-00445
in the name of the Plaintiff and turn over possession thereof to the latter.
Defendant is hereby further ordered to pay the Plaintiff the sum of P2,500.00 a month from the time she withheld
possession of the store in litigation in June 1996 until she vacates the same and turn over possession thereof to the
Plaintiff.
Defendant is finally ordered, to pay the sum of P5,000.00 to the Plaintiff by way of Attorney's fees; and to pay the costs.
SO ORDERED.
On Juliet's appeal to the RTC, the latter, in its decision of July 29, 1995, affirmed that of the MTC. Undaunted, Juliet then went
to the CA in CA-G.R. SP No. 48675.
As stated at the threshold hereof, the CA, in its Decision of October 24, 2000, 3 reversed that of the RTC, to wit:
WHEREFORE, the petition is GRANTED. The assailed decision of the Regional Trial Court is hereby reversed and set
aside. Petitioner, Juliet Waeyan is entitled to possess the property and maintain therein her business.
SO ORDERED.
Partly says the CA in its reversal disposition:
It is undisputed that the parties lived together as husband and wife without the benefit of marriage from 1986 to 1995 and
that they acquired certain properties which must be divided between them upon the termination of their common law
relationship.
xxx

xxx

xxx

. . . their property relations cannot be governed by the provision of the Civil Code on conjugal partnership... but by the rule
on co-ownership.
xxx

xxx

xxx

. . . the parties' share in respect of the properties they have accumulated during their cohabitation shall be equal unless
there is proof to the contrary.

To the CA, John's evidence failed to establish that he alone spent for the construction of the annex structure. Hence, the same
pertained to both, and being a co-owner herself, Juliet cannot be evicted therefrom, adding that if ever, John's cause of action
should have been for a sum of money "because he claims that Juliet still owes him the payment for the extension." According to
the CA, ejectment cannot lie against Juliet because Juliet's possession of the premises in dispute was not by virtue of a
contract, express or implied, nor did she obtain such possession thru force, intimidation, threat, strategy or stealth.
Hence, John's present recourse, submitting that the CA erred in
1. not giving effect to the parties' Memorandum of Agreement which should have been binding between them albeit
unsigned by both;
2. in holding that the subject premises (annex structure housing the sari-sari store) is owned by the two of them in
common;
3. in ruling that the parties should settle their common properties in a separate action for partition even as the community
character of the subject premises has not been proven.
We AFFIRM with modification.
Essentially, the issues raised center on the core question of whether or not the property subject of the suit pertains to the
exclusive ownership of petitioner, John. Departing from the factual findings of the two courts before it, the CA found that the
premises in dispute is owned in common by Juliet and John, the latter having failed to establish by the required quantum of
proof that the money spent for the construction thereof solely came from him. Being a co-owner of the same structure, Juliet
may not be ejected therefrom.
While the question raised is essentially one of fact, of which the Court normally eschews from, yet, given the conflicting factual
findings of the three courts below, the Court shall go by the exception 4 to the general rule and proceed to make its own
assessment of the evidence.
First and foremost, it is undisputed that the parties hereto lived together as husband and wife from 1986 to 1995 without the
benefit of marriage. Neither is it disputed that sometime in December 1991, Juliet left for Korea and worked thereat, sending
money to John which the latter deposited in their joint account. In fact, Juliet was still in Korea when the annex structure was
constructed in 1992.
Other than John's bare allegation that he alone, thru his own funds and money he borrowed from his relatives, spent for the
construction of the annex structure, evidence is wanting to support such naked claim. For sure, John even failed to reveal how
much he spent therefor. Neither did he divulge the names of the alleged relatives from whom he made his borrowings, let alone
the amount of money he borrowed from them. All that petitioner could offer by way of reinforcing his claim of spending his own
funds and borrowed money in putting up the subject structure was the affidavit executed by a certain Manuel Macaraeg to the
effect that petitioner borrowedP30,000.00 from him. Even then, Macaraeg stated in his affidavit that it was sometime in 1990
when John borrowed said amount from him. With the petitioner's own admission that the subject structure was constructed only
in 1992, or two years after he borrowed P30,000.00 from Macaraeg, it is even doubtful whether the amount he allegedly
borrowed from the latter went into the construction of the structure in dispute. More, it is noted that while petitioner was able to
present in evidence the Macaraeg affidavit, he failed to introduce similar affidavits, if any, of his close relatives from whom he
claimed to have made similar borrowings. For sure, not a single relative came forward to confirm petitioner's tale. In short, there
is a paucity of evidence, testimonial or documentary, to support petitioner's self-serving allegation that the annex structure which
housed the sari-sari store was put up thru his own funds and/or money borrowed by him. Sure, petitioner has in his favor the tax
declaration covering the subject structure. We have, however, ruled time and again that tax declarations do not prove ownership
but at best an indicia of claims of ownership. 5 Payment of taxes is not proof of ownership, any more than indicating possession
in the concept of an owner.6 Neither tax receipts nor declaration of ownership for taxation purposes are evidence of ownership
or of the right to possess realty when not supported by other effective proofs. 7
In this connection, Article 147 of the Family Code is instructive. It reads:
Art. 147. When a man and a woman who are capacitated to marry each other, live exclusively with each other as husband
and wife without the benefit of marriage or under a void marriage, their wages and salaries shall be owned by them in
equal shares and the property acquired by both of them through their work or industry shall be governed by the rules on
co-ownership.
In the absence of proof to the contrary, properties acquired while they lived together shall be presumed to have been
obtained by their joint efforts, work or industry, and shall be owned by them in equal shares. For purposes of this Article, a
party who did not participate in the acquisition by other party of any property shall be deemed to have contributed jointly in
the acquisition thereof if the former's efforts consisted in the care and maintenance of the family and of the household.
The law is clear. In the absence, as here, of proofs to the contrary, any property acquired by common-law spouses during their
period of cohabitation is presumed to have been obtained thru their joint efforts and is owned by them in equal shares. Their
property relationship is governed by the rules on co-ownership. And under this regime, they owned their properties in common
"in equal shares." Being herself a co-owner of the structure in question, Juliet, as correctly ruled by the CA, may not be ejected
therefrom.
True it is that under Article 4878 of the Civil Code, a co-owner may bring an action for ejectment against a co-owner who takes
exclusive possession and asserts exclusive ownership of a common property. It bears stressing, however, that in this case,
evidence is totally wanting to establish John's or Juliet's exclusive ownership of the property in question. Neither did Juliet
obtain possession thereof by virtue of a contract, express or implied, or thru intimidation, threat, strategy or stealth. As borne by
the record, Juliet was in possession of the subject structure and the sari-sari store thereat by virtue of her being a co-owner
thereof. As such, she is as much entitled to enjoy its possession and ownership as John.
We, however, disagree with the ruling of the CA that the subject Memorandum of Agreement, being unsigned by Juliet and
John, has no binding effect between them.
It is a matter of record that pursuant to said Agreement, Juliet did pay John the amount of P232,397.66, as initial payment for
John's share in their common properties, with the balance of P196,472.34 payable in twelve monthly installments beginning
November 1995. It is also a matter of record that the Agreement was signed by the witnesses thereto. Hence, the irrelevant
circumstances that the Agreement was left unsigned by Juliet and John cannot adversely affect its binding force or effect
between them, as evidently, Juliet's initial payment ofP232,397.66 to John was in fulfillment of what the parties had agreed upon
thereunder. However, and as correctly held by the CA, Juliet's failure to pay John the balance of the latter's share in their
common properties could at best give rise to an action for a sum of money against Juliet, or for rescission of the said agreement
and not for ejectment.
WHEREFORE, the petition is DENIED and the assailed CA Decision is AFFIRMED, except that portion thereof denying effect to
the parties' Memorandum of Agreement for being unsigned by both.

G.R. No. L-50127-28 March 30, 1979


VICTOR JUANIZA, Heirs of Josefa P. Leus etc., et al., plaintiffs and appellees,
vs.
EUGENIO JOSE, THE ECONOMIC INSURANCE COMPANY, INC., and ROSALIA ARROYO, defendants and appellants.
Victoriano O. Javier and Ricardo A. Fabros, Jr. for appellees.
Luis Viscocho and Francisco E. Rodrigo, Jr. for appellants.

DE CASTRO, J.:
This case was certified by the Court of Appeals to this Court on the ground that the questions raised in the appeal of the
decision of the Court of First Instance of Laguna are purely questions of law.
Eugenio Jose was the registered owner and operator of the passenger jeepney involved in an accident of collision with a freight
train of the Philippine National Railways that took place on November 23, 1969 which resulted in the death to seven (7) and
physical injuries to five (5) of its passengers. At the time of the accident, Eugenio Jose was legally married to Socorro Ramos
but had been cohabiting with defendant-appellant, Rosalia Arroyo, for sixteen (16) years in a relationship akin to that of husband
and wife.
In the resulting cages for damages filed in the Court of First Instance of Laguna, decision was rendered, the dispositive part of
which reads as follows:
(4) In Civil Case No. SP-867 ordering defendants Eugenio Jose and Rosalia Arroyo jointly and severally to pay
plaintiff Victor Juaniza the sum of P1,600.00 plus legal interest from date of complaint until fully paid and costs of
suit;
(5) In Civil Case No. SP-872, ordering defendants Eugenio Jose and Rosalia Arroyo jointly and severally to pay the
respective heirs of the deceased Josefa P. Leus, Fausto Retrita, Nestor del Rosario Aonuevo and Arceli de la
Cueva in the sum of P12,000.00 for the life of each of said deceased, with legal interest from date of complaint, and
costs of suit. (pp. 47-48, Rello).
Motion for reconsideration was filed by Rosalia Arroyo praying that the decision be reconsidered insofar as it condemns her to
pay damages jointly and severally with her co-defendant, but was denied. The lower court based her liability on the provision of
Article 144 of the Civil Code which reads:
When a man and woman driving together as husband and wife, but they are not married, or their marriage is void
from the beginning, the property acquired by either or both of them through their work or industry or their wages and
salaries shall be governed by the rules on co-ownership.
Rosalia Arroyo then filed her appeal with the Court of Appeals which, as previously stated, certified the same to Us, the question
raised being purely legal as may be seen from the lone assigned error as follows:
The lower court erred in holding defendant-appellant Rosalia Arroyo liable 'for damages resulting from the death and
physical injuries suffered by the passengers' of the jeepney registered in the name of Eugenio Jose, on the
erroneous theory that Eugenio Jose and Rosalia Arroyo, having lived together as husband and wife, without the
benefit of marriage, are co- owners of said jeepney. (p. 2, Appellant's Brief).
The issues thus to be resolved are as follows: (1) whether or not Article 144 of the Civil Code is applicable in a case where one
of the parties in a common-law relationship is incapacitated to marry, and (2) whether or not Rosalia who is not a registered
owner of the jeepney can be held jointly and severally liable for damages with the registered owner of the same.
It has been consistently ruled by this Court that the co-ownership contemplated in Article 144 of the Civil Code requires that the
man and the woman living together must not in any way be incapacitated to contract marriage. (Camporedondo vs. Aznar, L11483, February 4, 1958, 102 Phil. 1055, 1068; Osmea vs. Rodriguez, 54 OG 5526; Malajacan vs. Rubi, 42 OG 5576). Since
Eugenio Jose is legally married to Socorro Ramos, there is an impediment for him to contract marriage with Rosalia Arroyo.
Under the aforecited provision of the Civil Code, Arroyo cannot be a co-owner of the jeepney. The jeepney belongs to the
conjugal partnership of Jose and his legal wife. There is therefore no basis for the liability of Arroyo for damages arising from the
death of, and physical injuries suffered by, the passengers of the jeepney which figured in the collision.
Rosalia Arroyo, who is not the registered owner of the jeepney can neither be liable for damages caused by its operation. It is
settled in our jurisprudence that only the registered owner of a public service vehicle is responsible for damages that may arise
from consequences incident to its operation, or maybe caused to any of the passengers therein. (De Peralta vs. Mangusang, L18110, July 31, 1964, 11 SCRA 598; Tamayo vs. Aquino, L-12634 and L-12720, May 29, 1959; Roque vs. Malibay Transit, L8561, November 18,1955; Montoya vs. Ignacio, L-5868, December 29, 1953).
WHEREFORE, in view of the foregoing, Rosalia Arroyo is hereby declared free from any liability for damages and the appealed
decision is hereby modified accordingly. No costs.

G.R. No. 137650

April 12, 2000

GUILLERMA TUMLOS, petitioner,


vs.
SPOUSES MARIO FERNANDEZ and LOURDES FERNANDEZ, respondents.

PANGANIBAN, J.:
Under Article 148 of the Family Code, a man and a woman who are not legally capacitated to marry each other, but who
nonetheless live together conjugally, may be deemed co-owners of a property acquired during the cohabitation only upon proof
that each made an actual contribution to its acquisition. Hence, mere cohabitation without proof of contribution will not result in a
co-ownership.
The Case
Before us is a Petition for Review under Rule 45 of the Rules of Court, assailing the November 19, 1998 Decision of the Court
of Appeals 1 (CA), which reversed the October 7, 1997 Order of the Regional Trial Court (RTC). 2The dispositive part of the CA
Decision reads:
WHEREFORE, the instant petition is GRANTED, and the questioned orders of the court a quo dated October 7, 1997 and
November 11, 1997, are hereby REVERSED and SET ASIDE. The judgment of the court a quo dated June 5, 1997 is
hereby REINSTATED. Costs against the private respondents. 3
The assailed Order of the RTC disposed as follows:
Wherefore, the decision of this Court rendered on June 5, 1997 affirming in toto the appealed judgment of the [MTC] is
hereby reconsidered and a new one is entered reversing said decision of the [MTC] and dismissing the complaint in the
above-entitled case. 4
Petitioner also assails the February 14, 1999 CA Resolution denying the Motion for Reconsideration.
The Facts
The Court of Appeals narrates the facts as follows:
[Herein respondents] were the plaintiffs in Civil Case No. 6756, an action for ejectment filed before Branch 82 of the MTC
of Valenzuela, Metro Manila against [herein Petitioner] Guillerma Tumlos, Toto Tumlos, and Gina Tumlos. In their
complaint dated July 5, 1996, the said spouses alleged that they are the absolute owners of an apartment building located
at ARTE SUBDIVISION III, Lawang Bato, Valenzuela, Metro Manila; that through tolerance they had allowed the
defendants-private respondents to occupy the apartment building for the last seven (7) years, since 1989, without the
payment of any rent; that it was agreed upon that after a few months, defendant Guillerma Tumlos will pay P1,600.00 a
month while the other defendants promised to pay P1,000.00 a month, both as rental, which agreement was not complied
with by the said defendants; that they have demanded several times [that] the defendants . . . vacate the premises, as
they are in need of the property for the construction of a new building; and that they have also demanded payment of
P84,000.00 from Toto and Gina Tumlos representing rentals for seven (7) years and payment of P143,600.00 from
Guillerma Tumlos as unpaid rentals for seven (7) years, but the said demands went unheeded. They then prayed that the
defendants be ordered to vacate the property in question and to pay the stated unpaid rentals, as well as to jointly pay
P30,000.00 in attorneys fees.
[Petitioner] Guillerma Tumlos was the only one who filed an answer to the complaint. She averred therein that the
Fernandez spouses had no cause of action against her, since she is a co-owner of the subject premises as evidenced by
a Contract to Sell wherein it was stated that she is a co-vendee of the property in question together with [Respondent]
Mario Fernandez. She then asked for the dismissal of the complaint.
After an unfruitful preliminary conference on November 15, 1996, the MTC required the parties to submit their affidavits
and other evidence on the factual issues defined in their pleadings within ten (10) days from receipt of such order,
pursuant to section 9 of the Revised Rule on Summary Procedure. [Petitioner] Guillerma Tumlos submitted her
affidavit/position paper on November 29, 1996, while the [respondents] filed their position paper on December 5, 1996,
attaching thereto their marriage contract, letters of demand to the defendants, and the Contract to Sell over the disputed
property. The MTC thereafter promulgated its judgment on January 22, 1997[.]
xxx

xxx

xxx

Upon appeal to the [RTC], [petitioner and the two other] defendants alleged in their memorandum on appeal that
[Respondent] Mario Fernandez and [Petitioner] Guillerma had an amorous relationship, and that they acquired the
property in question as their "love nest." It was further alleged that they lived together in the said apartment building with
their two (2) children for around ten (10) years, and that Guillerma administered the property by collecting rentals from the
lessees of the other apartments, until she discovered that [Respondent Mario] deceived her as to the annulment of his
marriage. It was also during the early part of 1996 when [Respondent Mario] accused her of being unfaithful and
demonstrated his baseless [jealousy].
In the same memorandum, [petitioner and the two other] defendants further averred that it was only recently that Toto
Tumlos was temporarily accommodated in one of the rooms of the subject premises while Gina Tumlos acted as a nanny
for the children. In short, their presence there [was] only transient and they [were] not tenants of the Fernandez spouses.
On June 5, 1997, the [RTC] rendered a decision affirming in toto the judgment of the MTC.
The [petitioner and the two other defendants] seasonably filed a motion for reconsideration on July 3, 1997, alleging that
the decision of affirmance by the RTC was constitutionally flawed for failing to point out distinctly and clearly the findings of
facts and law on which it was based vis--vis the statements of issues they have raised in their memorandum on appeal.
They also averred that the Contract to Sell presented by the plaintiffs which named the buyer as "Mario P. Fernandez, of
legal age, married to Lourdes P. Fernandez," should not be given credence as it was falsified to appear that way.
According to them, the Contract to Sell originally named "Guillerma Fernandez" as the spouse of [Respondent Mario]. As
found by the [RTC] in its judgment, a new Contract to Sell was issued by the sellers naming the [respondents] as the
buyers after the latter presented their marriage contract and requested a change in the name of the vendee-wife. Such
facts necessitate the conclusion that Guillerma was really a co-owner thereof, and that the [respondents] manipulated the
evidence in order to deprive her of her rights to enjoy and use the property as recognized by law.
xxx

xxx

xxx

The [RTC], in determining the question of ownership in order to resolve the issue of possession, ruled therein that the
Contract to Sell submitted by the Fernandez spouses appeared not to be authentic, as there was an alteration in the name
of the wife of [Respondent] Mario Fernandez. Hence, the contract presented by the [respondents] cannot be given any
weight. The court further ruled that Guillerma and [Respondent Mario] acquired the property during their cohabitation as

husband and wife, although without the benefit of marriage. From such findings, the court concluded that [Petitioner]
Guillerma Tumlos was a co-owner of the subject property and could not be ejected therefrom.
The [respondents] then filed a motion for reconsideration of the order of reversal, but the same was denied by the [RTC].
As earlier stated, the CA reversed the RTC. Hence, this Petition filed by Guillerma Tumlos only.

Ruling of the Court of Appeals


The CA rejected petitioner's claim that she and Respondent Mario Fernandez were co-owners of the disputed property. The CA
ruled:
From the inception of the instant case, the only defense presented by private respondent Guillerma is her right as a co-owner of
the subject property[.]
xxx

xxx

xxx

This claim of co-ownership was not satisfactorily proven by Guillerma, as correctly held by the trial court. No other evidence was
presented to validate such claim, except for the said affidavit/position paper. As previously stated, it was only on appeal that
Guillerma alleged that she cohabited with the petitioner-husband without the benefit of marriage, and that she bore him two (2)
children. Attached to her memorandum on appeal are the birth certificates of the said children. Such contentions and documents
should not have been considered by the . . . (RTC), as they were not presented in her affidavit/position paper before the trial
court (MTC).
xxx

xxx

xxx

However, even if the said allegations and documents could be considered, the claim of co-ownership must still fail. As [herein
Respondent] Mario Fernandez is validly married to [Respondent] Lourdes Fernandez (as per Marriage Contract dated April 27,
1968, p. 45, Original Record), Guillerma and Mario are not capacitated to marry each other. Thus, the property relations
governing their supposed cohabitation is that found in Article 148 of Executive Order No. 209, as amended, otherwise known as
the Family Code of the Philippines[.]
xxx

xxx

xxx

It is clear that actual contribution is required by this provision, in contrast to Article 147 of the Family Code which states that
efforts in the care and maintenance of the family and household are regarded as contributions to the acquisition of common
property by one who has no salary or income or work or industry (Agapay v. Palang, 276 SCRA 340). The care given by one
party [to] the home, children, and household, or spiritual or moral inspiration provided to the other, is not included in Article 148
(Handbook on the Family Code of the Philippines by Alicia V. Sempio-Diy, 1988 ed., p. 209). Hence, if actual contribution of the
party is not proved, there will be no co-ownership and no presumption of equal shares (Agapay, supra at p.
348, citing Commentaries and Jurisprudence on the Civil Code of the Philippines Volume I by Arturo M. Tolentino, 1990 ed., p.
500).
In the instant case, no proof of actual contribution by Guillerma Tumlos in the purchase of the subject property was presented.
Her only evidence was her being named in the Contract to Sell as the wife of [Respondent] Mario Fernandez. Since she failed
to prove that she contributed money to the purchase price of the subject apartment building, We find no basis to justify her coownership with [Respondent Mario]. The said property is thus presumed to belong to the conjugal partnership property of Mario
and Lourdes Fernandez, it being acquired during the subsistence of their marriage and there being no other proof to the
contrary (please see Article 116 of the Family Code).
The court a quo (RTC) also found that [Respondent Mario] has two (2) children with Guillerma who are in her custody, and that
to eject them from the apartment building would be to run counter with the obligation of the former to give support to his minor
illegitimate children, which indispensably includes dwelling. As previously discussed, such finding has no leg to stand on, it
being based on evidence presented for the first time on appeal.
xxx

xxx

xxx

Even assuming arguendo that the said evidence was validly presented, the RTC failed to consider that the need for support
cannot be presumed. Article 203 of the Family Code expressly provides that the obligation to give support shall be demandable
from the time the person who has a right to receive the same needs it for maintenance, but it shall not be paid except from the
date of judicial or extrajudicial demand. . . .
1wphi1.nt

In contrast to the clear pronouncement of the Supreme Court, the RTC instead presumed that Guillerma and her children
needed support from [Respondent Mario]. Worse, it relied on evidence not properly presented before the trial court (MTC).
With regard to the other [defendants], Gina and Toto Tumlos, a close perusal of the records shows that they did not file any
responsive pleading. Hence, judgment may be rendered against them as may be warranted by the facts alleged in the
complaint and limited to what is prayed for therein, as provided for in Section 6 of the Revised Rules on Summary Procedure.
There was no basis for the public respondent to dismiss the complaint against them. 7 (emphasis in the original)
The Issues
In her Memorandum, petitioner submits the following issues for the consideration of the Court:
I. The Court of Appeals gravely erred and abused its discretion in not outrightly dismissing the petition for review filed by
respondents.
II. The Court of Appeals erred in finding that petitioner is not the co-owner of the property in litis.
III. Corollary thereto, the Court of Appeals erred in applying Art. 148 of the Family Code in the case at bar.
IV. The Court of Appeals erred in disregarding the substantive right of support vis--vis the remedy of ejectment resorted
to by respondents. 8
In resolving this case, we shall answer two questions: (a) Is the petitioner a co-owner of the property? (b) Can the claim for
support bar this ejectment suit? We shall also discuss these preliminary matters: (a) whether the CA was biased in favor of
respondents and (b) whether the MTC had jurisdiction over the ejectment suit.
The Courts Ruling
The Petition has no merit.
Preliminary Matters
Petitioner submits that the CA exhibited partiality in favor of herein respondents. This bias, she argues, is manifest in the
following:

1. The CA considered the respondents Petition for Review 9 despite their failure to attach several pleadings as well as the
explanation for the proof of service, despite the clear mandate of Section 11 10 of Rule 13 of the Revised Rules of Court
and despite the ruling in Solar Team Entertainment, Inc. v. Ricafort. 11
2. It allowed respondents to submit the pleadings that were not attached.
3. It considered respondents' Reply dated May 20, 1998, which had allegedly been filed out of time.
4. It declared that the case was submitted for decision without first determining whether to give due course to the Petition,
pursuant to Section 6, Rule 42 of the Rules of Court. 12
The CA, for its part, succinctly dismissed these arguments in this wise:
It is too late in the day now to question the alleged procedural error after we have rendered the decision. More importantly,
when the private respondent filed their comment to the petition on April 26, 1998, they failed to question such alleged
procedural error. Neither have they questioned all the resolutions issued by the Court after their filing of such comment.
They should, therefore, be now considered in estoppel to question the same. 13
We agree with the appellate court. Petitioner never raised these matters before the CA. She cannot be allowed now to
challenge its Decision on grounds of alleged technicalities being belatedly raised as an afterthought. In this light, she cannot
invoke Solar 14 because she never raised this issue before the CA.
More important, we find it quite sanctimonious indeed on petitioners part to rely, on the one hand, on these procedural
technicalities to overcome the appealed Decision and, on the other hand, assert that the RTC may consider the new evidence
she presented for the first time on appeal. Such posturing only betrays the futility of petitioner's assertion, if not its absence of
merit.
One other preliminary matter. Petitioner implies that the court of origin, the Municipal Trial Court (MTC), did not have jurisdiction
over the "nature of the case," alleging that the real question involved is one of ownership. Since the issue of possession cannot
be settled without passing upon that of ownership, she maintains that the MTC should have dismissed the case.
This contention is erroneous. The issue of ownership may be passed upon by the MTC to settle the issue of possession.
disposition, however, is not final insofar as the issue of ownership is concerned, 16 which may be the subject of another
proceeding brought specifically to settle that question.

15

Such

Having resolved these preliminary matters, we now move on to petitioners substantive contentions.
First Issue:
Petitioner as Co-owner
Petitioners central theory and main defense against respondents' action for ejectment is her claim of co-ownership over the
property with Respondent Mario Fernandez. At the first instance before the MTC, she presented a Contract to Sell indicating
that she was his spouse. The MTC found this document insufficient to support her claim. The RTC, however, after considering
her allegation that she had been cohabiting with Mario Fernandez as shown by evidence presented before it, 17 ruled in her
favor.
On the other hand, the CA held that the pieces of evidence adduced before the RTC could no longer be considered because
they had not been submitted before the MTC. Hence, the appellate court concluded that "[t]he claim of co-ownership was not
satisfactorily proven . . ." 18
We agree with the petitioner that the RTC did not err in considering the evidence presented before it. Nonetheless, we reject her
claim that she was a co-owner of the disputed property.
Evidence Presented on
Appeal Before the RTC
In ruling that the RTC erred in considering on appeal the evidence presented by petitioner, the CA relied on the doctrine that
issues not raised during trial could not be considered for the first time during appeal. 19
We disagree. In the first place, there were no new matters or issues belatedly raised during the appeal before the RTC. The
defense invoked by petitioner at the very start was that she was a co-owner. To support her claim, she presented a Contract to
Sell dated November 14, 1986, which stated that Mario Fernandez was legally married to her. The allegation that she was
cohabiting with him was a mere elaboration of her initial theory.
In the second place, procedural rules are generally premised on considerations of fair play. Respondents never objected when
the assailed evidence was presented before the RTC. Thus, they cannot claim unfair surprise or prejudice.
Petitioner Not a Co-Owner Under
Article 144 of the Civil Code
Even considering the evidence presented before the MTC and the RTC, we cannot accept petitioner's submission that she is a
co-owner of the disputed property pursuant to Article 144 of the Civil Code. 20 As correctly held by the CA, the applicable law is
not Article 144 of the Civil Code, but Article 148 of the Family Code which provides:
Art. 148. In cases of cohabitation not falling under the preceding Article, 21 only the properties acquired by both of the
parties through their actual joint contribution of money, property, or industry shall be owned by them in common in
proportion to their respective contributions. In the absence of proof to the contrary, their contributions and corresponding
shares are presumed to be equal. The same rule and presumption shall apply to joint deposits of money and evidences of
credit.
If one of the parties is validly married to another, his or her share in the co-ownership shall accrue to the absolute
community or conjugal partnership existing in such valid marriage. If the party who acted in bad faith is not validly married
to another, his or her share shall be forfeited in the manner provided in the last paragraph of the preceding Article.
The foregoing rules on forfeiture shall likewise apply even if both parties are in bad faith.
Art. 144 of the Civil Code applies only to a relationship between a man and a woman who are not incapacitated to marry each
other, 22 or to one in which the marriage of the parties is void 23 from the beginning. 24 It does not apply to a cohabitation that
amounts to adultery or concubinage, for it would be absurd to create a co-ownership where there exists a prior conjugal
partnership or absolute community between the man and his lawful wife. 25
Based on evidence presented by respondents, as well as those submitted by petitioner herself before the RTC, it is clear that
Mario Fernandez was incapacitated to marry petitioner because he was legally married to Lourdes Fernandez. It is also clear
that, as readily admitted by petitioner, she cohabited with Mario in a state of concubinage. Therefore, Article 144 of the Civil
Code is inapplicable.

As stated above, the relationship between petitioner and Respondent Mario Fernandez is governed by Article 148 of the Family
Code. Justice Alicia V. Sempio-Diy points out 26 that "[t]he Family Code has filled the hiatus in Article 144 of the Civil Code by
expressly regulating in its Article 148 the property relations of couples living in a state of adultery or concubinage.
Hence, petitioners argument that the Family Code is inapplicable because the cohabitation and the acquisition of the
property occurred before its effectivity deserves scant consideration. Suffice it to say that the law itself states that it can be
applied retroactively if it does not prejudice vested or acquired rights. 27 In this case, petitioner failed to show any vested right
over the property in question. Moreover, to resolve similar issues, we have applied Article 148 of the Family Code
retroactively. 28
No Evidence of Actual Joint
Contribution
Another consideration militates against petitioners claim that she is a co-owner of the property. In Agapay, 29 the Court ruled:
Under Article 148, only the properties acquired by both of the parties through their actual joint contribution of
money, property or industry shall be owned by them in common in proportion to their respective contributions. It must be
stressed that the actual contribution is required by this provision, in contrast to Article 147 which states that efforts in the
care and maintenance of the family and household, are regarded as contributions to the acquisition of common property
by one who has no salary or income or work or industry. If the actual contribution of the party is not proved, there will be
no co-ownership and no presumption of equal shares. (emphasis ours)
In this case, petitioner fails to present any evidence that she had made an actual contribution to purchase the subject property.
Indeed, she anchors her claim of co-ownership merely on her cohabitation with Respondent Mario Fernandez.
Likewise, her claim of having administered the property during the cohabitation is unsubstantiated. In any event, this fact by
itself does not justify her claim, for nothing in Article 148 of the Family Code provides that the administration of the property
amounts to a contribution in its acquisition.
Clearly, there is no basis for petitioners claim of co-ownership. The property in question belongs to the conjugal partnership of
respondents. Hence, the MTC and the CA were correct in ordering the ejectment of petitioner from the premises.
Second Issue:
Support versus Ejectment
Petitioner contends that since Respondent Mario Fernandez failed to repudiate her claim regarding the filiation of his alleged
sons, Mark Gil and Michael Fernandez, his silence on the matter amounts to an admission. Arguing that Mario is liable for
support, she advances the theory that the childrens right to support, which necessarily includes shelter, prevails over the right
of respondents to eject her.
We disagree. It should be emphasized that this is an ejectment suit whereby respondents seek to exercise their possessory
right over their property. It is summary in character and deals solely with the issue of possession of the property in dispute.
Here, it has been shown that they have a better right to possess it than does the petitioner, whose right to possess is based
merely on their tolerance.
1wphi1.nt

Moreover, Respondent Mario Fernandez' alleged failure to repudiate petitioner's claim of filiation is not relevant to the present
case. Indeed, it would be highly improper for us to rule on such issue. Besides, it was not properly taken up below. 30 In any
event, Article 298 31 of the Civil Code requires that there should be an extrajudicial demand. 32 None was made here. The CA
was correct when it said:
1wphi1

Even assuming arguendo that the said evidence was validly presented, the RTC failed to consider that the need for
support cannot be presumed. Article [298] of the [New Civil Code] expressly provides that the obligation to give support
shall be demandable from the time the person who has a right to receive the same need it for maintenance, but it shall not
be paid except from the date of judicial and extrajudicial demand. 33
WHEREFORE, the Petition is DENIED and the appealed Decision AFFIRMED. Costs against petitioner.
SO ORDERED.

G.R. No. 140153

March 28, 2001

ANTONIO DOCENA and ALFREDA DOCENA, petitioners,


vs.
HON. RICARDO P. LAPESURA, in his capacity as Presiding Judge of the RTC, Branch III, Guian, Eastern Samar;
RUFINO M. GARADO, Sheriff IV; and CASIANO HOMBRIA, respondents.
GONZAGA-REYES, J.:
This is a petition for review on certiorari under Rule 45 of the Rules of Court seeking the nullification of the Court of
Appeals1 Resolutions dated June 18, 1999 and September 9, 1999 which dismissed the Petition for Certiorari and
Prohibition2 under Rule 65 and denied the corresponding motion for reconsideration, respectively.
The antecedent facts are as follows:
On June 1, 1977, private respondent Casiano Hombria filed a Complaint .for the recovery of a parcel of land against his
lessees, petitioner-spouses Antonio and Alfreda Docena. 3 The petitioners claimed ownership of the land based on occupation
since time immemorial.4 A certain Guillermo Abuda intervened in the case. In a Decision dated November 24, 1989, the trial
court ruled in favor of the petitioners and the intervenor Abuda. 5 On appeal, the Court of Appeals reversed the judgment of the
trial court and ordered the petitioners "to vacate the land they have leased from the plaintiff-appellant [private respondent
Casiano Hombria], excluding the portion which the petitioners reclaimed from the sea and forms part of the shore, as shown in
the Commissioner's Report, and to pay the plaintiff-appellant the agreed rental of P1.00 per year from the date of the filing of
the Complaint until they shall have actually vacated the premises." 6 The Complaint in Intervention of Abuda was dismissed. 7
On May 22, 1995, private respondent Hombria filed a Motion for Execution of the above decision which has already become
final and executory.8 The motion was granted by the public respondent judge, and a Writ of Execution was issued therefor.
However, the public respondent sheriff subsequently filed a Manifestation requesting that he "be clarified in the determination of
that particular portion which is sought to be excluded prior to the delivery of the land adjudged in favor of plaintiff Casiano
Hombria" in view of the defects in the Commissioner's Report and the Sketches attached thereto. 9 After requiring the parties to
file their Comment on the sheriff's Manifestation, the public respondent judge, in a Resolution dated August 30, 1996, held that
"xxx no attempt should be made to alter or modify the decision of the Court of Appeals. What should be delivered therefore to
the plaintiff xxx is that portion leased by the defendant-appellees from the plaintiff-appellant excluding the portion that the
defendant-appellee have reclaimed from the sea and forms part of the shore as shown in the commissioner's report
xxx."10 Pursuant to the Resolution, the public respondent sheriff issued an alias Writ of Demolition. The petitioners filed a Motion
to Set Aside or Defer the Implementation of Writ of Demolition. This motion was denied by the public respondent judge in an
Order dated November 18, 1998, a copy of which was received by the petitioners on December 29, 1998. 11 Also on December
29, 1998, the public respondent judge, in open court, granted the petitioners until January 13, 1999 to file a Motion for
Reconsideration.12 On January 13, 1999, petitioners moved for an extension of the period to file a motion for reconsideration
until January 28, 1999.13 The motion was finally filed by the petitioners on January 27, 1999, but was denied by the trial court in
an Order dated March 17, 1999.14 A copy of the Order was received by the petitioners on May 4, 1999. 15
A Petition for Certiorari and Prohibition was filed by the petitioners with the Court of Appeals, alleging grave abuse of discretion
on the part of the trial court judge in issuing the Orders dated November 18, 1998 and March 17, 1999, and of the sheriff in
issuing the alias Writ of Demolition. In a Resolution dated 4 June 18, 1999, the Court of Appeals dismissed the petition on the
grounds that the petition was filed beyond the 60-day period provided under Section 4 of Rule 65 of the 1997 Revised Rules of
Civil Procedure as amended by Bar Matter No. 803 effective September 1, 1998, and that the certification of non-forum
shopping attached thereto was signed by only one of the petitioners. 16 The Motion for Reconsideration filed by the petitioners
was denied by the Court of Appeals in a Resolution dated September 9, 1999. 17
Hence this petition.

1wphi1.nt

The sole issue in this case is whether or not the Court of Appeals erred in dismissing the Petition for Certiorari and Prohibition.
The petition is meritorious.
The Court of Appeals dismissed the Petition for Certiorari upon the following grounds, viz: (1) the petition was filed beyond the
60-day period provided under Sec. 4, Rule 65 of the 1997 Revised Rules of Civil Procedure as amended by Bar Matter No. 803
effective September l, 1998; and (2) the certification of non-forum shopping was signed by only one of the petitioners.
Upon the first ground, the Court of Appeals stated in its Resolution dated June 18, 1999 that:
xxx the 60-day period is counted not from the receipt of the Order denying their Motion for Reconsideration but from the
date of receipt of the Order of November 18, 1998 which was on December 29, 1998, interrupted by the filing of the
Motion for Reconsideration on January 27, 1999. The Motion for "Reconsideration was denied in an Order dated March
17, 1999 received by the petitioners on May 4, 1999. Counting the remaining period, this petition should have been filed
on June 4, 1999 but it was filed only on June 14, 1999 or ten (10) days beyond the 60-day period computed in accordance
with Bar Matter No. 803.
xxx

xxx

xxx18

The petitioners agree that the counting of the 60-day period should commence on December 29, 1998, the date of the receipt
by the petitioners of the assailed trial court order, interrupted by the filing of a motion for reconsideration on January 27, 1999,
and resume upon receipt by the petitioners of the denial of the motion by the trial court on May 4, 1999; however, the petitioners
contend that from December 29, 1998 up to January 27, 1999, only the 15-day period allowed for the filing of a motion for
reconsideration19 should be deemed to have elapsed considering the grant by the trial court of an extension of the period to file
the motion until January 13, 1999. Hence, on May 4, 1999, .the petitioners still had 45 days to file a petition for
certiorari and/or prohibition, and the filing made on June 14, 1999 was timely.
We hold that the Petition for Certiorari and Prohibition has been timely filed.
A.M. No. 00-2-03-SC, which took effect on September 1, 2000, amended Section 4 of Rule 65 of the 1997 Revised Rules of
Civil Procedure20 to provide thus:
SEC. 4. When and where petition filed. -- The petition shall be filed not later than sixty (60) days from notice of the
judgment, order or resolution. In case a motion for reconsideration or new trial is timely filed, whether such motion is
required or not, the sixty (60) day period shall be counted from notice of the denial of said motion.
The petition shall be filed in the Supreme Court or, if it relates to the acts or omissions of a lower court or of a corporation,
board, officer or person, in the Regional Trial Court exercising jurisdiction over the territorial area as defined by the
Supreme Court. It may also be filed in the Court of Appeals whether or not the same is in aid of its appellate jurisdiction, or
in the Sandiganbayan if it is in aid of its appellate jurisdiction. If it involves the acts or omissions of a quasi-judicial agency,
unless otherwise provided by law or these rules, the petition shall be filed in and cognizable only by the Court of Appeals.
No extension of time to file the petition shall be granted except for compelling reason and in no case exceeding fifteen
(15) days. [Emphasis ours]

In the case of Systems Factors Corporation versus NLRC,21 we held that the abovequoted Resolution, being procedural in
nature, is applicable to actions pending and undetermined at the time of their passage. The retroactive application of procedural
laws such as this Resolution is not violative of any right of a person who may feel adversely affected thereby, as no vested right
may attach to nor arise from procedural laws. 22 The ruling in the Systems Factors case was reiterated in the recent case of Unity
Fishing Development Corporation, et. al. vs. Court of Appeals, et. al. 23 Applying the Resolution to the case at bar, the 60-day
period for the filing of a petition for certiorari and prohibition should be reckoned from the date of receipt of the order denying the
motion for reconsideration, i.e., May 4, 1999, and thus, the filing made on June 14, 1999 was well within the 60-day
reglementary period.
Anent the ground that the certification of non-forum shopping was signed by only one of the petitioners, it is the contention of
the petitioners that the same is sufficient compliance with the requirements of Sections 1 and 2 of Rule 65 (Petition for Certiorari
and Prohibition) in relation to Section 3 of Rule 46 (Original Cases Filed in the Court of Appeals). The petitioners argue that
since they are spouses with joint or indivisible interest over the alleged conjugal property subject of the original action which
gave rise to the petition for certiorari and prohibition, the signing of the certificate of non-forum shopping by only one of them
would suffice, especially considering the long distance they had to travel just to sign the said certificate. 24 Moreover, there is
substantial compliance with the Rules of Court where the certification was signed by the husband who is the statutory
administrator of the conjugal property.25
It has been our previous ruling that the certificate of non-forum shopping should be signed by all the petitioners or plaintiffs in a
case, and that the signing by only one of them is insufficient. In the case of Efren Loquias, et. al. vs. Office of the Ombudsman,
et. al.,26 we held that the signing of the Verification and the Certification on Non-Forum Shopping by only one of the petitioners
constitutes a defect in the petition.27 The attestation contained in the certification on non- forum shopping requires personal
knowledge by the party executing the same,28 and the lone signing petitioner cannot be presumed to have personal knowledge
of the filing or non-filing by his co-petitioners of any action or claim the same as or similar to the current petition. To merit the
Court's consideration, petitioners must show reasonable cause for failure to personally sign the certification.
In the case at bar, however, we hold that the subject Certificate of Non-Forum Shopping signed by the petitioner Antonio
Docena alone should be deemed to constitute substantial compliance with the rules. 29 There are only two petitioners in this case
and they are husband and wife. Their residence is the subject property alleged to be conjugal in the instant verified petition. The
Verification/Certification on Non-Forum Shopping 30 attached to the Petition for Certiorari and Prohibition was signed only by the
husband who certified, inter alia, that he and his wife have not commenced any other action or proceeding involving the same
issues raised in the petition in any court, tribunal or quasi- judicial agency; that to the best of their knowledge no such action is
pending therein; and that he and his wife undertake to inform the Court within five (5) days from notice of any similar action or
proceeding which may have been filed.
The property subject of the original action for recovery is conjugal. Whether it is conjugal under the New Civil Code or the
Family Code, a fact that cannot be determined from the records before us, it is believed that the certificate on non-forum
shopping filed in the Court of Appeals constitutes sufficient compliance with the rules on forum-shopping.
Under the New Civil Code, the husband is the administrator of the conjugal partnership. 31 In fact, he is the sole administrator,
and the wife is not entitled as a matter of right to join him in this endeavor.32 The husband may defend the conjugal partnership
in a suit or action without being joined by the wife. 33 Corollarily, the husband alone may execute the necessary certificate of
non-forum shopping to accompany the pleading. The husband as the statutory administrator of the conjugal property could have
filed the petition for certiorari and prohibition 34alone, without the concurrence of the wife. If suits to defend an interest in the
conjugal properties may be filed by the husband alone, with more reason, he may sign the certificate of non-forum shopping to
be attached to the petition.
Under the Family Code, the administration of the conjugal property belongs to the husband and the wife jointly. 35However, unlike
an act of alienation or encumbrance where the consent of both spouses is required, joint management or administration does
not require that the husband and wife always act together. Each spouse may validly exercise full power of management alone,
subject to the intervention of the court in proper cases as provided under Article 124 of the Family Code. 36 It is believed that
even under the provisions of the Family Code, the husband alone could have filed the petition for certiorari and prohibition to
contest the writs of demolition issued against the conjugal property with the Court of Appeals without being joined by his wife.
The signing of the attached certificate of non-forum shopping only by the husband is not a fatal defect.
More important, the signing petitioner here made the certification in his behalf and that of his wife. The husband may reasonably
be presumed to have personal knowledge of the filing or non-filing by his wife of any action or claim similar to the petition for
certiorari and prohibition given the notices and legal processes involved in a legal proceeding involving real property. We also
see no justifiable reason why he may not lawfully undertake together with his wife to inform the court of any similar action or
proceeding which may be filed. If anybody may repudiate the certification or undertaking for having been incorrectly made, it is
the wife who may conceivably do so.
In view of the circumstances of this case, namely, the property involved is a conjugal property, the petition questioning the writ of
demolition thereof originated from an action for recovery brought against the spouses, and is clearly intended for the benefit of
the conjugal partnership, and the wife, as pointed out in the Motion for Reconsideration in respondent court, was in the province
of Guian, Samar, whereas the petition was prepared in Metro Manila, a rigid application of the rules on forum shopping that
would disauthorize a husband's signing the certification in his behalf and that of his wife is too harsh and is clearly uncalled for.
It bears stressing that the rules on forum shopping, which were designed to promote and facilitate the orderly administration of
justice, should not be interpreted with such absolute literalness as to subvert its own ultimate and legitimate objective. 37
The petitioner's motion for the issuance of a temporary restraining order to put on hold the demolition of the subject property is
principally anchored on their alleged right to the nullification of the assailed orders and writs issued by the public
respondents.38 As the existence of the right being asserted by the petitioners is a factual issue proper for determination by the
Court of Appeals, the motion based thereon should likewise be addressed to the latter court.
WHEREFORE, premises considered, the petition is hereby GRANTED. The Court of Appeals Resolutions dated June 18, 1999
and September 9, 1999 are hereby SET ASIDE and the case is REMANDED to the Court of Appeals for further proceedings.
SO ORDERED.

G.R. No. 162084

June 28, 2005

APRIL MARTINEZ, FRITZ DANIEL MARTINEZ and MARIA OLIVIA MARTINEZ, petitioners,
vs.
RODOLFO G. MARTINEZ, respondent.
DECISION
CALLEJO, SR., J.:
This is a petition for review on certiorari of the Decision1 of the Court of Appeals (CA) in CA-G.R. SP No. 59420 setting aside
and reversing the decision of the Regional Trial Court (RTC) of Manila, Branch 30, in Civil Case No. 00-96962 affirming, on
appeal, the decision of the Metropolitan Trial Court (MTC) of Manila in Civil Case No. 164761 (CV) for ejectment.
The Antecedents
The spouses Daniel P. Martinez, Sr. and Natividad de Guzman-Martinez were the owners of a parcel of land identified as Lot
18-B-2 covered by Transfer Certificate of Title (TCT) No. 54334, as well as the house constructed thereon. 2 On March 6, 1993,
Daniel, Sr. executed a Last Will and Testament 3 directing the subdivision of the property into three lots, namely, Lots 18-B-2-A,
18-B-2-B and 18-B-2-C. He then bequeathed the three lots to each of his sons, namely, Rodolfo, Manolo and Daniel, Jr.;
Manolo was designated as the administrator of the estate.
In May 1995, Daniel, Sr. suffered a stroke which resulted in the paralysis of the right side of his body. Natividad died on October
26, 1996.4 Daniel, Sr. passed away on October 6, 1997. 5
On September 16, 1998, Rodolfo found a deed of sale purportedly signed by his father on September 15, 1996, where the latter
appears to have sold Lot 18-B-2 to Manolo and his wife Lucila. 6 He also discovered that TCT No. 237936 was issued to the
vendees based on the said deed of sale.7
Rodolfo filed a complaint8 for annulment of deed of sale and cancellation of TCT No. 237936 against his brother Manolo and his
sister-in-law Lucila before the RTC. He also filed a criminal complaint for estafa through falsification of a public document in the
Office of the City Prosecutor against Manolo, which was elevated to the Department of Justice. 9
On motion of the defendants, the RTC issued an Order 10 on March 29, 1999, dismissing the complaint for annulment of deed of
sale on the ground that the trial court had no jurisdiction over the action since there was no allegation in the complaint that the
last will of Daniel Martinez, Sr. had been admitted to probate. Rodolfo appealed the order to the CA. 11
On October 4, 1999, Rodolfo filed a Petition with the RTC of Manila for the probate of the last will of the deceased Daniel
Martinez, Sr.12
In the meantime, the spouses Manolo and Lucila Martinez wrote Rodolfo, demanding that he vacate the property. Rodolfo
ignored the letter and refused to do so. This prompted the said spouses to file a complaint for unlawful detainer against Rodolfo
in the MTC of Manila. They alleged that they were the owners of the property covered by TCT No. 237936, and that pursuant to
Presidential Decree (P.D.) No. 1508, the matter was referred to thebarangay for conciliation and settlement, but none was
reached. They appended the certification to file action executed by the barangay chairman to the complaint.
In his Answer13 to the complaint filed on October 11, 1999, Rodolfo alleged, inter alia, that the complaint failed to state a
condition precedent, namely, that earnest efforts for an amicable settlement of the matter between the parties had been exerted,
but that none was reached. He also pointed out that the dispute had not been referred to the barangay before the complaint was
filed.
On October 20, 1999, the spouses Martinez filed an Amended Complaint in which they alleged that earnest efforts toward a
settlement had been made, but that the same proved futile. Rodolfo filed his opposition thereto, on the ground that there was no
motion for the admission of the amended complaint. The trial court failed to act on the matter.
The spouses Martinez alleged in their position paper that earnest efforts toward a compromise had been made and/or exerted
by them, but that the same proved futile.14 No amicable settlement was, likewise, reached by the parties during the preliminary
conference because of irreconcilable differences. The MTC was, thus, impelled to terminate the conference. 15
On February 21, 2000, the trial court rendered judgment in favor of the spouses Martinez. The fallo of the decision reads:
WHEREFORE, premises considered, judgment is rendered in favor of plaintiff. The defendant, including any person claiming
right under him, is ordered:
1) To vacate the subject premises;
2) To pay plaintiff the sum of P10,000.00 a month starting July 17, 1999, the date of last demand until he vacates the
same;
3) To pay the sum of P10,000.00 as and for attorneys fees; and
4) Costs of suit.
SO ORDERED.16
The trial court declared that the spouses Martinez had substantially complied with Article 151 of the Family Code of the
Philippines17 based on the allegations of the complaint and the appended certification to file action issued by
the barangay captain.
Rodolfo appealed the decision to the RTC. On May 31, 2000, the RTC rendered judgment affirming the appealed decision. He
then filed a petition for review of the decision with the CA, alleging that:
1. THE RTC ERRED IN AFFIRMING THE DECISION OF THE MTC WHICH FOUND WITHOUT MERIT THE DEFENSE OF
PETITIONER THAT THERE IS NO ALLEGATION IN THE COMPLAINT THAT PETITIONER HAS UNLAWFULLY WITHHELD
POSSESSION OF THE PROPERTY FROM RESPONDENTS A REQUIREMENT IN [AN] UNLAWFUL DETAINER SUIT.
2. THE RTC ERRED IN AFFIRMING THE DECISION OF THE MTC WHICH FOUND THAT PETITIONERS POSSESSION OF
THE PROPERTY IS BY MERE TOLERANCE OF RESPONDENTS.
3. THE RTC ERRED IN AFFIRMING THE DECISION OF THE MTC WHICH FOUND THAT THE RESPONDENTS HAVE A
CAUSE OF ACTION.
4. THE RTC ERRED IN AFFIRMING THE DECISION OF THE MTC WHICH DID NOT RESOLVE THE SIXTH ISSUE, TO WIT,
"Whether or not this Court has jurisdiction over this case considering that the allegations in the complaint makes out a case
of accion publiciana."
5. THE RTC ERRED IN AFFIRMING THE DECISION OF THE MTC WHICH HAS NO JURISDICTION OVER THE CASE.

6. THE RTC ERRED IN AFFIRMING THE DECISION OF THE MTC WHICH FOUND THAT THE MANDATORY
REQUIREMENT OF CONCILIATION HAS BEEN COMPLIED WITH.
7. THE RTC ERRED IN AFFIRMING THE DECISION OF THE MTC WHICH FOUND THAT THERE WAS SUBSTANTIAL
COMPLIANCE WITH THE KATARUNGANG PAMBARANGAY LAW.
8. THE RTC ERRED IN AFFIRMING THE DECISION OF THE MTC WHICH FOUND THAT THE PENDENCY OF CIVIL CASE
NO. 98-91147 AND SPECIAL PROCEEDINGS NO. 99-95281, INVOLVING THE PETITIONER AND RESPONDENTS AND
INVOLVING THE SAME PROPERTY DID NOT DIVEST THE MTC OF AUTHORITY TO DECIDE THE CASE.
9. THE RTC ERRED IN AFFIRMING THE DECISION OF THE MTC WHICH GRANTED THE RELIEF PRAYED FOR BY THE
RESPONDENTS.
10. THE RTC ERRED IN AFFIRMING THE DECISION OF THE MTC. 18
On November 27, 2003, the CA rendered judgment granting the petition and reversing the decision of the RTC. The appellate
court ruled that the spouses Martinez had failed to comply with Article 151 of the Family code. The CA also held that the defect
in their complaint before the MTC was not cured by the filing of an amended complaint because the latter pleading was not
admitted by the trial court.
Upon the denial of their motion for reconsideration of the said decision, the spouses Martinez filed the present petition for review
on certiorari, in which they raise the following issues:
I.
WHETHER OR NOT THE CERTIFICATION TO FILE ACTION AND THE ALLEGATIONS IN THE COMPLAINT THAT THE
CASE PASSED [THROUGH] THE BARANGAY BUT NO SETTLEMENT WAS REACHED, ARE SUFFICIENT COMPLIANCE
TO PROVE THAT, INDEED, EARNEST EFFORTS WERE, IN FACT, MADE BUT THE SAME HAVE FAILED PRIOR TO THE
FILING OF THE COMPLAINT.
II.
WHETHER OR NOT THE COURT OF APPEALS GRAVELY AND SERIOUSLY ERRED IN FINDING THAT THERE WAS NONCOMPLIANCE WITH THE REQUIREMENT PROVIDED FOR UNDER ARTICLE 151 OF THE FAMILY CODE, CONSIDERING
THAT ONE OF THE PARTIES TO A SUIT IN THIS CASE IS NOT A MEMBER OF THE SAME FAMILY.19
The petitioners alleged that they substantially complied with Article 151 of the Family Code, since they alleged the following in
their original complaint:
2. In compliance with P.D. 1508, otherwise known as the "Katarungang Pambarangay," this case passed [through] the Barangay
and no settlement was forged between plaintiffs and defendant as a result of which Certification to File Action was issued by
Barangay 97, Zone 8, District I, Tondo, Manila. xxx" (Underscoring supplied) 20
Further, the petitioners averred, they alleged in their position paper that they had exerted earnest efforts towards a compromise
which proved futile. They also point out that the MTC resolved to terminate the preliminary conference due to irreconcilable
difference between the parties. Besides, even before they filed their original complaint, animosity already existed between them
and the respondent due to the latters filing of civil and criminal cases against them; hence, the objective of an amicable
settlement could not have been attained. Moreover, under Article 150 of the Family Code, petitioner Lucila Martinez had no
familial relations with the respondent, being a mere sister-in-law. She was a stranger to the respondent; hence, there was no
need for the petitioners21 to comply with Article 151 of the Family Code.
The petition is meritorious.
Article 151 of the Family Code provides:
Art. 151. No suit between members of the same family shall prosper unless it should appear from the verified complaint or
petition that earnest efforts toward a compromise have been made, but that the same have failed. If it is shown that no such
efforts were, in fact, made, the case must be dismissed.
This rule shall not apply to cases which may not be the subject of compromise under the Civil Code.
The phrase "members of the family" must be construed in relation to Article 150 of the Family Code, to wit:
Art. 150. Family relations include those:
(1) Between husband and wife;
(2) Between parents and children;
(3) Among other ascendants and descendants; and
(4) Among brothers and sisters, whether of the full or half-blood.
Article 151 of the Family code must be construed strictly, it being an exception to the general rule. Hence, a sister-in-law or
brother-in-law is not included in the enumeration. 22
As pointed out by the Code Commission, it is difficult to imagine a sadder and more tragic spectacle than a litigation between
members of the same family. It is necessary that every effort should be made toward a compromise before a litigation is allowed
to breed hate and passion in the family and it is known that a lawsuit between close relatives generates deeper bitterness than
between strangers.23
Thus, a partys failure to comply with Article 151 of the Family Code before filing a complaint against a family member would
render such complaint premature.
In this case, the decision of the CA that the petitioners were mandated to comply with Article 151 of the Family code and that
they failed to do so is erroneous.
First. Petitioner Lucila Martinez, the respondents sister-in-law, was one of the plaintiffs in the MTC. The petitioner is not a
member of the same family as that of her deceased husband and the respondent:
As regards plaintiffs failure to seek a compromise, as an alleged obstacle to the present case, Art. 222 of our Civil Code
provides:
"No suit shall be filed or maintained between members of the same family unless it should appear that earnest efforts toward a
compromise have been made, but that the same have failed, subject to the limitations in Article 2035."

It is noteworthy that the impediment arising from this provision applies to suits "filed or maintained betweenmembers of the
same family." This phrase, "members of the same family," should, however, be construed in the light of Art. 217 of the same
Code, pursuant to which:
"Family relations shall include those:
(1) Between husband and wife;
(2) Between parent and child;
(3) Among other ascendants and their descendants;
(4) Among brothers and sisters."
Mrs. Gayon is plaintiffs sister-in-law, whereas her children are his nephews and/or nieces. Inasmuch as none of them is
included in the enumeration contained in said Art. 217 which should be construed strictly, it being an exception to the general
rule and Silvestre Gayon must necessarily be excluded as party in the case at bar, it follows that the same does not come
within the purview of Art. 222, and plaintiffs failure to seek a compromise before filing the complaint does not bar the same. 24
Second. The petitioners were able to comply with the requirements of Article 151 of the Family Code because they alleged in
their complaint that they had initiated a proceeding against the respondent for unlawful detainer in theKatarungang
Pambarangay, in compliance with P.D. No. 1508; and that, after due proceedings, no amicable settlement was arrived at,
resulting in the barangay chairmans issuance of a certificate to file action. 25 The Court rules that such allegation in the
complaint, as well as the certification to file action by the barangay chairman, is sufficient compliance with article 151 of the
Family Code. It bears stressing that under Section 412(a) of Republic Act No. 7160, no complaint involving any matter within the
authority of the Lupon shall be instituted or filed directly in court for adjudication unless there has been a confrontation between
the parties and no settlement was reached. 26
IN LIGHT OF ALL THE FOREGOING, the petition is GRANTED. The Decision of the Court of Appeals in CA-G.R. SP No.
59420 is REVERSED AND SET ASIDE. The Decision of the Metropolitan Trial Court of Manila, as affirmed on appeal by the
Regional Trial Court of Manila, Branch 30, in Civil Case No. 164761(CV) is REINSTATED. No costs.
SO ORDERED.

G.R. No. 125465 June 29, 1999


SPOUSES AUGUSTO HONTIVEROS and MARIA HONTIVEROS, petitioners,
vs.
REGIONAL TRIAL COURT, Branch 25, Iloilo City and SPOUSES GREGORIO HONTIVEROS and TEODORA
AYSON, respondents.

MENDOZA, J.:
On December 3, 1990, petitioners, the spouses Augusto and Maria Hontiveros, filed a complaint for damages against private
respondents Gregorio Hontiveros and Teodora Ayson before the Regional Trial Court of Iloilo City, Branch 25, where it was
docketed as Civil Case No. 19504. In said complaint, petitioners alleged that they are the owners of a parcel of land, in the town
of Jamindan, Province of Capiz, as shown by OCT No. 0-2124, issued pursuant to the decision of the Intermediate. Appellate
Court, dated April 12, 1984, which modified the decision of the Court of First Instance of Capiz, dated January 23, 1975, in a
land registration case 1 filed by private respondent Gregorio Hontiveros; that petitioners were deprived of income from the land as a
result of the filing of the land registration case; that such income consisted of rentals from tenants of the land in the amount of
P66,000.00 per year from 1968 to 1987, and P595,000.00 per year thereafter; and that private respondents filed the land registration
case and withheld possession of the land from petitioners in bad faith. 2
In their answer, private respondents denied that they were married and alleged that private respondent Hontiveros was a
widower while private respondent Ayson was single. They denied that they had deprived petitioners of possession of and
income from the land. On the contrary, they alleged that possession of the property in question had already been transferred to
petitioners on August 7, 1985, by virtue of a writ of possession, dated July 18, 1985, issued by the clerk of court of the Regional
Trial Court of Capiz, Mambusao, the return thereof having been received by petitioners' counsel; that since then, petitioners
have been directly receiving rentals from the tenants of the land, that the complaint failed to state a cause of action since it did
not allege that earnest efforts towards a compromise had been made, considering that petitioner Augusto Hontiveros and
private respondent Gregorio Hontiveros are brothers; that the decision of the Intermediate Appellate Court in Land Registration
Case No. N-581-25 was null and void since it was based upon a ground which was not passed upon by the trial court; that
petitioners' claim for damages was barred by prescription with respect to claims before 1984; that there were no rentals due
since private respondent Hontiveros was a possessor in good faith and for value; and that private respondent Ayson had
nothing to do with the case as she was not married to private respondent Gregorio Hontiveros and did not have any proprietary
interest in the subject property. Private respondents prayed for the dismissal of the complaint and for an order against
petitioners to pay damages to private respondents by way of counterclaim, as well as reconveyance of the subject land to
private respondents. 3
On May 16, 1991, petitioners filed an Amended Complaint to insert therein an allegation that "earnest efforts towards a
compromise have been made between the parties but the same were unsuccessful."
In due time, private respondents filed an Answer to Amended Complaint with Counterclaim, in which they denied, among other
things, that earnest efforts had been made to reach a compromise but the parties was unsuccessful.
On July 19, 1995, petitioners moved for a judgment on the pleadings on the ground that private respondents' answer did not
tender an issue or that it otherwise admitted the material allegations of the complaint. 4 Private respondents opposed the motion
alleging that they had denied petitioners' claims and thus tendered certain issues of fact which could only be resolved after
trial. 5
On November 23, 1995, the trial court denied petitioners' motion. At the same time, however, it dismissed the case on the
ground that the complaint was not verified as required by Art. 151 of the Family Code and, therefore, it did not believe that
earnest efforts had been made to arrive at a compromise. The order of the trial court reads: 6
The Court, after an assessment of the diverging views and arguments presented by both parties, is of the opinion
and so holds that judgment on the pleadings is inappropriate not only for the fact that the defendants in their answer,
particularly in its paragraph 3 to the amended complaint, specifically denied the claim of damages against them, but
also because of the ruling in De Cruz vs. Cruz, G.R. No. 27759, April 17, 1970 (32 SCRA 307), citing Rili vs.
Chunaco, 98 Phil. 505, which ruled that the party claiming damages must satisfactorily prove the amount thereof
and that though the rule is that failure to specifically deny the allegations in the complaint or counter-claim is
deemed an admission of said allegations, there is however an exception to it, that is, that when the allegations refer
to the amount of damages, the allegations must still be proved. This ruling is in accord with the provision of Section
1, Rule 9 of the Rules of Court.
That while the plaintiffs in their amended complaint alleged that earnest efforts towards a compromise with the
defendants were made, the fact is that their complaint was not verified as provided in Article 151 of the Family Code.
Besides, it is not believed that there were indeed earnest efforts made to patch up and/or reconcile the two feuding
brothers, Gregorio and Augusto, both surnamed Hontiveros.
The submission of the plaintiffs that, assuming no such earnest efforts were made, the same is not necessary or
jurisdictional in the light of the ruling in Rufino Magbaleta, et al., petitioner, vs. Hon. Arsenio M. Ganong, et al.,
respondents, No. L-44903, April 22, 1977, is, to the mind of this Court, not applicable to the case at bar for the fact is
the rationale in that case is not present in the instant case considering these salient points:
a) Teodora Ayson, the alleged wife of defendant Gregorio Hontiveros and allegedly not a member of the Hontiveros
Family, is not shown to be really the wife of Gregorio also denied in their verified answer to the amended complaint.
b) Teodora Ayson has not been shown to have acquired any proprietary right or interest in the land that was litigated
by Gregorio and Augusto, unlike the cited case of Magbaleta where it was shown that a stranger to the family
acquired certain right;
c) In the decision rendered by the appellate court no mention was made at all of the name of Teodora Ayson as partawardee of Lot 37 that was adjudged to Gregorio other than himself who was therein described as a widower.
Moreover, Teodora was never mentioned in said decision, nor in the amended complaint and in the amended motion
for judgment on the pleadings that she ever took any part in the act of transaction that gave rise to the damages
allegedly suffered by the plaintiffs for which they now claim some compensation.
WHEREFORE, in the light of all the foregoing premises, the Court orders, as it hereby orders, the dismissal of this
case with cost against the plaintiffs.
SO ORDERED.
Petitioners moved for a reconsideration of the order of dismissal, but their motion was denied. 7 Hence, this petition for review
on certiorari. Petitioner contend:
I. THE REGIONAL TRIAL COURT PALPABLY ERRED IN DISMISSING THE COMPLAINT ON THE
GROUND THAT IT DOES NOT ALLEGE UNDER OATH THAT EARNEST EFFORTS TOWARD A

COMPROMISE WERE MADE PRIOR TO THE FILING THEREOF AS REQUIRED BY ARTICLE 151 OF
THE FAMILY CODE.
II. THE REGIONAL TRIAL COURT PALPABLY ERRED IN NOT DENYING THE MOTION FOR
JUDGMENT ON THE PLEADINGS AND ORDERING A TRIAL ON THE MERITS.
Private respondents raise a preliminary question. They argue that petitioners should have brought this case on appeal to the
Court of Appeals since the order of the trial court judge was actually a decision on the merits. On the other hand, even if petition
for certiorari were the proper remedy, they contend that the petition is defective because the judge of the trial court has not been
impleaded as a respondent. 8
Private respondents' contention is without merit. The petition in this case was filed pursuant to Rule 45 of the Rules of Court. As
explained in Atlas Consolidated Mining Development Corporation v. Court of Appeals: 9
Under Section 5, subparagraph (2)(e), Article VIII of the 1987 Constitution, the Supreme Court is vested with the
power to review, revise, reverse, modify, or affirm on appeal or certiorari as the law or the Rules of Court may
provide, final judgments and orders of lower courts in all cases in which only an error or question of law is involved.
A similar provision is contained in Section 17, fourth paragraph, subparagraph (4) of the Judiciary Act of 1948, as
amended by Republic Act No. 5440. And, in such cases where only questions of law are involved, Section 25 of the
Interim Rules and Guidelines implementing Batas Pambansa Blg. 129, in conjunction with Section 3 of Republic Act
No. 5440, provides that the appeal to the Supreme Court shall be taken by petition for certiorari which shall be
governed by Rule 45 of the Rules of Court.
The rule, therefore, is that direct appeals to this Court from the trial court on questions of law have to be through the
filing of a petition for review on certiorari. It has been held that:
. . . when a CFI (RTC) adjudicates a case in the exercise of its original jurisdiction, the correct mode of
elevating the judgment to the Court of Appeals is by ordinary appeal, or appeal by writ of error, involving
merely the filing of a notice of appeal except only if the appeal is taken in special proceedings and
other cases wherein multiple appeals are allowed under the law, in which even the filing of a record on
appeal is additionally required. Of course, when the appeal would involve purely questions of law or any
of the other cases (except criminal cases as stated hereunder) specified in Section 5(2), Article X of the
Constitution, it should be taken to the Supreme Court by petition for review oncertiorari in accordance
with Rules 42 and 45 of the Rules of Court.
By way of implementation of the aforestated provisions of law, this Court issued on March 9, 1930 Circular No. 2-90,
paragraph 2 of which provides:
2. Appeals from Regional Courts to the Supreme Court. Except in criminal cases where the penalty
imposed is life imprisonment or reclusion perpetua, judgments of regional trial courts may be appealed
to the Supreme Court only by petition for review oncertiorari in accordance with Rule 45 of the Rules of
Court in relation to Section 17 of the Judiciary Act of 1948, as amended, this being the clear intendment
of the provision of the Interim Rules that (a)ppeals to the Supreme Court shall be taken by petition
for certiorariwhich shall be governed by Rule 45 of the Rules of Court.
Under the foregoing considerations, therefore, the inescapable conclusion is that herein petitioner adopted the
correct mode of appeal in G.R. No. 88354 by filing with this Court petition to review oncertiorari the decision of the
Regional Trail Court of Pasig in Civil Case No. 25528 and raising therein purely questions of law.
In Meneses v. Court of Appeals, it was held: 10
It must also be stressed that the trial court's order of 5 June 1992 dismissing the petitioner's complaint was, whether
it was right or wrong, a final order because it had put an end to the particular matter resolved, or settled definitely the
matter therein disposed of and left nothing more to be done by the trial court except the execution of the order. It is a
firmly settled rule that the remedy against such order is the remedy of appeal and not certiorari. That appeal may be
solely on questions of law, in which case it may be taken only to this Court; or on questions of fact and law, in which
case the appeal should be brought to the Court of Appeals. Pursuant to Murillo v. Consul, the appeal to this Court
should be by petition for review on certiorari in accordance with Rule 45 of the Rules of Court.
As private respondents themselves admit, the order of November 23, 1995 is a final order from which an appeal can be taken. It
is final in the sense that it disposes of the pending action before the court and puts an end to the litigation so that nothing more
was left for the trial court to do. 11 Furthermore, as the questions raised as the questions of law, petition for review on certiorari is the
proper mode of appeal. These questions are: (1) whether after denying petitioners' motion for judgment on the pleadings, the trial
court could dismiss their complaint motu proprio for failure to comply with Art. 151 of the Family Code which provides that no suit
between members of the same family shall prosper unless it appears from the complaint, which must be verified, that earnest efforts
towards a compromise have been made but the same have failed; and (2) whether Art. 151 applies to this case. These questions do
not require an examination of the probative value of evidence presented and the truth or falsehood of facts asserted which questions
of fact would entail. 12
On the other hand, petitioners contend that the trial court erred in dismissing the complaint when no motion to that effect was
made by any of the parties. They point out that, in opposing the motion for judgment on the pleadings, private respondents did
not seek the dismissal of the case but only the denial of petitioners' motion. Indeed, what private respondents asked was that
trial be held on the merits.
Of course, there are instances when the trial court may order the dismissal of the case even without a motion to that effect filed
by any of the parties. In Baja v. Macandog, 13 this Court mentioned these cases, to wit:
The court cannot dismiss a case motu proprio without violating the plaintiff's right to be heard, except in the following
instances: if the plaintiff fails to appear at the time of the trial; if he fails to prosecute his action for unreasonable
length of time; or if he fails to comply with the rules or any order of the court; or if the court finds that it has no
jurisdiction over the subject matter of the suit.
However, none of these exceptions appears in this case.
Moreover, the trial court itself found that "judgment on the pleadings is inappropriate not only for the fact that [private
respondents] in their answer . . . specifically denied the claim of damages against them, but also because of the [rule] . . . that
the party claiming damages must satisfactorily prove the amount thereof. . . . " Necessarily, a trial must be held.
Rule 19 of the Rules of Court provides:

14

Sec. 1. Judgment on the pleadings. Where an answer fails to tender an issue, or otherwise admits the material
allegation of the adverse party's pleadings, the court may, on motion of the party, direct judgment on such pleading.
But in actions for annulment of marriage or for legal separation the material facts alleged in the complaint shall
always be proved.

Under the rules, if there is no controverted matter in the case after the answer is filed, the trial court has the discretion to
grant a motion for judgment on the pleadings filed by a party. 15 When there are actual issues raised in the answer, such as
one involving damages, which require the presentation of evidence and assessment thereof by the trial court, it is improper for
the judge to render judgment based on the pleadings alone. 16 In this case, aside from the amount of damages, the following
factual issues have to be resolved, namely, (1) private respondent Teodora Ayson's participation and/or liability, if any to
petitioners and (2) the nature, extent, and duration of private respondents' possession of the subject property. The trial court,
therefore, correctly denied petitioners' motion for judgment on the pleadings.
However, the trial court erred in dismissing petitioners' complaint on the ground that, although it alleged that earnest efforts had
been made toward the settlement of the case but they proved futile, the complaint was not verified for which reason the trial
court could not believe the veracity of the allegation.
The absence of the verification required in Art. 151 does not affect the jurisdiction of the court over the subject matter of the
complaint. The verification is merely a formal requirement intended to secure an assurance that matters which are alleged are
true and correct. If the court doubted the veracity of the allegations regarding efforts made to settle the case among members of
the same family, it could simply have ordered petitioners to verify them. As this Court has already ruled, the court may simply
order the correction of unverified pleadings or act on it and waive strict compliance with the rules in order that the ends of justice
may be served. 17 Otherwise, mere suspicion or doubt on the part of the trial court as to the truth of the allegation that earnest efforts
had been made toward a compromise but the parties' efforts proved unsuccessful is not a ground for the dismissal of an action. Only
if it is later shown that such efforts had not really been exerted would the court be justified in dismissing the action. Thus, Art. 151
provides:
No suit between members of the same family shall prosper unless it should appear from the verified complaint or
petition that earnest efforts toward a compromise have been made, but that the same have failed. It if is shown that
no such efforts were in fact made, the case must be dismissed.
This rule shall not apply to cases which may not be the subject of compromise under the Civil Code.
Moreover, as petitioners contend, Art. 151 of the Family Code does not apply in this case since the suit is not exclusively among
the family members. Citing several cases 18 decided by this Court, petitioners claim that whenever a stranger is a party in the case
involving the family members, the requisite showing the earnest efforts to compromise is no longer mandatory. They argue that since
private respondent Ayson is admittedly a stranger to the Hontiveros family, the case is not covered by the requirements of Art. 151 of
the Family Code.
We agree with petitioners. The inclusion of private respondent Ayson as defendant and petitioner Maria Hontiveros as plaintiff
takes the case out of the ambit of Art. 151 of the Family Code. Under this provision, the phrase "members of the same family"
refers to the husband and wife, parents and children, ascendants and descendants, and brothers and sisters, whether full or
half-blood. 19 As this Court held in Guerrero v. RTC, Ilocos Norte, Br. XVI: 20
As early as two decades ago, we already ruled in Gayon v. Gayon that the enumeration of "brothers and sisters" as
member of the same family does not comprehend "sisters-in-law." In that case, then Chief Justice Concepcion
emphasized that "sisters-in-law" (hence, also "brother-in-law") are not listed under Art. 217 of the New Civil Code as
members of the same family. Since Art. 150 of the Family Code repeats essentially the same enumeration of
"members of the family," we find no reason to alter existing jurisprudence on the mater. Consequently, the court a
quo erred in ruling that petitioner Guerrero, being a brother-in-law of private respondent Hernando, was required to
exert earnest efforts towards a compromise before filing the present suit.
Religious relationship and relationship by affinity are not given any legal effect in this jurisdiction. 21Consequently, private
respondent Ayson, who is described in the complaint as the spouse of respondent Hontiveros, and petitioner Maria Hontiveros,
who is admittedly the spouse of petitioner Augusto Hontiveros, are considered strangers to the Hontiveros family, for purposes
of Art. 151.
Petitioners finally question the constitutionality of Art. 151 of the Family Code on the ground that it in effect amends the Rules of
Court. This, according to them, cannot be done since the Constitution reserves in favor of the Supreme Court the power to
promulgate rules of pleadings and procedure. Considering the conclusion we have reached in this case, however, it is
unnecessary for present purposes to pass upon this question. Courts do not pass upon constitutional questions unless they are
the very lis mota of the case.
WHEREFORE, the petition is GRANTED and the Order, dated November 23, 1995 of the Regional Trial Court of Iloilo City,
Branch 25 is SET ASIDE and the case is remanded to the trial court for further proceedings not inconsistent with this decision.

1wphi1.nt

G.R. NO. 129242

January 16, 2001

PILAR S. VDA. DE MANALO, ANTONIO S. MANALO, ORLANDO S. MANALO, and ISABELITA MANALO ,petitioners,
vs.
HON. COURT OF APPEALS, HON. REGIONAL TRIAL COURT OF MANILA (BRANCH 35), PURITA S. JAYME, MILAGROS
M. TERRE, BELEN M. ORILLANO, ROSALINA M. ACUIN, ROMEO S. MANALO, ROBERTO S. MANALO, AMALIA
MANALO and IMELDA MANALO, respondents.
DE LEON, JR., J.:
This is a petition for review on certiorari filed by petitioners Pilar S. Vda De Manalo, et. Al., seeking to annul the Resolution 1 of
the Court of Appeals 2 affirming the Orders 3 of the Regional Trial Court and the Resolution 4 which denied petitioner' motion for
reconsideration.
The antecedent facts 5 are as follows:
Troadio Manalo, a resident of 1996 Maria Clara Street, Sampaloc, Manila died intestate on February 14, 1992. He was survived
by his wife, Pilar S. Manalo, and his eleven (11) children, namely: Purita M. Jayme, Antonio Manalo, Milagros M. Terre, Belen M.
Orillano, Isabelita Manalo, Rosalina M. Acuin, Romeo Manalo, Roberto Manalo, Amalia Manalo, Orlando Manalo and Imelda
Manalo, who are all of legal age.
1wphi1.nt

At the time of his death on February 14, 1992, Troadio Manalo left several real properties located in Manila and in the province
of Tarlac including a business under the name and style Manalo's Machine Shop with offices at No. 19 Calavite Street, La
Loma, Quezon City and at NO. 45 General Tinio Street, Arty Subdivision, Valenzuela, Metro Manila.
On November 26, 1992, herein respondents, who are eight (8) of the surviving children of the late Troadio Manalo, namely;
Purita, Milagros, Belen Rocalina, Romeo, Roberto, Amalia, and Imelda filed a petition 6 with the respondent Regional Trial Court
of Manila 7 of the judicial settlement of the estate of their late father, Troadio Manalo, and for the appointment of their brother,
Romeo Manalo, as administrator thereof.
On December 15, 1992, the trial court issued an order setting the said petition for hearing on February 11, 1993 and directing
the publication of the order for three (3) consecutive weeks in a newspaper of general circulation in Metro Manila, and further
directing service by registered mail of the said order upon the heirs named in the petition at their respective addresses
mentioned therein.
On February 11, 1993, the date set for hearing of the petition, the trial court issued an order 'declaring the whole world in
default, except the government," and set the reception of evidence of the petitioners therein on March 16, 1993. However, the
trial court upon motion of set this order of general default aside herein petitioners (oppositors therein) namely: Pilar S. Vda. De
Manalo, Antonio, Isabelita and Orlando who were granted then (10) days within which to file their opposition to the petition.
Several pleadings were subsequently filed by herein petitioners, through counsel, culminating in the filling of an Omnibus
Motion8 on July 23, 1993 seeking; (1) to seat aside and reconsider the Order of the trial court dated July 9, 1993 which denied
the motion for additional extension of time file opposition; (2) to set for preliminary hearing their affirmative defenses as grounds
for dismissal of the case; (3) to declare that the trial court did not acquire jurisdiction over the persons of the oppositors; and (4)
for the immediate inhibition of the presiding judge.
On July 30, 1993, the trial court issued an order 9 which resolved, thus:
A. To admit the so-called Opposition filed by counsel for the oppositors on July 20, 1993, only for the purpose of
considering the merits thereof;
B. To deny the prayer of the oppositors for a preliminary hearing of their affirmative defenses as ground for the dismissal
of this proceeding, said affirmative defenses being irrelevant and immaterial to the purpose and issue of the present
proceeding;
C. To declare that this court has acquired jurisdiction over the persons of the oppositors;
D. To deny the motion of the oppositors for the inhibition of this Presiding Judge;
E. To set the application of Romeo Manalo for appointment as regular administrator in the intestate estate of the deceased
Troadio Manalo for hearing on September 9, 1993 at 2:00 o'clock in the afternoon.
Herein petitioners filed a petition for certiorari under Rule 65 of the Rules of Court with the Court of Appeals, docketed as CAG.R. SP. No. 39851, after the trial court in its Order 10 dated September 15, 1993. In their petition for improperly laid in SP.
PROC. No. 92-63626; (2) the trial court did not acquire jurisdiction over their persons; (3) the share of the surviving spouse was
included in the intestate proceedings; (4) there was absence of earnest efforts toward compromise among members of the
same family; and (5) no certification of non-forum shopping was attached to the petition.
Finding the contentions untenable, the Court of Appeals dismissed the petition for certiorari in its Resolution 11promulgated on
September 30, 1996. On May 6, 1997 the motion for reconsideration of the said resolution was likewise dismissed. 12
The only issue raised by herein petitioners in the instant petition for review is whether or not the respondent Court of Appeals
erred in upholding the questioned orders of the respondent trial court which denied their motion for the outright dismissal of the
petition for judicial settlement of estate despite the failure of the petitioners therein to aver that earnest efforts toward a
compromise involving members of the same family have been made prior to the filling of the petition but that the same have
failed.
Herein petitioners claim that the petition in SP. PROC. No. 92-63626 is actually an ordinary civil action involving members of the
same family. They point out that it contains certain averments, which, according to them, are indicative of its adversarial nature,
to wit:
X

Par. 7. One of the surviving sons, ANTONIO MANALO, since the death of his father, TROADIO MANALO, had not made
any settlement, judicial or extra-judicial of the properties of the deceased father TROADIO MANALO.
Par. 8. xxx the said surviving son continued to manage and control the properties aforementioned, without proper
accounting, to his own benefit and advantage xxx.
X

Par. 12. That said ANTONIO MANALO is managing and controlling the estate of the deceased TROADIO MANALO to his
own advantage and to the damage and prejudice of the herein petitioners and their co-heirs xxx.
X

Par. 14. For the protection of their rights and interests, petitioners were compelled to bring this suit and were forced to
litigate and incur expenses and will continue to incur expenses of not less than, P250,000.00 and engaged the services of
herein counsel committing to pay P200,000.00 as and attorney's fees plus honorarium of P2,500.00 per appearance in
court xxx.13
Consequently, according to herein petitioners, the same should be dismissed under Rule 16, Section 1(j) of the Revised Rules
of Court which provides that a motion to dismiss a complaint may be filed on the ground that a condition precedent for filling the
claim has not been complied with, that is, that the petitioners therein failed to aver in the petition in SP. PROC. No. 92-63626,
that earnest efforts toward a compromise have been made involving members of the same family prior to the filling of the
petition pursuant to Article 222 14 of the Civil Code of the Philippines.
The instant petition is not impressed with merit.
It is a fundamental rule that in the determination of the nature of an action or proceeding, the averments 15 and the character of
the relief sought 16 in the complaint, or petition, as in the case at bar, shall be controlling. A careful srutiny of the Petition for
Issuance of Letters of Administration, Settlement and Distribution of Estatein SP. PROC. No. 92-63626 belies herein petitioners'
claim that the same is in the nature of an ordinary civil action. The said petition contains sufficient jurisdictional facts required in
a petition for the settlement of estate of a deceased person such as the fat of death of the late Troadio Manalo on February 14,
1992, as well as his residence in the City of Manila at the time of his said death. The fact of death of the decedent and of his
residence within he country are foundation facts upon which all the subsequent proceedings in the administration of the estate
rest.17The petition is SP.PROC No. 92-63626 also contains an enumeration of the names of his legal heirs including a tentative
list of the properties left by the deceased which are sought to be settled in the probate proceedings. In addition, the relief's
prayed for in the said petition leave no room for doubt as regard the intention of the petitioners therein (private respondents
herein) to seek judicial settlement of the estate of their deceased father, Troadio Manalo, to wit;
PRAYER
WHEREFORE, premises considered, it is respectfully prayed for of this Honorable Court:
a. That after due hearing, letters of administration be issued to petitioner ROMEO MANALO for the administration of the
estate of the deceased TROADIO MANALO upon the giving of a bond in such reasonable sum that this Honorable Court
may fix.
b. That after all the properties of the deceased TROADIO MANALO have been inventoried and expenses and just debts, if
any, have been paid and the legal heirs of the deceased fully determined, that the said estate of TROADIO MANALO be
settled and distributed among the legal heirs all in accordance with law.
c. That the litigation expenses of these proceedings in the amount of P250,000.00 and attorney's fees in the amount of
P300,000.00 plus honorarium of P2,500.00 per appearance in court in the hearing and trial of this case and costs of suit
be taxed solely against ANTONIO MANALO. 18
Concededly, the petition in SP. PROC. No. 92-63626 contains certain averments which may be typical of an ordinary civil action.
Herein petitioners, as oppositors therein, took advantage of the said defect in the petition and filed their so-called Opposition
thereto which, as observed by the trial court, is actually an Answer containing admissions and denials, special and affirmative
defenses and compulsory counterclaims for actual, moral and exemplary damages, plus attorney's fees and costs 19 in an
apparent effort to make out a case of an ordinary civil action and ultimately seek its dismissal under Rule 16, Section 1(j) of the
Rules of Court vis--vis, Article 222 of civil of the Civil Code.
It is our view that herein petitioners may not be allowed to defeat the purpose of the essentially valid petition for the settlement
of the estate of the late Troadio Manalo by raising matters that as irrelevant and immaterial to the said petition. It must be
emphasized that the trial court, siting as a probate court, has limited and special jurisdiction 20 and cannot hear and dispose of
collateral matters and issues which may be properly threshed out only in an ordinary civil action. In addition, the rule has always
been to the effect that the jurisdiction of a court, as well as the concomitant nature of an action, is determined by the averments
in the complaint and not by the defenses contained in the answer. If it were otherwise, it would not be too difficult to have a case
either thrown out of court or its proceedings unduly delayed by simple strategem. 21 So it should be in the instant petition for
settlement of estate.
Herein petitioners argue that even if the petition in SP. PROC. No. 92-63626 were to be considered as a special proceeding for
the settlement of estate of a deceased person, Rule 16, Section 1(j) of the Rules of Court vis--visArticle 222 of the Civil Code
of the Philippines would nevertheless apply as a ground for the dismissal of the same by virtue of ule 1, Section 2 of the Rules
of Court which provides that the 'rules shall be liberally construed in order to promote their object and to assist the parties in
obtaining just, speedy and inexpensive determination of every action and proceedings.' Petitioners contend that the term
"proceeding" is so broad that it must necessarily include special proceedings.
The argument is misplaced. Herein petitioners may not validly take refuge under the provisions of Rule 1, Section 2, of the
Rules of Court to justify the invocation of Article 222 of the Civil Code of the Philippines for the dismissal of the petition for
settlement of the estate of the deceased Troadio Manalo inasmuch as the latter provision is clear enough. To wit:
Art. 222. No suit shall be filed or maintained between members of the same family unless it should appear that earnest efforts
toward a compromise have been made, but that the same have failed, subject to the limitations in Article 2035(underscoring
supplied).22
The above-quoted provision of the law is applicable only to ordinary civil actions. This is clear from the term 'suit' that it refers to
an action by one person or persons against another or other in a court of justice in which the plaintiff pursues the remedy which
the law affords him for the redress of an injury or the enforcement of a right, whether at law or in equity. 23 A civil action is thus an
action filed in a court of justice, whereby a party sues another for the enforcement of a right, or the prevention or redress of a
wrong.24 Besides, an excerpt form the Report of the Code Commission unmistakably reveals the intention of the Code
Commission to make that legal provision applicable only to civil actions which are essentially adversarial and involve members
of the same family, thus:
It is difficult to imagine a sadder and more tragic spectacle than a litigation between members of the same family. It is
necessary that every effort should be made toward a compromise before litigation is allowed to breed hate and passion in
the family. It is know that lawsuit between close relatives generates deeper bitterness than stranger. 25
It must be emphasized that the oppositors (herein petitioners) are not being sued in SP. PROC. No. 92-63626 for any cause of
action as in fact no defendant was imploded therein. The Petition for issuance of letters of Administration, Settlement and
Distribution of Estate in SP. PROC. No. 92-63626 is a special proceeding and, as such, it is a remedy whereby the petitioners
therein seek to establish a status, a right, or a particular fact. 26 the petitioners therein (private respondents herein) merely seek
to establish the fat of death of their father and subsequently to be duly recognized as among the heirs of the said deceased so
that they can validly exercise their right to participate in the settlement and liquidation of the estate of the decedent consistent
with the limited and special jurisdiction of the probate court.
1wphi1.nt

WHEREFORE, the petition in the above-entitled case, is DENIED for lack of merit, Costs against petitioners.

A.M. No. 1022-MJ May 7, 1976


REDENTOR ALBANO, complainant,
vs.
MUNICIPAL JUDGE PATROCINIO C. GAPUSAN of Dumalneg, Ilocos Norte, respondent.
RESOLUTION
AQUINO, J.:
Redentor Albano in a verified complaint dated August 18, 1975 charged Municipal Judge Patrocinio C. Gapusan of Dumalneg
and Adams, Ilocos Norte (1) with incompetence and Ignorance of the law for having prepared and notarized a document
providing for tile personal separation of husband and wife and the extrajudicial liquidation of their conjugal partnership and (2)
with having allegedly influenced Judge Zacarias A. Crispin of the Court of First Instance of Ilocos Norte in deciding two criminal
cases.
Malpractice as a notary. In 1941 or five years before his appointment to the bench, respondent Gapusan notarized a
document for the personal separation of the spouses Valentina Andres and Guillermo Maligta of Barrio 6, Vintar, Ilocos Norte
and for the extrajudicial liquidation of their conjugal partnership.
It was stipulated in that document that if either spouse should commit adultery or concubinage, as the case may be, then the
other should refrain from filing an action against the other.
Judge Gapusan denied that he drafted the agreement. He explained that the spouses had been separated for a long time when
they signed the separation agreement and that the wife had begotten children with her paramour. He said that there was a
stipulation in the agreement that the spouses would live together in case of reconciliation. His belief was that the separation
agreement forestalled the occurrence of violent incidents between the spouses.
Albano in filing the malpractice charge is in effect asking this Court to take belated disciplinary action against Judge Gapusan as
a member of the bar or as a notary. (He was admitted to the bar in 1937).
There is no question that the covenents contained in the said separation agreement are contrary to law, morals and good
customs (Biton vs. Momongan, 62 Phil. 7). Those stipulations undermine the institutions of marriage and the family, "Marriage is
not a mere contract but an inviolable social institution". "The family is a basic social institution which public policy cherishes and
protects." (Arts. 52 and 216, Civil Code). Marriage and the family are the bases of human society throughout the civilized world
(Adong vs. Cheong Seng Gee, 43 Phil. 43; Ramirez vs. Gmur, 42 Phil. 855, 864; Goitia vs. Campos Rueda, 35 Phil. 252, 254;
Brown vs. Yambao, 102 Phil. 168).
To preserve the institutions of marriage and the family, the law considers as void "any contract for personal separation between
husband and wife" and "every extrajudicial agreement, during the marriage, for the dissolution of the conjugal partnership" (Art.
221, Civil Code). Before the new Civil Code, it was held that the extrajudicial dissolution of the conjugal partnership without
judicial sanction was void (Quintana vs. Lerma, 24 Phil. 285; De Luna vs. Linatoc, 74 Phil. 15).
A notary should not facilitate the disintegration of a marriage and the family by encouraging the separation of the spouses and
extrajudically dissolving the conjugal partnership. Notaries were severely censured by this Court for notarizing documents which
subvert the institutions of marriage and the family (Selanova vs. Mendoza, Adm. Matter No. 804-CJ, May 19, 1975, 64 SCRA
69; Miranda vs. Fuentes, Adm. Case No. 241, April 30, 1966, 16 SCRA 802; Biton vs. Momongan, supra,, Panganiban vs.
Borromeo, 58 Phil. 367; In re Santiago, 70 Phil. 66; Balinon vs. De Leon, 94 Phil. 277).
Respondent Gapusan as a member of the bar should be censured for having notarized the void separation agreement already
mentioned.
However, his notarization of that document does not warrant any disciplinary action against him as a municipal judge (he was
appointed in 1946 as justice of the peace) especially considering that his appointment to the judiciary was screened by the
Commission on Appointments (See Ty vs. San Diego, Adm. Matter No. 169-J, June 29, 1972).
Alleged misconduct in influencing CFI Judge. Albano complains that Judge Gapusan took advantage of his intimacy with
Judge Crispin. He implies that by reason of that intimacy Judge Crispin acquitted of frustrated murder the defendants in
Criminal Case No. 102-III, People vs. Freddie Gapusan Gamboa, et al. and convicted Albano (complainant herein) of double
frustrated murder with triple attempted murder in Criminal Case No. 70-III.
Albano said that Freddie Gapusan, an accused in the first criminal case above-mentioned and a complaining witness in the
other case against Albano, is a relative of Judge Gapusan. He revealed that after the acquittal decision was rendered by Judge
Crispin in Criminal Case No. 102 III, the relatives of the accused in that case were saying that their relationship to Judge
Gapusan, a friend of Judge Crispin, proved to be "worthwhile and useful".
Judge Gapusan admitted in his answer that he is close to Judge Crispin because they used to be members of the Municipal
Judges League (when it was headed by Judge Crispin) and because the latter used to be an Executive Judge (with supervision
over municipal judges). Respondent said that his association with Judge Crispin "was purely official".
Judge Gapusan also admitted that Freddie Gapusan is his distant relative. He denied that he influenced Judge Crispin in
rendering his decisions in the two criminal cases.
It is manifest that Alliano's imputation that Judge Gapusan influenced Judge Crispin is anchored on mere suspicion. If he has
any evidence that Judge Crispin committed any irregularity due to the alleged influence exerted by Judge Gapusan, then Albano
should have complained against Judge Crispin's actuations. He should riot vent his ire on Judge Gapusan alone.
When an officer or court allows itself to enter upon the sea of suspicion, it permits itself to enter upon a sea which has no shore,
and the embarkation is without a rudder or compass to control the direction or to ascertain its bearing." (Dy Keng vs. Collector of
Customs, 40 Phil, 118, 123).
A person has freedom to choose his friends and to hobnob with them. It is not a crime nor unethical per se for a municipal judge
to fraternize with a Judge of the Court of First Instance. Whether the fraternization resulted in an unjust verdict rendered by the
Judge of the Court of First Instance due to the sinister or corruptive influence of the municipal judge cannot be shown by mere
inference, or conjecture. It should be Substantiated by solid evidence. The unjustness of the decision should be indubitably
established.
The second charge should be dismissed for being speculative and unfair to Judge Crispin. (He retired in September, 1975).
WHEREFORE, the respondent, as a member of the bar, is for having notarized the above-mentioned void agreement. The
second charge is dismissed.

The Family Home


G.R. No. 86355 May 31, 1990
JOSE MODEQUILLO, petitioner,
vs.
HON. AUGUSTO V. BREVA FRANCISCO SALINAS, FLORIPER ABELLAN-SALINAS, JUANITO CULAN-CULAN and
DEPUTY SHERIFF FERNANDO PLATA respondents.
Josefina Brandares-Almazan for petitioner.
ABC Law Offices for private respondents.

GANCAYCO, J.:
The issue in this petition is whether or not a final judgment of the Court of Appeals in an action for damages may be satisfied by
way of execution of a family home constituted under the Family Code.
The facts are undisputed.
On January 29, 1988, a judgment was rendered by the Court of Appeals in CA-G.R. CV No. 09218 entitled"Francisco Salinas,
et al. vs. Jose Modequillo, et al.," the dispositive part of which read as follows:
WHEREFORE, the decision under appeal should be, as it is hereby, reversed and set aside. Judgment is hereby
rendered finding the defendants-appellees Jose Modequillo and Benito Malubay jointly and severally liable to
plaintiffs-appellants as hereinbelow set forth. Accordingly, defendants-appellees are ordered to pay jointly and
severally to:
1. Plaintiffs-appellants, the Salinas spouses:
a. the amount of P30,000.00 by way of compensation for the death of their son Audie Salinas;
b. P10,000.00 for the loss of earnings by reason of the death of said Audie Salinas;
c. the sum of P5,000.00 as burial expenses of Audie Salinas; and
d. the sum of P5,000.00 by way of moral damages.
2. Plaintiffs-appellants Culan-Culan:
a. the sum of P5,000.00 for hospitalization expenses of Renato Culan- Culan; and
b. P5,000.00 for moral damages.
3. Both plaintiff-appellants Salinas and Culan-Culan, P7,000.00 for attorney's fees and litigation expenses.
All counterclaims and other claims are hereby dismissed.

The said judgment having become final and executory, a writ of execution was issued by the Regional Trial Court of Davao City
to satisfy the said judgment on the goods and chattels of the defendants Jose Modequillo and Benito Malubay at Malalag,
Davao del Sur.
On July 7, 1988, the sheriff levied on a parcel of residential land located at Poblacion Malalag, Davao del Sur containing an
area of 600 square meters with a market value of P34,550.00 and assessed value of P7,570.00 per Tax Declaration No. 8700801359, registered in the name of Jose Modequillo in the office of the Provincial Assessor of Davao del Sur; and a parcel of
agricultural land located at Dalagbong Bulacan, Malalag, Davao del Sur containing an area of 3 hectares with a market value of
P24,130.00 and assessed value of P9,650.00 per Tax Declaration No. 87-08-01848 registered in the name of Jose Modequillo
in the office of the Provincial Assessor of Davao del Sur. 2
A motion to quash and/or to set aside levy of execution was filed by defendant Jose Modequillo alleging therein that the
residential land located at Poblacion Malalag is where the family home is built since 1969 prior to the commencement of this
case and as such is exempt from execution, forced sale or attachment under Articles 152 and 153 of the Family Code except for
liabilities mentioned in Article 155 thereof, and that the judgment debt sought to be enforced against the family home of
defendant is not one of those enumerated under Article 155 of the Family Code. As to the agricultural land although it is
declared in the name of defendant it is alleged to be still part of the public land and the transfer in his favor by the original
possessor and applicant who was a member of a cultural minority was not approved by the proper government agency. An
opposition thereto was filed by the plaintiffs.
In an order dated August 26, 1988, the trial court denied the motion. A motion for reconsideration thereof was filed by defendant
and this was denied for lack of merit on September 2, 1988.
Hence, the herein petition for review on certiorari wherein it is alleged that the trial court erred and acted in excess of its
jurisdiction in denying petitioner's motion to quash and/or to set aside levy on the properties and in denying petitioner' motion for
reconsideration of the order dated August 26, 1988. Petitioner contends that only a question of law is involved in this petition.
He asserts that the residential house and lot was first occupied as his family residence in 1969 and was duly constituted as a
family home under the Family Code which took effect on August 4, 1988. Thus, petitioner argues that the said residential house
and lot is exempt from payment of the obligation enumerated in Article 155 of the Family Code; and that the decision in this
case pertaining to damages arising from a vehicular accident took place on March 16, 1976 and which became final in 1988 is
not one of those instances enumerated under Article 155 of the Family Code when the family home may be levied upon and
sold on execution. It is further alleged that the trial court erred in holding that the said house and lot became a family home only
on August 4, 1988 when the Family Code became effective, and that the Family Code cannot be interpreted in such a way that
all family residences are deemed to have been constituted as family homes at the time of their occupancy prior to the effectivity
of the said Code and that they are exempt from execution for the payment of obligations incurred before the effectivity of said
Code; and that it also erred when it declared that Article 162 of the Family Code does not state that the provisions of Chapter 2,
Title V have a retroactive effect.
Articles 152 and 153 of the Family Code provide as follows:
Art. 152. The family home, constituted jointly by the husband and the wife or by an unmarried head of a family, is the
dwelling house where they and their family reside, and the land on which it is situated.
Art. 153. The family home is deemed constituted on a house and lot from the time it is occupied as a family
residence. From the time of its constitution and so long as any of its beneficiaries actually resides therein, the family
home continues to be such and is exempt from execution, forced sale or attachment except as hereinafter provided
and to the extent of the value allowed by law.

Under the Family Code, a family home is deemed constituted on a house and lot from the time it is occupied as a family
residence. There is no need to constitute the same judicially or extrajudicially as required in the Civil Code. If the family actually
resides in the premises, it is, therefore, a family home as contemplated by law. Thus, the creditors should take the necessary
precautions to protect their interest before extending credit to the spouses or head of the family who owns the home.
Article 155 of the Family Code also provides as follows:
Art. 155. The family home shall be exempt from execution, forced sale or attachment except:
(1) For non-payment of taxes;
(2) For debts incurred prior to the constitution of the family home;
(3) For debts secured by mortgages on the premises before or after such constitution; and
(4) For debts due to laborers, mechanics, architects, builders, material men and others who have rendered service
or furnished material for the construction of the building.
The exemption provided as aforestated is effective from the time of the constitution of the family home as such, and lasts so
long as any of its beneficiaries actually resides therein.
In the present case, the residential house and lot of petitioner was not constituted as a family home whether judicially or
extrajudicially under the Civil Code. It became a family home by operation of law only under Article 153 of the Family Code. It is
deemed constituted as a family home upon the effectivity of the Family Code on August 3, 1988 not August 4, one year after its
publication in the Manila Chronicle on August 4, 1987 (1988 being a leap year).
The contention of petitioner that it should be considered a family home from the time it was occupied by petitioner and his family
in 1969 is not well- taken. Under Article 162 of the Family Code, it is provided that "the provisions of this Chapter shall also
govern existing family residences insofar as said provisions are applicable." It does not mean that Articles 152 and 153 of said
Code have a retroactive effect such that all existing family residences are deemed to have been constituted as family homes at
the time of their occupation prior to the effectivity of the Family Code and are exempt from execution for the payment of
obligations incurred before the effectivity of the Family Code. Article 162 simply means that all existing family residences at the
time of the effectivity of the Family Code, are considered family homes and are prospectively entitled to the benefits accorded to
a family home under the Family Code. Article 162 does not state that the provisions of Chapter 2, Title V have a retroactive
effect.
Is the family home of petitioner exempt from execution of the money judgment aforecited No. The debt or liability which was the
basis of the judgment arose or was incurred at the time of the vehicular accident on March 16, 1976 and the money judgment
arising therefrom was rendered by the appellate court on January 29, 1988. Both preceded the effectivity of the Family Code on
August 3, 1988. This case does not fall under the exemptions from execution provided in the Family Code.
As to the agricultural land subject of the execution, the trial court correctly ruled that the levy to be made by the sheriff shall be
on whatever rights the petitioner may have on the land.
WHEREFORE, the petition is DISMISSED for lack of merit. No pronouncement as to costs.

[G.R. No. 97898. August 11, 1997]

FLORANTE F. MANACOP, petitioner, vs. COURT OF APPEALS and E & L MERCANTILE, INC., respondents.
DECISION
PANGANIBAN, J.:

May a writ of execution of a final and executory judgment issued before the effectivity of the Family Code be executed on a
house and lot constituted as a family home under the provision of said Code?
Statement of the Case
This is the principal question posed by petitioner in assailing the Decision of Respondent Court of Appeals in CA-G.R. SP No.
18906 promulgated on February 21, 1990 and its Resolution promulgated on March 21, 1991, affirming the orders issued by the trial
court commanding the issuance of various writs of execution to enforce the latters decision in Civil Case No. 53271.
[1]

The Facts
Petitioner Florante F. Manacop and his wife Eulaceli purchased on March 10, 1972 a 446-square-meter residential lot with a
bungalow, in consideration of P75,000.00. The property, located in Commonwealth Village, Commonwealth Avenue, Quezon City, is
covered by Transfer Certificate of Title No. 174180.
[2]

[3]

On March 17, 1986, Private Respondent E & L Mercantile, Inc. filed a complaint against petitioner and F.F. Manacop Construction
Co., Inc. before the Regional Trial Court of Pasig, Metro Manila to collect an indebtedness of P3,359,218.45. Instead of filing an
answer, petitioner and his company entered into a compromise agreement with private respondent, the salient portion of which
provides:
c. That defendants will undertake to pay the amount of P2,000,000.00 as and when their means permit, but expeditiously as possible as their
collectibles will be collected. (sic)
On April 20, 1986, the trial court rendered judgment approving the aforementioned compromise agreement. It enjoined the
parties to comply with the agreement in good faith. On July 15, 1986, private respondent filed a motion for execution which the lower
court granted on September 23, 1986. However, execution of the judgment was delayed. Eventually, the sheriff levied on several
vehicles and other personal properties of petitioner. In partial satisfaction of the judgment debt, these chattels were sold at public
auction for which certificates of sale were correspondingly issued by the sheriff.
On August 1, 1989, petitioner and his company filed a motion to quash the alias writs of execution and to stop the sheriff from
continuing to enforce them on the ground that the judgment was not yet executory. They alleged that the compromise agreement
had not yet matured as there was no showing that they had the means to pay the indebtedness or that their receivables had in fact
been collected. They buttressed their motion with supplements and other pleadings.
On August 11, 1989, private respondent opposed the motion on the following grounds: (a) it was too late to question the
September 23, 1986 Order considering that more than two years had elapsed; (b) the second alias writ of execution had been
partially implemented; and (c) petitioner and his company were in bad faith in refusing to pay their indebtedness notwithstanding that
from February 1984 to January 5, 1989, they had collected the total amount of P41,664,895.56. On September 21, 1989, private
respondent filed an opposition to petitioner and his companys addendum to the motion to quash the writ of execution. It alleged that
the property covered by TCT No. 174180 could not be considered a family home on the grounds that petitioner was already living
abroad and that the property, having been acquired in 1972, should have been judicially constituted as a family home to exempt it
from execution.
On September 26, 1989, the lower court denied the motion to quash the writ of execution and the prayers in the subsequent
pleadings filed by petitioner and his company. Finding that petitioner and his company had not paid their indebtedness even though
they collected receivables amounting to P57,224,319.75, the lower court held that the case had become final and executory. It also
ruled that petitioners residence was not exempt from execution as it was not duly constituted as a family home, pursuant to the Civil
Code.
Hence, petitioner and his company filed with the Court of Appeals a petition for certiorari assailing the lower courts Orders of
September 23, 1986 and September 26, 1989. On February 21, 1990, Respondent Court of Appeals rendered its now questioned
Decision dismissing the petition for certiorari. The appellate court quoted with approval the findings of the lower court that: (a) the
judgment based on the compromise agreement had become final and executory, stressing that petitioner and his company had
collected the total amount of P57,224,319.75 but still failed to pay their indebtedness and (b) there was no showing that petitioners
residence had been duly constituted as a family home to exempt it from execution. On the second finding, the Court of Appeals
added that:
x x x. We agree with the respondent judge that there is no showing in evidence that petitioner Maacops residence under TCT 174180 has been
duly constituted as a family home in accordance with law. For one thing, it is the clear implication of Article 153 that the family home continues
to be so deemed constituted so long as any of its beneficiaries enumerated in Article 154 actually resides therein. Conversely, it ceases to continue
as such family home if none of its beneficiaries actually occupies it. There is no showing in evidence that any of its beneficiaries is actually
residing therein. On the other hand, the unrefuted assertion of private respondent is that petitioner Florante Maacop had already left the country
and is now, together with all the members of his family, living in West Covina, Los Angeles, California, U.S.A.
Petitioner and his company filed a motion for reconsideration of this Decision on the ground that the property covered by TCT No.
174180 was exempt from execution. On March 21, 1991, the Court of Appeals rendered the challenged Resolution denying the
motion. It anchored its ruling on Modequillo v. Breva, which held that all existing family residences at the time of the effectivity of
the Family Code are considered family homes and are prospectively entitled to the benefits accorded to a family home under the
Family Code.
[4]

Applying the foregoing pronouncements to this case, the Court of Appeals explained:
The record of the present case shows that petitioners incurred the debt of P3,468,000.00 from private respondent corporation on February 18,
1982 (Annex `A, Petition). The judgment based upon the compromise agreement was rendered by the court on April 18, 1986 (Annex `C, Ibid).
Paraphrasing the aforecited Modequillo case, both the debt and the judgment preceded the effectivity of the Family Code on August 3, 1988.
Verily, the case at bar does not fall under the exemptions from execution provided under Article 155 of the Family Code.
Undeterred, petitioner filed the instant petition for review on certiorari arguing that the Court of Appeals
misapplied Modequillo. He contends that there was no need for him to constitute his house and lot as a family home for it to be
treated as such since he was and still is a resident of the same property from the time it was levied upon and up to this moment.
The Issue
As stated in the opening sentence of this Decision, the issue in this case boils down to whether a final and executory decision
promulgated and a writ of execution issued before the effectivity of the Family Code can be executed on a family home constituted
under the provisions of the said Code.
The Courts Ruling

We answer the question in the affirmative. The Court of Appeals committed no reversible error. On the contrary, its Decision and
Resolution are supported by law and applicable jurisprudence.
No Novel Issue
At the outset, the Court notes that the issue submitted for resolution in the instant case is not entirely new. In Manacop v. Court
of Appeals, petitioner himself as a party therein raised a similar question of whether this very same property was exempt
from preliminary attachment for the same excuse that it was his family home. In said case, F.F. Cruz & Co., Inc. filed a complaint for
a sum of money. As an incident in the proceedings before it, the trial court issued a writ of attachment on the said house and lot. In
upholding the trial court (and the Court of Appeals) in that case, we ruled that petitioner incurred the indebtedness in 1987 or prior to
the effectivity of the Family Code on August 3, 1988. Hence, petitioners family home was not exempt from attachment by sheer
force of exclusion embodied in paragraph 2, Article 155 of the Family Code cited in Modequillo, where the Court categorically ruled:
[5]

Under the Family Code, a family home is deemed constituted on a house and lot from the time it is occupied as a family residence. There is no
need to constitute the same judicially or extrajudicially as required in the Civil Code. If the family actually resides in the premises, it is, therefore,
a family home as contemplated by law. Thus, the creditors should take the necessary precautions to protect their interest before extending credit to
the spouses or head of the family who owns the home.
Article 155 of the Family Code also provides as follows:
Art. 155. The family home shall be exempt from execution, forced sale or attachment except:
(1) For nonpayment of taxes;
(2) For debts incurred prior to the constitution of the family home;
(3) For debts secured by mortgages on the premises before or after such constitution; and
(4) For debts due to laborers, mechanics, architects, builders, materialmen and others who have rendered service or furnished material for
the construction of the building.
The exemption provided as aforestated is effective from the time of the constitution of the family home as such, and lasts so long as any of its
beneficiaries actually resides therein.
In the present case, the residential house and lot of petitioner was not constituted as a family home whether judicially or extrajudicially under the
Civil Code. It became a family home by operation of law only under Article 153 of the Family Code. It is deemed constituted as a family home
upon the effectivity of the Family Code on August 3, 1988 not August 4, one year after its publication in the Manila Chronicle on August 4, 1987
(1988 being a leap year).
The contention of petitioner that it should be considered a family home from the time it was occupied by petitioner and his family in 1960 is not
well-taken. Under Article 162 of the Family Code, it is provided that `the provisions of this Chapter shall also govern existing family residences
insofar as said provisions are applicable. It does not mean that Articles 152 and 153 of said Code have a retroactive effect such that all existing
family residences are deemed to have been constituted as family homes at the time of their occupation prior to the effectivity of the Family Code
and are exempt from execution for the payment of obligations incurred before the effectivity of the Family Code. Article 162 simply means that
all existing family residences at the time of the effectivity of the Family Code, are considered family homes and are prospectively entitled to the
benefits accorded to a family home under the Family Code. Article 162 does not state that the provisions of Chapter 2, Title V have a retroactive
effect.
Is the family home of petitioner exempt from execution of the money judgment aforecited? No. The debt or liability which was the basis of the
judgment arose or was incurred at the time of the vehicular accident on March 16, 1976 and the money judgment arising therefrom was rendered
by the appellate court on January 29, 1988. Both preceded the effectivity of the Family Code on August 3, 1988. This case does not fall under the
exemptions from execution provided in the Family Code. (Underscoring supplied.)
[6]6

Article 153 of the Family Code Has No Retroactive Effect


Petitioner contends that the trial court erred in holding that his residence was not exempt from execution in view of his failure to
show that the property involved has been duly constituted as a family home in accordance with law. He asserts that the Family
Code and Modequillo require simply the occupancy of the property by the petitioner, without need for its judicial or extrajudicial
constitution as a family home.
[7]

Petitioner is only partly correct. True, under the Family Code which took effect on August 3, 1988, the subject property became
his family home under the simplified process embodied in Article 153 of said Code. However, Modequillo explicitly ruled that said
provision of the Family Code does not have retroactive effect. In other words, prior to August 3, 1988, the procedure mandated by
the Civil Code had to be followed for a family home to be constituted as such. There being absolutely no proof that the subject
property was judicially or extrajudicially constituted as a family home, it follows that the laws protective mantle cannot be availed of
by petitioner. Since the debt involved herein was incurred and the assailed orders of the trial court issued prior to August 3, 1988,
the petitioner cannot be shielded by the benevolent provisions of the Family Code.
[8]

[9]

List of Beneficiary-Occupants Restricted to Those Enumerated in the Code


In view of the foregoing discussion, there is no reason to address the other arguments of petitioner other than to correct his
misconception of the law. Petitioner contends that he should be deemed residing in the family home because his stay in the United
States is merely temporary. He asserts that the person staying in the house is his overseer and that whenever his wife visited this
country, she stayed in the family home. This contention lacks merit.
The law explicitly provides that occupancy of the family home either by the owner thereof or by any of its beneficiaries must
be actual. That which is actual is something real, or actually existing, as opposed to something merely possible, or to something
which is presumptive or constructive. Actual occupancy, however, need not be by the owner of the house specifically. Rather, the
property may be occupied by the beneficiaries enumerated by Article 154 of the Family Code.
[10]

Art. 154. The beneficiaries of a family home are:


(1)

The husband and wife, or an unmarried person who is the head of the family; and

(2)
Their parents, ascendants, descendants, brothers and sisters, whether the relationship be legitimate or illegitimate, who are living
in the family home and who depend upon the head of the family for lead support.
This enumeration may include the in-laws where the family home is constituted jointly by the husband and wife. But the law
definitely excludes maids and overseers. They are not the beneficiaries contemplated by the Code. Consequently, occupancy of a
family home by an overseer like Carmencita V. Abat in this case is insufficient compliance with the law.
[11]

[12]

WHEREFORE, the petition is hereby DENIED for utter lack of merit. This Decision is immediately executory. Double costs
against petitioner.

VILMA G. ARRIOLA and


ANTHONY RONALD G.
ARRIOLA,
Petitioners,

G.R. No. 177703


Present:
YNARES-SANTIAGO, J.,
Chairperson,
AUSTRIA-MARTINEZ,
CORONA,
NACHURA, and
REYES, JJ.

- versus -

JOHN NABOR C. ARRIOLA,


Respondent.

Promulgated:
January 28, 2008
x------------------------------------------------x

DECISION

AUSTRIA-MARTINEZ, J.:
Before this Court is a Petition for Review on Certiorari under Rule 45 of the Rules of Court, assailing the November 30, 2006
Decision[1] and April 30, 2007 Resolution[2] of the Court of Appeals in CA-G.R. SP No. 93570.
The relevant facts are culled from the records.
John Nabor C. Arriola (respondent) filed Special Civil Action No. 03-0010 with the Regional Trial Court, Branch 254, LasPias City
(RTC) against Vilma G. Arriola and Anthony Ronald G. Arriola (petitioners) for judicial partition of the properties of decedent
Fidel Arriola (the decedent Fidel). Respondent is the son of decedent Fidel with his first wife Victoria C. Calabia, while petitioner Anthony
is the son of decedent Fidel with his second wife, petitioner Vilma.
On February 16, 2004, the RTC rendered a Decision, the dispositive portion of which reads:
WHEREFORE, premises considered, judgment is hereby rendered:
1. Ordering the partition of the parcel of land covered by Transfer Certificate of Title No. 383714 (84191) left by the
decedent Fidel S.Arriola by and among his heirs John Nabor C. Arriola, Vilma G. Arriola and Anthony Ronald G. Arriola in equal
shares of one-third (1/3) each without prejudice to the rights of creditors or mortgagees thereon, if any;
2. Attorney's fees in the amount of TEN THOUSAND (P10,000.00) PESOS is hereby awarded to be reimbursed by the
defendants to the plaintiff;
3. Costs against the defendants.
SO ORDERED.[3]

The decision became final on March 15, 2004.[4]


As the parties failed to agree on how to partition among them the land covered by TCT No. 383714 (subject land), respondent sought
its sale through public auction, and petitioners acceded to it. [5] Accordingly, the RTC ordered the public auction of the subject land. [6] The
public auction sale was scheduled on May 31, 2003 but it had to be reset when petitioners refused to include in the auction the house (subject
house) standing on the subject land.[7] This prompted respondent to file with the RTC an Urgent Manifestation and Motion for Contempt of
Court,[8] praying that petitioners be declared in contempt.
The RTC denied the motion in an Order [9] dated August 30, 2005, for the reason that petitioners were justified in refusing to have the
subject house included in the auction, thus:
The defendants [petitioners] are correct in holding that the house or improvement erected on the property should not be
included in the auction sale.
A cursory reading of the aforementioned Decision and of the evidence adduced during the ex-parte hearing clearly show that
nothing was mentioned about the house existing on the land subject matter of the case. In fact, even plaintiff's [respondent's]
initiatory Complaint likewise did not mention anything about the house. Undoubtedly therefore, the Court did not include the house
in its adjudication of the subject land because it was plaintiff himself who failed to allege the same. It is a well-settled rule that the
court can not give a relief to that which is not alleged and prayed for in the complaint.
To hold, as plaintiff argued, that the house is considered accessory to the land on which it is built is in effect to add to
plaintiff's [a] right which has never been considered or passed upon during the trial on the merits.
In the absence of any other declaration, obvious or otherwise, only the land should be partitioned in accordance to[sic] the
aforementioned Decision as the house can not be said to have been necessarily adjudicated therein. Thus, plaintiff can not be
declared as a co-owner of the same house without evidence thereof and due hearing thereon.
The Decision of the Court having attained its finality, as correctly pointed out, judgment must stand even at the risk that it
might be erroneous.
WHEREFORE, the Urgent Manifestation and Motion for Contempt of Court filed by plaintiff is hereby DENIED for lack of
merit.
SO ORDERED.[10]

The RTC, in its Order dated January 3, 2006, denied respondent's Motion for Reconsideration.[11]
Respondent filed with the CA a Petition for Certiorari[12] where he sought to have the RTC Orders set aside, and prayed that he
be allowed to proceed with the auction of the subject land including the subject house.
In its November 30, 2006 Decision, the CA granted the Petition for Certiorari, to wit:
WHEREFORE, the petition is GRANTED. The assailed orders dated August 30, 2005 and January 3, 2006 issued by the
RTC, in Civil Case No. SCA 03-0010, are REVERSED and SET ASIDE, and the sheriff is ordered to proceed with the public
auction sale of the subject lot covered by TCT No. 383714, including the house constructed thereon.
SO ORDERED.[13] (Emphasis supplied.)

Petitioners filed a motion for reconsideration but the CA denied the same in its Resolution[14] of April 30, 2007.
Hence, the present petition on the sole ground that the CA erred in holding that the RTC committed grave abuse of discretion in
denying the motion for contempt of court.
The assailed CA Decision and Resolution must be modified for reasons other than those advanced by petitioners.
The contempt proceeding initiated by respondent was one for indirect contempt. Section 4, Rule 71 of the Rules of Court prescribes
the procedure for the institution of proceedings for indirect contempt, viz:
Sec. 4. How proceedings commenced. Proceedings for indirect contempt may be initiated motu proprio by the court against
which the contempt was committed by an order or any other formal charge requiring the respondent to show cause why he should
not be punished for contempt.
In all other cases, charges for indirect contempt shall be commenced by a verified petition with supporting particulars and
certified true copies of documents or papers involved therein, and upon full compliance with the requirements for filing initiatory
pleadings for civil actions in the court concerned. If the contempt charges arose out of or are related to a principal action pending
in the court, the petition for contempt shall allege that fact but said petition shall be docketed, heard and decided separately, unless
the court in its discretion orders the consolidation of the contempt charge and the principal action for joint hearing and
decision. (Emphases supplied.)

Under the aforecited second paragraph of the Rules, the requirements for initiating an indirect contempt proceeding are a) that it be
initiated by way of a verified petition and b) that it should fully comply with the requirements for filing initiatory pleadings for civil
actions. In Regalado v. Go,[15] we held:
As explained by Justice Florenz Regalado, the filing of a verified petition that has complied with the requirements for the
filing of initiatory pleading, is mandatory x x x:
This new provision clarifies with a regularity norm the proper procedure for
commencing contempt proceedings. While such proceeding has been classified as special civil action under the former
Rules, the heterogenous practice tolerated by the courts, has been for any party to file a motion without paying any
docket or lawful fees therefore and without complying with the requirements for initiatory pleadings, which is now
required in the second paragraph of this amended section.
xxxx
Henceforth, except for indirect contempt proceedings initiated motu propio by order of or a formal
charge by the offended court, all charges shall be commenced by a verified petition with full compliance with the
requirements therefore and shall be disposed in accordance with the second paragraph of this section.
xxxx
Even if the contempt proceedings stemmed from the main case over which the court already acquired jurisdiction,
the rules direct that the petition for contempt be treated independently of the principal action. Consequently, the necessary
prerequisites for the filing of initiatory pleadings, such as the filing of a verified petition, attachment of a certification on
non-forum shopping, and the payment of the necessary docket fees, must be faithfully observed.
xxxx
The provisions of the Rules are worded in very clear and categorical language. In case where the indirect contempt charge is
not initiated by the courts, the filing of a verified petition which fulfills the requirements on initiatory pleadings is a
prerequisite. Beyond question now is the mandatory requirement of a verified petition in initiating an
indirect contempt proceeding. Truly, prior to the amendment of the 1997 Rules of Civil Procedure, mere motion without complying
with the requirements for initiatory pleadings was tolerated by the courts. At the onset of the 1997 Revised Rules of Civil
Procedure, however, such practice can no longer be countenanced. [16] (Emphasis ours.)

The RTC erred in taking jurisdiction over the indirect contempt proceeding initiated by respondent. The latter did not comply with
any of the mandatory requirements of Section 4, Rule 71. He filed a mere Urgent Manifestation and Motion for Contempt of Court, and not
a verified petition. He likewise did not conform with the requirements for the filing of initiatory pleadings such as the submission of a
certification against forum shopping and the payment of docket fees. Thus, his unverified motion should have been dismissed outright by
the RTC.
It is noted though that, while at first the RTC overlooked the infirmities in respondent's unverified motion for contempt, in the end, it
dismissed the motion, albeit on substantive grounds. The trouble is that, in the CA decision assailed herein, the appellate court committed
the same oversight by delving into the merits of respondent's unverified motion and granting the relief sought therein. Thus, strictly
speaking, the proper disposition of the present petition ought to be the reversal of the CA decision and the dismissal of respondent's
unverified motion for contempt filed in the RTC for being in contravention of Section 4, Rule 71.

However, such simplistic disposition will not put an end to the dispute between the parties. A seed of litigation has already been sown
that will likely sprout into another case between them at a later time. We refer to the question of whether the subject house should be
included in the public auction of the subject land. Until this question is finally resolved, there will be no end to litigation between the
parties. We must therefore deal with it squarely, here and now.
The RTC and the CA differed in their views on whether the public auction should include the subject house. The RTC excluded the
subject house because respondent never alleged its existence in his complaint for partition or established his co-ownership thereof. [17] On the
other hand, citing Articles 440,[18] 445[19] and 446[20] of the Civil Code, the CA held that as the deceased owned the subject land, he also owned
the subject house which is a mere accessory to the land. Both properties form part of the estate of the deceased and are held in co-ownership
by his heirs, the parties herein. Hence, the CA concludes that any decision in the action for partition of said estate should cover not just the
subject land but also the subject house.[21] The CA further pointed out that petitioners themselves implicitly recognized the inclusion of the
subject house in the partition of the subject land when they proposed in their letter of August 5, 2004, the following swapping-arrangement:
Sir:
Thank you very much for accommodating us even if we are only poor and simple people. We are very much pleased with the
decision of Presiding Judge Manuel B. Fernandez, Jr., RTC Br. 254, Las Pias, on the sharing of one-third (1/3) each of a land
covered by Transfer Certificate of Title No. 383714 (84191) in Las Pias City.
However, to preserve the sanctity of our house which is our residence for more than twenty (20) years, we wish to request
that the 1/3 share of John Nabor C. Arriola be paid by the defendants depending on the choice of the plaintiff between item (1) or
item (2), detailed as follows:
(1) Swap with a 500-square meters [sic] lot located at Baras Rizal x x x.
(2) Cash of P205,700.00 x x x.
x x x x.[22]

We agree that the subject house is covered by the judgment of partition for reasons postulated by the CA. We qualify, however, that
this ruling does not necessarily countenance the immediate and actual partition of the subject house by way of public auction in view of
the suspensive proscription imposed under Article 159 of The Family Code which will be discussed forthwith.
It is true that the existence of the subject house was not specifically alleged in the complaint for partition. Such omission
notwithstanding, the subject house is deemed part of the judgment of partition for two compelling reasons.
First, as correctly held by the CA, under the provisions of the Civil Code, the subject house is deemed part of the subject land. The
Court quotes with approval the ruling of the CA, to wit:
The RTC, in the assailed Order dated August 30, 2005 ratiocinated that since the house constructed on the subject lot was not
alleged in the complaint and its ownership was not passed upon during the trial on the merits, the court cannot include the house in
its adjudication of the subject lot. The court further stated that it cannot give a relief to[sic] which is not alleged and prayed for in the
complaint.
We are not persuaded.
To follow the foregoing reasoning of the RTC will in effect render meaningless the pertinent rule on accession. In
general, the right to accession is automatic (ipso jure), requiring no prior act on the part of the owner or the principal. So that
even if the improvements including the house were not alleged in the complaint for partition, they are deemed included in the
lot on which they stand, following the principle of accession. Consequently, the lot subject of judicial partition in this case
includes the house which is permanently attached thereto, otherwise, it would be absurd to divide the principal, i.e., the lot,
without dividing the house which is permanently attached thereto. [23] (Emphasis supplied)

Second, respondent has repeatedly claimed that the subject house was built by the deceased. [24] Petitioners nevercontroverted such
claim. There is then no dispute that the subject house is part of the estate of the deceased; as such, it is owned in common by the latter's
heirs, the parties herein,[25] any one of whom, under Article 494[26] of the Civil Code, may, at any time, demand the partition of the subject
house.[27] Therefore, respondent's recourse to the partition of the subject house cannot be hindered, least of all by the mere technical
omission of said common property from the complaint for partition.
That said notwithstanding, we must emphasize that, while we treat the subject house as part of the co-ownership of the parties, we
stop short of authorizing its actual partition by public auction at this time. It bears emphasis that an action for partition involves two
phases: first, the declaration of the existence of a state of co-ownership; and second, the actual termination of that state of co-ownership
through the segregation of the common property.[28] What is settled thus far is only the fact that the subject house is under the co-ownership
of the parties, and therefore susceptible of partition among them.
Whether the subject house should be sold at public auction as ordered by the RTC is an entirely different matter, depending on the
exact nature of the subject house.
Respondent claims that the subject house was built by decedent Fidel on his exclusive property. [29] Petitioners add that said house has
been their residence for 20 years.[30] Taken together, these averments on record establish that the subject house is a family home within the
contemplation of the provisions of The Family Code, particularly:
Article 152. The family home, constituted jointly by the husband and the wife or by an unmarried head of a family, is the
dwelling house where they and their family reside, and the land on which it is situated.
Article 153. The family home is deemed constituted on a house and lot from the time it is occupied as a family residence.
From the time of its constitution and so long as any of its beneficiaries actually resides therein, the family home continues to be such

and is exempt from execution, forced sale or attachment except as hereinafter provided and to the extent of the value allowed by
law. (Emphasis supplied.)

One significant innovation introduced by The Family Code is the automatic constitution of the family home from the time of its
occupation as a family residence, without need anymore for the judicial or extrajudicial processes provided under the defunct Articles 224 to
251 of the Civil Code and Rule 106 of the Rules of Court. Furthermore, Articles 152 and 153 specifically extend the scope of the family
home not just to the dwelling structure in which the family resides but also to the lot on which it stands. Thus, applying these concepts, the
subject house as well as the specific portion of the subject land on which it stands are deemed constituted as a family home by the deceased
and petitioner Vilma from the moment they began occupying the same as a family residence 20 years back.[31]
It being settled that the subject house (and the subject lot on which it stands) is the family home of the deceased and his heirs, the same
is shielded from immediate partition under Article 159 of The Family Code, viz:
Article 159. The family home shall continue despite the death of one or both spouses or of the unmarried head of the
family for a period of ten years or for as long as there is a minor beneficiary, and the heirs cannot partition the same unless the
court finds compelling reasonstherefor. This rule shall apply regardless of whoever owns the property or constituted the family
home. (Emphasis supplied.)

The purpose of Article 159 is to avert the disintegration of the family unit following the death of its head. To this end, it preserves the
family home as the physical symbol of family love, security and unity by imposing the following restrictions on its partition: first, that the
heirs cannot extra-judicially partition it for a period of 10 years from the death of one or both spouses or of the unmarried head of the family,
or for a longer period, if there is still a minor beneficiary residing therein; and second, that the heirs cannot judicially partition it during the
aforesaid periods unless the court finds compelling reasons therefor. No compelling reason has been alleged by the parties; nor has the RTC
found any compelling reason to order the partition of the family home, either by physical segregation or assignment to any of the heirs or
through auction sale as suggested by the parties.
More importantly, Article 159 imposes the proscription against the immediate partition of the family home regardless of its
ownership. This signifies that even if the family home has passed by succession to the co-ownership of the heirs, or has been willed to any
one of them, this fact alone cannot transform the family home into an ordinary property, much less dispel the protection cast upon it by the
law. The rights of the individual co-owner or owner of the family home cannot subjugate the rights granted under Article 159 to the
beneficiaries of the family home.

Set against the foregoing rules, the family home -- consisting of the subject house and lot on which it stands -- cannot be partitioned at
this time, even if it has passed to the co-ownership of his heirs, the parties herein. Decedent Fidel died on March 10, 2003.[32] Thus, for 10
years from said date or until March 10, 2013, or for a longer period, if there is still a minor beneficiary residing therein, the family home he
constituted cannot be partitioned, much less when no compelling reason exists for the court to otherwise set aside the restriction and order
the partition of the property.
The Court ruled in Honrado v. Court of Appeals[33] that a claim for exception from execution or forced sale under Article 153 should be
set up and proved to the Sheriff before the sale of the property at public auction. Herein petitioners timely objected to the inclusion of the
subject house although for a different reason.
To recapitulate, the evidence of record sustain the CA ruling that the subject house is part of the judgment of co-ownership and
partition. The same evidence also establishes that the subject house and the portion of the subject land on which it is standing have been
constituted as the family home of decedent Fidel and his heirs. Consequently, its actual and immediate partition cannot be sanctioned until
the lapse of a period of 10 years from the death of Fidel Arriola, or until March 10, 2013.
It bears emphasis, however, that in the meantime, there is no obstacle to the immediate public auction of the portion of the subject land
covered by TCT No. 383714, which falls outside the specific area of the family home.
WHEREFORE, the petition is PARTLY GRANTED and the November 30, 2006 Decision and April 30, 2007 Resolutionof the
Court of Appeals are MODIFIED in that the house standing on the land covered by Transfer Certificate of Title No. 383714
is DECLARED part

of

the

co-ownership

of

the

parties John Nabor C. Arriola, Vilma G. Arriola and

Anthony

G. Arriola butEXEMPTED from partition by public auction within the period provided for in Article 159 of the Family Code.

Paternity and Filiation

Ronald

BELEN SAGAD ANGELES,


Petitioner,

G.R. No. 153798


Present:

- versus -

PANGANIBAN, J., Chairman


SANDOVAL-GUTIERREZ,
CORONA,
CARPIO-MORALES, and
GARCIA, JJ.
Promulgated:

ALELI CORAZON ANGELES


MAGLAYA,
Respondent.
September 2, 2005
x----------------------------------------------------------------------------------x

DECISION
GARCIA, J.:

In this petition for review on certiorari under Rule 45 of the Rules of Court, petitioner Belen Sagad
Angeles seeks to set aside the Decision dated May 29, 2002 [1] of the Court of Appeals in CA G.R. CV No. 66037,
reversing an earlier Order of the Regional Trial Court at Caloocan City which dismissed the petition for the
settlement of the intestate estate of Francisco Angeles, thereat commenced by the herein respondent Aleli
Corazon Angeles-Maglaya.
The legal dispute between the parties started when, on March 25, 1998, in the Regional Trial Court (RTC)
at Caloocan City, respondent filed a petition [2] for letters of administration and her appointment as
administratrix of the intestate estate of Francisco M. Angeles (Francisco, hereinafter). In the petition, docketed
as Special Proceedings No. C-2140 and raffled to Branch 120 of the court, respondent alleged, among other
things, the following:
1. That Francisco, a resident of 71 B. Serrano St., Grace Park, Caloocan, died intestate on January 21, 1998
in the City of Manila, leaving behind four (4) parcels of land and a building, among other valuable properties;
2. That there is a need to appoint an administrator of Franciscos estate;
3. That she (respondent) is the sole legitimate child of the deceased and Genoveva Mercado, and, together
with petitioner, Belen S. Angeles, decedents wife by his second marriage, are the surviving heirs of the
decedent; and
4. That she has all the qualifications and none of the disqualifications required of an administrator.
Petitioner opposed the basic petition and prayed that she, instead of respondent, be made the
administratrix of Franciscos estate. [3] In support of her opposition and plea, petitioner alleged having married
Francisco on August 7, 1948 before Judge Lucio M. Tianco of the Municipal Court of Rizal, a union which was
ratified two (2) months later in religious rites at the Our Lady of Grace Parish in Caloocan City, and that
Francisco represented in their marriage contract that he was single at that time. Petitioner also averred that
respondent could not be the daughter of Francisco for, although she was recorded as Franciscos legitimate
daughter, the corresponding birth certificate was not signed by him. Pressing on, petitioner further alleged that
respondent, despite her claim of being the legitimate child of Francisco and Genoveva Mercado, has not
presented the marriage contract between her supposed parents or produced any acceptable document to prove
such union. And evidently to debunk respondents claim of being the only child of Francisco, petitioner likewise
averred that she and Francisco had, during their marriage, legally adopted

Concesa A. Yamat, et al. Petitioner

thus urged that she, being the surviving spouse of Francisco, be declared as possessed of the superior right to
the administration of his estate.
In her reply to opposition, respondent alleged, inter alia, that per certification of the appropriate offices,
the January to December 1938 records of marriages of the Civil Registrar of Bacolor, Pampanga where the
alleged 1938 Francisco-Genoveva wedding took place, were destroyed.

In the same reply, respondent

dismissed as of little consequence the adoption adverted to owing to her having interposed with the Court of
Appeals a petition to nullify the decree of adoption entered by the RTC at Caloocan. [4]
Issues having been joined, trial ensued. Respondent, as petitioner a quo, commenced the presentation of
her evidence by taking the witness stand. She testified having been born on November 20, 1939 as the
legitimate child of Francisco M. Angeles and Genoveva Mercado, who died in January 1988. [5] She also testified
having been in open and continuous possession of the status of a legitimate child.
testified on her behalf, namely: Tomas Angeles, [6] Francisco Yaya,[7]

Four (4) other witnesses

Jose O. Carreon[8] and Paulita Angeles de la

Cruz.[9] Respondent also offered in evidence her birth certificate which contained an entry stating that she was
born at the Mary Johnston Hospital, Tondo, Manila, to Francisco Angeles and Genoveva Mercado and whereon
the handwritten word Yes appears on the space below the question Legitimate? (Legitimo?); pictures taken

during respondents wedding as bride to Atty. Guillermo T. Maglaya; and a copy of her marriage contract.
Likewise offered were her scholastic and government service records.
After respondent rested her case following her formal offer of exhibits, petitioner filed a Motion to
Dismiss under Section 1(g), Rule 16 of the Rules of Court. In it, she prayed for the dismissal of the petition for
letters of administration on the ground that the petition failed to state or prove a cause of action, it being her
stated position that [P]etitioner [Corzaon], by her evidence, failed to establish her filiation vis--vis the
decedent, i.e., that she is in fact a legitimate child of Francisco M. Angeles.[10]
To the motion to dismiss, respondent interposed an opposition, followed by petitioners reply, to which
respondent countered with a rejoinder.
Eventually, in an Order dated July 12, 1999,[11] the trial court, on its finding that respondent failed to prove
her filiation as legitimate child of Francisco, dismissed the petition, thus:
WHEREFORE, the instant petition is hereby ordered DISMISSED for failure of the [respondent] to state a cause of action
in accordance with Section 1(g) of Rule 16 of the 1997 Rules of Civil of Procedure. (Word in bracket added]

Respondent then moved for reconsideration, which motion was denied by the trial court in its Order of
December 17, 1999.[12] Therefrom, respondent went on appeal to the Court of Appeals where her recourse was
docketed as CA-G.R. CV No. 66037.
As stated at the threshold hereof, the Court of Appeals, in its assailed Decision dated May 29, 2002,
reversed and set aside the trial courts order of dismissal and directed it to appoint respondent as

[13]

administratrix of the estate of Francisco, to wit:


WHEREFORE, the appealed order of dismissal is REVERSED. The Trial Court is hereby ordered to appoint petitionerappellant Aleli Corazon Angeles as administratrix of the intestate estate of Francisco Angeles.
SO ORDERED.

The appellate court predicated its ruling on the interplay of the following main premises:
1. Petitioners Motion to Dismiss filed with the trial court, albeit premised on the alleged failure of the
underlying petition for letter of administration to state or prove a cause of action, actually partakes of a
demurrer to evidence under Section 1 of Rule 33; [14]
2. Petitioners motion being a demurer, it follows that she thereby waived her right to present opposing
evidence to rebut respondents testimonial and documentary evidence; and
3. Respondent has sufficiently established her legitimate filiation with the deceased Francisco.
Hence, petitioners instant petition for review on certiorari, on the submission that the Court of Appeals
erred: (1) in reversing the trial courts order of dismissal; [15] (2) in treating her motion to dismiss as a demurrer
to evidence; (3) in holding that respondent is a legitimate daughter of Francisco; and (4) in decreeing
respondents appointment as administratrix of Franciscos intestate estate.
We resolve to grant the petition.
The principal issue tendered in this case boils down to the question of whether or not respondent is the
legitimate child of decedent Francisco M. Angeles and Genoveva Mercado. The Court of Appeals resolved the
issue in the affirmative and, on the basis of such determination, ordered the trial court to appoint respondent as
administratrix of Franciscos estate.
We are unable to lend concurrence to the appellate courts conclusion on the legitimate status of
respondent, or, to be precise, on her legitimate filiation to the decedent. A legitimate child is a product of, and,
therefore, implies a valid and lawful marriage. Remove the element of lawful union and there is strictly no
legitimate filiation between parents and child. Article 164 of the Family Code cannot be more emphatic on the
matter: Children conceived or born during the marriage of the parents are legitimate.
In finding for respondent, the Court of Appeals, citing and extensibly quoting from Tison vs. Court of
Appeals,[16] stated that since petitioner opted not to present any contrary evidence, the presumption on
respondents legitimacy stands unrebutted.[17]
Following is an excerpt from Tison:
It seems that both the court a quo and respondent appellate court have regrettably overlooked the universally recognized
presumption on legitimacy. There is no presumption of the law more firmly established and founded on sounder morality and more

convincing than the presumptionthat children born in wedlock are legitimate. And well-settled is the rule that the issue of
legitimacy cannot be attacked collaterally.
The rationale for this rule has been explained in this wise:
The presumption of legitimacy in the Family Code . . .
actually fixes a status for the child born in wedlock, and that civil status cannot be attacked collaterally. xxx
xxx

xxx

xxx

Upon the expiration of the periods provided in Article 170 [of the Family Code], the action to impugn the
legitimacy of a child can no longer be bought. The status conferred by the presumption, therefore, becomes fixed, and
can no longer be questioned. The obvious intention of the law is to prevent the status of a child born in wedlock from
being in a state of uncertainty. It also aims to force early action to settle any doubt as to the paternity of such child so that
the evidence material to the matter . . . may still be easily available.
xxx
xxx
xxx
Only the husband can contest the legitimacy of a child born to his wife . . . .(Words in bracket added; Emphasis
ours)

Contextually, the correct lesson of Tison, which the appellate court evidently misapplied, is that: (a) a
child is presumed legitimate only if conceived or born in wedlock; and (b) the presumptive legitimacy of such
child cannot be attacked collaterally.
A party in whose favor the legal presumption exists may rely on and invoke such legal presumption to
establish a fact in issue. He need not introduce evidence to prove that fact. [18] For, a presumption is prima
facie proof of the fact presumed. However, it cannot be over-emphasized, that while a fact thus prima
facie established by legal presumption shall, unless overthrown, stand as proved, [19] the presumption of
legitimacy under Article 164 of the Family Code [20] may be availed only upon convincing proof of the factual
basis therefor, i.e., that the childs parents were legally married and that his/her conception or birth occurred
during the subsistence of that marriage. Else, the presumption of law that a child is legitimate does not arise.
In the case at bench, the Court of Appeals, in its decision under review, did not categorically state from
what facts established during the trial was the presumption of respondents supposed legitimacy arose.

But

even if perhaps it wanted to, it could not have possibly done so. For, save for respondents gratuitous assertion
and an entry in her certificate of birth, there is absolutely no proof of the decedents marriage to respondents
mother, Genoveva Mercado. To stress, no marriage certificate or marriage contract doubtless the best
evidence of Franciscos and Genovevas marriage, if one had been solemnized [21] was offered in evidence. No
priest, judge, mayor, or other solemnizing authority was called to the witness box to

declare that he

solemnized the marriage between the two. None of the four (4) witnesses respondent presented could say
anything about, let alone affirm, that supposed marriage. At best, their testimonies proved that respondent
was Franciscos daughter. For example, Tomas Angeles and Paulita Angeles de la Cruz testified that they know
respondent to be their cousin because his (Tomas) father
Franciscos siblings, told them so. [22]

and

her

(Paulitas)

mother, who are both

And one Jose Carreon would testify seeing respondent in 1948 in

Franciscos house in Caloocan, the same Francisco who used to court Genoveva before the war.[23] In all, no
evidence whatsoever was presented of the execution of the Francisco Angeles-Genoveva Mercado marriage
contract; when and where their marriage was solemnized; the identity of the solemnizing officer; the persons
present, and like significant details.
While perhaps not determinative of the issue of the existence of marriage between Francisco and
Genoveva, we can even go to the extent of saying that respondent has not even presented a witness to testify
that her putative parents really held themselves out to the public as man-and-wife. Clearly, therefore, the Court
of Appeals erred in crediting respondent with the legal presumption of legitimacy which, as above explained,
should flow from a lawful marriage between Francisco and Genevova. To reiterate, absent such a marriage, as
here, there is no presumption of legitimacy and, therefore, there was really nothing for petitioner to rebut.
Parenthetically, for all her unyielding stance that her mother and Francisco Angeles were married in 1938,
respondent never, thru the years, even question what would necessarily be a bigamous Francisco-Belen Sagad
marriage. Ironical as it may seem, respondent herself undermined her very own case. As it were, she

made

certain judicial admission negating her own assertion as well as the appellate courts conclusion - that
Francisco was legally married to Genoveva. As may be recalled, respondent had declared that her mother
Genoveva died in 1988, implying, quite clearly, that when Francisco contracted marriage with petitioner Belen
S. Angeles in 1948, Genoveva and Francisco were already spouses. Now, then, if, as respondent maintained
despite utter lack of evidence, that Genoveva Mercado and Francisco were married in 1938, it follows that the
marriage of Francisco to petitioner Belen Angeles in 1948, or prior to Genovevas death, would necessarily have
to be bigamous, hence void,[24] in which case petitioner could not be, as respondent alleged in her petition for
letters of administration, a surviving spouse of the decedent. We quote the pertinent allegation:
4. The surviving heirs of decedent are the petitioner [Corazon] herself who is 58 years old, and BELEN S. Angeles, the
surviving spouse of deceased Francisco M. Angeles by his second marriage, who is about 77 years old . . . .YEARS OLD . . .
(Emphasis and word in bracket added)

We can concede, because Article 172 of the Family Code appears to say so, that the legitimate filiation of
a child can be established by any of the modes therein defined even without direct evidence of the marriage of
his/her supposed parents. Said article 172 reads:
Art. 172. The filiation of legitimate children is established by any of the following:
1.
2.

The record of birth appearing in the civil register or a final judgments; or


An admission of legitimate filiation in a public document or a private handwritten instrument and signed by the
parent concerned.

In the absence of the foregoing evidence, the legitimate filiation shall be proved by:
1.

The open and continuous possession of the status of a legitimate child; or

2.

Any other means allowed by the Rules of Court and special laws.

Here, respondent presented, in support of her claim of legitimacy, a copy of her Birth Certificate dated
November 23, 1939 issued by the Civil Registrar of the City of Manila (Exh. E). In it, her birth was recorded as
the legitimate child of Francisco Angeles and Genoveva Mercado. And the word married is written in the
certificate to indicate the union of Francisco and Genoveva.
Petitioner, however, contends, citing jurisprudence,

that [I]t was error for the Court of Appeals to have

ruled . . . that [respondents] Birth Certificate indubitably establishes that she is the legitimate daughter of
Francisco and Genoveva who are legally married.
The contention commends itself for concurrence. The reason is as simple as it is elementary: the Birth
Certificate presented was not signed by Francisco against whom legitimate filiation is asserted. Not even by
Genoveva. It was signed by the attending physician, one Rebecca De Guzman, who certified to having attended
the birth of a child. Such certificate, albeit considered a public record of a private document is, under Section
23, Rule 132 of the Rules of Court, evidence only of the fact which gave rise to its execution: the fact of birth of
a child.[25]Jurisprudence teaches that a birth certificate, to be considered as validating proof of paternity and as
an instrument of recognition, must be signed by the father and mother jointly, or by the mother alone if the
father refuses.[26] Dr. Arturo Tolentino, commenting on the probative value of the entries in a certificate of birth,
wrote:
xxx if the alleged father did not intervene in the making of the birth certificate, the putting of his name by the mother or
doctor or registrar is void; the signature of the alleged father is necessary. [27]

The conclusion reached by the Court of Appeals that the Birth Certificate of respondent, unsigned as it
were by Francisco and Genoveva, establishes and indubitably at that - not only respondents filiation to
Francisco but even her being a legitimate daughter of Francisco and Genoveva, taxes credulity to the limit. In a
very real sense, the appellate court regarded such certificate as defining proof of filiation, and not just filiation
but of legitimate filiation, by inferring from it that Francisco and Genoveva are legally married. In the apt words
of petitioner, the appellate court, out of a Birth Certificate signed by a physician who merely certified having
attended the birth of a child who was born alive at 3:50 P.M. , created a marriage that of Francisco and
Genoveva, and filiation (that said child) is the daughter of Francisco[28]
It cannot be over-emphasized that the legitimate filiation of a child is a matter fixed by law itself. [29] It
cannot, as the decision under review seems to suggest, be made dependent on the declaration of the attending
physician or midwife, or that of the mother of the newborn child. For then, an unwed mother, with or without the
participation of a doctor or midwife, could veritably invest legitimate status to her offspring through the simple
expedient of writing the putative fathers name in the appropriate space in the birth certificate. A long time
past, this Court cautioned against according a similar unsigned birth certificate prima facie evidentiary value of
filiation:
Give this certificate evidential relevancy, and we thereby pave the way for any scheming unmarried mother to extort money
for her child (and herself) from any eligible bachelor or affluent pater familias. How? She simply causes the midwife to state in the
birth certificate that the newborn babe is her legitimate offspring with that individual and the certificate will be accepted for
registration . . . . And any lawyer with sufficient imagination will realize the exciting possibilities from such mischief of such prima
facie evidence when and if the father dies in ignorance of the fraudulent design xxx [30]

Just like her Birth Certificate, respondent can hardly derive comfort from her marriage contract to Atty.
Maglaya and from her student and government records which indicated or purported to show that Francisco
Angeles is her father. The same holds true for her wedding pictures which showed Francisco giving respondents
hands in marriage. These papers or documents, unsigned as they are by Francisco or the execution of which he

had no part, are not sufficient evidence of filiation or recognition. [31] And needless to stress, they cannot support
a finding of the legitimate union of Francisco and Genoveva.
The argument may be advanced that the aforesaid wedding pictures, the school and service records and
the testimony of respondents witnesses lend support to her claim of enjoying open and continuous possession
of the status of a child of Francisco. The Court can even concede that respondent may have been the natural
child of Francisco with Genoveva. Unfortunately, however, that angle is not an, or at issue in the case before us.
For, respondent peremptorily predicated her petition for letters of administration on her being a legitimate child
of Francisco who was legally married to her mother, Genoveva, propositions which we have earlier refuted
herein.
If on the foregoing score alone, this Court could very well end this disposition were it not for another
compelling consideration which petitioner has raised and which we presently take judicially notice of.
As may be recalled, respondent, during the pendency of the proceedings at the trial court, filed with the
Court of Appeals a petition for the annulment of the decision of the RTC Caloocan granting the petition of
spouses Francisco Angeles and petitioner Belen S. Angeles for the adoption of Concesa A. Yamat and two others.
In that petition, docketed with the appellate court as CA-G.R. SP No. 47832 and captioned Aleli Corazon
Angeles Maglaya vs. Hon Jaime T. Hamoy, Consesa A. Yamat, Teodora A. Santos, Franco Angeles and Belen S.
Angeles, respondent alleged that as legitimate daughter of Francisco, she should have been notified of the
adoption proceedings.
Following a legal skirmish, the Court of Appeals referred the aforementioned annulment case to RTC,
Caloocan for reception of evidence. Eventually, in a Decision[32] dated December 17, 2003, the Court of Appeals
dismissedCA-G.R. SP No. 47832 on the ground, inter alia, that herein respondent is not, contrary to her claim, a
legitimate daughter of Francisco, nor a child of a lawful wedlock between Francisco M. Angeles and
Genoveva Y. Mercado. Wrote the appellate court in that case:
Petitioner [Aleli Corazon Maglaya] belabors with repetitious persistence the argument that she is a legitimate child or the
only daughter of Francisco M. Angeles and Genoveva Y. Mercado . . . .
In the case at bench, other than the self-serving declaration of the petitioner, there is nothing in the record to support
petitioners claim that she is indeed a legitimate child of the late Francisco M. Angeles and Genoveva Y. Mercado. xxx In other
words, Francisco M. Angeles was nevermarried before or at anytime prior to his marriage to Belen Sagad, contrary to the claim of
petitioner that Francisco M. Angeles and Genoveva Y. Mercado were married in 1938
While petitioner may have submitted certifications to the effect that the records of marriages during the war years . . . were
totally destroyed, no secondary evidence was presented by petitioner to prove the existence of the marriage between Francisco M.
Angeles and Genoveva Y. Mercado, even as no witness was presented to confirm the celebration of such marriage . . . .
Petitioner presented pictures. x x x However, it is already settled law that photographs are not sufficient evidence of filiation
or acknowledgment.
To be sure, very little comfort is provided by petitioners birth certificate and even her marriage contract.. . . Reason: These
documents were not signed by Francisco . . . . Equally inconsequential are petitioners school records . . . . all these lacked the
signatures of both Francisco and Genoveva . . . .
xxx

xxx

xxx

Having failed to prove that she is the legitimate daughter or acknowledged natural child of the late Francisco M. Angeles,
petitioner cannot be a real party in interest in the adoption proceedings, as her consent thereto is not essential or required. (Emphasis
in the original; words in bracket added)

Significantly, the aforesaid December 17, 2003 Decision of the appellate court in CA-G.R. SP No.47832 was
effectively affirmed by this Court via its Resolution dated August 9, 2004 in G.R. No. 163124, denying Aleli
Corazon Maglayas petition for Review on Certiorari, [33] and Resolution dated October 20, 2004, [34] denying with
FINALITY her motion for reconsideration. Another Resolution dated January 24, 2005 resolved to NOTE
WITHOUT ACTION Maglayas second motion for reconsideration.
In the light of the ruling of the Court of Appeals in CA-G.R. SP No. 47832, as affirmed with finality by this
Court in G.R. No. 163124, there can be no serious objection to applying in this case the rule on conclusiveness
of judgment,[35] one of two (2) concepts embraced in the res judicata principle.

Following the rule on

conclusiveness of judgment, herein respondent is precluded from claiming that she is the legitimate daughter of
Francisco and Genoveva Mercado. In fine, the issue of herein respondents legitimate filiation to Francisco and
the latters marriage to Genoveva, having been judicially determined in a final judgment by a court of
competent jurisdiction, has thereby become res judicata and may not again be resurrected or litigated between
herein petitioner and respondent or their privies in a subsequent action, regardless of the form of the latter. [36]
Lest it be overlooked, the same ruling of the appellate court in CA-G.R. SP No. 47832, as sustained by this
Court in G.R. No. 163124, virtually confirms the ratio of the trial courts order of dismissal in Special
Proceedings (SP) No. C-2140, i.e, that respondent failed to establish that she is in fact a legitimate child of

Francisco. Accordingly, the question of whether or not the Motion to Dismiss[37] interposed by herein petitioner,
as respondent in SP No. C-2140, is in the nature of a demurer to evidence has become moot and academic. It
need not detain us any minute further.
Finally, it should be noted that on the matter of appointment of administrator of the estate of the
deceased, the surviving spouse is preferred over the next of kin of the decedent. [38] When the law speaks of
next of kin, the reference is to those who are entitled, under the statute of distribution, to the decedents
property;[39] one whose relationship is such that he is entitled to share in the estate as distributed, [40] or, in
short,

an heir. In resolving, therefore, the issue of whether an applicant for letters of administration is a next

of kin or an heir of the decedent, the probate court perforce has to determine and pass upon the issue of
filiation. A separate action will only result in a multiplicity of suits. Upon this consideration, the trial court acted
within bounds when it looked into and pass upon the claimed relationship of respondent to the late Francisco
Angeles.
WHEREFORE, the herein assailed decision of the Court of Appeals is hereby REVERSED and SET ASIDE,
and the order of the trial court dismissing Special Proceedings No. C-2140 REINSTATED.

G.R. No. L-2474

May 30, 1951

MARIANO ANDAL, assisted by mother Maria Dueas as guardian ad litem, and MARIA DUEAS, plaintiffs,
vs.
EDUVIGIS MACARAIG, defendant.
Reyes and Dy-Liaco for appellants.
Tible, Tena and Borja for appellees.
BAUTISTA ANGELO, J.:
Mariano Andal, a minor, assisted by his mother Maria Dueas, as guardian ad litem, brought an action in the Court of First
Instance of Camarines Sur for the recovery of the ownership and possession of a parcel of land situated in the barrio of Talacop,
Calabanga, Camarines Sur.
The complaint alleges that Mariano Andal is the surviving son of Emiliano Andal and Maria Dueas; that Emiliano Andal died on
September 24, 1942; that Emiliano Andal was the owner of the parcel of land in question having acquired it from his mother
Eduvigis Macaraig by virtue of a donation propter nuptias executed by the latter in favor of the former; that Emiliano Andal had
been in possession of the land from 1938 up to 1942, when Eduvigis Macaraig, taking advantage of the abnormal situation then
prevailing, entered the land in question.
The lower court rendered judgment in favor of the plaintiffs (a) declaring Mariano Andal the legitimate son of Emiliano Andal and
such entitled to inherit the land in question; (b) declaring Mariano Andal owner of said land; and (c) ordering the defendant to
pay the costs of suit. Defendant took the case to this Court upon the plea that only question of law are involved.
It appears undisputed that the land in question was given by Eduvigis Macaraig to her son Emiliano Andal by virtue of a
donation propter nuptias she has executed in his favor on the occasion of his marriage to Maria Dueas. If the son born to the
couple is deemed legitimate, then he is entitled to inherit the land in question. If otherwise, then the land should revert back to
Eduvigis Macaraig as the next of kin entitled to succeed him under the law. The main issue, therefore, to be determined hinges
on the legitimacy of Mariano Andal in so far as his relation to Emiliano Andal is concerned. The determination of this issue much
depends upon the relationship that had existed between Emiliano Andal and his wife during the period of conception of the child
up to the date of his birth in connection with the death of the alleged father Emiliano Andal.
The following facts appear to have been proven: Emiliano Andal became sick of tuberculosis in January 1941. Sometime
thereafter, his brother, Felix, went to live in his house to help him work his house to help him work his farm. His sickness
became worse that on or about September 10, 1942, he became so weak that he could hardly move and get up from his bed.
On September 10, 1942, Maria Duenas, his wife, eloped with Felix, and both went to live in the house of Maria's father, until the
middle of 1943. Since May, 1942, Felix and Maria had sexual intercourse and treated each other as husband and wife. On
January 1, 1943, Emiliano died without the presence of his wife, who did not even attend his funeral. On June 17, 1943, Maria
Dueas gave birth to a boy, who was given the name of Mariano Andal. Under these facts, can the child be considered as the
legitimate son of Emiliano?
Article 108 of the Civil Code provides:
Children born after the one hundred and eighty days next following that of the celebration of marriage or within the three
hundred days next following its dissolution or the separation of the spouses shall be presumed to be legitimate.
This presumption may be rebutted only by proof that it was physically impossible for the husband to have had access to
his wife during the first one hundred and twenty days of the three hundred next preceding the birth of the child.
Since the boy was born on June 17, 1943, and Emiliano Andal died on January 1, 1943, that boy is presumed to be the
legitimate son of Emiliano and his wife, he having been born within three hundred (300) days following the dissolution of the
marriage. This presumption can only be rebutted by proof that it was physically impossible for the husband to have had access
to his wife during the first 120 days of the 300 next preceding the birth of the child. Is there any evidence to prove that it was
physically impossible for Emiliano to have such access? Is the fact that Emiliano was sick of tuberculosis and was so weak that
he could hardly move and get up from his bed sufficient to overcome this presumption?
Manresa on this point says:
Impossibility of access by husband to wife would include (1) absence during the initial period of conception, (2) impotence
which is patent, continuing and incurable, and (3) imprisonment, unless it can be shown that cohabitation took place
through corrupt violation of prison regulations. Manresa, 492-500, Vol. I, cited by Dr. Arturo Tolentino in his book
"Commentaries and Jurisprudence on the Civil Code, Vol. 1, p.90)."
There was no evidence presented that Emiliano Andal was absent during the initial period of conception, specially during the
period comprised between August 21, 1942 and September 10, 1942, which is included in the 120 days of the 300 next
preceding the birth of the child Mariano Andal. On the contrary, there is enough evidence to show that during that initial period,

Emiliano Andal and his wife were still living under the marital roof. Even if Felix, the brother, was living in the same house, and
he and the wife were indulging in illicit intercourse since May, 1942, that does not preclude cohabitation between Emiliano and
his wife. We admit that Emiliano was already suffering from tuberculosis and his condition then was so serious that he could
hardly move and get up from bed, his feet were swollen and his voice hoarse. But experience shows that this does not prevent
carnal intercourse. There are cases where persons suffering from this sickness can do the carnal act even in the most crucial
stage because they are more inclined to sexual intercourse. As an author has said, "the reputation of the tuberculosis towards
eroticism (sexual propensity) is probably dependent more upon confinement to bed than the consequences of the disease." (An
Integrated Practice of Medicine, by Hyman, Vol. 3, p.2202). There is neither evidence to show that Emiliano was suffering from
impotency, patent, continuous and incurable, nor was there evidence that he was imprisoned. The presumption of legitimacy
under the Civil Code in favor of the child has not, therefore, been overcome.
We can obtain the same result viewing this case under section 68, par. (c) of Rule 123, of the Rules of Court, which is
practically based upon the same rai'son d'etre underlying the Civil Code. Said section provides:
The issue of a wife cohabiting with the husband who is not impotent, is indisputably presumed to be legitimate, if not born
within one hundred eighty days immediately succeeding the marriage, or after the expiration of three hundred days
following its dissolution.
We have already seen that Emiliano and his wife were living together, or at least had access one to the other, and Emiliano was
not impotent, and the child was born within three (300) days following the dissolution of the marriage. Under these facts no
other presumption can be drawn than that the issue is legitimate. We have also seen that this presumption can only be rebutted
by clear proof that it was physically or naturally impossible for them to indulge in carnal intercourse. And here there is no such
proof. The fact that Maria Dueas has committed adultery can not also overcome this presumption (Tolentino's Commentaries
on the Civil Code, Vol. I, p. 92).
In view of all the foregoing, we are constrained to hold that the lower court did not err in declaring Mariano Andal as the
legitimate son of the spouses Emiliano Andal and Maria Dueas.
Wherefore, the decision appealed from is affirmed, without pronouncement as to costs.
G.R. No. 105625 January 24, 1994
MARISSA BENITEZ-BADUA, petitioner,
vs.
COURT OF APPEALS, VICTORIA BENITEZ LIRIO AND FEODOR BENITEZ AGUILAR, respondents.
Reynaldo M. Alcantara for petitioner.
Augustus Cesar E. Azura for private respondents.

PUNO, J.:
This is a petition for review of the Decision of the 12th Division of the Court of Appeals in CA-G.R. No. CV No. 30862 dated May
29, 1992. 1
The facts show that the spouses Vicente Benitez and Isabel Chipongian owned various properties especially in Laguna. Isabel
died on April 25, 1982. Vicente followed her in the grave on November 13, 1989. He died intestate.
The fight for administration of Vicente's estate ensued. On September 24, 1990, private respondents Victoria Benitez-Lirio and
Feodor Benitez Aguilar (Vicente's sister and nephew, respectively) instituted Sp. Proc. No. 797 (90) before the RTC of San
Pablo City, 4th Judicial Region, Br. 30. They prayed for the issuance of letters of administration of Vicente's estate in favor of
private respondent Aguilar. They alleged, inter alia, viz.:
xxx xxx xxx
4. The decedent is survived by no other heirs or relatives be they ascendants or descendants, whether legitimate,
illegitimate or legally adopted; despite claims or representation to the contrary, petitioners can well and truly
establish, given the chance to do so, that said decedent and his spouse Isabel Chipongian who pre-deceased him,
and whose estate had earlier been settled extra-judicial, were without issue and/or without descendants whatsoever,
and that one Marissa Benitez-Badua who was raised and cared by them since childhood is, in fact, not related to
them by blood, nor legally adopted, and is therefore not a legal heir; . . .
On November 2, 1990, petitioner opposed the petition. She alleged that she is the sole heir of the deceased Vicente Benitez
and capable of administering his estate. The parties further exchanged reply and rejoinder to buttress their legal postures.
The trial court then received evidence on the issue of petitioner's heirship to the estate of the deceased. Petitioner tried to prove
that she is the only legitimate child of the spouses Vicente Benitez and Isabel Chipongian. She submitted documentary
evidence, among others: (1) her Certificate of Live Birth (Exh. 3); (2) Baptismal Certificate (Exh. 4); (3) Income Tax Returns and
Information Sheet for Membership with the GSIS of the late Vicente naming her as his daughter (Exhs. 10 to 21); and (4) School
Records (Exhs. 5 & 6). She also testified that the said spouses reared an continuously treated her as their legitimate daughter.
On the other hand, private respondents tried to prove, mostly thru testimonial evidence, that the said spouses failed to beget a
child during their marriage; that the late Isabel, then thirty six (36) years of age, was even referred to Dr. Constantino Manahan,
a noted obstetrician-gynecologist, for treatment. Their primary witness, Victoria Benitez-Lirio, elder sister of the late Vicente,
then 77 years of age, 2 categorically declared that petitioner was not the biological child of the said spouses who were unable to physically procreate.
On December 17, 1990, the trial court decided in favor of the petitioner. It dismissed the private respondents petition for letters
and administration and declared petitioner as the legitimate daughter and sole heir of the spouses Vicente O. Benitez and
Isabel Chipongian. The trial court relied on Articles 166 and 170 of the Family Code.
On appeal, however, the Decision of the trial court was reversed on May 29, 1992 by the 17th Division of the Court of Appeals.
The dispositive portion of the Decision of the appellate court states:
WHEREFORE, the decision appealed from herein is REVERSED and another one entered declaring that appellee
Marissa Benitez is not the biological daughter or child by nature of the spouse Vicente O. Benitez and Isabel
Chipongian and, therefore, not a legal heir of the deceased Vicente O. Benitez. Her opposition to the petition for the
appointment of an administrator of the intestate of the deceased Vicente O. Benitez is, consequently, DENIED; said
petition and the proceedings already conducted therein reinstated; and the lower court is directed to proceed with
the hearing of Special proceeding No. SP-797 (90) in accordance with law and the Rules.
Costs against appellee.
SO ORDERED.
In juxtaposition, the appellate court held that the trial court erred in applying Articles 166 and 170 of the Family Code.

In this petition for review, petitioner contends:


1. The Honorable Court of Appeals committed error of law and misapprehension of facts when it failed to apply the
provisions, more particularly, Arts. 164, 166, 170 and 171 of the Family Code in this case and in adopting and
upholding private respondent's theory that the instant case does not involve an action to impugn the legitimacy of a
child;
2. Assuming arguendo that private respondents can question or impugn directly or indirectly, the legitimacy of
Marissa's birth, still the respondent appellate Court committed grave abuse of discretion when it gave more weight
to the testimonial evidence of witnesses of private respondents whose credibility and demeanor have not convinced
the trial court of the truth and sincerity thereof, than the documentary and testimonial evidence of the now petitioner
Marissa Benitez-Badua;
3. The Honorable Court of Appeals has decided the case in a way not in accord with law or with applicable decisions
of the supreme Court, more particularly, on prescription or laches.
We find no merit to the petition.
Petitioner's insistence on the applicability of Articles 164, 166, 170 and 171 of the Family Code to the case at bench cannot be
sustained. These articles provide:
Art. 164. Children conceived or born during the marriage of the parents are legitimate.
Children conceived as a result of artificial insemination of the wife with sperm of the husband or that of a donor or
both are likewise legitimate children of the husband and his wife, provided, that both of them authorized or ratified
such insemination in a written instrument executed and signed by them before the birth of the child. The instrument
shall be recorded in the civil registry together with the birth certificate of the child.
Art. 166. Legitimacy of child may be impugned only on the following grounds:
1) That it was physically impossible for the husband to have sexual intercourse with his wife within the first 120 days
of the 300 days which immediately preceded the birth of the child because of:
a) the physical incapacity of the husband to have sexual intercourse with his wife;
b) the fact that the husband and wife were living separately in such a way that sexual intercourse was
not possible; or
c) serious illness of the husband, which absolutely prevented sexual intercourse.
2) That it is proved that for biological or other scientific reasons, the child could not have been that of the husband
except in the instance provided in the second paragraph of Article 164; or
3) That in case of children conceived through artificial insemination, the written authorization or ratification of either
parent was obtained through mistake, fraud, violence, intimidation, or undue influence.
Art. 170. The action to impugn the legitimacy of the child shall be brought within one year from the knowledge of the
birth or its recording in the civil register, if the husband or, in a proper case, any of his heirs, should reside in the city
or municipality where the birth took place or was recorded.
If the husband or, in his default, all of his heirs do not reside at the place of birth as defined in the first paragraph or
where it was recorded, the period shall be two years if they should reside in the Philippines; and three years if
abroad. If the birth of the child has been concealed from or was unknown to the husband or his heirs, the period
shall be counted from the discovery or knowledge of the birth of the child or of the fact of registration of said birth,
which ever is earlier.
Art. 171. The heirs of the husband may impugn the filiation of the child within the period prescribed in the preceding
Article only in the following case:
1) If the husband should die before the expiration of the period fixed for bringing his action;
2) If he should die after the filing of the complaint, without having desisted therefrom; or
3) If the child was born after the death of the husband.
A careful reading of the above articles will show that they do not contemplate a situation, like in the instant case, where a child is
alleged not to be the child of nature or biological child of a certain couple. Rather, these articles govern a situation where a
husband (or his heirs) denies as his own a child of his wife. Thus, under Article 166, it is the husband who can impugn the
legitimacy of said child by proving: (1) it was physically impossible for him to have sexual intercourse, with his wife within the
first 120 days of the 300 days which immediately preceded the birth of the child; (2) that for biological or other scientific reasons,
the child could not have been his child; (3) that in case of children conceived through artificial insemination, the written
authorization or ratification by either parent was obtained through mistake, fraud, violence, intimidation or undue influence.
Articles 170 and 171 reinforce this reading as they speak of the prescriptive period within which the husband or any of his
heirs should file the action impugning the legitimacy of said child. Doubtless then, the appellate court did not err when it refused
to apply these articles to the case at bench. For the case at bench is not one where the heirs of the late Vicente are contending
that petitioner is not his child by Isabel. Rather, their clear submission is that petitioner was not born to Vicente and Isabel. Our
ruling in Cabatbat-Lim vs. Intermediate Appellate Court, 166 SCRA 451, 457 cited in the impugned decision is apropos, viz.:
Petitioners' recourse to Article 263 of the New Civil Code [now Article 170 of the Family Code] is not well-taken. This
legal provision refers to an action to impugn legitimacy. It is inapplicable to this case because this is not an action to
impugn the legitimacy of a child, but an action of the private respondents to claim their inheritance as legal heirs of
their childless deceased aunt. They do not claim that petitioner Violeta Cabatbat Lim is an illegitimate child of the
deceased, but that she is not the decedent's child at all. Being neither legally adopted child, nor an acknowledged
natural child, nor a child by legal fiction of Esperanza Cabatbat, Violeta is not a legal heir of the deceased.
We now come to the factual finding of the appellate court that petitioner was not the biological child or child of nature of the
spouses Vicente Benitez and Isabel Chipongian. The appellate court exhaustively dissected the evidence of the parties as
follows:
. . . And on this issue, we are constrained to say that appellee's evidence is utterly insufficient to establish her
biological and blood kinship with the aforesaid spouses, while the evidence on record is strong and convincing that
she is not, but that said couple being childless and desirous as they were of having a child, the late Vicente O.
Benitez took Marissa from somewhere while still a baby, and without he and his wife's legally adopting her treated,
cared for, reared, considered, and loved her as their own true child, giving her the status as not so, such that she
herself had believed that she was really their daughter and entitled to inherit from them as such.
The strong and convincing evidence referred to us are the following:

First, the evidence is very cogent and clear that Isabel Chipongian never became pregnant and, therefore, never
delivered a child. Isabel's own only brother and sibling, Dr. Lino Chipongian, admitted that his sister had already
been married for ten years and was already about 36 years old and still she has not begotten or still could not bear a
child, so that he even had to refer her to the late Dr. Constantino Manahan, a well-known and eminent obstetriciangynecologist and the OB of his mother and wife, who treated his sister for a number of years. There is likewise the
testimony of the elder sister of the deceased Vicente O. Benitez, Victoria Benitez Lirio, who then, being a teacher,
helped him (he being the only boy and the youngest of the children of their widowed mother) through law school,
and whom Vicente and his wife highly respected and consulted on family matters, that her brother Vicente and his
wife Isabel being childless, they wanted to adopt her youngest daughter and when she refused, they looked for a
baby to adopt elsewhere, that Vicente found two baby boys but Isabel wanted a baby girl as she feared a boy might
grow up unruly and uncontrollable, and that Vicente finally brought home a baby girl and told his elder sister Victoria
he would register the baby as his and his wife's child. Victoria Benitez Lirio was already 77 years old and too weak
to travel and come to court in San Pablo City, so that the taking of her testimony by the presiding judge of the lower
court had to be held at her residence in Paraaque, MM. Considering, her advanced age and weak physical
condition at the time she testified in this case, Victoria Benitez Lirio's testimony is highly trustworthy and credible, for
as one who may be called by her Creator at any time, she would hardly be interested in material things anymore and
can be expected not to lie, especially under her oath as a witness. There were also several disinterested neighbors
of the couple Vicente O. Benitez and Isabel Chipongian in Nagcarlan, Laguna (Sergio Fule, Cecilia Coronado, and
Benjamin C. Asendido) who testified in this case and declared that they used to see Isabel almost everyday
especially as she had drugstore in the ground floor of her house, but they never saw her to have been pregnant, in
1954 (the year appellee Marissa Benitez was allegedly born, according to her birth certificate Exh. "3") or at any time
at all, and that it is also true with the rest of their townmates. Ressureccion A. Tuico, Isabel Chipongian's personal
beautician who used to set her hair once a week at her (Isabel's) residence, likewise declared that she did not see
Isabel ever become pregnant, that she knows that Isabel never delivered a baby, and that when she saw the baby
Marissa in her crib one day she went to Isabel's house to set the latter's hair, she was surprised and asked the latter
where the baby came from, and "she told me that the child was brought by Atty. Benitez and told me not to tell about
it" (p. 10, tsn, Nov. 29, 1990).
The facts of a woman's becoming pregnant and growing big with child, as well as her delivering a baby, are matters
that cannot be hidden from the public eye, and so is the fact that a woman never became pregnant and could not
have, therefore, delivered a baby at all. Hence, if she is suddenly seen mothering and caring for a baby as if it were
her own, especially at the rather late age of 36 (the age of Isabel Chipongian when appellee Marissa Benitez was
allegedly born), we can be sure that she is not the true mother of that baby.
Second, appellee's birth certificate Exh. "3" with the late Vicente O. Benitez appearing as the informant, is highly
questionable and suspicious. For if Vicente's wife Isabel, who wads already 36 years old at the time of the child's
supposed birth, was truly the mother of that child, as reported by Vicente in her birth certificate, should the child not
have been born in a hospital under the experienced, skillful and caring hands of Isabel's obstetrician-gynecologist
Dr. Constantino Manahan, since delivery of a child at that late age by Isabel would have been difficult and quite risky
to her health and even life? How come, then, that as appearing in appellee's birth certificate, Marissa was
supposedly born at the Benitez home in Avenida Rizal, Nagcarlan, Laguna, with no physician or even a midwife
attending?
At this juncture, it might be meet to mention that it has become a practice in recent times for people who want to
avoid the expense and trouble of a judicial adoption to simply register the child as their supposed child in the civil
registry. Perhaps Atty. Benitez, though a lawyer himself, thought that he could avoid the trouble if not the expense of
adopting the child Marissa through court proceedings by merely putting himself and his wife as the parents of the
child in her birth certificate. Or perhaps he had intended to legally adopt the child when she grew a little older but did
not come around doing so either because he was too busy or for some other reason. But definitely, the mere
registration of a child in his or her birth certificate as the child of the supposed parents is not a valid adoption, does
not confer upon the child the status of an adopted child and the legal rights of such child, and even amounts of
simulation of the child's birth or falsification of his or her birth certificate, which is a public document.
Third, if appellee Marissa Benitez is truly the real, biological daughter of the late Vicente O. Benitez and his wife
Isabel Chipongian, why did he and Isabel's only brother and sibling Dr. Nilo Chipongian, after Isabel's death on April
25, 1982, state in the extrajudicial settlement
Exh. "E" that they executed her estate, "that we are the sole heirs of the deceased ISABEL CHIPONGIAN because
she died without descendants or ascendants?" Dr. Chipongian, placed on a witness stand by appellants, testified
that it was his brother-in-law Atty. Vicente O. Benitez who prepared said document and that he signed the same only
because the latter told him to do so (p. 24, tsn, Nov. 22, 1990). But why would Atty. Benitez make such a statement
in said document, unless appellee Marissa Benitez is not really his and his wife's daughter and descendant and,
therefore, not his deceased wife's legal heir? As for Dr. Chipongian, he lamely explained that he signed said
document without understanding completely the meaning of the words "descendant and ascendant" (p. 21, tsn, Nov.
22, 1990). This we cannot believe, Dr. Chipongian being a practicing pediatrician who has even gone to the United
States (p. 52, tsn, Dec. 13, 1990). Obviously,
Dr. Chipongian was just trying to protect the interests of appellee, the foster-daughter of his deceased sister and
brother-in-law, as against those of the latter's collateral blood relatives.
Fourth, it is likewise odd and strange, if appellee Marissa Benitez is really the daughter and only legal heir of the
spouses Vicente O. Benitez and Isabel Chipongian, that the latter, before her death, would write a note to her
husband and Marissa stating that:
even without any legal papers, I wish that my husband and my child or only daughter will inherit what is
legally my own property, in case I die without a will,
and in the same handwritten note, she even implored her husband
that any inheritance due him from my property when he die to make our own daughter his sole
heir. This do [sic] not mean what he legally owns or his inherited property. I leave him to decide for
himself regarding those.
(Exhs. "F-1", "F-1-A" and "F-1-B")
We say odd and strange, for if Marissa Benitez is really the daughter of the spouses Vicente O. Benitez and Isabel
Chipongian, it would not have been necessary for Isabel to write and plead for the foregoing requests to her
husband, since Marissa would be their legal heir by operation of law. Obviously, Isabel Chipongian had to implore
and supplicate her husband to give appellee although without any legal papers her properties when she dies, and
likewise for her husband to give Marissa the properties that he would inherit from her (Isabel), since she well knew
that Marissa is not truly their daughter and could not be their legal heir unless her (Isabel's) husband makes her so.
Finally, the deceased Vicente O. Benitez' elder sister Victoria Benitez Lirio even testified that her brother Vicente
gave the date
December 8 as Marissa's birthday in her birth certificate because that date is the birthday of their (Victoria and
Vicente's) mother. It is indeed too much of a coincidence for the child Marissa and the mother of Vicente and Victoria

to have the same birthday unless it is true, as Victoria testified, that Marissa was only registered by Vicente as his
and his wife's child and that they gave her the birth date of Vicente's mother.
We sustain these findings as they are not unsupported by the evidence on record. The weight of these findings was not negated
by documentary evidence presented by the petitioner, the most notable of which is her Certificate of Live Birth (Exh. "3")
purportedly showing that her parents were the late
Vicente Benitez and Isabel Chipongian. This Certificate registered on December 28, 1954 appears to have been signed by the
deceased Vicente Benitez. Under Article 410 of the New Civil Code, however, "the books making up the Civil Registry and all
documents relating thereto shall be considered public documents and shall be prima facieevidence of the facts therein stated."
As related above, the totality of contrary evidence, presented by the private respondents sufficiently rebutted the truth of the
content of petitioner's Certificate of Live Birth. of said rebutting evidence, the most telling was the Deed of Extra-Judicial
Settlement of the Estate of the Deceased Isabel Chipongian (Exh. "E") executed on July 20, 1982 by Vicente Benitez, and
Dr. Nilo Chipongian, a brother of Isabel. In their notarized document, they stated that "(they) are the sole heirs of the deceased
Isabel Chipongian because she died without descendants or ascendants". In executing this Deed, Vicente Benitez effectively
repudiated the Certificate of Live Birth of petitioner where it appeared that he was petitioner's father. The repudiation was made
twenty-eight years after he signed petitioner's Certificate of Live Birth.
IN VIEW WHEREOF, the petition for review is dismissed for lack of merit. Costs against petitioner.

[G.R. No. 123450. August 31, 2005]

GERARDO B. CONCEPCION, petitioner, vs. COURT OF APPEALS and MA. THERESA ALMONTE,respondents.
DECISION
CORONA, J.:

The child, by reason of his mental and physical immaturity, needs special safeguard and care, including appropriate legal
protection before as well as after birth. In case of assault on his rights by those who take advantage of his innocence and
vulnerability, the law will rise in his defense with the single-minded purpose of upholding only his best interests.
[1]

This is the story of petitioner Gerardo B. Concepcion and private respondent Ma. Theresa Almonte, and a child named Jose
Gerardo. Gerardo and Ma. Theresa were married on December 29, 1989. After their marriage, they lived with Ma. Theresas
parents in Fairview, Quezon City. Almost a year later, on December 8, 1990, Ma. Theresa gave birth to Jose Gerardo.
[2]

[3]

[4]

Gerardo and Ma. Theresas relationship turned out to be short-lived, however. On December 19, 1991, Gerardo filed a petition to
have his marriage to Ma. Theresa annulled on the ground of bigamy. He alleged that nine years before he married Ma. Theresa on
December 10, 1980, she had married one Mario Gopiao, which marriage was never annulled. Gerardo also found out that Mario
was still alive and was residing in Loyola Heights, Quezon City.
[5]

[6]

[7]

Ma. Theresa did not deny marrying Mario when she was twenty years old. She, however, averred that the marriage was a sham
and that she never lived with Mario at all.
[8]

The trial court ruled that Ma. Theresas marriage to Mario was valid and subsisting when she married Gerardo and annulled her
marriage to the latter for being bigamous. It declared Jose Gerardo to be an illegitimate child as a result. The custody of the child
was awarded to Ma. Theresa while Gerardo was granted visitation rights.
[9]

Ma. Theresa felt betrayed and humiliated when Gerardo had their marriage annulled. She held him responsible for the
bastardization of Gerardo. She moved for the reconsideration of the above decision INSOFAR ONLY as that portion of the
decision which grant(ed) to the petitioner so-called visitation rights between the hours of 8 in the morning to 12:00 p.m. of any
Sunday. She argued that there was nothing in the law granting visitation rights in favor of the putative father of an illegitimate
child. She further maintained that Jose Gerardos surname should be changed from Concepcion to Almonte, her maiden name,
following the rule that an illegitimate child shall use the mothers surname.
[10]

[11]

Gerardo opposed the motion. He insisted on his visitation rights and the retention of Concepcion as Jose Gerardos surname.
Applying the best interest of the child principle, the trial court denied Ma. Theresas motion and made the following
observations:
It is a pity that the parties herein seem to be using their son to get at or to hurt the other, something they should never do if they want to assure the
normal development and well-being of the boy.
The Court allowed visitorial rights to the father knowing that the minor needs a father, especially as he is a boy, who must have a father figure to
recognize something that the mother alone cannot give. Moreover, the Court believes that the emotional and psychological well-being of the
boy would be better served if he were allowed to maintain relationships with his father.
There being no law which compels the Court to act one way or the other on this matter, the Court invokes the provision of Art. 8, PD 603 as
amended, otherwise known as the Child and Youth Welfare Code, to wit:
In all questions regarding the care, custody, education and property of the child, his welfare shall be the paramount consideration.
WHEREFORE, the respondents Motion for Reconsideration has to be, as it is hereby DENIED.

[12]

Ma. Theresa elevated the case to the Court of Appeals, assigning as error the ruling of the trial court granting visitation rights to
Gerardo. She likewise opposed the continued use of Gerardos surname (Concepcion) despite the fact that Jose Gerardo had
already been declared illegitimate and should therefore use her surname (Almonte). The appellate court denied the petition and
affirmed in toto the decision of the trial court.
[13]

On the issue raised by Ma. Theresa that there was nothing in the law that granted a putative father visitation rights over his
illegitimate child, the appellate court affirmed the best interest of the child policy invoked by the court a quo. It ruled that [a]t
bottom, it (was) the childs welfare and not the convenience of the parents which (was) the primary consideration in granting
visitation rights a few hours once a week.
[14]

The appellate court likewise held that an illegitimate child cannot use the mothers surname motu proprio. The child, represented
by the mother, should file a separate proceeding for a change of name under Rule 103 of the Rules of Court to effect the correction
in the civil registry.
[15]

Undaunted, Ma. Theresa moved for the reconsideration of the adverse decision of the appellate court. She also filed a motion to
set the case for oral arguments so that she could better ventilate the issues involved in the controversy.

After hearing the oral arguments of the respective counsels of the parties, the appellate court resolved the motion for
reconsideration. It reversed its earlier ruling and held that Jose Gerardo was not the son of Ma. Theresa by Gerardo but by Mario
during her first marriage:
It is, therefore, undeniable established by the evidence in this case that the appellant [Ma. Theresa] was married to Mario Gopiao, and that she
had never entered into a lawful marriage with the appellee [Gerardo] since the so-called marriage with the latter was void ab initio. It was
[Gerardo] himself who had established these facts. In other words, [Ma. Theresa] was legitimately married to Mario Gopiao when the child Jose
Gerardo was born on December 8, 1990. Therefore, the child Jose Gerardo under the law is the legitimate child of the legal and subsisting
marriage between [Ma. Theresa] and Mario Gopiao; he cannot be deemed to be the illegitimate child of the void and non-existent marriage
between [Ma. Theresa] and [Gerardo], but is said by the law to be the child of the legitimate and existing marriage between [Ma. Theresa] and
Mario Gopiao (Art. 164, Family Code). Consequently, [she] is right in firmly saying that [Gerardo] can claim neither custody nor visitorial rights
over the child Jose Gerardo. Further, [Gerardo] cannot impose his name upon the child. Not only is it without legal basis (even supposing the child
to be his illegitimate child [Art. 146, The Family Code]); it would tend to destroy the existing marriage between [Ma. Theresa] and Gopiao, would
prevent any possible rapproachment between the married couple, and would mean a judicial seal upon an illegitimate relationship.
[16]

The appellate court brushed aside the common admission of Gerardo and Ma. Theresa that Jose Gerardo was their son. It gave
little weight to Jose Gerardos birth certificate showing that he was born a little less than a year after Gerardo and Ma. Theresa were
married:
We are not unaware of the movants argument that various evidence exist that appellee and the appellant have judicially admitted that the minor is
their natural child. But, in the same vein, We cannot overlook the fact that Article 167 of the Family Code mandates:
The child shall be considered legitimate although the mother may have declared against its legitimacy or may have been sentenced as an
adulteress. (underscoring ours)
Thus, implicit from the above provision is the fact that a minor cannot be deprived of his/her legitimate status on the bare declaration of the
mother and/or even much less, the supposed father. In fine, the law and only the law determines who are the legitimate or illegitimate
children for ones legitimacy or illegitimacy cannot ever be compromised. Not even the birth certificate of the minor can change his status for
the information contained therein are merely supplied by the mother and/or the supposed father. It should be what the law says and not what a
parent says it is. (Emphasis supplied)
[17]

Shocked and stunned, Gerardo moved for a reconsideration of the above decision but the same was denied. Hence, this
appeal.
[18]

The status and filiation of a child cannot be compromised. Article 164 of the Family Code is clear. A child who is conceived or
born during the marriage of his parents is legitimate.
[19]

[20]

As a guaranty in favor of the child and to protect his status of legitimacy, Article 167 of the Family Code provides:
[21]

Article 167. The child shall be considered legitimate although the mother may have declared against its legitimacy or may have been sentenced as
an adulteress.
The law requires that every reasonable presumption be made in favor of legitimacy. We explained the rationale of this rule in
the recent case of Cabatania v. Court of Appeals :
[22]

[23]

The presumption of legitimacy does not only flow out of a declaration in the statute but is based on the broad principles of natural justice and the
supposed virtue of the mother. It is grounded on the policy to protect the innocent offspring from the odium of illegitimacy.
Gerardo invokes Article 166 (1)(b) of the Family Code. He cannot. He has no standing in law to dispute the status of Jose
Gerardo. Only Ma. Theresas husband Mario or, in a proper case, his heirs, who can contest the legitimacy of the child Jose
Gerardo born to his wife. Impugning the legitimacy of a child is a strictly personal right of the husband or, in exceptional cases, his
heirs. Since the marriage of Gerardo and Ma. Theresa was void from the very beginning, he never became her husband and thus
never acquired any right to impugn the legitimacy of her child.
[24]

[25]

[26]

[27]

The presumption of legitimacy proceeds from the sexual union in marriage, particularly during the period of conception. To
overthrow this presumption on the basis of Article 166 (1)(b) of the Family Code, it must be shown beyond reasonable doubt that
there was no access that could have enabled the husband to father the child. Sexual intercourse is to be presumed where personal
access is not disproved, unless such presumption is rebutted by evidence to the contrary.
[28]

[29]

[30]

The presumption is quasi-conclusive and may be refuted only by the evidence of physical impossibility of coitus between
husband and wife within the first 120 days of the 300 days which immediately preceded the birth of the child.
[31]

To rebut the presumption, the separation between the spouses must be such as to make marital intimacy impossible. This may
take place, for instance, when they reside in different countries or provinces and they were never together during the period of
conception. Or, the husband was in prison during the period of conception, unless it appears that sexual union took place through
the violation of prison regulations.
[32]

[33]

[34]

Here, during the period that Gerardo and Ma. Theresa were living together in Fairview, Quezon City, Mario was living in Loyola
Heights which is also in Quezon City. Fairview and Loyola Heights are only a scant four kilometers apart.
Not only did both Ma. Theresa and Mario reside in the same city but also that no evidence at all was presented to disprove
personal access between them. Considering these circumstances, the separation between Ma. Theresa and her lawful husband,
Mario, was certainly not such as to make it physically impossible for them to engage in the marital act.
Sexual union between spouses is assumed. Evidence sufficient to defeat the assumption should be presented by him who
asserts the contrary. There is no such evidence here. Thus, the presumption of legitimacy in favor of Jose Gerardo, as the issue of
the marriage between Ma. Theresa and Mario, stands.
Gerardo relies on Ma. Theresas statement in her answer to the petition for annulment of marriage that she never lived with
Mario. He claims this was an admission that there was never any sexual relation between her and Mario, an admission that was
binding on her.
[35]

[36]

Gerardos argument is without merit.


First, the import of Ma. Theresas statement is that Jose Gerardo is not her legitimate son with Mario but her illegitimate son with
Gerardo. This declaration an avowal by the mother that her child is illegitimate is the very declaration that is proscribed by
Article 167 of the Family Code.
The language of the law is unmistakable. An assertion by the mother against the legitimacy of her child cannot affect the
legitimacy of a child born or conceived within a valid marriage.
Second, even assuming the truth of her statement, it does not mean that there was never an instance where Ma. Theresa could
have been together with Mario or that there occurred absolutely no intercourse between them. All she said was that she never lived
with Mario. She never claimed that nothing ever happened between them.
Telling is the fact that both of them were living in Quezon City during the time material to Jose Gerardos conception and birth.
Far from foreclosing the possibility of marital intimacy, their proximity to each other only serves to reinforce such possibility. Thus,
the impossibility of physical access was never established beyond reasonable doubt.
Third, to give credence to Ma. Theresas statement is to allow her to arrogate unto herself a right exclusively lodged in the
husband, or in a proper case, his heirs. A mother has no right to disavow a child because maternity is never uncertain. Hence, Ma.
Theresa is not permitted by law to question Jose Gerardos legitimacy.
[37]

[38]

Finally, for reasons of public decency and morality, a married woman cannot say that she had no intercourse with her husband
and that her offspring is illegitimate. The proscription is in consonance with the presumption in favor of family solidarity. It also
promotes the intention of the law to lean toward the legitimacy of children.
[39]

[40]

Gerardos insistence that the filiation of Jose Gerardo was never an issue both in the trial court and in the appellate court does
not hold water. The fact that both Ma. Theresa and Gerardo admitted and agreed that Jose Gerardo was born to them was
immaterial. That was, in effect, an agreement that the child was illegitimate. If the Court were to validate that stipulation, then it
would be tantamount to allowing the mother to make a declaration against the legitimacy of her child and consenting to the denial of
filiation of the child by persons other than her husband. These are the very acts from which the law seeks to shield the child.
Public policy demands that there be no compromise on the status and filiation of a child. Otherwise, the child will be at the
mercy of those who may be so minded to exploit his defenselessness.
[41]

The reliance of Gerardo on Jose Gerardos birth certificate is misplaced. It has no evidentiary value in this case because it was
not offered in evidence before the trial court. The rule is that the court shall not consider any evidence which has not been formally
offered.
[42]

Moreover, the law itself establishes the status of a child from the moment of his birth. Although a record of birth or birth
certificate may be used as primary evidence of the filiation of a child, as the status of a child is determined by the law itself, proof of
filiation is necessary only when the legitimacy of the child is being questioned, or when the status of a child born after 300 days
following the termination of marriage is sought to be established.
[43]

[44]

[45]

Here, the status of Jose Gerardo as a legitimate child was not under attack as it could not be contested collaterally and, even
then, only by the husband or, in extraordinary cases, his heirs. Hence, the presentation of proof of legitimacy in this case was
improper and uncalled for.
In addition, a record of birth is merely prima facie evidence of the facts contained therein. As prima facie evidence, the
statements in the record of birth may be rebutted by more preponderant evidence. It is not conclusive evidence with respect to the
truthfulness of the statements made therein by the interested parties. Between the certificate of birth which is prima facie evidence
of Jose Gerardos illegitimacy and the quasi-conclusive presumption of law (rebuttable only by proof beyond reasonable doubt) of his
legitimacy, the latter shall prevail. Not only does it bear more weight, it is also more conducive to the best interests of the child and
in consonance with the purpose of the law.
[46]

[47]

It perplexes us why both Gerardo and Ma. Theresa would doggedly press for Jose Gerardos illegitimacy while claiming that they
both had the childs interests at heart. The law, reason and common sense dictate that a legitimate status is more favorable to the
child. In the eyes of the law, the legitimate child enjoys a preferred and superior status. He is entitled to bear the surnames of both
his father and mother, full support and full inheritance. On the other hand, an illegitimate child is bound to use the surname and be
under the parental authority only of his mother. He can claim support only from a more limited group and his legitime is only half of
that of his legitimate counterpart. Moreover (without unwittingly exacerbating the discrimination against him), in the eyes of society,
a bastard is usually regarded as bearing a stigma or mark of dishonor. Needless to state, the legitimacy presumptively vested by
law upon Jose Gerardo favors his interest.
[48]

[49]

It is unfortunate that Jose Gerardo was used as a pawn in the bitter squabble between the very persons who were passionately
declaring their concern for him. The paradox was that he was made to suffer supposedly for his own sake. This madness should
end.
This case has been pending for a very long time already. What is specially tragic is that an innocent child is involved. Jose
Gerardo was barely a year old when these proceedings began. He is now almost fifteen and all this time he has been a victim of
incessant bickering. The law now comes to his aid to write finis to the controversy which has unfairly hounded him since his infancy.
Having only his best interests in mind, we uphold the presumption of his legitimacy.
As a legitimate child, Jose Gerardo shall have the right to bear the surnames of his father Mario and mother Ma. Theresa, in
conformity with the provisions of the Civil Code on surnames. A persons surname or family name identifies the family to which he
belongs and is passed on from parent to child. Hence, Gerardo cannot impose his surname on Jose Gerardo who is, in the eyes of
the law, not related to him in any way.
[50]

[51]

The matter of changing Jose Gerardos name and effecting the corrections of the entries in the civil register regarding his
paternity and filiation should be threshed out in a separate proceeding.
In case of annulment or declaration of absolute nullity of marriage, Article 49 of the Family Code grants visitation rights to a
parent who is deprived of custody of his children. Such visitation rights flow from the natural right of both parent and child to each
others company. There being no such parent-child relationship between them, Gerardo has no legally demandable right to visit
Jose Gerardo.
Our laws seek to promote the welfare of the child. Article 8 of PD 603, otherwise known as the Child and Youth Welfare Code, is
clear and unequivocal:
Article 8. Childs Welfare Paramount. In all questions regarding the care, custody, education and property of the child, his welfare shall be the
paramount consideration.
Article 3 (1) of the United Nations Convention on the Rights of a Child of which the Philippines is a signatory is similarly
emphatic:
Article 3
1. In all actions concerning children, whether undertaken by public or private social welfare institutions, courts of law, administrative
authorities or legislative bodies, the best interests of the child shall be a primary consideration.
The State as parens patriae affords special protection to children from abuse, exploitation and other conditions prejudicial to their
development. It is mandated to provide protection to those of tender years. Through its laws, the State safeguards them from every
one, even their own parents, to the end that their eventual development as responsible citizens and members of society shall not be
impeded, distracted or impaired by family acrimony. This is especially significant where, as in this case, the issue concerns their
filiation as it strikes at their very identity and lineage.
[52]

WHEREFORE, the petition is hereby DENIED. The September 14, 1995 and January 10, 1996 resolutions of the Court of
Appeals in CA-G.R. CV No. 40651 are hereby AFFIRMED.

[G.R. No. 138961. March 7, 2002]

WILLIAM LIYAO, JR., represented by his mother Corazon Garcia, petitioner, vs. JUANITA TANHOTI-LIYAO, PEARL
MARGARET L. TAN, TITA ROSE L. TAN AND LINDA CHRISTINA LIYAO, respondents.
DECISION
DE LEON, JR., J.:

Before us is a petition for review on certiorari assailing the decision dated June 4, 1999 of the Court of Appeals in CA-G.R. C.V.
No. 45394 which reversed the decision of the Regional Trial Court (RTC) of Pasig, Metro Manila, Branch 167 in declaring William
Liyao, Jr. as the illegitimate (spurious) son of the deceased William Liyao and ordering Juanita Tanhoti-Liyao, Pearl Margaret L. Tan,
Tita Rose L. Tan and Linda Christina Liyao to recognize and acknowledge William Liyao, Jr. as a compulsory heir of the deceased
William Liyao and entitled to all successional rights as such and to pay the costs of the suit.
[1]

On November 29,1976, William Liyao, Jr., represented by his mother Corazon G. Garcia, filed Civil Case No. 24943 before the
RTC of Pasig, Branch 167 which is an action for compulsory recognition as the illegitimate (spurious) child of the late William Liyao
against herein respondents, Juanita Tanhoti-Liyao, Pearl Margaret L. Tan, Tita Rose L. Tan and Linda Christina Liyao. The
complaint was later amended to include the allegation that petitioner was in continuous possession and enjoyment of the status of
the child of said William Liyao, petitioner having been recognized and acknowledged as such child by the decedent during his
lifetime."
[2]

[3]

The facts as alleged by petitioner are as follows:


Corazon G. Garcia is legally married to but living separately from Ramon M. Yulo for more than ten (10) years at the time of the
institution of the said civil case. Corazon cohabited with the late William Liyao from 1965 up to the time of Williams untimely demise
on December 2, 1975. They lived together in the company of Corazons two (2) children from her subsisting marriage, namely:
Enrique and Bernadette, both surnamed Yulo, in a succession of rented houses in Quezon City and Manila. This was with the
knowledge of William Liyaos legitimate children, Tita Rose L. Tan and Linda Christina Liyao-Ortiga, from his subsisting marriage with
Juanita Tanhoti Liyao. Tita Rose and Christina were both employed at the Far East Realty Investment, Inc. of which Corazon and
William were then vice president and president, respectively.
Sometime in 1974, Corazon bought a lot from Ortigas and Co. which required the signature of her husband, Ramon Yulo, to
show his consent to the aforesaid sale. She failed to secure his signature and, had never been in touch with him despite the
necessity to meet him. Upon the advice of William Liyao, the sale of the parcel of land located at the Valle Verde Subdivision was
registered under the name of Far East Realty Investment, Inc.
On June 9, 1975, Corazon gave birth to William Liyao, Jr. at the Cardinal Santos Memorial Hospital. During her three (3) day stay
at the hospital, William Liyao visited and stayed with her and the new born baby, William, Jr. (Billy). All the medical and hospital
expenses, food and clothing were paid under the account of William Liyao. William Liyao even asked his confidential secretary, Mrs.
Virginia Rodriguez, to secure a copy of Billys birth certificate. He likewise instructed Corazon to open a bank account for Billy with
the Consolidated Bank and Trust Company and gave weekly amounts to be deposited therein. William Liyao would bring Billy to the
office, introduce him as his good looking son and had their pictures taken together.
[4]

[5]

[6]

During the lifetime of William Liyao, several pictures were taken showing, among others, William Liyao and Corazon together
with Billys godfather, Fr. Julian Ruiz, William Liyaos legal staff and their wives while on vacation in Baguio. Corazon also presented
pictures in court to prove that that she usually accompanied William Liyao while attending various social gatherings and other
important meetings. During the occasion of William Liyaos last birthday on November 22, 1975 held at the Republic Supermarket,
William Liyao expressly acknowledged Billy as his son in the presence of Fr. Ruiz, Maurita Pasion and other friends and said, Hey,
look I am still young, I can still make a good looking son." Since birth, Billy had been in continuous possession and enjoyment of the
status of a recognized and/or acknowledged child of William Liyao by the latters direct and overt acts. William Liyao supported Billy
and paid for his food, clothing and other material needs. However, after William Liyaos death, it was Corazon who provided sole
support to Billy and took care of his tuition fees at La Salle, Greenhills. William Liyao left his personal belongings, collections,
clothing, old newspaper clippings and laminations at the house in White Plains where he shared his last moments with Corazon.
[7]

[8]

[9]

Testifying for the petitioner, Maurita Pasion declared that she knew both Corazon G. Garcia and William Liyao who were
godparents to her children. She used to visit Corazon and William Liyao from 1965-1975. The two children of Corazon from her
marriage to Ramon Yulo, namely, Bernadette and Enrique (Ike), together with some housemaids lived with Corazon and William
Liyao as one family. On some occasions like birthdays or some other celebrations, Maurita would sleep in the couples residence
and cook for the family. During these occasions, she would usually see William Liyao in sleeping clothes. When Corazon, during the
latter part of 1974, was pregnant with her child Billy, Maurita often visited her three (3) to four (4) times a week in Greenhills and later
on in White Plains where she would often see William Liyao. Being a close friend of Corazon, she was at the Cardinal Santos
Memorial Hospital during the birth of Billy. She continuously visited them at White Plains and knew that William Liyao, while living
with her friend Corazon, gave support by way of grocery supplies, money for household expenses and matriculation fees for the two
(2) older children, Bernadette and Enrique. During William Liyaos birthday on November 22, 1975 held at the Republic Supermarket
Office, he was carrying Billy and told everybody present, including his two (2) daughters from his legal marriage, Look, this is my

son, very guapo and healthy. He then talked about his plan for the baptism of Billy before Christmas. He intended to make it
engrande and make the bells of San Sebastian Church ring. Unfortunately, this did not happen since William Liyao passed away
on December 2, 1975. Maurita attended Mr. Liyaos funeral and helped Corazon pack his clothes. She even recognized a short
sleeved shirt of blue and gray which Mr. Liyao wore in a photograph as well as another shirt of lime green as belonging to the
deceased. A note was also presented with the following inscriptions: To Cora, Love From William. Maurita remembered having
invited the couple during her mothers birthday where the couple had their pictures taken while exhibiting affectionate poses with one
another. Maurita knew that Corazon is still married to Ramon Yulo since her marriage has not been annulled nor is Corazon legally
separated from her said husband. However, during the entire cohabitation of William Liyao with Corazon Garcia, Maurita had not
seen Ramon Yulo or any other man in the house when she usually visited Corazon.
[10]

[11]

[12]

[13]

[14]

[15]

Gloria Panopio testified that she is the owner of a beauty parlor and that she knew that Billy is the son of her neighbors, William
Liyao and Corazon Garcia, the latter being one of her customers. Gloria met Mr. Liyao at Corazons house in Scout Delgado,
Quezon City in the Christmas of 1965. Gloria had numerous occasions to see Mr. Liyao from 1966 to 1974 and even more so when
the couple transferred to White Plains, Quezon City from 1974-1975. At the time Corazon was conceiving, Mr. Liyao was worried that
Corazon might have another miscarriage so he insisted that she just stay in the house, play mahjong and not be bored. Gloria taught
Corazon how to play mahjong and together with Atty. Brillantes wife and sister-in-law, had mahjong sessions among themselves.
Gloria knew that Mr. Liyao provided Corazon with a rented house, paid the salary of the maids and food for Billy. He also gave
Corazon financial support. Gloria knew that Corazon is married but is separated from Ramon Yulo although Gloria never had any
occasion to see Mr. Yulo with Corazon in the house where Mr. Liyao and Corazon lived.
Enrique Garcia Yulo testified that he had not heard from his father, Ramon Yulo, from the time that the latter abandoned and
separated from his family. Enrique was about six (6) years old when William Liyao started to live with them up to the time of the
latters death on December 2, 1975. Mr. Liyao was very supportive and fond of Enriques half brother, Billy. He identified several
pictures showing Mr. Liyao carrying Billy at the house as well as in the office. Enriques testimony was corroborated by his sister,
Bernadette Yulo, who testified that the various pictures showing Mr. Liyao carrying Billy could not have been superimposed and that
the negatives were in the possession of her mother, Corazon Garcia.
Respondents, on the other hand, painted a different picture of the story.
Linda Christina Liyao-Ortiga stated that her parents, William Liyao and Juanita Tanhoti-Liyao, were legally married. Linda grew
up and lived with her parents at San Lorenzo Village, Makati, Metro Manila until she got married; that her parents were not separated
legally or in fact and that there was no reason why any of her parents would institute legal separation proceedings in court. Her
father lived at their house in San Lorenzo Village and came home regularly. Even during out of town business trips or for
conferences with the lawyers at the office, her father would change his clothes at home because of his personal hygiene and habits.
Her father reportedly had trouble sleeping in other peoples homes. Linda described him as very conservative and a strict
disciplinarian. He believed that no amount of success would compensate for failure of a home. As a businessman, he was very
tough, strong, fought for what he believed in and did not give up easily. He suffered two strokes before the fatal attack which led to
his death on December 2, 1975. He suffered a stroke at the office sometime in April-May 1974 and was attended by Dr. Santiago Co.
He then stayed in the house for two (2) to three (3) months for his therapy and acupuncture treatment. He could not talk, move, walk,
write or sign his name. In the meantime, Linda and her sister, Tita Rose Liyao-Tan, ran the office. She handled the collection of rents
while her sister referred legal matters to their lawyers. William Liyao was bedridden and had personally changed. He was not active
in business and had dietary restrictions. Mr. Liyao also suffered a milder stroke during the latter part of September to October 1974.
He stayed home for two (2) to three (3) days and went back to work. He felt depressed, however, and was easily bored. He did not
put in long hours in the office unlike before and tried to spend more time with his family.
[16]

Linda testified that she knew Corazon Garcia is still married to Ramon Yulo. Corazon was not legally separated from her
husband and the records from the Local Civil Registrar do not indicate that the couple obtained any annulment of their marriage.
Once in 1973, Linda chanced upon Ramon Yulo picking up Corazon Garcia at the company garage. Immediately after the death of
Lindas father, Corazon went to Lindas office for the return of the formers alleged investments with the Far East Realty Investment,
Inc. including a parcel of land sold by Ortigas and Company. Linda added that Corazon, while still a Vice-President of the company,
was able to take out documents, clothes and several laminated pictures of William Liyao from the office. There was one instance
when she was told by the guards, Mrs. Yulo is leaving and taking out things again. Linda then instructed the guards to bring Mrs.
Yulo to the office upstairs but her sister, Tita Rose, decided to let Corazon Garcia go. Linda did not recognize any article of clothing
which belonged to her father after having been shown three (3) large suit cases full of mens clothes, underwear, sweaters, shorts
and pajamas.
[17]

[18]

Tita Rose Liyao-Tan testified that her parents were legally married and had never been separated. They resided at No. 21
Hernandez Street, San Lorenzo Village, Makati up to the time of her fathers death on December 2, 1975. Her father suffered two
(2) minor cardio-vascular arrests (CVA) prior to his death. During the first heart attack sometime between April and May 1974, his
speech and hands were affected and he had to stay home for two (2) to three (3) months under strict medication, taking aldomet,
serpadil and cifromet which were prescribed by Dr. Bonifacio Yap, for high blood pressure and cholesterol level control. Tita Rose
testified that after the death of Mr. Liyao, Corazon Garcia was paid the amount of One Hundred Thousand Pesos (P100,000.00)
representing her investment in the Far East Realty Investment Inc. Tita Rose also stated that her family never received any formal
demand that they recognize a certain William Liyao, Jr. as an illegitimate son of her father, William Liyao. After assuming the position
of President of the company, Tita Rose did not come across any check signed by her late father representing payment to lessors as
rentals for the house occupied by Corazon Garcia. Tita Rose added that the laminated photographs presented by Corazon Garcia
are the personal collection of the deceased which were displayed at the latters office.
[19]

[20]

The last witness who testified for the respondents was Ramon Pineda, driver and bodyguard of William Liyao from 1962 to 1974,
who said that he usually reported for work at San Lorenzo Village, Makati to pick up his boss at 8:00 oclock in the morning. At past
7:00 oclock in the evening, either Carlos Palamigan or Serafin Villacillo took over as night shift driver. Sometime between April and
May 1974, Mr. Liyao got sick. It was only after a month that he was able to report to the office. Thereafter, Mr. Liyao was not able to
report to the office regularly. Sometime in September 1974, Mr. Liyao suffered from another heart attack. Mr. Pineda added that as a
driver and bodyguard of Mr. Liyao, he ran errands for the latter among which was buying medicine for him like capasid and aldomet.
On December 2, 1975, Mr. Pineda was called inside the office of Mr. Liyao. Mr. Pineda saw his employer leaning on the table. He
tried to massage Mr. Liyaos breast and decided later to carry and bring him to the hospital but Mr. Liyao died upon arrival thereat.
Mrs. Liyao and her daughter, Linda Liyao-Ortiga were the first to arrive at the hospital.
Mr. Pineda also declared that he knew Corazon Garcia to be one of the employees of the Republic Supermarket. People in the
office knew that she was married. Her husband, Ramon Yulo, would sometimes go to the office. One time, in 1974, Mr. Pineda saw
Ramon Yulo at the office garage as if to fetch Corazon Garcia. Mr. Yulo who was also asking about cars for sale, represented himself
as car dealer.
Witness Pineda declared that he did not know anything about the claim of Corazon. He freely relayed the information that he saw
Mr. Yulo in the garage of Republic Supermarket once in 1973 and then in 1974 to Atty. Quisumbing when he went to the latters law
office. Being the driver of Mr. Liyao for a number of years, Pineda said that he remembered having driven the group of Mr. Liyao,
Atty. Astraquillo, Atty. Brillantes, Atty. Magno and Atty. Laguio to Baguio for a vacation together with the lawyers wives. During his
employment, as driver of Mr. Liyao, he does not remember driving for Corazon Garcia on a trip to Baguio or for activities like
shopping.
On August 31, 1993, the trial court rendered a decision, the dispositive portion of which reads as follows:
WHEREFORE, judgment is hereby rendered in favor of the plaintiff and against the defendants as follows:
(a) Confirming the appointment of Corazon G. Garcia as the guardian ad litem of the minor William Liyao, Jr.;
(b) Declaring the minor William Liyao, Jr. as the illegitimate (spurious) son of the deceased William Liyao;
(c) Ordering the defendants Juanita Tanhoti Liyao, Pearl Margaret L. Tan, Tita Rose L. Tan and Christian Liyao, to recognize, and
acknowledge the minor William Liyao, Jr. as a compulsory heir of the deceased William Liyao, entitled to all succesional rights as
such; and
(d) Costs of suit.[21]

In ruling for herein petitioner, the trial court said it was convinced by preponderance of evidence that the deceased William Liyao
sired William Liyao, Jr. since the latter was conceived at the time when Corazon Garcia cohabited with the deceased. The trial court
observed that herein petitioner had been in continuous possession and enjoyment of the status of a child of the deceased by direct
and overt acts of the latter such as securing the birth certificate of petitioner through his confidential secretary, Mrs. Virginia
Rodriguez; openly and publicly acknowledging petitioner as his son; providing sustenance and even introducing herein petitioner to
his legitimate children.
The Court of Appeals, however, reversed the ruling of the trial court saying that the law favors the legitimacy rather than the
illegitimacy of the child and the presumption of legitimacy is thwarted only on ethnic ground and by proof that marital intimacy
between husband and wife was physically impossible at the period cited in Article 257 in relation to Article 255 of the Civil Code. The
appellate court gave weight to the testimonies of some witnesses for the respondents that Corazon Garcia and Ramon Yulo who
were still legally married and have not secured legal separation, were seen in each others company during the supposed time that
Corazon cohabited with the deceased William Liyao. The appellate court further noted that the birth certificate and the baptismal
certificate of William Liyao, Jr. which were presented by petitioner are not sufficient to establish proof of paternity in the absence of
any evidence that the deceased, William Liyao, had a hand in the preparation of said certificates and considering that his signature
does not appear thereon. The Court of Appeals stated that neither do family pictures constitute competent proof of filiation. With
regard to the passbook which was presented as evidence for petitioner, the appellate court observed that there was nothing in it to
prove that the same was opened by William Liyao for either petitioner or Corazon Garcia since William Liyaos signature and name
do not appear thereon.
His motion for reconsideration having been denied, petitioner filed the present petition.
It must be stated at the outset that both petitioner and respondents have raised a number of issues which relate solely to the
sufficiency of evidence presented by petitioner to establish his claim of filiation with the late William Liyao. Unfortunately, both parties
have consistently overlooked the real crux of this litigation: May petitioner impugn his own legitimacy to be able to claim from the
estate of his supposed father, William Liyao?
We deny the present petition.
Under the New Civil Code, a child born and conceived during a valid marriage is presumed to be legitimate. The presumption of
legitimacy of children does not only flow out from a declaration contained in the statute but is based on the broad principles of natural
justice and the supposed virtue of the mother. The presumption is grounded in a policy to protect innocent offspring from the odium
of illegitimacy.
[22]

[23]

The presumption of legitimacy of the child, however, is not conclusive and consequently, may be overthrown by evidence to the
contrary. Hence, Article 255 of the New Civil Code provides:
[24]

Article 255. Children born after one hundred and eighty days following the celebration of the marriage, and before three hundred days following
its dissolution or the separation of the spouses shall be presumed to be legitimate.
Against this presumption no evidence shall be admitted other than that of the physical impossibility of the husband having access to his wife
within the first one hundred and twenty days of the three hundred which preceded the birth of the child.
This physical impossibility may be caused:
1) By the impotence of the husband;
2) By the fact that husband and wife were living separately in such a way that access was not possible;
3) By the serious illness of the husband.
Petitioner insists that his mother, Corazon Garcia, had been living separately for ten (10) years from her husband, Ramon Yulo,
at the time that she cohabited with the late William Liyao and it was physically impossible for her to have sexual relations with
Ramon Yulo when petitioner was conceived and born. To bolster his claim, petitioner presented a document entitled, Contract of
Separation, executed and signed by Ramon Yulo indicating a waiver of rights to any and all claims on any property that Corazon
Garcia might acquire in the future.
[25]

[26]

The fact that Corazon Garcia had been living separately from her husband, Ramon Yulo, at the time petitioner was conceived
and born is of no moment. While physical impossibility for the husband to have sexual intercourse with his wife is one of the grounds
for impugning the legitimacy of the child, it bears emphasis that the grounds for impugning the legitimacy of the child mentioned in
Article 255 of the Civil Code may only be invoked by the husband, or in proper cases, his heirs under the conditions set forth under
Article 262 of the Civil Code. Impugning the legitimacy of the child is a strictly personal right of the husband, or in exceptional
cases, his heirs for the simple reason that he is the one directly confronted with the scandal and ridicule which the infidelity of his
wife produces and he should be the one to decide whether to conceal that infidelity or expose it in view of the moral and economic
interest involved. It is only in exceptional cases that his heirs are allowed to contest such legitimacy. Outside of these cases, none even his heirs - can impugn legitimacy; that would amount o an insult to his memory.
[27]

[28]

[29]

It is therefor clear that the present petition initiated by Corazon G. Garcia as guardian ad litem of the then minor, herein petitioner,
to compel recognition by respondents of petitioner William Liyao, Jr, as the illegitimate son of the late William Liyao cannot prosper. It
is settled that a child born within a valid marriage is presumed legitimate even though the mother may have declared against its
legitimacy or may have been sentenced as an adulteress. We cannot allow petitioner to maintain his present petition and subvert
the clear mandate of the law that only the husband, or in exceptional circumstances, his heirs, could impugn the legitimacy of a child
born in a valid and subsisting marriage. The child himself cannot choose his own filiation. If the husband, presumed to be the father
does not impugn the legitimacy of the child, then the status of the child is fixed, and the latter cannot choose to be the child of his
mothers alleged paramour. On the other hand, if the presumption of legitimacy is overthrown, the child cannot elect the paternity of
the husband who successfully defeated the presumption.
[30]

[31]

Do the acts of Enrique and Bernadette Yulo, the undisputed children of Corazon Garcia with Ramon Yulo, in testifying for herein
petitioner amount to impugnation of the legitimacy of the latter?
We think not. As earlier stated, it is only in exceptional cases that the heirs of the husband are allowed to contest the legitimacy
of the child. There is nothing on the records to indicate that Ramon Yulo has already passed away at the time of the birth of the
petitioner nor at the time of the initiation of this proceedings. Notably, the case at bar was initiated by petitioner himself through his
mother, Corazon Garcia, and not through Enrique and Bernadette Yulo. It is settled that the legitimacy of the child can be impugned
only in a direct action brought for that purpose, by the proper parties and within the period limited by law.
Considering the foregoing, we find no reason to discuss the sufficiency of the evidence presented by both parties on the
petitioners claim of alleged filiation with the late William Liyao. In any event, there is no clear, competent and positive evidence
presented by the petitioner that his alleged father had admitted or recognized his paternity.
WHEREFORE, the instant petition is DENIED. The assailed decision of the Court of Appeals in CA-G.R. CV No. 45394 is hereby
AFFIRMED. No costs.

G.R. No. 157037

May 20, 2004

ROSALINA P. ECETA, petitioner,


vs.
MA. THERESA VELL LAGURA ECETA, respondent.
DECISION
YNARES-SANTIAGO, J.:
This petition for review on certiorari assails the Decision 1 of the Court of Appeals in CA-G.R. CV No. 50449 which affirmed with
modification the trial courts ruling that respondent Maria Theresa Vell Lagura Eceta is entitled to one-eight (1/8) portion of the
disputed property.
The antecedent facts are as follows:
Petitioner Rosalina P. Vda. De Eceta was married to Isaac Eceta sometime in 1926. During the subsistence of their
marriage, they begot a son, Vicente. The couple acquired several properties, among which is the disputed property
located at Stanford, Cubao, Quezon City covered by Transfer Certificate of Title No. 61036. Isaac died in 1967 leaving
behind Rosalina and Vicente as his compulsory heirs.
In 1977, Vicente died. During his lifetime, however, he sired Maria Theresa, an illegitimate daughter. Thus at the time of
his death, his compulsory heirs were his mother, Rosalina, and illegitimate child, Maria Theresa.
In 1991, Maria Theresa filed a case before the Regional Trial Court of Quezon City, Branch 218, for "Partition and Accounting
with Damages"2 against Rosalina alleging that by virtue of her fathers death, she became Rosalinas co-heir and co-owner of
the Cubao property. The case was docketed as Civil Case No. Q-91-8922.
In her answer, Rosalina alleged that the property is paraphernal in nature and thus belonged to her exclusively.
During the pre-trial conference, the parties entered into a stipulation of facts wherein they both admitted their relationship to one
another, i.e., that Rosalina is Maria Theresas grandmother.3
After trial on the merits, the court a quo rendered judgment, the dispositive portion of which reads:
WHEREFORE, premises considered, judgment is hereby rendered as follows:
a) Theresa Eceta and Rosalina Eceta are the only surviving co-heirs and co-owners over the parcel of land and
improvements thereon subject of this case;
b) Maria Theresa Eceta is entitled to one fourth share of said property;
c) Rosalina Eceta is ordered to account for the value corresponding to the one-fourth undivided share of Theresa
Eceta in the monthly rentals of the property with interest and must commence from the filing of this case;
d) Parties are ordered within fifteen days from receipt of this decision to amicably agree upon a written partition and
to submit the same for approval, parties shall appoint a commissioner to effect said partition of the property between
the parties;
e) The counterclaim by defendant Rosalina is hereby dismissed. 4
Rosalina appealed the decision to the Court of Appeals, which affirmed with modification the trial courts ruling, thus:
WHEREFORE, premises considered, the appealed Decision is hereby AFFIRMED subject to the MODIFICATION that the
one-fourth (1/4) share erroneously decreed to Appellee is hereby REDUCED to one-eight (1/8) undivided share of the
entire disputed property, covered by TCT No. 61036, in accordance with law.
Her motion for reconsideration having been denied, Rosalina is now before us by way of petition for review wherein she submits
the following issues:
a. Whether the certified xerox copy from a xerox copy of the certificate of live birth (Exhibit A) is competent evidence to
prove the alleged filiation of the respondent as an "illegitimate daughter" of her alleged father Vicente Eceta.
b. Whether the admission made by petitioner that respondent is her granddaughter is enough to prove respondents
filiation with Vicente Eceta, the only son of petitioner.

c. Whether the action for recognition has already prescribed.


The petition has no merit.
We note Rosalinas attempt to mislead the Court by representing that this case is one for compulsory recognition, partition and
accounting with damages.5 Notably, what was filed and tried before the trial court and the Court of Appeals is one for partition
and accounting with damages only. The filiation, or compulsory recognition by Vicente Eceta of Maria Theresa, was never put in
issue. In fact, both parties have already agreed and admitted, as duly noted in the trial courts pre-trial order, 6 that Maria Theresa
is Rosalinas granddaughter.
Notwithstanding, Maria Theresa successfully established her filiation with Vicente by presenting a duly authenticated birth
certificate.7 Vicente himself signed Maria Theresas birth certificate thereby acknowledging that she is his daughter. By this act
alone, Vicente is deemed to have acknowledged his paternity over Maria Theresa, thus:
The filiation of illegitimate children, like legitimate children, is established by (1) the record of birth appearing in the civil
register or a final judgment; or (2) an admission of legitimate filiation in a public document or a private handwritten
instrument and signed by the parent concerned. In the absence thereof, filiation shall be proved by (1) the open and
continuous possession of the status of a legitimate child; or (2) any other means allowed by the Rules of Court and special
laws. The due recognition of an illegitimate child in a record of birth, a will, a statement before a court of record, or in any
authentic writing is, in itself, a consummated act of acknowledgement of the child, and no further court action is required.
In fact, any authentic writing is treated not just a ground for compulsory recognition; it is in itself a voluntary recognition
that does not require a separate action for judicial approval. 8
In view of the foregoing, we find no necessity to discuss the other issues submitted.
WHEREFORE, the petition for review on certiorari is DENIED. The decision of the Court of Appeals in CA-G.R. CV No. 50449,
which affirmed with modification the decision of the Regional Trial Court of Quezon City, Branch 218 in Civil Case No. Q-918922, is AFFIRMED in toto.

G.R. No. 140500. January 21, 2002]

ERNESTINA BERNABE, petitioner,


BERNABE, respondent.

vs. CAROLINA

ALEJO

as

guardian

ad litem for

the

minor

ADRIAN

DECISION
PANGANIBAN, J.:

The right to seek recognition granted by the Civil Code to illegitimate children who were still minors at the time the Family Code
took effect cannot be impaired or taken away. The minors have up to four years from attaining majority age within which to file an
action for recognition.
Statement of the Case
Before us is a Petition for Review on Certiorari under Rule 45 of the Rules of Court, praying for (1) the nullification of the July 7,
1999 Court of Appeals (CA) Decision in CA-GR CV No. 51919 and the October 14, 1999 CA Resolution denying petitioners
Motion for Reconsideration, as well as (2) the reinstatement of the two Orders issued by the Regional Trial Court (RTC)
of Pasay City (Branch 109) concerning the same case. The dispositive portion of the assailed Decision reads as follows:
[1]

[2]

[3]

[4]

WHEREFORE, premises considered, the order of the lower court dismissing Civil Case No. 94-0562 is REVERSED and SET ASIDE. Let the
records of this case be remanded to the lower court for trial on the merits. [5]
The Facts
The undisputed facts are summarized by the Court of Appeals in this wise:
The late Fiscal Ernesto A. Bernabe allegedly fathered a son with his secretary of twenty-three (23) years, herein plaintiff-appellant
Carolina Alejo. The son was born on September 18, 1981 and was named Adrian Bernabe. Fiscal Bernabe died on August 13, 1993, while his
wife Rosalina died on December 3 of the same year, leaving Ernestina as the sole surviving heir.
On May 16, 1994, Carolina, in behalf of Adrian, filed the aforesaid complaint praying that Adrian be declared an acknowledged illegitimate son
of Fiscal Bernabeand as such he (Adrian) be given his share in Fiscal Bernabes estate, which is now being held by Ernestina as the sole surviving
heir.
On July 16, 1995, the Regional Trial Court dismissed the complaint, ruling that under the provisions of the Family Code as well as the case
of Uyguangco vs. Court of Appeals, the complaint is now barred x x x.[6]
Orders of the Trial Court
In an Order dated July 26, 1995, the trial court granted Ernestina Bernabes Motion for Reconsideration of the trial courts
Decision and ordered the dismissal of the Complaint for recognition. Citing Article 175 of the Family Code, the RTC held that the
death of the putative father had barred the action.
In its Order dated October 6, 1995, the trial court added that since the putative father had not acknowledged or recognized
Adrian Bernabein writing, the action for recognition should have been filed during the lifetime of the alleged father to give him the
opportunity to either affirm or deny the childs filiation.
Ruling of the Court of Appeals
On the other hand, the Court of Appeals ruled that in the interest of justice, Adrian should be allowed to prove that he was the
illegitimate son of Fiscal Bernabe. Because the boy was born in 1981, his rights are governed by Article 285 of the Civil Code, which
allows an action for recognition to be filed within four years after the child has attained the age of majority. The subsequent
enactment of the Family Code did not take away that right.
Hence, this appeal.

[7]

Issues

In her Memorandum, petitioner raises the following issues for our consideration:
[8]

Whether or not respondent has a cause of action to file a case against petitioner, the legitimate daughter of the putative father, for recognition and
partition with accounting after the putative fathers death in the absence of any written acknowledgment of paternity by the latter.
II

Whether or not the Honorable Court of Appeals erred in ruling that respondents had four years from the attainment of minority to file an action
for recognition as provided in Art. 285 of the Civil Code, in complete disregard of its repeal by the [express] provisions of the Family Code and
the applicable jurisprudence as held by the Honorable Court of Appeals.
III

Whether or not the petition for certiorari filed by the petition[er] is fatally defective for failure to implead the Court of Appeals as one of the
respondents.[9]
The Courts Ruling
The Petition has no merit.
First and Second Issues: Period to File Action for Recognition
Because the first and the second issues are interrelated, we shall discuss them jointly.
Petitioner contends that respondent is barred from filing an action for recognition, because Article 285 of the Civil Code has been
supplanted by the provisions of the Family Code. She argues that the latter Code should be given retroactive effect, since no vested
right would be impaired. We do not agree.
Article 285 of the Civil Code provides the period for filing an action for recognition as follows:
ART. 285. The action for the recognition of natural children may be brought only during the lifetime of the presumed parents, except in the
following cases:
(1)

If the father or mother died during the minority of the child, in which case the latter may file the action before the expiration of four
years from the attainment of his majority;

(2)

If after the death of the father or of the mother a document should appear of which nothing had been heard and in which either or
both parents recognize the child.

In this case, the action must be commenced within four years from the finding of the document.
The two exceptions provided under the foregoing provision, have however been omitted by Articles 172, 173 and 175 of the
Family Code, which we quote:
ART. 172. The filiation of legitimate children is established by any of the following:
(1)

The record of birth appearing in the civil register or a final judgment; or

(2)

An admission of legitimate filiation in a public document or a private handwritten instrument and signed by the parent concerned.

In the absence of the foregoing evidence, the legitimate filiation shall be proved by:
(1)

The open and continuous possession of the status of a legitimate child; or

(2)

Any other means allowed by the Rules of Court and special laws.

ART. 173. The action to claim legitimacy may be brought by the child during his or her lifetime and shall be transmitted to the heirs should the
child die during minority or in a state of insanity. In these cases, the heirs shall have a period of five years within which to institute the action.
The action already commenced by the child shall survive notwithstanding the death of either or both of the parties.
ART. 175. Illegitimate children may establish their illegitimate filiation in the same way and on the same, evidence as legitimate children.
The action must be brought within the same period specified in Article 173, except when the action is based on the second paragraph of Article
172, in which case the action may be brought during the lifetime of the alleged parent.
Under the new law, an action for the recognition of an illegitimate child must be brought within the lifetime of the alleged parent.
The Family Code makes no distinction on whether the former was still a minor when the latter died. Thus, the putative parent is given
by the new Code a chance to dispute the claim, considering that illegitimate children are usually begotten and raised in secrecy and
without the legitimate family being aware of their existence. x x x The putative parent should thus be given the opportunity to affirm
or deny the childs filiation, and this, he or she cannot do if he or she is already dead.
[10]

Nonetheless, the Family Code provides the caveat that rights that have already vested prior to its enactment should not be
prejudiced or impaired as follows:
ART. 255. This Code shall have retroactive effect insofar as it does not prejudice or impair vested or acquired rights in accordance with the Civil
Code or other laws.
The crucial issue to be resolved therefore is whether Adrians right to an action for recognition, which was granted by Article 285
of the Civil Code, had already vested prior to the enactment of the Family Code. Our answer is affirmative.
A vested right is defined as one which is absolute, complete and unconditional, to the exercise of which no obstacle exists, and
which is immediate and perfect in itself and not dependent upon a contingency x x x. Respondent however contends that the filing
of an action for recognition is procedural in nature and that as a general rule, no vested right may attach to [or] arise from
procedural laws.
[11]

[12]

Bustos v. Lucero distinguished substantive from procedural law in these words:


[13]

x x x. Substantive law creates substantive rights and the two terms in this respect may be said to be synonymous. Substantive rights is a term
which includes those rights which one enjoys under the legal system prior to the disturbance of normal relations. Substantive law is that part of the
law which creates, defines and regulates rights, or which regulates the rights and duties which give rise to a cause of action; that part of the law
which courts are established to administer; as opposed to adjective or remedial law, which prescribes the method of enforcing rights or obtains
redress for their invasion.[14] (Citations omitted)
Recently, in Fabian v. Desierto, the Court laid down the test for determining whether a rule is procedural or substantive:
[15]

[I]n determining whether a rule prescribed by the Supreme Court, for the practice and procedure of the lower courts, abridges, enlarges, or
modifies any substantive right, the test is whether the rule really regulates procedure, that is, the judicial process for enforcing rights and duties
recognized by substantive law and for justly administering remedy and redress for a disregard or infraction of them. If the rule takes away a vested

right, it is not procedural. If the rule creates a right such as the right to appeal, it may be classified as a substantive matter; but if it operates as a
means of implementing an existing right then the rule deals merely with procedure. [16]
Applying the foregoing jurisprudence, we hold that Article 285 of the Civil Code is a substantive law, as it gives Adrian the right to
file his petition for recognition within four years from attaining majority age. Therefore, the Family Code cannot impair or
take Adrians right to file an action for recognition, because that right had already vested prior to its enactment.
Uyguangco v. Court of Appeals is not applicable to the case at bar, because the plaintiff therein sought recognition as an
illegitimate child when he was no longer a minor. On the other hand, in Aruego Jr. v. Court of Appeals the Court ruled that an action
for recognition filed while the Civil Code was in effect should not be affected by the subsequent enactment of the Family Code,
because the right had already vested.
[17]

[18]

Not Limited to Natural Children


To be sure, Article 285 of the Civil Code refers to the action for recognition of natural children. Thus, petitioner contends that the
provision cannot be availed of by respondent, because at the time of his conception, his parents were impeded from marrying each
other. In other words, he is not a natural child.
A natural child is one whose parents, at the time of conception, were not disqualified by any legal impediment from marrying
each other. Thus, in De Santos v. Angeles, the Court explained:
[19]

A childs parents should not have been disqualified to marry each other at the time of conception for him to qualify as a natural child. [20]
A strict and literal interpretation of Article 285 has already been frowned upon by this Court in the aforesaid case
of Aruego, which allowed minors to file a case for recognition even if their parents were disqualified from marrying each other. There,
the Complaint averred that the late Jose Aruego Sr., a married man, had an extramarital liason with Luz Fabian. Out of this
relationship were born two illegitimate children who in 1983 filed an action for recognition. The two children were born in 1962 and
1963, while the alleged putative father died in 1982. In short, at the time of their conception, the two childrens parents were legally
disqualified from marrying each other. The Court allowed the Complaint to prosper, even though it had been filed almost a year after
the death of the presumed father. At the time of his death, both children were still minors.
Moreover, in the earlier case Divinagracia v. Rovira, the Court said that the rules on voluntary and compulsory acknowledgment
of natural children, as well as the prescriptive period for filing such action, may likewise be applied to spurious children. Pertinent
portions of the case are quoted hereunder:
[21]

The so-called spurious children, or illegitimate children other than natural children, commonly known as bastards, include those adulterous
children or those born out of wedlock to a married woman cohabiting with a man other than her husband or to a married man cohabiting with a
woman other than his wife. They are entitled to support and successional rights. But their filiation must be duly proven.
How should their filiation be proven? Article 289 of the Civil Code allows the investigation of the paternity or maternity or spurious children
under the circumstances specified in articles 283 and 284 of the Civil Code. The implication is that the rules on compulsory recognition of natural
children are applicable to spurious children.
Spurious children should not be in a better position than natural children. The rules on proof of filiation of natural children or the rules on
voluntary and compulsory acknowledgment for natural children may be applied to spurious children.
That does not mean that spurious children should be acknowledged, as that term is used with respect to natural children. What is simply meant is
that the grounds or instances for the acknowledgment of natural children are utilized to establish the filiation of spurious children.
A spurious child may prove his filiation by means of a record of birth, a will, a statement before a court of record, or in any authentic writing.
These are the modes of voluntary recognition of natural children.
In case there is no evidence on the voluntary recognition of the spurious child, then his filiation may be established by means of the
circumstances or grounds for compulsory recognition prescribed in the aforementioned articles 283 and 284.
The prescriptive period for filing the action for compulsory recognition in the case of natural children, as provided for in article 285 of the Civil
Code, applies to spurious children.[22] (Citations omitted, italics supplied)
Thus, under the Civil Code, natural children have superior successional rights over spurious ones. However, Rovira treats them
as equals with respect to other rights, including the right to recognition granted by Article 285.
[23]

To emphasize, illegitimate children who were still minors at the time the Family Code took effect and whose putative parent died
during their minority are thus given the right to seek recognition (under Article 285 of the Civil Code) for a period of up to four years
from attaining majority age. This vested right was not impaired or taken away by the passage of the Family Code.
Indeed, our overriding consideration is to protect the vested rights of minors who could not have filed suit, on their own, during
the lifetime of their putative parents. As respondent aptly points out in his Memorandum, the State as parens patriae should protect
a minors right. Born in 1981, Adrian was only seven years old when the Family Code took effect and only twelve when his alleged
father died in 1993. The minor must be given his day in court.
[24]

Third Issue: Failure to Implead the CA


Under Section 4(a) of Rule 45 of the current Rules of Court, it is no longer required to implead the lower courts or judges
x x x either as petitioners or respondents. Under Section 3, however, the lower tribunal should still be furnished a copy of the
petition. Hence, the failure of petitioner to implead the Court of Appeals as a party is not a reversible error; it is in fact the correct
procedure.
WHEREFORE, the Petition is hereby DENIED and the assailed Decision and Resolution AFFIRMED. Costs against petitioner.
SO ORDERED.

G.R. No. 124853 February 24, 1998


FRANCISCO L. JISON, petitioner,
vs.
COURT OF APPEALS and MONINA JISON, respondents.

DAVIDE, JR., J.:


This is a petition for review under Rule 45 of the Rules of Court of the 27 April 1995 decision of the Court of Appeals (CA) in CAG.R. CV No. 32860 1 which reversed the decision of Branch 24 of the Regional Trial Court (RTC) of Iloilo City in Civil Case No. 16373. 2 The latter dismissed the complaint
of private respondent Monina Jison (hereafter MONINA) for recognition as an illegitimate child of petitioner Francisco Jison (hereafter FRANCISCO).

In issue is whether or not public respondent Court of Appeals committed reversible error, which, in this instance, necessitates an
inquiry into the facts. While as a general rule, factual issues are not within the province of this Court, nevertheless, in light of the
conflicting findings of facts of the trial court and the Court of Appeals, this case falls under an exception to this rule? 3
In her complaint 4 filed with the RTC on 13 March 1985, MONINA alleged that FRANCISCO had been married to a certain Lilia Lopez Jison since 1940. At the end of 1945 or
the start of 1946, however, FRANCISCO impregnated Esperanza F. Amolar (who was then employed as the nanny of FRANCISCO's daughter, Lourdes). As a result, MONINA was
born on 6 August 1946, in Dingle, Iloilo, and since childhood, had enjoyed the continuous, implied recognition as an illegitimate child of FRANCISCO by his acts and that of his
family. MONINA further alleged that FRANCISCO gave her support and spent for her education, such that she obtained a Master's degree, became a certified public accountant
(CPA) and eventually, a Central Bank examiner. In view of FRANCISCO's refusal to expressly recognize her, MONINA prayed for a judicial declaration of her illegitimate status and
that FRANCISCO support and treat her as such.

In his answer, 5 FRANCISCO alleged that he could not have had sexual relations with Esperanza Amolar during the period specified in the complaint as she had ceased to be
in his employ as early as 1944, and did not know of her whereabouts since then; further, he never recognized MONINA, expressly or impliedly, as his illegitimate child. As
affirmative and special defenses, FRANCISCO contended that MONINA had no right or cause of action against him and that her action was barred by estoppel, laches and/or
prescription. He thus prayed for dismissal of the complaint and an award of damages due to the malicious filing of the complaint.

After MONINA filed her reply, 6 pre-trial was conducted where the parties stipulated on the following issues:
1. Did Francisco Jison have any sexual relation[s] with Esperanza Am[o]lar about the end of 1945 or the start of 1946?
2. Is Monina Jison the recognized illegitimate daughter of Francisco Jison by the latter's own acts and those of his family?
3. Is Monina Jison barred from instituting or prosecuting the present action by estoppel, laches and/or prescription?
4. Damages. 7
At trial on the merits, MONINA presented a total of eleven (11) witnesses, namely: herself, Ruben Castellanes, Sr., Adela
Casabuena, Arsenio Duatin, Zafiro Ledesma, Danthea Lopez, Romeo Bilbao, Rudy Tiangson, Alfredo Baylosis, Dominador
Zavariz and Lope Amolar.
Ruben Castellanes, Sr., a 63-year old resident of Iloilo City, testified that he had worked for FRANCISCO for a total of six (6)
years at Nelly Garden, FRANCISCO's Iloilo residence. Towards the end of the Japanese occupation, FRANCISCO's wife
suffered a miscarriage or abortion, thereby depriving FRANCISCO of consortium; thereafter, FRANCISCO's wife managed a
nightclub on the ground floor of Nelly Garden which operated daily from 6:00 p.m. till 3:00 a.m. of the following day, thereby
allowing FRANCISCO free access to MONINA's mother, Esperanza Amolar, who was nicknamed Pansay.
Adela Casabuena, a 61-year old farmer, testified that she served as the yaya ("nanny") of Lourdes from July 1946 up to
February 1947. Although Pansay had left Nelly Garden two (2) weeks before Adela started working for the Jisons, Pansay
returned sometime in September 1946, or about one month after she gave birth to MONINA, to ask FRANCISCO for support.
As a result, Pansay and Lilia Jison, FRANCISCO's wife, quarreled in the living room, and in the course thereof, Pansay claimed
that FRANCISCO was the father of her baby. To which, Lilia replied: "I did not tell you to make that baby so it is your fault."
During the quarrel which lasted from 10:30 till 11:00 a.m., FRANCISCO was supposedly inside the house listening.
Arsenio Duatin, a 77-year old retired laborer, testified that from 1947 until 1977, he worked as FRANCISCO's houseboy at the
latter's house on 12th Street, Capitol Subdivision, Bacolod City. Arsenio met MONINA in 1967, when Felipe Lagarto, the
bookkeeper at Nelly Garden, informed Arsenio that MONINA, FRANCISCO's daughter, would arrive at Bacolod City with a letter
of introduction from Lagarto.
Initially, Arsenio identified seven (7) black-and-white photographs (Exhs. X-5 to X-11) of MONINA, 8 and as he paid for the telephone bills, he
likewise identified six (6) telephone cards (Exhs. G to L). Arsenio then declared that when MONINA arrived in Bacolod City, she introduced herself to him as FRANCISCO's
daughter. She stayed at FRANCISCO's house, but when the latter and his wife would come over, Arsenio would "conceal the presence of MONINA because Mrs. Jison did not like
to see her face." Once, Arsenio hid MONINA in the house of FRANCISCO's sister, Mrs. Luisa Jison Alano, in Silay City; another time, at the residence of FRANCISCO's cousin,

Mrs. Concha Lopez Cuaycong. Finally, Arsenio declared that the last time he saw MONINA was when she left for Manila, after having finished her schooling at La Salle College in
Bacolod City.

On re-direct and upon questions by the court, Arsenio disclosed that it was FRANCISCO who instructed that MONINA be
hidden whenever FRANCISCO and his wife were around; that although FRANCISCO and MONINA saw each other at the
Bacolod house only once, they called each other "through long distance;" and that MONINA addressed FRANCISCO as
"Daddy" during their lone meeting at the Bacolod house and were "affectionate" to each other. Arsenio likewise declared that
MONINA stayed at FRANCISCO's Bacolod house twice: first for a month, then for about a week the second time. On both
occasions, however, FRANCISCO and his wife were abroad. Finally, Arsenio recalled that FRANCISCO likewise bade Arsenio
to treat MONINA like his (FRANCISCO's) other daughters.
The testimony of Zafiro Ledesma, a 74-year old banker and former mayor of Iloilo City, initially touched on how he and his wife
were related to FRANCISCO, FRANCISCO's wife and MONINA. Zafiro first identified Exhibit R, a diagram of the family trees of
the Jison and Lopez families, which showed that former Vice-President Fernando Lopez was the first cousin of FRANCISCO's
wife, then told the court that the family of Vice-President Lopez treated MONINA "very well because she is considered a
relative . . . by reputation, by actual perception." Zafiro likewise identified Exhibits X-13 to X-18, photographs taken at the 14
April 1985 birthday celebration of Mrs. Fernando Lopez, which showed MONINA with the former Vice-President and other
members of the Lopez family.
Zafiro further testified that while MONINA lived with Mrs. Cuaycong, the latter paid for some of MONINA's school needs and
even asked MONINA to work in a hospital owned by Mrs. Cuaycong; and that another first cousin of FRANCISCO's wife, a
certain Remedios Lopez Franco, likewise helped MONINA with her studies and problems, and even attended MONINA's
graduation in 1978 when she obtained a masteral degree in Business Administration, as evidenced by another photograph
(Exh. X-12). Moreover, upon Remedios' recommendation, MONINA was employed as a secretary at Merchant Financing
Company, which was managed by a certain Danthea Lopez, the wife of another first cousin of FRANCISCO's wife, and among
whose directors were Zafiro himself, his wife and Danthea's husband. In closing, Zafiro identified MONINA's Social Security
Record (Exh. W), which was signed by Danthea as employer and where MONINA designated Remedios as the beneficiary.
Danthea Lopez, a 58-year old housekeeper, declared that FRANCISCO was the first cousin of her husband, Eusebio D. Lopez;
and that she came to know MONINA in the latter part of 1965 when Remedios Franco recommended MONINA for employment
at Merchant Financing Co., which Danthea managed at that time. Remedios introduced MONINA to Danthea "as being
reputedly the daughter of Mr. Frank Jison;" and on several occasions thereafter, Remedios made Danthea and the latter's
husband understand that MONINA was "reputedly the daughter of [FRANCISCO]" While MONINA worked at Merchant
Financing, Danthea knew that MONINA lived with Remedios; however, in the latter part of 1966, as Remedios left for Manila
and MONINA was still studying at San Agustin University, Danthea and her husband invited MONINA to live with them. During
MONINA's 6-month stay with them, she was not charged for board and lodging and was treated as a relative, not a mere
employee, all owing to what Remedios had said regarding MONINA's filiation. As Danthea understood, MONINA resigned from
Merchant Financing as she was called by Mrs. Cuaycong, a first cousin of Danthea's husband who lived in Bacolod City.
Romeo Bilbao, a 43-year old seaman, testified that he had worked for FRANCISCO from 1969 up to 1980 at Nelly Garden in
various capacities: as a procurement officer, hacienda overseer and, later, as hacienda administrator. Sometime in May, 1971,
Romeo saw and heard MONINA ask "her Daddy" (meaning FRANCISCO) for the money he promised to give her, but
FRANCISCO answered that he did not have the money to give, then told MONINA to go see Mr. Jose Cruz in Bacolod City.
Then in the middle of September that year, FRANCISCO told Romeo to pick up Mr. Cruz at the Iloilo pier and bring him to the
office of Atty. Benjamin Tirol. At said office, Atty. Tirol, Mr. Cruz and MONINA entered a room while Romeo waited outside. When
they came out, Atty. Tirol had papers for MONINA to sign, but she refused. Atty. Tirol said that a check would be released to
MONINA if she signed the papers, so MONINA acceded, although Atty. Tirol intended not to give MONINA a copy of the
document she signed. Thereafter, Mr. Cruz gave MONINA a check (Exh. Q), then MONINA grabbed a copy of the document she
signed and ran outside. Romeo then brought Mr. Cruz to Nelly Garden. As to his motive for testifying, Romeo stated that he
wanted to help MONINA be recognized as FRANCISCO'S daughter.
Rudy Tingson, a 45-year old antique dealer, testified that in 1963-1964, he was employed by FRANCISCO's wife at the Baguio
Military Institute in Baguio City; then in 1965, Rudy worked at FRANCISCO's office at Nelly Garden recording hacienda
expenses, typing vouchers and office papers, and, at times, acting as paymaster for the haciendas. From the nature of his work,
Rudy knew the persons receiving money from FRANCISCO's office, and clearly remembered that in 1965, as part of his job,
Rudy gave MONINA her allowance from FRANCISCO four (4) times, upon instructions of a certain Mr. Lagarto to give MONINA
P15.00 a month. Rudy likewise recalled that he first met MONINA in 1965, and that she would go to Nelly Garden whenever
FRANCISCO's wife was not around. On some of these occasions, MONINA would speak with and address FRANCISCO as
"Daddy," without objection from FRANCISCO. In fact, in 1965, Rudy saw FRANCISCO give MONINA money thrice. Rudy further
declared that in April 1965, FRANCISCO's office paid P250.00 to Funeraria Bernal for the funeral expenses of MONINA's
mother. Finally, as to Rudy's motives for testifying, he told the court that he simply wanted to held bring out the truth "and
nothing but the truth," and that MONINA's filiation was common knowledge among the people in the office at Nelly Garden.
On re-direct, Rudy declared that the moneys given by FRANCISCO's office to MONINA were not reflected in the books of the
office, but were kept in a separate book, as Mr. Lagarto explained that FRANCISCO's wife and children "should not know [of]
this." Rudy further revealed that as to the garden "meetings" between FRANCISCO and MONINA, Rudy saw MONINA kiss
FRANCISCO on the cheek both upon arriving and before leaving, and FRANCISCO's reaction upon seeing her was to smile
and say in the Visayan dialect: "Kamusta ka iha?" ("How are you, daughter?"); and that MONINA was free to go inside the
house as the household staff knew of her filiation, and that, sometimes, MONINA would join them for lunch.
Alfredo Baylosis, a 62-year old retired accountant, testified that he worked for FRANCISCO at Central Santos-Lopez in Iloilo
from 1951 up to 1961, then at Nelly Garden from 1961 until 1972. Alfredo first served FRANCISCO as a bookkeeper, then when
Mr. Lagarto died in 1967 or 1969, Alfredo replaced Mr. Lagarto as office manager.
Alfredo knew MONINA since 1961 as she used to go to Nelly Garden to claim her P15.00 monthly allowance given upon
FRANCISCO's standing order. Alfredo further declared that MONINA's filiation was pretty well-known in the office, that he had
seen MONINA and FRANCISCO go from the main building to the office, with FRANCISCO's arm on MONINA's shoulder; and
that the office paid for the burial expenses of Pansay, but this was not recorded in the books in order to hide it from
FRANCISCO's wife. Alfredo also disclosed that the disbursements for MONINA's allowance started in 1961 and were recorded
in a separate cash book. In 1967, the allowances ceased when MONINA stopped schooling and was employed in Bacolod City
with Miller, Cruz & Co., which served as FRANCISCO's accountant-auditor. Once, when Alfredo went to the offices of Miller,
Cruz & Co. to see the manager, Mr. Atienza, and arrange for the preparation of FRANCISCO's income tax return, Alfredo
chanced upon MONINA. When Alfredo asked her how she came to work there, she answered that "her Daddy," FRANCISCO,
recommended her, a fact confirmed by Mr. Atienza Alfredo then claimed that Mr. Jose Cruz, a partner at Miller, Cruz & Co., was
the most trusted man of FRANCISCO.
Dominador Savariz, a 55-year old caretaker, testified that he worked as FRANCISCO's houseboy at Nelly Garden from
November 1953 up to 1965. One morning in April 1954, MONINA and her mother Pansay went to Nelly Garden and spoke with
FRANCISCO for about an hour, during which time, Dominador was vacuuming the carpet about six (6) to seven (7) meters
away. Due to the noise of the vacuum cleaner, FRANCISCO and MONINA spoke in loud voices, thus Dominador overheard
their conversation. As FRANCISCO asked Pansay why they came, Pansay answered that they came to ask for the
"sustenance" of his child MONINA. FRANCISCO then touched MONINA's head and asked: "How are you Hija?," to which
MONINA answered: "Good morning, Daddy." After FRANCISCO told Pansay and MONINA to wait, he pulled something from his
wallet and said to Pansay. "I am giving this for a child."

In May 1954, Dominador saw MONINA at Mr. Lagarto's office where Dominador was to get "the day's expenses," while
MONINA was claiming her allowance from Mr. Diasnes. The next month, Dominador saw MONINA at Nelly Garden and heard in
the office that MONINA was there to get her allowance "from her Daddy." In December 1960, Dominador saw MONINA at Nelly
Garden, in the room of Don Vicente (father of FRANCISCO's wife), where she asked for a Christmas gift "and she was calling
Don Vicente, Lolo (grandfather)." At that time, FRANCISCO and his wife were not around. Then sometime in 1961, when
Dominador went to Mr. Legarto's office to get the marketing expenses, Dominador saw MONINA once more claiming her
allowance.
Dominador further testified that in February 1966, after he had stopped working for FRANCISCO, Dominador was at Mrs.
Franco's residence as she recommended him for employment with her sister, Mrs. Concha Cuaycong. There, he saw MONINA,
who was then about 15 years old, together with Mrs. Franco's daughter and son. Mrs. Franco pointed at MONINA and asked
Dominador if he knew who MONINA was. Dominador answered that MONINA was FRANCISCO's daughter with Pansay, and
then Mrs. Franco remarked that MONINA was staying with her (Mrs. Franco) and that she was sending MONINA to school at
the University of San Agustin.
Lope Amolar, a 50-year old resident of Dingle, Iloilo, and the younger brother of Esperanza Amolar (Pansay), testified that he
worked for FRANCISCO as a houseboy from March to November 1945 at Nelly Garden. Thereafter, FRANCISCO sent Lope to
work at Elena Apartments in Manila. By November 1945, Pansay was also working at Elena Apartments, where she revealed to
Lope that FRANCISCO impregnated her. Lope then confronted FRANCISCO, who told Lope "don't get hurt and don't cause any
trouble, because I am willing to support your Inday Pansay and my child." Three (3) days after this confrontation, Lope asked for
and received permission from FRANCISCO to resign because he (Lope) was hurt.
On 21 October 1986, MONINA herself took the witness stand. At that time, she was 40 years old and a Central Bank Examiner.
She affirmed that as evidenced by certifications from the Office of the Local Civil Registrar (Exhs. E and F) and baptismal
certificates (Exhs. C and D), she was born on 6 August 1946 in Barangay Tabugon, Dingle, Iloilo, to Esperanza Amolar (who
passed away on 20 April 1965) and FRANCISCO. 9 MONINA first studied at Sagrado where she stayed as a boarder. While at Sagrado from 1952 until 1955
(up to Grade 4), her father, FRANCISCO, paid for her tuition fees and other school expenses. She either received the money from FRANCISCO or from Mr. Lagarto, or saw
FRANCISCO give money to her mother, or Mr. Lagarto would pay Sagrado directly. After Sagrado, MONINA studied in different schools, 10 but FRANCISCO continuously answered
for her schooling.

For her college education, MONINA enrolled at the University of Iloilo, but she later dropped due to an accident which required a
week's hospitalization. Although FRANCISCO paid for part of the hospitalization expenses, her mother shouldered most of
them. In 1963, she enrolled at the University of San Agustin, where she stayed with Mrs. Franco who paid for MONINA's tuition
fees. However, expenses for books, school supplies, uniforms and the like were shouldered by FRANCISCO. At the start of
each semester, MONINA would show FRANCISCO that she was enrolled, then he would ask her to canvass prices, then give
her the money she needed. After finishing two (2) semesters at University of San Agustin, as evidenced by her transcript of
records (Exh. Z showing the FRANCISCO was listed as Parent/Guardian [Exh. Z-1], she transferred to "De Paul College," just
in front of Mrs. Franco's house, and studied there for a year. Thereafter, MONINA enrolled at Western Institute of Technology
(WIT), where she obtained a bachelor's degree in Commerce in April 1967. During her senior year, she stayed with Eusebio and
Danthea Lopez at Hotel Kahirup, owned by said couple. She passed the CPA board exams in 1974, and took up an M.B.A. at
De La Salle University as evidenced by her transcript (Exh. AA), wherein FRANCISCO was likewise listed as "Guardian" (Exhs.
AA-1 and AA-2).
MONINA enumerated the different members of the household staff at Nelly Garden, to wit: Luz, the household cook; the
houseboys Silvestre and Doming; the housemaid Natang; the yaya of the adopted triplets, Deling; the yaya of Lolo Vicente,
Adelina; and others. MONINA likewise enumerated the members of the office staff (Messrs. Baylosis, Lagarto, Tingson,
Diasnes, Jalandoni, Supertisioso, Doroy, and other), and identified them from a photograph marked as Exhibit X-2. She then
corroborated the prior testimony regarding her employment at Merchant Financing Co., and her having lived at Hotel Kahirup
and at Mrs. Cuaycong's residence in Bacolod City, while working at the hospital owned by Mrs. Cuaycong.
MONINA further testified that in March 1968, she went to Manila and met FRANCISCO at Elena Apartments at the corner of
Romero and Salas Streets, Ermita. She told FRANCISCO that she was going for a vacation in Baguio City with Mrs. Franco's
mother, with whom she stayed up to June 1968. Upon her return from Baguio City, MONINA told FRANCISCO that she wanted
to work, so the latter arranged for her employment at Miller & Cruz in Bacolod City. MONINA went to Bacolod City, was
interviewed by Mr. Jose Cruz, a partner at Miller & Cruz, who told her she would start working first week of September, sans
examination. She resigned from Miller & Cruz in 1971 and lived with Mrs. Cuaycong at her Forbes Park residence in Makati.
MONINA went to see FRANCISCO, told him that she resigned and asked him for money to go to Spain, but FRANCISCO
refused as she could not speak Spanish and would not be able find a job. The two quarreled and FRANCISCO ordered a helper
to send MONINA out of the house. In the process, MONINA broke many glasses at the pantry and cut her hand, after which,
FRANCISCO hugged her, gave her medicine, calmed her down, asked her to return to Bacolod City and promised that he would
giver her the money.
MONINA returned to Bacolod City by plane, using a Filipinas Orient Airways plane ticket (Exh. M) which FRANCISCO gave.
She called Mr. Cruz, then Atty. Tirol, as instructed by Mr. Cruz. These calls were evidenced by PLDT long distance toll card
(Exhs. G to L), with annotations at the back reading; "charged and paid under the name of Frank L. Jison" and were signed by
Arsenio Duatin (Exhs. G-1 to L-1). PLDT issued a certification as to the veracity of the contents of the toll cards (Exh. BB).
Likewise introduced in evidence was a letter of introduction prepared by Mr. Cruz addressed to Atty. Tirol, on MONINA's behalf
(Exh. N).
MONINA also declared that Atty. Tirol then told her that she would have to go to Iloilo and sign a certain affidavit, before Mr.
Cruz would turn over the money promised by FRANCISCO. She went to Atty. Tirol's office in Iloilo, but after going over the draft
of the affidavit, refused to sign it as it stated that she was not FRANCISCO's daughter. She explained that all she had agreed
with FRANCISCO was that he would pay for her fare to go abroad, and that since she was a little girl, she knew about her
illegitimacy. She started crying, begged Atty. Tirol to change the affidavit, to which Atty. Tirol responded that he was also a father
and did not want this to happen to his children as they could not be blamed for being brought into the world. She then wrote a
letter (Exh. O) to FRANCISCO and sent it to the latter's Forbes Park residence (Bauhinia Place) by JRS courier service (Exhs.
O-5 to O-7). MONINA subsequently met FRANCISCO in Bacolod City where they discussed the affidavit which she refused to
sign. FRANCISCO told her that the affidavit was for his wife, that in case she heard about MONINA going abroad, the affidavit
would "keep her peace."
MONINA then narrated that the first time she went to Atty. Tirol's office, she was accompanied by one Atty. Fernando
Divinagracia, who advised her that the affidavit (Exh. P) 11 would "boomerang" against FRANCISCO "as it is contrary to law." MONINA returned to Bacolod
City, then met with Atty. Tirol once more to reiterate her plea, but Atty. Tirol did not relent. Thus, on the morning of 20 or 21 September 1971, she signed the affidavit as she was
jobless and needed the money to support herself and finish her studies. In exchange for signing the document, MONINA received a Bank of Asia check for P15,000.00 (Exh. Q),
which was less than the P25,000.00 which FRANCISCO allegedly promised to give. As Atty. Tirol seemed hesitant to give her a copy of the affidavit after notarizing it, MONINA
merely grabbed a copy and immediately left.

MONINA then prepared to travel abroad, for which purpose, she procured letters of introduction (Exhs. S and T) from a cousin,
Mike Alano (son of FRANCISCO's elder sister Luisa); and an uncle, Emilio Jison (FRANCISCO's elder brother), addressed to
another cousin, Beth Jison (Emilio's daughter), for Beth to assist MONINA. Exhibit S contained a statement (Exh. S-1) expressly
recognizing that MONINA was FRANCISCO's daughter. Ultimately though, MONINA decided not to go abroad, opting instead to
spend the proceeds of the P15,000.00 check for her CPA review, board exam and graduate studies. After finishing her graduate
studies, she again planned to travel abroad, for which reason, she obtained a letter of introduction from former Vice President
Fernando Lopez addressed to then United States Consul Vernon McAnnich (Exh. V).
As to other acts tending to show her filiation, MONINA related that on one occasion, as FRANCISCO's wife was going to arrive
at the latter's Bacolod City residence, FRANCISCO called Arsenio Duatin and instructed Arsenio to hide MONINA. Thus,

MONINA stayed with Mrs. Luisa Jison for the duration of the stay of FRANCISCO's wife. MONINA also claimed that she knew
Vice President Fernando Lopez and his wife, Mariquit, even before starting to go to school. Thus, MONINA asked for a
recommendation letter (Exh. U) from Mrs. Mariquit Lopez for possible employment with Mrs. Rosario Lopez Cooper, another
second cousin of FRANCISCO. In Exhibit U, Mrs. Lopez expressly recognized MONINA as FRANCISCO's daughter. As
additional proof of her close relationship with the family of Vice President Lopez, MONINA identified photographs taken at a
birthday celebration on 14 April 1985.
MONINA finally claimed that she knew the three (3) children of FRANCISCO by wife, namely, Lourdes, Francisco, Jr., (Junior)
and Elena, but MONINA had met only Lourdes and Junior. MONINA's testimony dealt lengthily on her dealings with Junior and
the two (2) occasions when she met with Lourdes. The last time MONINA saw FRANCISCO was in March 1979, when she
sought his blessings to get married.
In his defense, FRANCISCO offered his deposition taken before then Judge Romeo Callejo of the Regional Trial Court of
Manila, Branch 48. As additional witnesses, FRANCISCO presented Nonito Jalandoni, Teodoro Zulla, Iigo Supertisioso,
Lourdes Ledesma, Jose Cruz and Dolores Argenal.
FRANCISCO declared that Pansay's employment ceased as of October, 1 1944, and that while employed by him, Pansay
would sleep with the other female helpers on the first floor of his residence, while he, his wife and daughter slept in a room on
the second floor. At that time, his household staff was composed of three (3) female workers and two (2) male workers. After
Pansay left in October 1944, she never communicated with him again, neither did he know of her whereabouts. FRANCISCO
staunchly denied having had sexual relations with Pansay and disavowed any knowledge about MONINA's birth. In the same
vein, he denied having paid for MONINA's tuition fees, in person or otherwise, and asserted that he never knew that Mr. Lagarto
paid for these fees. Moreover, FRANCISCO could not believe that Lagarto would pay for these fees despite absence of
instructions or approval from FRANCISCO. He likewise categorically denied that he told anyone, be it Danthea Lopez, Zafiro
Ledesma, Concha Cuaycong or Remedios Franco, that MONINA was his daughter.
FRANCISCO also disclosed that upon his return from the United States in 1971, he fired Alfredo Baylosis upon discovering that
Alfredo had taken advantage of his position during the former's absence. FRANCISCO likewise fired Rudy Tingson and Romeo
Bilbao, but did not give the reasons therefor.
Finally, FRANCISCO denied knowledge of MONINA's long distance calls from his Bacolod residence; nevertheless, when he
subsequently discovered this, he fired certain people in his office for their failure to report this anomaly. As regards the caretaker
of his Bacolod residence, FRANCISCO explained that since MONINA lived at Mrs. Cuaycong's residence, the caretaker thought
that he could allow people who lived at the Cuaycong residence to use the facilities at his (FRANCISCO's) house.
Nonito Jalandoni, bookkeeper and paymaster at Nelly's Garden from 1963 up to 1974, then from 1980 up to 1986, the assistant
overseer of Hacienda Lopez, testified that he did not know MONINA; that he learned of her only in June 1988, when he was
informed by FRANCISCO that MONINA had sued him; and that he never saw MONINA at Nelly's Garden, neither did he know
of any instructions for anyone at Nelly's Garden to give money to MONINA.
Teodoro Zulla, FRANCISCO's bookkeeper and paymaster from 1951 up to 1986, testified that FRANCISCO dismissed Alfredo
Baylosis due to certain unspecified discrepancies; and that he never saw MONINA receive funds from either Mr. Lagarto or Mr.
Baylosis. Upon questions from the trial court, however, Teodoro admitted that he prepared vouchers for only one of
FRANCISCO's haciendas, and not vouchers pertaining to the latter's personal expenses.
Iigo Supertisioso testified that he worked for FRANCISCO at Nelly's Garden from 1964 up to 1984 as a field inspector,
paymaster, cashier and, eventually, officer-in-charge (OIC). He confirmed Alfredo Baylosis' dismissal due to these unspecified
irregularities, then denied that FRANCISCO ever ordered that MONINA be given her allowance. Likewise, Iigo never heard
FRANCISCO mention that MONINA was his (FRANCISCO's) daughter.
Lourdes Ledesma, FRANCISCO's daughter, testified that she saw (but did not know) MONINA at the Our Lady of Mercy
Hospital, on the occasion of the birth of Lourdes' first son, Mark. Over lunch one day, Lourdes' aunt casually introduced Lourdes
and MONINA to each other, but they were referred to only by their first names. Then sometime in 1983 or 1984, MONINA
allegedly went to Lourdes' house in Sta. Clara Subdivision requesting for a letter of introduction or referral as MONINA was then
job-hunting. However, Lourdes did not comply with the request.
Jose Cruz, a partner at Miller, Cruz & Co., testified that MONINA worked at Miller & Cruz from 1968 up to 1971, however, he did
not personally interview her before she was accepted for employment. Moreover, MONINA underwent the usual screening
procedure before being hired. Jose recalled that one of the accountants, a certain Mr. Atienza, reported that MONINA claimed to
be FRANCISCO's daughter. Jose then told Mr. Atienza to speak with MONINA and see if he (Mr. Atienza) could stop her from
spreading this rumor. Mr. Atienza reported that he spoke with MONINA, who told him that she planned to leave for the United
States and needed P20,000.00 for that purpose, and in exchange, she would sign a document disclaiming filiation with
FRANCISCO. Thus, Jose instructed Mr. Atienza to request that MONINA meet with Jose, and at that meeting, MONINA
confirmed Mr. Atienza's report. Jose then informed Atty. Tirol, FRANCISCO's personal lawyer, about the matter.
Atty. Tirol told Jose to send MONINA and her lawyer to his (Atty. Tirol's) office in Iloilo. Jose then wrote out a letter of introduction
for MONINA addressed to Atty. Tirol Jose relayed Atty. Tirol's message to MONINA through Mr. Atienza, then later, Atty. Tirol told
Jose to go to Iloilo with a clerk for P15,000.00 Jose complied, and at Atty. Tirol's office, Jose saw MONINA, Atty. Tirol and his
secretary reading some documents. MONINA then expressed her willingness to sign the document, sans revisions. Jose
alleged that he drew the P15,000.00 from his personal funds, subject to reimbursement from and due to an understanding with
FRANCISCO.
Dolores Argenal, a househelper at Nelly Garden from May 1944 up to May 1946, testified that she knew that Pansay was
Lourdes' nanny; that Lourdes slept in her parents' room; that she had not seen FRANCISCO give special treatment to Pansay;
that there was no "unusual relationship" between FRANCISCO and Pansay, and if there was any, Dolores would have easily
detected it since she slept in the same room as Pansay. Dolores further declared that whenever FRANCISCO's wife was out of
town, Pansay would bring Lourdes downstairs at nighttime, and that Pansay would not sleep in the room where FRANCISCO
slept. Finally, Dolores declared that Pansay stopped working for FRANCISCO and his wife in October, 1944.
The reception of evidence having been concluded, the parties filed their respective memoranda.
It need be recalled that Judge Catalino Castaeda, Jr. presided over trial up to 21 October 1986, thereby hearing only the
testimonies of MONINA's witnesses and about half of MONINA's testimony on direct examination. Judge Norberto E. Devera,
Jr. heard the rest of MONINA's testimony and those of FRANCISCO's witnesses.
In its decision of 12 November 1990 12 the trial court, through Judge Devera, dismissed the complaint with costs against MONINA. In the opening paragraph thereof,
it observed:

This is a complaint for recognition of an illegitimate child instituted by plaintiff Monina Jison against defendant Francisco
Jison. This complaint was filed on March 13, 1985 at the time when plaintiff, reckoned from her death of birth, was already
thirty-nine years old. Noteworthy also is the fact that it was instituted twenty years after the death of plaintiff's mother,
Esperanza Amolar. For the years between plaintiff's birth and Esperanza's death, no action of any kind was instituted
against defendant either by plaintiff, her mother Esperanza or the latter's parents. Neither had plaintiff brought such an
action against defendant immediately upon her mother's death on April 20, 1965, considering that she was then already
nineteen years old or, within a reasonable time thereafter. Twenty years more had to supervene before this complaint was
eventually instituted.

The trial court then proceeded to discuss the four issues stipulated at pre-trial, without, however, summarizing the testimonies of
the witnesses nor referring to the testimonies of the witnesses other than those mentioned in the discussion of the issues.
The trial court resolved the first issue in the negative, holding that it was improbable for witness Lope Amolar to have noticed
that Pansay was pregnant upon seeing her at the Elena Apartments in November 1945, since Pansay was then only in her first
month of pregnancy; that there was no positive assertion that "copulation did indeed take place between Francisco and
Esperanza;" and that MONINA's attempt to show opportunity on the part of FRANCISCO failed to consider "that there was also
the opportunity for copulation between Esperanza and one of the several domestic helpers admittedly also residing at Nelly's
Garden at that time." The RTC also ruled that the probative value of the birth and baptismal certificates of MONINA paled in light
of jurisprudence, especially when the misspellings therein were considered.
The trial court likewise resolved the second issue in the negative, finding that MONINA's evidence thereon "may either be one
of three categories, namely: hearsay evidence, incredulous evidence, or self-serving evidence." To the first category belonged
the testimonies of Adela Casabuena and Alfredo Baylosis, whose knowledge of MONINA's filiation was based, as to the former,
on "utterances of defendant's wife Lilia and Esperanza allegedly during the heat of their quarrel," while as to the latter, Alfredo's
conclusion was based "from the rumors going [around] that plaintiff is defendant's daughter, front his personal observation of
plaintiff's facial appearance which he compared with that of defendant's and from the way the two (plaintiff and defendant) acted
and treated each other on one occasion that he had then opportunity to closely observe them together." To the second category
belonged that of Dominador Savariz, as:
At each precise time that Esperanza allegedly visited Nelly's Garden and allegedly on those occasions when defendant's
wife, Lilia was in Manila, this witness was there and allegedly heard pieces of conversation between defendant and
Esperanza related to the paternity of the latter's child. . .
The RTC then placed MONINA's testimony regarding the acts of recognition accorded her by FRANCISCO's relatives under the
third category, since the latter were never presented as witnesses, for which reason the trial court excluded the letters from
FRANCISCO's relatives (Exhs. S to V).
As to the third issue, the trial court held that MONINA was not barred by prescription for it was of "the perception . . . that the
benefits of Article 268 accorded to legitimate children may be availed of or extended to illegitimate children in the same manner
as the Family Code has so provided;" or by laches, "which is [a] creation of equity applied only to bring equitable results,
and . . . addressed to the sound discretion of the court [and] the circumstances [here] would show that whether plaintiff filed this
case immediately upon the death of her mother Esperanza in 1965 or twenty years thereafter in 1985, . . . there seems to be no
inequitable result to defendant as related to the situation of plaintiff."
The RTC ruled, however, that MONINA was barred by estoppel by deed because of the affidavit (Exh. P/Exh. 2) which she
signed "when she was already twenty-five years, a professional and . . . under the able guidance of counsel."
Finally, the RTC denied FRANCISCO's claim for damages, finding that MONINA did not file the complaint with malice, she
having been "propelled by an honest belief, founded on probable cause."
MONINA seasonably appealed to the Court of Appeals (CA-G.R. CV No. 32860) and sought reversal of the trial court's decision
on the grounds that:
I
THE TRIAL COURT WAS ERRONEOUSLY PREDISPOSED TO ADJUDGE THIS CASE AGAINST APPELLANT DUE TO
ITS MISPERCEPTION THAT APPELLANT'S DELAY IN FILING HER COMPLAINT WAS FATAL TO HER CASE.
II
THE TRIAL COURT ERRED IN ITS REJECTION OF THE TESTIMONIES OF APPELLANT'S WITNESSES AS TAILORMADE, INADEQUATE AND INCREDIBLE.
III
THE TRIAL COURT ERRED IN ITS REJECTION OF THE ADMISSIBILITY OF THE CERTIFIED COPIES OF PUBLIC
DOCUMENTS PRESENTED BY APPELLANT AS PART OF HER EVIDENCE.
IV
THE TRIAL COURT ERRED IN ITS REQUIREMENT THAT A WITNESS TO THE ACTUAL ACT O COPULATION
BETWEEN THE APPELLEE AND APPELLANT'S MOTHER SHOULD HAVE POSITIVELY TESTIFIED TO SAID EFFECT.
V
THE TRIAL COURT ERRED IN REJECTING THE ADMISSIBILITY OF THE DULY IDENTIFIED NOTES AND LETTER OF
THE RELATIVES OF THE APPELLEE AS HEARSAY.
VI
THE TRIAL COURT ERRED IN CONCLUDING THAT APPELLANT'S AFFIDAVIT (EXH. P) SERVED AS A BAR AGAINST
HER CLAIM FOR RECOGNITION INSTEAD OF REINFORCING SAID CLAIM. 13
Expectedly, FRANCISCO refuted these alleged errors in his Appellee's Brief. 14
In its decision of 27 April 1995, 15 the Court of Appeals initially declared that as no vested or acquired rights were affected, the instant case was governed by Article
175, in relation to Articles 172 and 173, of the Family Code. 16 While the Court of Appeals rejected the certifications issued by the Local Civil Registrar of Dingle, Iloilo (Exhs. E and
F) as FRANCISCO did not sign them, said court focused its discussion on the other means by which illegitimate filiation could be proved, i.e., the open and continuous possession
of the status of an illegitimate child or, by any other means allowed by the Rules of Court and special laws, such as "the baptismal certificate of the child, a judicial admission, a
family bible wherein the name of the child is entered, common reputation respecting pedigree, admission by silence, testimonies of witnesses . . ." 17 To the Court of Appeals, the
"bottom line issue" was whether or not MONINA established her filiation as FRANCISCO's illegitimate daughter by preponderance of evidence, as to which issue said court found:

[N]ot just preponderant but overwhelming evidence on record to prove that [MONINA] is the illegitimate daughter of
[FRANCISCO] and that she had continuously enjoyed such status by direct acts of [FRANCISCO] and/or his relatives.
In so ruling, the Court of Appeals observed that the testimonies of Lope Amolar, Adela Casabuena and Dominador Savariz were
already sufficient to establish MONINA's filiation:
As adverted to earlier, the trial court discredited Lope Amolar's testimony by saying that Lope could not have detected
Esperanza's pregnant state in November, 1945 since at that point in time [sic] she was still in the initial stage of
pregnancy. Apparently, the trial court paid more emphasis on the date mentioned by Lope Amolar than on the tenor and
import his testimony. As . . . Lope . . . was asked about an incident that transpired more than 41 years back, [u]nder the
circumstances, it is unreasonable to expect that Lope could still be dead right on the specific month in 1945 that [he] met
and confronted his sister. At any rate, what is important is not the month that they met but the essence of his testimony
that his sister pointed to their employer [FRANCISCO] as the one responsible for her pregnancy, and that upon being
confronted, [FRANCISCO] assured him of support for Esperanza and their child. It would appear then that in an attempt to
find fault with Lope's testimony, the trial court has fallen oblivious to the fact that even [FRANCISCO], in his deposition, did

not deny that he was confronted by Lope about what he had done to Esperanza during which he unequivocally
acknowledged paternity by assuring Lope of support for both Esperanza and their child.
The Court of Appeals further noted that Casabuena and Savariz "testified on something that they personally observed or
witnessed," which matters FRANCISCO "did not deny or refute." Finally, said court aptly held:
Taking into account all the foregoing uncontroverted testimonies
. . . let alone such circumstantial evidence as [MONINA's] Birth Certificates . . . and Baptismal Certificates which invariably
bear the name of [FRANCISCO] as her father, We cannot go along with the trial court's theory that [MONINA's] illegitimate
filiation has not been satisfactorily established.
xxx xxx xxx
Significantly, [MONINA's] testimony finds ample corroboration from [FRANCISCO's] former employees, Arsenio Duatin,
Rudy Tingson and Alfredo Baylosis. . . .
xxx xxx xxx
Carefully evaluating appellant's evidence on her enjoyment of the status of an illegitimate daughter of [FRANCISCO] visa-vis [FRANCISCO's] controversion thereof, We find more weight in the former. The positive testimonies of [MONINA] and
[her] witnesses . . . all bearing on [FRANCISCO's] acts and/or conduct indubitably showing that he had continuously
acknowledged [MONINA] as his illegitimate daughter have not been succeessfully [sic] refuted. In fact, [FRANCISCO]
himself, in his deposition, only casually dismissed [MONINA's] exhaustive and detailed testimony as untrue, and with
respect to those given by [MONINA's] witnesses, he merely explained that he had fired [them] from their employment.
Needless to state, [FRANCISCO's] vague denial is grossly inadequate to overcome the probative weight of [MONINA's]
testimonial evidence.
Even the affidavit (Exh 2) which [FRANCISCO] had foisted on the trial court . . . does not hold sway in the face of
[MONINA's] logical explanation that she at first did agree to sign the affidavit which contained untruthful statements. In
fact, she promptly complained to [FRANCISCO] who, however explained to her that the affidavit was only for the
consumption of his spouse . . . Further, the testimony of Jose Cruz concerning the events that led to the execution of the
affidavit . . . could not have been true, for as pointed out by [MONINA] she signed the affidavit . . . almost five months after
she had resigned from the Miller, Cruz & Co. . . .
At any rate, if [MONINA] were not his illegitimate daughter, it would have been uncalled for, if not absurd, for
[FRANCISCO] or his lawyer to have secured [MONINA's] sworn statement . . . On the contrary, in asking [MONINA] to
sign the said affidavit at the cost of P15,000. [FRANCISCO] clearly betrayed his intention to conceal or suppress his
paternity of [MONINA] . . .
In fine, We hold that [MONINA's] filiation as [FRANCISCO's] illegitimate daughter has been conclusively, established by
the uncontroverted testimonies of Lope Amolar, Adela Casabuena and Dominador Savariz to the effect that appellee
himself had admitted his paternity of the appellee, and also by the testimonies of appellant; Arsenio Duatin, Romeo Bilbao,
Rudy Tingson and Alfredo Baylosis unerringly demonstrating that by his own conduct or overt acts like sending appellant
to school, paying for her tuition fees, school uniforms, books, board and lodging at the Colegio del Sagrado Corazon de
Jesus, defraying appellant's hospitalization expenses, providing her with [a] monthly allowance, paying for the funeral
expenses of appellant's mother, acknowledging appellant's paternal greetings and calling appellant his "Hija" or child,
instructing his office personnel to give appellant's monthly allowance, recommending appellant for employment at the
Miller, Cruz & Co., allowing appellant to use his house in Bacolod and paying for her long distance telephone calls, having
appellant spend her vacation in his apartment in Manila and also at his Forbes residence, allowing appellant to use his
surname in her scholastic and other records (Exhs. Z, AA, AA-1, to AA-5, W & W-5), appellee had continuously recognized
appellant as his illegitimate daughter. Added to these are the acts of [FRANCISCO's] relatives acknowledging or treating
[MONINA] as [FRANCISCO's] daughter (Exh. U) or as their relative (Exhs. T & V). On this point, witness Zafiro Ledesma,
former Mayor of Iloilo City, whose spouse belongs to the Lopez clan just like [FRANCISCO], testified that [MONINA) has
been considered by the Lopezes as a relative. He identified pictures of the appellee in the company of the Lopezes (Exhs
X-16 & X-17). Another witness, Danthea H. Lopez, whose husband Eusebio Lopez is appellee's first cousin, testified that
appellant was introduced to her by appellee's cousin, Remedios Lopez Franco, as the daughter of appellee Francisco
Jison, for which reason, she took her in as [a] secretary in the Merchant's Financing Corporation of which she was the
manager, and further allowed her to stay with her family free of board and lodging. Still on this aspect, Dominador Savariz
declared that sometime in February, 1966 appellee's relative, Ms. Remedios Lopez Franco pointed to appellant as the
daughter of appellee Francisco Jison.
Finally, the Certifications of the Local Civil Registrar of Dingle (Exhs E and F) as well as [MONINA's] Baptismal
Certificates (Exhs C & D) which the trial court admitted in evidence as part of [MONINA's] testimony, may serve as
circumstantial evidence to further reinforce [MONINA's] claim that she is [FRANCISCO's] illegitimate daughter by
Esperanza Amolar.
True it is that a trial judge's assessment of the credibility of witnesses is accorded great respect on appeal. But the rule
admits of certain exceptions. One such exception is where the judge who rendered the judgment was not the one who
heard the witnesses testify. [citations omitted] The other is where the trial court had overlooked, misunderstood or
misappreciated some facts or circumstances of weight and substance which, if properly considered, might affect the result
of the case. [citations omitted] In the present case, both exceptions obtain. All of [MONINA's] witnesses . . . whose
testimonies were not given credence did not testify before the judge who rendered the disputed judgment . . .
The Court of Appeals then decreed:
WHEREFORE, premises considered, the judgment of the trial court is SET ASIDE and another one is hereby entered for
appellant Monina Jison, declaring her as the illegitimate daughter of appellee Francisco Jison, and entitled to all rights and
privileges granted by law.
Costs against appellee.
SO ORDERED.
His motion for reconsideration having been denied by the Court of Appeals in its resolution of 29 March 1996, 18FRANCISCO filed the
instant petition. He urges us to reverse the judgment of the Court of Appeals, alleging that said court committed errors of law:

I.
. . . IN REVERSING THE DECISION OF THE TRIAL COURT AND DECLARING PRIVATE RESPONDENT AS THE
ILLEGITIMATE CHILD OF PETITIONER, CONSIDERING [THE] IMPOSSIBILITY OF SEXUAL CONTACT BETWEEN
THE PETITIONER AND THE PRIVATE RESPONDENTS MOTHER AT THE TIME CONCEPTION WAS SUPPOSED TO
HAVE OCCURRED.
II.

. . . IN REVERSING THE TRIAL COURT'S FINDING CONSIDERING THAT PRIVATE RESPONDENTS TESTIMONIAL
EVIDENCE OF PATERNITY AND FILIATION IS NOT CLEAR AND CONVINCING.
III.
. . . IN GIVING CREDENCE TO DOCUMENTARY EVIDENCE PRESENTED BY THE PRIVATE RESPONDENT AS
EVIDENCE OF FILIATION CONSIDERING THAT THE SAME ARE HEARSAY, SELF-SERVING AND CANNOT BIND THE
PETITIONER UNDER THE BASIC RULES OF EVIDENCE.
IV.
. . . IN INTERPRETING THE PRIVATE RESPONDENTS SWORN STATEMENT (EXH. "P" /EXH. "2") IN A MANNER NOT
IN CONSONANCE WITH THE RULINGS OF THE HONORABLE SUPREME COURT.
V.
. . . IN NOT CONSIDERING THE LONG AND UNEXPLAINED DELAY IN THE FILING OF THE PRESENT PATERNITY
SUIT AS EQUIVALENT TO LACHES.
As regards the first error, FRANCISCO insists that taking into account the second paragraph of MONINA's complaint wherein
she claimed that he and Pansay had sexual relations "by about the end of 1945 or the start of 1946," it was physically
impossable for him and Pansay to have had sexual contact which resulted in MONINA's birth, considering that:
The normal period of human pregnancy is nine (9) months. If as claimed by private respondent in her complaint that her
mother was impregnated by FRANCISCO "at the end of 1945 or the start of 1946", she would have been born sometime
in late September or early October and not August 6, 1946 . . . The instant case finds factual and legal parallels
in Constantino vs. Mendez, 19 thus: . . .
FRANCISCO further claims that his testimony that Pansay was no longer employed by him at the time in question was
unrebutted, moreover, other men had access to Pansay during the time of or even after her employment by him.
As to the second error, FRANCISCO submits that MONINA's testimonial evidence is "shaky, contradictory and unreliable," and
proceeds to attack the credibility of her witnesses by claiming, in the main, that: (a) Lope Amolar could not have
detected Pansay pregnancy in November 1945 when they met since she would have been only one (1) month pregnant then;
(b) Dominador Savariz did not in fact witness the meeting between FRANCISCO, Pansay and MONINA; (c) Zafiro Ledesma had
an ulterior motive in testifying for MONINA as he owned a bank in Iloilo which was then under Central Bank supervision and
MONINA was the Bank Examiner assigned to Iloilo; and (d) Danthea Lopez was not related to him by blood and whatever
favorable treatment MONINA received from Danthea was due to the former's employment at Merchants' Financing Company
and additional services rendered at Kahirup Hotel; besides Danthea admitted that she had no personal knowledge as to the
issue of paternity and filiation of the contending parties, hence Sections 39 and 40 20 of Rule 130 of the Rules of Court did not come into play.
FRANCISCO likewise re-echoes the view of the trial court as regards the testimonies of Adela Casabuena and Alfredo Baylosis.

FRANCISCO further asserts that MONINA's testimony that he answered for her schooling was self-serving and uncorroborated
by any receipt or other documentary evidence; and assuming he did, such should be interpreted as a manifestation of kindness
shown towards the family of a former household helper.
Anent the treatment given by his relatives to MONINA as his daughter, FRANCISCO points to the fact that Pansay was the
former laundrywoman of Mrs. Franco; MONINA resided with the families of Eusebio Lopez and Concha Cuaycong because she
was in their employ at Kahirup Hotel and Our Lady of Mercy Hospital, respectively; MONINA failed to present Mrs. Franco,
Eusebio Lopez and Mrs. Cuaycong; and MONINA's employment at the accounting firm of Miller, Cruz & Co. was attributable to
her educational attainment, there being absolutely no evidence to prove that FRANCISCO ever facilitated her employment
thereat. Hence, in light of Baluyot v. Baluyot,21 the quantum of evidence to prove paternity by clear and convincing evidence, not merely a preponderance
thereof, was not met.

With respect to the third assigned error, FRANCISCO argues that the Court of Appeals' reliance on the certifications of the Local
Civil Registrar (Exhs. E and F) and Baptismal Certificates (Exhs. C and D) as circumstantial evidence is misplaced. First, their
genuineness could not be ascertained as the persons who issued them did not testify. Second, in light of Reyes v. Court of
Appeals, 22 the contents of the baptismal certificates were hearsay, as the data was based only on what was told to the priest who solemnized the baptism, who likewise was
not presented as a witness. Additionally, the name of the father appearing therein was "Franque Jison," which was not FRANCISCO's name. Third, in both Exhibits E and F, the
names of the child's parents were listed as "Frank Heson" and "Esperanza Amador" (not Amolar). FRANCISCO further points out that in Exhibit F, the status of the child is listen as
"legitimate," while the father's occupation as "laborer." Most importantly, there was no showing that FRANCISCO signed Exhibits E and F or that he was the one who reported the
child's birth to the Office of the Local Civil Registrar. As to MONINA's educational records, FRANCISCO invokes Baas v. Baas 23 which recognized that school records are
prepared by school authorities, not by putative parents, thus incompetent to prove paternity. And, as to the photographs presented by MONINA, FRANCISCO cites Colorado
v. Court of Appeals, 24 and further asserts that MONINA did not present any of the persons with whom she is seen in the pictures to testify thereon; besides these persons were, at
best, mere second cousins of FRANCISCO. He likewise assails the various notes and letters written by his relatives (Exhs. S to V) as they were not identified by the authors.
Finally, he stresses that MONINA did not testify as to the telephone cards (Exhs. G to L) nor did these reveal the circumstances surrounding the calls she made from his residence.

Anent the fourth assigned error, FRANCISCO contends that the Court of Appeals' interpretation of MONINA's affidavit of 21
September 1971 ran counter to Dequito v. Llamas, 25 and overlooked that at the time of execution, MONINA was more than 25 years old and assisted by
counsel.

As to the last assigned error, FRANCISCO bewails the Court of Appeals' failure to consider the long and unexplained delay in
the filing of the case.
In her comment, MONINA forcefully refuted FRANCISCO's arguments, leading FRANCISCO to file his reply thereto.
On 20 November 1996, we gave due course to this petition and required the parties to submit their respective memoranda,
which they subsequently did.
A painstaking review of the evidence and arguments fails to support petitioner.
Before addressing the merits of the controversy, we first dispose of preliminary matters relating to the applicable law and the
guiding principles in paternity suits. As to the former, plainly, the Family Code of the Philippines (Executive Order No. 209)
governs the present controversy. As correctly cited by the Court of Appeals, Uyguangco 26 served as a judicial confirmation of Article 256 of the
Family Code 27 regarding its retroactive effect unless there be impairment of vested rights, which does not hold true here, it appearing that neither the putative parent nor the child
has passed away and the former having actually resisted the latter's claim below.

Under Article 175 of the Family Code, illegitimate filiation, such as MONINA's, may be established in the same way and on the
same evidence as that of legitimate children. Article 172 thereof provides the various forms of evidence by which legitimate
filiation is established, thus:
Art. 172. The filiation of legitimate children is established by any of the following:
(1) The record of birth appearing in the civil register or a final judgment; or
(2) An admission of legitimate filiation in a public document or a private handwritten instrument signed by the
parent concerned.
In the absence of the foregoing evidence, the legitimate filiation shall be proved by:

(1) The open and continuous possession of the status of a legitimate child; or
(2) Any other means allowed by the Rules of Court and special laws.
This Article reproduces, with amendments, Articles 265, 266 and 267 of the Civil Code.
For the success of an action to establish illegitimate filiation under the second paragraph. which MONINA relies upon given that
she has none of the evidence mentioned in the first paragraph, a "high standard of proof" 28 is required. Specifically, to prove open and
continuous possession of the status of an illegitimate child, there must be evidence of the manifestation of the permanent intention of the supposed father to consider the child as
his, by continuous and clear manifestations of parental affection and care, which cannot be attributed to pure charity. Such acts must be of such a nature that they reveal not only
the conviction of paternity, but also the apparent desire to have and treat the child as such in all relations in society and in life, not accidentally, but continuously. 29

By "continuous" is meant uninterrupted and consistent, but does not require any particular length of time. 30
The foregoing standard of proof required to establish one's filiation is founded on the principle that an order for recognition and
support may create an unwholesome atmosphere or may be an irritant in the family or lives of the parties, so that it must be
issued only if paternity or filiation is established by clear and convincing evidence. 31
The foregoing discussion, however, must be situated within the general rules on evidence, in light of the burden of proof in civil
cases, i.e., preponderance of evidence, and the shifting of the burden of evidence in such cases. Simply put, he who alleges the
affirmative of the issue has the burden of proof, and upon the plaintiff in a civil case, the burden of proof never parts. However,
in the course of trial in a civil case, once plaintiff makes out aprima facie case in his favor, the duty or the burden of evidence
shifts to defendant to controvert plaintiff's prima facie case, otherwise, a verdict must be returned in favor of plaintiff. Moreover,
in civil cases, the party having the burden of proof must produce a preponderance of evidence thereon, with plaintiff having to
rely on the strength of his own evidence and not upon the weakness of the defendant's. The concept of "preponderance of
evidence" refers to evidence which is of greater weight, or more convincing, that which is offered in opposition to it; at bottom, it
means probability of truth. 32
With these in mind, we now proceed to resolve the merits of the instant controversy.
FRANCISCO's arguments in support of his first assigned error deserve scant consideration. While it has been observed that
unlawful intercourse will not be presumed merely from proof of an opportunity for such indulgence, 33 this does not favor FRANCISCO. Akin
to the crime of rape where, in most instances, the only witnesses to the felony are the participants in the sexual act themselves, in deciding paternity suits, the issue of whether
sexual intercourse actually occurred inevitably redounds to the victim's or mother's word, as against the accused's or putative father's protestations. In the instant case, MONINA's
mother could no longer testify as to the fact of intercourse, as she had, unfortunately, passed away long before the institution of the complaint for recognition. But this did not mean
that MONINA could no longer prove her filiation. The fact of her birth and her parentage may be established by evidence other than the testimony of her mother. The paramount
question then is whether MONINA's evidence is coherent, logical and natural. 34

The complaint stated that FRANCISCO had carnal knowledge of Pansay "by about the end of 1945." We agree with MONINA
that this was broad enough to cover the fourth quarter of said year, hence her birth on 6 August 1946 could still be attributed to
sexual relations between FRANCISCO and MONINA's mother. In any event, since it was established that her mother was still in
the employ of FRANCISCO at the time MONINA was conceived as determined by the date of her birth, sexual contact between
FRANCISCO and MONINA's mother was not at all impossible, especially in light of the overwhelming evidence, as hereafter
shown, that FRANCISCO fathered MONINA, has recognized her as his daughter and that MONINA has been enjoying the open
and continuous possession of the status as FRANCISCO's illegitimate daughter.
We readily conclude that the testimonial evidence offered by MONINA, woven by her narration of circumstances and events that
occurred through the years, concerning her relationship with FRANCISCO, coupled with the testimonies of her witnesses,
overwhelmingly established the following facts:
1) FRANCISCO is MONINA's father and she was conceived at the time when her mother was in the employ of the former;
2) FRANCISCO recognized MONINA as his child through his overt acts and conduct which the Court of Appeals took
pains to enumerate, thus:
[L]ike sending appellant to school, paying for her tuition fees, school uniforms, books, board and lodging at the
Colegio del Sagrado de Jesus, defraying appellant's hospitalization expenses, providing her with [a] monthly
allowance, paying for the funeral expenses of appellant's mother, acknowledging appellant's paternal
greetings and calling appellant his "Hija" or child, instructing his office personnel to give appellant's monthly
allowance, recommending appellant to use his house in Bacolod and paying for her long distance telephone
calls, having appellant spend her long distance telephone calls, having appellant spend her vacation in his
apartment in Manila and also at his Forbes residence, allowing appellant to use his surname in her scholastic
and other records (Exhs Z, AA, AA-1 to AA-5, W & W-5) . . .
3) Such recognition has been consistently shown and manifested throughout the years publicly,
an uninterrupted manner.

35

spontaneously, continuously and in

36

Accordingly, in light of the totality of the evidence on record, the second assigned error must fail.
There is some merit, however, in the third assigned error against the probative value of some of MONINA's documentary
evidence.
MONINA's reliance on the certification issued by the Local Civil Registrar concerning her birth (Exhs. E and F) is clearly
misplaced. It is settled that a certificate of live birth purportedly identifying the putative father is not competent evidence as to
the issue of paternity, when there is no showing that the putative father had a hand in the preparation of said certificates, and
the Local Civil Registrar is devoid of authority to record the paternity of an illegitimate child upon the information of a third
person. 37 Simply put, if the alleged father did not intervene in the birth certificate, e.g., supplying the information himself, the inscription of his name by the mother or doctor or
registrar is null and void; the mere certificate by the registrar without the signature of the father is not proof of voluntary acknowledgment on the latter's part. 38 In like manner,
FRANCISCO's lack of participation in the preparation of the baptismal certificates (Exhs. C and D) and school records (Exhs. Z and AA) renders these documents incompetent to
prove paternity, the former being competent merely to prove the administration of the sacrament of baptism on the date so specified. 39 However, despite the inadmissibility of the
school records per se to prove the paternity, they may be admitted as part of MONINA's testimony to corroborate her claim that FRANCISCO spent for her education.

We likewise disagree with the ruling of the Court of Appeals that the certificates issued by the Local Civil Registrar and the
baptismal certificates may be taken as circumstantial evidence to prove MONINA's filiation. Since they areper se inadmissible in
evidence as proof of such filiation, they cannot be admitted indirectly as circumstantial evidence to prove the same.
As to Exhibits "S," "T," "U" and "V," the various notes and letters written by FRANCISCO's relatives, namely Mike Alano, Emilio
Jison, Mariquit Lopez and Fernando Lopez, respectively, allegedly attesting to MONINA's filiation, while their due execution and
authenticity are not in issue, 40 as MONINA witnessed the authors signing the documents, nevertheless, under Rule 130, Section 39, the contents of these documents
may not be admitted, there being no showing that the declarants-authors were dead or unable to testify, neither was the relationship between the declarants and MONINA shown
by evidence other than the documents in question. 41 As to the admissibility of these documents under Rule 130, Section 40, however, this requires further elaboration.

Rule 130, Section 40, provides:


Sec. 40. Family reputation or tradition regarding pedigree. The reputation or tradition existing in a family previous to the
controversy, in respect to the pedigree of any one of its members, may be received in evidence if the witness testifying
thereon be also a member of the family, either by consanguinity or affinity.Entries in family bibles or other family books or
charts, engravings on rings, family portraits and the like may be received as evidence of pedigree. (emphasis supplied)

It is evident that this provision may be divided into two (2) parts: the portion containing the first underscored clause which
pertains to testimonial evidence, under which the documents in question may not be admitted as the authors thereof did not
take the witness stand; and the section containing the second underscored phrase. What must then be ascertained is whether
Exhibits S to V, as private documents, fall within the scope of the clause "and the like" as qualified by the preceding phrase
"[e]ntries in family bibles or other family books or charts, engravings on rights [and] family portraits,"
We hold that the scope of the enumeration contained in the second portion of this provision, in light of the rule ofejusdem
generis, is limited to objects which are commonly known as "family possessions," or those articles which represent, in effect, a
family's joint statement of its belief as to the pedigree of a person. 42 These have been described as objects "openly exhibited and well known to the
family," 43 or those "which, if preserved in a family, may be regarded as giving a family tradition." 44 Other examples of these objects which are regarded as reflective of a family's
reputation or tradition regarding pedigree are inscriptions on tombstones, 45 monuments or coffin plates. 46

Plainly then, Exhibits S to V, as private documents not constituting "family possessions" as discussed above, may not be
admitted on the basis of Rule 130, Section 40. Neither may these exhibits be admitted on the basis of Rule 130, Section 41
regarding common reputation, 47 it having been observed that:
[T]he weight of authority appears to be in favor of the theory that it is the general repute, the common reputation in the
family, and not the common reputation in community, that is a material element of evidence going to establish pedigree. . .
. [Thus] matters of pedigree may be proved by reputation in the family, and not by reputation in the neighborhood or
vicinity, except where the pedigree in question is marriage which may be proved by common reputation in the
community. 48
Their inadmissibility notwithstanding, Exhibits "S" to "V," inclusive, may, in like manner as MONINA's school records, properly be
admitted as part of her testimony to strengthen her claim that, indeed, relatives of FRANCISCO recognized her as his daughter.
We now direct our attention to MONINA's 21 September 1971 affidavit (Exh. P/Exh. 2), subject of the fourth assigned error,
where she attests that FRANCISCO is not her father. MONINA contends that she signed it under duress, i.e., she was jobless,
had no savings and needed the money to support herself and finish her studies. Moreover, she signed Exhibit P upon the
advice of Atty. Divinagracia that filiation could not be waived and that FRANCISCO's ploy would "boomerang" upon him. On the
other hand, FRANCISCO asserts that full credence should be afforded Exhibit P as MONINA was already 25 years old at the
time of its execution and was advised by counsel; further, being a notarized document, its genuineness and due execution could
not be questioned. He relies on the testimony of Jose Cruz, a partner at the accounting firm of Miller & Cruz, who declared that
he intervened in the matter as MONINA was spreading rumors about her filiation within the firm, which might have had
deleterious effects upon the relationship between the firm and FRANCISCO.
On this issue, we find for MONINA and agree with the following observations of the Court of Appeals:
Even the affidavit (Exh 2) which [FRANCISCO] had foisted on the trial court . . . does not hold sway in the face of
[MONINA's] logical explanation that she at first did agree to sign the affidavit which contained untruthful statements. In
fact, she promptly complained to [FRANCISCO] who, however explained to her that the affidavit was only for the
consumption of his spouse . . .
At any rate, if [MONINA] were not his illegitimate daughter, it would have been uncalled for, if not absurd, for
[FRANCISCO] of his lawyer to have secured [MONINA's] sworn statement . . . On the contrary, in asking [MONINA] to
sign the said affidavit at the cost of P15,000. [FRANCISCO] clearly betrayed his intention to conceal or suppress his
paternity of [MONINA] . . .
Indeed, if MONINA were truly not FRANCISCO's illegitimate daughter, it would have been unnecessary for him to have gone to
such great lengths in order that MONINA denounce her filiation. For as clearly established before the trial court and properly
appreciated by the Court of Appeals, MONINA had resigned from Miller & Cruz five (5) months prior to the execution of the
sworn statement in question, hence negating FRANCISCO's theory of the need to quash rumors circulating within Miller & Cruz
regarding the identity of MONINA's father. Hence, coupled with the assessment of the credibility of the testimonial evidence of
the parties discussed above, it is evident that the standard to contradict a notarial document, i.e. clear and convincing evidence
and more than merely preponderant, 49 has been met by MONINA
Plainly then, the burden of evidence fully shifted to FRANCISCO.
Two (2) glaring points in FRANCISCO's defense beg to be addressed: First, that his testimony was comprised of mere denials,
rife with bare, unsubstantiated responses such as "That is not true," "I do not believe that," or "None that I know." In declining
then to lend credence to FRANCISCO's testimony, we resort to a guiding principle in adjudging the credibility of a witness and
the truthfulness of his statements, laid down as early as 1921:
The experience of courts and the general observation of humanity teach us that the natural limitations of our inventive
faculties are such that if a witness undertakes to fabricate and deliver in court a false narrative containing numerous
details, he is almost certain to fall into fatal inconsistencies, to make statements which can be readily refuted, or to expose
in his demeanor the falsity of his message.
For this reason it will be found that perjurers usually confine themselves to the incidents immediately related to the
principal fact about which they testify, and when asked about collateral facts by which their truthfulness could be tested,
their answers not infrequently take the stereotyped form of such expressions as "I don't know" or "I don't remember." . . . 50
Second, the reasons for the dismissals of Tingson, Baylosis and Savariz were unspecified or likewise unsubstantiated, hence
FRANCISCO's attempt to prove ill-motive on their part to falsely testify in MONINA's favor may not succeed. As may be
gleaned, the only detail which FRANCISCO could furnish as to the circumstances surrounding the dismissals of his former
employees was that Baylosis allegedly "took advantage of his position" while FRANCISCO was in the United States. But aside
from this bare claim, FRANCISCO's account is barren, hence unable to provide the basis for a finding of bias against
FRANCISCO on the part of his former employees.
As to FRANCISCO's other witnesses, nothing substantial could be obtained either. Nonito Jalandoni avowed that he only came
to know of MONINA in June 1988; 51 that during his employment at Nelly Garden from 1963 up to 1974, he did not recall ever having seen MONINA there, neither did
he know of any instructions from FRANCISCO nor Mr. Lagarto (FRANCISCO's office manager before passing away) regarding the disbursement of MONINA's
allowance. 52 Teodoro Zulla corroborated Jalandoni's testimony regarding not having seen MONINA at Nelly Garden and MONINA's allowance; declared that Alfredo Baylosis was
dismissed due to discrepancies discovered after an audit, without any further elaboration, however; but admitted that he never prepared the vouchers pertaining to FRANCISCO's
personal expenses, merely those intended for one of FRANCISCO's haciendas. 53 Then, Iigo Superticioso confirmed that according to the report of a certain Mr. Atienza, Baylosis
"was dismissed by Mr. Jison for irregularities," while Superticioso was informed by FRANCISCO that Tingson was dismissed for loss of confidence. Superticioso likewise denied
that MONINA received money from FRANCISCO's office, neither was there a standing order from FRANCISCO to release funds to her. 54

It is at once obvious that the testimonies of these witnesses for FRANCISCO are likewise insufficient to overcome MONINA's
evidence. The former merely consist of denials as regards the latter's having gone to Nelly Garden or having received her
allowance from FRANCISCO's office, which, being in the form of negative testimony, necessarily stand infirm as against positive
testimony; 55 bare assertions as regards the dismissal of Baylosis; ignorance of FRANCISCO's personal expenses incapable of evincing that FRANCISCO did not provide
MONINA with an allowance; or hearsay evidence as regards the cause for the dismissals of Baylosis and Tingson. But what then serves as the coup de grace is that despite
Superticioso's claim that he did not know MONINA, 56 when confronted with Exhibit H, a telephone toll ticket indicating that on 18 May 1971, MONINA called a certain "Eing" at
FRANCISCO's office, Superticioso admitted that his nickname was "Iing" and that there was no other person named "Iing" in FRANCISCO's office. 57

All told, MONINA's evidence hurdled "the high standard of proof" required for the success of an action to establish one's
illegitimate filiation when relying upon the provisions regarding "open and continuous possession'' or "any other means allowed
by the Rules of Court and special laws;" moreover, MONINA proved her filiation by more than mere preponderance of evidence.

The last assigned error concerning laches likewise fails to convince. The essential elements of laches are: (1) conduct on the
part of the defendant, or of one under whom he claims, giving rise to the situation of which the complaint seeks a remedy; (2)
delay in asserting the complainant's rights, the complainant having had knowledge or notice of the defendant's conduct as
having been afforded an opportunity to institute a suit; (3) lack of knowledge or notice on the part of the defendant that the
complaint would assert the right in which he bases his suit; and (4) injury or prejudice to the defendant in the event relief is
accorded to the complaint, or the suit is not held barred. 58 The last element is the origin of the doctrine that sale demands apply only where by reason of the
lapse of time it would be inequitable to allow a party to enforce his legal rights. 59

As FRANCISCO set up, laches as an affirmative defense, it was incumbent upon him to prove the existence of its elements.
However, he only succeeded in showing MONINA's delay in asserting her claim, but miserably failed to prove the last element.
In any event, it must be stressed that laches is based upon grounds of public policy which requires, for the peace of society, the
discouragement of state claims, and is principally a question of the inequity or unfairness of permitting a right or claim to be
enforced or asserted. There is no absolute rule as to what constitutes laches; each case is to be determined according to its
particular circumstances. The question of laches is addressed to the sound discretion of the court, and since it is an equitable
doctrine, its application is controlled by equitable considerations. It cannot be worked to defeat justice or to perpetuate fraud and
injustice. 60 Since the instant case involves paternity and filiation, even if illegitimate, MONINA filed her action well within the period granted her by a positive provision of law. A
denial then of her action on ground of laches would clearly be inequitable and unjust.

WHEREFORE, IN VIEW OF THE FOREGOING, the petition is hereby DENIED and the challenged decision of the Court of
Appeals of 27 April 1995 in CA-G.R. CV No. 32860 is AFFIRMED.

JENIE SAN JUAN DELA


CRUZ and minor
CHRISTIAN DELA CRUZ
AQUINO, represented by
JENIE SAN JUAN DELA
CRUZ,
Petitioners,

G.R. No. 177728


Present:
QUISUMBING, J., Chairperson,
CARPIO MORALES,
CHICO-NAZARIO,
LEONARDO-DE CASTRO, and
PERALTA,* JJ.

versus
RONALD PAUL S. GRACIA, in his
Promulgated:
capacity as City Civil Registrar of
Antipolo City,
July 31, 2009
Respondent.
x------------------------------------------------x

DECISION
CARPIO MORALES, J.:
For several months in 2005, then 21-year old petitioner Jenie San Juan Dela Cruz (Jenie) and then 19-year old Christian Dominique
Sto. Tomas Aquino (Dominique) lived together as husband and wife without the benefit of marriage. They resided in the house of
Dominiques parents Domingo B. Aquino and Raquel Sto. Tomas Aquino at Pulang-lupa, Dulumbayan, Teresa, Rizal.
On September 4, 2005, Dominique died.[1] After almost two months, or on November 2, 2005, Jenie, who continued to live with
Dominiques parents, gave birth to her herein co-petitioner minor child Christian Dela Cruz Aquino at the Antipolo Doctors Hospital,
Antipolo City.
Jenie applied for registration of the childs birth, using Dominiques surname Aquino, with the Office of the City Civil
Registrar, Antipolo City, in support of which she submitted the childs Certificate of Live Birth,[2] Affidavit to Use the Surname of the
Father[3] (AUSF) which she had executed and signed, and Affidavit of Acknowledgment executed by Dominiques father Domingo Butch
Aquino.[4] Both affidavits attested, inter alia, that during the lifetime of Dominique, he had continuously acknowledged his yet unborn child,
and that his paternity had never been questioned. Jenie attached to the AUSF a document entitled AUTOBIOGRAPHY which Dominique,
during his lifetime, wrote in his own handwriting, the pertinent portions of which read:
AQUINO, CHRISTIAN DOMINIQUE S.T.
AUTOBIOGRAPHY
IM CHRISTIAN DOMINIQUE STO. TOMAS AQUINO, 19 YEARS OF AGE TURNING 20 THIS COMING OCTOBER
31, 2005.[5] I RESIDE AT PULANG-LUPA STREET BRGY. DULUMBAYAN, TERESA, RIZAL. I AM THE YOUNGEST IN
OUR FAMILY. I HAVE ONE BROTHER NAMED JOSEPH BUTCH STO. TOMAS AQUINO. MY FATHERS NAME IS
DOMINGO BUTCH AQUINO AND MY MOTHERS NAME IS RAQUEL STO. TOMAS AQUINO. x x x.
xxxx
AS OF NOW I HAVE MY WIFE NAMED JENIE DELA CRUZ. WE MET EACH OTHER IN OUR HOMETOWN,
TEREZA RIZAL. AT FIRST WE BECAME GOOD FRIENDS, THEN WE FELL IN LOVE WITH EACH OTHER, THEN WE
BECAME GOOD COUPLES. AND AS OF NOW SHE IS PREGNANT AND FOR THAT WE LIVE TOGETHER IN OUR
HOUSE NOW. THATS ALL.[6](Emphasis and underscoring supplied)

By letter dated November 11, 2005,[7] the City Civil Registrar of Antipolo City, Ronald Paul S. Gracia (respondent), deniedJenies
application for registration of the childs name in this wise:

7. Rule 7 of Administrative Order No. 1, Series of 2004 (Implementing Rules and Regulations of Republic Act No. 9255 [An Act
Allowing Illegitimate Children to Use the Surname of their Father, Amending for the Purpose, Article 176 of Executive Order
No. 209, otherwise Known as the Family Code of the Philippines]) provides that:
Rule 7. Requirements for the Child to Use the Surname of the Father
7.1 For Births Not Yet Registered
7.1.1 The illegitimate child shall use the surname of the father if a public document is executed by the father, either at the
back of the Certificate of Live Birth or in a separate document.
7.1.2 If admission of paternity is made through a private handwritten instrument, the child shall use the surname of the
father, provided the registration is supported by the following documents:
a. AUSF[8]
b. Consent of the child, if 18 years old and over at the time of the filing of the document.
c. Any two of the following documents showing clearly the paternity between the father and the child:
1. Employment records
2. SSS/GSIS records
3. Insurance
4. Certification of membership in any organization
5. Statement of Assets and Liability
6. Income Tax Return (ITR)
In summary, the child cannot use the surname of his father because he was born out of wedlock and the father unfortunately died
prior to his birth and has no more capacity to acknowledge his paternity to the child (either through the back of Municipal Form No.
102 Affidavit of Acknowledgment/Admission of Paternity or the Authority to Use the Surname of the Father). (Underscoring
supplied)

Jenie

and

the

child

promptly

filed

complaint [9] for

injunction/registration

of

name

against

respondent

before

the RegionalTrial Court of Antipolo City, docketed as SCA Case No. 06-539, which was raffled to Branch 73 thereof. The complaint alleged
that,inter alia, the denial of registration of the childs name is a violation of his right to use the surname of his deceased father under Article
176 of the Family Code, as amended by Republic Act (R.A.) No. 9255,[10] which provides:
Article 176. Illegitimate children shall use the surname and shall be under the parental authority of their mother, and shall be
entitled to support in conformity with this Code. However, illegitimate children may use the surname of their father if their filiation
has been expressly recognized by the father through the record of birth appearing in the civil register, or when an admission in
a public document or private handwritten instrument is made by the father. Provided, the father has the right to institute an
action before the regular courts to prove non-filiation during his lifetime. The legitime of each illegitimate child shall consist of onehalf of the legitime of a legitimate child. (Emphasis and underscoring supplied)

They maintained that the Autobiography executed by Dominique constitutes an admission of paternity in a private handwritten instrument
within the contemplation of the above-quoted provision of law.
For failure to file a responsive pleading or answer despite service of summons, respondent was declared in default.
Jenie thereupon presented evidence ex-parte. She testified on the circumstances of her common-law relationship with Dominique and
affirmed her declarations in her AUSF that during his lifetime, he had acknowledged his yet unborn child. [11] She offered Dominiques
handwritten Autobiography (Exhibit A) as her documentary evidence-in-chief. [12] Dominiques lone brother, Joseph Butch S.T. Aquino,
also testified, corroborating Jenies declarations.[13]
By Decision[14] of April 25, 2007, the trial court dismissed the complaint for lack of cause of action as the Autobiography
was unsigned, citing paragraph 2.2, Rule 2 (Definition of Terms) of Administrative Order (A.O.) No. 1, Series of 2004 (the Rules and
Regulations Governing the Implementation of R.A. 9255) which defines private handwritten document through which a father may
acknowledge an illegitimate child as follows:
2.2 Private handwritten instrument an instrument executed in the handwriting of the father and duly signed by him where
he expressly recognizes paternity to the child. (Underscoring supplied)

The trial court held that even if Dominique was the author of the handwritten Autobiography, the same does not contain any express
recognition of paternity.
Hence, this direct resort to the Court via Petition for Review on Certiorari raising this purely legal issue of:
WHETHER OR NOT THE UNSIGNED HANDWRITTEN STATEMENT OF THE DECEASED FATHER OF MINOR
CHRISTIAN DELA CRUZ CAN BE CONSIDERED AS A RECOGNITION OF PATERNITY IN A PRIVATE HANDWRITTEN
INSTRUMENT WITHIN THE CONTEMPLATION OF ARTICLE 176 OF THE FAMILY CODE, AS AMENDED BY R.A. 9255,
WHICH ENTITLES THE SAID MINOR TO USE HIS FATHERS SURNAME.[15] (Underscoring supplied)

Petitioners contend that Article 176 of the Family Code, as amended, does not expressly require that the private handwritten
instrument containing the putative fathers admission of paternity must be signed by him. They add that the deceaseds handwritten
Autobiography, though unsigned by him, is sufficient, for the requirement in the above-quoted paragraph 2.2 of the Administrative
Order that the admission/recognition must be duly signed by the father is void as it unduly expanded the earlier-quoted provision of
Article 176 of the Family Code.[16]

Petitioners further contend that the trial court erred in not finding that Dominiques handwritten Autobiography contains a clear and
unmistakable recognition of the childs paternity.[17]
In its Comment, the Office of the Solicitor General (OSG) submits that respondents position, as affirmed by the trial court, is in
consonance with the law and thus prays for the dismissal of the petition.

It further submits that Dominiques Autobiography merely

acknowledged Jenies pregnancy but not [his] paternity of the child she was carrying in her womb.[18]
Article 176 of the Family Code, as amended by R.A. 9255, permits an illegitimate child to use the surname of his/her father if the
latter had expressly recognized him/her as his offspring through the record of birth appearing in the civil register, or through anadmission
made in a public or private handwritten instrument. The recognition made in any of these documents is, in itself, a consummated act of
acknowledgment of the childs paternity; hence, no separate action for judicial approval is necessary.[19]
Article 176 of the Family Code, as amended, does not, indeed, explicitly state that the private handwritten instrument acknowledging
the childs paternity must be signed by the putative father. This provision must, however, be read in conjunction with related provisions of
the Family Code which require that recognition by the father must bear his signature, thus:
Art. 175. Illegitimate children may establish their illegitimate filiation in the same way and on the same evidence as legitimate
children.
xxxx
Art. 172. The filiation of legitimate children is established by any of the following:
(1) The record of birth appearing in the civil register or a final judgment; or
(2) An admission of legitimate filiation in a public document or a private handwritten instrument and signed by the
parent concerned.
x x x x (Emphasis and underscoring supplied)

That a father who acknowledges paternity of a child through a written instrument must affix his signature thereon is clearly implied
in Article 176 of the Family Code. Paragraph 2.2, Rule 2 of A.O. No. 1, Series of 2004, merely articulated such requirement; it did not
unduly expand the import of Article 176 as claimed by petitioners.
In the present case, however, special circumstances exist to hold that Dominiques Autobiography, though unsigned by
him,substantially satisfies the requirement of the law.
First, Dominique died about two months prior to the childs birth. Second, the relevant matters in the Autobiography, unquestionably
handwritten by Dominique, correspond to the facts culled from the testimonial evidence Jenie proffered. [20] Third, Jenies testimony
is corroborated by the Affidavit of Acknowledgment of Dominiques father Domingo Aquino and testimony of his brother Joseph Butch
Aquino whose hereditary rights could be affected by the registration of the questioned recognition of the child.These circumstances
indicating Dominiques paternity of the child give life to his statements in his Autobiography that JENIE DELA CRUZ is MY WIFE as
WE FELL IN LOVE WITH EACH OTHER and NOW SHE IS PREGNANT AND FOR THAT WE LIVE TOGETHER.
In Herrera v. Alba,[21] the Court summarized the laws, rules, and jurisprudence on establishing filiation, discoursing in relevant part:
Laws, Rules, and Jurisprudence
Establishing Filiation
The relevant provisions of the Family Code provide as follows:
ART. 175.
Illegitimate children may establish their illegitimate filiation in the same way and on the same evidence as
legitimate children.
xxxx
ART. 172.

The filiation of legitimate children is established by any of the following:

(1) The record of birth appearing in the civil register or a final judgment; or
(2) An admission of legitimate filiation in a public document or a private handwritten instrument and signed by the parent
concerned.
In the absence of the foregoing evidence, the legitimate filiation shall be proved by:
(1) The open and continuous possession of the status of a legitimate child; or
(2) Any other means allowed by the Rules of Court and special laws.
The Rules on Evidence include provisions on pedigree. The relevant sections of Rule 130 provide:
SEC. 39. Act or declaration about pedigree. The act or declaration of a person deceased, or unable to testify, in respect to
the pedigree of another person related to him by birth or marriage, may be received in evidence where it occurred before the
controversy, and the relationship between the two persons is shown by evidence other than such act or declaration. The word
"pedigree" includes relationship, family genealogy, birth, marriage, death, the dates when and the places where these facts occurred,
and the names of the relatives. It embraces also facts of family history intimately connected with pedigree.

SEC. 40.
Family reputation or tradition regarding pedigree. The reputation or tradition existing in a family previous
to the controversy, in respect to the pedigree of any one of its members, may be received in evidence if the witness testifying thereon
be also a member of the family, either by consanguinity or affinity. Entries in family bibles or other family books or charts,
engraving on rings, family portraits and the like, may be received as evidence of pedigree.
This Court's rulings further specify what incriminating acts are acceptable as evidence to establish filiation. In Pe Lim v. CA, a
case petitioner often cites, we stated that the issue of paternity still has to be resolved by such conventional evidence as the
relevant incriminating verbal and written acts by the putative father. Under Article 278 of the New Civil Code, voluntary
recognition by a parent shall be made in the record of birth, a will, a statement before a court of record, or in any authentic
writing. To be effective, the claim of filiation must be made by the putative father himself and the writing must be the writing
of the putative father. A notarial agreement to support a child whose filiation is admitted by the putative father was considered
acceptable evidence. Letters to the mother vowing to be a good father to the child and pictures of the putative father cuddling the
child on various occasions, together with the certificate of live birth, proved filiation. However, a student permanent record, a written
consent to a father's operation, or a marriage contract where the putative father gave consent, cannot be taken as authentic writing.
Standing alone, neither a certificate of baptism nor family pictures are sufficient to establish filiation. (Emphasis and underscoring
supplied.)

In the case at bar, there is no dispute that the earlier quoted statements in Dominiques Autobiography have been made and written by
him. Taken together with the other relevant facts extant herein that Dominique, during his lifetime, and Jenie were living together as
common-law spouses for several months in 2005 at his parents house in Pulang-lupa, Dulumbayan, Teresa, Rizal; she was pregnant when
Dominique died on September 4, 2005; and about two months after his death, Jenie gave birth to the child they sufficiently establish that
the child of Jenie is Dominiques.
In view of the pronouncements herein made, the Court sees it fit to adopt the following rules respecting the requirement of affixing the
signature of the acknowledging parent in any private handwritten instrument wherein an admission of filiation of a legitimate or illegitimate
child is made:
1)

Where the private handwritten instrument is the lone piece of evidence submitted to prove filiation, there should be strict

compliance with the requirement that the same must be signed by the acknowledging parent; and
2)

Where the private handwritten instrument is accompanied by other relevant and competent evidence, it suffices that the claim of

filiation therein be shown to have been made and handwritten by the acknowledging parent as it is merely corroborative of such other
evidence.
Our laws instruct that the welfare of the child shall be the paramount consideration in resolving questions affecting him. [22]Article
3(1) of the United Nations Convention on the Rights of a Child of which the Philippines is a signatory is similarly emphatic:
Article 3
1. In all actions concerning children, whether undertaken by public or private social welfare institutions, courts of law,
administrative authorities or legislative bodies, the best interests of the child shall be a primary consideration.[23] (Underscoring
supplied)

It is thus (t)he policy of the Family Code to liberalize the rule on the investigation of the paternity and filiation of children,especially
of illegitimate children x x x.[24] Too, (t)he State as parens patriae affords special protection to children from abuse, exploitation and other
conditions prejudicial to their development.[25]

In the eyes of society, a child with an unknown father bears the stigma of dishonor. It is to petitioner minor childs best interests to
allow him to bear the surname of the now deceased Dominique and enter it in his birth certificate.
WHEREFORE, the petition is GRANTED. The City Civil Registrar of Antipolo City is DIRECTED to immediately enterthe
surname of the late Christian Dominique Sto. Tomas Aquino as the surname of petitioner minor Christian dela Cruz in his Certificate of Live
Birth, and record the same in the Register of Births.

[G. R. No. 4275. March 23, 1909.]


PAULA CONDE, Plaintiff-Appellee, vs. ROMAN ABAYA, Defendant-Appellant.
DECISION
ARELLANO, C.J.:
From the hearing of the appeal interposed by Roman Abaya in the special proceedings brought in the Court of First Instance of
La Laguna for the settlement of the intestate estate and the distribution of the property of Casiano Abaya it appears: chanrobles
virtualawlibrary
I.
As antecedents: chanrobles virtualawlibrary that Casiano Abaya, unmarried, the son of Romualdo Abaya and Sabina
Labadia, died on the 6th of April 1899; that Paula Conde, as the mother of the natural children Jose and Teopista Conde, whom
she states she had by Casiano Abaya, on the 6th of November, 1905, moved the settlement of the said intestate succession;
that an administrator having been appointed for the said estate on the 25th of November, 1905, Roman Abaya, a son of the said
Romualdo Abaya and Sabina Labadia, the parents of the late Casiano Abaya, came forward and opposed said appointment and
claimed it for himself as being the nearest relative of the deceased; that this was granted by the court below on the 9th of
January, 1906; that on the 17th of November, 1906, Roman Abaya moved that, after due process of law, the court declare him
to be the sole heir of Casiano Abaya, to the exclusion of all other persons, especially of Paula Conde, and to be therefore
entitled to take possession of all the property of said estate, and that it be adjudicated to him; and that on November 22, 1906,
the court ordered the publication of notices for the declaration of heirs and distribution of the property of the estate.
II.
That on the 28th of November, 1906, Paula Conde, in reply to the foregoing motion of Roman Abaya, filed a petition
wherein she stated that she acknowledged the relationship alleged by Roman Abaya, but that she considered that her right was
superior to his and moved for a hearing of the matter, and, in consequence of the evidence that she intended to present she
prayed that she be declared to have preferential rights to the property left by Casiano Abaya, and that the same be adjudicated
to her together with the corresponding products thereof.
III.
That the trial was held, both parties presenting documentary and oral evidence, and the court below entered the
following judgment: chanrobles virtualawlibrary
That the administrator of the estate of Casiana Abaya should recognize Teopista and Jose Conde as being natural children of
Casiano Abaya; that the Petitioner Paula Conde should succeed to the hereditary rights of her children with respect to the
inheritance of their deceased natural father Casiano Abaya; and therefore, it is hereby declared that she is the only heir to the
property of the said intestate estate, to the exclusion of the administrator, Roman Abaya.
IV.
That Roman Abaya excepted to the foregoing judgment, appealed to this court, and presented the following statement
of errors: chanrobles virtualawlibrary
1.
The fact that the court below found that an ordinary action for the acknowledgment of natural children under articles
135 and 137 of the Civil Code, might be brought in special probate proceedings.
2.
The finding that after the death of a person claimed to be an unacknowledged natural child, the mother of such
presumed natural child, as heir to the latter, may bring an action to enforce the acknowledgment of her deceased child in
accordance with articles 135 and 137 of the Civil Code.
3.
The finding in the judgment that the alleged continuous possession of the deceased children of Paula Conde of the
status of natural children of the late Casiano Abaya, has been fully proven in these proceedings; and
4.
On the hypothesis that it was proper to adjudicate the property of this intestate estate to Paula Conde, as improperly
found by the court below, the court erred in not having declared that said property should be reserved in favor of relatives of
Casiano Abaya to the third degree, and in not having previously demanded securities from Paula Conde to guarantee the
transmission of the property to those who might fall within the reservation.
As to the first error assigned, the question is set up as to whether in special proceedings for the administration and distribution
of an intestate estate, an action might be brought to enforce the acknowledgment of the natural child of the person from whom
the inheritance is derived, that is to say, whether one might appear as heir on the ground that he is a recognized natural child of
the deceased, not having been so recognized by the deceased either voluntarily or compulsory by reason of a preexisting
judicial decision, but asking at the same time that, in the special proceeding itself, he be recognized by the presumed legitimate
heirs of the deceased who claim to be entitled to the succession opened in the special proceeding.
According to section 782 of the Code of Civil Procedure
If there shall be a controversy before the Court of First Instance as to who the lawful heirs of the deceased person are, or as to
the distributive share to which each person is entitled under the law, the testimony as to such controversy shall be taken in
writing by the judge, under oath and signed by witness. Any party in interest whose distributive share is affected by the
determination of such controversy, may appeal from the judgment of the Court of First Instance determining such controversy to
the Supreme Court, within the time and in the manner provided in the last preceding section.
This court has decided the present question in the manner shown in the case of Juana Pimental vs. Engracio Palanca (5 Phil.
Rep. 436.) cralaw
The main question with regard to the second error assigned, is whether or not the mother of a natural child now deceased, but
who survived the person who, it is claimed, was his natural father, also deceased, may bring an action for the acknowledgment
of the natural filiation in favor of such child in order to appear in his behalf to receive the inheritance from the person who is
supposed to be his natural father.

In order to decide in the affirmative the court below has assigned the following as the only foundation: chanrobles
virtualawlibrary
In resolving a similar question Manresa says: chanrobles virtualawlibrary An acknowledgment can only be demanded by the
natural child and his descendants whom it shall benefit, and should they be minors or otherwise incapacitated, such person as
legally represents them; the mother may ask it in behalf of her child so long as he is under her authority. On this point no
positive declaration has been made, undoubtedly because it was not considered necessary. A private action is in question and
the general rule must be followed. Elsewhere the same author adds: chanrobles virtualawlibrary It may so happen that the child
dies before four years have expired after attaining majority, or that the document supporting his petition for acknowledgment is
discovered after his death, such death perhaps occurring after his parents had died, as is supposed by article 137, or during
their lifetime. In any case such right of action shall pertain to the descendants of the child whom the acknowledgment may
interest. (See Commentaries to arts. 135 and 137, Civil Code. Vol. I.) cralaw
The above doctrine, advanced by one of the most eminent commentators of the Civil Code, lacks legal and doctrinal foundation.
The power to transmit the right of such action by the natural child to his descendants cannot be sustained under the law, and
still less to his mother.
It is without any support in law because the rule laid down in the code is most positive, limiting in form, when establishing the
exception for the exercise of such right of action after the death of the presumed parents, as is shown hereafter. It is not
supported by any doctrine, because up to the present time no argument has been presented, upon which even an approximate
conclusion could be based.
Although the Civil Code considerably improved the condition of recognized natural children, granting them rights and actions
that they did not possess under the former laws, they were not, however, placed upon the same plane as legitimate ones. The
difference that separates these two classes of children is still great, as proven by so many articles dealing with the rights of the
family and with succession in relation to the members thereof. It may be laid down as a legal maxim, that whatever the code
does not grant to the legitimate children, or in connection with their rights, must still less be understood as granted to recognized
natural children or in connection with their rights. There is not a single exception in its provisions.
If legitimacy is the attribute that constitutes the basis of the absolute family rights of the child, the acknowledgment of the natural
child is, among illegitimate ones, that which unites him to the family of the father or the mother who recognizes him, and affords
him a participation in the rights of the family, relatively advantageous according to whether they are alone or whether they
concur with other individuals of the family of his purely natural father or mother.
Thus, in order to consider the spirit of the Civil Code nothing is more logical than to establish a comparison between an action to
claim the legitimacy, and one to enforce acknowledgment.
Art. 118.
The action to claim its legitimacy may be brought by the child at any time of its lifetime and shall be transmitted
to its heirs, should it die during minority or in a state of insanity. In such cases the heirs shall be allowed a period of five years in
which to institute the action.
The action already instituted by the child is transmitted by its death to the heirs, if it has not lapsed before then.
Art. 137.
The actions for the acknowledgment of natural children can be instituted only during the life of the presumed
parents, except in the following cases: chanrobles virtualawlibrary
1.
If the father or mother died during the minority of the child, in which case the latter may institute the action before the
expiration of the first four years of its majority.
2.
If, after the death of the father or mother, some instrument, before unknown, should be discovered in which the child is
expressly acknowledged.
In this case the action must be instituted within the six months following the discovery of such instrument.
On this supposition the first difference that results between one action and the other consists in that the right of action for
legitimacy lasts during the whole lifetime of the child, that is, it can always be brought against the presumed parents or their
heirs by the child itself, while the right of action for the acknowledgment of a natural child does not last his whole lifetime, and,
as a general rule, it cannot be instituted against the heirs of the presumed parents, inasmuch as it can be exercised only during
the life of the presumed parents.
With regard to the question at issue, that is, the transmission to the heirs of the presumed parents of the obligation to admit the
legitimate filiation, or to recognize the natural filiation, there exists the most radical difference in that the former continues during
the life of the child who claims to be legitimate, and he may demand it either directly and primarily from the said presumed
parents, or indirectly and secondarily from the heirs of the latter; while the second does not endure for life; as a general rule, it
only lasts during the life of the presumed parents. Hence the other difference, derived as a consequence, that an action for
legitimacy is always brought against the heirs of the presumed parents in case of the death of the latter, while the action for
acknowledgment is not brought against the heirs of such parents, with the exception of the two cases prescribed by article 137
transcribed above.
So much for the passive transmission of the obligation to admit the legitimate filiation, or to acknowledge the natural filiation.
As to the transmission to the heirs of the child of the latters action to claim his legitimacy, or to obtain the acknowledgment of
his natural filiation, it is seen that the code grants it in the first case, but not the second. It contains provisions for the
transmission of the right of action which, for the purpose of claiming his legitimacy inheres in the child, but it does not say a
word with regard to the transmission of the right to obtain the acknowledgment of the natural filiation.
Therefore, the respective corollary of each of the two above-cited articles is: chanrobles virtualawlibrary (1) That the right of
action which devolves upon the child to claim his legitimacy under article 118, may be transmitted to his heirs in certain cases
designated in the said article; (2) That the right of action for the acknowledgment of natural children to which article 137 refers,
can never be transmitted, for the reason that the code makes no mention of it in any case, not even as an exception.
It is most illogical and contrary to every rule of correct interpretation, that the right of action to secure acknowledgment by the
natural child should be presumed to be transmitted, independently, as a rule, to his heirs, while the right of action to claim
legitimacy from his predecessor is not expressly, independently, or, as a general rule, conceded to the heirs of the legitimate
child, but only relatively and as an exception. Consequently, the pretension that the right of action on the part of the child to
obtain the acknowledgment of his natural filiation is transmitted to his descendants is altogether unfounded. No legal provision
exists to sustain such pretension, nor can an argument of presumption be based on the lesser claim when there is no basis for
the greater one, and when it is only given as an exception in well-defined cases. It is placing the heirs of the natural child on a
better footing than the heirs of the legitimate one, when, as a matter of fact, the position of a natural child is no better than, nor
even equal to, that of a legitimate child.
From the express and precise precepts of the code the following conclusions are derived: chanrobles virtualawlibrary
The right of action that devolves upon the child to claim his legitimacy lasts during his whole life, while the right to claim the
acknowledgment of a natural child lasts only during the life of his presumed parents.
Inasmuch as the right of action accruing to the child to claim his legitimacy lasts during his whole life, he may exercise it either
against the presumed parents, or their heirs; while the right of action to secure the acknowledgment of a natural child, since it
does not last during his whole life, but depends on that of the presumed parents, as a general rule can only be exercised
against the latter.
Usually the right of action for legitimacy devolving upon the child is of a personal character and pertains exclusively to him, only
the child may exercise it at any time during his lifetime. As an exception, and in three cases only, it may be transmitted to the
heirs of the child, to wit, if he died during his minority, or while insane, or after action had been already instituted.
An action for the acknowledgment of a natural child may, as an exception, be exercised against the heirs of the presumed
parents in two cases: chanrobles virtualawlibrary first, in the event of the death of the latter during the minority of the child, and

second, upon the discovery of some instrument of express acknowledgment of the child, executed by the father or mother, the
existence of which was unknown during the life of the latter.
But as such action for the acknowledgment of a natural child can only be exercised by him. It cannot be transmitted to his
descendants, or to his ascendants.
In support of the foregoing the following authorities may be cited: chanrobles virtualawlibrary
Sanchez Roman, in his Treatise on Civil Law, propounds the question as to whether said action should be considered
transmissive to the heirs or descendants of the natural child, whether he had or had not exercised it up to the time of his death,
and decides it as follows;
There is an entire absence of legal provisions, and at most, it might be deemed admissible as a solution, that the right of action
to claim the acknowledgment of a natural child is transmitted by analogy to his heirs on the same conditions and terms that it is
transmitted to the descendants of a legitimate child, to claim his legitimacy, under article 118, but nothing more; because on this
point nothing warrants placing the heirs of a natural child on a better footing than those of the legitimate child, and even to
compare them would not fail to be a strained and questionable matter, and one of great difficulty for decision by the courts, for
the simple reason that for the heirs of the legitimate child, the said article 118 exists, while for those of the natural child, as we
have said, there is no provision in the code authorizing the same, although on the other hand there is none that prohibits it.
(Vol. V.) cralaw
Diaz Guijarro and Martinez Ruiz in their work on The Civil Code as construed by the supreme court of Spain, commenting
upon article 137, say: chanrobles virtualawlibrary
Article 118, taking into account the privileges due to the legitimacy of children, grants them the right to claim said legitimacy
during their lifetime, and even authorizes the transmission of said right for the space of five years to the heirs thereof, if the child
die during his minority or in a state of insanity. But as article 137 is based on the consideration that in the case of a natural child,
ties are less strong and sacred in the eyes of the law, it does not fix such a long and indefinite period for the exercise of the
action; it limits it to the life of the parents, excepting in the two cases mentioned in said article; and it does not allow, as does
article 118, the action to pass on to the heirs, inasmuch as, although it does not prohibit it, and for that reason it might be
deemed on general principles of law to consent to it, such a supposition is inadmissible for the reason that a comparison of both
articles shows that the silence of the law in the latter case is not, nor can it be, an omission, but a deliberate intent to establish a
wide difference between the advantages granted to a legitimate child and to a natural one.
(Ibid., Vol. II, 171.) cralaw
Navarro Amandi (Cuestionario del Codigo Civil) raises the question: chanrobles virtualawlibrary Can the heirs of a natural child
claim the acknowledgment in those cases wherein the father or mother are under obligation to acknowledge? And
says: chanrobles virtualawlibrary
Opinions are widely divergent. The court of Rennes held (on April 13, 1844) that the right of investigation forms a part of the
estate of the child, and along with his patrimony is transmitted to his heirs. The affirmation is altogether too categorical to be
admissible. If it were correct the same thing would happen as when the legitimacy of a child is claimed, and as already seen,
the right of action to demand the legitimacy is not transmitted to the heirs in every case and as an absolute right, but under
certain limitations and circumstances. Now, were we to admit the doctrine of the court of Rennes, the result would be that the
claim for natural filiation would be more favored than one for legitimate filiation. This would be absurd, because it cannot be
conceived that the legislator should have granted a right of action to the heirs of the natural child, which is only granted under
great limitations and in very few cases to those of a legitimate one. Some persons insist that the same rules that govern
legitimate filiation apply by analogy to natural filiation, and that in this conception the heirs of the natural child are entitled to
claim it in the cases prescribed by article 118. The majority, however, are inclined to consider the right to claim acknowledgment
as a personal right, and consequently, not transmissive to the heirs. Really there are not legal grounds to warrant the
transmission. (Vol. 2, 229.) cralaw
In a decision like the present one it is impossible to bring forward the argument of analogy for the purpose of considering that
the heirs of the natural child are entitled to the right of action which article 118 concedes to the heirs of the legitimate child. The
existence of a provision for the one case and the absence thereof for the other is a conclusive argument that inclusio unius est
exclusio alterius, and it cannot be understood that the provision of law should be the same when the same reason does not hold
in the one case as in the other.
The theory of the law of transmission is also entirely inapplicable in this case. This theory, which in the Roman Law expressed
the general rule that an heir who did not accept an inheritance during his lifetime was incapacitated from transmitting it to his
own heirs, included at the same time the idea that if the inheritance was not transmitted because the heir did not possess it,
there were, however, certain things which the heir held and could transmit. Such was the law and the right to accept the
inheritance, for the existing reason that all rights, both real and personal, shall pass to the heir; quia haeres representat
defunctum in omnibus et per omnia. According to article 659 of the Civil Code, the inheritance includes all the property, rights,
and obligations of a person, which are not extinguished by his death. If the mother is the heir of her natural child, and the latter,
among other rights during his lifetime was entitled to exercise an action for his acknowledgment against his father, during the life
of the latter, or after his death in some of the excepting cases of article 137, such right, which is a portion of his inheritance, is
transmitted to his mother as being his heir, and it was so understood by the court of Rennes when it considered the right in
question, not as a personal and exclusive right of the child which is extinguished by his death, but as any other right which might
be transmitted after his death. This right of supposed transmission is even less tenable than that sought to be sustained by the
argument of analogy.
The right of action pertaining to the child to claim his legitimacy is in all respects superior to that of the child who claims
acknowledgment as a natural child. And it is evident that the right of action to claim his legitimacy is not one of those rights
which the legitimate child may transmit by inheritance to his heirs; it forms no part of the component rights of his inheritance. If it
were so, there would have been no necessity to establish its transmissibility to heirs as an exception in the terms and conditions
of article 118 of the code. So that, in order that it may constitute a portion of the childs inheritance, it is necessary that the
conditions and the terms contained in article 118 shall be present, since without them, the right that the child held during his
lifetime, being personal and exclusive in principle, and therefore, as a general rule not susceptible of transmission, would and
should have been extinguished by his death. Therefore, where no express provision like that of article 118 exists, the right of
action for the acknowledgment of a natural child is, in principle and without exception, extinguished by his death, and cannot be
transmitted as a portion of the inheritance of the deceased child.
On the other hand, it said right of action formed a part of the childs inheritance, it would be necessary to establish the doctrine
that the right to claim such an acknowledgment from the presumed natural father and from his heirs is an absolute right of the
heirs of the child, not limited by certain circumstances as in the case of the heirs of a legitimate child; and if it is unreasonable to
compare a natural child with a legitimate one to place the heirs of a natural child and his inheritance on a better footing than
those of a legitimate child would not only be unreasonable, but, as stated in one of the above citations, most absurd and illegal
in the present state of the law and in accordance with the general principles thereof.
For all of the foregoing reasons we hereby reverse the judgment appealed from in all its parts, without any special ruling as to
the costs of this instance.

G.R. No. 72078 June 27, 1994


EUTIQUIO MARQUINO and MARIA TERENAL-MARQUINO Survived by: LUZ. T. MARQUINO, ANA T. MARQUINO and
EVA T. MARQUINO, petitioners,
vs.
THE HON. INTERMEDIATE APPELLATE COURT, FIRST CIVIL CASES DIVISION, BIBIANA ROMANO-PAGADORA,
Survived By: PEDRO PAGADORA, EMY R. PAGADORA, JUNE R. PAGADORA, EDGAR R. PAGADORA, MAY R.
PAGADORA, MAGO R. PAGADORA, ARDEN R. PAGADORA, and MARS R. PAGADORA,respondents.
Lenin R. Victoriano for petitioners.
Herminiano D. Silva for respondents.

PUNO, J.:
For resolution are the following issues: (1) the effect of the death of the natural child during the pendency of her action for
recognition; and (2) the effect of the death of the putative parent also during the pendency of the case.
The facts are as follows:
Respondent Bibiana Romano-Pagadora filed Civil Case No. 5197, an action for Judicial Declaration of Filiation, Annulment of
Partition, Support, and Damages against petitioner Eutiquio Marquino on January 10, 1971 before the then Court of First
Instance of Negros Occidental. Also impleaded as defendants, were Maria Terenal-Marquino, wife of Eutiquio Marquino, and
their legitimate children Luz, Ana, and Eva, all surnamed Terenal-Marquino.
The records show that Bibiana was born on December 2, 1926 at Piapi, Dumaguete City, of Gregoria Romano and allegedly of
Eutiquio Marquino. 1 At that time, Eutiquio was still single. Bibiana became personally known to the Marquino family when she was hired as domestic helper in their
household at Luke Wright Street, Dumaguete City. She always received financial assistance from them. Thus, she claimed that she enjoyed continuous possession of the status of
an acknowledged natural child by direct and unequivocal acts of her father and his family. The Marquinos, on the other hand, strongly denied her allegations.

During the pendency of the case and before respondent Bibiana could finish presenting her evidence, she died on March 17,
1979. On March 23, 1979, her heirs were ordered substituted for her as parties-plaintiffs. On
May 17, 1983, petitioners filed a Motion to Dismiss. They averred that the action for recognition is intransmissible to the heirs
being a personal act. 2 The trial court dismissed the case.
Respondents appealed to the respondent Intermediate Appellate Court (now Court of Appeals). On August 20, 1983, Eutiquio
Marquino died while the case was pending appeal. On June 17, 1985, respondent court invoking the case of Banaga
vs. Pascacio, (No. 4848-R, July 31, 1954, 50 O.G. No. 12, p. 5908) reversed the controverted order. It ruled:
[A]fter the death of the natural child, the heirs of said deceased natural child, cannot bring the action to compel
recognition, but may however, continue the action already filed to compel recognition.
xxx xxx xxx
Summarizing, We hold that the death of the putative parent while the case against him for recognition of his alleged
child is pending will not extinguish the action but the same can be continued with the heirs substituted for said
deceased parents because:
a) the law does not require that the case be brought and decided while the putative parent is alive;
b) that would be adding another requisite for the action which is not sanctioned by the law or
jurisprudence;
c) it would be unfair to the plaintiff child to have his action for recognition depend on the speed of the
Court in disposing of the case and on a fortuitous event. This is because if the court takes, let us say, 10
years to decide the case, the chances that the defendant parent would survive the case is very much
less, especially if he was already of advanced age at the time the action is brought;
d) there are no compelling reasons not to allow substitution of the deceased parent with his heirs, for
with the death of the defendant parent, the effects of recognition will practically be limited to
successional rights.
WHEREFORE, finding merit in this appeal, we hereby SET ASIDE the Order of the trial Court dated August 13, 1983
and remand the case to the Court of origin for continuation of the trial by the heirs of plaintiff against the heirs of
defendant Eutiquio Marquino, without pronouncement as to costs.
SO ORDERED. 3

The Motion for Reconsideration was denied on May 19, 1985. Hence, this petition for review on certiorari.
Petitioners hold respondent court to be in error, in these respects:
I
IN RULING THAT AFTER THE DEATH OF THE NATURAL CHILD, THE HEIRS OF SAID DECEASED NATURAL
CHILD, CANNOT BRING THE ACTION TO COMPEL RECOGNITION, BUT THEY MAY HOWEVER, CONTINUE
THE ACTION ALREADY FILED TO COMPEL RECOGNITION.
II
IN RULING THAT THE DEATH OF THE PUTATIVE PARENT WHILE THE CASE AGAINST HIM FOR
RECOGNITION OF HIS ALLEGED CHILD IS PENDING WILL NOT EXTINGUISH THE ACTION BUT THE SAME
CAN BE CONTINUED WITH THE HEIRS SUBSTITUTED FOR SAID DECEASED PARENT. 4
The Court writes finis to this controversy after twenty-three (23) years of protracted litigation.
The first issue to be resolved is whether or not the right of action to compel recognition is intransmissible in character.
Article 285 of the Civil Code provides that an action for recognition of natural children may be brought only during the lifetime of
the presumed parents, except in the following cases:
(1) If the father or mother died during the minority of the child, in which case the latter may file the action before the expiration of
four years from the attainment of his majority;
(2) If after the death of the father or of the mother a document should appear of which nothing had been heard and in which
either or both parents recognize the child.
In this case, the action must be commenced within four years from discovery of the document.
The rationale for the rule is to give the alleged parents opportunity to be heard. The reason for the exceptions is to protect the
heirs. 5
In Conde vs. Abaya, 6 we held that the right of action for the acknowledgment of natural children to which Article 285 (Article 137, Old Civil code) refers, can never be
transmitted. The reason is that the code makes no mention of it in any case, not even as an exception.

In the case at bench, it is evident that Bibiana was a natural child. She was born out of wedlock on December 2, 1926, of
Gregoria Romano and allegedly of Eutiquio Marquino who at that time was single. Bibiana sued for compulsory recognition
while Eutiquio was still alive. Sadly, she died on March 17, 1983 before she could present her proof of recognition. Her death
tolled the action considering its personal nature and intransmissibility. As explained in the case of Conde vs. Abaya, 8 viz.:
It is most illogical and contrary to every rule of correct interpretation that the right of action to secure
acknowledgment by the natural child should be presumed to be transmitted, independently, as a rule to his heirs,
while the right to claim legitimacy from his predecessor is not expressly, independently, or, as a general rule
conceded to the heirs of the legitimate child, but only relatively and as an exception. Consequently, the pretension
that the right of action on the part of the child to obtain the acknowledgment of his natural filiation is transmitted to
his descendants, is altogether unfounded. No legal provision exists to sustain such pretension, nor can an argument
of presumption be based on the lesser claim when there is no basis for the greater one, and when it is only given as
an exception in well-defined cases. It is placing the heirs of the natural child on a better footing than the heirs of the
legitimate one, when, as a child is not better than, nor even equal to, that of a legitimate child.
This ruling was reiterated in the recent case of Heirs of Raymundo C. Banas vs. Heirs of Bibiano Banas 9thus:
Granting that, after the death of Bibiano Banas Raymundo could file an action for compulsory recognition against
Bibiano's heirs, still plaintiffs-appellants cannot invoke Raymundo's right to file such action, because it is not
transmissible to the natural child's heirs; the right is purely a personal one to the natural child.
The second issue for resolution is whether or not after the death of the putative father the action for recognition of a natural child
can be continued against the heirs of the former.
We rule against its continuance. In an action for compulsory recognition, the party in the best position to oppose the same is the
putative parent
himself. 10 The need to hear the side of the putative parent is an overwhelming consideration because of the unsettling effects of such an action on the peace and harmonious
relationship in the family of the putative parent. For this reason, Article 285 provides only two (2) exceptions when an action for recognition transcends the death of the putative
parent. Neither of these exceptions obtains in the case at bench. Firstly, the death of Eutiquio did not occur during the minority of Bibiana. In fact, she was already forty-five (45)
years old when the recognition case was filed on January 10, 1971. Secondly, no document was discovered, before unknown, in which Bibiana was expressly acknowledged as a
natural child. Consequently, the respondent court erred in ruling that the action can still be continued against the heirs of Eutiquio. 11

Our public policy at that time supports the rule limiting actions for recognition during the lifetime of the presumed parents, to
quote:
Public policy, indeed public necessity, demands that before an illegitimate child be admitted into a legitimate family,
every requisite of the law must be completely and fully complied with. No one should ever be permitted upon
doubtful evidence to take from legitimate children the property which they and their parents have, by industry,
fidelity, and frugality, acquired. To do so would in many instances where the legitimate children had "labored
unsparingly in order that they might have the comforts of life and joys of home," be manifestly contrary to every
plainest principles of justice. And again, if this can ever be done upon oral testimony alone, after the lips of the
alleged father and mother have been closed by death, such testimony must be clear, strong, and convincing. 12
Our law providing for the intransmissibility of an action for recognition, however, has been superseded by the New Family Code
which took effect on August 3, 1988. Under Article 173 of the Family Code, it is now provided:
The action to claim legitimacy may be brought by the child during his or her lifetime and shall be transmitted to the
heirs should the child die during minority or in a state of insanity. In these cases, the heirs shall have a period of five
(5) years within which to institute the action.
The action commenced by the child shall survive notwithstanding the death of either or both of the parties.
(Emphasis supplied)
Pursuant to this provision, the child can bring the action during his or her entire lifetime (not during the lifetime of the parents)
and even after the death of the parents. In other words, the action does not prescribe as long as he lives. 13
Be that as it may, Article 173 of the Family Code cannot be given retroactive effect so as to apply to the case at bench because
it will prejudice the vested rights of petitioners transmitted to them at the time of the death of their father, Eutiquio Marquino.
"Vested right" is a right in property which has become fixed and established and is no longer open to doubt or controversy. 14 It
expresses the concept of present fixed interest, which in right reason and natural justice should be protected against arbitrary State action. 15

WHEREFORE, the decision of the Court of Appeals dated June 17, 1985 is REVERSED and SET ASIDE. The Complaint in
Civil Case No. 5197 of the then Court of First Instance of Negros Occidental is DISMISSED.

A.M. No. MTJ-92-716 October 25, 1995


MA. BLYTH B. ABADILLA, complainant,
vs.
JUDGE JOSE C. TABILIRAN, JR., Presiding Judge, 8th MCTC, Manukan and Jose Dalman, 9th Judicial Region,
Manukan, Zamboanga del Norte, respondent.

PER CURIAM:
We have a list of these crooked judges whose actuations have been found to be patently wrong and indefensible. There ought
to be no objection or compunction in weeding them out from the service. If they are not booted out now, it will take from here to
eternity to clean this Augean stable. 1
Indeed, our judicial structure is supposed to be manned by magistrates chosen for their probity, integrity, impartiality, dedication
and learning. And so, any judge wanting in any of these qualities should be broomed off and out of the bench in order to
improve the judicial landscape. Screening off the misfits, considering the great number of judges and justices in the country at
present, is the arduous and Herculean task of this Court. The effort if dramatized with rectitude and sincerity should bring about
the strengthening of the people's abiding faith in democracy and the integrity of our courts of justice.
The herein administrative case arose from a complaint, dated September 8, 1992, filed by Ma. Blyth B. Abadilla, a Clerk of
Court assigned at the sala of respondent, Judge Jose C. Tabiliran, Jr., of the 8th Municipal Circuit Trial Court, Manukan,
Zamboanga del Norte. Respondent stands charged with "gross immorality, deceitful conduct, and corruption unbecoming of a
judge."
In her verified complaint, complainant Abadilla, in respect to the charge of gross immorality on the part of the respondent,
contends that respondent had scandalously and publicly cohabited with a certain Priscilla Q. Baybayan during the existence of
his legitimate marriage with Teresita Banzuela. Adding ignominy to an ignominious situation, respondent allegedly shamefacedly
contracted marriage with the said Priscilla Baybayan on May 23, 1986. Complainant claims that this was a bigamous union
because of the fact that the respondent was then still very much married to Teresita Banzuela.
Furthermore, respondent falsely represented himself as "single" in the marriage contract (Exh. "A") and dispensed with the
requirements of a marriage contract by invoking cohabitation with Baybayan for five years.
Of persuasive effect on the charge of immorality is the fact that, earlier, respondent's wife filed a complaint in the case entitled,
Teresita B. Tabiliran vs. Atty. Jose C. Tabiliran, Jr., 115 SCRA 451. Respondent stood charged therein for abandoning the family
home and living with a certain Leonora Pillarion with whom he had a son.
In respect of the charge of deceitful conduct, complainant claims that respondent caused to be registered as "legitimate", his
three illegitimate children with Priscilla Baybayan, namely:
Buenasol B. Tabiliran born on July 14, 1970
Venus B. Tabiliran born on Sept. 7, 1971
Saturn B. Tabiliran born on Sept. 20, 1975
by falsely executing separate affidavits stating that the delayed registration was due to inadvertence, excusable
negligence or oversight, when in truth and in fact, respondent knew that these children cannot be legally registered as
legitimate.
The following acts are alleged to have constituted the charge of corruption:
(1) Utilizing his office time, while being a judge, in the private practice of law by the preparation and notarization of documents,
out of which he charged fees beyond the authorized rates allowed as Ex-Officio Notary Public. These acts which, according to
the charge, amount to the private practice of law, prejudice public interest.
Complainant submitted the following documents in support of these allegations:
a) Affidavit of Ponciana Geromo (Annex "B"), attesting to the fact that respondent Judge Tabiliran prepared a
Simultaneous Deed of Sale, (Annex "C", Doc. No. 901, Page No. 77, Book No. V, Series of 1991 of ExOfficio Notary Public Jose C. Tabiliran, Jr.) and collect P600.00 from the vendees (par. 10(a) a-1 Complaint, p. 9
records);
b) Receipt prepared under instruction of the respondent showing that he received P250.00 thru MCTC Aide Ely O.
Inot for preparation and notarization of Joint Affidavit declaring the correct ages of Carlo Manzano, Lodmila Cinco,

Kadapi Amad, Jul Samud and Amman Eddai dated November 12, 1991, when the legal fees therefor should have
been P10.00 only (Annex "D") (par. 10(a) a-2 Complaint, p. 9 records);
c) Another receipt (Annex "E") prepared thru the direction of the respondent dated November 12, 1991, showing that
said respondent received from Reynaldo Subebe the sum of P150.00 for preparation and notarization by him of a
Joint Affidavit declaring the correct age of Agata Luna, Rosie Miranda and Jose Juneser Adrias (par. 10(a) a-c
Complaint, p. 9 records);
d) Still another receipt (Annex "F") dated November 12, 1991, signed by the respondent himself showing that he
received from Nelly Baradas the sum of P50.00 for preparation and notarization of Joint Affidavit attesting to the
correct age of one Luzviminda Jacoba (par. 10(a) a-d Complaint, p. 9 records);
e) Another receipt (Annex "G") dated November 12, 1991, issued by the respondent, showing that he received from
Torres P. Modai the sum of P50.00, thru the same Ely O. Inot, MCTC Aide, for preparation of Joint Affidavit attesting
to the correct age of Flores Jalampangan (par. 10 (a) a-e Complaint, pp. 9 & 10 records).
(2) Accepting bribes from parties-litigants in his Court as supported by an affidavit (Annex "M") executed by a certain Calixto
Calunod, a court aide, stating that he saw Edna Siton, complainant in a criminal case tried by respondent, hand over to the
latter a bag of fish and squid which respondent Judge received.
(3) Preparing an Affidavit of Desistance in a case filed with his sala out of which he collected the amount of P500.00 from the
accused Antonio Oriola, as supported by the affidavits of Arcelita Salvador, the complainant therein, and Benito Sagario, one of
the persons present when the accused perpetrated the acts aforesaid. (Submitted as Annexes "I" and "J", respectively.)
Complainant manifests that the commission by the respondent of the foregoing acts renders him unfit to occupy the exalted
position of a dispenser of justice. By the example shown by the respondent, the public had allegedly lost confidence in the
administration of justice, perceiving as is evident to see that the person occupying the position of a judge lacks the morality and
probity required of one occupying such a high office.
Respondent, in his comment, dated December 25, 1992, declared that his cohabitation with Priscilla Baybayan is not and was
neither bigamous nor immoral because he started living with Priscilla Baybayan only after his first wife had already left and
abandoned the family home in 1966 and, since then, and until the present her whereabouts is not known and respondent has
had no news of her being alive. He further avers that 25 years had already elapsed since the disappearance of his first wife
when he married Priscilla Baybayan in 1986.
Respondent cited Sec. 3(w), Rule 131 of the Rules of Court and Art. 390 of the Civil Code in order to show the legality of his
acts:
After the absence of seven years, it being unknown whether or not the absentee still lives, he is considered dead for
all purposes except for those of succession. (Rule 131, Sec. 3(w), Rules of Court.)
After an absence of seven years, it being unknown whether or not the absentee still lives, he shall be presumed
dead for all purposes, except for those of succession. (Art. 390, Civil Code.)
The case of Jones vs. Hortiguela, 64 Phil. 179, where this Court held that for the purpose of the civil marriage law, it is not
necessary to have the former spouse judicially declared an absentee is to respondent's mind, a case in point.
He admits that he indicated in his marriage contract that he was then "single", but he denied the charge that he acted with
deceit or false misrepresentation, claiming that, since there were only three words to choose from, namely: Single, Widow or
Divorced, he preferred to choose the word "single", it being the most appropriate. Besides, both he and Priscilla executed a joint
affidavit wherein his former marriage to Banzuela was honestly divulged.
On the charge of corruption, respondent submitted certifications (Annexes "4" & "5") from the Mayor of Manukan, Zamboanga
del Norte, attesting to the fact that there was no Notary Public in Manukan and, as such, respondent may be allowed to notarize
documents. He denied having charged exorbitant fees. He claims that all the amounts received by him were used to subsidize
office expenses, since the funds he had been receiving from the municipal government were not enough to cover expenses in
maintaining his office. Respondent submitted a certification (Annex "6") from the Accounting Department of the Municipal
Government of Manukan to the effect that his yearly expenditures were more than the yearly appropriations.
Respondent finds support in Canon 4, Rule 4.01 of the Code of Judicial Conduct which states:
A Judge may, with due regard to official duties, engage in activities to improve . . . the administration of justice.
Respondent vehemently denies the charge of bribery claiming that it was inconceivable for him to receive a bag full of fish and
squid since his residence was 42 kilometers from Jose Dalman where his courtroom or office was located. It takes one an hour
and a half by bus to reach Katipunan and so, by the time he reaches his house, the fish and the squid should have become
rotten. In support of his denials, respondent submitted as Annex "8", an affidavit of Ely D. Inot, their court Interpreter who
declared:
xxx xxx xxx
3. That last June 6, 1991, I was with the Municipal Judge, Jose C. Tabiliran, Jr., from the morning until we went
home in the afternoon and we in fact dined together in the local Carenderia of Jose Dalman as it is the usual ways
of the Judge to eat lunch together with the court personnel;
4. That when we went home in the afternoon of that day we were also together riding in a bus, the Lillian Express
and until I drop in Roxas and he proceeded to Katipunan where his residence is;
5. That all the time during that day I did not noticed him bringing anything except his "Hand Bag" which he used to
carry in going to the office; (Annex "8", Affidavit of Ely O. Inot, December 17, 1992.)
xxx xxx xxx
Finally, respondent tags as a fabricated lie the charge that he prepared an Affidavit of Desistance in a case pending in his sala
and thereafter charged the accused, Antonio Oriola, the sum of P500.00 for legal services. The complainant, he said, was the
one who induced Arcelita Salvador (the complainant in the rape case) to execute an affidavit (Annex "I") in support of the charge
of corruption against respondent.
Complainant's filing of the present case was motivated by revenge and resentment because, earlier, respondent filed an
administrative case (A.M. No. P-91-597) against her for "Insubordination and Serious Misconduct". The Supreme Court decided
to reprimand her with a warning that a repetition of her acts will be severely dealt with. Respondent claims that the complainant
had nevertheless repeatedly continued to do acts of insubordination in the following manner:
1) She continues to keep court records and has kept refusing to hand them over to respondent inspite of verbal and
written orders;

2) She refused to receive a memorandum from the Vice-Mayor requiring the Clerk of Court to submit an Annual
report;
3) She refused to prepare the said annual report required of her as Clerk of Court;
4) She continue to refuse to obey just and lawful orders of the Court.
On April 12, 1993, by resolution of this Court En Banc, the herein administrative case was referred to Executive Judge Jesus O.
Angeles of the Regional Trial Court, Dipolog City, for investigation, report and recommendation. Judge Angeles found
respondent guilty only on two (2) counts of corruption: (1) for acting as notary public and collecting fees for his services; and (2)
for preparing an affidavit of desistance in a case pending in his Court and receiving payment for it.
In his report and recommendation dated August 3, 1993, Executive Judge Angeles found that:
ON GROSS IMMORALITY:
In contracting marriage with Priscilla Q. Baybayan on May 23, 1986, (p. 13 of the records), respondent did not hide
the fact that he was married to Teresita T. Banzuela, having disclosed it in his affidavit jointly executed with Priscilla
Q. Baybayan on May 23, 1986 (p. 115 of the records), particularly paragraph 4 thereof which reads:
4. That affiant Jose C. Tabiliran, Jr., was formerly married to Teresita T. Banzuela but who left and abandoned their
family home sometime in 1965 in Katipunan, Zamboanga del Norte, and until now at present her whereabouts is not
known.
It was therefore a marriage contracted under Article 83 (2) of the Civil Code which, although bigamous, remains
valid until automatically terminated by the recording of the affidavit of reappearance of the absent spouse (Art. 42,
Family Code). Respondent's assertion that since 1965 to the present, his first wife Teresita T. Banzuela had left their
conjugal dwelling and did not return, her whereabouts being unknown, was not controverted. Living as husband and
wife pursuant to an authorized bigamous marriage, respondent cannot be said to be acting in an immoral and
scandalous manner, and the immoral stigma of extra-marital union since 1969 duly declared in their aforesaid joint
affidavit, may be considered cleansed by their marriage in 1986, if Art. 1395 of the Civil Code on ratification on
contracts in general is allowed to be applied, it being ratification of marital cohabitation. Article 76 of Civil Code, now
Art. 34 of the Family Colde was intended to facilitate and encourage the marriage of persons who have been living
in a state of concubinage for more than five years (Tolentino, Civil Code, Book I, 1974 Ed., p. 245, cited in Ernesto
L. Pineda, Family Code, 1992 Ed., p. 38). Indicating his civil status in the marriage contract as "single" is hardly
considered a misrepresentation of fact, specially to the solemnizing officer, Municipal Mayor Jacinto C. Ruedas, Jr.
to whom the aforesaid joint affidavit was submitted.
ON DECEITFUL CONDUCT:
Respondent's children begotten with Priscilla Q. Baybayan, namely: Buenasol B. Tabiliran, Venus B. Tabiliran and
Saturn B. Tabiliran, all of whom were born before their marriage, were disclosed and made known to the solemnizing
officer and the latter himself, in his affidavit dated May 23, 1986 (p. 116 of the records) which supports the marriage
contract of respondent with Priscilla Q. Baybayan, having shown such fact.
Exhibit P which purports to be an affidavit of Lydia T. Zanoria dated May 27, 1993, consisting of three pages, was
submitted by the complainant for the purpose of proving her charge that the respondent falsely executed his three
separate affidavits, namely: Exhibit K dated May 24, 1983 regarding the late registration of birth of his daughter
Buenasol B. Tabiliran; Exhibit M dated May 28, 1988 regarding the late registration of birth of his third child Saturn B.
Tabiliran; and his affidavit dated May 27, 1988, Exhibit O, in reference to the late registration of birth of his second
child Venus B. Tabiliran, stating inadvertence, excusable negligence or oversight as the reasons for the delayed
registration of their births, without however presenting said affiant Mrs. Zanoria, consequently denying respondent
the opportunity to cross examine her. Her affidavit is not among those brought out in the pre-hearing conference,
and was not discussed during the hearing itself, submitting it only after the investigation proper was terminated. The
supposed affiant claimed she was the government midwife who attended to the births of respondent's three children,
denying, as the affidavit shows, negligence, inadvertence or oversight on her part to register their birth on time. Not
having been presented for respondent to confront her, or an opportunity to do so, Exhibit P cannot be considered
evidence of the charge. An affidavit is hearsay unless the affiant is presented (People vs. Villeza, 127 SCRA 349), or
admitted by the party against whom it is presented.
ON CORRUPTION:
1. Acting as Notary Public during office hours, and collecting fees:
Respondent has admitted having prepared the documents and collected fees, in the instances specified in par. 10 of
the complaint, namely: (1) affidavit of Ponciana Geromo; (2) Joint Affidavit of Carlo Manzano, Lodmila Cinco, Kadapi
Amad, Jul Samud and Amman Eddai; (3) Joint Affidavit of Agata Luna, Rosie Miranda and Jose Juneser Adrias; (4)
Joint Affidavit on the correct age of Luzviminda Jacoba; and (5) Joint Affidavit on the correct age of Flores
Jalampangan, but not necessarily on the accuracy of the amounts therein stated as having been collected by him
from them (please see Pre-Hearing Order of May 20, 1993 of the Investigating Judge). Seeking justification of his
acts, respondent submitted Annexes 4 & 5 of his comments (pp. 118 and 119, records) which are certifications of
Manukan Mayor Eugene U. Caballero attesting that in the absence of a Notary Public in Manukan town, respondent
who is a Judge thereat was allowed "to prepare and ligalize (sic) documents".
He declared "the fees derived from the preparation and notarization of documents were mostly used by respondent
to buy supplies and materials of his Office", explaining that his office needs cannot be sustained by the
appropriations of the local government which are inadequate. On page 120 of the records, his Annex 6 shows a
shortage in his appropriations for supplies. And supplies from the Supreme Court can only be obtained if secured
personally but has to assume the expenses for transportation, freight and handling.
Respondent Judge maintains that the Code of Judicial conduct does not prohibit him from acting as Notary Public,
and the fees he has received were much lower than the rates prescribed by the Integrated Bar of the Philippines,
Zamboanga del Norte Chapter, submitting Annex 3, p. 117 of the records, to prove it.
Further justifying his act under Canon 4, Rule 4.01 of the Code of Judicial Conduct which provides that a judge may,
with due regard to official duties, engaged in activities to improve the administration of justice, respondent claims
that due to his efforts, he was able to secure an extension room of his office covering a floor area of 24 square
meters, from the Sangguniang Pampook of Region IX based in Zamboanga City, costing P19,000.00 per
certification shown in his Annex 7 (page 121 of the records).
In the light of 1989 Code of Judicial Conduct vis-a-vis the power of Municipal Trial Court Judges and Municipal
Circuit Trial Court Judges to act in the capacity of Notary Public Ex-Officio, the Honorable Supreme Court in A.M.
No. 89-11-1303, MTC, Dec. 19, 1989, has ruled:
MTC and MCTC Judges assigned to municipalities or circuits with no lawyers or notaries public may, in their
capacity as notary public ex-officio perform any act within the competency of a regular Notary Public, provided that:

(1) all notarial fees charged be for the account of the Government and turned-over to the municipal treasurer
(Lapea, Jr. vs. Marcos, Adm. Matter No. 1969-MJ, June 29, 1982, 114 SCRA 572); and (2) certification be made in
the notarized documents attesting to the lack of any lawyer or notary public in such municipality or circuit.
Although absence of a notary public commissioned for, and residing in Manukan town, even in Jose Dalman which
is within his circuit is confirmed, respondent Judge while he may be justified in so acting as notary public, did not,
however, comply with requirement No. 1 which obliged him to charge for the account of the Government and turnover to the municipal treasurer all notarial fees. And there is no way of determining the truth of his assertion that the
notarial fees he collected were "mostly used" to buy supplies and materials for his office, absent any accounting.
2. Accepting Bribe from Parties-litigants:
Admitting the existence of Annex H found on page 21 in the records, respondent, however, denied the imputation
therein contained by affiant Calixto Calunod that he received a sando bag full of fish and squid from a certain Edna
Siton who had a case with respondent's court as complainant in a certain criminal case. Instead of calling the affiant
himself, complainant presented the Court Interpreter Ely O. Inot, who "confirmed that there was squid and fish
contained in a plastic bag which was left in Aseniero Carenderia by a person unknown to her and some members of
the Court staff. When informed by the carenderia owner that the stuff was intended for Judge Tabiliran, the latter told
them to cook it, and they afterwards partook of it without the Judge who already boarded the passenger bus".
(Record of Proceedings, p. 1, par. No. 1, dated June 11, 1993). Being her witness, complainant is bound by her
testimony. This particular charge is, therefore, not proved.
3. Preparing Affidavit of Desistance and Collecting Fee for his Services:
Under this count, two affidavits both sworn before 2nd Asst. Provincial Fiscal Valeriano B. Lagula were submitted:
one by Arcelita Salvador, complainant in an attempted rape case who was categorical in her declaration that
respondent Judge asked and received from Pitoy Oriola, brother of accused Antonio Oriola the amount of P500.00
after the Judge prepared the affidavit of desistance and motion to dismiss which he made her sign (Annex I, p. 40
records). Benito Sagario who was present executed another separate affidavit, Annex J found on page 41 in the
records, confirming it. In admitting the affidavit, respondent, however, denied the imputation, asserting that it is false,
but without confronting them or presenting witnesses to dispute their accusation. He could have demanded that the
affiants, including the persons they mentioned were present in the transaction, namely: accused Antonio Oriola, his
brother Pitoy Oriola, Ignacio Salvador, and INC Minister Antonio Calua be required to appear for his confrontation,
but respondent chose not, contented himself only with the explanation that it was just the handiwork of complainant
Abadilla and her husband, a major in the military who is an active member of the Iglesia Ni Cristo of which affiant
Arcelita Salvador also belonged, which is bare and unsubstantiated. No other conclusion can be drawn other than
holding, as the Investigating Judge does, that this particular charge is true. Evidently, Judge Tabiliran wants to avoid
meeting them by way of confrontation. If he is innocent, and is certain the charge is fabricated, he will surely raise
hell to insist that he confronts them face to face. Clearly, his deportment betrays his insistence of innocence.
On Respondent's Counterclaim:
It was not proven. On the contrary, the controverting evidence shows that the records of Criminal Case No. 2279
referred to in his Annex 9, p. 123 of the records, were not in the possession of complainant. Quite obviously, Ely O.
Inot, respondent's Court Interpreter tried to cover up the fact that the same were already being kept by Judge
Tabiliran before he issued the memorandum, Annex 9. Complainant, who is respondent's Clerk of Court was not,
therefore, in a position to comply with his Order.
Also, Mrs. Abadilla's failure to prepare the annual report of the Court in 1992 as called for in Annexes 10 and 10-A
was, contrary to respondent's claim, not by reason of her obstinate refusal to obey her superior but, by sheer
impossibility to comply, considering that monthly reports upon which the annual report shall be based, were not
prepared by her, not because of her refusal to do so which is among those included in her job description, but
because the Judge himself took the work from her for no other reason than to establish the false impression that the
complainant is disobedient to the Judge, and does not attend to her duties.
By and large, there is no harmony in their office. Complainant and respondent are not in talking terms. They are
hostile to each other. Respondent's complaint that Mrs. Abadilla spat saliva in front of him whenever they meet each
other; destroying the Court dry seal by throwing it at him one time she was mad; showing face; and sticking out her
tongue to him, are all puerile acts which the undersigned cannot conclude as sufficiently established even with the
testimony of Mrs. Ely O. Inot which is far from being definite and categorical, whose actuation is understandable
because Judge Tabiliran, being her superior, has moral ascendancy over her (Record of Proceedings, June 11,
1993).
The undersigned believes that the problem is on Judge Tabiliran, and not on Mrs. Abadilla, who has been in the
service as Clerk of Court under a previous Judge of the same Court for quite long without any complaint having
been filed. The evidence disputing his counterclaim tends to show that respondent tried to build up a situation of
undesirability against his Clerk of Court whom he wanted pulled out from her position in his Court.
Other Matters Not Covered By The Complaint And Comments:
The authority to investigate being confined only to matters alleged in the complaint on the basis of which respondent
filed his comments, other matters not therein covered which complainant brought out by way of presenting
documentary exhibits, (from Exhibit AAA to HHH), are not subject of this report and recommendation.
RECOMMENDATION:
The charge of GROSS IMMORALITY and DECEITFUL CONDUCT have not been proven, but the undersigned
believes evidence is sufficient to sustain pronouncement of guilt on two counts of CORRUPTION, namely: acting as
notary public and collecting fees for his services in preparing affidavit of desistance of a case in his Court. Likewise,
acts of oppression, deceit and false imputation against his Clerk of Court are found duly established.
WHEREFORE, suspension of the respondent Judge from the service for a period of three months is recommended.
THE FOREGOING CONSIDERED, We hold the respondent culpable for gross immorality, he having scandalously and openly
cohabited with the said Priscilla Baybayan during the existence of his marriage with Teresita B. Tabiliran.
Contrary to his protestations that he started to cohabit with Priscilla Baybayan only after his first wife, Teresita Tabiliran, had long
abandoned him and the conjugal home in 1966, it appears from the record that he had been scandalously and openly living with
said Priscilla Baybayan as early as 1970 as shown by the fact that he begot three children by her, namely Buenasol, Venus and
Saturn, all surnamed Tabiliran. Buenasol was born on July 14, 1970; Venus was born on September 7, 1971; while Saturn was
born on September 20, 1975. Evidently, therefore, respondent and Priscilla Baybayan had openly lived together even while
respondent's marriage to his first wife was still valid and subsisting. The provisions of Sec. 3(w) of the Rules of Court and Art.
390 of the Civil Code which provide that, after an absence of seven years, it being unknown whether or not the absentee still
lives, the absent spouse shall be considered dead for all purposes, except for those of succession, cannot be invoked by
respondent. By respondent's own allegation, Teresita B. Tabiliran left the conjugal home in 1966. From that time on up to the
time that respondent started to cohabit with Priscilla Baybayan in 1970, only four years had elapsed. Respondent had no right to

presume therefore that Teresita B. Tabiliran was already dead for all purposes. Thus, respondent's actuation of cohabiting with
Priscilla Baybayan in 1970 when his marriage to Teresita B. Tabiliran was still valid and subsisting constitutes gross immoral
conduct. It makes mockery of the inviolability and sanctity of marriage as a basic social institution. According to Justice Malcolm:
"The basis of human society throughout the civilized world is that of marriage. It is not only a civil contract, but is a new relation,
an institution on the maintenance of which the public is deeply interested. Consequently, every intendment of the law leans
toward legalizing matrimony." (Civil Code 1993 Ed., Volume 1, p. 122, Ramon C. Aquino).
By committing the immorality in question, respondent violated the trust reposed on his high office and utterly failed to live up to
the noble ideals and strict standards of morality required of the law profession. (Imbing v. Tiongson, 229 SCRA 690).
As to respondent's act of eventually marrying Priscilla Baybayan in 1986, We are not in a position to determine the legality
thereof, absent all the facts for a proper determination. Sufficient for Our consideration is the finding of the Investigating Judge,
that the said marriage is authorized under Art. 83 (2) of the Civil Code.
With respect to the charge of deceitful conduct, We hold that the charge has likewise been duly established. An examination of
the birth certificates (Exhs. "J", "L", & "M") of respondent's three illegitimate children with Priscilla Baybayan clearly indicate that
these children are his legitimate issues. It was respondent who caused the entry therein. It is important to note that these
children, namely, Buenasol, Venus and Saturn, all surnamed Tabiliran, were born in the year 1970, 1971, and 1975,
respectively, and prior to the marriage of respondent to Priscilla, which was in 1986. As a lawyer and a judge, respondent ought
to know that, despite his subsequent marriage to Priscilla, these three children cannot be legitimated nor in any way be
considered legitimate since at the time they were born, there was an existing valid marriage between respondent and his first
wife, Teresita B. Tabiliran. The applicable legal provision in the case at bar is Article 269 of the Civil Code of the Philippines
(R.A. 386 as amended) which provides:
Art. 269. Only natural children can be legitimated. Children born outside of wedlock of parents who, at the time of
the conception of the former, were not disqualified by any impediment to marry each other, are natural.
Legitimation is limited to natural children and cannot include those born of adulterous relations (Ramirez vs. Gmur, 42 Phil.
855). The Family Code: (Executive Order, No. 209), which took effect on August 3, 1988, reiterated the above-mentioned
provision thus:
Art. 177. Only children conceived and born outside of wedlock of parents who, at the time of the conception of the
former, were not disqualified by any impediment to marry each other may be legitimated.
The reasons for this limitation are given as follows:
1) The rationale of legitimation would be destroyed;
2) It would be unfair to the legitimate children in terms of successional rights;
3) There will be the problem of public scandal, unless social mores change;
4) It is too violent to grant the privilege of legitimation to adulterous children as it will destroy the sanctity of marriage;
5) It will be very scandalous, especially if the parents marry many years after the birth of the child. (The Family
Code, p. 252, Alicia v. Sempio Diy).
It is clear, therefore, that no legal provision, whether old or new, can give refuge to the deceitful actuations of the
respondent.
It is also erroneous for respondent to state that his first wife Teresita disappeared in 1966 and has not been heard from since
then. It appears that on December 8, 1969, Teresita filed a complaint against respondent entitled,Tabiliran vs. Tabiliran (G.R.
No. 1155451) which was decided by this Court in 1982. In the said case, respondent was sued for abandonment of his family
home and for living with another woman with whom he allegedly begot a child. Respondent was, however, exonerated because
of the failure of his wife to substantiate the charges. However, respondent was reprimanded for having executed a "Deed of
Settlement of Spouses To Live Separately from Bed", with a stipulation that they allow each of the other spouse to live with
another man or woman as the case may be, without the objection and intervention of the other. It was also in the same case
where respondent declared that he has only two children, namely, Reynald Antonio and Jose III, both surnamed Tabiliran, who
are his legitimate issues. Thus, his statements in his affidavits marked as Exhs. "M-4" and "O-4" that Saturn and Venus are his
third and second children respectively, are erroneous, deceitful, misleading and detrimental to his legitimate children.
With respect to the charge of corruption, We agree with the findings of the Investigating Judge that respondent should be found
culpable for two counts of corruption: (1) acting as Notary Public; and (2) collecting legal fees in preparing an Affidavit of
Desistance of a case in his court.
Respondent himself admitted that he prepared and notarized the documents (Annexes "C", "D", "E", "F" and "G") wherein he
charged notarial fees. Though he was legally allowed to notarize documents and charge fees therefor due to the fact that there
has been no Notary Public in the town of Manukan, this defense is not sufficient to justify his otherwise corrupt and illegal acts.
Section 252 of the Notarial Law expressly provides thus:
Sec. 252. Compensation of Notaries Public No fee, compensation, or reward of any sort, except such as is
expressly prescribed and allowed by law, shall be collected or received for any service rendered by a notary public.
Such money collected by notaries public proper shall belong to them personally. Officers acting as notaries public
ex-officio shall charge for their services the fees prescribed by law and account therefor as for Government funds.
(Notarial Law, Revised Administrative Code of the Philippines, p. 202.)
Respondent's failure to properly account and turn over the fees collected by him as Ex-Officio notary to the municipal
government as required by law raises the presumption that he had put such fund to his personal use.
With respect to the charge that respondent prepared an Affidavit of Desistance in a rape case filed before his sala for which he
collected the amount of P500.00 from the complainant therein, respondent merely denied the said imputation but failed to offer
any evidence to support such denial. Denial, if unsubstantiated by clear and convincing evidence, is a negative and self-serving
evidence which deserves no weight in law and cannot be given greater evidentiary value over the testimony of credible
witnesses who testify on affirmative matters (People v. Amaguin, 229 SCRA 166). It is unfortunate that respondent had failed to
adhere to, and let this remind him once again of Canon 2 of the Code of Judicial Conduct, to wit:
Canon 2
A judge should avoid impropriety and the appearance of impropriety in all activities.
WHEREFORE, the Court finds respondent Judge Jose C. Tabiliran, Jr. guilty of gross immorality, deceitful conduct and
corruption and, consequently, orders his dismissal from the service. Such dismissal shall carry with it cancellation of eligibility,
forfeiture of leave credits and retirement benefits, and disqualification from re-employment in the government-service, all without
prejudice to criminal or civil liability.

Adoption
Republic Act No. 8552

February 25, 1998

AN ACT ESTABLISHING THE RULES AND POLICIES ON THE DOMESTIC ADOPTION OF FILIPINO CHILDREN AND FOR
OTHER PURPOSES
Be it enacted by the Senate and House of Representatives of the Philippines in Congress assembled::
ARTICLE I
GENERAL PROVISIONS
Section 1. Short Title. This Act shall be known as the "Domestic Adoption Act of 1998."
Section 2. Declaration of Policies. (a) It is hereby declared the policy of the State to ensure that every child remains under
the care and custody of his/her parent(s) and be provided with love, care, understanding and security towards the full and
harmonious development of his/her personality. Only when such efforts prove insufficient and no appropriate placement or
adoption within the child's extended family is available shall adoption by an unrelated person be considered.
(b) In all matters relating to the care, custody and adoption of a child, his/her interest shall be the paramount consideration in
accordance with the tenets set forth in the United Nations (UN) Convention on the Rights of the Child; UN Declaration on Social
and Legal Principles Relating to the Protection and Welfare of Children with Special Reference to Foster Placement and
Adoption, Nationally and Internationally; and the Hague Convention on the Protection of Children and Cooperation in Respect of
Intercountry Adoption. Toward this end, the State shall provide alternative protection and assistance through foster care or
adoption for every child who is neglected, orphaned, or abandoned.
(c) It shall also be a State policy to:
(i) Safeguard the biological parent(s) from making hurried decisions to relinquish his/her parental authority over his/her
child;
(ii) Prevent the child from unnecessary separation from his/her biological parent(s);
(iii) Protect adoptive parent(s) from attempts to disturb his/her parental authority and custody over his/her adopted child.
Any voluntary or involuntary termination of parental authority shall be administratively or judicially declared so as to
establish the status of the child as "legally available for adoption" and his/her custody transferred to the Department of
Social Welfare and Development or to any duly licensed and accredited child-placing or child-caring agency, which entity
shall be authorized to take steps for the permanent placement of the child;
(iv) Conduct public information and educational campaigns to promote a positive environment for adoption;
(v) Ensure that sufficient capacity exists within government and private sector agencies to handle adoption inquiries,
process domestic adoption applications, and offer adoption-related services including, but not limited to, parent
preparation and post-adoption education and counseling; and
(vi) Encourage domestic adoption so as to preserve the child's identity and culture in his/her native land, and only when
this is not available shall intercountry adoption be considered as a last resort.
Section 3. Definition of Terms. For purposes of this Act, the following terms shall be defined as:
(a) "Child" is a person below eighteen (18) years of age.
(b) "A child legally available for adoption" refers to a child who has been voluntarily or involuntarily committed to the
Department or to a duly licensed and accredited child-placing or child-caring agency, freed of the parental authority of
his/her biological parent(s) or guardian or adopter(s) in case of rescission of adoption.
(c) "Voluntarily committed child" is one whose parent(s) knowingly and willingly relinquishes parental authority to the
Department.
(d) "Involuntarily committed child" is one whose parent(s), known or unknown, has been permanently and judicially
deprived of parental authority due to abandonment; substantial, continuous, or repeated neglect; abuse; or incompetence
to discharge parental responsibilities.
(e) "Abandoned child" refers to one who has no proper parental care or guardianship or whose parent(s) has deserted
him/her for a period of at least six (6) continuous months and has been judicially declared as such.

(f) "Supervised trial custody" is a period of time within which a social worker oversees the adjustment and emotional
readiness of both adopter(s) and adoptee in stabilizing their filial relationship.
(g) "Department" refers to the Department of Social Welfare and Development.
(h) "Child-placing agency" is a duly licensed and accredited agency by the Department to provide comprehensive child
welfare services including, but not limited to, receiving applications for adoption, evaluating the prospective adoptive
parents, and preparing the adoption home study.
(i) "Child-caring agency" is a duly licensed and accredited agency by the Department that provides twenty four (24)-hour
residential care services for abandoned, orphaned, neglected, or voluntarily committed children.
(j) "Simulation of birth" is the tampering of the civil registry making it appear in the birth records that a certain child was
born to a person who is not his/her biological mother, causing such child to lose his/her true identity and status.
ARTICLE II
PRE-ADOPTION SERVICES
Section 4. Counseling Service. The Department shall provide the services of licensed social workers to the following:
(a) Biological Parent(s) Counseling shall be provided to the parent(s) before and after the birth of his/her child. No
binding commitment to an adoption plan shall be permitted before the birth of his/her child. A period of six (6) months shall
be allowed for the biological parent(s) to reconsider any decision to relinquish his/her child for adoption before the
decision becomes irrevocable. Counseling and rehabilitation services shall also be offered to the biological parent(s) after
he/she has relinquished his/her child for adoption.
Steps shall be taken by the Department to ensure that no hurried decisions are made and all alternatives for the child's
future and the implications of each alternative have been provided.
(b) Prospective Adoptive Parent(s) Counseling sessions, adoption fora and seminars, among others, shall be provided
to prospective adoptive parent(s) to resolve possible adoption issues and to prepare him/her for effective parenting.
(c) Prospective Adoptee Counseling sessions shall be provided to ensure that he/she understands the nature and
effects of adoption and is able to express his/her views on adoption in accordance with his/her age and level of maturity.
Section 5. Location of Unknown Parent(s). It shall be the duty of the Department or the child-placing or child-caring agency
which has custody of the child to exert all efforts to locate his/her unknown biological parent(s). If such efforts fail, the child shall
be registered as a foundling and subsequently be the subject of legal proceedings where he/she shall be declared abandoned.
Section 6. Support Services. The Department shall develop a pre-adoption program which shall include, among others, the
above mentioned services.
ARTICLE III
ELIGIBILITY
Section 7. Who May Adopt. The following may adopt:
(a) Any Filipino citizen of legal age, in possession of full civil capacity and legal rights, of good moral character, has not
been convicted of any crime involving moral turpitude, emotionally and psychologically capable of caring for children, at
least sixteen (16) years older than the adoptee, and who is in a position to support and care for his/her children in keeping
with the means of the family. The requirement of sixteen (16) year difference between the age of the adopter and adoptee
may be waived when the adopter is the biological parent of the adoptee, or is the spouse of the adoptee's parent;
(b) Any alien possessing the same qualifications as above stated for Filipino nationals: Provided, That his/her country has
diplomatic relations with the Republic of the Philippines, that he/she has been living in the Philippines for at least three (3)
continuous years prior to the filing of the application for adoption and maintains such residence until the adoption decree
is entered, that he/she has been certified by his/her diplomatic or consular office or any appropriate government agency
that he/she has the legal capacity to adopt in his/her country, and that his/her government allows the adoptee to enter
his/her country as his/her adopted son/daughter: Provided, Further, That the requirements on residency and certification
of the alien's qualification to adopt in his/her country may be waived for the following:
(i) a former Filipino citizen who seeks to adopt a relative within the fourth (4th) degree of consanguinity or affinity; or
(ii) one who seeks to adopt the legitimate son/daughter of his/her Filipino spouse; or
(iii) one who is married to a Filipino citizen and seeks to adopt jointly with his/her spouse a relative within the fourth
(4th) degree of consanguinity or affinity of the Filipino spouse; or
(c) The guardian with respect to the ward after the termination of the guardianship and clearance of his/her financial
accountabilities.
Husband and wife shall jointly adopt, except in the following cases:
(i) if one spouse seeks to adopt the legitimate son/daughter of the other; or
(ii) if one spouse seeks to adopt his/her own illegitimate son/daughter: Provided, However, that the other spouse has
signified his/her consent thereto; or
(iii) if the spouses are legally separated from each other.
In case husband and wife jointly adopt, or one spouse adopts the illegitimate son/daughter of the other, joint parental authority
shall be exercised by the spouses.
Section 8. Who May Be Adopted. The following may be adopted:
(a) Any person below eighteen (18) years of age who has been administratively or judicially declared available for
adoption;
(b) The legitimate son/daughter of one spouse by the other spouse;
(c) An illegitimate son/daughter by a qualified adopter to improve his/her status to that of legitimacy;
(d) A person of legal age if, prior to the adoption, said person has been consistently considered and treated by the
adopter(s) as his/her own child since minority;
(e) A child whose adoption has been previously rescinded; or

(f) A child whose biological or adoptive parent(s) has died: Provided, That no proceedings shall be initiated within six (6)
months from the time of death of said parent(s).
Section 9. Whose Consent is Necessary to the Adoption. After being properly counseled and informed of his/her right to
give or withhold his/her approval of the adoption, the written consent of the following to the adoption is hereby required:
(a) The adoptee, if ten (10) years of age or over;
(b) The biological parent(s) of the child, if known, or the legal guardian, or the proper government instrumentality which
has legal custody of the child;
(c) The legitimate and adopted sons/daughters, ten (10) years of age or over, of the adopter(s) and adoptee, if any;
(d) The illegitimate sons/daughters, ten (10) years of age or over, of the adopter if living with said adopter and the latter's
spouse, if any; and
(e) The spouse, if any, of the person adopting or to be adopted.
ARTICLE IV
PROCEDURE
Section 10. Hurried Decisions. In all proceedings for adoption, the court shall require proof that the biological parent(s) has
been properly counseled to prevent him/her from making hurried decisions caused by strain or anxiety to give up the child, and
to sustain that all measures to strengthen the family have been exhausted and that any prolonged stay of the child in his/her
own home will be inimical to his/her welfare and interest.
Section 11. Case Study. No petition for adoption shall be set for hearing unless a licensed social worker of the Department,
the social service office of the local government unit, or any child-placing or child-caring agency has made a case study of the
adoptee, his/her biological parent(s), as well as the adopter(s), and has submitted the report and recommendations on the
matter to the court hearing such petition.
At the time of preparation of the adoptee's case study, the concerned social worker shall confirm with the Civil Registry the real
identity and registered name of the adoptee. If the birth of the adoptee was not registered with the Civil Registry, it shall be the
responsibility of the concerned social worker to ensure that the adoptee is registered.
The case study on the adoptee shall establish that he/she is legally available for adoption and that the documents to support
this fact are valid and authentic. Further, the case study of the adopter(s) shall ascertain his/her genuine intentions and that the
adoption is in the best interest of the child.
The Department shall intervene on behalf of the adoptee if it finds, after the conduct of the case studies, that the petition should
be denied. The case studies and other relevant documents and records pertaining to the adoptee and the adoption shall be
preserved by the Department.
Section 12. Supervised Trial Custody. No petition for adoption shall be finally granted until the adopter(s) has been given by
the court a supervised trial custody period for at least six (6) months within which the parties are expected to adjust
psychologically and emotionally to each other and establish a bonding relationship. During said period, temporary parental
authority shall be vested in the adopter(s).
The court may motu proprio or upon motion of any party reduce the trial period if it finds the same to be in the best interest of
the adoptee, stating the reasons for the reduction of the period. However, for alien adopter(s), he/she must complete the six (6)month trial custody except for those enumerated in Sec. 7 (b) (i) (ii) (iii).
If the child is below seven (7) years of age and is placed with the prospective adopter(s) through a pre-adoption placement
authority issued by the Department, the prospective adopter(s) shall enjoy all the benefits to which biological parent(s) is entitled
from the date the adoptee is placed with the prospective adopter(s).
Section 13. Decree of Adoption. If, after the publication of the order of hearing has been complied with, and no opposition
has been interposed to the petition, and after consideration of the case studies, the qualifications of the adopter(s), trial custody
report and the evidence submitted, the court is convinced that the petitioners are qualified to adopt, and that the adoption would
redound to the best interest of the adoptee, a decree of adoption shall be entered which shall be effective as of the date the
original petition was filed. This provision shall also apply in case the petitioner(s) dies before the issuance of the decree of
adoption to protect the interest of the adoptee. The decree shall state the name by which the child is to be known.
Section 14. Civil Registry Record. An amended certificate of birth shall be issued by the Civil Registry, as required by
the Rules of Court, attesting to the fact that the adoptee is the child of the adopter(s) by being registered with his/her surname.
The original certificate of birth shall be stamped "cancelled" with the annotation of the issuance of an amended birth certificate
in its place and shall be sealed in the civil registry records. The new birth certificate to be issued to the adoptee shall not bear
any notation that it is an amended issue.
Section 15. Confidential Nature of Proceedings and Records. All hearings in adoption cases shall be confidential and
shall not be open to the public. All records, books, and papers relating to the adoption cases in the files of the court, the
Department, or any other agency or institution participating in the adoption proceedings shall be kept strictly confidential.
If the court finds that the disclosure of the information to a third person is necessary for purposes connected with or arising out
of the adoption and will be for the best interest of the adoptee, the court may merit the necessary information to be released,
restricting the purposes for which it may be used.
ARTICLE V
EFFECTS OF ADOPTION
Section 16. Parental Authority. Except in cases where the biological parent is the spouse of the adopter, all legal ties
between the biological parent(s) and the adoptee shall be severed and the same shall then be vested on the adopter(s).
Section 17. Legitimacy. The adoptee shall be considered the legitimate son/daughter of the adopter(s) for all intents and
purposes and as such is entitled to all the rights and obligations provided by law to legitimate sons/daughters born to them
without discrimination of any kind. To this end, the adoptee is entitled to love, guidance, and support in keeping with the means
of the family.
Section 18. Succession. In legal and intestate succession, the adopter(s) and the adoptee shall have reciprocal rights of
succession without distinction from legitimate filiation. However, if the adoptee and his/her biological parent(s) had left a will, the
law on testamentary succession shall govern.
ARTICLE VI
RESCISSION OF ADOPTION
Section 19. Grounds for Rescission of Adoption. Upon petition of the adoptee, with the assistance of the Department if a
minor or if over eighteen (18) years of age but is incapacitated, as guardian/counsel, the adoption may be rescinded on any of

the following grounds committed by the adopter(s): (a) repeated physical and verbal maltreatment by the adopter(s) despite
having undergone counseling; (b) attempt on the life of the adoptee; (c) sexual assault or violence; or (d) abandonment and
failure to comply with parental obligations.
Adoption, being in the best interest of the child, shall not be subject to rescission by the adopter(s). However, the adopter(s)
may disinherit the adoptee for causes provided in Article 919 of the Civil Code.
Section 20. Effects of Rescission. If the petition is granted, the parental authority of the adoptee's biological parent(s), if
known, or the legal custody of the Department shall be restored if the adoptee is still a minor or incapacitated. The reciprocal
rights and obligations of the adopter(s) and the adoptee to each other shall be extinguished.
The court shall order the Civil Registrar to cancel the amended certificate of birth of the adoptee and restore his/her original
birth certificate.
Succession rights shall revert to its status prior to adoption, but only as of the date of judgment of judicial rescission. Vested
rights acquired prior to judicial rescission shall be respected.
All the foregoing effects of rescission of adoption shall be without prejudice to the penalties imposable under the Penal Code if
the criminal acts are properly proven.
ARTICLE VII
VIOLATIONS AND PENALTIES
Section 21. Violations and Penalties. (a) The penalty of imprisonment ranging from six (6) years and one (1) day to twelve
(12) years and/or a fine not less than Fifty thousand pesos (P50,000.00), but not more than Two hundred thousand pesos
(P200,000.00) at the discretion of the court shall be imposed on any person who shall commit any of the following acts:
(i) obtaining consent for an adoption through coercion, undue influence, fraud, improper material inducement, or other
similar acts;
(ii) non-compliance with the procedures and safeguards provided by the law for adoption; or
(iii) subjecting or exposing the child to be adopted to danger, abuse, or exploitation.
(b) Any person who shall cause the fictitious registration of the birth of a child under the name(s) of a person(s) who is not
his/her biological parent(s) shall be guilty of simulation of birth, and shall be punished by prision mayor in its medium period and
a fine not exceeding Fifty thousand pesos (P50,000.00).
Any physician or nurse or hospital personnel who, in violation of his/her oath of office, shall cooperate in the execution of the
abovementioned crime shall suffer the penalties herein prescribed and also the penalty of permanent disqualification.
Any person who shall violate established regulations relating to the confidentiality and integrity of records, documents, and
communications of adoption applications, cases, and processes shall suffer the penalty of imprisonment ranging from one (1)
year and one (1) day to two (2) years, and/or a fine of not less than Five thousand pesos (P5,000.00) but not more than Ten
thousand pesos (P10,000.00), at the discretion of the court.
A penalty lower by two (2) degrees than that prescribed for the consummated offense under this Article shall be imposed upon
the principals of the attempt to commit any of the acts herein enumerated. Acts punishable under this Article, when committed
by a syndicate or where it involves two (2) or more children shall be considered as an offense constituting child trafficking and
shall merit the penalty of reclusion perpetua.
Acts punishable under this Article are deemed committed by a syndicate if carried out by a group of three (3) or more persons
conspiring and/or confederating with one another in carrying out any of the unlawful acts defined under this Article. Penalties as
are herein provided, shall be in addition to any other penalties which may be imposed for the same acts punishable under other
laws, ordinances, executive orders, and proclamations.
When the offender is an alien, he/she shall be deported immediately after service of sentence and perpetually excluded from
entry to the country.
Any government official, employee or functionary who shall be found guilty of violating any of the provisions of this Act, or who
shall conspire with private individuals shall, in addition to the above-prescribed penalties, be penalized in accordance with
existing civil service laws, rules and regulations: Provided, That upon the filing of a case, either administrative or criminal, said
government official, employee, or functionary concerned shall automatically suffer suspension until the resolution of the case.
Section 22. Rectification of Simulated Births. A person who has, prior to the effectivity of this Act, simulated the birth of a
child shall not be punished for such act: Provided, That the simulation of birth was made for the best interest of the child and
that he/she has been consistently considered and treated by that person as his/her own son/daughter: Provided, further, That
the application for correction of the birth registration and petition for adoption shall be filed within five (5) years from the
effectivity of this Act and completed thereafter: Provided, finally, That such person complies with the procedure as specified in
Article IV of this Act and other requirements as determined by the Department.
ARTICLE VIII
FINAL PROVISIONS
Section 23. Adoption Resource and Referral Office. There shall be established an Adoption Resources and Referral Office
under the Department with the following functions: (a) monitor the existence, number, and flow of children legally available for
adoption and prospective adopter(s) so as to facilitate their matching; (b) maintain a nationwide information and educational
campaign on domestic adoption; (c) keep records of adoption proceedings; (d) generate resources to help child-caring and
child-placing agencies and foster homes maintain viability; and (e) do policy research in collaboration with the Intercountry
Adoption Board and other concerned agencies. The office shall be manned by adoption experts from the public and private
sectors.
Section 24. Implementing Rules and Regulations. Within six (6) months from the promulgation of this Act, the Department,
with the Council for the Welfare of Children, the Office of Civil Registry General, the Department of Justice, Office of the Solicitor
General, and two (2) private individuals representing child-placing and child-caring agencies shall formulate the necessary
guidelines to make the provisions of this Act operative.
Section 25. Appropriations. Such sum as may be necessary for the implementation of the provisions of this Act shall be
included in the General Appropriations Act of the year following its enactment into law and thereafter.
Section 26. Repealing Clause. Any law, presidential decree or issuance, executive order, letter of instruction, administrative
order, rule, or regulation contrary to, or inconsistent with the provisions of this Act is hereby repealed, modified, or amended
accordingly.
Section 27. Separability Clause. If any provision of this Act is held invalid or unconstitutional, the other provisions not
affected thereby shall remain valid and subsisting.

Section 28. Effectivity Clause. This Act shall take effect fifteen (15) days following its complete publication in any newspaper
of general circulation or in the Official Gazette.

Republic Act 8043


The Inter-Country Adoption Act of 1995
Files:

"AN ACT ESTABLISHING THE RULES TO GOVERN INTER-COUNTRY ADOPTION OF FILIPINO CHILDREN, AND
FOR OTHER PURPOSES"
Section 1. Short Title. This Act shall be known as the "Inter-Country Adoption Act of 1995."
Sec. 2. Declaration of Policy. It is hereby declared the policy of the State to provide every neglected and
abandoned child with a family that will provide such child with love and care as well as opportunities for growth and
development. Towards this end, efforts shall be exerted to place the child with an adoptive family in the Philippines.
However, recognizing that inter-country adoption may be considered as allowing aliens not presently allowed by law to
adopt Filipino children if such children cannot be adopted by qualified Filipino citizens or aliens, the State shall take
measures to ensure that inter-country adoptions are allowed when the same shall prove beneficial to the child's best
interests, and shall serve and protect his/her fundamental rights.
Sec. 3. Definition of Terms. As used in this Act. the term:
(a) Inter-country adoption refers to the socio-legal process of adopting a Filipino child by a foreigner or a Filipino
citizen permanently residing abroad where the petition is filed, the supervised trial custody is undertaken, and the
decree of adoption is issued outside the Philippines.
(b) Child means a person below fifteen (15) years of age unless sooner emancipated by law.
(c) Department refers to the Department of Social Welfare and Development of the Republic of the Philippines.
(d) Secretary refers to the Secretary of the Department of Social Welfare and Development.
(e) Authorized and accredited agency refers to the State welfare agency or a licensed adoption agency in the
country of the adopting parents which provide comprehensive social services and which is duly recognized by the
Department.
(f) Legally-free child means a child who has been voluntarily or involuntarily committed to the Department, in
accordance with the Child and Youth Welfare Code.
(g) Matching refers to the judicious pairing of the adoptive child and the applicant to promote a mutually satisfying
parent-child relationship.
(h) Board refers to the Inter-country Adoption Board.
ARTICLE II
THE INTER-COUNTRY ADOPTION BOARD
Sec. 4. The Inter-Country Adoption Board. There is hereby created the Inter-Country Adoption Board, hereinafter
referred to as the Board to act as the central authority in matters relating to inter-country adoption. It shall act as the
policy-making body for purposes of carrying out the provisions of this Act, in consultation and coordination with the
Department, the different child-care and placement agencies, adoptive agencies, as well as non-governmental
organizations engaged in child-care and placement activities. As such, it shall:
(a) Protect the Filipino child from abuse, exploitation, trafficking and/or sale or any other practice in connection with
adoption which is harmful, detrimental, or prejudicial to the child;
(b) Collect, maintain, and preserve confidential information about the child and the adoptive parents;
(c) Monitor, follow up, and facilitate completion of adoption of the child through authorized and accredited agency;
(d) Prevent improper financial or other gain in connection with an adoption and deter improper practices contrary to

this Act;
(e) Promote the development of adoption services including post-legal adoption;
(f) License and accredit child-caring/placement agencies and collaborate with them in the placement of Filipino
children;
(g) Accredit and authorize foreign adoption agency in the placement of Filipino children in their own country; and
(h) Cancel the license to operate and blacklist the child-caring and placement agency or adoptive agency involved
from the accreditation list of the Board upon a finding of violation of any provision under this Act.
Sec. 5. Composition of the Board. The Board shall be composed of the Secretary of the Department as ex officio
Chairman, and six (6) other members to be appointed by the President for a nonrenewable term of six (6) years:
Provided, That there shall be appointed one (1) psychiatrist or psychologist, two (2) lawyers who shall have at least the
qualifications of a regional trial court judge, one (1) registered social worker and two (2) representatives from nongovernmental organizations engaged in child-caring and placement activities. The members of the Board shall receive a
per diem allowance of One thousand five hundred pesos (P1,500) for each meeting attended by them: Provided, further,
That no compensation shall be paid for more than four (4) meetings a month.
Sec. 6. Powers and Functions of the Board. The Board shall have the following powers and functions:
(a) to prescribe rules and regulations as it may deem reasonably necessary to carry out the provisions of this Act,
after consultation and upon favorable recommendation of the different agencies concerned with the child-caring,
placement, and adoption;
(b) to set the guidelines for the convening of an Inter-country Adoption Placement Committee which shall be under
the direct supervision of the Board;
(c) to set the guidelines for the manner by which selection/matching of prospective adoptive parents and adoptive
child can be made;
(d) to determine a reasonable schedule of fees and charges to be exacted in connection with the application for
adoption;
(e) to determine the form and contents of the application for inter-country adoption;
(g) to institute systems and procedures to prevent improper financial gain in connection with adoption and deter
improper practices which are contrary to this Act;
(h) to promote the development of adoption services, including post-legal adoption services,
(i) to accredit and authorize foreign private adoption agencies which have demonstrated professionalism, competence
and have consistently pursued non-profit objectives to engage in the placement of Filipino children in their own country:
Provided, That such foreign private agencies are duly authorized and accredited by their own government to conduct
inter-country adoption: Provided, however, That the total number of authorized and accredited foreign private adoption
agencies shall not exceed one hundred (100) a year;
(j) to take appropriate measures to ensure confidentiality of the records of the child, the natural parents and the
adoptive parents at all times;
(k) to prepare, review or modify, and thereafter, recommend to the Department of Foreign Affairs, Memoranda of
Agreement respecting inter-country adoption consistent with the implementation of this Act and its stated goals,
entered into, between and among foreign governments, international organizations and recognized international nongovernmental organizations;
(l) to assist other concerned agencies and the courts in the implementation of this Act, particularly as regards
coordination with foreign persons, agencies and other entities involved in the process of adoption and the physical
transfer of the child; and
(m) to perform such other functions on matters relating to inter-country adoption as may be determined by the
President.
ARTICLE III
PROCEDURE
Sec. 7. Inter-Country Adoption as the Last Resort. The Board shall ensure that all possibilities for adoption of
the child under the Family Code have been exhausted and that inter-country adoption is in the best interest of the child.
Towards this end, the Board shall set up the guidelines to ensure that steps will be taken to place the child in the
Philippines before the child is placed for inter-country adoption: Provided, however, That the maximum number that
may be allowed for foreign adoption shall not exceed six hundred (600) a year for the first five (5) years.
Sec. 8. Who May be Adopted. Only a legally free child may be the subject of inter-country adoption. In order that
such child may be considered for placement, the following documents must be submitted to the Board:
(a)Child study;
(b)Birth certificate/foundling certificate;
(c)Deed of voluntary commitment/decree of abandonment/death certificate of parents;
(d)Medical evaluation /history;
(e)Psychological evaluation, as necessary; and
(f)Recent photo of the child.
Sec. 9. Who May Adopt. An alien or a Filipino citizen permanently residing abroad may file an application for intercountry adoption of a Filipino child if he/she:

(a) is at least twenty-seven (27) years of age and at least sixteen (16) years older than the child to be adopted, at
the time of application unless the adopter is the parent by nature of the child to be adopted or the spouse of such
parent:
(b) if married, his/her spouse must jointly file for the adoption;
(c) has the capacity to act and assume all rights and responsibilities of parental authority under his national laws, and
has undergone the appropriate counseling from an accredited counselor in his/her country;
(d) has not been convicted of a crime involving moral turpitude;
(e) is eligible to adopt under his/her national law;
(f) is in a position to provide the proper care and support and to give the necessary moral values and example to all
his children, including the child to be adopted;
(g) agrees to uphold the basic rights of the child as embodied under Philippine laws, the U.N. Convention on the
Rights of the Child, and to abide by the rules and regulations issued to implement the provisions of this Act;
(h) comes from a country with whom the Philippines has diplomatic relations and whose government maintains a
similarly authorized and accredited agency and that adoption is allowed under his/her national laws; and
(i) possesses all the qualifications and none of the disqualifications provided herein and in other applicable Philippine
laws.
Sec. 10. Where to File Application. An application to adopt a Filipino child shall be filed either with the Philippine
Regional Trial Court having jurisdiction over the child, or with the Board, through an intermediate agency, whether
governmental or an authorized and accredited agency, in the country of the prospective adoptive parents, which
application shall be in accordance with the requirements as set forth in the implementing rules and regulations to be
promulgated by the Board.
The application shall be supported by the following documents written and officially translated in English.
(a) Birth certificate of applicant(s);
(b) Marriage contract, if married, and divorce decree, if applicable;
(c) Written consent of their biological or adoptive children above ten (10) years of age, in the form of sworn
statement;
(d) Physical, medical and psychological evaluation by a duly licensed physician and psychologist;
(e) Income tax returns or any document showing the financial capability of the applicant(s);
(f) Police clearance of applicant(s);
(g) Character reference from the local church/minister, the applicant's employer and a member of the immediate
community who have known the applicant(s) for at least five (5) years; and
(h) Recent postcard-size pictures of the applicant(s) and his immediate family;
The Rules of Court shall apply in case of adoption by judicial proceedings.
Sec. 11. Family Selection/Matching. No child shall be matched to a foreign adoptive family unless it is
satisfactorily shown that the child cannot be adopted locally. The clearance, as issued by the Board, with the copy of the
minutes of the meetings, shall form part of the records of the child to be adopted. When the Board is ready to transmit
the Placement Authority to the authorized and accredited inter-country adoption agency and all the travel documents of
the child are ready, the adoptive parents, or any one of them, shall personally fetch the child in the Philippines.
Sec. 12. Pre-adoptive Placement Costs. The applicant(s) shall bear the following costs incidental to the placement
of the child;
(a) The cost of bringing the child from the Philippines to the residence of the applicant(s) abroad, including all travel
expenses within the Philippines and abroad; and
(b) The cost of passport, visa, medical examination and psychological evaluation required, and other related
expenses.
Sec. 13. Fees, Charges and Assessments. Fees, charges, and assessments collected by the Board in the exercise
of its functions shall be used solely to process applications for inter-country adoption and to support the activities of the
Board.
Sec. 14. Supervision of Trial Custody. The governmental agency or the authorized and accredited agency in the
country of the adoptive parents which filed the application for inter-country adoption shall be responsible for the trial
custody and the care of the child. It shall also provide family counseling and other related services. The trial custody
shall be for a period of six (6) months from the time of placement. Only after the lapse of the period of trial custody
shall a decree of adoption be issued in the said country a copy of which shall be sent to the Board to form part of the
records of the child.
During the trial custody, the adopting parent(s) shall submit to the governmental agency or the authorized and
accredited agency, which shall in turn transmit a copy to the Board, a progress report of the child's adjustment. The
progress report shall be taken into consideration in deciding whether or not to issue the decree of adoption.
The Department of Foreign Affairs shall set up a system by which Filipino children sent abroad for trial custody are
monitored and checked as reported by the authorized and accredited inter-country adoption agency as well as the
repatriation to the Philippines of a Filipino child whose adoption has not been approved.

Sec. 15. Executive Agreements. The Department of Foreign Affairs, upon representation of the Board, shall cause
the preparation of Executive Agreements with countries of the foreign adoption agencies to ensure the legitimate
concurrence of said countries in upholding the safeguards provided by this Act.
ARTICLE IV
PENALTIES
Sec. 16. Penalties.
(a) Any person who shall knowingly participate in the conduct or carrying out of an illegal adoption, in violation of the
provisions of this Act, shall be punished with a penalty of imprisonment ranging from six (6) years and one (1) day to
twelve (12) years and/or a fine of not less than Fifty thousand pesos (P50,000), but not more than Two hundred
thousand pesos (P200.000), at the discretion of the court. For purposes of this Act, an adoption is illegal if it is effected
in any manner contrary to the provisions of this Act or established State policies, its implementing rules and regulations,
executive agreements, and other laws pertaining to adoption. Illegality may be presumed from the following acts:
(1)consent for an adoption was acquired through, or attended by coercion, fraud, improper material inducement;
(2)there is no authority from the Board to effect adoption;
(3)the procedures and safeguards placed under the law for adoption were not complied with; and
(4)the child to be adopted is subjected to, or exposed to danger, abuse and exploitation.
(b)Any person who shall violate established regulations relating to the confidentiality and integrity of records,
documents and communications of adoption applications, cases and processes shall suffer the penalty of imprisonment
ranging from one (1) year and one (1) day to two (2) years, and/or a fine of not less than Five thousand pesos
(P5,000), but not more than Ten thousand pesos (P10,000), at the discretion of the court.
A penalty lower by two (2) degrees than that prescribed for the consummated felony under this Article shall be imposed
upon the principals of the attempt to commit any of the acts herein enumerated.
Acts punishable under this Article, when committed by a syndicate or where it involves two or more children shall be
considered as an offense constituting child trafficking and shall merit the penalty of reclusion perpetua.
Acts punishable under this Article are deemed committed by a syndicate if carried out by a group of three (3) or more
persons conspiring and/or confederating with one another in carrying out any of the unlawful acts defined under this
Article.Penalties as are herein provided shall be in addition to any other penalties which may be imposed for the same
acts punishable under other laws, ordinances, executive orders, and proclamations.
Sec. 17. Public Officers as Offenders. Any government official, employee or functionary who shall be found guilty
of violating any of the provisions of this Act, or who shall conspire with private individuals shall, in addition to the
above-prescribed penalties, be penalized in accordance with existing civil service laws, rules and regulations: Provided,
That upon the filing of a case, either administrative or criminal, said government official, employee or functionary
concerned shall automatically suffer suspension until the resolution of the case.
ARTICLE V
FINAL PROVISIONS
Sec. 18. Implementing Rules and Regulations. The Inter-country Adoption Board, in coordination with the
Council for the Welfare of Children, the Department of Foreign Affairs, and the Department of Justice, after due
consultation with agencies involved in child-care and placement, shall promulgate the necessary rules and regulations to
implement the provisions of this Act within six (6) months after its effectivity.
Sec. 19. Appropriations. The amount of Five million pesos (P5,000,000) is hereby appropriated from the proceeds
of the Lotto for the initial operations of the Board and subsequently the appropriations of the same shall be included in
the General Appropriations Act for the year following its enactment.
Sec. 20. Separability Clause. If any provision, or part hereof is held invalid or unconstitutional, the remainder of
the law or the provision not otherwise affected, shall remain valid and subsisting.
Sec. 21. Repealing Clause. Any law, decree, executive order, administrative order or rules and regulations contrary
to, or inconsistent with the provisions of this Act are hereby repealed, modified or amended accordingly.
Sec. 22. Effectivity Clause. This Act shall take effect fifteen (15) days after its publication in two (2) newspapers of
general circulation.

G.R. No. L-18753

March 26, 1965

VICENTE B. TEOTICO, petitioner-appellant,


vs.
ANA DEL VAL, ETC., oppositor-appellant.
Antonio Gonzales for petitioner-appellant.
J.C. Zulueta, G. D. David and N. J. Quisumbing for oppositor-appellant.
BAUTISTA ANGELO, J.:
Maria Mortera y Balsalobre Vda. de Aguirre died on July 14, 1955 in the City of Manila leaving properties worth P600,000.00.
She left a will written in Spanish which she executed at her residence at No. 2 Legarda St., Quiapo, Manila. She affixed her
signature at the bottom of the will and on the left margin of each and every page thereof in the presence of Pilar Borja, Pilar C.
Sanchez, and Modesto Formilleza, who in turn affixed their signatures below the attestation clause and on the left margin of
each and every page of the will in the presence of the testatrix and of each other. Said will was acknowledged before Notary
Public Niceforo S. Agaton by the testatrix and her witnesses.
In said will the testatrix made the following preliminary statement: that she was possessed of the full use of her mental faculties;
that she was free from illegal pressure or influence of any kind from the beneficiaries of the will and from any influence of fear or
threat; that she freely and spontaneously executed said will and that she had neither ascendants nor descendants of any kind
such that she could freely dispose of all her estate.
Among the many legacies and devises made in the will was one of P20,000.00 to Rene A. Teotico, married to the testatrix's
niece named Josefina Mortera. To said spouses the testatrix left the usufruct of her interest in the Calvo building, while the
naked ownership thereof she left in equal parts to her grandchildren who are the legitimate children of said spouses. The
testatrix also instituted Josefina Mortera as her sole and universal heir to all the remainder of her properties not otherwise
disposed of in the will.
On July 17, 1955, Vicente B. Teotico filed a petition for the probate of the will before the Court of First Instance of Manila which
was set for hearing on September 3, 1955 after the requisite publication and service to all parties concerned.
Ana del Val Chan, claiming to be an adopted child of Francisca Mortera, a deceased sister of the testatrix, as well as an
acknowledged natural child of Jose Mortera, a deceased brother of the same testatrix, filed on September 2, 1955 an opposition
to the probate of the will alleging the following grounds: (1) said will was not executed as required by law; (2) the testatrix was
physically and mentally incapable to execute the will at the time of its execution; and (3) the will was executed under duress,
threat or influence of fear.
Vicente B. Teotico, filed a motion to dismiss the opposition alleging that the oppositor had no legal personality to intervene. The
probate court, after due hearing, allowed the oppositor to intervene as an adopted child of Francisca Mortera, and on June 17,
1959, the oppositor amended her opposition by alleging, the additional ground that the will is inoperative as to the share of Dr.
Rene Teotico because the latter was the physician who took care of the testatrix during her last illness.
After the parties had presented their evidence, the probate court rendered its decision on November 10, 1960, admitting the will
to probate but declaring the disposition made in favor of Dr. Rene Teotico void with the statement that the portion to be vacated
by the annulment should pass to the testatrix's heirs by way of intestate succession.
Petitioner Teotico, together with the universal heir Josefina Mortera, filed a motion for reconsideration of that part of the decision
which declares the portion of the estate to be vacated by the nullity of the legacy made to Dr. Rene Teotico as passing to the
legal heirs, while the oppositor filed also a motion for reconsideration of the portion of the judgment which decrees the probate
of the will. On his part, Dr. Rene Teotico requested leave to intervene and to file a motion for reconsideration with regard to that
portion of the decision which nullified the legacy made in his favor.
The motions for reconsideration above adverted to having been denied, both petitioner and oppositor appealed from the
decision, the former from that portion which nullifies the legacy in favor of Dr. Rene Teotico and declares the vacated portion as
subject of succession in favor of the legal heirs, and the latter from that portion which admits the will to probate. And in this
instance both petitioner and oppositor assign several errors which, stripped of non-essentials, may be boiled down to the
following: (1) Has oppositor Ana del Val Chan the right to intervene in this proceeding?; (2) Has the will in question been duly
admitted to probate?; (3) Did the probate court commit an error in passing on the intrinsic validity of the provisions of the will
and in determining who should inherit the portion to be vacated by the nullification of the legacy made in favor of Dr. Rene
Teotico?
These issues will be discussed separately.

1. It is a well-settled rule that in order that a person may be allowed to intervene in a probate proceeding he must have an
interest in the estate, or in the will, or in the property to be affected by it either as executor or as a claimant of the estate (Ngo
The Hua v. Chung Kiat Hua, et al., L-17091, September 30, 1963); and an interested party has been defined as one who would
be benefited by the estate such as an heir or one who has a claim against the estate like a creditor (Idem). On the other hand,
in Saguinsin v. Lindayag, et al., L-17750, December 17, 1962, this Court said:
According to Section 2, Rule 80 of the Rules of Court, a petition for letters of administration must be filed by an "interested
person." An interested party has been defined in this connection as one who would be benefited by the estate, such as an
heir, or one who has a claim against the estate, such as a creditor (Intestate Estate of Julio Magbanwa 40 O.G. 1171).
And it is well settled in this jurisdiction that in civil actions as well as special proceedings, the interest required in order that
a person may be a party thereto must be material and direct, and not merely indirect or contingent (Trillana vs.
Crisostomo, G.R. No. L-3370, August 22, 1951; Rapinosa vs. Barrion, 70 Phil. 311).
The question now may be asked: Has oppositor any interest in any of the provisions of the will, and, in the negative, would she
acquire any right to the estate in the event that the will is denied probate?
Under the terms of the will, oppositor has no right to intervene because she has no interest in the estate either as heir, executor,
or administrator, nor does she have any claim to any property affected by the will, because it nowhere appears therein any
provision designating her as heir, legatee or devisee of any portion of the estate. She has also no interest in the will either as
administratrix or executrix. Neither has she any claim against any portion of the estate because she is not a co-owner thereof,
and while she previously had an interest in the Calvo building located in Escolta, she had already disposed of it long before the
execution of the will.
1wph1.t

In the supposition that, the will is denied probate, would the oppositor acquire any interest in any portion of the estate left by the
testatrix? She would acquire such right only if she were a legal heir of the deceased, but she is not under our Civil Code. It is
true that oppositor claims to be an acknowledged natural child of Jose Mortera, a deceased brother of the deceased, and also
an adopted daughter of Francisca Mortera, a deceased sister of the testatrix, but such claim cannot give her any comfort for,
even if it be true, the law does not give her any right to succeed to the estate of the deceased sister of both Jose Mortera and
Francisca Mortera. And this is so because being an illegitimate child she is prohibited by law from succeeding to the legitimate
relatives of her natural father. Thus, Article 992 of our Civil Code provides: "An illegitimate child has no right to inherit ab
intestato from the legitimate children and relatives of his father or mother; ... ." And the philosophy behind this provision is well
expressed in Grey v. Fabie, 68 Phil. 128, as follows:
Between the natural child and the legitimate relatives of the father or mother who acknowledged it, the Code denies any
right of succession. They cannot be called relatives and they have no right to inherit. Of course, there is a blood tie, but
the law does not recognize it. On this, article 943 is based upon the reality of the facts and upon the presumption will of
the interested parties; the natural child is disgracefully looked down upon by the legitimate family; the legitimate family is,
in turn, hated by the natural child; the latter considers the privileged condition of the former and the resources of which it is
thereby deprived; the former, in turn, sees in the natural child nothing but the product of sin, a palpable evidence of a
blemish upon the family. Every relation is ordinarily broken in life; the law does no more than recognize this truth, by
avoiding further grounds of resentment. (7 Manresa, 3d., p. 110.)
The oppositor cannot also derive comfort from the fact that she is an adopted child of Francisca Mortera because under our law
the relationship established by adoption is limited solely to the adopter and the adopted and does not extend to the relatives of
the adopting parents or of the adopted child except only as expressly provided for by law. Hence, no relationship is created
between the adopted and the collaterals of the adopting parents. As a consequence, the adopted is an heir of the adopter but
not of the relatives of the adopter.
The relationship established by the adoption, however, is limited to the adopting parent, and does not extend to his other
relatives, except as expressly provided by law. Thus, the adopted child cannot be considered as a relative of the
ascendants and collaterals of the adopting parents, nor of the legitimate children which they may have after the adoption,
except that the law imposes certain impediments to marriage by reason of adoption. Neither are the children of the
adopted considered as descendants of the adopter. The relationship created is exclusively between the adopter and the
adopted, and does not extend to the relatives of either. (Tolentino, Civil Code of the Philippines, Vol. 1, p. 652).
Relationship by adoption is limited to adopter and adopted, and does not extend to other members of the family of either;
but the adopted is prohibited to marry the children of the adopter to avoid scandal. (An Outline of Philippine Civil Law by
Justice Jose B. L. Reyes and Ricardo C. Puno, Vol. 1, p. 313; See also Caguioa, Comments and Cases on Civil Law
1955, Vol 1, pp. 312-313; Paras, Civil Code of the Philippines, 1959 ed., Vol. 1, p. 515)
It thus appears that the oppositor has no right to intervene either as testamentary or as legal heir in this probate proceeding
contrary to the ruling of the court a quo.
2. The next question to be determined is whether the will Exhibit A was duly admitted to probate. Oppositor claims that the same
should not have been admitted not only because it was not properly attested to but also because it was procured thru pressure
and influence and the testatrix affixed her signature by mistake believing that it contained her true intent.
The claim that the will was not properly attested to is contradicted by the evidence of record. In this respect it is fit that we state
briefly the declarations of the instrumental witnesses.
Pilar Borja testified that the testatrix was in perfect state of health at the time she executed the will for she carried her
conversation with her intelligently; that the testatrix signed immediately above the attestation clause and on each and every
page thereof at the left-hand margin in the presence of the three instrumental witnesses and the notary public; that it was the
testatrix herself who asked her and the other witnesses to act as such; and that the testatrix was the first one to sign and later
she gave the will to the witnesses who read and signed it.
Pilar G. Sanchez also testified that she knew the testatrix since 1945; that it was the testatrix herself who asked her to be a
witness to the will; that the testatrix was the first one to sign and she gave the will later to the witnesses to sign and afterwards
she gave it to the notary public; that on the day of the execution of the will the testatrix was in the best of health.
Modesto Formilleza also testified that he was asked by the testatrix to be one of the witnesses to the will; that he read and
understood the attestation clause before he signed the document, and all the witnesses spoke either in Spanish or in Tagalog.
He finally said that the instrumental witnesses and the testatrix signed the will at the same time and place and identified their
signatures.
This evidence which has not been successfully refuted proves conclusively that the will was duly executed because it was
signed by the testatrix and her instrumental witnesses and the notary public in the manner provided for by law.
The claim that the will was procured by improper pressure and influence is also belied by the evidence. On this point the court a
quo made the following observation:
The circumstance that the testatrix was then living under the same roof with Dr. Rene Teotico is no proof adequate in law
to sustain the conclusion that there was improper pressure and undue influence. Nor is the alleged fact of isolation of the
testatrix from the oppositor and her witnesses, for their supposed failure to see personally the testatrix, attributable to the
vehemence of Dr. Rene Teotico, to exclude visitors, took place years after the execution of the will on May 17, 1951.
Although those fact may have some weight to support the theory of the oppositor, yet they must perforce yield to the

weightier fact that nothing could have prevented the testatrix, had she really wanted to from subsequently revoking her
1951 will if it did not in fact reflect and express her own testamentary dispositions. For, as testified to by the oppositor and
her witnesses, the testatrix was often seen at the Escolta, in Quiapo and Sta. Cruz, Manila, walking and accompanied by
no one. In fact, on different occasions, each of them was able to talk with her.
We have examined the evidence on the matter and we are fully in accord with the foregoing observation. Moreover, the mere
claim that Josefina Mortera and her husband Rene Teotico had the opportunity to exert pressure on the testatrix simply because
she lived in their house several years prior to the execution of the will and that she was old and suffering from hypertension in
that she was virtually isolated from her friends for several years prior to her death is insufficient to disprove what the
instrumental witnesses had testified that the testatrix freely and voluntarily and with full consciousness of the solemnity of the
occasion executed the will under consideration. The exercise of improper pressure and undue influence must be supported by
substantial evidence and must be of a kind that would overpower and subjugate the mind of the testatrix as to destroy her free
agency and make her express the will of another rather than her own (Coso v. Deza, 42 0. G. 596). The burden is on the person
challenging the will that such influence was exerted at the time of its execution, a matter which here was not done, for the
evidence presented not only is insufficient but was disproved by the testimony of the instrumental witnesses.
3. The question of whether the probate court could determine the intrinsic validity of the provisions of a will has been decided by
this Court in a long line of decisions among which the following may be cited:
Opposition to the intrinsic validity or legality of the provisions of the will cannot be entertained in Probate proceeding
because its only purpose is merely to determine if the will has been executed in accordance with the requirements of the
law." (Palacios v. Palacios, 58 0. G. 220)
... The authentication of a will decides no other questions than such as touch upon the capacity of the testator and the
compliance with those requisites or solemnities which the law prescribes for the validity of wills. It does not determine nor
even by implication prejudge the validity or efficiency of the provisions, these may be impugned as being vicious or null,
notwithstanding its authentication. The questions relating to these points remain entirely unaffected, and may be raised
even after the will has been authenticated. ...
From the fact that the legalization of a will does not validate the provisions therein contained, it does not follow that such
provision lack the efficiency, or fail to produce the effects which the law recognizes when they are not impugned by
anyone. In the matter of wills it is a fundamental doctrine that the will of the testator is the law governing the interested
parties, and must be punctually complied with in so far as it is not contrary to the law or to public morals. (Montaano v.
Suesa, 14 Phil. 676, 679-680)
To establish conclusively as against everyone, and once for all, the facts that a will was executed with the formalities
required by law and that the testator was in a condition to make a will, is the only purpose of the proceedings under the
new code for the probate of a will. (Sec. 625.) The judgment in such proceedings determines and can determine nothing
more. In them the court has no power to pass upon the validity of any provisions made in the will. It can not decide, for
example, that a certain legacy is void and another one is valid. (Castaeda v. Alemany, 3 Phil. 426, 428)
Pursuant to the foregoing precedents the pronouncement made by the court a quo declaring invalid the legacy made to Dr.
Rene Teotico in the will Exhibit A must be set aside as having been made in excess of its jurisdiction. Another reason why said
pronouncement should be set aside is that the legatee was not given an opportunity to defend the validity of the legacy for he
was not allowed to intervene in this proceeding. As a corollary, the other pronouncements touching on the disposition of the
estate in favor of some relatives of the deceased should also be set aside for the same reason.
WHEREFORE, with the exception of that portion of the decision which declares that the will in question has been duly executed
and admitted the same to probate, the rest of the decision is hereby set aside. This case is ordered remanded to the court a
quo for further proceedings. No pronouncement as to costs.

G.R. No. 92326 January 24, 1992


REPUBLIC OF THE PHILIPPINES, petitioner,
vs.
COURT OF APPEALS and ZENAIDA C. BOBILES, respondents.
The Solicitor General for petitioner.
Mariano B. Miranda for private respondent.

REGALADO, J.:
Dissatisfied with the decision of respondent Court of Appeals promulgated on February 20, 1990

which affirmed in toto the decision of


Branch 2 of the Regional Trial Court of Legaspi City granting the petition of herein private respondent to adopt the minor Jason Condat, petitioner seeks the reversal thereof in the
present petition for review on certiorari.
2

On February 2, 1988, Zenaida Corteza Bobiles filed a petition to adopt Jason Condat, then six (6) years old and who had been
living with her family since he was four (4) months old, before the Regional Trial Court of Legaspi City, docketed therein as
Special Proceeding No. 1386. 3
The court a quo, finding the petition to be sufficient in form and substance, issued an order dated February 15, 1988 setting the
petition for hearing on March 28, 1988. 4 The order was duly published, with copies thereof seasonably served on the Solicitor General; Assistant Provincial Fiscal
Mediavillo, Jr. of Albay; Salvador Condat, father of the child; and the social worker assigned to the court. A copy of said order was posted on the bulletin board of the court and in
the other places it had required for that purpose. Nobody appeared to oppose the petition. 5

Compliance with the jurisdictional requirements having been proved at the hearing, the testimonies of herein private
respondent, together with that of her husband, Dioscoro Bobiles, and one Ma. Luz Salameno of the Department of Social
Welfare and Development were taken and admitted in the proceedings.
On March 20, 1988, the trial court rendered judgment disposing as follows:
ACCORDINGLY, it is declared that henceforth, the minor child, JASON CONDAT, be freed from all legal obligations
of obedience and maintenance with respect to his natural parents, and be, to all intents and purposes, the child of
the spouses Dioscoro and Zenaida Bobiles, and the surname of the child be changed to "Bobiles" which is the
surname of the petitioner.
Furnish the Office of the Solicitor General, Manila, the Department of Social Welfare and Development, Regional
Office, Region V, Legaspi City, and the Local Civil Registrar of Tiwi, Albay, with copies of this decision. 6
Herein petitioner appealed to the Court of Appeals which, as earlier stated, affirmed the aforesaid decision of the court below.
Hence, this present petition with the following assignment of errors:
1. The Honorable Court of Appeals erred in ruling that the Family Code cannot be applied retroactively to the petition
for adoption filed by Zenaida C. Bobiles; and
2 The Honorable Court of Appeals erred in affirming the trial court's decision which granted the petition to adopt
Jason Condat in favor of spouses Dioscoro Bobiles and Zenaida C. Bobiles. 7
The petition for adoption was filed by private respondent Zenaida C. Bobiles on February 2, 1988, when the law applicable was
Presidential Decree No. 603, the Child and Youth Welfare Code. Under said code, a petition for adoption may be filed by either
of the spouses or by both of them. However, after the trial court rendered its decision and while the case was pending on appeal
in the Court of Appeals, Executive Order No. 209, the Family Code, took effect on August 3, 1988. Under the said new law, joint
adoption by husband and wife is mandatory.
On the foregoing consideration, petitioner contends that the petition for adoption should be dismissed outright for it was filed
solely by private respondent without joining her husband, in violation of Article 185 of the Family Code which requires joint
adoption by the spouses. It argues that the Family Code must be applied retroactively to the petition filed by Mrs. Bobiles, as the
latter did not acquire a vested right to adopt Jason Condat by the mere filing of her petition for adoption. We are not persuaded.
Preliminarily, we observe that petitioner's theory implies that the non-inclusion of Dioscoro Bobiles as a co-petitioner is a
jurisdictional defect, hence its prayer for an outright dismissal on that score. It could not be taking exception only on the ground
of non-joinder since petitioner must be aware that non-joinder is not a ground for the dismissal of an action or a special
proceeding. 8 We further apprehend that this objection has been raised for the first time on appeal in respondent court. Nonetheless, we shall clarify petitioner's misgivings as postulated in its aforestated assignment of errors.

Article 246 of the Family Code provides for retroactive effect of appropriate relevant provisions thereof, subject to the
qualification that such retrospective application will not prejudice or impair vested or acquired rights in accordance with the Civil
Code or other laws.
A vested right is one whose existence, effectivity and extent does not depend upon events foreign to the will of the holder.

The
term expresses the concept of present fixed interest which in right reason and natural justice should be protected against arbitrary State action, or an innately just and imperative
right which enlightened free society, sensitive to inherent and irrefragable individual rights, cannot deny. 10 Vested rights include not only legal or equitable title to the enforcement
of a demand, but also an exemption from new obligations created after the right has vested. 11

Under the Child and Youth Welfare Code, private respondent had the right to file a petition for adoption by herself, without
joining her husband therein. When Mrs. Bobiles filed her petition, she was exercising her explicit and unconditional right under
said law. Upon her filing thereof, her right to file such petition alone and to have the same proceed to final adjudication, in
accordance with the law in force at the time, was already vested and cannot be prejudiced or impaired by the enactment of a
new law.
When private respondent filed her petition in Special Proceeding No. 1386, the trial court acquired jurisdiction thereover in
accordance with the governing law. Jurisdiction being a matter of substantive law, the established rule is that the jurisdiction of
the court is determined by the statute in force at the time of the commencement of the action. 12 We do not find in the present case such facts
as would constitute it as an exception to the rule.

The first error assigned by petitioner warrants a review of applicable local and foreign jurisprudence. For that purpose, we start
with the premise that Article 185 of the Family Code is remedial in nature. Procedural statutes are ordinarily accorded a
retrospective construction in the sense that they may be applied to pending actions and proceedings, as well as to future
actions. However, they will not be so applied as to defeat procedural steps completed before their enactment. 13
Procedural matters are governed by the law in force when they arise, and procedural statutes are generally retroactive in that
they apply to pending proceedings and are not confined to those begun after their enactment although, with respect to such
pending proceedings, they affect only procedural steps taken after their enactment. 14
The rule that a statutory change in matters of procedure will affect pending actions and proceedings, unless the language of the
act excludes them from its operation, is not so extensive that it may be used to validate or invalidate proceedings taken before it
goes into effect, since procedure must be governed by the law regulating it at the time the question of procedure arises. 15
The jurisdictional, as distinguished from the purely procedural, aspect of a case is substantive in nature and is subject to a more
stringent rule. A petition cannot be dismissed by reason of failure to comply with a law which was not yet in force and effect at
the time. As long as the petition for adoption was sufficient in form and substance in accordance with the law in governance at
the time it was filed, the court acquires jurisdiction and retains it until it fully disposes of the case. 16 To repeat, the jurisdiction of the court is
determined by the statute in force at the time of the commencement of the action. Such jurisdiction of a court, whether in criminal or civil cases, once it attaches cannot be ousted
by subsequent happenings or events, although of a character which would have prevented jurisdiction from attaching in the first instance. 17

On the second issue, petitioner argues that, even assuming that the Family Code should not apply retroactively, the Court of
Appeals should have modified the trial court's decision by granting the adoption in favor of private respondent Zenaida C.
Bobiles only, her husband not being a petitioner. We do not consider this as a tenable position and, accordingly, reject the same.
Although Dioscoro Bobiles was not named as one of the petitioners in the petition for adoption filed by his wife, his affidavit of
consent, attached to the petition as Annex "B" and expressly made an integral part thereof, shows that he himself actually joined
his wife in adopting the child. The pertinent parts of his written consent read as follows:
xxx xxx xxx
2. That my wife, ZENAIDA O. CORTEZA BOBILES and I mutually desire to adopt as our child, a boy named JASON
CONDAT, still a minor being six (6) years old, likewise residing at 18 C. Imperial Street, Legaspi City, Albay, also in
the Philippines;
3. That we are filing the corresponding Petition for Adoption of said minor child, JASON CONDAT, before the
Juvenile and Domestic Relations court, now the Regional Trial Court in Legaspi City, Albay in the Philippines;
4. That I, Dioscoro C. Bobiles as the husband and father, am giving my lawful consent to this adoption of said minor
child, JASON CONDAT;
5. That further, my wife ZENAIDA O. CORTEZA BOBILES, and I have continuously reared and cared for this minor
child, JASON CONDAT since birth;
6. That as a result thereof, my wife and I have developed a kind of maternal and paternal love for the boy as our
very own, exercising therein the care, concern and diligence of a good father toward him;
7. That I am executing this document, an AFFIDAVIT OF CONSENT for whatever it is worth in the premises as to
the matter of adoption of this minor child, JASON CONDAT, by my wife ZENAIDA O. CORTEZA BOBILES and by
me, DIOSCORO C. BOBILES, in any court of justice; (Emphasis supplied.) 18
xxx xxx xxx

The foregoing declarations, and his subsequent confirmatory testimony in open court, are sufficient to make him a co-petitioner.
Under the circumstances then obtaining, and by reason of his foreign residence, he must have yielded to the legal advice that
an affidavit of consent on his part sufficed to make him a party to the petition. This is evident from the text of his affidavit.
Punctiliousness in language and pedantry in the formal requirements should yield to and be eschewed in the higher
considerations of substantial justice. The future of an innocent child must not be compromised by arbitrary insistence of rigid
adherence to procedural rules on the form of pleadings.
We see no reason why the following doctrines in American law should not apply to this case and, for that matter, in our
jurisdiction. It is a settled rule therein that adoption statutes, as well as matters of procedure leading up to adoption, should be
liberally construed to carry out the beneficent purposes of the adoption institution and to protect the adopted child in the rights
and privileges coming to it as a result of the adoption. 19 The modern tendency of the courts is to hold that there need not be more than a substantial
compliance with statutory requirements to sustain the validity of the proceeding; to refuse would be to indulge in such a narrow and technical construction of the statute as to defeat
its intention and beneficial results or to invalidate proceedings where every material requirement of the statute was complied with.

In support of this rule it is said that it is not the duty of the courts to bring the judicial microscope to bear upon the case in order
that every slight defect may be enlarged and magnified so that a reason may be found for declaring invalid an act consummated
years before, but rather to approach the case with the inclination to uphold such acts if it is found that there was a substantial
compliance with the statute. 20 The technical rules of pleading should not be stringently applied to adoption proceedings, and it is deemed more important that the
petition should contain facts relating to the child and its parents, which may give information to those interested, than that it should be formally correct as a pleading. Accordingly, it
is generally held that a petition will confer jurisdiction if it substantially complies with the adoption statute, alleging all facts necessary to give the court jurisdiction. 21

In determining whether or not to set aside the decree of adoption the interests and welfare of the child are of primary and
paramount consideration. 22 The welfare of a child is of paramount consideration in proceedings involving its custody and the propriety of its adoption by another, and the
courts to which the application for adoption is made is charged with the duty of protecting the child and its interests and, to bring those interests fully before it, it has authority to
make rules to accomplish that end. 23 Ordinarily, the approval of the adoption rests in the sound discretion of the court. This discretion should be exercised in accordance with the
best interests of the child, as long as the natural rights of the parents over the child are not disregarded. In the absence of a showing of grave abuse, the exercise of this discretion
by the approving official will not be disturbed. 24

In the case at bar, the rights concomitant to and conferred by the decree of adoption will be for the best interests of the child.
His adoption is with the consent of his natural parents. 25 The representative of the Department of Social Welfare and Development unqualifiedly
recommended the approval of the petition for adoption 26 and the trial court dispensed with the trial custody for several commendatory reasons, especially since the child had been
living with the adopting parents since infancy. 27 Further, the said petition was with the sworn written consent of the children of the adopters.

The trial court and respondent court acted correctly in granting the petition for adoption and we find no reason to disturb the
same. As found and aptly stated by respondent court: "Given the facts and circumstances of the case and considered in the
light of the foregoing doctrine, 28 We are of the opinion and so hold that the decree of adoption issued by the court a quo would go a long way towards promoting the
welfare of the child and the enhancement of his opportunities for a useful and happy life."

29

Adoption statutes, being humane and salutary, hold the interests and welfare of the child to be of paramount consideration.
They are designed to provide homes, parental care and education for unfortunate, needy or orphaned children and give them
the protection of society and family in the person of the adopted, as well as to allow childless couples or persons to experience
the joys of parenthood and give them legally a child in the person of the adopted for the manifestation of their natural parental
instincts. Every reasonable intendment should be sustained to promote and fulfill these noble and compassionate objectives of
the law. 30
WHEREFORE, the instant petition is hereby DENIED.

G.R. No. 85044 June 3, 1992


MACARIO TAMARGO, CELSO TAMARGO and AURELIA TAMARGO, petitioners,
vs.
HON. COURT OF APPEALS, THE HON. ARISTON L. RUBIO, RTC Judge, Branch 20, Vigan, Ilocos Sur; VICTOR
BUNDOC; and CLARA BUNDOC, respondents.

FELICIANO, J.:
On 20 October 1982, Adelberto Bundoc, then a minor of 10 years of age, shot Jennifer Tamargo with an air rifle causing injuries
which resulted in her death. Accordingly, a civil complaint for damages was filed with the Regional Trial Court, Branch 20, Vigan,
Ilocos Sur, docketed as Civil Case No. 3457-V, by petitioner Macario Tamargo, Jennifer's adopting parent, and petitioner
spouses Celso and Aurelia Tamargo, Jennifer's natural parents against respondent spouses Victor and Clara Bundoc,
Adelberto's natural parents with whom he was living at the time of the tragic incident. In addition to this case for damages, a
criminal information or Homicide through Reckless Imprudence was filed [Criminal Case No. 1722-V] against Adelberto Bundoc.
Adelberto, however, was acquitted and exempted from criminal liability on the ground that he bad acted without discernment.
Prior to the incident, or on 10 December 1981, the spouses Sabas and Felisa Rapisura had filed a petition to adopt the minor
Adelberto Bundoc in Special Proceedings No. 0373-T before the then Court of First Instance of Ilocos Sur. This petition for
adoption was grunted on, 18 November 1982, that is, after Adelberto had shot and killed Jennifer.
In their Answer, respondent spouses Bundoc, Adelberto's natural parents, reciting the result of the foregoing petition for
adoption, claimed that not they, but rather the adopting parents, namely the spouses Sabas and Felisa Rapisura, were
indispensable parties to the action since parental authority had shifted to the adopting parents from the moment the successful
petition for adoption was filed.
Petitioners in their Reply contended that since Adelberto Bundoc was then actually living with his natural parents, parental
authority had not ceased nor been relinquished by the mere filing and granting of a petition for adoption.
The trial court on 3 December 1987 dismissed petitioners' complaint, ruling that respondent natural parents of Adelberto indeed
were not indispensable parties to the action.
Petitioners received a copy of the trial court's Decision on 7 December 1987. Within the 15-day reglementary period, or on 14
December 1987, petitioners filed a motion for reconsideration followed by a supplemental motion for reconsideration on 15
January 1988. It appearing, however, that the motions failed to comply with Sections 4 and 5 of Rule 15 of the Revised Rules of
Court that notice of the motion shall be given to all parties concerned at least three (3) days before the hearing of said
motion; and that said notice shall state the time and place of hearing both motions were denied by the trial court in an Order
dated 18 April 1988. On 28 April 1988, petitioners filed a notice of appeal. In its Order dated 6 June 1988, the trial court
dismissed the notice at appeal, this time ruling that the notice had been filed beyond the 15-day reglementary period ending 22
December 1987.
Petitioners went to the Court of Appeals on a petition for mandamus and certiorari questioning the trial court's Decision dated 3
December 1987 and the Orders dated 18 April 1988 and 6 June 1988, The Court of Appeals dismissed the petition, ruling that
petitioners had lost their right to appeal.
In the present Petition for Review, petitioners once again contend that respondent spouses Bundoc are the indispensable
parties to the action for damages caused by the acts of their minor child, Adelberto Bundoc. Resolution of this Petition hinges on
the following issues: (1) whether or not petitioners, notwithstanding loss of their right to appeal, may still file the instant Petition;
conversely, whether the Court may still take cognizance of the case even through petitioners' appeal had been filed out of time;
and (2) whether or not the effects of adoption, insofar as parental authority is concerned may be given retroactive effect so as to
make the adopting parents the indispensable parties in a damage case filed against their adopted child, for acts committed by
the latter, when actual custody was yet lodged with the biological parents.
1. It will be recalled that, petitioners' motion (and supplemental motion) for reconsideration filed before the trial court, not having
complied with the requirements of Section 13, Rule 41, and Section 4, Rule 15, of the Revised Rules of Court, were
considered pro forma and hence did not interrupt and suspend the reglementary period to appeal: the trial court held that the
motions, not having contained a notice of time and place of hearing, had become useless pieces of paper which did not interrupt
the reglementary period. 1 As in fact repeatedly held by this Court, what is mandatory is the service of the motion on the opposing counsel indicating the time and place
of hearing. 2

In view, however, of the nature of the issue raised in the instant. Petition, and in order that substantial justice may be served, the
Court, invoking its right to suspend the application of technical rules to prevent manifest injustice, elects to treat the notice of
appeal as having been seasonably filed before the trial court, and the motion (and supplemental motion) for reconsideration
filed by petitioner in the trial court as having interrupted the reglementary period for appeal. As the Court held in Gregorio v.
Court of Appeals: 3

Dismissal of appeal; purely on technical grounds is frowned upon where the policy of the courts is to encourage
hearings of appeal on their merits. The rules of procedure ought not be applied in a very rigid technical sense, rules
of procedure are used only to help secure not override, substantial justice. if d technical and rigid enforcement of the
rules is made their aim would be defeated. 4
2. It is not disputed that Adelberto Bundoc's voluntary act of shooting Jennifer Tamargo with an air rifle gave rise to a cause of
action on quasi-delict against him. As Article 2176 of the Civil Code provides:
Whoever by act or omission causes damage to another, there being fault or negligence, is obliged to pay for the
damage done. Such fault or negligence, if there is no pre-existing contractual relation between the parties, is called
a quasi-delict . . .
Upon the other hand, the law imposes civil liability upon the father and, in case of his death or incapacity, the mother, for any
damages that may be caused by a minor child who lives with them. Article 2180 of the Civil Code reads:
The obligation imposed by article 2176 is demandable not only for one's own acts or omissions, but also for those of
persons for whom one is responsible.
The father and, in case of his death or incapacity, the mother, are responsible for the damages caused by the minor
children who live in their company.
xxx xxx xxx
The responsibility treated of in this Article shall cease when the person herein mentioned prove that they observed
all the diligence of a good father of a family to prevent damage. (Emphasis supplied)
This principle of parental liability is a species of what is frequently designated as vicarious liability, or the doctrine of "imputed
negligence" under Anglo-American tort law, where a person is not only liable for torts committed by himself, but also for torts
committed by others with whom he has a certain relationship and for whom he is responsible. Thus, parental liability is made a
natural or logical consequence of the duties and responsibilities of parents their parental authority which includes the
instructing, controlling and disciplining of the child. 5 The basis for the doctrine of vicarious liability was explained by the Court in Cangco v. Manila Railroad
Co. 6 in the following terms:

With respect to extra-contractual obligation arising from negligence, whether of act or omission, it is competent for
the legislature to elect and our Legislature has so elected to limit such liability to cases in which the person
upon whom such an obligation is imposed is morally culpable or, on the contrary, for reasons of public policy. to
extend that liability, without regard to the lack of moral culpability, so as to include responsibility for the negligence of
those persons whose acts or omissions are imputable, by a legal fiction, to others who are in a position to exercise
an absolute or limited control over them. The legislature which adopted our Civil Code has elected to limit extracontractual liability with certain well-defined exceptions to cases in which moral culpability can be directly
imputed to the persons to be charged. This moral responsibility may consist in having failed to exercise due care in
one's own acts, or in having failed to exercise due care in the selection and control of one's agent or servants, or in
the control of persons who, by reasons of their status, occupy a position of dependency with respect to the person
made liable for their conduct. 7 (Emphasis Supplied)
The civil liability imposed upon parents for the torts of their minor children living with them, may be seen to be based upon
the parental authority vested by the Civil Code upon such parents. The civil law assumes that when an unemancipated
child living with its parents commits a tortious acts, the parents were negligent in the performance of their legal and natural
duty closely to supervise the child who is in their custody and control. Parental liability is, in other words, anchored upon
parental authority coupled with presumed parental dereliction in the discharge of the duties accompanying such authority.
The parental dereliction is, of course, only presumed and the presumption can be overtuned under Article 2180 of the Civil
Code by proof that the parents had exercised all the diligence of a good father of a family to prevent the damage.
In the instant case, the shooting of Jennifer by Adelberto with an air rifle occured when parental authority was still lodged in
respondent Bundoc spouses, the natural parents of the minor Adelberto. It would thus follow that the natural parents who had
then actual custody of the minor Adelberto, are the indispensable parties to the suit for damages.
The natural parents of Adelberto, however, stoutly maintain that because a decree of adoption was issued by the adoption court
in favor of the Rapisura spouses, parental authority was vested in the latter as adopting parents as of the time of the filing of the
petition for adoption that is, before Adelberto had shot Jennifer which an air rifle. The Bundoc spouses contend that they were
therefore free of any parental responsibility for Adelberto's allegedly tortious conduct.
Respondent Bundoc spouses rely on Article 36 of the Child and Youth Welfare Code

8 which reads as follows:

Art. 36. Decree of Adoption. If, after considering the report of the Department of Social Welfare or duly licensed
child placement agency and the evidence submitted before it, the court is satisfied that the petitioner is qualified to
maintain, care for, and educate the child, that the trial custody period has been completed, and that the best
interests of the child will be promoted by the adoption, a decree of adoption shall be entered, which shall be
effective he date the original petition was filed. The decree shall state the name by which the child is thenceforth to
be known. (Emphasis supplied)
The Bundoc spouses further argue that the above Article 36 should be read in relation to Article 39 of the same Code:
Art. 39. Effect of Adoption. The adoption shall:
xxx xxx xxx
(2) Dissolve the authority vested in the natural parents, except where the adopter is the spouse of the surviving
natural parent;
xxx xxx xxx
(Emphasis supplied)
and urge that their Parental authority must be deemed to have been dissolved as of the time the Petition for adoption was filed.
The Court is not persuaded. As earlier noted, under the Civil Code, the basis of parental liability for the torts of a minor child is
the relationship existing between the parents and the minor child living with them and over whom, the law presumes, the
parents exercise supervision and control. Article 58 of the Child and Youth Welfare Code, re-enacted this rule:
Article 58 Torts Parents and guardians are responsible for the damage caused by the child under their parental
authority in accordance with the civil Code. (Emphasis supplied)
Article 221 of the Family Code of the Philippines

has similarly insisted upon the requisite that the child, doer of the tortious act, shall have beer in the actual
custody of the parents sought to be held liable for the ensuing damage:

Art. 221. Parents and other persons exercising parental authority shall be civilly liable for the injuries and damages
caused by the acts or omissions of their unemancipated children living in their companyand under their parental
authority subject to the appropriate defenses provided by law. (Emphasis supplied)
We do not believe that parental authority is properly regarded as having been retroactively transferred to and vested in the
adopting parents, the Rapisura spouses, at the time the air rifle shooting happened. We do not consider that retroactive effect
may be giver to the decree of adoption so as to impose a liability upon the adopting parents accruing at a time when adopting
parents had no actual or physically custody over the adopted child. Retroactive affect may perhaps be given to the granting of
the petition for adoption where such is essential to permit the accrual of some benefit or advantage in favor of the adopted child.
In the instant case, however, to hold that parental authority had been retroactively lodged in the Rapisura spouses so as to
burden them with liability for a tortious act that they could not have foreseen and which they could not have prevented (since
they were at the time in the United States and had no physical custody over the child Adelberto) would be unfair and
unconscionable. Such a result, moreover, would be inconsistent with the philosophical and policy basis underlying the doctrine
of vicarious liability. Put a little differently, no presumption of parental dereliction on the part of the adopting parents, the
Rapisura spouses, could have arisen since Adelberto was not in fact subject to their control at the time the tort was committed.
Article 35 of the Child and Youth Welfare Code fortifies the conclusion reached above. Article 35 provides as follows:
Art. 35. Trial Custody. No petition for adoption shall be finally granted unless and until the adopting parents are
given by the courts a supervised trial custody period of at least six months to assess their adjustment and emotional
readiness for the legal union. During the period of trial custody, parental authority shall be vested in the adopting
parents. (Emphasis supplied)
Under the above Article 35, parental authority is provisionally vested in the adopting parents during the period of trial
custody, i.e., before the issuance of a decree of adoption, precisely because the adopting parents are given actual custody of
the child during such trial period. In the instant case, the trial custody period either had not yet begun or bad already been
completed at the time of the air rifle shooting; in any case, actual custody of Adelberto was then with his natural parents, not the
adopting parents.
Accordingly, we conclude that respondent Bundoc spouses, Adelberto's natural parents, were indispensable parties to the suit
for damages brought by petitioners, and that the dismissal by the trial court of petitioners' complaint, the indispensable parties
being already before the court, constituted grave abuse of discretion amounting to lack or excess of jurisdiction.
WHEREFORE, premises considered, the Petition for Review is hereby GRANTED DUE COURSE and the Decision of the Court
of Appeals dated 6 September 1988, in C.A.-G.R. No. SP-15016 is hereby REVERSED and SET ASIDE. Petitioners' complaint
filed before the trial court is hereby REINSTATED and this case is REMANDED to that court for further proceedings consistent

G.R. No. 164948

June 27, 2006

DIWATA RAMOS LANDINGIN Petitioner,


vs.
REPUBLIC OF THE PHILIPPINES, Respondent.
DECISION
CALLEJO, SR., J.:
Assailed in this petition for review on certiorari under Rule 45 of the Rules of Court is the Decision 1 of the Court of Appeals in
CA-G.R. CV No. 77826 which reversed the Decision 2 of the Regional Trial Court (RTC) of Tarlac City, Branch 63 in Civil Case
No. 2733 granting the Petition for Adoption of the petitioner herein.
The Antecedents
On February 4, 2002, Diwata Ramos Landingin, a citizen of the United States of America (USA), of Filipino parentage and a
resident of Guam, USA, filed a petition3 for the adoption of minors Elaine Dizon Ramos who was born on August 31,
1986;4 Elma Dizon Ramos, who was born on September 7, 1987; 5 and Eugene Dizon Ramos who was born on August 5,
1989.6 The minors are the natural children of Manuel Ramos, petitioners brother, and Amelia Ramos.
Landingin, as petitioner, alleged in her petition that when Manuel died on May 19, 1990, 7 the children were left to their paternal
grandmother, Maria Taruc Ramos; their biological mother, Amelia, went to Italy, re-married there and now has two children by
her second marriage and no longer communicated with her children by Manuel Ramos nor with her in-laws from the time she
left up to the institution of the adoption; the minors are being financially supported by the petitioner and her children, and
relatives abroad; as Maria passed away on November 23, 2000, petitioner desires to adopt the children; the minors have given
their written consent8 to the adoption; she is qualified to adopt as shown by the fact that she is a 57-year-old widow, has children
of her own who are already married, gainfully employed and have their respective families; she lives alone in her own home in
Guam, USA, where she acquired citizenship, and works as a restaurant server. She came back to the Philippines to spend time
with the minors; her children gave their written consent 9 to the adoption of the minors. Petitioners brother, Mariano Ramos, who
earns substantial income, signified his willingness and commitment to support the minors while in petitioners custody.
Petitioner prayed that, after due hearing, judgment be rendered in her favor, as follows:
WHEREFORE, it is most respectfully prayed to this Honorable Court that after publication and hearing, judgment be rendered
allowing the adoption of the minor children Elaine Dizon Ramos, Elma Dizon Ramos, and Eugene Dizon Ramos by the
petitioner, and ordering that the minor childrens name follow the family name of petitioner.
Petitioner prays for such other reliefs, just and equitable under the premises. 10
On March 5, 2002, the court ordered the Department of Social Welfare and Development (DSWD) to conduct a case study as
mandated by Article 34 of Presidential Decree No. 603, as amended, and to submit a report thereon not later than April 4, 2002,
the date set for the initial hearing of the petition. 11 The Office of the Solicitor General (OSG) entered its appearance 12 but
deputized the City Prosecutor of Tarlac to appear in its behalf. 13 Since her petition was unopposed, petitioner was allowed to
present her evidence ex parte.14
The petitioner testified in her behalf. She also presented Elaine Ramos, the eldest of the adoptees, to testify on the written
consent executed by her and her siblings.15 The petitioner marked in evidence the Affidavit of Consent purportedly executed by
her children Ann, Errol, Dennis and Ricfel Branitley, all surnamed Landingin, and notarized by a notary public in Guam, USA, as
proof of said consent.16
On May 24, 2002, Elizabeth Pagbilao, Social Welfare Officer II of the DSWD, Field Office III, Tarlac, submitted a Child Study
Report, with the following recommendation:
In view of the foregoing, undersigned finds minors Elaine, Elma & Eugene all surnamed Ramos, eligible for adoption because of
the following reasons:
1. Minors surviving parent, the mother has voluntarily consented to their adoption by the paternal aunt, Diwata Landingin
this is in view of her inability to provide the parental care, guidance and support they need. An Affidavit of Consent was
executed by the mother which is hereto attached.
2. The three minors subject for adoption have also expressed their willingness to be adopted and joins the petitioners in
Guam, USA in the future. A joint Affidavit of consent is hereto attached. The minors developed close attachment to the
petitioners and they regarded her as second parent.

3. The minors are present under the care of a temporary guardian who has also family to look after. As young adolescents
they really need parental love, care, guidance and support to ensure their protection and well being.
In view of the foregoing, it is hereby respectfully recommended that minors Elaine D. Ramos, Elma D. Ramos and Eugene D.
Ramos be adopted by their maternal aunt Diwata Landingin. Trial custody is hereby further recommended to be dispensed with
considering that they are close relatives and that close attachments was already developed between the petitioner and the 3
minors.17
Pagbilao narrated what transpired during her interview, as follows:
The mother of minors came home together with her son John Mario, this May 2002 for 3 weeks vacation. This is to enable her
appear for the personal interview concerning the adoption of her children.
The plan for the adoption of minors by their paternal aunt Diwata Landingin was conceived after the death of their paternal
grandmother and guardian. The paternal relatives including the petitioner who attended the wake of their mother were very
much concerned about the well-being of the three minors. While preparing for their adoption, they have asked a cousin who has
a family to stay with minors and act as their temporary guardian.
The mother of minors was consulted about the adoption plan and after weighing the benefits of adoption to her children, she
voluntarily consented. She realized that her children need parental love, guidance and support which she could not provide as
she already has a second family & residing in Italy. Knowing also that the petitioners & her children have been supporting her
children up to the present and truly care for them, she believes her children will be in good hands. She also finds petitioners in a
better position to provide a secured and bright future to her children. 18
However, petitioner failed to present Pagbilao as witness and offer in evidence the voluntary consent of Amelia Ramos to the
adoption; petitioner, likewise, failed to present any documentary evidence to prove that Amelia assents to the adoption.
On November 23, 2002, the court, finding merit in the petition for adoption, rendered a decision granting said petition. The
dispositive portion reads:
WHEREFORE, it is hereby ordered that henceforth, minors Elaine Dizon Ramos, Elma Dizon Ramos, Eugene Dizon Ramos be
freed from all legal obligations obedience and maintenance from their natural parents and that they be declared for all legal
intents and purposes the children of Diwata Ramos Landingin. Trial custody is dispensed with considering that parent-children
relationship has long been established between the children and the adoptive parents. Let the surnames of the children be
changed from "Dizon-Ramos" to "Ramos-Landingin."
Let a copy of this decision be furnished the Local Civil Registrar of Tarlac, Tarlac for him to effect the corresponding
changes/amendment in the birth certificates of the above-mentioned minors.
SO ORDERED.19
The OSG appealed20 the decision to the Court of Appeals on December 2, 2002. In its brief 21 for the oppositor-appellant, the
OSG raised the following arguments:
I
THE TRIAL COURT ERRED IN GRANTING THE PETITION FOR ADOPTION DESPITE THE LACK OF CONSENT OF THE
PROPOSED ADOPTEES BIOLOGICAL MOTHER.
II
THE TRIAL COURT ERRED IN GRANTING THE PETITION FOR ADOPTION DESPITE THE LACK OF THE WRITTEN
CONSENT OF THE PETITIONERS CHILDREN AS REQUIRED BY LAW.
III
THE TRIAL COURT ERRED IN GRANTING THE PETITION FOR ADOPTION DESPITE PETITIONERS FAILURE TO
ESTABLISH THAT SHE IS IN A POSITION TO SUPPORT THE PROPOSED ADOPTEES.
On April 29, 2004, the CA rendered a decision22 reversing the ruling of the RTC. It held that petitioner failed to adduce in
evidence the voluntary consent of Amelia Ramos, the childrens natural mother. Moreover, the affidavit of consent of the
petitioners children could not also be admitted in evidence as the same was executed in Guam, USA and was not authenticated
or acknowledged before a Philippine consular office, and although petitioner has a job, she was not stable enough to support
the children. The dispositive portion of the CA decision reads:
WHEREFORE, premises considered, the appealed decision dated November 25, 2002 of the Regional Trial Court, Branch 63,
Tarlac City in Spec. Proc. No. 2733 is hereby REVERSED and SET ASIDE.
SO ORDERED.23
Petitioner filed a Motion for Reconsideration 24 on May 21, 2004, which the CA denied in its Resolution dated August 12, 2004. 25
Petitioner, thus, filed the instant petition for review on certiorari 26 on September 7, 2004, assigning the following errors:
1. THAT THE HONORABLE LOWER COURT HAS OVERLOOKED AND MISAPPLIED SOME FACTS AND
CIRCUMSTANCES WHICH ARE OF WEIGHT AND IMPORTANCE AND WHICH IF CONSIDERED WOULD HAVE
AFFECTED THE RESULT OF THE CASE.
2. THAT THE HONORABLE LOWER COURT ERRED IN CONCLUDING THAT THE PETITIONER-APPELLEE IS NOT
FINANCIALLY CAPABLE TO SUPPORT THE THREE CHILDREN.27
The issues raised by the parties in their pleadings are the following: (a) whether the petitioner is entitled to adopt the minors
without the written consent of their biological mother, Amelia Ramos; (b) whether or not the affidavit of consent purportedly
executed by the petitioner-adopters children sufficiently complies with the law; and (c) whether or not petitioner is financially
capable of supporting the adoptees.
The Courts Ruling
The petition is denied for lack of merit.
It has been the policy of the Court to adhere to the liberal concept, as stated in Malkinson v. Agrava, 28 that adoption statutes,
being humane and salutary, hold the interest and welfare of the child to be of paramount consideration and are designed to
provide homes, parental care and education for unfortunate, needy or orphaned children and give them the protection of society
and family in the person of the adopter as well as to allow childless couples or persons to experience the joys of parenthood
and give them legally a child in the person of the adopted for the manifestation of their natural parental instincts. Every
reasonable intendment should thus be sustained to promote and fulfill these noble and compassionate objectives of the law. 29

However, in Cang v. Court of Appeals,30 the Court also ruled that the liberality with which this Court treats matters leading to
adoption insofar as it carries out the beneficent purposes of the law to ensure the rights and privileges of the adopted child
arising therefrom, ever mindful that the paramount consideration is the overall benefit and interest of the adopted child, should
be understood in its proper context and perspective. The Courts position should not be misconstrued or misinterpreted as to
extend to inferences beyond the contemplation of law and jurisprudence. Thus, the discretion to approve adoption proceedings
is not to be anchored solely on best interests of the child but likewise, with due regard to the natural rights of the parents over
the child.31
Section 9 of Republic Act No. 8552, otherwise known as the Domestic Adoption Act of 1998, provides:
Sec. 9. Whose Consent is Necessary to the Adoption. - After being properly counseled and informed of his/her right to give or
withhold his/her approval of the adoption, the written consent of the following to the adoption is hereby required:
(a) The adoptee, if ten (10) years of age or over;
(b) The biological parent(s) of the child, if known, or the legal guardian, or the proper government instrumentality which
has legal custody of the child;
(c) The legitimate and adopted sons/daughters, ten (10) years of age or over, of the adopter(s) and adoptee, if any;
(d) The illegitimate sons/daughters, ten (10) years of age or over, of the adopter, if living with said adopter and the latters
souse, if any;
(e) The spouse, if any, of the person adopting or to be adopted.
The general requirement of consent and notice to the natural parents is intended to protect the natural parental relationship from
unwarranted interference by interlopers, and to insure the opportunity to safeguard the best interests of the child in the manner
of the proposed adoption.32
Clearly, the written consent of the biological parents is indispensable for the validity of a decree of adoption. Indeed, the natural
right of a parent to his child requires that his consent must be obtained before his parental rights and duties may be terminated
and re-established in adoptive parents. In this case, petitioner failed to submit the written consent of Amelia Ramos to the
adoption.
We note that in her Report, Pagbilao declared that she was able to interview Amelia Ramos who arrived in the Philippines with
her son, John Mario in May 2002. If said Amelia Ramos was in the Philippines and Pagbilao was able to interview her, it is
incredible that the latter would not require Amelia Ramos to execute a Written Consent to the adoption of her minor children.
Neither did the petitioner bother to present Amelia Ramos as witness in support of the petition.
Petitioner, nonetheless, argues that the written consent of the biological mother is no longer necessary because when Amelias
husband died in 1990, she left for Italy and never came back. The children were then left to the guidance and care of their
paternal grandmother. It is the paternal relatives, including petitioner, who provided for the childrens financial needs. Hence,
Amelia, the biological mother, had effectively abandoned the children. Petitioner further contends that it was by twist of fate that
after 12 years, when the petition for adoption was pending with the RTC that Amelia and her child by her second marriage were
on vacation in the Philippines. Pagbilao, the DSWD social worker, was able to meet her, and during the meeting, Amelia
intimated to the social worker that she conformed to the adoption of her three children by the petitioner.
Petitioners contention must be rejected. When she filed her petition with the trial court, Rep. Act No. 8552 was already in effect.
Section 9 thereof provides that if the written consent of the biological parents cannot be obtained, the written consent of the
legal guardian of the minors will suffice. If, as claimed by petitioner, that the biological mother of the minors had indeed
abandoned them, she should, thus have adduced the written consent of their legal guardian.
Ordinarily, abandonment by a parent to justify the adoption of his child without his consent, is a conduct which evinces a settled
purpose to forego all parental duties.33 The term means neglect and refusal to perform the filial and legal obligations of love and
support. If a parent withholds presence, love, care, the opportunity to display filial affection, and neglects to lend support and
maintenance, the parent, in effect, abandons the child. 34
Merely permitting the child to remain for a time undisturbed in the care of others is not such an abandonment. 35To dispense with
the requirement of consent, the abandonment must be shown to have existed at the time of adoption. 36
In this case, petitioner relied solely on her testimony and that of Elaine Ramos to prove her claim that Amelia Ramos had
abandoned her children. Petitioners testimony on that matter follows:
Q Where is the mother of these three children now?
A She left for Italy on November 20, 1990, sir.
Q At the time when Amelia Ramos left for Italy, was there an instance where she communicated with the family?
A None, sir.
Q How about with her children?
A None, sir.
Q Do you know what place in Italy did she reside?
A I do not know, sir.
Q Did you receive any news about Amelia Ramos?
A What I know, sir, was that she was already married with another man.
Q From whom did you learn that?
A From others who came from Italy, sir.
Q Did you come to know whether she has children by her second marriage?
A Yes, sir, she got two kids.37
Elaine, the eldest of the minors, testified, thus:
Q Where is your mother now?
A In Italy, sir.
Q When did your mother left for Italy?

A After my father died, sir.


Q How old were you when your mother left for Italy in 1990?
A Two years old, sir.
Q At the time when your mother left for Italy, did your mother communicate with you?
A No, sir.38
However, the Home Study Report of the DSWD Social Worker also stated the following:
IV. Background of the Case:
xxxx
Since the mother left for Italy, minors siblings had been under the care and custody of their maternal grandmother. However,
she died in Nov. 2001 and an uncle, cousin of their deceased father now serves as their guardian. The petitioner, together with
her children and other relatives abroad have been supporting the minor children financially, even during the time that they were
still living with their natural parents. Their mother also sends financial support but very minimal. 39
xxxx
V. Background Information about the Minors Being Sought for Adoption:
xxxx
As the eldest she tries her best to be a role model to her younger siblings. She helps them in their lessons, works and has fun
with them. She also encourages openness on their problems and concerns and provides petty counseling. In serious problems
she already consult (sic) her mother and petitioner-aunt. 40
xxxx
In their 5 years of married life, they begot 3 children, herein minors, Amelia recalled that they had a happy and comfortable life.
After the death of her husband, her in-laws which include the petitioner had continued providing support for them. However
being ashamed of just depending on the support of her husbands relatives, she decided to work abroad. Her parents are also in
need of financial help as they are undergoing maintenance medication. Her parents mortgaged their farm land which she used
in going to Italy and worked as domestic helper.
When she left for Italy in November 1990, she entrusted her 3 children to the care & custody of her mother-in-law who returned
home for good, however she died on November 2000.
While working in Italy, she met Jun Tayag, a married man from Tarlac. They became live-in partners since 1995 and have a son
John Mario who is now 2 years old. The three of them are considered Italian residents. Amelia claimed that Mr. Tayag is
planning to file an annulment of his marriage and his wife is amenable to it. He is providing his legitimate family regular support.
Amelia also sends financial support ranging from P10,000-P15,000 a month through her parents who share minimal amount of
P3,000-P5,000 a month to his (sic) children. The petitioner and other paternal relatives are continuously providing support for
most of the needs & education of minors up to present. 41
Thus, when Amelia left for Italy, she had not intended to abandon her children, or to permanently sever their mother-child
relationship. She was merely impelled to leave the country by financial constraints. Yet, even while abroad, she did not
surrender or relinquish entirely her motherly obligations of rearing the children to her now deceased mother-in-law, for, as
claimed by Elaine herself, she consulted her mother, Amelia, for serious personal problems. Likewise, Amelia continues to send
financial support to the children, though in minimal amounts as compared to what her affluent in-laws provide.
Let it be emphasized, nevertheless, that the adoption of the minors herein will have the effect of severing all legal ties between
the biological mother, Amelia, and the adoptees, and that the same shall then be vested on the adopter. 42 It would thus be
against the spirit of the law if financial consideration were to be the paramount consideration in deciding whether to deprive a
person of parental authority over his/her children. More proof has to be adduced that Amelia has emotionally abandoned the
children, and that the latter will not miss her guidance and counsel if they are given to an adopting parent. 43 Again, it is the best
interest of the child that takes precedence in adoption.
Section 34, Rule 132 of the Rules of Court provides that the Court shall consider no evidence which has not been formally
offered. The purpose for which the evidence is offered must be specified. The offer of evidence is necessary because it is the
duty of the Court to rest its findings of fact and its judgment only and strictly upon the evidence offered by the parties. Unless
and until admitted by the court in evidence for the purpose or purposes for which such document is offered, the same is merely
a scrap of paper barren of probative weight. Mere identification of documents and the markings thereof as exhibits do not confer
any evidentiary weight on documents unless formally offered. 44
Petitioner failed to offer in evidence Pagbilaos Report and of the Joint Affidavit of Consent purportedly executed by her children;
the authenticity of which she, likewise, failed to prove. The joint written consent of petitioners children 45 was notarized on
January 16, 2002 in Guam, USA; for it to be treated by the Rules of Court in the same way as a document notarized in this
country it needs to comply with Section 2 of Act No. 2103, 46 which states:
Section 2. An instrument or document acknowledged and authenticated in a foreign country shall be considered authentic if the
acknowledgment and authentication are made in accordance with the following requirements:
(a) The acknowledgment shall be made before (1) an ambassador, minister, secretary of legation, charg d affaires,
consul, vice-consul, or consular agent of the Republic of the Philippines, acting within the country or place to which he is
accredited, or (2) a notary public or officer duly authorized by law of the country to take acknowledgments of instruments
or documents in the place where the act is done.
(b) The person taking the acknowledgment shall certify that the person acknowledging the instrument or document is
known to him, and that he is the same person who executed it, and acknowledged that the same is his free act and deed.
The certificate shall be under his official seal, if he is by law required to keep a seal, and if not, his certificate shall so
state. In case the acknowledgment is made before a notary public or an officer mentioned in subdivision (2) of the
preceding paragraph, the certificate of the notary public or the officer taking the acknowledgment shall be authenticated by
an ambassador, minister, secretary of legation, charg de affaires, consul, vice-consul, or consular agent of the Republic
of the Philippines, acting within the country or place to which he is accredited. The officer making the authentication shall
certify under his official seal that the person who took the acknowledgment was at the time duly authorized to act as
notary public or that he was duly exercising the functions of the office by virtue of which he assumed to act, and that as
such he had authority under the law to take acknowledgment of instruments or documents in the place where the
acknowledgment was taken, and that his signature and seal, if any, are genuine.

As the alleged written consent of petitioners legitimate children did not comply with the afore-cited law, the same can at best be
treated by the Rules as a private document whose authenticity must be proved either by anyone who saw the document
executed or written; or by evidence of the genuineness of the signature or handwriting of the makers. 47
Since, in the instant case, no further proof was introduced by petitioner to authenticate the written consent of her legitimate
children, the same is inadmissible in evidence.
In reversing the ruling of the RTC, the CA ruled that petitioner was not stable enough to support the children and is only relying
on the financial backing, support and commitment of her children and her siblings. 48 Petitioner contradicts this by claiming that
she is financially capable as she has worked in Guam for 14 years, has savings, a house, and currently earns $5.15 an hour
with tips of not less than $1,000.00 a month. Her children and siblings have likewise committed themselves to provide financial
backing should the need arise. The OSG, again in its comment, banks on the statement in the Home Study Report that
"petitioner has limited income." Accordingly, it appears that she will rely on the financial backing of her children and siblings in
order to support the minor adoptees. The law, however, states that it is the adopter who should be in a position to provide
support in keeping with the means of the family.
Since the primary consideration in adoption is the best interest of the child, it follows that the financial capacity of prospective
parents should also
be carefully evaluated and considered. Certainly, the adopter should be in a position to support the would-be adopted child or
children, in keeping with the means of the family.
According to the Adoption Home Study Report 49 forwarded by the Department of Public Health & Social Services of the
Government of Guam to the DSWD, petitioner is no longer supporting her legitimate children, as the latter are already adults,
have individual lives and families. At the time of the filing of the petition, petitioner was 57 years old, employed on a part-time
basis as a waitress, earning $5.15 an hour and tips of around $1,000 a month. Petitioners main intention in adopting the
children is to bring the latter to Guam, USA. She has a house at Quitugua Subdivision in Yigo, Guam, but the same is still being
amortized. Petitioner likewise knows that the limited income might be a hindrance to the adoption proceedings.
Given these limited facts, it is indeed doubtful whether petitioner will be able to sufficiently handle the financial aspect of rearing
the three children in the US. She only has a part-time job, and she is rather of age. While petitioner claims that she has the
financial support and backing of her children and siblings, the OSG is correct in stating that the ability to support the adoptees is
personal to the adopter, as adoption only creates a legal relation between the former and the latter. Moreover, the records do
not prove nor support petitioners allegation that her siblings and her children are financially able and that they are willing to
support the minors herein. The Court, therefore, again sustains the ruling of the CA on this issue.
While the Court recognizes that petitioner has only the best of intentions for her nieces and nephew, there are legal infirmities
that militate against reversing the ruling of the CA. In any case, petitioner is not prevented from filing a new petition for adoption
of the herein minors.

IN RE: PETITION FOR


ADOPTION OF MICHELLE P.
LIM,

G.R. Nos. 168992-93


Present:

MONINA P. LIM,

PUNO, C.J., Chairperson,


Petitioner.
CARPIO,
x - - - - - - - - - - - - - - - - - - - - - - - x CORONA,
LEONARDO-DE CASTRO, and
IN RE: PETITION FOR
BERSAMIN, JJ.
ADOPTION OF MICHAEL JUDE
P. LIM,
Promulgated:
MONINA P. LIM,
Petitioner.
May 21, 2009
x - - - - - - - - - - - - - - - - - - - - - - - - - - - - - - - - - - - - - - - - - - - - - - - - - -x

DECISION
CARPIO, J.:
The Case
This is a petition for review on certiorari filed by Monina P. Lim (petitioner) seeking to set aside the Decision [1] dated 15 September
2004 of the Regional Trial Court, General Santos City, Branch 22 (trial court), in SPL. PROC. Case Nos. 1258 and 1259, which dismissed
without prejudice the consolidated petitions for adoption of Michelle P. Lim and Michael Jude P. Lim.

The Facts
The following facts are undisputed. Petitioner is an optometrist by profession. On 23 June 1974, she married Primo Lim (Lim). They
were childless. Minor children, whose parents were unknown, were entrusted to them by a certain Lucia Ayuban (Ayuban). Being so eager to
have a child of their own, petitioner and Lim registered the children to make it appear that they were the childrens parents. The
children[2] were named Michelle P. Lim (Michelle) and Michael Jude P. Lim (Michael). Michelle was barely eleven days old when brought to
the clinic of petitioner. She was born on 15 March 1977. [3] Michael was 11 days old when Ayuban brought him to petitioners clinic. His date
of birth is 1 August 1983.[4]
The spouses reared and cared for the children as if they were their own. They sent the children to exclusive schools. They used the
surname Lim in all their school records and documents. Unfortunately, on 28 November 1998, Lim died. On 27 December 2000, petitioner
married Angel Olario (Olario), an American citizen.
Thereafter, petitioner decided to adopt the children by availing of the amnesty [5] given under Republic Act No. 8552[6] (RA 8552) to
those individuals who simulated the birth of a child. Thus, on 24 April 2002, petitioner filed separate petitions for the adoption of Michelle
and Michael before the trial court docketed as SPL PROC. Case Nos. 1258 and 1259, respectively. At the time of the filing of the petitions
for adoption, Michelle was 25 years old and already married, while Michael was 18 years and seven months old.
Michelle and her husband gave their consent to the adoption as evidenced by their Affidavits of Consent. [7] Michael also gave his
consent to his adoption as shown in his Affidavit of Consent. [8] Petitioners husband Olario likewise executed an Affidavit of Consent [9] for
the adoption of Michelle and Michael.

In the Certification issued by the Department of Social Welfare and Development (DSWD), Michelle was considered as an abandoned
child and the whereabouts of her natural parents were unknown.[10] The DSWD issued a similar Certification for Michael.[11]

The Ruling of the Trial Court


On 15 September 2004, the trial court rendered judgment dismissing the petitions. The trial court ruled that since petitioner had
remarried, petitioner should have filed the petition jointly with her new husband. The trial court ruled that joint adoption by the husband and
the wife is mandatory citing Section 7(c), Article III of RA 8552 and Article 185 of the Family Code.
Petitioner filed a Motion for Reconsideration of the decision but the motion was denied in the Order dated 16 June 2005. In denying
the motion, the trial court ruled that petitioner did not fall under any of the exceptions under Section 7(c), Article III of RA 8552. Petitioners
argument that mere consent of her husband would suffice was untenable because, under the law, there are additional requirements, such as
residency and certification of his qualification, which the husband, who was not even made a party in this case, must comply.
As to the argument that the adoptees are already emancipated and joint adoption is merely for the joint exercise of parental authority,
the trial court ruled that joint adoption is not only for the purpose of exercising parental authority because an emancipated child acquires
certain rights from his parents and assumes certain obligations and responsibilities.
Hence, the present petition.

Issue
Petitioner appealed directly to this Court raising the sole issue of whether or not petitioner, who has remarried, can singly adopt.

The Courts Ruling


Petitioner contends that the rule on joint adoption must be relaxed because it is the duty of the court and the State to protect the
paramount interest and welfare of the child to be adopted. Petitioner argues that the legal maxim dura lex sed lex is not applicable to
adoption cases. She argues that joint parental authority is not necessary in this case since, at the time the petitions were filed, Michelle was
25 years old and already married, while Michael was already 18 years of age. Parental authority is not anymore necessary since they have
been emancipated having attained the age of majority.
We deny the petition.
Joint Adoption by Husband and Wife
It is undisputed that, at the time the petitions for adoption were filed, petitioner had already remarried. She filed the petitions by herself,
without being joined by her husband Olario. We have no other recourse but to affirm the trial courts decision denying the petitions for
adoption. Dura lex sed lex. The law is explicit. Section 7, Article III of RA 8552 reads:
SEC. 7. Who May Adopt. - The following may adopt:
(a) Any Filipino citizen of legal age, in possession of full civil capacity and legal rights, of good moral character, has not been
convicted of any crime involving moral turpitude, emotionally and psychologically capable of caring for children, at least sixteen
(16) years older than the adoptee, and who is in a position to support and care for his/her children in keeping with the means of the
family. The requirement of sixteen (16) year difference between the age of the adopter and adoptee may be waived when the adopter
is the biological parent of the adoptee, or is the spouse of the adoptees parent;
(b) Any alien possessing the same qualifications as above stated for Filipino nationals: Provided, That his/her country has
diplomatic relations with the Republic of the Philippines, that he/she has been living in the Philippines for at least three (3)
continuous years prior to the filing of the application for adoption and maintains such residence until the adoption decree is entered,
that he/she has been certified by his/her diplomatic or consular office or any appropriate government agency that he/she has the legal
capacity to adopt in his/her country, and that his/her government allows the adoptee to enter his/her country as his/her adopted
son/daughter: Provided, further, That the requirements on residency and certification of the aliens qualification to adopt in his/her
country may be waived for the following:
(i) a former Filipino citizen who seeks to adopt a relative within the fourth (4 th) degree of consanguinity or affinity; or
(ii) one who seeks to adopt the legitimate son/daughter of his/her Filipino spouse; or
(iii) one who is married to a Filipino citizen and seeks to adopt jointly with his/her spouse a relative within the fourth
(4 ) degree of consanguinity or affinity of the Filipino spouses; or
th

(c) The guardian with respect to the ward after the termination of the guardianship and clearance of his/her financial
accountabilities.
Husband and wife shall jointly adopt, except in the following cases:
(i) if one spouse seeks to adopt the legitimate son/daughter of the other; or

(ii) if one spouse seeks to adopt his/her own illegitimate son/daughter: Provided, however, That the other spouse has
signified his/her consent thereto; or
(iii) if the spouses are legally separated from each other.
In case husband and wife jointly adopt, or one spouse adopts the illegitimate son/daughter of the other, joint parental
authority shall be exercised by the spouses. (Emphasis supplied)

The use of the word shall in the above-quoted provision means that joint adoption by the husband and the wife is mandatory. This is
in consonance with the concept of joint parental authority over the child which is the ideal situation. As the child to be adopted is elevated to
the level of a legitimate child, it is but natural to require the spouses to adopt jointly. The rule also insures harmony between the spouses. [12]
The law is clear. There is no room for ambiguity. Petitioner, having remarried at the time the petitions for adoption were filed, must
jointly adopt. Since the petitions for adoption were filed only by petitioner herself, without joining her husband, Olario, the trial court was
correct in denying the petitions for adoption on this ground.
Neither does petitioner fall under any of the three exceptions enumerated in Section 7. First, the children to be adopted are not the
legitimate children of petitioner or of her husband Olario. Second, the children are not the illegitimate children of petitioner. And third,
petitioner and Olario are not legally separated from each other.
The fact that Olario gave his consent to the adoption as shown in his Affidavit of Consent does not suffice. There are certain
requirements that Olario must comply being an American citizen. He must meet the qualifications set forth in Section 7 of RA 8552 such as:
(1) he must prove that his country has diplomatic relations with the Republic of the Philippines; (2) he must have been living in the
Philippines for at least three continuous years prior to the filing of the application for adoption; (3) he must maintain such residency until the
adoption decree is entered; (4) he has legal capacity to adopt in his own country; and (5) the adoptee is allowed to enter the adopters country
as the latters adopted child. None of these qualifications were shown and proved during the trial.
These requirements on residency and certification of the aliens qualification to adopt cannot likewise be waived pursuant to Section 7.
The children or adoptees are not relatives within the fourth degree of consanguinity or affinity of petitioner or of Olario. Neither are the
adoptees the legitimate children of petitioner.

Effects of Adoption
Petitioner contends that joint parental authority is not anymore necessary since the children have been emancipated having reached the
age of majority.

This is untenable.

Parental authority includes caring for and rearing the children for civic consciousness and efficiency and the development of their
moral, mental and physical character and well-being. [13] The father and the mother shall jointly exercise parental authority over the persons
of their common children.[14] Even the remarriage of the surviving parent shall not affect the parental authority over the children, unless the
court appoints another person to be the guardian of the person or property of the children.[15]
It is true that when the child reaches the age of emancipation that is, when he attains the age of majority or 18 years of
age[16] emancipation terminates parental authority over the person and property of the child, who shall then be qualified and responsible
for all acts of civil life.[17] However, parental authority is merely just one of the effects of legal adoption. Article V of RA 8552 enumerates
the effects of adoption, thus:
ARTICLE V
EFFECTS OF ADOPTION
SEC. 16. Parental Authority. - Except in cases where the biological parent is the spouse of the adopter, all legal ties between
the biological parent(s) and the adoptee shall be severed and the same shall then be vested on the adopter(s).
SEC. 17. Legitimacy. - The adoptee shall be considered the legitimate son/daughter of the adopter(s) for all intents and
purposes and as such is entitled to all the rights and obligations provided by law to legitimate sons/daughters born to them without
discrimination of any kind. To this end, the adoptee is entitled to love, guidance, and support in keeping with the means of the family.
SEC. 18. Succession. - In legal and intestate succession, the adopter(s) and the adoptee shall have reciprocal rights of
succession without distinction from legitimate filiation. However, if the adoptee and his/her biological parent(s) had left a will, the
law on testamentary succession shall govern.

Adoption has, thus, the following effects: (1) sever all legal ties between the biological parent(s) and the adoptee, except when the
biological parent is the spouse of the adopter; (2) deem the adoptee as a legitimate child of the adopter; and (3) give adopter and adoptee
reciprocal rights and obligations arising from the relationship of parent and child, including but not limited to: (i) the right of the adopter to
choose the name the child is to be known; and (ii) the right of the adopter and adoptee to be legal and compulsory heirs of each other.
[18]

Therefore, even if emancipation terminates parental authority, the adoptee is still considered a legitimate child of the adopter with all

the rights[19] of a legitimate child such as: (1) to bear the surname of the father and the mother; (2) to receive support from their parents; and
(3) to be entitled to the legitime and other successional rights. Conversely, the adoptive parents shall, with respect to the adopted child, enjoy
all the benefits to which biological parents are entitled[20] such as support[21] and successional rights.[22]

We are mindful of the fact that adoption statutes, being humane and salutary, hold the interests and welfare of the child to be of
paramount consideration. They are designed to provide homes, parental care and education for unfortunate, needy or orphaned children and
give them the protection of society and family, as well as to allow childless couples or persons to experience the joys of parenthood and give
them legally a child in the person of the adopted for the manifestation of their natural parental instincts. Every reasonable intendment should
be sustained to promote and fulfill these noble and compassionate objectives of the law.[23] But, as we have ruled in Republic v. Vergara:[24]
We are not unmindful of the main purpose of adoption statutes, which is the promotion of the welfare of the children.
Accordingly, the law should be construed liberally, in a manner that will sustain rather than defeat said purpose. The law must also be
applied with compassion, understanding and less severity in view of the fact that it is intended to provide homes, love, care and
education for less fortunate children. Regrettably, the Court is not in a position to affirm the trial courts decision favoring adoption in
the case at bar, for the law is clear and it cannot be modified without violating the proscription against judicial
legislation. Until such time however, that the law on the matter is amended, we cannot sustain the respondent-spouses petition for
adoption. (Emphasis supplied)

Petitioner, being married at the time the petitions for adoption were filed, should have jointly filed the petitions with her husband. We cannot
make our own legislation to suit petitioner.
Petitioner, in her Memorandum, insists that subsequent events would show that joint adoption could no longer be possible because
Olario has filed a case for dissolution of his marriage to petitioner in the Los Angeles Superior Court.
We disagree. The filing of a case for dissolution of the marriage between petitioner and Olario is of no moment. It is not equivalent to a
decree of dissolution of marriage. Until and unless there is a judicial decree for the dissolution of the marriage between petitioner and Olario,
the marriage still subsists. That being the case, joint adoption by the husband and the wife is required. We reiterate our ruling above that
since, at the time the petitions for adoption were filed, petitioner was married to Olario, joint adoption is mandatory.

Support
G.R. No. L-6706

March 29, 1953

ALFREDO JAVIER, petitioner,


vs.
HON. ANTONIO G. LUCERO, Judge of the Court of First Instance of Cavite; SALUD R. ARCA and ALFREDO JAVIER,
JR., respondents.
David F. Barrera for petitioner.
Jacinto, Santillan and Roxas for respondents.
BENGZON, J.:
In an action for alimony (Civil Case No. 5150, Cavite), the respondent judge, after hearing the parties and their evidence,
ordered Alfredo Javier to give a monthly allowance of P60 to his wife Salud R. Arca and their son Alfredo Javier, Jr.
On April 14, 1953 the husband filed a notice of appeal, and on May 6, 1953, he submitted the appeal bond and the record on
appeal. Meanwhile the wife and the son presented on April 30, 1953 a motion for "support pendente lite" even pending the final
determination of the case on appeal". Whereupon on May 8, 1953, the judge directed Alfredo Javier to pay the monthly
pensions notwithstanding the pendency of his appeal.
Here comes Alfredo Javier with a petition for certiorari challenging such directive and arguing, in his own words:
"1. The status of Salud R. Arca as wife of the petitioner is being contested;
"2. Alfredo Javier Jr. is over 21 years old on March 31, 1953 and no longer entitled to be supported; and
"3. Even granting that Alfredo Javier, Jr. is entitled to support even if over 21 years of age to complete his education or training
for some profession, trade or vocation, the support could not be paid because the decision is vague or silent on that point.
The facts, as found in the action for support, are these:
On November 19, 1937, plaintiff Salud R. Arca and defendant Alfredo Javier had their marriage solemnized by Judge
Mariano Nable of the Municipal Court of Manila. At the time of their marriage, they had already begotten a son named
Alfredo Javier Junior who was born on December 2, 1931. Sometime in 1938, defendant Alfredo Javier left for the United
States on board a ship of the United States navy, for it appears that he had joined the United States Navy since 1927,
such at the time of his marriage with plaintiff Salud R. Arca, defendant Alfredo Javier was already a enlisted man in the
United States Navy. Because of defendant Alfredo Javier's departure for the United States in 1938, his wife Salud R. Arca,
who is from Tanza, Cavite, chose to live with defendant's parents at Naic, Cavite. But for certain incompatibility of
character (frictions having occurred between plaintiff Salud R. Arca and defendant's folks) plaintiff Salud R. Arca had
found it necessary to leave defendant's parents' abode and transfer her residence to Tanza, Cavite her native place.
Since then the relation between plaintiff Salud R. Arca and defendant Alfredo Javier become strained such that on August
13, 1940 defendant Alfredo Javier brought an action for divorce against plaintiff Salud R. Arca before the Circuit Court of
Mobile County, State of Alabama, USA, docketed as Civil Case No. 14313 of that Court and marked as Exhibit 2 (c) in this
case. Having received a copy of the complaint for divorce on September 23, 1940, plaintiff Salud R. Arca answering the
complaint alleged in her answer that she received a copy of the complaint on September 23, 1940 although she was
directed to file her answer thereto on or before September 13, 1940. In that answer she filed, plaintiff Salud R. Arca
averred among other things that defendant Alfredo Javier was not a resident of Mobile County, State of Alabama, for the
period of twelve months preceding the institution of the complaint, but that he was a resident of Naic, Cavite, Philippines.
Another avernment of interest, which is essential to relate here, is that under paragraph 5 of her answer to the complaint
for divorce, Salud R. Arca alleged that it was not true that the cause of their separation was desertion on her part but that
if the defendant Alfredo Javier was in the United States at that time and she was not with him then it was because he was
in active duty as an enlisted man of the United States Navy, as a consequence of which he had to leave for the United
States without her. She further alleged that since his departure from the Philippines for the United States, he had always
supported her and her co-plaintiff Alfredo Javier Junior through allotments made by the Navy Department of the United
States Government. She denied, furthermore, the allegation that she had abandoned defendant's home at Naic, Cavite,
and their separation was due to physical impossibility for they were separated by about 10,000 miles from each other. At
this juncture, under the Old civil Code, the wife is not bound to live with her husband if the latter has gone to ultra-marine
colonies. Plaintiff Salud R. Arca, in her answer to the complaint for divorce by defendant Alfredo Javier, prayed that the
complaint for divorce be dismissed. However, notwithstanding Salud R. Arca's averments in her answer, contesting the
jurisdiction of the Circuit Court of Mobile County, State of Alabama, to take cognizance of the divorce proceeding filed by
defendant Alfredo Javier, as shown by her answer marked Exhibit 2 (d), nevertheless the Circuit Court of Mobile County
rendered judgment decreeing dissolution of the marriage of Salud R. Arca and Alfredo Javier, and granting the latter a
decree of divorce dated April 9, 1941, a certified copy of which is marked Exhibit 2(f). Thereupon, the evidence discloses
that some time in 1946 defendant Alfredo Javier returned to the Philippines but went back to the United States.
In July, 1941, that is after securing a divorce from plaintiff Salud R. Arca on April 9, 1941 defendant Alfredo Javier
married Thelma Francis, an American citizen and bought a house and lot at 248 Brooklyn, New York City. In 1949, Thelma
Francis, defendant's American wife, obtained a divorce from him for reasons not disclosed by the evidence, and later on,
having retired from the United States Navy, defendant Alfredo Javier returned to the Philippines, armed with two decrees

of divorce one against his first wife Salud R. Arca and the other against him by his second wife Thelma Francis
issued by the Circuit Court of Mobile County, State of Alabama, USA, defendant Alfredo Javier married Maria Odvina
before Judge Natividad Almeda-Lopez of the Municipal Court of Manila on April 19, 1950, marked Exhibit 2 (b).
At the instance of plaintiff Salud R. Arca an information for bigamy was filed by the City fiscal of manila on July 25, 1950
against defendant Alfredo Javier with the Court of First Instance of Manila, docketed as Criminal Case No. 13310 and
marked Exhibit 2 (a). However, defendant Alfredo Javier with the Court of First Instance of Manila was acquitted of the
charge of bigamy in a decision rendered by the Court of First Instance of Manila through Judge Alejandro J. Panlilio, dated
August 10, 1951, predicated on the proposition that the marriage of defendant Alfredo Javier with Maria Odvina was made
in all good faith and in the honest belief that his marriage with plaintiff Salud R. Arca had been legally dissolved by the
decree of divorce obtained by him from the Circuit Court of Mobile County, State of Alabama, USA, which had the legal
effect of dissolving the marital ties between defendant Alfredo Javier and plaintiff Salud R. Arca. At this juncture, again, it
is this Court's opinion that defendant Alfredo Javier's acquittal in that Criminal Case No. 13310 of the Court of First
Instance of Manila by Judge Panlilio was due to the fact that the accused had no criminal intent in contracting a second or
subsequent marriage while his first marriage was still subsisting.
Turning now to the petition for certiorari, we perceive that, as to its first ground the respondent judge declared in his decision
that Alfredo Javier and Salud Arca were married on November 19, 1937 when they had already a natural son named Alfredo
Javier Junior, born December 2, 1931, and that, notwithstanding a decree of divorce which the husband Alfredo obtained in the
United States in 1941, their marriage still subsists. Such being the situation, the principle in Francisco vs. Zandueta, 61 Phil.,
752 on which petitioner entirely relies is not controlling, inasmuch as the existence of the married relation and the paternity had
been established at least prima facie (cf. Sanchez vs. Zulueta, 68 Phil., 112.) Besides, as respondents point out, this is strictly
not alimony pendente lite, under Rule 63, but execution of judgment pending appeal, under Rule 39. 1
In connection with the second ground of the petition, respondents observe that under the new Civil Code, article 290 support
also includes the education of the person to be supported "until he complete his education or training for some profession, trade
or vocation even beyond the age of majority" and on the basis of this article support was granted to Alfredo Javier Junior. Said
the Court, "while it is true that plaintiff Alfredo Javier Junior, who was born on December 2, 1931, has reached the age of
majority on December 2, 1952, yet, under the last part of article 290 of the new Civil Code, support may be given him even
beyond the age of majority in order to enable him to complete his education, for some trade or profession."
Now then, was the order issued in excess of jurisdiction or with grave abuse of discretion? The court undoubtedly has
jurisdiction, inasmuch as it was issued before the record on appeal was submitted. (Sumulong vs. Imperial, 51 Phil., 251;
Syquia vs. Concepcion, 60 Phil., 186). Did the judge abuse his discretion?
Unquestionably, Alfredo Javier, Jr. is the son of petitioner Alfredo Javier, and if financial assistance is to be rendered only at the
termination of the appeal his education, or the completion thereof, would be unduly delayed. That is good reason for immediate
execution. Petitioner claims that according to the records Alfredo Javier Jr. "is no longer studying". Yet probably he stopped
going to school due to lack of means, since the petitioner himself admits that his son is just a pre-law graduate.
But the real grievance of petitioner is contained in the last portion of his pleading, which says, "What Alfredo Javier now tries to
avoid is to support a woman who has desperately tried to put him in jail, when she accused him of bigamy." Such disgust is
easily understandable. But compliance with legal and contractual duties is not always pleasant.
Under the New Civil Code articles 303 and 921 the wife forfeits her husband's support after "she has accused (him) of a crime
for which the law prescribes imprisonment for six years or more, and the accusation has been found to be false." Admittedly, he
married a third time without the first marriage having been dissolved; but he was cleared of the bigamy charge for lack of
criminal intent, inasmuch as he believed his divorce obtained in the U.S., had already ended his first marriage to Salud r. Arca.
Such acquittal is no different from an acquittal on reasonable doubt, which in our opinion, and in the opinion of a member of the
code Commission that framed the New Civil code, would not be ground to forfeit her right to support. 2
Of course, the question whether Alfredo Javier's prosecution for bigamy and subsequent acquittal extinguished his obligation to
maintain his complaining spouse will definitely be decided when the main case (No. 5150) is reviewed on appeal. Other aspects
of the issue could then undoubtedly be the subject of research and elucidation. Nevertheless, we briefly explain our first
impressions or provisional conclusion in the task of examining the alleged misuse by respondent judge of his prerogatives. It is
markworthy that the son has not forfeited his right to support.
As the issues are presently framed, petitioner has failed to sustain the burden of demonstrating the judge's clear error or
grievous mistake in ordering execution of his judgment pending appeal. Costs against petitioner.

G.R. No. 11263

November 2, 1916

ELOISA GOITIA DE LA CAMARA, plaintiff-appellant,


vs.
JOSE CAMPOS RUEDA, defendant-appellee.
Eduardo Gutierrez Repide and Felix Socias for appellant.
Sanz, Opisso and Luzuriaga for appellee.

TRENT, J.:
This is an action by the wife against her husband for support outside of the conjugal domicile. From a judgment sustaining the
defendant's demurrer upon the ground that the facts alleged in the complaint do not state a cause of action, followed by an
order dismissing the case after the plaintiff declined to amend, the latter appealed.
It was urged in the first instance, and the court so held, that the defendant cannot be compelled to support the plaintiff, except in
his own house, unless it be by virtue of a judicial decree granting her a divorce or separation from the defendant.
The parties were legally married in the city of Manila on January 7, 1915, and immediately thereafter established their residence
at 115 Calle San Marcelino, where they lived together for about a month, when the plaintiff returned to the home of her parents.
The pertinent allegations of the complaint are as follows:
That the defendant, one month after he had contracted marriage with the plaintiff, demanded of her that she perform
unchaste and lascivious acts on his genital organs; that the plaintiff spurned the obscene demands of the defendant and
refused to perform any act other than legal and valid cohabitation; that the defendant, since that date had continually on
other successive dates, made similar lewd and indecorous demands on his wife, the plaintiff, who always spurned them,
which just refusals of the plaintiff exasperated the defendant and induce him to maltreat her by word and deed and inflict
injuries upon her lips, her face and different parts of her body; and that, as the plaintiff was unable by any means to induce
the defendant to desist from his repugnant desires and cease from maltreating her, she was obliged to leave the conjugal
abode and take refuge in the home of her parents.
Marriage in this jurisdiction is a contract entered into in the manner and with the solemnities established by General Orders No.
68, in so far as its civil effects are concerned requiring the consent of the parties. (Garcia vs. Montague, 12 Phil. Rep., 480,
citing article 1261 of Civil Code.) Upon the termination of the marriage ceremony, a conjugal partnership is formed between the
parties. (Sy Joc Lieng vs. Encarnacion, 16 Phil. Rep., 137.) To this extent a marriage partakes of the nature of an ordinary
contract. But it is something more than a mere contract. It is a new relation, the rights, duties, and obligations of which rest not
upon the agreement of the parties but upon the general law which defines and prescribes those rights, duties, and
obligations .Marriage is an institution, in the maintenance of which in its purity the public is deeply interested. It is a relation for
life and the parties cannot terminate it at any shorter period by virtue of any contract they may make .The reciprocal rights
arising from this relation, so long as it continues, are such as the law determines from time to time, and none other. When the
legal existence of the parties is merged into one by marriage, the new relation is regulated and controlled by the state or
government upon principles of public policy for the benefit of society as well as the parties. And when the object of a marriage is
defeated by rendering its continuance intolerable to one of the parties and productive of no possible good to the community,
relief in some way should be obtainable. With these principles to guide us, we will inquire into the status of the law touching and
governing the question under consideration.
Articles 42 to 107 of the Civil Code are not in force in the Philippine Islands (Benedicto vs. De la Rama, 3 Phil .Rep., 34).
Articles 44 to 78 of the Law of Civil Marriage of 1870, in force in the Peninsula, were extended to the Philippine Islands by royal
decree on April 13, 1883 (Ebreo vs. Sichon, 4 Phil. Rep., 705). Articles 44, 45, and 48 of this law read:
ART. 44. The spouses are obliged to be faithful to each other and to mutually assist each other.
ART. 45. The husband must live with and protect his wife. (The second paragraph deals with the management of the
wife's property.)
ART. 48. The wife must obey her husband, live with him, and follow him when he charges his domicile or residence.
Notwithstanding the provisions of the foregoing paragraph, the court may for just cause relieve her from this duty when the
husband removes his residence to a foreign country.
And articles 143 and 149 of the Civil Code are as follows:
ART. 143. The following are obliged to support each other reciprocally to the whole extent specified in the preceding
article.
1. The consorts.
xxx

xxx

xxx

ART. (149) 49. The person obliged to give support may, at his option, satisfy it, either by paying the pension that may be
fixed or by receiving and maintaining in his own home the person having the right to the same.

Article 152 of the Civil Code gives the instances when the obligation to give support shall cease. The failure of the wife to live
with her husband is not one of them.
The above quoted provisions of the Law of Civil Marriage and the Civil Code fix the duties and obligations of the spouses. The
spouses must be faithful to, assist, and support each other. The husband must live with and protect his wife. The wife must obey
and live with her husband and follow him when he changes his domicile or residence, except when he removes to a foreign
country. But the husband who is obliged to support his wife may, at his option, do so by paying her a fixed pension or by
receiving and maintaining her in his own home. May the husband, on account of his conduct toward his wife, lose this option
and be compelled to pay the pension? Is the rule established by article 149 of the Civil Code absolute? The supreme court of
Spain in its decision of December 5, 1903, held:.
That in accordance with the ruling of the supreme court of Spain in its decisions dated May 11, 1897, November 25, 1899,
and July 5, 1901, the option which article 149 grants the person, obliged to furnish subsistence, between paying the
pension fixed or receiving and keeping in his own house the party who is entitled to the same, is not so absolute as to
prevent cases being considered wherein, either because this right would be opposed to the exercise of a preferential right
or because of the existence of some justifiable cause morally opposed to the removal of the party enjoying the
maintenance, the right of selection must be understood as being thereby restricted.
Whereas the only question discussed in the case which gave rise to this appeal was whether there was any reason to
prevent the exercise of the option granted by article 149 of the Civil Code to the person obliged to furnish subsistence, to
receive and maintain in his own house the one who is entitled to receive it; and inasmuch as nothing has been alleged or
discussed with regard to the parental authority of Pedro Alcantara Calvo, which he ha not exercised, and it having been
set forth that the natural father simply claims his child for the purpose of thus better attending to her maintenance, no
action having been taken by him toward providing the support until, owing to such negligence, the mother was obliged to
demand it; it is seen that these circumstances, together with the fact of the marriage of Pedro Alcantara, and that it would
be difficult for the mother to maintain relations with her daughter, all constitute an impediment of such a nature as to
prevent the exercise of the option in the present case, without prejudice to such decision as may be deemed proper with
regard to the other questions previously cited in respect to which no opinion should be expressed at this time.
The above was quoted with approval in United States and De Jesus vs. Alvir (9 Phil. Rep., 576), wherein the court held that the
rule laid down in article 149 of the Civil Code "is not absolute." but it is insisted that there existed a preexisting or preferential
right in each of these cases which was opposed to the removal of the one entitled to support. It is true that in the first the person
claiming the option was the natural father of the child and had married a woman other than the child's mother, and in the second
the right to support had already been established by a final judgment in a criminal case. Notwithstanding these facts the two
cases clearly established the proposition that the option given by article 149 of the Civil Code may not be exercised in any and
all cases.
Counsel for the defendant cite, in support of their contention, the decision of the supreme court of Spain, dated November 3,
1905. In this case Don Berno Comas, as a result of certain business reverses and in order no to prejudice his wife, conferred
upon her powers to administer and dispose of her property. When she left him he gave her all the muniments of title, mortgage
credits, notes, P10,000 in accounts receivable, and the key to the safe in which he kept a large amount of jewels, thus depriving
himself of all his possessions and being reduced in consequence to want. Subsequently he instituted this civil action against his
wife, who was then living in opulence, for support and the revocation of the powers heretofore granted in reference to the
administration and disposal of her property. In her answer the wife claimed that the plaintiff (her husband) was not legally in a
situation to claim support and that the powers voluntarily conferred and accepted by her were bilateral and could not be
canceled by the plaintiff. From a judgment in favor of the plaintiff the defendant wife appealed to the Audencia Territorialwherein,
after due trial, judgment was rendered in her favor dismissing the action upon the merits. The plaintiff appealed to the supreme
court and that high tribunal, in affirming the judgment of the Audencia Territorial, said:
Considering that article 143, No. 1, of the Civil Code, providing that the spouses are mutually obliged to provide each
other with support, cannot but be subordinate to the other provisions of said Code which regulates the family organization
and the duties of spouses not legally separated, among which duties are those of their living together and mutually helping
each other, as provided in article 56 of the aforementioned code; and taking this for granted, the obligation of the spouse
who has property to furnish support to the one who has no property and is in need of it for subsistence, is to be
understood as limited to the case where, in accordance with law, their separation has been decreed, either temporarily or
finally and this case, with respect to the husband, cannot occur until a judgment of divorce is rendered, since, until then, if
he is culpable, he is not deprived of the management of his wife's property and of the product of the other property
belonging to the conjugal partnership; and
Considering that, should the doctrine maintained in the appeal prevail, it would allow married persons to disregard the
marriage bond and separate from each other of their own free will, thus establishing, contrary to the legal provision
contained in said article 56 of the Civil Code, a legal status entirely incompatible with the nature and effects of marriage in
disregard of the duties inherent therein and disturbing the unity of the family, in opposition to what the law, in conformity
with good morals, has established; and.
Considering that, as the spouses D. Ramon Benso and Doa Adela Galindo are not legally separated, it is their duty to
live together and afford each other help and support; and for this reason, it cannot be held that the former has need of
support from his wife so that he may live apart from her without the conjugal abode where it is his place to be, nor of her
conferring power upon him to dispose even of the fruits of her property in order therewith to pay the matrimonial expenses
and, consequently, those of his own support without need of going to his wife; wherefore the judgment appealed from,
denying the petition of D. Ramon Benso for support, has not violated the articles of the Civil Code and the doctrine
invoked in the assignments of error 1 and 5 of the appeal.
From a careful reading of the case just cited and quoted from it appears quite clearly that the spouses separated voluntarily in
accordance with an agreement previously made. At least there are strong indications to this effect, for the court says, "should
the doctrine maintained in the appeal prevail, it would allow married persons to disregard the marriage bond and separate from
each other of their own free will." If this be the true basis upon which the supreme court of Spain rested its decision, then the
doctrine therein enunciated would not be controlling in cases where one of the spouses was compelled to leave the conjugal
abode by the other or where the husband voluntarily abandons such abode and the wife seeks to force him to furnish support.
That this is true appears from the decision of the same high tribunal, dated October 16, 1903. In this case the wife brought an
action for support against her husband who had willfully and voluntarily abandoned the conjugal abode without any cause
whatever. The supreme court, reversing the judgment absolving the defendant upon the ground that no action for divorce, etc.,
had been instituted, said:
In the case at bar, it has been proven that it was Don Teodoro Exposito who left the conjugal abode, although he claims,
without however proving his contention, that the person responsible for this situation was his wife, as she turned him out
of the house. From this state of affairs it results that it is the wife who is party abandoned, the husband not having
prosecuted any action to keep her in his company and he therefore finds himself, as long as he consents to the situation,
under the ineluctable obligation to support his wife in fulfillment of the natural duty sanctioned in article 56 of the Code in
relation with paragraph 1 of article 143. In not so holding, the trial court, on the mistaken ground that for the fulfillment of
this duty the situation or relation of the spouses should be regulated in the manner it indicates, has made the errors of law
assigned in the first three grounds alleged, because the nature of the duty of affording mutual support is compatible and
enforcible in all situations, so long as the needy spouse does not create any illicit situation of the court above described.
lawphil.net

If we are in error as to the doctrine enunciated by the supreme court of Spain in its decision of November 3, 1905, and if the
court did hold, as contended by counsel for the defendant in the case under consideration, that neither spouse can be
compelled to support the other outside of the conjugal abode, unless it be by virtue of a final judgment granting the injured one a
divorce or separation from the other, still such doctrine or holding would not necessarily control in this jurisdiction for the reason
that the substantive law is not in every particular the same here as it is in Spain. As we have already stated, articles 42 to 107 of
the Civil Code in force in the Peninsula are not in force in the Philippine Islands. The law governing the duties and obligations of
husband and wife in this country are articles 44 to 78 of the Law of Civil Marriage of 1870 .In Spain the complaining spouse has,
under article 105 of the Civil Code, various causes for divorce, such as adultery on the part of the wife in every case and on the
part of the husband when public scandal or disgrace of the wife results therefrom; personal violence actually inflicted or grave
insults: violence exercised by the husband toward the wife in order to force her to change her religion; the proposal of the
husband to prostitute his wife; the attempts of the husband or wife to corrupt their sons or to prostitute their daughters; the
connivance in their corruption or prostitution; and the condemnation of a spouse to perpetual chains or hard labor, while in this
jurisdiction the only ground for a divorce is adultery. (Benedicto vs. De la Rama, 3 Phil .Rep., 34, 45.) This positive and absolute
doctrine was announced by this court in the case just cited after an exhaustive examination of the entire subject. Although the
case was appealed to the Supreme Court of the United States and the judgment rendered by this court was there reversed, the
reversal did not affect in any way or weaken the doctrine in reference to adultery being the only ground for a divorce. And since
the decision was promulgated by this court in that case in December, 1903, no change or modification of the rule has been
announced. It is, therefore, the well settled and accepted doctrine in this jurisdiction.
But it is argued that to grant support in an independent suit is equivalent to granting divorce or separation, as it necessitates a
determination of the question whether the wife has a good and sufficient cause for living separate from her husband; and,
consequently, if a court lacks power to decree a divorce, as in the instant case, power to grant a separate maintenance must
also be lacking. The weakness of this argument lies in the assumption that the power to grant support in a separate action is
dependent upon a power to grant a divorce. That the one is not dependent upon the other is apparent from the very nature of
the marital obligations of the spouses. The mere act of marriage creates an obligation on the part of the husband to support his
wife. This obligation is founded not so much on the express or implied terms of the contract of marriage as on the natural and
legal duty of the husband; an obligation, the enforcement of which is of such vital concern to the state itself that the laws will not
permit him to terminate it by his own wrongful acts in driving his wife to seek protection in the parental home. A judgment for
separate maintenance is not due and payable either as damages or as a penalty; nor is it a debt in the strict legal sense of the
term, but rather a judgment calling for the performance of a duty made specific by the mandate of the sovereign. This is done
from necessity and with a view to preserve the public peace and the purity of the wife; as where the husband makes so base
demands upon his wife and indulges in the habit of assaulting her. The pro tanto separation resulting from a decree for separate
support is not an impeachment of that public policy by which marriage is regarded as so sacred and inviolable in its nature; it is
merely a stronger policy overruling a weaker one; and except in so far only as such separation is tolerated as a means of
preserving the public peace and morals may be considered, it does not in any respect whatever impair the marriage contract or
for any purpose place the wife in the situation of a feme sole.
The foregoing are the grounds upon which our short opinion and order for judgment, heretofore filed in this case, rest.

[G.R. No. 127578. February 15, 1999]

MANUEL DE ASIS, petitioner, vs. COURT OF APPEALS, HON. JAIME T. HAMOY, Branch 130, RTC, Kalookan City and GLEN
CAMIL ANDRES DE ASIS represented by her mother/guardian VIRCEL D. ANDRES, respondents.
DECISION
PURISIMA, J.:

Petition for certiorari under Rule 65 of the Revised Rules of Court seeking to nullify the decision of the Court of Appeals which affirmed the trial courts
Orders, dated November 25, 1993 and February 4, 1994, respectively, denying petitioners Motion to Dismiss the Complaint in Civil Case No. C-16107,
entitledGlen Camil Andres de Asis, etc. vs. Manuel de Asis, and the motion for reconsideration.
The pertinent facts leading to the filing of the petition at bar are, as follows:
On October 14, 1988, Vircel D. Andres, (the herein private respondent) in her capacity as the legal guardian of the minor, Glen Camil Andres de Asis, brought
an action for maintenance and support against Manuel de Asis, docketed as Civil Case No. Q-88-935 before the Regional Trial Court of Quezon City, Branch 94,
alleging that the defendant Manuel de Asis (the petitioner here) is the father of subject minor Glen Camil Andres de Asis, and the former refused and/or failed to
provide for the maintenance of the latter, despite repeated demands.
In his Answer, petitioner denied his paternity of the said minor and theorized that he cannot therefore be required to provide support for him.
On July 4, 1989, private respondent Vircel D. Andres, through counsel, sent in a manifestation the pertinent portion of which, reads;

1. That in his proposed Amended Answer, defendant (herein petitioner) has made a judicial admission/declaration that
1) defendant
denies that the said minor child (Glen Camil) is his child; 2) he (petitioner) has no obligation to the plaintiff Glen Camil xxx.
2. That with the aforesaid judicial admissions/declarations by the defendant, it seems futile and a useless exercise to claim support from said
defendant.
3. That under the foregoing circumstances it would be more practical that plaintiff withdraws the complaint against the defendant subject to
the condition that the defendant should not pursue his counterclaim in the above-entitled case, xxx. [1]
By virtue of the said manifestation, both the plaintiff and the defendant agreed to move for the dismissal of the case. Acting thereupon, the Regional Trial
Courta quo issued the following Order of August 8, 1989, dismissing Civil Case No. Q-88-935 with prejudice, to wit:

Acting on the manifestation of Atty. Romualdo C. delos Santos, counsel for the defendant, that counsel for the plaintiff Atty. Ismael J. Andres has
no objection that this case be withdrawn provided that the defendant will withdraw the counterclaim, as prayed for, let the case be dismissed with
prejudice.
SO ORDERED.[2]
On September 7, 1995, another Complaint for maintenance and support was brought against Manuel A. de Asis, this time in the name of Glen Camil Andres de
Asis, represented by her legal guardian/mother, Vircel D. Andres. Docketed as Civil Case No. C-16107 before Branch 130 of the Regional Trial Court of Kalookan,
the said Complaint prayed, thus:

WHEREFORE, premises considered, it is respectfully prayed that judgment be rendered ordering defendant:
1. To pay plaintiff the sum of not less than P2,000.00 per month for every month since June 1, 1987 as support in arrears which defendant failed
to provide plaintiff shortly after her birth in June 1987 up to the present;
2. To give plaintiff a monthly allowance of P5,000.00 to be paid in advance on or before the 5th of each and every month;
3. To give plaintiff by way of support pendente lite, a monthly allowance of P5,000.00 per month, the first monthly allowance to start retroactively
from the first day of this month and the subsequent ones to be paid in advance on or before the 5th of each succeeding month;
4. To pay the costs of suit.
Plaintiff prays for such other relief just and equitable under the premises. [3]
On October 8, 1993, petitioner moved to dismiss the Complaint on the ground of res judicata, alleging that Civil Case C-16107 is barred by the prior judgment
which dismissed with prejudice Civil Case Q-88-935.
In the Order dated November 25, 1993 denying subject motion to dismiss, the trial court ruled that res judicata is inapplicable in an action for support for the
reason that renunciation or waiver of future support is prohibited by law. Petitioners motion for reconsideration of the said Order met the same fate. It was likewise
denied.
Petitioner filed with the Court of Appeals a Petition for Certiorari. But on June 7, 1996, the Court of Appeals found the said Petition devoid of merit and
dismissed the same.
Undaunted, petitioner found his way to this court via the present petition, posing the question whether or not the public respondent acted with grave abuse of
discretion amounting to lack or excess of jurisdiction in upholding the denial of the motion to dismiss by the trial court, and holding that an action for support cannot
be barred by res judicata.
To buttress his submission, petitioner invokes the previous dismissal of the Complaint for maintenance and support, Civil Case Q-88-935, filed by the mother
and guardian of the minor, Glen Camil Andres de Asis, (the herein private respondent). In said case, the complainant manifested that because of the defendants
judicial declaration denying that he is the father of subject minor child, it was futile and a useless exercise to claim support from defendant. Because of such
manifestation, and defendants assurance that he would not pursue his counterclaim anymore, the parties mutually agreed to move for the dismissal of the
complaint. The motion was granted by the Quezon City Regional Trial Court, which then dismissed the case with prejudice.

Petitioner contends that the aforecited manifestation, in effect, admitted the lack of filiation between him and the minor child, which admission binds the
complainant, and since the obligation to give support is based on the existence of paternity and filiation between the child and the putative parent, the lack thereof
negates the right to claim for support. Thus, petitioner maintains that the dismissal of the Complaint by the lower court on the basis of the said manifestation bars the
present action for support, especially so because the order of the trial court explicitly stated that the dismissal of the case was with prejudice.
The petition is not impressed with merit.
The right to receive support can neither be renounced nor transmitted to a third person. Article 301 of the Civil Code, the law in point, reads:

Art. 301. The right to receive support cannot be renounced, nor can it be transmitted to a third person. Neither can it be compensated with what
the recipient owes the obligor. xxx
Furthermore, future support cannot be the subject of a compromise.
Article 2035, ibid, provides, that:

No compromise upon the following questions shall be valid:


(1) The civil status of persons;
(2) The validity of a marriage or legal separation;
(3) Any ground for legal separation
(4) Future support;
(5) The jurisdiction of courts;
(6) Future legitime.
The raison d etre behind the proscription against renunciation, transmission and/or compromise of the right to support is stated, thus:

The right to support being founded upon the need of the recipient to maintain his existence, he is not entitled to renounce or transfer the right for
this would mean sanctioning the voluntary giving up of life itself. The right to life cannot be renounced; hence, support, which is the means to
attain the former, cannot be renounced.
xxx

To allow renunciation or transmission or compensation of the family right of a person to support is virtually to allow either suicide or the
conversion of the recipient to a public burden. This is contrary to public policy.[4]
In the case at bar, respondent minors mother, who was the plaintiff in the first case, manifested that she was withdrawing the case as it seemed futile to claim
support from petitioner who denied his paternity over the child. Since the right to claim for support is predicated on the existence of filiation between the minor
child and the putative parent, petitioner would like us to believe that such manifestation admitting the futility of claiming support from him puts the issue to rest and
bars any and all future complaint for support.
The manifestation sent in by respondents mother in the first case, which acknowledged that it would be useless to pursue its complaint for support, amounted to
renunciation as it severed the vinculum that gives the minor, Glen Camil, the right to claim support from his putative parent, the petitioner. Furthermore, the
agreement entered into between the petitioner and respondents mother for the dismissal of the complaint for maintenance and support conditioned upon the
dismissal of the counterclaim is in the nature of a compromise which cannot be countenanced. It violates the prohibition against any compromise of the right to
support.

Thus, the admission made by counsel for the wife of the facts alleged in a motion of the husband, in which the latter prayed that his obligation to
support be extinguished cannot be considered as an assent to the prayer, and much less, as a waiver of the right to claim for support. [5]
It is true that in order to claim support, filiation and/or paternity must first be shown between the claimant and the parent. However, paternity and filiation or
the lack of the same is a relationship that must be judicially established and it is for the court to declare its existence or absence. It cannot be left to the will or
agreement of the parties.

The civil status of a son having been denied, and this civil status, from which the right to support is derived being in issue, it is apparent that no
effect can be given to such a claim until an authoritative declaration has been made as to the existence of the cause. [6]
Although in the case under scrutiny, the admission may be binding upon the respondent, such an admission is at most evidentiary and does not conclusively
establish the lack of filiation.
Neither are we persuaded by petitioners theory that the dismissal with prejudice of Civil Case Q-88-935 has the effect of res judicata on the subsequent case
for support. The case of Advincula vs. Advincula[7] comes to the fore. In Advincula, the minor, Manuela Advincula, instituted a case for acknowledgment and
support against her putative father, Manuel Advincula. On motion of both parties and for the reason that the plaintiff has lost interest and is no longer interested in
continuing the case against the defendant and has no further evidence to introduce in support of the complaint, the case was dismissed. Thereafter, a similar case
was instituted by Manuela, which the defendant moved to dismiss, theorizing that the dismissal of the first case precluded the filing of the second case.
In disposing such case, this Court ruled, thus:

The new Civil Code provides that the allowance for support is provisional because the amount may be increased or decreased depending upon
the means of the giver and the needs of the recipient (Art. 297); and that the right to receive support cannot be renounced nor can it be
transmitted to a third person; neither can it be compensated with what the recipient owes the obligator (Art. 301). Furthermore, the right to
support can not be waived or transferred to third parties and future support cannot be the subject of compromise (Art. 2035; Coral v. Gallego, 38
O.G. 3135, cited in IV Civil Code by Padilla, p. 648, 1956 Ed.). This being true, it is indisputable that the present action for support can be
brought, notwithstanding the fact the previous case filed against the same defendant was dismissed. And it also appearing that the dismissal of
Civil Case No. 3553, was not an adjudication upon the merits, as heretofore shown, the right of herein plaintiff-appellant to reiterate her suit for
support and acknowledgment is available, as her needs arise. Once the needs of plaintiff arise, she has the right to bring an action for support,
for it is only then that her cause of action accrues.xxx
xxx

It appears that the former dismissal was predicated upon a compromise. Acknowledgment, affecting as it does the civil status of persons and
future support, cannot be the subject of compromise. (pars. 1 & 4, Art. 2035, Civil Code). Hence, the first dismissal cannot have force and
effect and can not bar the filing of another action, asking for the same relief against the same defendant.(emphasis supplied)
Conformably, notwithstanding the dismissal of Civil Case 88-935 and the lower courts pronouncement that such dismissal was with prejudice, the second
action for support may still prosper.
WHEREFORE, the petition under consideration is hereby DISMISSED and the decision of the Court of Appeals AFFIRMED. No pronouncement as to costs.

Parental Authority
MARIE ANTONETTE ABIGAIL
C. SALIENTES, ORLANDO B.
SALIENTES, and ROSARIO C.
SALIENTES,
Petitioners,
- versus LORAN S.D. ABANILLA,
HONORABLE JUDGE PEDRO
SABUNDAYO, JR., REGIONAL
TRIAL COURT,
BRANCH
203,MUNTINLUPA CITY,
Respondents.

G.R. No. 162734


Present:
QUISUMBING, J., Chairperson,
CARPIO,
CARPIO MORALES,
TINGA, and
VELASCO, JR., JJ.

Promulgated:
August 29, 2006

x- - - - - - - - - - - - - - - - - - - - - - - - - - - - - - - - - - - - - - - - - - - - - - - - - - -x

DECISION
QUISUMBING, J.:

The instant petition assails the Decision[1] dated November 10, 2003 of the Court of Appeals in CA-G.R. SP No. 75680, which
dismissed the petition for certiorari against the orders of the Regional Trial Court in Special Proceedings No. 03-004. Likewise assailed is the
Court of Appeals Resolution[2] dated March 19, 2004 denying reconsideration.
The facts of the case are as follows:
Private respondent Loran S.D. Abanilla and petitioner Marie Antonette Abigail C. Salientes are the parents of the minor Lorenzo
Emmanuel S. Abanilla. They lived with Marie Antonettes parents, petitioners Orlando B. Salientes and Rosario C.Salientes. Due to in-laws
problems, private respondent suggested to his wife that they transfer to their own house, but MarieAntonette refused. So, he alone left the
house of the Salientes. Thereafter, he was prevented from seeing his son.
Later, Loran S.D. Abanilla in his personal capacity and as the representative of his son, filed a Petition for Habeas Corpus and
Custody,[3] docketed as Special Proceedings No. 03-004 before the Regional Trial Court of Muntinlupa City. On January 23, 2003, the trial
court issued the following order:
Upon verified Petition for a Writ of Habeas Corpus by Petitioners, the Respondents Marie Antonette Abigail C. Salientes,
Orlando B.Salientes and Rosario C. Salientes are hereby directed to produce and bring before this Court the body of minor Lorenzo
Emmanuel SalientesAbanilla on January 31, 2003 at 1:00 oclock in the afternoon and to show cause why the said child should not
be discharged from restraint.
Let this Writ be served by the Sheriff or any authorized representative of this Court, who is directed to immediately make a
return.
SO ORDERED.[4]

Petitioners moved for reconsideration which the court denied.


Consequently, petitioners filed a petition for certiorari with the Court of Appeals, but the same was dismissed on November 10,
2003. The appellate court affirmed the February 24, 2003 Order of the trial court holding that its January 23, 2003 Order did not award the
custody of the 2-year-old child to any one but was simply the standard order issued for the production of restrained persons. The appellate
court held that the trial court was still about to conduct a full inquiry, in a summary proceeding, on the cause of the minors detention and the
matter of his custody. The Court of Appeals ruled thus:
WHEREFORE, the petition is hereby DISMISSED for lack of merit.

SO ORDERED.[5]

Petitioners moved for reconsideration, which was denied on March 19, 2004.
Hence, petitioners interposed this appeal by certiorari anchored on the following grounds:
1.

The Court of Appeals erred in not pronouncing the respondent judge gravely abused his discretion, amounting to lack or in
excess of jurisdiction in issuing an order for the petitioner-mother to first show cause why her own three-year old child in her
custody should not be discharged from a so-called restraint despite no evidence at all of restraint and no evidence of
compelling reasons of maternal unfitness to deprive the petitioner-mother of her minor son of tender years. The assailed
orders, resolutions and decisions of the lower court and the Court of Appeals are clearly void;

2.

The Court of Appeals erred in not pronouncing that the respondent judge gravely abused his discretion in issuing a writ of
habeas corpus which clearly is not warranted considering that there is no unlawful restraint by the mother and considering
further that the law presumes the fitness of the mother, thereby negating any notion of such mother illegally restraining or
confining her very own son of tender years. The petition is not even sufficient in substance to warrant the writ. The assailed
orders are clearly void.

3.

Contrary to the Court of Appeals decision, the Sombong vs. CA case supports rather than negates the position of the
petitioners.

4.
5.
6.

Contrary to the Court of Appeals decision, summary proceeding does violence to the tender-years-rule
The Court of Appeals failed to consider that the private respondent failed to present prima facie proof of any compelling
reason of the unfitness of the petitioner-mother;
The Court of Appeals failed to see that the New Rules on Custody SUFFICES AS REMEDY.[6]

Plainly put, the issue is: Did the Court of Appeals err when it dismissed the petition for certiorari against the trial courts orders
dated January 23, 2003 and February 24, 2003?
Petitioners contend that the order is contrary to Article 213 [7] of the Family Code, which provides that no child under seven years of
age shall be separated from the mother unless the court finds compelling reasons to order otherwise. They maintain that herein respondent
Loran had the burden of showing any compelling reason but failed to present even a prima facie proof thereof.
Petitioners posit that even assuming that there were compelling reasons, the proper remedy for private respondent was simply an
action for custody, but not habeas corpus. Petitioners assert that habeas corpus is unavailable against the mother who, under the law, has the
right of custody of the minor. They insist there was no illegal or involuntary restraint of the minor by his own mother. There was no need for
the mother to show cause and explain the custody of her very own child.
Private respondent counters that petitioners argument based on Article 213 of the Family Code applies only to the second part of his
petition regarding the custody of his son. It does not address the first part, which pertains to his right as the father to see his son. He asserts
that the writ of habeas corpus is available against any person who restrains the minors right to see his father andvice versa. He avers that
the instant petition is merely filed for delay, for had petitioners really intended to bring the child before the court in accordance with the new
rules on custody of minors, they would have done so on the dates specified in the January 23, 2003and the February 24, 2003 orders of the
trial court.
Private respondent maintains that, under the law, he and petitioner Marie Antonette have shared custody and parental authority over
their son. He alleges that at times when petitioner Marie Antonette is out of the country as required of her job as an international flight
stewardess, he, the father, should have custody of their son and not the maternal grandparents.
As correctly pointed out by the Court of Appeals, the assailed January 23, 2003 Order of the trial court did not grant custody of the
minor to any of the parties but merely directed petitioners to produce the minor in court and explain why they are restraining his liberty. The
assailed order was an interlocutory order precedent to the trial courts full inquiry into the issue of custody, which was still pending before it.
Under Rule 41, Section 1[8] of the Rules of Court, an interlocutory order is not appealable but the aggrieved party may file an
appropriate special action under Rule 65. The aggrieved party must show that the court gravely abused its discretion in issuing the
interlocutory order. In the present case, it is incumbent upon petitioners to show that the trial court gravely abused its discretion in issuing
the order.
Habeas corpus may be resorted to in cases where rightful custody is withheld from a person entitled thereto. [9] Under Article 211[10] of
the Family Code, respondent Loran and petitioner Marie Antonette have joint parental authority over their son and consequently joint
custody. Further, although the couple is separated de facto, the issue of custody has yet to be adjudicated by the court. In the absence of a
judicial grant of custody to one parent, both parents are still entitled to the custody of their child. In the present case, private respondents
cause of action is the deprivation of his right to see his child as alleged in his petition. [11] Hence, the remedy of habeas corpus is available to
him.
In a petition for habeas corpus, the childs welfare is the supreme consideration. The Child and Youth Welfare Code [12]unequivocally
provides that in all questions regarding the care and custody, among others, of the child, his welfare shall be the paramount consideration. [13]
Again, it bears stressing that the order did not grant custody of the minor to any of the parties but merely directed petitioners to
produce the minor in court and explain why private respondent is prevented from seeing his child. This is in line with the directive in

Section 9[14] of A.M. 03-04-04-SC[15] that within fifteen days after the filing of the answer or the expiration of the period to file answer, the
court shall issue an order requiring the respondent (herein petitioners) to present the minor before the court. This was exactly what the court
did.
Moreover, Article 213 of the Family Code deals with the judicial adjudication of custody and serves as a guideline for the proper
award of custody by the court. Petitioners can raise it as a counter argument for private respondents petition for custody. But it is not a basis
for preventing the father to see his own child. Nothing in the said provision disallows a father from seeing or visiting his child under seven
years of age.
In sum, the trial court did not err in issuing the orders dated January 23, 2003 and February 24, 2003. Hence, the Court of Appeals
properly dismissed the petition for certiorari against the said orders of the trial court.
WHEREFORE, the petition is DENIED. The Decision dated November 10, 2003 and the Resolution dated March 19, 2004of the
Court of Appeals in CA-G.R. SP No. 75680 are AFFIRMED. Costs against petitioners.

G.R. No. 115640 March 15, 1995


REYNALDO ESPIRITU and GUILLERMA LAYUG, petitioners,
vs.
COURT OF APPEALS and TERESITA MASAUDING, respondents.

MELO, J.:
This case concerns a seemingly void marriage and a relationship which went sour. The innocent victims are two children horn
out of the same union. Upon this Court now falls the not too welcome task of deciding the issue of who, between the father and
mother, is more suitable and better qualified in helping the children to grow into responsible, well-adjusted, and happy young
adulthood.
Petitioner Reynaldo Espiritu and respondent Teresita Masauding first met sometime in 1976 in Iligan City where Reynaldo was
employed by the National Steel Corporation and Teresita was employed as a nurse in a local hospital. In 1977, Teresita left for
Los Angeles, California to work as a nurse. She was able to acquire immigrant status sometime later. In 1984, Reynaldo was
sent by his employer, the National Steel Corporation, to Pittsburgh, Pennsylvania as its liaison officer and Reynaldo and Teresita
then began to maintain a common law relationship of husband and wife. On August 16, 1986, their daughter, Rosalind Therese,
was born. On October 7, 1987, while they were on a brief vacation in the Philippines, Reynaldo and Teresita got married, and
upon their return to the United States, their second child, a son, this time, and given the name Reginald Vince, was born on
January 12, 1988.
The relationship of the couple deteriorated until they decided to separate sometime in 1990. Teresita blamed Reynaldo for the
break-up, stating he was always nagging her about money matters. Reynaldo, on the other hand, contended that Teresita was a
spendthrift, buying expensive jewelry and antique furniture instead of attending to household expenses.
Instead of giving their marriage a second chance as allegedly pleaded by Reynaldo, Teresita left Reynaldo and the children and
went back to California. She claims, however, that she spent a lot of money on long distance telephone calls to keep in constant
touch with her children.
Reynaldo brought his children home to the Philippines, but because his assignment in Pittsburgh was not yet completed, he
was sent back by his company to Pittsburgh. He had to leave his children with his sister, co-petitioner Guillerma Layug and her
family.
Teresita claims that she did not immediately follow her children because Reynaldo filed a criminal case for bigamy against her
and she was afraid of being arrested. The judgment of conviction in the bigamy case was actually rendered only on September
29, 1994. (Per Judge Harriet O. Demetriou, Branch 70, RTC, Pasig, pp. 210-222,Rollo). Teresita, meanwhile, decided to return
to the Philippines and on December 8, 1992 and filed the petition for a writ of habeas corpus against herein two petitioners to
gain custody over the children, thus starting the whole proceedings now reaching this Court.
On June 30, 1993, the trial court dismissed the petition for habeas corpus. It suspended Teresita's parental authority over
Rosalind and Reginald and declared Reynaldo to have sole parental authority over them but with rights of visitation to be
agreed upon by the parties and to be approved by the Court.
On February 16, 1994, the Court of Appeals per Justice Isnani, with Justices de Pano and Ibay-Somera concurring, reversed
the trial court's decision. It gave custody to Teresita and visitation rights on weekends to Reynaldo.
Petitioners now come to this Court on a petition for review, in the main contending that the Court of Appeals disregarded the
factual findings of the trial court; that the Court of Appeals further engaged in speculations and conjectures, resulting in its
erroneous conclusion that custody of the children should be given to respondent Teresita.
We believe that respondent court resolved the question of custody over the children through an automatic and blind application
of the age proviso of Article 363 of the Civil Code which reads:
Art. 363. In all questions on the care, custody, education and property of the children, the latter's welfare shall be
paramount. No mother shall be separated from her child under seven years of age, unless the court finds compelling
reasons for such measure.
and of Article 213 of the Family Code which in turn provides:
Art. 213. In case of separation of the parents parental authority shall be exercised by the parent designated by the
Court. The Court shall take into account all relevant considerations, especially the choice of the child over seven
years of age unless the parent chosen is unfit.
The decision under review is based on the report of the Code Commission which drafted Article 213 that a child below seven
years still needs the loving, tender care that only a mother can give and which, presumably, a father cannot give in equal
measure. The commentaries of a member of the Code Commission, former Court of Appeals Justice Alicia Sempio-Diy, in a
textbook on the Family Code, were also taken into account. Justice Diy believes that a child below seven years should still be
awarded to her mother even if the latter is a prostitute or is unfaithful to her husband. This is on the theory that moral dereliction
has no effect on a baby unable to understand such action. (Handbook on the Family Code of the Philippines, 1988 Ed., p. 297.)

The Court of Appeals was unduly swayed by an abstract presumption of law rather than an appreciation of relevant facts and
the law which should apply to those facts. The task of choosing the parent to whom custody shall be awarded is not a ministerial
function to be determined by a simple determination of the age of a minor child. Whether a child is under or over seven years of
age, the paramount criterion must always be the child's interests. Discretion is given to the court to decide who can best assure
the welfare of the child, and award the custody on the basis of that consideration. In Unson III vs. Navarro (101 SCRA 183
[1980]), we laid down the rule that "in all controversies regarding the custody of minors, the sole and foremost consideration is
the physical, education, social and moral welfare of the child concerned, taking into account the respective resources and social
and moral situations of the contending parents", and in Medina vs. Makabali (27 SCRA 502 [1969]), where custody of the minor
was given to a non-relative as against the mother, then the country's leading civilist, Justice J.B.L. Reyes, explained its basis in
this manner:
. . . While our law recognizes the right of a parent to the custody of her child, Courts must not lose sight of the basic
principle that "in all questions on the care, custody, education and property of children, the latter's welfare shall be
paramount" (Civil Code of the Philippines. Art. 363), and that for compelling reasons, even a child under seven may
be ordered separated from the mother (do). This is as it should be, for in the continual evolution of legal institutions,
the patria potestas has been transformed from the jus vitae ac necis (right of life and death) of the Roman law,
under which the offspring was virtually a chattel of his parents into a radically different institution, due to the
influence of Christian faith and doctrines. The obligational aspect is now supreme. As pointed out by Puig Pena, now
"there is no power, but a task; no complex of rights (of parents) but a sum of duties; no sovereignty, but a sacred
trust for the welfare of the minor."
As a result, the right of parents to the company and custody of their children is but ancillary to the proper discharge
of parental duties to provide the children with adequate support, education, moral, intellectual and civic training and
development (Civil Code, Art. 356).
(pp. 504-505.)
In ascertaining the welfare and best interests of the child, courts are mandated by the Family Code to take into
account all relevant considerations. If a child is under seven years of age, the law presumes that the mother is the best
custodian. The presumption is strong but it is not conclusive. It can be overcome by "compelling reasons". If a child is over
seven, his choice is paramount but, again, the court is not bound by that choice. In its discretion, the court may find the chosen
parent unfit and award custody to the other parent, or even to a third party as it deems fit under the circumstances.
In the present case, both Rosalind and Reginald are now over seven years of age. Rosalind celebrated her seventh birthday on
August 16, 1993 while Reginald reached the same age on January 12, 1995. Both are studying in reputable schools and appear
to be fairly intelligent children, quite capable of thoughtfully determining the parent with whom they would want to live. Once the
choice has been made, the burden returns to the court to investigate if the parent thus chosen is unfit to assume parental
authority and custodial responsibility.
Herein lies the error of the Court of Appeals. Instead of scrutinizing the records to discover the choice of the children and rather
than verifying whether that parent is fit or unfit, respondent court simply followed statutory presumptions and general
propositions applicable to ordinary or common situations. The seven-year age limit was mechanically treated as an arbitrary cut
off period and not a guide based on a strong presumption.
A scrutiny of the pleadings in this case indicates that Teresita, or at least, her counsel are more intent on emphasizing the
"torture and agony" of a mother separated from her children and the humiliation she suffered as a result of her character being
made a key issue in court rather than the feelings and future, the best interests and welfare of her children. While the bonds
between a mother and her small child are special in nature, either parent, whether father or mother, is bound to suffer agony
and pain if deprived of custody. One cannot say that his or her suffering is greater than that of the other parent. It is not so much
the suffering, pride, and other feelings of either parent but the welfare of the child which is the paramount consideration.
We are inclined to sustain the findings and conclusions of the regional trial court because it gave greater attention to the choice
of Rosalind and considered in detail all the relevant factors bearing on the issue of custody.
When she was a little over 5 years old, Rosalind was referred to a child psychologist, Rita Flores Macabulos, to determine the
effects of uprooting her from the Assumption College where she was studying. Four different tests were administered. The
results of the tests are quite revealing. The responses of Rosalind about her mother were very negative causing the
psychologist to delve deeper into the child's anxiety. Among the things revealed by Rosalind was an incident where she saw her
mother hugging and kissing a "bad" man who lived in their house and worked for her father. Rosalind refused to talk to her
mother even on the telephone. She tended to be emotionally emblazed because of constant fears that she may have to leave
school and her aunt's family to go back to the United States to live with her mother. The 5-1/2 page report deals at length with
feelings of insecurity and anxiety arising from strong conflict with the mother. The child tried to compensate by having fantasy
activities. All of the 8 recommendations of the child psychologist show that Rosalind chooses petitioners over the private
respondent and that her welfare will be best served by staying with them (pp. 199-205, Rollo).
At about the same time, a social welfare case study was conducted for the purpose of securing the travel clearance required
before minors may go abroad. Social Welfare Officer Emma D. Estrada Lopez, stated that the child Rosalind refused to go back
to the United States and be reunited with her mother. She felt unloved and uncared for. Rosalind was more attached to her Yaya
who did everything for her and Reginald. The child was found suffering from emotional shock caused by her mother's infidelity.
The application for travel clearance was recommended for denial (pp. 206-209, Rollo).
Respondent Teresita, for her part, argues that the 7-year age reference in the law applies to the date when the petition for a writ
of habeas corpus is filed, not to the date when a decision is rendered. This argument is flawed. Considerations involving the
choice made by a child must be ascertained at the time that either parent is given custody over the child. The matter of custody
is not permanent and unalterable. If the parent who was given custody suffers a future character change and becomes unfit, the
matter of custody can always be re-examined and adjusted (Unson III v. Navarro, supra, at p. 189). To be sure, the welfare, the
best interests, the benefit, and the good of the child must be determined as of the time that either parent is chosen to be the
custodian. At the present time, both children are over 7 years of age and are thus perfectly capable of making a fairly intelligent
choice.
According to respondent Teresita, she and her children had tearful reunion in the trial court, with the children crying, grabbing,
and embracing her to prevent the father from taking them away from her. We are more inclined to believe the father's contention
that the children ignored Teresita in court because such an emotional display as described by Teresita in her pleadings could not
have been missed by the trial court. Unlike the Justices of the Court of Appeals Fourth Division, Judge Lucas P. Bersamin
personally observed the children and their mother in the courtroom. What the Judge found is diametrically opposed to the
contentions of respondent Teresita. The Judge had this to say on the matter.
And, lastly, the Court cannot look at petitioner [Teresita] in similar light, or with more understanding, especially as her
conduct and demeanor in the courtroom (during most of the proceedings) or elsewhere (but in the presence of the
undersigned presiding judge) demonstrated her ebulent temper that tended to corroborate the alleged violence of
her physical punishment of the children (even if only for ordinary disciplinary purposes) and emotional instability,
typified by her failure (or refusal?) to show deference and respect to the Court and the other parties (pp. 12-13, RTC
Decision)

Respondent Teresita also questions the competence and impartiality of the expert witnesses. Respondent court, in turn, states
that the trial court should have considered the fact that Reynaldo and his sister, herein petitioner Guillerma Layug, hired the two
expert witnesses. Actually, this was taken into account by the trial court which stated that the allegations of bias and unfairness
made by Teresita against the psychologist and social worker were not substantiated.
The trial court stated that the professional integrity and competence of the expert witnesses and the objectivity of the interviews
were unshaken and unimpeached. We might add that their testimony remain uncontroverted. We also note that the
examinations made by the experts were conducted in late 1991, well over a year before the filing by Teresita of the habeas
corpus petition in December, 1992. Thus, the examinations were at that time not intended to support petitioners' position in
litigation, because there was then not even an impending possibility of one. That they were subsequently utilized in the case a
quo when it did materialize does not change the tenor in which they were first obtained.
Furthermore, such examinations, when presented to the court must be construed to have been presented not to sway the court
in favor of any of the parties, but to assist the court in the determination of the issue before it. The persons who effected such
examinations were presented in the capacity of expert witnesses testifying on matters within their respective knowledge and
expertise. On this matter, this Court had occasion to rule in the case of Sali vs. Abukakar, et al. (17 SCRA 988 [1966]).
The fact that, in a particular litigation, an NBI expert examines certain contested documents, at the request, not of a
public officer or agency of the Government, but of a private litigant, does not necessarily nullify the examination thus
made. Its purpose, presumably, to assist the court having jurisdiction over said litigation, in the performance of its
duty to settle correctly the issues relative to said documents. Even a non-expert private individual may examine the
same, if there are facts within his knowledge which may help, the court in the determination of said issue. Such
examination, which may properly be undertaken by a non-expert private individual, does not, certainly become null
and void when the examiner is an expert and/or an officer of the NBI.
(pp. 991-992.)
In regard to testimony of expert witnesses it was held in Salomon, et al. vs. Intermediate Appellate Court, et al. (185 SCRA 352
[1990]):
. . . Although courts are not ordinarily bound by expert testimonies, they may place whatever weight they choose
upon such testimonies in accordance with the facts of the case. The relative weight and sufficiency of expert
testimony is peculiarly within the province of the trial court to decide, considering the ability and character of the
witness, his actions upon the witness stand, the weight and process of the reasoning by which he has supported his
opinion, his possible bias in favor of the side for whom he testifies, the fact that he is a paid witness, the relative
opportunities for study and observation of the matters about which he testifies, and any other matters which reserve
to illuminate his statements. The opinion of the expert may not be arbitrarily rejected; it is to be considered by the
court in view of all the facts and circumstances in the case and when common knowledge utterly fails, the expert
opinion may be given controlling effect (20 Am. Jur., 1056-1058). The problem of the credibility of the expert witness
and the evaluation of his testimony is left to the discretion of the trial court whose ruling thereupon is not reviewable
in the absence of an abuse of that discretion.
(p. 359)
It was in the exercise of this discretion, coupled with the opportunity to assess the witnesses' character and to observe their
respective demeanor that the trial court opted to rely on their testimony, and we believe that the trial court was correct in its
action.
Under direct examination an February 4, 1993, Social Worker Lopez stated that Rosalind and her aunt were about to board a
plane when they were off-loaded because there was no required clearance. They were referred to her office, at which time
Reginald was also brought along and interviewed. One of the regular duties of Social Worker Lopez in her job appears to be the
interview of minors who leave for abroad with their parents or other persons. The interview was for purposes of foreign travel by
a 5-year old child and had nothing to do with any pending litigation. On cross-examination, Social Worker Lopez stated that her
assessment of the minor's hatred for her mother was based on the disclosures of the minor. It is inconceivable, much less
presumable that Ms. Lopez would compromise her position, ethics, and the public trust reposed on a person of her position in
the course of doing her job by falsely testifying just to support the position of any litigant.
The psychologist, Ms. Macabulos, is a B.S. magna cum laude graduate in Psychology and an M.A. degree holder also in
Psychology with her thesis graded "Excellent". She was a candidate for a doctoral degree at the time of the interview. Petitioner
Reynaldo may have shouldered the cost of the interview but Ms. Macabulos services were secured because Assumption
College wanted an examination of the child for school purposes and not because of any litigation. She may have been paid to
examine the child and to render a finding based on her examination, but she was not paid to fabricate such findings in favor of
the party who retained her services. In this instance it was not even petitioner Reynaldo but the school authorities who initiated
the same. It cannot be presumed that a professional of her potential and stature would compromise her professional standing.
Teresita questions the findings of the trial court that:
1. Her morality is questionable as shown by her marrying Reynaldo at the time she had a subsisting marriage with
another man.
2. She is guilty of grave indiscretion in carrying on a love affair with one of the Reynaldo's fellow NSC employees.
3. She is incapable of providing the children with necessities and conveniences commensurate to their social
standing because she does not even own any home in the Philippines.
4. She is emotionally unstable with ebullient temper.
It is contended that the above findings do not constitute the compelling reasons under the law which would justify depriving her
of custody over the children; worse, she claims, these findings are non-existent and have not been proved by clear and
convincing evidence.
Public and private respondents give undue weight to the matter of a child under 7 years of age not to be separated from the
mother, without considering what the law itself denominates as compelling reasons or relevant considerations to otherwise
decree. In the Unson III case, earlier mentioned, this Court stated that it found no difficulty in not awarding custody to the
mother, it being in the best interest of the child "to be freed from the obviously unwholesome, not
to say immoral influence, that the situation where [the mother] had placed herself . . . might create in the moral and social
outlook of [the child] who was in her formative and most impressionable stage . . ."
Then too, it must be noted that both Rosalind and Reginald are now over 7 years of age. They understand the difference
between right and wrong, ethical behavior and deviant immorality. Their best interests would be better served in an environment
characterized by emotional stability and a certain degree of material sufficiency. There is nothing in the records to show that
Reynaldo is an "unfit" person under Article 213 of the Family Code. In fact, he has been trying his best to give the children the
kind of attention and care which the mother is not in a position to extend.
The argument that the charges against the mother are false is not supported by the records. The findings of the trial court are
based on evidence.

Teresita does not deny that she was legally married to Roberto Lustado on December 17, 1984 in California (p. 13,
Respondent's Memorandum; p. 238, Rollo; pp. 11, RTC Decision). Less than a year later, she had already driven across the
continental United States to commence living with another man, petitioner Reynaldo, in Pittsburgh. The two were married on
October 7, 1987. Of course, to dilute this disadvantage on her part, this matter of her having contracted a bigamous marriage
later with Reynaldo, Teresita tried to picture Reynaldo as a rapist, alleging further that she told Reynaldo about her marriage to
Lustado on the occasion when she was raped by Reynaldo. Expectedly, Judge Harriet Demetriou of the Pasig RTC lent no
weight to such tale. And even if this story were given credence, it adds to and not subtracts from the conviction of this Court
about Teresita's values. Rape is an insidious crime against privacy. Confiding to one's potential rapist about a prior marriage is
not a very convincing indication that the potential victim is averse to the act. The implication created is that the act would be
acceptable if not for the prior marriage.
More likely is Reynaldo's story that he learned of the prior marriage only much later. In fact, the rape incident itself is unlikely
against a woman who had driven three days and three nights from California, who went straight to the house of Reynaldo in
Pittsburgh and upon arriving went to bed and, who immediately thereafter started to live with him in a relationship which is
marital in nature if not in fact.
Judge Bersamin of the court a quo believed the testimony of the various witnesses that while married to Reynaldo, Teresita
entered into an illicit relationship with Perdencio Gonzales right there in the house of petitioner Reynaldo and respondent
Teresita. Perdencio had been assigned by the National Steel Corporation to assist in the project in Pittsburgh and was staying
with Reynaldo, his co-employee, in the latter's house. The record shows that the daughter Rosalind suffered emotional
disturbance caused by the traumatic effect of seeing her mother hugging and kissing a boarder in their house. The record also
shows that it was Teresita who left the conjugal home and the children, bound for California. When Perdencio Gonzales was
reassigned to the Philippines, Teresita followed him and was seen in his company in a Cebu hotel, staying in one room and
taking breakfast together. More significant is that letters and written messages from Teresita to Perdencio were submitted in
evidence (p.12, RTC Decision).
The argument that moral laxity or the habit of flirting from one man to another does not fall under "compelling reasons" is neither
meritorious nor applicable in this case. Not only are the children over seven years old and their clear choice is the father, but the
illicit or immoral activities of the mother had already caused emotional disturbances, personality conflicts, and exposure to
conflicting moral values, at least in Rosalind. This is not to mention her conviction for the crime of bigamy, which from the
records appears to have become final (pp. 210-222, Rollo).
Respondent court's finding that the father could not very well perform the role of a sole parent and substitute mother because
his job is in the United States while the children will be left behind with their aunt in the Philippines is misplaced. The
assignment of Reynaldo in Pittsburgh is or was a temporary one. He was sent there to oversee the purchase of a steel mill
component and various equipment needed by the National Steel Corporation in the Philippines. Once the purchases are
completed, there is nothing to keep him there anymore. In fact, in a letter dated January 30, 1995, Reynaldo informs this Court
of the completion of his assignment abroad and of his permanent return to the Philippines (ff.
p. 263, Rollo).
The law is more than satisfied by the judgment of the trial court. The children are now both over seven years old. Their choice of
the parent with whom they prefer to stay is clear from the record. From all indications, Reynaldo is a fit person, thus meeting the
two requirements found in the first paragraph of Article 213 of the Family Code. The presumption under the second paragraph
of said article no longer applies as the children are over seven years. Assuming that the presumption should have persuasive
value for children only one or two years beyond the age of seven years mentioned in the statute, there are compelling reasons
and relevant considerations not to grant custody to the mother. The children understand the unfortunate shortcomings of their
mother and have been affected in their emotional growth by her behavior.
WHEREFORE, the petition is hereby GRANTED. The decision of the Court of Appeals is reversed and set aside, and the
decision of Branch 96 of the Regional Trial Court of the National Capital Judicial Region stationed in Quezon City and presided
over by the Honorable Lucas P. Bersamin in its Civil Case No. Q-92-14206 awarding custody of the minors Rosalind and
Reginald Espiritu to their father, Reynaldo Espiritu, is reinstated. No special pronouncement is made as to costs.

G.R. No. L-47745 April 15, 1988


JOSE S. AMADORA, LORETA A. AMADORA, JOSE A. AMADORA JR., NORMA A. YLAYA PANTALEON A. AMADORA,
JOSE A. AMADORA III, LUCY A. AMADORA, ROSALINDA A. AMADORA, PERFECTO A. AMADORA, SERREC A.
AMADORA, VICENTE A. AMADORA and MARIA TISCALINA A. AMADORA, petitioners
vs.
HONORABLE COURT OF APPEALS, COLEGIO DE SAN JOSE-RECOLETOS, VICTOR LLUCH SERGIO P. DLMASO JR.,
CELESTINO DICON, ANIANO ABELLANA, PABLITO DAFFON thru his parents and natural guardians, MR. and MRS.
NICANOR GUMBAN, and ROLANDO VALENCIA, thru his guardian, A. FRANCISCO ALONSO, respondents.
Jose S. Amadora & Associates for petitioners.
Padilla Law Office for respondents.

CRUZ, J.:
Like any prospective graduate, Alfredo Amadora was looking forward to the commencement exercises where he would ascend
the stage and in the presence of his relatives and friends receive his high school diploma. These ceremonies were scheduled
on April 16, 1972. As it turned out, though, fate would intervene and deny him that awaited experience. On April 13, 1972, while
they were in the auditorium of their school, the Colegio de San Jose-Recoletos, a classmate, Pablito Damon, fired a gun that
mortally hit Alfredo, ending all his expectations and his life as well. The victim was only seventeen years old. 1
Daffon was convicted of homicide thru reckless imprudence .

Additionally, the herein petitioners, as the victim's parents, filed a civil action for damages
under Article 2180 of the Civil Code against the Colegio de San Jose-Recoletos, its rector the high school principal, the dean of boys, and the physics teacher, together with Daffon
and two other students, through their respective parents. The complaint against the students was later dropped. After trial, the Court of First Instance of Cebu held the remaining
defendants liable to the plaintiffs in the sum of P294,984.00, representing death compensation, loss of earning capacity, costs of litigation, funeral expenses, moral damages,
exemplary damages, and attorney's fees . 3On appeal to the respondent court, however, the decision was reversed and all the defendants were completely absolved . 4

In its decision, which is now the subject of this petition for certiorari under Rule 45 of the Rules of Court, the respondent court
found that Article 2180 was not applicable as the Colegio de San Jose-Recoletos was not a school of arts and trades but an
academic institution of learning. It also held that the students were not in the custody of the school at the time of the incident as
the semester had already ended, that there was no clear identification of the fatal gun and that in any event the defendant, had
exercised the necessary diligence in preventing the injury. 5
The basic undisputed facts are that Alfredo Amadora went to the San Jose-Recoletos on April 13, 1972, and while in its
auditorium was shot to death by Pablito Daffon, a classmate. On the implications and consequences of these facts, the parties
sharply disagree.
The petitioners contend that their son was in the school to show his physics experiment as a prerequisite to his graduation;
hence, he was then under the custody of the private respondents. The private respondents submit that Alfredo Amadora had
gone to the school only for the purpose of submitting his physics report and that he was no longer in their custody because the
semester had already ended.
There is also the question of the identity of the gun used which the petitioners consider important because of an earlier incident
which they claim underscores the negligence of the school and at least one of the private respondents. It is not denied by the
respondents that on April 7, 1972, Sergio Damaso, Jr., the dean of boys, confiscated from Jose Gumban an unlicensed pistol
but later returned it to him without making a report to the principal or taking any further action . 6 As Gumban was one of the companions of
Daffon when the latter fired the gun that killed Alfredo, the petitioners contend that this was the same pistol that had been confiscated from Gumban and that their son would not
have been killed if it had not been returned by Damaso. The respondents say, however, that there is no proof that the gun was the same firearm that killed Alfredo.

Resolution of all these disagreements will depend on the interpretation of Article 2180 which, as it happens, is invoked by both
parties in support of their conflicting positions. The pertinent part of this article reads as follows:
Lastly, teachers or heads of establishments of arts and trades shall be liable for damages caused by their pupils and
students or apprentices so long as they remain in their custody.
Three cases have so far been decided by the Court in connection with the above-quoted provision, to wit: Exconde v.
Capuno 7 Mercado v. Court of Appeals, 8 and Palisoc v. Brillantes. 9 These will be briefly reviewed in this opinion for a better resolution of the case at bar.
In the Exconde Case, Dante Capuno, a student of the Balintawak Elementary School and a Boy Scout, attended a Rizal Day
parade on instructions of the city school supervisor. After the parade, the boy boarded a jeep, took over its wheel and drove it so
recklessly that it turned turtle, resulting in the death of two of its passengers. Dante was found guilty of double homicide with
reckless imprudence. In the separate civil action flied against them, his father was held solidarily liable with him in damages
under Article 1903 (now Article 2180) of the Civil Code for the tort committed by the 15-year old boy.
This decision, which was penned by Justice Bautista Angelo on June 29,1957, exculpated the school in an obiter dictum (as it
was not a party to the case) on the ground that it was riot a school of arts and trades. Justice J.B.L. Reyes, with whom Justices
Sabino Padilla and Alex Reyes concurred, dissented, arguing that it was the school authorities who should be held liable
Liability under this rule, he said, was imposed on (1) teachers in general; and (2) heads of schools of arts and trades in
particular. The modifying clause "of establishments of arts and trades" should apply only to "heads" and not "teachers."
Exconde was reiterated in the Mercado Case, and with an elaboration. A student cut a classmate with a razor blade during
recess time at the Lourdes Catholic School in Quezon City, and the parents of the victim sued the culprits parents for damages.

Through Justice Labrador, the Court declared in another obiter (as the school itself had also not been sued that the school was
not liable because it was not an establishment of arts and trades. Moreover, the custody requirement had not been proved as
this "contemplates a situation where the student lives and boards with the teacher, such that the control, direction and
influences on the pupil supersede those of the parents." Justice J.B.L. Reyes did not take part but the other members of the
court concurred in this decision promulgated on May 30, 1960.
In Palisoc vs. Brillantes, decided on October 4, 1971, a 16-year old student was killed by a classmate with fist blows in the
laboratory of the Manila Technical Institute. Although the wrongdoer who was already of age was not boarding in the
school, the head thereof and the teacher in charge were held solidarily liable with him. The Court declared through Justice
Teehankee:
The phrase used in the cited article "so long as (the students) remain in their custody" means the protective
and supervisory custody that the school and its heads and teachers exercise over the pupils and students for as
long as they are at attendance in the school, including recess time. There is nothing in the law that requires that for
such liability to attach, the pupil or student who commits the tortious act must live and board in the school, as
erroneously held by the lower court, and the dicta in Mercado (as well as in Exconde) on which it relied, must now
be deemed to have been set aside by the present decision.
This decision was concurred in by five other members,

10

including Justice J.B.L. Reyes, who stressed, in answer to the dissenting opinion, that even
students already of age were covered by the provision since they were equally in the custody of the school and subject to its discipline. Dissenting with three others, 11 Justice
Makalintal was for retaining the custody interpretation in Mercado and submitted that the rule should apply only to torts committed by students not yet of age as the school would
be acting only in loco parentis.

In a footnote, Justice Teehankee said he agreed with Justice Reyes' dissent in the Exconde Case but added that "since the
school involved at bar is a non-academic school, the question as to the applicability of the cited codal provision to academic
institutions will have to await another case wherein it may properly be raised."
This is the case.
Unlike in Exconde and Mercado, the Colegio de San Jose-Recoletos has been directly impleaded and is sought to be held liable
under Article 2180; and unlike in Palisoc, it is not a school of arts and trades but an academic institution of learning. The parties
herein have also directly raised the question of whether or not Article 2180 covers even establishments which are technically not
schools of arts and trades, and, if so, when the offending student is supposed to be "in its custody."
After an exhaustive examination of the problem, the Court has come to the conclusion that the provision in question should
apply to all schools, academic as well as non-academic. Where the school is academic rather than technical or vocational in
nature, responsibility for the tort committed by the student will attach to the teacher in charge of such student, following the first
part of the provision. This is the general rule. In the case of establishments of arts and trades, it is the head thereof, and only
he, who shall be held liable as an exception to the general rule. In other words, teachers in general shall be liable for the acts of
their students except where the school is technical in nature, in which case it is the head thereof who shall be answerable.
Following the canon ofreddendo singula singulis "teachers" should apply to the words "pupils and students" and "heads of
establishments of arts and trades" to the word "apprentices."
The Court thus conforms to the dissenting opinion expressed by Justice J.B.L. Reyes in Exconde where he said in part:
I can see no sound reason for limiting Art. 1903 of the Old Civil Code to teachers of arts and trades and not to
academic ones. What substantial difference is there between them insofar as concerns the proper supervision and
vice over their pupils? It cannot be seriously contended that an academic teacher is exempt from the duty of
watching that his pupils do not commit a tort to the detriment of third Persons, so long as they are in a position to
exercise authority and Supervision over the pupil. In my opinion, in the phrase "teachers or heads of establishments
of arts and trades" used in Art. 1903 of the old Civil Code, the words "arts and trades" does not qualify "teachers" but
only "heads of establishments." The phrase is only an updated version of the equivalent terms "preceptores y
artesanos" used in the Italian and French Civil Codes.
If, as conceded by all commentators, the basis of the presumption of negligence of Art. 1903 in someculpa in
vigilando that the parents, teachers, etc. are supposed to have incurred in the exercise of their authority, it would
seem clear that where the parent places the child under the effective authority of the teacher, the latter, and not the
parent, should be the one answerable for the torts committed while under his custody, for the very reason/that the
parent is not supposed to interfere with the discipline of the school nor with the authority and supervision of the
teacher while the child is under instruction. And if there is no authority, there can be no responsibility.
There is really no substantial distinction between the academic and the non-academic schools insofar as torts committed by
their students are concerned. The same vigilance is expected from the teacher over the students under his control and
supervision, whatever the nature of the school where he is teaching. The suggestion in the Exconde and Mercado Cases is that
the provision would make the teacher or even the head of the school of arts and trades liable for an injury caused by any
student in its custody but if that same tort were committed in an academic school, no liability would attach to the teacher or the
school head. All other circumstances being the same, the teacher or the head of the academic school would be absolved
whereas the teacher and the head of the non-academic school would be held liable, and simply because the latter is a school of
arts and trades.
The Court cannot see why different degrees of vigilance should be exercised by the school authorities on the basis only of the
nature of their respective schools. There does not seem to be any plausible reason for relaxing that vigilance simply because
the school is academic in nature and for increasing such vigilance where the school is non-academic. Notably, the injury subject
of liability is caused by the student and not by the school itself nor is it a result of the operations of the school or its equipment.
The injury contemplated may be caused by any student regardless of the school where he is registered. The teacher certainly
should not be able to excuse himself by simply showing that he is teaching in an academic school where, on the other hand, the
head would be held liable if the school were non-academic.
These questions, though, may be asked: If the teacher of the academic school is to be held answerable for the torts committed
by his students, why is it the head of the school only who is held liable where the injury is caused in a school of arts and trades?
And in the case of the academic or non- technical school, why not apply the rule also to the head thereof instead of imposing
the liability only on the teacher?
The reason for the disparity can be traced to the fact that historically the head of the school of arts and trades exercised a closer
tutelage over his pupils than the head of the academic school. The old schools of arts and trades were engaged in the training
of artisans apprenticed to their master who personally and directly instructed them on the technique and secrets of their craft.
The head of the school of arts and trades was such a master and so was personally involved in the task of teaching his
students, who usually even boarded with him and so came under his constant control, supervision and influence. By contrast,
the head of the academic school was not as involved with his students and exercised only administrative duties over the
teachers who were the persons directly dealing with the students. The head of the academic school had then (as now) only a
vicarious relationship with the students. Consequently, while he could not be directly faulted for the acts of the students, the
head of the school of arts and trades, because of his closer ties with them, could be so blamed.
It is conceded that the distinction no longer obtains at present in view of the expansion of the schools of arts and trades, the
consequent increase in their enrollment, and the corresponding diminution of the direct and personal contract of their heads with
the students. Article 2180, however, remains unchanged. In its present state, the provision must be interpreted by the Court

according to its clear and original mandate until the legislature, taking into account the charges in the situation subject to be
regulated, sees fit to enact the necessary amendment.
The other matter to be resolved is the duration of the responsibility of the teacher or the head of the school of arts and trades
over the students. Is such responsibility co-extensive with the period when the student is actually undergoing studies during the
school term, as contended by the respondents and impliedly admitted by the petitioners themselves?
From a reading of the provision under examination, it is clear that while the custody requirement, to repeat Palisoc v. Brillantes,
does not mean that the student must be boarding with the school authorities, it does signify that the student should be within the
control and under the influence of the school authorities at the time of the occurrence of the injury. This does not necessarily
mean that such, custody be co-terminous with the semester, beginning with the start of classes and ending upon the close
thereof, and excluding the time before or after such period, such as the period of registration, and in the case of graduating
students, the period before the commencement exercises. In the view of the Court, the student is in the custody of the school
authorities as long as he is under the control and influence of the school and within its premises, whether the semester has not
yet begun or has already ended.
It is too tenuous to argue that the student comes under the discipline of the school only upon the start of classes
notwithstanding that before that day he has already registered and thus placed himself under its rules. Neither should such
discipline be deemed ended upon the last day of classes notwithstanding that there may still be certain requisites to be satisfied
for completion of the course, such as submission of reports, term papers, clearances and the like. During such periods, the
student is still subject to the disciplinary authority of the school and cannot consider himself released altogether from
observance of its rules.
As long as it can be shown that the student is in the school premises in pursuance of a legitimate student objective, in the
exercise of a legitimate student right, and even in the enjoyment of a legitimate student right, and even in the enjoyment of a
legitimate student privilege, the responsibility of the school authorities over the student continues. Indeed, even if the student
should be doing nothing more than relaxing in the campus in the company of his classmates and friends and enjoying the
ambience and atmosphere of the school, he is still within the custody and subject to the discipline of the school authorities
under the provisions of Article 2180.
During all these occasions, it is obviously the teacher-in-charge who must answer for his students' torts, in practically the same
way that the parents are responsible for the child when he is in their custody. The teacher-in-charge is the one designated by
the dean, principal, or other administrative superior to exercise supervision over the pupils in the specific classes or sections to
which they are assigned. It is not necessary that at the time of the injury, the teacher be physically present and in a position to
prevent it. Custody does not connote immediate and actual physical control but refers more to the influence exerted on the child
and the discipline instilled in him as a result of such influence. Thus, for the injuries caused by the student, the teacher and not
the parent shag be held responsible if the tort was committed within the premises of the school at any time when its authority
could be validly exercised over him.
In any event, it should be noted that the liability imposed by this article is supposed to fall directly on the teacher or the head of
the school of arts and trades and not on the school itself. If at all, the school, whatever its nature, may be held to answer for the
acts of its teachers or even of the head thereof under the general principle ofrespondeat superior, but then it may exculpate
itself from liability by proof that it had exercised the diligence of abonus paterfamilias.
Such defense is, of course, also available to the teacher or the head of the school of arts and trades directly held to answer for
the tort committed by the student. As long as the defendant can show that he had taken the necessary precautions to prevent
the injury complained of, he can exonerate himself from the liability imposed by Article 2180, which also states that:
The responsibility treated of in this article shall cease when the Persons herein mentioned prove that they observed
all the diligence of a good father of a family to prevent damages.
In this connection, it should be observed that the teacher will be held liable not only when he is acting in loco parentis for the law
does not require that the offending student be of minority age. Unlike the parent, who wig be liable only if his child is still a
minor, the teacher is held answerable by the law for the act of the student under him regardless of the student's age. Thus, in
the Palisoc Case, liability attached to the teacher and the head of the technical school although the wrongdoer was already of
age. In this sense, Article 2180 treats the parent more favorably than the teacher.
The Court is not unmindful of the apprehensions expressed by Justice Makalintal in his dissenting opinion in Palisoc that the
school may be unduly exposed to liability under this article in view of the increasing activism among the students that is likely to
cause violence and resulting injuries in the school premises. That is a valid fear, to be sure. Nevertheless, it should be repeated
that, under the present ruling, it is not the school that will be held directly liable. Moreover, the defense of due diligence is
available to it in case it is sought to be held answerable as principal for the acts or omission of its head or the teacher in its
employ.
The school can show that it exercised proper measures in selecting the head or its teachers and the appropriate supervision
over them in the custody and instruction of the pupils pursuant to its rules and regulations for the maintenance of discipline
among them. In almost all cases now, in fact, these measures are effected through the assistance of an adequate security force
to help the teacher physically enforce those rules upon the students. Ms should bolster the claim of the school that it has taken
adequate steps to prevent any injury that may be committed by its students.
A fortiori, the teacher himself may invoke this defense as it would otherwise be unfair to hold him directly answerable for the
damage caused by his students as long as they are in the school premises and presumably under his influence. In this respect,
the Court is disposed not to expect from the teacher the same measure of responsibility imposed on the parent for their
influence over the child is not equal in degree. Obviously, the parent can expect more obedience from the child because the
latter's dependence on him is greater than on the teacher. It need not be stressed that such dependence includes the child's
support and sustenance whereas submission to the teacher's influence, besides being coterminous with the period of custody is
usually enforced only because of the students' desire to pass the course. The parent can instill more las discipline on the child
than the teacher and so should be held to a greater accountability than the teacher for the tort committed by the child.
And if it is also considered that under the article in question, the teacher or the head of the school of arts and trades is
responsible for the damage caused by the student or apprentice even if he is already of age and therefore less tractable than
the minor then there should all the more be justification to require from the school authorities less accountability as long as
they can prove reasonable diligence in preventing the injury. After all, if the parent himself is no longer liable for the student's
acts because he has reached majority age and so is no longer under the former's control, there is then all the more reason for
leniency in assessing the teacher's responsibility for the acts of the student.
Applying the foregoing considerations, the Court has arrived at the following conclusions:
1. At the time Alfredo Amadora was fatally shot, he was still in the custody of the authorities of Colegio de San Jose-Recoletos
notwithstanding that the fourth year classes had formally ended. It was immaterial if he was in the school auditorium to finish his
physics experiment or merely to submit his physics report for what is important is that he was there for a legitimate purpose. As
previously observed, even the mere savoring of the company of his friends in the premises of the school is a legitimate purpose
that would have also brought him in the custody of the school authorities.
2. The rector, the high school principal and the dean of boys cannot be held liable because none of them was the teacher-incharge as previously defined. Each of them was exercising only a general authority over the student body and not the direct

control and influence exerted by the teacher placed in charge of particular classes or sections and thus immediately involved in
its discipline. The evidence of the parties does not disclose who the teacher-in-charge of the offending student was. The mere
fact that Alfredo Amadora had gone to school that day in connection with his physics report did not necessarily make the
physics teacher, respondent Celestino Dicon, the teacher-in-charge of Alfredo's killer.
3. At any rate, assuming that he was the teacher-in-charge, there is no showing that Dicon was negligent in enforcing discipline
upon Daffon or that he had waived observance of the rules and regulations of the school or condoned their non-observance. His
absence when the tragedy happened cannot be considered against him because he was not supposed or required to report to
school on that day. And while it is true that the offending student was still in the custody of the teacher-in-charge even if the
latter was physically absent when the tort was committed, it has not been established that it was caused by his laxness in
enforcing discipline upon the student. On the contrary, the private respondents have proved that they had exercised due
diligence, through the enforcement of the school regulations, in maintaining that discipline.
4. In the absence of a teacher-in-charge, it is probably the dean of boys who should be held liable especially in view of the
unrefuted evidence that he had earlier confiscated an unlicensed gun from one of the students and returned the same later to
him without taking disciplinary action or reporting the matter to higher authorities. While this was clearly negligence on his part,
for which he deserves sanctions from the school, it does not necessarily link him to the shooting of Amador as it has not been
shown that he confiscated and returned pistol was the gun that killed the petitioners' son.
5. Finally, as previously observed, the Colegio de San Jose-Recoletos cannot be held directly liable under the article because
only the teacher or the head of the school of arts and trades is made responsible for the damage caused by the student or
apprentice. Neither can it be held to answer for the tort committed by any of the other private respondents for none of them has
been found to have been charged with the custody of the offending student or has been remiss in the discharge of his duties in
connection with such custody.
In sum, the Court finds under the facts as disclosed by the record and in the light of the principles herein announced that none
of the respondents is liable for the injury inflicted by Pablito Damon on Alfredo Amadora that resulted in the latter's death at the
auditorium of the Colegio de San Jose-Recoletos on April 13, 1972. While we deeply sympathize with the petitioners over the
loss of their son under the tragic circumstances here related, we nevertheless are unable to extend them the material relief they
seek, as a balm to their grief, under the law they have invoked.
WHEREFORE, the petition is DENIED, without any pronouncement as to costs. It is so ordered.

G.R. No. L-33722 July 29, 1988


FEDERICO YLARDE and ADELAIDA DORONIO petitioners,
vs.
EDGARDO AQUINO, MAURO SORIANO and COURT OF APPEALS, respondents.
Buenaventura C. Evangelista for petitioners.
Modesto V. Cabanela for respondent Edgardo Aquino.
Manuel P. Pastor for respondent Mauro Soriano.

GANCAYCO, J.:
In this petition for review on certiorari seeking the reversal of the decision of the Court of Appeals in CA-G.R. No. 36390-R
entitled "Federico Ylarde, et al. vs. Edgardo Aquino, et al.," a case which originated from the Court of First Instance of
Pangasinan, We are again caned upon determine the responsibility of the principals and teachers towards their students or
pupils.
In 1963, private respondent Mariano Soriano was the principal of the Gabaldon Primary School, a public educational institution
located in Tayug, Pangasinan-Private respondent Edgardo Aquino was a teacher therein. At that time, the school was fittered
with several concrete blocks which were remnants of the old school shop that was destroyed in World War II. Realizing that the
huge stones were serious hazards to the schoolchildren, another teacher by the name of Sergio Banez started burying them
one by one as early as 1962. In fact, he was able to bury ten of these blocks all by himself.
Deciding to help his colleague, private respondent Edgardo Aquino gathered eighteen of his male pupils, aged ten to eleven,
after class dismissal on October 7, 1963. Being their teacher-in-charge, he ordered them to dig beside a one-ton concrete block
in order to make a hole wherein the stone can be buried. The work was left unfinished. The following day, also after classes,
private respondent Aquino called four of the original eighteen pupils to continue the digging. These four pupils Reynaldo
Alonso, Francisco Alcantara, Ismael Abaga and Novelito Ylarde, dug until the excavation was one meter and forty centimeters
deep. At this point, private respondent Aquino alone continued digging while the pupils remained inside the pit throwing out the
loose soil that was brought about by the digging.
When the depth was right enough to accommodate the concrete block, private respondent Aquino and his four pupils got out of
the hole. Then, said private respondent left the children to level the loose soil around the open hole while he went to see Banez
who was about thirty meters away. Private respondent wanted to borrow from Banez the key to the school workroom where he
could get some rope. Before leaving. , private respondent Aquino allegedly told the children "not to touch the stone."
A few minutes after private respondent Aquino left, three of the four kids, Alonso, Alcantara and Ylarde, playfully jumped into the
pit. Then, without any warning at all, the remaining Abaga jumped on top of the concrete block causing it to slide down towards
the opening. Alonso and Alcantara were able to scramble out of the excavation on time but unfortunately fo Ylarde, the concrete
block caught him before he could get out, pinning him to the wall in a standing position. As a result thereof, Ylarde sustained the
following injuries:
1. Contusion with hematoma, left inguinal region and suprapubic region.
2. Contusion with ecchymosis entire scrotal region.
3. Lacerated wound, left lateral aspect of penile skin with phimosis
4. Abrasion, gluteal region, bilateral.
5. Intraperitoneal and extrapertitoneal extravasation of blood and urine about 2 liters.
6. Fracture, simple, symphesis pubis
7. Ruptured (macerated) urinary bladder with body of bladder almost entirely separated from its neck.
REMARKS:
1. Above were incurred by crushing injury.
2. Prognosis very poor.
(Sgd.) MELQUIADES A. BRAVO
Physician on Duty. 1
Three days later, Novelito Ylarde died.
Ylarde's parents, petitioners in this case, filed a suit for damages against both private respondents Aquino and Soriano. The
lower court dismissed the complaint on the following grounds: (1) that the digging done by the pupils is in line with their course

called Work Education; (2) that Aquino exercised the utmost diligence of a very cautious person; and (3) that the demise of
Ylarde was due to his own reckless imprudence. 2
On appeal, the Court of Appeals affirmed the Decision of the lower court.
Petitioners base their action against private respondent Aquino on Article 2176 of the Civil Code for his alleged negligence that
caused their son's death while the complaint against respondent Soriano as the head of school is founded on Article 2180 of the
same Code.
Article 2176 of the Civil Code provides:
Art. 2176. Whoever by act or omission causes damage to another, there being fault or negligence, is obliged to pay
for the damage done. Such fault or negligence, if there is no pre- existing contractual relation between the parties, is
called a quasi-delict and is governed by the provisions of this Chapter.
On the other hand, the applicable provision of Article 2180 states:
Art. 2180. x x x
xxx xxx xxx
Lastly, teachers or heads of establishments of arts and trades shall be liable for damages caused by their pupils and
students or apprentices, so long as they remain in their custody. 3
The issue to be resolved is whether or not under the cited provisions, both private respondents can be held liable for damages.
As regards the principal, We hold that he cannot be made responsible for the death of the child Ylarde, he being the head of an
academic school and not a school of arts and trades. This is in line with Our ruling in Amadora vs. Court of Appeals, 4 wherein this
Court thoroughly discussed the doctrine that under Article 2180 of the Civil Code, it is only the teacher and not the head of an academic school who should be answerable for torts
committed by their students. This Court went on to say that in a school of arts and trades, it is only the head of the school who can be held liable. In the same case, We explained:

After an exhaustive examination of the problem, the Court has come to the conclusion that the provision in question
should apply to all schools, academic as well as non-academic. Where the school is academic rather than technical
or vocational in nature, responsibility for the tort committed by the student will attach to the teacher in charge of such
student, following the first part of the provision. This is the general rule. In the case of establishments of arts and
trades, it is the head thereof, and only he, who shall be held liable as an exception to the general rule. In other
words, teachers in general shall be liable for the acts of their students except where the school is technical in nature,
in which case it is the head thereof who shall be answerable. Following the canon of reddendo singula
sinquilis 'teachers' should apply to the words "pupils and students' and 'heads of establishments of arts and trades
to the word "apprentices."
Hence, applying the said doctrine to this case, We rule that private respondent Soriano, as principal, cannot be held liable for
the reason that the school he heads is an academic school and not a school of arts and trades. Besides, as clearly admitted by
private respondent Aquino, private respondent Soriano did not give any instruction regarding the digging.
From the foregoing, it can be easily seen that private respondent Aquino can be held liable under Article 2180 of the Civil Code
as the teacher-in-charge of the children for being negligent in his supervision over them and his failure to take the necessary
precautions to prevent any injury on their persons. However, as earlier pointed out, petitioners base the alleged liability of
private respondent Aquino on Article 2176 which is separate and distinct from that provided for in Article 2180.
With this in mind, the question We need to answer is this: Were there acts and omissions on the part of private respondent
Aquino amounting to fault or negligence which have direct causal relation to the death of his pupil Ylarde? Our answer is in the
affirmative. He is liable for damages.
From a review of the record of this case, it is very clear that private respondent Aquino acted with fault and gross negligence
when he: (1) failed to avail himself of services of adult manual laborers and instead utilized his pupils aged ten to eleven to
make an excavation near the one-ton concrete stone which he knew to be a very hazardous task; (2) required the children to
remain inside the pit even after they had finished digging, knowing that the huge block was lying nearby and could be easily
pushed or kicked aside by any pupil who by chance may go to the perilous area; (3) ordered them to level the soil around the
excavation when it was so apparent that the huge stone was at the brink of falling; (4) went to a place where he would not be
able to check on the children's safety; and (5) left the children close to the excavation, an obviously attractive nuisance.
The negligent act of private respondent Aquino in leaving his pupils in such a dangerous site has a direct causal connection to
the death of the child Ylarde. Left by themselves, it was but natural for the children to play around. Tired from the strenuous
digging, they just had to amuse themselves with whatever they found. Driven by their playful and adventurous instincts and not
knowing the risk they were facing three of them jumped into the hole while the other one jumped on the stone. Since the stone
was so heavy and the soil was loose from the digging, it was also a natural consequence that the stone would fall into the hole
beside it, causing injury on the unfortunate child caught by its heavy weight. Everything that occurred was the natural and
probable effect of the negligent acts of private respondent Aquino. Needless to say, the child Ylarde would not have died were it
not for the unsafe situation created by private respondent Aquino which exposed the lives of all the pupils concerned to real
danger.
We cannot agree with the finding of the lower court that the injuries which resulted in the death of the child Ylarde were caused
by his own reckless imprudence, It should be remembered that he was only ten years old at the time of the incident, As such, he
is expected to be playful and daring. His actuations were natural to a boy his age. Going back to the facts, it was not only him
but the three of them who jumped into the hole while the remaining boy jumped on the block. From this, it is clear that he only
did what any other ten-year old child would do in the same situation.
In ruling that the child Ylarde was imprudent, it is evident that the lower court did not consider his age and maturity. This should
not be the case. The degree of care required to be exercised must vary with the capacity of the person endangered to care for
himself. A minor should not be held to the same degree of care as an adult, but his conduct should be judged according to the
average conduct of persons of his age and experience. 5 The standard of conduct to which a child must conform for his own protection is that degree of care
ordinarily exercised by children of the same age, capacity, discretion, knowledge and experience under the same or similar circumstances.
charge the child Ylarde with reckless imprudence.

Bearing this in mind, We cannot

The court is not persuaded that the digging done by the pupils can pass as part of their Work Education. A single glance at the
picture showing the excavation and the huge concrete block 7 would reveal a dangerous site requiring the attendance of strong, mature laborers and not
ten-year old grade-four pupils. We cannot comprehend why the lower court saw it otherwise when private respondent Aquino himself admitted that there were no instructions from
the principal requiring what the pupils were told to do. Nor was there any showing that it was included in the lesson plan for their Work Education. Even the Court of Appeals made
mention of the fact that respondent Aquino decided all by himself to help his co-teacher Banez bury the concrete remnants of the old school shop. 8 Furthermore, the excavation
should not be placed in the category of school gardening, planting trees, and the like as these undertakings do not expose the children to any risk that could result in death or
physical injuries.

The contention that private respondent Aquino exercised the utmost diligence of a very cautious person is certainly without
cogent basis. A reasonably prudent person would have foreseen that bringing children to an excavation site, and more so,
leaving them there all by themselves, may result in an accident. An ordinarily careful human being would not assume that a
simple warning "not to touch the stone" is sufficient to cast away all the serious danger that a huge concrete block adjacent to

an excavation would present to the children. Moreover, a teacher who stands in loco parentis to his pupils would have made
sure that the children are protected from all harm in his company.
We close by categorically stating that a truly careful and cautious person would have acted in all contrast to the way private
respondent Aquino did. Were it not for his gross negligence, the unfortunate incident would not have occurred and the child
Ylarde would probably be alive today, a grown- man of thirty-five. Due to his failure to take the necessary precautions to avoid
the hazard, Ylarde's parents suffered great anguish all these years.
WHEREFORE, in view of the foregoing, the petition is hereby GRANTED and the questioned judgment of the respondent court
is REVERSED and SET ASIDE and another judgment is hereby rendered ordering private respondent Edagardo Aquino to pay
petitioners the following:
(1) Indemnity for the death of Child Ylarde P30,000.00
(2) Exemplary damages 10,000.00
(3) Moral damages 20,000.00

[G.R. No. 143363. February 6, 2002]

ST. MARYS ACADEMY, petitioner, vs. WILLIAM CARPITANOS and LUCIA S. CARPITANOS, GUADA DANIEL, JAMES
DANIEL II, JAMES DANIEL, SR., and VIVENCIO VILLANUEVA, respondents.
DECISION
PARDO, J.:

The Case
The case is an appeal via certiorari from the decision of the Court of Appeals as well as the resolution denying reconsideration,
holding petitioner liable for damages arising from an accident that resulted in the death of a student who had joined a campaign to
visit the public schools in Dipolog City to solicit enrollment.
[1]

The Facts
The facts, as found by the Court of Appeals, are as follows:
Claiming damages for the death of their only son, Sherwin Carpitanos, spouses William Carpitanos and Lucia Carpitanos filed on June 9, 1995 a
case against James Daniel II and his parents, James Daniel Sr. and Guada Daniel, the vehicle owner, Vivencio Villanueva and St. Marys Academy
before the Regional Trial Court ofDipolog City.
On 20 February 1997, Branch 6 of the Regional Trial Court of Dipolog City rendered its decision the dispositive portion of which reads as
follows:
WHEREFORE, PREMISES CONSIDERED, judgment is hereby rendered in the following manner:
1.
Defendant St. Marys Academy of Dipolog City, is hereby ordered to pay plaintiffs William Carpitanos and Luisa Carpitanos, the
following sums of money:
a. FIFTY THOUSAND PESOS (P50,000.00) indemnity for the loss of life of Sherwin S. Carpitanos;
b. FORTY THOUSAND PESOS (P40,000.00) actual damages incurred by plaintiffs for burial and related expenses;
c. TEN THOUSAND PESOS (P10,000.00) for attorneys fees;
d. FIVE HUNDRED THOUSAND PESOS (P500,000.00) for moral damages; and to pay costs.

2.
Their liability being only subsidiary, defendants James Daniel, Sr. and Guada Daniel are hereby ordered to pay herein plaintiffs the
amount of damages above-stated in the event of insolvency of principal obligor St. Marys Academy of Dipolog City;
3.
Defendant James Daniel II, being a minor at the time of the commission of the tort and who was under special parental authority of
defendant St. Marys Academy, is ABSOLVED from paying the above-stated damages, same being adjudged against defendants St. Marys
Academy, and subsidiarily, against his parents;
4.
Defendant Vivencio Villanueva is hereby ABSOLVED of any liability. His counterclaim not being in order as earlier discussed in this
decision, is hereby DISMISSED.
IT IS SO ORDERED. (Decision, pp. 32-33; Records, pp. 205-206).
From the records it appears that from 13 to 20 February 1995, defendant-appellant St. Marys Academy of Dipolog City conducted an enrollment
drive for the school year 1995-1996. A facet of the enrollment campaign was the visitation of schools from where prospective enrollees were
studying. As a student of St. Marys Academy, Sherwin Carpitanos was part of the campaigning group. Accordingly, on the fateful day, Sherwin,
along with other high school students were riding in a Mitsubishi jeep owned by defendant Vivencio Villanueva on their way
to Larayan Elementary School, Larayan, Dapitan City. The jeep was driven by James Daniel II then 15 years old and a student of the same
school. Allegedly, the latter drove the jeep in a reckless manner and as a result the jeep turned turtle.
Sherwin Carpitanos died as a result of the injuries he sustained from the accident. [2]
In due time, petitioner St. Marys academy appealed the decision to the Court of Appeals.

[3]

On February 29, 2000, the Court of Appeals promulgated a decision reducing the actual damages to P25,000.00 but otherwise
affirming the decision a quo, in toto.
[4]

On February 29, 2000, petitioner St. Marys Academy filed a motion for reconsideration of the decision. However, on May 22,
2000, the Court of Appeals denied the motion.
[5]

Hence, this appeal.

[6]

The Issues

1) Whether the Court of Appeals erred in holding the petitioner liable for damages for the death of Sherwin Carpitanos.
2) Whether the Court of Appeals erred in affirming the award of moral damages against the petitioner.

The Courts Ruling


We reverse the decision of the Court of Appeals.
The Court of Appeals held petitioner St. Marys Academy liable for the death of Sherwin Carpitanos under Articles 218 and
219 of the Family Code, pointing out that petitioner was negligent in allowing a minor to drive and in not having a teacher
accompany the minor students in the jeep.
[7]

[8]

Under Article 218 of the Family Code, the following shall have special parental authority over a minor child while under their
supervision, instruction or custody: (1) the school, its administrators and teachers; or (2) the individual, entity or institution engaged
in child care. This special parental authority and responsibility applies to all authorized activities, whether inside or outside the
premises of the school, entity or institution. Thus, such authority and responsibility applies to field trips, excursions and other affairs
of the pupils and students outside the school premises whenever authorized by the school or its teachers.
[9]

Under Article 219 of the Family Code, if the person under custody is a minor, those exercising special parental authority are
principally and solidarily liable for damages caused by the acts or omissions of the unemancipated minor while under their
supervision, instruction, or custody.
[10]

However, for petitioner to be liable, there must be a finding that the act or omission considered as negligent was the proximate
cause of the injury caused because the negligence must have a causal connection to the accident.
[11]

In order that there may be a recovery for an injury, however, it must be shown that the injury for which recovery is sought must be the legitimate
consequence of the wrong done; the connection between the negligence and the injury must be a direct and natural sequence of events, unbroken
by intervening efficient causes. In other words, the negligence must be the proximate cause of the injury. For, negligence, no matter in what it
consists, cannot create a right of action unless it is the proximate cause of the injury complained of. And the proximate cause of an injury is that
cause, which, in natural and continuous sequence, unbroken by any efficient intervening cause, produces the injury, and without which the result
would not have occurred.[12]
In this case, the respondents failed to show that the negligence of petitioner was the proximate cause of the death of the victim.
Respondents Daniel spouses and Villanueva admitted that the immediate cause of the accident was not the negligence of
petitioner or the reckless driving of James Daniel II, but the detachment of the steering wheel guide of the jeep.
In their comment to the petition, respondents Daniel spouses and Villanueva admitted the documentary exhibits establishing that
the cause of the accident was the detachment of the steering wheel guide of the jeep. Hence, the cause of the accident was not the
recklessness of James Daniel II but the mechanical defect in the jeep of Vivencio Villanueva. Respondents, including the spouses
Carpitanos, parents of the deceased Sherwin Carpitanos, did not dispute the report and testimony of the traffic investigator who
stated that the cause of the accident was the detachment of the steering wheel guide that caused the jeep to turn turtle.
Significantly, respondents did not present any evidence to show that the proximate cause of the accident was the negligence of
the school authorities, or the reckless driving of James Daniel II. Hence, the respondents reliance on Article 219 of the Family Code
that those given the authority and responsibility under the preceding Article shall be principally and solidarily liable for damages
caused by acts or omissions of the unemancipated minor was unfounded.
Further, there was no evidence that petitioner school allowed the minor James Daniel II to drive the jeep of respondent Vivencio
Villanueva. It was Ched Villanueva, grandson of respondent Vivencio Villanueva, who had possession and control of the jeep. He
was driving the vehicle and he allowed James Daniel II, a minor, to drive the jeep at the time of the accident.
Hence, liability for the accident, whether caused by the negligence of the minor driver or mechanical detachment of the steering
wheel guide of the jeep, must be pinned on the minors parents primarily. The negligence of petitioner St. Marys Academy was only
a remote cause of the accident. Between the remote cause and the injury, there intervened the negligence of the minors parents or
the detachment of the steering wheel guide of the jeep.
The proximate cause of an injury is that cause, which, in natural and continuous sequence, unbroken by any efficient intervening cause, produces
the injury, and without which the result would not have occurred. [13]
Considering that the negligence of the minor driver or the detachment of the steering wheel guide of the jeep owned by
respondent Villanueva was an event over which petitioner St. Marys Academy had no control, and which was the proximate cause
of the accident, petitioner may not be held liable for the death resulting from such accident.
Consequently, we find that petitioner likewise cannot be held liable for moral damages in the amount of P500,000.00 awarded by
the trial court and affirmed by the Court of Appeals.
Though incapable of pecuniary computation, moral damages may be recovered if they are the proximate result of the defendants
wrongful act or omission. In this case, the proximate cause of the accident was not attributable to petitioner.
[14]

For the reason that petitioner was not directly liable for the accident, the decision of the Court of Appeals ordering petitioner to
pay death indemnity to respondent Carpitanos must be deleted. Moreover, the grant of attorneys fees as part of damages is the
exception rather than the rule. The power of the court to award attorneys fees under Article 2208 of the Civil Code demands
factual, legal and equitable justification. Thus, the grant of attorneys fees against the petitioner is likewise deleted.
[15]

[16]

Incidentally, there was no question that the registered owner of the vehicle was respondent Villanueva. He never denied and in
fact admitted this fact. We have held that the registered owner of any vehicle, even if not used for public service, would primarily be
responsible to the public or to third persons for injuries caused the latter while the vehicle was being driven on the highways or
streets. Hence, with the overwhelming evidence presented by petitioner and the respondent Daniel spouses that the accident
occurred because of the detachment of the steering wheel guide of the jeep, it is not the school, but the registered owner of the
vehicle who shall be held responsible for damages for the death of Sherwin Carpitanos.
[17]

The Fallo
WHEREFORE, the Court REVERSES and SETS ASIDE the decision of the Court of Appeals and that of the trial court. The
Court remands the case to the trial court for determination of the liability of defendants, excluding petitioner St. Marys
Academy, Dipolog City.
[18]

No costs.

[19]

[G.R. No. 70890. September 18, 1992.]


CRESENCIO LIBI * and AMELIA YAP LIBI, Petitioners, v. HON. INTERMEDIATE APPELLATE COURT, FELIPE GOTIONG
and SHIRLEY GOTIONG, Respondents.
Alex Y. Tan, for Petitioners.
Mario D. Ortiz and Danilo V. Ortiz for Private Respondents.
SYLLABUS
1. CIVIL LAW; QUASI DELICT; LIABILITY OF PARENTS FOR CIVIL LIABILITY ARISING FROM CRIMINAL OFFENSES
COMMITTED BY THEIR MINOR CHILDREN; RULE. The parents are and should be held primarily liable for the civil liability
arising from criminal offenses committed by their minor children under their legal authority or control, or who live in their
company, unless it is proven that the former acted with the diligence of a good father of a family to prevent such damages. That
primary liability is premised on the provisions of Article 101 of the Revised Penal Code with respect to damages ex delicto
caused by their children 9 years of age or under, or over 9 but under 15 years of age who acted without discernment; and, with
regard to their children over 9 but under 15 years of age who acted with discernment, or 15 years or over but under 21 years of
age, such primary liability shall be imposed pursuant to Article 2180 of the Civil Code. Under said Article 2180, the enforcement
of such liability shall be effected against the father and, in case of his death or incapacity, the mother. This was amplified by the
Child and Youth Welfare Code which provides that the same shall devolve upon the father and, in case of his death or
incapacity, upon the mother or, in case of her death or incapacity, upon the guardian, but the liability may also be voluntarily
assumed by a relative or family friend of the youthful offender. However, under the Family Code, this civil liability is now, without
such alternative qualification, the responsibility of the parents and those who exercise parental authority over the minor offender.
For civil liability arising from quasi-delicts committed by minors, the same rules shall apply in accordance with Articles 2180 and
2182 of the Civil Code, as so modified.
DECISION
REGALADO, J.:
One of the ironic verities of life, it has been said, is that sorrow is sometimes a touchstone of love. A tragic illustration is provided
by the instant case, wherein two lovers died while still in the prime of their years, a bitter episode for those whose lives they
have touched. While we cannot expect to award complete assuagement to their families through seemingly prosaic legal
verbiage, this disposition should at least terminate the acrimony and rancor of an extended judicial contest resulting from the
unfortunate occurrence.
In this final denouement of the judicial recourse the stages whereof were alternately initiated by the parties, petitioners are now
before us seeking the reversal of the judgment of respondent court promulgated on January 2, 1985 in AC-G.R. CV No. 69060
with the following decretal portion:jgc:chanrobles.com.ph
"WHEREFORE, the decision of the lower court dismissing plaintiffs complaint is hereby reversed; and instead, judgment is
hereby rendered sentencing defendants, jointly and solidarily, to pay to plaintiffs the following amounts:chanrobles.com : virtual
law library
1. Moral damages, P30,000.000;
2. Exemplary damages, P10,000.00;
3. Attorneys fees, P20,000.00, and costs.
However, denial of defendants-appellees counterclaims is affirmed." 1
Synthesized from the findings of the lower courts, it appears that respondent spouses are the legitimate parents of Julie Ann
Gotiong who, at the time of the deplorable incident which took place and from which she died on January 14, 1979, was an 18year old first year commerce student of the University of San Carlos, Cebu City; while petitioners are the parents of Wendell
Libi, then a minor between 18 and 19 years of age living with his aforesaid parents, and who also died in the same event on the
same date.
For more than two (2) years before their deaths, Julie Ann Gotiong and Wendell Libi were sweethearts until December, 1978
when Julie Ann broke up her relationship with Wendell after she supposedly found him to be sadistic and irresponsible. During
the first and second weeks of January, 1979, Wendell kept pestering Julie Ann with demands for reconciliation but the latter
persisted in her refusal, prompting the former to resort to threats against her. In order to avoid him, Julie Ann stayed in the

house of her best friend, Malou Alfonso, at the corner of Maria Cristina and Juana Osmea Streets, Cebu City, from January 7
to 13, 1978.
On January 14, 1979, Julie Ann and Wendell died, each from a single gunshot wound inflicted with the same firearm, a Smith
and Wesson revolver licensed in the name of petitioner Cresencio Libi, which was recovered from the scene of the crime inside
the residence of private respondents at the corner of General Maxilom and D. Jakosalem streets of the same city.
Due to the absence of an eyewitness account of the circumstances surrounding the death of both minors, their parents, who are
the contending parties herein, posited their respective theories drawn from their interpretation of circumstantial evidence,
available reports, documents and evidence of physical facts.
Private respondents, bereaved over the death of their daughter, submitted that Wendell caused her death by shooting her with
the aforesaid firearm and, thereafter, turning the gun on himself to commit suicide. On the other hand, Petitioners, puzzled and
likewise distressed over the death of their son, rejected the imputation and contended that an unknown third party, whom
Wendell may have displeased or antagonized by reason of his work as a narcotics informer of the Constabulary Anti-Narcotics
Unit (CANU), must have caused Wendells death and then shot Julie Ann to eliminate any witness and thereby avoid
identification.chanrobles.com:cralaw:red
As a result of the tragedy, the parents of Julie Ann filed Civil Case No. R-17774 in the then Court of First Instance of Cebu
against the parents of Wendell to recover damages arising from the latters vicarious liability under Article 2180 of the Civil
Code. After trial, the court below rendered judgment on October 20, 1980 as follows:jgc:chanrobles.com.ph
"WHEREFORE, premises duly considered, judgment is hereby rendered dismissing plaintiffs complaint for insufficiency of the
evidence. Defendants counterclaim is likewise denied for lack of sufficient merit." 2
On appeal to respondent court, said judgment of the lower court dismissing the complaint of therein plaintiffs-appellants was set
aside and another judgment was rendered against defendants-appellees who, as petitioners in the present appeal by certiorari,
now submit for resolution the following issues in this case:chanrob1es virtual 1aw library
1. Whether or not respondent court correctly reversed the trial court in accordance with established decisional laws; and
2. Whether or not Article 2180 of the Civil Code was correctly interpreted by respondent court to make petitioners liable for
vicarious liability. 3
In the proceedings before the trial court, Dr. Jesus P. Cerna, Police Medico-Legal Officer of Cebu, submitted his findings and
opinions on some postulates for determining whether or not the gunshot wound was inflicted on Wendell Libi by his own suicidal
act. However, undue emphasis was placed by the lower court on the absence of gunpowder or tattooing around the wound at
the point of entry of the bullet. It should be emphasized, however, that this is not the only circumstance to be taken into account
in the determination of whether it was suicide or not.
It is true that said witness declared that he found no evidence of contact or close-contact of an explosive discharge in the
entrance wound. However, as pointed out by private respondents, the body of deceased Wendell Libi must have been washed
at the funeral parlor, considering the hasty interment thereof a little after eight (8) hours from the occurrence wherein he died.
Dr. Cerna himself could not categorically state that the body of Wendell Libi was left untouched at the funeral parlor before he
was able to conduct his autopsy. It will also be noted that Dr. Cerna was negligent in not conducting a paraffin test on Wendell
Libi, hence possible evidence of gunpowder residue on Wendells hands was forever lost when Wendell was hastily
buried.cralawnad
More specifically, Dr. Cerna testified that he conducted an autopsy on the body of Wendell Libi about eight (8) hours after the
incident or, to be exact, eight (8) hours and twenty (20) minutes based on the record of death; that when he arrived at the
Cosmopolitan Funeral Homes, the body of the deceased was already on the autopsy table and in the stage of rigor mortis; and
that said body was not washed, but it was dried. 4 However, on redirect examination, he admitted that during the 8-hour interval,
he never saw the body nor did he see whether said body was wiped or washed in the area of the wound on the head which he
examined because the deceased was inside the morgue. 5 In fact, on cross-examination, he had earlier admitted that as far as
the entrance of the wound, the trajectory of the bullet and the exit of the wound are concerned, it is possible that Wendell Libi
shot himself. 6
He further testified that the muzzle of the gun was not pressed on the head of the victim and that he found no burning or
singeing of the hair or extensive laceration on the gunshot wound of entrance which are general characteristics of contact or
near-contact fire. On direct examination, Dr. Cerna nonetheless made these clarification:jgc:chanrobles.com.ph
"Q Is it not a fact that there are certain guns which are so made that there would be no black residue or tattooing that could
result from these guns because they are what we call clean?
A Yes, sir. I know that there are what we call smokeless powder.
ATTY. ORTIZ:chanrob1es virtual 1aw library
Q Yes. So, in cases, therefore, of guns where the powder is smokeless, those indications that you said may not rule out the
possibility that the gun was closer than 24 inches, is that correct?
A If the . . . assuming that the gun used was .. the bullet used was a smokeless powder.
Q At any rate, doctor, from . . . disregarding those other matters that you have noticed, the singeing, etc., from the trajectory,
based on the trajectory of the bullet as shown in your own sketch, is it not a fact that the gun could have been fired by the
person himself, the victim himself, Wendell Libi, because it shows a point of entry a little above the right ear and point of exit a
little above that, to be very fair and on your oath?
A As far as the point of entrance is concerned and as far as the trajectory of the bullet is concerned and as far as the angle or
the manner of fire is concerned, it could have been fired by the victim." 7
As shown by the evidence, there were only two used bullets 8 found at the scene of the crime, each of which were the bullets
that hit Julie Ann Gotiong and Wendell Libi, respectively. Also, the sketch prepared by the Medico-Legal Division of the National
Bureau of Investigation, 9 shows that there is only one gunshot wound of entrance located at the right temple of Wendell Libi.
The necropsy report prepared by Dr. Cerna states:chanrob1es virtual 1aw library
x

"Gunshot wound, ENTRANCE, ovaloid, 0.5 x 0.4 cm., with contusion collar widest inferiorly by 0.2 cm., edges inverted, oriented
upward, located at the head, temporal region, right, 2.8 cms. behind and 5.5 cms. above right external auditory meatus, directed
slightly forward, upward and to the left, involving skin and soft tissues, making a punch-in fracture on the temporal bone, right,
penetrating cranial cavity, lacerating extensively along its course the brain tissues, fracturing parietal bone, left, and finally

making an EXIT wound, irregular, 2.0 x 1.8 cms., edges (e)verted, parietal region, left, 2.0 cms. behind and 12.9 cms. above left
external auditory meatus.chanrobles virtualawlibrary chanrobles.com:chanrobles.com.ph
x

"Evidence of contact or close-contact fire, such as burning around the gunshot wound of entrance, gunpowder tatooing (sic),
smudging, singeing of hair, extensive laceration or bursting of the gunshot wound of entrance, or separation of the skin from the
underlying tissue, are absent." 10
On cross-examination, Dr. Cerna demonstrated his theory which was made of record, thus:jgc:chanrobles.com.ph
"Q Now, will you please use yourself as Wendell Libi, and following the entrance of the wound, the trajectory of the bullet and
the exit of the wound, and measuring yourself 24 inches, will you please indicate to the Honorable Court how would it have
been possible for Wendell Libi to kill himself? Will you please indicate the 24 inches?
WITNESS:chanrob1es virtual 1aw library
A Actually, sir, the 24 inches is approximately one arms length.
ATTY. SENINING:chanrob1es virtual 1aw library
I would like to make of record that the witness has demonstrated by extending his right arm almost straight towards his head."
11
Private respondents assail the fact that the trial court gave credence to the testimonies of defendants witnesses Lydia Ang and
James Enrique Tan, the first being a resident of an apartment across the street from the Gotiongs and the second, a resident of
the house adjacent to the Gotiong residence, who declared having seen a "shadow" of a person at the gate of the Gotiong
house after hearing shots therefrom.
On cross-examination, Lydia Ang testified that the apartment where she was staying faces the gas station; that it is the second
apartment; that from her window she can see directly the gate of the Gotiongs and, that there is a firewall between her
apartment and the gas station. 12 After seeing a man jump from the gate of the Gotiongs to the rooftop of the Tans, she called
the police station but the telephone lines were busy. Later on, she talked with James Enrique Tan and told him that she saw a
man leap from the gate towards his rooftop. 13
However, James Enrique Tan testified that he saw a "shadow" on top of the gate of the Gotiongs, but denied having talked with
anyone regarding what he saw. He explained that he lives in a duplex house with a garden in front of it; that his house is next to
Felipe Gotiongs house; and he further gave the following answers to these questions:chanrobles.com : virtual law library
"ATTY. ORTIZ: (TO WITNESS).
Q What is the height of the wall of the Gotiongs in relation to your house?
WITNESS:chanrob1es virtual 1aw library
A It is about 8 feet.
ATTY. ORTIZ: (TO WITNESS)
Q And where were you looking from?
WITNESS:chanrob1es virtual 1aw library
A From upstairs in my living room.
ATTY. ORTIZ (TO WITNESS)
Q From Your living room window, is that correct?
WITNESS:chanrob1es virtual 1aw library
A Yes, but not very clear because the wall is high." 14
Analyzing the foregoing testimonies, we agree with respondent court that the same do not inspire credence as to the reliability
and accuracy of the witnesses observations, since the visual perceptions of both were obstructed by high walls in their
respective houses in relation to the house of herein private respondents. On the other hand, witness Manolo Alfonso, testifying
on rebuttal, attested without contradiction that he and his sister, Malou Alfonso, were waiting for Julie Ann Gotiong when they
heard her scream; that when Manolo climbed the fence to see what was going on inside the Gotiong house, he heard the first
shot; and, not more than five (5) seconds later, he heard another shot. Consequently, he went down from the fence and drove to
the police station to report the incident. 15 Manolos direct and candid testimony establishes and explains the fact that it was he
whom Lydia Ang and James Enrique Tan saw as the "shadow" of a man at the gate of the Gotiong house.
We have perforce to reject petitioners effete and unsubstantiated pretension that it was another man who shot Wendell and
Julie Ann. It is significant that the Libi family did not even point to or present any suspect in the crime nor did they file any case
against any alleged "John Doe." Nor can we sustain the trial courts dubious theory that Wendell Libi did not die by his own hand
because of the overwhelming evidence testimonial, documentary and pictorial the confluence of which point to Wendell as
the assailant of Julie Ann, his motive being revenge for her rejection of his persistent pleas for a
reconciliation.chanrobles.com:cralaw:red
Petitioners defense that they had exercised the due diligence of a good father of a family, hence they should not be civilly liable
for the crime committed by their minor son, is not borne out by the evidence on record either.
Petitioner Amelita Yap Libi, mother of Wendell, testified that her husband, Cresencio Libi, owns a gun which he kept in a safety
deposit box inside a drawer in their bedroom. Each of these petitioners holds a key to the safety deposit box and Amelitas key
is always in her bag, all of which facts were known to Wendell. They have never seen their son Wendell taking or using the gun.
She admitted, however, that on that fateful night the gun was no longer in the safety deposit box. 16 We, accordingly, cannot but
entertain serious doubts that petitioner spouses had really been exercising the diligence of a good father of a family by safely
locking the fatal gun away. Wendell could not have gotten hold thereof unless one of the keys to the safety deposit box was
negligently left lying around or he had free access to the bag of his mother where the other key was.
The diligence of a good father of a family required by law in a parent and child relationship consists, to a large extent, of the
instruction and supervision of the child. Petitioners were gravely remiss in their duties as parents in not diligently supervising the
activities of their son, despite his minority and immaturity, so much so that it was only at the time of Wendells death that they

allegedly discovered that he was a CANU agent and that Cresencios gun was missing from the safety deposit box. Both
parents were sadly wanting in their duty and responsibility in monitoring and knowing the activities of their children who, for all
they know, may be engaged in dangerous work such as being drug informers, 17 or even drug users. Neither was a plausible
explanation given for the photograph of Wendell, with a handwritten dedication to Julie Ann at the back thereof, 18 holding
upright what clearly appears as a revolver and on how or why he was in possession of that firearm.
In setting aside the judgment of the court a quo and holding petitioners civilly liable, as explained at the start of this opinion,
respondent court waved aside the protestations of diligence on the part of petitioners and had this to say:jgc:chanrobles.com.ph
". . . It is still the duty of parents to know the activity of their children who may be engaged in this dangerous activity involving the
menace of drugs. Had the defendants-appellees been diligent in supervising the activities of their son, Wendell, and in keeping
said gun from his reach, they could have prevented Wendell from killing Julie Ann Gotiong. Therefore, appellants are liable
under Article 2180 of the Civil Code which provides:chanrob1es virtual 1aw library
The father, and in case of his death or incapacity, the mother, are responsible for the damages caused by their minor children
who live in their company.
"Having been grossly negligent in preventing Wendell Libi from having access to said gun which was allegedly kept in a safety
deposit box, defendants-appellees are subsidiarily liable for the natural consequence of the criminal act of said minor who was
living in their company. This vicarious liability of herein defendants-appellees has been reiterated by the Supreme Court in many
cases, prominent of which is the case of Fuellas v. Cadano, et. al. (L-14409, Oct. 31, 1961, 3 SCRA 361-367), which held
that:chanrob1es virtual 1aw library
The subsidiary liability of parents for damages caused by their minor children imposed by Article 2180 of the New Civil Code
covers obligations arising from both quasi-delicts and criminal offenses.
The subsidiary liability of parents arising from the criminal acts of their minor children who acted with discernment is
determined under the provisions of Article 2180, N.C.C. and under Article 101 of the Revised Penal Code, because to hold that
the former only covers obligations which arise from quasi-delicts and not obligations which arise from criminal offenses, would
result in the absurdity that while for an act where mere negligence intervenes the father or mother may stand subsidiarily liable
for the damages caused by his or her son, no liability would attach if the damage is caused with criminal intent. (3 SCRA 361362).
". . . In the instant case, minor son of herein defendants-appellees, Wendell Libi somehow got hold of the key to the drawer
where said gun was kept under lock without defendant-spouses ever knowing that said gun had been missing from that safety
box since 1978 when Wendell Libi had) a picture taken wherein he proudly displayed said gun and dedicated this picture to his
sweetheart, Julie Ann Gotiong; also since then, Wendell Libi was said to have kept said gun in his car, in keeping up with his
supposed role of a CANU agent . . ." chanrobles lawlibrary : rednad
x

"Based on the foregoing discussions of the assigned errors, this Court holds that the lower court was not correct in dismissing
herein plaintiffs-appellants complaint because as preponderantly shown by evidence, defendants-appellees utterly failed to
exercise all the diligence of a good father of the family in preventing their minor son from committing this crime by means of the
gun of defendants-appellees which was freely accessible to Wendell Libi for they have not regularly checked whether said gun
was still under lock, but learned that it was missing from the safety deposit box only after the crime had been committed."
(Emphases ours.) 19
We agree with the conclusion of respondent court that petitioners should be held liable for the civil liability based on what
appears from all indications was a crime committed by their minor son. We take this opportunity, however, to digress and
discuss its ratiocination therefor on jurisprudential dicta which we feel require clarification.
In imposing sanctions for the so-called vicarious liability of petitioners, respondent court cites Fuellas v. Cadano, Et. Al. 20 which
supposedly holds that" (t)he subsidiary liability of parents for damages caused by their minor children imposed by Article 2180 of
the New Civil Code covers obligations arising from both quasi-delicts and criminal offenses," followed by an extended quotation
ostensibly from the same case explaining why under Article 2180 of the Civil Code and Article 101 of the Revised Penal Code
parents should assume subsidiary liability for damages caused by their minor children. The quoted passages are set out two
paragraphs back, with pertinent underscoring for purposes of the discussion hereunder.chanrobles law library
Now, we do not have any objection to the doctrinal rule holding, the parents liable, but the categorization of their liability as
being subsidiary, and not primary, in nature requires a hard second look considering previous decisions of this court on the
matter which warrant comparative analyses. Our concern stems from our readings that if the liability of the parents for crimes or
quasi-delicts of their minor children is subsidiary, then the parents can neither invoke nor be absolved of civil liability on the
defense that they acted with the diligence of a good father of a family to prevent damages. On the other hand, if such liability
imputed to the parents is considered direct and primary, that diligence would constitute a valid and substantial defense.
We believe that the civil liability of parents for quasi-delicts of their minor children, as contemplated in Article 2180 of the Civil
Code, is primary and not subsidiary. In fact, if we apply Article 2194 of said code which provides for solidary liability of joint
tortfeasors, the persons responsible for the act or omission, in this case the minor and the father and, in case of his death of
incapacity, the mother, are solidarily liable. Accordingly, such parental liability is primary and not subsidiary, hence the last
paragraph of Article 2180 provides that" (t)he responsibility treated of in this article shall cease when the persons herein
mentioned prove that they observed all the diligence of a good father of a family to prevent damages."cralaw virtua1aw library
We are also persuaded that the liability of the parents for felonies committed by their minor children is likewise primary, not
subsidiary. Article 101 of the Revised Penal Code provides:jgc:chanrobles.com.ph
"ARTICLE 101. Rules regarding civil liability in certain cases.
x

First. In cases of subdivisions . . . 2, and 3 of Article 12, the civil liability for acts committed by . . . a person under nine years of
age, or by one over nine but under fifteen years of age, who has acted without discernment, shall devolve upon those having
such person under their legal authority or control, unless it appears that there was no fault or negligence on their part."
(Emphasis supplied.) 21
Accordingly, just like the rule in Article 2180 of the Civil Code, under the foregoing provision the civil liability of the parents for
crimes committed by their minor children is likewise direct and primary, and also subject to the defense of lack of fault or
negligence on their part, that is, the exercise of the diligence of a good father of a family.
That in both quasi-delicts and crimes the parents primarily respond for such damages is buttressed by the corresponding
provisions in both codes that the minor transgressor shall be answerable or shall respond with his own property only in the
absence or in case of insolvency of the former. Thus, for civil liability ex quasi delicto of minors, Article 2182 of the Civil Code

states that" (i)f the minor causing damage has no parents or guardian, the minor . . . shall be answerable with his own property
in an action against him where a guardian ad litem shall be appointed." For civil liability ex delicto of minors, an equivalent
provision is found in the third paragraph of Article 101 of the Revised Penal Code, to wit:jgc:chanrobles.com.ph
"Should there be no person having such . . . minor under his authority, legal guardianship or control, or if such person be
insolvent, said . . . minor shall respond with (his) own property, excepting property exempt from execution, in accordance with
civil law."cralaw virtua1aw library
The civil liability of parents for felonies committed by their minor children contemplated in the aforesaid rule in Article 101 of the
Revised Penal Code in relation to Article 2180 of the Civil Code has, aside from the aforecited case of Fuellas, been the subject
of a number of cases adjudicated by this Court, viz.: Exconde v. Capuno, Et Al., 22 Araneta v. Arreglado, 23 Salen, Et. Al. v.
Balce, 24 Paleyan, etc., Et. Al. v. Bangkili, Et Al., 25 and Elcano, et al, v. Hill, Et. Al. 26 Parenthetically, the aforesaid cases were
basically on the issue of the civil liability of parents for crimes committed by their minor children over 9 but under 15 years of
age, who acted with discernment, and also of minors 15 years of aye or over, since these situations are not covered by Article
101, Revised Penal Code. In both instances, this Court held that the issue of parental civil liability should be resolved in
accordance with the provisions of Article 2180 of the Civil Code for the reasons well expressed in Salen and adopted in the
cases hereinbefore enumerated that to hold that the civil liability under Article 2180 would apply only to quasi-delicts and not to
criminal offenses would result in the absurdity that in an act involving mere negligence the parents would be liable but not where
the damage is caused with criminal intent. In said cases, however, there are unfortunate variances resulting in a regrettable
inconsistency in the Courts determination of whether the liability of the parents, in cases involving either crimes or quasi-delicts
of their minor children, is primary or subsidiary.
In Exconde, where the 15-year old minor was convicted of double homicide through reckless imprudence, in a separate civil
action arising from the crime the minor and his father were held jointly and severally liable for failure of the latter to prove the
diligence of a good father of a family. The same liability in solidum and, therefore, primary liability was imposed in a separate
civil action in Araneta on the parents and their 14-year old son who was found guilty of frustrated homicide, but on the authority
of Article 2194 of the Civil Code providing for solidary responsibility of two or more persons who are liable for a quasi-delict.
However, in Salen, the father was declared subsidiarily liable for damages arising from the conviction of his son, who was over
15 but less than 18 years of age, by applying Article 2180 but, this time, disregarding Article 2194 of the Civil Code. In the
present case, as already explained, the petitioners herein were also held liable but supposedly in line with Fuellas which
purportedly declared the parents subsidiarily liable for the civil liability for serious physical injuries committed by their 13-year old
son. On the other hand, in Paleyan, the mother and her 19-year old son were adjudged solidarily liable for damages arising from
his conviction for homicide by the application of Article 2180 of the Civil Code since this is likewise not covered by Article 101 of
the Revised Penal Code. Finally, in Elcano, although the son was acquitted in a homicide charge due to "lack of intent, coupled
with mistake," it was ruled that while under Article 2180 of the Civil Code there should be solidary liability for damages, since the
son, "although married, was living with his father and getting subsistence from him at the time of the occurrence," but "is now of
age, as a matter of equity" the father was only held subsidiarily liable.
It bears stressing, however, that the Revised Penal Code provides for subsidiary liability only for persons causing damages
under the compulsion of irresistible force or under the impulse of an uncontrollable fear; 27 innkeepers, tavernkeepers and
proprietors of establishments; 28 employers, teachers, persons and corporations engaged in industry; 29 and principals,
accomplices and accessories for the unpaid civil liability of their co-accused in the other classes. 30
Also, coming back to respondent courts reliance on Fuellas in its decision in the present case, it is not exactly accurate to say
that Fuellas provided for subsidiary liability of the parents therein. A careful scrutiny shows that what respondent court quoted
verbatim in its decision now on appeal in the present case, and which it attributed to Fuellas, was the syllabus on the law report
of said case which spoke of "subsidiary" liability. However, such categorization does not specifically appear in the text of the
decision in Fuellas. In fact, after reviewing therein the cases of Exconde, Araneta and Salen and the discussions in said cases
of Article 101 of the Revised Penal Code in relation to Article 2180 of the Civil Code, this Court concluded its decision in this
wise:jgc:chanrobles.com.ph
"Moreover, the case at bar was decided by the Court of Appeals on the basis of evidence submitted therein by both parties,
independent of the criminal case. And responsibility for fault or negligence under Article 2176 upon which the present action was
instituted, is entirely separate and distinct from the civil liability arising from fault or negligence under the Penal Code (Art.
2177), and having in mind the reasons behind the law as heretofore stated, any discussion as to the minors criminal
responsibility is of no moment."cralaw virtua1aw library
Under the foregoing considerations, therefore, we hereby rule that the parents are and should be held primarily liable for the
civil liability arising from criminal offenses committed by their minor children under their legal authority or control, or who live in
their company, unless it is proven that the former acted with the diligence of a good father of a family to prevent such damages.
That primary liability is premised on the provisions of Article 101 of the Revised Penal Code with respect to damages ex delicto
caused by their children 9 years of age or under, or over 9 but under 15 years of age who acted without discernment; and, with
regard to their children over 9 but under 15 years of age who acted with discernment, or 15 years or over but under 21 years of
age, such primary liability shall be imposed pursuant to Article 2180 of the Civil Code. 31
Under said Article 2180, the enforcement of such liability shall be effected against the father and, in case of his death or
incapacity, the mother. This was amplified by the Child and Youth Welfare Code which provides that the same shall devolve
upon the father and, in case of his death or incapacity, upon the mother or, in case of her death or incapacity, upon the
guardian, but the liability may also be voluntarily assumed by a relative or family friend of the youthful offender. 32 However,
under the Family Code, this civil liability is now, without such alternative qualification, the responsibility of the parents and those
who exercise parental authority over the minor offender. 33 For civil liability arising from quasi-delicts committed by minors, the
same rules shall apply in accordance with Articles 2180 and 2182 of the Civil Code, as so modified.
In the case at bar, whether the death of the hapless Julie Ann Gotiong was caused by a felony or a quasi-delict committed by
Wendell Libi, respondent court did not err in holding petitioners liable for damages arising therefrom. Subject to the preceding
modifications of the premises relied upon by it therefor and on the bases of the legal imperatives herein explained, we conjoin in
its findings that said petitioners failed to duly exercise the requisite diligentissimi patris familias to prevent such damages.
ACCORDINGLY, the instant Petition is DENIED and the assailed judgment of respondent Court of Appeals is hereby
AFFIRMED, with costs against petitioners.
SO ORDERED.

Use of Surnames
G.R. No. L-18008

October 30, 1962

ELISEA LAPERAL, petitioner,


vs.
REPUBLIC OF THE PHILIPPINES, oppositor.
Martin B. Laurea and Associates for petitioner.
Office of the Solicitor General for oppositor.
BARRERA, J.:
On May 10, 1960, Elisea Laperal filed in the Court of First Instance of Baguio (Sp Proc. No. 433) a petition which reads:
1. That petitioner has been a bona fide resident of the City of Baguio for the last three years prior to the date of the filing of
this petition;
2. That petitioner's maiden name is ELISEA LAPERAL; that on March 24, 1939, she married Mr. Enrique R. Santamaria;
that in a partial decision entered on this Honorable Court on January 18, 1958, in Civil Case No. 356 of this Court, entitled
'Enrique R. Santamaria vs. Elisea L. Santamaria' Mr. Enrique Santamaria was given a decree of legal separation from her;
that the said partial decision is now final;
3. That during her marriage to Enrique R. Santamaria, she naturally used, instead of her maiden name, that of Elisea L.
Santamaria; that aside from her legal separation from Enrique R. Santamaria, she has also ceased to live with him for
many years now;
4. That in view of the fact that she has been legally separated from Mr. Enrique R. Santamaria and has likewise ceased to
live with him for many years, it is desirable that she be allowed to change her name and/or be permitted to resume using
her maiden name, to wit: ELISEA LAPERAL.
WHEREFORE, petitioner respectfully prayed that after the necessary proceedings are had, she be allowed to resume
using her maiden name of Elisea Laperal.
The petition was opposed by the City Attorney of Baguio on the ground that the same violates the provisions of Article 370
(should be 372) of the Civil Code, and that it is not sanctioned by the Rules of Court.
In its decision of October 31, 1960, the court denied the petition for the reason that Article 372 of the Civil Code requires the
wife, even after she is decreed legally separated from her husband, to continue using the name and surname she employed
before the legal separation. Upon petitioner's motion, however, the court, treating the petition as one for change of name,
reconsidered its decision and granted the petition on the ground that to allow petitioner, who is a businesswoman decreed
legally separated from her husband, to continue using her married name would give rise to confusion in her finances and the
eventual liquidation of the conjugal assets. Hence, this appeal by the State.
The contention of the Republic finds support in the provisions of Article 372 of the New Civil Code which reads:
ART. 372. When legal separation has been granted, the wife shall continue using her name and surname employed
before the legal separation. (Emphasis supplied)
Note that the language of the statute is mandatory that the wife, even after the legal separation has been decreed, shall
continue using her name and surname employed before the legal separation. This is so because her married status is
unaffected by the separation, there being no severance of the vinculum. It seems to be the policy of the law that the wife should
continue to use the name indicative of her unchanged status for the benefit of all concerned.
The appellee contends, however, that the petition is substantially for change of her name from Elisea L. Santamaria, the one
she has been using, since her marriage, to Elisea Laperal, her maiden name, giving as reason or cause therefor her being
legally separated from the husband Enrique R. Santamaria, and the fact that they have ceased to live together for many years.
There seems to be no dispute that in the institution of these proceedings, the procedure prescribed in Rule 103 of the Rules of
Court for change of name has been observed. But from the petition quoted in full at the beginning of these opinion, the only
reason relied upon for the change of name is the fact that petitioner is legally separated from her husband and has, in fact,
ceased to live with him for many years. It is doubtful, to say the least, whether Rule 103 which refers to change of name in
general, may prevail over the specific provisions of Article 372 of the New Civil Code with regards to married women legally
separated from their husbands. Even, however, applying Rule 103 to this case, the fact of legal separation alone which is the
only basis for the petition at bar is, in our opinion, not a sufficient ground to justify a change of the name of herein petitioner,
for to hold otherwise would be to provide an easy circumvention of the mandatory provisions of Article 372.
It is true that in the second decision which reconsidered the first it is stated that as the petitioner owns extensive business
interests, the continued used of her husband surname may cause undue confusion in her finances and the eventual liquidation
of the conjugal assets. This finding is however without basis. In the first place, these were not the causes upon which the

petition was based; hence, obviously no evidence to this effect had been adduced. Secondly, with the issuance of the decree of
legal separation in 1958, the conjugal partnership between petitioner and her husband had automatically been dissolved and
liquidated. (Art. 106[2], Civil Cod). Consequently, there could be no more occasion for an eventual liquidation of the conjugal
assets.
WHEREFORE, the order of the lower court of December 1, 1960, granting the petition, is hereby set aside and the petition
dismissed. Without costs. So ordered.

G.R. No. L-32054 May 15, 1974


TERESITA LLANETA (known also as TERESITA LLANETA FERRER and TERESITA FERRER), petitioner,
vs.
The Honorable CORAZON JULIANO AGRAVA, as Presiding Judge of the Juvenile and Domestic Relations Court of
Manila, respondent.
Pascual G. Mier for petitioner.
Office of the Solicitor General Felix Q. Antonio, Acting Assistant Solicitor General Ricardo L. Pronove, Jr. and Trial Attorney
Quirino B. Maglente, Jr. for respondent.

CASTRO, J.:p
From the denial by the respondent Juvenile and Domestic Relations Court of Manila, in its special proceeding H-00237, of her
petition for change of name, Teresita Llaneta has come to this Court on appeal by certiorari.
Teresita's mother, one Atanacia Llaneta, was once married to Serafin Ferrer with whom she had but one child named Victoriano
Ferrer. In 1942 Serafin Ferrer died, and about four years later Atanacia had relations with another man out of which Teresita was
born. Shortly after Teresita's birth, Atanacia brought her and Victoriano to Manila where all of them lived with Atanacia's motherin-law, Victoria vda. de Ferrer. Teresita was raised in the household of the Ferrer's, using the surname of Ferrer in all her
dealings and throughout her schooling. When she was about twenty years old, she applied for a copy of her birth certificate in
Irosin, Sorsogon, where she was born, as she was required to present it in connection with a scholarship granted to her by the
Catholic Charities. It was then that she discovered that her registered surname is Llaneta not Ferrer and that she is the
illegitimate child of Atanacia and an unknown father.
On the ground that her use thenceforth of the surname Llaneta, instead of Ferrer which she had been using since she acquired
reason, would cause untold difficulties and confusion, Teresita petitioned the court below on March 18, 1969 for change of her
name from Teresita Llaneta to Teresita Llaneta Ferrer. After trial duly had, the respondent judge denied her petition; hence the
present recourse.
The petitioner has established that she has been using the surname Ferrer for as long as she can remember; that all her
records, in school and elsewhere, put her name down as Teresita Ferrer; that her friends and associates know her only as
Teresita Ferrer; and that even the late Serafin Ferrer's nearest of kin (who apparently have kept Teresita's illegitimacy a secret
from her) have tolerated and still approve of her use of the surname Ferrer. Indeed, a sudden shift at this time by the petitioner
to the name Teresita Llaneta (in order to conform to that appearing in her birth certificate) would result in confusion among the
persons and entities she deals with and entail endless and vexatious explanations of the circumstances of her new surname. 1 In
her official dealings, this would likewise mean a lifelong fending with the necessary affidavits. Moreover, it is a salutary law that would allow Teresita, inspite of her illegitimate birth,
to carry on in society without her unfortunate status being bandied about at every turn. 2

The respondent court places reliance on the doctrine, expounded in three decisions of this Court, 3 that disallows such change of name as
would give the false impression of family relationship. The principle remains valid but only to the extent that the proposed change of name would in great probability cause
prejudice or future mischief to the family whose surname it is that is involved or to the community in general. In the case at bar, however, the late Serafin Ferrer's widowed mother,
Victoria, and his two remaining brothers, Nehemias and Ruben, have come forward in earnest support of the petition. Adequate publication of the proceeding has not elicited the
slightest opposition from the relatives and friends of the late Serafin Ferrer. Clearances from various Government agencies show that Teresita has a spotless record. And the State
(represented by the Solicitor General's Office), which has an interest in the name borne by every citizen within its realm for purposes of identification, interposed no opposition at
the trial after a searching cross-examination, of Teresita and her witnesses. Whether the late Serafin Ferrer, who died some five years before Teresita was born, would have
consented or objected to her use of his surname is open to speculation. One thing, however, is beyond cavil: those living who possess the right of action to prevent the surname
Ferrer from being smeared are proud to share it with her.

ACCORDINGLY, the judgment a quo is reversed, and the petition of Teresita Llaneta for change of her name to Teresita Llaneta
Ferrer is hereby granted. Let a copy of this decision be forwarded to the civil registrar of Irosin, Sorsogon, for this information
and proper action. No costs.

Civil Registrar
Republic Act No. 9048
Republic of the Philippines
Congress of the Philippines Metro Manila Eleventh Congress
Third Regular Session
Begun and held in Metro Manila, on Monday, the twenty-fourth day of July, two thousand.
[REPUBLIC ACT NO. 9048]
AN ACT AUTHORIZING THE CITY OR MUNICIPAL CIVIL REGISTRAR OR THE CONSUL GENERAL TO CORRECT A CLERICAL
OR TYPOGRAPHICAL ERROR IN AN ENTRY AND/OR CHANGE OF FIRST NAME OR NICKNAME IN THE CIVIL REGISTER
WITHOUT NEED OF A JUDICIAL ORDER, AMENDING FOR THIS PURPOSE ARTICLES 376 AND 412 OF THE CIVIL CODE
OF THE PHILIPPINES.
Be it enacted by the Senate and the House of Representatives of the Philippines in Congress assembled:
SECTION 1. Authority to Correct Clerical or Typographical Error and Change of First Name or Nickname - No
entry in a civil register shall be changed or corrected without a judicial order, except for clerical or typographical errors
and change of first name or nickname which can be corrected or changed by the concerned city or municipal civil
registrar or consul general in accordance with the provisions of this Act and its implementing rules and regulations.
SECTION 2. Definition of Terms - As used in this Act, the following terms shall mean:
1.

2.
3.

4.
5.
6.

"City or Municipal civil registrar" refers to the head of the local civil registry office of the city or municipality,
as the case may be, who is appointed as such by the city or municipal mayor in accordance with the provisions of
existing laws.
"Petitioner" refers to a natural person filing the petition and who has direct and personal interest in the
correction of a clerical or typographical error in an entry or change of first name or nickname in the civil register.
"Clerical or typographical error" refers to a mistake committed in the performance of clerical work in writing,
copying, transcribing or typing an entry in the civil register that is harmless and innocuous, such as misspelled
name or misspelled place of birth or the like, which is visible to the eyes or obvious to the understanding, and can
be corrected or changed only by reference to other existing record or records: Provided, however, That no
correction must involve the change of nationality, age, status or sex of the petitioner.
"Civil Register" refers to the various registry books and related certificates and documents kept in the archives
of the local civil registry offices, Philippine Consulates and of the Office of the Civil Registrar General.
"Civil registrar general" refers to the Administrator of the National Statistics Office which is the agency
mandated to carry out and administer the provision of laws on civil registration.
"First name" refers to a name or nickname given to a person which may consist of one or more names in
addition to the middle and last names.

SECTION 3. Who May File the Petition and Where. - Any person having direct and personal interest in the
correction of a clerical or typographical error in an entry and/or change of first name or nickname in the civil register
may file, in person, a verified petition with the local civil registry office of the city or municipality where the record being
sought to be corrected or changed is kept.
In case the petitioner has already migrated to another place in the country and it would not be practical for such
party, in terms of transportation expenses, time and effort to appear in person before the local civil registrar keeping the
documents to be corrected or changed, the petition may be filed, in person, with the local civil registrar of the place
where the interested party is presently residing or domiciled. The two (2) local civil registrars concerned will then
communicate to facilitate the processing of the petition.
Citizens of the Philippines who are presently residing or domiciled in foreign countries may file their petition, in
person, with the nearest Philippine Consulates.
The petitions filed with the city or municipal civil registrar or the consul general shall be processed in accordance
with this Act and its implementing rules and regulations.
All petitions for the clerical or typographical errors and/or change of first names or nicknames may be availed of
only once.
SECTION 4. Grounds for Change of First Name or Nickname. - The petition for change of first name or nickname
may be allowed in any of the following cases:

1. The petitioner finds the first name or nickname to be ridiculous, tainted with dishonor or extremely difficult to
write or pronounce.
2. The new first name or nickname has been habitually and continuously used by the petitioner and he has been
publicly known by that by that first name or nickname in the community: or
3. The change will avoid confusion.
SECTION 5. Form and Contents of the Petition. - The petition shall be in the form of an affidavit, subscribed and
sworn to before any person authorized by the law to administer oaths. The affidavit shall set forth facts necessary to
establish the merits of the petition and shall show affirmatively that the petitioner is competent to testify to the matters
stated. The petitioner shall state the particular erroneous entry or entries, which are sought to be corrected and/or the
change sought to be made.
The petition shall be supported with the following documents:
1. A certified true machine copy of the certificate or of the page of the registry book containing the entry or entries
sought to be corrected or changed.
2. At least two (2) public or private documents showing the correct entry or entries upon which the correction or
change shall be based; and
3. Other documents which the petitioner or the city or municipal civil registrar or the consul general may consider
relevant and necessary for the approval of the petition.
In case of change of first name or nickname, the petition shall likewise be supported with the documents
mentioned in the immediately preceding paragraph. In addition, the petition shall be published at least once a week for
two (2) consecutive weeks in a newspaper of general circulation. Furthermore, the petitioner shall submit a certification
from the appropriate law enforcement agencies that he has no pending case or no criminal record.
The petition and its supporting papers shall be filed in three (3) copies to be distributed as follows: first copy to
the concerned city or municipal civil registrar, or the consul general; second copy to the Office of the Civil Registrar
General; and third copy to the petitioner.
SECTION 6. Duties of the City or Municipal Civil Registrar or the Consul General. - The city or municipal civil
registrar or the consul general to whom the petition is presented shall examine the petition and its supporting
documents. He shall post the petition in a conspicuous place provided for that purpose for ten (10) consecutive days
after he finds the petition and its supporting documents sufficient in form and substance.
The city or municipal civil registrar or the consul general shall act on the petition and shall render a decision not
later than five (5) working days after the completion of the posting and/or publication requirement. He shall transmit a
copy of his decision together with the records of the proceedings to the Office of the Civil Registrar General within five
(5) working days from the date of the decision.
SECTION 7. Duties and Powers of the Civil Registrar General. - The civil registrar general shall, within ten (10)
working days from receipt of the decision granting a petition, exercise the power to impugn such decision by way of an
objection based on the following grounds:
1. The error is not clerical or typographical;
2. The correction of an entry or entries in the civil register is substantial or controversial as it affects the civil status
of a person; or
3. The basis used in changing the first name or nickname of a person does not fall under SECTION 4.
The civil registrar general shall immediately notify the city or municipal civil registrar or the consul general of the
action taken on the decision. Upon receipt of the notice thereof, the city or municipal civil registrar or the consul general
shall notify the petitioner of such action.
The petitioner may seek reconsideration with the civil registrar general or file the appropriate petition with the
proper court.
If the civil registrar general fails to exercise his power to impugn the decision of the city or municipal civil
registrar or of the consul general within the period prescribed herein, such decision shall become final and executory.
Where the petition is denied by the city or municipal civil registrar or the consul general, the petitioner may either
appeal the decision to the civil registrar general or file the appropriate petition with the proper court.
SECTION 8. Payment of Fees. - The city or municipal civil registrar or the consul general shall be authorized to collect
reasonable fees as a condition for accepting the petition. An indigent petitioner shall be exempt from the payment of the
said fee.
SECTION 9. Penalty Clause. - A person who violates any of the provisions of this Act shall, upon conviction, be
penalized by imprisonment of not less than six (6) years but not more than twelve (12) years, or a fine of not less than
Ten thousand pesos (P10,000.00) but not more than One Hundred Thousand pesos (P100,000.00), or both, at the
discretion of the court.
In addition, if the offender is a government official or employee he shall suffer the penalties provided under civil
service laws, rules and regulations.
SECTION 10. Implementing Rules and Regulations. - The civil registrar general shall, in consultation with the
Department of Justice, the Department of Foreign Affairs, the Office of the Supreme Court Administrator, the University
of the Philippines Law Center and the Philippine Association of Civil Registrars, issue the necessary rules and regulations
for the effective implementation of this Act not later than three (3) months from the effectivity of this law.
SECTION 11. Retroactivity Clause. - This Act shall have retroactive effect insofar as it does not prejudice or impair
vested or acquired rights in accordance with the Civil Code and other laws.
SECTION 12. Separability Clause. - If any portion or provision of this Act is declared void or unconstitutional, the
remaining portions or provisions thereof shall not be affected by such declaration.
SECTION 13. Repealing Clause. - All laws, decrees, orders, rules and regulations, other issuances, or parts thereof
inconsistent with the provisions of this Act are hereby repealed or modified accordingly.
SECTION 14. Effectivity Clause. - This Act shall take effect fifteen (15) days after its complete publication in at least
two (2) national newspapers of general circulation.

RULE 108
Cancellation Or Correction Of Entries In The Civil Registry
Section 1. Who may file petition. Any person interested in any act, event, order or decree concerning the civil status of
persons which has been recorded in the civil register, may file a verified petition for the cancellation or correction of any entry
relating thereto, with the Court of First Instance of the province where the corresponding civil registry is located.
Section 2. Entries subject to cancellation or correction. Upon good and valid grounds, the following entries in the civil
register may be cancelled or corrected: (a) births: (b) marriage; (c) deaths; (d) legal separations; (e) judgments of annulments of
marriage; (f) judgments declaring marriages void from the beginning; (g) legitimations; (h) adoptions; (i) acknowledgments of
natural children; (j) naturalization; (k) election, loss or recovery of citizenship; (l) civil interdiction; (m) judicial determination of
filiation; (n) voluntary emancipation of a minor; and (o) changes of name.
Section 3. Parties. When cancellation or correction of an entry in the civil register is sought, the civil registrar and all persons
who have or claim any interest which would be affected thereby shall be made parties to the proceeding.
Section 4. Notice and publication. Upon the filing of the petition, the court shall, by an order, fix the time and place for the
hearing of the same, and cause reasonable notice thereof to be given to the persons named in the petition. The court shall also
cause the order to be published once a week for three (3) consecutive weeks in a newspaper of general circulation in the
province.
Section 5. Opposition. The civil registrar and any person having or claiming any interest under the entry whose cancellation
or correction is sought may, within fifteen (15) days from notice of the petition, or from the last date of publication of such notice,
file his opposition thereto.
Section 6. Expediting proceedings. The court in which the proceeding is brought may make orders expediting the
proceedings, and may also grant preliminary injunction for the preservation of the rights of the parties pending such
proceedings.
Section 7. Order. After hearing, the court may either dismiss the petition or issue an order granting the cancellation or
correction prayed for. In either case, a certified copy of the judgment shall be served upon the civil registrar concerned who shall
annotated the same in his record.

REPUBLIC OF THEPHILIPPINES,
Petitioner,

- versus -

JENNIFER B. CAGANDAHAN,
Respondent.

G.R. No. 166676


Present:
QUISUMBING, J., Chairperson,
CARPIO MORALES,
TINGA,
VELASCO, JR., and
BRION, JJ.
Promulgated:
September 12, 2008

x- - - - - - - - - - - - - - - - - - - - - - - - - - - - - - - - - - - - - - - - - - - - - - - - - - -x

DECISION
QUISUMBING, J.:
This is a petition for review under Rule 45 of the Rules of Court raising purely questions of law and seeking a reversal of the
Decision[1] dated January 12, 2005 of the Regional Trial Court (RTC), Branch 33 of Siniloan, Laguna, which granted the Petition for
Correction of Entries in Birth Certificate filed by Jennifer B. Cagandahan and ordered the following changes of entries in Cagandahans birth
certificate: (1) the name Jennifer Cagandahan changed to Jeff Cagandahan and (2) gender from female to male.
The facts are as follows.
On December 11, 2003, respondent Jennifer Cagandahan filed a Petition for Correction of Entries in Birth Certificate [2] before the
RTC, Branch 33 of Siniloan, Laguna.
In her petition, she alleged that she was born on January 13, 1981 and was registered as a female in the Certificate of Live Birth but
while growing up, she developed secondary male characteristics and was diagnosed to have Congenital Adrenal Hyperplasia (CAH) which is
a condition where persons thus afflicted possess both male and female characteristics. She further alleged that she was diagnosed to have
clitoral hyperthropy in her early years and at age six, underwent an ultrasound where it was discovered that she has small ovaries. At age
thirteen, tests revealed that her ovarian structures had minimized, she has stopped growing and she has no breast or menstrual development.
She then alleged that for all interests and appearances as well as in mind and emotion, she has become a male person. Thus, she prayed that
her birth certificate be corrected such that her gender be changed from female to male and her first name be changed from Jennifer to Jeff.
The petition was published in a newspaper of general circulation for three (3) consecutive weeks and was posted in conspicuous places
by the sheriff of the court. The Solicitor General entered his appearance and authorized the Assistant Provincial Prosecutor to appear in his
behalf.
To prove her claim, respondent testified and presented the testimony of Dr. Michael Sionzon of the Department of Psychiatry,
University of the PhilippinesPhilippine General Hospital. Dr. Sionzon issued a medical certificate stating that respondents condition is
known as CAH. He explained that genetically respondent is female but because her body secretes male hormones, her female organs did not
develop normally and she has two sex organs female and male. He testified that this condition is very rare, that respondents uterus is not
fully developed because of lack of female hormones, and that she has no monthly period. He further testified that respondents condition is
permanent and recommended the change of gender because respondent has made up her mind, adjusted to her chosen role as male, and the
gender change would be advantageous to her.
The RTC granted respondents petition in a Decision dated January 12, 2005 which reads:
The Court is convinced that petitioner has satisfactorily shown that he is entitled to the reliefs prayed [for]. Petitioner has adequately
presented to the Court very clear and convincing proofs for the granting of his petition. It was medically proven that petitioners body produces male
hormones, and first his body as well as his action and feelings are that of a male. He has chosen to be male. He is a normal person and wants to be
acknowledged and identified as a male.
WHEREFORE, premises considered, the Civil Register of Pakil, Laguna is hereby ordered to make the following corrections in the birth
[c]ertificate of Jennifer Cagandahan upon payment of the prescribed fees:
a)

By changing the name from Jennifer Cagandahan to JEFF CAGANDAHAN; and

b)

By changing the gender from female to MALE.

It is likewise ordered that petitioners school records, voters registry, baptismal certificate, and other pertinent records are hereby amended to
conform with the foregoing corrected data.
SO ORDERED.[3]

Thus, this petition by the Office of the Solicitor General (OSG) seeking a reversal of the abovementioned ruling.
The issues raised by petitioner are:
THE TRIAL COURT ERRED IN GRANTING THE PETITION CONSIDERING THAT:
I.
THE REQUIREMENTS OF RULES 103 AND 108 OF THE RULES OF COURT HAVE NOT BEEN COMPLIED WITH; AND,
II.
CORRECTION OF ENTRY UNDER RULE 108 DOES NOT ALLOW CHANGE OF SEX OR GENDER IN THE BIRTH CERTIFICATE,
WHILE RESPONDENTS MEDICAL CONDITION, i.e., CONGENITAL ADRENAL HYPERPLASIA DOES NOT MAKE HER A MALE.[4]

Simply stated, the issue is whether the trial court erred in ordering the correction of entries in the birth certificate of respondent to
change her sex or gender, from female to male, on the ground of her medical condition known as CAH, and her name from Jennifer to
Jeff, under Rules 103 and 108 of the Rules of Court.
The OSG contends that the petition below is fatally defective for non-compliance with Rules 103 and 108 of the Rules of Court
because while the local civil registrar is an indispensable party in a petition for cancellation or correction of entries under Section 3, Rule 108
of the Rules of Court, respondents petition before the court a quo did not implead the local civil registrar.[5]The OSG further contends
respondents petition is fatally defective since it failed to state that respondent is a bona fide resident of the province where the petition was
filed for at least three (3) years prior to the date of such filing as mandated under Section 2(b), Rule 103 of the Rules of Court. [6] The OSG
argues that Rule 108 does not allow change of sex or gender in the birth certificate and respondents claimed medical condition known as
CAH does not make her a male.[7]
On the other hand, respondent counters that although the Local Civil Registrar of Pakil, Laguna was not formally named a party in the
Petition for Correction of Birth Certificate, nonetheless the Local Civil Registrar was furnished a copy of the Petition, the Order to publish
on December 16, 2003 and all pleadings, orders or processes in the course of the proceedings, [8] respondent is actually a male person and
hence his birth certificate has to be corrected to reflect his true sex/gender, [9] change of sex or gender is allowed under Rule 108, [10] and
respondent substantially complied with the requirements of Rules 103 and 108 of the Rules of Court.[11]
Rules 103 and 108 of the Rules of Court provide:

Rule 103
CHANGE OF NAME
SECTION 1. Venue. A person desiring to change his name shall present the petition to the Regional Trial Court of the province in which he
resides, [or, in the City of Manila, to the Juvenile and Domestic Relations Court].
SEC. 2. Contents of petition. A petition for change of name shall be signed and verified by the person desiring his name changed, or some
other person on his behalf, and shall set forth:
(a)
That the petitioner has been a bona fide resident of the province where the petition is filed for at least three (3) years prior to the date
of such filing;
(b)

The cause for which the change of the petitioner's name is sought;

(c)

The name asked for.

SEC. 3. Order for hearing. If the petition filed is sufficient in form and substance, the court, by an order reciting the purpose of the petition,
shall fix a date and place for the hearing thereof, and shall direct that a copy of the order be published before the hearing at least once a week for
three (3) successive weeks in some newspaper of general circulation published in the province, as the court shall deem best. The date set for the
hearing shall not be within thirty (30) days prior to an election nor within four (4) months after the last publication of the notice.
SEC. 4. Hearing. Any interested person may appear at the hearing and oppose the petition. The Solicitor General or the proper provincial or
city fiscal shall appear on behalf of the Government of the Republic.
SEC. 5. Judgment. Upon satisfactory proof in open court on the date fixed in the order that such order has been published as directed and
that the allegations of the petition are true, the court shall, if proper and reasonable cause appears for changing the name of the petitioner, adjudge
that such name be changed in accordance with the prayer of the petition.
SEC. 6. Service of judgment. Judgments or orders rendered in connection with this rule shall be furnished the civil registrar of the
municipality or city where the court issuing the same is situated, who shall forthwith enter the same in the civil register.
Rule 108
CANCELLATION OR CORRECTION OF ENTRIES
IN THE CIVIL REGISTRY
SECTION 1. Who may file petition. Any person interested in any act, event, order or decree concerning the civil status of persons which has
been recorded in the civil register, may file a verified petition for the cancellation or correction of any entry relating thereto, with the Regional Trial
Court of the province where the corresponding civil registry is located.
SEC. 2. Entries subject to cancellation or correction. Upon good and valid grounds, the following entries in the civil register may be
cancelled or corrected: (a) births; (b) marriages; (c) deaths; (d) legal separations; (e) judgments of annulments of marriage; (f) judgments declaring
marriages void from the beginning; (g) legitimations; (h) adoptions; (i) acknowledgments of natural children; (j) naturalization; (k) election, loss or
recovery of citizenship; (l) civil interdiction; (m) judicial determination of filiation; (n) voluntary emancipation of a minor; and (o) changes of name.
SEC. 3. Parties. When cancellation or correction of an entry in the civil register is sought, the civil registrar and all persons who have or
claim any interest which would be affected thereby shall be made parties to the proceeding.
SEC. 4. Notice and publication. Upon the filing of the petition, the court shall, by an order, fix the time and place for the hearing of the
same, and cause reasonable notice thereof to be given to the persons named in the petition. The court shall also cause the order to be published once a
week for three (3) consecutive weeks in a newspaper of general circulation in the province.
SEC. 5. Opposition. The civil registrar and any person having or claiming any interest under the entry whose cancellation or correction is
sought may, within fifteen (15) days from notice of the petition, or from the last date of publication of such notice, file his opposition thereto.
SEC. 6. Expediting proceedings. The court in which the proceedings is brought may make orders expediting the proceedings, and may also
grant preliminary injunction for the preservation of the rights of the parties pending such proceedings.
SEC. 7. Order. After hearing, the court may either dismiss the petition or issue an order granting the cancellation or correction prayed for. In
either case, a certified copy of the judgment shall be served upon the civil registrar concerned who shall annotate the same in his record.

The OSG argues that the petition below is fatally defective for non-compliance with Rules 103 and 108 of the Rules of Court because
respondents petition did not implead the local civil registrar. Section 3, Rule 108 provides that the civil registrar and all persons who have
or claim any interest which would be affected thereby shall be made parties to the proceedings. Likewise, the local civil registrar is required
to be made a party in a proceeding for the correction of name in the civil registry. He is an indispensable party without whom no final
determination of the case can be had.[12] Unless all possible indispensable parties were duly notified of the proceedings, the same shall be
considered as falling much too short of the requirements of the rules. [13] The corresponding petition should also implead as respondents the
civil registrar and all other persons who may have or may claim to have any interest that would be affected thereby. [14] Respondent, however,
invokes Section 6,[15] Rule 1 of the Rules of Court which states that courts shall construe the Rules liberally to promote their objectives of
securing to the parties a just, speedy and inexpensive disposition of the matters brought before it. We agree that there is substantial
compliance with Rule 108 when respondent furnished a copy of the petition to the local civil registrar.
The determination of a persons sex appearing in his birth certificate is a legal issue and the court must look to the statutes. In this
connection, Article 412 of the Civil Code provides:
ART. 412. No entry in a civil register shall be changed or corrected without a judicial order.

Together with Article 376[16] of the Civil Code, this provision was amended by Republic Act No. 9048[17] in so far asclerical or
typographical errors are involved. The correction or change of such matters can now be made through administrative proceedings and
without the need for a judicial order. In effect, Rep. Act No. 9048 removed from the ambit of Rule 108 of the Rules of Court the correction
of such errors. Rule 108 now applies only to substantial changes and corrections in entries in the civil register.[18]
Under Rep. Act No. 9048, a correction in the civil registry involving the change of sex is not a mere clerical or typographical error. It
is a substantial change for which the applicable procedure is Rule 108 of the Rules of Court.[19]
The entries envisaged in Article 412 of the Civil Code and correctable under Rule 108 of the Rules of Court are those provided in
Articles 407 and 408 of the Civil Code:
ART. 407. Acts, events and judicial decrees concerning the civil status of persons shall be recorded in the civil register.
ART. 408. The following shall be entered in the civil register:
(1) Births; (2) marriages; (3) deaths; (4) legal separations; (5) annulments of marriage; (6) judgments declaring marriages void from the beginning;
(7) legitimations; (8) adoptions; (9) acknowledgments of natural children; (10) naturalization; (11) loss, or (12) recovery of citizenship; (13) civil
interdiction; (14) judicial determination of filiation; (15) voluntary emancipation of a minor; and (16) changes of name.

The acts, events or factual errors contemplated under Article 407 of the Civil Code include even those that occur after birth.[20]
Respondent undisputedly has CAH. This condition causes the early or inappropriate appearance of male characteristics. A person,
like respondent, with this condition produces too much androgen, a male hormone. A newborn who has XX chromosomes coupled with
CAH usually has a (1) swollen clitoris with the urethral opening at the base, an ambiguous genitalia often appearing more male than female;
(2) normal internal structures of the female reproductive tract such as the ovaries, uterus and fallopian tubes; as the child grows older, some
features start to appear male, such as deepening of the voice, facial hair, and failure to menstruate at puberty. About 1 in 10,000 to 18,000
children are born with CAH.
CAH is one of many conditions[21] that involve intersex anatomy. During the twentieth century, medicine adopted the term
intersexuality to apply to human beings who cannot be classified as either male or female. [22] The term is now of widespread
use. According to Wikipedia, intersexuality is the state of a living thing of a gonochoristic species whose sex chromosomes, genitalia,
and/or secondary sex characteristics are determined to be neither exclusively male nor female. An organism with intersexmay have
biological characteristics of both male and female sexes.
Intersex individuals are treated in different ways by different cultures. In most societies, intersex individuals have been expected to
conform to either a male or female gender role. [23] Since the rise of modern medical science in Western societies, someintersex people with
ambiguous external genitalia have had their genitalia surgically modified to resemble either male or female genitals. [24] More commonly,
an intersex individual is considered as suffering from a disorder which is almost always recommended to be treated, whether by surgery
and/or by taking lifetime medication in order to mold the individual as neatly as possible into the category of either male or female.
In deciding this case, we consider the compassionate calls for recognition of the various degrees of intersex as variations which should
not be subject to outright denial. It has been suggested that there is some middle ground between the sexes, a no-mans land for those
individuals who are neither truly male nor truly female. [25] The current state of Philippine statutes apparently compels that a person be
classified either as a male or as a female, but this Court is not controlled by mere appearances when nature itself fundamentally negates such
rigid classification.
In the instant case, if we determine respondent to be a female, then there is no basis for a change in the birth certificate entry for
gender. But
if
we
determine,
based
on
medical
testimony
and
scientific
development
showing the respondent to be other than female, then a change in the
subjects birth certificate entry is in order.
Biologically, nature endowed respondent with a mixed (neither consistently and categorically female nor consistently and
categorically male) composition. Respondent has female (XX) chromosomes. However, respondents body system naturally produces high
levels of male hormones (androgen). As a result, respondent has ambiguous genitalia and the phenotypic features of a male.
Ultimately, we are of the view that where the person is biologically or naturally intersex the determining factor in his gender
classification would be what the individual, like respondent, having reached the age of majority, with good reason thinks of his/her
sex. Respondent here thinks of himself as a male and considering that his body produces high levels of male hormones (androgen) there is
preponderant biological support for considering him as being male. Sexual development in cases of intersex persons makes the gender
classification at birth inconclusive. It is at maturity that the gender of such persons, like respondent, is fixed.

Respondent here has simply let nature take its course and has not taken unnatural steps to arrest or interfere with what he was born
with. And accordingly, he has already ordered his life to that of a male. Respondent could have undergone treatment and taken steps, like
taking lifelong medication,[26] to force his body into the categorical mold of a female but he did not. He chose not to do so. Nature has
instead taken its due course in respondents development to reveal more fully his male characteristics.
In the absence of a law on the matter, the Court will not dictate on respondent concerning a matter so innately private as ones
sexuality and lifestyle preferences, much less on whether or not to undergo medical treatment to reverse the male tendency due to CAH. The
Court will not consider respondent as having erred in not choosing to undergo treatment in order to become or remain as a female. Neither
will the Court force respondent to undergo treatment and to take medication in order to fit the mold of a female, as society commonly
currently knows this gender of the human species. Respondent is the one who has to live with his intersexanatomy. To him belongs the
human right to the pursuit of happiness and of health. Thus, to him should belong the primordial choice of what courses of action to take
along the path of his sexual development and maturation. In the absence of evidence that respondent is an incompetent [27] and in the
absence of evidence to show that classifying respondent as a male will harm other members of society who are equally entitled to protection
under the law, the Court affirms as valid and justified the respondents position and his personal judgment of being a male.
In so ruling we do no more than give respect to (1) the diversity of nature; and (2) how an individual deals with what nature has
handed out. In other words, we respect respondents congenital condition and his mature decision to be a male. Life is already difficult for
the ordinary person. We cannot but respect how respondent deals with his unordinary state and thus help make his life easier, considering
the unique circumstances in this case.
As for respondents change of name under Rule 103, this Court has held that a change of name is not a matter of right but of judicial
discretion, to be exercised in the light of the reasons adduced and the consequences that will follow. [28] The trial courts grant of respondents
change of name from Jennifer to Jeff implies a change of a feminine name to a masculine name. Considering the consequence that
respondents change of name merely recognizes his preferred gender, we find merit in respondents change of name. Such a change will
conform with the change of the entry in his birth certificate from female to male.
WHEREFORE, the Republics petition is DENIED. The Decision dated January 12, 2005 of the Regional Trial Court, Branch 33 of
Siniloan, Laguna, is AFFIRMED. No pronouncement as to costs.
SO ORDERED.

You might also like